Across Vol 5

You might also like

Download as pdf or txt
Download as pdf or txt
You are on page 1of 786

rmation on PGMEE books, visit our website www.jaypeebrothers.

com, for detailed information on PGMEE books, visit our website www.jaypeebrothers.com

rBestselling book on the subject


Thoroughly revised & updated edition including
G S to Subjectwise Synopsis & Latest Exam Pattern Questions

^ACROSS
A Complete Review of Short Subjects
A Unique book for NEET, DNB, AIIMS, PGI, FMGE and State-level examinations

Physiology Volume
Biochemistry
5
8 th Edition

Saumya Shukla • Anurag Shukla


Thoroughly revised and updated edition w ith a new layout and tw o new sub specialities added (Radiotherapy
& Nuclear M edicine).
Concise, u p -to -d a te and accurate.
W ritten in simple language w ith precise explanation.
Systematic presentation of the text.
Book covers all questions from 2001 onwards
All MCQs o f All India (NBE Pattern), AIIM S, PGI, DNB, and State-based MCQs up to 2 01 6 have been included

A m u s t-b u y b o o k f o r A ll In d ia , A IIM S , P G I, J IP M E R , D N B a n d S t a t e E n tr a n c e E x a m s
Across
Volume 5
Physiology Biochemistry

Eighth Edition

Saumya Shukla
DNB Radiodiagnosis
Assistant Professor
Nizam's Institute of Medical Sciences
Hyderabad, India

Anurag Shukla
MS MCh (Orth), UK

Siddharth Dixit
DO, DNB (Ophthalmology)
Glaucoma Consultant, L V Prasad Eye Institute,
Hyderabad, India

Khushi Shukla

W m ) The Health Sciences Publisher


New Delhi | London | Panam a
ja y p e e ; Jaypee Brothers Medical Publishers (P) Ltd

Headquarters
Jaypee Brothers Medical Publishers (P) Ltd.
4838/24, Ansari Road, Daryaganj
New Delhi 110 002, India
Phone: +91-11-43574357
Fax: +91-11-43574314
E-mail: jaypee@jaypeebrothers.com

Overseas Offices Jaypee-Highlights Medical Publishers Inc.


J.P. Medical Ltd. City of Knowledge, Bid. 235, 2nd Floor, Clayton
83, Victoria Street, London Panama City, Panama
SW1H OHW(UK) Phone: +1 507-301-0496
Phone: +44-20 3170 8910 Fax: +1 507-301-0499
Fax: +44(0)20 3008 6180 E-mail: cservice@jphmedical.com
E-mail: info@jpmedpub.com

Jaypee Brothers Medical Publishers (P) Ltd. Jaypee Brothers Medical Publishers (P) Ltd.
17/1-B, Babar Road, Block-B, Shaymali Bhotahity, Kathmandu, Nepal
Mohammadpur, Dhaka-1207, Bangladesh Phone: +977-9741283608
Mobile: +08801912003485 E-mail: kathmandu@jaypeebrothers.com
E-mail: jaypeedhaka@gmail.com

Website: www.jaypeebrothers.com
Website: www.jaypeedigital.com
© 2017, Jaypee Brothers Medical Publishers
The views and opinions expressed in this book are solely those of the original contributor(s)/author(s) and do not
necessarily represent those of editor(s) of the book.
All rights reserved. No part of this publication may be reproduced, stored or transmitted in any form or by any means,
electronic, mechanical, photocopying, recording or otherwise, without the prior permission in writing of the publishers.
All brand names and product names used in this book are trade names, service marks, trademarks or registered trademarks
of their respective owners. The publisher is not associated with any product or vendor mentioned in this book.
Medical knowledge and practice change constantly. This book is designed to provide accurate, authoritative information
about the subject matter in question. However, readers are advised to check the most current information available on
procedures included and check information from the manufacturer of each product to be administered, to verify the
recommended dose, formula, method and duration of administration, adverse effects and contraindications. It is the
responsibility of the practitioner to take all appropriate safety precautions. Neither the publisher nor the author(s)/editor(s)
assume any liability for any injury and/or damage to persons or property arising from or related to use of material in this
book.
This book is sold on the understanding that the publisher is not engaged in providing professional medical services. If
such advice or services are required, the services of a competent medical professional should be sought.
Every effort has been made where necessary to contact holders of copyright to obtain permission to reproduce copyright
material. If any have been inadvertently overlooked, the publisher will be pleased to make the necessary arrangements
at the first opportunity. The CD/DVD-ROM (if any) provided in the sealed envelope with this book is complimentary and
free of cost. Not meant for sale.
Inquiries for bulk sales may be solicited at: jaypee@jaypeebrothers.com

Across Volume 5

Seventh Edition: 2015

Eighth Edition: 2017


ISBN: 97 8 -9 3 -5 2 7 0 -1 4 0 -7

P rinted a t Sanat Printers


Contents
Physiology

1. Central Nervous System 3


2. Nerves and Muscles Physiology 44
3. Respiratory System 64
4. Cardiovascular System 91
5. Digestive System 133
6. Excretory System 152
7. Endocrine System 170
8. General Physiology 205
9. Cell Physiology 210
Review Notes 233

Biochemistry

1. Carbohydrate Metabolism 419


2. Fat Metabolism 449
3. Structure, Function and Replication of Informational Macromolecules 471
4. Cytogenetics and Molecular Genetics,Recombinant DNA & Genomic Technology 511
5. Structure & Functions o f Proteins and Enzymes 530
6. Metabolism of Protein & Amino Acids 553
7. General Biochemistry 574
Review Notes 587
PHYSIOLOGY

MCQs with Answers and Explanations


Chapter -1 CENTRAL NERVOUS SYSTEM

Q U E S T IO N S

Higher Function C.VPL nucleus □


D.Anterior nucleus of Thalamus □
A ttention and concentration is m ediated by: 9. W indow of lim bic system (Assam 04, WB -03)
A. Hypothalamus (AIIMS 14) □ A. Hypothalmus □
B. Frontal lobe □ B. Amygdala □
C. Parietal lobe □ C. Hippocampus □
D. Basal ganglia □ D. Thalmus □
10. Functions of lim bic system : (PGI 06)
H um an brain is more intelligent than monkey's
brain due to: (PGI 03) A. Emotion □
B. Memory □
A. Larger brain □
C. Higher function □
B. Increased convulations □
D. Planned motor activity □
C. Increased brain area compared to body surface
11. Structure of brain involved in emotion :
area □
A. Neocortex (PGI 04) □
D. More blood supply □
B. Limbic system □
True about function of brain are a ll except: (AI 06)
C. Thalamus □
A. Sensitive to hypoxia □
D. Hippocampus □
B. Dependent on glucose □
E. Basal ganglia □
C. Uses fatty acids in starvation □
12. Self stim ulation could be induced experim entally
D. Does not store energy □ most effectively from which part of the brain OR
Sem atosensory area I largest representation is for- Area of brain responsible for self stim ulation
A. Arm (NBE P 13) D rew ard on induction (i.e. reward centre) is?
B. Leg □ A. Periaqueductal area (AI 12; AIIMS 1 4 )0
C. Back □ B. Medial forebrain bundle □
D. Head □ C. Periventricular region of hypothalamus □
Ablation of the 'som atosensory area I' of the D. Mesencephalon □
cerebral cortex leads to: 13. W hich of the following statem ents about 'Sham
(NBE P 15,14,13; AIIMS 14; AI 02) Range' is Not true: (AI 12)
A. Total loss of pain sensation □ A. Caused by Hypothalamic stimulation □
B. Total loss of touch sensation □ B. Abolished by Decortication □
C. Loss of tactile localization but not of two point C. Pathological Rage Reaction □
discrimination □ D. Association with sympathetic stimulation □
D. Loss of tactile localization and two point 14. W hich of the following nucleus of Hypothalam us is
discrimination □ most closely related to sexual activity in males
The inability to perceive the texture & shape an A. Preoptic nucleus (Al 12) □
object occurs in lesion of: (AIIMS 06) B. Supraoptic nucleus □
A. Lateral spino-thalamic tract □ C. Lateral hypothalamic nuclei □
B. Nucleus gracils □ D. Posterior hypothalamic nuclei □
C. Spino reticular tract □ 15. The m echanism of learning and m em ory, include all
D. Nucleus cuneatus/tractus cuneatus □ except: (AI 07)
Pt. is able to recognize person by name but not by A. Changes in level of neurotransmitter at
face. Lesion is in - (NBE P 13) synapse □
A. Post parietal region □ B. Increasing protein synthesis □
B. Occipital □ C. Recruitment by multiplication of neurons □
C. Frontal lobe □ D. Spatial Reorganization of synapse □
D. Temporal lobe □ 16. The processing of short term m em ory to long term
m em ory is done in OR Unable to consolidate long
term m em ory indicate dam age to OR
Emotions & Memory
Consolidation of long term mem ory occur in:
A. Prefrontal cortex (NBE P 15,14,13) □
The nucleus involved in Papez circuit is:
B. Hippocampus (AIIMS 14,12; AI 0 7 )0
A. Pulvinar (Al 07) □
C. Neocortex □
B. Intralaminar □
D. Amygdala □
4 a A Complete Review of Short Subjects

17. All are true statem ent regarding mem ory except 25. Neurophysiological defects present in right lobe
A.Structural changes occur in synapses during the involvem ent are A/E (AIIMS 09)
development of long-term memory □ A. Visuospatial defect □
B. Short term memory last as long as the person B. Anosognosa □
continues to think (PGI 1 1 )0 C. Dyscalculia □
C. The site for storage of long term memory is in D. Dysgraphia □
temporal lobe □
Aphasia
D. Bilateral loss of hippocampal function leads to
loss of the ability to encode events of the recent 26. Broca's area is present in- (NBE P 13)
past in long term memory □ A. Superior temporal gyrus O
E. Intermediate memory results from temporary B. Precentral gyrus a
change in synapse □ C. Post central gyrus o
18. Associative learning- (NBE P 13) D. Inferior frontal gyrus □
A. Associated with consciousness □ 27. Broca's area is concerned with: (AI 07)
B. Includes skills and habits □ A. Word formation 0
C. Relation of one stimulus to another □ B. Comprehension □
D. Facilitation of recognition of words □ C. Repetition □
19. Striatum damage affects prim ing- (NBE P 13,15) D. Reading □
A. Procedural memory □ 28. M otor aphasia refers to defect in: (AIIMS 08)
B. Short term memory □ A. Peripheral speech apparatus □
C. Long term memory □ B. Verbal expression □
D. Explicit memory □ C. Auditory comprehension □
D. Verbal comprehension □
Sleep
Hearing & Vision
20. Key regulators of sleep are located in
29. Endolym ph in ear equivalent to- (NBE P 13)
A. Hypothalamus (AI 09, AIIMS 1 2 )0
A. ICF O
B. Thalamus □
B. Lymph □
C. Putamen □
C. CSF □
D. Limbic cortex □
D. Blood □
21. W hich of the following is likely to happen to the
30. Inner hair cells depolarises due to- (NBE P 13)
'Sleep-w ake cycle' if all environm ent/external cues
A. K+ influx O
are rem oved: (AIIMS 12, AI12)
B. Na+ influx a
Will remain unchanged at 24 hrs □
C. K+efflux 0
Will be prolonged to greater than 24 hrs cycle
D. Voltage gated ca channels □
length
31. Sem icircular canals are stim ulated by- (NBE P 13)
C. Will be shortened to less than 24 hours cycle
A. Gravity O
length
B. Linear acceleration a
D. Will become irregular/does not continue
C. Rotation a
22 . W hich of the following nucleus controls the
D. Sound a
circadian rhythm ? OR D ium al variation of ACTH
32. Endo-cochlear potential is: (AIIMS 16)
depends on- (NBE P 15,13; AIIMS 08,13,15)
A. +45 mV □
A. Supraoptic nucleus □
B. -45m V □
B. Paraventricular nucleus □
C. -60m V □
C. Suprachiasmatic nucleus □
D. +85 mV □
D. Premamillary nucleus □
33. W here do the peripheral processes of spiral
Cereberal Dominance ganglion end? (AIIMS 16)

23. Left lobe is responsible for (AIIMS 12)


A. Appreciation of Music □
B. Spatial orientation □
C. Visual Stimuli Processing □
D. Written and Spoken Language □
24. Agnosia is caused by lesion of- (NBE P 13)
A. Representative hemisphere, left □
B. Categorial hemisphere, left □
C. Representative hemisphere, right □
D. Categorial hemisphere, right □
Physiology: Central Nervous System ■ 5

A. C □ 43. Relative colour and lum inosity of photoreceptive


B. D □ input under changing light conditions are regulated
C. B □ and maintained by: (AIIMS 03)
D. A □ A. Muller cells □
34. True stereopsis is perceived due to the following: B. Amacrine cells □
A. Overlay of contours (AIIMS 06) □ C. Ganglion cells □
B. Motion parallax □ D. Retinal astrocytes □
C. Bi- nasal disparity □ 44. During the dark phase of visual cycle, w hich form
D. Linear perspective □ of vitam in A com bines with opsin to make
35. The fibers from the contralateral nasal hemiretina Rhodopsin: (AIIMS 04)
project to the following layers of the lateral A. All trans-Retinaldehyde □
geniculate nucleus: B. All trans-Retinol □
A. Layers 2 ,3 & 5. (AI 05) □ C. 11-cis-Retinaldehyde □
B. Layers 1 ,2 & 6 . □ D. 11-cis-Retinol □
C. Layers 1, 4 & 6 . □ 45. Initiation of visual im pulse is associated with
D. Layers 4 ,5 & 6 . □ A. Condensation of opsin with vitamin A
36. M ost afferent fibers from the lateral geniculate aldehyde (AI 09) □
nucleus term inate in the prim ary visual cortex is: B. Photoisomerization and hydrolysis of visual
A. Layer 1 (A I0 6 )D purple □
B. Layer 2 & 3 □ C. I NADP □
C. Layer 4 □ D. i NAD □
D. Layer 5 & 6 □ 46. Photoreceptors on exposure to dark release-
37. The parvocellular pathway, from the lateral A. Acetylcholine (NBE P 13) □
geniculate nucleus to the visual cortex, carries B. Glutamate □
signals for the detection of: (AI 06) C. Adrenaline □
A. Movement, depth and flicker □ D. Epinephrine □
B. Color vision, shape and fine details □ 47. Transducin is required for- (NBE P 13)
C. Temporal frequency □ A. Small □
D. Luminance contrast □ B. Hearing □
38. The parvocellular pathway from lateral geniculate C. Small □
nucleus to visual cortex is m ost sensitive for the D. Vision □
stim ulus of: (AI 05) 48. Rods and cones differ in all, Except:
A. Color contrast. □ A. Signal transduction (AIIMS 09) □
B. Luminance contrast. □ B. Light sensitivity □
C. Temporal frequency. □ C. Wavelength □
D. Saccadic eye movements. □ D. Acuity □
39. The blobs of the visual cortex are associated with:
A. Ocular dominance (AI 06) □ Color Vision, Taste & Olfaction
B. Orientation □
C. Color processing □ 49. For color vision, which is true? (AIIMS 09)
D. Saccadic eye movements □ A. Independent of wavelength of light □
40. Retinal cells which secrete acytyl choline- B. Depends on intensity discrimination □
A. Bipolar cells (NBE P 13) □ C. Involves opponent color cells □
B. Ganglion cells □ D. Minimum at fixation point □
C. Amacrine cells □ 50. Visible range of electrom agnetic spectrum of human
D. H cells □ eye (PGI 04)
41. Which is the first order neuron in optic pathway: A. 370 - 740 nm □
A. Bipolar cells (NBE P 13) O B. 7 4 0 -1 1 4 0 nm □
B. Ganglionic cells □ C. 2 0 0 -3 4 0 nm □
C. Cells of lateral geniculate body □ D. 200 - 370 nm □
D. Astrocytes □ E. 370 - 770 nm □
42. Second order neuron- (NBE P 13) 51. W hich of the follow ing are Chem oreceptors-
A. Optic nerve □ A. Rods (NBE P 13) □
B. MGB □ B. Cones □
C. LGB □ C. Taste buds □
D. Layer of retina □ D. Muscle spindle □
6 ■ A Complete Review of Short Subjects

52. Salty taste is due to- (NBE P 13) C. Emotions □


A. Ca+2 channels □ D. Maintenance of equilibrium □
B. Na+ channels □ E. Coordination of movements □
C. G-protein □ 62. W hich of the following clearly states the role of
D. H+ Channels □ basal ganglia in m otor function:
53. The receptors/buds responsible for carrying bitter A. Planning (NBE P 13; PGI 05,13,14) □
taste sensation are situated- (NBE P 13) B. Skilled function □
A. At the tip of the tongue □ C. Coordinate function □
B. Just behing the tip □ D. Balance & Coarse movements □
C. At the sides □ 63. True about spinocerebellar tract is:
D. At the posterior aspect □ A.Equilibrium (AIIMS 09,10) □
54. Which is incorrectly m atched- (NBE P 13) B. Smoothens and coordinates movement □
A. Eyes-rods& cones □ C. Learning induced by change in vestibulocular
B. Olfaction-hair cells □ reflex □
C. Tongue-Na channels □ D. Planning and programming □
D. Carotid body-chemoreceptor □ 64. Lesion of Globus pallidus causes- (NBE P 13)
A. Chorea □
Equilibrium & Basal Ganglia B. Ballismus □
C. Athetosis □
55. Static equilibrium is due to- (NBE P 13) D. Akinesia □
A. Macula □ 65. Disease of Basal ganglia dysfunction are all except-
B. Cupula □ A. Alziemer's disease (NBE P 13) □
C. End organ of corti □ B. Parkinsonism □
D. Cristae ampulla □ C. Chorea. □
56. Nucleus of basal ganglia ar e:(PGI 04,13; NBEP 14,13) D. Athetosis □
A. Dentate □
B. Thalamus □
Blood Brain Barrier
C. Caudate nucleus / Putamen/ GP □
D. Red nucleus □
66. W hich of the following is not perm eable through
57. The efferent fiber bundle of the substantia nigra
the Blood Brain Barrier: (AIIMS 10)
transm its dopam ine to one of the following areas:
A. Water □
A. Thalamus (AIIMS 04) □
B. Corpus striatum □ B. Lipophilic drugs □
C. Tegmentum of pons □ C. Gas □
D. Tectum of midbrain □ D. Proteins □
58. The highest density of glutam ate receptors in basal 67. "Blood brain barrier" is present at all of the
ganglia are found in (AI 12) follow ing sites except: (AIIMS 14,12; AI 2K)
A. Substantia nigra □ A. Hebenular nucleus □
B. Globus pallidus interna □ B. Subfornical organ □
C. Globus pallidus externa □ C. Cerebellum □
D. Putamen □ D. Pontine nucleus □
59. W hich of the follow ing basal ganglia nucleus is 68. C ircum entricular organ is- (NBE P 13)
prim arily glutam inergic (AI12) A. Anterior pituitary □
A. Putamen □ B. Posterior pituitary □
B. Subthalamic Nucleus □ C. Pineal gland □
C. Globus pallidus □ D. None □
D. Substantial Nigra □ 69. Vom iting centre- (NBE P 13)
60. W hich of the follow ing acts as the m ajor A. Area postrema □
neurotransm itter in substantia nigra (nigrostriatal B. Suprachiasmatic □
pathway): (NBE P 14,13) C. Medial nuclei □
A. Dopamine (AIIMS 0 7 ,14)U D. Lateral nuclei □
B. Noradrenaline □ 70. V om iting centre is situated in the- (NBE P 13)
C. Acetylcholine □ A. Hypothalamus □
D. Serotonin □ B. Amygdala □
61. Functions of basal ganglia include : (PGI 02) C. Rons □
A. Gross motor □ D. Medulla □
B. Skilled movements □
Physiology: Central Nervous System ■ 7

Cerebellum, Posture & Decortication C. Resting tremor □


D. Ataxia/Hypotonia □
71. Flocculo nodular lobe has direct connection with :
81. W hich of the follow ing does not carry
A. Red nucleus (AI 08) □
proprioceptive im pulses? (NBE P 13)
B. Inferior olivary nucleus □
A. Olivo cerebellar tract □
C. Vestibular nucleus □
B. Tecto cerebellar tract □
D. Dentate nucleus □
C. Spino cerebellar tract □
72. V estibulocerebellar tract term inates in the... of
D. Cuneo cerebellar tract □
cerebellum - (JIPMER 04)
82. D estruction of anterior cerebellum in a decerebrate
A. Flocculus □
anim al leads to- (NBE P 13)
B. Lingula □
C. Nodulus □ A.No effect on rigidity □
B.Increased flexor muscle tone □
D. Uvula □
E. All of the above C.Increased rigidity via alpha motor neurouns □
73. Cerebellar connection to other parts of the brain is D.Decrease in rigidity □
projected through which cell OR O utput from the 83. True about spinocerebellar tract is- (NBE P 13)
cerebellum is solely from:: (AI 04) A. Equilibrium □
A. Golgi cells □ B. Smoothens and coordinates movement □
B. Basket cells □ C. Learining induced by change in vestibulocular
C. Purkinje cells □ reflex □
D. Oligodendrocytes □ D. Planning and programming □
74. Purkinje cells from the cerebellum end in OR 84. Vestibulo ocular reflex is concerned with-
Purkinje fibres are inhibitory for: (NBE P 14,13) A. Archicerebellum (NBE P 13) □
A. Extrapyramidal system (Jipmer 05) □ B. Flocculonodular lobe □
B. Cranial nerve nuclei □ C. Neocerbellum □
C. Deep Cerebellar nuclei □ D. Occipital lobe □
D. Cerebral cortex □ 85. Setting posture before planned m ovem ent-
75. All are true about cerebellum except (AI 04; PGI 05) A. Premotor cortex (NBE P 13) □
A. It has 3 layers, 4 nucleus & 5 cells □ B. Motor cortex □
B. Climbing fibres are afferent inputs □ C. Frontal □
C. Mossy fibres inhibit Purkinje cells □ D. Supplementary motor cortex □
D. Climbing fibres excite Purkinje cells □ 86. The m aintenance of posture in a norm al adult
76. W hich is a cerebellar nucleus- (NBE P 13) hum an being depends upon: (AI 06)
A. Caudate □ A. Integrity of reflex arc □
B. Subthalamic □ B. Muscle power □
C. Globus pallidus □ C. Type of muscle fibers □
D. Dentate □ D. Joint movements in physiological range □
77. Cells present in Cerebellar cortex are all except. 87. Posture is m aintained by- (NBE P 13)
A. Purkinje (AIIMS 08) □ A. Crosse extensor reflex □
B. Bipolar □ B. Stretch reflex □
C. Granule □ C. Golgi tendon refkex □
D. Golgi □ D. None □
78. True about cereb ellu m : (PGI 04) 88. In decorticate anim al which reflex is lost? (NBE P 13)
A. Cerebral cortex have mostly inhibitory effects □ A. Hoping and placing reflex □
B. Co-ordination □ B. Tonic neck reflex □
C. Planning of motor movements □
C. Tonic labyrinthine reflex □
D. Decreased tone □
D. Stretch reflex □
E. Excitatory effect from deep nuclei □
89. Postural reflex w ith integrating center in spine-
79. Which one of the following clearly states the role of
A. Righting reflex (NBE P 13) □
cerebellum in m otor perform ance: (AIIMS 04)
B. Tonic labyrinthine reflex □
A. Planning and programming of movement □
C. Righting reflex □
B. Convert abstract thought into voluntary action □
D. Crossed extensor reflex □
C. Initiation of skilled voluntary action □
90. All of the follow ing are true about Kluver-Buey
D. Smoothens and coordinates ongoing
syndrom e except: (NBE P 13)
movements □
A. Hypersexuality □
80. Lesions of the lateral cerebellum cause all of the
B. Hyperactivity □
following, Except: (NBE P 15,13,14; AI 10; AIIMS 10)
A. Incoordination □ C. Hypermetamorphosis □
B. Intention tremor/Dymetria □ D. Placidity □
8 m A Complete Review of Short Subjects

Appetite C. Gamma-amino butyric acid □


D. Taurine □
91. The principal polypeptide that increase food intake
101. The hyperkinetic features of the Huntington's
are all except- (NBE P 13)
disease are due to the loss of: (AI 06)
A. Neuropeptide Y □
A. Nigrostriatal dopaminergic system □
B. Leptin □
B. Intrastriatal cholinergic system □
C. Orexin A □ C. GABA-ergic and cholinergic system □
D. Ghrelin □ D. Intrastriatal GABA-ergic and cholinergic syste □
92. Appetite is stimulated by all of the following
peptides, Except: (AI 09, AIIMS 10)
Thalamus & Thirst Control
A. Agouti - Related Peptide (AGRP) □
B. Melanocyte Stimulating hormone(MSH) T
102. Pain and temperature in thalamus is controlled by
C. Melanin Concentrating Hormone(MCH) □
nucleus- (NBE P 13)
D. Neuropeptide Y □
A. VPM □
93. W hich horm one is secreted from stom ach that
B. VPL □
control intake of food and stim ulate growth:
C. Anterior □
A.
Orexin (NBE P 15,13; AIIMS 10) C l
D. Posterior □
B.
Insulin like growth hormone □ 103. Location of osm oreceptor is OR Injection of
C.
Cholecystokinin □ hypertonic saline in which region of hypothalam us
D.
Ghrelin □ produces intense thirst: (AIIMS 07, 02)
94. Regarding Gherlin w hich of false- (NBE P 13) A. Posterior region □
A. Secreted from oxyntic cells □ B. Paraventricular □
B. Increases fat deposition C] C. Preoptic nucleus of anterior hypothalamus □
C. Stimulates appetite □ D. Supraoptic □
D. Secretion increased in anorexia □ 104. Drinking can be induced by: (AIIMS 04)
95. The cell bodies of orexinergic neurons are present A. Electrical stimulation of the posterior
in: (AI 06) hypothalamus □
1
A. Locus ceruleus L -j B. Osmotic stimulation of supraoptic nucleus □
B. Dorsal raphe □ C. Lesions in the paraventicular nucleus □
C. Lateral hypothalamic area iJ
D. Neuronal lesion of the preoptic nucleus □
D. Hippocampus !” J 105. Vasopressin induced thirst is through- (NBE P 13)
A. Subfomicial nuclei □
Neurotransmitter
B. Posterior hypothalamus osmoreceptors □
96. Which is not a peptide neurotransmitters- C. Pretectal nucleus □
A. Enkephalin (NBE P 13) D
B. Substance P ij Temperature Regulation
C. Endorphin n
106. Temperature at which regulating mechanism does
D. Serotonin iC! not work- (NBE P 13)
97. All are true about neuropeptide Y except: A. 40°C □
A. Consist of 36amino acids (AIIMS 10) □ B. 35°C □
B. Decreases thermogenesis □ C. 30°C □
C. Decreased in starvation □ D. 25°C □
D. Same effect like melanocorticotropin u 107. Hyperthermia- (NBE P 13)
98. Substance P is increased in response to pain in A. Temperature>41.5 □
periphery by which of the following? B. >40 with autonomic dysfunctuion □
A. Mast cells (AIIMS 09) □ C. No change in hypothalamic thermostat □
B. Endothelium i 'l
D. One which normalizes with antipyretic □
C. Plasma □ 108. Function of preptic nucleus of hypothalamus-
D. Nerve terminals □ A. Temperature regulation (NBE P 13) O
99. The main excitatory neurotransmitter in the CNS is B. Thirst □
A. Glycine (PGI 04, AI 04) □ C. GI stimulation □
B. Acetylcholine □ D. Satiety centre □
C. Aspartate □ 109. Lesion of preoptic nucleus of hypothalamus causes-
D. Glutamate □ A. Hyperphagia (NBE P 13) □
100. The inhibitory neurotransmitter in CNS neurons is B. Hyperdypsia □
A. Glutamate (AIIMS 04) □ C. Hyperthermia □
B. Aspartate □ D. Hyperglycemia □
Physiology: Central Nervous System h 9

110. W hich of the following nucleus controls the A. Liver F)


circadian rhythm ? (NBE P 13) B. Spleen ■1
A. Supraoptic nucleus □ C. Heart □
B. Paraventricular nucleus □ D. Small intestine F1
C. Suprachiasmatic nucleus □ 121. Non-shivering therm ogenesis is m ediated by:
D. Premamillary nucleus □ A. ai receptor. (NBE P 15,13; PGI 0 3 ,14,13) □
111. Vasopressin is secreted by- (NBE P 13) B. ($2 receptor
A. Supraoptic □ C. p3 adrenergic receptor &UCP1
B. Preoptic □ D. UCP-2
C. Paraventricular □ 122. All the follow ing can occur in a neonate for heat
D. Posterior nucleus □ production except: (AIIMS 12)
112. A/E is under the control of hypothalam us: A. Shivering 1
A. Increases heart rate with exercise □ B. Breakdown of brown fat withadrenaline
B. Food intake (AIIMS 05, AI 07) □ secretion '
C. Pituitary hormone (hypophyseal) regulation □ C. Universal flexion like a fetus
D. Temperature regulation □ D. Cutaneous vasoconstriction U!
113. Sweating a result of exertion is m ediated through 123. The hormone associated with cold adaption is:
OR D uring acclim itisation, decreased sw eating is A. Growth hormone (AIIMS 03) IF
due to down regulation of: (NBE P 14,13) B. Thyroxine F1
A. Adrenal hormones (PGI 15,13,09) □ C. Insulin.
B. Sympathetic cholinergic □ D. Melanocyte Stimulating Hormone. -
C. Sympathetic adrenergic (ACh) □ 124. True regarding animals that are chronically exposed
D. Parasympathetic cholinergic (NA) □ to cold? (AIIMS 11)
114. H eat loss from the body depends m ostly on: A. Increased sympathetic stimulation rJ
A. Thermoregulatory center (AI 2K) □ B. Increased vagal action □
B. Warming of air during inspiration □ C. Increased insulin levels in blood
C. On the environmental temperature □
D. Decrease blood supply to adipose tissue i
D. Radiation and evaporation □
125. A 10 degree decrease in tem perature causes decrease
115. M ain m echanism in therm oregulation-heat loss is-
in cerebral m etabolic rate by: (AIIMS 02)
A. Radiation (NBE P 13) □
A. 10% □
B. Evaporation □
B. 30% □
C. Conduction □
C. 50% □
D. Convection □
D. 70% □
116. The first physiological response to high
126. In hum an being, the least useful physiological
environm ental tem perature is: (AI 05)
response to low environm ental tem perature is:
A. Sweating. □
A. Shivering (AI 06) □
B. Vasodilatation. □
B. Vasoconstriction
C. Decreased heat production. □
C. Release of thyroxine 1
D. Non-shivering thermogenesis. □
D. Piloerection □
117. U nder physiological conditions heat acclim atization
127. W hich of the following site doesn't contain brown
is accom plished A/E. (AIIMS 08)
adipose tissues? (AIIMS 15)
A. Decreased Renal Blood Flow □
A. Scapula -l
B. Increased urine sodium □
B. Subcutaneous tissue -j
C. Increased aldosterone secretion □
C. Around blood vessel □
D. Excessive sweating □
D. Around adrenal cortex IF
118. The physiological effect in unacclim atised person
128. Adrenergic beta-receptors having lipolysis property
suddenly exposed to cold is: (AIIMS 01)
in fat cells is: (AIIMS 15)
A. Tachycardia □
A. Alpha-1 1j
B. Shift of blood from shell to core □
B. Alpha-2 F1
C. Non shivering thermogenesis □
C. Beta-1 ~-
D. Hypertension □
D. Beta-3
119. Nonshivering thermogenesis in adults is due to
129. A dverse effects of hypotherm ia are all except:
A. Thyroid hormone (AI 02) □
A. Cardiac arrhythmias (AIIMS11) > ■
B. Brown fat between the shoulders □
B. Renal failure □
C. Noradrenaline □
C. Decreased peripheral resistance □
D. Muscle metabolism □
D. Reversible coagulopathy □
120. Endogenous non-shivering therm ogens are secreted
by all e x c e p t: (AI 03)
10 ■ A Complete Review of Short Subjects

Cells of CNS 138. Low CSF protein may be seen in all of the following
conditions, Except: (AI 10)
130. Features of Neuroglia cells include a ll except: A. Recurrent Lumbar Puncture □
A. Protoplasmic astrocytes are found in grey B. Hypothyroidism □
matter (NBE P 15,13; AI 08) □ C. Pseudotumor cerebri □
B. Oligodendrocytes are derived from ectoderm and D. Infants □
form myelin □ 139. Regarding CSF all the follow ing statements are true
C. Microglia are mesodermal in origin □ except (AIIMS 07)
D. Central neuroglial cells are derived from A. Its pH is less than blood □
Schwann Cells □ B. No neutrophils present normally □
131. Function of microglia in CNS: (NBE P 15; PGI 03) C. Formed by arachnoid villi □
A. Phagocytosis (like macrophage) □ D. Persistent leakage can cause headache □
B. Myeline synthesis □ 140. CSF pressure is mainly regulated by:
C. Fibrosis □ A. Rate of CSF formation (AIIMS 12,11, 08) □
B. Rate of CSF absorption □
C. Cerebral blood flow □
Nerve Regeneration
D. Venous pressure □
141. True about cerebrospinal fluid: (PGI 12)
132. A politician is shot in the back at level of D8
A. Produced by choroid plexus □
vertebral body. Im m ediately after the shot he loses
B. Travels from subarachnoid space to third
all the sensation below the level of transaction. He
ventricle □
is put on treatment. No recovery is noted even after
C. Absorbed by arachnoid villi □
proper treatm ent. All of the follow ing are the
possible reasons for his non recovery, except: D. Flow from spinal cord to brain □
A. Glial scar formation (AIIMS 12) □ 142. Concentration of potassium in children's CSF is:
B. Absence of growth factors □ A. 3 mEq/L (PGI 12) □
C. Lack of myelin inhibiting substance □ B. 5 mEq/L □
D. Absence of endoneural tube □ C. 10 mEq/L □
D. 15 mEq/L □
E. 20 mEq/L □
Cerebrospinal Fluid (CSF)
143. Enzymes found in CSF: (AIIMS 12)
A. ALP+CK-MB □
133. About CSF following statement (s) true is A/E:
B. CK+ LDH □
(NBE P 15,13;PGI 08)
C. Deaminase and peroxidase □
A. Specific gravity is 1.003-1.0008 □
D. GGT+ALP □
B. CSF/plasma glucose ratio is 0.6 to 0.8 (2/3) □
C. CSF volume is 150ml, daily production is Intracranial Tension & Transection
550ml/day & pH is 7.33 □
144. All are seen in cerebral herniation, except:
D. CSF/plasma protein ratio is 0.3 □
A. Neck stiffness (AIIMS 01) □
134. CSF pressure (lumbar) - (AIIMS 2K)
B. Loss of consciousness □
A. 70 -180 mm CSF □
C. Pupil dilatation □
B. 50 -100 mm CSF □
D. Autonomic disturbances □
C. > 200 mm CSF □ 145. Increase in intracranial pressure is NO T associated
D. 150 - 200 mm CSF □ with: (AIIMS 14,08,02; NBE P 15,14)
135. CSF does not pass through - (AI 09) A. Deterioration of consciousness □
A. Ventricles □ B. Tachycardia □
B. Venous sinuses □ C. Respiratory depression □
C. Epidural spaces □ D. Increase in BP □
D. Subarachnoid space □ 146. Increased BP & decreased heart rate is seen in-
136. CSF is principally secreted by - (NBE P 15,13) A. Hemorrhage (NBE P 13) □
A. Choroid plexus (AIIMS 14) □ B. High altitude □
B. Arachnoid granulation □ C. Raised intracranial pressure □
C. Floor of fourth ventricle □ D. Anemia □
D. Periaqueductal grey □ 147. In an encephala isole preparation, the transections is
137. The normal adult CSF pressure is: done at: (AIIMS 14)
A. 1-2 mm Hg (AIIMS 03) □ A. First cervical spinal segment □
B. 6-12 mm Hg □ B. Level of medulla □
C. 15-30 mm Hg □ C. Midpontine level □
D. 730 mm Hg □ D. Midcollicular level □
Physiology: Central Nervous System ■ 11

EEG 156. In the given image, wave B represents: (AIIMS 15)

148. W hat are the EEC w aves recored for parietooccipital


rgion with subject awake & eye closed- (NBE P 13) EMG

A. Alpha waves □ EEG


B. Beta waves □
C. Delta waves □
A B C D
D. Theta waves □
A. REM □
149. True about EEG (Buerger) alpha (a) wave is:
B. NREM □
A. Frequency is 8 -1 3 /sec. (PGI 2K) D
C. Quiet wakefulness □
B. Voltage is 50 microvolts usually □
D. Awake □
C. Seen in awake patients at rest with eyes closed
157. About paradoxical sleep. True is: (AIIMS 07)
and mind wandering □
A. Prominent beta waves □
D. These are Buerger (EEG) curves seen in
B. Prominent alpha waves □
hippocampus. □
C. Also known as NREM sleep. □
E. Found in emotional stress. □
D. Low amplitude, mixed frequency waves. □
150. True about EEG (Buerger) P w ave is /are (PGI 01)
158. Nightmare is seen in : (AI 04)
A. Frequency is 13-30 per second □
A. REM sleep □
B. Frequency is < 13 per second □
B. Stage II NREM sleep □
C. Predominant activity in REM sleep □
C. Stage IV NREM sleep □
D. Predominant activity in NRFM sleep □
D. Stage I NREM sleep □
151. Beta w aveform s in electroencephalogram designate
159. W hich one of the follow ing phenomena is closely
w hich of the following states of the patient:
associated with slow wave sleep:
A. Deep Anaesthesia (AIIMS 05) □
A. Dreaming (AIIMS 04) □
B. Surgical Anaesthesia □
B. Atonia □
C. Light Anaesthesia, eyes closed, relaxed □
C. Sleep walking □
D. Awake/alert state □
D. Irregular heart rate □
152. D elta waves are seen in : (AI 07)
160. The normal adult human electroencephalogram
A. Deep sleep □ (EEG): (AIIMS 04)
B. REM sleep □ A. Will not show high frequency waves during
C. Awake state □ Stage 3 sleep □
D. Stage IN REM sleep □ B. Shows alpha rhythm when aperson is awake but
153. EEG rhythm having low est frequency is-
inattentive □
A. Alpha (DNB 14, A l 04) □
C. Has lower frequency waves during mental
B. Beta □ activity □
C. Delta □ D. Is predominated by large amplitude waves
D. Theta □ during REM sleep □
154. Slow wave sleep associated with: (PGI 06)
161. Polysomnography contains all of the follow ing tests
A. Dreams □ except: (AIIMS 16)
B. Cardiac arrhythmia □ A. Electroencephalography □
C. Penile intumescence □ B. Pulse oximetry □
D. Delta activity □ C. Electrooculography □
155. The EEG recorded show n below is norm ally
D. Arterial pCC>2 measurement □
recordable during w hich stage of sleep?
(AIIMS 15,14,03) Pain / Analgesia & Receptors
K Compl ex

Spi ke 14 Hz i 162. Free nerve endings carrying nociceptive fibers are


Sl eep spindles seen in: (AIIMS 14)
A. Intestine □
B. Spleen □
C. Liver □
D. M esentery □
163. Nerve ending sensitive to noxious stimuli are
A. Stage I □ present in all except: (PGI 09)
B. Stage II □ A. Stomach □
C. Stage III □ B. Intestine □
D. Stage IV □ C. Mesentry q
12 ■ A Complete Review of Short Subjects

D. Liver □ 173. Massage and the application of linim ents to painful


E. Brain □ areas in the body relieves pain due to: (AIIMS 04)
164. Pain-sensitive intracranial structure is: A. Stimulation of endogenous analgesic system □
A. Piamater (NBE P 15,13; PGI 15; AI 06) □ B. Release of endorphins by the first order neurons
B. Pial vassels □ in the brain stem □
C. Duramater □ C. Release of glutamate and substance P in the
D. Brain matter □ spinal cord □
165. Vanilloid receptors are activated by: D. Inhibition by large myelinated afferent fibres □
A. Pain (AIIMS 0 6 )0 174. Which of the following is effect of p (mu) receptor
B. Vibration □ of opioids: (AIIMS 02)
C. Touch □ A. Sedation □
D. Pressure □ B. Dysphoria □
166. Perception of norm al sensory stim uli as painful is C. Miosis □
called (AIIMS 08) D. Bradycardia □
A. Hyperalgesia □ 175. Na+ & /or K+ is /are involved in mechanism of action
of which of the following receptor (s): (PGI 09)
B. Allodynia □
A. GABAA receptor □
C. Hyperpathia □
B. Nm receptor □
D. Causalgia □
C. Nn receptor □
167. Repetitive stim ulation increases pain sensation, The
D. 5-HT3 receptor □
probable cause is:
E. Dopamine □
A. Hyper-sensitization (AIIMS 09) □
176. All of the follow ing are thermal modalities except:
B. Decreased reflex time □
A. Cold receptor (PGI 05,14,15) □
C. Increase in threshold of pain □
B. Warm receptor □
D. Decreased receptor area □
C. Pain receptor □
168. True about Visceral pain: (PGI 11)
D. Stretch receptor □
A. It is poorly localized □
E. Pressure receptor □
B. Resembles "fast pain" produced by noxious
177. Which of the following phrase adequately describes
stimulation of the skin □
Pacinian corpuscles i.e. Pacinian corpuscles transmit
C. Mediated by B fibers in the dorsal roots of the which sensation:: (AIIMS 04)
spinal nerves □ A. A type of pain receptors □
D. Causes relaxation of nearby skeletal muscles □ B. Slowly adapting touch receptors □
E. Shows relatively rapid adaptation □ C. Rapidly adapting touch receptors □
169. Ascending pain pathw ay is inhibited in dorsal D. Located in the joints □
midbrain by encephalin and: (PGI 10) 178. M eissenePs corpuscle are for- (NBE P 13)
A. 5-HT (serotonin) □ A. Touch □
B. Noradrenaline □ B. Temperature □
C. Substance P □ C. Pressure □
D. Glutamate □ D. Progrioception □
E. Adenosine □ 179. The mechanostretch receptors in joints and
170. Hot w ater bottle relieves pain of abdom inal spasm ligaments are: (AIIMS 14)
by: (AI 10) A. Fast adapting □
A. Simulation of adrenergic fibers □ B. Slow adapting □
B. Stimulation of cholinergic fibers □ C. Non adapting □
C. Inhibition of cold receptors □ D. Adapt differentially for different stresses □
D. Inhibition of heat receptors □ 180. During hip replacement, loss of joint and ligament
171. M echanism of analgesia is by; (PGI 05) receptors leads to: (AIIMS 14)
A. Nociceptin stimulation □ A. No loss of sensation □
B. Nocistatin stimulation □ B. Loss of all sensations □
C. O.L.R.I □ C. Joint sensation lost in stable position □
D. Anadamide receptors □ D. Decrease in movements & decrease in sensation □
E. Nicotinic & cholinergic receptors □ 181. The increase in threshold of a receptor when a series
172. Transcutaneous Electrical Nerve Stim ulation of stimuli of subthreshold intensity are applied in
(TENS) is based on: (AIIMS 13) succession is called OR Increase in threshold level on
A. Central pain □ applying subthreshold, slowly rising stimulus is k/a:
B. Referred pain □ A. Adaptation (AIIMS 08, 0 6 )0
C. Gate controlled theory of pain □ B. Accomodation □
D. Allodynia □ C. Initiation □
D. Resistance □
Physiology: Central Nervous System ■ 13

Spinal Cord & Tracts A. Vibration & Proprioception □


B. Temperature / Crude Sense & Pain □
182. Kinesthetic sensation is: (AI 04) C. Fine touch □
A. Transmitted by the (3-type of sensory nerve □ D. Position sense □
B. Located in Merkl's disc □ 192. Concious proprioception is carried by: (NBE P 14)
C. Transmitted by Meissner's corpuscles □ A. Dorsal column fibres (AI 03)□
D. Means abnormal perception of sensation □ B. Anterior spinothalamic tract □
183. Pyramids are formed by- (NBE P 13) C. Lateral spinothalamic tract □
A. Arcuate nucleus □ D. Vestibular tract □
B. Vestibular nuclei □ 193. Which of the follow ing sensation are transmitted by
C. Interstitial cells of cajal □ the Dorsal Tract/ Posterior column: (AIIMS 15)
D. Lateral corticospinal tract □ A. Fine touch, Propioception (NBE P 15,14) □
184. UM N includes: (PGI 02) B. Pain (PGI 14) □
A. Pyramidal cells □ C. Temperature □
B. Peripheral nerves □ D. All of the above □
C. Anterior horn cells □ 194. Which of the follow ing tracts is concerned with pain
D. Glial Cells □ & temperature (NBE P 15,13; AI 08; AIIMS 14)
E. Schwann cells □ A. Pyramidal tract □
185. True about lateral corticospinal tract: B. Anterior spinothalmic tract □
A. Crossed (PGI 06) □ C. Lateral spinothalmic tract □
B. Uncrossed □ D. Dorsal spinocerebellar tract □
C. Stops in midthoracic region □ 195. A lesion of ventrolateral part of spinal cord w ill lead
D. Crossed at midspinal level □ to loss (below the level of lesion) of: (AI 03)
E. Only 20% crossed □ A. Pain sensation on the ipsilateral side □
186. Cortcospinal tract lesion leads to: (PGI 10) B. Proprioception on the contralateral side □
A. Spaticity □ C. Pain sensation on the contralateral side □
B. Extensor plantar response □ D. Propriception on the ipsilateral side □
C. Cogwheel rigidity □ 196. An anterolateral (ventrolateral) corodotomy
D. Resting tremors □ relieving pain in right leg is effective because it
interrupts the: (NBE P 14,13)
E. Exaggerated tendon reflexes □
A. Left dorsal column (Al 12, AIIMS 14, 05) □
187. Features of pyram idal tract lesion are all except
A. Clasp knife rigidity (AI 08) □ B. Left ventral spinothalmic tract □
B. Involuntary movements □ C. Left lateral spinothalmic tract □
C. Positive Babiniski sign □ D. Right lateral spinothalmic tract □
D. Exaggerated reflexes & increased tone □ 197. All are true about brown sequard syndrome
A. Ipsilateral loss of joint sensation (PGI 09) □
188. Following are the features of corticospinal
involvement except: (AI 03) B. Contralateral loss of joint sensation □
A. Cog-wheel rigidity □ C. Ipsilateral loss of pain & temperature □
B. Spasticity □ D. Segmental sign are bilateral □
C. Planter extensor response □ E. Contralateral loss of vibration □
D. Exaggerated deep tendon reflexes. □ 198. All are seen in Brown - Sequard syndrome except
189. Amyotrophic lateral sclerosis involves: (AI 03) A. I/L Pyrimidal tract features (AIIMS 1 1 ) 0
A. Upper motor neuron and lower motor neuron B. C/L Dorsal column □
both □ C. C/L Spinothalamic tract □
B. Posterior column only □ D. I/L Plantar extensor □
C. Lower motor neuron only □ 199. U nlikely to be involved in lesion of anterior spinal
artery is: (PGI 09,15)
D. Raphae nucleus □
A. Pain and temp □
190. True about medial lemniscus system; (PGI 12)
A. Formed from fasciculus gracilis and fasciculus B. Vibration and proprioception □
cuneatus □ C. Pyramidal tract □
D. Sphincters □
B. Carries discriminative touch and
proprioception □
C. Convey pain and temperature □ Reflexes
D. Joins spinothalamic tract □
E. Decussate at lower medulla □ 200. Salivation by dog seen when food is given along
191. Posterior column lesion which of the follow ing is with ringing of bell is : (AIIMS 05)
not affected: (AIIMS 15,14; PGI 08, 14,15) A. Conditioned reflex □
14 ■ A Complete Review of Short Subjects

B. Reinforcement □ 208. Post ganglionic fibers are longer in? (NBE P 13)
C. Habituation □ A. Para sympathetic system □
D. Innate reflex □ B. Sympathetic system □
201. Decrease reflex response after repetitive C. Both have same length □
stimulation- (NBE P 13) D. It is variable □
A. Summation □ 209. All are effects of sympathetic stimulation, EXCEP.
B. Fatigue □ A. Increased conduction velocity (NBE P 13) D
C. Irradiation □ B. Increased heart rate (AIIMS 03,14) □
D. Occlusion □ C. Increased refractory period □
202. A/E are true properties of spinal cord reflex : D. Increased contractility of heart □
A. Memory (AIIMS 04) □ 210. During Flight or Fight reaction, which of the
B. Summation □ follow ing is responsible for increase in local blood
C. Delay □ flow: (AI 02)
D. Fatigue □ A. Sympathetic system mediated Cholinergic
203. The first reflex response to appear as spinal shock release □
wears off in hum ans is: (AI 06, NBE P 13) B. Local hormones □
A. Tympanic reflex □ C. Para-sympathetic cholinergic □
B. Withdrawal reflex / Bulbocavernous □ D. Endocrine factors only □
C. Neck righting reflex □ 211. a - adrenoreceptor stimulation effects are :
D. Labyrinthine reflex □ A. Vasoconstriction (PGI 0 4 )0
B. Vasodilatation □
Sensory Reception C. Broncho constriction □
D. Alveolar bronchoconstriction □
204. The distance by which two touch stimuli m ust be E. Bronchodilatation □
separated to be perceived as two separate stimuli is 212. Not a direct effect of Ach- (NBE P 13)
greastes at- (NBE P 13) A. Decrease BP □
A. The lips □ B. Increase contraction of heart □
B. The palm of the hand □ C. Decrease heart rate □
C. The back of scapula □ D. Decrease conduction □
D. The dorsum of the hand □ 213. Sympathetic stimulation increases blood flow to all
205. If a single spinal nerve is cut, the area of tactile loss except: (AIIMS 13)
is always greater than the area of loss of painful A. Skin □
sensations, because: (AIIMS 04) B. Coronary circulation □
A. Tactile information is carried by myelinated fast C. Cerebral □
conducting fibres □ D. Renal □
B. Tactile receptors adapt quickly □ 214. Catecholamines- (NBE P 13)
C. Degree of overlap of fibres carrying tactile A. Decrease gut contraction □
sensation is much less □ B. Bronchoconstriction □
D. In the primary sensory cortex tactile sensation is C. Decrease plasma glucose □
represented on a larger area □ D. Inhibit lipoysis □
215. Epinephrine reduces insulin by - (NBE P 13)
Autonomic Nervous System A. Alpha action predominantly □
B. Beta action predominantly □
206. Parasym pathetic supply is from- (NBE P 13) C. Alpha and beta □
A. C234 □ D. Muscarinic receptors □
216. All is/are true about ANS except: (AIIMS 09)
B. T234 □
A. Ciliary, otic, submandibular and pterygopalatine
C. L234 □
ganglia contain postganglionic parasympathetic
D. S234 □
207. True regarding autonom ic system is: (AIIMS 08, 06) neurons □
A. High centre of integration is medulla B. Celiac and superior & inferior mesenteric ganglia
oblongata □ contain post ganglionic sympathetic neurons □
B. Conduction autonomic fibres is sameas in C. Acetyl choline is neurotransmitter for both pre &
somatic motor fibres. □ post ganglionic synapses in parasympathetic
division as well as preganglionic sympathetic
C. Preganglionic parasympathetic fibres are
more lengthy. □ neurons. □
D. NE is the NT of all postganglionic sympathetic
D. Ratio of preganglionic and post ganglionic
neurons including merocrine sweat glands. □
fibres is 20:1 □
Physiology: Central Nervous System ■ 15

217. Nerve fibres innervating sw eat glands release the 220. Tyram ine acts by (AIIMS 03)
following at their endings- (NBE P 13) A. Inhibit acetylcholinesterase □
A. Noradrenaline □ B. Stimulate di receptor □
B. Acetycholine □ C. Release norepinephrine □
C. Doparmine □ D. Stimulate (3 receptor □
D. Histamine □ 221. True about Galvanic Skin Response: (PGI 09)
218. Simulation of post ganglion sym pathetic neurons A. Indicator of thermal sweating □
leads to: (PGI 02) B. T by sweating because of sweat has electrolytes □
A. Fast EPSP □ C. Fall when sympathetic activity is high □
B. Slow EPSP □ D. Fall is associated with yogic practice □
C. Fast IPSP □
D. Slow IPSP □
E. Very slow EPSP □
219. Slow IPSP in autonom ic is generated by: (PGI 05)
A. Nicotinic cholinergic □
B. Muscarinic cholinergic □
C. Dopamine □
D. GnRH □
E. Adrenaline □
16 ■ A Complete Review of Short Subjects

ANSWERS AND EXPLANATIONS:

Higher Function

1. B i.e. Frontal lobe [Ref: Ganong 24/e p. 294 [

Frontal lobe is the largest lobe. It is responsible for attention, concentration^, personality, behaviour, planning,
judgem ent, motivation, organization and word choice (expressive language)Q.

2. A i.e. Larger brain [Ref: Ganong 22/e P-272]


r ■ ■ .

Human brain is more intelligent than monkey's brain d/t larger brain, greater size o f association area and higher
brain/body w eight ratio®.

3. C i.e. Use fatty acid in starvation [Ref: NMS biochemistry 3/e P-516, Lippincotts 2/e P- 294,188, Ganong 22/e P-283[

Brain uses ketone bodies (not fa tty acid) in starvation®.

4. D i.e. Head and Face 5. D i.e. Loss of tactile localization & two point discrim ination
[Ref: Harrison 16/e P-143; Harrison 15/e P-131; Ganong 24/e P-168]

f t
Largest representation in somatosensory area I (post central gyrus) is for hand (thumb especially) and face (esp
mouth parts concerned with speech i.e. lips, tongue etc)Q. Trunk & back has very small representation.
A blation o f som atosensory area I (SI) o f cerebral cortex leads to loss o f tactile localization and tw o point
discrimination®.

6. D i.e. Nucleus cuneatus [Ref: Ganong 22/e P-147;Textbook of Human neuroanatomy I.B Singh 6/e P 12]
r ■- - — ■ ■■■ —

The middle part of cuneate nucleus, the pars rotunds which receives impulses from digits, palm, dorsum o f hand &
forearm is specific fo r stereognosis®.

Stereognosis

Stereognosis is determination of texture & size of an obect, is function of cerebral cortex.


The sensory information of stereognosis to brain is carried by p oster ior column®. Which includes tractus gracilis & tractus

[Mn: "G racilis for low er segm ent & cuneatus for upper segm ent - G comes after C"]

7. D i.e. Tem poral lobe [Ref: Ganong 23/ep 298]

Right inferior temporal lobe (in right handed person) stores visual inputs (representation) of objects especially faces. So
lesion in this area causes prosopagnosia i.e. inability to recognize fam iliar facesQ. However, they can recognize people
from their voice or names. They can also show autonomic responses (anxiety) on seeing familiar faces (but not on seeing
unfamiliar faces).

Emotions

8. D i.e. A nterior nucleus of thalmus


Physiology: Central Nervous System ■ 17

Papez Circuit

Fomix of Mammillary __ Anterior Cingulate gyrus


->
Hippocampus * body thalmic nucleus
(Limbic system) (Hypothalmus) (Thalmus) (Limbic system)
4_____
★ So the limbic system affects, and is affected by thalmus & hypothalmus
via papez circuit. Anterior nucleus o f thalmus® is involved in Papez circuit.

9. B i.e. Amygdala 10. A i.e. Emotion B i.e., Memory C i.e., Higher function
11. B i.e. Limbic system; D i.e. Hippocampus 12. B i.e. M edial forebrain bundle
13. B i.e. Abolished by Decortication 14. A i.e.Preoptic nucleus
[Ref: Ganong’s 24/e p 309; Guyton 12/e p 717; Gallagher & Nelson 2/e p 348; Chaudhary 6/e p 354]

Am ygdala is the w indow through which lim bic system sees the place o f person in the world®. f t
Reward center (ie area of brain responsible for self stimulation reward on induction) includes m edial forebrain
bundle®, especially in lateral and ventro m edial nuclei o f hypothalamus®.
The hypothalam us and limbic system® are responsible for instinctual behaviour & emotions. Limbic system is
responsible for emotions (em otional behavior & m otivational drives) and memory (higher functions)®.
Sham (false) rage is an outburst of pathological (inappropriate, uncontrolled & violent) anger & fear in decorticate
animals on mild peripheral stimulation. It can be caused by stimulation of punishment center of hypothalamus and
presents with features o f sym pathetic stimulation®.
Preoptic nucleus (MPOA) o f hypothalamus® is most closely related to sexcual activity in males.

15. C i.e. Recruitment by multiplication of neurons 16. B i.e. Hippocampus


17. C i.e. The site for storage of long term memory is in temporal lobe [Ref: Ganong 24/e p 283-90; Guyton 12/e p705-708;Berne
& Levy 6/e p210-14;Harrison 16/e p-2394; journal of Neuroscience vol.16, no-10 p. 3189-98]

- Mechanism of learning and long term memory include gene activation leading to Tprotein synthesis & spatial
reorganization (restructuring) of synapse changing their sensitivity by increasing the num ber of transmitter vesicles /
vesicle release sites /level of neurotransmitter at synapse /presynaptic terminals as well as changes in dendritic spine
that permit transmission of stronger signals. Change in num ber of neuron & their connectivity is significant in initial
week / month / years of life but this recruitment o f neurons is from already existing p o ol (not by multiplication)
mainly®
- Hippocampus (in medial temporal lobe) is the site where short term memory is processed and converted into long term
memory®. While the encoding process for short term explicit memory involves hippocampus, long term memories are
stored in various parts o f neocortex®. Hippocampus causes consolidation of long term memoryQ.
- Short term memory lasts as long as the person continues to think. Intermediate memory results from temporary changes
in synapses, whereas structural changes occur in synapses during development of long term memory.

18. C i.e. Relation of one stimulus to another 19. A i.e. Procedural memory
18 " A Complete Review of Short Subjects

Classification (Types & Forms) of Memory


Physiological classification of memory, classifies it into

hippocampus (& medial temporal lobes of brain) for retention It does not involve awareness
It is for factu al knowledge about people, places and things. It involves memory of (consciousness) and its retention does
various details of an integrated thought, example includes memory o f an important not involve processing in
event or experience. Such as memory of hippocampus®
1) Surroundings, 2) Causes of experience, 3) Meaning of experience, 4) Time It is important for training reflexive
relationship and 5) Memory of one's deductions that were left in the person's motor or perceptual skills such as all
mind. skills developed for hitting a tennis /
Explicit memories that are required initially during activities like learning cricket ball, including automatic
bicycle riding can become implicit once the task is thoroughly learned. memories to ( 1) sight the ball, (2 )
Explicit memory can be divided into calculate the speed of ball &
positioning of bat, (3) deduce rapidly
the motions of body to hit the ball- all
Semantic (Fact) Memory Episodic (Event) Memory
these activated instantly based on
- For facts like words, rules & -For event and experiences previous learning of game - and then
language - May be short to long term moving on to next stroke while
-M ay be short to long term -Anterior thalamic nuclei, prefrontal forgetting the details of previous
- Inferolateral temporal lobe is also cortex & m edial tem poral lobe is stroke.
involved (along with also involved It can be of 4 types
hippocampus)

Priming Procedural Memory Associative Learning Non-associative ie


_________ Learning______
Recognition of Includes habits & skills, It involves classical & operant It involves habituation &
words or objects by which after acquisition, conditioning in which relationship sensitization and is
prior exposure to become unconscious and between one stimulus and another dependent on various
them is dependent automatic. Procedural is leamtQ. It is dependent on reflex pathways
on memory is processed in amygdala for emotional responses
neocortex striatum (part o f basal and cerebellum for motor (skeletal
ganglia)Q_______________ musculature) responses

Sleep

20. A i.e. Hypothalamus


Ref: Harrison's 17lh/e p.172-73; Textbook of Physiology Gabriel C Ezeilo 472-80; Guyton l l lh/e p. 740-41; Ganong 23rd/e p. 236-39

Hypothalmic ventro lateral preoptic (VLPO) neurons play a key executive role in sleep regulation®.

21. B i.e. W ill be prolonged to greater than 24 hours cycle length » A i.e. W ill remain unchanged at 24 hrs
22. C i.e. Suprachiasmatic nucleus [Ref.Ganong 24/e p.278-79; Best & Taylor 13/e p.869-70,657,1205; Berne & Levy 6/e p.207-8, 655]^
-----------------------------------------------------------------------------------------------------------------------------------------------
- In the liver circadian rhythm is influenced by pattern of food intake, but in alm ost all other cells the circadian rhythm
is entertained by paired suprachiasm atic nuclei®.
- Endogenous (built in or self sustained) circadian periodicity o f sleep w ake cycle is about 25 hours (i.e. slightly greater
than 24 hours). It is entrained (adjusted or synchronized) by external (environmental) cues (or Zeitgebers = time
givers) mainly the day-night (or light-dark) cycle to attain 24 hour periodicity. Therefore, if all external or
environmental cues (Zeitgebers) are removed or in a constant environment, the sleep wake cycle becomes free
running and slightly drifts away from 24 hour cycle to become > 24 (to 25) cycle.
Physiology: Central Nervous System ■ 19

Cerebral Dominance

23. D i.e. Written and Spoken Language 24. C i.e. Representative hemisphere, right
25. C i.e. Dyscalculia [Ref: Berne & Levy 6/e p 213-208; Ganong 24/e p 291-294; Guyton 12/e p 701-704]

- Categorical (dominant) hemisphere (ie left lobe in most cases) is responsible for sequential - analytic processes such
as language (understanding language, verbal memory, motor speech, written & spoken language) and mathematical
ability. Therefore lesions of left (dominant) side produce aphasias (receptive & expressive), dyslexia and dyscalculia.
- Representational (non dominant mostly right) hemisphere is responsible for visuo-spatial orientation & musical (art)
awareness. Its lesion causes anosognosia and dysgraphia.

Aphasia

26. D i.e. Inferior frontal gyrus 27. A i.e. Word formation 28. B i.e. Verbal expression
[Ref: Fuller 2^/10; Harrison 16/e P -146,147; Ganong 22/e P-274-275]

Broca's ( nonfluent/motor'/ expressive)


aphasia is caused by lesion in inferior fron tal
gyrus ®. It is a disorder o f w ord finding
(form ation) resulting in inadequate verbal
expression ®.

B ro c a ’s
a re a

Hearing & Vision

29. A i.e. ICF

Perilymph Endo lymph


Rich in Na+ & resembles ECF (& CSF)® Rich in K+ and resembles ICF®
- Communicates with CSF via aqueduct of cochlea - Secreted by stria vascularis (of cochlea) and dark
- Fills space between bony & membranous labyrinth i.e. cells (of utricle & ampulatted ends of SCCs)
scale vestibule & tympani - Fills membranous labyrinth including sale media
(cochlear duct)

30. A i.e. K+ influx 31. C i.e. Rotation [Ref: Ganong 24/ep 202-205]

Inner Hair cells (in auditory pathway) depolarize due to influx of K + ion ( » » ) and Ca++®

Hair cells (HCs) Sensory Receptors of Ear

- HCs (6 Patches) are mechanoreceptors, which signal .


Hearing In organ of Corti
- Base of HCs is in contact with afferent neuron and
Horizontal acceleration® In Utricle®
supporting cells (surmounted by otolith membrane
Verticle acceleration® In Saccule
embded with calcium carbonate crystals or otoliths).
Rotational acceleration® (A patch) in all 3 SCCs®
- Apical end of HCs have 30-150 rod shaped processes
(hairs or stereocilia). Kinocilium (not found in cochlea) is largest non motile, true hair process with clubbed end. It has
1 central and 9 circumferential pairs of microtubules. All stereocilia are of same height (along perpendicular axis) but along
an axis towards kinocilium they increase in height progressively
- Tips of adjacent stereocilia are connected by tip links controlling opening of mechanosensitive cation transduction channel in
taller stereocilia.
20 ■ A Complete Review of Short Subjects

- Movement of shorter stereocilia towards taller stereocilia (i.e. deflection of stereocilium towards kinocilium) cause
opening (or increased open time) of transduction channel resulting in influx of K+ (Most abundant endolymph
cation)*? and Ca++(In lesser amount). This produces depolarization of hair cells. RMP of hair cells is -60 mV and it is
decreased to-50 mv.
- Depolarization of HCs cause opening of voltage gated (not transduction) Ca++ channels at their base allowing more
Ca++ influx resulting in release of NT-glutamate. Finally voltage sensitive K+ channels at base cause K+ efflux
restoring Resting membrane potential (RMP).
- But a /1 recent text, myosin based molecular motor in taller cilia moves the transduction channel towards base and so
releasing tension in tip link. This closes Ca++ channel and restores resting state.
- Movement of stereocilia away from kinocilium causes closure of transduction channel and hyper polarization*?

32. D i.e. +85 mV IRef: Guyton 11 /ep 657]

Endocochlear potential is positive (+) 80 to 100 mV*? voltage seen in cochlear endolymphatic space (i.e. b/w endolymph
& perilymph) with positive inside scala media and negative outside. It is generated by continuous secretion of K + ions
into scale media by stria vascularis.

S cala
33. A i.e. C [Ref: Ganong 24/e p 208]
V estibular vestibular
m em b ra n e |
- Sensory neurons that arborize around hair cells have Scala media
their cell bodies located in spiral ganglion (cochlear duct)
- 90-95% of peripheral process of spiral ganglion (these
sensory neurons) innervate inner hair cell*? (i.e. C in
diagram) and only 5-10% innervates more numerous
outer hair cells (B in diagram).

Basilar O u ter Internal spiral Spiral


m em b ran e hair cells tunnel tim bus

34. C i.e. Binasal disparity [Ref: Ganong 22/e P-167- 68]

True stereospsis is perceived due to binasal disparity *?.


$
35. C i.e. Layers 1,4,6 36. C i.e. Layer 4 37. B i.e. Colour vision, shape, fine details
38. A i.e. Color contrast 39. C i.e. Colour processing [Ref: Neuroanatomy by l.B.S 6/e P-233; Ganong 22/e P-160- 61] .

• Fibres from contralateral nasal hemiretina project to the layers (laminae) 1,4 and 6 o f lateral geniculate body *?.
• Most afferent fibers from the lateral geniculate body terminate in the primary visual cortex layer 4*?.
• The parvocellular pathway from layers 3 to 6 of lateral geniculate nucleus to the visual cortex, carries signals for
color vision (contrast), texture, shape and fine detail*?.
• B lobs are arranged in a m osaic in visual cortex and are concerned with colour vision.
x.

40. C i.e. Amacrine cells 41. A i.e. Bipolar cells 42. A i.e. Optic nerve [Ref: Guyton 12 ep-617: Ganong 24/ep 179-92]

Visual Pathway • Amacrine cells secrete 5 inhibitory neurotransmitters


Sensory (Photo) Rods & cones ACh*?, GABA, Glycine, Dopamine & Indolamine.
Receptors • Only ganglionic cell in retina produce propagating
1st Order Neuron Bipolar cell axon*? (in retina) action potential (always depolarizing). All other cells
2nd Order Neuron Ganglionic cell axon*? (form optic develop graded local potential which is decremental,
disc-nerve-chiasma-tract)*? non propogative & does not follow all or none law.
3rd Order Neuron LGB nerve cell axons (form optic
• Photoreceptors (rods & cones) release glutamate at
radiation) their synapse with bipolar cells (in dark)*?
Physiology: Central Nervous System b-. 21

43. B i.e. Amacrine cells [Ref: Guyton lV h/e p. 635-6371 .

[S ' l
Amacrine cells serve important function of increasing visual acuity by modulating photoreceptive signals, serving to
adjust or m aintain the relative colour & luminosity inputs under changing light conditions®. So amacrine cells regulate
and maintain relative color luminosity o f photoreceptive input under changing light conditions ®.

44. C i.e. 11-Cis - Retinalaldehyde [Ref: Lippincott 2/e P 332; Ganong 22/e P-158, Chaudhuri 5/e P-506]

In the dark phase - 11-Cis- retinal® exists combined with opsin. In the light phase - 11 trans - retinal exist combined
with opsin.

45. B i.e. Photoisomerization and hydrolysis of visual purple [Ref: Harrison- 409; Lippincott's 4,h/e p. 381- 84; Garner &
Klintworth pathology of ocular disease 3le pg- 1066-67; Robbin's 7/ed pg- 451 ]
f ™ ■■
Initiation of visual cycle is associated with condensation o f opsin w ith vitamin A aldehyde (11-cis retinaldehyde)®
whereas, initiation of visual (nerve) impulse is associated with photochemical isomerization & hydrolysis (bleaching) of visual
purple (rhodopsin) resulting in release of all trans retinal & opsin.

46. B i.e. Glutamate 47. D i.e. Vision 48. A i.e. Signal transduction
[Ref: Ganong 23rdje p. 190-92; Neuroscience: Mark, Bear, Barry 3/e p. 296; CNS Per Broda 3/e p. 188; Guyton llth /e p. 626-31, 636]

• Both rods cones release glutamate at their synapses with bipolar cells and remain in depolarized state (-40 mv
membrane potential) in darkQ. So photo transduction is a unique type of sensory transmission that is associated with
decreased neurotransmitter (glutamate) release from photoreceptors in light. Unlike other receptors, the photoreceptors
(rods and cones) & bipolar cells are hyperpolarized (instead of depolarized) and do not produce action potential®.
Instead they produce hyperpolarizing local graded potential^
• Process o f phototransduction (response to light) in rods and cones is virtually the same®. And the sequence of events
are:
Incident light —» structural change in retinene 1 of photopigment —> conformational change of photopigment opsin —>
activation of transducin (G tl) in rods /Gt2 in conesQ —> activation of photodiesterase —>-l intracellular C-GM P—»
closure of Na+ channels —>4- intracellular Na+ —» hyperpolarization —» dcreased release of synaptic transmitter
(glutamate)Q —>response in bipolar cells.

Color Vision & Taste


49. C i.e. Involves opponent color cells [Ref: Ganong 23rdje p. 195-98; Gabriel C Ezeilo p. 441, 449-50; 439, Guyton llt h /e p.638-32]

Colour vision involves opponent color coding cone cells®.

50. A i.e. 370-740 nm [Ref: Guyton 10/e P- 6431

The visible range is 350nm-600nm®

51. C i.e. Taste buds 52. B i.e. Na+channels 53. D i.e. At the posterior aspect [Ref: Ganong 24/ep 223-24]

(R) Receptors (O n * ) Substance R-Type


Salty Sodium chloiid e Ligand gated (ionotropic) channels: Amiloride sensitive sodium
(Na+) selective epithelial channels (ENaC)Q
Anterior half of tongueQ ★
Sour HCL Ligand gated (ionotropic) channel: ENaC permitting entry of
proton (H+ions)Q; H+ can also block K+ sensitive channels; HCN
Edges of tongueQ ★
(hyper polarization-activated cyclic nucleotide gated cation channel)
Sweet Glucose etc GPCR (metabotropic) receptors: T1R3 (Sac locus) family of G
protein coupled receptors (GPCR) coupling to G protein
Tip of tongueQ ★
Gustducin
Bitter Poisons GPCR (T2R family) interact with gustducin e.g. strychnine; some
are membrane permeabe & K+ channel blockers
Back of tongueQ *
Unami L-glutamate Metabotropic glutamate Receptors (M-Glu R4)
22 ■ A Complete Review of Short Subjects

54. B i.e. Olfaction-hair cells

Olfactory receptors are unique as receptor cell itself (not hair cell)Q is neuron.

Equilibrium & Basal Ganglia

55. A i.e. Macula [Ref: Ganong 23/e p. 206]

Macula (Otolith Organ = Stretch receptors) in M aintain Static equilibrium i.e. head position in reference to
Utricle & Saccule gravityQ
Cristae at ampullated ends of Semicircular Canals Respond to angular (rotational) acceleration & deaccelerationQ.

56. C i.e. Caudate nucleus /Putamen/GP 57. B i.e. Corpus striatum


58. D i.e. Putamen 59. B i.e. Subthalamic Nucleus
[Ref: Ganong's 24/e p 136-142, 243-45; Guyton 12/e p 689-94; Best & Taylor 13/e p 1194-95; Berne & Levy 6/e p 188-193]

• - 6/ 1-
- Basal ganglia consists of - caudate nucleus®, putamen, globus pallidus, subthalm ic nucleus, and substantia nigra®. ^
Striatum (term derived from the striated appearance of these nuclei) refers to the caudate nucleus and putamen.
Whereas the combination of putamen and globus pallidus is often referred to as lentiform nucleus
- Efferent fiber bundle of substantia nigra (pars compacta) transmit dopaminergic fibers to corpus striatum /striatum
(dopaminergic nigro striatal projection).
- Subthalamic nucleus (a diencephalic grey matter part of basal ganglia) is the only nucleus of basal ganglia that actually
produce excitatory neurotransmitter glutamate. Subthalamic nucleus receives inhibitory (GABA - nergic) imput from
globus pallidus external segment (GPe) and has excitatory (glutaminergic) projections to both GPe and GPi (globus
pallidus internal segment). Subthalamic nucleus plays an important role in stimulation of indirect SNpc-GPi
(substantia nigra - pars compacta - globus pallidus internal segment) pathway.
- Although the majority of striatal neurons are GABA nergic medium spiny neurons, the highest density of glutamate
receptors in basal ganglia is found in corpus striatum (putamen + caudate nucleus)®, which receives 2 main inputs
(both of which are excitatory glutaminergic) from cerebral cortex (cortico striatal pathway) and from intralaminar
nuclei of thalamus (thalamostriatal pathway).

60. A i.e. Dopamine [Ref. Ganong 22/e P-213]

Dopamine is the m ajor neurotransmitter in substantia nigra® (nigrostriatal pathway)®.

61. B i.e. Skilled movement [Ref: Ganong 23/e p. 242-57; Guyton llth /e p. 700]
62. A i.e. Planning 63. B i.e. Smoothens and coordinates movement

Basal ganglia is concerned with planning and organizing voluntary (skilled) movements^ whereas, spinocerebellum
(spinocerebellar tract) smoothen & coordinates movements^

Functions of
_________I__
Basal ganglia Cerebellum Limbic system •Gross motor function^
- Planning and programming of - Concerned with -Olfaction —>by cortico motro area
movementsQ equilibrium^ - Autonomic responses •Sensory relay station^ -
- Subconscious execution of learned - Co- ordination of (along with Thalamus
pattern of movement^ motor function^ hypothalamus) •Short term memoryQ —>
- Cognitive control of sequence of motor - Emotion of rage, fear and Hippocampus
pattern. motivation

64. C i.e. Athetosis 65. A i.e. Alziemer's disease


Disorder Basal Ganglia (Part) involved
Parkinson's Disease Dopaminergic neurons in Substantia nigra pars compactaQ (most severely
affecting putamen)
Athetosis Globus pallidusQ » > Subthalmic nucleus
Ballismus Subthalmic nuclei^
Huntington Chorea Striatum (Caudate nucleus » Putamen)
Physiology: Central Nervous System ■ 23

Dysfunction in any of three biochemical pathways of basal ganglia [i.e. (1) nigrostriatal dopaminergic system, (2)
intrastriatal cholinergic system and (3) GABA ergic system] may result in two type of motor disorders i.e.
1. Hypokinetic disorders like akinesia & bradykinesia.
2. Hyperkinetic disorders like athetosis, ballism us, chorea, cog-wheel rigidity & resting tremorsQ

Blood Brain Barrier (BBB)

66. D i.e. Proteins [Ref: Ganong 23je p-572-74; Guyton 11 je p-766]

Blood-CSF Barrier & Blood-Brain Barriers are highly permeable to water, CO 2 _______ BBB is Formed by
(carbondioxide), O 2 (oxygen^ and most lipid soluble (lipophilic) substances - Tight junction of the
such as free forms of steroid hormones, alcohol and anesthetics^; slightly endothelial cellsQ
permeable to electrolytes like Na+ (sodium), K+ (potassium) and Cl- (chloride); - Foot process of the astrocytes
and almost totally impermeable to plasma proteins, protein bound forms of around the capillaries Q
steroid hormones, protein antibodies and most non lipid soluble large organic (astrocytes send processes to
molecules (i.e. impermeable to, in general, all proteins and polypeptides)Q. It blood vessels where they
'Slows penetration of some smaller molecules like urea.________________________ induce capillaries)___________

67. B i.e. Subfornical organ 68. B i.e. Posterior pituitary 69. A i.e. Area postrema 70. D i.e. Medulla
[Ref: Ganong 23/e P-573-74; MTx]

- Circumventricular organs without bloor brain barrier includes Median em inence of hypothalamus, Subfornical
organ (SFO), Organum Vasculosum of lamina Terminalis (OVLT, supraoptic crest), Area postrema (chemoreceptor
trigger zone = CTZ = Vomiting Center)Q, Posterior pituitaryQ (Neurohypolysis).
j M edulla contains centers for swallowing, vomiting, coughing, sneezing & VMC (vasomotor center)._____________

Cerebellum, Posture & Decortication

71. C i.e. Vestibular nucleus 72. A i.e. Flocculus; C i.e. Nodulus ' 73. C i.e. Purkinje cells
74. C i.e. Cerebellar nuclei 75. C i.e. M ossy fibres inhibit Purkinje cells 76. D i.e. Dentate
77. B i.e. Bipolar 78. A i.e.Cerebral cortex; B i.e.Co-ordination; D i.e. Decreased tone; E i.e. Excitatory effect
79. D i.e. Smoothens and coordinates ongoing movement 80. C i.e. Resting tremor
Ref: Ganong 23/e p. 255-58; Guyton 12 SAE 699-709; Gray's 40je p. 300; Snell’s NA 8/e p. 223-225; IBS NA 6/e p. 258j

- Flocculonodular lobe constitutes the vestibulo- cerebellum. ☆ Cerebellum has 4 deep nuclei: - dentate,
Output from flocculonodular lobe goes directly to the emboliform, fastigial & globoseQ (Mn: 'DEFG').
brainstemQ (vestibular nucleus), bypassing the deep
Cerebellar cortex contains 5 types of neuron cells
cerebellar nuclei and is concerned with maintenance of
Mn: "G o Park & Basket Granular starts"
equlibriumQ.
- Cerebellar connection to other parts of the brain is projected
GO Golgi cells
through (or output from the cerebellum is solely from)
Park & Purkinje cells
purkinje cellsQ. Purkinje cells are amongst the biggest
Basket Basket cells
neurons in the body. Their axons are the only output from the
Granular Granule cells
cerebellar cortex from where they are projected to deep
Stars Stellate cells
cerebellar nucleiQ.
- Efferents arising from cerebellar cortex (purkinje fibers) end
in deep cerebellar nuclei & have inhibitory effectsQ. Whereas
output arising in deep cerebellar nuclei is always excitatoryQ
and project to centre outside the cerebellum to brainstem &
thalamus.
- The afferent input to the cerebellum is excitatory and comes
through - Clim bing fibers and Mossy fibers.________________
24 h A Complete Review of Short Subjects

- Cerebellum smoothens & coordinates ongoing movement in


motor performance this is the reason why lesion of
cerebellum causes I) A taxia (lack of coordination of
movement) 2) A synergia (lack of coordination b/w
different group of muscles), 3) A stasia (Unsteady
voluntary movement) 4) D ysd iad och okin esia = Rebound
Phenomenon (Inability to do rapid alternate successive
movement).

- Cerebellar lesions are associated with unsteady-


drunken gait, slurred-scanning speech, dissection-
decomposition of movement (i.e. b/o difficulty in Diagram of neural connections in the cerebellum. Plus (+)
performing action that involve simultaneous and minus (-) sign indicate whether endings are excitory
movement at more than one joint patient dissect such or inhibitory. BC basket cell; GC, Golgi cell; GR, granule
motion & carry them out one joint at a time). Slurred- cell; NC, cell in deep nucleus; PC, Purkinje cell. Note that
scanning speech, hypotonic, dysmetria (Past PCs and BCs are inhibitory. The connections of the stellate
pointing) & intentional tremorsQ whereas, resting cells, which are not shown, are similar to those of the
tremors occur in lesions of basal ganglia^. basket cells, except that they end for the most part on
Purkije cell dendrites.

81. B i.e. Tecto cerebellar tract

Spino/Olivo/Cuneo-Cerebellar (Mnz SOCC) tracts carry proprioceptive impulses whereas tecto cerebellar tract carry
auditory & visual impulses.

Afferent Tracts Transmits


Vestibulo-cerebellar Vestibular impulses from labyrinths, direct and via v estibu lar nuclei®
Dorsal /Ventral spinocerebellar Proprioceptive and exteroceptive impulses from body
Cuneo cerebellar Proprioceptive impulses, especially from head and neck
Tectocerebellar Auditory and visual impulses via inferior and superior colliculi
Pontocerebellar Impulses from motor and other parts of cerebral cortex via pontine nuclei
Olivocerebellar ProprioceptiveQ input from whole body via relay in inferior olive

82. C i.e. Increased rigidity via alpha motor neurouns [Ref: Ganong 24/e p. 241-43]

M id-Collicular Decerebration Decorticate Rigidity


• Complete transaction of brain stem (i.e. lesion) • Decortication (i.e. removal of cerebral cortex) produces
between superior & inferior colliculi permits - Flexion of upper extremities at elbow (d/t rubrospinal
brain stem pathways to function independent from excitation) &
all inputs from cortex (corticospinal & - Extensor hyperactivity in lower limbs (cause similar to
corticobulbar tracts) and red nucleus (rubrospinal midcollicular decerebration)
tract) mainly to distal muscles of Extremities • It is seen in hemiplegia d/t internal capsular bleed.
• Inhibitory & excitatory reticulospinal pathway to
postural extensor muscles remain intact leading __________________Decerebellate Rigidity________________
to hyperactivity & rigidity in extensor muscles of
all 4 extremitiesQ • Extensor muscle hyperactivity is exaggerated by
• Reticulospinal tract activates y motor neurons, removing anterior lobe of cerebellum in decerebrate
which indirectly (via la spindle afferent activity) animals (decerebellate rigidity)Q d/t elimination of
activate a motor neurons (k/a Gamma loop). cortical inhibition of cerebellar fastigial nucleus
Sections of dorsal root eliminate extensor rigidity • This shows that decerebrate rigidity also 1/1 direct
by breaking this myotactic stretch reflex. It is seen activation of a motor neuronsQ (independent of gamma
in uncal herniation d/t large cerebral tumor or loop)Q. So subsequent dorsal root cut does not reverses
hemorrhage. rigidityQ

83. B i.e. Smoothens and coordinates movement 84. B i.e. Flocculonodular lobe
Physiology: Central Nervous System ■ 25

Functional D ivision of Cerebellum (3 parts)

1. Vestibulo-cerebellum 2. Spinocerebellum 3. Neocerebellum/Cerebro-cerebellum

Phylogenetically the - Includes rest of verm is & Phylogenetically the newest® part of cerebellum
oldest® p art o f cerebellum ad jacen t m ed ial p ortio n o f Includes la ter a l p ortion o f cerebellar
Includes floccu lu s & hemispheres® hemisphere®
nodule in the verm is so - Receives propioceptive infut Interact with motor cortex in plann in g &
a lso k/a F loccu lon odu lar from all body & motor plan program m ing movements®
lobe® from motor cortex. By Output through D entate nucleus® to ventrolateral
Concerned with comparing both it nucleus of thalamus
equilibrium , g a it & sm oothen s & coord in ates S p in o c e r e b e llu m
learning induced changes m ovem en ts that are ongoing
M o to r
in VOR (Vestibulo- - Output through Emboliform, e x e c u tio n

Ocular Reflex) Fastigial or Globose nuclei to


Output passes directly to brainstem
brainstem

V e s lib u lo c e r e b e llu m

85. A i.e. Premotor cortex

Control of Posture & Movements

Part Function
Cortical Association Areas Com m ands fo r V oluntary m ovem en ts originate®
Basal ganglia & neocerebellum (lateral Are part of a feedback circuit to premotor & motor cortex that is concerned
portion of cerebellum) & cortex with planning & organizing volu n tary movement®
Supplementary motor area Involved primarily in programming motor sequences. Lesions 1/t awkwardness
in performing complex activities & difficulty with bimanual co - ordination
Premotor Cortex Concerned with settin g postu re a t the start o f a plann ed m ovem en t and
with getting individual ready to perform®
Somatosensory area (Posterior parietal Lesion causes inability to execute learned sequences of movements as eating with
cortex) knife & fork. Neurons in area 5 is concerned with aiming hand toward an object
& manipulating it, where as area 7 is concerned with hand- eye co-ordination.
Spinocerebellum (medial & intermediate Movements is sm ooth en ed & co- ordinated®
portion of cerebellum)
Cortico spinal & Cortico-bulbar system Primary pathway for the in itiation o f sk illed volu n tary movements®.
Rubrospinal, Reticulo-spinal, Tecto-spinal Are main brainstem pathways concerned with posture & co- ordination
& Vestibulo spinal tracts

86. A i.e. Integrity of Reflex arc: [Ref: Ganong 22/e P-206; Chaudhari 7jep 602] 87. B i.e. Stretch reflex ,
/ ----------------------------------------------------------------------------------------------------------------------------------------------------------------------------
- 'Postural Reflexes not only maintain the body in an upright balanced position but also provide the constant adjustments necessary
to maintain a stable postural background for voluntary activity'- Ganong 22nd/207
- Integrity o f the postural reflex arc is therefore the single best answer o f choice.
- Posture is maintained by altering postural (stretch) reflexesQ. In any posture some muscles are contracted & others are
relaxed. For example in erect posture antigravity muscle (extensors) of neck, spine, hip & knee have high tone.

88. A i.e. Hoping and placing reflex [Ref: Ganong 24/e p 240-41 & Internet] 89. D i.e. Crossed extensor reflex

In decorticate animals, Postural reflex integrated in cerebral cortex i.e. long loop stretch reflex & hopping-placing
reflexes are lostQ
26 ■ A Complete Review of Short Subjects

Integrating Center Postural Reflexes


Cerebral cortex Long Loop Stretch reflex, Hopping & placing reflexQ
Mid Brain Rightining reflex
Medulla oblongata Tonic Neck & Labyrinthine reflexesQ
Spinal cord Muscle tone, Positive supporting reaction & crossed extensor reflexesQ

Postural Reflexes
• Optical rightning reflex, placing reactions & hopping reactions are integrated in cerebral cortex
- Labyrinthine / Neck / Body on head / Body on body - rightining reflexes are integrated in midbrain
- Tonic neck reflex, tonic labyrinthine reflexes & streth reflex are integrated in medulla.
- Negative supporting reaction, positive supporting (magnet) reaction and stretch reflexesQ are integrated in spinal
cord
• Antigravity reflexes in medulla and locomotor reflex in midbrain & thalamus

90. B i.e. Hyperactivity [Ref: MTx]

Kluver-Bucy Syndrome i.e. Bilateral ablation of Amygdale (anterior temporal lobe) Clinical Presentation

1. Hyperorality Other Features include


- Loss of fear, extreme curosity, forgets rapidly
Tendency to place everything in mouth
- Visual agnosia, altered dietary habits (d /1
hyperorality)
2. Hypersexuality
- Depression, apathy, agitation & anxiety
Strong sex drive so much so that it attempts to copulate even - Sensory-motor-memory deficit.
with wrong (immature, wrong sex, different species) anmials
& inanimate objects.

3. Hypermetamorphosis

4. Placidity (Tameness or Docility)


Opposite of hyperactivity

Appetite

91. B i.e. Leptin 92. B i.e. Melanocyte Stimulating hormone (MSH) 93. D i.e. Ghrelin
94. B i.e. Increases fat deposition 95. C i.e. Lateral Hypothalamic Area
[Ref: Harrison's 17lll/ed pg-255, 464; Guyton 11 /e p. 867-70, 923-24; Handbook of obesity 2/ed pg-302; Ganong 23/e 384, 448, 237[
----------------------------------------------------------------------
- M SH reduce apetite whereas MCH increase itO. M SH (a-melanocyte stimulating hormone) and CART (cocaine &
amphetamine regulated transcript) releasing POMC (pro-opio-melano-cortin) neurons are anorexigenic /anti-orexigenic
(i.e. produce satiety & decrease appetite). Whereas, AGRP (agouti-related-peptide) and NPY (neuropeptide Y) releasing
neurons of arcuate nuclei of hypothalamus are orexigenic (i.e. increase appetite & cause obesity).
- Ghrelin, is released by oxyntic cells of stomachQ, especially during fasting (in anorexia) and stimulates appetiteQ
(orexigenic). It is also produced in hypothalamus & stimulates growth hormone (GH) secretion & activity^. CCK,
Peptide YY/PYY (secreted from entire gitract but especially from ileum & colon), glucagon like peptide (secreted by
intestines d/t presence of food) and insulin (produced in response to GLP) are gastrointestinal hormonal factors, which
suppress feeding i.e. cause satiety (anorexigenic). Whereas, ghrelin another gastrointestinal hormone produced mainly
by stomach is orexigenic (i.e. increase appetite).
- 'Orexins are synthesized in neurons located in the lateral hypothalamus'. And neurons that produce orexigenic
substances NPY & AGRP have their cell bodies in arcuate nuclei and project to paraventricular nuclei of
hypothalamus.
Physiology: Central Nervous System ■ 27

Regulation of Food Intake

- Hypothalmus contains hunger (feeding) and satiety centers. The lateral nuclei of hypothalamus serve as a feeding
(hunger) center and ventromedial nuclei of hypothalamus serve as the satiety center. Stimulation of feeding center &
/or destruction of satiety center cause voracious appetite (hyperphagia) & obesity. Whereas, destruction of feeding center &
/or stimulation of satiety center cause anorexia (complete satiety /lack of desire for food) & progressive inanition (weight
loss).
- Lesions of paraventricular nuclei of hypothalamus cause excessive eating and lesions of dorsomedial nuclei usually
depress eating behaviour.
- Arcuate nuclei of hypothalamus, are the sites where multiple hormones released from gastrointestinal tract and adiose tissue
converge to regulate food intake & energy expenditure. It has 2 types of neurons.

POMC /CART - (anorexigenic) Neurons AGRP/NPY (orexigenic) - Neurons


Propiomelanocortin (POMC) neurons produce a-M SH (a- These neurons produce orexigenic substances
melanocyte stimulating hormone) and CART (cocaine and neuropeptide Y (NPY) & agouti related protein
amphetamine regulated transcript), decreasing food intake (anorexigenic) (AGRP), increasing food intake & reducing
and increasing energy expenditure. energy expenditure.
I
a- MSH released by POMC neurons stimulate melanocortin receptor - AGRP is a natural antagonist of MCR-3 &
(MCR), esp. MCR-3 & (MCR-4) present in neurons of paraventricular M CR-4 and so increases appetite & obesity
nuclei (PVN) of hypothalamus, which then activate neuronal by inhibiting the stimulatory effects of a-
pathways that project to nucleus tractus solitarius (NTS), thereby M SH on M CR receptors.
increasing sympathetic activity & energy expenditure. So defective - When energy stores of body are low,
signaling of melanocortin pathway & /or mutations of MCR-4 are orexigenic neurons release NPY to stimulate
a/w obesity. Whereas excessive activation of melanocortin system appetite.
reduces appetite, (eg anorexia a/w cancers & infections.)_____________
Insulin, leptin and CCK hormones inhibit AGRP- NPY neurons and Gherlin hormone secreted from stomach
stimulate POMC-CART neurons, thereby reducing food intake (mainly) activates AGRP-NPY neurons &
stimulate food ntake.

Appetite Stimulatory Peptides (Orexigenic) Satiety Causing Peptides (Antiorexigenic)

These peptides increase appetite/ food intake and obesity These peptides decrease food intake and inaniation

AOrB Orexin A & Orexin B A MS a M SH (Melanocyte stimulating Hormone)

Why Neuropeptide Y (NPY) Gastro Gastrointestinal peptides such as


Neuro - Glucagon
MCH Melanin Concentrating Hormone (MCH) - Glucagons like peptide (GLP)
- Insulin
Gala Galanin (GAL) - Cholecystokinin (CCK)
Galanin like peptie (GALP) - Somatostatin
Ghrelin - Peptide YY (PYY)
a Gout Agouti Related Peptide (Ag RP) - GRP
patient Aminoacids (glutamate & a amino butyric • CART (Cocaine / Amphetamine
CR
acid) (Chief Regulated Transcript)
Resident) • CRH (Corticotropin Releasing
Ended Endorphin B
Hormone)
Or Orexin A or B
• CGRP (Calcitonin Gene Related
Released Gonadotropin Releasing Hormone peptide)
His
Slept Serotonin, Leptin
Gonadal
On Oxytocin, Norepinephrine
Growth Growth Hormone Releasing Hormone tense Neuro tensin
(GHRH) Bomb Bombesin
Cortisol
.............................. ........... „ . .. .„ . , M n - " Ah 4S Gastro Chief Resident (CR) Slept On
Tense Bom b"
O r Released His Gonadal Growth"
28 a A Complete Review of Short Subjects

Variety of Cytokines can induce cachexia (anorexia + other inflammatory responses)


- Tumor necrosis factor (TNF)
- Interleukin 6 (IL 6 )
- Inteleukin 1 (IL1)
- Interferon (IFN)
- Ciliary netrotrophic factor (CNTF)
- Leukemia inhibitory factor (LIF)

Neurotransmitter
96. D i.e. Serotonin

Large M olecule NTs Small M olecule Neurotransmitters


Neuropeptides e.g. Enkephalins, p-Endorphins, 1 • Amino acids e S' AsPartate' Glutamate, Glycine, GABA
Substance PQ & Hypothalamic releasing Z Amines e'S' SerotonineQ, Dopamine, Epinephrine, Nor
hormone epinephrine
3. Acetylcholine
97. C i.e. Decreased in starvation [Ref: Guyton 11/e 868-70, 563-64, 874, 956-57; Gaong 23/e p 145, 379; 22/e p-114, 96, 351, 224,
238-39, 230, 602; Harrison 17/e p-464; Martin obesity surgery p-52; Sembulingm 5/e p 273]
f N
Neuropeptide Y (NPY) is a 36 am inoacid ® polypeptide, which increases during starvation (Martin-52)/ and feeding *
(Ganong 22/e p 238) to increase the fo o d intake and reduce energy expenditure (i.e. basal /resting metabolic rate, adaptive
thermogenesis, thermic effect of exercise and energy cost of metabolizing and storing food). NP-Y decreases during
satiety.
NPY & MCH (melanin concentrating hormone) increase appetite whereas, M SH (a-metanocyte stimulating hormone)
decrease appetite. NPY decrease the firing of POMC neurons, thereby decreasing the activity o f melanocortin (i.e. MSH
secreting) pathw ay. (So the option D has some mistake).

98. D i.e. Nerve terminals [Ref: Ganong 23rd/e p. 143,175, 448; Gabriel C Ezeilo p. 146, 248, 251,427; MTx]

The synaptic neurotransmitter released by primary afferent nerve fibers in dorsal horn grey matter (substentia
gelatinosa) subserving fast mild pain is glutamate and the transmitter subserving slow severe pain is substance P.
Substance P is involved in pain transmission, neuroendocrinal regulation, axon reflex and peristalsis (intestinal)Q.
V-
99. D i.e. Glutamate 100. C i.e. G am m a-am ino butyric acid [Ref: Ganong 22/e P -1 0 7 ,109]

- Glutamate is the main excitatory transmitter in brain & I Neurotransmitters of CNS


spinal cord®, responsible for 75% excitatory transmission in
brain. Excitatory amino ad d Inhibitory amino add
- GABA is the m ajor inhibitory mediator in brain®. - Glutamate® - Gama amino butyric acid
- Aspartate® (GABA)®
- Glycine®
101. C i.e. GABA-ergic and cholinergic system [Ref: Ganong 22/e P-215]
‘In Huntington’s disease, a loss o f the intrastriatal GABA ergic and cholinergic neurons occurs'.
The loss of GABA ergic pathway to the external pallidum releases inhibition, permitting the hyperkinetic features of the
disease to develop. Degeneration of Nigrostriatal dopaminergic system causes Parkinson's disease®.

Thirst Control
102. B i.e. VPL [Ref: Ganong 24/ep 169-170]
- Ventro-lateral Spinothalmic tract (from nociceptors & thermo-receptors) carrying pain and temperature synapse in
VPL (Ventral-posterior-lateral) nucleus. Some fibers also project to centrolateral nucleus of thalamus (through
spinoreticular pathway). Thalamic pain syndrome is chronic pain on contralateral side of body during recovery from
thalamic infarct.
- Dorsal column carrying touch, vibration & proprioception synapse in gracilis & cuneate nuclei. 2nd order neuron
(medial lemniscus) cross midline and end in contralateral VPL.
Physiology: Central Nervous System ■ 29

103. C i.e. Preoptic nucleus of anterior hypothalamus [Ref: Ganong 22/e P- 244-245,615; Harrison 16/e P- 2097-2097]
104. B i.e. Osmotic stimulation of supraoptic nucleus (most probably) 105. A i.e. Subfom icial nuclei

Thirst

Temperature Regulation

106. D i.e. 25°C

Zone of thermo neutrality i.e. zone of least thermoregulatory effort is narrow range of ambient temperature (25 to
27°C)Q in which normal body temperature can be maintained by physical (not regulatory) mechanism.

107. C i.e. No change in hypothalamic thermostat [Ref: Harrison 17je p. 118]

Hyperthermia is uncontrolled increase in body temperature that exceeds body's ability to lose heat d/t failed
thermoregulatory mechanism, unlike fever; there is no change in hypothalamic thermostat (thermoregulatory) settings,
so antipyretics are not effective^.

108. A i.e. Temperature regulation 109. C i.e. Hyperthermia 110. C i.e. Suprachiasmatic nucleus
111. A i.e. Supraoptic [Ref: Gygton 12/ep 716]

Hypothalamus Part Function Hypothalamic part Hormone Released


Anterior hypothalamic (Pre- Temperature regulation Supraoptic (SO) Vasopressin/ADHQ
optic area) (thermostatic detection & control Nuclei (some by PV nuclei)
center of body temperature)Q Para ventricular (PV) Oxytocin (also by
- Anterior hypothalamus - Responds to heat so lesion 1/1 nuclei SO), TRH, CRH
hyperthermia Periventricular Somatostatin, GnRH
- Posterior hypothalamus - Responds to cold so lesion 1/1 Nuclei esp. Arcuate
(Shivering center) hypothermia Nucleus
Supraoptic Crest/OVLT in Osmoreceptors (regulating Preoptic area GnRH
anterior hypothalamus Vasopressin/ADH secretion) Dorsal & Posterior Catecholamine's
Preoptic nucleus^ (Ganong)/ Thirst center hypothalamus
Lateral superior
hypothalamus (Guyton)
Lateral Hypothalamus Feeding center
Ventromedial Nucleus Satiety center
Suprachiasmatic Nucleus^ Circadian rhythmQ
30 "A Complete Review of Short Subjects

112. A i.e. Increases heart rate with exercise [Ref: Gnnong 22/e P- 228, 610]

Increase in heart rate with exercise is under medullary control.^

113. B i.e. Sympathetic cholinergic (ACh) 114. C i.e. On the environmental temperature 115. A i.e. Radiation
116. B i.e. Vasodilatation 117. B i.e. Increased urine sodium 118. B i.e. Shift of blood from shell to core
119. C i.e. Noradrenalin 120. B i.e. Spleen 121. C i.e. 03 receptor & UCP -1
122. A i.e. Shivering 123. B i.e. Thyroxine 124. A i.e. Increased sympathetic stimulation
125. D i.e. 70% 126. D i.e. Piloerection
[Ref: Ganong 24/e p 316-19; Lem/ & Berne 6/e p 664, 228; Best & Taylor 13/e p 1168, 287; Guyton 12/e p 865-867-77; Lee 12/e p 292]

- Sweat glands are innervated by sympathetic postganglionic cholinergic fibres.QStimulation of sympathetic system
result in sweating. The major thermoregulatory responses to high environmental temperature include cutaneous
vasodilatation & sweating. Cutaneous vasodilatation is first physiological response.Q & prerequisite for sweating to
occur.
- Heat acclimatization (ie person exposed to hot weather for 1 to 6 weeks) results in excessive sweat production,
increased secretion of aldosterone, decrease in sodium chloride (salt) concentration in sweat and urineQ and
decreased renal blood flow.
- Heat loss from body depends mostly on the environmental temperature^. Heat is lost from the body when
environmental temperature falls below the body temperature.^
- Modes of heat lost from skin in a nude person (at normal room temperature) include radiation (60%)Q, Conduction
(to air =15% & direct conduction to solid objects e.g. chair= 3%) and evaporation (sweating =22% )Q. Conduction of heat
from body to air is self limited unless air currents (convection) continually moves air away from skin.
- Cutaneous vasoconstriction shifts the blood from shell to core and decrease heat lossQ in person suddenly exposed to
cold.
- With each fall of 1°C, metabolism is reduced by 6.7%Q. So in 10°C fall it will decrease by 67% (~70%)Q
- In heat acclimatization there is decrease in urine sodium.
- Endogenous non sievering thermogens are secreted by liver, intestine and muscles (eg cardiac) but not by spleen.
Nonshivering thermogenesis is seen in brown fat, which is abundant in infants & is mediated by UCP -1 (Uncoupler
protein) and 03- adrenergic receptorsQ. Stimulating the sympathetic nervous system, with liberation of nor epinephrine
and epinephrine, increases the metabolic rate of many tissues of the body. These hormones stimulate glycogenolysis in
liver, muscle other tissue and increase cellular activity, (thermogenesis) Q.
- Neonates cannot produce heat by shiveringQ. However, neonates can assume universal flexion posture like fetus (to
decrease surface area) & cutaneous vasoconstriction to decrease heat loss as well as non shivering thermogenesis (ie
breakdown of brown fat with adrenaline secretion) for heat production.
- Thyroxine is the hormone associated with long term cold adaptation^.
- Skin vasoconstriction, sympathetic stim ulation, increased thyroxine output shivering, non exercise activity
thermogenesis or NEAT (increased energy expenditure in form of small, fidgety movements; also responsible for
slower weight gain), non shivering thermogenesis (important in neonates with brown fat), voluntary behavioural
responses such as universal flexion or curling up in a ball posture = more important in animals and neonates),
piloerection (horripilation = goose pimples) and shunt vessels (counter current exchange of blood from artery to deep
venae comitantes) are physiological responses to low body temperature (cold).
- Piloerection, shunt vessels and curling up are important in animals but not in humans as a physiological response to
coldQ

127. B i.e. Subcutaneous tissue 128. D i.e. Beta-3

Non Shivering Thermogenesis

Is a cold induced increase in oxygen consumption to generate heat and is mediated by sympathetic nervous system.
Depends directly on the availability of brown fat and is a common feature in neonates.
The magnitude of this type of thermogenesis in adult human being, w h o h as virtu ally no brow n fa t, is less than 15%.Q
Physiology: Central Nervous System ■ 31

Sites Mechanism
• Sites of brown adipose tissue Brown fat contains large num ber of mitochondria and many small globules of
deposition
fat instead of one large fat globule. In brown fat cells the process of oxidative
- between the shoulder blades
phosphrylation in mitochondria is mainly uncoupled ie on sympathetic
- around the neck
stimulation, mitochondria produce large amount of heat but almost no ATP,
- behind the sternum
therefore almost all the released energy in oxidative phosphorylation becomes
- along the great vessels in thorax &
heat.
abdomen
- around the kidney & adrenal glandQ Sympathetic Stim ulation & Release of Epinephrine and Norepinephrine
(Brown adipose tissue is found only
I
in neonates - Samson wright)
Pa adrenergic receptor mediated Indirect action in cells of brown fat
• Norephinephrine stimulates Direct action on muscle & liver cells with large no of mitochondria
glycogenolysis in liver, muscle® & I I
other tissue.
-Increased glycogenolysis, lipolysis UCP-1 mediated uncoupling of
• Thyroxine
& fatty acid oxidation^ oxidative phosphorylation
• Intestine® producing large amounts of heat
-Increased BMR and heat production
but no ATP

129. C i.e. Decreased peripheral resistance [Ref Harrison 18/e p 166-167]

Hypothermia may lead to cardiac arrhythmias, reversible coagulopathy, increased pulmonary> systemic vascular

resistance, decreased cardiac output and renal blood flow, renal failure (in severe), cold induced diuresis (in mild cases)
increased haematocrit (2% for each 1°C drop), dehydration, areflexia, maladaptive behavior, leftward shift of
oxyhemoglobin dissociation curve, and decreased bowel motality and secondary dilation.

Cells of CNS
130. D i.e. Central neuroglial cells are derived from schwann cells [Ref: Ganong 23/e P-80-82]

Arise from cells of the neural tube (Ependymal cells or Neuroepithelial cell) - These do not develop from cells of
,____________________ I_____________ neural tube
1
Astrocytes®. Oligodendrocytes® - They probably come from the bone
marrow and enter the nervous system
- Send proceses to blood where they induce from circulating blood vessels.
capillaries to form the tight junction that - Form myelin in CNS®
I
form the blood-brain barrier®. Two types: - Unlike Schwann cells which
Microglia® (Macrophage o f CNS)
i) Fibrous astrocytes - Found in w hite form myelin between two
nodes of ranvier on single Scavanger cells resembling tissue
matter® & Rich in intermediate filament
neuron (in PNS) macrophage®and are responsible for
ii) Protoplasm ic astrocytes - found in gray phagocytosis®. These small sized glial
oligodendrocytes form
matter Q - Have granular cytoplasm cells are of mesodermal origin &
myelin on many neighbouring
- They produce substances that are trophic axons. derived from circulating monocytes.
to neuron® Example includes Gitter, Rod &
Siderophage cells.
32 ■ A Complete Review of Short Subjects

Nerve Regeneration

132. C i.e. Lack of myelin inhibiting substance IRef: Gaming 24/e p. 93-95; Guyton 12 SAE p. 75-76; Berne 6/e p. 63]

Factors responsible for non recovery (non-regeneration) of CNS neurons (i.e. neurons in brain & spinal cord) include
absence of growth promoting factors/ Schwann cells/ neurilemma = endoneural tube or H enle's sheath and presence
of myelin associated inhibitors of regeneration^ (i.e. Nogo, MAG= myelin associated glycoprotein, OM gp =
oligodendrocyte myelin glycoprotein & transmembrane semaphorin 4D)/ glial scar form ations /astrocytic proliferation/
microglial activation/inflammation & invasion of immure cells .

Cerebro Spinal Fluid (CSF)

133. D i.e. CSF/plasma protein ratio is 0.3 134. A i.e. 70-180 mm CSF 135. C i.e. Epidural space
136. A i.e. Choroid plexus 137. B i.e. 6 - 1 2 mmHg 138. B i.e. Hypothyroidism
139. C i.e. Formed by arachnoid villi 140. Bi.e. Rate of CSF absorption
[Ref: Guyton 11/e p. 763-67; Ganong 23rd/e p. 571-72; Ganong 22/e p. 612-14; Shinde & Chatterjea 7/e p. 676; Harrison 17/e
Appendix table 6; Wallach 8/e p 315; MTx]
I'/*
CSF is principally secreted by choroid plexusQ; it does not pass through epidural spaces. Cerebro spinal fluid (CSF)
pressure is mainly regulated by rate of CSF absorption by arachnoidal villiQ. The normal pressure in CSF when one is
lying in horizontal position averages lOmmHg or 130 mm of waterQ. Although this may range from 65-195 m illiliters
of water.
Rate of CSF formation is 550ml/day (2.3ml/hr or 0.38ml/min)Q; it s specific gravity is 1.006-1.007Q, pH 7.33Q (i.e. <
plasma pH 7.4) and CSF/Plasma glucose ratio is -0.66Q (CSF sugar is 2/3 of plasma sugar). CSF/Plasma protein ratio
is 0.003Q.
Hyperthyroidism cause decreased whereas, hypothyroidism/myxoedema is associated with increased CSF protein.

141. A i.e. Produced by choroid plexus; C i.e. Absorbed by arachnoid villi 142. A i.e. 3 mEq/L IRef: Adam's 10/e p. 16-20]

CSF is produced by choroid plexus, flow from 3rd ventricle to subarachnoid space and brain to spinal cord and is
absorbed by arachnoid villQ. Concentration of K+ in CSF is 3 mEq/L in children & aduItsQ.

143. B i.e. CK+ LDH [Ref: Adam's 10/e p. 17-18; Henray's Clinical DM/LM 22/e p. 6301

Enzymes found in CSF include creatine kinase (CK) BB, lactate dehydrogenase (LDH), adenosine deaminase, enolase,
neopterin, lysozyme and alpha fetoprotein.

Intracranial Tension & Transection

144. A i.e. Neck stiffness [Ref: Harrison 16/e P-1626; Fuller 2mt/e P-199, 200]

Neck stiffness is seen in those condition in which the meninges are irritated e.g. meningitis not in cerebral herniation Q.

145. B i.e. Tachycardia [Ref: Ganong 22/e P- 609] 146. C i.e. Raised intracranial pressure

When intracranial pressure is elevated to more than 33 mmHg over a short period, cerebral blood is significantly
reduced 1/t deterioration of consciousness. The resultant ischemia stimulates the vasomotor area & systemic blood
pressure rises.Q
Stimulation of vagal outflow produces bradycardia & respiration is slowedQ. The blood pressure rise helps to
maintain the cerebral blood flow (Cushing reflex)Q.
Physiology; Central Nervous System ■ 33

147. A i.e. First cervical spinal segment

Encephale An animal with caudal medulla or


Isole (Line A) post medullary (i.e. at 1st cervical
Transection spine segment o f spinal cord)
transection.
It remains alert, has sleep wake
cycles, normal pupillary reactions &
EEG and its respiration is
maintained artificially.
Cerveau Isole Mesencephalon transection
(Line B) separating forebrain from more
Transection caudal structures at mid point of
colliculi. Animal breathes
spontaneously but is unresponsive,
with abnormal pupils (usually
dilated) and a continuous sleep
pattern in EEG.

EEG

148. A i.e. Alpha waves [Ref: Ganong 24/e p. 274; Guyton 11/e p. 764]
149. A i.e. Frequency is 8-13 /sec.; B i.e. Voltage is 50 microvolts usually; C i.e. Predominant activity in REM sleep
150. A i.e. Frequency is 13-30 per second; C i.e. Predominant activity in REM sleep
151. D i.e. Awake /alert state 152. A i.e. Deep sleep 153. C i.e. Delta
154. D i.e. Delta activity 155. B i.e. Stage II 156. B i.e. NREM (stage 2) see diagram

- Alpha (o) EEG or Berger's rhythm has frequency of 8-13 Hz, voltage of 50 micro volts usually and is seen in
parietooccipital region of awake patient at rest with eyes closed, and mind wandering1?.
- Theta waves are seen in hippocampus^ and during emotional stress.
- 0 waves have >14 H z frequency*? are seen in awake alert patient at rest with eyes open and in REM (paradoxical)
sleep*?.
- Lowest frequency (3-5 Hz) delta waves are seen in stage III/IV NREM deep sleep (or slow wave sleep)*?.
- Sleep spindles & K- complexes are found in stage 2 NREM sleep.

Various EEG Patterns [Mnemonic: "B A T Dance"]


Order in which frequency decreases & amplitude increases ----------- ►

Beta (0) Wave Alpha (a) Wave Theta Wave Delta Wave
Recorded - From parietal & - Recorded from parieto - Seen in hippocampus'? Seen in very deep
from/seen frontal*? region occipital region1?. - In parietal and temporal NREM sleep1? &
in during specific - Seen in almost all normal regions of normal infant1?, serious organic
activation of these adults when they are children1 ? & drowsiness, brain disease, in
parts of brain awake and in quiet resting emotional stress in adults subcortical transactions
- Seen in drowsiness state of cerebration especially separating the ceberal
& children & disappointment and cortex from thalamus.
- Seen in awake patient at
patients awake, at rest*? with mind frustration
rest with eyes open*? wandering and eyes
closed*?
Frequency >14Hz*? 8-13 Hz*? - 4 - 7 Hz - 3-5 H z (min)*?
Amplitude Low amplitude1? - High amplitude*? - High amplitude1? - Large
amplitude(Max)1?
Voltage - 50 micro volts (usually) to - - 2 to 4 times greater
100 than most other types
34 ■ A Complete Review of Short Subjects

157. D i.e. Low amplitude, mixed frequency wave. 158. A i.e. REM sleep 159. C i.e. Sleep walking
160. A i.e. W ill not show high frequency waves during stage -3 sleep; B i.e. Shows alpha rhythm when a person is awake
but inattentive. [Ref: Harrison 16/e P-154-155; Guyton 11/ep 741-743; 10/e P-692; Ganong 22/e p.193-198; Ahuja 5/e P -140,148]

REM (paradoxical) sleep have predominantly mixed frequency low amplitude waves ((3 like activity). Nightmare is
seen in REM sleep and sleep w alking occurs in NREM sleep®.

*EEG Pattern In Sleep Normal EEG


- A lpha w av e is most
striking component &
I.:'":'■'rij. 1:.'i''-.vii'v. ■ -V’1,..
reflects the synchronized
18^^ v■ :■•:'''/iafeiiilliiiKS .1111 IMinlj h I !»-
Stage I: (dozing): First & lighest stage of sleep. - Light phase of sleep, but arousal is brain activity.
- Predominatly theta waveQ is seen. difficult^. - The alpha wave wax &
Stage II: (Unequivocal) - Characterized by - Mixed frequency, low amplitude^ wave spontaneously &
- Sleep spindlesQ waves on EEG, predominantly |3- are slowed down with
- K- complex (easily evoked) Q Iike activity^. sleep, disappears during
Stage III: (deep sleep transition) - Also known as DesynchronizedQ or deep sleep & are
- 5 (delta) wave first appear^ paradoxical sleep (because EEG is suppressed completely with
- K- complex (with strong stimuli rapid) eye opening or attention
only) - A - wave sleep directed to some specific
Stage IV: (Cerebral sleep) - Active sleep mental activity eg light
stimulation and are
- Predominant 5 (delta) activity^.
replaced by (3 waves.
It is called dreamless sleep but dreams do Dreaming is seen. Recall of vivid
occurQ, which are mostly not registered in dream imagery > 80%. Dreams are - Alpha block: When eyes
memoryQ. Imagery may be reported but lack associated with more bodily muscle is opened the alpha
vividness & detail activity <2. rhythm is replaced by
alpha block (a fast
-ve Increased autonomic activity, presence
irregular low voltage
of penile erection & bursts of eye
activity)
movements and muscle tone
- Desynchronization:
decreased.
When alpha pattern is
Regular respiration, -I BP, I HR Irregular respiration, variable BP &
replaced by another wave
HR, arrhythmia present
pattern during sensory
Sleep disorder during NREM IV Sleep disorder of REM
stimulation/ mental
1. Sleep walking (Som nam bulism ^ 1. Night maresQ
concentration, it is called
2. Sleep talking (Somniloquy)Q 2. NarcolepsyQ: The hallmark of this
dysynchronization. This
3. Night terror^ (Pavor nocturnus) disorder being decreased sleep
is also called alerting or
4. BruxismQ (tooth grinding) latency
arousal response.
5. Nocturnal enuresisQ (bed wetting) 3. Nocturnal penile trumesceneQ

* NREM / Slow wave sleep is called dreamless sleep but dream s d o occur®.
- NREM dreams are n ot registered® in the memory.
- REM dreams are associated with m ore b od ily m uscle activity®.
161. D i.e. Arterial pCC>2 measurement [Ref: Ganong 24/ep 276]

- Polysomnography (PSG) (noninvasive) procedure includes EEG (brain), EOG (eye), EMG (muscle) & ECG (heart)
monitoring.
Arterial PaCC>2 measurement is an invasive procedure.
Physiology: Central Nervous System ■ 35

Pain and Analgesia

162. D i.e. Mesentery

- Pain receptors (i.e. nerve endings sensitive to noxious stumuli) are absent in liver parenchyma, lung alveoli, brain
parenchyma, arachnoidmater, piamater, pial veins, & choroids plexus®.So these are pain insensitive organs/structures.
■ Free nerve endings carrying nociceptive fibers are seen in mesentery®.

Pain Sensitive/Insensitive Viscera

• Pain receptors (nociceptors) are free nerve endings® widely distributed in superficial layers of skin and certain internal
tissues, such as periosteum, arterial walls, joint surfaces and falx and tentorium in the cranial vault. Most other deep
tissues are only sparsely supplied with pain endings.
•Visceral pain originating in thoracic and abdominal cavities is transmitted through small type C pain fibers and so
can only be of chronic aching- suffering type of pain. Stumuli which cause true visceral pain are - ischemia, chemical
damage, spasm of smooth muscle of hollow viscus (eg gut, gall bladder, bile duct, ureter etc), over distention of a
hollow viscus and stretching of the connective tissue® surrounding or within the viscus.
• Insensitive viscera - Few viscera like parenchyma of liver and alveoli of lungs® are almost completely insensitive to
pain of anytype. Yet the liver capsule, bile ducts, parietal pleura and bronchi are very sensitive to pain.
• Intracranial structures insensitive to pain are - brain parenchyma, ventricular ependyma, duramater over convexity of
skull, piamater, pial veins, arachnoidmater and choroids plexus®. Whearas, scalp, aponeurotica, duramater around
venous sinuses & vessels, falx & tentorium cerebri, cranial venous sinuses with afferent veins, arteries at the base of
brain and arteries of duramater including middle meningeal artery, proximal large pial arteries and 2, 3, 5, 7, 9, 10
cranial nerves are pain sensitive.
• Due to a relative deficiency of A 5 nerve fibers in deep structures there is only a little rapid, bright pain. Instead deep
& visceral pain are poorly localized, nauseating, a/w autonomic symptoms like sweating and changes in blood
pressure and may initiate reflex contraction of nearby skeletal muscle (i.e. reflex spasm or guarding). Ischemic muscle
pain like angina pectoralis occurs d/t accumulation of a chemical agent Lewis's P factor (whose identity is not settled
but it could be K+). Visceral pain often radiates to other somatic structure that developed from same embryonic segment
or dermatome - (dermatomal rule). This occurs d/t convergence of somatic and visceral pain fibers on same 2nd order
neurons in lamina I-VIII of dorsal horn that project to the thalamus & then to somatosensory cortex (convergence -
projection theory). Somatic nociceptive fibers normally donot activate the second order neurons but prolonged visceral
pain facilitate somatic fiber endings - Causing stimulation of 2nd order neuron, and ofcourse brain cannot determine the
source of stimulus is viscera or area of referral.
• Viscera have no proprioceptors and few temperature, touch and pain receptors (sparsely distributed than in somatic
structures), Afferent fibers from viscera reach CNS via sympathetic pathway (from structures b/w the pain lines i.e.
lower esophagus to distal colon, lungs, diaphragm, hepatobiliary system, Kidney, ureter, bladder, uterus & fallopian
tubes etc) and parasympathetic pathway (from structures above thoracic & below pelvic pain line i.e. larynx, trachea,
cervix & upper vagina, bladder triagnone, prostate, urethra, testicle, upper food pipe & rectum etc). Their cell bodies are
located in dorsal roots & homologous cranial nerve ganglia, (esp facial, glossopharyngeal, vagus nerves, thoracic, upper
lumbar & sacral roots).

163. D, E i.e. Liver, Brain 164. C i.e.Duramater 165. A i.e. Pain


166. B i.e. Allodynia 167. A i.e. Hyper - sensitization 168. A i.e. It is poorly localized .
r
- Vanilloid receptors - VR1 are activated not only by pain causing agents capsaicin® but also by protons & temperatures
above 43°C®.
- Perception of normal innocuous sensory stumuli (eg touch) as painful® is known as allodynia or different pain;
whereas hyperalgesia is a hypesensitization of pain receptors®, thus lowering their threshold and causing exaggerated
response on minor pain producing stumili®.
- Visceral pain (of thoracic & abdominal cavities) is transmitted through small type C fibers and so can only be of chronic
aching- suffering type showing slow adaptation®. Due to relative deficiency of A8 nerve fibers in deep (visceral)
structures there is only very little rapid, bright, fast pain. Visceral pain is poorly localized® (often referred &
radiating), nauseating & a/w autonomous symptoms and spasm (reflex contraction of nearby skeletal muscle)®
36 ■ A Complete Review of Short Subjects

169. A > > B i.e. 5-HT (serotonin) > > Noradrenaline 170. A i.e. Simulation of adrenergic fibers
IRef: Guyton 11/e 59, 564, 602-3; Ganong 23/167-179,150-143, 262-267; Harrison 17/83]

Ascending pain pathway is inhibited in dorsal midbrain by enkephalins and serotonin (5HT)Q. (Guyton). Serotonergic &
Noradrenergic neurons in brain stem activate opioid (enkephalin) intem eurons and suppress the activity of
spinothalamic projection neurons. (Ganong p - 178- fig 11.5)
>■ -1

171. B i.e. Nocistatin stimulation; D i.e. Anandamide receptor; E i.e. Nicotinic & Cholinergic receptors
172. C i.e. Gate controlled theory of pain 173. D i.e. Inhibition by large myelinated afferent fibres
[Ref: Ganong 23/e 262-267,179 Q-2; Guyton 11/e 59]

< ;----------------------------------------------------------------------------------
- Analgesia is produced by nocistatin stimulation (ie inhibition of pain receptors), nicotinic-cholinergic (a4 & P2)
agonism and anandamide- cannabinoid agonism.
- TENS (i.e. trans cutaneous electrical nerve stimulation), Massage and application of linim ents or hot bottles to
painful areas in the body relieves pain due to gate control mechanism i.e. simultaneous stimulation of large
myelinated A|i fibers1? from non painful tactile sensation block/close/inhibit the gate to painful stimuli.

174. C i.e. M iosis > A i.e. Sedation [Ref: Ganong 23/e p. 144; KDT 5/e P-428]

M iosis is produced by p. (mu) opioid receptor and sedation is produced by p and k (kappa) opioid receptors.

175. B, C, D, E i.e. Nm receptor, Nn receptor, 5-HT3 receptor, Dopamine [Ref: Ganong 23/e 133; 141-42; KDT 6/e p-48, 96,163,
415; Guyton ll/ep -5 6 1 ]
-----------------------------------------------------------------------------------------------------------------------------------------------
Na+ and/or K+ are involved in nicotinic cholinergic (Nn & Nm) receptor, serotonin (5HT) receptor, dopamine (D2)
receptor, norepinephrine (ai & a 2) receptor, glutamate and GABA b receptors but not in GABA a and glycine receptors.

176. A i.e. Cold receptor B i.e. Warm receptor C i.e. Pain receptor [Ref: Ganong 23/e p. 167-168; Guyton 11/e p. 607-8]

[ Cold, warmth (heat) and pain receptors are thermal receptors.

177. C i.e. Rapidly adapting touch receptors [Ref: Ganong 23/e p. 150-49; Guyton 11/ep. 576-574; 585-86]
178. A i.e. Touch 179. B i.e. Slow adapting

Pacinion Corpuscles are rapidly adapting touch receptors <?. It responds only to transient touch, not to sustained
pressure because the corpuscle rapidly adapts and dissipates the pressure.

*Tactile / Touch Receptors


(Touch receptors are most numerous in the skin of fingers & lips & scarce in the skin of trunk)

Rapidly adapting Slowly adapting receptors


- Detect change in stimulus strength and are also known as - They are also known as 'tonic receptors’
'Rate receptors', 'Movement receptors' or phasic receptors. - They continue to transmit impulses to the brain as long as
- They adapt rapidly so cannot be used to transmit a stimulus is present. Examples include
continuous signal because these receptors are stimulated 1) Muscle Spindle Input (from stretch receptors in joint &
only when the stimulus strength changes. ligaments) maintaining posture & Nociceptor input are
- Examples : Light touch is rapidly adapting (otherwise it slowly adapting [Mn- "Free Rough Disc"]
would be distracting) [Mnemonic: "M y hair pack"] 2) Free nerve endings - found every where in the skin can
1) M eissener corpuscle - An elongated encapsulated nerve detect touch & pressure.
ending, detects touch*?. 3) R uffini's ending - Detects touch and pressure
2) Hair end organ - detects touch 4) M erckel's disc - are expanded tip tactile receptors which
3) Pacinian corpuscle*? - detects deep pressure & vibration detect touch, and are found in areas that contain large
* Pressure is maintained touch. numbers of Meissner's corpuscles. These are often grouped
together in Iggo dome receptor which projects upwards.
Physiology: Central Nervous System ■ 37

180. A i.e. No loss o f sensation IRef: Ganong 23/e p. 158; Campbell 11/e p. 2348]

During joint replacement (hip, knee etc) there is no loss of sensation1? despite loss of joint & ligament receptors.

181. B i.e. Accomodation IRef: Ganong 22/e P- 56, 324; Guyton 11/e p. 575- 76, 668, 608, 599, 568 -69, Sembulingam 4/e p. 695, 720, 892, 702;
www.lib.mcg.cdulcduleshuphysiolprogramlsectionlllch4lslch4-65.html

Increase in threshold level (of a receptor) on applying series o f subthreshold slow ly rising stimulus in succession is called
accomodation®.

Spinal Cord & Tracts


182. A i.e. Transmitted by the {J - type of sensory nerve [Ref: Ganong 23/e p. 89; Guyton 11/e p. 594-95]

Kinesthetic sensation is ability to perceive the extent, direction or rate of movements, i.e. perception of touch and pressure 1
and is therefore transmitted by A/3 type o f fibers®.

183. D i.e. Lateral corticospinal tract 184. A i.e. Pyramidal cells 185. A i.e. Crossed
186. A, B, E i.e. Spaticity, Extensor plantar response, Exaggerated tendon reflexes
187. B i.e. Involuntary movements 188. A i.e. Cog-Wheel rigidity
[Ref: Chaudhuri 5/e P- 525-29; Ganong 22/e P- 192-204, 253; Hutchinson's 20/e p 268; Guyton 11/e p.687- 89; Harrison 17/e p 147]

- Upper motor neurons (UMN) consists of pyramidal (corticospinal) tract1? (pyramidal cells & their neurons). 80% of
corticospinal tract fibers cross in low er medulla to form lateral corticospinal tract1?.
- Pyramidal (Corticospinal) tract lesion leads to increased muscle tone, spasticity (or clasp knife rigidity), clonus,
exaggerated deep tendon reflexes and extensor plantar ref lex (i.e. positive Babinski sign)®.
- Involuntary movements (i.e. athetosis, ballism and chorea), tremor at rest and cogwheel (lead-pipe) rigidity® are
hyperkinetic whereas, akinesia, and bradykinesia are hypokinetic features of extrapyram idal system (EPS) like basal
ganglia lesion or disease (eg Parkinson disease). W ing beating tremor or asterixis, dysarthria, unsteady gait, and
rigidity occur in hepato lenticular degeneration (Wilson's disease) involving putamen of lenticular nucleus.

189. A i.e. Upper motor neuron and lower motor neuron both.

Amyotrophic lateral sclerosis [ALS] is the most common form of progeressive motor neuron disease. It involves both
upper m otor neuron (UMN) & low er m otor neuron (LMN)®.

190. A i.e. Formed from fasciculus gracilis...; B i.e. Carries discriminative touch...; E i.e. Decussate at lower medulla
191. B i.e. Temperature, Crude sense [Ref: Ganong 24/e p. 167-169; Guyton 12/SAE p. 693-94]
192. A i.e. Dorsal column fibers 193. A i.e. Fine touch & Propioception 194. C i.e. Lateral spinothalmic tract

Dorsal column medial lemniscus pathway is formed from fasciculus gracilis and fasciculus cuneatus (1st order,
sensory neurons). The 2nd order neurons (medial lemniscus) cross the midline (decussate) a t lower medulla®. It carries
fine & discrim inative touch, proprioception and jo in t position sense.
Posterior (dorsal) column-medial lemniscal system carries fine touch, vibration, join t position and proprioception
(conscious proprioception)1?.

Somatic - Sensory Pathways

Dorsal Column-Medial Lemniscal System (Pathway) Antero Lateral Spinothalam ic Tract

column of the spinal cord to medulla, where they synapse in gracilus & cuneate Anterior (Ventral) Lateral
nuclei. The first order neurons in dorsal columns are k/a fasciculus gracilis Spinothalamic Spinothalamic
and fasciculus cuneatus. The second order neurons from the gracile & cuneate Tract Tract
nuclei cross the midline (decussate) at lower medulla1? and ascend in the - Crude touch® - Pain® (pin prick)
medial lemniscus to end in contralateral VPL (ventral posterior lateral) nucleus - Crude pressure® - Temperature
etc in thalamus (sensory relay area called ventrobasal complex). (capable only of /Thermal
Additional fibers from sensory nuclei of trigeminal nerve (subserving same crude localizing sensation (both
functions from head) join medial leminiscus. Third order neurons project from ability on the warm & cold)
VB complex to somatosensory area I (post central gyrus of cerebral cortex) mainly and surface of body) - Itch, Tickle
also to SSA II (lateral parietal cortex). + Sexual sensations also travel in
anterolateral system.
38 ■ A Complete Review of Short Subjects

- Fine (light) touch ® (requiring a high degree of localization of stimulus or requiring transmission of fine gradations of
intensity)
- Fine (light) pressure®
- Two point discrim ination (discrim inative touch, tactile discrimination or discrete localization)
- Phasic sensations such as vibration (pallesthesia)
- Sensations that signal movement against the skin
- Joint-position sense®
- Proprioception® (static position sense i.e. conscious perception of orientation of different parts of body with respect to
one another; and rate of m ovem ent sense /k in esth esia or dynam ic proprioception).__________________________________

195. C i.e. Pain sensation on the contralateral side 196. C i.e. Left lateral spinothalamic tract

- A lesion of antero (ventro) lateral part of spinal cord (i.e. anterolateral ascending spinothalamic tract) will lead to
loss o f pain, temperature (& ± crude touch)® below the level of lesion beginning 1-2 /2-6 segments below.
- Antero (ventro-) lateral cordotomy is effective in relieving pain because it cuts /interrupts lateral (not anterior)
spinothalanic tract carrying pain (& temperature) from the opposite side o f body®. This is the reason why, to produce
relief o f pain in right leg the contralateral (i.e. left) lateral spinothalam ic tract is cut or interrupted®.

197. A i.e. Ipsilateral loss of joint sensation 198. B i.e. C/L Dorsal column
[Ref: Ganong 23/e p. 171; Guyton 11/ep 606; Harrison 77/ep- 2589; Chaudhari 5/e p 592-93; Fuller p -1 5 3 ,165]

Brown Sequard Syndrome, a functional hemisection of cord results in ipsilateral loss of dorsal column - medial
lemniscus (i.e. discriminative touch, vibration, proprioception, kinesthetic and position sensations, discrete localization)
and corticospinal-pyramidal-motor tract (i.e. weakness and spasticity) modalities and contralateral loss of spinothalamic
(i.e. pain, temperature) modalities.

199. B i.e. Vibration and Proprioception [Ref: Harrison 16je P-2440] .

-----------------------------------------------------------------------------------------------------------------
In anterior spinal artery syndrome all spinal cord function (motor, sensory and autonomic) are lost below the level of
lesion with the striking exception of retained vibration and position sense®.]As posterior column is supplied by
posterior spinal artery]

Reflexes

200. D i.e. Innate reflex [Ref: Ganong 22/e P-267, 268]


- Salivation by dog on seeing food- innate reflex (unconditioned reflexJQ.
- Salivation by dog on ringing the bell only (without food) is conditioned reflex ®.

201. B i.e. Fatigue 202. A i.e. Memory [Ref: Chaudhuri 5/e P- 477; Ganong 24/e p. 286]

Properties of spinal cord reflex includes summation, delay and fatigue but not memoryQ as memory is strictly a
function of brain (stored in neocortex, amygdala & corpus striatum) not seen in spinal reflexes.
Physiology Central Nervous System ■ 39

Properties of Spinal Cord Reflex

Delay1? Tension produced in a muscle by simultaneous


stimulation of two afferents may be
- Between the application of stimulus & starting of the
response there is a time - interval.
- Most of the delay occurs at the synapse (central delay). But
More than sum of Less than sum of
is may also be peripheral delay (in afferent or efferent
response produced responses produced
limb).
separately b/o separately b/o
Sublim inal Fringe Occlusion
Fatigue1?
When a nerve is repetatively stimulated at a rapid rate,
flexor response appears within a few millisecond because
of spinal reflex. Then in the next few seconds the reflex
begins to decrease b/o fatigue of synaptic transmission1?.
In Occlusion, Fiber 1,2,3 are excited by Nerve A
Summation1?: Two types stimulation and Fiber 3 , 4 , 5 are stimulated by
a) Temporal summation - A single subthreshold stimulus Nerve B. So there is less response on simultaneous
fails to produce a response two (or more) subthreshold stimulation.
stimuli, applied in quick succession, can evoke a response. In Sublim inal Fringe, Fiber 3 is excited when both
b)Spatial summation - Two subthreshold stimuli, applied A & B are stimulated only
simultaneously but at different spots, can evoke a response.
Adaptations
Irradiation Repeated application of stimuli may cause response
intensity to
Stimulus intensity Reflex contraction r
Low Few (flexor) muscle Decreased (i.e. Increase (i.e.
Strong Large no. of fibers (withdrawl) Habituation) Sensitization)

If stimuli is non— If stimuli is injurious


injurious

203. B i.e. Withdrawal Reflex/ Bulbocavernous [Ref: Ganong 24/e P-235]

Transaction of spinal cord in all vertebrates is followed by spinal shock, in which all spinal reflexes are profoundly ☆
depressed. The first reflex response to appear as spinal shock wears o ff in human is the Bulbocavernous > Cremastric >
Babinski > Withdrawl reflexQ (ie a slight contraction of leg flexors and adductors in response to a noxious stimulus.

Sensory Reception

204. C i.e. The back of scapula [Ref: Ganong 24/e 162]

Tactile acuity (indicated by 2 point discrim ination threshold or minimum distance at which 2 points touched
simultaneously are perceived separate) measured by two point threshold test. It is dependent on number of touch
receptors (TR).
I 4,
TR are most abundant on finger tips so two point TR is least and so TPD distance is >65mm on back
discrimination distance is 2mm<? (of scapula)1?___________________________________

205. C i.e. Degree of overlap of fibres carrying tactile sensation is much less. [Ref: Adam's Neurology 10/e 153-159]

Following injury to a cutaneous nerve, the area of sensory loss is always less than its anatomic distribution because of overlap
from adjacent nerve. But area o f sensory loss fo r touch > pain®. The area o f tactile loss, is greater than th at o f pain®
because in tactile fibers there is much less degree of overlap of fibers and theres is lack of collateralization ( ie rapid
collateral regeneration)1?. Whereas, in pain fibers there is greater overlap and rapid collateral regeneration.
40 ■ A Complete Review of Short Subjects

Autonomic Nervous System


206. D i.e. S234 207. C i.e. Preganglionic parasympathic fibres are more lengthy
208. 6 i.e. Sympathetic system 209. C i.e. Increased refractory period
210. A i.e. Sympathetic mediated cholinergic release 211. A i.e. Vasoconstriction
212. B i.e. Increase contraction of heart 213. A i.e. Skin [Ref: Ganong 24/e p. 256-58; BRS 5/e p. 276-265; Guyton 12/SAE p. 608]
[P a r a s y m p h e lc : A N S |

Parasympathetic division is k/a


S p h m c le r p u p ila e a n d
E d in g e r w e s lp h a l
craniosacral (involving 3, 7 , 9 , 10 CN & S 2, ( A c c e s s o r y o c c u lo m o to r) n u c le u s | C lla ry g a n g ljo r T | c ilia fV ™ s c le

S 3, S 4 sacral roots) or cholinergic system S u p e rio r s a liv a to r y n u c le u s


■ 0 ------------- L a c rim a l a n d n a s a l g la n d s

o f fa c ia l n e rv e fS p h e n o (p le r y g o ) p a la tin e g a n g lio n -]
(uses ACh).
/ yv S u b m a x illa ry , s u b m a n d ib u la r
In fe rio r s a liv a to r y n u c le u s o f

Preganglionic parasympathetic fibers are g lo s o p h a ry n g e a l n e rv e

more lengthy (longer) than post ganglionic


fibers with a ratio of 1 : 8 to 9 (i.e. 1 pre
ganglionic fiber diverges to 8 to 9 post
ganglonic fibers). Whereas, postganglionic
fibers are longer in sympathetic system^.
Flight or fight reaction is caused by
sym pathetic system m ediated cholinergic
release Q. Sympathetic stimulation causes
increased heart rate, conduction velocily and
contractility but decreased refractory
period Q.
Vasoconstriction is a i adrenergic
stimulation response whereas vasodilation
and bronchodilation are {fe adrenergic
stimulation responses.

Sympathetic stimulation cause


vasoconstriction and so decreases blood
flow in skin and splanchnic vesslesQ

C o lo n . B la d d e r.
K id n e y,
s e x o rg a n s

L a rg e in te s tin e ,
P e lv ic B la d d e r, G e n ta l
s p la n c h n ic tis s u e
n e rv e

Heart/Skin Parsympathetic (Cholinergic/ Ach) Sympathetic (Adrenergic)


Response Response Receptor
S-A node Decrease in heart rate, vagal arrestQ Increase in heart rate Pi
Atria & ventricles Decrease in contractility and Increase in contractility and Pu P2
(usually) increase in conduction conduction velocity
velocityQ
A-V node, His-Purkinje Decrease in conduction velocityQ Increase in conduction velocity P.
System
Arterioles (vascular
smooth muscles)
- Skin, splanchnic vessels - Contracts ai
- Skeletal muscle vessels Dilation/ Contracts p2M /a,
Systemic veins - Dila tion /Contracts p2/ ai,a 2
Sweat glands Generalized secretionsQ Slight localized secretion in M
thermoregulation & stress (apocrine)
Pilomotor Muscles - Contraction al
Physiology Central Nervous System ■ 41

214. A i.e. Decrease gut contraction [Ref: MTx Autonomic System]

Effector organ Parsym pathetic (Cholinergic) Sym pathetic (Adrenergic)


Response R eceptor R esponse
GIT (Stomach & Intestine)
Motility & tone Increases (Gut co n tractio n ^ oci, CLi, $2 (Pi in S. intestine) Decrease (usually)Q
Sphincters Relaxation ai Contraction
Walls Contrtacts Relax
Secretion Stim ulation^ ai Inhibition

Gallbladder and ducts Contraction P2 Relaxation


Urinary bladder
Detrusor Contraction P2 Relaxation usually
Trigone and sphincter Relaxation ai Contraction
Uterus (Motality & tone) Increases, Contraction di
(pregnant) P2
Relaxation (pregnant & non
pregnant)
Male sex organs (Penis & Erection ai Ejaculation
seminal vesicle)
Spleen capsule - Con traction /Relaxa tion di/p 2
Adrenal medulla - - NE & Epinephrine
secretion
Liver - di, p2 Glycogenolysis,
gluconeogenesis
Pancreas
Acini Increased secretions a Decreased secretion
Islets cc2 D ecreased insulin and
glucagon secretion^,
increase plasm a glucose
(mostly)
P2 Increased insulin and
glucagons secretion
Salivary glands Profuse, w atery secretion ai Thick, viscous secretion

P Amylase secretion
Lacrim al & naso pharyngeal Secretion -
glands
Adipose tissue - Pa LipolysisQ

Juxtaglom erular cells - Pi Increased rennin


secretion
Pineal gland - P Increased m elatonin
synthesis and secretion

Eye
di Mydriasis
Radial muscle of iris (dilator
pupillae)
Contraction (miosis)Q
Sphincter muscle of iris
(pupillae)
Contraction for near vision P2 Relaxation for far vision
Ciliary muscle
(accomodation)

Lungs: Bronchial muscle Contraction^ P2 Relaxa tionQ

215. A i.e. Alpha action predom inantly

Sympathetic (adrenergic) system decreases secretion o f insulin & glucagon by 0 2 receptor action^ (and increases it by p2
receptor action) on p cells (for insulin) or a cells (for glucagon).
42 ■ A Complete Review of Short Subjects

a - Actions Adrenergic P-A ction


Vaso constriction® 1/t T BP (ai) Arterioles & veins V asod ilation 0 1/t IB P (p2)
Arrhythmia at high dose, otherwise Heart Cardiac stimulation ((3i) 1/t THR, force &
little effect (ai) conduction velocity
Lung Bronchodilation® (p2)
M ydriasisQ d/t contraction of Eye No effect on iris & ciliar muscles. Enhanced
radialis muscle (aO & decreased aqueous secretion
aqueous secretion
Intestinal relaxation, contraction of Intestine Intestinal relaxation (p2)
sphincters Bladder
Trigone contraction Uterus Detrusor relaxation
Contraction Spleenic capsule Relaxation (p2)
Contraction Pilomotor & Relaxation ((32)
Contraction Nictitating
membrane

Transmission facilitated (TACH Neuromuscular Active state prolonged in fast contracting


release) muscle, abbreviated in slow contracting
muscle tremor (p2)
Ejaculation Male sex organ Ptylin secretion
K+ & water secretion (ai) Salivary Gland Liver glycogenolysis^ (p2) ->
Liver glycogenolysis (±) Metabolism hyperlactacidemia
Muscle - glycogenolysisQ (p2) —>
hyperlactacidemia
Fat- lipolysis® (P3) —>T FFA, Calorigenesis®

Pancreas Increase insulin (mild) & glucagon secretion


Insulin secretion inhibited (02)°
(fc)Q
Kidney Renin release

216. D i.e. NE is the NT of all postganglionic sympathetic neurons including merocrine sweat glands
217. B i.e. Acetycholine

- Ciliary, Otic, Submandibular and pTerygopalatine ganglia contain postganglionic parasympathetic neurons
(Mn = " C O ST " = Parasympathetic ganglia).
- Celiac and superior & inferior mesenteric ganglia contain post ganglionic sympathetic neurons. (Mn=Seal (Celiarv)
Mess = Sympathetic ganglia).
- Acetyl choline is neurotransmitter for both pre & post ganglionic synapses in parasympathetic division as well as
preganglionic sympathetic neuronsQ.
- Nor epinephrine is the neurotransmitter o f p ost ganglionic neurons, w ith the exception o f merocrine (eccrine) sw eat
glandsQ and some blood vessels that receive cholinergic sympathetic innervationsQ. However, apocrine sweat glands
of axilla and most blood vessels are innervated by post ganglionic sympathetic adrenergic fibers.

- Riley-Day syndrome (familial dysautonomia 1/t abnormal sweating & BP instability), H om er syndrom e (interrupted
sy m pathetic innervations to face)®, peptic ulcer disease (d/t increased parasympathetic tone), Raynauds disease (?
increased sympathetic activity), botulism and Hirschsprung disease are possible diseases resulting from ANS
dysfunction.
Physiology Central Nervous System ■ 43

Chemical NTs of Nervous System E ffector


CN S
O rgan

ACh: Cholinergic Norepinephrine SNS


• Released by all Major NT of most post L ACh
neurons whose ganglionic sympathetic
axons come out of neurons including apocrine
(via blood) E, NE,
CNS eg sweat glands of axilla (but Dopamine,
- a motor neurons of excluding Peptide
SNSQ A drenal M ed u lla
- Preganglionic Merocrine NT is ACh (A N S: Sym pathetic)
sympathetic and sweat (acetyl
parasympathetic glands & choline)Q
neuronsQ few blood NE
- y motor neurons vessels)
- Cranial motor Renal & NT is ACh
neurons. M esenteric dopamine A N S: Sy m p athetic
• All postganglionic Vasculature
parasympathetic Adrenal NT is
neurons and few medulla adrenaline A Ch
postganglionic or
sympathetic epinephrine
A N S: P arasym pathetic
neurons of
merocrine(eccrine) Epinephrine
sweat glands and
some blood vessels By chromaffin cells of adrenal
also release ACH. medulla

218. A i.e. Fast EPSP, B i.e. Slow EPSP, D i.e. Slow IPSP 219. C i.e. Dopamine [Ref: Ganong 24/e P - 121-24]

- Stim ulation of post ganglionic sympathetic neurons leads to fast EPSP, slow EPSP, late slow EPSP and slow IPSPQ.
- Slow IPSP is generated by dopamine^.

220. C i.e. Release norepinephrine [Ref: Ganong 22/e P-230J

Tyramine, ephedrine & amphetamine (TEA) augment autonomic activity by releasing norepinephrine Q from endings
of post-ganglionic neurons.

221 . A, B, D i.e. Indicator of thermal sweating, T by sweating because of sweat has electrolytes, Fall is associated with yogic

Galvanic skin response is an indicator of thermal sweating^. G SR is increased by sweating (as in high sympathetic ☆
activity) because sweat has electrolytes, and falls in relaxed states like during yoagQ.
Chapter -2 NERVES AND MUSCLES PHYSIOLOGY

QUESTIONS

Neurons & Types of Nerve Fibres 9. Nerve Conduction is slowest in- (NBE P 13)
A. C □
1. Dorsal root ganglia have- (NBE P 13) B. A alpha □
A. Unipolar □ C. A beta □
B. Multipolar □ D. A delta □
C. Bipolar □ 10. Group B Nerve fibres are situated in: (AIIMS 02)
D. Pseudounipolar □ A. Muscle spindles □
2. Neurons in sympathetic ganglia are- (NBE P 13) B. Fibres carrying pain sensation □
A. Unipolar □ C. Preganglionic autonomic fibres □
B. Bipolar □ D. Post ganglionic autonomic fibres. □
C. Pseudounipolar □ 11. In Intersegmental reflex, the afferents come from:
D. Multipolar □ A. Golgi tendon (AIIMS 09) □
3. In which type of nerve fibres is conduction blocked B. Ia fibres □
maximally by pressure OR If one of your nerves is C. lib fibres □
compressed and this leads to paresthesia for D. Muscle spindles □
sometime, the type of nerve fiber affected is 12. Nerve fibers involved in proprioception:
probably OR Person wakes up with pain, A. Type A a fiber (NBE P 15,14; PGI 15,13,08) □
parasthesia, tingling of the arms, he had slept with B. Type B fiber □
arm below the head. Which fibres are involved?: C. Type C fiber □
(NBE P 15,14,13; PGI 01, SG PG I06) D. Type A(3 fiber □
A. Aa □ 13. Most sensitive nerve fiber to hypoxia- (NBE P 13)
B. Ab □ A. A □
C. Ag □ B. B □
D. C □ C. C □
4. A man slept with head over forearm, next moming D. All are equally sensitive □
he complains of tingling, numbness over forearm. It 14. Group A nerve fibers most susceptible to:
is caused by- (NBE P 13) A. Pressure (AIIMS 05) □
A. Sensitivity to hypoxia is A>B>C □ B. Hypoxia □
B. Sensitivity to pressure is A>B>C □ C. Local anesthetics □
C. Sensitivity to hypoxia is C>B>A □ D. Temperature □
D. Sensitivity to pressure is B>A>C □ 15. The afferent fibers which are m ost sensitive to local
5. True about Nerve fibre 'C ' is (TN 08, AP 01, UP 02) anesthetic belong to Group OR Post ganglionic
A. Most susceptible to Hypoxia (BHU 03) □ sympathetic fibres are: (AIIMS 04; NBE P 15,13)
B. Unmyelinated □ A. A □
C. Preganglionic autonomic □ B. B □
D. Not for temp. & pain senses □ C. C □
6. Nociception from abdomen is transmitted by- D. D □
A. A gamma (NBE P 13) □ 16. The correct order of susceptibility of the different
B. Aa □ types of nerve fibres to local anesthetics is:
C. C fibres □ A. B > C > A (AIIMS 08) □
D. B □ B. A > B > C □
7. Burning pain is carried by which type of fibres- C. C > A > B □
A. A alpha (NBE P 13) □ D. C > B > A □
B. A delta □ 17. A gamma nerve diameter is- (NBE P 13)
C. A beta □ A. 13-20 □
D. C □ B. 4-13 □
8. Maximum conduction occurs in which fibres C. 3-6 □
A. B-fibres (UP 09, Bihar 03, Delhi 04) □ D. 0.2-1.0 □
B. Aa fibre □ 18. Thickening of axon leads to- (NBE P 13)
C. Sympathetic fibre □ A. Increased speed of conduction □
D. C-fibre □ B. Decreased speed of conduction □
Physiology: Nerves and Muscles ■ 45

C. Increased absolute refractory period □ C. Occurs only in antigravity extensor muscles □


D. Unmyelination □ D. Excited by both stretch and contraction of the
19. Increased velocity of conduction in a nerve is muscles in which it is located. □
favored by: (PGI 08) 27. Find faulty statement regarding muscle spindle
A. Increased capacitance □ (SGPGI-02, Jipmer-01, UP-04, Bihar 05)
B. Decreased capacitance □ A. Central zone has no active & myosin □
C. Increased resistance □ B. Peripheral zone has no actin & myosin □
D. Decreased resistance □ C. Type 1 nuclear bag fibers has low myosin ATP
E. Increased velocity □ ase activity □
20. W hich of the follow ing is an am ineneuro- D. Type 2 nuclear bag fiber has has high level of
transm itter (AIIMS 03) myosin ATPase activity □
A. Acetylcholine □ 28. Contractile part of intrafusal fiber
B. GABA □ A. Ends (WB-01, Assam-03) O
C. Glutamate □ B. Center (SG PG I09, UP-03) □
D. Epinephrine □ C. Both □
D. None □
Muscle Spindle 29. Find false statement regarding sensory endings
A. Flower spray is secondary (Jhar-04, Jipmer 08) □
21. Which of the follow ing is true regarding alpha and B. Primary ending is annulospiral □
gamma motor neurons during initiation of C. Primary ending conduct la fibers □
voluntary movements? (AIIMS 15) D. Annulospiral wrap the ends □
A. Alpha motor neurons are activated first followed 30. Central wraping is done by (PGI 08, DNB 12)
by gamma motor neurons □ A. Flower spray endings □
B. Gamma motor neurons are activated first B. Annulospiral ending □
followed by alpha motor neurons □ C. Golgitendon □
C. Both are activated together □ D. Type II fiber □
D. Only alpha motor neurons get activated □ 31. Dynamic response is due to
22. The intrafusal fibers of the striated skeletal muscles A. Primary ending (AMU-04, WB-03, UP-06) □
are innervated by one o f the follow ing type of motor
B. Secondary ending □
neurons. Choose the correct answ er (AIIMS 04)
C. Tertiary ending □
A. Alpha □
D. All □
B. Beta □
32. True is all except regarding intrafusal fibers (UP-01)
C. Gamma □
A. Nuclear chain fibers are shorter & thinner □
D. Delta □
B. Nuclear bag fibers are lesser(PGI 06, WB 09) □
23. You are testing knee jerk reflex in a patient. The
C. Primary endings excited by bag fibers only □
afferents in deep tendon reflexes are carried by:
D. Secondary endings excited bychain fibers only □
A. Dynamic intrafusal fibers (AIIMS 14) □
33. Group II sensory fibers are attached to
B. Golgi tendon organ □
A. Annulospiral ending (AIIMS 04) □
C. Nuclear bag and static chain fibers □
D. Both Golgi tendon and muscle spindle □ B. Flower spray ending □
24. Lowest most level of integration of stretch reflex is C. Golgi tendon □
at D. Pacinian corpsucle □
A. Cerebral cortex □ 34. Plate endings are feature of (PGI 14,12, SGPG1-03)
B. Medulla □ A. Nuclear chain fibers (NBE P 14,13; UP 05) □
C. Lower medulla □ B. Nuclear bag fibers □
D. Spinal cord □ C. Extrafusal □
25. What is true regarding the gamma efferent neuron? D. Golgi tendon □
A. An 'A ' group (Jipmer 05) □ 35. Leksell efferents go to (Bihar 03, MP 04)
B. Motor neuron with a smaller diameter than that A. Intemeurons □
of alpha efferent neurons □ B. Golgi bottle cells □
C. Innervates intrafusal fibres □ C. Golgi tendon organ □
D. Innervates muscle fibres that stretch annulospiral D. Spindle □
endings □ 36. Nuclear bag fibres are related to: (AIIMS 14)
E. All of the above □ A. Force □
26. Muscle spindle is - (AI 2K) B. Length □
A. Receptor for a variety of multisynaptic reflexes □ C. Tone □
B. Receptor for myotatic or stretch receptor reflex □ D. Length and velocity □
46 ■ A Complete Review of Short Subjects

37. Muscle spindle (stretch reflex) detects: Nerve Injury


A. Tension (AIIMS 09,15,13; NBE P 15,14) □
B. Length □ 47. Most susceptible tissue to hypoxia-
C. Proprioception □ A. Muscle (NBE P 15,14; AIIMS 15,13) □
D. Stretch Length □ B. Neurons □
C. Hepatocytes □
Golgi Tendon D. Myocytes □
48. Neuronal degeneration is seen in all of the
38. In stretch reflex, the stimulus is- (NBE P 13) follow ing except OR Best prognosis in nerve injury:
A. Change in muscle length □ (AI 05; NBE P 15,14)
B. Change in muscle tension □ A. Crush nerve injury & Neurotemesis □
C. Pain in muscle □ B. Axontemesis □
D. Cessation of blood supply □ C. Senescence. □
39. Golgi tendon organs detect (AIIMS 08,15,13,14) D. Neuropraxia. □
A. Static muscle length (NBE P 15,14,13) □ 49. Axonotmesis includes discontinuity in: (NBE P 14)
□ A. Perineurium (PGI 06) □
B. Dynamic muscle length
□ B. Epineurium □
C. Muscle tension
C. Endoneurium □
D. Muscle action □
D. Axon & Myelin sheath □
40. Regarding Golgi tendon organ true is (AIIMS 11, 06)
50. Motor march is seen in- (NBE P 13)
A. Senses dynamic length of muscle □
A. Axontemesis □
B. Involved in reciprocal innervation □
B. Neurotemesis □
C. a-motor neuron stimulation □
C. Neuropraxia □
D. Senses muscle tension □
D. Nerve regeneration □
41. True statement about golgi tendon organ is
51. Difference between neuropraxia and
A. High threshold (PGI 08, WB-03, KA-05) □
neuronotomesis is: (PGI 10)
B. Dynamic response only □
A. EMG finding □
C. Detects length change □ B. Sensory loss □
D. 3-25 muscle fibers. □ C. Motor loss □
42. Number of golgi tendon organs per 100 muscle D. Nerve conduction studiesin distal fragment □
fibres- (NBE P 13) E. Axon integrity □
A. 1-20 □ 52. First change observed in distal part, of a cut nerve
B. 200-400 □ is: (AI 04; AIIMS 02)
C. ■ 50-60 □ A. Axonal degeneration f/b myelin degeneration □
D. 80-100 □ B. Chromatolysis □
43. Golgi tendon organ true is A/E (Jipmer 04, TN 05) C. Degeneration of neurilema □
A. Static & dynamic response (Assam-06) □ D. Schawn cells proliferation & Sprouting □
B. Activated by both stretch andcontraction of
muscle □ Nerve Conduction & N.M. Junction
C. Responsible for stretch reflex □
D. Transmit to lb fibers. □ 53. If a supramaximal stimulus is applied to an
44. Tendon organ, correct is A/E (Raj 05) excitable tissue like a nerve or a muscle and it elicits
A. Ramified nobby nerve endings □ a response, then the tissue is said to be in:
B. Regulates tension (SGPGI06, Andhra 04) □ A. Absolute refractory period (NBE P 13) □
C. Weak stretch activates inversestretch reflex □ B. Relative refractory period □
D. Myelinated lb fibers □ C. Latent period □
45. Autogenic inhibition is function of (PGI 02) D. After-depolarization □
A. Intrafusal fiber (AIIMS-03, UP 05) □ 54. If a supramaximal stimulus is applied to an
B. Extrafusal fiber □ excitable tissue like a nerve or a muscle and it elicits
C. Golgi tendon organ □ a response, then the tissue is said to be in:
D. Muscle spindle □ A. Absolute refractory period (NBE P 13) □
46. Crossed extensor reflex is a - (AIIMS 02, JIPMER 09) B. Relative refractory period (AIIMS 14)0
A. Withdrawal reflex □ C. Latent period □
B. Postural reflex □ D. After-depolarization □
C. Monosynaptic reflex □ 55. Action potential is generated in excitable cells.
D. Sympathetic □ These cells are: (PGI 11)
A. Nerves □
Physiology: Nerves and Muscles ■ 47

B. Muscle □ C. P 0 4- □
C. Glands □ D. Cl- □
D. Contractile tissue □ 66. Excitable tissue at rest is least permeable to:
E. Neuroglial cells □ A. Na+ (AI 06, AIIMS 13) □
56. "N erve terminals release chem icals was discovered B. K+ □
by: (PGI 10) C. Ca++ □
A. Dale □ D. CP □
B. Withering □ 67. Resting membrane potential in nerve fibre
C. Domagk □ A. Is equal to the potential of ventricular muscle
D. Langley □ fibre (UP 2K, SG P G I04) □
E. Loewi □ B. Can be measured by surface electrodes □
57. Na+ channel concentration is highest at- (NBE P 13) C. Increases as extra cellular K+ increases □
A. Dendrites □ D. Depends upon K+ equilibrium □
B. Soma □ 68. A travelling nerve im pulse does not depolarize the
C. Axon hillock □ area immediately behind it, because : (AIIMS 02)
D. Axon □ A. It is hyperpolarized □
58. True regarding Na+ ion- (NBE P 13) B. It is refractory □
A. Responsible for Donnan effect □ C. It is not self propagating □
B. Responsible for Resting membrane potential □ D. The condition is always orthodromic □
C. Responsible for Depolarization □ 69. True about nerve im pulse is: (AIIMS 94)
D. Does not help other ions in transport □ A. Travels in one direction along axon □
59. Initiation of nerve im pulse occurs at the axon B. If current is increased too slow nerve respond
hillock because : (AI 04, 02) fast □
A.It has a lower threshold than the rest of axon □ C. Travels in one direction at synapse □
B.It is unmyelinated □ D. Travels with the speed of electric current □
C.Neurotransmitter release occurs here □ 70. Unidirectional flow o f a nerve impulse is at:
D.None of the above □ A. Synapse (Al 06) □
60. Nerve depolarization is due to- (NBE P 13) B. Axon □
A. Opening of sodium channels □ C. Dendrites □
B. Opening of chloride channels □ D. All of the above □
C. Opening of potassium channels □ 71. Orthodromic conduction is
D. Opening of calcium channels □ (AMU 04, SGPGI 04;(NBE P 15,14,13; AIIMS 14)
61. W hich is TRUE regarding nerve conduction: A. An axon can conduct impulse in one direction
A.All or none phenomenon (AIIMS 04) □ only □
B. Conduction independent of amplitude □ B. An axon can conduct impulse in both direction □
C. Propagated action potential is generated in C. The jumping of depolarization from node to
dendrites □ node □
D. Faster in unmyelinated fibres □ D. The point at which a runaway spike potential □
62. Action potential is: (Raj 09, AMU 04, MAHE 02) 72. Synaptic conduction is mostly orthodromic because:
A. Decremental phenomenon □ A. Dendrities cannot be depolarized (AI 03) □
B. Doesn't obey all or none phenomenon □ B. Once repolarized, an area cannot be depolarized □
C. K+ goes from ECF to ICF □ C. The strength of antidromic impulse is less □
D. Threshold stimulus is required □ D. Chemical mediator is localised only in the
63. W hich ion causes repolarization - (NBE P 13) presynaptic terminal. □
A. Magnesium □ 73. Function of Dendrites: (PGI 03,12)
B. Calcium □ A. It generates the action potential □
C. Potassium □ B. Wave activity in E.E.G. □
D. Sodium □ C. ied Retrograde firing □
64. Resting mem brane potential of neuron is close to D. Ted Retrograde firing □
the isoelectrical (equilibrium ) potential of - E. Involved in memory □
A. Na (NBE P 14; (AIIMS 08) □ 74. In a neuron, graded electrogenesis occurs at-
B. CI □ A. Soma-dendritic zone (NBE P 13) □
C. K+ □ B. Initial segment □
D. Mg ++ □ C. Axon □
65. M ost diffusable ion in excitable tissue is : D. Nerve ending □
A. Na+ (AI 08) □ 75. M ost permeable to pure phospholipid bi layer is-
B. K> □ A. Oxygen (NBE P 13) □
48 ■ A Complete Review of Short Subjects

B. Na □ 83. TRUE about Renshaw cell inhibition is:


C. Cl □ A. Add on collateral sensation (AIIMS 2K) □
D. Water □ B. Increases by local anaesthetics. □
76. True about Nissl granule: (PGI 10,14) C. Has memory for spinal cord. □
A. Involves in RNA synthesis □ D. Inhibition of feedback propogation. □
B. Present in axon □ 84. Renshaw inhibition- (NBE P 13)
C. Present in dendrite □ A. Feedback facilitation □
D. Involves in protein synthesis □ B. Feedforward inhibhition □
E. Structurally they are endoplasmic reticulum □ C. Feedforward facilitation □
77. Node of Ranvier is seen in- (NBE P 13) D. Feedback inhibhition □
A. Cell body □ 85. Presynaptic facilitation is caused by- (NBE P 13)
B. Dendrites □ A. Prolonged opening of calcium channels □
C. Axons □ B. Prolonged opening of chloride channels □
D. Terminal butons □
C. Prolonged opening of sodium channels □
78. Which of the following statements is true for
D. Prolonged closure of potassium channels □
excitatory postsynaptic potentials (EPSP): (AI 03)
86. Synaptic potentials can be recorded by: (AIIMS 05)
A. Are self propagating □
A. Patch clamp technique □
B. Show all or none response □
B. Voltage clamp technique □
C. Are proportional to the amount of transmitter
C. Microelectrode □
released by the presynaptic neuron □
D. Are inhibitory at presynaptic terminal □ D. EEG □
79. Simulation of post ganglion sympathetic neurons 87. Type principle that is the spinal cord dorsal roots
leads to: (PGI 02) are sensory and the ventral roots are motor is known
A. Fast EPSP □ a s- (NBE P 13)
B. Slow EPSP □ A. Laplace's law □
C. Fast IPSP □ B. Bell-Magendle's law □
D. Slow IPSP □ C. Frank Starling's law □
E. Very slow EPSP □ D. Weber-Fechner's law □
80. Equal strength stimulus of which type is least likely 88. Phantom lim b sensations are best described by:
to produce a nerve stimulation: (AIIMS 02) A. Weber Fechner law (AIIMS 05) □
A. □ B. Power law □
C. Bell-Magendie law □
B.
D. Law of projection □
89. W eber Fechner law is related with : (AI 03)
D. A. Amplitude □
81. Which of the following statements is true about B. Surface area □
presynaptic inhibition? (AIIMS 08) C. Number of sensory fibre involvement □
A. It results due to failure of the action potential to D. Stimulus discrimination □
reach the synapse. □
90. W eber fechner law is: (AIIMS 08)
B. It occurs due to hyperpolarisation of presynaptic
A. Magnitude of stimulus strength perceived is
membrane. □
approximately proportionate to the log of the
C. It occurs due to inhibition of release of
intensity of stimulus strength □
neurotransmitter from presynaptic terminal. □
B. Magnitude of stimulus strength perceived is
D. It occurs due to blockade of neurotransmitter
directly proportional to the intensity of stimulus
receptors □
strength □
82. Regarding presynaptic inhibition which of the
C. Threshold of receptor is directly proportional to
following combination is true: (AIIMS 01)
stimulus strength □
a) Affected by axo-axonic transmission.
D. Threshold of receptor is inversely proportional to
b) Not affected by pharmacologic agent
stimulus strength □
c) Prolonged by anaesthetic agents
91. Intensity of sensory stimulation is directly related
d) Not prolonged by strychnine.
to: (AIIMS 02)
e) Inhibited by picrotoxin
A. Duration of action potential (AP) □
f) It will increase EPSP
A. a, c and f are true. □ B. Frequency of AP □
B. a, d, e are true. □ C. Amplitude of AP □
C. d, c and b are true. □ D. All of the above. □
D. a, d and b are true. □ 92. Spontaneous release of acetylcholine at the neuro­
muscular junction produces: (AI 05)
Physiology: Nerves and Muscles ■ 49

A. Miniature end-plate potential □


B. Action potential □
C. Post-tetanic potential □
D. Resting membrane potential □
Neuromuscular Junction

93. A short duration high frequency stimulation with 5


Hz of perforating substance pathway in
A. band (AIIMS 16) □
hippocampus leads to: (AIIMS 14)
B. I-band □
A. Long term potentiation □
C. M-band □
B. Post tetanic potentiation □
D. H-zone □
C. Long term depression □
101. Thin filam ent consists of all except- (NBE P 13)
D. Habituation □ A. Actin □
94. Release of synaptic vesicles from the presynaptic B. Troponin □
terminals is inhibited by? (AIIMS 13) C. Myosin □
A. Preventing depolarization of nerve terminal □ D. Tropomyosin □
B. Inhibition of conduction of nerve implse □ 102. Which of the follow ing is not a Sarcolemmal
C. Prevention of Ca2+ influx □ Proteins? (AI 08)
D. Prevention of Na+ influx □ A. Sarcoglycan □
95. Contraction of muscle can be caused by- (NBE P 13) B. Dystrophin □
A. Release of noradrenaline □ C. Dystroglycan □
B. Release of acetylcholine □ D. Perlecan □
C. Release of serotonin □ 103. M yosin filam ent has a fixed length of- (NBE P 13)
A. 0.16nm □
D. Release of histamine □
B. 1.6microm □
96. Which of the following events happens when a
C. 16nm □
nerve impulse arrives at the Neuromuscular
D. ,6 mm □
Junction: (Al 12)
104. The band which disappears on muscular contraction
A. Release of Calcium from the Sarcoplasmic
is (AIIMS 14, PGI 15)
Reticulum □
A. A □
B. Acetylcholine binds to receptors on the
B. H □
postsynaptic sarcolemmal membrane □
C. I □
C. Initiation of formation of cross bridges to allow D. M or CM □
muscle contraction □ 105. In M uscle contraction all are true except
D. Efflux of calcium through synaptic vesicles into A. A bond remains unchanged [PGI 14,03] □
the synaptic cleft □ B. H zone disappears □
97. Skeletal muscle contraction ends when- (NBE P 13) C. I band becomes wider □
A. Ions move out of cytoplasm □ D. Two Z lines come closer □
B. Ach is absorbed from the NMJ □ 106. Which protein prevents contraction by covering
C. Closure and indrawing of receptors □ binding sites on actin and m y osin : (AI 08, NBE P 14)
D. Decreased calcium outside reticulum □ A. Troponin □
98. Myasthenia gravis is a disorder of: (AI 05) B. Calmodulin □
A. Motor neuron □ C. Thymosin □
D. Tropomyosin □
B. Neuromuscular junction □
107. Tropomyosin: (AI 02)
C. Peripheral nerve □
A. Helps in the fusion of actin and myosin □
D. Spinal cord □
B. Covers myosin & prevents attachments of actin &
Muscle Contraction myosin □
C. Slides over myosin □
99. Sarcomere extends betw een- (NBE P 13) D. Causes Ca2+ release □
A. A band & I band □ 108. The site where myosin heads bind to actin in
B. Two Z lines □ skeletal muscles are covered by OR Actin's active
C. Two I bands □ site are covered by: (AI 08; NBE P 15,13; AIIMS 15)
A. Tropomyosin □
D. None □ B. Troponin □
100. In the follow ing electron m icrograph of the
C. Calcium □
sarcom ere, correctly identify the area labeled as "E ".
D. None of the above □
50 ■ A Complete Review of Short Subjects

109. All of the following are true about excitation 118. The difference between skeletal an smooth muscle
contraction coupling except: (AIIMS 13) contraction and relaxation is/are:
A. Accetylcholine is released at the nerve terminalD A. Troponin (+ve) (NBE P 15,13;PGI 06)□
B. Calcium is pumped back into the sacroplasmic B. Myosin light chain kinase □
reticulum during relaxation □ C. Ca2+ □
C. Calcium is released from sacroplasmic reticulum D. Actin & Myosin □
during contraction □ 119. True regarding excitation contraction coupling in
D. Calcium binds to tropomyosin to initiate muscle smooth muscles is : (AIIMS 02)
contraction □ A. Presence of troponin is essential □
110. Which of the following triggers muscle contraction B. Sustained contraction occurs with high calcium
A. Ca binding tropomyosin (AIIMS 15,13; A l 08) □ concentration □
B. Ca binding troponin C □ C. Phosphorylation of actin is required for
C. ATP breakdown □ contraction □
D. Ca binding troponin I □ D. Presence of cellular calcium is essential to cause
111. Type I muscle fibers are- (NBE P 13) muscle contraction □
A. Red □ 120. The force of muscle contraction can be increased by
B. Anerobic □ all of the following excep t: (AI 04)
C. Large □ A. Increasing the frequency of activation of motor
D. Glycolytic □ units □
112. Features of slow twinching muscle is/are (PGI 11) B. Increasing the number of motor units activated □
A. Contain large amounts of myoglubin □ C. Increasing the amplitude of action potentials in
B. Composed of smaller fibers □ the motor neurons □
C. Contain larger amount of glycolytic enzyme □ D. Recruiting larger motor units □
D. White in colour □ 121. Twitch of a single motor unit iscalled : (AI 06)
E. It has high ATPase activity □ A. Myoclonic-jerk □
113. The properties of fast twitch muscle are: (PGI 11,13) B. Fasciculation □
A. Extensive sarcoplasmic reticulum □ C. Tremor □
B. Extensive blood supply □ D. Chorea □
C. Large glycolytic pathway □ 122. Tetanic contraction is due to accumulation of -
D. Abundant mitochondria □ A. Na+ (NBE P 13) O
E. Made up of large so of small fibers with some B. Ca +2 □
long fibre muscles □ C. K+ □
114. Fast fatigue fibres are recruited during walking: D. CL □
A. In the beginning (AIIMS 14) □ 123. There is mutation of gene coding for the ryanodine
B. In the end □ receptors in malignant hyperthermia. Which of the
C. Throughout the walking process □ following statements best explains the increased
D. When small neurons are excited □ heat production in malignant hyperthermia?
115. True for smooth muscle contraction is A/E (PGI 09) A. Increased muscle metabolism by excess of Ca
A. Latch bridge are less cycle less ATP bridges □ ions (AIIMS 05)D
B. Dephosphorylation causes definite relaxation □ B. Thermic effect of blood □
C. LC phosphatase is essential for binding □ C. Increased sympatheticdischarge □
D. Slow & prolonged contractile response D. Mitochondria thermogenesis □
independent of nerve supply □ 124. When the tension in a muscle fibre is maximum, its
E. Length tension relationship does not exist □ length is called as- (NBE P 13)
116. Increase in cytosolic calcium from intracellular A. Equlibrium length □
storage, during smooth m uscle contraction is/are B. Optimum length □
due to: (PGI 08) C. Initial length □
A. CAMP □ D. None □
B. CGMP □ 125. Considering the latent period of a muscle twitch to
C. CCMP □ be 10 ms, contraction time 40 ms and relaxation time
D. IP3- DAG □ 50 ms, what will be the tetanizing frequency for this
E. C a 2+ channel □ muscle? (AIIMS 15)
117. Action of calmodulin is: (PGI 02) A. 25 Hz □
A. Ca2+ dependent □ B. 50 Hz □
B. Through calmodulin dependent kinases □ C. 100 Hz □
C. Through CAMP dependent kinases □ D. 75 Hz □
D. Through CGMP dependent kinases □
Physiology: Nerves and Muscles ■ 51

ANSWERS AND EXPLANATIONS:

Neurons & Types of Nerve Fibres

1. D i.e. Pseudounipolar 2. D i.e. M ultipolar IRef: BRS Anatomy p ! 8 ]

Unipolar Neurons Pseudounipolar N Bipolar N M ultipolar Neuron


Invertebrates Dorsal root ganglionQ Bipolar cells (retina) Purkinje (cerebellar) cells pyramidal
cells of hyppocampus, sympathetic
ganglion & chain, motor neuron
M n="PM S"

3. B i.e. Ab 4. B i.e. i.e. Sensitivity to pressure is A>B>C 5. B i.e. Unmyelinated 6. C i.e. C fibres
7. D i.e. C 8. B i.e. Aa fibre 9. A i.e. C 10. C i.e. Preganglionic autonomic fibres
11. D i.e. Muscle spindle 12. A i.e. A a fibers 13. B i.e. B 14. A i.e. Pressure
15. C i.e. 'C ' 16. D i.e. C > B > A 17. C i.e. 3-6 [Ref: Ganong 24/e p. 92-94; Berbe 6/e p. 77; Guyton 12 SAE p. 731
r
A (A a Afi, Ay, AS) and B fibers are m yelinated but C fibers are unmyelinated. ------------------------------------------ f t
Fiber diameter and hence conduction velocity in decreasing order is A (A a> Af}> Aj ) > B > C .
Relative decreasing order of susceptibility of nerve fibers to conduction block produced by various agents is:
pressure (ABC = A > B > C); hypoxia (BAC = B > A > C); and local anesthetics (CBA = C > B > A). That means A
fibers are most susceptible to pressure, B to hypoxia and C to LA.
Pressure on a nerve (eg Saturday night or Sunday morning palsy) can cause loss of conduction in large diameter
motor (Aa), touch and pressure (A3) fibersQ while pain sensation (A8) remains relatively intact.
Type A a fibers are involved in proprioception^; group B are preganglionic autonom ic Q and group C are post
ganglionic autonomic (sympathetic) & dorsal root fibersQ. Visceral pain (nociception) e.g. from abdomen is carried
by c fibersQ.
Thermo receptors for cold are A delta (A8) fibers for hot (heat/warmth/bum) are on C fibersQ. Visceral pain
(nociception) eg from abdomen is carried by C fibers.

Thermoreceptors for cold are A delta (A5) fibers and for hot (heat/warmth/bum) are on C fibersQ.

Mammalian Nerve Fibers: Types & Function

Numerical (N) & Erlanger - Gasser (EG) Fiber Type Classification of Sensory Neurons

N Type EG Type Origin Function M yeli­ Fiber Velo­ Spike Absolute


nated Dia meter city Dura­ Refractory
(pm) (m/s) tion period
(ms)
la Aa Annulospiral ending o f 12-20 70-120
muscle spindle® (primary Proprioception, Yes
muscle spindle receptor somatic motor
a/w extrafusal muscle
fiber)
lb Aa Golgi-tendon organ Q
II Ap - Flow er spray ending o f Touch, pressure, Yes 5-12 30-70 0. 4-0.5 0.4-1
muscle spindle Q motor
(Secondary muscle
spindle receptor)
- Fibers from most
cutaneous mechano
receptor (i.e. discrete
cutaneous tactile,
receptor)
52 ■ A Complete Review of Short Subjects

Ill AS Nociceptors of Pricking pain, Thin 2-5 12-30


neospinothalamic tract, cold
cold thermoreceptor, free (temperature),
nerve endings of crude crude touch
touch & pressure (skin
mechanoreceptors)
IV C (dorsal Nociceptors of Slow pain, Unmyeli­ 0.4-1.2 0.5-2 2 2
root) paleospinothalamic tract, temperature nated
warmth receptor (hot), crude
touch, itch &
tickle

Classification of Motor Fibers (Neurons)

Motor EG type
type
a Aa Annulospiral ending o f Proprioception, Yes 12-20 70-120
muscle spindle® (primary somatic motor
receptor a/w extrafusal
muscle fiber)

Y Ay Associated with intrafusal Motor to muscle Yes 3-6Q 15-30


muscle fiber spindle

Classification of Autonomic (Motor) Neurons

Preganglionic B Preganglionic autonomic® Yes <3 3-15 1.2 1.2


Post ganglionic C Postganglionic sympathetic® No 0.3-1.3 0.7-2.3 2 2

Features

• Greater the diameter of a given nerve fibre, Relative susceptibility of mammalian A, B and C nerve fibres to
greater is the speed o f conduction Q. D iam eter & conduction block produced by various agents:
hence the velocity of conduction in decreasing
Suscepti-bility to M ost Inter-mediate Least
order is
Susceptible Susceptible
A o > Ap > Ay > A8 > B > C Hypoxia B A C
• Large axons are primarily concerned with: Pressure A B C
- Proprioceptive sensation Local anesthetics C B A
- Somatic motor function
- Conscious touch & pressure • Touch is carried by -|— AB
• Small axons are primarily concerned with 1— A5
- Pain • Pain by— T A 8
- Temperature L C fibres [PA DC]
- Autonomic function
• Refractory period & spike duration • A (Aa, Ap, Ay Ag) and B fibers are myelinated but C fibers are
Minimum - A a unmyelinated.
Maximum - C fibres

18. A i.e. Increased speed of conduction 19. A & D i.e. Increased capacitance & decreased resistance
[Ref: Gabriel C Ezeilo p. 33; Mathews- cellular physiology of nerve and muscle 4/ed pg-50; Ganong 23rd/e p.89-83]
* - - ■ ■ ■ ■ ““
V elocity of conduction in a nerve is increased by myelination, increase in diam eter (thick nerve) and capacitance and
decrease in resistance®.
Physiology: Nerves and Muscles ■ 53

Factors Affecting Nerve Conduction Velocity

I
Fiber Diameter Degree of M yelination
I I
Ability of an action potential to spread is proportional to amount of - Nerve conduction velocity is directly
cytosol. I___________________ proportional to degree of myelination
I---------------------- I - Degree of myelination and so the conduction
Small diameter fibers have less Large diameter fiber offers lesser velocity decrease from A (a P y 8 ) to B to C (eg
cytosol (& more membrane) which resistance and higher capacitance A a fibers are heavily myelinated where C
means less capacitance (and more through its thick axoplasmic core fibers are unmyelinated)
resistance) causing slower and so the conduction velocity is
conduction velocity faster.

20. D i.e. Epinephrine [Ref: Ganong 24/e p. 136-53; Berne 6/e p. 95-103]

Histamine, epinephrine, norepinephrine, serotonin and dopam ine are biogenic mono-amine neurotransmitters.
(M n" HENS do are monoamine N Ts").

Muscle Spindle
21. C i.e. Both are activated together 22. C i.e. Gamma [Ref: Ganong 24/ep 232, Guyton 13/ep 694]

- The main motor supply o f intrafusal fibres is V (gamma) neurons.Q Y motor neuron exclusively supply intrafusal
fibresQ. Intrafusal fibres are also supplied by p fibres, p fibres also supply the extrafusal fibres [p for both]. The main
motor supply of extrafusal fibres are a neurons.

- During initiation of voluntary movements, in response to descending excitatory inputs to spinal motor circuits i.e.
whenever signals are transmitted from motor cortex etc, both gamma (y) and alpha (a) motor neurons are stimulated
simultaneously. This co activation effect of a and y neurons causes simultaneous contraction of both extrafusal
skeletal muscle fibers and intrafusal muscle spindle fibersQ

23. A i.e. Dynamic intrafusal fibers

Afferent in deep tendon reflex (eg knee jerk) is carried by dynamic intrafusal fibers (nuclear bag fibers type 1)Q.

24. D i.e. Spinal cord 25. E i.e. All 26. B i.e. Receptor for stretch receptor reflex
27. B i.e. Peripheral zone has no action & myosin 28. A i.e. Ends 29. D i.e. Annulospiral wrap the ends
30. B i.e. Anulospiral ending 31. A i.e. Primary ending 32. C i.e. Primary endings excited by bag fibers only
33. B i.e. Flower spray ending 34. B i.e. Nuclear bag fibers 35. D i.e. Spindle
36. D i.e. Length and velocity
37. B i.e. Length [Ref: Guyton 12/SAE p. Utib,770-72; Ganong 24/e p. 229-33]

Golgi tendon organ function as a transducer in feedback circuit that regulate muscle force (or tension)® in a fashion

analogus to the muscle spindle (dynamic nuclear bag fibers) feed back circuit that regulates muscle length and velocity®.
Mn - "T for T & L f o r i "

M uscle Spindle

Each muscle spindle is 3-10mm long structure consisting of 3 elements (1) tiny intrafusal muscle fibers =ifm f (3-12 in
number) with contractile polar ends and noncontractile center; (2) large diameter myelinated type la and II afferent nerves
originating in the central portion o/ifmf and (3) small diameter myelinated y motor efferent nerves supplying the polar
contractile region of ifmfLMuscle spindle is a sense organ which sends information about muscle lengthQ and rate of change
of length (i.e. speed of movementQ. Because the change in muscle length is associated with change in joint angle,
therefore muscle spindle also provide information on join t position (proprioception). Muscle spindle acts through
monosynaptic muscle stretch (myotatic) reflexQ.
54 ■ A Complete Review of Short Subjects

Intrafusal Muscle Fibers Afferent (Sensory) Nerve Endings Efferent (Motor) Nerves
(IFMF)
• Central (mid) part o f Central sensory receptor portion of muscle spindle * Main motor supply of muscle
fibers has few or no actin is stimulated by stretching of midportion either by spindle is through 3-6|i diameter
& myosin filam ents, so lengthening of whole muscle or even by contraction gamma m otor neuron (type A- y
the ends o f intrafusal of end portion of intrafusal fibers. m otor fibers)®. Because of their
fibers are contractile characteristic smaller size they are
Two type of sensory nerve endings are
whereas central portions called ^efferent of LeKell or
are not®. It's central small motor nerve system. It has
portion functions as Group la: Primary Group II:
2 histological types of motor end
sensory receptor. (Annulospiral) Endings Secondary (Flower
plates.________________
• IFMFs lie parallel to Are terminations of rapidly spray) Endings
regular contractile units of conducting (70-120 m/s) Are terminations of Gamma- Gamma-Static
muscle ie extrafusal fibers 17p diameter group la group II sensory Dynamic (y-d) (y-S)
attached to its tendons. afferent fibers®. It wraps fibers® (8 p diameter) Plate endings Trail network
(encircles) the central May encircle fiber on nuclear bag endings on
But it does not contribute to
position o f nuclear bag but usually it spreads dynamic nuclear chain
the overall contractile force
(dynamic and static) & fibers fibers and static
of muscle._ Two types of like branches on a
nuclear bag
intrafusal fibers are nuclear chain fibers®. bush.
fibers
So annulospiral endings Located near the
•Alpha motor neurons (type Aa-
are innervated (excited) by ends but only on
Nuclear bag Nuclear fibers) are other type of anterior
both nuclear bag (dynamic nuclear chain fibers®
fibers chain motor neuron of 14pdiameter
& static) and nuclear chain &static nuclear bag
I fibers mainly supplying extrafusal fibers
fibers. fibers. These do not
- Several I and causing contraction of muscle
Ia fibers are very sensitive innervate dynamic
nuclei are Shorter & nuclear bag fibers®. fibers.
to velocity (speed) of
congregated thinner® There are upto 8 • So the muscle can be made to
change in muscle length
in expanded (about group II secondary contract by stimulation of a motor
during stretch (dynamic
central bag. half) as endings in each neuron (innervating extrafusal
response).
Typically 2 compared muscle spindle. fiber) or ymotor neuron (indirectly
When the length of spindle
(1-3) bag to bag receptor increases suddenly Static response d/t via stretch reflex). Intrafusal
fibers occurs fibers and (i.e., dynamic response to slow stretching of fibers are less strong & lesser in
per spindle. lacks a rate of change of receptor muscle spindle number to cause muscle
- There are 2 definite length), only primary stimulate both contraction. However, their
subtypes of bag. endings are stimulated® primary & secondary contraction by y neuron deforms
nuclear bag Nuclei are ending® annulspiral endings & initiate
fibers: static arranged stretch reflex.
and in a chain
F lo w e r s p ra y
dynamic. througho e n d in g s
Nuclear bag ut A n n u lo s p ira l II S e c o n d a ry
fiber 1 with receptor e n d in g s a ffe re n t
low level o f area. (fro m all e x c e p t
myosin d y n a m ic
3-9 fibers
n u c le a r b a g fib e rs )
ATPase per la P rim a ry
activity and spindle a ffe re n t
nuclear bag (fro m all n u c le a r b ag
a n d n u c le a r c h a in fib ers
fiber 2 with
high level o f y- S ta tic e ffe re n t
y -D y n a m ic e ffe re n t
myosin (w iith p la te e n d in g s ) (trail e n d in g s )
ATPase P E ffe re n t
activity®.
To e x tra
fusal
N u c le a r c hain
fib ers
fib ers
N u c le a r bag
in tra fu s a l fib ers

Mammalian M uscle Spindle: Afferents & Efferents


Physiology. Nerves and Muscles ■ 55

Golgi Tendon
38. A i.e. Change in muscle length 39. C i.e. Muscle tension 40. D i.e. Senses muscle tension
41. D i.e. 3-25 muscle fibers. 42. A i.e. 1-20 43. C i.e. Responsible for stretch reflex
44. C i.e. Weak stretch activates reflex 45. C i.e. Golgi tendon organ
IRef: Ganong 24/e p. 230-233; Guyton 12/SAE p. 770-74; Oxford physiology 3je p.2541

: —
- Muscle spindle (responsible for stretch reflex) located in muscle belly send information about muscle length and rate ^
^
(speed or velocity) of change of length. Whereas golgi tedon organs located in muscle tendons transmit information
about muscle tendon tension or rate of change of tension^.
- Golgi tendon organ has low threshold and strong stretch is required to produce relaxation (i.e. inverse stretch reflex
or autogenic protective inhibitory negative feedback lengthening reaction).
- Golgi tendon (or ramified net like knobby nerve endings) are both activated by passive stretch and active
contraction of musdeQ and has both dynamic as well as static response.
j Each goligi tendon is attached to 3-25 muscle fibers.___________________________________________________________

46. A i.e. Withdrawal reflex [Ref: Guyton & Hall 11 Ie p.681 - 82; Ganong 22/e p. 135]
Crossed extensor reflex is withdrawl reflexQ.

Nerve Injury
47. B i.e. Neurons
Hypoxic Susceptibility of Cells

Type High susceptibility Intermediate Low susceptibility


Damage time 3-5 minutes 20-120 minutes Hours
Cells Neurons (nerve cell)Q Myocardium (Myocyte), Fibroblast, epidermis,
Hepatocyte, Renal epithelial cell skeletal muscle
48. D i.e. Neuropraxia 49. D i.e. Axon & Myelin Sheath 50. A i.e. Axontemesis
51. A, D, E i.e. EMG finding, Nerve conduction studies.., Axon integrity 52. A i.e. Axonal degeneration
[Ref: G uyton 12/SA E p. 74-76; Sam son W right 13/e p. 288; Chnudhan 5/e p. 46; Apley's 8le P-231; Sem bulingam 4/e p. 687; C SD T 11 /<?p. 128, 913;
M alwshwnri 3/e p. 50-511
r -v
- Neuropraxia refers to physiological disruption of conduction only, no structural changes or degeneration occur.
Neuronal degeneration is seen inQ - Crush nerve injury, Fetal development, Senescence.
- No neuronal degeneration (anatomical disruption or axonal disintegrity) is seen in neuroprexia. Axontemesis
includes discontinuity in axons and myelin sheathsQ. Whereas, in neurontemesis there is complete discontinuation of
nerve (ie axon with myelin sheath & neural tube or perineurium, epineurium & endoneurium).
- Type of nerve injury can be differentiated by EMG & nerve conduction studies.
- l sl change occurring in distal part of cut nerve is axonal degeneration followed by (2 nd change) myelin degeneration^.

Seddon's Classification of Nerve Injury

Features Neuropraxia Axontemesis Neurontemesis


Pathology - No anatomical disruption Axons & myelin sheath - Complete division of nerve
- Both axon & sheath intact disrupted but endoneural - Axon & neural tube (i.e. perineurium,
- Physiological disruption of perineurium and epineurium, & endoneural sheath)
conduction only epineurium sheath (neural both are divided.
tube) is intactQ
Degeneration No degeneration Q Degeneration present Degeneration present proximal +
proximal + distal (Wallerian) Distal (Wallerian)
Neuroma No In continuity End or side neuroma
Tinel's signQ / Absent Advancing (present)Q Static
Motor march
Prognosis - Excellent - Good /fair /poor Poor
- Recovery is complete - Occurs as regenerating
usually with in 6 weeks axons grow into intact
sheath
56 ■ A Complete Review of Short Subjects

- Electrodiagnostic studies (like nerve conduction study & EMG) are used to distinguish between partial & complete
lesionQ and to establish a baseline for monitoring subsequent functional recovery^. However, these are not helpful
until 2-3 weeks after an acute injury.
- If a muscle lose its nerve supply (d/t nerve injury), EMG shows denervation potential at 3rd week; which exludes
neuropraxia but however does not differentiate between axontmesis and neurontemesis.
- All types of injuries may cause sensory, m otor or both loss depending on type of nerve damaged.

★ Each axon is covered by myelin sheath (in myelinated nerve only) &/or neurilemma (Schwann sheath). Each nerve
fiber (containing many axons) is covered by endoneurium. Each fasciculus (containing many nerve fibres) is covered by
perineurium. And each nerve (containing mamy fasciculus) is covered by epineurium.

Nerve Conduction & N.M. Junction

53. B i.e. Relative refractory period 54. B i.e. Relative refractory period
55. A i.e. Nerves; B i.e. Muscle; C i.e. Glands; D i.e. Contractile tissue
[Ref: Ganong 24/e p. 88; Guyton 12/e p. 64-65, 666; Chaudhary 6/e p. 17; Best & Taylor 13/e p.155-621

f
Action potential develops in excitable cells such as nerves (neurons), muscle (skeletal, smooth and cardiac), and other
contractile tissues such as few glandsQ. Glial cells such as neuroglial cells do not generate action potential.

Refractory Period (RP)

Absolute RP Relative RP
It correspond to period from time the firing level is reached until - From the end of absolute R.P to start of after
repolarization is l/ 3rd complete depolarization.
No stimulus no matter how strong, w ill excite the nerve. - In this period only very strong stimulus can
cause excitation^

56. E i.e. Loewi


Donor heart R ecip ie n t heart

Otto Loewi discovered that nerve terminals or synapse 4. Add flu id to


. S tim u la te re cip ien t heart
release chemicals. In his most famous experiment, Loewi vagus ^
took fluid from one frog heart (vagus stoff or later found to 3. R em ove
Fluid
be acetylcholine) and applied it to another, slowing the sam ple
second heart and showing that synaptic signaling used
chemical messengers.
J V is
. H eart rate slow s 5. H eart rate slows!

O tto L oew i's Experim ent

57. C i.e. Axon hillock 58. C i.e. Responsible for Depolarization 59. A i.e. It has lower threshold than rest of axon
60. A i.e. Opening of sodium channels 61. A i.e. All or none phenomenon 62. D i.e. Threshold stimulus is required
63. C i.e. Potassium 64. C i.e. K+ 65. B i.e. K+ potassium ion is most diffusible ion.
66. A i.e. Na+ 67. D i.e. Depends on potassium ion equilibrium 68. B i.e. It is refractory
69. C i.e. Travels in one direction at synapse 70. A i.e. Synapse 71. A i.e. An Axon can conduct impulse in ...
72. D i.e. Chemical mediator is located only in the presynaptic terminal
[R ef: G a n on g 2 4 /e p. 8 6 -9 2 ; G u y ton 12/S A E p. 71 -7 3 ; B ern e 6/e p. 6 5 -7 6 ; C h au d h ari 5 /e p. 4 6 9 -7 0 ]
Physiology: Nerves and Muscles ■ 57

- Initiation o f nerve impulse (depolarization) occurs at axon hillock because it has lower threshold for excitation than
the rest of axon (neuron)*? mainly because of greater concentration of voltage gated Na+ channels.
- Development of AP (i.e. depolarization spike) occurs d/t Na+ influx (from opening of Na+ channels)*? and rapid
return of membrane towards resting membrane potential/RMP (i.e. repolarization) is d/t closing of Na+ Channels and
opening of K+ channel*?.
- Development of action potential (or nerve conduction) is an "all or none phenomenon"*? ie neither AP occurs below
threshold intensity nor does the further increase in intensity of stimulus produces increase in AP.
- Because of more open K+ channels and greater membrane perm eability to K+(in comparison to Na+) at rest, the
resting membrane potential in neurons is usually about -70 mV, which is close to equilibrium (isoelectric) potential
for K+*? - Summary of Ganong 24/e p. 88
- Order of permeability in excitable tissue at rest: K + > Cl- > Na+*?
- The nerve impulse is self propagating*? due to circular current flow and succive electronic depolarization to the firing
level of membrane ahead of action potential.
- Once initiated moving impulse does not depolarize the area behind it to the firing level, because this area is
refractory*?.
- Axons can conduct nerve impulses in either direction*? (anti-dromic conduction) unlike synapses, which permit
conduction of nerve impulses in one direction (orthodromic) only because the neurotransmitter substance
responsible for conduction is present in vesicles o f presynaptic neurons only*?. The post synaptic neuron does not
contain any neurotransmitter*?. It has protein molecules embedded in it which serve as receptor for the
neurotransmitter.
- Orthodromic conduction means conduction in one direction only, from one neuron called the presynaptic neuron to
another neuron called the postsynaptic neuron*?.___________________________________________________________________
PRESYNAPTIC NEURON --------------> SYNAPTIC CLEFT > POST SYNAPTIC NEURON
(SYNAPTIC KNOB) - N O neurotransm itter vesicles
- N eu rotran sm itter vesicles - Receptors present for them.
- Electrical synapses can transmit signals in either direction.*?

73. B i.e. Wave activity in EEG (C) 4-ed retrograde firing (E) Involved in memory
[Ref: Ganong 24/e P-125-26, 84-85; Guyton 12/SAE P-671-72, 676; Berne 6/e p. 53-54]

Functions of dendrites are learning & long term potentiation of memory, decreased retrograde firing and production of
ware activity in EEG*?.

74. B i.e. Initial segment 75. A i.e. Oxygen 76. C, D, E i.e. Present in dendrite; Involves in protein synthesis;
77. C i.e. Axons Structurally they are endoplasmic reticulum

- From Cell body (Soma) of neuron arises 5-7 Dendrites and Axon Hillock (initial thick segment of axon=Nerve
impulse generator)*?. Nodes of Ranvier are periodic gaps in myelin sheath of axons*?.
- Dendrites & soma are receptor zone. Axon hillock is nerve impulse generator (graded electro genesis), Axon is
transm itter and nerve terminal is neurotransmitter (NT) release zone.
- Nissl body (or granule) /tigroid body is large rough endoplasmic reticulum (i.e. containing ribosomes), seen in soma
(or perikaryon, or cyton, or cell body, the bulbous end of a neuron, containing the cell nucleus)*? and dendrites*?, of
neurons but not in axon or axon hillock*?. They are the site of protein synthesis*? involved in for production of
neurotransmitters such acetylcholine.

78. C i.e. Are proportional to the amount of transmitter released by the presynaptic neuron
[Ref: Ganong 24/e P-122-23,161, 259; Berne 6/e p. 85, 89-90,158, 222]
f---------------------------------------------------------------------------------------------------------------------------------------------------------------------------
Excitatory post synaptic potential (EPSP) or generator (receptor) potential (which is EPSP in Pacinian corpuscles) is a
nonpropagating localized depolarizing potential, the magnitude of which is proportional to the intensity of stimulus
(i.e. the amount of transmitters released by the pre synaptic neurons)*?. So EPSP or GP(RP) are graded potentials rather
than "all or none" action potential.
58 " A Complete Review of Short Subjects

79 .

80.

81. C i.e. It occurs due to inhibition of release of neurotransmitter from presynaptic terminal.
82. B i.e. a, d, e 83. D i.e. Inhibition of feedback propogation
84. D i.e. Feedback inhibhition 85. A i.e. Prolonged opening of calcium channels
[Ref: G anong 2 4 je p l 34-36; Gabriel C Ezeilo p. 45-46; Gin/ton l l t h / e p. 567-66; Sem bulingu m 3/e P- 642; Chaudlniri 5/e P- 472, 4731

Indirect preynaptic inhibition occurs due to decrease or inhibition of release of excitatory neurotransmitter from pr<
synaptic terminalQ (because of GABA mediated increased CI- influx, increased K+ outflux, decreased size of action
potential, decreased Ca++ entery, decreased NT release causing decrease in EPSP). Whereas direct reciprocal
postsynaptic inhibition occurs d/t release of inhibitory neurotransmitter (glycine / GABA) on post synaptic membrane
producing hyperpolarization & IPSP.
Pricrotoxin (not LA) inhibits or antagonizes presynaptic inhibition whereas strychnine antagonizes postsynaptic
inhibition^.
Presynaptic facilitation is caused by prolonged opening of Ca++ channels'^ and early closing of K+ channels^.
Renshaw cell inhibition is negative feedback inhibition or inhibition of feedback propogation^.

86 . C i.e. Microelectrode [Ref: G anong 24/e P-87-91 ]

By electrical, chemical or m echanical stim ulus, two types of physio chemical disturbances are produced in n e rv e s -1)
Local, non propagated potential - synaptic, electrotonic or generator potential, 2) Propogated disturbance - action
potential (nerve impulse). The electric events in neurons are rapid being measured in millisecond (ms) and the potential
changes are small, measured in m illivolts (mv). The electrical activity can be measured by (i) M icroelectrodeP with tip
diam eter of less than 1 micron (ii) Electronic am plifier (iii) Cathode ray oscilloscope

87. B i.e. Bell-M agendle's law 88 . D i.e. Law of projection 89. D i.e. Stimulus discrimation
90. A i.e. Magnitude of stimulus strength perceived is approximately proportionate to the log of the intensity of stimulus
strength [Ref: G anong 23rd/e p. 152-54; 22/ed pg-126; Guyton l l t h / e p. 594-96; Gabriel C Ezeilo p. 4521
91. B i.e. Frequency of AP

Weber Fechner law "M agnitude of sensation felt is proportionate to the log of intensity of the stim ulus". So stimulus
strength discrimination (gradation) is dependent on frequency of APQ, number of receptors
activated and amount of receptor potential.
Bell Magentic law In the spinal cord - dorsal roots are sensory and ventral roots are motor [DS VM]
Law of projection Describe p han tom lim b sensationQ

92. A i.e. Miniature end plate potential [Ref: G anong 22/e P-1171

Small quanta (packets) of acetylcholine (Ach) are released randomly from the nerve cell membrane at rest, each producing a
minute depolarizing spike called a m iniature end- p la te p o ten tia l.Q
Physiology: Nerves and Muscles ■ 59

Neuromuscular Junction

93. A i.e. Long term potentiation

Synaptic Plasticity and Learning

1. Post tetanic Potentiation (PTP) 3. Long Term Depression


- Production of enhanced post synaptic potentiation in - LTD is opposite to LTP characterized by decreased
response to brief tetanizing train of stim uli in pre synaptic strength.
synaptic neuron (which causes Ca++ to accumulate in - It is produced by slower stimulation of presynaptic
presynaptic neuron) neurons and a/w sm aller rise in intracellular C a++
- Lasts upto 60 seconds._____________________________ than LTP. It is d/t dephosphorylation of AMPA
2. Long term Potentiation (LTP) receptors.
- Like post tetanic potentiation but is much more - In cerebellum , LTD of clim bing fibers causes decrease
prolonged and can last for daysQ. firing of parallel fibers
- Unlike PTP it is initiated by increase in intracellular C aH+ - First noted in hippocampus but occurs throughout
in postsynaptic (rather than presynaptic) neuron. brain.
- Studied m axim ally in hippocampus (perforating 4. Habituation
substance pathway)Q.
5. Sensitizationz

94. C i.e. Prevention of Ca2+ influx 95. B i.e. Release of acetylcholine


96. B i.e. Acetylcholine binds to receptors on the postsynaptic sarcolemmal membrane
I Ref: G anong 24 fe p 1 1 5 , 1 0 4 , 1 2 7 - 2 9 ; G u yton 1 2 / e p 84-88; B erne & Levy 6/e p 234, Best & Taylor 13/e p 8 0 -8 1 1 A_

( ---------------------------------------------------------------------------------------
-
Action potential conducted along the nerve fiber enhances the endocytic release of ACh (acetyl choline) from vesicles
(packets) into the neuromuscular cleft by enhancing the perm eability of presynaptic membrane to Ca++ ions (1/t Ca++
influx not efflux through voltage gated channels).
- Sequence of events when a nerve impulse arrives at neuromuscular junction is: Calcium influx in presynaptic neural
membrane though voltage gated Ca++ channels —> Exocytosis of ACh from synaptic vesicles into synaptic cleft —»
ACh binds to Nm receptors on post synaptic sarcolemmal membrane which are ligand gated Na+ channels —» Na+
influx; development of EPP & current sink —> AP conducted in both directions and enter muscle fiber via T-tubules
—» Ca++ released (efflux) from sarcoplasmic reticulum (ryanodine receptor channels) in response to voltage change
sensed by DHPR on T tubules —> Binding of Ca++ to troponin C, uncovering myosin binding sites of actin 1/t formation
of cross linkage (bridging) that allows muscle contraction.
- R e le a s e o f s y n a p tic v e s ic le s fr o m p r e s y n a p tic term in a ls is in h ib ite d b y p re v en tio n o f c a lc iu m in flu x 0; examples include
Lambert-Eaton syndrome (LEMS) & use of aminoglycoside antibiotics. (Although preventing depolarization/
conduction/ Na+ influx can also prevent NM transmission but the best answer is prevention of Ca++ influx because of
above examples). LEMS, usually a/w cancer has autoimmune damage of neuromuscular voltage gated Ca++ channels
decreasing Ca++ influx and acetylcholine release resulting in muscle weakness. Aminoglycosides can also impair Ca++
channel function. Whereas aminopyridines used in treatment of LEMS facilitate ACH release (so improve muscle
strength) by blocking presynaptic K+ channels and activating voltage gated Ca++ channels.
V__________________________________________________________________________________________________/

97. D i.e. Decreased calcium outside reticulum [Ref: M t x o f N M ] transmission & m uscle contra ction]

ACh degradation at NMJ by ACh Esterase prevents continued muscle reexcitation (contraction)Q, whereas pumping
back of Ca++ from sarcoplasm into sarcoplasmic reticulum by ATP dependent pump (resulting in decreased Ca++
outside reticulum) ends skeletal muscle contraction (i.e. causes relaxation).

98. B i.e. Neuromuscular junction [Ref: Harrison 16]e P- 2518]

Myasthenia gravis is a n eu ro m u scu la r d is o r d e r Q characterized by weakness and fatiguability of skeletal muscle. The
underlying defect is a decrease in the number of available nicotinic Ach receptor at the N eurom uscular junction due to an
a n tib o d y m e d ia te d a u to im m u n e a t t a c k .0
60 " A Complete Review of Short Subjects

Muscle Contraction

99. B i.e. Two Z lines 100. B i.e. I-band

Skeletal Muscle: M yofibrils

I,Z AHM Skeletal muscle in relaxed state

I. Sacromere -» I
I (Light) Band - A (Dark) Band-
Z line M line Z line
It is formed by thin In this band myosin & action filaments I □ A ct in
I
filament mainly overlap I
■t-
actin. - This band is dark because of Myosin
The light (Isotropic) overlapping and presence of — i—
I band is divided myosin.___________
into two by Z line. H Zone - \—
Myosin
Z Line - This is a narrow lighter area at the
each myofibril is middle of A band
divided into number There is no actin in this zone.
K- -H
of compartments by M Line- H zone
z-line. A transverse line seen in the middle H -» «-
1 1 band A band
zone
★ So in Q of AIIMS 2016, A = A Band, B = Actin Filament, C Sarcomere is the portion of m yofibril in between two
adjacent Z lines. It is the structural and functional unit
= Sacromere width, D = H zone and E = Isotropic (Light)
I BandQ.
o f skeletal muscle 0 .
Sarcomere = Vi I band + one full A band + I band

101. C i.e. Myosin

Skeletal M uscle Morphology

• A muscle consist of large number of muscle fibers and each muscle fibres contains large number of myofibrils. Each
myofibril contains two types of filament:
- Actin (active, thin) Q
- M yosin (M for mota, thick) Q
• Myosin & actin can become connected with each other by cross bridges. Top of cross bridge is called myosin head 6 " it has
sites for attachment for actin and ATP.
• Each myosin filament is surrounded by six actin filaments.

__________ Thin Complex Thick: Myosin II Filaments


Contains 3 different proteins Myosin II Filaments

Actin Troponin Tropomyosin

- In its monomer form it has - Has three subunits I,T, & C In the resting state
binding sites for myosin, - Troponin I bind with actin tropomyosin molecule lie
Troponin & ATP - Troponin C binds with Ca3+ & undergoes on top of active site of
- Two polymers of actin wind confirmational change and shifts the tropomyosin actin filament.®
with each other to form thin molecule from the active site of actin, allowing
filament. myosin to bind & contraction proceed.
Physiology: Nerves and Muscles ■ 61

102. D i.e. Perlecan I Ref: G anong 22/e p. 68; Guyton VI /e p. 72, 90; Harrison 17/e p. 2683-84]

Caplin, dystrophin, dysferlin, dystroglycan, sacroglycan, caveoli & integrins are sacrolemmal proteins®. Where as
perlecan is a large multidomain proteoglycan, which is synthesized by both vascular endothelium and smooth muscle
cells and deposited in extra cellular matrix®.

103. B i.e. 1.6 microm

Myosin filament has a fixed (uniform) length of 1.5 microns®.

104. B i.e. H 105. C i.e. I band becomes wider 106. D i.e. Tropomyosin
107. B i.e. Covers myosin and prevents attachment of actin and myosin 108. A i.e. Tropomyosin
109. D i.e. Calcium binds to tropomyosin to initiate muscle contraction 110. B i.e. Calcium binding troponin C
IRef: G anong 24/e P- 97-104; C h a u dh u ri 5/e P- 428-30; S em buling u m 3/e P -127] .

In each sarcomere, during muscle contraction - H zone disappears (greatly narrowed ), I band is reduced (narrowed) ■
and A band remains unchanged®.
Tropomyosin protein prevents muscle contraction by covering the active sites of actin where myosin head binds to
actin® and thus preventing attachment of both.
Calcium (Ca++) binding to troponin C (not tropomyosin) initiates or triggers muscle contraction®.
During excitation-contraction coupling, ACh (acetylcholine) is released at the nerve terminal. Calcium is pumped
back into the sacroplasmic reticulum during relaxation. Calcium is released from sacroplasmic reticulum during
contraction

111. A i.e. Red 112. A i.e. Contain large amounts of myoglubin; B i.e. Composed of smaller fibers
113. A i.e. Extensive sarcoplasmic reticulum; C i.e. Large glycolytic pathway
IRef: B ern e & Levy 6/e p 243-44; Guyton 12/e p 7 9 ,1 0 3 6 ; G a n o n g 24/e p 106-107]
r
Fast twitch (type II) fibers have larger (more) diameter, number, sarcoplasmic reticulum, SERCA1, glycolytic
pathway & myosin ATPase activity. Type IIA are red oxidative-glycolytic whereas, II B are white glycolytic.
Slow twitch red glycolytic® (type I) fibers have smaller (lesser) diameter, number, force /velocity of contraction &
fatigability; glycolysis, SR, SERCA1 and ATPase activity but their m itochondrial content, capillary density,
myoglobin (responsible for red color) and oxidative (aerobic) capacity are higher.

114. B i.e. In the end IRef: G a n o n g 24/e p. 107; Gu yton 12/e p. 1036]

According to 'Size Principle' for recruitment of motor units during muscle contraction, a general schem e follow ed during
a specific m uscle action is__________________________________________________________________________________________________
(1) Initial recruitment of S (slow) muscle unit to produce controlled contraction. Eg. during standing.
i
(2) Followed by recruitment of FR (Fast Resistant to fatigue) units, resulting in more powerful response over a short
period of time. Eg during initiation of walking.
1
(3) Lastly (in the end) FF (Fast Fatiguable) units are recruited for most dem anding tasks eg running, jumping.

C la s s if ic a tio n o f F ib e r T y p e s in S k e le t a l M u s c le s
F e a tu re T y p e I: S lo w O x id a tiv e (S O ) T y p e IIA : F a st O x id a tiv e T y p e IIB : F a st G ly c o ly tic
R ed G ly c o ly tic (F O G ) R ed (F G ) W h ite
Color R ed (d/t myoglobin) R ed (d/t myoglobin) W h ite
S p e e d o f c o n tra c tio n (time to develop S lo w tw itch F ast tw itch F a st tw itch
peak tension)
M y o s in A T P a s e a c tiv ity (rate) S lo w F ast F ast
O x id a tiv e ca p a city , a c tiv ity o f H ig h Very high (a/t Berne): but L ow
e n z y m e s o f o x id a tiv e oxidative capacity is
p h o s p h o r y la tio n , g e n e r a tio n o f moderate a/t Ganong
a e ro b ic e n e rg y , m ito c h o n d r ia l
c o n te n t,m y o g lo b in , n u m b e r
(d e n sity ) o f c a p illa r ie s
62 ■ A Complete Review of Short Subjects

Glycolytic capacity & activity of Moderate High High


glycolytic enzymes
Fatigability Low (slow) b/o dependence of - High (rapid) b/o dependence
oxidative phosphorylation on glycolytic pathway
Used for More sustaind activity, for Occasionally & for brief
endurance eg maintaince of periods eg jumping , sprinters
posture, marathon, swimming
Associated motor unit type Slow (s) Fast resistant to fatigue (FR) Fast fatigable (FF)
Number of fibers; Fiber diameter; Lesser (smaller, lower) Higher (larger)
Force & velocity of contraction &
Fatigability
Diffusion distance (diameter) Modera te Small Large
Innervating nerve cell diameter & Smaller (fast) Larger (very fast)
conduction velocity
Excitability of nerve High Low
Sarcoplasmic reticulum content; Moderate High High
Calcium pumping capacity of SR;
SERCA 1 activity and Calcium
reuptake into SR
Relaxation time Slower Quicker (d/t SERCA1)
Begin to develop tension at Lower Ca++ concentration Higher Ca++ conc.
Low affinity Ca++ binding sites in 1 2
troponin C
Deliver Prolonged strength of Extreme amount of power for
contraction for minutes to few seconds to a minute
hours ie endurance
Example muscles Soleus muscle Lateral rectus > gastrocnemius
Sprinter, jumping
Sports activity Marathon, swimming

115. B & C i.e. Dephosphorylation causes definite relaxation & LC phosphatase is essential for binding
116. D & E i.e. IP3- DAG & Ca2+ channel Ref: Guyton llth /e p. 90, 98-99; Gabriel C Ezeilo P.56-57; Ganong 23rd/e p. 110-12,125
v V
- During smooth muscle contraction increase in cytosolic calcium is due to influx through voltage or ligand gated plasm a
membrane Ca++ channels ® and release of Ca++ from sarcoplasmic reticulum activated by IP3 receptor calcium channels ®
- In smooth muscles, length tension relationship does not exist & and contraction (slow & prolonged) and relaxation are
independent o f nerve supplyQ. M yosin light chain (MLC) phosphyrylation by MLC kinase ( not phosphatese) is
essential^ for its actin-binding. However MLC dephosphorylation by MLC phosphatese does not necessarily l/t
relaxation®. Latch bridges are less (attachment- detachment) cycle, less ATP requiring bridge, which maintain
prolonged contractions without much expenditure of energy.

117. A i.e. Ca2+ dependent; B i.e. Through calmodulin dependent kinase [Ref: Ganong 24/e P- 56-58; Chaudhuri 5/e P-455]

Calmodulin is a calcium (dependent) binding protein which acts through calmodulin dependent protein kinases® like
Ca++/ calmodulin kinase I, II, III; MLC kinase and phosphorylase kinase.

118. A i.e. Troponin (+ve) 119. D i.e. Presence of cellular calcium essential to cause muscle contraction.

In smooth muscles troponin is not required for contraction^. Presence of intracellular Ca2+ is required for the
contraction of all the three types of muscles -sm ooth, skeletal & cardiacQ.

Excitation - Contraction Coupling

_______________________In Smooth muscle____________________ __________In Cardia /Skeletal muscle_________


AP causes release of stored Ca2+—»Ca2+ binds to calmodulinQ —> Ca2+ ions initiate the process of contraction by
activates calmodulin dependent myosin light chain kinase —» binding with troponin CQ —> uncover myosin
phosphorylates myosin. Phosphorylation of myosin is essential binding sites on actin —> actin gets attached to
for contraction.^! myosin —» contraction_________________________
* Sustained contraction:- is produced d/t latch bridge mechanism. Q By which myosin cross bridges remains attached
to actin for some time after cytoplasm ic Ca2+ concentration falls.
Physiology Nerves and Muscles ■ 63

Skeleton M uscles Smooth M uscles


Contain Actin (A), Myosin (M), Tropomyosin (TM) & troponin Contain A, M, Tm but lack troponin*?
RMP > (-90mv), chronaxie is longer, contraction is phasic & EPP RMP < (-50 mv), tonic sustained contraction and
present EPP absent
Automaticity (A), Rythmicity (R), & Cross striations (C) absent & RAC and multiple nucleus present
has single nucleus

120. C i.e. Increasing the amplitude of action potential in the motor neurons.

Action potential follows an all or none law. If sufficient amplitude is reached to produces a response, increasing the
amplitude further will not have any effect on the function to follow.

121. B i.e. Fasciculation

Fasciculation is visible or palpable twitch within a single muscle due to sp on tan eou s discharge o f one m o to r unitSi
[Each single motor neuron & the muscle fibres it innervates constitute a motor unit]
Fibrillation Fine, irregular contraction of individual fibers, they are not visible grossly.
Tremor Rhythmic abnormal involuntary movement.
Chorea Rapid, jerky, semipurposive irregular movement more commonly occurring in the distal part.

122. B i.e. Ca+2


123. A i.e. Increased muscle metabolism by excess of Ca2+ ions. [Ref: Harrison 16/e P-105,390; Goodman & Gilman's 10/e P -204].

In malignant hyperthermia (d/1 mutation in gene coding ryanodine receptor) increased heat production is produced by
increased muscle metabolism by uncontrolled (excess) release of Ca++ ions from sarcoplasmic reticulum*?.

124. B i.e. Optimum length

Initial length Length at relaxed state


Optimal length Length at which tension in muscle i.e. force
generated by muscle is maximal*?.

125. A i.e. 25 Hz [Ref: Ganong 24/e p. 104]

- Tetany means continuous contraction o f muscle (fibre) without latency & relaxation (i.e. no latent period &
relaxation time)*?.
- So tetanizing frequency (i.e. the frequency at which muscles can contract) depends only on contraction time of muscle
(i.e. twitch duration) and not on latency period & relaxation time.___________________________________________________
- Because twitch (contraction) duration = 40 miliseconds = 0.04 seconds
So tetanizing frequency = 1/0.04 = 25 Hz*?
Chapter: 3 RESPIRATORY SYSTEM

QUESTIONS

Transport of Gases B. 60mmHg □


C. 80mmHg □
1. Defference in the amount of O 2 inspired and CO 2 D. lOOmmHg □
expired- (NBE P 13) 10 . Gas used to measure diffusion in lung:
A. 20ml/min □ A. CO (AIIMS 10) □
B. 50ml/min □ B. NO □
C. 75 ml/min □ C. c o 2 □
D. 100 ml/min □ D. Nitrogen □
2. Largest fraction of CO 2 is present in blood as or CO 2 is 11. CO 2 diffuse more easily than O 2 because:
primarily transported in the arterial blood as A. Less dense (AIIMS 1 0 ) 0
A. Dissolved CO 2. (AIIMS 03, AI 05) □ B. More soluble in plasma □
B. Carbonic Acid. □ C. Less molecular weight of CO 2 □
C. Carbamino-hemoglobin. □ D. Less p C 0 2 in the alveoli □
D. Bicarbonate □ 12 . Carbon monoxide transfer factor is increased in:
3. Haldane effect- (NBE P 13) A. Anaemia (NBE P 14; AIIMS 16,14; PGI 13,15) □
A. Effect of 2,3-BPG □ B. L—>R shunt & Polycythemia □
B. Dissociation of CO 2 on oxygenation □ C. R->L shunt □
C. Dissociation of O 2 on additionof CO 2 □ D. Pulmonary embolism □
D. Chloride shift □ 13. Arterial blood 0 2 in ml of 0 2 per dL: (PGI 01)
4. Venous blood with high hematocrit is seen in- A. 12.1 □
A. RBC high chloride. (NBE P 13) □ B. 19.8 □
B. Plasma high Na □ C. 15.6 □
C. Plasma high HCO 3 □ D. 27.8 □
D. RBC high K □ 14. The Normal value of PO 2 in healthy man is :
5. Normal diffusion of CO 2 at rest- (NBE P 13) A. 45 mm Hg (AIIMS 2K) □
A. 20-25ml/min □ B. 110 m m H g □
B. 50-100ml/min □ C. 80 mm Hg □
C. 100-200ml/min □ D. 60 mm Hg □
D. 300-400ml/min □ 15. Amount of dissolved oxygen transported in 100 ml of
6. Which of the following does not occur as the blood plasma in a subject breathing 100% oxygen at 4 ATA
passes through systemic capillaries? (AIIMS 07) (Atmospheric pressure) will be: (AI 12)
A. Increased protein content □ A. 9 ml □
B. Shift of Hemoglobin dissociation curve to left □ B. 6 ml □
C. Increased hematocrit □ C. 3 ml □
D. Decreased pH □ D. 0.3 ml □
7. Hemoglobin unlike myoglobin shows: (PGI 02) 16. The additional amount of oxygen transported in 100
A. Sigmoid curve of oxygen dissociation □ ml of blood in a subject breathing 1 0 0 % oxygen under
B. Positive coopera tivity □ hyperbaric conditions of 4 ATA compared to
C. Hills coefficient of one □ normobaric conditions (1ATA) w ill be: (AI 12)
D. None of above □ A. 9 ml □
8. The normal value of P50 on the oxyhaemoglobin B. 6 ml □
dissociation curve in an adult is: (AIIMS 04) C. 3 ml □
A. 1.8 kPa □ D. 0.3 ml □
B. 2H kPa □ 17. Percentage of O 2 carried in chemical combination
C. 3.6 kPa □ A. 97% (PGI 0 4 ) 0
D. 4.5 kPa □ B. 3% □
9. One intern calculated the concentration of O 2 in blood C. 6 6 % □
as 0.0025 ml/ml of blood. Considering atmospheric D. 33% □
pressure as 760mmHg, how much approx. O 2 tension 18. Arterial carbon dioxide level (PGI 04)
could have been in the blood? (AI 04) A. 40 mm Hg □
A. 40mmHg □ B. 37 mm Hg □
Physiology: Respiratory System ■ 65

c. 45 mm Hg □ __________ 0 2 Dissociation Curve______________


D. 60 mm Hg □
□ 25. The sigmoid nature of Hb - 0 2 dissociation curve is
E. 42 mm Hg
because o f : (NBE P 14,13; AI 09, AIIMS 11)
19. O2 delivery to tissues depends on all/except:
A. Binding of one O 2 molecule increase the affinity for
A. Cardiac output (AIIMS 07) □
□ the next O 2 molecule. □
B. Type of fluid administered
□ B. Alpha chain has more affinity for O 2 thanbeta
C. Hemoglobin concentration
□ chain □
D. Affinity of hemoglobin for O 2
C. Beta chain has more affinity for O 2 than alpha
20 . The fraction of inspired air in mouth-to-mouth
respiration is: (AI 06, NBE P 13) chain and Haemoglobin is acidic in nature □
D. Binding of one oxygen molecule decreasethe
A. 0.16 □
B. 0.19 □ affinity of binding other O 2 molecules □
26. Increased oxygen delivery to tissues in reponse to
C. 0.21 □
increased CO 2 is- (NBE P 13)
D. 0.26 □
A. Bohr effect □
21. Mouth-to-mouth respiration provides an oxygen
concentration of: (DNB 14, AI 04) B. Haldane Effect □
A. 16% □ C. Hamburger effect □
D. Chloride shift □
B. 20% □
27. Bohr Effect is? (NBE P 13)
C. 22% □
A. Facilitates oxygen transport □
D. 24% □
22 . Arterial blood gas of a 5 year old child done at sea level B. Facilitates CO 2 transport □
gives the following results: pH 7.41, PaC>2 100 mmHg, C. Facilitates chloride transport □
and PaCC>2 40 mm Hg. The child is being ventilated D. None □
with 80% oxygen. What is the (A-a) P 0 2 (AIIMS 05) 28. In anemia, the concentration of 2 , 3-DPG is-
A. 570.4 mm Hg □ A. Decreased (DNB 09) □
B. 520.4 mm Hg □ B. Increased □
C. 470.4 mm Hg □ C. A or b □
D. 420.4 mm Hg □ D. Not changed □
23. What is charle's law- (NBE P 13) 29. Role o f 2 ,3 DPG in hem oglobin - (Al 06)
A. PV=constant □ A. Unloading oxygen to tissues □
B. P/T=constant □ B. Increased affinity for oxygen □
C. PV=nRT □ C. Buffering capacity □
D. None □ D. Osmotic fraglity □
30. An increase in the concentration o f 2-3
24. In the follow ing graph, the curve A represents the
normal relationship between Alveolar ventilation and Biphosphoglycerate (2, 3 DPG) may be seen in all of
p C 02, when p 0 2 is 100 mm Hg. O f pH is changed the follow ing except. (Al 09, NBE P 12)
from 7.4 to 7.3, where w ill the original curve shift? A. Anemia □
(AIIMS 16) B. Hypoxia □
C. Inosine □
D. Hypoxanthine. □
In hyperventilation (PGI 07)
A. p50 and Hb affinity for O 2 increases □
B. p50 and Hb affinity for O 2 decreases □
C. p50 increases and O 2 affinity decreases □
D. p50 decreases and O 2 affinity increases □
E. no change □
Increase in P50 in oxygenation curve is due to
decrease in- (NBE P 13)
A. PH □
B. Oxygen □
C. Temperature □
D. C 0 2 □
Fetal hemoglobin has higher affinity for oxygen due
to: (AIIMS 09)
A. C □ □
A. Decreased 2, 3 DPG concentration
B. A □ □
B. Reduced pH
C. B □ □
C. Increased release of carbon dioxide
D. D □ □
D. Oxygen dissociation curve is shifted to right
66 ■ A Complete Review of Short Subjects

34. During exercise increase in O 2 delivery to muscles is 44. O 2 delivery to tissue is decreased by A/E: (NBE P 14)
because of all e x c e p t: (AI 2K, DNB 09) A. l ed Haemoglobin level (PGI 03,13) □
A. Oxygen dissociation curve shifts to left □ B. led Pa02 □
B. Increased stroke volume □ C. Ted pH. □
C. Increased extraction of oxygen from the blood □ D. T e d HCO 3 & Ted Pa C 0 2 □
D. Increased blood flow to muscles □ 45. The factor responsible for the left shift of Hb - 0 2
35. Oxygen dissociation curve shifts to right in all except: dissociation curve is: (AIIMS 14,15, 09)
A. Diabetic ketoacidosis (AI 2K, DNB 11) □ A. Increase in 2,3 DPG in RBC □
B. Blood transfusion □ B. Fall in temperature □
C. High altitude □ C. Fall in pH □
D. Anaemia □ D. Increase level of CO 2 in blood □
36. W hich compound shifts the Oxygen dissociation 46. Which of the follow ing statement (s) is /are true about
curve to the right: (NBE P 14,13; A I 04) Hb - 0 2 dissociation curve:
A. 1, Phosphoglycerate □ (NBE P 15,13,14,13; PGI 12,08)
B. 2, 3 DPG □ A. Fetal Hb & hypothermia Shifts curveto Left □
C. 1, 3 DPG □ B. Hyperthermia shifts curve to Left □
D. Glyceraldehyde □ C. Hypercarbia shifts curve to Left □
37. Oxygen dissociation curve shift to right in A/E:
D. Left shift cause more 0 2 release to tissue □
A. Hypothermia/Fetal Hb □
B. Hypercarbia (NBE P 15,14,13; PGI 14,13)0
Lung Volume & Capacities
C. Anemia □
D. Sickle Hb □
47. Tidal volume is calculated by: (AI 06)
38. O 2 dissociation curve is shifted to right in all except:
A. Inspiratory capacity minus the inspiratory reserve
A. Hypercapnea /Hypercarbia (TCO 2) □
volume □
B. Rise in temperature (AIIMS 08,14; NBE P 15,13) □
B. Total lung capacity minus the reserve volume □
C. Raised2, 3 DPG level □
C. Functional residual capacity minus residual
D. Metabolic alkalosis (Decreased H+ ion) □
volume □
39. The im portant feature of 2.3 Diposphoglycerate
includes: (Al 07) D. Vital capacity minus expiratory reserve volumes □
48. Volume of air taken in and given out during normal
A. Higher concentration in adult blood □
respiration is referred to as:
B. Contribution to Bohr effect □
A. IRV (NBE P 15,13; PGI 0 2 ,14,13) □
C. Increases affinity of O 2 to haemoglobin □
B. TV □
D. Associated with foetal blood to promote
C. ERV □
oxygenation. □
40. True about Hb dissociation curve is:
D. VC □
49. Functional residual capacity is: (AIIMS 03)
A. Acidosis shifts O 2 dissociation curve to right □
A. Volume remaining after forced expiration □
B. T CO 2 shifts the curve to left (PGI 08,09) □
B. Tidal volume + volume inspired forcefully □
C. Hypoxia shifts curve to left □
C. Volume remaining after normal expiration □
D. 2.3 DPG has no effect on curve □
41. All of the following factors influence hem oglobin
D. Tidal volume + volume expired by forced
dissociation curve, except: (PGI 12, AIIMS 06) expiration. □
A. Chloride ion concentration □ 50. Functional residual capacity of lung is defined as;
B. CO 2 tension (pressure) □ A. Volume expired after normal expiration □
C. Temperature □ B. Volume remaining after forced expiration □
D. 2-3 DPG levels □ C. ERV + RV (PGI 07) □
E. Acidosis □ D. Tidal volume + volume inspired forcefully □
42. An increase in which of the following param eters will 51. Total lung capacity depends upon : (AI 08)
shift the O 2 dissociation curve to the left: A. Size of airway □
A. Temperature (NBE P 14, AI 03) □ B. Closing volume □
B. Partial pressure of CO 2 □ C. Lung compliance □
C. 2,3 DPG concentration □ D. Residual volume □
D. Oxygen affinity of hemoglobin □ 52. Regarding pulmonary function test all are TRUE,
43. Oxygen affinity is increased by all of the following EXCEPT: (AIIMS 09)
except: (NBE P 13, AI 05) A. Total lung volume increases in emphysema □
A. Alkalosis □ B. Compliance decreases in interstitial lung
B. Hypoxia □ disease. □
C. Increased HbF □ C. Compliance is total lung distensibility □
D. Hypothermia □ D. FEVI is forced expiratory rate at one minute □
Physiology: Respiratory System ■ 67

53. Which of the following is used to measure the C. Decreased FEV □


resistance to small airways OR Best denotes airway D. Decreased vital capacity □
resistance (NBE P 15,13; AIIMS 15,07) 63. Pulmonary function changes seen in Emphysema are
A. Vital capacity □ A. tTLC (AI 06) □
B. FEV, □ B. J-RV □
C. Max. mid respiratory flow rates. □ C. Tf e v , □
D. Closing volume □ D. T v c □
54. Respiratory minute volume of lung is- (NBE P 13) 64. A person is having normal lung compliance and
A. 6 L □ increased airway resistance. The most economical way
B. 4L □ of breathing for him: (AIIMS 02)
C. 500mL □ A. Rapid & deep □
D. 125L □ B. Rapid & shallow □
55. Total alveolar ventilation volume (in L/min) is: C. Slow & deep □
A. 1.5 (AI 10, NBE P 13) □
D. Slow & sallow □
B. 3.5 □
C. 4.2 □
Test for Pulmonary Compliance
D. 5.0 □
56. Calculate the Alveolar ventillation per minute of a
patient with respiratory rate 14/min, tidal vol. 500 ml 65. In body Plethysmography, a person is asked to expire
with a vital capacity 7000 m l: (AIIMS 01) against a closed glottis. Whiat will be the change in
A. 4900 ml □ the pressure in the lung and the box (AI 11)
B. 2000 ml □ A. Increase in both □
C. 7700 ml □ B. Decrease in both □
D. 7000 ml □ C. Increase in lung decrease in box □
57. In normal adult Vd/Vt ratio is: (PGI 13, NBE P 13) D. Decrease in lung increase in box □
A. 20 □ 66 . Vital capacity is sum of- (NBE P 13)
B. 0.35 □ A. Inspiratory reserve volme, Tidal volume and
Expiratory reserve volume □
C. 40 □
B. Tidal volume, Inspiratory reserve volume and
D. 50 □
Residual volume □
58. Critical Closing volume is (AIIMS 12)
C. Expiratory rerve volume, Inspiratory reserve
A. Volume at the end of forceful expiration □
volume and Residual volume □
B. Volume at the end of forceful inspiration □
D. Residual volume, Inspiratory volume, and
C. Volume remaining after Functional Residual
Expiratory volume. □
Capacity is measured □
67. Spirometry measure (s): (PGI 12)
D. Close to Residual Volume □
59. True about Hyaline membrane disease: A. Tidal volume □
(AIIMS 10, NBE P 13) B. TLC □
A. FRC above the closing volume □ C. ERV □
B. FRC below the closing volume □ D. FRC □
E. IRV □
C. FRC is equal to closing volume □
68 . Spirometry can demonstrate and measure all of the
D. FRC doesn't depend on closing volume □
follow ing except: (NBE P 15,14,13; AI 10,03; AIIMS 15)
60. Pulmonary function abnormalities in interstitial lung
A. Tidal volume □
diseases include all of the follow ing except:
A. Reduced vital capacity (AIIMS 05) □ B. Residual volume □
B. Reduced FEVI/FVC ratio □ C. Vital capacity □
C. Reduced diffusion capacity □ D. Inspiratory reserve capacity □
D. Reduced total lung capacity □
69. Routine spirometry can't estimate : (PGI 08, 04)
61. Set of data which correctly defines restrictive lung A. FRC □
disease is: (AI 08, NBE P 13) B. VC □
A. TFRC,T compliance of lung tissue □ C. RV □
B. TFEV i /FVC, ^compliance of lung tissue □ D. ERV □
C. IFEV i /FVC, ^compliance of lung tissue □ E. FEV, □
D. TTLC, RV is i □ 70. Nitrogen washout method is used for estimating:
62. In upper air way obstruction all of the following A. Dead space volume (PGI 04) □
changes are seen excep t: (AI 09) B. Functional residual capacity □
A. Decreased Maximum breathing capacity □ C. Tidal volume □
B. RV decreased □ D. Diffusion capacity □
68 ■ A Complete Review of Short Subjects

71. Measurement of anatomic dead space is by- Pressure During Breathing Cycle
A. O 2 breath test (NBE P 13) □
76. Intrapleural pressure is- (NBE P 13)
B. Helium dilution test □
A. Transpulmonary pressure+Alveolar pressure □
C. N 2 breath test □
B. Transpulmonary pressure -Alveolar pressure □
D. PCO 2 □
C. Transmural pressure+Alveolar pressure □
72. Regarding Dead space Volume in a normal
D. Alveolar pressure-Transpulmonary pressure □
individual- (NBE P 13)
77. True about inspiration; the intrapleural pressure
A. Anatomical dead space>Physiological Dead space □
becomes: (PGI 06)
B. Anatomical dead space=Physiological Dead space □
A. More - ve □
C. Anatomical dead space>Physiological Dead space □
D. Anatomical dead space is not related to
B. More +ve □
Physiological Dead space □
C. Same □
73. Which of the following methods is used for
D. Initially positive, then negative □
calculation of anatomical dead space? (AIIMS 15)
E. No relation □
78. The intrapleural pressure is negative both during
A. Xenon dilution technique □
inspiration and expiration because: (AIIMS 05)
B. Bohler's method □
A. Intrapulmonary pressure is always negative □
C. Spirometry □
B. Thoracic cage and lungs are elastic structure □
D. Single breath nitrogen test □
C. Transpulmonary pressure determines the
74. Examine the shape of the flow volume curve
negativity □
carefully. This flow volume loop indicates one of the
D. Surfactant prevents the lungs to collapse □
following: (AIIMS 04)
79. Normal Intrapleural Pressure is Negative because
A. The chestwall and lungs recoil in opposite
directions (A ll 2 )0
B. The surfactant prevents pulmonary collapse □
C. Intrapulmonary Pressure is Negative □
D. Tranplmonary Pressure is Negative □
80. Negative Intrapleural pressure is maintained by:
(AIIMS 10; AI 12)
A. Uniform distribution of surfactant over alveoli □
B. Cartilaginous rings in large airways □
C. Lymphatic drainage of pleural cavity □
VITAL CAPACITY D. Negative intraveolar pressure □
81. Increased airway resistance is/are caused by (PGI 11)
A. Normal □ A. Forced expiration □
B. Extrathoracic obstruction □ B. Dense air □
C. Intrathoracic obstruction □ C. Low lung volume □
D. Fixed large airway obstruction □ D. High lung volume □
75. The given graph likely depicts which of the following 82. More resistance in expiration is due to: (AIIMS 03)
disease? (AIIMS 15) A. Increased compression of airway □
A. Bronchial asthma □ B. Due to change from linear to turbulent flow □
B. Emphysema □ C. Saturation with moisture □
C. Interstitial lung disease □ D. Increased rate of flow during expiration □
D. Normal study □ 83. Respiration stops in the last stage of expiration, in
forced expiration b/c of: (AIIMS 08)
A. Respiratory muscle fatigue □
B. Collapse of alveoli □
C. Dynamic compression of airways □
D. Breaking effect of inspiratory muscles □
Expiration
84. True about normal expiration (PGI 10)
A. At the end of normal expiration air in lung is ERV □
B. At the end of normal expiration air in lung is ERV □
C. In expiration pleural pressure is equal to alveolar
pressure □
D. Muscles that elevate the chest cage are classified as
muscles of expiration □
Physiology: Respiratory System ■ 69

Work Load of Breathing C. Posterior lobe of lung □


D. Middle of the lung □
85. Effort during normal respiration is done due to - 94. Ventilation perfusion ratio is least at- (NBE P 15,13)
A. Lung elasticity (DNB 04, Delhi 13) □ A. Apex Cl
B. Respiratory air passages □ B. Middle lob □
C. Alveolar air spaces □ C. Base □
D. Creating negative pleural pressure □ D. None □
86 . Which of the follow ing adaptations w ill be apt to 95. True statement regarding Pulmonary ventilation is
increase the work capacity at high altitude: (Al 07) A. PaC>2 is maximum at the apex. (AI 04) □
A. Increasing workload, decreasing duration of B. V/Q is maximum at the base. □
exercise □ C. Ventilation per unit lung volume is
B. Increasing workload, increasing duration of maximum at the apex □
exercise □ D. Blood circulation is minimum at the base. □
C. Decreasing workload, increasing duration of 96. Not a stimulus for pulmonary vasoconstriction:
exercise □ A. Hypoxemia (NBE P 15,13; AIIMS 0 4 , 15) □
D. Decreasing workload, decreasing duration of B. Hypercapnia (Hypercarbia) □
exercise □ C. PGI2 □
87. When gases flow through an orifice which factor to D. Thromboxane A 2 □
least likely to effect turbulence- (NBE P 13)
A. Density of gas □ Hypoxia
B. Viscosity of gas □
97. All of the follow ing cause Hyperventilation except:
C. Pressure of gas □
A. Decreased pH in CSF (DNB 10) □
D. Diameter of orifice □
B. Decreased plasma HCO 3 □
C. CO poisoning □
Compliance
D. Increased adrenergic levels □
98. Arterial blood gas analysis in Carbon monoxide
88 .Specific lung compliance is decreased in all except poisoning shows: (AIIMS 07)
A. Pulmonary congestion (AIIMS 15, AI 1 1 ) 0 A. PO 2 less, O 2saturation normal □
B. Chronic bronchitis □ B. PO 2 normal, O 2saturation less with normal or
C. Pulmonary fibrosis □ slightly decreased PCO 2 □
D. Decreased surfactant □ C. PO 2 less, O 2 saturation normal □
89. Compliance of lung is measured by- (NBE P 14,13) D. PO 2 less, O2 saturation decreased □
A. Elasticity □ 99. Which o f the follow ing variants of hypoxia does not
B. Amount of air □ stimulate peripheral chemoreceptors OR Oxygen
C. Blood flow □ therapy is least useful in: (NBE P15,14,13; AI 07)
D. Presence if fluid □ A. Hypoxic hypoxia (AIIIMS 14) D
90. Compliance of lungs is- (AIIMS 14; NBE P 13) B. Anaemic hypoxia □
A. 200ml/cm water □ C. Stagnant hypoxia □
B. 500ml/cm water □ D. Histotoxic hypoxia □
C. 800 ml/cm water □ 100 . Anemic hypoxia is due t o : (AI 06)
D. 1000 ml/cm water □ A. T P 0 2 in arterial blood □
B. TPO 2 in arterial blood □
Ventilation & Perfusion C. TPCO 2 in arterial blood □
D. 1 O 2 content in arterial blood □
91. High Oxygen tension in alveoli is due to: 101. Which is the best parameter for analysis of Hypoxic
A. Right to left shunt (AIIMS 04) □ hypoxia: (AI 04)
B. Ventilation perfusion mismatch □ A. Arterial PO 2 □
C. Bronchial Asthma □ B. Arterial PCO 2 □
D. Inappropriate gas exchange □ C. Venous PO 2 □
92. PO 2is m axim um - (NBE P 15,14,13) D. A-V Difference □
A. Base of lung □ 10 2 . Which of the follow ing conditions leads to tissue
B. Posterior lobe □ hypoxia without alteration of oxygen content of
C. Apex of lung □ blood? (AIIMS 05)
D. Middle lobe □ A. CO poisoning □
93. Ventilation perfusion ratio is maximum a t : B. Met Hb □
A. Apex of lung (NBE P 15,14; AI 10) O C. Cyanide poisoning □
B. Base of lung □ D. Respiratory acidosis □
70 ■ A Complete Review of Short Subjects

103. Which combination of the following statements is A. 10.9 & 4.1 □


correct with reference to hypoxia: (AIIMS 04) B. 10.9 & 5.1 □
a) When it is severe, causes stimulation of the C. 10.9 & 6.1 □
sympathetic nervous system D. 8.9 & 4.1 □
b) It leads to the accumulation of hydrogen and E. 8.9 & 6.1 □
lactate ions 112. Cyanosis in trauma is interpreted as:
c) It causes decrease in cerebral blood flow A. Early sign of hypoxia (AIIMS 10) □
d) If it is chronic, causes rightward shift of oxygen Hb B. Latesign of Hypoxia □
curve C. Absence of cyanosis indicates adequate airway /
A. All of the above statements are correct □ ventilation □
B. b&c D D. Absence of cyanosis indicates good tissue
C. a,b & d □ oxygenation □
D. b,c & d □ 113. Concentration of methemoglobin to appears cyanosis
104. Tachycardia is caused by hypoxia due to: A. 5 gm/dl (PGI 07) D
A. Reflexly through peripheral chemoreceptors □ B. 2 gm/dl □
B. Diffuse vasodilatation (AIIMS 08) □ C. 1.5 gm/dl □
C. Through central chemoreceptor □ D. 12 gm/dl □
D. Secondarily after by hyperventilation □ 114. Cause of methaemoglobinemia are all excep t:
105. The neurons may get irreversibly damaged if exposed A. Nitrites (NBE P 13) □
to significant hypoxia f o r : (AIIMS 02) B. Phenacetin □
A. 8 min □ C. Sulfonamide □
B. 2 min □ D. Phenytoin □
C. 30 sec □ 115. Antidote for sodium nitrate poisoning is : (NBE P 13)
D. 15 sec □ A. Methylene blue IV □
106. Which of the follow ing is most prone to hypoxic B. Egg albumin □
injury (PGI 04) C. EDTA □
A. Thalamus □ D. Animal charcoal □
B. Hippocampus □ 116. Apnoea is defined as: (PGI 03)
C. Caudate nucleus □ A. Stoppage of heart beat □
D. Cerebellum □ B. Cessation of respiration □
107. Hypoxemia does not depend on: C. Irregular respiration □
A. P aC 0 2 (AIIMS 10) □ D. RR □
B. Altitude □ 117. In which of the follow ing a reduction in arterial
C. Hb □ oxygen tension occurs? (AI 05)
D. F i0 2 □ A. Anaemia. □
B. CO poisoning. □
Cyanosis
C. Moderate exercise. □
108. Condition where severe hypoxaemia occurs without D. Hypoventilation. □
cyanosis (PGI 13,15; NBE P 15,13)
CO, O2 and CO2 Toxicity
A. CO poisoning & Anemia □
B. High altitude □ 118. Which of the follow ing statements about Carbon
C. Interstitial lung disease □ Monoxide poisoning is not true: (AI 12)
D. Pulmonary A. V. malformation □ A. Carboxy haemoglobin (COHb) causes left shift
109. Cyanosis does not occur in severe anemia because: of oxyhaemoglobin dissociation curve □
A. Hypoxia stimulates erythropoietin production □ B. Partial pressuresas low as 0.6 mm Hg can be
B. Oxygen carrying capacity of available Hb is lethal □
increased (AI 09 ) □ C. Tissue toxicity plays an important role in clinical
C. Critical concentration of Hb required to produce CO poisoning □
cyanosis is reduced □ D. Hyperbaric oxygen is used for treatment □
D. Oxygen Hemoglobin curve shift to the right □ 119. Regarding carbon monoxide toxicity true are A/E
110 . Central cyanosis is seen if: (PGI 01) A. Cytochrome toxicity is lethal (PGI 08) □
A. Methemoglobin 0.5 gm/dl □ B. Treated by 5% CO 2 □
B. 0 2 saturation < 85% □ C. PO2 is decreased □
C. 0 2 saturation < 94% □ D. Shift H b 0 2 dissociation cure to left □
D. Hb - 4 gm% □ E. Gaseous vasodilator □
1 1 1 . Central cyanosis not occurs when total Hb and 120 . Transport of Carbon monoxide (CO) is diffusion
reduced Hb level is respectively (in gm%): (PGI 12) limited because: (AIIMS 09)
A. High affinity of CO for haemoglobin □
Physiology: Respiratory System ■ 71

B. Alveolar membrane is less permeable to CO □ 130. If a cat apneustic center is destroyed along with
C. CO crosses epithelial barrier slowly □ cutting of vagi. Which of the follow ing statement is
D. On exposure to air there is sudden increase in correct regarding the breathing pattern seen in cat?
partial pressure □ A. Prolonged inspiratory spasm (PGI 05) □
1 2 1 . Carbon monoxide poisoning is a type of- (NBE P 13) B. Prolonged expiratory spasm □
A. Anemic hypoxia □ C. Slow and shallow respiration □
B. Histotoxic hypoxia □ D. Animal will die □
C. Hypoxic hypoxia □ 131. Inspiratory depth is halted by- (NBE P 13)
D. Stagnant hypoxia □ A. Pneumotaxic centre □
122 . Hyperbaric oxygen is dangerous because it: B. Apneustic centre □
A. Decreases displacement of O 2 from kHb □ C. Inspiratory centre □
B. Decreases respiratory drive (PGI 04) □ D. Expiratory centre □
C. Enzyme damage (□ 132. Transection at mid pons level results in?
D. Is toxic to tissures □ A. Hyperventlation (AIIMS 09) □
123. Toxic effects of high oxygen tension include all o f the B. Apneusis □
follow ing except: (AI 07) C. Rapid and shallow breathing □
A. Pulmonary edema □ D. Hypoxia □
B. Decreased cerebral blood flow □ 133. What will be the effect on respiration if a
C. Retinal damage □ transmission is made between the pons & medulla
D. CNS excitation and convulsion □ A. Apnoea (AIIMS 04) □
Smoking B. Irregular & gasping □
C. No effect □
124. Physiological response to sm oking, are all, except: D. Slow & deep □
A. Decreased HDL (AIIMS 09) □ 134. CO 2 increases ventilation by acting mainly on
B. Increased hematocrit □ receptors of- (NBE P 13)
C. Increased heart rate and increased catecholamine A. Apneustic center □
release □
B. Pneumotaxic center □
D. Decreased carboxyhemoglobin □
C. Ventral surface of medulla □
Respiratory Failure D. DRG □
135. True regarding respiratory center is (AIIMS 03)
125. The gradient of alveolar arterial oxygen tension
A. Directly stimulated by fall in PaCh □
increases in A/E: (AIIMS 08)
B. Inhibited during swallowing □
A. Diffusion defect □
C. Connected with cardiac center □
B. Right - Left shunt □
D. Situated in midbrain □
C. Hypoventilation □
136. Central Chemoreceptors are mot sensitive to
D. Ventilation perfusion abnormality □
follow ing changes in blood: (AI 09)
126. Alveolar hypoventilation is present in A/E:
A. TPCO 2 □
A. Bulber poliomyelitis (AIIMS 05) □
B. IPC O 2 □
B. COPD □
□ C. ?H + □
C. Kyphoscoliosis
D. Lobar pneumonia □ D. t P 0 2 □
127. CO 2 retention is seen in A/E: (PGI 10, 01) 137. The primary direct stimulus for excitation of central
A. Carbon monoxide poisoning/ High altitude □ chemoreceptors regulating ventilation is:
B. Respiratory / Ventilatory failure □ A. tH+ (AIIMS 11, AI 09) D
C. Pulmonary edema □ B. TC 02 □
D. Drowning □ c. to 2 □
128. H ypercarbia is characterized by: (AIIMS 01) D. IC O 2 □
A. Miosis □ 138. M ost important stimulus to peripheral chemoreceptrs
B. Cool extremities □ A. PO 2 (NBE P 13) □
C. Bradycardia □ B. C 02 □
D. Hypertension □ C. pH □
D. HCO 3 □
Regulation Of Breathing
139. Peripheral & central chemoreceptors may both
129. Pacem aker of respiration OR Spontaneous rhythm ic contribute to increased ventilation that occurs as a
respiration initiated in? (AIIMS 10,14,15) result of decreased- (NBE P 13)
A. Dorsal nuclear group □ A. Arterial BP □
B. Apneustic centre □ B. Arterial tension □
C. Pnemotaxic centre □ C. Arterial O 2 concentration □
D. Pre Botzinger complex □ D. H+ □
72 « A Complete Review of Short Subjects

140. Which of the following does NOT stimulate C. Dipalmitoyl cephaline □


peripheral chemoreceptors: (NBE P 15,13; AI 05) D. Dipalmitoyl serine □
A. Hypoxia □ 150. Pulmonary surfactant (PGI 08)
B. Hypocapnia □ A. Secreted from 26th weeks of gestation & maintain
C. Acidosis □ minimum alveolar integrity □
D. Low perfusion pressure □ B. In Hyaline membrane disease deficiencyoccurs □
141. Administration of pure O 2 to hypoxic patients is C. Theraputic application seen □
dangerous because: (PGI 09) D. Abundant secretion in bronchoalveolar
A. Apnea occurs due to hypostimulation of peripheral carcinoma □
chemoreceptors □ E. It is mucin & secreted by pneumocyte 1 □
B. Pulmonary edema □ 151. Stability of alveoli is maintained by?
C. DPG □ A. Lung compliance (AIIMS 09) □
D. Convulsions □ B. Negative intrapleural pressure □
142. Tidal volume excessive load is prevented by C. Increase in alveolar surface area by the surfactant □
activation of which of the follow ing receptors: D. Residual air in alveoli □
A. J receptor (AIIMS 11) □ 152. Correct about action of surfactant? (AIIMS 07)
B. Thoracic muscle spindle □ A. Binds 0 2 □
C. Bronchial stretch receptors □ B. Lubricates C 0 2 flow □
D. Arterial baroreceptor □ C. Make capillary surface hydrophilic □
143. Herring Breuer reflex is an increase in- (NBE P 13) D. Breaks structure of water in alveoli □
A. Duration of inspiration □ 153. Alveoli are kept dry because of- (NBE P 13)
B. Duration of expiration □ A. Surfactants □
C. Depth of inspiration □ B. Glycoproteins □
D. Depth of expiration □ C. Buffers □
144. "Inflation of lungs induces further inflation" is D. Bohr's Effect □
explained by: 154. Hyaline membrane contains: (N E E T 12, A l 04)
A. Hering-Breuer inflation reflex (AIIMS 03) □ A. Albumin □
B. Hering-Breuer deflation reflex □ B. Fibrin □
C. Head's paradoxical reflex □ C. Globulin □
D. J-reflex □ D. WBCs □
145. All of the follow ing affect resting ventilation except
A. Stretch receptors (AIIMS 10) □
Non Respiratory Functions of Lung
B. J receptor □
C. Oxygen □
D. PCO 2 □ 155. Which of the follow ing is the best-known metabolic
function of the lung: (AIIMS 04)
A. Inactivation of serotonin □
Surfactant
B. Conversion of angiotensin-I to angiotensin-II □
C. Inactivation of bradykinin □
146. Surfactant production in lungs starts at:
D. Metabolism of basic drugs by cytochrome P-450
A. 28 weeks (AI 04,NBE P 14,13) □
system □
B. 32 wks □
156. An important non-respiratory function of Lungs:
C. 34 wks □
A. Anion balance (A I10)D
D. 36 wks □
B. Sodium balance □
147. Surfactant is produced by : (NBE P 14,13; AI 04)
C. Potassium balance □
A. Type II pneumocytes □
D. Calcium balance □
B. Type I pneumocytes □
157. Which of the following is true for the pulmonary
C. Macrophages □
neuroendocrine cells? (AIIMS 13)
D. Endothelial cells □
A. Contain catecholamines □
148. Surfacant is made up of O R In human body, the
B. Contain serotonin □
action of surfactant is done by: (NBE P 15; AIMS 13,06)
C. Have only cholinergic nerve supply □
A. Fibrin □
D. Respond to a decrease in PCb □
B. Mucoprotein □
158. True is all except about Acid-Base disorder
C. Phospholipids & Proteins □
A. pH determined by Pco 2 and HCO 3- (AIIMS 10) □
D. Fibrinogen □
B. Respiratory acidosis is compensated by I HCO 3- □
149. Main part of surfactant is formed by- (NBE P 15)
C. Metabolic acidosis is compensated by 'LPco2 □
A. Dipalmitoyl phosphatidyl choline □
D. Buffering may be intra & extra cellular □
B. Phosphotidyl-inositol □
Physiology: Respiratory System ■ 73

159. Uncompensated metabolic acidosis shows- (NBE P 13) 165. In high altitude mountain sickness, feature of
A. Increased pH with increased HCO3. □ pulmonary edema is. (PGI 06)
B. Increased pH with Decreased HCO3- □ A. Decreased pulmonary capillary permeability □
C. Decreased pH with increased HCO3. □ B. Increased pulmonary capillary pressure □
D. Decreased pH with Decreased HCO3- □ C. Normal left atrial pressure □
D. Increases left ventricular back pressure □
High Altitude 166. Compensating mechanisms involved in
acclimatization to altitude: (PGI 04)
160. During acclimatization to high altitude all of the A. Hyperventilation □
following take place except: (AI 04) B. Hypoventilation □
A. Increase in minute ventilation □ C. Respiratory depression □
B. Increase in the sensitivity of central D. Respiratory acidosis □
chemoreceptors □ E. Respiratory alkalosis □
C. Increase in the sensitivity of carotidbody to
hypoxia □ Decompression Sickness
D. Shift in the oxygen dissociation curve to theleft □
161. A person goes to the mountains. When he reaches 167. In caissons disease pain in joint is because of-
about 5000 ft. he develops dyspnea. Which of A. Nitrogen bubble (NBE P 14,13) □
following correctly explains for the symptoms: B. Oxygen bubble □
A. CNS depression (AIMS 2K) □ C. Carbon monoxide □
B. CO2 wash out. □ D. Air in joint □
C. Increased work of breathing □ 168. Caisson's disease (NBE P 14,13; AIIMS 13)
D. Increased blood flow to the pulmonary tissues □ A. Gas embolism d/t N2 release from tissue □
162. A mountaineer ascents 18,000 feet in 2 days without B. Amniotic fluid embolism □
supplemental oxygen. At the height of ascent the C. Tumor embolism □
changes are: (PGI 03) D. Air embolism d/t CO2 release from tissue. □
A. Ted Pa C 0 2 □ 169. Decompression sickness is seen in: (PGI 02)
B. led Barometric pressure □ A. Diver □
C. led inspired O2 □ B. Pilot □
D. led PaC>2 D C. Diver and pilot □
E. Ted PH □ D. Diver, pilot and mountaineer □
163. Which of the following is seen in high attitude E. Diver, pilot, mountaineer and astronauts □
climbers: 170. Nitrogen narcosis is caused due to: (AIIMS 11,08)
A. Hyperventilation (PGI 01) □ A. Nitrogen inhibits dismutase enzyme □
B. Decreased PaCC>2 □ B. Increase production of nitrous oxide □
C. Pulmonary edema □ C. Increased solubility of nitrogen in nerve cell
D. Hypertension □ membrane □
E. Bradycardia □ D. Decrease in oxygen free radicals □
164. A person unacclimatised develops pulmonary edema-
A. 19-21 days (NBE P 13) O
B. 2nd-3rd month □
C. 2-3 days □
D. 6-7 davs □
74 ■ A Complete Review of Short Subjects

ANSWERS & EXPLANATIONS:

Transport of Gases

1. B i.e. 50ml/min

Amount of O 2 enters the body/min = 250 ml/min®. Amount of CO 2 excreted by lung/min = 200 ml/min® (288 L/day)®.So
difference in amount of O 2 inspired & CO 2 expired is 50ml/minQ. CO 2 is 20 times more soluble than O 2.

2. D i.e. Bicarbonate [Ref: Ganong 24/e P-644-46; Sembulingum 3/e P-577,582; Chaudhuri5/e P-139]
3. B i.e. Dissociation of CO 2 on oxygenation 4. A i.e. RBC high chloride


----------------------------------------------------------------------------------------------------------------------------------------------------------------------------- K
Transport of CO 2 in the form of bicarbonate ions accounts for approximately 70% of transported carbon dioxide from
the tissues to lungs Q. So CO 2 is transported as plasma HCO 3 ' > RBC>HCOs- >Carbamino compound > dissolved CO 2 Q

Transport of Carbon Dioxide


Carbon dioxide is transported in the blood as: HCO 3- (70%) > carbamino compounds (23%) > dissolved CO2 (7%)

Fate of CO 2 in Blood Haldane Effect/CDH Chloride Shift/ Hamburger


(Christian, Douglas, Haldane) Effect Phenomenon
In Plasma If the Hb is oxygenated, the CO 2 • When the negatively charged (HCO 3 -)
- Dissolved dissociation curve shifts to the right, that bicarbonate ions move out of red blood cell
- Carbamino is, the blood begins to lose some CO 2 as it into the plasma, to maintain the electrolytic
compound with becomes oxygenated (Deoxygenated Hb equilibrium the negatively charged chloride
plasma protein binds more H* than oxyhemoglobin®, ions move into the Red blood Cells from
- Hydration, H+ binding of O 2 to Hb reduces its affinity plasma (In plasma plenty of sodium chloride
buffered, HCO 3- in for C 0 2) is present).
plasma T is s u e
P C 0 2 = 46m m H g
• This process is mediated by band 3 membrane
protein
In Red Blood Cells • Chloride shift occurs in: 1 secondQ
- Dissolved • Hematocrit of venous blood is: greater (by
- Carbamino-Hb 3%)than that of arterial bloodQ
- Hydration, H+ • PH of venous blood (7.36) is lower than
buffered, 70% of that of arterial blood (7.40)Q
HCO 3- enters the
plasma
- CF shifts into P 0 2= 4 0 m m H g P 0 2= 4 6 m m H g
C O jC o n te n t2 4 0 m l% C 0 2c o n (e n t= 5 2 m l%
Cells, mosm in cell
increases Diffusion of carbon dioxide from tissue to
________________capillary________________

★ For each CO 2 molecule added to RBC there is increase in one osmotically active particle in cell either HCO 3- or CF.
So RBC take up water & swell. For this reason plus a fact that some arterial fluid returns via lymphatics rather than
the veins, the hematocrit of venous blood is 3% greater than arterial bloodQ.

5. D i.e. 300-400ml/min

- The normal diffusing capacity (D l) for O 2 is about 25ml/min/mmHg; and it increases to 2 to 3 times during exercise
because of recruitment and distension of pulmonary capillaries. To convert D lc o to D lo 2, the value obtained is
multiplied by 1.23.
- Both O 2 & CO 2 are highly soluble in lipids, therefore membrane barrier offers no hindrance to their diffusion; instead
the limitation is their diffusion through tissue water (plasma). CO 2 is 20 times more soluble in water than O 2
(Solubility of CO 2/O2 in water=0.06/0.003= 20).
- This explains the fact that although CO 2 has higher (but not very different) molecular weight than O 2, it diffuses
nearly 20 times more rapidly across the alveolar capillary barrier than O 2Q. So diffusion capacity of -400-500 ml/minQ.
Physiology: Respiratory System ■ 75

6. B i.e. Shift of hemoglobin dissociation curve to left [Ref: Ganong 24/e p. 641-47; Guyton 11/e p. 189, 508]

In systemic capillaries, O2 is released (i.e., O2 dissociation curve shifted to right) and CO2 is acquired <2 (CO 2 dissociation
curve shifted to left). In systemic capillaries, there is decrease in pffQ and increase in hem atocrit, protein content, H+, CV,
& HCO3 ion concentration0.

7. A i.e. Sigmoid curve of oxygen dissociation & B i.e. Positive co-operativity [Ref: Ganong 24/e 644; Sembulingum 3/581-82J

Hemoglobin (unlike myoglobin) shows sigmoid curve o f O 2dissociation and positive co-operative binding <2.

Unlike Myoglobin, Hemoglobin shows:


- Tetram eric structure
- Can bind four O2 m olecule0
- Sigmoid saturation kinetics 0
- Co-operativity or co-operative binding
kinetics: property that perm its it fo itin d a
maximal quantity of O j at respiratory organs &
to deliver a maxim al quantity of O 2 at
peripheral tissue. Here binding of Hb with O j
facilitates binding of other O ; molecules thus
positive co-operativity. Dissociation curve of hem oglobin and myoglobin

8. C i.e., 3.6 K Pa P50 is the PO 2 at which hemoglobin is half saturated with O 2.

P 50 is the PO 2 at which hemoglobin is h a lf (50%) saturated with O 2®. The value ofPso is 25 mm Hg(~ 3.6 KpaQ)

9. C i.e. 80 mm Hg [Ref: Ganong 22/e P-667]

PO 2 (mm Hg) Dissolved O 2 (m l/d l) % Saturation G iv e n c o n c e n tra tio n o f o x y g en is in m l/ m l


1° ~\ 0.03 13.5 an d n eed s to b e co n v erted in to m l/ d l.

20 0.06 35 0.0025 ml
30 0.09 57
0.0025 ml/ml =
1/100 dl
40 r* D iffe re n ce o f 0.12 V T a b le o f 3 75
(as 1 dl = 100 m l)
50 10 0.15 83.5
= 0 .0 0 2 5 x 100 m l/ d l = 0.25 m l/ d l.
60 0.18 89
70 0.21 92.7 F ro m th e tab le a v alu e o f d isso lv e d o xy g en o f
80 0.240 94.5 0 .2 5 m l/ d l c o rre sp o n d s to 9 4 .5 7 sa tu ra tio n of
90 0 .2 7 96.5 H b and 80 m m H g of P O 2.
100 J 0 .3 0 J 97.5

10. A i.e. CO [Ref: John West 8/e p 26-33; Michel Levitzky: P ulm onary physiology 6/e p 132; G anong 23/e p 613;

13. B i.e. 19.8 14. C i.e. 80 mm Hg 15. A i.e. 9 ml


16. B i.e. 6 ml 17. A i.e. 97%; 18. A i.e. 40 mmHg
IRef: G anong 24/e p. 641-43; Guyton 12/e p. 485-99; John West 9 /e p .7 8 - 8 4 , 151-52; Chaudhuri5/e P-135; Sem bulingu m 3/e P -5 7 7 1
76 ■ A Complete Review of Short Subjects

- Arterial blood contains 19.8 m l CVdl of which f t Feature Arterial Blood Venous Blood
19.5ml bound to Hb and 0.29 m l dissolved in Po 2 (Oxygen 95 (±5)mm Hg 40 (± 2 ) mm Hg
plasmaQ. Tension)
- 97 %Q of O 2 is transported in chemical combination O 2 content 19.8 ml/dlQ 15.22 ml/dl
with Hb and only 3% is transported in dissolved (ml/dL or (19.5 ml Hb bound (15.1 ml Hb bound and
state in water of plasma & cells. ml/lOOml) & 0.29 ml plasma 0.12 ml plasma
- Pco 2 in arterial blood is 40mm Hg. dissolved) dissolved)
- The total amount of plasma dissolved O 2 transported Hb saturation (%) 95 (±2)% 75%
in 1 0 0 ml (ldl) of plasma in an individual breathing Pco 2 (CO 2 tension) 40 mmHg 46 mmHg
100% O 2 at 1 atm (760 mm Hg) is 2.28 ml (=0.003 P02 CO 2 content 49.02 ml/dl (46.4 52.68 ml/dl
= 0.003 x 760) and at 4 atm is 9.12 ml (0.003 x 760 x Hb bound + 2.62 (49.7 Hb bound + 2.98
4). So the difference is of - 6 ml. plasma dissolved) plasma dissolved)
- Arterial blood pH is 7.4 ± 0.02 N2 content 0.98 0.98
(plasma dissolved) (plasma dissolved)

19. B i.e. Type of fluid administered [Ref: Ganong 22/e p. 666-69]

Oxygen delivery to tissues depends on

Lung Cardiovascular system Blood


I I I
- Amount of O 2 entering Blood flow to tissue i.e., Capacity of blood to carry O 2, which depends on
the lungs (ventilation) - Cardiac output® - Amount of dissolved O 2 in plasma
- Adequacy of pulmonary - Peripheral vascular resistance® - Amount o f hemoglobin®
gas exchange (diffusion) i.e., degree of constriction of - Affinity o f Hb fo r O2 ®
vascular bed in the tissue. Which is represented by 0 2 -H b dissociation curve
and depends on 2-3, BPG, pH and temperature®.

20. A i.e. 0.16 21. A i.e. 16 %[Ref: Sembulingum 3/e P- 574, 612]

Mouth - to mouth respiration provides 16% of O 2. f t O 2 Content (ml%) in


In mouth- to - mouth respiration the resuscitor
takes a deep breath & exhales into the subject's I I
mouth. It provides 16% (15.7%) o f O 2 (amount of O 2 Inspired air Alveolar air Expired air
that is present in expired air). So the fraction is I I I
0.16Q. 20.84% 13.60% 15.7%®

22. D i.e. 420. 4 mm Hg [Ref: Chaudhuri 5/e P-141 ]

Where:
PA0 2 can be calculated by alveolar gas equation . PAOz = PO 2 of the alveolar air
PaOz = FIO 2 x (PB - PHzO) - PaCO? . FIO 2 = Fraction of O 2 in the air
R . PB = Barometric pressure (N-760 mm Hg)
. PH20 = Water vapour pressure (N- 47 mm H2O)
The data provided in the question is: . PaCOz = Partial pressure of CO 2 (N - 40 mm Hg)
• PaC>2 = 100 mm Hg . R = Respiratory quotient
• PaCC>2 = 40 mm Hg ( CO 2 output 200 ml 'j
• FiC>2 = 8 mm Hg 0 2 uptake 250ml J
FiC>2 - Fraction of inspired Oxygen
At sea level (760 mm Hg pressure), the composition of air - O 2 = 20%; N 2 = 80%

20 Substituting these values


So Fi0 2 is = 0.2 Pa02 = 0.8 x (760-47) - 40/0.8
100
= 520.4 mm Hg
In this question, the child is being ventilated with 80% O 2
Therefore Pa 0 2 - Pa02 = (520.4 mm Hg-lOOmm Hg)
80 = 420.4 mm Hg.
So Fi0 2 is = 0.8
100
Physiology: Respiratory System ■ 77

23. B i.e. P/T=constant

Chari's Law Boyle's law Dalton's Partial Pressure Law


At constant volume (V) At constant temperature In a mixture of gas, each gas exerts a
Pressure (P )« Absolute P 1/V or pressure (k/a partial pressure) a/t its
temperature (T) own concentration (as if it is
PV= Constant
present alone)_____________________
P/T = Constant Avogadro's Hypothesis
Gas equation All gases at same temperature (T),
PV=nRT, pressure (P) & dryness contain same
where R is gas constant and n is number of molecules, e.g. at STD
gram molecules of gas (0°C , 760 mmHg, dry) a gram
molecule of any gas occupies 22.4
liters

24. D i.e. D [Ref: Guyton 11/ep 519-20]

• In graph drawn between Alveolar ventilation & Alveolar


PC 02
- At same pH, line tilts towards right
with increase in P 0 2 and viceversa
- W ith decrease in pH, the fam ily of
lines (curves) displace towards leftQ
and viceversa.
• So curve of P 0 2 100 will remain parallel but shift towards left
with decrease in pH. This means original curve A (at P 0 2 100
mm Hg) will become D at lower pH.

0 2 - Dissociation Curve

25. A i.e. Binding of one 0 2 molecule increase affinity for the next 0 2 molecule [Ref:Ganong 23j 609-11; Sembulingum 3/ 581-82]

Oxygen - hemoglobin dissociation curve is sigmoid (or S) shaped because binding o f one oxygen m olecule to heme
increases the affinity o f second heme molecule fo r oxygen 0 an so on.

26. A i.e. Bohr effect 27. A i.e. Facilitates oxygen transport

Bohr effect: The presence of C 0 2 (1 pH) decreases the affinity of Hb for O 2, this enhances further release of 0 2 to
the tissue and the 0 2 dissociation curve is shifted to right. This is known as Bohr effect. So decrease in 0 2 affinity o f
hemoglobin when pH o f blood fa lls is called Bohr effectQ. And it is caused by the fact that deoxygenated hemoglobin
binds H+ more actively than does oxyhemoglobin. All the factors, which shift the 0 2- dissociation curve to the right
enhances the Bohr effect.
- Binding o f 0 2 to hem oglobin reduces its affinity fo r CO20 or in other words C 0 2 disociation curve shifts to right (i.e.
blood begins to lose some C 0 2 as it becomes oxygenated) is called Haldane - Christian - Douglas effect.

28. B i.e. Increased 29. A i.e. Unloading oxygen to tissues 30. D i.e. Hypoxanthine
[Ref: Ganong 23rd/e p. pg- 609-11; Lippincott Biochemistry 4lh/e p. 31-32, 147, 294; Wintrobe's clinical hematology 12/ed pg-675]

31.
While inosine increase /maintain 2 , 3 - BPG content, hypoxanthine does notQ.

D i.e. p5o decreases & 0 2 affinity increases 32. A i.e. PH 33. A i.e. Decreased 2 ,3 DPG concentration
a
34. A i.e. Oxygen dissociation curve shifts to left 35. B i.e. Blood transfusion 36. B i.e. 2,3, DPG
37. A i.e. Hypthermia/HBF 38. D i.e. M atabolic alkalosis 39. A i.e. Higher concentration are found in adult blood
40. A i.e. Acidosis shifts 0 2 dissociation curve to right. 41. A i.e. Chloride ion Concentration
42. D i.e. Oxygen affinity of hemoglobin 43. B i.e. Hypoxia 44. D i.e. tP a C 0 2 & T H C O 3-
45. B i.e. Fall in Temperature 46. A i.e. Fetal Hb & Hypothermia shifts curve to Left
[Ref: Best & Taylor 13/e p 604-5; West respiratory physiology 9/e p 79-82; Egan's p 234; Ganong 24/e p 643-48; Berne & Levy 6/e
p461-64; Nelson 17je p 1603-2; Chattarjea 7/e p 149; Kumar & Clark 6/e p 430]
78 ■ A Complete Review of Short Subjects

- Alkalosis (increased pH), hypothermia (decreased L eft S h ift (In creased Q a ffin ity & D ecreased fo ) |
Acule A lkalo sis ( f>H)
temperature), hypocarbia (decreased p CO 2, and D ecreased pC O j (H ypocarbia)
D ecreased lem perature(H ypolhurm ia)
decreased 2 ,3 , DPG cause left shift of O 2 dissociation D ecreased 2, 3 D PG (organic phosphate) level

curve (ie decreased P50 - increased O 2 affinity = M e (hem oglob in


Fetal H b (H bF)
decreased tissue delivery of O 2) Stored b lood
H b R an jer St H b Q tesap e ak e
- Hypocarbia due to hyperventilation shifts 0 2
dissociation curve towards left i.e., P50 decreases & Normal
oxygen affinity increases^.
- O 2 saturation of maternal blood in placenta is so low that IRight Shift (Decreased Oa affinity &Increased PsoTl
the fetus might suffer hypoxia damage if fetal RBCs did Acute A cidosis N^pH) eg d iab etic ketoacidosis
- Increased C O 2 (H ypercarbia)
not have a greater O 2 affinity than adult RBCs. The - Increased tem perature (H ypertherm ia)
cause of this greater O2 affinity is p oor binding o f 2 , 3 - Increased 2 ,3 D PG (H igh levels)
- H orm ones (GH, A ndrogens, Thyroid hormones)
DPG by ypolypeptide chains that replace fl chains in • H ighlaltitude, H ypoxia, Exercise
fe ta l Hb®. Because HbF binds less with 2 ,3 DPG & so is - A nem ia (H b S= sickle hem oglobin), HbM

able to bind O2 more accounting for left shifted O 2 PO2


dissociation curve at birth. Adult Hb (HbAi) has more 2,
3 DPG as compared to HbF (fetal hemoglobin) and it is
better at delivering O2 to tissue. H bS (sickle cell anemia)
shift the O 2 dissociation curve to right.
- O 2 affinity of HbM < HbA < Hb Chesapeake & Rainier. Because Mb- O 2 disociation curve lies left of Hb 0 2 dissociation
curve, it takes up O 2 from Hb in blood.
- Acidosis (decreased pH), hyperthermia (increased temperature), hyper carbia (increased CO 2) & high levels of 2 ,3 ,
biphosphoglycerate (2,3 BPG = organic phosphate) d/t exercise, high altitude or anemia (sickle cell Hb) cause right
shift of O 2 dissociation curve (ie increased P50 = decreased O 2 affinity = increased tissue delivery of O 2)

Factors Shifting O 2 Dissociation Curve

Towards Left ie Towards Right ie 2 ,3 DPG


Increasing O 2 Decreasing O 2 affinity to Is found in red blood cells. It is responsible for lowering the
affinity to Hb and hemoglobin and oxygen affinity o f hemoglobin®, thus allowing Hb to release
decreasing release = promoting release of O 2 in oxygen (unloading)Q more efficiently. It is
Decreased p50 tissue = Increased p50
- Acute Alkalosis - Acute Acidosis (|pH) eg High 2 ,3 DPG Low 2 ,3 DPG in
(TpH) diabetic ketoacidosis High altitude, Hypoxia, Hormones Fetal hemoglobin
- Decreased pCC>2 - Increased CO 2 (GH, Androgen, TH) (HbF)®
(Hypocarbia) (Hypercarbia) Stored blood®
Anemia (HbS) - Acidosis (ipH )
- Decreased - Increased temperature
Pregnancy - Exercise A lkalosis ( TpH)®
temperature (Hyperthermia)
- Increased 2 ,3 DPG (High O 2 delivery to tissue depends on
(Hypothermia)
- Decreased 2 ,3 DPG levels)
- Hormones (GH, Amount Adequacy of Blood Capacity of blood carry
(organic phosphate)
Androgens, Thyroid of O 2 pulmonary gas flow 02
level
- M ethemoglobin hormones) entering exchange to tissue
- High altitude, Hypoxia, lung depends on
- Fetal Hb (HbF)
- Stored blood Exercise I
- Anemia (HbS=sickle 1----------
- Hb Ranier & Hb Amount of Hb in blood Affinity of Hb for O 2
Chesapeake hemoglobin), HbM
dissolved O 2 I
★ Exercise: Oxygen dissociation Curve shifts to the Factors which cause left
right in exercising muscles. Stroke volume is shift of O 2 dissociation
increased during isotonic muscle exercise. Blood flow curve decrease O 2
to muscles is increased. delivery to tissue
Physiology: Respiratory System ■ 79

Lung Volume and Capacities

47. A i.e. Inspiratory Capacity minus Inspiratory Reserve volume 48. B i.e. TV
49. C i.e. Volume remaining after normal expiration 50. C i.e. ERV + RV
51. C i.e. Compliance of lung 52. D i.e. FEV] is forced expiratory rate at one minute
[Ref: Ganong 24/e P-629-31; Sembulingum 3/e P-561-62; Chaudhuri5/e P-154-155]

- Volume o f air taken in & given out during normal respiration is called as tidal volume (TV)®. TV = IC - IRVQ.
- Functional residual capacity (FRC) is volume remaining after normal (tidal) expiration®. FRC = ERV + RVQ.
- Total lung capacity (TLC) depends on lung compliance (distensibility)®. Compliance decreases in interstitial lung
diseases.
- FEVi or timed vital capacity is fraction o f VC expired (or forced expiratory volume) in 1 second®.

53. C i.e. Max mid respiratory flow rates [Ref: Harrison 16lh/1498, 1500, Ganong 22/e P-651, Sembulingum 3/e P- 567, 570]

Resistance to sm all air ways is best measured by maximal mid expiratory flo w rates (MMFR) fo llo w ed by FEVi.®
In early obstructive disease, which originates in the small airways FEV1/FVC may be normal but PEP 25-75% - (Max mid
expiratory flow rate) may be depressed.

54. A i.e. 6 L 55. C i.e. 4.2 56. A i.e. 4900 ml [Ref: Ganong 24/e P-629-30; Sembulingum 3/e P- 567,570]

Alveolar ventilation (rest) 4.2L/min.® Alveolar ventilation = (TV - dead space volume) x RR
Total lung capacity 4 - 6 lit.® = (500 - 150 ml) x 14
Respiratory minute volume (rest) 6 L/min.® = 4900 ml
Maximum voluntary ventilation 125-170 L/min.®
★ Normally we consider dead space to be 150 ml. But
Timed vital capacity - In 1 sec (FeVi) 83%®
also know that 'Dead space volume o f a person is equal
- In 3 sec 97%®
to twice his body weight i.e. a person weighing 70 kg
Work of quiet breathing 0.5 kg - m/min®
has a dead space volume of 140 ml.
Maximum work of breathing 10 kg- ml breath

57. B i.e. 0.35 [Ref: Best & Taylors 12/e P-527]

V j = Dead space volume & V, = Tidal Volume, so Vd/Vt = 150 ■0.3


100
Normal value of Vd/Vt in adults = 0.2 - 0.35®

58. D i.e. Close to Residual volume 59. B i.e. FRC below the closing volume
IR ef: G a n on g 2 4 /e p 6 3 3 -3 4 : Joh n W est 9 /e p 1 6 9 -7 0 ; N elson 1 8/e p 732 -3 5 ; C lo h erty 6 /e p 3 3 6 ; R og ers T P IC 3 /e p 2 8 7 -8 8 ; B e r n e & L ev y 6 /e p 431-4341/

f t
- Closing volume (CV) or critical CV is minimal volume of gas necessary to prevent small airway collapse. In young adults it
is 10% of vital capacity (ie = 480 ML) and becomes equal to ~ 40% of VC (ie = FRC) at the age of 65 years. In R D S
(hyaline membrane disease), the FRC decreases below closing volume (FRC < CV) indicating collapse of some
alveolar segments during tidal breathing.
- Volume of air in lung at the end of (1) force ful inspiration is TLC; (2) forceful expiration is residual volume (RV) and ;
(3) normal tidal expiration is FRC. RV is 1200mL and FRC is 2200 mL. Therefore normally closing volume is more close
to RV than to FRC.

60. B i.e. Reduced FEV1/FVC ratio 61. B i.e.? FEVi/FVC, ^Compliance of lung tissue
62. B i.e. RV decreased 63. A i.e.? TLC [Ref: Berne & Levy 6/e p. 443; Harrison 16/e P-1500] ,

The hallmark of restrictive lung disease (eg interstitial lung disease) is decrease in lung volumes primarily TLC, RV
and VC, decreased lung compliance (distensibility), reduced diffusion capacity but increased FEVi/FVC ratioQ.
The hallmark of obstructive lung disease (eg emphysema) are increase in lung volumes RV & TLC® (but not VC which
is decreased)®, decrease in expiratory flow rates (FEF25-75) & decrease in F E V i/ F V C Q .
80 ■ A Complete Review of Short Subjects

64. C i.e. Slow and deep [Ref: Harrison 16/e P-1549]

As the lung deflates, the intrapleural pressure rises from negative to zero —> this creates a situation when the bronchioles
are no more stretched —>bronchodilation ceases —»bronchial narrowing —>increased air flow resistance —> drop of rate of
airflow.
Slow and deep breathing is betterQ than rapid and shallow breathing:__________
- Because of the dead space, rapid shallow breathing produces much less alveolar
ventilation than slow deep breathing at the same respiratory minute volume.
- Slow and deep breathing encounter less airway resistance.____________________

Tests for Pulmonary Compliance


65. C i.e. Increase in lung decrease in box [RefiBerne 6/ 431-32;Sembulingam 5/ 665-66; Ganong 23j 539: John West 8/14-16,165-66J

Because of Boyle's law (PiVi = P2V 2), during expiration when lung volume decreases, the pressure inside the lung
increases and pressure inside the air tight box decreases because the gas volume in box increases - in body
plethysmography in which the subject sits in an air tight chamber.

66. A i.e. Inspiratory reserve volme, Tidal volume and Expiratory reserve volume
67. A, C, E i.e. Tidal volume, ERV, IRV 6 8 . B i.e. Residual Volume 69. A i.e. FRC C i.e. RV
70.

71. C i.e. N 2 breath test 72. B ie. Anatomical dead space=Physiological dead space 73. D ie. Single breath nitrogen test

Measurement of Dead Space Volume

Anatomical Dead Space Physiological Dead Space (Volume)

Single Breath N 2 Method (Fowler's Method) Bohr's Method (Equation)


- After quiet expiration, subject takes a single deep : TV (FaCCVFeCO;)
breath of pure (1 0 0 %) O 2 and breathes it out FaC 0 2
slowly & steadily. N2 analysis in expired air is
done : Ty (Pa C 0 2 - PeCQ 2
- Pure O 2 fills dead space with 100% O 2 and it comes PaCOj
out first in expiration so initial volume of air
TV= tidal volume, Fa C O 2 & Fe CC>2= C O 2 Concentration in
(during expiration) that is N 2 free gives
alveolar air (A) and expired air (E). P a C 0 2 & PeCC>2= Partial
anatomical dead spaceQ.
- In healthy individuals, normally Anatomical pressure of CO 2 in artery (a) & expired air (e)
dead space= Physiological dead space=150mlQ ★ Since concentration of gas is a partial pressure (FA/ e=
P a /e ) and alveolar Pc02=arterial PCO 2 (So PaC 0 2 = PaCCb).
Physiology: Respiratory System ■ 81

74. 8 i.e. Extrathoracic obstruction [Ref: Morgan 3/e P-542]

Pressure - Volume Curve

O
Vital Capacity Vital Capacity
Normal Extrathoracic obstruction Varialble intrathoracic obstruction Fixed large airway obstruction

75. C i.e. Interstitial lung disease [Ref: Ganong 24/e p. 631

Pressure Volume Curve

Compliance Decreased i.e. Curve Shifted Compliance Increased i.e Curve Shifted to
to Downwards & Right Upwards & Left
Pulmonary interstitial fibrosisQ Emphysema^
Interstitial lung disease^ Normal aging lung
Alveolar edema (prevents inflation of some alveoli) During an asthma attackQ
Pulmonary congestion (increased pulmonary
venous pressure)®
If lung remains unventilated for long period
especially if lung volume is low
D ecreased surfactant ® (causing increased surface
tension and alveolar atelectasis)
Deformities of thorax eg kyphosis, scoliosis
Paralysis of respiratory muscles
Pleural effusion pneumo/hydro/hemo-thorax TrarmKmflpressure(cmHjO)

Pressure During Breathing Cycle


76. D i.e. Alveolar pressure-Transpulmonary pressure
77. A i.e. More - ve 78. B i.e. Thoracic cage and lungs are elastic structures.
79. A i.e. The chest wall and lungs recoil in opposite directions 80. C i.e. Lymphatic drainage of pleural cavity
[Ref: Guyton 11/e p. 489-90; Ganong 23/e p. 599; John West 8/e p 114-15; 110-11, Berne & Levy 6/e p-433-435; Chaudhari 5/e p 121]
The pressure difference between the air spaces (alveolar pressure = Pa) and pressure surrounding the lung (i.e. pleural
pressure = Ppi) is called transmural pressure across lung or transpulmonary /trans lung pressure ( P l ) P l = P a - Ppl The
lung requires positive transpulmonary pressure (i.e. relatively more alveolar or lesser pleural pressure) to increase its
volume and the lung volume increases with increasing Pl.

So, Pleural (Intra Pleural) Pressure (Ppl) = Alveolar Pressure (Pa) - Trans Pulmonary/ Lung /Mural Pressure (i.e. Pl)

Normal negative intrapleural pressure is maintained by tf


opposing elastic recoil forces o f chest w all and lung® (ie the
Expansile Elastic Recoil
expansile recoil tendency o f chest wall and retractile recoil
tendency of Chest Wall
tendency of lung) along with efficient drainage of excessive
intrapleural fluid by lymphatic pump and osmotic forces
across pleural membrane maintaining a slight suction and
only a thin layer of serous fluid between parietal pleural surface
Thin layer of Intapleural
of thoracic cavity and visceral pleural surface of lung. fluid (maintaining slight
The basic cause of negative pressure in most tissue spaces of suction) d/t effeclent
drainage caused by
body (including intrapleural cavity) is lym phatic drainage (i.e.
1. Lymphatic pump
pumping o f fluid from space by lymphatics)®. 2. Osmotic forces
Intra pleural pressure is negative during the normal breathing
cycle (i.e. both during inspiration and expiration) because o f
elasticity o f lung and chest wall®.
Negative Intrapleural Pressure
82 ■ A Complete Review of Short Subjects

81. A, B, C i.e. Forced expiration, Dense air, Low lung volume


82. A i.e. Increased compression of airways 83. C i.e. Dynamic compression of airways
IRef: John West 8/e p 114-17; 96-97; Berne & Levy 6/e p 439-41; Harrison 18"' /2091; Chaudhary 6/e p 145]

Increased Airway resistance (Raw) is caused by low (decreased) lung volume such as during forced expiration^
because as the lungs compress, the airways also compress. Airway resistance is also increased if inspired air is more
dense or viscous and flow is turbulent^
Decreased airway resistance is caused by high (increased) lung volume eg during inspiration because expanding
lungs exert a traction on airways (trachea bronchial system) causing them to dilate (thereby decreasing resistance).
Airway resistance is also decreased if inspired air is less dense or viscous and flow is laminarQ.
Expiatory flow is effort independent and flow limited. Airway resistance is greater during exhalation than during
inspiration because of dynamic compression of airwaysQ which stops last stage of forced expiration.
Expiration in quiet breathing is passive and requires no muscle activity. Lung recoil pulls the chest back and the airway
pressure becomes slightly positive.

Expiration

84. B i.e. At the end of normal expiration air in lung is ERV [Ref: Ganong 23/e 591-94; Berne & Levy 6/e p-425-35; Guyton 11 /e
471-74; John West8/e p 114, 96-97; Gabriel p 178-184]

‘-----------------------------------------------------------------------------------------------------------------------------------------------------------------------------------
- Muscles that elevate the chest cage (i.e. pull the ribs upwards and farwards, thus increasing both AP and lateral f r
diameters = external intercostals) are muscles of inspiration whereas muscles that depress the chest cage (i.e. internal
intercostals which pull ribs downward and inwards, decreasing the thoracic volume) are muscles of expiration.
- Elastic chest wall has a tendency to move outward which is balanced by inward recoil of alveoli. At FRC both are
balanced
- Pleural pressure is alw ays lesser than alveolar pressure Q, because it is a suction (slightly negative) pressure holding the
lungs open
- Amount of air remaining in lungs after a normal tidal expiration is functional residual capacity (FRC), which is equal
to the sum of residual volume (RV) and expiratory reserve volume (ERV). ERV is the amount of air that can be enhaled
with maximal effort in excess of tidal expiration. RV is amount of air remaining in lungs after maximum expiration;
which keeps alveoli inflated between breathes and mixes with fresh air on net inspiration FRC = RV + ERV

Workload of Breathing

85. A i.e. Lung elasticity 86 . D i.e., Decrease workload and decrease duration of exercise
[Ref: Guyton 11/e p. 539- 41; John West 8/e p. 98-100; Ganong 22/e p.657]

Efforts during respiration (inspiration) is done due to lung elasticity i.e. to expand elastic lung.
At high altitudes, work capacity is increased by acclim atization, increased rate o f Oi upake the body can achieve,
decreased respiratory w ork load and decreased duration o f exercise Q.

87. C i.e. Pressure of gas

Reynolds number (Re) _ 2 x radius x velocity x density ^ 2 rvd


Viscosity q
Turbulent flow (Re>2000) is seen in high flow rates & large airways and laminar flow is seen in small airways & low
velocity.

Compliance

88 . B i.e. Chronic bronchitis (Answer of exclusion) 89. A i.e. Elasticity 90. A i.e. 200ml/cm water
[Ref: Berne & Levy 6/e p 431-32; Gabriel p 182-83; John West 8/e p 99, 117; Guyton 11/e p 473-75; Ganong 23/e p 595-971
Physiology: Respiratory System ■ 83

- Unit of compliance (i.e. stretchability or elasticity) is mL (or liter) per cm H2O. The compliance of normal human (both)
lung is 0.2L/cm H2 O (200 tnL/Cm H2 O,)® which means every time the transpulmonary pressure increases 1 cm H2O, the
lung volume will expand 200 ml (0.2 L) after 10-20 seconds.
- Specific lung compliance is decreased in pulmonary fibrosis, pulmonary congestion and decreased surfactant®.

Ventilation & Perfusion

91. B i.e. Ventilation - Perfusion mismatch [Ref: Harrison 16/e pg 1503,1504; Ganong 22/e pg 662,663]

- f t
In ventilation perfusion mismatch the O2 contained in the alveoli, cannot be transmitted to arteries. This leads to
accumulation o f O2 in the alveoli®, causing increased O2 tension.

92. C i.e Apex of lung 93. A i.e. Apex of lung 94. B i.e. Base 95. A i.e. PaC>2 is maximum at apex
[Ref: Ganong 22/e P- 663,662, Sembulingum 3/e P- 572]

&
Ventilation = Minimum
Ventilation Perfusion Ratio is maximum (3) at the Apex Perfusion = M inimum
V/Q ratio = Maximum0
of lungQ and minimum (0 .6 ) at base. Pa0 2 is maximum at
apexQ. Average V/P ratio for whole lung is 0.8Q.
Ventilation = Maximum
Perfusion = Maximum
V/Q ratio = Minimum0
Lung

Ventilation/Perfusion Ratio

The ratio of pulmonary ventilation to pulmonary blood flow for the whole lung at rest is - 0.8 (4.2 L/min ventilation
divided by 5.5 L/min pulmonary blood flow)Q

If the ventilation of an alveolus is reduced If perfusion is reduced relative to ventilation:


- PO2 falls because of less O2 delivery - PO2 rises in alveolus because less O2 enters the blood
- PCO2 rises because less CO2 is expired from alveolus, - PCO2 falls because less CO2 is delivered.

Ventilation as well as perfusion, in an upright position declines from the bases to the apices of the lung (in linear
fashion). But the change in blood from apex to base is greater than the relative change in ventilation, therefore V/Q
ratio is low at the base and high at the apex®.
In an upright position
- Ventilation per unit lung volume is greatest at base
- Perfusion per unit lung volume is greatest at base
PaC>2 is maximum at apex®.
High V- P ratio at the apex accounts for predilection of the tuberculosis for this area.
In patient with unilateral lung disease the patient is positioned such that the good lung is in dependent position.
Arterial oxygenation is improved in this position due to effect of gravity on ventilation.
(The situation in infants is opposite, i.e. infants do better with the diseased lung in the dependent position)

96. C i.e. P G I; I Ref: G anong 22/e P -663-664; K D T 5/e 158-160]

The unique feature of pulmonary vasculature is that the hypoxia tends to produce Vasconstriction (where as in other
system it produces Vasodilation). PGI2 is uniformly vasodilator®.
84 ■ A Complete Review of Short Subjects

Substance Causing

Vasodilation (Nitric oxide Vasoconstriction PGE 2 & PGF 2 a Causes v asod ilation
________ mediated)________ in m o st but n ot all
- 0 2 - adrenergic stimulation Mn: 2-a, fl, I - a l Adrenergic Mn: v ascu lar beds. PGF 2
- p2- adrenergic stimulation 2-Hi, Hy - ThromboxaneQ Oi acau ses
- Prostacyclin (P G k )0 Na, Va (TX-Az) a 2, a ii v asocon striction in
- Histamine - Angiotensin II cd 4 m any larger veins.®
- Endothelium derived . LTC 4,L T D 4 E-A PGI 2 Is Uniformly
hyperpolarizing factor - Endothelin io2tco2 Vasodilator.
- Atrial natriuretic peptide [ETA]Q PG Are inherently
- Vasopressin - Hypoxia® endoperoxidases Vasoconstrictor, but
- Hypercapnia® (G 2 and H2) often produce
vasodilatation or
biphasic response.
t x a 2& Consistently
Leucotrienes produces
(LTC 4 & LTD4) vasoconstriction

Hypoxia

97. C i.e. CO poisoning 98. B i.e. PO 2 normal 0 2 , saturation less with normal or slightly decreases PCO 2
99. B i.e. Anemic hypoxia 100. D i.e. i 0 2 content in arterial blood 101. A i.e. Arterial P 0 2
102. C i.e. Cyanide poisoning 103. C i.e. a,b & d 104. B i.e. D iffuse vasodilation
105. A i.e. 8 min. 106. B i.e. Hippocampus 107. C i.e. Hb
[Ref: Berne & Levy 6/e p 454-65; John West 8/e p 57, 90,134; Gabriel p 512-17; 69,194; Ganong 23/e p 616-23; Guyton 11/e p 530-
31; Harrison 17/e p 1591; Lee 12/e p 292.

- Arterial hypoxemia is defined as an arterial P02 (partial pressure of oxygen) less than 80 mmHg, who is breathing
room air at sea level. And because the partial pressure of a gas is determined by the soluble fraction of gas and not by
the amount carried chemically bound to hemoglobin - the Hb does not determine P 0 2 and hypoxemia does not depend
on Hb level®.
- In CO poisoning effective hemoglobin that can carry O2 decreases Q (because CO occupies Hb binding site) since P 0 2 or
arterial blood O2 remains normal Q, peripheral chemoreceptors are not stimulated and hyperventilation is not seen.
- In hypoxia, with the reduction of PO 2 cerebral blood flow increases (d/t vaso dilatation P to maintain O 2 delivery in the
brain.
- The cerebral cortex can tolerate acute hypoxia for 5-10 min at 28°CQ, 20 min at 20°C, and 50 min at 50°C.
- Hypoxic ischemia almost invariably involve hippocampus. The hippocampal CA1 neurons are vulnerable to even brief
episodes of hypoxic ischemia.

Cyanosis

108. A i.e. CO poisoning & Anemia 109. C i.e. Critical concentration of Hb required to produce cyanosis is reduced
110. B i.e. 0 2 saturation < 85% 111. A, D i.e. 10.9 & 4.1; 8.9 & 4.1
112. B i.e. Late sign of Hypoxia [Ref: Ganong 23/e 612; Harrison 17/e p 230-31; Guyton 11/531; John West 8/ 78; Wintrobe 6/e p.
S-------------------------------------------------------------------------------------------------------------------------------------------------------------------
- The critical concentration of reduced hemoglobin required to produce cynosis is 4 (Harrison) / 5 (wintrobe) gram/ dl. In
severe anemia, the total concentration o f hemoglobin may be < 4-5gm/ dl and so cynosis w ill not be manifested because
the critical concentration can not be reached®.
- Cyanosis is a late sign o f hypoxia®; so it is never advisible to wait for cyanosis to appear to diagnose hypoxia.
- Cyanosis occurs when concentration of reduced hemoglobin is > 4-5 gm % in capillary blood. Whereas when
concentration of reduced Hb is less than 4/5 gm % (like 4.1) cyanosis would not occur.
Physiology: Respiratory System ■ 85

113. C i.e. 1.5 g/dl 114. D i.e. Phenytoin 115. A i.e. Methylene blue [Ref: Sembulingam 4/e p. 72 - 73]

Concentration of methemoglobin to cause cyanosis is 1.5 gm/dlG. " it


Causes of methemoglobinemia includes nitrites, sulfonamides and phenacetin not phenytoin®. Antidote o fN a -
nitrate poisoning is methylene blue IV®.

116. B i.e. Cessation of respiration

117.
Apnea is cessation or stoppage o f respiration®

D i.e. Hypoventilation IRef: Chaudhuri5/e P-143]


$
Arterial O 2 tension may be reduced either from a defect in oxygenation (Type I failure) or Ventilation (type II
failure)=hypoventilation.

CO, 0 2 and C 02 Toxicity

118. C i.e. Tissue toxicity plays an important role .. 119. A & C i.e. Cytochrome toxicity is lethal & PO 2 is decreased
120. A i.e. High affinity of CO for haemoglobin 121. A i.e. Anemic hypoxia
Ref: Gnnong 23rdje p. 621, 564, 525; Guyton 12/e p. 501; 11 th/e p. 509-12,196, 492, 499

- Transport and toxicity o f carbon monoxide (CO) is lim ited to its diffusing capacity because hemoglobin combines with

this gas so rapidly® that its partial pressure never has time to build up and is essentially zero in blood. The affinity of
Hb for CO is 210 greater than its affinity for O 2. So the p artial pressure as low as 0.6mmHg o f CO may be lethal®. CO is
toxic to tissue cytochrom es a t level that is 1000 times o f lethal dose. Therefore tissue toxicity plays no role in clinical
CO poisoning®. It is treated by hyperbaric pure oxygen and 5% CO 2.
- Carbon monoxide (CO) poisoning shifts oxyhemoglobin dissocation curve tow ards left® (i.e. reduces amount of O 2
released in tissue). It reduces the amount of Hb that can carry O 2 and causes anemic hypoxia.

122. D i.e. Is toxic to tissues 123. B i.e. Decreased cerebral flow


[R ef: Sembulingum 3/e p. 595; Guyton 11/e p. 547; Chaudhari 5/e p. 148, 228; Goodman Gillman 11/e p. 393- 94]

Decreased cerebral blood flow due to cerebral vasoconstriction caused by high O 2 tension is its protective effect® (not
toxic). Which protects brain from free radicle injury.

Smoking

124. D i.e. Decreased carboxyhemoglobin [Ref: Harrison's 17/ed pg-2738; The risk o f passive smoking: Roy / Shepherd p. 71;
www.springerUnk.com/content / a 74832479802343 K/]

Smoking decreases HDL and increases RBC, hemotocrit, adrenaline, heart rate, catecholamines and
carboxy hemogolobinQ.

Respiratory Failure

125. C i.e Hypoventilation 126. D i.e Lobar Pneumonia.


127. A i.e. Carbon monoxide poisoning/ High altitude [Ref: Ganong 23/e p. 618-21; Harrison 17/e p. 1676; Guyton 11/e p. 531-32;
API-Mcdicinc 6/e p. 278; John West 8/e p. 85-87; Berne & Levy 6/e p. 465-66; 645-47, 447, 441; Gabriel p. 201-202]
86 "A Complete Review of Short Subjects

--------------------------------------------------------------------------------------- ;----------------------------------------------------------------------------------- s V
- Type II respiratory failure, which occurs d / 1 hypoventilation (i.e. failure of ventilation) is characterized by
hypoxemia with hypercapnia (CO 2) retention but normal (not increased) alveolar-arterial oxygen gradient (Pas02 )
- Hypercapnia (hypercarbia or CO 2 retention) is defined as an elevation in arterial partial pressure of carbon dioxide
(Paw). It characteristically occurs secondary to inadequate alveolar ventilation (hypoventilation), in type II respiratory
failure.
- Type II respiratory failure (or alveolar hypoventilation = alveolar ventilatory failure = hypercapnia = hypercarbia =
CO 2 retention = respiratory acidosis) or normal PA-a02 is seen in reduced compliance of chest like kyphoscoliosisQ,
reduced lung compliance like pulmonary (alveolar) edema®, obstructive lung disease like COPD®, weakness of
respiratory muscle (like bulbar poliomyelitis®), bronchospasm and decreased central respiratory drive. Drowning may
cause bronchospasm & pulmonary edema & so CO 2 retention.
- Type I respiratory failure results from failure in exchange (diffusion) of respiratory gases at the alveolar-capillary
junction as a result of disease o f lung parenchyma (like pneumonia), or vasculature (like right to left shunt), and
ventilation-perfusion mismatch®. It is characterized by increased alveolar-arterial oxygen gradient (P a -u o i) ®,
hypoxemia and normal or low Paco 2-
- At high altitudes alveolar CO 2 decreases b/o hyperventilation.

128. D i.e. Hypertension [Ref: Lee's 12/e P-42; Sembulingum 3/e P-5951
/"
Hypercarbia (hypercapnia) is characterized by hypertension, tachycardia, and mydriasis d/t sym pathetic stimulation®. If
also increase intracranial tension.

Regulation of Breathing

129. D i.e. Pre Botzinger complex 130. A i.e. Prolonged inspiratory spasm 131. A i.e. Pneumotaxic centre
[Ref:Ganong 23/e 626-27; Guyton l l / e p 514-16; John West 8/e p-124-26; Berne 6/e p 469-72; Gabriel p 197-99; MTx]

Spontaneous automatic rhythmic respiration is initiated by a small group of synaptically coupled pacemaker cells in the
pre-Biotzinger complex (Pre-BOTC) situated on either side of medulla between nucleus ambiguous and lateral reticular
nucleus.
--------------------------------------------------------------------------------------------------------
Pre Botzinger complex Pacemaker of respiration^ i.e. spontaneous automatic
Medullary (Pre BOTC) rhythmic respiration is initiated^
Respiratory Dorsal Respiratory Mainly cause rhythmic inspiratory discharge^
Centers (MRC) Group (DRG)
Ventral Respiratory Mainly controls forceful expiration^ & some inspiration
group (VRG)
Pontine Pneumotaxic center Halt (limit) inspiratory depthQ by fine tuning
Respiratory (PC) respiratory rhythm and switching between inspiration &
Centers (PRC) expiration
Apneustic center (AC) Excites inspiration center (DRG) to prolong inspiration

132. B i.e. Apneusis 133. B i.e. Irregular and gasping


[Ref: Samson Wright 13/168; Ganong 23/ 626-27; Sembulingum 3/e P-585-587; NMS physiology 4/e P-253-57; Guyton 9/e P 525-26/

■ % T i------------------------
The main component of respiratory control pattern generator responsible M u lb rain '
[ Net cfleet on
for automatic respiration are located in the medulla. After transection Pneum otaxic respiration

of brain stem at inferior border of pons spontaneous respiration centre


(u p p e r puriM /-
N orm al rhythm , slight
slow in g, increase in
continues, albeit som ew hat tidal \o lu m e
A pneustic centre I f vagus also cut.
irregular and gasping.® Whereas, transection at mid (upper) pons (lo w e r p o n s )
I ■ inspiration is sustained
V (1 e. apneusis)
leads to slow and deep breathing® (vagi intact) or apneustic breathing (ia s p in g rcspitaiion
\
(or prolonged respiratory spasm)®, when vagi are also cut. In sp ira io ry and
expiratory \~
neurons
in m edulla \
Effects o f transection o f the b rainstem at different
\
levels on resptrotion Note the a Ldditionol effect
o f cutting the vagus (the H e n n g-B reu e r mflotion
reflex).
Spin al cord
Physiology: Respiratory System » 87

Effect of Lesion/ Transection

Above (rostral) pons In Pons & medulla where autonomic centers are situated Below
medulla
- Loss of voluntary control
Respiration
- Respiration remains normal At upper or mid-pons At inferior border of
stops______
•Pneumotaxis center damage: respiration pons & upper border
becom es s lo w e r & tid a l v o lu m e g r e a te r (s lo w of medulla
& deep)® bu t n o ch an g e in r a te o r rhythm . - Spontaneous
If pneumotaxis center is damaged along with respiration
vagus - this causes removal of inhibitory continues, although
influence on Apneustic center and leads to som e what-
p r o lo n g e d re sp ir a to ry s p a s m that resem bles irreg u la r an d
breath holding k/a A p n eu sis o r A p n eu stic g a sp in g ® (Ganong)
brea th in g ® i.e. breathing stops in deep - Rhythmic respiration
inspiration which m ay be interrupted by persis but the
occasional expiration. pattern is less
If only vegi are cut d ep th o f in sp ira to n is sm ooth ®
in creased®

134. C i.e. Ventral surface of medulla 135. B i.e. Inhibited during swallowing 136. A i.e. TPCO 2
137. A i.e. T h + 138. A i.e. P 0 2 139. D i.e. H+
140. B i.e. Hypocapnia 141. A i.e. Apnea occurs due to hypostimulation of peripheral chemo receptor
[Ref: John West 8/e p 126-31; Ganong 23/e p. 627-29; Guyton 11/e 517-18; Berne b Levy 6/e p 471-72; Gabriel p-2001

- Respiratory center is inhibited during swallowing to prevent food aspiration 0
- Reduction in partial pressure of oxygen is most potent stimulant for the peripheral chemoreceptor and send impulses to
respiratory centers. If the O 2 dissolved in blood is high (as in hyperbaric O 2) the peripheral receptors will not be
stimulated and lack of respiratory drive leads to apnea.Q
- Central chemoreceptors (in ventral surface of medulla )0 are sensitive to dissolved C 0 2 (Pco 2>in blood (but not to
P 02, H+ concentration or pH of blood) and respond to pH (i.e. H+ concentration) of CSF (i.e. ECF or interstitial
fluid)°. Therefore primary indirect stimulus for excitation of central chemoreceptor is increase in Pco 2 in blood and
primary direct (and more important) stimulus is increase in H+ ion concentration (or decrease in pH) of CSF°.
- Peripheral chemoreceptors (carotid & aortic bodies) respond to decrease in P 02 (most important stimulus) and pH (T
H+ ion concentration in blood )0 and increase in arterial Pco 2°- Hypercapnia (TCO 2) does not stimulate either
peripheral or central chemoreceptors (only hypercapnia stimulate central receptors)0.
- If the peripheral chemo receptor is denervated the response to - (1) Arterial hypoxemia (P 02) and pH is completely
abolished0. (2 ) Hyper capnia (arterial Paw) is reduced by 30%.

142. C i.e. Bronchial stretch receptors 143. B i.e. Duration of expiration 144. C i.e Head's paradoxical reflex
145. B i.e. J receptor/Rtf. B est& T a y lo r 1 3 /6 5 3 -6 4 9 ; G a n on g 2 4 /6 6 4 -6 6 ; G abriel p l9 8 - 9 9 ; B ern e& L ev y 6 /4 7 2 - 7 4 ; jo h n W est 8/129-31 ;G u y t o n l l j 5 2 1 -1 6 1

- Tidal volume excessive load is prevented by (or lung maintains the tidal volume by) activation of slowly adapting
pulmonary (or bronchial) stretch receptors0 .
- "Inflation of lungs induces further inflation" due to elastic recoil of alveoli. With inflation of the lungs, there is
augmentation of respiratory effort= Head's paradoxical reflex.
- Excessive inflation of lungs cause reflex inhibition of inflation (& increase duration of expiration)0- this is Hering
Breuer inflation reflex mediated by vagus. Conversely, excessive deflation of lungs cause reflex inhibition of
expiration (decrease duration)- this is Hering- Breuer deflation reflex.
- Physiological role of J (juxta-capilary/alveolar) reflex is uncertain but it occurs in pathological states such as
pulmonary congestion, pulmonary edem a° and embolism when the J recptors are stimulated by large lung hyper
inflation, or intravascular chemicals like capsaicin, bradykinin & serotonin etc.
- Slowly adapting pulmonary stretch receptors are stimulated by lung inflation (not hyper inflation) - Ganong p 632
table 37.2. Hyper inflation (over stretching) of lungs stimulate stretch receptors and lead to Hering - Breuer inflation
reflex. However, more recent work indicates HB reflex may be important in new bom babies but are largely inactive
in adult human unless the tidal volume exceeds 1 liter Qohn west) /1.5 liter (Guyton), as in exercise. So we can extract
inference that, although stretch receptors get stimulated by (normal-quiet) inflation, the HBI reflex occurs only after lung
hyper inflation.
88 A Complete Review of Short Subjects

Surfactant

146. A i.e. 28 Weeks 147. A i.e. Type II pneumocyte 148. C i.e. Phospholipids
149. A i.e. Dipalmitoyl phosphatidyl choline
150. A i.e. Secreted from 26lh week..; B i.e. In hyaline membrane disease deficiency ...; C i.e. Theraputic application is seen
151. C i.e. Increase in alveolar surface area by the surfactant 152. D i.e. Breaks structure of water in alveoli
153. A i.e. Surfactants [Ref: Mtx; Ganong 23/e 596-98; Guyton 11/e 474; Croftan Doughlas 5/e p 19-20, 41, 85; Gabriel p 181-83; John
West 8/e p 99-104, 10; Berne & Levy 6/e p 426-27; hangman's 11/ep 2041

Surfactant is produced by type II pneumocytes after 20 Components %


weeks of gestational ageQ is a mixture of phospholipids 1. Phospholipids eg 77
and protein. It maintains alveolar stability (integrity) - D ip alm itoy l p h osp h atid y l cholin e
by decreasing the surface tension in alveoli (i.e. (DPPC)Q 62
increasing alveolar surface area by not dissolving - Phosphatidyl glycerol (PG) 5
uniformly in fluid linning the alveoli = breaking the - O ther phospholipids 10
structure of water in alveoli = keeping alveoli dry)Q. It 2. Neutral lipids eg cholesterol and its 13
is deficient in hyaline membrane disease where it is ester
therapeutically usedQ. Surfactant breaks structure of 3. Protein: surfactant proteins (SP) like 8
water in alveoliQ. SP-A, SP-B, SP-C, SP-D
V_____________________________________________________
4. Carbohydrate 2
5. Fatty acid, Calcium

154. B i.e. Fibrin [Ref: Ganong 23/c p.598; Gabriel p. 182; hangman 11/e p. 206Robbins 6/e P 472[

Hyaline membranes are made up of fibrinogen^ and fibrinQ mixed with cell debris derived from necrotic type II
pneumocytes. In HMD, functional residual capacity (FRC) is below closing volume (CV)Q.

Non Respiratory Functions of Lung

155. B i.e. Conversion of angiotensin - I to angiotension II.


156. B i.e. Sodium balance [Ref: Ganong 23/e p 605-6; John West 8/e p 49-51; Berne & Levy 6/e p 477-83; Pulmonary physiology, Lange
6/e p 215-25; Lecture notes on human physiology 4/e p 400; Respiratory System: Devis & Moore (2003) p 28-29]

- One of the most important metabolic (non-respiratory) function of lungs is activation of relatively physiologically
inactive decapeptide angiotensin I to 50 times more potent (active) vasoconstrictor, pressure (TBP), aldosterone
stimulating octapeptid angiotensin II by extracellular ACE on the surface of endothelial cells in pulmonary circulation.
- As a part of renin - angiotensin-aldesterone system , lungs indirectly regulate sodium and water balance (and BP).
However, lungs also directly a ffect Na+ balaneQ via epithelial Na+ channels (E NaCs). Absorption of excess fluid from
alveolar lumen and airways require active transepithelial transport of N a+ by ENaCs (mainly of alveolar type II and
possibly type I cells.)

Prolonged immaturity of ENaCs AR, cystic fibrosis disease is characterized by thick, tenacious and dehydrated airway
in IR D S or hyaline membrane secretions. In CF, mutations in CFTR (cystic fibrosis trans membrane conductance
disease result in fluid regulator), which is a regulated Cl" channel, causes decreased ability to secrete C l' and
accumulation and failure of gas therefore enhance Na+ and water absorption out of airway. This reduces pericillary
exchange. fluid and results in thick, sticky and inspissated mucous that cannot be cleared from the
m ucociliary clearance system 1/t recurrent infections.__________________________________

157. B i.e. Contain se ro to n in » D i.e. Respond to a decrease in PO 2[Rcf:Berne 6/e p.421-25; CRMR vol 4(2008) p. 174-186 PNEC
system: Curtz et al]

Pulmonary neuroendocrinal cell (PNEC) like Kultschitzky cells secrete biological amines including dopamine & 5-
hydroxy tryptamine (serotonin). These cells are more numerous in fetus (than adults), have dual nerve supply and are
cells of origin for bronchial carcinoid.
90 A Complete Review of Short Subjects

Caisson's Disease is n itrogen (N 2) g a s em b o lism 6 seen in deep sea divers an d a v ia to rs (pilots)Q on sudden
decompression.
- Air em bolim occurs in neuro-cranial surgery and escape from submarines (i.e. person breathing compressed air from a
tank). Because the veins in the skull are in non-collapsible chamber, so they cannot collapse and a negative (suction)
pressure develops in dural sinuses (- 10 mmHg in saggital sinus in standing position). Therefore if dural sinus is
opened during surgery, air can be sucked (causing air em bolism ) in venous system. Similarly when a person ascends
(from deep sea in submarine) the gases in lung expand and sometimes rupture pulmonary blood vessel, forcing the
gases (air) to enter the vessels and cause air embolism.
^ ^

170. C i.e. Increased solubility of nitrogen in nerve cell membrane [Ref: Guyton llt h /e p. 545- 46; Gabriel p. 518]

Nitrogen narcosis (toxicity) or raptures of death sets in at around 120 feet and below sea surface because at high tissue
concentrations, N 2 behave as anesthetic gases and show increased solubility in lipid (fatty) substances of neuronal
membrane and makes them less excitable by altering ionic conductance^.
90 ■ A Complete Review of Short Subjects

Caisson's Disease is nitrogen (N2) g a s em bolism ® seen in deep sea divers and a v iators (pilots)® on sudden
decompression.
- Air embolim occurs in neuro-cranial surgery and escape from submarines (i.e. person breathing compressed air from a
tank). Because the veins in the skull are in non-collapsible chamber, so they cannot collapse and a negative (suction)
pressure develops in dural sinuses (-10 mmHg in saggital sinus in standing position). Therefore if dural sinus is
opened during surgery, air can be sucked (causing air embolism) in venous system. Similarly when a person ascends
(from deep sea in submarine) the gases in lung expand and sometimes rupture pulmonary blood vessel, forcing the
gases (air) to enter the vessels and cause air embolism.
>- -j

170. C i.e. Increased solubility of nitrogen in nerve cell membrane [Ref: Guyton llth /e p. 545- 46; Gabriel p. 518]

Nitrogen narcosis (toxicity) or raptures of death sets in at around 120 feet and below sea surface because at high tissue
concentrations, N 2 behave as anesthetic gases and show increased solubility in lipid (fatty) substances of neuronal
membrane and makes them less excitable by altering ionic conductance^.
Chapter -4 CARDIOVASCULAR SYSTEM

QUESTIONS

Electrical Activity of Heart C. Epicardium -> Septum -> Endocardium □


D. Septum -> Epicardium -> Endocardium □
1. The following statements are true regarding the SA 10. Repolarization in isolated muscle piece fibre proceeds
node except: (AIIMS 05) from: (PGI 08)
A. Is located at the right border of the ascending A. Epicardium to endocardium □
aorta □ B. Endocardium to epicardium □
B. It contains specialized nodal cardiac muscle □ C. Left to right □
C. It is supplied by the artial branches of the right D. Right to left □
coronary artery □ 11. Preload leads to- (NEB P 13)
D. It initiates cardiac conduction □ A. Isovolulmetric relaxation □
2. Prepotential in SA node is due to all EXCEPT? B. Isovolumetric contraction □
A. Ca2+ spark (NBE P 13) D C. Peripheral resistance □
B. Fast sodium channels opening □ D. Parasympathetic nervous system activation □
C. K+ □ 12. During the cardiac cycle the opening o f the aortic
D. Transient Ca chnnel opening □ valve takes place at the : (AI 04)
3. Hyperpolarizing cyclic nucleotide (HCN) gated A. Beginning of systole □
channels have a role in: (AIIMS 16) B. End of isovolumetric contraction □
A. Cardiac rhythm generation □ C. End of diastole □
B. Generation of mitochondrial action potential □ D. End of diastasis □
C. Memory formation □ 13. O f the follow ing which one correlates with
D. Myocardial muscle contraction □ isovolumic contraction phase: (AIIMS 08)
4. Inward flow of Na+ in heart leads to- (NBE P 13) A. AV opening and aortic and pulmonary valve
A. Plateu phase □ closure. □
B. Action potential □ B. AV closure and aortic & pulmonary valve
C. Repolarization □ opening. □
D. No change □ C. Both valves are closed □
5. Plateau phase of ventricular muscle is d/f opening of D. Both valves are open □
A. Na+channel (NBE P 14,13; PGI 15,12,11) □ 14. Isovolumic relaxation phase of the cardiac cycle ends
B. K+ channel □ w ith :
C. Ca++ - Na+ channel □ A. Peak of 'C' waves (AI 2K, NEET13) □
D. Closure of K+ channel □ B. Opening of A.V. valve □
E. Closure of CL channel □ C. Closure of semilunar valve □
6 . S.A. node acts as a pacemaker of the heart because of D. Beginning of T wave. □
the fact that it: (NBE P 15,13; AI 05) 15. Isometric relaxation is defined as- (NBE P 13)
A. Is capable of generating impulses spontaneously □ A. Relaxation of both atria
B. Has rich sympathetic innervations. □ B. Relaxation of both atria with all valves open
C. Has poor cholinergic innervations. □ C. Relaxation of LV with mitral & aortic valve closed
D. Generates impulses at the highest rate. □ 16. Einthovens law- (NBE P 13)
7. Highest conduction rate in m/s is observed in: A. I+III=II □
A. SAnode (NBE P 15,14,13; PGI 05, AI 05) □ B. I+III=II □
B. AV node □ C. 1+11+111=0 □
C. Bundle of His □ D. I+III=avL □
D. Purkinje system □ 17. Einthoven's triangle, what is the value of lead III
8 . Least conduction velocity is seen in OR Gatekeeper of when lead l = 2 mv and Lead =1 mv- (NBE P 13)
the heart is: (NBE P 15,14,13; PGI 08) A. 1 □
A. AV node □ B. 2 □
B. Purkinje fibres □ C. 3 □
C. Bundle of his □ D. 4 □
D. Ventricular myocardial fibres □ 18. C' wave in JV P is due to (AI 09)
9. Which of the following is the order of activation after A. Atrial contraction □
stimulation of Purkinji fibrs is : (AIIMS 05) B. Atrial filling with tricuspid valvesclosed □
A. Septum -> Endocardium -> Epicardium □ C. Rapid right ventricular filling □
B. Endocrdium -> Septum -> Epicardium □ D. Bulging of tricuspid valve into the rightatrium □
92 ■ A Complete Review of Short Subjects

19. The feature of JVP in cardiac tamponade is. (PGI 09) 28. The ECG of a 40 year old male was recorded using
A. Prominent x descent with prominent 'y'descent □ standard bipolar limb leads. The sum of voltage of the
B. Prominent x descent with absent 'y' descent □ three standard leads was found to be 5 millivolts. This
C. Absent x descent with prominent 'y' descent □ indicates: (AIIMS 05)
D. Absent x descent with absent 'y' descent □ A. A normal heart □
20. A wave in JV P is due to- (NBE P 13) B. Right ventricular hypertrophy □
A. Atrial systole □ C. Left ventricular hypertrophy □
B. Atrial diastole □ D. Increased cardiac muscle mass □
C. Ventricular systole □ 29. On increasing vagal tone following occurs in
D. Ventricular diastole □ pacemaker- (NBE P 13)
21. The left atrial V wave is larger than 'a' wave in A. Increased Na+ increased slop □
comparison to right atrium because: (AIIMS 14) B. Decreased Na+ decreased slop □
A. Left side is high pressure system □ C. Increased Na+ decreased slop □
B. Left atrium is more compliant □ D. Decreased Na+ increased slop □
C. Left atrium is compressed posteriorly by 30. Vagal stimulation of the heart causes: (AIIMS 08)
pulmonary veins □ A. Increased heart rate □
D. Right atrial pressure is transmitted into the B. Increased R - R interval in ECG □
pulmonary circulation □ C. Increased force of heart contraction □
22. True statement about instantaneous means vector: D. Increased cardiac output □
A. Equal & same as mean QRS vector (PGI 11) □ 31. Hypokalemia ECG changes all except- (NBE P 13)
B. It is drawn through the centre of vector in a A. Tall T waves □
direction from base toward apex □ B. Prolonged QRS interval □
C. Summated vector of generated potential at C. Depressed ST segment □
particular instant cause by inflowing septal D. Prominent U waves □
depolarization □ 32. A cardiologist asks for measurement of
D. When a vector is exactly horizontal and directed electromechanical systole (QS 2) and left ventricular
toward the person's left side, the vector is said to ejection time (LVET), and pre-ejection period (PEP). The
extend in the direction of 0 °. □ cardiologist is informed by the technician that the
23. Ventricular contraction lasts for: (PGI 11) carotid transducer is not functioning. Which of the
A. Beginning of Q wave to end of S wave □ following readings could not be obtained
B. Beginning of Q wave to the end of T wave □ A. QS 2 (AIIMS 02) □
C. Beginning of P wave to end of T wave □ B. QS 2 and LVET □
D. Beginning of R wave to the end of T wave, if Q C. QS 2 and PEP □
wave is absent □ D. LVET and PEP □
E. Beginning of P wave to the end of S wave □ 33. Duration of 2nd heart sound is - (NBE P 13)
24. Q RS complex indicates : (AI 08) A. 0.15sec □
A. Atrial repolarization □ B. 0.12sec □
B. Atrial depolarization □ C. 0.08sec □
C. Ventricular repolarization □ D. O.lsec □
D. Ventricular depolarization □ 34. 3rd heart sound is d/t- (NBE P 13)
25. P wave in absent in- (NBE P 13) A. Closure of av valve □
A. Atrial fibrillation □ B. Closure of aortic valve □
B. CCF □ C. Mid diastolic flow in the ventricle □
C. Atrial flutter □ D. Atrial contraction □
D. PSVT □ 35. Fourth heart sound is caused by- (NBE P 13)
26. Normal duration of PR interval- (NBE P 13) A. Closure of AV valves □
A. 0.12-0.2sec □ B. Closure of semilunar valves □
B. 0.2-0.3sec □ C. Rapid ventricular filling □
C. 0.3-0.4sec □ D. Atrial contraction □
D. 0.4-0.5sec □ 36. Which of the follow ing is true about fourth heart
27. Electromechanical Systole is best defined as the sound 'S4': (AI 07)
interval between (AI 12) A. Can be heard by the unaided ear □
A. Q wave and S2 (Q-S2 Interval) □ B. Frequency is greater than 20 Hz □
B. Q wave and SI (Q-Sl Interval) □ C. Heard during ventricular filling phase □
C. Q wave and beginning of T wave □ D. Heard during ventricular ejection phase □
D. Q wave and R wave □ 37. During cardiac imaging the phase of minimum
motion of heart is: (AI 10)
A. Late systole O
Physiology: Cardiovascular System ■ 93

B. Mid systole □ C. Eating □


C. Late diastole □ D. Standing from lying position □
D. Mid diastole □ 48. Cardiac output is increased in- (NBE P 13)
A. Sleep □
Heart as a Pump B. Pregnancy □
C. Sitting □
38. Cardiac index in a normal person is D. Standing □
A. 2.1 (NBE P 13,14; TN 08, KA 07) □ 49. W hich scientific principle is the basis for
B. 3.2 □ Therm odilution method used in measurement of
C. 4.6 □ cardiac output by pulmonary catheter? (AIIMS 12)
D. 5.9 □ A. Hagen-Poisseuille Principle □
39. In a patient with cardiac output 5 liters/minute and B. Stewart-Hamilton Principle □
body surface area 1.7 m2 what w ill be the cardiac C. Bernoulli's Principle □
index (PGI 04) D. Universal Gas Equation □
A. 3 liter/min/m 2 □ 50. Starlings law im plies- (NBE P 13)
B. 4 liter/min/m 2 □ A. Increased VR-increased CO □
C. 5 liter/min/m 2 □ B. Increased discharge-increased CO □
D. 2.5 liter/mkin/m 2 □ C. Increased HR-increased CO □
E. 4.7 liter/min/m 2 □ D. Increased BP-increased CO □
40. Amount of blood in heart- (NBE P 13) 51. Preload is increased by- (NBE P 13)
A. 250-300ml □ A. Increased blood volume □
B. 500-600ml □ B. Increased total peripheral resistance □
C. 1-2 litre □ C. Standing □
D. 100-200ml □ D. Sitting □
41. Ejection fraction is- (NBE P 13) 52. End diastolic volume increases due to (AIIMS 12)
A. SV/EDV □ A. Increase in intrapericardial pressure □
B. EDV/SV □ B. Decrease in ventricular compliance □
C. ESV/EDV □ C. Decrease in total blood volume □
D. SV/EDV □ D. Increase in negative intra thoracic pressure □
42. What is the normal left ventricular ejection fraction? 53. A ll of the follow ing factors normally increase the
A. 20% (NBE P 13) □ length of the ventricular cardiac muscle fibers except:
B. 30% □ A. Increased venous tone. (AI 05) □
C. 50% □ B. Increased total blood volume. □
D. 65% □ C. Increased negative intrathoracic pressure. □
43. Basal cardiac output in an adult in nearly; D. Lying-to-standing change in posture □
A. 7.5 litre (PGI 09) □ 54. Single most important factor in control of automatic
B. 5 litre □ contractility of heart is: (AI 03)
C. 12 litre □ A. Myocordial wall thickness □
D. 10 litre □ B. Right atrial volume □
44. Cardiac output in L/min divided by heart rate equals: C. SA node pacemaker potential □
A. Cardiac efficiency (NBE P 15,13; AIIMS 14; AI 05) □ D. Sympathetic stimulation □
B. Mean stroke volume □ 55. Discharge from Baroreceptors causes inhibition of
C. Cardiac index □ A. Caudal Ventrolateral Medulla 64712; NBE P 14) □
D. Mean arterial pressure □ B. Rostral Ventrolateral Medulla □
45. Cardiac index is defined as: (PGI 06) C. Nucleus ambiguous □
A. Stroke volume M2/BSA □ D. Nucleus tractus solitarus □
B. C.O.P. per unit body surface area □ 56. Which of the follow ing statements about vasomotor
C. Syst press/M 2 BSA □ centre (VMC) of medulla is true (AIIMS 11, AI 09)
D. End diastolic volume □ A. Independent of corticohypothlamic inputs □
46. The cardiac output can be determined by all except B. Influenced by baroceptor signals butnot by
A. Fick's principle (PGI 09) □ chemoreceptors □
B. V/Q ratio □ C. Acts along with the cardiovagal centre (CVC) to
C. Echocardiography □ maintain Blood pressure. □
D. Thermodilution □ D. Essentially Silent in sleep □
47. Cardiac output decreases during: (AI 12) 57. Baroreceptor reflex, true is- (NBE P 13)
A. High Environmental temperature □ A. From aortic and carotid body □
B. Anxiety and excitement □ B. Causes arterial vasoconstriction when BP falls n
94 ■ A Complete Review of Short Subjects

C. Causes decrease in HR when BP increases □ A. End diastolic volume of ventricles □


D. Causes increase in HR when BP increases □ B. Endo systolic volume □
58. Baroreceptor stimulation produces; (PGI 15,13,05) C. Volume of blood in aorta □
A. Decreased heart rate & BP □ D. Ventricular ejection volume □
B. Increased heart rate & BP □ 67. Sympathetic stimulation causes all of the following
C. Increased cardiac contractility □ except: (NBE P 14, AI 09)
D. Decreased cardiac contractility □ A. Increase in heart rate □
59. Clamping of the carotid arteries below the carotid B. Increase in blood pressure □
sinus is likely to produce; (AIIMS 12) C. Increase in total peripheral resistance □
A. Increase in discharge of carotid sinus afferent D. Increase in venous capacitance □
nerves □ 68. Stroke volume is decreased in- (NBE P 13)
B. Decreased heart rate and blood pressure □ A. Lying down □
C. Increase in vasomotor centre activity □ B. Standing □
D. Baroreceptor adaptation □ C. Hypervolemia □
60. Clamping of the carotid arteries below (proximal) the D. Increased muscle tone □
carotid sinus is likely to produce: (AI 12) 69. W hen a person changes position from standing to
A. Increase in Blood pressure and increase in Heart lying down position, follow ing occurs: (AIIMS 07)
Rate □ A. Heart rate increases and settles at a higher
B. Decrease in Blood pressure and decrease in Heart level □
Rate □ B. Venous return to the heart rises immediately □
C. Increase in Blood pressure and decrease in Heart C. Cerebral blood flow becomes more than that in
Rate □ standing position and settles at a higher level □
D. Decrease in Blood Pressure and increase in Heart D. Decrease in blood flow to the lung apex □
Rate □ 70. A shift of posture from supine to upright posture is
61. Volume receptors are- (NBE P 13) associated with cardiovascular adjustments. Which of
A. Affected by total cardiovascular output □ the follow ing is NOT true in this context: (AIIMS 03)
B. Stimulated by atrial systole and diastole □ A. Rise in central venous pressure. □
C. Stimulated by left ventricular contraction □ B. Rise in heart rate. □
D. Stimulated by aortic pressure □ C. Decrease in cardiac output. □
62. A Patient with increased BP and decreased heart rate D. Decrease in stroke volume. □
is likely to have: (PGI 10,15) 71. When a normal person changes position from
A. Increased ICT □ standing to lying down (supine), follow ing change is
B. Deep sea diving □ seen: (AIIMS 13, AI 09)
C. Brain tumor □ A. Heart rate increases □
D. Head trauma □ B. Venous return to heart increases immediately □
63. The triad of hypertension, bradycardia and irregular C. Cerebral blood flow increases □
respiration is seen in: (AIIMS 14) D. Blood flow at apices of lung decreases □
A. Cushing's reflex □ 72. In the given circuit, the inward flow pressure is
B. Bezold-Zarisch reflex □ 100mm Hg and the outward flow pressure is 10 mm
C. Herring-Bruer reflex □ Hg. Each of the parallel circuit has a resistance of 5
D. Bainbridge's reflex □ mm Hg/mL/min. Calculate the flow across the circuit:
64. Which of the follow ing statement is least correct: (AIIMS 14)
5 m m H g /m l/m in
(AI 08)
A. Vagal stimulation decreases rate of contraction □
B. Noradrenaline increases force of contraction □
C. Denervated heart has more heart rate □
D. During Exercise Systole is shortened more than
diastole □
65. TRUE regarding human heart (AI 08)
A. Conduction of impulse from endocardium to
inwards □
B. During exercise duration of systole is reduced
more than diastole □
C. HR increases with parasympathetic
denervation □ B. 90 mL □
D. Vagal stimulation decreases force of contraction □ C. 3.6 mL □
66 . Volume determining preload is; (PGI 08) D. 135 mL □
Physiology: Cardiovascular System ■ 95

Blood Pressure C. Mean circulatory filling pressure □


D. Perfusion pressure □
73. Which of the follow ing is true about measurement of 82. Mean circulatory filling pressure is?
blood pressure? (AIIMS 14) A. Difference between systemic and pulmonary
A. Bladder in the blood pressure cuff should cover arterial pressure (AIIMS 15,13,08) □
more than 80% of the arm area □ B. Difference between central venous pressure and
B. The person should be comfortably sitting and central arterial pressure □
blood pressure cuff should be at a higher level than C. Mean atrial pressure □
the heart □ D. Arterial pressure taken at the point when heart
C. Caffeine intake induces decrease in blood pressure□ stops beating □
D. Blood pressure increases during sleep □ 83. Venous return (VR) is given by formula - (DNB 09)
74. The blood pressure measured by a A. VR = (R V R -R A P )/ M F SP □
sphygmomanometer: (AI 08) B. VR = (M FSP-R V R)/R A P □
A. Is lower than the intraarterial pressure □ C. VR = (MFSP - RAP)/RVR □
B. Is higher than the intraarterial pressure □ D. None □
C. Is same as the intraarterial pressure □ 84. Blood pressure is defined as the product of:
D. Is the same with different cuff sizes □ A. Systolic pressure x pulse (NBE P 14,13; PGI 08) □
75. True about blood pressure measurement is all/except: B. Diastolic pressure x pulse rate □
A. Cuff width should be 40% of arm C. Pulse pressure x pulse rate □
circumference (AIIMS 07) □ D. Cardiac output x peripheral resistance □
B. Diastolic blood pressure is indicated by fourth 85. Spuriously high BP is seen in A/E : (AIIMS 01)
Korotkoff sound □ A. Auscultatory gap □
C. Small cuff measures spuriously elevated Diastolic B. Small cuff □
blood pressure □ C. Thick calcified vessels □
D. Monkenberg sclerosis causes pseudohypertension □ D. Obesity □
76. True statement regarding arterial BP; (PGI 13) 86. W hich is true about measurement of BP with
A. Diurnal variation present □ syphgnomonmeter versus intraarterial pressure
B. Low when taken with small cuff □ measurements: (AI 01)
C. Standing increases both SBP & DBP □ A. Less than Lntravascular pressure □
D. Should be measured in both arms □ B. More than intravascular pressure □
E. Exercise increases both SBP & DBP □ C. Equal to intravascular pressure □
77. W indkessel effect is seen in- (NBE P 13) D. Depends upon blood flow □
A. Large elastic vessels □ 87. In a study to measure BP, two students Rakesh and Arif
B. Capacitance vessels □ conduct a study on a dog. Rakesh measures BP using a
C. Throughfare channels □ mercury sphygmomanometer on the right femoral
D. Capillaries □ artery and Arif measures using a pressure transducer
78. During Diastole, the arterial pressure is maintained and pulse tracing on the left femoral artery. The mean
by (AI 09) arterial pressure for both Rakesh and Arif is the same
A. Elastic Recoil of aorta □ i.e. 100 mmHg. After 5 minutes of injection of
B. Musculature of arteries □ adrenaline, Rakesh measures blood pressure 130 mmHg
C. Constriction of capillaries □ and Arif as 120 mmHg. The difference of 10 mmHg is
D. Contraction of left ventricle □ explained by: (AIIMS 02)
79. Mean arterial pressure is : (NBE P 14; AI 06) A. Falsely high values at low pressure in pulse
A. Systolic + (Diastolic BP)/2 □ tracing □
B. Systolic + 1/3rd Pulse pressure □ B. Falsely low values at high pressure in pulse
C. Diastolic + (Systolic BP)/2 □ tracings □
D. Diastolic + 1/3rd Pulse pressure □ C. Femoral artery is more sensitive to adrenaline on
80. Mean arterial pressure is calculated as: right side □
A. (SBP + 2DBP)/3 (NBE P 15; AIIMS 11, 06) □ D. Ventricular filling affects diastole period □
B. (DBP + 2SBP)/3 □ 88. As a part of space-research program, a physiologist
C. (SBP + 3DBP)/2 □ was asked to investigate the effect of flight-induced
D. (DBP + 3SBP)/2 □ stress on blood pressure. Accordingly, the blood
81. Which of the follow ing defines the pressure in the pressure of the cosmonauts were to be measured
vascular system in the absence of blood flow? (Asked twice: once before the take-off and once after the
twice) (AIIMS 15,14; NBE P 13) spacecraft entered the designated orbit around the
A. Pulse pressure □ earth. For a proper comparison, the preflight blood
B. Critical closing pressure □ pressure should be recorded in: (AI 03)
A. The lying down position □
96 ■ A Complete Review of Short Subjects

B. The sitting position □ 97. All are vasodilators except- (NBE P 13)
C. The standing position □ A. Prostacyclin □
D. Any position, as long as the post-flight recording is B. NO □
made in the same position □ C. Bradykinin □
89. SI unit for measuring Blood Pressure is : (AI 02) D. Endothelin □
A. Torr □ 98. Which is not synthesized by vascular endothelium-
B. mmHg □ A. Prostacyclin (NBE P 13) D
C. kPa □ B. Angiotensin 2 □
D. Barr □ C. Endothelin □
90. Pressure on carotid sinus cause. (PGI 09, AI 10) D. Heparin □
A. Hyperapnea □
B. Reflex bradycardia □ Circulation Through Special Regions
C. Tachycardia □
D. Dyspnea □ 99. Normal interstitial pressure is- (NBE P 13)
91. Two students, Vineet and Kamlesh were asked to A. 10 to 15 mmHg □
demonstrate in dogs the role of sinus nerve in hypo­ B. -5 to 0 mmHg □
volemic shock. Vineet severed the sinus nerve when C. 20 to 30 mmHg □
the mean blood pressure (MBP) was 85 mm Hg and D. -10 to -20 mmHg □
Kamlesh cut the sinus nerve when the mean blood 100. Which of the follow ing favours filtration at the
pressure was 60 mm Hg. On cutting the sinus nerve arteriolar end of the capillary bed. (AIIMS 06)
A. Vineet recorded the increase in MBP but Kamlesh A. Decrease in hydrostatic pressure of capillaries □
recorded a decrease in MBP □ B. Increase in hydrostatic pressure of capillaries □
B. Vineet recorded a decrease in MBP but Kamlesh C. Increase in oncotic pressure of capillaries □
recorded an increase in MBP □ D. Decrease in oncotic pressure of interstitium □
C. Both recorded an increase in MBP (AI 03) □ 101. Which of the follow ing causes maximal cerebral
D. Both recorded a decrease in MBP □ vasodilation: (AI 12)
92. A pilot in Sukhoi aircraft is experiencing negative G. A. Hypercarbia □
Which of the follow ing physiological events will B. Hypoxia □
manifest in such situation? (AIIMS 04, A I 06) C. Lactic Acidosis □
A. The hydrostatic pressure in veins of lower limb D. Exercise □
increases □ 102. Cerebral blood flow is regulated by all except:
B. The cardiac output decreases □ A. Blood pressure (AIIMS 07, 08) □
C. Black out occurs □ B. Arterial PCO 2 □
D. The cerebral arterial pressure rises □ C. Potassium ions □
93. Shape of the arterial pulse is influenced by: D. Cerebral metabolic rate □
A. Viscosity of blood (AIIMS 06) □ 103. The best method to access the adequacy of
B. Velocity of blood □ replacement of fluid a case of shock: (AIIMS 2K)
C. Arterial wall expansion □ A. Decrease in thirst □
D. Cross sectional area of artery. □ B. Increased PaC>2 □
94. A 0.5 litre blood loss in 30 minutes w ill lead to C. Increase in urine output □
A. Increase in HR, decrease in BP (AI 01) □ D. Blood pressure □
B. Slight increase in HR, normal BP □ 104. Which of the follow ing is NOT correct regarding
C. Decrease in HR and BP □ capillaries: (AI 01)
D. Prominent increase in HR □ A. Greatest cross sectional area □
B. Contain 25% of blood □
Cardiovascular Regulatory Mechanism C. Contains less blood than veins □
D. Have single layer of cells bounding the lumen □
95. True regarding endothelin-1 are all except: 105. Which of the follow ing statements is true about
A. Bronchodilatation (AIIMS 07) □ cap illaries: (AI 06)
B. Vasoconstriction □ A. Contain 5% of total blood volume □
C. Decreased GFR □ B. Contain 10 % of total blood volume □
D. Has inotropic effect □ C. Velocity of blood flow is maximum □
96. Nitric Oxide is produced in: (PGI 14,12, 08) D. Offer maximum resistance to blood flow □
A. Endothelium □ 106. All are true regarding capillaries except:
B. Plasma □ (NBE P 15,13; AI 04; AIIMS 13)
C. Platelets □ A. Have large total cross-sectional area □
D. Serum □ B. Contain larger quantity of blood than veins □
Physiology: Cardiovascular System ■ 97

C. Site of gaseous exchange □ 116. What is the rise of blood flow if the radius of blood
D. Lined by endothelium □ vessel is increased by 50%? (AIIMS 15)
107. Which of the following statements about cutaneous A. 5 times □
shunt vessels is true: (AIIMS 11, AI 09) B. 10 times □
A. Perform nutritive function □ C. 20 times □
B. Have role in thermoregulation □ D. 100 times □
C. Not under the control of autonomic nervous 117. True about shear stress on vessel: (PGI 13)
system □ A. Parallel to the long axis of blood vessel □
D. These vessels are evenly distributed throughout B. Perpendicular to long axis of blood vessel □
the skin □ C. Increases with increase in axial velocity of fluid □
108. The velocity of blood is maximum in the: D. Poiseuille-Hagen equation correlate flow with
A. Large veins (AIIMS 05) □ viscosity □
B. Small veins □ E. Depend upon viscosity □
C. Venules □ 118. Flow is lam inar in small vessels because
D. Capillaries □ A. Reynolds number is >2000 (AI 11) □
109. Maximum difference of BP occurs between: B. Total cross sectional area of small vessels is
A. Descending Aorta & common iliac artery □ smaller □
B. Femoral artery & femoral vein (PGI 07) □ C. Diameter of smaller vessels is less □
C. Arterial end & venous end □ D. Effective velocity in small vessels is less □
D. Capillaries & venules □ 119. W hich of the follow ing increases turbulence in blood
E. Venules & vein □ flow (AIIMS 06)
110. Microcirculation consist of- (NBE P 13) A. Reynolds number less than 2000 □
A. Capillaries □ B. Decreases in velocity of blood □
B. Capillaries venules and arterioles □ C. Decrease in density of blood □
C. Aorta □ D. Increase in diameter of blood vessel □
D. Arteries and veins □ 120. W hich of these mediate the precapillary sphincter
111. Arteriole is- (NBE P 13) relaxation: (AIIMS 08)
A. Conduction vessel □ A. Local hormones □
B. Resistance vessel □ B. Sympathetic stimulations □
C. Exchange vessels □ C. Catcholamines □
D. Capacitance vessel □ D. Capillary filling □
112. Maximum reservoir of blood OR Hightest compliance 121. Coronary blood flow, true is- (NBE P 13)
is seen in which vessel - (NBE P 13) A. 250ml/min □
A. Veins □ B. Maximum during systole □
B. Aorta □ C. Adenosine decreases it □
C. Heart □ D. More than skin □
D. Capillaries □ 122. Blood flow to skeletal muscles (ml/min)- (NBE P 13)
113. Gas exchange in tissues takes place at- (NBE P 13) A. 100 □
A. Artery □ B. 200 □
B. Capillary □ C. 400 □
C. Vein □ D. 800 □
D. Venules □ 123. Brain blood supply- (NBE P 13)
114. All are true regarding circulation except (PGI 09) A. 55ml/100 gm/min □
A. Pulsatile flow affects gene transcription □ B. 400ml/lOOgrn/min □
B. Transfused blood is mainly distributed C. 100ml/lOOgrn/min □
in arteries □ D. 200ml/100gm/min □
C. Hematocrit markedly change peripheral 124. True about blood flow in various organs.
resistance □ A. Liver>Kidney>Brain > Heart (PGI 13,09,02) □
D. Increased viscosity increases mean blood B. Liver > Brain > Kidney > Heart □
pressure □ C. Kidney > Brain > Heart > Liver □
E. Non pulsatile flow is recommended □ D. Liver > Heart > Brain > Kidney □
115. Distribution of blood flow in mainly regulated by 125. The local control in blood flow is seen in A/E :
the: (AI 05) A. Skin (AI 05,AIIMS 09) □
A. Arteries. □ B. Muscle □
B. Arterioles. □ C. Splanchnic vessels □
C. Capillaries. □ D. Cerebrum □
D. Venules □
98 ■ A Complete Review of Short Subjects

126. According to myogenic hypothesis of renal C. Decreased vital capacity in supine position □
autoregulation, the afferent arterioles contract in D. Most blood is in pulmonary capillaries □
response to stretch induced by: (AIIMS 05) 135. During heavy exercise the cardiac output (CO)
A. No release □ increases upto five fold while pulmonary arterial
B. Narodrenaline release □ pressure rises very little. This physiological ability of
C. Opening of Ca2+ channels □ the pulmonary circulation is best explained by:
D. Adenosine release □ A. Increase in the number of open capillaries □
127. P.G. increasing the renal blood flow and causing renal B. Sympathetically mediated greater distensibility of
vasodilation is/are: (PGI 03) pulmonary vessels (AI 10) □
A. PGI2 □ C. Large amount of smooth muscle in pulmonary
B. PGE 2 □ arterioles □
C. TXA 2 □ D. Smaller surface area of pulmonary circulation □
D. PGFia □ 136. Pulmonary micro-circulation differs from systemic
E. PGF 2 □ circulation in having: (AI 05)
128. Regulation of coronary circulation is? (NBE P 13) A. Resistance low, pulsatile flow high □
A. Autonomic □ B. Resistance low, capillary pressure low □
B. Autoregulatory □ C. Resistance high, pulsatile flow low □
C. Hormonal □ D. Resistance high, capillary pressure high □
D. Sympathelic □ 137. All of the following statements about bronchial
129. Which one of the following is the correct statement circulation are true, Except: (AI 10)
regarding coronary food flow? (AIIMS 06) A. Contribute 2% of systemic circulation □
A. Coronary blood flow is directly related to B. Contribute to gaseous exchange □
perfusion pressure and inversely related to C. Cause venous admixing of blood □
resistance □ D. Provide nutritive function to lung □
B. Coronary blood flow is inversely related to 138. Deoxigenated Blood is carried by all except:
perfusion pressure and inversely related to A. Pulmonary Artery (AI 11) □
resistance □ B. Umbilical Vein □
C. Coronary blood flow is directly related to C. Right ventricle □
perfusion pressure and also to resistance □ D. Umbilical Art. □
D. Coronary blood flow is inversely related to both 139. Max. blood supply to liver is by: (PGI 2K)
pressures and resistance □ A. Portal Vein □
130. True about pulmonary circulation: (PGI 03) B. Hepatic A □
A. It receives 30% of cardiac output □ C. Splenic A □
B. Hypoxia causes vasoconstriction □ D. Mesenteric A □
C. Blood volume in lung is 450 ml. □ 140. Wedged hepatic venous pressure represents pressure
D. Pulmonary capillaries contain most of the blood in: (AIIMS 04)
volume in lung □ A. Main portal vein □
E. It as low resistance □ B. Main hepatic vein □
131. True regarding vascularity of lung is: (AIIMS 02) C. Sinusoids □
A. Hypoxia causes vasodilation □ D. Central vein radicles □
B. Pulmonary resistance is half of the systemic 141. Effectiveness of blood brain barrier is by:
vascular resistance □ A. Tight endothelial function (AIIMS 2K) □
C. Perfusion is more in the apical lobe than in base □ B. Microglial cell □
D. Distended pulmonary veins in the lower lobe □ C. Thick basement membrane □
132. Pulmonary circulation differs from systemic D. Tight arrangement of astrocyfes □
circulation (AIIMS 06, 08) 142. Protein filtration across cerebral capillaries is limited
A. Pulmonary vasodilation in hypoxia □ by: (AIIMS 05, 02)
B. Pulmonary vasoconstriction in hypoxia □ A. Fibrous tissue □
C. Decreased blood volume during systole □ B. Foot process of astrocytes □
D. Increased basal vasoconstrictor tone □ C. Low BP □
133. Physiological dead space in lung is D. High CSF pressure □
A. Zone 1 (]ipmer 06, UP08) □ 143. W hich of the following statements about myocardial
B. Zone 2 □ oxygen demand is true: (AI 09)
C. Zone 3 □ A. Correlates with heart rate □
D. Zone 4 □ B. Is directly proportional to external cardiac work □
134. True is all except about lung circulation C. Is negligible when heart is at rest □
A. V/P ratio at rest is 0.8 (PGI 09, WB05, KA03) □ D. Depends upon duration of systole □
B. In apex ventilation is less than at base □
Physiology: Cardiovascular System ■ 99

144. True regarding myocardial O 2 demand? (AIIMS 11) B. Aortic regurgitation □


A. Inversely related to heart rate □ C. Mitral stenosis □
B. Has constant relation to external cardiac work □ D. Aortic stenosis □
C. Directly proportional to duration of systole □ 154. In the follow ing diagram, left ventricular pressure is
D. Is negligible at rest □ equal to diastolic blood pressure indicated by which
145. About Myocardial Oxygen Consumption true is: point? (AIIMS 15)
A. Inverse relation with heart rate (AIIMS 0 7 )0
B. Inverse relation to mean systolic arterial
pressure
C. Constant relation to external work done
D. Negligible in quiescent heart
E. IDirectly proportional to mean arterial pressure
146. Myocardial oxygen demand depend upon:
A. Preload (PGI 09) □
B. Afterload □
C. Intramyocardial tension □
D. Myocardial muscle mass □
E. Blood Hb concentration □
147. Blood supply during exercise is increased in:
A. Cutaneous circulation (AI 05) □
B. Hepato-splanchnic circulation □ □
C. Renal circulation □ B. B □
D. Coronary circulation □ C. C □
148. Aerobic capacity is maximally increased b y : D. D □
A. Regular 3 minute exercise (A I0 7 )D 155. Calculate the ejection fraction from the given volume
B. Spurts of exercise □ pressure curve: (AIIMS 15)
C. Prolonged exercises □
D. Sternous exercises □
149. In a patient with a transplanted heart which of these
are the reasons for increased cardiac output during
exercise: (PGI 02)
A. Reinnervation of transplanted heart by vagus □
B. Intrinsic mechanism □
C. Epinephrine from medulla □
D. Bainbridge reflex □
E. Frank-Starling mechanism □
150. Blood in splanchnic area during exercise is decreased
due to : (AI 07)
A. Venoconstriction with decreased blood flow □
B. Venodilation with decreased blood flow □
C. Venodilation with increased blood flow □ A. 40% □
D. Venodilation with normal blood flow □ B. 50% □
151. The vasodilatation produced by carbon dioxide is C. 55% □
maximum in one of the following: (AI 05) D. 60% □
A. Kidney. □
B. Brain. □ Circulating Body Fluid
C. Liver. □
D. Heart. □ 156. Lymphocytes are produced in- (NBE P 13)
152. Patients having acute cardiac failure do not show A. Lymph node □
oedema, because : (AI 04) B. Thymus □
A. The plasma oncotic pressure is high □ C. Bone marrow □
B. There is renal compensation □ D. None □
C. There is an increase in cardiac output □ 157. In Fetal life RBC are produced in all except-
D. There is a fall in the systemic capillary hydrostatic A. Liver (NBE P 1 3 ) 0
pressure □ B. Lymph node □
153. The pressure-volume curve is shifted to the left in C. Spleen □
A. Mitral regurgitation (AIIMS 04) □ D. Bone marrow □
100 " A Complete Review of Short Subjects

158. Erythopoiesis in gestation age takes place in- 169. The follow ing image shows cell count in different
A. Yolk sac (NBE P 13) □ squares of a Neubaueris chamber after charging with
B. Placenta □ 20 times diluted blood. What is the total leukocyte
C. Amniotic sac □ count? (AIIMS 16)
D. Chorion □
159. Site of RBC formation in 20 year old healthy male is
A. Flat bones (AI 07) □
B. Long bones □
C. Liver □
D. Yolk sac □
160. Heme is converted to bilirubin mainly in: (AI 05)
A. Kidney. □
B. Liver. □
C. Spleen. □
D. Bone marrow. □
A. 6000 per cu mm □
161. Metalloproteins help in jaundice by the following B. 2400 per cu mm □
m echanism : (AI 01) C. 3000 per cu mm □
A. Increased glucoronyl transferase activity □ D. 14200 per cu mm □
B. Inhibit heme oxygenase □ 170. In this Leishman's stained blood smear, identify the
C. Decrease RBC lysis □ cell which is raised markedly in worm infestations.
D. Increase Y and Z receptors □
162. Which of the following protein inhibits heme loss
from plasma- (NBE P 13)
A. Ferritin □
B. Hemopexin □
C. Ceruloplasmin □
D. Hmosiderin □
163. What is true of iron: (PGI 2K)
A. It is stored in ferritin □
B. It is absorbed by Transferrin in the intestine □ A. C (AIIMS 16) □
C. Spleen in major storage organ □ B. B □
D. Fe++ is excreted in urine □ C. D □
164.2nd M ost Common Hb in adult is- (NBE P 13) D. A □
A. o2(3 □ 171. Which of the follow ing situations w ill lead to
B. a2Y2 □ increased viscosity of blood: (AIIMS 05)
C. a262 □ A. Fasting state □
D. p2y2 □ B. Hypoglycemia □
165. True about fetal Hb (PGI 12)
C. Multiple myeloma □
A. It is made up of 2 alpha and 2beta subunit □ D. Amyloidogenesis □
B. It is made up of 2 alpha and 2 gamma subunit □
172. Albumin is important factor in maintaining osmotic
C. It is made up of 2 alpha and 2 delta subunit □
pressure, has: (AIIMS 08)
D. It has high affinity for oxygen □
A. Low molecular weight andhigh blood
E. It has low affinity for oxygen □
concentration □
166. The type of hemoglobin that has least affinity for 2,3-
B. Low molecular weight and low blood
Diphosphoglycerate (2,3-DPG) or (2,3-BPG) is:
concentration. □
A. Hg A. (AI 05) □
B. Hg F. □ C. High molecular weight and low blood
C. H B. □ concentration. □
D. Hg A2. □ D. High molecular weight and high blood
167. Embryonic hemoglobin is mainly composed of? concentration. □
A. Alpha and gamma chains (AIIMS 13) □ 173. Thrombosthenin is a/an: (AIIMS 07)
B. Alpha and beta chains □ A. Thrombosis preventing protein □
C. Zeta and epsilon chains □ B. Contractile protein □
D. Alpha and delta chains □ C. Coagulation protein □
168. ADT test is for: (PGI 08) D. Protein for regulating platelets production □
A. Bence jounes protein □ 174. Central lymphoid organs are: (PGI 03)
B. Fetabl Hb □ A. Bone marrow □
C. Albumin □ B. Lymph node □
D. Myoglobin □ C. Thymus □
Physiology Cardiovascular System ■ 101

D. Spleen □ 185. Function of lymphatics (PGI 02)


E. Tonsil □ A. Increases oncotic pressure □

175. True about cytokines are all except: (PGI 07) B. Carries protein II
A. It is polypeptide □ C. Immunocompetent cells C
B. Have autocrine and paracrine effects □ D. Fat absorption C
C. Involve in leukocytic movements □ E. Preventive role in infection C
D. It takes part in intrinsic enzymatic reaction □
Clotting & Anticlotting
i/o. n eip er ana cytotoxic cens Deiong to:
186. All are vitamin K dependent clotting factors of
A. T cells (PGI 07) □
hepatic origin except: (PGI 15,13,09,
B. B cells □
A. II □
C. Monocytes □
B. VII □
D. Macrophages □
c _.
V 111
vV i i i n
l_ l
177. Lysozymes are found in: (PGI 05)
n x n

A. Eukaryotes □
187. Not a vit. K dependent factor; (NBE P 13,15; PGI 08,
B. Prokaryotes □
A. II □
C. Plants □ B. VII □
D. Bacteriophage □ C. IX □
E. Virus □ D. XII □
178. G-CSF and GM CSF in haematopoisis causes: 188. Activity of factor V III procoagulant is deficient in:
A. Leucocytosis (AI 09, 06)D A. Haemophilia (PGI 08) □
B. Erythrocytosis □ B. Von Willibrand's disease □
C. Leucopenia □ C. ITP □
D. Thrombocytosis □ D. Sickle cell anemia □
179. Macrophages release(s): (PGI 12) 189. True about Haemophilia A are all e x c e p t :
A. IL-2 n A. PTT increased (AI 01) □
B. IL -6 □ B. PT increased □
C. IL-1 □ C. Clotting time is increased. □
D. TNF-ot □ D. Serum levels of factor VIII are decreased. □
E. Interferon □
190. Which of the follow ing is not involved in intrinsic
180. Neutrophil secretes: (PGI 02) pathway?
A. Superoxide dismutase □
A. Factor XII (Al 09, AIIMS 15,13) □
B. Myeloperoxidase □
B. Factor XI □

C. Lysozymal enzyme □
C. Factor IX □

D. Catalase □
D. Factor VII □

E. Cathepsin G 191. The substance that is present in both serum and



piasma is:
181. Which is/are true of NK cell: (PGI 12)
A. Fibrinogen (AI 07) □
A. It is a large glanular lymphocyte □
B. Factor VII □
B. Releases granzymes and perforins □
C. Factor V □
C. Depends on antibody for killing □
n P a r l-n r TT f~ l
D. Depend on thymus for killing □
192. H alf life of factor V III- (NBE P 13)
E. Can kill virus infected and cancer cells □
A. 2-4 □
182. Absolute monocytosis is seen in: (PGI 06)
B. 8-12 □
A. Infectious mononucleosis □
C. 6 minutes □
B. Kala-azar □
D. 60 days □
C. TB □
193. Which of the follow ing helps in bridging the fibrin in
D. Brucellosis □
a clot and stabilizes the clot O R The clot formed is not
183. Source of TNF are (PGI 04) stable unless extensive cross-linking occurs. This
A. Endothelium □ extensive cross-linking of blood clot is done by?
B. Neutrophil □ A. Factor XIII (AIIMS 15,14; A I 09)D
C. Macrophage □ B. Factor V □

D. Basophils □ C. Factor VIII □

E. B Cells □ D. Factor III n


184. Life span of neutrophils is: (PGI 07) 194. Conversionof fibrinogen to fibrin is by- (NBE P 13)
A. 6 hours □ A. Prothrombin □

B. 6 days □ B. Factor XIII □

C. 10 days □ C. Thrombin □

D. 15 days □ D. Kallikrein □
102 ■ A Complete Review of Short Subjects

195. Which of the following coagulation factors causes 204. The genotype of a person with blood group 'A'
cross linking and stabilization of clot (AI 08) should be:
A. Factor XIII □ A. OO (AI 12) □
B. Thrombin □ B. BO □
C. Factor VIII □ C. AB □
D. Factor IX □ D. AO □
196. Which of the following anticoagulants prevent 205. Memory cell in immune system are long lived and
clotting of blood in the normal vascular system:647 12) escape apoptosis because of: (PGI 09)
A. Antithrombin III □ A. Insulin like growth factor □
B. Plasminogen □ B. Fibroblast growth factor (FGF) □
C. Fibrinogen □ C. Platelet derived growth factor □
D. Heparin □ D. Nerve growth factor □
197. Thrombomodulin I is produced by all of the 206. Erythropoiesis is promoted by all of the following
following EXCEPT (AI 08) except: (AIIMS 04)
A. Splanchnic circulation □ A. ACTH □
B. Skin circulation □ B. Thyroxine □
C. Cerebral circulation □ C. Oestrogen □
D. Muscle circulation □
D. Prolactin □
198. The blood within the vessels does not clot normally
207. What is not true for extrasystole in ventricle :
because: (AI 08)
A. Falls to produce radial pulse (AIIMS 09) □
A. Vitamin K antagonists are present in plasma □
B. Hints at serious heart ailment □
B. Thrombin has a positive feedback on
C. Associated with abnormal QRS complex □
plasminogen. □
D. Tendency to be followed by a compensatory
C. Sodium citrate in plasma chelates calcium ions □
pause □
D. Vascular endothelium is smooth and coated with
glycocalyx □
208. Reticulocytes are stained with: (AIIMS 04)
A. Methyl violet □
199. Thrombin activity is inhibited by (AI 08)
A. Chymotrypsin □ B. Brilliant cresyl blue □
B. Heparin cofactor II □ C. Sudan Black □
C. Alpha 2 antitrypsin □ D. Indigocarmine □
D. Alpha 2 macroglobulin □ 209. A procoagulant not normally circulating in the plasma
200. Thromboxane A2 causes- (NBE P 13) is: (AIIMS 03)
A. Vasodilatation and platelet aggregartion □ A. Prothrombin □
B. Vasodilatation and anti platelet aggregatory □ B. Fibrinogen □
C. Vasoconstriction and anti platelet aggregatory □ C. Antithemophilic factor □
D. Vasoconstriction and platelet aggregartion □ D. Factor V □
201. During homeostasis, platelet affects the coagulation E. None of the above □
area: (PGI 04) 210. Furasol DA is : (AI 06)
A. Platelet adhesion to exposed endothelium □ A. Free radical □
B. Clot retraction □ B. Artificial blood □
C. Activation of prothrombinase complex □ C. CO antagonist □
D. Vasoconstriction □ D. Used to increase 0 2 delivery to tissue □
E. Conversion of fibrinogen to fibrin □ 211. Which of the following is true regarding hydroxyl
202. Although more than 400 blood groups have been ethyl starch : (AI 02)
identified, the ABO blood group system remains the A. It is an anesthetic agent □
most important in clinical medicine because: B. It is a plasma expander □
A. It was the first blood group system to be C. It is a crystalloid □
discovered. (AIIMS 03)H D. Used as a nutritional agent □
B. It has four different blood groups A,B,AB,0(H) □ 212. Hydroxyethyl starch is a (AI 04)
C. ABO (H) antigens are present in most body tissues A. Vasodilator □
and fluids. □ B. Inotrope □
D. ABO (H) antibodies are invariably present in C. Plasma expander □
plasma when persons RBC lacks the corresponding D. Diuretic □
antigen. □ 213. Ringer lactate contains all EXCEPT:
203. True about ABO blood group antigen: (PGI 11) A. Chloride (AIIMS 07) □
A. Found on RBC membrane □ B. Sodium □
B. Glycprotein in nature □ C. Bicarbonate □
C. Highly immunogenic □ D. Potassium □
D. Autosomal recessive inheritance □
Physiology: Cardiovascular System ■ 105

Normal Spread o i Cardiac Excitation Conduction Rates (Speeds) in Cardiac Tissue’1'


(Depolarization or Electrical Activity)
Tissue Conduction rate Relative
- Depolarization begins in SA node (located (m/s) value
subepicardially) which is the pacemaker of heart as S A node 0.05 2 nd least ®
it generates impulses a t the highest rate®. Atrial pathway 1
- The excitation (depolarization) spreads through AV node 0 .0 2 -0 .0 5 L east ®
atria to converge on AVnode. Because of slow Bundle of His 1
conduction in AV node, there is a AV node delay of Purkinje system 4Q Highest®
about 0 .1 s before depolarization spreads to Ventricular 1
ventricle. This delay is shortened by sym pathetic muscle
stimulation and lengthened by vagal stimulation. A
Conduction Rate in decreasing ord er Purkinje fibers Q > Bundle
marked loss of conduction is seen when there is lack
of His=Ventricular muscle=Atrial pathway> SA node > AV node®
of Inh in depolarization phase (0 ).
Rhythm icity of Heart
- After AV node, the wave of excitation spreads from
top of septum in rapidly conducting Purkinje Tissue Rate of im pulse generation
fibers to all parts o f ventricle in 0.08 - 0 . 1s. SA node 70-80/minutes (Highest)®
- Ventricular depolarization (excitation) starts from AV node 40-60/minutes
left side o f interventricular septumQ and moves first Bundle of His 40/minutes
to the right across the mid portion of septum. It Purkinje system 24/minutes
then travels down the septum to the apex of heart.
From apex, the wave of depolarization returns Rate of im pulse generation to decreasing ord er
upwards along the ventricular walls to AV groove, SA node® > AV node > Bundle of His > purkinje system®
proceeding from endocardial to epicardial surface. AV node and purkinje fibres are also capable of generating impulses
Therefore, posterobasal portion of the left spontaneously but SA node discharges most rapidly (70-80/min)
& depolarizes the AV node and purkinje fibres before they can
ventricle, pulmonary conus and uppermost
portion of interventicular septum are all the last reach their own threshold for self excitation.

Order of Depolarization of Septum Order of Activation Normal Spread of


• Left —» Mid - » Right of Ventricular Electrical Activity
• Upper —» Lower —>Upper most M uscle
First the Purkinje Order of spread o f cardiac excitation
Order of Depolarization (Activation) of system inside the SA node -> Atrium -> AV node
Ventricular M uscle septumQ —>Bundle of His —» Purkinje system
Endocardial surface of both ventricles are activated
rapidly but the wave of excitation spreads from
I
Entire Endocardium®
—^Ventricular muscles
• Order of activation of ventricular
endocardium to epicardium at a slower velocity (0.3 muscle
of ventricle
- 0 .4 m/sec) through • Depolarization starts at the left side o f
Epicardial surface of right ventricle is depolarized ventricular interventricular septum ® —> Moves first
(activated) earlier than left because of its thinner ■muscle to the right across the mid portion of
wall. septum - » Activation of anteroseptal
Epicardium®
- Apical and central epicardial regions of both region of ventricular mayocardium (apex)
ventricles are activated earlier than their respective —> Major portion of ventricular
basal regions. myocardium from endocardium to
epicardium®.
Purkinje system inside the inter ventricular ■ Last part to be depolarized are:
septum (excluding basal portion) is depolarized R ight
1-P osterobasal portion o f left
first (along with papillary muscles) ventricle® (epicardial region)
I 2-Pulmonary Conus
From SA node to AV
Entire endocardium of ventricles 3-Upper m ost (basal) portion o f
node; Left to Right;
I interventricular septum®
Endo to Epi cardiumQ
through ventricular muscle

Epicardium
104 ■ A Complete Review of Short Subjects

3. A i.e. Cardiac rhythm generation IRef: Ganong 24/ep 523]

- Hyper polarization activated cyclic Nucleotide Gated (HCN) channels also k/a H channels or Funny (F) channels d/t
unusual (funny) activation (in hyper polarization phase), are nonselective cation channels found in plasma m embranes
of heart & brain cells. It is encoded by 4 genes HCN 1, 2, 3._____________________________________________________________
- It is also k/a pacemaker channel as it generates rhythmic activity in SA/AV nodes and neuronal rhythm icity in CNS. It
produces funny/hyperpolarizing current (If or Ih).

4. B i.e. Action potential 5. C i.e. Ca++-Na+ Channel [Ref: Ganong 24/e p - 522-23; Guyton 12/e p 115-117,102-103]

In Cardiac (e.g. Ventricular) Muscle

Phase Is due to
Rapid depolarization Na+influx through rapidly opening
phase or development fast Na+channels (N a+current, INa)
of action potential (O)
Initial rapid Inactivation of N a+channel
repolarization phase (1 )
Platue phase (2) C a++influx (Ca++current, lea) through
m ore slowly opening (slow) Ca++
channels (also called calcium-
sodium channels)
Slow repolarization K+ efflux through m ultiple types of
phase (3) K + channels, allow s the cell to return
to resting m em brane potential

Myocardial fibers (cells) have RMP of approximately -90mv.

6. D i.e. Generates impulses at the highest rate. 7. D i.e. Purkinje system


8. A i.e. AV node 9. A i.e. Septum - Endocardium - Epicardium
[Ref: Ganong 24/e p. 524; Berne <&Levy 6/e p. 308]

- Sinus node (located sub epicardiallyj® controls the beat of heart because its rate o f discharge is greater than that o f any
other part o f heart®. Therefore it is the pacemaker of heart.
- Purkinje fibers has fa stest conduction®. Conduction Rate in decreasing ord er Purkinje fibers ® > Bundle of His =
Ventricular muscle = Atrial pathway> SA node > AV node®.So AV node is k/a "Gate keeper of heart".
- Ventricular depolarization (excitation) starts from left side o f interventricular septum® and moves first to the right
across the mid portion of septum. It then travels down the septum to the apex of heart. From apex, the wave of
depolarization returns upwards along the ventricular walls to AV groove, proceeding from endocardial to epicardial
surface. Therefore, posterobasal portion of the left ventricle, pulmonary conus and uppermost portion of
interventicular septum are all the last parts of heart to be depolorized (excited).

10. B i.e. Endocardium to epicardium [Ref: Harrison 16/e P-1314; Best and Taylor's 12/e P 177, Guyton 8/27]

Repolarization in isolated ventricular muscle fibre is from endocardium to epicardium®, whereas in whole heart the
direction is from epicardium to endocardium.

Direction of Depolarization & Repolarization

Isolated Ventricular Muscle Fiber Whole Heart

Depolarization Repolarization Depolarization Repolarization


(QRS com plex) (T-wave)
From endocardium to Endocardium to I I
epicardium® epicardium® Endocardium to Epicardium to
epicardium® endocardium
Physiology: Cardiovascular System 103

ANSWERS & EXPLANATIONS:

Electrical Activity of Heart

1. A i.e. Is located at the right border of ascending aorta [Ref: Moore 4/e P-137; BDC 4/e Vol. IP-245-50]

S. A node is located at the junction o f superior venacave with right atrium® just deep to the epicardium, near the
superior end of sulcus terminalis and AV node is located in the right postero-inferior region of interatrial septum near the
opening of coronary sinus.

2. B i.e. Fast sodium channels opening [Ref: Ganong 24/ep 523]

Prepotential or Pacemaker potential i.e. the action potential in SA & AV nodes are d/t Ca**, w ith no contribution by ☆
N a* influx®, so there is no sharp, rapid depolarizing spike before the plateau, as there in other parts. Prepotentials are
prominent only in SA & AV nodes, but latent pacemakers are present in other parts of conducting system. However, atrial
& ventricular muscles do not have prepotentialsQ

Pacemaker (Pre) Potential


A_ A_
The various parts of conducting system and myocardium are capable
mV
of spontaneous discharge. However SA node norm ally discharges
60
m ost rapidly ®, depolarization spreading from it to the other regions Sympalhelic
stimulation
before they discharge. The SA node is there fore the normal cardiac
pacemaker, its rate of discharge determining the rate at which the .A .
heart beats. mV
-60
Rhythmically discharging cells have a membrane potential, that after Vagal
stimulation
each impulse, declines to the firing level. Thus this prepotential or
pacemaker potential triggers the next impulse. E ffect o f s y m p a th e tic (n o ra d re n e rg ic )
and vagal (c h o lin e rg ic ) s tim u la tio n o n th e m e m b r a n e
Repolarization At the peak of each impulse IK ( the K+ current, K + potential o f th e SA n o d e .
efflux) by opening of K + channels begins &brings
about repolarization
Depolarization IK then declines (K + efflux decreases), the
(Start) membrane begins to depolarize forming the first
part of prepotential
Slow depolarization starts d/t opening of Funny (F)/Hyper
polarization (H) channels (so called because of their unusual/funny
activation by hyper polarization and can pass both Na+ & K +). As
funny current (Ih) increases, depolarization begins forming first part
of prepotential
Depolarization - The calcium current (lea) d /1 opening of T
(Late) (transient) channels completes the prepotential and
lea d/t opening of L (long lasting) channels
produce the impulse.
Local Ca++ release (Ca** spark) from sarcoplasmic
reticulum occurs during prepotentialQ

When the cholinergic (vagal) fibers to nodal tissue are stimulated, the membrane becomes hyperpolarized and the slope of
prepotentials decreased, because Ach acts on M 2 muscarinic receptors and open K+ channels. The resulting IKAch counters the
decay of IK. In addition activation of M 2 receptors decreases C- AMP and slows opening of Ca** channels resulting in
decrease in firing rate.
Sympathetic stimulation increases the rate of spontaneous discharge and makes the membrane potential fall more rapidly (d/t
opening of L channels)
Discharge frequency also increase with temperature rise (1/t tachycardia) and digitalis exerts effect like that of vagal
simulation particularly on the AV node.
106 ■ A Complete Review of Short Subjects

11. B i.e. Isovolumetric contraction

Preload Tension on muscle when it starts to contract. For heart it is usually end diastolic pressure when
ventricles has filled
After load Load against which muscle contracts. For heart it is pressure in aorta or resistance in circulation

12. B i.e. End of isovolumetric contraction 13. C i.e. Both value are closed
14. B i.e. Opening of A.V. value [Ref: Ganong 24/e P- 540-45; Chaudhuri 5/e P - 176; Sembulingum 3/e P- 436]
15. C i.e. Relaxation of LV with mitral & aortic valve closed

During isovolumic contraction phase both semilunar (=aortic & pulmonary) valves and AV (mitral & tricuspid) ☆
valves are dosedQ. Opening of aortic & pulmonary valves take place at the end of isovolumetric contraction & or at
the beginning of ventricular ejection^.
Similarly during isovolumetric ventricular relaxation phase both AV and sem ilunar valves are closed. It ends with
opening of AV valvesQ, permitting the ventricles to fill.

Mechanical Events in Cardiac Cycle (= 0.80 sec)

Ventricular systole=0.27 sec Diastole = 0.53 sec


Valves Isovolumetric Ventricular Proto-diastole Isovolumetric Late Atrial Systole
Ventricular Ejection = 0.22 0.04 sec Ventricular Diastole VF VF
Contraction = sec Relaxation
0.05sec =0.08sec
AV Closed Closed Closed Closed Open Open
(Bi/Tricuspid)
Aortic & Closed Open Closed Closed Closed Closed
Pulmonary

Valvular event Cardiac events ECG JVP


(valves) (Jugular Venous
Pressure curve)
Opening of AV End of isovolumic relaxation phased End of T waveQ V-Y descents
valveQ (phase b/w T wave and new p
wave)
Closure of AV valve -End of diastole Later half of R wave End of V descent
- Beginning of isovolumic contraction
Opening of semilunar End of isovolumic contraction^ ST segment Peak of 'C' waveQ
valve
Closure of semilunar - Beginning of isovolumic relaxation phase Later half of T wave
valve - Beginning of diastole

16. A i.e. I+III=II 17. A i.e. 1 [Ref: Guyton 12/e p 125]

Einthoven's Equilateral Triangle Einthoven's Bipolar (Standard) Limb Leads between


With the heart in center as moving dipole, it is
formed by two shoulder & pubis. RA(-)& RA (-) & LA (-) &
But electrodes are connected to left arm (LA), right LA (+) LL (+) LL (+)
arm (RA) and left leg (LL) for convenience. i i i
All 3 comers always have O potentials and so are Lead I Lead II Lead III
k/a indifferent electrode (central terminal of
Einthoven's Law for Sum of Electrical Potential
Wilson).
(Voltage) in Leads_____________________________
Physiology: Cardiovascular System ■ 107

18. D i.e. Bulging of tricuspid valve into the right atrium [Ref: Harrison 16/e P- 1305,1306]
19. B i.e. Prominent X descent with absent 'Y ' descent. 20. A i.e. Atrial systole

'C 'wave in JVP is due to bulging of tricuspid (AV) valve into right atriumQ during right ventricular isovolumetric
contraction.
JVP in cardiac temponade has prominent x descent with absent Y descent^.

Jugular Venous Pattern (JVP)

a waves: Due to distention produced by right atrial contraction^.


c waves: Are positive waves produced by bulging of tricuspid valve
into right atrium during right ventricular^ isovolumetric
contraction.
v waves: Due to increasing volume of blood in the right atrium
during ventricular systole when the tricuspid valve is
closed.Q
a -x Due both to atrial relaxation and to the downward
descent: displacement of the tricuspid valve during ventricular
systole.
Accentuated in Reduced with Features of JV P in:
_______ I________
- Constrictive pericarditis - Right ventricular dilatation
- Cardiac temponade Cardiac Constrictive
Reversed with
- Restrictive cardiomyopathy temponade pericarditis
- Tricuspid Regurgitation
- Prominent 'X' - Prominent 'Y'
v -y By the opening of the tricuspid valve and the subsequent
descent descent
descent: rapid inflow of blood into the right ventricle.
- Absent 'Y' - Present 'X'
Accentuated in Reduced with
descent descent
- Tricuspid regurgitation - Tricuspid stenosis
- Constrictive pericarditis - Right / Atrial Myxoma
(Suggests obstruction to right
ventricular filling)
y - a descent: Due to continuous diastolic inflow of blood into great veins,
right atrium and ventricle which are all in free
communication during diastole.
21. A i.e. Left side is high pressure system [Ref: Braunwald 9je p. 1110-12]

In comparison to right atrium, the left atrial V wave is larger than a wave (in JV P) because 1ft side is high pressure
system.

22. A, B,C, D i.e. Instantaneous mean vector is equal & same as mean Q RS vector; It is drawn through the centre o f vector
in a direction from base toward apex; Summated vector of generated potential at particular instant cause by inflow ing
septal depolarization; When a vector is exactly horizontal and directed toward the person's left side, the vector is said
to extend in the direction o f 0 °.
23. B i.e. Beginning of Q wave to the end of T wave; D i.e. Beginning of R wave to the end of T wave, if Q wave is absent
24. D i.e. Ventricular depolarization [Ref: Ganong 24/e p 526-28; Guyton 12/e p. 129-137,121-123; Harrison 18/e p. 1832; Berne 6 /e p 310-11]

QRS complex (wave) reflects ventricular depolarization 0


Q T interval (or period of electrical systole of ventricles) is closely correlated with the mean action potential duration o f
ventricular myocytes. Contraction of ventricles lasts from the beginning o f Q wave (or R wave, if Q wave is absent) to the
end of T wave: an interval k/a QT interval.

25. A i.e. Atrial fibrillation 26. A i.e. 0.12-0.2sec

P wave (in ECG) indicate atrial depolarization. It is characteristically absent in hyperkalemia (TK+), atrial fibrillation^,
SA block, AV node rhythm, and ventricular tachycardia^.
108 " A Complete Review of Short Subjects

ECG

Intervals /Wave Normal Duration (sec) Events in the heart


Average Range
P wave Atrial depolarization^
T wave Ventricular RepolarizationQ
PR (PQ) interval 0.18 0 .1 2 -0 .2 0 Q Atrio ventricular conduction (from onset of atrial activation to
onset of ventricular excitation)
Q RS complex (wave) 0.08 Q to 0.10 Ventricular depolarization (& atrial repolarization is masked)
Q T (RT) interval 0.40 to 0.43 Ventricular action potential or contraction^
(Ventricular depolarization + Ventricular repolarization)
ST interval (QT minus 0.32 Platue phase of ventricle action potential
QRS) (Ventricular repolarization)

27. A i.e Q wave and S2 (Q-S2 Interval) [Ref: Ganong 24/e p 542-43; Katz 5/e p 340-42]

Electromechanical systole is best defined y QS2 interval 0


J
28. D i.e. Increased cardiac muscle mass [Ref: Ganong 22/e P- 550; Guyton 10/e P 127]
1■
When the sum of voltages of all the QRS complexes of the three standard leads is greater than 4 m illivolts, the ECG is considered
as high voltage electrocardiogram. Most often due to increased cardiac muscle mass Q.

The voltage in the three standard leads is measured from the peak of R wave tothe bottom of S wave = 0.5- 2.0 millivolts.
Highest voltage is in lead II, lowest in lead III.

29. B i.e. Decreased Na+ decreased slop


30. B i.e. Increased R - R interval in ECG [Ref: Guyton llth /e p. 113; Ganong23rd/e p. 502-4; 494; Gabriel CEzeilo p. 122,124]

*01
• SA node (develops from right side of embryo) is supplied by right vagus and AV node (develops from left so) is
supplied by left vagus. Vagal or parasympathetic stimulation of heart causes decreased conduction (negative
dromotropic), decreased force o f contraction, cardiac out put, ejection fraction and heart rateQ (negative
chronotropic) and increased R - R interval on ECGQ. It also increases refractory period.

• R R interval is used to define a cardiac cycle and determine heart rate from ECG. A Heart beat occurs at every R wave, one
small division represents 0.04 s & 1 minute is represented by 1500 (60/ 0.04) small divisions. So heart rate (HR) is

HR = Number o f ' RR intervals in 1 minute (or 1500 small divisions)


HR = 1500
Number of small division in one R R interval

So increased R R interval means decreased heart rate (bradycardia)Q and decreased R R interval means increased heart
rate (tachycardia)________________________________________________________________________________________________
• Pacemaker (pre) Potential is mainly d /1 Na+ influx. Vagal stimulation decreases Na+ influx and cause decrease in
slope (=flattening) of pacemaker (pre) potentials. So time taken to reach threshold is increased & HR is decreased.
• Sympathetic stimulation causes positive chronotropic (THR d/t increased slope of phase 4 of pacemaker potential),
positive ionotropic (T contractility), positive dromotropic (T conduction velocity), positive bathmotropic (increased
automaticity) but decreased refractory period

31. A i.e. Tall T waves [Ref: Harrison 17/ep 282]


Prolonged PR interval, QRS widening & depressed/sagged ST segment is seen in, both hypo & hyperkalemia. So both
are differentiated by

____________________ Hypokalemia________________ __________________ Hyperkalemia________


- Prominent U wave (earliest)Q - Tall-tented (Narrow & peaked) T waves
- Flat & inverted T waveQ & Prolonged QT interval - Sine wave configuration
- Ventricular systole____________________
Physiology: Cardiovascular System ■ 109

32. D i.e. LVET and PEP. [Ref: Ganong 22/e P- 568]

I f carotid transducer is not functioning w e w ould not be able to measure LVET and PEP 0

33. B i.e. 0.12sec 34. C i.e. Mid diastolic flow in the ventricle 35. C i.e. Rapid ventricular filling
36. C i.e. Heard during ventricular filling [Ref: Guyton 11/e p.269-70; Ganong 22/e p. 569; Harrison 16/e p. 1308]

f ----------------------------------------------------------------------------------------------------------------------------------------------------------------------
- 3rd heart sound is d/t mid (middle third) diastolic flow (rapid filling) of ventricle0 .
- S4 is a low pitched presystolic sound which can alm ost never be heard w ith a stethoscope (& ear) because o f its
weakness and very low frequency - usually £20 cycles/ second (Hz)°. It is caused by in thrashing o f blood into
ventricles (ventricular filling0, which initiates vibration) when the atria contracts0 .

Feature 1 st heart sound 2 nd heart sound 3rd heat sound 4th heart sound *
Character Low pitched Shorter high Soft low pitched weak Arterial heart sound
(frequency) slightly pitched "dup" rumbling
prolonged "lub"
Duration 0.14 (0.15)second 0 .1 1 (0 .1 2 ) sec 0.1 sec -
Frequency 25-45 Hz 50 Hz - < 20 Hz®
Cause Sudden closure of Closure of aortic & R apid ventricular Ventricular filling® d/t atrial
mitral & tricuspid pulmonary valves filling® d/t inthrushing contraction causing
valves of blood from atria inthrushing of blood
Timing Start of ventricular Just after end of Begining o f middle Immediately before 1st heart
systole ventricular systole third o f diastole 0 sound (presystolic)
Heard Si & S 2 can be heard with stethoscope S3 & S4 can not be heard by stethoscope (ear)®. S 3 oftenly
and S4 sometimes can be recorded in phonocardiorgam

37. D i.e. Mid diastole [Ref: Braunwald (2002) p 227; M D C T : A practical approach (2006) p-213; MDCT: From protocol to practice
(2008) p 213; CT of CVS (2007) p 110; B ecker: Multislice CT 3/e p 197]

f -----------------------------------------------------------------------------------------------------------------------------------------------------------------------------
- During cardiac imaging (such as M RI, CT, electron beam tomography), mid diastolic phase (or diastasis) of cardiac
cycle is usually (but not always) associated with lowest (minimum) mean motion o f heart .0
- Optimum phase (i.e. with minimum cardiac motion) for cardiac and coronary vessel imaging is m id-diastole
(diastasis) at low or intermediate heart rates; but is variable and may occur in late systole at high heart rates.

_______________ Heart As Pump_______________

38. B i.e. 3.2 39. A i.e. 3 liters /min /m2 [Ref: Guyton 10/e P - 551]

_ , Cardiac Output
Cardiac Index = =— = 3L /min /m2
Body Surface Area 1.7

40. B i.e. 500-600ml


Heart contains 12% of blood volume i.e. -6 0 0 m l° (of 5L).

41. A i.e. SV/EDV 42. D i.e. 65% 43. B i.e. 5 litre 44. B i.e. M ean stroke volume
45. B i.e. COP per unit body surface area 46. B i.e. V/Q ratio [Measures ventilation perfusion ratio]

Cardiac output 5L/min ° The output of the heart per unit time. It can be measured by Fick's principle,
thermodilution technique and Doppler echocardiography®.
Stroke volume 70 ml® The amount of blood pumped out of each ventricle per beat. It is difference
between end diastolic & end systolic ventricle volume (i.e. differences b/w
volume of blood in left ventricle at the end of diastole i.e. 1 2 0 ml and at the end
of systole i.e. 50 ml)
M ean stroke volum e 0 = Cardiac output/Heart rate
110 ■ A Complete Review of Short Subjects

Ejection Percentage of stroke volume (SV) that is ejected by each strokeQ. it is valuable index of ventricular
Fraction (EF) pump function^ and decreases in failing heart

E F= Stroke volume (SV ) x l 0 0 = 7 0 x l 0 0 = 60% «


End diastolic ventricle volume (ED V) 120

Cardiac Index 3.2L/min/m2® Cardiac output/ body surface area®

47. D i.e. Standing from lying position 48. B i.e. Pregnancy

Cardiac Output

I
Measured by Affecting conditions
___________l _ _
I------------- I
Direct method Indirect method No change Increase Decrease
(In animal only) (In human beings)
I I I I I
- By cardiometer By Fick's principle Q Sleep Anxiety & Sitting /Standing
- By Flow meter By indicator dilution Moderate excitement from lying
technique change Eating position^
By thermodilution in Exercise Rapid arrhythmia
technique^ environmental High temperature Heart disease
By Doppler- temperature PregnancyQ
echocardiographyQ Epinephrine

49. B i.e . S e tw a r t-H a m ilto n P r in c ip le : [R ef: G a n on g 2 4 /e p 5 4 5 -4 6 ; G u yton 12/e p 240-41 & N et]

Thermodilution (cold saline) method, a type of indicator (dye /radioactive isotope) dilution method measures cardiac
output by pulmonary artery catheter using Stewart - Hamilton principle®.

50. A i.e. Increased VR-increased CO 51. A i.e. Increased blood volume


52. D i.e. Increase in negative intrathoracic pressure 53. D i.e. Lying to standing change in posture
[R ef: G an on g 2 4 /e p. 5 4 0 ,5 4 7 ; 2 3 /e p. 5 1 5 ; 2 2 /e p. 5 7 2 ; G u yton 's 12 S A E p. 168, 207-111

When a person moves from a supine position to standing position volume of blood pools in the lower extremity

because of high compliance of veins & venous return decreases. As a result of decreased venous return the length o f
ventricular cardiac muscle fibres is decreased® and so the cardiac output in accordance with Frank-Sterling law.
Preload or end diastolic volume/pressure or venous return or length o f ventricular cardiac muscle fiber increases due to
increase in negative intrathoracic pressure®, increased total blood volume^ and increased venous tone. Whereas EDV
decreases on standing, increased intrapericardial pressure and decreased ventricular compliance.
Increase in negative intrathoracic pressure (ie more expansion of thorax) would increase the pressure gradient and thus
would increase the venous return to heart, thereby increasing preload or EDV.

Preload /Venous Return /End Diastolic Volume (EDV) or Pressure (EDP) /


Length of Ventricular Cardiac Muscle Fiber

Preload is the degree of tension on the muscle which begins to contract. For cardiac contraction it is end diastolic pressure
(EDP) of filled ventricles which inturn corresponds to ventricular filling (end diastolic volume= EDV) or venous return or
stretch/length of ventricular cardiac muscle fiber. According to Frank Starling law, force of cardiac contraction (so
stroke volume & cardiac output) is proportional to muscle fiber length (preload). After load is the load against which
the muscle exerts its contractile force. For heart it is pressure in aorta. Preload/EDV/EDP/VR/VCMF length is
Physiology: Cardiovascular System ■ 111

Increase by factors causing increased Decreased by factors causing decreased venous


venous return return

- Increased total blood volumes - Increased venous compliance


- Increased venous tone (or stiffness) or decreased - Decreased ventricular compliance (ie increased
venous com pliances ventricular stiffness) due to MI, myopathy etc.
- Muscular activity or increased pumping action of - Increased intrapericardial pressure due to cardiac
skeletal muscleQ temponade, pericardial effusion, constrictive
- Increased negative intrathoracic pressure (ie more pericarditis or tumor
expanded chest) - Standing (from lying)Q
- Strong atrial contraction
- Squatting

54. D i.e. Sympathetic stimulation [Ref: Ganong 24/e P- 547-50]

Sympathetic stimulation is the single most important factor in control of automatic myocardial cantractilityQ.

Catecholamines liberated by adrenergic (sympathetic) stimulation cause cardiac acceleration (increased heart rate = +
chronotropic action) and increased strength of cardiac contraction (increased myocardial contractility = + inotropic
action).

55. B i.e. Rostral Ventrolateral M edulla 56. C i.e. Acts along with the cardiovagal centre (CVC) to maintain BP
57. C i.e. Causes decrease in HR when BP increases 58. A i.e. Decreased HR & BP; D i.e. Decreased cardiac contractility
[ Ref: Textbook of physiology: Gabriel C ezeilo (oxford) p- 142- 43, 385,481; Guyton llth /e p. 212-13, 210; Ganong 23/e P-556-59]
■ K
- Vasomotor centre (VMC) controlling sympathetic outflow acts along w ith cardiovagal centre (CVC) controlling
parasympathetic out flow to maintain blood pressures.
- Baroreceptors, chemoreceptors and corticohypothalmic inputs influence VMCQ.
- Baroreceptors are tonically active stretch receptors located in w alls of aortic arch & carotid sinus (not body).
Discharge from baroreceptors cause inhibition of C l neurons o f vasomotor center (VMC) situated in upper (rostral)
half o f ventrolateral mdullaG and facilitation of cardiovagal centre (CVC) or nucleus ambiguous.
- Baroreceptors are the stretch receptors in the walls of heart & blood vessels eg carotid sinus & aortic arch receptors,
receptors in walls of atria at enterance of SVC, IVC & pulmonary veins and receptors in pulmonary circulation (cardio­
pulmonary receptors). These are stimulated by distension of the structure in which they are located 1/ 1 vagal
innervation of heart and producing vasodilation, venodilation, a drop in BP, bradycardia & decrease cardiac output
(contractility) Q.

Baroreceptor Discharge (+)


I
Through NTS (Nucleus Tractus Solitaries) cause

r
Inhibition of (-) VMC=Vasomotor Center C l Neurons Facilitation (+) of CVC=Cardio Vagal Center or
in Upper/Rostral- Ventro-Lateral M edulla (RVLM) Nucleus Ambiguous___________________________
I I
Vasodilatation & 4-BPS Bradycardia & Decreased Cardiac Output (Contractility)

59. C i.e. Increase in vasomotor centre activity


60. A i.e. Increase in Blood pressure and increase in Heart Rate: [Ref: Ganong 24/e p 589-93; Guyton 12/e p 206]

Stretching of carotid sinus baroreceptors [d/t


v% C lam ping distal (a b o v e )
C arotid sinus in internal
clamping of bilateral carotid arteries above C arotid artery
(distal to) carotid sinus] stimulate their firing C lam ping proxim al (below ) (S tim ulates vagal
Carotid sinus p a rasym pathetic center
which leads to peripheral vasodilation decreased (S tim ulates v asom otor •V asodilation
heart rate, decreased strength of contraction, center: s ym pathetic respnse) -d e c re a s e d s trength of
decreased cardiac output and decreased blood •Vasoconstriction contraction
-In c rea s ed s trength of •D e c re a s e d H R , C o , B P
pressure (BP) by excitation of vagal contraction
-In c rea s ed H R , C o, BP
parasympathetic center.
112 ■ A Complete Review of Short Subjects

- Clamping of carotid arteries proximal (below) the carotid sinus, decreases pressure within the sinus reducing its
discharge. This intum stimulates vasomotor center in medulla and inhibits vagal parasympathetic center. This induces
reciprocal compensatory response causing peripheral vasoconstriction, increased heart rate, increased strength of
contraction, increased cardiac output and increased blood pressure (BP). When bilateral vagatomy is also performed
the BP rise >500/200mmHg and is unstable.
- Bilateral destruction of visceral afferent nucleus ie nucleus tractus solitaries (NTS), which is the site of termination of
baro receptor afferents, also causes a marked (severe may be even fatal) hypertension.

61. A i.e. Affected by total cardiovascular output

Volume (Low pressure) receptors are located in walls of right and left atrium at the entrance of SVC and IVC and
pulmonary veins as well as in pulmonary circulation. In low pressure part of circulation, these are k/a
Cardiopulmonary receptors. These are stimulated by distension or pressure rise in structure (total cardiovascular
output)Q.

62. A, C, D i.e. Increased ICT, Brain tumor, Head trauma 63. A i.e. C u sh in g 's re fle x
[Ganong 24/e 607; Guyton 11/e 213; Gabriel p 150-171; Berne & Levy 6/e p- 362]

Elevated intracranial pressure, as caused by space occupying lesion like brain tumor or head trauma with associated
intracranial bleeding (eg epidural /subdural/cerebral- haemorrhage), results in triad of irregular respiration, increased
system ic blood pressure ( TBP) and bradycardia® (rather than tachycardia), This response is called Cushing's reflex or
phenomenon.

64. D i.e. During exercise systole is shortened more than diastole [Ref: Guyton 11/e p. 112-13; Ganong 22/e 603-05, 566-68
65. C i.e. Heart rate increases with parasympathetic denervation Chaudhauri 6/e p. 191, 202; KDT 6/e p. 478]

The duration of systole is much more fixed than that of diastole and when the heart rate is increased (eg. in exercise),
diastole is shortened to a much greater degree.Q

66. A i.e End diastolic volume of ventricles [Ref: Ganong 22/e P- 572-573]
/---------------------------------------------------------------------------------------------------
According to frank-starling law, the length of muscle fibers (extent of the pre-load) is proportionate to the end diastolic
volumeQ

67. D i.e. Increase in venous capacitance


Effect of Sympathetic Stimulation On Feature Sympathetic Parasympathetic
(adrenergic) (cholinergic)
Heart Blood vessels stimulation stimulation
- Increase the heart rate - Venocons friction:
Heart rate Increased^ Decreased (main)
(+ve chronotropic decrease in venous
action) capacitance® (chronotropic)
- Increases the strength of - V asoconstriction: Conduction Increased^ Decreased
cardiac contraction (+ve in creased b lo o d pressure ®
inotropic action)Q & increased peripheral Force of contraction Increased^ Decreased (to a
resistance (Ionotropic) lesser extent)

68. B i.e. Standing 69. B i.e. Venous return to heart rises immediately
70. A i.e. Rise in central venous pressure [Ref: Ganong 22/e p.630 - 31, 615-18, 662; Guyton 11/e p. 762]
------------------------------------------------------------------------------------ — ---------------------------------------------------------------------------------s V
- When a person changes posture from standing to lying down position, heart rate decreases and venous return to 1
heart rises.
- On assumption of erect posture, the force of gravity opposes the return of blood (i.e.4- in central venous p re ssu re )Q
Physiology: Cardiovascular System ■ 113

Cardiovascular Effect on Standing Supine (Lying Standing


Feature down) Position (Upright) Position

• K i
400 ml
Peripheral venous 300 -500 ml blood pools in
pooling venous capacitance vessels
of lower extremities
Central blood pool Same amount of blood
decreases 400 ml
Central venous Decreases 3mm Hg
pressure
Arterial blood Sudden decrease followed 1 ___
pressure by a relative gradual rise
and settles at a lower level.
Stroke volume Decrease by 40 %Q
40%
Cardiac output (& Decrease by 25%®
25%
venous return)
Abdominal & limb Decrease by 25%
25%
flow
Heart rate Increase by 25® 25%

Total peripheral Increase by 25%® 25%


resistance

Abdominal & lim b Increase


resistance
Small vein pressure Increase by 10 mmHg 10 mmHg

71. B i.e. Venous return to heart increases immediately

In standing posture, 300- 500ml o f blood p ools in venous capacitance vessels o f low er extremities, which is im m ediately
returned tow ards heart# on assuming lying down posture.

72. B i.e. 90 mL [Ref: Ganong 24/e p. 573]

Five (5) resistance of 5mm Hg/ml/min connected in parallel. So net resistance in circuit (circulation) according to Ohm's

J_ _ J_ J_ J_ J_ J_
Law would be
R Rj R2 Rj R4 Rj

1 I------
:— 1 1---
1 1— 1, O / „
1 = 1 mmHg/ml/min •
-

5 5 5 5 5

„ , , Effective Perfusion Pressure


Now, Flow in any portion of vascular system = —
Net Resistance

Pjow _ Inward flow pressure - Outward flow pressure _ 100-10 _ 9 Qmj/mjn


Net Resistance 1
114 « A Complete Review of Short Subjects

Blood Pressure

73. A i.e. Bladder in the blood pressure cuff should cover more than 80% of the arm area
74. B i.e . Is h ig h e r th a n in tra a r te r ia l p re s s u r e 75. B i.e . D ia s to lic B P is in d ic a te d b y 4 ,h K o v o tk o f f's so u n d
76. A i.e . D iu m a l v a r ia tio n p re s e n t; D i.e . S h o u ld b e m e a su re d in b o th a rm s; E i.e . E x e rc ise in c r e a s e s b o th S B P & D B P
77. A i.e. Large elastic vessels [Ref: Ganong 24/e p. 578-79; Jain 5/e p. 360-62; Snow Screening fo r disease (2004) p.135; M edicine fo r anesthesia
4/285; Guyton U /p l6 6 -6 7 ,175-76}

Blood pressure measured by auscultatory method using sphygmomanometer tends to be higher than true intra­
arterial pressure measured by arterial cannulation, because some cuff pressure gets dissipated between the cuff and
arteryQ, in the soft tissue.
Systolic pressure is best indicated by 1st Korotkoff's sound and diastolic pressure in adults by 5th korotkoff's
soundQ. Where as diastolic pressure in children, in adults after exercise, hyperthyroidism & aortic insufficiency best
correlates with 4th Korotkoff's sound.
Using relatively small cuff, obesity and persons with thick calcified & sclerotic vessels that are difficult to
compress (eg in elderly, atherosclerosis, diabetics and M onkenberg's arteriosclerosis)Q are the reasons of spuriously
high blood pressure (Pseudohypertension).

Blood Pressure

Auscultatory method described by Nicolay Korotkoff is standard method for BP measurement. It uses inflatable cuff (Riva-
Rocci-cuf) attached to a mercury sphygmomanometer, which is gold standard but may be supplemented by aneroid
sphygmomanometer.

Technique Korotkoff's sound Source of error


- Person should be comfortably seated, with >A re p ro d u c e d b y tu rb u le n t flo w in artery . False High values are found in
back & arm supported, legs uncrossed & T h e so u n d g ra d u a lly b e c o m e lo u d e r, th en • T h ic k o v e rly in g so ft tissu e c a u sin g m o re
upper arm a t the level o f right atrium®. d u ll & m u ffled . d issip a tio n o f p re ssu re eg. ob ese
- Snugly w rapped cu ff sh o u ld a llo w o n ly one 1. F a in t, clear, ta p p in g so u n d s • H ard s c le r o tic v e s s e ls w ith lo w
fin g er to b e slip p e d b / w it & skin . It in d ica te s s y s t o lic BP® c o m p r e s s ib ility su ch as
- T h e c u ff is rapidly inflated to 30 mm H g a b o v e 2. M u rm u r / sw ish in g so u n d s - E ld erly
th e p o in t a t w h ich ra d ia l p u lse d isa p p ea rs. 3. M o re in te n se , c risp e r so u n d s - A th e ro sc le ro sis
Deflation ra te sh o u ld b e 2-3 mm/second. - D iab e tics
4. D istin c t a b ru p t m u ffle f so u n d s
- T w o re a d in g s sh o u ld b e a t le a st 1 m in u te - M o k en b erg 's a r terio sc lero sis® ,
Indicates diastolic BP in children, in
ap art. • R e ta tiv e ly s m a ll s iz e o f c u f f (b la d d e r)
adults after exercise, hyperthyroidism , b
- BP should be measured in both arms® to eg sta n d a rd a rm c u ff u se d in thigh
aortic insufficiency.
rule out diseases of blood vessels like • T o o slo w in fla tio n rate 1/t too h ig h
5. N o sou n d
Takayasu's disease. d ia sto lic p re ssu re
B e s t c o r r e la te s w ith d i a s t o l i c B P in
Appropriate Cuff size False low values are found in
n o rm a l adults® .
• L arg e (w id e ) c u ff eg sta n d a rd arm cu ff
R a tio o f w id th o f c o m p re s sio n ca v ity o f c u ff It te n d s to g iv e v a lu e s fo r s y s t o lic p ressu re
used in fo rearm .
(b la d d er) to c irc u m fe re n ce o f ex tre m ity is o f t h a t a r e lo w e r th an true in tra a r te r ia l
• T o o fa s t D e fla tio n rate lea d s to too low
critica l im p o rta n c e . A c c o rd in g to A m erica n p ressu re £r d ia s t o lic v a lu e s t h a t a r e higher®.
sy sto lic p re ssu re.
H ea rt A sso cia tio n Diumal variation
I D ium al variation o f 5-10 mm Hg in systolic Auscultatory gap
B la d d e r w id th s h o u ld B la d d e r le n g th
BP (SBP)® w ith lo w e s t v a lu e s in e a rly It is th e in te rv a l o f p re ssu re w h ere
b e 40% o f circu m feren ce sh o u ld b e 80% o f
m o m in g a n d p e a k v a lu e s d u rin g a fte rn o o n is K o r o tk o ff's so u n d s in d ica tin g true sy sto lic
o r 1.2 tim e s o f d ia m e t e r arm
c o m m o n . N ig h t w o r k e r s h a v e re v e rse rh y th m . p re ssu re fa d e a w a y an d re a p p e a r a t a
o f extrem ity® _______________ circu m ference®

L Changes in Exercise and Posture


lo w e r p re ssu re. T h e im p ro p e r
in te rp re ta tio n o f th is g ap m a y 1/t erro rs
Length to width ratio is 2 :1 m a d e in f a ls e ly lo w re c o r d in g o f sy s to lic
Exercise increases both SBP and DBP®. On
If th e p e r so n 's lim b m e a su re m e n t is o n the B P . It u su a lly o c cu rs at h ig h p re ssu re s,
standing arterial BP decreases®
b o rd e rlin e o f tw o d iffe re n t c u ff siz e s the w h en th ere is sile n ce a t p re ssu re s g rea te r
(G a n o n g ). O n sta n d in g S B P d e cre a se s b u t D B P
ch a n c e o f e rro r is decreased if the larger o f two in c re a se s (A K Ja in ). D B P ch a n g e s w ith b o d y th an sy sto lic B P , e x c e p t th at th e p u lse can
cu ff sizes is used. still b e p alp ated .
p o stu re i.e. in c re a se s on sta n d in g , re m ain s
S o in o b e se a n d re co rd in g BP in th igh w id e r n o rm a l on sittin g an d d e cre a se s in lyin g
c u ff siz e p ro v id e m o re a c cu ra te re su lts. p o stu re (A K Ja in ).
Physiology: Cardiovascular System ■ 115

78. A i.e. Elastic Recoil of aorta [Ref: Ganong 23rd/e p. 507; 543- 47; 'Human Physiology ' by Muthayya 3rd /342; Cardiovascular
system at a glance 2nd /9 Cardiovascular Physiology by Mohrman (2006) /50]

Windkessel (Elastic reservoir) effect is the recoiling o f large elastic vessel w all o f aorta and large arteries during
d iastole ® that have been stretched during systole, which m aintains continuous forw ard flow , and arterial pressure during
diastole®.

79. B i.e. Systolic +l/3rd pulse pressure 80. A i.e. (SBP + 2DBP) 13 [Ref: Chaudhuri 5fe P- 244]

Mean arterial pressure (BP) = D iastolic BP + 113rd pulse pressure Q


= Diastolic BP +l/3rd (Systolic BP - Diastolic BP)

= DBP+ ^ - ^ = ^ + DBP - — = § K + 1 DBP= I(SB P + 2D B P )


3 3 3 3 3 3 3
Mean BP: It is the 'weighed' mean pressure of the blood. So like BP it depends on cardiac output & peripheral
resistance^. In a cardiac cycle, the systole lasts for 0.3sec and diastole for 0.5 sec. Therefore in a subject having a cardiac
cycle of 0.8 sec, the mean BP cannot be taken as (systolic BP + diastolic BP)/2
Difference between systolic and diastolic pressure is k/a Pulse Pressure.

81. C i.e. M ean circulatory filling pressure


82. D i.e. Arterial pressure taken at the point when heart stops beating [Ref: Guyton llth /e p. 239- 40; Gabriel C Ezeilo p.158]

Mean circulatory filling pressure (MCFP) is the arterial pressure taken a t the poin t when heart stops beating®. It is the
equilibrium pressure reached throughout the cardiovascular system (i.e. everywhere in circulation = both systemic and
pulmonary) when cardiac output (i.e. heart pumping) has stopped completely.Q

83. C i.e. V R = (MSFP-RAP)/RVR 84. D i.e. Peripheral resistance x cardiac output


85. A i.e. Auscultatory gap 86. B i.e. More than intravascular pressure
[Ref: Ganong 24/e P- 578-82, Chaudhuri 5/e P- 210; Bijlani 1/e P 189]
/ \
- Venous return (VR) = (MSFP - RAP)/RVR; whereas blood pressure (BP) = cardiac output x peripheral resistances.
- BP appears spuriously (falsely) low in auscultatory gap® and falsely high in obesity, calcified thick vessels and small
cuff.
Direct intraarterial method measures accurate BP. Whereas indirect sphygmomanometer method gives falsely high
readings.

87. B i.e. Falsely low values at high pressure in pulse tracing [Ref Ganong 22/e P- 588- 589]

Bernoulli's principle: In a tube or a blood vessel the total


energy (kinetic energy offlow+ potential energy) is constant
When a vessel is narrowed (e.g. in adrenalin administration),
the velocity of flow in the narrowed portion increases & so
the kinetic energy (1/2 mv2). As the total energy is constant
(Bernoulli's principle), potential energy decreases in
narrowed segment.
Therefore when the velocity of flow is increased the lateral When fluid flows through the narrow portion of the tube, the
pressure distending the wall at the constriction is decreased and kinetic energy of flow increases as the velocity increases, and the
potential energy is reduced. Consequently, the measured
narrowing tends to maintain itself.
pressure (P) is lower than it would have been at that point if the
Pressure transducer measures arterial pressure directly, so it tube had been narrowed. The dashed line indicates what the
gives falsely low value after adrenaline injection (at high pressure drop due to friction forces would have been if the tube
pressure) had been of uniform diameter.

88. A i.e. The lying down position

For two values to be comparable (to remove any bias) we must remove as many confounding variables as possible.
The confounding variable in this question is gravity. Gravity is present on earth but is not present in space.
The best position in which the effect of gravity is nullified is lying down.
116 ■ A Complete Review of Short Subjects

89. C i.e. KPa [Ref: Taber's Medical Dictionary 12216]

Pressure in SI unit is measured in pascal [Pa]. Blood pressure in SI unit is measured in K Pa Q

90. B i.e. Reflex bradycardia [Ref: Ganong 24/e P-588-93; Wright 13/e p. 124-25]

94. B i.e. Slight increase in H.R, normal BP [Ref: Harrison 16/e P-616, Bailey & love 23/e P275]

Blood loss of upto 20% of blood volume are normally tolerated by redistribution of blood flow, mediated by
vasospasmQ.

500ml blood accounts for about 10% of total blood volume (5 litres)
Blood loss <15% 15-30%Q 30-40% >40%
BP No change No change Reduced Markedly reduced
Heart rate Slight Increase 100-120 (slight) > 120 , weak > 120 , thready
Respiratory rate Normal Increased >20 20-30

Cardiovascular Regulatory Mechanism


95. A i.e. Bronchodilation [Ref: Ganong 22/e p.599-600,446, Guyton & Hall 11/ep. 202,322, Harrison 16/e p. 1366,1509, 213 ,
www.cvphysiology.com]

Endothelin 1 is broncho constrictor (not bronchodilator) & vaso constrictor^ (so -lGFR)Q. It is inotropic and stimulates
ANP and aldosterone secretion.

96. A i.e. Endothelium [Ref: Ganong 22/e P-598] 97. D i.e. Endothelin

Nitric oxide is synthesized from arginine by the enzyme NO synthetase (NOS) in endothelial cellsQ. It causes
vasodilation.

Synthesis of NO/Endothelium Derived Relaxing Factor

C
GTP
ACh
B rad yk in in > C a 2+
CGM P
S h e a r stress 4
N O S (in e n d o th e liu m ) k S ''
L -A r g in in e + 0 2 + N A D P H — j ------ ► C itru lin e + NO + N A D P •S o lu b le guanyl c y c la s e - S m o o th m u scle r e la x a tio n a
(v a s o d ila tio n )
Thiol, Tetrahydrobioprotein
FAD, FM N
Physiology: Cardiovascular System ■ 117

Factors Affecting the Caliber of Arteriole Precapillary Sphincter C aliber


Precapillary sphincters are layer of smooth muscle fibres present
Constriction Dilatation
just at the point where a capillary originates from a metarteriole.
There is no in nervation on p recap illary sphin cter f i
- Decreased local - Increased K+,
Their dilatation or constriction (to control the blood flow) depends
temperature Local adenosine.
on lo c a l horm on es fi, circulating substance, end product of
- Autoregulation factors - Increased CO 2
metabolism, hydrogen ions etc.
- Decreased O 2
Catecholamines causes sphincter constriction.
- Decreased local
pH
- Decreased local Effects of Prostaglandins
temperature
Blood Platelets Bronchi Kidney Uterus
- Endothelin-IQ Endothelin - NOQ
vessels
- Platelet products - ProstacyclinQ
PGE2 Vasodilation Variable Dilatation Vasodila­ Contraction
serotonin - KininsQeg
effect tation Softening
- Thromboxane BradykininQ
Renin of cervix
a 2q
release
- Epinephrin - Epinephrin in
PFFjd Vasodilation Constriction Contraction
(except in sk e le ta l m uscle Softening
skeletal muscle Circulating & liver® of cervix
& liver) hormones - Substance P® PGh Vasodilation Antiaggre­ Dilatation Vasodilation
- Norepinephrin - Histamine® gatory Renin
- AVP - ANP release
- Angiotensin II - VIP TXAz Vasocons­ Aggregation Constriction Vasocons­ “
- Neuropeptide triction triction
Y

98. B i.e. Angiotensin 2

Angiotensin converting enzyme/ACE (not angiotensin II) is synthesized in endothelium cells o f lung capillaries
(mainly)Q.

Endothelial Cell Function/Activity (By production of)

Anticoagulant- Antithrombotic Vasoconstriction Cell growth stimulation


- Prostacyclin (PGI2), HeparinQ, ACEQ & Endothelin^ PDGF, CSF, FGF
- Protein S, Thrombomodulin, NO
Vasodilatation
Cell growth inhibitors
Prothrombotic Activity
Prostacycline, NOQ Heparin, TGFp
vW factor & Tissue factor
Inflam m ation & Immunity
IL1 & 6 , HLA, VCAM-1, ICAM, E & P
Selections

Circulation Through Special Regions

99. B i.e. -5 to 0 mmHg

In most tissues, interstitial pressure (i.e. interstitial fluid hydrostatic pressure) is normally sub atmospheric (or
negative) ranging from-5 mm Hg to-1 mm HgQ.

100. B i.e. Increase in hydrostatic pressure of capillaries [Ref: Ganong 22 je p. 592; Guyton & Hall 11/e p. 184-85]

High capillary hydrostatic pressure & interstitial fluid colloid osmotic pressure tend to cause osmosis (filteration) of
fluid outward through capillary membrane^.
118 ■ A Complete Review of Short Subjects

101. A i.e. Hypercarbia 102. A i.e. Blood pressure » C i.e. Potassium ions
[Ref: Ganong 22/e p.611- 19, 597; Lee's anesthesia 12/e p-441; Lange, anesthesia 4/e p- 615]

Cerebral blood flow is insensitive to hormones, sym pathetic nerve activity and virtually remain constant despite
changes in arterial blood pressure Q.
Cerebral circulation shares many features of coronary circulation. Both heart & bain have a high metabolic rate, extract
& use large amount of O 2 & have limited ability to use anaerobic glycolysis for metbolism. Their vasculature have a
limited ability to constrict in response to sympathetic nerve stimulation but an excellent ability to auto regulate blood
flo w a t arterial blood pressure from 50-60 mmHg to 150-160 mm Hg®.
Potassium ion (K+) & lactate cause vasodilation in skeletal muscle, whereas adenoside may cause vasodilation in cardiac
muscle but not in skeletal muscle.
Since ions (H*, K+, HCO 3'), proteins, polypeptides and protein bound form s do not cross blood brain barrier as easily as
water, CO2, O2 and lipid soluble substances, the ions have little effect on cerebral blood flow®. Cerebral blood flow,
like other areas of body, is highly related to metabolic rate of tissue. Reaction of CBF to CO2, H+ and K+Q (i.e., chemicals
released by increased brain activity) is a part of overall process of matching braines metabolic needs to its blood supply
of nutrients & O2.
Hypercarbia (TCO 2 concentration in arterial blood) is the m ost potent cerebral vasodilator Qbecause of ease of
permeability across BBB. CO 2 causes vasodilation by increasing the H+ ion concentration in cerebral vessels.
However, H+ ions in blood do not cross BBB easily
The 10-30% increase in blood flow in areas of brain excited by mental or visual activity and peripheral nerve
stimulation, may be related to these three substances released from active nerve cells. Cerebral vasculature also dilates
when the O2 content o f arterial blood is reduced but the vasodilatory effect is much less pow erful than o f elevated
co2«.
103. C i.e. Increase in urine output [Ref: Shwartz 8/e P-132,133 CSDT Pg 217, Harrison 16/e P-1600-1617; Oxford Surgery P-120]

Urine output is quantitative and relatively reliable indicator o f organ perfusion Q

The most reliable clinical guide in assessing hypovolumic shock is skin perfusion, (skin on the extremeties becomes pale, cool and
moist).
As a general principle, measurements of blood pressure and pulse are less reliable than change in urine output in assessing the
severity of shock.
B.P : Hypotension is not a reliable early sign of hypovolumea. In healthy patient blood volume must decrease by 30-40%
before hypotension occur.
CVP: CVP measurement helps in distinguishing b/w Cardiogenesis shock and hypovolumic shock.
"A Central venous catheter can give information about the relationship b/w intravascular volume & right ventricular function but
should not be used to assess either factor independently."
CVP is not a good indicator in septic shock because patient can develop pulmonary edema when it is in the normal range.

104. B i.e. Contains 25% of blood 105. A i.e. Contains 5% of blood volume
106. B i.e. Contains larger quantities of blood than veins [Ref: Ganong 23/e P-537-543]

Vessel Percentage of Cross- sectional area Maximum blood is contained in the veins (venous
blood (cm)2 system)Q, while capillaries contain only 5% of blood
Aorta 2% 4.5 volume.
Artery 8% 20 Capillaries
Arteriole 1% 400 Maximum cross - sectional area Q(1000 times of
Capillary 5%Q 4500 (maximumP aorta)
Venule 54% (maximum)® 18 Minimum velocity in circulation is seen in capillaries
Vein (Inverse relationship with cross- sectional area)
Vena cava - Are lined by single layer o f endothelial cells fi

107. B i.e. Have role in thermoregulation


[R ef: A n in tro d u ction to c a rd io v a scu lar p h y siolog y - Levick, B u tterw o rth , H ein em a n n 1995; ‘C ard io v a scu la r P h y sio lo g y in E x ercise & Sport' by Bell
(2 0 0 8 ) / 60, 61; C ase B a sed M ed ica l P h y siolog y ' by B ell (20 0 5 ) / 115, N eu roim m u n o lo g y o f the skin (20 0 8 ) / 2 4]

Cutaneous (anterior- venous anastomosis) shunt vessels play an important role in temperature thermoregulation Q.
Physiology: Cardiovascular System ■ 119

108. A i.e. Large veins [Ref: Ganong 22/e P- 584]

The velocity of blood is inversely proportional


mal to the total cross - sectional area at that
tb point.
i
1
Order of Velocity of Blood______________ Order of Cross - Sectional Area:
Velocity is inversely proportional to total cross section
area. So, velocity is highest in aorta, declines steadily in Capillaries (max)® > Arteriole > Artery > Vena cava >
smaller vessels & lowest in capillaries. Velocity increases Aorta
again as blood enters veins & is relatively high in vena
cava but not so high as in aorta. ★ Mean velocity of blood in the aorta is = 40 cm /sec® .
Aorta > Vena cava® > Artery > Arteriole > Capillary Average normal arm to tongue circulation time is 15
(min)® seconds.

109. B i.e. Femoral artery and femoral vein [Ref: Ganong 22/e p. 586 - 9 5 , 21/e p. 589]


Blood pressure in decreasing order is: A orta > arteries > arterioles > capillaries > venule > vein > vena cava®.
Blood pressure in aorta is 120/80 mmHg. It falls very slightly in the large & medium sized arteries becauswe their
$
resistance to flow is small. But BP falls rapidly in small arteries & arterioles, which are the main site o f peripheral
resistance® against which the heart pumps. So these are referred to as resistance vessels. The mean pressure at the end
of arteriole is 30-38 mmHg and pulse pressure 5 mmHg. Capillary pressure (BP) at arteriolar end is 32 mmHg with pulse
pressure 5 mmHg and at venous end is 15 mmHg with pulse pressure 0 mmHg. In capillary, blood velocity is 0.07 cm/s
and transit time 1 - 2 seconds. The pressure in venules is 12-18mm Hg which falls steadily in larger veins to about 5.5
mmHg in great veins outside the thorax. The pressure in great veins at their entrance into right atrium in thorax
(central venous pressure) averages 4.6 mm Hg.
• Both arterial pressure and peripheral venous pressure are affected by gravity, in a way that it is increased 0.77 mm Hg
fo r each centimeter® below the right atrium and decreased by same amount for each centimeter above the right atrium
• W indkessel (elastic reservoir) effect is the recoiling of elastic vessel wall of aorta & large arteries during diastole that
have been stretched during systole, which maintains continuous forward flow.

110. B i.e. Capillaries venules and arterioles 111. B i.e. Resistance vessel 112. A i.e. V eins 113. B i.e. Capillary
114. B, C, & E i.e. Transfused blood is mainly distributed in arteries, Hematocrit markedly change peripheral resistance, &
Non pulsatile flow is recommended [Ref: Ganong 23rd/e p. 542- 47; Guyton 12/e p. 157]

• A large amount of blood can be added to veins and therefore are called capacitance vessels. The sm all arteries and
arterioles® are referred to as resistance vessels because they are the principal site of peripheral resistance. So
increased radius o f resistance vessel increases capillary blood flow®
• When extra blood is transfused, < 1% of it is distributed in arterial system (High pressure system) and all the rest is
found in systemic veins, pulmonary circulation, & heart chamber other than left ventricle (Low pressure system)
• Resistance to blood flow is determined by vascular hindrance (radius of blood vessels) and viscosity of blood. The viscosity
depends mostly on hematocrit (i.e.% volume of blood occupied by RBC) In large vessels, hem atocrit increase causes
appreciable increase in viscosity® and so increased mean blood pressure®. H ow ever, in vessels smaller than 100pm
diameter (i.e. arteriole, capillaries & venules) - the viscosity change per unit change in hem atocrit is much less than it
is in large bore vessels®, d/t difference in nature of flow through smaller vessels. So the net change in viscosity
(Poisenill- Hagen formula) per unit change in hematocrit is considerable smaller in vivo than in vitro. This is why
hem atocrit changes have relatively little effect on peripheral resistance®. Except when the changes are large.
• Mean velocity of blood flow in proximal aorta is 40 cm/s (ranges from 120 cm/s during systole to negative at the time
of transient back flow before the aortic valve closes in diastole) the flow is phasic. Pulsatile flo w maintains optim al
tissue function (by gene transcription)®. If an organ is perfused by a non pulsatile flow, inflammatory markers are
produced, there is gradual rise in vascular resistance & ultimately tissue perfusion fails.
120 ■ A Complete Review of Short Subjects

O ar Functional Classification of Vessel Wall

Vessel Type Include Feature


Elastic Arteries Aorta, Pulmonary artery & Elastic recoil maintains continuous blood
(Windkessel Vessels)Q major branches flow & diastolic arterial pressureQ
Conduct (muscular) Arteries Main arteries eg cerebral, Thicker muscle in tunica media
coronary, popliteal, radial
Resistance vessels Sm allest terminal arteries & - Single layer of muscle in tunica media
arteriolesQ - Regulate local blood flow to match local
demands
Arteriovenous anstomosis Shunt vesselsQ (dia 20-135) mm - Connect arteriole to venule by passing
capillaries
- Present in skin & nasal mucosa
- Regulate temperatureQ
Exchange vessels CapillariesQ - Largest cross section areaQ
- Provide 0 2 and nutrition to tissuesQ
Capacitance (Compliance) Veins - Store large volumes of bloodQ
vessels
Capacitance V. VeinsQ Store large amount of blood
Microcirculation Arteriole —> Metarteriole —> Precapillary sphincter —» Capillary -» VenuleQ

115. B i.e. Arterioles [Ref: Ganong 24/e p. 575; Chaudhuri 5/e P-198] 116. A i.e. 5 times

Distribution of blood flow is mainly regulated by arteriolesQ, by alteration in their diameter. They can increase or
decrease, the peripheral resistance to blood flow. Also called precapillaiy resistance vessel Q.

Arterioles are major site of resistance to blood flow, often termed as seat o f peripheral vascular resistance,® Small change
in their caliber cause large change in total peripheral resistance & affect distribution of blood flow. Poiseuille-Hagen
Formula.
8nL R = Resistance L= Length of the tube
R= = viscosity
Ttr r = radius of vessel T|

8nL
K= ■constant
F= Pa ~ P. n
R F = Flow, P a - Pb = Pressure difference between two ends of tube

R °c -------- \— and Flow (F) - radius (r)4

So, on increasing radius 50% (i.e. to 1.5 r) would increases flow to (1.5)4 = 5 timesQ

117. A i.e. Parallel to the long axis of blood vessel; C i.e. Increases with increase in axial velocity of fluid; D i.e. Poiseuille-
Hagen equation correlate flow with viscosity; E i.e. Depend upon viscosity
[Ref: Ganong 24/e p. 573-75]

Flowing blood creates a shear stress (y) on the endothelium that is parallel to the long axis o f vessel®. y is
proportional to viscosity (Tf) times the shear rateQ (dy/dr, ie the rate at which the axial velocity increases from the
vessel wall toward the lumen). |Y= Tj (dy/dr)
Changes in shear stress, cyclic strain & stretch activate endothelial genes such as endothelin 1, growth factors (PDGF a/p,
TGF P) and integrins etc.
Poiseuille- Hagen formula correlates flow (F) with viscosity (T|)Q, pressure difference b/w two ends of tube (P a -P b ),
radius (r) & length (1) of tube. F= (Pa-P b) xOi/Slxd/TpxlrVL) .
Physiology: Cardiovascular System ■ 121

118. D i.e. Effective velocity in small vessels is less [Ref: Ganong 23/e p 539-42; Berne & Levy 6/e p 331-34; Gabriel p 134-35]
119. D i.e. Increase in diameter of blood vessels

- Probability of turbulence increases with Reynolds num ber > 3000, increase in velocity (above critical level), density of
blood and diameter of vessel or with decrease in viscosity.
- Average velocity of flow is inversely proportional to the total cross sectional area of the vessel. Therefore the average
velocity of flow is high in aorta declines steadily in sm aller vessels and is lowest in capillaries. So the cause of laminar
flow in small vessels mainly is large area of cross section and less effective velocity^.

120. A i.e. Local hormones [Ref: Ganong 22/e P- 577, 603, Guyton 10/e P/162]

Precapillary sphincter relaxation is mediated by local hormonesQ, circulating substances, end product of metabolisms &
H 2 ions as there is no neural innervations.

121. A i.e. 250 ml/min 122. D i.e. 800 123. A i.e. 55ml/100 gm/min
124. A i.e. Liver > Kidney > Brain > Heart

Overall total blood flow in various organs is liver (1500 ml/min) > kidney > skeletal muscle (840)Q > brain > skin ☆
(460ml/min) > heart/ myocardium/coronary (250mL/min)Q.
Coronary flow is maximum during diastoleQ. Adenosine increases it by coronary vasodilatation esp. during
ischemia.
Brain blood supply is 750 ml/min or 54 ml/100gm/minQ.

Overall Blood Flow and Oxygen Consumption by Various Organs

Effects of Exercise on Cardiac Output


Over all (ml/min) Per unit mass (ml j 100 gm/min) Cardiac output Q uiet Exercise Effect
Blood Flow O 2 Consumption Blood flow O 2 consumption standing
Liver > kidney Liver > skeletal Cardiac output 5900 24,000 TP
Kidney HeartQ > Kidney
> skeletal ms > ms> Brain > ml/min
>Heart > > Brain
Brain > skin > Heart liver > Blood flow to:
HeartQ Brain Active skeletal 650 20,850 TT
muscle
Heart 250 1000 TP
Resistance A-V O 2 difference Brain 750 Unchanged
750
A bso lu te Per/kg (ml/L) Skin 500 500 Unchanged
H e a rt: Skeletal m s : Max Heart : MaxQ Inactive skeletal 650 300 I
M ax Kidney : Min K id ney: Min muscle
L iv e r : M in Kidney, Liver, 3100 600 w
GIT ect.
Resting blood flow and O2 consumption of various organs in a 63-kg adult man with a mean arterial blood pressure of 90
mm Hg and an O 2 consumption of 250 mL/min
Ateriovenous Oxygen Resistance
Mass Blood Flow Oxygen Consumption (R units)* Percentage of Total
Region
(kg) mL/min mL/100 D ifference mL/min mL./lOO Absolute per kg Cardiac Oxygen
g/min mL/L g/min Output Consumption
Liver 2.6 1500 57.7 34 51 2.0 3.6 9.4 27.8 20.4
Kidneys 0.3 1260 420.0 14 18 6.0 4.3 1.3 23.3 7.2
Brain 1.4 750 54.0Q 62 46 3.3 7.2 10.1 13.9 18.4
Skin 3.6 462 12.8 25 12 0.3 11.7 42.1 8.6 4.8
Skeletal muscle 31.0 840Q 2.7 60 50 0.2 6.4 198.4 15.6 20.0
Heart muscle 0.3 250 84.0 114 29 9.7 21.4 6.4 4.7 11.6
Rest of body 23.8 336 1.4 129 44 0.2 16.1 383.2 6.2 17.6
Whole body 63.0 5400 8.6 46 250 0.4 1.0 63.0 100.0 100.0
122 ■ A Complete Review of Short Subjects

125. A i.e. Skin Autoregulation of GFR & flenal Blood Flow


126. C i.e. O pening of Ca2+ channels
[Ref: Guyton 10/e P-293) M yogenic autoregulation Tubulo glom erular
Ability o f individual bloods vessels to resist stretching during feed back
The capacity of tissues to regulate
increased arterial pressure.______________________________
their own blood flow is referred to as
Stretching of vascular wall This feed back
autoregulation. This capacity is well
( when arterial pressure T es) mechanism links
developed in kidney,® also seen in
mesentry, skeletal muscle ®, i changes in sodium
brain®, liver®, & myocardium. But TM ovement of Ca2+ from ECF into cell® chloride conc at the
no autoregulation is seen in skin®. 1 macula densa with
Contraction of smooth muscle the control of renal
127. A i.e. PGI2 & B i.e. PGE2
I arteriolar resistance.
PGE2 and PGI2 cause renal - Prevent overdistension of vessel This ensures a
vasodilation, increasing renal - Tes vascular resistance constant delivery of
blood flow and release renin®. J- NaCl to the distal
Prevent T in renal blood flow & GFR tubule.

128. B i.e. A utoregulatory [Ref: Sembulingum 3/e P- 516-17;Ganong 22/e P-620- 21 ]


129. A i.e. Coronary blood flow is directly related to perfusion pressure & inversely related to resistance

Factors regulating Coronary Blood Flow

Auto regulation (Chem ical Regulation) N eural Regulation


- Need for 0 2 (i.e. hypoxia) is the most - Nervous factors: sympathetic stimulation increases and parasympathetic
important factor maintaining blood flow stimulation decreases coronary blood flow.
(by causing vasodilation) - Coronary perfusion pressure is the balance between mean arterial
- M etabolites eg adenosine, K+, H*, CO 2 pressure & resistance offered to blood flow in heart. The mean arterial
causes vasodilation & increase blood flow pressure particularly in aorta plays an important role in maintaining
circulation where as resistence decrease it®.

130. B i.e. Hypoxia causes Vasoconstriction, C i.e. Blood volume in lungs is 450 ml; E i.e. It has low resistance
131. D i.e. Distended pulmonary veins in the lower lobes 132. B i.e. Pulmonary vasoconstriction in hypoxia
133. A i.e. Zone 1 134. D i.e. Most blood is in pulmonary capillaries
135. A i.e. Increase in the number of open capillaries 136. B i.e. Resistance low, Capillary pressure low
[R ef: B ern e & L ev y 6 /e p 4 2 1 -2 3 ; G u y ton 1 1 /e 4 8 6 -8 7 ; L an ge: P u lm o n a ry p h y siolog y 6 /e p 9 5 -9 7 ; fo h n , W est 8 /e p 3 6 -4 1 ; G a n on g 2 3 /e 6 0 2 -0 4 ]

Lung contains 450 ml (9% of total ) blood volume of which only 70 ml is in pulmonary capillaries and rest is
equally divided b/w arteries and veins.
Pulmonary circulation is low pressure low resistance system. Pulmonary vascular resistance is only one tenth®.
Pulmonary microcirculation is characterized by rem arkably low vascular resistance and low pressure and is very
pulsatile®. Hypoxia causes vasoconstriction in pulmonary small arteries & arterioles®.
Perfusion & ventilation both decrease from base to apex of lung®. Both V & P are more in lower lobes but V/P ratio
is highest in apex®. V/P ratio is 0.8 for whole lung at rest.
Zone 1 of no blood flow is physiological (alveolar) dead space®.
During heavy exercise the cardiac output (CO) increases upto three folds while pulmonary arterial pressure rises
very little because pulmonary circulation can accommodate significant amount of blood by phenomenon of
recruitment (i.e. increasing the number of open capillaries conducting blood) and distension (i.e. increase in caliber of
already open vessel).

137. B i.e. Contribute to gaseous exchange [R ef: B ern e b L ev y 6 /e p 4 2 2 -2 3 ; G u yton 11/e p. 4 8 3 ; P h o d es b B ell M ed ic a l p h y s io lo g y 3 /e p -3 7 1 ;
M ille r : L u n g fu n c tio n 6 /e p 2 8]______________________________________________________________________________________________________________________

Bronchial circulation does not take p art in gaseous exhange®. Gaseous exchange takes place in pulmonary circulation. j
138. B i.e. Um bilical Vein
---------------------------------------------------------------------------------------------------------------------------------------------
- Um blical vein and pulm onary veins carry oxygenated blood®. W hereas um bilical and pulm onary arteries ca n y
deoxygenated blood®.
Physiology: Cardiovascular System ■ 123

- Deoxygenated blood (with 62% O 2 saturation) leave via um bilical arteries to be oxygenated at placenta whereas,
umbilical veins carry oxygenated blood from the placenta with HbC>2 saturation of ~ 80% (as compared to 98%
saturation in adults arterial circulation).
- Right ventricle pump deoxygenated blood through pulmonary artery into pulmonary capillaries where it is
oxygenated. And then oxygenated blood is carried through pulmonary veins to left atrium.

139. A i.e. Portal vein [Ref: BDC- 4/e vol II p 253]

| Blood supply of liver through: P ortal vein is 80%® and Hepatic artery is 20%

140. C i.e. Sinusoids [Ref: Davidson 17je P 498]

Wedged Hepatic Venous Pressure

It is the pressure recorded by a catheter wedged in a hepatic vein. It reflects the contribution o f hepatic sinusoids to portal
venous pressure ,® It is elevated in sinusoidal & presinusoidal portal hypertension,

Portal venous pressure = Wedged hepatic venous pressure - free hepatic venous pressure.

141. A i.e. Tight endothelial function 142. B i.e. Foot process of astrocytes

Blood brain barrier (BBB) is formed by tight junction of endothelial cells and foot process of astrocytesQ, limiting
filtration of protein across capillaries.

143. A i.e. Correlates with heart rate 144. B i.e. Has constant relation to externalcardiac work
145. E i.e. Directly proportional to mean arterial pressure
[Ref: Guyton 11/e 235-37; 249-53; Faw zy G.Cardiac anesthesia : Principles and clinical practice p. 25; C ardiovascular physiology - H eller (lange) 5/ed
pg-62-64; Internal m edicine 5/ed pg- 192-193; Textbok o f physiology G abriel (oxford) p-113; Ganong 23rd/e p. 519]

- Myocardial O 2 demand /consumption is primarily determined by heart rate, contractility of myocardium (or
ventricles) (i.e. inotropy), intramyocardial wall tension (which is directly proportional to after load or intra cavitatory
pressure and preload or ventricular radius and inversely proportional to myocardial muscle mass or wall thickness)
and ventricular work per beat (i.e. stroke volume X mean arterial pressure). For left ventricle mean arterial pressure of
aorta and for right ventricle MAP in pulmonary artery is used.
- M yocardial O2 demand is not negligible a t rest®. It has a constant relation (not directly proportional) to external
cardiac work (during ejection phase of cardiac cycle). Myocardial O 2 demand is directly proportional to heart rate but
has no relation to duration o f systole®

146. A, B, C, D > E i.e. Preload, Afterload, Intramyocardial tension, Myocardial muscle mass > Blood Hb concentration

Anemia, anxiety, arterio-venous fistula (AVF), beriberi, hyperthyroidism, paget's disease, pregnancy, and pulmonary
disease commonly cause high cardiac output due to reduced total peripheral resistane.
Anemia decreases total peripheral resistance by reducing viscosity of blood (d/t decreased RBC concentation) and
causing local vasodilation (d/t diminished delivery of O2 to tissue).
Long term increased work load (but not so much excess that damages the heart) causes the heart muscle to increase
in mass and contractile strength in the same way that heavy exercise causes skeletal muscle to hypertrophy.

147. D i.e Coronary circulation [Ref: Ganong 22/e P-634 ]

During exercise blood is shunted from regions that do not require im m ediate support to areas with increased demands,
(e.g. skeletal muscles, heart)®.

148. A i.e. Regular 3 minute exercise

Regular 3 minute exercise have been well documented to bring about an increase in the aerobic capacity

149. C i.e. Epinephrine from medulla [Ref: Harrison 16/e P- 1377; Schwartz 8/e P-324; Ganong 22/e P- 667]
----------------------------------------------------------------------------------------------------------------------------------------
In cardiac transplantation, the recipient heart remains denervated, so cardiac function differs from that of the
innervated heart during both rest & exercise______________________________________________________________
124 ■ A Complete Review of Short Subjects

150. A i.e. Venoconstriction with decreased blood flow [Ref: Ganong 22/e P-634]
— ■■■ . . . . . .
During vigrous exercise, constriction of vessels in splanchnic organ, and decreased b lo o d ' storage' in liver & other
portion of splanchnic bed occurs^, which may increase the volume of actively circulating blood, perfusion the muscles
by as much as 30%. Contraction of the capacitance vessels in the viscera can pump a litre of blood into the arterial
circulation, in less than a minute.

151. B i.e. Brain [Ref Ganong 22/e P-599]


The direct vasodilator action of CO 2 is most pronounced, in the skin and brainQ

The metabolic changes that produce vasodilatation, in most tissue, is decrease in O 2 & pH
T CO 2 is associated with cutaneous & cerebral vosodilatation Q , but vasoconstriction elsewhere & usually a slow rise
in BP.

152. D i.e. There is fall in systemic capillary hydrostatic pressure

Edema normally occurs due to increased hydrostatic pressure that pushes fluid out or due to reduced oncotic pressure
that fails to pull the fluid. Acute heart failure is usually predominantly systolic and sudden reduction in cardiac output
often result in systemic hypotension without peripheral edema. Acute heart failure is associated with a fall in hydrostatic
pressure and therefore explains lack of peripheral edema.

153. D i.e. Aortic stenosis [Ref: Ganong 24/e p. 542; Guyton 9/e P114, Braunwal 6/e P 479, 480,1675]
™ ^

- In pressure volume curve, during diastole in period of ventricular filling (from d-»a) pressure increases. Pressure
rises sharply to reach diastolic BP during isometric contraction (form a-»b)Q and (from b—>c) during period of
ejection. At C, the aortic valves close and pressure falls during isovolunetric relaxation (from c-»d).
^ In AS, pressure-volume (P-V) curve is shifted to the left, whereas in M R and AR, PV curve shifts to right.

Pressure Volume Curve (P-V Curve)

P-V curve denotes the pumping mechanics of the left ventricle (since left ventricle is the main chamber, all studies are
referred to the left ventricle). P-V Curve is shifted to left (i.e. more pressure would be created for the same volume).
When contractility of the chamber increases
- Increased sympathetic activity
- Concentric hypertrophy of the chamber
- Decreased compliance of the chamber

Left ventricular volume (ml)

155. D i.e. 60%

EDV=End-Diastolic volume (point a)=130 ml SV 80


So, Ejection Fraction = 0.6 = 60%
ESV= End-Systolic volume (point d)= 50 ml EDV 130
SV = Stroke volume= EDV=ESV= 130-50=80 ml
Physiology: Cardiovascular System ■ 125

Circulating Body Fluid

156. C i.e. Bone marrow

Bone Marrow
• It is one of the largest and most active organ approaching weight & size of liver. Active cellular marrow is called red
marrow, inactive fatty marrow is K/a yellow marrow
• Bone marrow is source o f all blood cells in adultsQ. However, liver and yolk sac produce blood cells in fetal &
neonatal life
Precursor/Lineage Form ★ Both B & T lymphocyte origin in bone marrow
Erythroid RBC in adults (yolk sac & liver in fetus). But they
Megakaryocyte Platelett mature in Bone marrow (B=B cells) and Thymus
Myeloid Neutrophil, Monocyte, (T=Thymus) respectively
Eosinophil, Basophil
Lymphoid Lymphocytes^ ( B & T cells)Q
• Normally 75% marrow cells belong to white blood producing myeloid series & only 25% are maturing RBCs. Even
though in circulation RBCs are 500 times more than WBCs d/t the fact that average life span of WBCs is short and
RBC is long.
• Cellularity & RBC production after 15 years onward is Vertebrae > sternum > rib > femur shaft > tibia shaft

157. B i.e. Lymph node 158. A i.e. Yolk sac 159. A i.e. Flat bones
[Ref: Guyton l i f e p. 420; Ganong 22/e p. 515-16; Chaudhary 5/ep-20-21]

The marrow of the long bones except fo r the _______________ Erythropoesis_____________


proximal humerus and tibia (Guyton) / upper It starts in 3rd week of intra uterine life. Upto middle
humerus & fem ur (Ganong) becomes inactive trimester of gestation blood is formed out of
(fatty) and produces no more red blood cells medullary cavity (extram edullary ery th ro p oesis).
after about age 20 years. B eyon d this ag e m o st
RBC are p rodu ced in m arrow o f m em branous
f l a t bon es such a s vertebrae, sternum, ribs &
ilia®.

Before birth After birth


Age Site Age Site
3weeks - 3 months M esoderm o f y o lk Birth -5 years R ed bon e m a rrow <2
(Intra vascular sa c 0 within the (myeloid ★ All marrow of long & flat bone is red at this stage.
phase) blood vessel. phase)
3-5 months L iver (main), spleen 5-20 years R ed bon e m a rrow Q of all flat & long bones
(Hepatic phase) & lym ph n od es0 (±) ★ There is gradual decrease of red marrow in long
bones and so gradually increasing production from
flat bones
5-9 months R ed b on e m a rrow 0 >20 years - Red marrow of flat membranous bones such as
(Myeloid phase) ★ All marrow of vertebrae, sternum, ribs & iliamSi.
long & flat - Very little production in upper humerus, tibia &
bones is red at femur
this stage

160. C i.e. Spleen [Ref: Lippincott's 3/e P-280, Chaudhuri 5/e P-33]

Breakdown of heme to bilirubin occurs in macrophages of the reticuloendothelial system m ain ly in the sp leen Q also in
the liver and bone marrow.______________________________________________________________________________________
126 A Complete Review of Short Subjects

161. B i.e. Inhibit heme oxygenase [Ref: Care of newborn by meharban sing /189, Ghai 5/e P151 ]

I Tin & Zn porphyrins and mesoporphyrins inhibits the activity o f heme oxygenase Q and decrease formation of bilirubin

Best method for Estimation of Hb come in blood is - Cyanmethhemoglobin method®

162. B i.e. Hemopexin

Heme Binding Proteins - haptoglobin (1st) and hemopexin (2nd line of defense) inhibit heme loss from plasma by
binding heme (so preventing free heme in plasma).

163. A i.e. It is stored in ferritin [Ref Ganong 22je P- 477]


Iron

★ Iron is deposited in all cells of body specially in reticuloendothelial cells & liver hepatocytes.

164. C i.e. a262 165. B i.e. It is made up of 2 alpha and 2 gamma subunit
166. B i.e. HbF 167. C i.e. Zeta and epsilon chains [Ref: Ganong 24/e p. 559; Bijlani 3/e Pg 271, Lehninger's 4/e p 173]

- Embryonic hemoglobin contains epsilon & alpha (&Ck); or ☆ Hemoglobin type Tetramer (globin) chain
epsilon & zeta (& £); or zeta & gam m a ( £ ji ) chains®. Adult Hb A 012P2 = most commonQ
- Fetal hemoglobin (HbF) differs from adult hemoglobin (HbA) Adult Hb A2 0262 = 2 nd mcQ
in that it has gamma (7 ) chains in place of beta (0) chains. So Fetal Hb F CX272
HBF is 0272 , whereas HbA is Obffc tetramer^. Embryonic Hb (EHb)
- 2,3 BPG greatly reduces the affinity of Hb to O 2. SO the O 2 EHb Gower 1 &2
content (at a given PO 2) for HbF is greater than that of adult EHb Gower 2 CX2E2
hemoglobin because HbF binds 2,3- BPG less avidlyG. EHb Portland C2Y2
- In young embryos additional £ and e chains form Gower 1
hemoglobin (£2 6 2 ) and Gower 2 hemoglobin (a 2 E2).

168. B i.e. Fetal Hb [Ref: Samson Wright 3/e p-198]

Alkali denaturation test (ADT) - done for fetal hemoglobin. HbF is a lk ali denaturation resistant®.

169. A i.e. 6000 per cu mm [Ref: Paul 2/ep 57-58]

Neubauer (Hemocytometer) Square Chamber

This counting grid carved in glass is 3mm x 3mm is size with 9 subdivision squares 1mm x 1 mm. For counting of

TLC (WBC-Number) RBC & Platelets


Four com er squares are usedQ. Big central square is used which is
Volume of 4 comer squares is 0.4 cu.mm (=4 x.01 cu mm) split in 25 square of 0.2 mm. Each of
these 25 squares are subdivided into
Number of WBCs = Cell count in 4 com er squares x Dilution Factor 16 small squares. (So=25 x 16=400
0.4 total)
25 + 35 + 40 + 20
W BCs: x20 = 6000 cells /dL°
0.4

170. A i.e. C (Eosinophils) [Ref: Guyton 11/ep 436, Ganong 24/ep 558]

Parasitic infestations (=worm infections) causes eosinophilia (i.e. increased count of eosinophils)Q.
Physiology Cardiovascular System ■ 127

Identification of RBC & WBCs In Lab

RBCs (Erythrocytes) WBCs (Leukocytes)


Most numerous (4.8-5.5 million/mL), biconcave disks Much less in number (5-10 thousand/mL) with -700:1
(doughnut with a hole) with no nucleus and small (8 RBC: WBC ratio. On the basis of cytoplasmic
gm dia) size. appearance it can be divided into 2 groups

Granular Leukocytes I Agranular Leukocytes


With relatively clear cytoplasm, this group
Feature Neutrophil Eosino (Acido) phil Basophil includes -
Size 10-12 nm 10-12 pm 8-10 pm - Lymphocytes 20-25 % of the WBCs, stains
Count Most (70%) 2-4% <1% dark and is round or slightly indented with
Cytoplasm Lightly stained Red-Orange large, Large blue- the cytoplasm appearing as a rim around the
(Granules) with very fine uniform granules black round nucleus. The round, uniform nucleus and
pale lilac, that do not block granules that small amount of cytoplasm surrounding it
granules nucleus block bilobed are the best identifying characteristics.
Nucleus M ultiple (2-5) Mostly 2 (but may nucleus^ - M onocytes comprise 3-8% of the WBCs, and
finely connected be 3) connected & largest 12-20 pm in diameter. The nucleus is
lobes they are more uniform, indented or kidney shaped, not rounded, and
rarely uniform in rounded nuclear surrounded by foamy cytoplasm. Their size
size lobesQ is the most easily identifiable characteristic.

’H
§ • .V
M <
c
• s> m •* - \. i
■- . »
% e

L y m p h o cyte M on ocyte Basop hil E osin oph il N eutrophil P latelets

171. C i.e. M ultiple myeloma [Ref Ganong 22/e P- 585]

In m ultiple myeloma, M protein increase in the blood, this may cause cryoglobulinemia which leas to increase in
viscosity of blood

172. A i.e. Low molecular weight and high blood concentration [Ref: Guyton 10/e P 4 6 ,189]

Osmotic pressure is determined by the number of particles in solution and not by the mass of the particle, that means 1 gm of a
heavy molecular weight protein will contain a lesser no. of particles than 1 gm of a light molecular weight protein, &
consequently will have a lesser osmotic pressure.

Plasma protein MW Concentration


Contribution to plasma
osmolality
Albumin 69,000 3.5-5 gm% 80%
Globulin 1,40,000 2.0 gm% 20%
Fibrinogen 4,00,000 0.3 gm% 0%
Of the there major types of plasma protein, albumin with minimum molecular weight contributes, max to plasma oncotic
pressure.

173. B i.e. Contractile protein [Ref: Guyton 10/e P 419]

Thrombosthenin - is a contractile protein found within platelets .<2 It helps the platelets to contract during clot
formation. Other contractile proteins found within platelets are- (i) Actin, (ii) Myosin
128 ■ A Complete Review of Short Subjects

174. A i.e. Bone marrow; C i.e Thymus

Lymphoid Organs

Primary /Central Secondary/ Peripheral


Bone macro wQ Bone marrow, Spleen, Lymph node ★ Bone marrow serve as both central
ThymusQ Payers patches, Waldeyer's ring & peripheral lymphoid organ,
(tonsils & adenoid)

175. D i.e It takes part in intrinsic enzymatic reaction [Ref: Ganong 22/e P- 521-5251 176. A i.e. T cells

Cytokines Lymphocyte

They are polypeptide ® in nature.


Induced their effect in three ways: T- lymphocyte B- lym phocyte
Autocrine, paracrine & endocrine I I
effects.® responsible for cellular immunity Responsible for
Mediates natural immunity humoral
I 1 I I
Affects leucocytic movement® T helper (TH) CD4 on the surface Memory Cytotoxic T cell immunity
Requlate lymphocytic growth I------------1------------ 1 T cell I
activation & differentiation. TH-1 TH-2 - Destroy
Activate inflammatory cells & I I transplanted &
stimulate hematopoesis. Concerned Interacts with B other foreign
with cellular cell in relation to cells.
immunity humeral immunity - CDs on the
surface

177. A i.e. Eukaryotes C i.e. Plants D i.e. Bacteriophage [Ref: Robbins 7/e P- 61; Harper 26/e P- 621 ]

Lysozyme is an enzyme found in neutrophils o f eukaryotes, plants; & in bacteriophage virus®. It hydrolyzes link between
N- acetyl muramic acid & N acetyl D glucosamine found in certain bacterial walls.

178. A i.e. Leucocytosis [Ref: Ganong 22/e P-516[

White Blood Cells

Granulocytes (Polymorphonuclear Non- granular lecocytes


Leukocyte, PMNs)
- Neutrophils (60-70%) Lymphocytes (25-30%)
- Eosinophils (1-4%) Mn: "BEN" Monocytes (2-8%)_____
- Basophils (0.2 - 1%) ♦ T is su e m acrophage are d e riv e d from m o n o cy tes;
an d m ast cells are re la ted to b a so p h ils.
G-CSF: Granulocyte colony stimulating factor
GM-CSF: Granulocyte- Monocyte colony stimulating factor.

Thus G -C SF or GM-CSF will stimulate the synthesis of either granulocytes or monocytes hence leucocytosis is seen

179. B i.e. IL6; C i.e. IL 1; D i.e. TN Fa [Ref: Ganong's 24/e p. 73; jawetz 26/e p. 130]

----------------------------------------------------------------------------------------!-------------------------------------------------------------------------------------K
Macrophage release tumor necrosis factor (TNF)a, interleukin (IL)l/6/8/10/12/23, GM CSF (granulocyte-macrophage
colony stimulating factor), GCSF, transforming growth factor P (TGF p), VEGFA and chemokines like IL8 (CXCL 8),
rantes (CCL5), CXCL 9/10/11.

180. B i.e. Myeloperoxidase; C i.e. Lysozyme; E i.e. Cathepsin G. [Ref: Chandrasoma Taylor 3/e P-409]

Neutrophils secrete myeloperoxidase, lysozyme and cathepsin GQ.


Physiology: Cardiovascular System * 129

181. A i.e. It is a large glanular lymphocyte; 6 i.e. Releases granzymes and perforins; E i.e. Can k ill virus infected and cancer
cells [Ref: Best & Taylor 13/e p. 375-76, 1226; Jawetz 26/e p. 125-26]

Natural K iller (NK) cells are large granular lymphocyte^ which k ill virus infected cells and cancer cells m ainly by cell
mediated immunity using perforin and granzymeQ. They do not depend on thymus or antibody for killing^ (although
play role in ADCC).

182. B i.e. Kalazar; C i.e. TB; D i.e. Brucellosis [Ref: Harrison 17/e p. 355; Harsh mohan 5/e p. 411]

Absolute monocyte count > 800/pl is called m onocytosis, and it is associated with
Tuberculosis (TB), Brucellosis, k a la z ar (visceral leishm aniasis )®, subacute bacterial endocarditis, Rockey mountain
spotted fever, & Malaria
M alignancies, leukemia, myeloprotiferative syndromes, hemolytic anemias, chronic idiopathic neutropenias , &
granulomatous diseases such as sarcoidosis, regional enteritis (IBD) and collagen vascular diseases.

183. B i.e. Neutrophil; C i.e. Macrophage; D i.e. Basophils; E i.e. B- Cells 184. A i.e. 6 Hours [Ref: Ganong 22/e P-516]
[Ref: Robbins 7/e 202, 522; Harrison 15/e P - 1815]
The average half life of neutrophils in the
Sources of Tum or Necrosis Factor (TNF) circulation is 6 Hours.®
_L
I
To maintain normal circulating blood
TNF-a TN F-3
I I level, production of 100 billion
- M onocytes /macrophage® neutrophils /day is necessary.
- T cells
- B asop h ils ®, Eosinophils - B - cells H alf life o f transfused p la telet is 4 days®
- Mast cell
- Natural killer (NK) cells
- Keratinocytes, Fibroblast
- Thymic epithelial cells
- T and f i - cells®

185. B i.e. Carries protein; D i.e. Fat absorption; E i.e. preventive role in infection. [Ref: Bijlani 2/e P 174- 75]

Lymphatics
k/as second circulatory system.
I
Feature Function
Network of blind ended thin endothelial Collects proteins, lipids & other large molecules® which leaks
tubes. out of capillaries into interstitial space & returning these to
Endothelial lining is not fenestrated circulation. This
Unidirectional flow (d/1 valves) Help to conserve these substance
All lympatics eventually open into venous Restore blood volume & composition
system Prevent rising o f osm otic pressure in interstitial spaces®
Intercellular junction are permeable to large A bsorption o f lipid in sm all intestine.®
molecules. Carry large undesirable particulate matter and bacteria to the
Basement membrane is discontinuous. nearest lymph node where they can be handled by macrophage
or lymphocytes (essential role in defence mechanism o f body)®

186. C i.e. V III 187. D i.e. XII [Ref: Guyton 11/e p. 464; Ganong 22/e P—544]

- V it K dependent factors: fa c to r I I (prothrombin), VII, IX, X protein C and S®


- V it K is a necessary cofactor for the enzyme that catalyzes carboxylation of glutamic residue in these proteins,
necessary for their action.
- PT is first to be prolonged in Vit K deficiency.® ( factor VII has short tl /2)

188. A i.e. H em ophilia [Ref: Ganong 22/e P— 544; Harrison 16/e P -6 8 0 ,681]

H em ophilia A is a disorder o f coagulation th at occurs due to deficiency or reduced activity o f fa cto r VIII®.
a
130 ® A Complete Review of Short Subjects

189. B i.e. PT is increased [Ref Ganong Physiology 23nl/e p. 531-35; 22/e P- 540-45; Harrison 16/e P-680,681;Chaudhuri 5/e P-58]

Hemophilia A occurs d/t deficiency of factor VIII. Factor VIII is involved in intrinsic pathw ay which is measured by
p t t .q

190. D i.e. Factor VII [Ref: Ganong 23rd/e p. 531- 35; Robbins 7/ed pg-128; clinical hematology 4/ed pg-355]

Factor 7 is involved in extrinsic pathw ay and factors 8, 9,11 and 12 are involved in extrinsic pathw ay o f coagulation®.

191. B i.e. Factor VII [Ref: Ganong 22/e p. 539- 40, 542- 53; Guyton ll/e p. 293, 458- 65; Chandrasoma Taylor 3 126/

Serum has essentially the same composition as plasma except that

its clotting fa cto r II,V, VIII and fibrinogen (I) have been removed®
it has a higher serotonin® content b/o breakdown of platelets during clotting.

Plasma Serum
It is the extracellular fluid portion of blood that contains It is the fluid that remains after clot retraction, i.e., if the
immense number of ions, inorganic & organic molecules. It is whole blood is allowed to clot in a tube and clot is
rich in clotting factors, and can clot__________________ removed, the remaining fluid is serum. Serum cannot
Plasma = Whole blood - Cellular component clot.
★ Since test tube (invitro) clotting activates only intrinsic Serum = Whole blood - Clot
pathw ay, the factor VII (7) that is exclusve to extrinsic Serum = Plasma - Clotting factors 1, 2, 5 & 8
system is not consumed®.
192. B i.e. 8-12

Half life of clotting factors (in hours) in increasing order is Factors III (6hrs), Factor V III (8-12 hrs)Q, Factor V & IX (24 ☆
hrs), WVF (30 hrs), Factor X (45-50 hrs), Factor II (60 hrs), Factor XI (50-84 hrs), Factor I (100-150 hrs) and Factor XII (150
hrs), (Mn: Factor 8 has half life of 8-12 hours)Q.

193. A i.e. Factor XIII [Ref: Ganong 24/e p. 577]

Factor XIII (B) helps in bridging the fibrin in a clot and stabilizes the clot by covalently cross linking fibrinQ. It
requires Ca++.

194. C i.e. Thrombin


195. A i.e. Factor XIII [Ref: Ganong 23rd/e p. 531- 35; 22/e p. 543; Prothombin
Extrinsic pathway
Harrison 17/e p. 364; Guyton 11/ep. 459- 50] Prothombin /
C a’

Factor X III (13) also known as fibrin stabilizing factor,


☆ I activator

Thrombin
\
Intrinsic pathway

when gets activated (by thrombin) cause form ation o f F ib r in m o n o m e r (has


a u to m a tic c ap ab ility to
covalent cross linkages and provide three dimensional Fibrinogen.
p o ly m e riz e & fo rm w eak
strength to fibrin meshw ork (stbilization)® non c o v a len t h y d rog en
Factor XIII (a) converts a loose mesh of interlacing fibrin XIII - XIII activated -
b on ds)

strands to a tightly dense aggregate by formation of multiple


covalent bonds between fibrin monomers and cross Covantely bonded cross c< I
Fibrin fibers
linkages between fibrin fibers. This also requires calcium. linked fibrin fibers S ta b i liz a ti o n

196. A i.e. Antithrombin III 197. C i.e. Cerebral circulation


198. D i.e. Vascular endothelium is smooth & coated with glycocalyx [Ref: Ganong 24/e p. 566-69; Guyton 12/e p. 456]

Blood clotting is prevented by endothelium d/t their sm oothness and its glycocalyx- thrombomodulin layer®. ☆
Thrombomodulin, a thrombin binding protein, is expressed by all endothelial cells except those in the cerebral
microcirculation®.
Vascular endothelium is sm ooth and is coated with a layer o f glycocalyx (mucopolysaccharide)® which prevents
activation of intrinsic system and repels clotting factor & platelets, thereby preventing clotting of blood with in vessels.
Antithrombin (AT) III is the m ost important anticoagulant in blood which prevents clotting o f blood in normal
vascular system®. Heparin is powerful anticoagulant but its concentration in blood is low, so by itself it has little or no
anticoagulant property in normal vascular system. However, heparins binding with ATIII increases effectiveness of AT
III upto 1000 times. Fibrin (not fibrinogen) also has antithrombin action during clot formation.
Physiology Cardiovascular System 131

199. D i.e. Alpha-2 macroglobulin » B i.e. Heparin cofactor II [Ref: Harper 27/e p. 611;Devlin 5/e p. 1041 ]

• There are 4 naturally occurring thrombin inhibitors in normal plasma


- Antithrombin IIIQ is most important & contributes -75% of antithrombin activity. It also inhibits factors IXa, Xa(
XIa, Xlla and Vila complexed with tissue factors.
- Ob-macroglobulinQ contributes most of remainder antithrombin activity.
- Heparin cofactor II and di antitrypsin are minor inhibitors under physiological conditions
• Heparin (acid proteoglycan) binds to cationic site of antithrombin III and greately potentiates its activity^. So,
strongly cationic polypeptide such as protamine strongly bind to heparin & inhibit its anticoagulant effect.
• Contrary to heparin (which acts through antithrombin III), direct thrombin inhibitors such as bivaluridin, lepirudin,
and argatroban directly inhibits the active site of thrombin. So they have no effect on factors 9 ,1 0 ,1 1 ,1 2 and 7.
• Throm bin catalyzes conversion of fibrinogen to fibrin, and is also involved in a regulatory mechanism of coagulation in
this way. Thrombin forms a complex with thrombomodulin, a glycoprotein on endothelial surface which activates
protein C (APC) —» activated protein C combines with protein S and degrade factor Va and Villa limiting their action in
coagulation. So deficiency of either protein C or protein S can cause venous thrombosis.
• Factor V leiden, has a glutamine residue in place of arginine at position 506, which makes it resistant to inactivation by APC.
So this condition has increased risk of venous thrombosis.

200. D i.e. Vasoconstriction and platelet aggregartion


201. A i.e. Platelet adhesion to exposed endothelium B i.e. Clot retraction C i.e. Activation of prothrombinase complex D i.e.
Vasoconstriction [Ref: Harper's Biochemistry 26/e P- 601; Ganong 22/e P- 542; Guyton 10/e P- 418 - 23]

Thromboxane (TX) A 2 causes vasoconstriction & platelet aggregation^. Whereas prostacycline (PGI2) has reverse action
ie vasodilatation & inhibition of platelet aggregation.

Role of Platelets in Hemostasis

- Vasoconstriction G d /1 release of TXAi and serotonin


- Adherence: The glycoprotein of cell membrane platelet cause adherence to exposed endothelium (but repulses to
normal endothelium)
- Platelet plug formation or aggregation closing small vascular holes due to thromboxane A2 (TxA 2)Q.
I
- Conversion of prothrombin to thrombin: A ctivation o f prothrom binase com plex Q
(consisting of platelet anionic phospholipids Ca++, factor Va, factor Xa and prothrombin)
on the surface of activated platelet 1/t conversion of prothrombin into thrombin
- Stabilization of fibrin: Fibrin stabilization factor released from platelet cause stablization
of fibrin (Polymerization of fibrin)
- Thrombin causes conversion of fibrinogen to fibrin
i
- Fibrinous organization o f blood clot d/t growth factors secreted by platelet
- Clot retraction & contraction d/t contractile proteins actin, myosin and thrombosthenin

202. D i.e. ABO [H] antibodies are invariably present in plasma when person RBC lacks the corresponding antigen.
203. A, B, C i.e. Found on RBC membrane; Glycprotein in nature; Highly immunogenic
204. D i.e. AO [R e f: B est & T a y lo r 1 3 /e p 3 7 9 ; G u y ton 1 2/e p. 4 4 5 -4 6 ; G a n o n g 2 4 /e p. 5 6 0 -6 1 ; A n a n th a n a ra y a n a n 8 /e p. 185-86; L ip p in cott 4 /e p. 16 5 ]

---------------------------------------------------------------------------------------- !---------------!--------------------- !------------------------------ h i


- Highly immunogenic ABO blood group antigen is due to complex oligosaccharide (glycoprotein) found on RBC
membrane. It is inherited as m edelian autosomal dominants^.
- ABO blood group system remains the most important blood group system in clinical practice because ABO [H]
antibodies are invariably present in plasma when person's R BC 's lack the corresponding antigen. And because of
these preformed antibodies the reaction is immediate.
- Genotype of a person with A blood group may either be AA (homozygous) or AO (heterozygous). The genotype in B
blood group may be BB (Hamozygous) or BO (heterozygous); in AB blood group AB and in blood group O the
genotype is OO.
132 ■ A Complete Review of Short Subjects

205. D i.e. Nerve growth factor IR ef: T rocia M et al. N erv e g ro w th fa c to r in hibits a p op tosis in m em o ry B cells j B iochem 20 0 3 ; 19 :276(4 2 1 :3 9 0 2 7 -3 6 ].

• Nerve growth factor inhibits apoptosis in memory B cellsQ via inactivation of p38 MAPK, prevention of
antiapoptotic factor Bcl-2 phosphorylation (i.e. maintaining its integrity) & cytochrome c release

Although other growth factors like FGF, PDGF and IGF can also prolong the survival, it is the nerve growth factor that
is most important in immortalization of memory cells.
Growth factor activate P13 kinase -> activate Akt - » inactivates procaspase 9 & inhibit Fast synthesis

206. C i.e. Oestrogen [Ref: A.K. Jain clinical hematology 2/e P 150, water & Emile P664]

Erythropoiesis is promoted by ACTH, Prolactin, TSH (thyroxine stimulating hormone) but not oestrogenQ.
£
207. B i.e. Hints at serious heart ailment

[ventricular extrasystole (premature beat) in the absence of ischemic heart disease is usually b e n ig n Q ^ ^

208. B i.e. Brilliant Cryesyl blue [Ref: Samson Wright 13/e P 33,34]

Reticulocytes- They young red cells are so called because on vital staining with cresyl blue a network of reticulum is
apparent in the cytoplasm as clumps of small dots or as a faint thread connecting two small nodes. Staining solution
include: Brilliant Cresyl blueQ (Supravital staining), Sodium citrate, NaCl, Distilled H 2O
★ Myeloid: Erythroid ratio in Red bone marrow is 3 :1 Q

209. E i.e. None [Ref: Ganong 22/e P- 539]

Plasma - is fluid portion of blood, containing an immense number of ions, inorganic molecules & organic molecules that
are in transit to various part of the body.
Serum - If whole blood is allowed to clot and the clot is removed, the remaining fluid is serum.
It has same composition as plasma except its fibrinogen, factor 2,5,& 8 have been removed
It has high serotonin content d/t breakdown of platelets during clotting.

210. D i.e. Used to increase oxygen delivery to tissue 211. B i.e. Plasma expander
212. C i.e. Plasma expander [Ref: KDT 5/e P-582-584; Trauma Management by Asensio P-70-82]

Plasma expanders are high molecular weight substances which exert colloidal osmotic pressure and when infused
intravenously retain fluid in the vascular compartment. Substances employed as Plasma expanders include: Human
albumin, Dextran®, Hydroxyethyl starch (Hetastarch )Q, Degraded gelatin polymer, Polyvinyl pyrrolidane.

213. C i.e. Bicarbonate [Ref: O.P Ghai 4/e P 182]

Composition of Ringers lactate: Calcium chloride Q, Potassium chloride Q, Sodium chloride Q, Sodium lactate Q.
C hap ters DIGESTIVE SYSTEM

QUESTIONS

Saliva, Oesophagus and Stomach B. K+ □


C. Ca++ □
1. Lysozyme is present in- (NBE P 13) D. Mg++ □
A. Saliva □ 11. Pepsinogen is secreted by- (NBE P 13)
B. Human milk □ A. Parietal cells □
C. Tears □ B. Mucus cells □
D. Mucus □ C. Chief cells. □
2. Maximum potassium ions secretion is seen in: D. Oxyntic cells □
A. Saliva (AIIMS 09) □ 12. Intrinsic Factor of Castle is secreted by which of the
B. Gastric secretions □ following cells in gastric glands: (NBE P 15,13; AI 09)
C. Jejunal secretions □ A. Chief cells (DNB 15,14) □
D. Colonic secretions □ B. Parietal (Oxyntic) cells □
3. Highest concentration of potassium is in? C. Enterochromaffin cells □
A. Bile (AIIMS 09) □ D. B cells □
B. Pancreatic juice □ 13. G oblet cells secrete- (NBE P 13)
C. Terminal ileal secretions □ A. HCI □
D. Rectal fluid □ B. Pepsin □
4. Salivary amylase is inactivated by- (NBE P 13) C. Mucus □
A. Enteropeptidase □ D. Serolonin □
B. Low pH of stomach □ 14. W hich of the follow ing enzymes is stable at acidic
C. High pH of intestine □ low pH: (AI 07)
D. None A. Pepsin/pepsinogen □
5. A man cannot digest carbohydrate. The enzyme B. Trypsin □
deficient is- (NBE P 13) C. Chymotrypsin □
A. Lipase □ D. Carboxypeptidase □
B. Amylase □ 15. Chymotrypsinogen in activated into chymotrypsin by
C. Pepsin □ A. Trypsin (AIIMS 01) □
D. Trypsin □ B. Pepsin □
6. Which of the follow ing is not true about the salivary C. Fatty acids □
gland output in SjO gren's syndrome? (AIIMS 16) D. Bile salts □
A. Increase in sodium concentration □ 16. Trypsinogen is converted to trypsin by- (NBE P 13)
B. Increase in phosphate concentration □ A. Pepsin □
C. Decreased output of salivary glands<0.5ml/min □ B. Enterokinase □
D. Increase in IgA concentration □ C. HCL □
7. Effect of aceytylcholine on LES- (NBE P 13) D. None □
A. Contraction □ 17. Pancreatic secretion contain - (NBE P 14; PGI 04)
B. Relaxation □ A. Trypsin & Lipase □
C. No effect □ B. Enteropeptidase □
D. Contraction followed by relaxation □ C. Pepsin □
8. Lower esophageal sphincter: (AIIMS 04) D. Renin □
A. Has no tonic activity □ 18. All of the follow ing are secreted proenzyme form
B. Has a tone which is provided by the sympathetic except: (NBE P 14,13; Delhi 06)
system □ A. Trypsin □
C. Relaxes on increasing abdominal pressure □ B. Chymotrypsin □
D. Relaxes ahead of the peristaltic wave □ C. Pepsin □
9. Rennin is present in- (NBE P 13) D. Ribonuclease/Amylase □
A. Gastric juice □ 19. Following are gastrointestinal hormones excep t:
B. Liver □ A. CCK- PZ (DNB 14,13) □
C. Kidney □ B. GIP □
D. Lung □ C. Motilin □
10. Normal gastric juice contains all except: D. Chymotrypsin □
A. Na+ (AI 05, AIIMS 06) □
134 ■ A Complete Review of Short Subjects

20. Chymotrypsinogen is a (AIIMS 11, 06) B. Gastric antral cells □


A. Zymogen □ C. Pituitary □
B. Carboxypeptidase □ D. All . □
C. Transaminase □ 31. Gastric secretion is : (NBE P 13; UP 02, UPSC 09)
D. Elastase □ A. Inhibited by curare □
21. All of the following are trypsin inhibitors, Except: B. Stimulated by nor adrenaline □
A. Alpha - 1 antitrypsin (AI 08) □ C. Increased by stomach antral distention □
B. Alpha - 1 - antiproteinase □ D. Stimulated by an increase in tonic activity □
C. Enterokinase □ 32. Which of the follow ing is TRUE regarding gastric
D. Egg - white □ emptying: (NEET 14, AIIMS 08)
22. Cephalic phase of gastric secretion- (NBE P 13) A. Decreased by CCK. □
A. On food entering stomach □ B. Decreased by gastrin □
B. On food entering intestine □ C. Increased by secretin □
C. On seeing food □ D. Decreased by insulin. □
D. On stress □ 33. Stimulation for gastric emptying : (DNB 01)
23. Cephalic phase of gastric secretion- (NBE P 13) A. Secretin □
A. 20% □ B. CCK □
B. 70% □ C. Gastrin □
C. 10% □ D. Distension □
D. 100% □ 34. Gastric emptying sequence into duodenum-
24. Cephalic phase of gastric secretion is mediated by A. Fat>protein>carbohydrate (NBE P 13) □
A. Neurohormones (AI 02) □ B. Fat<carbohydrate<protein □
B. Parasympathetic □ C. Protein<fat<carbohydrate □
C. Sympathetic □ D. Protein>fat>carbohydrate □
D. Gastrin □ 35. Physiological gastrectomy is: (PGI 2K)
25. The gastric phase of gastric HC1 secretion is brought A. Ligate all major arteries □
about by (NBE P 15,13; AIIMS 03,14,15) B. Antrectomy □
A. Neural factors □ C. Upper 1/3 of stoma resected □
B. Gastrin Hormonal factors □ D. Ligation of 4-out of five arteries □
C. Gastric distension □ 36. Figure below represents the pH of the digestive juices
D. Presence of proteins in the stomach □ aspirated from the alimentary tract as a function of
26. Which of these statements are correct regarding acid position along the alimentary tract during digestion
secretion in stomach: (PGI 06, 02) of a meal: (AI 03)
A. Gastrin increases acid secretion □
B. Secretin decreases acid secretion □
C. H 2 blockers decrease acid secretion □
D. Total acid secretion reflexes functional parietal cell
mass □
E. Somatostatin increases acid secretion □
27. Gastric secretion is increased by all except- (NBE P 13)
A. Histamine □
B. Acetylcholine □
C. Gastrin □
D. HCL □
A. A typical value for Y2 is 9.0 □
28. HCL secretion is stimulated by: (NBE P 14; PGI 08)
B. A typical value for Y3 is 10.0 □
A. Secretin □
C. The segment C represents the pylorus □
B. Somatostatin □
D. The digestive enzymes activein segment A are
C. Histamine & Gastrin □
inactivated in segment B. □
D. VIP □
29. Gastric secretions stimulated by all of the following Pancreatic Secretions
except: (NBE P 13,14,15; AIIMS 14,15)
A. Secretin □ 37. Pancreatic lipase hydrolyses ester linkage of triacid
B. Gastric (antral) distension □ glycerides at position- (NBE P 13)
C. Gastrin □ A. 1&2 □
D. Vagal stimulus □ B. 1&3 □
30. Gastrin is produced by: (NBE P 15,14,13) C. 2&3 □
A. Pancreas □ D. Only 3 □
Physiology: Digestive System ■ 135

38. Amount of water lost in stools- (NBE P 13) C. Contraction of pyloic sphincter
A. 50ml □ D. Gastric secretion increase
B. 200ml □ 48. I cells secrete- (NBE P 13)
C. 300ml □ A. Secretin
D. 350ml □ B. Gastrin n
39. Daily pancreatic secretion- (NBE P 13) C. CCK □
A. 1.5L □ D. Motilin n
B. 2.5 L □ 49. Best stimuli for CCK secretion is: (NBE P 13)
C. 5.0 L □ A. Acid
D. 10 L □ B. Protein
40. True about secretin is: (PGI 02) C. Bile
A. Increased gallbladder contraction and HCO 3 rich D. Fat L1
pancreatic fluid □ 50. Fat in the duodenum lumen : (AI 09)
B. Increased gastrin secretion □ A. Stimulates gall bladder contraction
C. Gastric hypermotility □ B. Inhibits gall bladder contraction
D. Increase enzyme rich pancreatic fluid □ C. Inhibits CCK secretion
41. All are true about secretion except: (PGI 08,15) D. Releases Secretin
A. Inhibits gastric emptying (NBE P 14,13) □ 51. CCK-PZ causes all of the follow ing except
B. Increases bicarbonate rich pancreatic secretion □ A. Gall bladder contraction (AI 10) C
C. Potentiates action of CCK □ B. Pancreatic enzyme secretion
D. T ses bile salt & bile acids □ C. Increased gastrin secretion
E. Increases bile secretion □ D. Decreased lower esophageal sphinictor □
42. The duodenum secretes a hormone which has 52. G allbladder contraction is stimulated by OR The
follow ing effects except: (DNB 14,11, AI 09) most important hormone that increases gallbladder
A. Causes copious pancreatic juice rich in contraction after a fatty meal is: (NBE P 15; AIIMS 14)
bicarbonate and poor in enzymes □ A. Gastrin □
B. Increases gastric motility □ B. Secretin 13
C. Causes gall bladder to contract and sphincter C. Vagus □
of oddi to relax □ D. Cholycystokinin □
D. Leads to meagre flow of pancreatic juice rich
in enzymes □ Gall Bladder & Bile
43. Actions of cholecystokinin include all o f the
53. Which is/are not present in bile: (PGI 12)
following except: ( AIIMS 14)
A. Contraction of gall bladder □
A. Bile salt □
B. Bile pigment □
B. Secretion of pancreatic juice rich in enzymes □
C. Cholesterol □
C. Increases the secretion of enterokinase and
D. Phospholipids □
augments the action of gastrin □
E. Cholecystokinin □
D. Stimulated gastric emptying □
44. Which of the follow ing statements, regarding 54. Secretion of bile into bile canaliculus is by-
'secretin' is least correct? (AI 08) A. Osmotic gradient (NBE P 13) □
A. Increases bicarbonate rich secretion □ B. Facilitated diffusion □
B. Inhibit gastric acid secretion □ C. Active transport across the membrane □
C. Increases gastric acid secretion □ D. Simple diffusion □
D. Causes contraction of pyloric sphincter □ 55. M ost important stimulant for bile secretion is:
45. Most potent stimulus for secretin is - (NBE P 13) A. Cholecystokinin (AIIMS 05, jipmer 09) □
A. Dilatation of intestine □ B. Secretin □
B. Acid chime/Acid □ C. Bile acid □
C. Protein □ D. Bile salt r3

D. Fat □ 56. Bilirubin is secreted by: (AIIMS 08, DNB 13)


46. Pancreatic juice rich in water and electrolytes poor in A. Bile Salts □
enzymes is secreted in response to : (NBE P 14) B. Bile pigments □
A. Pancreatozymin □ C. Secretin C
B. Cholecystokinin □ D. CCK. □
C. Secretin □ 57. Maximum contraction of gall bladder is seen with:
D. Proteins □ A. CCK (PGI 2K) f-3
47. Secretin does not cause : (UP 08, AI 10) B. Secretin □
A. Bicarbonate secretin □ C. Gastrin rI
B. Auguments the action of CCK □ D. Enterogastrone :
136 ■ A Complete Review of Short Subjects

58. Bilirubin is derived from: (PGI 02) 68. All of the following statements are true for 'Intestinal
A. Myoglobin □ M otility' except: (NBE P 15,14; AI 07)
B. Hemoglobin □ A. Does not depend on Gastric motility □
C. Muscle □ B. Increased by Distension □
D. Cholesterol □ C. Increased by Acetylcholine □
E. Amino acids D. Increased by cholecystokinin □
59. In blood bilirubin is with- (N BE P 13) 69. Intestinal motility is increased by- (N BE P 13)
A. Protein □ A. Secretin □
B. Steroid □ B. Gastrin □
C. Vitamin □ C. CCK □
D. Carbohydrates □ D. None □
60. Role of bile salts- (N B E P 13) 70. After a meal rich in carbohydrate, insulin secretion is
A. Vit B12 absorption □ stimulated by: (Jipmer 03, WB 05)
B. Formationof lipid bilayer □ A. CCK □
C. Emulsification of lipids □ B. Serotonin □
D. Fatry acid degradation □ C. VIP □
61. Bile acid has a detergent action due to: D. GLP-1 (7-36) amide □
A. Formation of soap (AIIMS 07) □ 71. M ajor regulator of interdigestive myoelectric
B. Formation of zwitter ion □ complexes (N BE P 1 4 ,1 5 ; T N 04, SG PG I 03, UP 05)
C. Amphipathic nature of bile acids □ A. VIP □
D. Formation of medium chain triglycerides. □ B. GIP □
C. Motilin □
Liver D. Neurotensin □
E. GRP □
62. Fat is maximum snythesised in- (N BE P 13) (N BE P 13)
72. M otilin secretion decreased in-
A. Liver □ A. Thirsty □
B. Adipose tissue □ B. Starving □
C. Intestine □ C. Ingested meal □
D. Muscle □ D. Interdigestive state □
63. Function of hepatic stellate cells is/are: 73. Myentric plexus is present in- (N BE P 13)
A. Formation of sinusoids (PGI 01) □ A. Muscularis externa □
B. Vit-A storage □ B. Submucosa □
C. Increases blood perfusion □ □
C. Mucosa
D. Phagocytosis □ D. Serosa □
64. Liver synthesizes all, except: (AIIMS 07) 74. Small intestinal peristalsis is controlled by
A. C3 complement component □ A. Myentric plexus (N E E T 13, Kerala 04) □
B. Hap Toglobin □ B. Meissners plexus □
C. Fibrinogen □ C. Vagus nerve □
D. Immunoglobulin □ D. Para sympathetic system □
65. Conjugated hyperbilirubinemia in infancy is seen in: 75. Pacemaker of the G.I.T. is located in: (PGI 08,15)
A. Gilbert syndrome (PGI 06) □ A. Cardiac end of stomach □
B. Criggler- najjar syndrome □ B. Long muscle of small intestine □
C. Dubin- johnson syndrome □ C. Pyloric end of stomach □
D. Rotor syndrome □ D. Central control of CBD origin □
E. Neonatal hepatitis □ E. Fundus of stomach □
66. Abnormal synthesis function of liver: 76. Chyme is propelled forward in small intestine by
A. TPT (PGI 06) □ A. Segmentation (PGI 08, 14) □
B. Hyperbilirubinemia □ B. Haustrations □
C. Acute phase reactant □ C. Migratory motor complexes(MMC) □
D. Kupffer cells □ D. Peristalsis □
77. M ajor initiating response for peristalsis is- (NBE P 13)
Motality
A. Hormonal □
67. Gastro-colic reflex is related to: (AIIMS 03, Orissa 08) B. Local stretching of gut □
A. Mass peristalsis □ C. Neural □
B. Segmental movement □ D. None □
C. Pendular movement □ 78. What is responsible for clearing & flushing food from
D. Colonic cases □ the intestinal lumen in the interdigestive period-
Physiology: Digestive System ■ 137

A. Gastrin (NBE P 13) □ B. Mannose □


B. Migration motor complexes □ C. Fructose □
C. Secretin □ D. Lactose □
D. CCK □ 87. The amount of water absorbed in the intestine in a
79. Migratory motor complexes in the gut reappear after day is: (NBE P 13, AIIMS 05)
intervals of: (AIIMS 16) A. 5 lit. □
A. 90 mins □ B. 1 lit. □
B. 120 mins □ C. 10 lit. □
C. 60 mins □ D. 8 lit. □
D. 150 mins □ 88. Maximum water & electrolyte reabsorption in the
80 . Enterogastric reflex is caused by all, except: Gastrointestinal tract occurs in OR Resection of which
A. Duodenum distension (PGI 08, AIIMS 04) □ intestional segment causes marked electrolyte
B. Alkaline pH in duodenum &hormones □ imbalance: (NBE P 15,13; AI 11,10)
C. Increased osmolality of chime □ A. Stomach □
D. Products of protein digestion & H+ ions bathing B. Jejunum □
duodenal mucosa □ C. Ileum □
81. Factors responsible for causing diarrhoea after D. Colon □
vagotomy are A/E: (NEET 13, PGI 08) 89. Maximum absorption of bile occurs at - (Bihar 03)
A. Rapid gastric emptying □ A. Jejunum □
B. Hypoacidity in duodenum □ B. Duodenum □
C. Irregular peristalsis □ C. Ileum □
D. None □ D. Colon □
82. All of the following are true about basal electrical 90. Which of the following is absorbed in the colon -
rhythm of intestines except: (AIIMS 15) A. Iron (AMU 04, ]har 06, MP 05) □
-15- B. Proteins □
Spike potentials
E lectrical
recording
mV C. Bile salts □
D. Electrolytes □
-50
91. Which of the following secretions has a very high pH?
A. Gastric juice (AI 06) □
M echanical
recording , 5 B. Pancreatic juice □
(tension)
C. Bile in gall bladder □
A.Tone of contraction isrelated toamplitude of the D. Saliva □
stimulus □ 92. The final sugars in intestinal chyme are: (Delhi 06)
B. Tone of contraction isrelated tofrequency of A. Glucose & fructose □
stimulation □ B. Ribose & mannose □
C. Frequency of contraction is 6per minute □ C. Ribose & xylulose □
D. Threshold of contraction is -5 0 mV □ D. Xylulose & fructose □
83. Slow waves/BER maximum in- (NBE P 13) 93. The only Sugar normaly absorbed in the intestine
A. Stomach □ against a concentration gradient is OR Which is
B. Ileums □ maximally absorbed from GIT:
C. Colon □ A. Xylose (NBE P 15,13, Jipmer 04) □
D. Rectum □ B. Mannose □
84. Maximum postprandial motility is seen in: C. Glucose (Flexose) □
A. Ascending colon (AIIMS 11, 08) □ D. Ribose (Pantose) □
B. Transverse colon □ 94. All of these are required for digestion of dietary fat:
C. Descending colon □ A. Bile pigments (PGI 02) □
D. Sigmoid colon □ B. Gastric lipase □
85. While doing sigmoidoscopy, if the rectum is inflated C. Colipase □
with gas increased peristalsis is seen in: (AIIMS 01) D. Bile salts □
A. Whole colon □ E. Pancreatic lipase □
B. Proximal colon □ 95. True about fat absorption (PGI 2K)
C. Distal colon □ A. Fat in stool > 6 gm/day indicate malabsorption □
D. Whole intestine □ B. C-glycine test is most sensitive test □
C. Major fat absorption occurs in proximal intestine
Small & Large Intestine (jejunum & duodenum) □
D. Steatorrhea means stool fat > 10 gm/day □
86. Best absorbable monosaccharide- (NBE P 13)
A. Glucose □
138 ■ A Complete Review of Short Subjects

96. Short chain fatty acid produced by bacteria are 106. Which of the following cannot be synthesized in the
maximally absorbed in : (NEET 15,13, Jipmer 02) body: (PGI 12)
A. Duodenum □ A. Vit K □
B. Colon □ B. Vit C □
C. Ileum □ C. Thiamine □
D. Jejunum □ D. Riboflavin □
97. What is Intrinsic Factor related to- (NBE P 13) E. Cyanocobalamin □
A. Cobalmin absorption □ 107. Which vitamin is synthesized in vivo in the body by
B. Vitamin D □ humans? (AIIMS 16)
C. Folate absorbtion □ A. Folic acid □
D. Vitamin C □ B. Niacin □
98. Gastric secretions are essential for absorption of- C. Pantothenic acid □
A. Cobalamin (NBE P 13) □ D. Cyanocobalamin □
B. Fat □ 108. Which of the following is not the primary function of
C. Thiamine □ gut flora? (AIIMS 11)
D. Folic acid □ A. Synthesis of shortchain fatty acids □
99. Vitamin B n is absorbed in: (NBE P 15,13; PGI 15,12,07) B. Decreased proliferation of epithelial cells □
A. Duodenum □ C. Production of vit. K □
B. Jejunum □ D. Fermentation of mucin □
C. Ileum □ 109. Most potent anti oxidant- (NBE P 13)
D. Stomach □ A. Vit A □
100. Glossitis and cheilitis seen with which Vit. B. Vit K □
deficiency: C. Vit E □
A. Vit. B2 (PGI 06) □ D. Vit C □
B. Vit. B12 □
C. Vit. K □ Diet
D. Vit D □
E. Vit. E □ 110. Which of these is the correct representation of the
101. Iron absorption is increased by: (AIIMS 04) normal oral glucose tolerance test? (AIIMS 16)
A. Fibre diet □
B. Vitamin C □
C. Phosphate □
D. Phytic acid □
102. Iron is actively absorbed in:
A. Stomach (NBE P 14,13; AIIMS 07,13,15) □
B. Duodenum and proximal jejunum □
C. Large intestine □
D. Ileum □
103. Function of peritoneum are A/E: (PGI 01)
A. Lubrication □
B. Hormone release □
C. Pain sensitive □
A. B
D. Enzymatic digestion □
B. C
E. Phagocytosis □
C. A
104. Which of these is the best test for assessment of
D. None of these
intestinal malabsorption? (AIIMS 16)
111. Which of the following statements about
A. D-xylose absorption test □
Recommended Dietary Allowances (RDA) is true
B. Fecal fat estimation □
A. RDA is statistically defined as two standard
C. Serum amylase levels □
deviations (SD) above Estimated Average
D. NBT-PABA test □
Requirement (EAR) (AI 09) □
Colonic Microflora B. RDA is defined as being equal to the Estimated
Average Requirement (EAR) □
105. Which vitamins are synthesised by intestinal bacteria- C. RDA is defined as being equal to Adequate Intake
A. Vit K and D (NBE P 13) D (AI) □
B. Vit K and B □ D. RDA is defined as the recommended minimum
C. Vit A and D □ requirement □
D. Vit K and C □
Physiology: Digestive System ■ 139

112. True about dietary fiber: (PGI 07) 121. Function of M etabolism includes all except
A. Soluble fiber T metabolism of sugar in GIT. □ A. Extraction of nutrients from food (AI 04) □
B. t bulk of stool. □ B. Breakdown of substrate □
C. Only soluble fibers are included in diet. □ C. Equilibrium of biochemical with intracellular
D. t GI transit time. □ components □
E. Not prevention against colonic cancer. □ D. Using building blocks for synthesis □
113. Longest transit time in G IT is seen in; (PGI 04,08) 122. Largest reserve of energy in body stored as:
A. Stomach □ A. Liver glycogen (PGI 07) □
B. Jejunum □ B. Muscle glycogen □
C. Colon □ C. Adipose tissue □
D. Ileum □ D. Blood glucose □
114. True about High roughage in diet is : (AI 02) E. Muscle protein □
A. Decreases stool transit time □ 123. M ain source of energy derived from: (PGI 08)
B. Increase stool transit time □ A. Fat □
C. Normalise stool transit □ B. Glycogen □
D. No effect on stool transit time □ C. Lactate □
115. Which of the follow ing plant components is not D. Acetone □
fermented (degraded) by gastrointestinal 124. Which dissacharies are not broken down in git-
microorganisms (AIIMS 11, AI 09) A. Lactulose (NBE P 15) □
A. Lignin □ B. Maltose □
B. Cellulose □ C. Sucrose □
C. Hemicellulose □ D. Latose □
D. Pectin □ 125. During starvation, graph of three substances are
116. TRUE regarding action of alpha amylase (AIIMS 04) plotted as below. The scale for A and B is on the left
A. Breaks glucose from carbohydrate end □ and the scale for C is on the right. W hich of the
B. Cleaves only at a 1-4 □ substances are represented by the curve B?
C. Cleaves only at a 1-6 □
D. b & c □ 10

117. Following constitute dietary fibres except:


A. Pectin (NBE P 15,13; AI 03)D
B. Cellulose □
C. Hemicellulose □
D. Riboflavin / Lactulose □
118. Dietary fibre contains: (AI04)
A. Collagen □
B. Pectin □
C. Proteoglycans □
D. Starch □
119. Colonic bacteria, on digestion o f dietary fibres would
Days
give- (AIIMS 01)
A. Free radicals □ A. RTj □
B. Glycerol □ B. T3 □
C. Butyrate □ C. T4 □
D. Sucrose □ D. DIT □
120. In the follow ing food items, which one has the
highest 'Glycemic Index'? (AIIMS 03)
A. Corn-flakes □
B. Brown rice □
C. Ice-cream. □
D. Whole wheat bread □
140 ■ A Complete Review of Short Subjects

ANSWERS & EXPLANATIONS:

Saliva

1. A i.e. Saliva [Ref: Guyton 12/ep 776]

Saliva contains thiocyanate ions and lysozyme (proteolytic) enzyme both of which destroy bacteria.

2. A i.e. Saliva 3. D i.e. Rectal fluid


[Ref: Ganong 23rd/e p. 440-42; Guyton llth /e p. 794; Swartz 8th/ed pg- 46; Irwin- Ripe's intensive care medicine 6/e pg- 915; Berne &
levy physiology 5/ed pg- 817]
f ------------------------------------------------------------------------------------------------------------------------------------------------------------------------
• Maximum concentration of K+ ion (i.e. 75 Meq/L or 80-95 mmol/1) is see in colonic secretions (or stool)® but this
accounts for only a small amount of potassium because of only 1 0 0 -2 0 0 ml quantity of large intestinal secretion per day.
Whereas saliva has 2 nd highest concentration of K+ ions (i.e. 25-30 meq/L at low flow rates and 15- 20 meq/L at high
flow rates) but this accounts for overall maximum potassium ion secretion® because of its high (1 to 1.5 L/day) turn
over.

4. B i.e. Low pH of stomach

Salivary amylase (Tylin) is inactivated by low pH (<4.5) of stomach^

5. B i.e. Amylase [Ref: Ganong 23/ep 4531

Digestion of dietary carbohydrate involves salivary & pancreatic amylaseQ

6. B i.e. Increase in phosphate concentration [Ref: Harrison 18/e ch 324: Ganong 24/e p 456-57]

- Na+& CL are extracted and K+ & H C O 3 - are added as saliva passes through duct. Saliva is hypotonic because duct is
relatively impermeable to water. At high secretion rates, there is less time for NaCl extraction, so tonicity of saliva rises
but it always stays hypotonic (to plasma).________________________________________________________________________
- In Sjogren's syndrome, output o f salivary gland decreases (<0.5 ml/min), PO 4-3 concentration remains sameQ, and
Na+ and IgA concentration increases (d/t decrease serous component of saliva).

Oesophagus and Stomach


7. A i.e. Contraction 8. D i.e. Relaxes ahead of the peristaltic wave. [Ref: Ganong 22/e P- 490; Guyton. 10/e P-730]

Lower oesophagus sphincter (LES) relaxes ahead of peristaltic waveQ allowing easy propulsion of food.

Lower Esophageal Sphincter (LES) Mechanism which Prevent GE Reflux


The lower 3-4 cm of the esophagus is thickened to form LES. The smooth Tonic activity o f LES®.
circular muscle fibres of gastroes ophageal function thicken to form LES. Valve like mechanism of short
It is not a true anatomic sphincter. portion o f esophagus that extend into
This remains tonically active® so that intraluminal pressure at lower end of the diaphragm®, when the gastric
esophagus is 30 mm Hg. (upper & middle portion remain relaxed). This tonic pressure increases this prevents
activity prevents reflux of gastric content into esophagus reflux.
The tone of LES is under Parasympathetic® neural control. Fibres of the crural portion o f
Vagal excitatory fibres (VEF): causes release of acetylchoine & esophageal diaphragm® (skeletal muscle)
sphincter contract^. surround esophagus at its lower end.
Vagal inhibitory fibres (VIF): causes release of NO and VIP results in They exert a pinch cock like action,
relaxation of LES. preventing reflux.
The tone of LES is also influenced by some G.I hormones & other endocrinal
mechanism.
When peristaltic swallowing wave passes down the esophagus. There is
receptive relaxation o f LES ahead o f the peristaltic wave®, which allow easy
propulsion of food into the stomach.
Physiology; Digestive System ■ 141

9. A i.e. Gastric juice 10. C i.e. Ca++ IRef: Ganong 24rJ/e p. 459-60]

Normal gastric juice contains Mg2+, Na+, K+, H+but no Ca++Q. Rennin (double 'n' not Renin) = Chymosin is proteolytic
enzyme secreted by chief cells of stomach. It is found in gastric juice and it coagulates/curdle mild in very young animals.

11. C i.e. Chief cells. 12. B i.e. Parietal (Oxyntic) Cells [Ref: G anong 23rd/e p. 431- 434; G abriel p. 260-61; G uyton l l t h j e p. 796;
Surgical pathology o f gastrointestinal tract 2/erf pg- 668, Grays 40"'je p. 11221

Intrinsic factor of Castle is secreted by parietal (oxyntic) cells ® and is essential for absorption o f vitamin BnQ; in its
absence, Addison's pernicious (megaloblastic) anemia® results.

Cells of Stomach and Secretion


The cells of the gastric glands secrete about 2500 ml o f gastric juice daily ®. Contents of Normal Gastric Juice
Esophagus
Cations®: Mg2+, Na\ K+, H+ (PH approx 1)
• Fundus
[Mn: Mujhe Na KaHo]
- Anions: Ch, H P042-, S 0 42-
Cardia
- Pepsins - Intrinsic - Mucus - Lipase
[Mn: Pepsi in my lips.]___________________ Body
Parietal cells
Parietal (Oxyntic - Present in the body & fundus of stomach (H CI
Intrinsic factor)
cells)® - Secrete HCI and intrinsic factor® Duodenum
Chief cells:
(Pepsinogen)
Chief (Zymogen/ - Present in the body & fundus
Peptic) cells - Secrete pepsinogen® (G aslrin+m ucus
and pepsinogen)
Enterochromaffin - Histamine secretion
Like cell (ECL) - Histamine is also a trigger of parietal cell Antrum (gastrin)
secretion via binding to H2 histamine receptor. Pylorus

★ Parietal & chief cell can also be stimulated by acetyl choline released Mucus is secreted in all parts of the
from enteric nerve endings in fundus stomach.

13. C i.e. Mucus 14. A i.e. Pepsin [Ref: Ganong 22/e p.471,468]

Pepsin is formed from pepsinogen in stomach by HCI activation. Optimal pH for pepsin is 1.6-3.2 (acidic) at which it
is stable ®. But its action is terminated when acidic gastric content is mixed with alkaline pancreatic juice in duodenum
Pancreatic enzymes - trypsin, chymotrypsin & carboxy peptidase are stable in alkaline pH®.
Cells with in Principal Gastric Glands in Body & Fundus
Gastric glands are simple branched tubular glands which usually open in group of 4 into a short common channel (or
gastric pit), through which they discharge their secretions into the lumen. Each gland has 3 zones containing 4 main cell
types (& secrete) i______________________________________
r
Outer zone Middle zone (Neck & Isthmus) Basal zone
(Luminal surface i.e. Isthmus
Parietal or HC1Q PepsinogenQ
& gastric pit) Chief/Peptic /
Oxyntic cell Intrinsic factor of CastleQ
I Acid, mbnsicfactor, pepsinogen
Zymogen cells
Mucus Neck Mucus®
cells HCO j 0 Neuro (Entero) Endocrine cells
Surfacemucouscel s
(mucus, trefoil peptide
bicarbonatesecretion) D cells Somatostatin®
Globlet cells Mucus Cell migration (f J L 0 I Mucousnee* cel's
(Intestine)
I[ $) I(stemMil compartment) Enterochromaff Serotonin®
b c tH in cells (ECO
Paneiat ce-le
Brunner's Gland Mucus (acd. minrtsiclaclcr
secfetioni
(Duodenum) Enterochromaff Histamine®
Panneth Cell in Guanylin in Like cells
crypts of —ECleall
(histaminesecretion) (ELC)
Lieberkuhn Gastrin®
Creel colls Gastrin (G)
(small intestine) (pepsinogensecretion,
cells.

Structure of a gastric gland from the fundus


and body of the stomach
142 ■ A Complete Review of Short Subjects

15. A i.e. Trypsin 16. B i.e. Enterokinase 17. A i.e. Trypsin & Lipase
18. D i.e. Ribonuclease /Amylase 19. D i.e. Chymotrypsin 20. A i.e. Zymogen
[Ref: Lippincott 4/e p. 64, 443, 248-49; Harper 29/e p 89, 519, 651-53; Ganong 22/e p. 468 - 69]

Chymotrypsinogen or prochymotrypsin is a proprotein /proenzyme /or zymogen. Chymotrypsinogen is activated to


chmotrypsin by trypsinQ. Chymotrypsin is enzyme not hormone.
Trypsinogen & pepsinogen proenzymes are activated by entero kinase and HCL (respectively) to trypsin & pepsin.
Trypsin & lipase are pancreatic enzymesQ

21 . C i.e. Enterokinase [Ref: Ganong 22/e p. 468; Harper 26/e p. 597; Chatterjee 7/e p. 435]

Enterokinase (enteropeptidase), a glycoprotein enzyme o f intestinal juice activates trypsin® at pH 5.5.


22 . C i.e. On seeing food 23. A i.e. 20%
24. B i.e. Parasympathetic 25. B i.e. Gastrin Hormonal [Ref: Guyton 11/e p. 798 - 99; Ganong 22/e p. 494- 95]

Gastrin Secretin Phase M ainly Mediated by Accounts For % of Total Secretion


Cephalic phase Parasympathetic (vagal) inputs originating from 20%
dorsal vagal complex in brainQ (on seeing food)Q.
Gastric phase Gastrin hormone mechenismQ. 70%
Intestinal Phase Small amount of gastrin released by duodenum <10 %
26. A i.e. Gastrin increases acid secretion; B i.e. Secretin decreases acid secretion; C i.e. H2 blocker decreases acid secretion
D i.e. Total acid secretion reflects functional parietal cell mass
27. D i.e. HCL 28. C i.e. Histamine & Gastrin 29. A i.e. Secretin
30. D i.e. All 31. C i.e. Increased by stomach (antral) distension 32. A i.e. Decreased by CCK
33. C i.e. Gastrin < D i.e. Distension [Ref: Ganong 24/e P- 502-503; 469-72; Guyton 12/SAE p.447, 409-101

Gastrin is produced by gastric antral cells (G cells) mainly but can also be produced by gastrinom as o f pancreas & ☆
pituitary glands®. Gastrin (& so gastric) secretions are increased by gastric (antral) distension, vagal stimulus® and
decreased by secretin and somatostatin®. Gastrin mainly stimulates gastric acid (HCL) and pepsin secretion®.
HCL secretion reflects parietal cell mass. It is stimulated by gastrin, acetylcholine (M3 receptor) & histamine (via H2
receptors) and inhibited by low pH (acid) & H2 blockersQ.
Gastric emptying is prom oted by gastric fo o d volume distending the stom ach w all (main) and gastrin®. It is inhibited
by CCK (main), secretin and GIPQ.
V.
34. A i.e. Fat > Protein > Carbohydrate [Ref: Ganong 241/ep 502]

Gastric emptying is fastest for food rich in carbohydrate (few hours). Protein rich food leaves more slowly and gastric
emptying is slowest for fat rich foodQ-
Hyperosmolality of food entering duodenum decrease gastric emptying.

35. A i.e. Ligate all major arteries [Ref: Ganong 22/e P- 488, 491 ]
36. D i.e. The digestive enzymes active in segment A are inactivated in segment B.

Segment A represents mouth & esophagus Salivary pH is 7®


Segment B represents stomach (since it shows a sharp dip in pH) Gastric pH is 1.2-13®.
Segment C represents duodenum where the alkaline pancreatic/bile Duodenal pH is approx 8®
secretion occur, which raises pH above that of saliva
Enzymes work efficiently only within narrow pH range. Therefore enzym es that work at pH of 7 (segment A) will be
inactivated at pH of 1.5 (segment B).

Pancreatic Secretions
37. B i.e. 1&3 [Ref: Ganong 23/ep 457]

Fat digestion begins in duodenum by pancreatic enzymes lipase & colipase. Pancreatic lipase hydrolyses 1 and 3 ester
linkage (bonds) of triglycerides (triglycerols)Q with relative ease (but bond 2 at a very low rate) producing FFAs (free
fatty acids) and 2 mono acylglycerol.
Colipase is activated by trypsin and colipase activates lipase.
Physiology: Digestive System ■ 143

38. B i.e. 200ml 39. A i.e. 1.5L

Daily secretion of Intestinal Juices D aily water turnover (ml) in G IT


Daily volume (L) pH
Saliva 1-1.5 L 6.0-7.0 Ingested 2000
Gastric secretion 1 .5 - 2.5 L 1.0-3.5 Endogenous secretions 7000
Pancreatic secretion 1 - 1 .5 L 8.0-8.3 Salivary glands 1500
Bile 1-0.5 7.8 Stomach 2500
Small intestine secretion 1.8 L "| 7.5-8.0 Bile 500
Brunner's gland secretion 0.2 L l - l 8.0-8.9 Pancreas 1500
Large intestinal secretion 0.2 L J 7.5-8.0 Intestine 1000
7000
Total 6 .7 -7
Total Input 9000
Reabsorbed 8800
Jejunum 5500
Ileum 2000
Colon 1300
8800
Balance in stool 200
Total input of water is 9 lit and 8.8 of this
fluid is reabsorbed.

40. A i.e. Increased gallbladder contraction and HCO 3 rich pancreatic fluid
41. D i.e. Tses bile salt & bile acids 42. B i.e. Increases gastric motility
43. C i.e. Increases the secretion o f enterokinase and augments the action of gastrin

Secretin hormone stimulates copious secretion of a very alkaline pancreatic juce that is rich in water & HCO 3 and poor
in enzymes and C1~Q. It also stimulates bile secretion (choleretics)Q but has no role in synthesis of bile salt and bile acids.
It augments action of CCKPZ but inhibits gastrin secretion and gastric emptying & motalityQ.

44. C i.e. Increase gastric acid secretion 45. B i.e. Acid chyme 46. C i.e. Secretin
47. D i.e. Gastric secretion increases 48. C i.e. CCK 49. B i.e. Protein
50. A i.e. Stim ulates gall bladder contraction 51. C i.e. Increased gastrin secretion 52. D i.e. Cholecystokinin
[Ref: Guyton 12/SAE p. 417; Ganong 24/e P- 468-71]

Presence of protein and fat in duodenum promotes the secretion of cholecyctokinin- pancreozymin (CCK-PZ) from
the I cells of jejunum (upper small intestine)Q. CCK-PZ is not found in b ile but it produces contraction of gall
bladderQ.

Cholecystokinin Pancreaticozymln (CCPZ) Secretin


• Released by mucosa of jejunumQ (from I cells)Q • Released from S cells of duodenal mucosaQ. It is
• Stim uli for secretion structurally similar to glucagons, GIP, VIP and GLI
- Presence of peptides & aminocids in small intestine^ • Stim uli for secretion
(main). - Acidity of chymeQ (main)
- Fatty acids, having more than 10 carbon atomsQ, in the - Products of protein digestionQ.
duodenum • Action
• Action - Stimulates bicarbonate richQ pancreatic secretion
- Secretion of pancreatic juice rich in enzymeQ. - Responsible for adding water & alkaline
- Contraction of gall bladder & release of bileQ. component to pancreatic juiceQ.
- Inhibits gastric emptyingQ (augments contraction of - Inhibits gastric acid secretion & gastric emptyingQ
pyloric sphincter) by causing contraction of pyloric sphincterQ
- Enhances small intestinal & colonic motilityQ. - Augments action of CCK-PZQ in producing
- Augments action of secretinQ in producing alkaline pancreatic secretion in of digestive enzyme.
pancreatic juice - Stimulates bile secretion (choleretics)Q
- Increases secretion of glucagonQ & Insulin.
144 ■ A Complete Review of Short Subjects

Gall Bladder & Bile

53. E i.e. Cholecystokinin [Ref: Berne 6/e p. 545-48; Ganong 24/e p. 464-65]

Bile is a micellar solution in which major solutes are bile acids, phosphatidylcholine and cholesterol in 1 0 : 3 : 1 ratio.

Bile acids may be primary (ie synthesized by hepatocytes) eg chenodeoxy cholic acid & cholic acid; or secondary bile
acids (ie formed by bacterial action in colon) eg lithocholic acid & ursodeoxycholic acid (formed from chenodeoxy CA)
and deoxy cholic acid (formed from cholic acid). Bile also contains glucronides of bile pigments bilirubin and biliverdin
(responsible for golden yellow color) as well as alkaline electrolyte solution resembling pancreatic juice.

54. C i.e. Active transport across the membrane 55. D i.e. Bile salt 56. A i.e. Bile salts 57. A i.e. CCK

Secretion of bile into bile canaliculus is by active transport across the membraneQ. Bile substances that are actively
secreted into bile across canalicular membrane are bile acids (primary force), conjugated bilirubin, cholesterol &
xenobiotics.
Substances that increase the secretion of bile are called as cholerecticsQ. Bile salts are amongst the most important
physiological cholerectionQ. Most potent stimulus for causing gall bladder contraction is CCK-PZQ.
v j

fo Cholagogues Choleretics
Substances that cause contraction of gall bladder^ Substances that increase secretion of bileQ
CholecytokininQ. - SecretinQ
- Fatty acidsQ 1 by Ting CCK-Pz - Vagal stimulation^
Amino acids J - Bile saltsQ (Most potent stimulus)Q.

58. A i.e. Myoglobin; B i.e. Hemoglobin. [Ref: Harper’s 26/e P- 278] 59. A i.e. Protein

The daily bilirubin formation in human is approx. 250- 350mg, deriving mainly (70-80%) from hemoglobin® but also
from ineffective erythropoiesis and from various other heme proteins such as cytochrome P450 and myoglobin®._______
1 gm o f Hemoglobin yields 35mg o f bilirubin®.

Heme (in RBC Hb, M yoglobin, Cytochrome P450) is broken by Heme oxygenase into
i
Biliverdin (By Biliverdin-Reduce into)
I
Bilirubin (Unconjugated): In blood it is transported in bound from with albumin (i.e. Bilirubin-Album in Complex)
to liver where there is carrier mediated uptake.
I
Conjugated with Glucuronic acid (to form Conjugated-Bilirubin) by UDP-GT1AI and excreted into bile. In intestine, it is
deconjugated by bacterial (1-glucuronidase into Urobilinogen (80% of which is excreted in feces, 20% reabsorbed to enter
enteroheaptic circulation and small amount excreted in urine)

60. C i.e. Emulsification of lipids 61. C i.e. Amphipathic nature of bile acids [Ref: MTx]

Bile acids (bile salts) have detergent (emulsification) action on the lipids due to their amphipathic nature^. They make
ingested lipids soluble & smaller by forming micelles, so that they can be transported to enterocytes for digestion.

Liver
62. A i.e. Liver [Ref: Guyton 12/ep 822]
In Lipid metabolism, liver causes- Synthesis of Liver (maximum)Q
1. Synthesis of TG s (mainly from carbohydrates » > proteins) fat » > Adipose tissue
2. Synthesis of other lipids (mainly cholesterol & phospholipids) from Storage of fat Adipose tissue
fatty acids (FA) (m axim u m )»> liver
3. Degrading FAs into compounds that can be used energy
Physiology: Digestive System ■ 145

63. B i.e. Vit - A storage I Ref: Robbins 7/e P-878]

Hepatic Cells

' 1________
________ Hepatic Stellate cell (also called Ito cells) _______ Kupffer Cells

- Are of mesenchym al origin, found in space of Disse. They have phagocytic


- Play a role in the storage and m etabolism o f Vit AQ. function Q.
- They transform into collagen producing myofibroblast when
there is inflammation and fibrosis of liver.

64. D i.e. Immunoglobulin IRef: Ganong 22/ep. 541; Guyton 11/ep. 854-57; Harper 27je p. 585- 91}

Alpha (a) and beta (f)) globulins are produced by hepatocytes (liver), where as immunoglobins or gamma globins are
produced by B- lymphocyte (plasma cell)Q.

65. C i.e. Dubin Johnson syndrome; D i.e. Rotor syndrome; E i.e Neonatal hepatitis IR ef: H a rrison 16/e p. 1820 - 21; O D G 5 /e p. 1 7 4 1

Disorders of Bilirubin M etabolism 1/t


,__________________________ I___________________________ .
Unconjugaed hyperbilirubinemia Mixed or Predominantly Conjugated hyper bilirubinem ia
- Hem olysis - Dubin Johnson syndrome Q
- Physiological neonatal jaundice® - Rotor syndrom e
- Breast milk jaundice - Benign recurrent intra hepatic cholestasis (BR1C)
- Drug eg pregnanediol, novobiocin, - Progressive fam ilial intrahepatic cholestasis (FIC)
chloram phenicol, gentam ycin (inhibital UGT 1A) - Biliary atresia
- Transient fam ilial neonatal hyper bilirubinem ia - N eonatal hepatitis Q, sepsis
(Lucey - Driscoll syndrom e)
- Crigler - N ajjar syndrome type I, & IIQ
- Gilbert's syndrome Q.
66. A i.e. T PT B i.e., Hyperbilirubinemia; C i.e. Acute phase reactant
IRef: Ganong 22/e p. 500- 01; Guyton & Hall 11/e p. 861 - 64; Harrison 16/e p. 1810-13]

Liver abnormalities are diagnosed by assessing prothrombin time (PTT), direct & total serum bilirubin ☆
(hyperbilirubinemia), albumin, & levels of acute phase reactants (serum alanine & aspartate amino transferase=ALT &
AST)Q

Motility
67. A i.e. Mass persistalsis 68. A i.e. Does not depend on gastric motality 69. C i.e. CCK
IRef: Berne 6/e p. 533; Guyton 11/e p. 776, 784-90, 822; Ganong 24/e p. 498-505]

After a meal, gastrocolic reflex (i.e. increased colonic motality) develops in form of a contraction ring appearing &
moving (like small intestinal peristalsis waves) aborally in colon for large distance resulting in movement of feaces
from splenic flexure to rectum in single attempt (k/a mass peristalsis).
- Intestinal motality depends on gastric motalityQ. It is increased by distension, ACh, CCK, gastrin, motilin,
substance P & serotoninQ.

70. D i.e. GLP-1 (7-36) amide 71. C i.e. M otilin [Ref: Ganong 22/e p. 486 - 87; Guyton 11/e p. 776] ^_

- Glucagon derivative G L P -1 (7-36) am ide (m ost potent) > gastric inhibitory peptide (GIP) also known as glucose -
dependent insulinotropic polypeptide » gastrin , CCK, secretin and glucagon stim ulates insulin secretion Q. But only
GLP-1 (7-36) amide & GIP stimulate insulin secretion when adminstered in dose that produce blood levels comparable
to those produced by oral glucose.
- M otolin is secreted by entero chromaffin cells & Mo cells in stomach, small intestine and colon during fasting. It is released
cyclically (so Ted levels) at intervals of 90-100 minutes in a fasted person which stimulates waves of gastrointestinal motality
called myoelectric interdigestive cmplexes (MIC). Erythromycin binds to motolin receptors so its derivatives may be of
value in treating decreased intestinal motality.

- Neurotensin is produced by neurons & cells of ileum. Its release is stim ulated by fatty acid and it increases ileal blood
flow and inhibits gastro intestinal motality.
146 ■ A Complete Review of Short Subjects

- G IP is produced by K cells in mucosa o f duodenum & jejunum. Its secretion is stimulated by glucose & fa t ° in
duodenum. It is named so because in large doses it inhibits gastric secretion & motality. How ever in small dose (that
are seen after meal) it does not have significant gastric inhibitory activity.

72. C i.e. Ingested meal [Ref: Ganong 24jep 472, 498]

Motolin, a 22 amino acid polypeptide hormone, is secreted by Entero chromaffin cells and Mo cells in stomach, small
intestine & colon. It acts on G protein receptors on enteric neurons in duodenum & colon and produce smooth muscle
contraction in stomach and intestine in period between mealsQ. Erythromycin binds to motolin receptors so in future
may derive com pounds to treat decreased gastric motility.

During fasting betw een periods of digestion, motor activity of gi smooth Ingestion of food (meal)
muscles migrate at rate of 5 cm/min from stom ach to distal ileum (MMC) to suppress/decrease motolin releaseQ,
clear luminal contents for next meal. so MMC is abolished (& peristalsis
M otilin (blood level) increase at interval of -lOOmin in interdigestive state returns) until digestion & absorption
(during contractile phase of MMC). It initiate & migrating motor complexQ. are complete.

73. A i.e. Muscularis extema 74. A i.e. Myentric plexus [Ref: Ganong 24/e p. 473]

Myenteric (Auerbach's) plexus is situated in outer (external) muscular layer in muscularis extema between and ☆
innervates outer longitudinal & m iddle circular muscular layersQ and is prim arily concerned w ith m otor control°.
Submucosal (meissner's) plexus situated between middle circular layer and mucosa is primarly concerned with control o f
intestinal secretion ° as it innervates glandular epithelium, intestinal endocrine cells & submucosal blood vessels.

75. C i.e. Pyloric end of stomach [Ref: Ganong 24/e P- 498-99; Schwartz 8/e P- 946-47]

About option B: Pacemaker cells (interstitial cells of Cajal) are present in Circular muscle layer°.
About option A & C : As they are absent in esophagus and proximal portion of stomach, cardiac end and fundus of
stomach can be ruled out. Peristalsis occurs from esophagus to rectumQ.

76. D i.e. Peristalsis 77. B i.e. Local stretching of gut 78. B i.e. Migration motor complexes 79. A i.e. 90 mins
[Ref: Ganong 24/e P- 497-505; Guyton 10/e 461-62; Sembulingum 3/e P- 217]
( 1
Peristalsis propels the sm all intestinal content (chyme)0 toward the large intestine. It is a reflex response that is
initiated when gut wall is stretched by contents of lumenQ. It occurs in all part of g.i tract from oesophagus to rectum
propelling the content forward. It can be increased or decreased by autonomic input to gut but its occurrence is
independent of extrinsic innervation. It is blocked only if the segment is reversed before it is sutured back (not by removed of
^ intestine segment & resuturing in original position).

Bowel Movements

V V Peristalsis Segmentation ■sj*V Migrating Motor Complex


/•'"'A
Contraction
Peristalsis propels the sm all intestinal Ring like contraction, - During fasting between periods of digestion, the
content (chyme) toward the large which occur regularly or pattern of electrical and motor activity in G.I
intestineQ. irregularly but in smooth muscle becomes modified so that cycles
It is a reflex response that is initiated when rhythmic fashion. of motor activity migrate from the stomach to
gut wall is stretched by contents of lumen. It They move the chyme to distal ileum. Each cycle is called migrating motor
occurs in all part of g.i tract from & fro and increase its complex. (MMC)
oesophagus to rectum propelling the content exposure to mucosal - They may clear the stomach and sm all intestine
forward. It can be increased or decreased by surface. of luminal contents in preparation for the next
autonomic input to gut but its occurrence is Helps in mixing o f chyme mealQ.
independent of extrinsic innervation. It is with digestive juices0. - They cause moderately active peristaltic waves
blocked only if the segment is reversed before it Retard movements of to sweep excess digestive secretions into colon &
is sutured back (not by removed of intestine intestinal contents to prevent their accumulation in upper GIT.
segment & resuturing in original position). provide time for digestion - MMCs occur at intervals of 90 minutes &
Peristaltic Rush & absorption.____________ migrate at rate of 5 cm/minQ. Gastric &
Haustration pancreatic secretion and bile flow increase
- Very intense peristaltic wave, occur when
during each MMC. They are immediately
intestine is obstructed. Mixing movements in stopped by ingestion of food (whereas
- Not seen in normal individual colon0. peristalsis, BER & spike potential return)
Physiology Digestive System ■ 147

80. B i.e. Alkaline pH in duodenum & Hormone


81. D i.e. None [Ref: Bailey & Love 24/e P-1044; Schwartz 8/e P-988; Sembulingum 3/e P- 215, Guyton 11 je P. 785-86]
------------------------------------------------------------------------------------------------------------------------------------
- Enterogastric reflex leading to stoppage of gastric emptying is caused by duodenal distension, acidity (i.e. H+ not
alkalinity) & high osm olarity o f duodenal chyme and presence o f protein & fa t breakdow n products in duodenum®.
- Post vagotomy diarrhea is caused by irregular peristalisis, rapid gastric emptying and decreased gastric acid secretic
(i.e. hypoacidity in duodenum)Q.

82. A i.e. Tone of contraction is related to amplitude of the stimulus

- Muscle contraction in intestine is mostly caused by spike potential (super imposed on BER) and only very rarely by
BER itself (alone). The tone of contraction depends on frequency of stimulus rather than amplitude^.
- In figure (given in Q) threshold for AP & contraction is -50mV. And 1 contraction per 10 seconds implies that frequency
is 6 contractions per minute.

Pacemaker (Interstital) Cell Basic Electrical Rhythm (BER)


of Cajal
- Cajal cells produce basic electrical rhythm
Are stellate mesenchymal (BER)Q, which is spontaneous rhythmic
cells with smooth muscle fluctuation in membrane potential between
like features that send long about - 65 and -45mv ®.
multiple branched processes - BER rarely cause muscle contraction, but spike
into intestinal smooth potential superimposed on the most
muscles depolarizing portion of the BER waves do
Pacemakar cells are absent increase muscle tension^
in esophagus and proxim al - Depolarization of each spike is d/1 Ca++ in
-V'- “V"
portion o f stomach®. flux ® and repolerization portion is d/t K*
-A— ---
In stomach and small -Ap- efflux®
-V - -V—
intestine they are located in V'— -V'- —Y - -Jp - 1 1
outer circular muscle layer® V- -Ar— - V - —V
Spike Other
—V\
Ar- --------- \r\_
near myentric plexus. —
V' *V - potential cells
In the colon they are at the
(intestine)
subm ucosal border o f Regional myoelectric patterns in Ca++ influx Depolarization Na+ influx
circular muscle layer®. the stomach K+ efflux Repolarization K+ efflux
In stomach & small
intestine, pacemaker
frequency is in descending Number of spikes & tension is increased by
gradient and the pacemaker acetyl choline and decreased by epinephrine.
with highest frequency BER cordinates peristaltic & other motor
usually dominates (like activity such as rhythm of segmentation etc.
heart) After vagotomy or stomach wall transection,
peristalsis (in stomach) becomes irregular &:
chaotic.

83. C i.e. Colon


84. D i.e. Sigmoid colon [Ref: Ganong 24/e p 505, 499; Berne & Levy 6/e p 534-36; Timothy R Koch - Colonic disease pg- 339]

Colonic motality is minimal in fasting period. Eating BER Part


stimulates colonic contractions. Fat component of meal is Rate/minute
major stimulus to colonic contractions. Protein ingestion 4/min stomach
inhibits colonic contraction. Maximum postprandial 12/min duodenum & proximal jejunum
increase in m otality is seen in sigm od coin > descending 8/min distal ileum
colon > transverse colon®. 9 /min caecum
16/min sigmoid colon (maximum)Q

85. C i.e. Distal colon [Ref: Sembulingum 3/e P-219; Guyton 10/e P 737, 710]

When the rectum is inflated with gas (CO2 ) in sigm oidoscopy, nerves are stimulated and causes increased peristalsis in the
descending colon, sigmoid (distal colon)® and rectum.
148 ■ A Complete Review of Short Subjects

Small & Large Intestine


86. A i.e. Glucose

Hexose sugars (glucose> fructose) are best absorbable (most rapidly absorbed) monosaccharide^. These are mainly
absorbed in small intestine (proximal part of jejunum)Q.

87. D i.e. 8 lit 88. B i.e. Jejunum 89. C i.e. Ileum 90. D i.e. Electrolytes
IRef: Ganong 23/e 441, 452; Swartz 8/e p 1021-24

- Total input of water is 9 lit and 8.8 of this fluid is Content Maximum
reabsorbed (the nearest answer is 8 liters). absorption
- Out of this 9 lit (2 lit water ingested +1.5 lit saliva + Calcium, Iron DuodenumQ
2.5 lit gastric juice + 0.5 lit bile + 1.5 lit pancreatic juice Water, Electrolyte (Na+, CI-), Amino JejumunQ
+ 1 lit small intestine secretion) 85% is absorbed in acids, Fats (TG,Ch),Sugar (mono &
small intestine (mostly jejunum ) and only 5-10% (1- disaccharide), water soluble vitamins
1.5 lit) is absorbed in colon (large intestine)^. Water (except Bn), Fat soluble vitamins
and electrolyte (NaCl) reabsorptions are linked. Bile, Bile salts. B n (vitamin) Ileum
- Ca2*, Cl', Fe2*, SO42', long chain fa tty acids and vitamins (except B 12) are mainly absorbed in upper sm all intestine (i.e.,
jejunum & duodenum)®.
- Sodium (Na*) is mainly absorbed is upper & low er sm all intestine (jejunum, ileum) and colon®. And K+ in upper mid
& lower small intestine.
- Sugars and amino acids are mainly absorbed in mid small intestine whereas bile salts, and vitamin B12 in low er sm all
intestine (ileum)®
91. B i.e. Pancreatic juice [Ref: Guyton 11/e P-794]

‘Pancreatic juice is essentially alkalin e with a pH as high as 8.0 to 8.3.

92. A i.e. Glucose & fructose 93. C i.e. Glucose [Ref: Ganong 22/e p. 470]

Final products of carbohydrate digestion in intestinal chyme are glucose & fructose®.
The glucose transport in intestine is an example of secondary active transport i.e., the energy of glucose transport is
provided indirectly by active transport of Na+ out of cell. Because glucose & Na+ share the same cotransporter (symport)
the sodium dependent glucose transporter (SGLT, Na+ - glucose cotransporter, cross cell membrane 12 times), the high
concentration of Na+ on mucosal surface of cells facillitate & low concentration inhibit hexose sugar influx into
epithelial cells.

94. C i.e. Colipase; D i.e. Bile salts; E i.e. Pancreatic lipase [Ref: Ganong 24/e P- 483-85, 465, 513; Lippincotts 2/e P 1.164]
95. A i.e. Fat in stool > 6 gm /day indicate malabsorption; C i.e. M ajor fat absorption occurs in proximal intestine.
96. B i.e. Colon [Ref: Harrison 16/e P- 1768, 238]
/ ----------------------------------------------------------------------------------------------------------------------------------------------------------------------
The lingual & gastric lipase is of little importance in fat digestion except in pancreatic insufficiency. Digestion of dietary
fat requires pancreatic lipase, colipae & bile salts (i.e. salts of bile acids). Whereas, bile pigments (bilirubin &
biliverdin) are breakdown products of haemoglobin (heme), give golden yellow color to bile and are secreted in bile in
water soluble glucuronide conjugates.

97. A i.e. Cobalmin absorption 98. A i.e. Cobalamin 99. C i.e. Ileum [Ref: Ganong 22/e P- 477, 496]

Most vitamins are absorbed in upper small intestine but vitamin Bn is absorbed in the ileum®. Intrinsic facto r is
secreted by p arietal or oxyntic cells o f gastric mucosa® and is estimated for vitamin B12 absorptions.__________

100. A i.e. B 2 [Ref: Harrison 16/e p. 404]

R iboflavin (vit B 2) deficiency manifests principally as mucocutaneous lesions o f mouth an skin such as glossitis & cheilitis®.
Physiology: Digestive System ■ 149

101. B i.e. Vitamin C 103. B i.e. Hormone release [Ref: Bailey & Love 24/e P-] 133]
102. B i.e. Duodenum and proximal jejunum
Function of Peritoneum
IRef: Robbins 7je p. 644; Ganong 22/e P- 477; KDT 5"‘/e P-
546; Harrison 16/e p. 588; OP Ghai 6/e p. 300] - Pain perception® (parietal peritoneum )
- Allow visceral m ovem ent
Iron is actively absorbed (via DMT 1) in proxim al sm all - Visceral lubrication®
intestine (duodenum & upper jejunum)®. Its absorption is - Fluid and particulate absorption
increased by ascorbic acid (vit C)®. - Inflam m atory & Im m une response (Phagocytosis)®.
- Fibrinolytic activity
- Storage of fat.

104. A i.e. D-Xylose absorption test [Ref: Harrison 18/e Chapter 294]

Daily fecal fat averages 15-25gm/d with small intestinal disease & exceeds 40gm /d with pancreatic exocrine insufficiency.
Evaluation of fat malabsorption:

1. Quantitative estimation of Best test for diagnosis of NBT-PABA, 1. Oral glycine-1 shows
fecal fat is Intestinal Malabsorption Bentiromide test and
B ile acid M alabsorption
- Gold standart test for (& document integrity of Serum Amylase tests for
diagnosis o f fat intestinal mucosa) I
Malabsorption^ I 2. Schilling Test is for
Pancreatic M alabsorption
Malabsorption of
D-Xylose Test
2. Sudan III Quantitative Vitamin B12
Test is Best for ScreeningQ

Colonic Microflora & Antioxidants

105. B i.e. V it K and B


106. B i.e. V it C; C i.e. Thiam in; D i.e. Riboflavin; E i.e. Cyanocobalamin [Ref: Harper 29/e p. 525, 532; Chatterjea 8/e p. 173]

Vitam ins cannot be synthesized by body and must therefore be supplied in the diet. Although vitamin Ki
(menaquinones) & biotin are synthesized by intestinal bacteria® o f most higher animal species.

107. B i.e. Niacin

Humans can produce some vitamins from their precursors consumed in food eg vitamin A (from beta carotene) and
niacin (from amino acid tryptophan).
Vitamin D is synthesized in skin from UV sunlight.

108. B i.e. Decreased proliferation of epithelial cells IR ef: B ern e & L ev y 6/e p 533, 5 3 8 -3 9 ; G a n o n g 2 4 /e p 4 7 3 ,4 8 5 ; jn w elz 2 3 /e p 198-991

Gut (colonic) microflora synthesize short chain fatty acids (SCFAs) by fermenting complex carbohydrate, resistant
starches and other components of dietary fibers & mucin that escape digestion in upper git. They also produce vitamin
K and biotin, detoxify xenobiotics, increase & maturate immunity, preventing allergy and growth of pathogenic bacteria. They
promote development of normal colonic epithelium and stimulate intestinal epithelial cell differentiation (ie increase
not decrease proliferation of epithelial cells)

109. C i.e. V it E [Ref: Guyton 12/e p 855]

Vitamin E (tocopherol) is the most potent (important) antioxidant in the bodyQ.


150 ■ A Complete Review of Short Subjects

GTT & Diet


110. C i.e. A

According to WHO

Plasma Glucose Condition Means


ISO-
125-
< 100 mg/dl NormalQ
100 - 110-125 Fasting Borderline
i
75- >125 mg/dl DiabeticQ
50- <180 mg/dl 1 Hour G TT Normal
25- < 140 mg/dl Normal
2 Hours GTT
140-200 Borderline
1 2 3 4
TtW(h) >200 mg/dl (Glucose DiabeticQ
A = Normal; B = Excessively rapid carbohydrate absorption; Tolerance Test)
C = Liver Disease

111. A i.e. RDA is statistically defined as two standard deviations (SD) above Estimated Average Requirement (EAR)
[Ref: Park's 20/ed pg- 547; Lippincott 4lh/e p. 357-58]

Recommended dietry allow ance (RDA) is set a t 2 SD above estim ated average requirement (EAR)®.

112. A i.e., Soluble fibers increases metabolism of sugar in G IT B i.e., Increases bulk of stool
113. C i.e. Colon 114. A i.e. Decreases stool transit time 115. A i.e. Lignin
116, B i.e. Cleaves only at (a 1-4) 117. D i.e. Riboflavin/Lactulose 118. B i.e. Pectin
[Ref: Ganong 22/e P- 508-510; Lippincott 4/e p. 365- 68; Advanced human nutrition 1/e p. 89; Bhagvan: Medical Biochemistry 3/e p.
154; Harper's 26/e P-474]

• Dietary Fiber consists of all plant cell wall components that cannot be digested by animals' own enzymes. These
include : 1) Cellulose^ 2) Hemicellulose Q 3) LigninQ 4) GumsQ 5) Pectins^ 6) PentosansQ
• High fiber diet aids water retention during passage of food along the gut, producing larger & softer faeces; thereby
also reducing the stool transit timeQ. Colon has the longest transit timeQ.
• A high fiber diet is associated with reduced incidence o f : a) diverticulosisQ b) cancer colon Q c) cardiovascular
disease^ d) diabetes mellitusQ

Lignin, a non carbohydrate type of Digestion /Hydrolysis by Digestion /


dietary fiber is neither Type of fibre enzymes in small intestine Fermenttion by colonic
digested/hydrolyzed (by bacteria
endogenous human enzymes) nor Soluble Pectin Aliginates Not digested Fermented
fermented/degraded by gastro "PA G " Gums
intestinal microorganisms. Insoluble Hemicellulose Not digested Fermented
"H CL" Cellulose
Lignin Not digested Not fermented

• a Amylase is a carbohydrate digesting enzyme found in saliva, pancreatic juice and intestinal juice. This enzyme hydrolyses a-
1—>4 glycosidic bonds.

119. C i.e Butyrates: [Ref. Harper 25/e P-656]

In herbivores, such as ruminants, fiber (mainly as cellulose) is a major source of energy after it has been digested by colonic
microorganisms to sm all chain fa tty acids (acetate, propionate and butyrate)Q which are absorbed into the portal vein.
Colonic fermentation may also contribute to human energy requirements.
Physiology Digestive System ■ 151

120. A i.e. Com -flakes [Ref. Journal of Nutritional Physiology & Biochemistry: Zeitschrift fur Natuforschung]
■ ft
Glycemic index in decreasing order (ie healthiness of food in increasing orders) is glucose (100), baked potato & com
flakes (85), white bread (70) and brown rice (55).

121. A i.e. Extraction of nutrients from food: [Ref: Sembulingum 3/e P-295-971

Extraction o f nutrients from fo o d is n ot m etabolism . M etabolism is oxidation o f nutrients from fo o d to produce energy.®

Metabolism is a combination of two processes, namely: Anabolism: synthetic reactions that build up substances.
$
Catabolism: reactions involving breakdown or oxidation of fuel molecules (breakdown of substrate). Both catabolism and
anabolism proceed only until equilibrium is achieved. Equilibrium is a mechanism for biological control of rate at which
these reactions occur.

122. C i.e. Adipose tissue [Ref: Lippincott 4/e p. 189, 329]

Fatty acid stored in adipose tissue in form o f neutral TAG, serve as the body's m ajor fu el storage reserve®.
a
123. B i.e. Glycogen [Ref: Lippincott 4th/ed pg-360; Harper 28,h/ed pg- 173,365-67] 124. A i.e. Lactulose

Main source o f energy in humans is carbohydrate. The primary role o f dietary carbohydrate is to provide enegyQ.
Lactulose is not broken dowin in G IT.

125. B i.e. T3 [Ref: Ganong 24/e p. 345]


With starvation, there is marked reduction in T 3 levels
200 -10
with a reciprocal rise in RT 3 (in thyroid profile).

160 -8

A = RT3; B=T3: C=T 4


* B
120 -6

1
80 -4

40 -2

L0
1------1------ 1------1------i------r
- 4 -2 8 2 4 6 8 10 +2 +4
Days
Chapter -6 EXCRETORY SYSTEM

QUESTIONS

Nephron & Glomerular Filtration C. PGF2 □


D. Vasopressin & Angiotensin II □
1. All of the following structures lie in the renal Following cells are responsible for acid secretion in
medulla, Except: (AIIMS 08) kidney- (Jipmer 02)
A. Juxtaglomerular apparatus □ A. I cells □
B. Loop of Henle □ B. P cells □
C. Collecting duct □ C. Mesangial cell □
D. Vasa Recta □ D. Pericytes □
2. Which of the following causes increase in both GFR 9. Which of the follow ing does NOT form a filtration
and renal plasma flow- (NBE P 13) barrier in nephrons: (NBE P 14; AI 08)
A. Increased plasma colloid osmotic pressure □ A. Podocytes □
B. Dilatation of afferent arteriales □ B. Mesangium □
C. Constriction of a afferent arterioles □ C. Endothelial cell □
D. Increased tubular pressure □ D. Basement membrane □
3. True about nephron /kidney is all except 10. All of these are actions of Atrial Natriuretic Peptide
(PGI 15,13; NBEP 14,15) except: (AIIMS 15)
A. Each kidney contains 1 million nephrons □ A. Afferent arteriole dilation □
B. DCT is 5mm long, 15% nephrons are B. Mesangial constriction □
juxtamedullary, filtration pressure in glomeruli is C. Decreased sodium absorption in PCT □
10 to 15 mm HG. □ D. Inhibition of sodium reabsorption in medullary
C. Effective renal plasma flow is 625 ml/min and collecting duct □
inulin clearance is same □ 11. W hich of the follow ing is freely filtered by Kidney
D. GFR increases when afferent arteriotes dilate and across glomerular capillaries: (PGI 09,07)
renal blood flow increases □ A. Albumin (across glomerular capillaries) □
4. Two substances that can probably be used to B. Globulin □
determine filtration fraction are - (NBE P 14,13) C. Creatinine □
A. Insulin and'mannitol (PGI 15,13,08) □ D. H C 0 3 □
B. Urea and diodrast □ E. Glucose □
C. PAH and phenol red □ 12. In a normal person at resting condition GFR is:
D. Inulin and PAH □ A. 125 ml/min (PGI 03) □
All are correct about glomerular filtration, except: B. 90 ml/min □
A. Glucose concentration of glomerular filtrate is C. 60m l/min □
same as plasma (AIIMS 11) □ D. 150ml/min □
B. Glomerular oncotic pressure is less than of 13. Inulin clearance closely resembles -
filtrate □ A. G. F. R. (NBE P 14) □
C. Constriction of afferent arteriole decreases B. Renal Plasma flow □
glomerular hydrostatic pressure □ C. Creatinine clearance □
D. Ureteric obstruction increases the hydrostatic D. P.A.H. clearance □
pressure of Bowman's space □ 14. Best test for GFR measurement is with;
All of the following statements about Renal A. Inulin (NBE P 15,13; PGI 03,15) □
physiology are true, Except: (AI 09) B. Hippuric acid □
A. Distal tubule always receives hypoosmotic C. Creatinine □
solution □ D. PAH □
B. The kidneys receive 5% of the cardiac output □ 15. In the formula for urea clearance, C = U x V/P, what
C. GFR is controlled by resistance in afferent and does U stands fon (AIIMS 15)
efferent arterioles □ A. Urinary concentration in g/24 hour □
D. The Glomerulus receives capillaries from the B. Urinary concentration in mg/ml □
afferent arteriole □ C. Urine osmolarity □
7. Relaxation of mesangial cells of kidney is brought D. Urine volume per minute □
about by - (NBE P15,14,13; AIIMS 15; Jipmer 02) 16. If the interstitial hydrostatic pressure is 2 mm
A. cAMP/Dopamine □ Hg,interstitial oncotic pressure is 7 mm Hg and
B. Endothelin □ capillary hydrostatic pressure is 25 mm Hg. What
Physiology: Excretory System ■ 153

should be the capillary oncotic pressure to allow a net Proximal Convoluted Tubule
filtration pressure of 3 mm Hg? (AIIMS 15)
A. 20 □ 24. Transport maximum (Tm) means - (WB 03, Jipmer 02)
B. 21 □ A. Maximum reabsorption & secretion □
C. 23 □ B. Maximum amount of glomerular filtration /min □
D. 27 □ C. Substance cleared from plasma/min □
17. What is implied, if a drug has more renal clearance D. Amount of toxic substances excreted /min □
than the GFR : (AIIMS 2K) 25. W hich of the follow ing has no Tm value -
A. Drug is reabsorbed in the tubules □ A. Albumin, arginine (AIIMS 07, DNB 14) □
B. Drug is secreted in the tubules □ B. Betahydroxybutyrate, glucose □
C. Drug is excreted in bile □ C. Glucose haemoglobin, phosphate □
D. Drug is neither secreted, nor resorbed. □ D. Sulfate, Uric acid □
18. Which of the follow ing statement is true? E. Urea □
A. Fluid coming from the descending limb of loop of 26. Tubular maximum for kidney in practice is actually
Henle is hypotonic (AIIMS 11, 08) □ less than the calculated value because -
B. Descending limb of loop of henle is permeable to A. Different nephrons have different transport
solutes □ maximum (AP 08, TN 07) □
C. If clearance of a substance is greater than GFR, then B. Depends on GFR □
tubular secretion must be present □ C. Depends on renal blood flow □
D. Clearance of a substance is always more than GFR D. Depends on blood Pressure □
if there is tubular secretion. □ 27. W hich of follow ing is not absorbed in PCT:
19. A substance is present in concentration of 2 mg% in A. Bicarbonate (AIIMS 10) □
the afferent arteriole and zero mg% in the efferent. B. Sodium □
True about the substance is: (AIIMS 02) C. Phosphate □
A. It is free filtered in glomerulus □ D. H+ □
B. Secreted in cortical nephron □ 28. Bicarbonate (H C 0 3 ) is maximally absorbed from:
C. Absorbed in PCT □ A. PCT (Indirectly) (AIIMS 07, 03,15,13) □
D. Impermeable in loop of Henk □ B. PCT (actively) (NBE P 15,14,13) □
20. The renal plasma flow (RPF) of a patient was to be C. Collecting duct □
estimated through the measurement of Para Amino D. Thick ascending loop of Henle □
Hippuric acid (PAH) clearance The technician 29. Active reabsorption of glucose occurs in the -
observed the procedures correctly but due to an error A. Distal tubule (NBE P 15,13; AP 08) □
in the weighing inadvertently used thrice the B. Proximal tubule □
recommended dose of PAH The RPF estimated is C. Loop of henle □
likely to be (AI03) D. Collecting ducts □
A. False-high □ 30. In the presence of vasopressin the greatest fraction of
B. False-low □ filtered water is re-absorbed which part of the
C. False-high or false-low depending on the GFR □ nephron: (AI 03, 01)
D. Correct and is unaffected by the PAH overdose □ A. Proximal tubule □
21. A negatively charged molecule is filtered with more B. Distal tubule □
difficulty compared to a positive one because- C. Loop of Henle □
A. Presence of negatively charged sialoproteins on the D. Collecting duct □
filtering membrane (NBE P 13) □ 31. W hich one of the follow ing statements regarding
B. Negatively charged molecules are larger □ water reabsorption in the tubules? (AI 05)
C. Postivelyl charged proteins on filtering A. The bulk of water reabsorption occurs secondary to
membrance □ N a+ reabsorption. □
D. Urine is acidic □ B. Majority of facultative reabsorption occurs in
22. The main cause of minimal change ds is: (NBE P 13) proximal tubule. □
A. Increase in pore size □ C. Obligatory reabsorption is ADH dependent. □
B. Loss of negative charge on membrane □ D. 20% of water is always reabsorbed irrespective of
C. Loss of cells □ water balance. □
D. Decreased circulation □ 32. The Principal site of absorption of Sodium is:
23. Substrate which is both secreted & filtered- A. Proximal convoluted tubule (AI 10; AIIMS 14) □
A. Uric Acid (NBE P 13) □
B. Distal convoluted tubule □
B. Glucose □ C. Loop of Henle (thick portion) □
C. Urea □
D. Collecting duct □
D. Na+ □
154 ■ A Complete Review of Short Subjects

33. The main site of bicarbonate reabsorption is: C. Osmotic gradient in medulla □
A. Proximal convoluted tubule. □ D. Secretion of uric acid □
B. Distal convoluted tubule. (AI 05) □ 43. Substance involved in countercurrent mechanism for
C. Cortical collecting duct. □ maintaining medullary gradient- (Orissa 04)
D. Medullary collecting duct. □ A. NaCl but no urea (NBE P 15,14,13)0
34. As fluid comes down the PCT, what is true- B. Urea □
A. Concentration of urea falls (NBE P 13) C. NaCi, urea but not Water □
B. Concentration of H C O 3 falls □ D. NaCl, urea, water □
C. Concentration of Na+ increases □ 44. The high sodium content of the filtrate in renal
D. Concentration of inulin decreases □ medulla is because: (AIIMS 06)
35. Following are true about HCOs-except: (PGI 08) A. At the loop of Henle, there is counter current
A. Extracellular concentration 25 m mol □ mechanism □
B. Intracellular concentration 10 m mol □ B. Increased blood flow to vasarecta □
C. 7.5% solution gives 2 nmols □ C. Increased excretion of Na+ from PCT □
D. In kidney, H C O 3- is produced by carbonic D. Increased absorption of Na from PCT □
anhydrase □ 45. Renal medullary hyperosmolarity is due to:
36. The most sensitive index for renal tubular function is: A. Increased Na+ (PGI 15,14, 04, 03) □
A. Specific gravity of urine (AIIMS 09) □ B. Increased Na content □
B. Blood urea □ C. Increased Glucose □
C. GFR □ D. Increased urea □
D. Creatinine clearance □ E. Increased Potassium □
37. A/E - a r e absorbed in the DCT: (AIIMS 07) 46. The prime driving force for counter current multiplier
A. Water □ system is: (AI 06)
B. Potassium □ A. Medullary hyperosmolarity □
C. Chloride □ B. Reabsorption of Na+ in thick ascending limb □
D. Sodium □ C. Action of ADH via aquaporin channels □
38. M ajor portion of glomerular filtrate is absorbed in - D. Urea recycling □
A. Loop of henle (AI 04) □ 47. Countercurrent mechanism is not seen in: (AIIMS 16)
B. Distal convoluted tubule □ A. Kidney □
C. Collecting duct □ B. Testes □
D. Proximal segment □ C. Eye □
39. Which of the following is true about Nephron D. Intestine □
function: (AI 09) 48. Which of the following is not a component of counter
A. Ascending thick limb is permeable to water □ current multiplier mechanism? (AIIMS 14)
B. Descending thin limb is impermeable to water □ A. Vasa-recta □
C. Osmolality of intra-tubular content in DCT is more B. Thick ascending limb of loop of Henle □
than surrounding interstitution □ C. Thin descending limb of loop of Henle □
D. Osmolality of intratubular content in PCT is D. Collecting duct □
isotonic to surrounding interstitium □ 49. Loop of Henle handles the following ions except:
40. Which of the following occurs along with glucose A. Na+ (PGI 07,14) □
transport into renal tubular cells: (AI 08, 01) B. K+ □
A. Sodium symport/Cotransport (NBE P 15,13) □ C. Cl- □
B. Sodium anteport □ D. Urea □
C. Potassium transport □
Distal Convoluted Tubule & Collecting Duct
D. Amino acid transport □
50. Macula densa in kidney is located in relation to -
Loop of Henle
A. PCT (NBE P 15,13;UPSC 0 7 ) 0
41. Counter current mechanism,all are true except- B. DCT □
A. Occurs due to opp flow in vasa recta and loop of C. Afferent arteriole □
henle (NBE P 13) D D. Efferent arteriole □
B. Seen in medullary nephron □ 51. The tubuloglomerular feedback is mediated by:fAI 06)
C. Helps in creating osmolarity gradient □ A. Sensing of Na+ concentration in the macula
D. Urea has no role □ densa □
42. Countercurrent mechanism in the kidney is B. Sensing of Cl+ concentration in macula densa □
responsible for- (NBE P 13) C. Sensing NaCl concentration in the maculadensa □
A. Absorption of glucose □ D. Opening up of voltage gated Na+ channels in
B. Maintenance of blood flow □ afferent arteriole □
Physiology: Excretory System ■ 155

52. Where in the kidney does acitve reabsorption of 62. Peristalsis of ureter depends on (AIIMS 07)
sodium ions occur - (AMU 01, UP 05) A. Sympathetic flow □
A. Collecting duct □ B. Parasympathetic flow □
B. Distal tubule □ C. Sympathetic and Parasympathetic flow □
C. Ascending limb of Henle □ D. Pacemaker in smooth muscle of renal calyces □
D. All of the above □ 63. Hypertonic urine is excreted due to absorption of
E. Only B and C □ water in: (AIIMS 03)
53. K+ secretion from loop of henle is decreased by- A. Collecting ducts □
A. Furosemide (N B E P 1 3 )H B. DCT □
B. Thiazide □ C. Ascending part of loop of Henley □
C. Spironolactone □ D. Descending part of loop of Henley □
D. Acetazolamide □ 64. Urinary concentrating ability o f the kidney is
54. Alodosterone mainly acts upon- (NBE P 13) increased by: (AI 05)
A. PCT □ A. ECF volume contraction. □
B. Loop of Henle □ B. Increase in RBF. □
C. Glomerulus □ C. Reduction of medullary hyperosmolarity. □
D. Collecting duct □ D. Increase in GFR. □
55. Where does ADH not act- (NBE P 13) 65. Substance that is completely reabsorbed from the
A. PCT □ kidney- (NBE P 13)
B. Collecting duct □ A. Na+ □
C. Collecting tubules □ B. K+ □
D. DCT □ C. Urea □
56. All of these are correct of renal physiology except: D. Glucose □
A. Sodium absorption occurs in DCT (PGI 02) □ 66. Least reabsorption out of- (NBE P 13)
B. Potassium is both secreted and absorbed in A. Glucose □
tubules □ B. Urea □
C. Glucose is reabsorbed in DCT □ C. Na+ □
D. Hb is not excreted as it is a large molecule □ D. H C 0 3 □
E. Amino acids are reabsorbed in CD □
57. The Principal site of acidification of urine is: Urine
A. Distal convoluted tubule (AI 04) □
B. Proximal convoluted tubule □ 67. Elimination of waste product from a normal person
C. Loop of Henle □ requires minimal amount of urine of: (PGI 05)
D. Collecting duct □ A. 100 ml □
58. Which one of the following is not responsible for B. 500 m □
concentration of urine in the kidney - (DNB 13) C. 200 ml □
A. Aldosterone □ D. 2000 ml □
B. Angiotensinll □ 68. Normal excretion of protein in urine per day is:
C. Vasopressin □ A. 100 mg (A I0 4 )D
D. Epinephrine □ B. 150 mg □
59. Several hormones regulate the tubular reabsorption of C. 400 mg □
water and electrolytes at different sites in the D. 600 mg □
nephron. Which of the follow ing combination is 69. The urge for micturition is felt when the bladder is
correct? (AIIMS 03) filled with - (DNB 05)
A. Angiotensin in distal tubule □ A. 100-200cc of urine □
B. Aldosterone in collecting ducts □ B. 200-300cc of urine □
C. ADH in proximal tubule □ C. 500-700cc of urine □
D. ANP in loop of Henle □ D. 700-800cc of urine □
60. Functionof vasopressin- (NBE P 13) 70. In a normal adult the marked desire for micturition is
A. Absorption of water in PCT □ felt - (AIIMS 06, DNB 09)
B. Absoption of water in loop of henle □ A. When about 100-200 cc f urine has collected in the
C. Absorption of water in collecting duct □ bladder □
D. Absorption of Na+ in PCT □ B. When about 300 - 400 cc of urine has collected in
61. ANP acts at which site- (NBE P 13) the bladder □
A. Glomerulus □ C. When about 600-800 cc of urine has collected - in
B. Loop of Henle □ the bladder □
C. PCT □ D. When about 1000 cc of urine has collected in the
D. Collection duct □ bladder □
156 ■ A Complete Review of Short Subjects

71. True statement among the follow ing is - (DNB 06) C. Lactic acidosis □
A. Bladder emptying cannot occur if volume < D. DKA □
100 ml. □ E. Cardiac failure □
B. Bladder muscles contains intrafusal fibres □ 80. Normal anion gap metabolic acidosis is seen in:
C. Pressure increase in blader very linerly with A. Diarrhoea (AIIMS 02) □
time □ B. Renal failure □
D. One time of fluid, intake :results in complete C. Lactic acidosis □
excretion in 1 hour □ D. Diabetic ketoacidosis □
81. In severe exercise, decrease in pH is due to-
Water and Acid Base Balance A. Respiratory acidosis (NBE P 13) □
B. Lactic acidosis □
72. Daily loss of water from skin in absence of sweating C. H+ retension □
is OR Insensible water loss per day is: D. H C O 3 excretion □
A. 1 litre (NBE P 15,13; AUMS 04) □ 82. Blood buffers are: (PGI 02)
B. 1.5 litres □ A. Bicarbonate □
C. 200-300 ml □ B. Plasma proteins □
D. 500-700 ml □ C. Phosphates □
73. Findings of Syndrome of Inappropriate Antidiuretic D. Hemoglobin □
Hormone Secretion (SIADH) includes (PGI 12) E. Oxygen
A. TUrine Na+ □ 83. A newly posted junior doctor had difficulty in
B. TSerum Na- □ finding out base deficit/excess for blood in a given
C. TUrine osmolality □ patient. An experienced senior resident advised a
D. TSerum osteolality □ quick method to determine acid base composition of
E. Postural hypotention □ blood based on PCO 2. Which of the following is the
74. True about free water clearance is: (AIIMS 07) likely method he suggested to predict acid base
A. Regulated by ADH □ composition of blood? (AI 04)
B. Regulated by aldosterone □ A. Red ford normogram □
C. Increased by furosemide □ B. DuBio's normogram □
D. None of the above □ C. Goldman constant field equation □
75. In metabolic acidosis, which of the following changes D. Siggard-Andersen normogram □
are seen: (AI 02) 84. If you calculate the plasma osmolality of a child with
A. Increased K+ excretion □ plasma Na+ 125 mEq//, glucose 108 mg/ dl, and BUN
B. Decreased K+ excretion □ (blood urea nitrogen) 140 mg/ dl, the most appropriate
C. Increased Na+ excretion □ answer would be:
D. Increased Na+ reabsorption □ A. 300 mOsm/kg (AIIMS 03) □
76. All of the following statements are correct about B. 306 mOsm/kg □
potassium balance, except: (AI 03) C. 312 mOsm/kg □
A. Most of potassium is intracellular □ D. 318 mOsm/kg □
B. Three quarter of the total body potassium is found
in skeletal muscle □ Endocrine Functions Of Kidney
C. Intracellular potassium is released into extra­
cellular space in response to severe injury □
85. Erythropoietin is secreted from- (NBE P 13)
D. Acidosis leads to movements of potassium from
A. Juxtraglomerular cells □
extracellular to intracellular fluid compartment. □
B. Macula densa □
77. Anion gap is mostly due to: (AI 06)
C. Interstitial cells □
A. Proteins □
D. Glomerulus □
B. Sulphates □
86. Erythropoietin secretion occurs when: (NBE P 13)
C. Phosphates □
A. i Tissue p 0 2 concentration □
D. Nitrates □
78. M etabolic alkalosis is seen in all except - (AI 08) B. T Tissue pH □
A. Thiazide diuretics □ C. T Tissue p C 0 2 concentration □
B. Uretersigmoidostomy □ D. T Hemoglobin □
C. Prolonged vomiting □ 87. Erythropoietin is secreted by A/E: (PGI 05)
D. Systemic antacid therapy □ A. Hemangioblastoma □
79. Increased anion gap is seen in: (PGI 03) B. Hepatoma □
A. Enterocutaneous fistula □ C. Renal cell carcinoma □
B. llleostomy fistula □ D. Adrenocortical tumours □
Physiology: Excretory System ■ 157

88. True about function of Angiotensin II: (PGI 10) 96. Renin is secreted b y : (NBE P 15,14,13; AIIMS 07,13)
A. Constriction of afferent arteriole □ A. PCT □
B. Autoregulation of GFR □ B. DCT □
C. Secreted from endothelial □ C. Collecting duct □
D. Release aldosterone □ D. Juxatglomerular apparatus □
E. Increased sodium and water reabsorption □ 97. M acula densa in kidney is located in relation to -
89. W hich is true about rennin: (PGI 03) A. PCT (NBE P 15,13) □
A. It helps to convert angiotensinogen to B. DCT □
Angiotensin-I □ C. Afferent arteriole □
B. Secreted by PCT □ D. Efferent arteriole □
C. T GFR causes T secretion of rennin □ 98. Lacis cells are located at- (NBE P 13)
D. -I plasma Na+ & H 2O □ A. JGA □
90. True regarding rennin angiotensin system is A/E B. Proximal tubule □
A. Angiotonin is octapeptide (PGI 09) □ C. Distal tubule □
B. ACE splits Leucine-valine bond □ D. Loop pf henle □
C. Aspartic acid is essential for rennin activity □ 99. True about juxta-glom erular apparatus of kidney:
D. PRA is better than PRC □ A. Important source of angiotensiogen (PGI 2 K )0
E. Catalysis site of ACE contain Zn++ □ B. Important source of rennin □
91. Physiologically inactive form is (WB 09) C. Consist of macula densa □
A. Angiotensin I □ D. Consist of afferent arteriole forming lacis cell □
B. Angiotensin II □ 100. Juxtaglom erular apparatus lies in relation to:
C. Angiotensin III □ A. Proximal convoluted tubule (AIIMS 07) □
D. Angiotensin IV □ B. Ascending loop of Henle Cl
92. True is all except (PGI 09) C. Descending loop of Henle □
A. Renin - liver □ D. Glomerulus □
B. Renin- kidney □ 101. Vasopressin acts by: (PGI 08)
C. Renin substrate - liver □ A. Water transport across collecting duct □
D. ACE- lung endothelium □ B. Water absorption at medullary ducts □
E. Angiotensinogen - Renal □ C. Water secretion at loop of Henle □
93. Angiotensin II causes all of the follow ing, Except: D. Water transport at PCT □
A. Stmulation of Thirst (AIIMS 11, A I 09) □ 102. W hich one of the follow ing is not a transport or
B. Increased ADH secretion □ binding protein? (AIIMS 05)
C. Vasodilatation □ A. Erythropoietin □
D. Aldosterone secretion □ B. Ceruloplasmin □
94. W hich causes raised angiotensin in blood- (NBE P 13) C. Lactoferrin □
A. Increased blood volume □ D. Transnsferrin □
B. Raised cardiac output □
C. Decreased blood pressure □
D. Increased sympathetic tone □
95. A/E one results in increased secretion of Renin
(AIIMS 14,13; NBE P 14,13)
A. Renal ischemia, and sympathetic nerve
stimulation □
B. Decreased amount of Na+ in DCT. □
C. Decreased amount of Na+ in PCT. □
D. Narrowing of afferent arterioles (or low
pressure) □
158 ■ A Complete Review of Short Subjects

ANSWERS & EXPLANATIONS:


Nephron & Glomerular Filtration
1. A i.e. Juxtaglomerular apparatus [Ref: Ganong 23rd/e p. 640-43; Gabriel C Ezeilo p. 208-10; Guyton lV h/e p. 309]

Loop of Henle, collecting duct & vasa recta lie in renal'medulla (pyramid) whereas, JG apparatus (JG cells, lacis cells,
macula densa of DCT) lie in renal cortex.

2. B i.e. Dilatation of afferent arteriales 3. C i.e. Effective renal plasma flow is 625 ml/min & inulin clearance is same
4. D i.e. Inulin & PAH [Ref: Ganong 24/e p. 675-80; Guyton 11/e p. 308-13]

Each kidney contains 1-1.3 m illion nephronsQ. PCT is 15mm and DCT is 5mm long, 15% are juxtamedullary and 85%
£
are cortical nephronsQ.
GFR increases with afferent arteriole dilation and increased renal blood flo w Q. Normal GFR measured by inulin
clearance (best) or creatinine clearance is 125ml/min (7.51/hr or 1 8 0 1/day). Net filtration pressure is 10 to 15mm Hg.
Renal plasma flow (RPF) i.e. renal blood flow or circulation can be measured by infusing PAH (P-aminohippuric
acid). Effective RPF is 625 ml/min and actual RPF is 700 ml/minQ.
Because filtration fraction is GFR/RPF (i.e. 0.16 to 0.20). It can be measured by inulin/creatine (for GFR) and PAH
(for RPF)Q.

5. B i.e. Glomerular oncotic pressure is less than of filtrate 6. B i.e. The kidneys receive 5% of the cardiac output
[R ef: G u y ton 12/e p. 3 1 4 -1 5 ; l l t h / e p. 31 4 , 3 1 9 ; G an on g 2 3 rd je p. 653- 57 ; G abriel p. 220, 211- 13; D isea se o f k id n ey & u rin ary tract 8 /e p g -1 0 ]

- Kidney receives 25% o f cardiac output at rest ® (i.e. 1.2 to 1.3 litres blood per minute). ☆
- Glucose is freely filtered across glomerular capillary membrane and therefore glucose concentration o f glomerular
filtrate is sam e as that o f plasma®.
- Glomerular (capillary) oncotic pressure (d/t plasma protein content) is higher than that of filtrate oncotic pressure in
Bowman's capsule (with almost O protein content).
- Constriction o f afferent arteriole decreases glom erular hydrostatic pressure (& GFR)®, whereas afferent arteriole
dilation & efferent arteriole constriction increase it.

7. A i.e. C-Amp & Dopamine 8. A i.e. I cells 9. B i.e. Mesangium [R ef: G a n o n g 2 4 / 707; R ob bin s 7/ 9 5 7 -9 5 8 ; M T x l

- M esangial cells do not form filteration barrier®. Relaxation o f mesangial cells is brought about by C-AMP, ANP,
dopamine and PGE 2 ®.
- Intercalated (I) or brown cells secrete acid & transport HCO 3’ in kidney®.

Specialized Cells of Nephron Mesangial Cells


I---------------------------- - Mesenchymal origin
In Glomerulus In Collecting Duct - Support® the entire glomeular
1 I------------------------------- 1 tuft.
Stellate /Mesengial cells are contractile, Principal (P) cells Intercalated (I) or - Secrete mesangial matrix®.
locate between basal lamina & Are more numerous Brown cells
- are contractile®
endothelium and regulate GFR. Agents relatively taller cells Are present in smaller
- are phagocytic®
causing their with few organelles, number, have more
- Capable o f proliferation®
I------------------------- 1 They are involved microvilli, cytoplasmic
Contraction Relaxation in Na* reabsorption & vesicles, & Filteration Barrier
- Endothelins - Dopamine vasopressin mitochondria. They are
- Angiotensin II stimulated water concercerned with acid The glomeular capillary wall is the
- ANP
- Vasopressin reabsorption secretion & "Filtering membrane" of nephron. It
- CAMP®
- Norepinephrine HCOs'transport® consists of following structures:
- PGE2
- Platelet activating Type I medullary interstitial cells 1. Fenestrated endothelial cells®
factor contain lipid droplets & secrete 2. Glomerular basement
- Platelet derived prostaglandin PGE? membraneQ (Consist of Type IV
growth factor Podocytes form epithelium of Bowman's collagen®)
- PGF2 capsule which along with capillary 3. Visceral epithelial cells®
- Throm boxane A 2 endothelium forms filterating membrane. (Podocytes).
- Leukotrienes C 4 & D 4 These layers are separated by basal
- Histamine lamine.
Physiology. Excretory System ■ 159

10. B i.e. M esangial constriction

ANP, BNP and CNP (on injection) increase Na+ excretion (i.e. cause natriuresis) by dilating afferent arterioles and
relaxing mesangial cells®.

Natriuretic Peptide (NP) Isolated from


Atrial NP (ANP) Heart (Atria), BrainQ
Brain (B type) NP (BNP) Ventricle (main), Brain
C- type NP (CNP) Brain, Pituitary, Kidney, Vascular endothelium

11. E i.e. Glucose; C i.e. Creatinine [Ref: Ganong 22/e p. 705-07; Guyton & Hall 11/e p. 316 -18; Sembulingam 4je p. 290-93]

Electrolytes such as sodium (Na+), water and small orgaic compounds like glucose, inulin and creatinine are freely
filtered across glomerular capillaries (i.e. filterability =1).

12. A i.e. 125ml/min [Ref: Ganong 22/e P-706; Guyton 10/e P2911

G FR in an normal person at resting condition is 125ml/min or 7.5 U hour or 180UdayQ. Whereas the normal urine
volume is about 1L/day. Thus 99% of filtrate is reabsorbed.

13. A i.e. GFR 14. A i.e. Inulin [Ref: Ganong 24je P-679J

ItiulinQ is the standard substance and extensively used to measure GFR. Clearance o f creatinine (C&) can also be used
for GFR measurement. However, clearance of creatinine will be slightly higher than inulin because of some tubular
secretion. Pcr (plasma creatinine) is an index of renal function.

15. B i.e. Urinary concentration in mg/ml

Renal Circulation Glomerular Filtration


Kidney receives 1.2-1.3 L/min blood ® (25% of cardiac Normal GFR is 125ml/minQ or 7.5L/hour or 180L/day. So per
output) at rest.Q day kidney filters fluid equal to 4 times of total body water, 15
Renal plasma flow (RPF) equals the amount of times of ECF, & 60 times of plasma volume.
substance excreted per unit of time divided by Inulin Q& creatinine can be used to measure GFR
arteriovenous difference for the substance across the
GFR - K f[{P cc ? t ) (^ gc 71 t t
kidney (as long as hematocrit is same).____________
Kf = glomerular ultrafiltration cofficient (it is product of
U c*V permeability & effective filtration area)
RPF =- Excretion ratio = S rac S rvc
S rac ~ S rvc ’ RAC Pec & P t = Hyprostatic pressure (P) in glomerular capillary
Uc = Concentration of substance in urine (mg/mL)Q, (gc) & tubule (t) respectively
V = Urine flow (ml/min), S r a c & S r v c = substance renal k gc & K T = Osmotic pressure ( n ) of plasma in glomerular
arterial & venous concentrations respectively. capillary (GC) & tubule (T) respectively.
• RPF can be measured by infusing p-am ino hippuric Net filtration pressure (P uf ) is 15 mm Hg0
acid (PAH)Q. As excretion ratio of PAH is high (0.9)
Puf ~ Pgc -n,G C
and its 90% is removed from artery in a single
circulation through kidney. And its renal arterial Afferent end Efferent end
concentration is equal to plasma concentration, so RPF (mmHg) (mmHg)
by PAH can also be measured by this formula, which is P gc 45 45
k/a effective renal plasma flow (ERPF) PT 10 10
Ur* x V 20 35
ERPF — ~ 625 mL/minO k gc

PAH plasm a concentration P uf 150 0

★ PT = Pressure in Bowman's capsule


ERPF 630
Actual RPF = = 700ml/min® • Filtration fraction is ratio of GFR/RPF, and is normally
Excretion ratio 0.9
0.16- 0.20
160 ■ A Complete Review of Short Subjects

16. A i.e. 20 [Ref: Ganong 24/e p. 680]

Hydrostatic Pressure in Glomerular Capillary (Pgc) _ Hydrostatic


Pressure in tubule interstitium (Pt) - Glomerular Capillary Oncotic
or Osmotic Pressure (ncc)

or PUP = P g c ~Pt ~ I I g c 3 = 25 - 2 - PIcc


3 = 23 - n cc
So P Ig c = 20

17. B i.e. Drug is secreted in the tubules [Ref: Gabriel C Ezeilo p. 212-15; Guyton llth/355-56; Ganong 23rd/645-47; KDT 5/e P 27]
18. C i.e. If clearance of a substance is greater than GFR, then tubular secretion must be present

If a drug is more in renal clearance than the amount filtered by glomerulus it If renal clearance is Tubular
has to be secreted in renal tubules more than GFR secretionQ
Tubular secretion - This is the active transfer of organic acids and bases in the If renal clearance is Tubular
proximal tubules. less than GFR reabsorptionQ
If renal clearance of drug is greater than 120 ml/min (GFR), additional tubular
secretion can be assumed to be occurring.

19. A i.e. It is freely filtered in glomerulus [Ref: Ganong 22/e P- 684]

If the substance is O mg% in efferent arteriole, it means that all of it has freely filtered in glomerulus.

20. B i.e. False -low

At low plasma PAH concentration, the measured RPF (renal plasma flow) is accurate. Whereas, at high (>30mg/dl)
plasma PAH concentrations the estimated RPF is falsely low because of saturated secretary mechanism & TM.

21. A i.e. Presence of negatively charged sialoproteins on the filtering membrane


22. B i.e. Loss of negative charge on membrane [Ref: Mtx; Ganong 23/ep 646]

The negative charges of basement membrane & podocytes restrict and repel large negatively charged and attract
positively charged moleculesQ. Negatively charged large molecules are filtered less easily than positively charged
molecules o f equal m olecular size®, and even neutral molecule are filtered much more easily than negatively charged
molecules of same size. So, negatively changes. M ain cause of minimal change nephropathy (disease) is loss of negative
charge on basement membrane^.

23. A i.e. Uric Acid [Ganong 23/ep 648-50]

Substrate which are both filtered (secreted) & absorbed are Creatinine, Uric acid & potassium (K+)Q. Mn "Sec UK or
CUK".

Proximal Convoluted Tubule

24. A i.e. Maximum reabsorption & secretion 25. E i.e. Urea [Ref: Guyton 11/e p. 330 - 32; Ganong 24/e p. 682-831
26. A i.e., D ifferent nephrons have different transport maximum 27. D i.e. H+ .
h T
Transport maximum (Tm) means maximum reabsorption & secretion Q. Tmfor kidney in practice is actually less than the
calculated value because not all nephrons have sam e T„fi. Passively absorbed substances e.g. urea, chloride and w ater do
not exhibit TmQ.

Actual renal threshold for glucose is less than its predicted value.
Physiology: Excretory System ■ 161

28. A i.e. PCT (indirect) 29. B i.e. Proximal tubule 30. A i.e. Proximal tubule
31. A i.e. The bulk of water reabsorption occurs secondary to Na+reabsorption 32. A i.e. Proximal convoluted tubule
33. A i.e. Proximal convoluted tubule 34. B i.e. Concentration of H C O 3 falls
[Ref: Ganong 24/e P- 685,22/e p. 717; Guyton 12/SAE p. 480-90; Sembulingum 3/e P- 254-55]

- Proximal convoluted tubule (early part) reabsorbs almost all of glucose, amino acids, protein, lactate & inorganic
phosphate and most of bicarbonate/ HCO3- (80-90%), Na+ (60%), H2O, K+ and Cl- ®. PCT is also an important site for
secretion of organic acids and bases such as oxalate, urate, bile salts, H+ and catecholamines, along with toxins,
drugs eg penicillin, salicylates and PAH. So its' a work horse of nephron®.
- Absorption of HCO3-UI PCT is indirect i.e. coupled with H+ Secretion®
- Greatest fraction of filtered water (60-70%) is reabsorbed in proximal tubule irrespective of presence or absence of
vasopressin®. Proximal convoluted tubule reabsorbs approximately 50-60% of water (bulk of water), which is
secondary (obligatory) to Na+reabsorption®.

35. C i.e. 7.5% solution gives 2n mols [Ganong 22/e P-686, 720]

Concentration of bicarbonate • In kidney bicarbonate is produced by carbonic anhydrase®.


- Intracellular : 10 mmol/LQ |TT ^ “ [ TT ^ | “ IT^
- ExtraceManZZmmol/LQ H j0 + C 0 ! ------------------------------ * P " * HCO,-

Carbonic anhydrase® Spontaneous

36. A i.e. Specific gravity of urine

The main function of renal tubules is concentration of urine and this can be measured by specific gravity of urine.

37. B i.e. Potassium [Ref: Ganong 22/e P- 724]

Potassium once filtered in the tubular fluid is reabsorbed only in the proximal tubules. Nowhere else it is reabsorbed so it
is not reabsorbed in the DCT, rather it is secreted into the tubular fluid in DCT®.

Water reabsorption in DCT: Early part of distal tubule is impermeable to water. Late DCT becomes perm eable under the
influence o f ADH.

38. D i.e. Proximal segment 39. D i.e. Osmolality of intratubular content in PCT is isotonic to surrounding interstitium

41. D i.e. Urea has no role 42. C i.e. Osmotic gradient in medulla 43. D i.e. NaCl, Urea, Water
44. A i.e. At the loop of Henle there is counter current mechanism
45. A i.e. Increased Na+; D i.e. Increased urea; E i.e. Increased K+
46. B i.e. Reabsorption of Na+ in thick ascending limb [Ref: Ganong 24/e p. 687-89; Guyton 12/SAE p. 482-87; MTx]
162 ■ A Complete Review of Short Subjects

- Counter current mechanism is seen in medullary nephronsQ, are occurs due to opposite flow in loop of Henle & vasa- " f t
rectaQ. Counter current multiplier at Loop of Henle generate and counter current exchanger at vasa-recta of medullary
capillaries maintain medullary osmotic pressure gradient^
- Renal medullary hyperosmolarity is maintained by counter current mechanism 1/t tNa+, Cl-, Urea & K+ concQ. The
Counter current multiplier mechanism at the loop of Henle is responsible for development of hyper osmolarity of
medullary interstitial fluid and medullary gradient. Hyperosmolarity is due to high Na+ conc. of the filtrate in renal
medullaQ.
- T h e operation of each loop of Henle as a counter current multiplier depends on the active transport of Na+ and Cl­
out of its thick ascending limb, high permeability of its thin descending limb to water and the inflow of tubular fluid
from the proximal tubule with outflow into the distal tubule'. The prime driving force for counter current multiplier is
reabsorption of Na+ in thick ascending limb. The prime objective for counter current multiplier system is to produce
Medullary hyperosmolarity.
\
47. C i.e. Eye [Ref: Ganong 24/ep 419, 687]

Counter current mechanism (CCM) is a system in which inflow and outflow run parallel & opposite (i.e. counter to) each
other in close proximity to create a concentration gradient. It is seen in many mammalian organs e.g. kidney, testis (b/w
spermatic arteries & pampiniform plexus) and gut (hair pin arrangement of arteries & vein in intestinal villi where O 2
directly diffuses from arterioles to veins).

48. A i.e. Vasa-recta

Loop of Henle (& collecting duct) Involved in Counter Current M ultiplier Mechanism
Vasa recta Involved in Counter Current Exchanger Mechanism.

Maintenance of Osmolarity

Medullary Gradient ~^T Counter Current Mechanism


>The osmolarity of cortical Counter current flow is flow in which the inflow runs parallel to, counter to, & in close
interstitium is same as that of proximity to the outflow for some distance._______
plasma (300 mosm/L). However Counter Current Multiplier Counter Current Exchanger
osmolality of medullary interstitium >Is formed by the lo o p ofH enleQ. Loop of Henle • Formed by the v a sa rectaQ.
is very high (1200 mosm/L) multiplies the Na+concentration within medulla, by Vasa recta retains the Na+ and
>Gradual increase in the osmolarity retaining the new Na+coming from glomerular urea in the medullary
(from 300 to 1200 mosm/L) of filtrate. interstitium & removes water
medullary interstitium is called • G radient o f increasing o sm o la lity along the from it. Since Na* & urea are
medullary gradient medullary pyramid is producedQ by operation of the exchanged for water between
>Counter - current mechanism is lo op s ofH en le^ as counter current multipliers. It acts ascending & descending limb
responsible for maintenance of by of vasa recta it is called
medullary gradient i.e. exchanger.
hypeosmotic renal medullary Active High Additional inflow of • G radient is maintainedQ by the
interstitium. transport of permeability tubular fluid into operation of v a sa rectaQ as
’ Factors responsible for Na+ and Cl­ of the thin the proximal tubule. counter current exchanger.
Medullary Interstitium out of its descending This causes the - The solutes diffuse out of the
Hyperosmolarity thick limb to water hyperosmotic fluid vessels conducting blood
1. Active transport of N a* & c o ­ ascending This allows initially formed in towards the cortex and into the
transport o f Cl', K*Q & other ions loop osmosis of the descending limb vessels descending into the
out of thick ascending limb of This initiates water out of to flow into pyramid
loop of Henle into medullary the increased the ascending limb. - Water diffuses out of the
interstitium. osmolarity of descending Once this fluid is in descending vessels & into
2. P assiv e diffu sion o f ureaQ from the renal limb thus ascending limb, ascending vessels
inner medullary collecting ducts medulla achieving further active - Therefore, the solute (Na*, CF,
into the medullary interstitium. hyperosmotic transport enhances K+, Urea) tends to recirculate in
3. D iffusion o f only sm a ll am ou n t o f fluid in the the medullary medulla and water tends to
w ater from m edu llary tubules Q descending osmolarity. bypass it, so that hypertonicity
into medullary interstitium (far limb is maintained
less than resorption of solutes into These steps are repeated over and over, with the net - It is a passive process and could
medullary interstitium) effect of adding more and more solute to medulla and not maintain gradient if the
4. Active transport of ions from creating a hyperosmotic renal medulla. process of counter current
collecting duct into medullary exchanger were to cease.
interstitium.
Physiology: Excretory System ■ 163

Loop of Henle as a counter current m ultiplier producing a gradient of Vasa recta as counter current
hyperosmolarity in the medullary interstitium (MI). exchangers in the kidney.
TDL Ml TAL

A 4 t B C ▼

300 300 300 40 0 4 0 0 4 — 20 0 300 300 200 350 3504— 150


300 300 300 40 0 4 0 0 4 — 20 0 300 300 200 350 350-4— 150
300 300 300 40 0 4 0 0 - 4 - 20 0 300 300 200 350 3504— 150
300 300 300 400 40 0-4— 200 300 300 200 350 3504- 150
300 300 300 400 4 0 0 4 -2 0 0 400 400 400 500 5004- 300
300 300 300 400 4 0 0 4 — 20 0 , 400 400 400 500 5004— 300
300 300 300 400 4 0 0 4 — 200 400 400 400 500 5004— 300
300 300 300 400 4 0 0 4 — 200 400 400 400 500 5004- 300

4
E i t F G t H

300 300 150 325 325 4— 125 300 300 125 312 3 1 2 4 — 112
300 300 150 325 325 4 - 125 325 325 225 375 37 5 4 - 175
350 350 300 425 425 4 - 225 325 325 225 375 3 7 5 4 - 175
350 350 300 425 425 4— 225 425 425 225 425 425 4 — 225
350 350 300 425 425 4— 225 425 425 225 425 425 4 — 225
350 350 300 425 425 4— 225 425 42 5 400 513 5 1 3 4 -3 1 3
500 500 500 600 600 4— 40 0 425 42 5 400 513 5 1 3 4 — 313
500 500 500 600 600 4— 40 0 600 600 600 700 700 4 — 500

TDL, thin descending limb; TA L, thick ascending limb. The process of


generation of the gradient is illustrated as occurring in hypothetical steps, N aCl and urea diffuse out of the
starting at A, where osm olality in both limbs and the intestitium is 300 ascending limb of the vessel and into
m som /kg of water. The pum ps in the thick ascending limb m ove Na+ and Cl­ the descending limb, w hereas water
in to the interstitium , increasing its osm olality to 400 M osm /kg, and this diffuses out of the descending and
equilibrates with the fluid in the thin descending limb. H ow ever, isotonic fluid into the ascending limb o f the
continues to flow into the thin descending limb and hypotonic fluid out o f the vascular loop.
thick ascending limb. Continued operation of the pumps makes the fluid
leaving the thick ascending limb even m ore hypotonic, while hypertonicity
accum ulates at the apex of the loop.

49. D i.e. Urea [Ref: Ganong 22/e P- 714; Sembulingum 3/e P- 258]

- f t
In the thick ascending limb, a carrier co-transports one Na+, one K + and 2CL® from the tubular lumen into the tubular
cells.

50. B i.e. DCT 51. C i.e. Sensing NaCl concentration in the macula densa [ Ref:Guyton l l ih/323, 324; Ganong 22/e p 700-13]

'To perform the function of auto regulation, the kidneys have a feed back mechanism (tubuloglomerular feed back)
that links changes in sodium chloride concentration at the macula densa (tubular component) w ith the control of renal
arteriolar resistance (glomerular component). Macula densa is located in DCT®.

52. D i.e. All 53. C i.e. Spironolactone 54. D i.e. Collecting duct
55. A i.e. PCT 56. C i.e. Glucose is reabsorbed in DCT; D i.e. Hb is not excreted ..; E i.e.Amino acids are reabsorbed...
[Ref: Ganong 22/e P- 709, 711;Chaudhuri 5/e P- 406, 412; Shinde & Chatterjea 6/e P-658]

- Na+ reabsorption occurs in all parts of nephron except descending lim b o f LH®. K+ reabsorption takes place in PCT

(maximum) and thick ascending lim b of LH®. K+ is secreted in thin descending limb of LH & DT.
- Glucose, amino acids, bicarbonates and Na+ are reabsorbed maximally in early PCT®.
Potassium sparing (retaining) natriuretics are diuretics which prevent loss of K +(unlike other diuretics) by inhibiting
aldosterone (so Na+-K+ Exchange) in collecting duct (e.g. spironolactone)® or by inhibiting ENaCs (e.g. amiloride).
Hemoglobinuria - excretion of free Hb follow ing increased plasm a level of free Hb (Hem oglobinem ia). There is renal
threshold for Hb. W hen this threshold is exceeded, hem oglobinuria results. If the kidneys are normal, the threshold is
about 155tng% of blood plasma. But it m ay be lowered when kidneys are dam aged. Hb excretion is n ot related to its
m olecu lar structure®.
164 ■ A Complete Review of Short Subjects

57. D i.e. Collecting duct [Ref: Ganong 22/e P- 720; Chaudhuri 5/e P -411]

Acidification of tubular fluid occurs in distal nephron, particularly the collecting duct, (CD).

58. D i.e. Epinephrine 59. B i.e. Aldosterone in collecting duct 60. C i.e. Absorption of water in collecting duct
61. D i.e. Collection duct [Ref: Guyton 10/e P 290, 304, 305, 335]

Hormone Site of Action in kidney Effect


ADH (Vasopressin) T Water reabsorption
Cortical collecting duct & Distal tubuleQ
Aldosterone T Water & NaCl reabsorption, K+Secretion
ANP M edullary collecting duct & Distal tubuleQ •iNaCl reabsorption
Angiotensin II Constricts Afferent Arterioles Reduce GFR
PT, DT, CD & Thick ascending limb TWater & NaCl reabsorption

62. D i.e. Pacemaker activity in smooth muscles of renalcalyces


[Ref: Guyton 11/e p. 312; Georgl Tanner: Medical Physiology 2/e p. 423, www.pubmedcentral.nih.gov/articlerender.fegi?
arcidzll68135]
• W alls o f renal calyces & pelvis contain specialized sm ooth muscle with inherent pacem aker activity 0. Urine flowing
from collecting duct into renal calyces stretches it and increases their inherent pacemaker activity which inturn initiates
peristaltic contractions Q that spread to renal pelvis & then downwards along the length of ureter.
• Walls of ureter contain smooth muscle and are innervated by both parasympathetic and sympathetic nerves as well as
by intramural plexus of neurons. As with other smooth muscles, peristaltic contractions in ureter are enhanced by
parasym pathetic stimulation & inhibited by sym pathetic stim ulation <3

63. A i.e. Collecting duct [Ref: Ganong 22/e P-716]

64.
Osmolality of urine depends on the action of vasopressin on the collecting ductsQ,

A i.e. ECF Volume Contraction [Ref: Guyton 10/e P 315]


$
The kidney has the ability to form urine
that is more concentrated than the Prerequisite for excreting a Mechanism
plasma. When there is contraction of concentrated urine I
ECF volume, the urinary concentrating I ECF volume contraction / water deficit
ability of kidney increases and it forms 1. High level of ADH: I
a concentrated urine with a smaller ADH increases permeability
T Plasma osmolality
volume, thereby m aintaining of distal tubules & CT to
homeostasisQ. I
water thus allowing these
segments to absorb water. T Secretion of ADH from post pituitary
Reduction in medullary hyperosmolarity
2. High osmolarity of renal I
leads to loss of oncotic pressure gradient
which is essential to produce medullary interstitial fluid: T Permeability of late distal tubule &
concentrated urine Provides the osmotic CD
gradient necessary for water I
Increase in RBF and T GFR indicate an reabsorption to occur
adequately filed vascular system and T Reabsorption of water from tubular
hence likely to decrease ADH secretion, lumen
therefore decreases kidney's i
concentrating ability. Passage of concentrated urine

65. D i.e. Glucose 66. B i.e. Urea [Ref: Ganong 24/ep 6881

In kidney, glucose & bicarbonate (HCO 3-) are completely (100%) reabsorbedQ > Water & Na+ (99.4%)> K+(99.3%)>
Cl- (99.2%)> Uric acid (9 8 % )» > urea (58%)Q.
Physiology: Excretory System ■ 165

Urine Water and Acid Base Balance


67. B i.e. 500ml IRef: Chaudhuri 5/e P- 420; Harrison 72. C i.e. 200 - 300 ml I Ref: Guyton 12/e p. 355]
16/e P- 259; Ganong 22/e p. 707, 726-727]
68. A i.e. 100 mg Same amount of solute can be excreted per 24 hour in urine
69. A i.e., 100 - 200 ml of urine volume of 500 ml (with concentration of 1400 mOsm/kg) or in
70. B i.e. When about 300-400 ml of urine is 23.3 lit volume (with concentration of 30 mOsm/kg)®.
collected in bladder
71. A i.e., Bladder emptying cannot occur if volume Daily intake and output of water (ml/day)
is <100 ml

- The m inimum volum e necessary to remove all Intake Output


waste products, with a norm al diet, is 500ml I I
/day® and is called obligatory volume. Output Normal Prolonged
Oliguria® is urinary output less than 500ml® in Intake Normal Prolonged Heavy
24 hrs. Heavy Exercise
- The am ount of protein in urine is norm ally less Exercise Insensible - 350® 350®
than lOOrng/ day®. M ost of this is not filtered Fluid 2100 - skin
and com es from shed tubular cells. ingested 200 200 Insensible -
- First urge to void is felt at a bladder volum e of From Lungs 350 650
~ 150 ml®, and a marked sense of fullness at metabolism Sweat 100® 5000
about 400 ml®. In adult the volum e of urine that Total 2300 - Feces 100 100
norm ally initiates a reflex contraction is about intake Urine 1400 500
300- 400 ml®. T otal output 2300 6600

73. A, C i.e. TUrine Na+, TUrine osmolality/Pe/. Harrison's 18lh/2908-10; Berne & Levy 6/e p 598; Best & Taylor 13/e p 850-53;
74. A i.e. Regulated by ADH Williams Endocrinology 12/e p 1698; Guyton 11/e p. 357- 60]

- f t
Concentration and dilution of urine i.e. free water and osmolar clearances are regulated by antidiuretic hormone
(ADH) or vasopressin®.
SIADH is characterized by relatively excessive ADH levels, euvolemia, hypoosmolality (decreased osmolality) of
serum, hyponatremia (4-serum Na+), inappropriately high urine osmolality and elevated urinary Na+ excretion®.

75. B i.e. Decreased Potassium excretion [Ref: Ganong 22/e P- 8, 720; Harrison 16/e P- 258]
76. D i.e. Acidosis leads to movement of potassium from extracellular to intracellular fluid compartment.

98% potassium (K+) is intracellular & about 3/4 th of total body K + lies in skeletal muscles®. Severe injury and
metabolic acidosis cause hyperkalemia ie 1/t movement of K> from intra to extracellular space ®.

77. A i.e. Proteins [Ref: Ganong 22/e P- 372, 736; Guyton 10/e 362; Harrison 16/e P- 264; Davidson's 17/e P 605]
78. B i.e. Uretersigmoidostomy 79. C i.e. Lactic acidosis; D i.e. DKA, E i.e. cardiac failure
80. A i.e. Diarrhoea 81. B i.e. Lactic acidosis 82. A i.e. Bicarbone; B i.e. Plasma Proteins; D i.e. Hemoglobin

Anion gap is mostly due to proteins®. Normal anion gap metabolic acidosis is seen in diarrhea, uretero ☆
sigmoidostomy and renal tubular acidosis®. High anion gap metabolic acidosis is seen in lactic acidosis (d/t cardiac
failures shock, acute respiratory failure) and diabetic ketoacidosis®.
Buffers in blood include bicarbonates, plasma proteins and hemoglobin®.
166 ■ A Complete Review of Short Subjects

Anion Gap (AG) M etabolic Acidosis (MA)

The concentration of anions and cations in


High - Anion Gap MA Non - Anion Gap MA
plasma must be equal to maintain electrical
neutrality. However, only certain cations (Na+) 1) Lactic acidosisO 1) Gastrointestinal bicarbonate
and anions (C1-, HCO 3-) are routinely measured - Shock loss
in clinical laboratory. Anion gap (AG) represents - Cardiac failureQ - DiarrhoeaQ
the unmeasured anions in the plasma (difference - Acute Respiratory failure - External pancreatic
between 'unmeasured cations & unmeasured - Metformin therapy drainageQ
- UreterosigmoidostomyQ
anions'). It is ca lcu latedQ as follows: 2) KetoacidosisO
- Drugs : Mg SO 4
AG = [Na+] - [HCOy + Cl ] - DiabeticO
CaCl2
The most important unmeasured cations: Ca2+, - AlcoholicO
Cholestyramine
Mg2+, K+. The most important unmeasured - StarvationQ
anions: Proteins (Albumin), sulphate, 2) Renal tubular acidosisQ
3) Ingested toxins
phosphates. Proteins are present in our body in 3) Ingestion of Ammonium
- Ethylene glycol
much more quantities as compared to other chlorideQ
(antifreeze)Q
unmeasured anions. 4) Carbonic anhydrase inhibitor
- MethanolQ
Represents unm easured an ions in p la sm a <2. AG - SalicylatesQ
is mostly due to p rotein sQ. Normal AG is 10 -12
4) Renal failureQ (acute &
m m ol /LO. D ecrease in serum album in by lg /d l
chronic)
decreases AG by 2 .5m eqll£.

83. D i.e. Siggard - Anderson normogram [Ref: Ganong 24/e P- 717-18]

Siggard Anderson Curve Nomogram is used to determine acid- base composition o f blood (pH, plotted on horizontal
axis) on the basis of Pco 2° (plotted on vertical a x is).

84. B i.e. 306 mOsm/kg [Ref: Ganong 22/e P- 6]

Osmolality (mOsm/kg) = 2[Na+ in mEq/1] + 0.055 [Glucose in mg/dl]+ 0.36 [BUN in mg/dl]

Osmolality = 2 x [Na+] + [Glucose] + [BUN]


18 2.8

Putting the values (of question) in equation

Osmolality = 2 x 125 + 108 + 140 = 306 mOsm/kg


18 2.8

Endocrine Function of Kidney

85. C i.e. Interstitial cells 86. A i.e. 4- Tissue PO 2


87. Di.e. Adrenocortical tumour concentration [Ref: Ganong 24/e P- 709; Harrison 16/e P- 627]

The only physiological stimulus for erythropoietin secretion is hypoxia or decreased partial pressure of oxygenQ.
Tumors producing erythropoietin include RCC a, hepatoma and cerebellar hemangioblastomaQ.
Physiology Excretory System ■ 167

Erythropoietin

Structure & Sources Erythropoietin Production


• It is a glycoprotein ® (165aa + 4 Oligosacchride
chains) Tumors producing Normal
• Most important source in adult is Kidney®. Most erythropoietin source
important source in fetal/ neonatal life is Liver®. - Renal cell carcinoma^ - 85% by interstitial cells
• Erythropoietin increases the number of (3% of cases) in peritubular capillary
erythropoietin - sensitive committed stem cells in bed of kidneyQ
- HepatomaQ(10%)
bone marrow that are converted to RBC. In the - 15% by perivenous
- Cerebellar
absence of erythropoietin, Erythroid stem cells show hepatycytes of liverQ
haemangioblastomad
apoptosis®
(15%)
• Erythropoietin is mostly inactivated in liverQ after
circulation V /i of 5 hours. However, RBC count
• Erythropoietin can also be extracted from salivary
increases after 2-3 days, as it is a slow process.
gland & spleen (but do not contain mRNA so do not
Stimuli for Secretion produce it)
• HypoxiaQ • It is also produced in brain, uterus & oviduct. But in
cases of renal failure, all other tissues cannot
• Cobalt salts / Androgen
compensate and anemia develops.
• Alkalosis of high altitude^
• Catecholamines
• Adenosine

88. A, B, C, D, E i.e. Constriction of afferent arteriole, Autoregulation of GFR, Secreted from endothelial, Release
aldosterone, Increased sodium and water reabsorption 89. A i.e. It helps to convert angiotensinogen to angiotensin I
90. B & D i.e. ACE splits Leucine- valine bond & PRA is better than PRC
91. A i.e. Angiotensin I 92. A & E i.e. Renin - liver & Angiotensinogen - Renal
[Ref: Ganong 23/e p. 670; Guyton l l / e 201-2, 223-24, 907; Robbins 7/e p 527; Harrison 16/e P-627]

Angiotensin I Bradykinin
<---------- ACE (Angiotensin converting enzyme expressed in lung endothelium)
Angiote nsin II (splits off histidyl leucine bond) Inactive metabolite
i
T V asocon striction A ld osteron e secretion® (salt and water retention)

lNa+ reabsorption® JW ater reabsorption®

TBlood Pressure

Renin - Angiotensin System


168 " A Complete Review of Short Subjects

93. C i.e. Vasodilatation [R ef: G an on g 2 3rd/e p. 669- 73; B oard R eview series, ph y siolog y 4/ed pg- 92- 93]

Angiotensin II increases secretion of ADH, (Vasopressin), ACTH, Aldosterone, increases thirst and vasoconstriction
(most potent or ~ 4-8 times more than noradrenaline on weight basis). It decreases rennin secretion.

94. D i.e. Increased sympathetic tone > C i.e. Decreased blood pressure 95. C i.e. Decreased amount of Na+ in PCT
96. D i.e. Juxta glomerular apparatus [R ef: G an on g 24/e P -706]

Renin is secreted by JG cells o f Juxta glom eru lar (JG) ap p aratu s®. Renin secretion is increased by renal ischemia,
narrowing (or low pressure) in afferent arterioles & increased sympathetic activity whereas its decreased by increased
Na+ & Cl" reabsorption across macula densa®.

Factors Affecting Renin Secretion

Renin secretion depends upon the rate of transport of N a+ and Ch across the m acu la densa. M acu la den sa is a part o f D CT.
Decreased delivery of N a+ & Ch to distal tubule is associated with increased rennin secretion (inversely proportional).

Stimulatory Inhibitory

Ted sympathetic activity via renal nerves - Ted afferent arteriolar pressure
Ted circulating catecholamines - Ted Na+ & CP reabsorption at macula densa
Prostaglandins® - A ngiotensin 77®
O ther conditions increasing secretions - V asopressin®
- Prostaglandin inhibitors such as
I----------- indom ethacin &- B- blockers eg propranolol
i Central venous 1 Renal arteriolar T Activity of - Pepstatin peptide & Enalkiren prevent
pressure (4-CVP) pressure renal nerves rennin from generating angiotensin I.
N a+ depletion in DCT, I I
diuretics Constriction of Psychological
Hemorrhage, upright posture, renal artery and stimuli
d eh yd ratio n , hypotension®, aorta
CHF, cirrhosis

97. B i.e. DCT 98. A i.e. JGA [R ef: G an on g 23"'/e p. 674; G uyton & H all U /e p.324-25, 10/e p.293]
99. B i.e. Important source of rennin; C i.e. Consist of macula densa 100. D i.e. Glomerulus

Juxta means next to or near. And so is the situation of juxta glomerular apparatus, which lies a t hilum o f glomerulus®. JG
apparatus consists of JG cells, macula densa (located in relation to DCT) and lacis cells. It is important source o f renin®.

Juxtaglomerular Apparatus

Its a m icroscopic structure found betw een the vascular pole of renal corpuscle and returning distal convoluted tubule of
same nephron. It regulates renal blood flo w an d g lom eru lar filtra tion rate because of its critical location - a t hilium o f
glomerulus®. JG apparatus consists of
Physiology Excretory System ■ 169

1
Juxta glomerular (JG) cells Macula densa Lacis cells
Are epitheliod cells located It is m odified region o f distal Also k/a
in the m edia o f afferent convoluted tubular (DCT) extraglomerular
arterioles as they enter epithelium at the point mesengial cells
glomeruti®. where afferent arteriole
They are agranular cells,
enters glomerulus and
Also known as granular strategically located
efferent arteriole leaves it®.
cells. They secrete renin®. between macula densa
This location marks the start
and JG cells at junction
Intra renal baroreceptor of DCT
mechanism decrease renin between afferent and
M acula densa senses sodium efferent arterioles®. And
secretion when arteriolar
chloride concentration®. so it may mediate signals
pressure at JG cells increase
Renin secretion is inversely b/w them
and vice versa
proportional to the amount
They also contain renin
Low pressure on JG cells of Na+ & CL entering DCT
stimulate renin secretion, from loop of henle, where Contain actin & myosin,
which inturn increase BP Macula cells are located. allowing them to
and maintain GFR via renin These electrolytes enter contract & modulate
angiotensin system macula cells via Na+ - K+ - action of JG apparatus
2C1- cotransporter. Possibly when stim ulated by
NO mediates signal b/w renal sym pathetic nerves
macula densa & JG cells for
renin secretion.

101. A i.e. Water transport across collecting duct [Ref: Ganong 22/e P- 716 244; Guyton 10/e P 302]

Vasopressin acts by increasing perm eability o f collecting ducts®, to water. The key to the action of vasopressin on
collecting duct is aquaporin -2. The overall effect is retention of water in excess of solute which results in decrease effective
osmotic pressure.

102. A i.e. Erythropoietin [Ref: Ganong 22/e P-188; Harper 26/e P-609, 620]

Erythropoietin is a plasma protein that act as a hormone. Its a m ajor regulator o f human erythropoiesis®.

Ceruloplasmin transport copper®. Transferrin transports iron®. Lactoferrin binds iron®.


Chapter-7 ENDOCRINE SYSTEM

QUESTIONS

Receptors & Second Messengers 10. True about intracellular receptors: (PGI 05)
A. Mainly on nuclear surface □
1. The following hormone does not have any B. Steroids act on them □
intracellular receptor: (AI 01) C. Estrogen does not act on it □
A. Vitamin D3 □ D. GH act on it □
B. Cortisone □ 11. Hormone synthesized as peptide precursor is/are:
C. Adrenaline □ A. Insulin (PGI 09) □
D. Thyroxine □ B. PTH □
2. Among the following all are hydrophilic hormones C. Renin □
that act on cytosolic receptors except one which is a D. Thyroid hormone □
lipophilic hormone that acts on nuclear receptor: E. Angiotensin II □
A. Thyroxine (AIIMS 11,14,13) □ 12. All the following mediate their action using cAMP as
B. Epinephrine/Adrenaline □ second messenger except: (AIIMS 09)
C. GH □ A. Corticotropin □
D. ACTH □ B. Dopamine □
3. Intracellular receptors are found in A/E: (PGI 06,15) C. Glucagon □
A. Insulin & Glucagon □ D. Vasopressin □
B. Corticosteriods □ 13. Which of the following is a membrane bound enzyme
C. Androgen □ that catalyzes the formation of cyclic AMP from ATP?
D. Thyroxine □ A. Tyrosine Kinase (NBE P 15,13; AI 10) □
4. Which hormone acts on cytoplasmic membrane B. Polymerase □
receptor- (NBE P 13) C. ATP synthase □
A. TSH □ D. Adenylate cyclase □
B. Thyroxine □ 14. Following modification occurs in Gs subunit which
leads to watery diarrhea in cholera (AIIMS 11)
C. Androgen □
A. ADP ribosylation □
D. Cortisol □
B. ATP-ADP transfer □
5. cGM P is second messenger for which hormone(s):
A. Somatostatin (AIIMS 14,13; PGI 09,15) □ C. Phosphorylation □
D. Dephosphorylation □
B. Atrial natriuretic factor (ANF) & NO □
15. Cholera toxin: (AIIMS 06)
C. Angiotensin II □
A. Increases the levels of intracellular cyclic GMP □
D. Antidiuretic hormone (ADH), Insulin, GH □
B. Acts through the receptor for opiates □
6. cAMP action mediates all except- (NBE P 13)
C. Causes continued activation of adenylate cyclase □
A. Glucagon □
D. Inhibits the enzyme phosphodiesterase □
B. Follicle stimulating hormone □
16. CAMP acts through: (PGI 06)
C. Leutinizing hormone □
A. Activation of protein kinase □
D. Estrogen □
B. Activation of adenylate cyclase □
7. Steroid receptor super family is present in:
C. T Ca2+ release. □
A. Vitamin D3 (PGI 12,06) □
D. PIP3 pathway □
B. Insulin □
17. Which one of the following acts as second messenger?
C. Glucagon □
A. Mg+t (AIIMS 06) □
D. Thyroid □
B. Cl- □
8. All are second messengers except;
C. Ca++ □
A. Guanylyl cyclase (AIIMS 12; PGI 13) □
D. PO 43- □
B. c AMP/CGMP □
18. Which set of hormones have nuclear receptor:
C. IP3 □
A. Estrogen, Thyroxin & Glucagon (PGI 08)D
D. DAG □
B. Estrogen, Thyroxin & TSH □
9. Regarding nitric oxide false is (AIIMS 07)
C. Estrogen, TSH & Gonadotropin releasing hormone
A. Derived from endothelium □
(GnRH) □
B. Acts by increasing c-AMP levels □
D. Retinoic acid, Thyroxin & Luteinizing hormone
C. Vasodilator □
(LH) □
D. Derived from arginine □
E. Testosterone, cortisol & Estogen □
Physiology: Endocrine System ■ 171

19. All of the follow ing bind to the steroid receptors B. Decreased IP3 □
except: (AIIMS 08) C. Increased GTPase activity □
A. Steroids □ D. Increased cAMP □
B. Transcriptional mediators □ 29. Adrenaline, noradrenaline and dopamine act
C. Transcriptional receptors □ through (AIIMS 11, 06)
D. Steroid response elements □ A. Single pass receptors □
20. C terminal end of androgen receptor is concerned B. Four pass receptors □
with: (AI 07) C. Seven pass receptor □
A. Ligand binding □ D. Ligand gated channel □
B. Increasing biological half life □ 30. True about Second messenger: (PGI 11)
C. Increasing the affinity of receptor to DNA □ A. Bind first messenger □
D. Increasing the level of transcription □ B. Integral protein □
21. Aldosterone receptors are present in all except: C. Hormone secreted by stimulation of other
A. Liver (AIIMS 07) □ hormones □
B. Colon □ D. Substance that increase or decrease function □
C. Hippocampus □ E. Intracellular receptor □
D. Distal nephron □ 31. Various cells respond differentially to a second
22. True about membrane receptors: (PGI 2K) messenger (such as increased cAMP) because they
A. Barrier function □ have different.
B. For specific action □ A. Receptors (AIIMS 03) □
C. Molecular transport □ B. Enzymatic composition □
D. None □ C. Nuclei □
23. Receptors on cell membrane that activate ion channel D. Membrane lipid □
after binding with agonists are 32. ANF is mediated by (ANF: Atrial natriuretic factor):
A. Nicotonic Cholinergic (PGI 03) □ A. Inositol phosphate (AIIMS 07) □
B. Muscarinic Cholinergic □ B. DAG □
C. Optoid (i receptors □ C. CyAMP □
D. GABA a □ D. CyGMP □
E. GABA b □ 33. Which of the follow ing act through tyrosine kinase
24. Action of a - subunit of G protein is: (AIIMS 08) receptor: (PGI 01)
A. Breakdown of GTP to GDP □ A. Insulin □
B. Conversion of GDP to GTP □ B. Glucagons □
C. Internalization of receptors □ C. GH □
D. Binding of agonist □ D. FSH □
25. Following are true about G protein except: (PGI 08) 34. cAMP action mediates all except: (PGI 2K)
A. G channels □ A. Glucagon □
B. Phosphorylase formation □ B. FSH □
C. Made up of 4 units □ C. LH □
D. Related to ras oncogene □ D. Estrogen □
26. True about G-protein receptor complex: 35. CyAMP acts as a second messenger of:
A. It interacts transmembrane domain. A. FSH (AIIMS 05) □
B. GTP to GDP (PGI 08) □ B. Thyroxine □
C. Adenyl cyclase activation leads to increased C. Growth hormone □
cAMP □ D. Insulin □
D. GPCR has no phospharylation property □ 36. Cyclic GM P is the second messenger of:
E. Helical structure □ A. Growth Hormone (AIIMS 04) □
27. True statement about the G-protein coupled receptor B. Follicle Stimulating Hormone (FSH) □
is: C. Insulin □
A. It binds to ligands at the cell surface (AIIMS 08) □ D. Thyroxin □
B. Association of all the three subunits (alpha, beta 37. W hich one of the following molecules is used for cell
and gamma) is essential for its action □ signaling? (AI 05)
C. G-protein acts as inhibitory and excitatory as alpha A. C 0 2. □
subunit exists in two forms □ B. 0 2. □
D. GTP binds to G protein in resting state □ C. NO. □
28. If there is a Gs alfa subunit gain-of-function D. N2. □
mutation, this results in: (AI 11) 38. All are second messengers except: (AIIMS 08)
A. Decreased cAMP □ A. Guanylyl cyclase □
172 ■ A Complete Review of Short Subjects

B. cAMP □ A. Phytates □
C. IP3 □ B. Vitamin D □
D. DAG □ C. Alkaline pH in gut □
Which of the following function as second D. Protein in diet □
messengers: (PGI 05) 48. True statement about calcium : (PGI 04; 15,13)
A. CAMP □ A. Absorbed in upper small intestine □
B. DAG □ B. Absorbed in lower small intestine □
C. IP3 □ C. Absorption increased by alkaline pH □
D. Ca2+ □ D. Absorption increased by acidic pH □
E. Mg2+ □ 49. Calcium sensing receptors is/are present at: (PGI 13,14)
Second messengers is/are: (PGI 12) A. PCT □
A. Mg+2 □ B. DCT □
B. PO 43- □ C. Loop of Henle □
C. cAMP □ D. Apical brush-border membrane of the intestinal
D. Ca2+ □ enterocyte □
E. CP ion □ E. Epithelium of intestine □
50. Which of the following would you expect to find in a
Calcium Metabolism & Parathyroid Gland patient whose diet has been low in calcium for 8
weeks?
41. A small Ca+2 binding protein that modifies the A. Increased phosphate levels (AIIMS 07) □
activity of many enzymes and other proteins in B. Raised calcitonin levels □
response to changes of Ca2+ concentration, is known C. Increased parathormone secretion □
as: (AI04) D. Activation of 24-25 dihydrocholecalciferol □
A. Cycline □ 51. True about Parathyroid hormone: (PGI 12)
B. Calmodulin □ A. It is steroidal in nature □
C. Collagen □ B. Stimulate 1,25 D3 formation □
D. Kinesin □ C. Inhibits Ca2+ absorption from the intestines □
42. Parathyroid hormone isresponsible for all actions D. Increases the resorption of bone □
except: (PGI 07) 52. Secondary hyperparathyroidism due to V it D
A. Absorption of phosphorous increase □ deficiency shows: (AI 01)
B. Vit D absorption increases □ A. Hypocalcemia □
C. Mobilizes calcium from bone □ B. Hypercalcemia □
D. Increase intestinal absorption of calcium □ C. Hypophosphatemia □
43. Calcium absorption is from. (PGI 09) D. Hyperphosphatemia □
A. Proximal small intestine □ 53. Which hormone regulate (s) serum calcium level:
B. Distal ileum □ A. PTH (PGI 13) D
C. Middle small intestine □ B. Calcitonin □
D. Ascending colon □
C. Adrenaline □
44. Calcium absorptionis increased by- (NBE P 13)
D. 1 ,25-Dihydroxy-cholecalciferol □
A. Proteins □
E. Procalcitonin □
B. Bile □
54. Which of these can cause hypocalcemia:
C. Acid □
A. Thyroxine (AIIMS 02) □
D. Phosphates □
B. Calcitonin □
45. True of the following: (PGI 01)
C. Parathyroid hormone □
A. Calcium reabsorbed in DCT □
D. Choloecalciferol □
B. 90% calcium excreted by glomerulus □
55. Hypocalcemia due to calcitonin is by: (PGI 09)
C. Parathormone (PTH) promotes absorption of Ca++
A. Increased excretion in kidney □
from intestine □
B. Decreased bone resorption □
D. PTH promote action of calcitonin □
C. Decreased intestinal absorption □
46. Which of the following is considered the active form
of calcium: (AI 08) D. Decreased renal reabsorption □
A. Ionised Calcium □ 56. Osteoclasts are inhibited by action on specific
B. Albumin bound Calcium □ receptor:
C. Phosphate bound Calcium □ A. Parathyroid hormone. (AIIMS 03, AI 05) □
D. Protein bound Calcium □ B. Calcitonin. □
47. Calcium absorption from gut is increased by all C. 1,25-dihydroxycholecaIciferol. □
except: (AIIMS 16) D. Tumor necrosis factor. □
Physiology: Endocrine System ■ 173

57. Sudden decrease in serum calcium is associated with: 67. Which of the follow ing hormones is an example of a
A. Increased thyroxine and PTH secretion (AI 0 2 )0 peptide hormone? (AIIMS 05)
B. Increased phosphate □ A. Parathormone □
C. Increased excitability of muscle and nerve □ B. Adrenaline □
D. Cardiac conduction abnormalities □ C. Cortisol □
58. In tetany hyperexcitability is due to (PGI 04) D. Thyroxine □
A. Low Ca++ causes T permeability to Na+ □ 68. Regarding transport of Ca++ across a membrane
B. Prevent K+ release □ follow ing are true: (PGI 03)
C. Prevent Na+ & K+ release □ A. Ca++ calmodulin binding □
D. iC a ++ produce generation of AP □ B. It is a passive mechanism □
59. Hypocalcemia is characterized by all except: C. Required hydrolysis of A.T.P. □
A. Numbness and tingling of circumoral region □ D. It is a symport □
B. Hyperactive tendon reflexes and positive E. It is an active process □
Chvostek's sign. (AIIMS 03) □ 69. Which one one o f the follow ing acts to increase the
C. Shortening of Q-T interval in ECG. □ release of Ca2+ from endoplasmic reticulum:
D. Carpopedal spasm □ A. Inositol triphosphate (AIIMS 04, AI 05) □
60. Which of the follow ing organ is not involved in B. iParathyroid hormone □
calcium metabolism: (PGI 01,10) C. 1 ,25-dihydroxy cholecalciferol □
A. Lung □ D. Diacyglycerol □
B. Liver □
C. Spleen □ Carbohydrate Metabolism &
D. Skin □ Pancreatic Hormone
E. Kidney □
70. ALL are true regarding pancreas except
61. Which o f the follow ing organs is not involved in
(AI 12, AIIMS 07,15,14,13; PGI 11,10, 08)
calcium homeostasis? (AI 06)
A. Delta (D) cells secrete somatostatin □
A. Kidneys □
B. Insulin & amylin are secreted by beta (B) cells
B. Skin □
whereas glucagon is secreted by alpha (A) cells. □
C. Intestines □
C. Pancreatic polypeptide hormone is secreted by F
D. Lungs □
(PP) cells. □
62. All are true about Vitamin D except: (AIIMS 08)
D. PP is secreted by D l cells. □
A. 25-hydroxylation occurs in the liverfNBE P 15,13)0
71. The mechanism that protects normal pancreas from
B. 1-a hydroxylation occurs in the kidneys □
autodigestion is : (AIIMS 03)
C. In the absence of sunlight, the minimum daily
A. Secretion of biocarbonate. □
requirement is 400-600 IU □
B. Protease inhibitors present in plasma. □
D. William's syndrome consists of mental retardation,
C. Proteolytic enzymes secreted in inactive form □
obesity and precocious puberty □
D. The resistance of pancreatic cells. □
63. Increased calcium levels lead to- (NBE P 13)
72. All of the follow ing enzymes are active within a cell
A. Increased 1,25 dihydroxycholecalciferol □
except
B. Increased 24,25 dihydroxycholecalciferol □
A. Trypsin (AIIMS 05) □
C. Decreased calcitonin □
B. Fumerase □
D. Increased parathormone □
C. Hexokinase □
64. Active form of vitamin D is: (AIIMS 07)
D. Alcohol dehydrogenase □
A. Cholecalciferol □
73. All are true about exocrine functions of pancreas
B. 24, 25 (OH)2 vit-D □
EXCEPT: (AIIMS 07)
C. l,25(OH)2 vit-D □
A. Osmolality is Vi of the osmolality of blood □
D. 25-OH v i t - D □
B. Secretes l ‘/2 litre of the alkaline fluid daily □
65. Function of Phospholamban is : (AIIMS 2K)
C. Secretin enhances bicarbonate secretion □
A. Regulates Na K Pump □
D. Secretion is rich in enzymes. □
B. Transports calcium out of the mitrochondria □
74. After 5 days of fasting a man undergoes oral GTT,
C. Binds actin with myosin □
true is all excep t: (AI 01)
D. Collects calcium into the sarcoplasmic
A. GH levels are increased □
reticulum □
B. Increased glucose tolerance □
66. Hypercalcemia associated with malignancy is most
C. Decreased insulin levels □
often mediated by: (AI 05)
D. Glucagon levels are increased □
A. Parathyroid hormone (PTH). □
75. A/E the follow ing changes are seen on the 5th day of
B. Parathyroid hormone related protein (PTHrP) □
fasting:
C. Interleukin - 6 (IL-6). □
A. Increase in FFA levels. (AIIMS 04) □
D. Calcitonin. □
174 ■ A Complete Review of Short Subjects

B. Decreased glucose tolerance □ D. Insulin □


C. Decreased growth hormone □ E. Glucagons □
D. Decreased level of Insulin □ 86. (PGI?':)
76. Noble prize for sequencing insulin aminoacid A. Causes neoglucogenesis. □
sequence & molecular structure was given to: B. Not useful for growth & development □
A. Banting & Macleod (AIIMS 11) O C. Required for transport of glucose, aminoacid,
B. Sanger O K+ & Na+ □
C. Charles Best O D. Catabolic hormone □
D. Paul Berg O 87.
77. Human insulin differs from beef insulin by how A. 3rd month (AIIMS 05) □
many number of amino acid residues: (AI 05) B. 5th month □
A. 1 0 C. 7th month □
B. 2 O D. 9th month □
C. 3 O 88. Increased ratio of insulin to glucagon causes:
D. 4 O A. Decreased levels of cyclic AMP (AI 0 6 )0
78. Half life of insulin is- (NBE P 13) B. Decreased levels of lipoprotein lipase □
A. l-2min O C. Decreased amino acid synthesis □
B. 4-6min 0 D. Enhanced lipolysis in adipose tissue □
C. 10-12min 0 89. Insulin promotes lipogenesis by all of the following
D. 12-16min O ways except: (AIIMS 08)
79. Insulin is secreted along with the follow ing molecule A. Decreasing intracellular cAMP □
in a 1 :1 ratio: (PGI 06) B. Increasing the transport of glucose into the cell □
A. Pancreatic polypeptide 0 C. Inhibits pyruvate dehydrogenase □
B. Glucagon 0 D. Increases activity of acetyl-CoA carboxylase □
C. GLP-1 0 90. Insulin secretion seen in (PGI 04)
D. Somatostatin 0 A. Glucose □
E. C- peptide 0 B. Vagal stimulation □
80. Glucose mediated insulin release is mediated C. Acetylcholine □
through: (AI 02) D. Adrenaline □
A. ATP dependent K+ channels 0 E. Calmodulin □
B. cAMP 0 91. D iet devoid of carbohydrates causes; (PGI 04)
C. Carrier modulators 0 A. Ketosis □
D. Receptor phosphorylation 0 B. DM □
81. Insulin secretion is inhibited by: (AI 10) C. No effect □
A. Secretion 0 D. Obesity □
B. Epinephrine O 92. A patient presents with hepatomegaly and
C. Growth hormone 0 hypoglycemia not responding to epinephrine. Most
D. Gastrin 0 probable diagnosis is
82. Insulin secretion is/are increased by all except: A. Gaucher's disease (AIIMS 10, PGI 10) □
A. Gastrin (PGI 10) □ B. Von Gierke's disease □
B. Secretin □ C. Anderson's disease □
C. VIP □ D. Pompes disease □
D. GIP □ 93. All are actions of insulin except OR insulin increases
E. CCK □ all except one which is decreased.:
83. Insulin secretion is decreased by: (PGI 03) A. Gluconeogenesis (AI 2K, NBE P 15,13)0
A. Glucagon O B. Glycolysis □
B. Glucose O C. Glycogenesis □
C. Adrenaline □ D. Lipogenesis □
D. Vagal stimulation □ 94.
84. Rapid infusion of insulin causes (AIIMS 10) A. Insulin (AI 2K) □
A. Hyperkalemia O B. Steroid □
B. Hypokalemia O C. Oestrogen □
C. Hypernatremia O D. Thyroxine □
D. Hyponatremia O 95. Not done by insulin: (AIIMS 10, 09)
85. All of these cause hyperglycemia except: A. Glycogen synthesis O
A. GH (PGI 02) □ B. Glycolysis O
B. Cortisol □ C. Lipogenesis O
C. Catecholamines □ D. Ketogenesis o
Physiology: Endocrine System ■ 175

96. What is role of insulin in lipid metabolism: B. Tetradotoxin □


A. Active lipoprotein lipase (PGI 08) □ C. Tetra Ethyl lead □
B. Increase lipolysis □ D. Choline □
C. Activate hormone sensitive lipase □ 107. Tetrodotoxin blocks (UP 2K)
D. Activate Acetyl CoA carboxylase □ A. yNa+ during action potential O
E. Decrease free fatty acid level □ B. yK+during action potential □
97. Insulin increases the activities of all of the follow ing C. yNa+ during resting state □
enzymes, EXCEPT: (Jipmer 04, A I 05) D. yK+during resting state □
A. Glucokinase □ 108. To synthesize insulin on a large scale basis, the most
B. Pyruvate carboxylase □ suitable starting material obtained fro the beta cells of
C. Glycogen synthase □ the pancreas is: (AIIMS 03, UP 04)
D. Acetyl-CoA carboxylase □ A. Genomic DNA □
98. GLUT2 is present mainly in- (NBE P 15,13) B. Total cellular RNA □
A. Beta cells of pancreas □ C. CDNA of insulin □
B. Placenta □ D. MRNA of insulin □
C. Skeletal muscle □ 109. Insulin resistance is seen in patients with liver
D. Cardiac muscle □ disease due to: (AIIMS 13)
99. Which of the follow ing glucose transporter is/are used A. Damaged hepatocytes □
by brain: (AIIMS 1 5 ,13P G I12) B. Hepatic steatosis □
A. GLUT1 & 3 □ C. Impaired insulin release □
B. GLUT 2 □ D. Low C peptide level □
C. GLUT 4 □ 1 1 0 .1st response to hypoglycemia- (NBE P 13)
D. GLUT 5 □ A. Decreased insulin □
100. Glucose transporter in myocyte? (AIIMS 14,12) B. Increased glucagon □
A. GLUT 1 (NBE P 1 5 ,1 3 ) 0 C. Increased cortisol □
B. GLUT 2 □ D. Increased nor epinephrine □
C. GLUT 3 □ 111. In surgical stress all hormone is increased except:
D. GLUT 4 □ A. Adrenaline (PGI 11) □
101. Insulin mediated glucose uptake occurs through B. ACTH □
A. G L U T -1 (NBE P 15,13; AIIMS 09) □ C. Epinephrine □
B. GLUT -2 □ D. Cortisol □
C. GLUT- 3 □ E. Insulin □
D. GLUT -4 □ 112. Stress-induced hyperglycemia is due to:
102. Insulin mediated glucose uptake is/are used in: A. Glucocorticoids (PGI 02) □
A. Adipose tissue & Skeletal muscle (PGI 12,15)0 B. GH □
B. Kidney (NBE P 1 5 ,1 3 )0 C. Thyroxine □
C. Brain □ D. Epinephrine □
D. Intestine □ E. Insulin □
103. Insulin does not facilitate glucose uptake in the 113. W hich of the follow ing hormones are increased due to
follow ing ex cep t: (AIIMS 14,08; NBE P 15,13) stress during surgery, especially in DM:
A. Liver □ A. Epinephrine (PGI 2K) □
B. Heart □ B. GH □
C. RBC □ C. Glucocorticoids □
D. Kidney □ D. Thyroxine □
104. Insulin acts on glucose metabolism by: (PGI 2K) 114. Endogenous triglycerides are maximum in:
A. T permeability of glucose across cell membrane □ A. VLDL (DNB 06, AI 04) □
B. T permeability across cell membrane against B. Chylomicron □
glucose gradient □ C. LDL □
C. T permeability of renal cells □ D. HDL □
D. T glucose transport to brain □
105. Na dependent glucose transport is /are inhibited by: Pineal Gland & Pituitary Gland
A. Oubain (PGI 08) □
B. Na azide □ 115. M elatonin is secreted by: (AI 02)
C. Phlorhizine □ A. Hypothalamus □
D. Phloretin □ B. Adrenal cortex □
106. Sodium channels are specifically blocked by - C. Pineal gland □
A. Nifedipine (AIIMS 05) □ D. Melanocytes □
176 ■ A Complete Review of Short Subjects

116. Exposure to darkness leads to increased melatonin B. Growth hormone □


secretion. It is brought about by: (AIIMS 03) C. Somatostatin □
A. Decreasing the activity of suprachiasmatic nuclei□ D. Insulin □
B. Increasing the serotonin N-acetyl transferase □ 127. Somatomedin mediates: (AIIMS 10, 09)
C. Decreasing the hydroxy-indole-o-methyl A. Deposition of chondroitin sulfate O
transferase, activity □ B. Lipolysis O
D. Blocking the release of norepinephrine from C. Gluconeogenesis o
sympathetic nerve terminals D. Decreased rate of glucose uptake by cells □
117. Acidophils secrete- (NBE P 13) 128. Growth hormone level decreased in- (NBE P 13)
A. GH □ A. Hypoglycemia □
B. TSH □ B. Fasting □
C. ACTH □ C. Sleep □
D. FSH □ D. Exercise □
118. Hypothalamus controls the hormone secretion of- 129. Secretion of Prolactin is affected by: (NBE P 15, AI 04)
A. Anterior hypophysis (NBE P 13) □ A. GnRH analogue □
B. Posterior hypophysis □ B. Dopamine □
C. Kidney □ C. Serotonin □
D. Pineal gland □ D. FSH □
119. Hypothalamus increase release of all hormones from
the pituitary except- (NBE P 13) Posterior Pituitary Hormone
A. TSH □
B. FSH □ 130. Pituicytes are seen in- (NBE P 13)
C. CRH □ A. Anterior lobe O
D. Prolactin □ B. Posterior lobe O
120. Basophilic cells of pituitary secretes A/E: (PGI 02,15) C. Intermediate lobe O
A. Prolactin & G H (NBE P 1 5 ,1 4 )0 D. All O
B. TSH □ 131. True about m ilk secretion is A/E: (NBE P 15,13; PGI 10)
C. ACTH □ A. Neuroendocrine part of post pituitary is
D. LH □ involved □
121. Hormone secreted by anterior pituitary/ B. Secretion by contraction of lactiferous sinus □
adenohypophysis are A/E: (NBE P 15,13,14) C. Oxytocin hormone cause ejection of milk □
A. Oxytocin - Vasopressin (AIIMS 15,13; PGI 03,14) □ D. Prolactin cause contraction of Myoepithelial
B. TSH, LH, FSH □ cells and not affected by emotion □
C. Gonadotropns □ 132. Which is wrongly matched pair about vasopressin
D. ACTH □ receptor: (AIIMS 10)
122. Posterior pituitary secretes: (NBE P 14; PGI 13) A. Vl-Vascular smooth muscles □
A. Oxytocin & ADH □ B. V2-collecting ducts □
B. Prolactin □ C. V3-anterior pituitary □
C. MSH □ D. V4-CNS □
D. FSH □ 133. When NaCl is injected in the internal carotid artery, it
123. Herring body is seen in- (NBE P 13) causes release of ADH by acting on: (AIIMS 01)
A. Pars intermedia □ A. Paramedian nucleus □
B. Pineal gland □ B. Anterior pitutary □
C. Pars nervosa □ C. Paraoptic nucleus □
D. Adenohypophysis □ D. Supraoptic nucleus □
124. ACTH level is highest during; (PGI 13, 09) 134. Greatest stimulator for ADH secretion:
A. Early morning O A. Hyperosmolarity (PGI 06) □
B. Evening O B. Hyponatremia □
C. Afternoon o C. Hypotension □
D. Night o D. Hypovolemia □
125. ACTH is increased in all except- (NBE P 13) 135. All of the following are true for ADH except:
A. Exercise O A. Post-operative increase in secretion □
B. Emotions O B. Neurosecretion (AIIMS 12) □
C. Evening □ C. Increased secretion when plasma osmolality is
D. Tumors □ low □
126. Acromegaly is due to excess of: (PGI 14) D. Act on disal tubule and icrease permeability □
A. Somatomedin □
Physiology: Endocrine System ■ 177

136. In which of the following form the Anti diuretic C. IB M R □


hormone (ADH) is circulated in plasma: (AI 04) D. T myocardial contractility □
A. Bound to neurophysin-I □ 146. TRH stim ulation testing is useful in diagnosis of
B. Bound to neurophysin-II □ disorders of follow ing hormones : (AI 02)
C. Bound to plasma albumin □ A. Insulin □
D. Free form □ B. ACTH □
137. A/E are caused by accidental transection o f pituitary C. Growth hormone □
stalk: (AIIMS 02) D. PTH □
A. Diabetes mellitus □
B. Polyuria □ Kidney & Adrenal Gland
C. Galactorrhea □ 147. W hich of the follow ing hormone is not secreted by
D. Diabetes insipidus □ the kidney: (AI 07; PGI 09,07,06)
138. Which of the follow ing is most important in sodium A. Renin □
and water retention- (NBE P 13) B. Angiotensin 1/ Vasopressin / Angiotensinogen/
A. Rennin angiotensin system □ 25-hydroxy vit D (cholecalciferol) / Granulocyte -
B. ANP □ Monocyte - CCF □
C. BNP □ C. Erythropoietin □
D. Vasopressin □ D. 1,2 5 DHCC □
148. All of the follow ing statements about adrenals are
Thyroid Gland true except: (AI 07)
A. Chromoffin granules are seen in
139. In Thyroid follicle for how long Thyroxine is stored- pheochromocytoma □
A. 2-3 weeks (NBE P 13) □ B. Adrenal medulla normally secretes epinephrine &
B. 2-3 days □ in excess of norepinephrine □
C. 2-3 months d C. Tumours of adrenal medulla secrete epinephrine in
D. 2-3 years □ excess of norepinephrine □
140. Thyroid hormones in blood is transported by: D. Adrenal medulla is not essential for life □
A. Albumin (PGI 01) □ 149. Hormones secreted by adrenal medulla are:
B. Globulin □ A. Glucagons (PGI 02,13) □
C. Prealbumin □ B. Cortisol □
D. Transferrin □ C. Norepinephrine □
E. Ceruloplasmin □ D. Insulin □
141. Regarding thyroid hormone all are true except: (AI 2K) E. Aldosterone □
A. T 3 is more avidly bound to nuclear receptors than 150. Destruction of zone glomerulosa w ill deplete-
T4 □ A. Aldosterone (NBE P 13) □
B. T 4 has the maximum plasma concentration □ B. Cortisol □
C. T 3 is more active than T4 □ C. Testosterone □
D. T 4 has shorter half life than T 3 □ D. Catecholamines □
142. W olf chaifoff effect is due to- (NBE P 13) 151. M ajor adrenal androgen is- (NBE P 13)
A. Suppression of TSH secretion □ A. Testosterone □
B. Decreased iodination of MIT □ B. 11-hydroxy derivative of androstenedione □
C. Decreased T 3 to T4 conversion □ C. 17-ketosteroid dehydroepiandrosterone □
D. Iodine intake □ D. Cortisol □
143. Sodium iodine symporter is not present in: (AIIMS 15) 152. In the follow ing sections of the adrenal gland, what
A. Pituitary gland □ hormone is secreted by the region marked as "A "?
B. Placenta □
C. Parotid □
D. Thyroid □
144. Iodine uptake is seen in follow ing organs:
A. Ovary (PGI 03; NBE P 15,14) □
B. Thyroid □
C. Parathyroid □
D. Salivary gland □
E. Mammary gland □
145. Thyroxine injected to rat produces A/E:
A. >14. lipolysis (PGI 2K) □
B. TT 0 2 consumption □
170 ■ A Complete Review of Short Subjects

A. Aldosterone (AIIMS 16) □ C. CYP11A, □


B. Cortisol □ D. ll-(3 hydroxysteroid dehydrogenase □
C. Testosterone □ 163. Least mineralocorticoid activity is seen in
D. Epinephrine □ A. Aldosterone (AIIMS 0 9 ) 0
153. Mineralocorticoid receptors are present in all of the B. Deoxycortisone □
following sites, Except: (AI 11, AIIMS 08) C. Dexamethasone □
A. Hippocampus (NBE P 15,14,13) □ D. 9 a Flurotisol □
B. Kidney □ 164. Epinephrine action in liver: (PGI 08)
C. Colon □ A. Glycogenoloysis □
D. Liver □ B. Gluconeogenesis □
154. What is effect of cortisol on metabolism: (PGI 08) C. Glycolysis □
A. T Neoglucogenesis □ D. Lipolysis □
B. T Lipogenesis □
C. T Proteolysis □ Reproductive Hormones
D. T Protein anabolism in liver □
E. T Glycolysis □ 165. Beta HCG is secreted by- (NBE P 13)
155. Effect of steroids on calcium- (NBE P 13) A. Ovary □
A. Increased plasma level □ B. Pituitary □
B. Increased absorption from gut □ C. Corpus luteum □
C. Increased excretion from kidney □ D. Placenta □
D. None □ 166. What is thelarche? (NBE P 13)
156. Anti-Inflammatory action of steroids due to- A. Pubertal breast enlargement ingirls □
A. Inhibiton of phospholipas A2 (NBE P 1 3 ) 0 B. Breast enlargement in pregnancy □
B. Inhibhition of cyclooxegenase □ C. Hormone induced breast enlargement in boys □
C. Increased activity of lipolipase □ D. Post hormonal therapy breast enlargement in
D. Inhbhition of lipo oxegenase □ postmenopausal females □
157. D ium al variation in Eosinophil count is related to: 167. W hich of the following sugar is/are found in amniotic
A. Cortisol (AI 12) □ fluid:
B. Thyroxine □ A. Mannose (PGI 13) □
C. Growth hormone □ B. Sucrose □
D. Testesterone □ C. Glucose □
158. In Cushing syndrome, which of the following are D. Fructose □
seen: (AI 09) E. Galactose □
A. TAldosterone □ 168. Which of the following is 21-Carbon or C-21/17-OH
B. Aldosterobe □ steroid: (PGI 14,15; NBE P 15,13; AIIMS 14)
C. T Epinephrine □ A. Testosterone □
D. -I Epinephrine □ B. Estrogen □
E. Edema □ C. Androgen □
159. Aldosterone synthesis is inhibited by: (PGI 03) D. Progesterone & Corticosteroid □
A. Renin □ 169. A middle aged woman on oral contraceptives for
B. Endothelin □ many years, developed neurological symptoms such
C. Dopamine □ as depression, irritability, nervousness and mental
D. Endorphin □ confusion. Her hemoglobin level was 8g/dl.
E. Hypernatremia □ Biochemical investigations revealed that she was
160. Hyperaldosteronism is associated with all except excreting highly elevated concentrations of
A. Hypermatrenia (AIIMS 10, NBE P 13) □ xanthurenic acid in urine. She also showed high
B. Hypokalemia □ levels of triglycerides and cholesterol in serum.
C. Hypertension □ All of the above findings are most probably related to
D. Metabolic acidosis □ vitamin B6 deficiency caused by prolonged oral
161. Excess secretion of aldosterone causes all except contraceptive use excep t: (AI 04)
A. Increased ECF (AIIMS 08) □ A. Increased urinary xanthurenic acidexcretion □
B. Very High Na+ in plasma □ B. Neurological symptoms by decreased synthesis of
C. Increased BP □ biogenic amines □
D. Natriuresis □ C. Decreased hemoglobin level □
162. Apparent mineralocorticoid excess is d/t D. Increased triglyceride and cholesterol level □
A. Sgk gene (WB 09, TN 08) □ 170. Before the onset of puberty, the GnRH neurons are
B. CYP11B2 □ under the inhibitory control of : (AI 04)
Physiology: Endocrine System ■ 179

A. Glycine □ Whether her cycle was ovulatory or not may be


B. Glutamate □ validly assessed by the reported serum level o f :
C. Gamma amino butyric acid (GABA) □ A. FSH (AIIMS 02) □
D. Beta-endorphin □ B. LH □
171. Which of the following is not seen in humans: C. Oestradiol □
A. Estrous cycle (AI 10) □ D. Progesterone □
B. Menstrual cycle □ 181. Estrogen is secreted during pregnancy, mostly by-
C. Endometrial cycle □ A. Maternal ovary (NBE P 13) □
D. Ovarian cycle □ B. Fetal ovary □
172. Ovulation is associated with sudden preovulatory C. Pituitary □
(surge) rise in: (NBE P 15,14,13; AIIMS 04,08,15) D. Hypothalanmus □
A. Prolactin/ GnRH/FSH □ 182. Most common end product of progesterone
B. Testosterone □ metabolism found in urine is: (AIIMS 13)
C. LH □ A. Pregnanetriol □
D. Oxytocin □ B. Pregnenolone □
173. Ovulation in a women with 28 day cycle occurs at- C. Pregnanediol □
A. 14 days prior to menstruation (NBE P 13) □ D. 17 hydroxypregnenolone □
B. Just before LH surge □ 183. There is a mid-cycle shift in the basal body
C. Just after corpus leuteal maturation □ temperature (BBT) after ovulation in women. This is
D. Due to progesterone rise □ caused b y :
174. Positive feedback action of estrogen for inducing A. FSH-peak (AIIMS 03) □
luteininzing hormone surge is associated with which B. LH-peak □
of the following steroid hormone ratios in peripheral C. Oestradiol □
circulation: (AI 12,03) D. Progesterone □
A. High estrogen : low progesterone □ 184. In post ovulatory phase thickness of endometrium is
B. Low esterogen : high progesterone □ because of- (NBE P 13)
C. Low esterogen : low progesterone □ A. Progesterone □
D. High esterogen : high progesterone □ B. Oestrogen □
175. Corpus luteum is maintained in pregnancy b y : (AI 07) C. FSH □
A. Progesterone (NBE P 15,14,13) □ D. LH □
B. LH □ 185. The gene coding for androgen receptors is located on
C. FSH □ A. Short arm of X- chromosome (AI 09) □
D. Estrogen □ B. Short arm of Y- chromosome □
176. Sertoli cells in the testis have receptors fo r C. Long arm of X- chromosome □
A. FSH (AIIMS 07) □ D. Long arm of Y- chromosome □
B. LH □ 186. The enzyme associated with the conversion of
C. Inhibin □ androgen to oestrogen in the growing ovarian follicle is
D. GnRH □ A. Desmolase (AI 04) □
177. Follicular stimulating hormone receptors are present B. Isomerase □
on: (AIIMS 06) C. Aromatase □
A. Theca cells □ D. Hydroxylase □
B. Granulose cells □ 187. Estrogen action on carbohydrate metabolism:
C. Leydig cells □ A. Increases uptake of glucose through increase in
D. Basement membrane of ovarian follicle □ insulin sensitivity (AIIMS 09) □
178. Ovarian reserve is best indicated by (AIIMS 10) B. Glycolysis increases □
A. LH □ C. Increasing central adipose deposition □
B. FSH □ D. Worsening of NIDDM □
C. LH/FSH ratio □ 188. Cause of vasodilation in spider nevi
D. Estrogen □ A. Testosterone (AIIMS 10) □
179. Luteal phase dominant hormone- (NBE P 13) B. Estrogen □
A. Estrogen □ C. FSH □
B. Progesterone □ D. Hepatotoxin □
C. Prolactin □ 189. FSH acts on which of the following:
D. Oxytocin □ A. Granulosa cell (AIIMS 02) □
180. The laboratory report shows values of gonadotropin B. Theca interna □
and ovarian hormones of the blood sample taken, on C. Endometrium □
day 20 of the menstrual cycle of a young woman. D. Myometrium □
100 ■ A Complete Review of Short Subjects

190. Sertoli cells play a key role in which of the following C. Epididymis □
process: (AI 09) D. Vas deferens □
A. Spermiogenesis O 201. Capacitance of sperms takes place in: (NBE P 15,13)
B. Testosterone secretion O A. Seminiferous tubules (AIIMS 10, 09) □
C. Secretion of seminal fluid O B. Epididymis □
D. Production of germ cells. O C. Vas deference □
191. Sertoli cells secrete: (AI 04; NBE P 15) D. Uterus □
A. Testosterone a 202. Spermatogenesis is maintained by all hormone(s)
B. Estrogen o except: (NBE P 14,13)
C. Androstenedione 0 A. Testosterone (PGI 0 8 ) 0
D. Inhibin o B. FSH □
192. Sertoli cells have receptors for: (NBE P 15; AI 04) C. LH □
A. Inhibin O D. Prolactin & Gonadotropin Releasing Hormone
B. Luteinizing hormone O (GnRH) □
C. Follicle stimulating hormone O 203. Dihydrotestosterone acts on- (NBE P 13)
D. Melatonin O A. Pituitary □
193. Sertoli cell feedback mechanism involves- (NBE P 13) B. Exterrnal genitalia □
A. Decreased LH O C. Internal genitalia □
B. Decreased FSH O D. Tests □
C. Decreased TRH O 204. Negative feedback in spermatogenesis by- (NBE P 13)
D. Decreased CRH O A. ABP □
194. Blood testis barrier is located between: (AIIMS 13) B. Inhibin □
A. Sertoli and sartoli cells O C. Progesterone □
B. Leydig and myoid cells O D. None □
C. Sertoli germ cells O 205. Prolonged administration of testosterone in a man
D. Sertoli spematid O causes: (AIIMS 10)
195. Testosterone production is mainly contributed by: A. Azoospermia □
A. Leydig cells (NBE P 15,13; Al 06) □ B. Increased GnRH □
B. Sertolie cells O C. Increased sperm mobility □
C. Seminiferous tubules O D. Increased spermiogenesis □
D. Epididymis O 206. Which of the following hormones is mainly
196. Which of the following statements can be regarded as responsible for skeletal maturation: (AIIMS 02)
primary action of Inhibin? (NBE P 14; AI 05) A. Testosterone □
A. It inhibits secretion of prolactin. O B. Estrogen □
B. It stimulates synthesis of estradiol1. □ C. Growth hormone □
C. It stimulates secretion of TSH. □ D. Testosterone/estrogen ratio □
□ 207. Change in sex hormone binding globulin will most
D. It inhibits secretion of FSH.
affect levels of which of these hormones? (AIIMS 16)
197. Ovary produces all except- (NBE P 13)
A. Testosterone □
A. Gonadotropin O
B. Estrogen □
B. Progesterone O
C. Progesterone □
C. Estrogen O
D. DHEA □
D. Inhibin B O
208. In contraction stress test, positive interpretation is:
198. M ales and females show difference in the age of onset
A. Early deceleration (AIIMS 2K) □
of puberty. The difference in the age of onset of
B. Early acceleration □
puberty amongst males may be explained by:
C. Persistent late deceleration □
A. Increased Activin - A levels (AI 07) □
□ D. Variable deceleration □
B. Decreased Follistatin levels
209. Change in sex hormone binding globulin w ill most
C. Increased Inhibin levels □
affect levels of which of these hormones? (AIIMS 16)
D. Easily releasable FSH pool □
A. Testosterone □
199. Semen is released by- (NBE P 15,13)
B. Estrogen □
A. Epididymis (AIIMS 14,12) □
C. Progesterone □
B. Testes □
D. DHEA □
C. Vas deferens □
210. Antibodies against sperms may develop after:
D. Prolactin □
A. Trauma (NBE P 14,13) □
200. Sperms become motile in: (NBE P 15,13; AIIMS 12)
B. Infection □
A. Prostate (AI 06) □
C. Vasectomy □
B. Seminal vesicles □
D. Orchidectomy □
Physiology: Endocrine System ■ 181

211. The commonest cause of primary amenorrhea with 220. Stress induced hyperglycemia is mediated through
ambiguous genitalia in a female with 46XX which hormone: (PGI 05)
chromosome: (AIIMS 2K) A. Insulin □
A. 21 hydroxylase def. C B. Glucagon □
B. 17 hydroxylase def. C C. Thyroxine □
C. 11 hydroxylase def. □ D. Epinephrine □
D. Desmolase hydrolase def. c E. Cortisol □
212. Velocity of human sperm- (NBE P 13) 221. During stress increased stimulation of cortisol release
A. 1-3 mm/min C cause: (PGI 04)
B. 4-6mm/min c A. Neoglucogenesis □
C. 6-9 mm/min □ B. Glycolysis □
D. 10-13 mm/min □ C. Protein breakdown □
213. PGs in semen are secreted by- (NBE P 13) D. Glucose utilization □
A. Prostate C E. Lipolysis □
B. Seminal vesicle C 222. In stress which hormone is increased (PGI 04)
C. Sperms c A. Insulin □
D. Testes c B. Vasopressin □
214. Not found in sem inal fluid: (PGI 13) C. Adrenaline □
A. Fructose C
B. Prostaglandins c Nitric Oxide
C. Spermine E
D. Citric acid c 223. Mechanism of action of Nitric oxide is through:
E. Inositol c A. cGMP (AI 2K) □
215. Which of the follow ing organs secretes zinc in large B. cAMP □
amount in man? (AIIMS 06) C. Ca++ □
A. Seminal vesicle C D. Tyrosine □
B. Prostate c 224. NO is synthesized by: (AIIMS 09)
C. Epididymis c A. Uracil □
D. Vas c B. Aspartate □
216. Immune rejection in fetus is prevented by- (NBE P 13) C. Guanosine □
A. HCG n D. Arginine □
B. HPL n 225. Nitric Oxide Synthase : (AI02)
C. Oestrogen n A. Is inhibited by Ca++ □
D. Progesterone n B. Catalyzes a dioxygenase reaction □
217. True about placental hormone is A/E (AI 10) C. Accepts electrons from NADH □
A. hCS is diabetogenic C D. Requires NADPH, FAD, FMN &Heme iron C
B. hCG rise 1/1 nausea C 226. True statement regarding Nitric oxide is: (AI 09)
C. Progesterone production require fetal A. NO is synthesized from arginine □
steroidogenic tissue c B. NO is spontaneous produced from NO 2 □
D. Luteal - placental shift at 8-10 weeks c C. NO causes vasoconstriction □
218. Intrauterine growth of fetus is affected by: D. NO is released from mitochondria □
A. Growth hormone (PGI 10) □ 227. True about nitric oxide: (PGI 01)
B. Insulin C A. Free radical □
C. Thyroxine C B. Vasodilator □
D. Glucocorticoids c C. Oxidizing agent □
E. Parathormone c D. Catalyst □
Stress
E. Platelet aggregator □
228. The primary action of Nitric oxide (NO) in the
219. Which of the following increases during surgical gastrointestinal tract is: (AI 10)
stress- (NBE P 13) A. Vasodilatation □
A. Cortisol C B. Vasoconstriction □
B. Glucagon □ C. Gastrointestinal smooth muscle relaxation □
C. Insulin □ D. Gastrointestinal slow smooth muscle contraction □
D. Gastrin □
182 ■ A Complete Review of Short Subjects

ANSWERS & EXPLANATIONS:

Receptors and Second Messengers

1. C i.e. Adrenalin 2. A i.e. Thyroxine 3. A i.e. Insulin & Glucagon 4. A i.e. TSH

Group II (Hydrophilic) Hormones that bind to Cell surface/ Cell Membrane Receptors

Second M essenger is

A. C-AMP - Adenyl Cyclase B. C-GMP C. Calcium or Phosphatidyl D. Kinase or Phosphatase


Alteration of intracellular inositol (or both) or Inositol Cascade
-A trial natriuretic
cAmp conc. leading to triphosphate (IP3)/Diacyl glycerol
factor®
alteration of protein kinase A (DAG) - phospholipase C
-Nitric oxide®
activity 0 Oxytocin® - Growth hormone (GH)®
[Mn: GAN]
- Corticotropin releasing God - Gonadotropin - Epidermal growth factor
hormone (CRH) releasing hormone (EGF)
Mn- "O God II (q)"
- Follicle Stimulating (GnRH) - Fibroblast growth factor
Plate Pe (4) Gas Co
hormone (FSH)® - Growth hormone (FGF)
Os)) Mat (Her) Press
- Melanocyte stimulating releasing hormone - Insulin like growth
And Release Caro
hormone (MSH) (GHRH) factors (IGF) I & II
Os^r)
- Thyroid stimulating II Angiotensin 11 - Nerve growth factor
hormone® (TSH) (vascular & (NGF)
- L u tein izin g h o rm o n e (LH)® smooth muscle) - Platelet derived growth
- Adrenaline® Plate Platelet derived factor (PDGF)
- Adrenocorticotropic Mn- "C - releasing, growth factor - Erythropoietin
hormone (ACTH, F M T - stimulating, (PGDF) - Prolactin®
corticotropin) L/AC/AD/PT Pe Substance P - Insulin®
- a 2 & (3 adrenergic hormone, V & A-2, Gas Gastrin - Chorionic
catecholamines secreting chronic Cho Cholecvstokinin somatomammotropin
- Paratharmone (PTH) glue & static Mat Muscarinic - Adiponectin & leptin
- Anti diuretic hormone lipotropic calcium." (Acetylcholine) - Macrophage Colony
(ADH) Press Vasopressin (ADH)® Stimulating Factor
- Vasopressin (V 2 receptor And Vi receptor, vascular (MCSF)
epithelial cells) smooth muscle (Mn - "Grow th EPIC")
- Angiotensin II (epitheliam Release Thyrotropin
cells) releasing hormone
- Secretin (TRH)
- Human choronic Caro Catecholamines ai
gonadotropin (HCG)
- GlucagonQ ★ a 2 & P adrenergic catecholamines, angiotensin II, acetyl choline & somatostatin inhibit
- Somatostatin adenyl acyclase (HI) whereas others stimulate adenyl acyclase (H5).
- Lipotropin
- Calcitonin

Group I (Lipophilic) Hormones that bind to Intracellular Receptors

Ligand - receptor complex directly provides the signal to specific genes whose rate of transcription is there by affected.

At Cytoplasmic Receptors At Nuclear Receptors

- G lucocorticoids & mineralocorticoids® Includes Thyroid (T3 & T 4)


- Androgens (testosterone), estrogen & progestins® hormones®
- Retinoic a d d (retinoids) ®
- V itD 3 (l,25-(O H )2 D 3) 0
Physiology Endocrine System ■ 183

5. B i.e. Atrial natriuretic factor & Nitric Oxide (NO) 6 . D i.e. Estrogen 7. A i.e. Vitam in D 3 ; D i.e. Thyroid
8. A i.e. Guanylyl cyclase 9. B i.e. Acts by increasing C- AMP 10. A i.e. M ainly on nuclear surface; B i.e. Steroid act
[Ref: Harper 28/e 428, 445-48; Gabriel p 281; Guyton 11/e 910-15; Ganong 23/e p. 50-54; anesthesia volume for detail]

!----------
- Cyclic A M P , C - G M P , IP 3/ D A G are secondary messengers. C-GMP acts on nitric oxide (NO) and ANF ( atrial
natriuretic factor)® as second messenger.
- Intra cellular receptors (steroid receptor super family) is present in thyroxine ( nuclear receptor) and gluco/mineralo-
corticoids, androgen estrogen, progestin, retinoic acid and vitamin D 3 (all cytoplasmic receptors).
- 3 classes of second messengers include (1) cyclic nucleotides (i.e. cAMP & cGMP); (2) IP 3 (inositol triphosphate) &
DAG (diacylglycerol) and (3) calcium ion (Ca2+). Adenyl and guanyl cyclases are not second messengers®, they
catalyze formation of cAMP & cGMP respectively.

11. A, B, C, E i.e. Insulin, PTH, Renin, Angiotensin II [Ref: Berne & Levy 6/e p-656-58; Guyton 11/e 907-224; Gabriel p 280-81]

- Sterod hormones are synthesized from cholesterol® and example includes adrenocortical hormones (i.e. cortisol &
aldosterone®), sex hormones (i.e. testosterone, estrogen & progesterone)® and vitamin D3 = 1,25- (OH)2
cholecalciferol (a secosteroid because one of the rings of steroid nucleus , the B ring, is open)
Amine hormones are derived from aminoacid tyrosine and examples include catecholam ines (adrenaline /epinephrine
and noradrenaline/nor epinephrine)® from adrenal medulla, thyroid hormones (triiodothyronine = T3,
tetraiodothyronine = T4 = thyroxine)® and prolactin inhibiting factor /PIF (dopamine) from hypothalamus.
- All remaining hormones are either low molecular weight proteins (i.e. poly peptides) or glycoproteins. They are
synthesized initially as larger peptide precursor® (eg prepro-insulin) and then cleaved off until the final smaller active
molecule is produced. Some are also coupled with carbohydrates to form glycoproteins.

13. D i.e. Adenylate cyclase [Ref: Ganong 22/e P-44-46]

Adenylate cyclase is an enzyme that occurs on the surface of cell membranesQ


It belongs to the family of cell membrane receptors that are linked to the effecter through one or more GTP activated G-
proteins (G-protein coupled receptor) ®
Cycle AMP is an important second messenger that is formed from ATP by the action of enzyme adenelyl cyclase.Q
- C yclic A M P acts by activatin g a cyclic n u cleo tid e - d ep en d en t p ro tein k in ase (protein kin ase-A ) w hich
catalyzes the ph osp hory lation.

14. A i.e. ADP ribosylation 15. C i.e. Causes continued activation of adenylate cyclase
16. A i.e., Activation of protein kinase [Ref: Harper 29/e p 732-33; Ganong 22/e P- 43; Harrison 16/e P- 755] .

Cholera toxin (A l subunit) catalyzes ADP ribosylation o f G s a subunit and causes persistent activation o f adenyl
cyclase®. This results in elevation of cAMP, which activates protein kinase A® and causes phosphorylation of CFTR and
Na+ - H+ exchanger. This intum leads to inhibition of Na+ absorption and enhancement of Cl~ secretion. Thus massive
amounts of NaCl accumulate inside intestine lumen contributing to liquid stools (watery diarrhoea) characteristic of
cholera.
184 ■ A Complete Review of Short Subjects

17. C i.e. Ca++[Ref: Harpers illustrated Biochemistry 28/e p. 428; 27je P-471,445; 26/e P-463,437;Ganong 22/e P-42- 45]

Ligand binding to receptor activates phospholipase- C (PLC) on the


S tim u la to ry
inner surface of membrane via Gq. The Bi & B2 forms of PLC are receptor
activated by G protein and catalyze hydrolysis of PIP 2 (phosphatidyl ISF

inositol 4, 5 diphosphate) to form IP 3 (Inositol 1 ,4, 5 - triphosphate) and

m PIP, -> DAG

DAG (Diacyl glycerol)


The IP 3 diffuses into sacroplasm ic reticulum & releases calcium into the
cytoplasm Q. DAG is also a 2nd messenger and it activates protein Gq, etc IP j Phosphoproteins
Tyrosine
kinase - C
Kinase
* Tyrosine kinase linked receptors can convert PIP 2 —»IP3 & DAG by Cytoplasm
activating PLCYi
CaBP «----- Ca
* IP 3 receptor resemble ryanodine receptor (a Ca++ channel in
t Physiologic
sarcoplasmic reticulum of skeletal muscle) except that IP3 receptor ER
effects
Physiologic effects
is half as large.

18. E i.e. Testosterone, cortisol & Estogen (nearest answer) [Ref: Harper 28/ed p. 457; Lehninger 5/e p. 422-456]
- f t
Glucagon, TSH , GnRH &LH all bind to cell surface receptors. So nuclear receptors (here it means intracellular steroidal
receptors i.e. both nuclear & cytoplasmic receptors) are used by testosterone, cortisol and estrogen (answer of exclusion).

19. B i.e. Transcriptional mediators [Ref: Harper 27/e p-464- 66, 476- 80, Molecular cell biology December 19(12); 8383-8392; L
costanzo physiology 3/e p-363]

Steroid receptors bind to hormones, hormone response elements (HRE), and transcription coregulator proteins such as co­
repressors (NCoR, SMRT), co- activators (CBP, P-300) and m ediator related protein Q.

• It is a steroid hormone receptor that belongs to nuclear receptor subfamily 3, group C, member 2 (NR3C2). It is activated by
mineralocorticoids eg. aldosterone & deoxycorticosterone, as well as by glucocorticoids (such as cortisol & cortisone) and
some progestins.
• Its activation in epithelial cells leads to expression of proteins regulating ionic and water transport (mainly epithelial
sodium channels. ENaC, Na+ - K+ pump, serum & glucocorticoid induced kinase or SGK1) resulting in absorption of
Na+ causing increased ECF volume & BP, and an excretion of K+ to maintain a normal salt concentration in body.

22. B i.e. For specific action; C i.e. M olecular transport 23. A i.e. Nicotinic cholinergic receptor (D) GABAa receptor
[Ref: Goodman & Gilman 10/e P34, 35; KDT S/e p. 42; Harper's 27/e P-445; 26/e P-435-436]

- Cell membrane receptors bind to specific molecule and transport it via endo/exo-cytosis giving specificity to process.
- Cell surface receptors with intrinsic ion channel include nicotinic cholinergic, glutamate, glycine, GABAa and 5 HT3
receptors °.

24. A i.e. Breakdown of GTP to GDP 25. C i.e. Made up of 4 units


[Ref: Ganong 22/e P- 41-44, Guyton 11/e p. 911; KDT 6/e p. 45-48]

[ g protein is made up of 3 subunits - a , fi, and ’f i. And the intrinsic GTPase activity of a subunit converts GTP to GDP®?
Physiology: Endocrine System ■ 185

26. A i.e. It interacts transmembrane; B i.e. GTP to GDP; C i.e. Adenyl cyclase activation ..; E i.e. Helical structure
27. A i.e. It binds to ligands at the cell surface

G protein coupled receptor (GPCR) binds to the ligand at the cell surface®. It is a seven transmembrane domain (7TM)
or haptohelical or serpentine receptor. Separation of a subunit from |i, ysubunits brings about biological effectQ. Py
subunits acts on adenyl acylase ( Ting cAMP, stimulating PKA and phosphorylating receptor)®. On ligand removal, the
intrinsic GTPase activity of a subunit inactivates itself by converting its bound GTP into GDP®.

28. D i.e. Increased cAMP [Ref: Harper 28/e 4481

G s-a subunit gain would cause increased adenyl cyclase level - ultimately resulting in increased c-AMP level®, (because
adenyl cyclase converts ATP to c-AMP).

29. C i.e. Seven pass - receptor [Ref: Guyton 11/e p. 910-13; Ganong 24/e p. 58-59]

Catecholamines (epinephrine, norepinephrine and dopamine) act through G protein coupled receptors (GPCRs), - f t

which span the cell membrane 7 times; hence also referred to as seven pass (seven helix) or serpentine receptors.

30. D i.e. Substance that increase or decrease function 31. B i.e. Enzymatic composition
32. D i.e. CyGMP 33. Ai.e. Insulin 34. D i.e. Estrogen
35. A i.e. FSH 36. Ci.e. Insulin 37. C i.e. NO
38. A i.e. Guanylyl cyclase 39. A i.e. CAMP B i.e. DAG C i.e. IP3 D i.e. Ca++ 40. C, D i.e. cAMP, Ca++
[Ref: Berne & Levy 6/e p 34-42; Best & Taylor 13/e p 821-25; Ganong 24/e p 54-63; Lehninger 4/e p434; Harper 27/e p 445-52, 428;
Lippincott 4/e p 422-43; Guyton 12/e p 888-91; KDT 6/e p 231-32]
1 '
- Second messengers (concept proposed by E.W. Sutherland) are intracellular signaling molecules mediating
intracellular hormone / ligand (1st messenger) function.
- Second messangers respond differentially to a hormone or drug because of receptors but cells respond to second
messangers differentially because o f different enzymatic composition®.
- A surprising finding in mid 1980s w as the role o f nitric oxide (NO) as an important biological messenger■Q.
- NO is a short lived messenger that act by stimulating a guanyl cyclase raising cGMP and stimulating protein kinase G®.
- C- AMP, C- GMP, IP 3, DAG, Ca++ and protein kinase® function as second messengers.

Calcium Metabolism and Parathyroid Gland

41. B i.e. Calmodulin [Ref: Harper's 27/e P- 471, 579; Ganong 24/e p. 57-58]

Calmodulin is a small Ca++ binding protein that modifies activity of many enzymes/ proteins in response to changes of
Ca++ concentration.

42. A i.e. Absorption of phosphorus increase 43. A i.e. Proximal small intestine
44. A i.e. Proteins 45. C i.e. Parathormone (PTH) promotes absorption of Ca2+ from intestine

Parathyroid hormone increases ☆ Parathyroid Hormone (PTH)


phosphate excretion in urine
due to decrease in reabsorption Bone Kidney Intestine
o f phosphate in the proxim al - PTH acts directly on bone to J.p o 43- TCa2+ absorption (via
tubules®. PTH promotes increase bone resorption®, reabsorptionQ. activation of V it D)Q,
absorption of calcium from - On a longer times cale, PTH - tC a 2+ reabsorption mainly from proximal
intestine® (mainly proximal stimulates osteoclast & in distal tubuleQ small intestine.Q
sm all intestine)® via activation osteoblast, with the effect on
of vitamin D (1,25, dihydroxy osteoclast predominating so more
cholecalciferol). Ca2+ is mobilized from bone.
186 ■ A Complete Review of Short Subjects

46. A i.e. Ionised calcium 47. C i.e Alkaline pH in gut


48. A i.e. Absorbed in upper small intestine; D i.e. Absorption increased by acidic pH 49. A,B,C i.e. PCT, DCT, LH
[Ref: Ganong 24/e p. 56-58, 377-78;Berne 6/e p. 700-2, 628-32; Harper 26/e p. 444; Chatterjee 7/e p. 875; Harrison 17/e p. 286/
^ ^ ^ ^ ^
Free ionized calcium is physiologically active form o f calcium®, that is a vital second messenger and is necessary for
blood coagulation, muscle contraction and nerve function.
Calcium is absorbed in upper sm all intestine (duodenum & proxim al jejunum) and the absorption is increased by
acidic pH®.
Calcium sensing receptors (CaSRs) are present in PTH secreting cells of parathyroid glands, calcitonin secreting
parafollicular cells in thyroid gland and caldtriol (Vit D) producing cells of proximal tubules (i.e. in the cells
producing hormone involved in regulation of serum calcium). Ca SRs are also present in thick ascending limb o f loop
o f Henle and distal tubule (DCT)® to directly regulate Ca++ homeostasis. Sm all intestine cells lack CaSRsQ.
The active transcellular Ca++ absorption is facilitated in enterocytes and nephrons by epithelial Ca++ channels TRPV 5
and 6 (on apical epithelium), calbindin D9k (in cytoplasm) and plasma membrane Ca ATPase (PMCA; in basolateral
membrane). The sodium/calcium exchanger (NCX) also transport calcium out of enterocytes.

Absorption of Calcium is

Decreased by Increased by
■Achlorhydria®
■Taking drugs that inhibit gastric acid secretion ■ ProteinQ (also increase Mg absorption)
Pancreatic & biliary non sufficiency where ingested calcium remains bound to - Hypocalcemia
unabsorbed fatty acid etc. - Acidic pH in stomach
Phosphates & oxalates 0 _ LactoseQ
Alkalis (Alkaline pH)Q . p j f j & y jt jyo
Hyper calcemia (TCa++)
High Mg++, PO 4-3,
Phytic acid, Caffeine, Dietary fibers

50. C i.e. Increased Parathormone secretion 51. B, D i.e. Stimulate 1 ,25 D3 formation, Increases the resorption of bone
[Ref: Best b Taylor 13/e p -886-888; Ganong 24/e p 381-84; Guyton 12/e p 962-661
V -J
- Synthesis & secretion of parathyroid hormone (PTH) are regulated primarily by serum ionized calcium concentration
sensed by calcium sensing receptor (CaSR) located on chief cell o f parathyroid gland. When ionized serum Ca++ levels
decrease below 1.3mM (eg d/t prolonged low calcium diet), PTH synthesis and secretion increases (to restore Ca++
levels). Whereas when extracellular serum Ca++ increases above 1.3 mM, PTH decreases & serum calcium diminishes.
- Parathyroid hormone (PTH) is a 84 aminoacids linear polypeptide hormone. PTH stimulates 1,25,
dihydroxycholecalciferol [1,25 (OHIj D 3] formation, promotes absorption of calcium mainly from proximal small
intestis and increases bone resorption. PTH also increases reabsorption o f calcium in distal tubules (via TRPV 6 =
Transient Receptor Potential Vanilloid type 6 channels) and decreases reabsorption of phosphate in proximal tubules
(via Na-Pi Ha). In nutshell it increases serum calcium & decreases serum phosphate®.

52. C i.e. Hypophosphatemia [Ref: Ganong 22/e P-392, 391 ]

PTH increases phosphate excretion in urine causing hypophosphatemia®. This phosphaturic action is due to decreased
reapsorption of phosphate in distal tubules.

Secondary Hyperparathyroidism d/t -V it D deficiency

- This results from decreased serum Ca2+.


- The released parathyroid hormone directs its activity to increase serum Ca2* at the cost of of loss of phosphate from the body
- hypophosphatemia®.
- PTH mobilizes both Ca2+ & phosphate from the bone to increased level of both in blood initially, while it reabsorbs the
filtered Ca2+ from renal tubules, it fails to reabsorb phosphate which is lost in urine.
Physiology: Endocrine System ■ 187

53. A i.e. PTH; B i.e. Calcitonin; D i.e. 1, 25 DHC IRef: Ganong 24/e p377-871
- f t
Seram calcium level is regulated by parathormone (PTH), vitamin D (i.e. 1 ,2 5 dihydroxy chole calciferol) and
calcitonin. V it D and PTH increase whereas calcitonin decreases serum Ca++ level.

54. B i.e. Calcitonin 55. B i.e. Decreased bone resorption 56. B i.e. Calcitonin [R ef: G a n o n g 2 2 /e p. 3 9 3 -3 9 4 ; K D T 5 /e P-301 ]

’ '
Calcitonin, a hormone secreted by parafollicular cells or C cells of thyroid gland, lowers the serum calcium levels.
Calcitonin exerts its Ca2+ lowering effect (hypocalcemia) by inhibiting bone resorption. This action is direct and
calcitonin inhibits the activity of osteoclasts. It also increases Ca2+ excretion in urine.

Effect of Hormone on Calcium & Phosphate Level

Hormone Serum Serum Bone cells on which their


Calcium phosphate receptors are present
V it D Increases^ Increases Osteoblast
PTH Increases'^ DecreaseQ Osteoblast
Calcitonin Deere asesQ Decreases O steoclasts

57. C i.e. Increased excitability of muscle and nerve.


58. A i.e. Low Ca++ C ausest Na+ permeability [Ref: Ganong 22/e P- 59; Gyuton 10/e P 315]

• A decrease in extra cellular Ca** Concentration increases the excitability o f nerve and muscle cells by decreasing the
amount o f depolarization necessary to initiate the change in Na* & K* conductance that produce the action potential®.
In fact Ca++needs to fall only 50% below the normal before spontaneous discharge occur from peripheral nerves, often
causing tetany. Conversely, an increase in ECF Ca** stabilizes the membrane by decreasing excitability®.
• The voltage gated calcium channels are slightly permeable to Na+ as well as Ca++; when they are activated both Na* and
Ca** flow to interior of the fiber. When there is deficit of Ca2* in ECF, Na* channels become activated (opened) by very little
change of membrane potential (from its very negative resting level to a less negative level). Therefore nerve fibers become
highly excitable. Sometime discharging repetitively without provocation rather than remaining in resting stage.
• Decreasing the external Na+ concentration decreases the size o f action potential® but has little effect on RMP.
Increasing the external K+ concentration decreases the RMP®.

59. C i.e. Shortening of QT interval in ECG. [Ref: Harrison 16/e P-2263; Schwartz 7/e P-69; Ganong 22/e P-382]

Q T interval (ventricular depolarization + repolarization) on ECG is prolonged in hypocalcemia®.(as Ca2+ is necessary


for contraction) and is shortened in hypercalcemia®.

60. C i.e. Spleen 61. D i.e. Lungs


62. D i.e. W illiams syndrome consists of mental retardation, obesity & precocious
63. B i.e. Increased 24,25 dihydroxycholecalciferol 64. C i.e. 1 , 25 (OHh vitamin D
[Ref: Harrison 17/e p. 2138; 16/e p- 385,1382, 2258, 2246; Harper 27je p- 453, 492 , Shils - Modern nutrition in health & disease 9/e
p-287, Ganong 22/e p. 387- 89; Braunwald's 8/e p-1085; Chatterjee Shinde 6/e p. 151; Guyton 11/e p. 983-85]

Spleen ( » Lung) are not involved in calcium metabolism (homeostasis)Q. But skin, kidney, liver and intestines ☆
are involved in calcium metabolism.
In liver vit D j (cholecalciferol) is hydroxylated to form 25, hydroxy cholecalciferol (calcidiol)Q, which is released in
circulation bound to a globulin, the main storage form of vitamin D. In kidney calcidiol undergoes either 1
hydroxylation to form active metabolite 1 ,25 dihydroxy cholecalciferol (calcitriol)Q or 24 hydroxylation to yield an
inactive (probably) metabolite 2 4,25 dihydroxy cholecalciferol (24- hydroxy calcidiol). W hen calcium level is high,
minor pathway becomes main i.e. little calcitriol is produced and kidney m ainly produce relatively inactive 24,25
dihydroxy cholicalciferolS.
188 ■ A Complete Review of Short Subjects

• W illiam's syndrome is autosomal D2, D3 (Diet) 7 - dehydrocholesterol (Skin)Q


dominant idiopathic hyper calcemia of | S u n l i g h t (UV)
infancy^. It is characterized by multiple
Pre Vitamin D3
congenital developmental defects such as
I (Thermal isomerization)
supravalvular aortic and peripheral
pulmonic stenosis, mental retardation, an Cholecalciferol (Vitamin D3)
elfin facies, loquacious personality and Liver j 25 HydroxylaseQ
hoarse voice, in association with hyper 25 (OH) V it D 3 = Calcidial = Main Storage form
calcemiaQ d/t abnormal sensitivity to
vitamin D. It is probably d/t mutations Kidney(PCT cells)
involving elastin locus on chromosome 7
24,25 (OH ) 2 24 H ydroxyla$j 1 a HydroxylaseQ
(7q 11.23). Other clinical features are ------
feeding difficulties, failure to thrive, vitamin D 3 M in o r Path (rate lim iting step)
gastrointestinal problems such as vomiting,
1,25 (OH ) 2 V it D 3 = Calcitriol = Active form
constipation, & colic along with auditory
hyperacusis, inguinal hernia, outgoing & — r—
Absorption of Negative feedback to Mineratization
engaging personality, strabismus, dental
Ca2+ from gutQ. - Parathormone (PTH) of bone
development abnormalities (eg.
Microdontia enamel hypoplasia Si - 25(OH)D, la-hydroxylase (i.e.
malocclusion) and narrowing of peripheral l a hydroxylase)______________
systemic and pulmonary arteries.

65. D i.e. Collects calcium into the sarcoplasmic reticulum. [Ref: Pretest physiology 9/e Q NO - 15]

Phospholamban is a protein contained within the sarcoplasmic reticulum that inhibits the activity of sarcoplasmic
calcium pumpQ. Inactivation of phospholamban results in an increase in calcium sequestration by sarcoplasmic
reticulum.Q
In cardiac muscle, the rapid sequestration of Ca2+ shortens the duration of contraction.
In smooth muscle, sequestration causes the muscle to relax.
Phospholamban has little effect on skeletal muscle contraction.

66. B i.e. Parathyroid hormone related protein (PTHrP) [Ref: Harrison 16/e P-566] .

Ar4
Hypercalcemia related to malignancy is also k/a Humoral hypercalcemia of malignancy (HHM) is most often
associated with over production of PTHrP (Parathyroid hormone related protein)Q.

67. A i.e. Parathormone [Ref: Harper 26/e P-438; Shinde 6/e P 490]

Hormones can be classified chemically into three major groups

Ainjxto Peptide/Pttrtein
- Glucocorticoids Derived from tyrosine - P arath orm on eQ
- Mineralocorticoids - Catecholamines - Calcitonin
- Estrogen - Thyroid hormone - Insulin
- Progesteron - Glucagon
- Androgen - Pituitary hormones

68. C i.e. Required hydrolysis of ATP, E i.e. It is an active process


69. A i.e. Inositol triphosphate [Ref: Ganong 24/e P-56-58; Sembulingum 3/e P - 19; NMS Biochemistry 4/e P 263]

- Calcium can enter cell from ECF passively down its electrochemical gradient. But the movement of calcium outside
of cytosol (ie across the plasma membrane or the membrane of internal store ie ER & mitochondria) is against
electrochemical gradient so is an active process requiring hydrolysis of ATPQ.
- In ositol triphosphate (IP 3 ) diffuses to the endoplasm ic reticulum, where it triggers the release o f calcium into
cytoplasm Q. I P 3 is the major second messenger that causes release of Ca++ from ER through the IP3 receptor activation
(a ligand gated channel). So one second messenger (IP 3) causes release of another second messenger (Ca++)Q.
Physiology: Endocrine System ■ 189

Carbohydrate Metabolism & Pancreatic Hormone

70. D i.e. PP is secreted by D l cells 71. C i.e. Proteolytic enzymes secreted in inactive form 72. A i.e. Trypsin
73. A i.e. Osmolality is Vi of the osmolality of blood [Ref: Ganong 23/e 328-30, 316; Berne & Levy 6/e p 675-76; Guyton 12/e p.
939; Robb in's 7jc 940-41] .

Pancreas is composed of 2 types of tissue (1) exocrine acini constituting Islets cell type Secretion
of gland, which secrete digestive enzymes e.g. trypsin and chymotrypsin into the A cells Alpha Glucagon*?
duodenum, and (2) endocrine islets of Langerhans, which secrete hormones (a)
directly into blood (and constitute 2-3% of gland). Remainder volume is B cells Beta (fi)Insulin*? &
constituted by ducts & blood vessels. amylin
Pancrease (exocrine part) secretes enzymatically inert proenzymes (eg D cells Delta (S) Somatostatin*?
trypsinogen) to prevent self digestion*?. PP cells Pancreatic
F cells
Pacreatic exocrine secretion is isotonic with blood plasma*?. It is alkaline w ith polypeptide*?
1.5 lit/day secretion rich in enzymes®. Secretin enhances its bicarbonate D l cells V IP
component.

74. B i.e. Increased glucose tolerance 75. C i.e. Decreased growth hormone [Ref: Shinde Chatterjee P-465]

On 5th day of fasting, blood glucose level will be low and all the physiological changes will work in the direction of
increasing the blood glucose level. So glucagon and growth hormone (GH) are increased*?. Whereas, insulin and glucose
tolerance is decreased*?. FFA levels are also increased*?.

76. B i.e. Sanger 77. A i.e. 1 78. B i.e. 4-6min 79. E i.e. C - Peptide
80. A i.e. ATP dependent K+ channels 81. B i.e. Epinephrine 82. C i.e. VIP
83. C i.e. Adrenalin 84. B i.e. Hypokalemia 85. D i.e. Insulin
86. C i.e. Required for transport of glucose, aminoacid, K+ & Na+ 87. A i.e. 3rd month
88. A i.e. Decreased levels of cyclic AMP 89. C i.e. Inhibits Pyruvate dehydrogenase
90. A i.e. Glucose B i.e. Vagal Stimulation C i.e. Acetyl choline [Ref: Lehninger 5/e p 903-904,93-44; 292; Ganong 23/e 319-28;
Guyton 11/e 366, 968-69; Greenspn Endocrinology 8/e p 115; Gabriel p 312]

Nobel prize for sequencing insulin aminoacid sequence & molecular structure was given to Sanger*?.
Insulin is secreted along with C-peptide in a 1:1 ratio*?. Pork insulin differs from human insulin by only 1 aminoacid
residue*?. H alf life of human insulin is 5 minutes*?.
Glucose mediated insulin release is mediated through ATP dependent K+ channels*?.
Insulin secretion is inhibited by epinephrine (adrenaline)*? and increased by gastrin, secretin, GIP, CCK*?, glucose,
vagal stimulation and acetyl choline*?.
Insulin promotes the transport of K+ (potassium) and phosphate (P 0 4 ) into cells leading to hypokalemia and
hypophosphatemia*?, (i.e. 1 K+ & I P 0 4).
Insulin secretion begins in fetus by 3rd month*?. Insulin is required for transport of glucose, Na+, K+ and aminoacids*?.
Increased insulin to glucagon ratio causes decreased level of cyclic AMP*?. Insulin promotes lipogenesis by
activating pyruvate dehydrogenase in adipose tissue*? (but not in liver).

91. A i.e. Ketosis [Ref: Ganong 22/e P-289-300; Harper 26/e P-189]

Three conditions lead to deficient intracellular glucose supplies & hence to ketoacidosis: Starvation, DM (Diabetes
M ellitus) and high fat low carbohydrate diet*?.

Low Carbohydrate D iet

- Cause defective intracellular glucose supply, and most of the calories are supplied by fat.
- When there is high rate of fatty acid oxidation the ketone bodies are formed in hepatic mitochondria to provide calories
(ATP) & they accumulate in blood (ketosis)®. Ketone bodies are important fuel in extrahepatic tissue. Small amount
of glucose abolishes the ketosis & for this reson carbohydrate is said to be antiketogenic®.
190 ■ A Complete Review of Short Subjects

92. B i.e. Von G ierke's disease 93. A i.e. Gluconeogenesis 94. A i.e. Insulin 95. D i.e. Ketogenesis
96. A, D, & E i.e. Active lipoprotein lipase, Activate Acetyl CoA carboxylase, & Decrease free fatty acid level
97. B i.e. Pyruvate carboxylase [Ref: Harper's 28/e p. 221-23,170-71, 416,150; Lippincott's 5/e p. 347; Chatterjea 7/e p. 549; Ganong
23/e p. 318-19; Satyanarayan 3/e p 671-72; Harrison's 17th/e p. 221-23]
- Insulin acts through activation of receptor tyrosine kinase activity*? (an enzyme receptor). Insulin prevents
ketogenesis and diabetes m ellitus (i.e. lack of insulin) promotes ketogenesis*?.
- Insulin increases lipogenesis (i.e. FA, TG & glycerol P synthesis) and decrease lipolysis and so free fatty acid level*?
- Insulin activate acetyl CoA carboxylase (rate lim iting enzyme of FA synthesis), activate lipoprotein lipase (so
increasing TG synthesis) but inhibits hormone sensitive lipase.*?
- Insulin decreases gluconeogenesis by inactivating pyruvate carboxylase*? etc and increases glycolysis by inducing
phosphofructokinase and pyruvate kinase enzymes*?.
- Patient with hepatomegaly & hypoglycemia not responding to epinephrine most probably has von G ierke's disease.

98. A i.e. Beta cells of pancreas 99. A i.e. GLUT1 & 3 100. D i.e. GLUT 4
101. D i.e. GLUT-4 102. A i.e. Adipose tissue & Skeletal muscle 103. B i.e. Heart
104. B i.e. T permeability across cell membrane against glucose gradient
[Ref: Harper's 28/e P - 171; 26/e P-160, Lippincott's 3/e P-95, 310, Ganong 24/e P 434}

- Insulin increases glucose uptake by increasing the number of glucose transporter in cell membrane. Direct insulin ☆
stimulated glucose uptake is mediated by Glut 4 and is seen only in: 1) Muscle: Skeletal muscle*? and cardiac muscle*?
2) Adipose tissue*?. Insulin acts on glucose metabolism by increasing permeability of glucose across cell
membrane*?.
- GLUT 3 is used by brain (placenta & kidney) and GLUT 1 is used by blood brain barrier (BBB, brain, placenta, kidney,
colon & RBC).

Glucose Transporters In Mammals

Facilitated Bidirection Function M ajor Sites of Expression Km


Diffusion
GLUT 1 Basal glucose uptake - Placenta, blood - brain barrier, brain®, 1-2
RBC, kidney, colon & other organs
GLUT 2 B cell glucose sensor; transport out of - B cells of islets*?, Liver, Sm all intestine, 12-20
intestinal & renal epithelial cells. Kidney
GLUT 3 Basal glucose uptake - Brain, placenta, kidney <1
GLUT 4 Insulin stim ulated glucose uptake *? - Skeletal & cardiac muscle® 5
- A dipose tissue®
GLUT 5 Fructose transport - Jejunum, Sperm 1-2
GLUT 6 None - Brain®, spleen, kidney -

GLUT 7 Glucose 6 -PO 4 transporter in - Liver -


endoplasm ic reticulum
Secondary Active Transport Na+ glucose cotransport (unidirectional -transporter)
SGLT-1 Active uptake o f glucose against Small intestine, renal tubule 0.1 - 1.0
SGLT-2 concentration gradient Renal tubule 1.6

105. C i.e. Phlorhizine 106. B i.e. Tetradotoxin 107. A i.e. y N a+during action potential
[Ref: Ezcilo p. 13, 28, 271, 293, 296; Shinde & Chatterjea 7/ed pg-134, 571; Ganong 23^/e p. 453-54, 676]

Phlorizin inhibits Na dependent glucose transport*?. Whereas phloretin inhibits Na+ independent GT (GLUT2).
Tetrodotoxin blocks sodium channels during action potential*?.

108. D i.e. mRNA of insulin [Ref: Lehninger 4/e P 318-19]

Insulin is synthesized on a large scale basis from complementary DNA's (cDNA), using recombinant technology. But
cDNA is not the starting material. cDNA are derived from mRNA <?.
Thus mRNA is starting material for insulin synthesis on a large scale basis.
Physiology: Endocrine System ■ 191

• Construction of cDNA from mRNA m RN A (from specific cells of an organism)


• This duplex DNA is inserted into an appropriate cloning i Reverse transcriptase
vector which produces large num ber cDNA clones. This Com plem entary DNA strand
method of producing DNA clones from RNA is J,

R ecom bin an t D N A techn ology. mRNA - DNA hybrid


I
mRNA is degraded by alkali
i DN A polym erase I
Double stranded DNA (cDNA)

109. B i.e. Hepatic steatosis IR ef: B ern e 6je p. 672, 686, 791]

Insulin resistance in patients w ith liver involvement (eg d/t obesity) is because of lipotoxicity leading to hepatic
steatosis or fatty liver and nonalcoholic steatotic hepatitis (NASH).

110. A i.e. Decreased insulin [R ef: H arrison 1 7 je p. 2 3 0 5 1

Decreased insulin is the first (1st) response to hypoglycemia^ followed successively by increase in glucagon,
epinephrine, cortical & GH.

111. E i.e. Insulin: 112. A i.e. Glucocorticoids; B i.e. GH; D i.e. Epinephrine
113. A i.e. Epinephrine; B i.e. GH; C i.e. Glucocorticoid.
[R ef: C S D T 11/e p 103-105; Best & T aylor's 13/e p 831, 840, 874; L ippin cott's 2 /e P -30; Lee's 386, Sam pson & W right 3/e P 5 1 9 -5 2 0 1

Glucagon, catecholamines (epinephrine & norepinephrine), cortisol, ACTH, GH (growth hormone) are increased in
stress (infection, surgery, hypoglycemia) whereas, insulin is suppressed (decreased)Q.

114. A i.e. VLDL [R ef: G an on g 22/e P -302]


S ----------------------------------------------------------------------
- Maximum triglycerides are in chylomicrons^ ---------------------------
- Exogenous (Dietary) triglycenrides are maximum in chylomicrons^
- Endogenous triglycerides are maximum in VLDLQ

Pineal Gland & Pituitary Gland

115. C i.e. Pineal gland [R ef: G an on g 22/e P -462I

M elatonin is synthesized & secreted by pineal glandQ.

116. B i.e. Increasing the serotonin N- acetyl transferase [R ef: G an on g 2 2 /e P -463-64]


/
Increased in N-acetyl transferase o f hydroxy indole-o-methyl (HOM) transferase activity in darkness 1/t increased
m elatonin secretionQ _________________________________________________________________________________________

117. A i.e GH 118. A i.e. Anterior hypophysis > B ie. Posterior hypophysis 119. D i.e. Prolactin
120. Ai.e. Prolactin & GH 121. A i.e. Oxytocin - Vasopressin 122. A i.e. Oxytocin & ADH
123. C i.e. Pars nervosa [R ef: G an on g 2 2 /e P- 396, B D C 4/e V olum e 3 P 99; G u yton 12/ep 898]

M ost anterior pituitary horm one secretion is increased by hypothalam us. But prolactin is under its inhibitory control
(prolactin inhibitory hormone=PIH). Grow th horm one has both GH RH & GH IH (i.e. releasing & inhibitory control).
RH = Releasing H orm one & IH = Inhibitory Hormone.
192 ■ A Complete Review of Short Subjects

Pituitary gland Hormone secreting cells of Anterior


Pituitary
Adenohypophysis Neurohypophysis /Pars Nervosa
(Anterior Pituitary) (Posterior Pituitary)
Acidophillic Cells Basophillic Cell®
Develops as an upward growth Develops as a dow nward growth
called Rathcke's pouch from from the floor of diencephalons & Cell type Hormone Cell type Hormone
ectoderm al roof of the connected to hypothalam us by Som atotrope Growth Corticotrope ACTH
stomodeum. neural pathways. hormone
Thy ro trope TSH
Hormones Hypothalmic Oxytocin & ADH is synthesized in
Lactotrope Prolactin Gonadotrope FSH, LH
secreted control supra- optic and paraventricular
TSH*? TRH nuclei o f o f hypothalamus® in
ACTH*? CRH ribosom es of cell bodies. After
★ Melanocyte stimulating hormone (MSH) is
FSH*? GnRH removing leader sequences in ER,
secreted by intermediate pituitary gland®.
LH*? Secretary granules (Herring bodies)
However, interm ediate lobe is rudimentary in
Prolactin*? PIH are packed in golgi apparatus and
humans & it appears that MSH is not secreted
Growth GRRH, transported by axoplasmic flow to
in adults.
hormone GHIH pars nervosa*?. Herring bodies store
ie hormones secreted by post.
Pituitary__________________________
Oxytosin®
Arginine vasopressin (ADH)®.

124. A i.e. Early moming [Ref: Ganong 22/e P-373] 125. C i.e. Evening

- ACTH level is highest during early moming®. About 75% of daily output occurs between 4 AM and 10 AM. Secretion is
least in the evening. The biological clock responsible for diumal variation of ACTH is located in suprachiasm atic nuclei
o f hypothalamus®.

A C TH secretion is increased in stress (1. mental or em otional stress, endogenous depression, 2. physical stress or
illness, exercise), alcohol abuse, obesity and ACTH producing tumors.

126. B i.e. Growth hormone [Ref: Ganong 22/e P-409, 402]

Tumors of somatortopes of the anterior pituitary secrete


large amount of growth hormone, leading in children to
gigantism and in adults to acromegaly.
Hypersecretion of GH is accompamied by
hypersecretion of prolactin in 20 - 40% of patient with
acromegaly.

Somatomedian: are polypeptide growth factors secreted


by liver & other tissues. The effect of growth hormone on
growth, cartilage and protein m etabolism depend on
interaction betw een GH & somatomedian.

127. A i.e. Deposition of chondroitin sulfate [Ref: Ezeilo p. 288, 310; Guyton llth /e p. 923-24; Ole Holger petergson 5/ep - 247;
Hans Hess: vol 17jpg- 223 (both on books, google.com) Ganong 23rd/e p. 382-84; 318, 321 ]
■ ■—■■ ....
Growth hormone exerts much of its effects (especially on growth, cartilage & protein metabolism) through intermediate
polypeptide growth factors also k/a somatomedin or insulin like growth factor secreted by liver & other tissues. The first
of somatomedin isolated was called sulfation factor because it stimulated incorporation of sulfate into cartilage (i.e.
deposition of chondroitin sulfate)*? and collagen formation.
Physiology: Endocrine System ■ 193

128. C i.e. Sleep [Ref: Ganong 24/ep 330]

- Going to sleep, Hypoglycemia, Fasting, ExerciseQ, Stress


REM SleepQ, Glucose, Cortisol, Free fatty - 2 -deoxyglucose, Protein meal, Arginine infusion, T blood level of
acids, GH, IGF-1, Medroxy progesterone amino acids, Lysine vasopressin, Pyrogen, Glucagon
- L Dopa & a adrenergic agonists, Apo morphine (dopamine
receptor agonist), Estrogen & Androgen
129. B i.e. Dopamine [Ref: Ganong 22/e P- 423]

Dopamine acts as hypothalamic prolactin inhibiting hormoneQ and decreases prolactin secretion.

L -D O PA Q Decreases prolactin release by formation of dopamine


Bromocriptine^ (& Dopamine agonists) Inhibits prolactin releas by stimulating dopamine receptors.
ChlorpromazineQ (& Dopamine antagonist) Enhances prolactin release by blocking dopamine receptors

Factors enhancing prolactin secretion: [Mnemonic: Nu PETS SHOP]


Nu - Nursing S - StressQ S - Sexual intercourse
P - PregnancyQ S - Strenuous excerciseQ H - Hypoglycemia
E - EstrogenQ S - Breast stimuIationQ in H - HypothyroidismQ
T - TRHQ non- lactating women O - OpiatesQ
S - SomatiostatinQ S - SleepQ P - PhenothiazineQ

_________ Posterior Pituitary Hormone_________


130. B i.e. Posterior lobe

Pituicytes are glial cells of posterior pituitary which assist in storage & release of ADH & oxytocin hormone.

131. D i.e. Prolactin cause contraction of M yoepithelial cells and n o t affected by emotion

M ilk secretion (ejection or let down) a neuroendocrinal reflexQ is caused by oxytocin secreted from posterior pituitaryQ.
Oxytocin, whose secretion is affected by emotions^ & genital stimulation, cause contraction of m yoepithelial cellsQ that
lie outside the alveoli and hence 1 / 1 milk ejection.

132. D i.e. V4-CNS [Ref: Ganong 23/e 279, 665-66; Guyton 11/e 358-62, 928-29; 734,1040-41; Katzung 10/e p 614-15[
133. D i.e. Supraoptic nucleus 134. A i.e. Hyperosmolarity 135. C i.e. Increased secretion when plasma osmolality is low
136. D i.e. Free form 137. A i.e. Diabetes mellitus

- ADH is formed primarily in supraoptic nuclei^, whereas oxytocin is formed mainly in paraventricular nucleiQ of
hypothalamus. Both hormones are secreted in free form or loosely bound to neurophysin (II & I respectively but
then immediately gets detached) to circulate in blood as free hormone formsQ.
- Hyperosmolarity is the greatest stimulator for ADH secretionQ. So injection o f hyperosmolar (concentrated)
electrolyte solution in artery supplying hypothalamus (supraoptic nucleus mainly) cause release of ADHQ.
- Neurosecretion of posterior pituitary hormone ADH (antidiuretic hormone or vaspression) mainly from
supraoptic nuclei o f hypothalamus is increased with increased plasma osmolality (hyperosmolality) and is
decreased with decreased plasma osmolality. Surgery, exercise, stress, pain also increaseADH secretionQ. ADH
increases the permeability o f distal tubule & collecting ducts to waterQ therefore retaining water in excess of
solute.
- Vasopressin (ADH) has 3 receptor - V ia receptor in vascular smooth musclesQ, (causing vasoconstriction), liver
(glycogenolysis) and area postrema of brain (decreasing cardiac output); Vib or V 3 receptor in anterior pituitaryQ
(increasing ACTH secretion); and V 2 receptor in principal cells of collecting ductsQ (augmenting movement of
water through water channel aquaporin- 2 ).
- Diabetes m ellitus results from complete or absolute lack of insulin d/t reduction in f) -cell mass of pacreaseQ. It
has nothing to do with pituitary.
194 ■ A Complete Review of Short Subjects

Hypothalamus mediated response Part o f hypothalamus responsible


Response to heatQ Anterior hypothalamus “1 Preoptic
Response to coldQ Posterior hypothalamus (Shivering center) -1 area
Circadian rhythmQ Suprachiasmatic nucleus^
GnRH is secretedQ Arcuate nucleus
Oxytocin released^ Paraventricular nucleus
ADH released^ mainly Supraoptic nucleus^
Satiety centreQ Ventromedial hypothalamus
Food intake is governed^ Ventromedial hypothalamus
Feeding (hunger) and thirst centreQ Lateral hypothalamus

M agnocellular neurons of Supraoptic nuclei & Paraventricular nuclei of hypothalamus form precursor molecule
i.e.
- Signal peptide + AVP + Neurophysin II + Glycopeptide (copeptin)
- Signal peptide + Oxytocin + Neurophysin I
I
Signal /leader sequence removed in ER, packaged into secretory granule (Herring bodies) in GA and transported
through axon by axoplasmic flow to reach nerve endings in posterior pituitary in several days.
I
Cleaved during transport and so storage granules contain free ADH/oxytocin, corresponding neurophysin & glycol
peptide (in vasopressin neurons) which are released in circulation.

138. D i.e. Vasopressin

Thirst -ADH (Vasopressin) Mechanism __________ Aldosterone (Renin-Angiotensin System)____________


It is a weak regulator of Na+-water b/o mineralo-corticoid
It is the m ajor (most important) regulator escape i.e.
of body Na+ and water balance^, 1. I Na+ with normal ECF volume is weak stimulator of
maintaining near perfect Na+ balance in aldosterone
body (even in absence of aldosterone) 2. Aldosterone T tubular reabsorption of both Na+ & water and so
expansion of ECF volume produces natriuresis
Aldosterone is key regulator of external K+ balance^ (absence of
which may r/i fatal hyperkalemia

_______________ Thyroid Gland_______________


139. C i.e. 2-3 months 140. A i.e. Albumin; B i.e. Globuin; C i.e. Prealbumin [Ref: Ganong 23fe P- 305.; Guyton 10/e P 833]

In thyroid follicles, thyroid hormones (thyroxin, Ts) Protein Plasma Amount of


are stored for 2-3 monthsQ (i.e. in amounts sufficient Concentration Circulating
to supply normal requirements for 2-3 months). (mg/dl) Hormone bound (%)
- Thyroid hormones in blood is transported combined
t4 T3
with plasma proteins. Normally 99.9% of T 4 (&T3) in
TBGQ 2 67® 46
plasma is protein bound. Plasma protein which bind
T 4 & T 3 are: 1) Albumin ® 2) Thyroxine binding TransthyretinQ (TBPA) 15 20 1
prealbumin® (TBPA) (new called transthyretin) AlbuminQ 3500 13 53
3)Thyroxine binding globulin® (TBG): affinity to bind
T 4 is maximum.
- Transferrin® [apotrans ferrin (0 -globulin) + Fe]:
transport iron to plasma®. Ceruloplasmin® [Copper
binding protein] transport Cu to plasma.

141. D i.e. T4 has shorter half life than T3 [Ref: Ganong 22/e P-319; KDT 5th/e P-228]

Plasma TU2 o f T4 is more (6-7 days)® than T 3 (l-2days).___________


Physiology: Endocrine System ■ 195

142. D i.e. Iodine intake [Ref: Ganong 24/ep 309]

Iodine is fastest acting thyroid inhibitor^. So excessive iodine intake inhibits its own transport in thyroid cells, alter
redox potential of cells, interfering iodination and thus reducing T3/T4 synthesis (W olff-Chaikoff e ffe c t).
All aspects of thyroid hormone synthesis are affected but most important effect is thyroid constipation (i.e. inhibition
of thyroid hormone reiease)Q.

143. A i.e. Pituitary 144. B i.e. Thyroid; D i.e. Salivary gland; E i.e. Mammary Gland. [Ref:Ganong 24/e P- 341 ]

Iodine uptake i.e. sodium-iodine symporter (Na-I symporter) is seen in thyroid gland, salivary gland, mammary
glandQ, gastric mucosa, placenta, ciliary body of eye & choroids plexus.

145. A i.e. 1 1 Lipolysis, C lB M R [Ref: Ganong 22/e P-324; Chaudhuri 5/e P- 262-263]

Thyroxine increases BM R, Lipolysis, O 2 consumption and myocardial contractilityQ.

146. C i.e. Growth hormone [Ref: Harrison 16/e P-2090, 20961

About 60% of patients with GH - secreting tumours may exhibit paradoxical GH response to TRH stim ulation.
TRH induced GH stimulation may occur in acrom egaly 0, renal failure & depression.

Kidney & Adrenal Gland


147. B i.e. Angiotensin I [Ref: Berne & Levy 6/e p 699, 655; Ganong 23/e 670; Guyton 11/e p 907; Gabriel p-145, 227]

Kidney produces rennin (& so Angiotensinogen Basal- Keratinocyte


angiotensin I), erythropoietin and 1,25- (Liver) 7- Dehydrocholesterol
dihydroxy vitamin D3
(Cholecalciferol)Q, Mn " RED 3". Renin | UV-B
Erythropoietin (peptide) increases (Kidney)
Vitamin D 3
erythrocyte production; rennin (peptide) Angiotensin I
converts angiotensinogen to angiotensin
I and 1,25 (OH )2 vitamin D (steroid) Hepatocyte
ACE (Lung-
causes bone mineralization by increasing endothelium)
intestinal Ca++ absorption. 25-Hydroxylase
- Liver produces 25-hydroxy vitamin D Angiotensin II 25-Hydroxy Vitam in D
(calcifediol), vitamin D binding protein (Lung- endothelium )
(DBP), angiotensinogenQ, insulin like
growth factor type I (IGF-1) &
testosterone. D B F (liv e r ) —^

- ACE (angiotensin converting enzyme)


and so angiotensin II is produced in
2 5 - ( O H ) Vit D - D B P
endothelium of mainly lungs (but
conversion also occurs in many other
parts). V

Kidney (proximal tubule cells)


- Heart produces atrial natriuretic peptide (ANP); adipose tissue
25 - (OH) Vit D
produce leptin & adiponectin; Stomach produce gastrin, somatostatin hydroxyl
24
& ghrelin; intestines produce secretion, cholecystokinin (CCK), H y d ro x y Vitam in D
l a Hydroxylase
glucagons like peptide (GLP) 1, and 2, motilin and glucose dependent la s e

insulinotropic peptide (GIP; gastric inhibitory peptide); adipose ’


tissue & mammary glands produce estradiol 17 (3-; liver & sebaceous 1, 25- Di hydroxyl ”*•1,24, 25-
gland produce testosterone and genital skin and prostate produce 5- Vitamin D Tri
D ihydrotestosterone (DHT). hydroxyl
- Granulocyte & granulocyte - macrophage colony stim ulating factors Vitam in D
(GCSF & GM - CSF respectively) are hem opoietic growth factors
(like erythropoietin). G - CSF is relatively specific and induces mainly
granulocytes. GM - CSF induces granulocytes, macrophages, and
eosinophils.
196 ■ A Complete Review of Short Subjects

148. B i.e. Adrenal medulla normally secretes epinephrine & in excess of.. 149. C i.e. Norepinephrine
150. A i.e. Aldosterone [Ref: Harrison 16/e P-2148; Ganong 22/e P-356] 151. C i.e. 17-ketosteroid dehydroepiandrosterone

Hormones of Adrenal Gland

Adrenal Cortex Adrenal M edulla


Zona glomerulosa Zona faciculate Zona reticularis - Norepinephrine®
- Epinephrine
- MineralocorticoidsQ - Glucocorticoids^ Androgens eg.
- AldosteroneQ - CortisolQ - Dopamine
- 17 Ketosteroid-
- Deoxycorticosterone - Corticosterone DHEA (Dehydro ★ Adrenals may also secrete small amount of
Epi Androsterone)Q estrogen, although most of the estrogen that are
is major adrenal not formed in the ovaries are produced in the
androgen cirulation from adrenal androstenedione.
- Androstenedione.

★ M ajor testicular androgen is testosterone®

152. A i.e. Aldosterone

Adrenal Gland can be divided into 3 cortical zones & medulla

Adrenal Cortex (Mn GFR from out to in) Adrenal Medulla


Zona Glomerulosa AldosteroneQ
Zona Fasiculata Corticosteroids mainly Epinephrine & NorepienphrineQ
Zona Reticularis Androgen (sex steroids) mainly

★ In zone glomerulosa inspite of mitochondrial Ilf)hydroxylase (P 450 C 11 or (YP 11 B l), a closely related mitochondrial
enyme aldosterone synthase (P 450 C 11 AS or CYP 11 B2), is found (gene of both enzymes found and chromosome 8). Zona
glomerulosa also lacks mitochondrial 17ahydroxylase (P450 C17 or cYP-17) that is why it makes aldosterone but fails to
make cortisol or sex hormoneQ.
★ Zona fasciculate has more 3/J- hydroxysteroid dehydrogenase (smooth endoplasmic reticulum enzyme activity) than zona
reticularis and zona reticularis has more of cofactors required for expression of 17, 20 lyase activity of 17a hydroxylase.
Therefore zona fasciculata makes more cortisol & corticosterone, & zone reticularis makes more androgenQ.

153. D i.e. Liver [Ref: Gabriel p. 305-6; Guyton llth /e p. 947-50; Ganong 23rd fe p. 342-48, 355-59; Stuart: molecular & cellular pediatric
endocrinology p-219; wikipedia] a
‘ ““ i.
• M ineralocorticoid (aldosterone) receptors are not found in liver®. Much of Aldosterone is metabolized in liver to
tetrahydroglucuronide derivative (inactive) and some is changed in liver & kidney to 18- glucuronide also k/a acid labile
conjugate' because unlike other breakdown products of steroids, it is converted back to free aldosterone at pH 1 by
hydrolysis. 40% tetrahydroglucuronide, 5% acid labile conjugate & < 1% free aldosterone appear in urine.
• M ineral corticoid receptors are present in collecting tubules (principal cells), distal tubules & collecting ducts o f
kidney, sw eat glands, salivary glands, and intestinal epithelium especially in colon ® causing conservation
(reabsorption) of Na+ & water and secretion (excretion) of K+ & HCOi . These receptors are also found in brain
(hippocampus), myocardium, muscle, vessels (peripheral vasculature) and brown adipose tissue®.
Physiology: Endocrine System « 197

154. A , C, & D i.e. T Neoglucogenesis, T Proteolysis, & T Protein anabolism in liver [Ref: G abriel p. 302-4; C hatterjea & S hin dc
7<i'/e p. 556-57; H arp er 2 8 ‘>'le p. 1 1 6 ,1 7 1 , 221-22, 430-31, 442- 43; K a tzu n g 10/e p64U; K D T 6»’/ed pg-277]

Glucocorticoid is anti-insulin hormone that is catabolic to peripheral tissues and anabolic to liver. So in liver it increases
glycogen and protein synthesis and gluconeogenesisQ whereas, in peripheral tissue it causes hyper glycemia (d/t
decreased glucose uptake and decreased glycolysis)Q, increased free fatty acids & aminoacids in plasma (d/t increased
lipolysis and proteolysis respectively)^

155. C i.e. Increased excretion from kidney IR ef: MT.x: G lu cocorticoids]

Glucocorticoids (steroid) facilitate or increase free water clearance (i.e. rapid excretion of water load) and uric acid
excretion. It also inhibits intestinal absorption and enhance renal excretion of Ca++Q (esp. on large dose). So prolonged
use of steroids cause osteoporosis^ (decreased bone matrix)Q.

156. A i.e. Inhibiton of phospholipase A2

Anti-inflammatory action of Steroids is d/t inhibition of phospholipase A-2Q

157. A i.e. Cortisol [R ef: G an on g 2 4 /e p 3 6 6 -3 6 9 ; W illiam s 7/e p 869]

|pium al variation in eosinophilic count is reciprocally (invrsely) related to circadian levels of cortisolQ.

158. A i.e. tA ldosterone; C i.e. T Epinephrine; E i.e. Edema [R ef: G an on g 2 3 r,i/e p. 3 49-51; H arrison 17/e p. 2 2 5 2 -5 5 ; 356, 359;
A C T H stim u lation o f ad ren al ep in ep h rin e & n orep in ep h rin e vol 23, N ov 2008; T ex tbook o f P hysiology G abriel C E zeilo p. 307-8]

In cushing syndrome there may be increase in ACTH (in ACTH dependent type), aldosterone (for 1-2 days),
deoxycorticosterone, angiotensin secretion and even epinephrine & norepinephrine leading to hypertension and
edemaQ.
- The salt and w ater retention plus facial obesity cause the characteristic plethoric, rounded moon faced appearance and
there may be significant K + depletion and weakness.
- O steoporosis, precipitation of diabetes, prom inent reddish purple striae on thin skin, poor wound healing, thin
extrem ities, pedulous abdom en, buffalo hum p, moon face and red cheeks are other features of cushing syndrom e.
- 3 prim ary regulatory factors involve in aldosterone secretion are ACTH, renin (via angiotensin 11) an d rise in p la sm a K+
con cen tration Q (G an on g - 356).
159. E i.e. Hypernatremia [R ef: G an on g 23/e P -357-58; C hau dhu ri 5 /e P -281; G u yton 11/e P -948-952]

Increase in plasm a N a* inhibits aldosterone secretion & acute decline in plasm a Na* stim ulates its secretionfi

160. D i.e. M etabolic acidosis


161. B i.e. Very High Na+ in plasma [Ref: Guyton T lth/e p. 948; Ganong 23rd/e p. 356-59; Gabriel p-305-07]

- Hyperaldosteronism (excess mineralocorticosteroid) leads to potassium (K*) depletion / hypokalem ia and sodium
(Na*) retention, usually w ithout edema but w ith w eakness tetany, polyuria, hypertension and hypokalem ic mild
m etabolic a lk a lo sis °.
- Hyperaldosteronism does not cause m etabolic acidoss °. And plasma Na* level is elevated only slightly if a t a llQ
because water is retained with osmolically active Na+ ions.

162. D i.e. 11 p hydroxysteroid dehydrogenase Ref: G an on g 2 3rd/e p. 355

Apparent mineralocorticoid excess is d/t inhibition or absence o f l l f i hydroxysteroid dehydrogenase type 2 °

163. C i.e. Dexamethasone.


Glucocorticoid (GC) & Mineralacorticoid (MC) Activity of Steroids

Adrenal Steroids GC MC Synthetic Steroids GC MC


Cortisol 1 1 Cortisone 0.7 0.8
Corticosterone 0.3 15 Prednisolone 4 0.8
Methyl Prednisone 5 -
Aldosterone 0.3 3000
Dexamethasone 25 -30 -
Deoxycorticosterone 0.2 100
Dehydroepiandrosterone - - 9 a- Fluorocortisol 10 125
198 * A Complete Review of Short Subjects

164. A i.e. Glycogenolysis [Ref: Ganong 22/e P- 360; KDT 5thje P-110]

M etabolic effects of Epinephrine /Norepinephrine

Liver Muscle Adipose tissue Stim ulation of metabolic rate


1 I 1 M etabolic effect results from
Glycogenolysis® (a, (32) Glycogenolysis^ Lipolysis Tglucagons & J-Insulin secretion
i i 1
Hyperglycemia® Hyperlactacidemia . TBlood FFA
• Calorigenesis

Reproductive Hormones

165. D i.e. Placenta [Ref: Ganong 23/e p. 4251

P-HCG (human chorionic gonadotropin) is secreted by syncytiotrophoblast of placentaQ.

166. A i.e. Pubertal breast enlargement in girls

- In girls, at puberty, thelarche (breast development/enlargement) is the first eventQ, followed by pubarche
(development of pubic & axillary hair) and then by menarche (first menstrual period). Estrogen is growth hormone of
breast responsible for thelarche.
- Adrenarche ie increased secretion of adrenal androgens also occurs in hum ans at puberty.

167. C i.e. Glucose [Ref: Williams 23/e p. 88; Gray's 40le p. 181; Dutta 7jc p. 391

GlucoseQ is the sugar found in amniotic fluid. ☆


168. D i.e. Progesterone; & Corticosterioid 169. D i.e. Increased triglyceride & cholesterol level in serum
[Ref: Harper's 27/e P- 446-50] [Ref: Lippincott's 3/e P- 376, Chatterjea 6/e P- 451, Dutta 5/e 5851

C- 21 - Corticosteroids 17-Hydroxy Increased triglyceride and cholesterol levels are best



steroids (gluco & steroids i.e. attributed to the metabolic effects of steroidal
mineralocorticoids)® OH gp at 17 contraceptivesQ rather than as consequence of decreased
- Progesterone® vitamin B 6 level. Effects of OCP on lipid metabolism
C - 19 - Testosterone 17-keto - Plasma lipid and lipoproteins are increased
steroids - Androgen steroids i.e. - Total cholesterol & triglyceride are increased.
Keto gp at 17 - Use of OCP is also associated with deficiency of
C - 18 Estrogen pyridoxine(Vit. B 6 ).
steroid:

D eficiency of Pyridoxine Causes


1
Increased xanthiuric acid excretion in urine:® Decreased Hb level / Anemia® N eurological Symptoms®
Kynurenine Succinyl COA + Glycine - Peripheral neuropathy
T Xanthurenic - Seizures in infants
acid Pyridoxal-P
- Abnormal ECG.
(excreted in 3- OH- Kynurenine
urine) a - amino - (3 Ketoadipate
B6 deficiency I
'+
J
ALA
★ In B6 deficiency heme
synthesis suffers.
3-OH- anthranillic acid
j
Niacin
J
H eme
Physiology: Endocrine System ■ 199

170. C i.e. Gamma amino butyric acid. [Ref: Ganong 22/e P-4191

Gn RH neurosecretion has been shown to be under the control of many neurotransmitter & neuropeptides:
- GABA - exerts an inhibitory action on GnRH secretion before onset of pubertyQ.
- Glutamate - has an excitatory effect^ & whose levels increase at the onset o f pubertyQ.
Gama amino butyric acid (GABA)Q appears to be an inhibitory neurotransmitter responsible for restricting LHRH release
before onset of puberty in female rhesus monkey.

171. A i.e. Estrous cycle [Ref: Levy & Berne 6 /ep 116-77; Novak's 14/ep 172; William's obstetrics 23/ep 36; Ganong 23/e p. 412-171

Feature Menstrual cycle Estrous Cycle


- Humans undergo menstrual cycle, ovarian cycle,
Occurs in Menstruating Non menstruating
uterine (endometrial) cycle, uterine cervx cycle, vaginal
mammals in eluding mammals other
cycle, anovulatory cycles and cyclic changes in breast,
humans & higher than primates
but not an estrous cycleQ.
prmates
- M ammals other than primates donot menstruate and
Fate of Shed as menstruation Reabsorbed
their sexual cycle is called an estrous (heat) cycleQ. It is
endometrium,
named for the conspicuous period of heat (estrus) at the
if conception
time of ovulation, normally the only time during which
does not occur
sexual interest of female is aroused.
Females are At any time of cycle Only around the
sexually unrelated to time of time of ovulation
aroused & ovulation (i.e. estrous phase
receptive or heat of cycle)

172. C i.e. LH 173. A i.e. 14 days prior to mensturation 174. A i.e. High estrogen, low progesterone
175. B i.e. LH [Ref: Ganong 22/e P-434-445;Chaudhuri 5/e P-318;Shaws 12/e P-37]

A surge in LH secretion triggers ovulationQ, and Action of LH are


ovulation normally occurs about 9 hours after the - Stimulates ovulationQ.
peak of LH surge at midcycle (i.e. 14 days prior to Formation & maintenance of corpus luteumQ
start of next menstruation)Q. L H surge causes Formation of progesterone in the corpus leutieum from
ovulationQ. For ovulation to occur the hormone the lutein cellsQ.
picture should be high estrogen & low progesteronQ.

176. A i.e. FSH 177. B i.e. Granulosa Cell [Ref: Ganong Physiology 22/e P - 446, 432; Chaudhuri Physiology 5/e P - 3181
Location of

I
FSH Receptors LH Receptors Sertoli cells have receptors for FSH &
testosteroneQ
- Granulosa cells in - On both theca interna and granulose
fem ales Q cellsQ of preovulatory follicles in _L
1
- Sertoli cells in m ales Q females FSH stimulates Testosterone
- Leyding cells in malesQ the first half of causes last half of
H y p o th a la m u s
GnRH spermatogenesis Spermatogenesis
★ FSH is responsible for early growth of
ovarian follicle in female; where as LH is
responsible for final maturation of ovarian
follicle & estrogen secretion from them. It
is also responsible for ovulation, initial
formation of corpus luteum and secretion
of progesterone.

— Broken line = inhibitory pathw ay; solid line = stimulatory


pathway
200 A Complete Review of Short Subjects

178. B i.e. FSH [Ref: Harrison 17/e p 2331; Berne & Levy 6/e p 776-77; William Gynaecology (2008) 1/e p 434; Daftari Reproductive
endocrinology p-250]

Ovarian reserve can be best assessed by measuring FSH levels on 3rd day o f menstrual cycle, clomiphene citrate challenge
/provocative test (measures FSH levels on cycle day 10 after antiestrogen clomiphene citrate 100 mg administration from
day 5 to 9). Serum inhibin B level and ultrasonic scanning to count the number of antral follicles in ovary are other
methods.

179. B i.e. Progesterone 180. D i.e. Progesterone [Ref: Shaw's 12/e P-167]

[ Whether the cycle is ovulatory or anovulatory can be studied by plasm a level o f progesterone and LH^T

Plasma Progesterone Plasma LH


Progesterone is secreted by Corpus luteum ® (which - LH is secreted by anterior pituitary gland
is formed after ovulation), placenta and in small - LH surge causes ovulation and formation of corpus
amount by follicle. luteum.
Its concentration rises after ovulation & peaks at - Its peak (LH surge) is reached about 24-36 hrs before
mid luteal phase (around day 2 0 ) and then declines ovulation®.
as luteum degenerates^. - After ovulation its level start falling because of negative
feedback of progesterone.

181. A i.e. Maternal ovary

Estrogen is primarily secreted by granulosa cells of ovarian (griffin) follicles (before ovulation), corpus luteum (after
fertilization in early pregnancy)^ and by placenta (in late pregnancy (after 8 weeks).

Hormonal Picture at the time of Ovulation /LH Surge

For ovulation to occur Estrogen should be high & Progesteron should be low®.

Estrogen Progesteron
At the time of ovulation, ova in the Graffian follicle needs Progesteron is secreted by the corpus luteum, which
to be well developed, which is accom plished by ' high is the rem nant of graffian follicles after the ova has
estrogen® been released. Thus progesterone would rise only
LH secretion is held in check by negative feedback effect after ovulation®.
of rising plasma estrogen. Large doses of progesterone inhibit LH secretion &
At 36-48 hours before ovulation estrogen feed back effect potentiate the inhibitory effect of estrogen,
becomes positive & initiates a burst o f LH secretion (LH preventing ovulation
surge)®

LH Surge After Ovulation Fate of Corpus Luteum


Just prior to ovulation there is an increase in estrogen and to some r
degree progesterone level. Fertilization occurs No fertilization
LH secretion is held in check by the negative feedback effect of
- Corpus luteum stays CL begins to
rising plasm a estrogen.
(maintained by LH & degenerate d/t
But at 36 - 48 hours before ovulation the estrogen feedback effect
placental HCG)Q & decrease in LH
becomes positive & this initiates the burst of LH secretion (LH Surge),
produce oestrogen & levels and
FSH secretion also peaks.
progesterone increased inhibin
This LH surge induces ovulation®.
hormones. secreted by luteal
Other indicators of Ovulation calls about 4 days
- By 8lh week placenta
before the next
grows sufficiienty &
Increase in basal body temperature® at the time of ovulation m enses (24"' day of
produce sufficient
Presence of secretory pattern® on biopsy of endometrium. hormones.
cycle) & eventually
Presence of thick cervical mucus® that forms a fern pattern® replaced by scar
- From now CL regress.
tissue forming
corpus albicans
Physiology Endocrine System ■ 201

182. C i.e. Pregnanediol IRef: Ganong 24/e p. 409; KDT 6/e p. 3081

Progesterone (21C steroid) is formed from cholesterol ( - » pregnenolone) has short half life and is converted in liver to
pregnanediol (most common end product) which is conjugated to glucronic acid and excreted in urine as sodium
pregnanediol-20 -glucuronide.

183. D i.e. Progesterone [Ref: Ganong 23/e P-410-14] 184. A i.e. Progesterone

- Progesterone is thermogenic & responsible for the rise in basal body temperatureQ at the time of ovulation

Increase in basal body tem perature at the time ovulation in reliable indicator of time of ovulation. The rise starts 1-2
days after ovulation. O ther indicators of ovulation - Secretory patternQ on biopsy of endom etrium , Thick cervical mucus

Preovulatory proliferative changes in endometrium are d/t estrogen


Postovulatory secretary changes in endometrium are d/t-progesterone (mainly)

185. C i.e. Long arm of X-Chromosome [Ref: Harrison 16/e P-2128]

The androgen receptor is a typical member of the steroid/ thyroid family and is encoded by a gene on the long arm o f
X -chrom osom e 6.

186. C i.e. Aromatase 187. B i.e. Glycolysis increases


[Ref: Gabriel C Ezeilo p. 366-73; Ganong 23rdje p. 348, 371, 392, 427, 416; Diabetes & Pregnancy 2/e p. 459 ]

Aromatase is the enzyme that catalyzes the conversion of testosterone to estradiol0 and androstenedione to estrone.
Estrogen increases glycolysis Q.

190. A i.e. Spermiogenesis 191. D i.e. Inhibin 192. C i.e. FSH 193. B i.e. Decreased FSH
Ganong 23rd/e p. 402-3; Grays 40lh/e p. 1265-67,1317-20; Smith urology 17thed pg-686-87; Basic Histology - test & Atlas 11/ed pg-
427; Gabriel p. 354-56; Chaudhari 6/ed pg-324; Endocrine physiology 11/ed pg-478
- Sertoli cells have receptors for FSH & Hypothalamus
testosterone^; secrete inhibin, M ullerian
inhibiting (regression) factor, & androgen binding
proteinQ. They are involved in orchestration and Gonadotropin Releasing Hormone (GnRH)
coordination of all key events in spermatogenesis
& spermiogenesisQ. 1_____________________
- Sperm atids donot divide but gradually mature into
Anterior pituitary
spermatozoa by a series of nuclear & cytoplasm ic
changes k/a spermiogenesis. All of these
LH (tropic for) FSH (tf)
maturational changes take place while spermatids
remain closely associated with sertoli cellsQ.
Sperm iogenesis involves - golgi, cap, acrosom al and Lyedig cells Sertoli cell
final phase. In the final phase of m aturation, excess
secrete Testosterone secrete Inhibin
cytoplasm is detached as a residual body that is
phagocytized & degraded by sertoli cellsQ
202 ■ A Complete Review of Short Subjects

194. A i.e. Sartioli & Sertoli cells [Ref: MTx]

Occluding tight junctions between sertoli cells forms the BTB (Blood Testis Barrier)Q.

195. A i.e. Leydig cells 196. D i.e. It inhibits secretion of FSH [Ref Ganong 22/e P-432; Chaudhuri 5je P- 311]
V-
- 'Between the tubules in the testes are nests of cells containing lipid granules, the 'Interstitial cells of Leydig' which
secretes testesterone into the blood stream'
- Inhibin is a polypeptide hormone and inhibits FSH secretionQ of anterior pituitary by direct action.___________________

Testes Inhibin - Secreted by sertoli cells in


males & granulose cells in
females^.
- Inhibits FSH secretionQ
Activin - Secreted by sertoli cells in males
and granulose cells in females
- Stimulates FSH secretion
- Androgen binding ★ Mature spermatozoa
Relaxin - Secreted in females by corpus
Mature into proteinQ are released from
luteum in ovary and from
primary - InhibinQ sertoli cellsQ
placentaQ.
spermatocytes - MISQ (contain aromatase ★ Mature spermatozoa
- Relaxes the symphysis &
cypia) acquire motility
sacroiliac joints during
Contribute to blood testes during passage in
pregnancy and helps in
barrier the epidydimisQ.
cervical ripeningQ.
(The spermatids mature into
- Structurally similar to insulin
spermatozoa in sertoli cells)
& IGFQ.

197. A i.e. Gonadotropin [Ref: Ganong 23/ep 632]

198. C i.e. Increased inhibin levels [Ref: Sex differences in FSH regulatory peptides in pubertal age boys & girls; Journal of human
reproduction vol. 19, No. 7, 1668- 1676,1107, July 2004; Guyton 11/e p.1006-7]

D elayed onset o f puberty in boys in due to increased production o f inhibin®, which results in greater negative feedback
on FSH as compared to girls.

199. C i.e. Vas deferens 200. C i.e. Epidydmis 201. D i.e. Uterus
[Ref-.Langman 10/e p.35; Ganong 23rd/e p. 404; Guyton llth /e p. 10001

Spermatogenesis & Meiosis® occurs in Seminiferous tubules® Semen release (i.e. spermatozoa
Sperms are stored, matured and gain m otility in Epididymis®. ejaculation) involves contraction of
Sperm ejaculation is d/t contraction of Vas deferensQ smooth muscles of vas deferensQ
Capacitation of sperm occur in Female genital tract®. (mediated by ligand gated cation channels
responding to ATP=P 2 X receptors)

202. D i.e. Prolactin & Gonadotropin Releasing Hormone (GnRH) [Ref: Guyton llth /e p. 999; Ganong 23rd/e p. 404]

Testosterone (androgen), estrogen, FSH, LH and growth hormones play essential roles in spermatogenesis^.

203. B i.e. Extermal genitalia


Physiology: Endocrine System a 203

Male Sex Hormone (Androgens)


I
Potent agent produced by Leydig cells of testis=Testosterone W eek agents produced in small amounts by
(responsible for growth of internal genitalia i.e. epididymis, vas adrenal cortex
deferens, seminal vesicle, spermatogenesis, erythropoeisis & LH - Dehydroepiandrosterone
inhibition - Androstenedione
I '
Converted by 5 a reductase to most potent D i hydro-testosterone (responsible for external genital growth i.e. penis,
scrotum, hair loss, pubertal sexual maturity & male behavior, and prostate growth in elderly).
I
Bone skeletal growth, muscle mass & epiphyseal fusion is caused by both. Androsterone is testosterone metabolite with
Vio activity.
204. B i.e. Inhibin 205. A i.e. Azoospermia [Ref: Berne & Levy 6/e p 766-67 ]
- Testosterone & inhibin are involved in negative feedback in spermatogenesis^. Because testosterone inhibits LH
secretion and inhibin inhibits FSH secretion and both are essential for spermatogenesis.
- Prolonged administration of testosterone in males causes azoospermia®l or oligospermia^.
206. B i.e. Estrogen [Ref: Nelson 16/e P-1688; Ganong 22/e P-406]

Skeletal m aturation, epiphyseal fusion & cessation o f growth® is caused by estrogen.


Growth spurt at puberty occurs because of androgen & estrogens, but estrogen ultimately terminate grow th by causing
the epiphyses to fuse to the long bones (epiphysial closure). Once the epiphyses have closed, linear growth ceases.
Patient with sexual precocity are apt to be dwarfed. On the other hand men who were castrated before puberty tend to be
tall because their estrogen production is decreased & their epiphysis remain open.
These observation suggest that estrogen, rather than androgen, are responsible for bone m aturation that ultim ately leads
to epiphyseal fusion and cessation o f growth®.
207. A i.e. Testosterone [Ref: Ganong 24/ep 424]

• Gonadal Steroid Binding G lobulin (GBG) or sex hormone binding globulin is a glycoprotein which binds to sex
hormones (in decreasing order affinity).____________________________________________________________________________
Dihydro testosterone (DHT) > Testosterone (65% b o u n d )» > Estradiol/Estrone (38%) » Androstenedione ( 8 % ) »
Progesterone & Cortical (0%)
• Corticosteroid Binding G lobulin (CBG) has affinity in decreasing order as following Cortisol (90%) » > >
Progesterone (18%) » > DHT, T and Androstenedione (0%).
• Albumin binding in decreasing order is; Androstenedione (90%)> Progesterone (80%) > Estradiol (60%) > Testosterone
(33%) > Cortical (6 %).
208. C i.e. Persistent late deceleration [Ref: Dutta 6/e P501 ]

In contraction stress (oxytocin challenge) test, positive result means persistent late detection ofFHR®.

209. A i.e. Testosterone 210. C i.e. Vasectomy [Ref: Ganong 22/e P- 428]

50% of men who have been vasectom ized develop antibodies against spermatozoa®. This holds significance, as because 1
of these antibodies reversal of vasectomy may not restore fertility, even when the reversal is successful.

211. A i.e. 21- hydroxylase deficiency [Ref: Harrison 16/e P-2128, 2145; Ganong 22/e P-365]

• Classic 21 - hydroxylase deficiency is most common cause of ambiguous genitalia in new bom (virilization) and
primary amenorrheaQ. i.e. Pseudohermaphroditism of fem ale child
• Primary amenorrhea with VirilizationQ in females is in: i) 21 - hydroxylase deficiency ii) 11 - Hydroxylase deficiency

212. A i.e. 1-3 mm/min 213. B i.e. Seminal vesicle 214. E i.e. Inositol 215. B i.e. Prostate [Ref: Ganong 24/e p. 422]

Zinc is secreted by prostate; and prostaglandins from sem inal vesicleQ. Sem en (seminal fluid) contains fructose,
prostaglandins, spermine and citric acid but no inositolQ.

Semen
Semen contains sperm and secretions of the seminal vesicles, prostate, cowper's gland and probably, the urethral glands. An
average volume ejaculation is 2.5- 3.5mL. Each mL of semen normally contains about 100 million® sperms. 50% of men
with counts of 20-40 million /mL and essentially all those with counts < 20 million jmL are sterile. Morphologically
204 ■ A Complete Review of Short Subjects

abnormal & or im m otile sperm atozoa also correlates with infertility. Human sperm m ove at a speed of -3 m m / m inQ
through the female genital tract. Sperm s reach uterine tubes 30- 6(j minutes after copulation. Com position of human semen ■

Colour : white, Seminal Vesicle From prostate Buffers


opalescent Secretions (contributes (20% of volume) - Phosphate
Specific gravity : 1.028 60% of total volume) 1 - Bicarbonate
PH : 7.35- 7.50Q I Sperm ine0
Sp erm -co u n t: Citric a c id 0
Fructose 0 (1.5- 6.5mg/mL)
~100m illion/m L w ith Cholesterol
Flavins
few er than 20% Phospholipids
Ergothionene
abn orm al f o m f i
Ascorbic acid Fibrinolysin
Contain hyaluronidase
Phosphoryl choline Fibrinogenase
Prostaglandins<2 ZincQ
Acid phosphatase

216. A i.e. HCG > D i.e. Progesterone [Ref: Multiple papers]

Immune rejection of fetus is prevented by hCG » Progesterone by inhibiting maternal immune responses to fetal
antigens^.

217. C i.e. Progesterone production require feta steroidogenic tissue


218. B, C, D i.e. Insulin, Thyroxine, Glucocorticoids, [Ref: Berne & Levy 6/e p 787-91; Ghai 7/e p l ]

Ironically, fetal growth hormone does not appear to regulate growth^, and anencephalic infants and GH-deficient
children typically have normal birth weight.
Fetal thyroxine, insulin and glucocorticoids^ play important role in intrauterine growth of fetus.

Stress
219. A i.e. Corticol > B i.e. Glucagon 220. B i.e. Glucagon; D i.e. Epinephrine; E i.e. Cortisol
[Ref: Ganong 22/e P-370; 374,145, 39, 349; Morgan 3/e P- 320]

Stress induced Hyperglycemia is mediated through increased epinephrine, cortisol and glucagon^ causing
gluconeogenesis

221. A i.e. Neoglucogenesis E i.e. Lipolysis C i.e. Protein breakdown [Ref: G a n o n g 22/e P-352, 370; KDT 5/e P- 256]
222. B i.e. Vasopressin; C i.e. Adrenaline {Ref: G a n o n g 22/e P - 245]
Vasopressin Secretion
I 1 I
Increased Decreased
- T Effective osm otic pressure of plasma - i Effective osm otic pressure of plasma
- i ECF volume - T ECF volume
- Pain , emotion, s t r e s s Q, exercise, standing - Alcohol
- Angiotensin II, clofibrate, carbam azapine

Nitric Oxide
223. A i.e. cGMP 224. D i.e. Arginine
225. D i.e. Requires NADPH, FAD, FMN & Heme iron 226. A i.e. NO is synthesized from arginine
227. A i.e. Free radical; B i.e. Vasodilator; C i.e. Oxidizing agent; D i.e. Catalyst
228. C i.e. Gastrointestinal smooth muscle relaxation [Ref: Berne & Levy 6/e P 100, 281; Gabriel p 145-46, 163, 248, 280; Harper
28/e 559-60, 55, 591; Guyton 11 /e 564,199-200, 332; Ganong 23/e p. 563-64,112; Lippincott 5/e p 150-51]

- Nitric oxide (NO) is synthesized from arginine (O 2, NADPH) by NO synthase (requiring FMN, FAD, heme & BH 4) in
nerves and vascular endothelial cells (hence called EDRF). It relaxes smooth muscles by increasing cyclic GMPQ.
- Nitric oxide (NO) and atrial natriuretic factor (ANF) are hormones that act through c-GMPQ.
- NO (along with VIP and PACAP) are inhibitory neurotransmitter in enteric (gastrointestinal) neurons that primarily
mediate gastrointestinal smooth muscle relaxation via c-GM P pathwayQ.
Chapter :8 GENERAL PHYSIOLOGY

QUESTIONS

BMR 9. Aerobic capacity is increased by: (AIIMS 04)


A. Prolonged exercise routine □
1. Basal M etabolic Rate depends most closely on: B. Strenuous exercise □
A. Lean body mass (NBE P 15,14; AI 10) □ C. Regular 3 minutes exercise □
B. Body mass index □ D. Spurt of exercise □
C. Obesity □ 10. Exercise causes which of the following?
D. Body surface area □ A. Increased blood flow to the muscles after half
2. Basal metabolic rate closely associated with? minute (AIIMS 07) □
A. Lean body mass (NBE P 14,13; AI 11) □ B. Increase in cerebral blood flow due to increase in
B. Body surface area □ systolic blood pressure □
C. Body mass index □ C. Increase in body temperature □
D. Body height □ D. Decreased O 2 consumption □
3. Decreased Basal M etabolic Rate is seen in: 11. Which of the following is TRUE regarding
A. Obesity (AI 10) □ physiological changes in the brain during moderate
B. Hyperthyroidism □ exercise?
C. Feeding □ A. Blood flow is decreased (AI 08) □
D. Exercise □ B. Blood flow is increased □
4. BM R depends on: (Al 05) C. Blood flow remains unaltered □
A. Body weight □ D. Blood flow initially increases & then decreases □
B. Surface area □ 12. Autoregulation means (UP 2K)
C. Amount of adipose tissue □ A. Maintains the blood flow □
D. Amount of lean body mass □ B. Vary with change in pressure □
5. Which of the following statements is true regarding C. Regulated by local metabolites □
basal metabolic rate: (PGI 01) D. Well developed in the skin □
A. Increased in starvation □ 13. In isometric exercise all are increased except:
B. It is not influenced by hormonal changes □ A. Heart rate (AIIMS 07) □
C. It is not affected by dietary changes □ B. Cardiac output □
D. Decreased by 50% in starvation □ C. Mean arterial pressure □
E. It is not affected by energy expenditure □ D. Systemic vascular resistance □
6. Energy expenditure in Resting state depends on 14. Strengthening muscle exercise- (NBE P 13)
A. Lean Body Mass (AIIMS 11, AI 09) □ A. Isotonic □
B. Adipose tissue □ B. Isometric □
C. Resting Heart Rate □ C. Aerobic □
D. Exercise □ D. None □
15. The main cause of increased blood flow to exercising
Aging muscles is : (AIIMS 10, AI 09)
A. Raised blood pressure □
7. Which of the following does not changes in the old B. Vasodilatation due to local metabolites □
age: C. Increased sympathetic discharge to peripheral
A. GFR, FEV, (AIIMS 0 7 ) 0 vessels □
B. Glucose tolerance □ D. Increased heart rate □
C. Haematocrit □ 16. All are true regarding blood supply increase in
D. Blood pressure □ muscles during exercise, except: (AIIMS 12)
A. Decreased beta adrenergic stimulation □
Exercise B. Local metabolites □
C. Increased arterial pressure □
D. Cholinergic stimulation □
6. In moderate excercise the respiratory rate is increased
due to response of : (NBE P 15,13) 17. A/E are the features of exercise: (AIIMS 09)
A. Propioception receptor in the joints □ A. Left shift of Hb-Ch dissociation curve □
B. Increase blood supply to muscle □
B. TPCO 2 in arterial blood □
C. Increase stroke volume □
C. TPO 2 in arterial blood □
D. Increase O 2 extraction □
D. J-receptor stimulation □
206 ■ A Complete Review of Short Subjects

18. Physiological changes during severe exercise are: 26. Low haptoglobin in haemorrhage is masked by
A. Hyperventilation in the beginning □ concurrent presence of
B. Hyperkalemia (PGI 03) □ A. Malnutrition □
C. le d Pa O 2. □ B. Pregnancy □
D. le d P a C 0 2 □ C. Obstructive biliary disease □
E. le d H2 C O 3 □ D. Liver parenchymal disease □
19. Inexercising muscle; true about metabolism is: 27. Histamine is formed by of histidine.
(NBE P 13,15; AIIMS 16,14; PGI 08) A. Methylatin (AI 08) □
A. Same in aerobic and anaerobic □ B. Reduction □
B. Fatty acids used mainly □ C. Oxidation □
C. Glycogen & (phospho) creatine kinase used D. Carboxylation □
anerobically □ 28. Which of the follow ing statements about Histamine is
D. All of the above □ true: (AI 08)
20. Exercise is also prescribed as an adjuvant treatment of A. Is found in Mast cells □
depression. Most probably it acts by: B. Increases gastric acid secretion □
A. Increasing pulse pressure (AIIMS 05) □ C. Related to arousal and blood pressure □
B. Improving hemodynamics □ D. All of the above □
C. Raising endorphin levels □ 29. 'F lare'in Triple response is mediated by : (AI02)
D. Inducing good sleep □ A. Axon reflex □
B. Arteriolar dilation □
Homeostasis C. Histamine release □
D. Local hormones □
21. Living body control system correct (PGI 09) 30. Lewis Triple Response is mediated by (AI 08)
deformity/disturbance: A. Histamine □
A. Immediately B. Axon reflex □
B. Completely C. Injury to endothelium □
C. Incompletely D. None of the above □
D. With gain 31. In Lewis Triple Response, redness when skin is
22. Which of the following is not mediated through scratched with a pointed object is seen due to
negative feed back mechanism: A. Axon Reflex causing VasoconstrictionfAfJMS 12) □
A. BP (NBE P 15,13; AIIMS 15; AI ODD B. Histamine Release due to local injury to mast
B. GH formation □ cells □
C. Thrombus formation □ C. Free Nerve Endings □
D. ACTH release □ D. Endothelial Damage leading to increase in
23. About the homeostatic mechanism of the body permeability □
following are true excep t: (AIIMS 02) 32. Which of the following statement about Bradykinin is
A. Values revolve around the mean □ true: (NBE P 15,14,13; AI 08)
B. Value of controlled variable is compared to the A. Causes pain □
reference value □ B. Causes bronchodilation □
C. Value of controlled variable oscillates near aset C. Causes vasoconstriction □
point □ D. Decreases vascular permeability □
D. System is stabilized by the positive feed back 33. 0 2 consumption is minimal when temp, gradient
mechanism □ between skin and external environment is:
24. Positive feed back is seen in A/E : A. < 0.50 (PGI 2K) □
A. LH Surge (AIIMS 01) □ B. < 2.50 □
B. Entry of Ca into sarcoplasmic reticulum □ C. < 1.5° □
C. Stimulation of gastric secretion of histamine& D. > 3.50 □
gastrin. □
D. Thrombolytic activity in coagulation cascade □

Inflammatory Mediators

25. 'C ' in CRP stands for (AIIMS 09)


A. Concanavalin A □
B. Cellular □
C. C polysaccharide of pneumococcus □
D. Chondroitin sulfate in series following ARP and
BRP □
General Physiology ■ 207

ANSWERS & EXPLANATIONS:

BMR

1. A i.e. Lean body mass 2. B i.e. Body surface area 3. A i.e. Obesity
4. B i.e. Surface area 5. D i.e. J, 50% in saturation 6. A i.e. Lean Body Mass
[Ref: Ganong 23rdje p. 461-63; Exercise physiology Thomas Poland vol 2; Guyton llth jep . 886-87; Marks- essential medical
Biochemistry 2/e pg-15; Manual of nutritional therapeutics 5/ep 80; Harrison 17/ed pg- 464; Chaudhuri 5/e P- 3431 .

-----------------------------------------------------------------------------------------------------------------------------------------
Resting energy expenditure or resting metabolic rate (RMR) and basal metabolic rate (BMR) depends more closely &
more logically on metabolically active fat free- lean body mass (i.e. muscle andinternal organs) than body surface
areaQ. However, body surface area is a reasonable determinant of lean body mass (So RMR/BMR depends on LBM >
Body mass > BSA > Body weight > BMI > Height).
Obesity (i.e. excess of metabolically inactive fat /adipose tissue) and starvation decrease BMRQ.

Aging

7. C i.e. Hematocrit [Ref: Chaudhuri 5/e P- 302]

Hematocrit does not changes with age Q

Exercise
8. A i.e. Propioception receptor in the joints. [Ref: Ganong Review of Medical Physiology 22/e P- 681 ]

- In moderate excercise the abrupt increase in ventilation at the start of excerdse is due to psychic stimuli and afferent
impulses from proprioceptors in muscles, tendons and jointsQ.

- Arterial pH, Pco 2 and P02 remain constant^ during moderate excercise because increase in ventilation is proportionate
to increase in O2 consumption.

9. C i.e. Regular 3 minute excercise 10. C i.e. Increase in body temperature


11. C i.e. Blood flow remains unchanged 12. A i.e. Maintains the blood flow
13. D i.e. Systemic vascular resistance 14. B i.e. Isometric
[Ref: Ganong 22/e p. 632 - 35, 681-83; Guyton 11 je p. 1055- 66; Fox physiology 8/e p-346; http: /www. pubmedcentral.nih.gov/
pagerender. fcgi? artid = 301529 & pageindex = 2# pageJ

• Aerobic capacity is increased by regular 3 minute exerciseQ (not by spurts of prolonged & strenuous exercise). Exercise
increases blood flow to muscles at or even before the start of exercise. It increases O2 consumption & body temperature.
• Cerebral blood flow is maintained due to auto regulation (between 60-160 mm Hg) in response to moderate
exerciseQ.
• Isometric exercise characteristically show modest increase in heart rate and cardiac output and pronounced increase in
systolic and diastolic arterial pressure. The relative increases in cardiac output and mean blood pressure are such that
(calculated) systemic vascular resistance is unchangedQ.
• Isotonic (or DCER= Dynamic Constant External Resistance) exercise includes all exercises which involve joint
movement like running, rowing etc. Whereas, isometric exercise are static (i.e. no joint movement is involved) and
include yoga postures, wall press/sit. So it strengthens muscles at that angle and benefits will not carry over to joint
actions beyond that range.
• Muscle strength (endurance) is increased both by isotonic & isometric exercises. Recent studies however report, that
strengthening occurs more in isometric exercises (because there is no help of supporting muscles & momentum) with
a benefit of less chances of muscle injuryQ. Muscle bulk is mainly increased by isotonic exercise.

15. B i.e. Vasodilatation due to local metabolites [Ref: Guyton 12/SAE p. 99, 208; Ganong 24/e p. 666-668]
16. A i.e. Decreased beta adrenergic stimulation
208 ■ A Complete Review of Short Subjects

- The main cause of increased blood flow to exercising muscle is vasodilation due to local metabolites^. The most
important local chemical effect is reduction of 0 2 in muscle tissue. Increased arterial pressure above normal, P (beta)
adrenergic receptor mediated vasodilation in exercising muscle and a (alpha) adrenergic receptor mediated
vasoconstriction in resting muscles are other factors responsible for increased blood supply in muscles during exerc
- Increased (not decreased) P2 adrenergic stimulation (in humans) and cholinergic stimulation (in some animals like
dogs, cats) are responsible for skeletal muscle vasodilation whereas, a 1 adrenergic stimulation cause skeletal muscle
vasoconstriction. Epinephrine excites more of p receptors (=vasodilation), whereas a vasoconstrictor receptors are
excited especially by norepinephrine.

17. A i.e. Left shift of H b - 0 2 dissociation Curve [Ref: Ganong 22/e P- 6811
18. A i.e. Hyperventilation in the beginning; B i.e. Hyperkalemia D i P aco 2 [Ref: Ganong 22/e P-681-82, Guyton 10/e P479-481 ].

A right shift o fH b - O z dissociation curve® is seen in excercising muscle. Right shift occurs because of accumulation o f
CO2 (4 pH)Q and increased temperature o f contracting muscle®.
Physiological changes during severe exercise include hyperventilation (in the beginning), Ted PaO^ iPaC O ^ 4pH,
IH C O 3” and hyperkalemia®.

19. C i.e. Glycogen and creatine kinase used anerobically I Ref: Ganong 24/cp. 106 - 108 ; Gm/ton w /e 9 4 U

Glycogen lactic acid system and phosphoryl creatinine stores (or creatine kinase) are used during anaerobic exercise.

20. C i.e. Raising endorphin level [Ref: Kaplan & Saddok's 7/e P -106,1071

Exercise increases release of endogenous opioids and is associated with mood enhancement.
- /?endorphin is the principal endogenous opioids
- The best documented function of endogenous opiates is in analgesia & alteration of pain perception

Homeostasis

21. All [Ref: Guyton 11/e 6-8; Ganong 23/e 58-60]

Thousands of homeostatic control systems operate within living body (in all cells, organs) to maintain the constancy
of internal environment. These may correct disturbance (deformity) completely or incompletely (with error & gain)
mostly using negative feed back. It is corrected immediately (adaptive control) or after a delay.
Gain of a control system: Most control systems are of negative feed back nature (i.e. negative to initiating stimulus).
If some factor (eg any ion, BP etc) becomes excessive or deficient, negative feed back consists of a series of changes
that return the factor towards normal mean value. The degree of effectiveness of control system to maintain constant
condition is determine by the gain of negative feedback. For example same amount of blood volume is transfused
into 2 persons with and without baroreceptor pressure control system has caused an arterior pressure to increase from
normal 100 mm Hg to 125 mm Hg and 175 mm Hg respectively. This means feedback control system has caused a
correction of -50 mmHg (i.e. from 175 to 125 mmHg) but still there is an error of 25 mm Hg (as there remains an
increase of +25 mmHg pressure even in normal functioning control system; which also means that control system is
not 100% effective in preventing change in this case). The gain is calculated by

Correction _50 (i.e. a disturbance that increases or decrese the arterial pressure does so only 1 /3rd
Gain = --------------- 7 7 7 =_2 as much as would occur if this control system were not present)
Error

Gain of temperature control system is much greater (-33) therefore it is much more effective than baroreceptor
pressure control system.

Feed Back Mechanism

22. C i.e. Thrombus formation 23. D i.e. system is stabilized by the positive feedback [Ref: Guyton 10/e P 6, 7]
General Physiology ■ 209

Positive feed back system leads to vicious cycle which destabilizes the homeostatic mechanism^. So it is the negative ☆
feed back system that stabilizes the body's homeostatic mechanism. Clotting o f blood (ie thrombus formation) is
mediated through positive feed back mechanism^.

24. C i.e. Stimulation of gastric secretion by histamine & gastrin [Ref: Ganong Physiology 22/e P- P470, Guyton 10fc P6I

Most control system of the body are negative feed back system. Release of acid in stomach by the action of gastrin is
negative feed back system, as Tin acid secretion inhibits further gastrin secretion.

Inflammatory Mediators
25. C i.e. C polysaccharide of pneumococcus
[Ref. Harrison 17"'/e p. 568-569, 638; Tillet & Francis : Serological reactions in pneumonia; J Exp Med 1930, 52: 561-5851

• C-reactive protein (CRP), discovered by Tillet and Francis in 1930, is so named because it reacts with the C
polysaccharide of pneumococci.Q

Acute phase proteins (or reactants) are proteins that increase during acute inflam m atory states or secondary to certain
types of tissue damages. These include - a l - antitrypsin, a l acid glycoprotein, CRP, haptoglobin and fibrinogenQ.
Interleukin-1 (1L-1), a polypeptide released from m ononuclear phagocytic cells, is the principal - but not the sole -
stim ulator of synthesis of m ajority of acute phase reactants by hepatocytes. Cytokine IL-6 are also involved. N uclear
factor Kappa-B (NFKB) is a transcription factor that has been involved in the stim ulation of synthesis of acute phase
proteins.
CRP (a type of pentraxin protein) is synthesized by liver in response to factors released by fat cells. It binds to
phosphocholine expressed on surface o f dead or dying cells in order to activate classic com plem ent pathw ay via C lq .
Its level is mainly determ ined by rate of production (rises within 6 hours, peak in 48 hours & P/2 is constant)

26. D i.e Liver parenchymal disease [Ref: Harrison 17"'/e p. 570; Gabriel p.711
s~
Haptoglobin (Hp) binds extracorpuscular (i.e. extracellular) hemoglobin (Hb), preventing free Hb from entering the
kidney. Hp-Hb complex is removed from plasma by haptocytes. Levels of haptoglobin falls rapidly in situations where
hemoglobin is constantly being released from RBC such as occurs in hemolytic anemia (haemorrhage). But in case of
parenchymal liver disease, Hp-Hb complex removal is deranged so reduction in haptoglobin level is masked.

27. D i.e. Carboxylation


28. D i.e. All [Ref: Gannong 22/e p. 107; Guyton 11/e p. 797-98, 450, 480, 202, 529, 285-86; Chaudhary 6/e p. 318; KDT 6/e p. 151-531

Histamine is formed by decarboxylation o f amino acid histidine^, and is found in mast cell, basophils,
enterochromaffin like cells of gastric mucosa and brainQ. It has been related to arousal, sexual behaviour^, regulation of
secretion of some anterior pituitary hormones, blood pressure, drinking & pain threshold.

29. A i.e. Axon reflex 30. A i.e. Histamine 31. Histamine Release due to local injury to mast cells
[Ref: Ganong 22/e P-593, 627; Chaudhuri 6/c P-249; BOS-Skin immune system 3/e p. 401-02; Ramdass (2007) p. 141]

Flare in triple response is caused by Arteriolar dilatation which is mediated by the axon reflexQ.
Histamine is primarily responsible for Lewis triple response. Axonal reflex only mediates flare part of triple
response.

32. A i.e. Causes Pain [Ref: Guyton 11/e p. 436, 202, 529, 322, 779, 795; Robbins 7/e p. 65, 74-75, 69]

Bradykinin increases vascular permeability, and causes contraction o f smooth muscle (utrine, intestine, broncho-
constriction), dilation of blood vessels and painQ when injected into the skin. These effects are similar to those of
histamine.

33 . A < 0.5°C (still searching)


Chapter-9 CELL PHYSIOLOGY

QUESTIONS

Cell Membrane C.Symmetrical arrangement of cell wall


components □
1. Eukaryotic plasma membrane is made up of all D. Not made up of amphipathic lipids □
except: 10. In cell membrane, following are true except: (PGI 05)
A. Carbohydrates (AIIMS 10, 09) □ A. Lipids are regularly arranged □
B. Triglycerides □ B . Lipids are symmetrical □
C. Lecithin □ C. Protein displaced laterally □
D. Cholesterol □ D. None □
2. Plasma membrane is mainly composed of: 11. Which of the following membrane has the highest
A. Cholesterol (NBE P 14,13; AIIMS 04) □ protein content per gram tissue: (AIIMS 02)
B. Carbohydrate □ A. Inner mitochondrial membrane □
C. Phospholipid □
B. Outer mitochondrial membrane □
D. Protein □
C. Plasma membrane □
3. On weight basis, the membrane contains protein
D. Myelin sheath □
and lipid in the ratio of- (NBE P 13)
12. The transmembrane region of protein is likely to
A. 1:2 □
have: (AI 04)
B. 1:1 □
A. A stretch of hydrophilic amino acids □
C. 2:1 □
B. A stretch of hydrophobic amino acids □
D. 4:1 □
4. Which of the follow ing is seen in association with C. A disulphide loop □
membrane raft? (AIIMS 12) D. Alternating hydrophilic and hydrophobic amino
A. Glycosylphosphatidylinositol (GPI) anchored acids □
protein □ 13. Lipids and proteins interact in membrane by
B. Mannose binding protein (MBP) □ A. Hydrophobic interactions (PGI 07) □
C. Guanosine triphosphate associated protein □ B. Both hydrophobic and covalent interactions □
D. Spectrin associated protein □ C. Covalent bonds □
5. Function of phospholipid in cell membrance is: D. H bonds □
A. Cell to cell variation (PGI 01) □ 14. In leucine zipper model, Leucine residue seen after
B. Transduction of Signals □ every (PGI 07)
C. Transmembrane preparation of protein □ A. 3 amino acids □
D. DNA replication □ B. 6 amino acids □
E. Enzyme activation at membrane surface □ C. 9 amno acids □
6. Membrane fluidity is increased by: (AIIMS05) D. 12 amino acids □
A. Stearic acid □ E. 7 amino acid □
B. Palmitic acid □ 15. Which important component of cell wall has
C. Cholesterol □ carbohydrate moiety: (PGI 01)
D. Linoleic acid □ A. Phosphoglyceride □
7. Transition temperature of lipid bilayers of cell B. Triacylglycerol □
membrane is increased by- (AIIMS 2K) C. Sphingomyelin □
A. Cholesterol □ D. Cholesterol □
B. Saturated fatty acids □ E. GM 2 Gangliosides □
C. Hydrocarbons □ 16. Glycophorin is present in: (PGI 09)
D. Unsaturated fatty acids □ A. Enterocyte □
8. Addition of PUFA in plasma membrane causes B. Hepatocyte □
A. Membrane becomes rigid (AIIMS 09) □ C. RBC □
B. Increase in fluidity of membrane □ D. Lymphocyte □
C. Decrease in fluidity of membrane □ 17. RBC membrane integrity is maintained by:
D. No change in fluidity of membrane □ A. Spectrin (AIIMS 02,01) □
9. True about lipid bilayer of cell wall: (PGI 01) B. Laminin □
A. Asymmetrical arrangement of cell wall C. Collagen □
component □ D. Elastin □
B. Lateral diffusion of ions □
Cell Physiology ■ 211

18. M ost permeable to pure phospholipid bilayer is- 27. Resting membrane potential is mainly due to:
A. Oxygen (NBE P 13) D A. Na+ (NBE P 15,13; PGI 03,15,13) □
B. Na □ B. K+ □
C. Cl □ C. Cl- □
D. Water □ D. M g++ □
28. RM P o f nerve cell is equal to which ion's
Transport Across Cell Membrane equilibrium potential...?- (NBE P 13)
A. K+ □
19. Osmolarity is defined as- (NBE P 13) B. Cl- □
A. Number of osmole per litre □ C. Na+ □
B. Number of osmole per kg □ D. Ca+ □
C. Weight of solute per litre □ 29. Resting membrane potential of neuron is-
D. Weight of solvent oper litre □ A. +70 mv (NBE P 13) D
20. Osmotic pressure of 1 mol of idea solute relative to B. -70 mv □
pure water is: (PGI 09) C. +90 mv □
A. 6.5 atm. □ D. -90 □
B. 22.4 atm. □ 30. What is/are effect on membrane when extracellular
C. 4 atm. □ concentration of K+ is decreased: (PGI 09, 08)
D. 2 atm. □ A. I magnitude of RMP □
E. 1 atm. □ B. T negativity of the membrane □
21. Cell volume & shape is maintained by- (NBE P 13) C. t magnitude of RMP □
A. Goldman effect □ D. 1 negativity of membrane □
B. Gibbs-Donnan effect □ 31. For sodium - potassium pump the coupling ratio is
C. Singer's effect □ A. 1 : 1 (Assam 04, AIIMS09) □
D. None □ B. 2 : 3 □
22. Potential developed due to movement of freely C. 3 : 2 □
diffusible ions across a semi-permeable membrane D. 1 : 4 □
is calculated using: (AIIMS 16) 32. K + homeostasis in human is: (PGI 05)
A. Nemst equation □ A. Most of the K+ are intracellular □
B. Gibbs equation □ B . Most of the K+are extra cellular □
C. Goldman-Hodgkin-Katz equation □ C. An essential electrolyte for different organ
D. Fick principle □ function. □
23. Equilibrium potential for an ion is calculated using: D. Actively secreted in the distal tubule □
A. Gibbs-Donnan equation (AIIMS 08,15) □ E. It maintains the Na+ K+ ATPaseactivity. □
B. Nerst equation □ 33. True about Na+ - K+ pump is that:
C. Goldman equation □ (PGI 04, 06; WB05; NBE P 15,13)
D. Fick law □ A. Involves ATPase (enzyme) activity □
24. The extracellular potassium concentration is 100 B. It can move Na+ in and out of cell □
mEq/mmol and intracellular potassium C. Electrically neutral □
concentration is 10 mEq/mmol. What w ill be the D. Pumps out one Na+ for one K+ □
equilibrium potential for potassium according to
E. Pumps 3 Na and 2k inside the cell □
N em st equation? (AIIMS 15)
34. True statement regarding mechanism of action of
A. 0 V □ Na/K ATpase: (NBE P 15,13; PGI 12)
B. -6 0 V □ A. Pumps 3 Na out & 2K+ in □
C. -9 0 V □ B. Pumps 3 K out & 2 N a+in □
D. +30 V □
C. Pumps 3 Na in & 2K+ out □
25. Fick's law of diffusion explains (AIIMS 13,14)
D. Pumps 2 K out & 3N a+ in □
A. Active diffusion along concentration gradient □ 35. True regarding Na+ - K+ pump (UP- 05, AMU -06)
B. Passive diffusion along concentration gradient □ A. Hetrodimer - homogenous □
C. Both of these □ B. Hetrodimer - hetrogenous □
D. None of these □ C. Homodimer - hetrogenous □
26. Fick's law, flux of Geomembrane increased in:
D. Homodimer - homogenous □
A. Concentration across (PGI 08) □ 36. Intracellular binding site on Na+ - K+ pump is
B. Temperature □ A. Na+ (JIPMER - 06, WB - 05) □
C. Increased size of molecule □ B. ATP □
D. Weight □
C. P 0 4 □
E. Area □ D. Ouabain □
212 ■ A Complete Review of Short Subjects

37. Binding site present on f) unit of Na+ - K+ pump is 47. Active transport across the cell membrane is
A. Na+ (TN-04, KA-05) □ mediated by: (AIIMS 01)
B. K+ □ A. G-proteins □
C. ATP □ B. Na+- K+ ATPase □
D. Glycosylation □ C. Carrier protein □
38. Which of them is TRUE about Na+-K + pump: D. Channel protein □
A. K+ is pumped against the gradient. (AIIMS 09) □ 48. Which of the follow ing statements about facilitated
B. 2K+ are exchanged with 5Na+ □ diffusion is true (AI 09)
C. Hypercalcemia causes arrest in Na+K+pump □ A. It is a form of active transport □
D. Increase in intracellular Na+ increases action B. It requires a carrier protein □
potential. □ C. Rate of transport is proportionate to the
39. The sodium- potassium pump is an example of: concentration gradient □
A. Active transport (AIIMS 06) □ D. Requires creatine phosphate □
B. Passive transport □ 49. Ionophores have following action except:
C. Facilitated diffusion □ A. Abolish proton gradient (PGI 07) □
D. Osmosis □ B. Inhibit ADP to ATP conversion □
40. True regarding transport across a cell membrane is : C. Hydrophilic in character □
A. Cl- with glucose symport (AIIMS 14; AI 08) □ D. Abolish pH gradient □
B. Na+ with glucose anteprot □ 50. Water travelling from extracellular to intracellular is
C. Na+ with glucose symport □ by- (NBE P 13)
D. K+ with glucose symport □ A. Co-transport □
41. Glucose is co-transported with Na+ ions. This is a B. Diffusion □
type of- (NBE P 13)
C. Filtration □
A. Secondary active transport □
D. Active transport □
B. Primary active transport □
51. Transport of neutral substances across the cell
C. Facilitated diffusion □
membrane occurs via (AI 08)
D. Simple diffusion □
A. Porins □
42. A cell membrane is damaged by insertion of
B. Lonophore □
microneedle, repair shall occur by which of the
C. Lipopolysaccharides □
following processes?
D. Diffusion □
A. Lateral movement of proteins (AIIMS 13) □
52. The emeiocytosis or reverse pinocytosis &
B. Resealing by lipid bilayer □
exocytosis requires which ion
C. Enzymatic reaction □
A. Na+ (NBE P 15,14,13; AIIMS 15,13,03)0
D. Hydrophobic interaction □
B. K + □
43. Which of the following increases particle diffusion
C. Ca++ □
across the cell membrane? (NBE P 13)
D. Mg++ □
A. Increasing size of particle □
53. Exocytosis (PGI 2K)
B. Decreasing lipid solubility of substance □
A. Extrusion of cell bound vesicles □
C. Increasing lipid solubility of substance □
B. Intrusion of liquid particles □
D. Decreasing size of opening in cell membrane □
C. Instrusion of solid particles □
44. All of the following transport process follow
D. All of the above □
'saturation kinetics' except: (AIIMS 05,13)
54. Clathrin is used in : (AIIMS 01)
A. Facilitated diffusion □
A. Receptor mediated endocytosis □
B. Na+ - Ca2+ exchanger □
B. Exocytosis □
C. Simple diffusion □
C. Cell to cell adhesion □
D. Na+ coupled active transport □
D. Plasma membrane □
45. True about facilitated diffusion are A/E:
55. The process by which fusion of part of a cell
A. Occur in direction of concentration gradient □
membrane occurs is/are : (PGI 05)
B. Does not require energy (PGI 08) □
A. Cell division □
C. Occur in direction opposite to electrical
B. Endocytosis □
gradient □
C. Exocytosis □
D. Facilitated by charge of molecule □
D. Virus replication □
46. All of the follow ing are true except: (NBE P 15,13)
56. The cell junctions allowing exchange of cytoplasmic
A. Facilitated diffusion requires energy □
molecules between the two cells are called-
B. Active transport is an energy dependent
A. Gap junctions (NBE P 13) □
process (AIIMS 14,13, 08) □
B. Tight junctions □
C. Co-transport is mediated via the use of carriers □
C. Focal junctions □
D. Glucose is transported via passive diffusion □
Cell Physiology ■ 213

57. All of the follow ing are involved in the 65. True statement regarding extra cellular fluid to
transmission of regulatory signals through the ECF, intracellular fluid is : (AI 08)
Except: (AI 09) A. ECF is rich K+ □
A. Synaptic signals through neurotransmitters □ B. ECF is more than ICF □
B. Endocrine signals through hormones □ C. ECF is rich in organic anion □
C. Direct contact through gap junctions □ D. High Na:K+ ratio is seen □
D. G protein coupled receptors □ 66. ICF is- (NBE P 13)
58. Nemst potential for K + is- (NBE P 13) A. 14L □
A. +90 □ B. 20% of body weight □
B. -90 □ C. 28L □
C. +70 □ D. 33% of body weight □
D. -70 □ 67. M ost osmotically active intracellular cation-
59. The ligand-receptor complex dissociates in the A. K+ (NBE P 13) □
endosome because: (AI 06) B. Na+ □
A. O f its large size □ C. Mg +2 □
B. The vesicle looses its clathrin cost □ D. Protein □
C. Of the acidic pH of the vesicle □ 68. Features of ECF as compared to ICF: (PGI 13)
D. Of the basic pH of the vesicle □ A. High K+ □
B. High Na+ □
Body Fluids/Extra Cellular Fluid C. High proteins □
D. High CP □
60. Volume of ICF in body- (NBE P 13) E. Nearly equal osmolarity in both fluid □
A. 0.2xbody wt □ 69. D 2O (Deuterium oxide) is used to measure volume
B. 0.4xbody wt □ of (AIM S 03)
A. Blood □
C. 0.6xbody wt □
B. Total body water □
D. 0.8xbody wt □
C. Extracellular fluid □
61. The follow ing are true about body water:
D. Intracellular fluid □
A. Water constitutes 60% of the body weight □
70. M ost accurate measurement of extracellular fluid
B. Plasma volume constitutes 10% of the total body
volume (ECF) can be done by using :
water (PGI 07) □
A. Sucrose. (NBE P 15,13; AIIMS 03) □
C. ECF volume can be determined by dilution
B. Mannitol. □
methods □
C. Inulin. □
D. 10% is intracellular water □
D. Aminopyrine □
62. True statements are : (NBE P 13; PGI 04)
71. Which of the follow ing methods is not used for
A. Totally body water constitutes about 60% of body
measurement of body fluid volumes? (AIIMS 05)
weight □
A. Antipyrine for total body water □
B. Most intracellular water is n the Musculoskeletal
B. Insulin for extracellular fluid □
system □
C. Evans blue for plasma volume □
C. Extra cellular water constitutes about 40% of
D. 125I-aIbumin for blood volume □
body weight □
72. Isotope used to measure RBC volume is- (NBE P 13)
D. Total plasma constitutes about 10% body
A. Cr51 □
weight □
B. H3 □
E. Extra cellular fluid can be measured by dilution
C. d 2o □
method □
D. I 135 □
63. True statements about ions composition in body: 73. Interstitial fluid volume can be determined by:
A. Intracellular & Extra cellular ions compositions (AIIMS 15)
are same (PGI 06) □ A. Radioactive iodine and radiolabelled water □
B. Phosphoru and Mg++ are major ions B. Radioactive water and radiolabelled albumin □
intracellularly □ C. Radioactive sodium and radioactive water □
C. Na+, Cl- principal ions in E.C.F. □ D. Radioactive sodium and radioactive labelled
D. Kidney tightly regulates Na+, K+, CF albumin □
composition □ 74. In a study to detect extracellular fluid volume, 10 gm
64. Predominant extracellular ions are A/E: mannitol was injected by intravenous route and
A. Na+ (NBE P 15,13; PGI 14,12,06) □ after waiting for adequate time for equilibration of
B. K+ & P 0 4- □ levels, concentration was measured as 50 mg/100 ml.
C. Cl- □ In this time, 10% mannitol was excreted. What is the
D. HCOy □ calculated volume of ECF? (AIIMS 15)
214 ■ A Complete Review of Short Subjects

A. 10 Litres □ 84. Basement membrane consists of all except:


B. 18 Litres □ A. Laminin (AIIMS 07) □
C. 42 Litres □ B. Nidogenin □
D. 52 Litres □ C. Entactin □
75. In a study, dye ABC is used to measure cardiac output D. Rhodopsin □
and blood volume. The dye is replaced with a new 85. Triple helix is seen in : (AIIMS u5)
dye XYZ which diffuses more rapidly out of the A. Collagen □
capillaries. This would affect the study by B. DNA □
A. Normal cardiac output, altered blood volume □ C. Elastin □
B. Altered cardiac output, normal blood volume D. RNA □
estimation (AIIMS 02) □ 86. Which of the following is not seen in the
C. Both altered □ interphotoreceptor matrix.: (AI 08)
D. Both normal □
A. Metalloproteinase □
76. Which of the following technique is used to study
B. Sialoprotein Associated with Rods andCones
current flow across a single ion channel? (AIIMS 14)
(SPARC) □
A. Patch clamp □
C. Tissue Inhibitors of Metalloproteinases (TIMP) □
B. Voltage clamp □
D. Memicane □
C. Iontophoresis □
87. Basement membrane (or collagen) degeneration is
D. Galvanometry □
mediated by: (NBE P 14,13; AI 07)
77. Osmoreceptor is located at which site -
A. Metalloproteinases □
A. Anterior hypothalamus □
B. Oxidases □
B. Renal medulla (AI 11, TN 04, AIIMS 13) □
C. Elastases □
C. Carotid body □
D. Atrial chamber □ D. Hydroxylases □
78. Ineffective osmols is (NBE P 14; Jipmer 02) 88. Type of collagen forming basement membrane of
A. Na+ □ kidney:
B. K> □ A. I (PGI 06) □
C. Urea □ B. II □
D. All □ C. Ill □
79. Osmotic adaptation is A/E (JIPMER- 04, WB 05) D. IV □
A. Due to osmolytes □ E. V □
B. In brain cells □ 89. The collagen triple helix structure is not found in:
C. Due to urea & glucose mainly □ A. Cytoplasm (AI 03) □
D. Protects against large H 2O shift □ B. Golgi apparatus □
80. About Sodium, true: (PGI 07) C. Lumen of endoplasmic reticulum □
A. Normal serum level is 135-145 mEq/L □ D. Intracellular vescicles □
B. Daily intake is 150 mmol of NaCl □ 90. Defect in collagen formation is seen in:
C. Major portion is extra cellular □ A. Scurvy (PGI 02) □
D. Major reserve is skeletal muscle □ B. Hunter's syndrome □
81. The majority of body sodium is present - C. Marfan's syndrome □
A. Extra cellular fluid (NBE P 15) □ D. Osteogenesis imperfecta □
B. Intra cellular fluid □ 91. Hyaluronic acid - mucopolysaccharide is present in
C. Plasma □ A. Cornea (AIIMS 10) □
D. Bone □ B. Mast cell □
Extracellular Matrix C. Vitreous humor □
D. Dermis □
82. Which of the following is/are extracellular matrix 92. Which of the following is the major proteoglycan of
protein: (PGI 09) synovial fluid- (NBE P 15)
A. Collagen □ A. Chondroitin sulfate □
B. Laminin □ B. Dermatan sulfate □
C. Fibronectin □ C. Heparna sulfate □
D. Integrin □ D. Hyaluronic acid □
E. Elastin □
93. Viscosity of synovial fluid depends upon:
83. Which of these is not a part of extracellular matrix:
A. N-acetyl galactosamine (AIIMS 14, 02) □
A. Laminin (AIIMS 02) □
B. N-acetyl glucosamine □
B. Fibronectin □
C. Glucuronic acid □
C. Integrins □
D. Hyaluronic acid □
D. Collagen □
Cell Physiology ■ 215

94. Danaparoid contains: (PGI 09) C. Nucleus □


A. Keratin sulphate □ D. Lysosome □
B. Chitin □ 104. All take place in m itochondria except
C. Dermatan sulphate □ A. Fatty acid oxidation (AIIMS 07) □
D. Heparan sulphate □ B. EMP pathway □
E. Keratin □ C. Electron transport chain □
95. Uronic acid level in urine is elevated in D. Citric acid cycle □
A. Tyrosinosis (AIIMS 07) □ 105. All of the follow ing occur in mitochondria except:
B. Maple syrup urine disease □ A. Citric acid cycle (Kreb's cycle) (AI 07) □
C. Nieman picks disease □ B. Glycogenolysis □
D. Mucopolysacchridosis. □ C. Fatty acid oxidation □
96. Products accumulated in H u n ter's syndrome are D. Electron transport chain □
A. Keratan sulfate (PGI 02) □ 106. All of the follow ing m etabolic functions occur in the
B. Dermatan sulfate m itochondria, Except: (NBE P 15; AIIMS 16; A I 12)
C. Chondroitin sulfate A. Beta-oxidation of fatty acids □
D. Heparan sulfate B. Biosynthesis of fatty acids □
E. Hyaluronic acid C. Protein synthesis/ DNA synthesis □
97. Elasticity of the com eal layer of skin is due to the D. Citric acid cycle □
presence of (AI 02) 107. Agranular endoplasmic reticulum is involved in the
A. Histidine □ synthesis of: (AI 07)
B. Keratin □ A. Protein □
C. Lysine □ B. Lipid □
D. Cysteine □ C. Carbohydrate □
98. Keratin of skin and nail differ because of D. Vitamin D □
A. Disulphide bond (AIIMS 2K) □ 108. Protein synthesis occurs in: (PGI 08)
B. Covalent bond □ A. Smooth ER □
C. Vander Waal bond □ B. Golgi bodies □
D. Hydrogen bond □ C. Rough ER □
D. Lysosomes □
Cell Organelle 109. W hich of the follow ing pathway occurs partly in
m itochondria and partly in cytosol- (NBE P 15)
99. Force generating proteins are : (AI 01)
A. Glycolysis □
A. Myosin and myoglobin □
B. Kreb's cycle □
B. Dynein and kinesin □
C. Ketogenesis □
C. Calmodelin and G protien □
D. Urea cycle □
D. Troponin □
110. Intracellular sorting and packing done for:
E. Lysosomes □ A. ER (PGI 07) a
1 0 0 . Axonal transport is- (NBE P 13)
A. Antegrade □
B. Golgi apparatus □
B. Retrograde □
C. Ribosome □
C. Antegrade & Retrograde □
D. Cytoplasm □
D. None □ E. Nucleus □
111. True regarding golgi apparatus is A/E (WB-04)
1 0 1 . Rapid axonal flow in the neurons is mediated by all
A. Cis is receiving end (Jipmer 09, A M U - 0 3 ) □
except (AIIMS 07)
B. Trans is secretory end □
A. Dynein □
C. Non polarized structure □
B. Kinesin □
C. Neurofilaments □
D. Situated near nucleus □
112. Sequence of vesicle transport is (TN 02, Kerala 03)
D. Microtubules □
A. ER —» Trans —» Cis —» lysosome □
E. Lysosome □ B. ER —» Cis —» Trans —» Cell membrane □
102. Agranular cytoplasmic reticulum is involved in the
synthesis of (AIIMS 06) C. ER —» Lysosome —» Trans —» Cis □
A. Protein □ D. Cis —» ER —> Trans —> Cell membrane □
B. Lipid □ 113. True about mitochondria is A/E:
C. Vitamin □ A. Site for ATP synthesis (NBE P 14,13; PGI 1 3 )0
D. Carbohydrate □ B. Isolation is done by hydrolysis in alkaline pH &
103. G lycolysis occurs in (AIIMS 07) synthesize fat □
A. Cytosol □ C. Protein synthesis □
B. Mitochondria □ D. Double stranded circular DNA □
216 ■ A Complete Review of Short Subjects

114. Which of the following liver enzymes is 120. In dividing cells, spindle is formed by : (AI 02)
predominantly mitochondrial: (AI 10) A. Ubiquitin □
A. SGOT (AST) □ B. Tubulin □
B. SGPT (ALT) □ C. Laminin □
C. GGT □ D. Keratin □
D. Nucleotidase □ 121. Not true about microtubules is /are: (PGI 2K)
115. Aggrecan in: (PGI 2K) A. Dynamic instability □
A. Receptor over platelets □ B. Polarity □
B. A molecule present in osteoid tissue □ C. Charged □
C. Granules in leukocytes □ D. GTP not required □
D. Important component of cartilage □ E. Form spindle □
116. Markers of plasma membrane are A/E: 122. Cell shape and motility are provided by (AI 05)
(NBEP 15,14,13; PGI 09,07) A. Microfilaments □
A. Galactosyl transferase & Glucokinase □ B. Microtubules □
B. 5 - Nucleotidase □ C. Golgi apparatus □
C. Adenyl cyclase □ D. Mitochondria □
D. Na+ K+ ATpase □
117. Enzyme marker for Golgi apparatus: Cell Cycle
A. Peroxidase (NBE P 15,14,13; PGI 0 3 , 1 4 , 1 5 ) 0
B. Galactosidase □ 123. True about cell cycle: (PGI 15,14,11)
C. Galactosyl transferase □ A. Sequence is G 1SG 2M □
D. Catalase □ B. M is Meiotic phase □
118. Marker Enzyme of Mitochondria are/is: C. DNA synthesis occur in S phase □
A. Na+ - K+ ATPase (PGI 0 5 ) 0 D. DNA synthesis occur in M phase □
B. Glutamic dehydrogenase □ E. Cell divide in M phase □
C. Lactate dehydrogenase □ 124. The correct sequence of cell cycle is:
D. No specific enzyme □ A. G0-G1-S-G2-M (AIIMS 04) □
119. Catabolism of H 2O 2 is carried out by (AI 04) B. G0-G1-G2-S-M □
A. Peroxisomes (NBE P 15,13) □ C. G0-M-G2-S-G1 □
B. Mitochondria □ D. G0-G1-S-M-G2 □
C. Endoplasmic reticulum □ 125. Which parts of cell cycle are fixed in duration:
D. Lysosomes □ A. G I (PGI 02) □
B. G2 □
C. S □
D. M □
ANSWERS AND EXPLANATIONS:

Cell Membrane

1. Ans. B i.e. Triglycerides 2. D i.e. Protein 3. B i.e. 1:1


[Ref: Ganong 23rd/ep. 32-34; Harper 28/e p. 407-11,124; Guyton 10/e P-10]
7
Cell Component Percentage
Triglycerides are not found in membrane
eukaryotic plasma membrane^.
Protein 55%
Plasma membrane is mainly
Lipid Phospholipids eg. lecithin (phasphatidylcholine), 25%
composed of proteinQ. Protein
phosphatidyl ethanolamine, phosphatidyl inositol,
equal or exceed the quantity of lipid
phosphatidyl serine and sphingomyelin®
in nearly all membranes except
Cholestrol 13%
myelin (an electrical insulator found
Glyco (sphingo) lipids eg. cerebrosides (galactosyl & 4%
on nerve fibers).
glucosyl ceramide) and ganglioside and other lipids
4 Oleic acid
Carbohydrate Glycoprotein & glycolipid 3%

Ratio o f Protein to Lipid in D ifferent Membranes

Inner mitochondrial membrane 3.2 (Max.)


a, ---------- M yelin ] 0 .2 3
Sarcoplasmic reticulum 2.0 c
G M ouse liver cell | 0.85
Outer mitochondrial membrane 1.1 ■O
e
Myelin § R etinal rods (bovine)
0.23 2
ra H um an erythrocyte
£ A m eba
★ In almost all the membranes of the body
proteins are equal or exceed the quantity of ^ ----------------- H eLa cells ^ ] 1 .5
lipid. The only exception is myelin (high M itochondrial ou ter m em brane ^ 1 .1
lipid content provide good insulation)®. Sacrop lasm ic reticulum |2.0
Whereas, the inner mitochondrial M itochondrial inn er m em brane 1 3.2
membrane has highest protein content
1 1 1 1-
per gram®.
0 1 2 3 4
R a tio o f P ro te in to L ip id

4. A i.e. Glycosylphosphatidylinositol (GPI) anchored protein [Ref: Harper 29/e p. 466, 564-65; Berne 6/e p. 5-7]


Membrane lipid rafts are cholesterol and sphingolipid (phospholipid) enriched micro-regions in the outer leaflet of
^
plasma membrane. Lipid rafts are rich in 2 types of integral membrane protein (1) GPI (glycosylphosphatidyl inositol)
anchored protein Qand (2) proteins anchored to membrane by two covalently attached long chain saturated fatty acids
(2 palmitoyl groups or one palmitoyl and one myristoyl group).

5. B i.e. Transduction of signals; E i.e. Enzyme activation at membrane surface


[Ref: Harper's 27/e P-429-30; Ganong 22/e P- 9; Shinde & Chatterjee 6/e P - 13,14]

Phospholipid in cell membrane is responsible for transduction of signals & enzyme activation at membrane surfaces®,
$
6. D i.e. Linoleic Acid 7. B i.e Saturated fatty acids 8. B i.e. Increased fluidity of membranes
[Ref. Harper’s 27/e P-428-29; 26/e P-422, 417]

Polyunsaturated fatty acids (PUFAs) eg linoleic acid addition increases the fluidity of membranes®. Whereas, saturated
FAs increase transition temperature (Tm) of lipid bilayer®.
218 ■ A Complete Review of Short Subjects

9. A i.e. Asymmetrical arrangement of cell wall component 10. B i.e. Lipids are symmetrical
IR ef: H arper's 27/e P- 427; 26/e P- 416-29; D ebjyoti D os: B iochem istry 8/e P-5151

In cell membrane there is asymmetrical arrangement of all cell components including lipids Q.

11. A i.e. Inner mitochondrial membrane [Ref. H arper's 27/e P -423; 26/e P-416],

In almost all the membranes of the body proteins are equal or exceed the quantity of lipid. The only exception is myelin
(high lipid content provide good insulation)Q. Whereas, the inner mitochondrial membrane has heighest protein
content per gramQ.

12. B i.e. A stretch of hydrophobic aminoacids 13. D i.e. Hydrogen bonds 14. E i.e. 7 aminoacid
IR ef: C hatterjee 7/e p. 12-13; H arper's 2 7 je P -425-430; 26/e P- 4501
~~ ■— ~ ■
- Trans membrane region of protein is likely to have a stretch of hydrophobic aminoacids, while hydrophilic amino
acids at ends.
- Lipids and proteins interact in membrane by hydrogen bonds. In leucine-zipper, leucine residue is seen after every 7
amino acid.

15. E i.e. Gm 2 - Gangliosides [R ef: H a rp er’s 2 7 je P- 423-251


— ■
Glycolipids are widely distributed particularly in nervous tissue as brain. They occur particularly in outer plasma
membrane where they contribute to cell surface carbohydrate^

16. C i.e. RBC 17. A i.e. Spectrin


IR ef: H arper's 27/e P -623-625; 26/e P- 615- 617; R obbin s 6je P- 607]

Glycophorins & anion exchange protein are single & multiple (10) pass type integral proteins of RBC respectively.
Integrity of RBC membrane is maintained by spectrinQ.

18. A i.e. Oxygen

• M iddle hydrophobic core (made up of phospholipid) bilayer) in cell membrane is most permeable to fat soluble
substances such O 2, C 0 2 and alcohol^. But its permeability (coefficient) to polar substances in decreasing order is:

H20 > Indole > Urea =Glycerol > Tryptophan > Glucose > CL > K+ > Na+

Urea (b/o free perm eability) and glucose (b/o consumption) are osmotic infective.

Transport Across Cell Membrane, RMP & Equilibrium Potential


19. A i.e. Number of osmole per litre

Osmolarity is numbers of osmoles per liter (in 1 liter) of solution (plasma), whereas osmolality is numer of osmoles per
kilogram of solvent (water). (Mn "Both Osmolar and liter end in r")Q-

20. B i.e. 22.4 [R ef: G arrett B iochem istry 3/ep- 37; B ern e & Levy 6/e p 15-18; G anong 2 3/e p. 4-6; G abriel p 16-17; G u tyon 11/e 52, 297-
Cell Physiology ■ 219

24. B i.e.-60V [Ref: Ganong 24/e p. 10; MTx]

Nerst Equation (Cl ) EMF = 61.5 l o g i C l j l


[CI-ol
• Because interior of cell is negative relative to exterior and NaCl
concentration is more in ECF and K+ concentration is more in ICF. (K+) EMF = 61.5 log [K-p]
Concentration & electrical gradients of various ions are [K+j]
Ion CG EG Concentration EP (mv)
Out In
For Cl-ion concentration ratio is
CI Inwards Outwards 125 9 -70
reversed (i.e. inside/outside) because
K+ Outwards Inwards 5.5 150 -90
valence is (- 1 ).
Na+ Inwards Inwards 150 15 + 60
• Sign of equilibrium potential is Now about question

Positive (+) if Negative (-) if (K+) EP =61.5 log —


° 10
- Negative (-) ion diffuse - (+) ion diffuse form inside
= 61.5 x log 10 = 61.5 x l
from inside to outside or to outside
= 61.5 = 60 mv
- Positive (+) ion diffuse - (-) ion diffuse from outside
from outside to inside to inside

I I I Donnan Effect Nernst Equation


• It is effect of a nondiffusible ion on distribution of other ions to Movement of each ion across the membrane
which membrane is permeableQ. For example a positive (+) charged depends on its concentration & electrical
non diffusible cation hinders diffusion of diffusible anion (-) & favors gradients, across the membrane. For example b/o
diffusion of diffusible cations (+). its higher concentration in ECF Cl" ion tends to
Membrane move inside the cells (concentration
1 B gradient=CG), where as relative negative cell
K+ K+ 1 Permeable interior push Cl" ion out of cell (electrical
CP cr
Protein Ion CG EG
(impermeable) Cl Inwards Out wards
Assuming that concentration of cations & anions are initially equal on K+ Outwards Inwards
both sides. Which means Cl" conc. is more on side A and it diffuses Na+ Inwards Inwards
down its concentration gradient from A to B, and some K+ moves with ■
ve charged CP because of its opposite charge. S o , The membrane potential at which influx & efflux
of an ion are equal (i.e., equilibrium b/w
[K +b] > [K a +] and furthermore.
electrical & concentration gradients exist) is k/a
[K+b] + [CPB] + [ProtB-] > [K+a] + [C1A ] equilibrium potential^
i.e., more osmotically active particles on side B than on side A. Electrical gradient that will balance a given
Donnan & G ibbs equation , for any pair of cations & anions of same concentration gradient (at 37°C) of univalent ion
valence, shows that in presence of a non diffusible ion, the diffusible can be calculated from Nemst equation
ions distribute themselves so that at equilibrium, their concentration
ratios are equal.
_ crB or k +J l c d = k ] b fl]
**• crA EMF = ±61.5/0#
• It has 3 effects on body
- There are more osmotically active particles inside the cells because of EMF = Electromotive force or equilibrium potential
presence of proteins (Pro-) and so rupture of cells d/t osmosis is R= Gas constant
prevented by Na f Kf ATPase pumpQ. (i.e. maintain cell shaped T= Absolute temperature
volume)Q. F= Faraday (coulombs per mole of charge)
- Plasma has more protein than interstitial fluid, which affects ion Zi = Valence of ion eg Cl has -1 & Na has +1
movement across capillary wall. Co = Concentration outside Ci = Concentration
- Because of asymmetrical distribution of ions across membrane at inside
equilibrium, an electrical difference exists across membrane whose
magnitude can be measured by Nernst equation.
220 ■ A Complete Review of Short Subjects

Goldmann Equation
When the membrane is permeable for several ions, the diffusion potential depends on (1) Polarity of ion (2) Permeability
(P) of each ion & (3) Concentration of each ion on inside (i) & outside (0). This means the degree of importance of each
ion in determining voltage is proportional to its membrane permeability and concentration gradient.
Goldman Hodgkins - Katz equation for calculating ^ ^ C N a '(i) PNa. + ct , ( tPr + ca WPU
membrane potential on the inside of membrane is EMF (mV)c\,,. PVi. + cr Pt. +C\.

25. B i.e. Passive diffusion along concentration gradient


26. A & E i.e. Concentration across & Area [Ref: Ganong 23rd/e p. 5; Gabriel C Ezeilo p. 15,1411
— . . . . . . .

According to Fick's law of diffusion, the net rate of diffusion is directly proportional to available surface area across
which diffusion is taking place and the concentration or chemical gradient but inversely proportional to thickness of
membrane®. Fick's law of diffusion explains passive diffusion along concentration gradient®.

27. B i.e. K+ 28. B i.e. CP 29. B i.e. -70 mv


[Ref: Ganong 22/e P- 8, 59; Gabriel C. Ezeilo p. 27-28; Sembulingum 3/e P-142]

t ---------------------------------------------------------------
The potential difference between inside and outside of the
w RMP (-) mv
Tissue
cell under resting condition is known as resting Inner Hair cell -150
membrane potential (RMP). K* channels maintain the Skeletal muscle, Ventricle® -90®
RMP®. RM P of nerve cell (-70 mv) is equal to Neuron (Nerve-cells) -70®
L equilibrium potential of Cl~® J Thyroid -5 0
Smooth muscles, SA node - 40 to - 30
RBC -1 0
Endocochlear Potential +85®

30. B & C i.e. T negativity of the membrane & T magnitude of RMP [R ef: G abriel C E zeilo p. 22, 25, 27; B ern e: P h y sio lo g y 5 /e p g -2 9 ]

Increasing extracellular concentration of K+ decreases magnitude o f RMP and negativity o f membrane i.e. (depolarize
cell)® whereas decreasing extracellular concentration of K+ increases RMP and negativity o f membrane (i.e. hyperpolarize
the cell )®. so relation is inverse but linear except at lower concentrations.

31. C i.e. 3 : 2 32. A i.e. Most of the K+ are intracellular C i.e. An essential electrohyte for different organ function
[Ref: Guyton 10/e P 338-39; Ganong 22/e P- 8, 69 35, 724; Harrison 15“'/258- 62]

M ost of the K+ ion are intracellular® and it is required for maintenance o f restingmembrane potential, nerve
conduction, muscle contraction etc. For Na-K pump the coupling ration is 3:2.

33. A i.e. Involves ATPase activity 34. A i.e. Pumps 3 Na out; Pumps 2 K in 35. B i.e. Hetrodimer hetrogenous
36. D i.e. Ouabain 37. D i.e. Glycosylation 38. A i.e. is K+ pumped against the gradient
39. A i.e. Active transport [Ref: Chaudhuri 5/e P-14; Ganong 22/e p. 33 - 35]

f•---------------------------------------------------------------------------------------------------------------------------------1
Na+ - K* pump!Na*-K+ ATPase, is an electrogenic pump ® that extrudes 3Na* from the cell & take 2K* into the
cell.(coupling ratio is 3:2)®. It is a heterodim er made up of hetrogenous a & f i units.
• It pumps both Na+ & K+ against their concentration gradient by utilizing energy from the hydrolysis o f ATP.®
• Increase in intracellular Na+, decrease the height of action potential. The pump keeps intracellular Na+ low and
contributes to small amount of membrane potential.
Cell Physiology ■ 221

a - Subunit (MW 1 lack) /3 - Subunit (MW 55,000)


a ] isoform is found in membranes of most subunit is widely distributed but absent in certain
cells, CC-, in muscle, heart, adipose tissue and astrocytes, vestibular cells of inner ear, & glycolytic fast
twitch muscles. Whereas fast twitch muscles contain only
brain; and a isoform in heart and brain
P 2 subunit
It transports Na+ & K+ and spans cell
It is a glycoprotein, has single membrane - spanning
membrane 10 times, with amino & carboxyl
terminal both located intracellularly. It domain & 3 extracellular glycosylation siteQ all of which
contains have attached carbohydrate residues.

Intracellular Extracellular
I I
Na+ binding siteQ - K+ binding siteQ
Phosphorylation - Ouabain binding
(Asp 376) siteQ siteQ
ATP binding siteQ

40. C i.e. Na+ with glucose symport [Ref: Sembulingum 3/e P - 19; MTx]

Na+ is cotransported (symported) with glucoseQ.Co (Sym) transport and antiport or counter transport are secondary
active transportQ.

Transport Osmosis Sim ple Facilitated l°A ctive Transport 2° Active Transport
D iffusion D iffusion
Move WaterQ O 2, CO 2, Fat Glucose, Unipart of ions - Symport (contra sport) of glucose/aa
some aa, Na+, K+, H+, Ca++ in cell along Na+Q
- Anti (counter) transport of H+, Ca++
out of cell against Na+

Sodium Co-transport (Symport)


In this along with Na+, another substance is carried by the carrier protein called symport. The energy is obtained from
diffusion process of Na+ due to concentration gradient across the cell membrane.

Carrier protein Substance carried by Sodium co transport


It has two receptor sites on Na+ - Co - transport of glucose
outer surface. One is for Na+ I I
& another site is for other - Glucose One sodiu m ion & one glu cose
substance - Amino acid m olecu le fro m ECF enter the cell.Q
- Chloride, Iodide
- Urate

42. B i.e. Resealing by lipid bilayer


[Ref: Berne 6/e p. 6-10; Journal of Cell Science (2002): Repairing a torn cell surface: make way lysosome to rescue]

After a cell membrane is damaged by insertion of microneedle, repair occurs by resealing of lipid bilayes through
vesicle- plasma membrane fusion (in cases of small breech) or through vesicle-vesicle fusion (in cases of large
disruption).
222 ■ A Complete Review of Short Subjects

' E n te rin g
R ep air Lipid Bilayer Resealing a th ro u g h b re e c h

- Calcium (Ca++) entering through


disruption signals repair, initiates
0 0 0 O u o O O 0 0 0
depolymerization of subcortical F actin
network and triggers accumulation of o 0 o
vesicles (powered by kinesin & myosin °o °
motor proteins) at the breach site S ig n a ls re p a ir: O e p o ly m e riz e s s u b c o rtic a l F a c tin

- In small breaches, vesicles fuse to plasma n e tw o rk ; A c c u m u la te s v e s ic le s

membrane (exocytotic reaction).


Whereas, in large disruptions vesicle-
vesicle fusion forms a large sheet of
E x o c y tic re a c tio n : V e s id e -P M fu s io n in s m a ll b re e c h i
vesicular membrane (homotypical
reaction) and this sheet fuses with plasma ooooooooooooooooooo ...
membrane completing resealing (patch V e s id e -v e s ic te fu s io n in la rg e d is ru p tio n

hypothesis).
43. C i.e. Increasing lipid solubility of substance 44. C i.e. Simple diffusion [Ref: Guyton 12/e P-45-47]
- Simple diffusion is movement of molecules across cell membrane following a concentration or chemical gradient
without any carrier protein. So rate of diffusion is directly proportional to (i.e. increases with increase in) (1)
concentration (chemical) gradient, (2) cross section area of membrane & prone size, (3) lipid solubility^. But it
decreases with (1) thickness of membrane, (2) polarity (charge) of substance and (3) large particle size.
- The rate of diffusion in simple diffusion keeps on increasing proportionately w ith the concentration of the diffusing
substance and does not follow saturation kinetics.
45. C i.e. Occur in direction opposite to electrical gradient 46. A i.e. Facilitated diffusion requires energy
47. C i.e. Carrier protein 48. B i.e. It requires a carrier protein
[Ref: Ganong 23rd/e p. 46; Guyton 11 th/e p.47; Harper 28lh/e p. 414-16; Chaudhari 6/ed pg-11; Sembulingum 3/e P-17; Harper's 27/e
P-430, 434; 26/e P-423]_____________________________________________________________________________________________

- When carrier proteins move substances in the direction of their chemical or electrical g rad ien ts no energy input is
required^ and the process is called facilitated diffusion.
- A carrier protein is responsible for both the facilitated diffusion and active transport.^

"Feature Sim ple D iffusion Facilitated D iffusion Active Transport


Type Passive Passive Active
ATP (energy) Not required Not required Required
expenditure
Concentration / Along EC gradient Along EC gradient Against EC gradient
Electrochemical (EC)
gradient
Direction Bidirectional (Can be) Bidirectional Unidirectional
Carrier proteins Not required Required Required
Saturability Not saturable Saturable Saturable
Maximum rate of Does Not have Vmax Have Vmax Have Vmax
transport (Vmax)
Rate of Transport Proportionate to - Proportionate only at low Not proportionate to
concentration gradient over concentrations before carrier protein concentration gradient
a wide range becomes saturated (difference)
- After Vmax becomes constant i.e. rate is
not proportionate over a wide range
Competitive Not inhibit Inhibited Inhibited
Inhibition
(by structurally
similar inhibitor)
Examples Diffusion of gasses (O 2, Glucose uptake by muscles, some Na+ / K+ ATPase
CO 2) in lung, movement of aminoacids H+ /K+ ATPase
fat
Cell Physiology ■ 223

49. C i.e. Hydrophilic in character [Ref: Harper's 27/c P-433-34; 26/e P-426; Shinde 6 /e P-15; Thomas M. Devlin 4/e P-211, 212, 213}

lonophores are hydrophobic in nature Q

50. B i.e. D iffusion 51. D i.e. D iffusion I Ref: Harper 27/e p. 108-10; 439-39]

-------------------------------------------------------------------------—
- Movement of water across cell membrane occurs by osmosis --------------------------------------------------------------------
(simple diffusion)*). f t
- Transport via simple passive diffusion is favoured by sm all size, lipid solubility (hydrophobicity) absence o f charge
(neutrality), and absence o f p olarity (nonpolar nature fi.

52. C i.e. Ca++ 53. A i.e. Extrusion of cell bound vesicles


54. A i.e. Receptor mediated endocytosis 55. B i.e. Endocytosis C i.e. Exocytosis
[Ref: Ganong 24/e p. 47-50; 22/e p. 28; Harper 27/e p. 436-38] .

7
Process by which fusion of part of cell membrane occurs are exocytosis & endocytosis. *) Exocytosis is extrusion of cell
bound vesicles. Clathrin is used for receptor m ediated endocytosis *). Reverse pinocytosis requires Ca*+Q.

56. A i.e. Gap junctions 57. C i.e. Direct contact through gap junctions
[Ref: Ganong 23rd/ep. 40-38; Gabriel C Ezeilo p. 9, 37, 56,113,117; Lehninger 5/e pg- 374-95]

Gap junctions are involved in transmission o f regulatory protein (Signal) from cell to cell w ithout entering ECFQ.

58. B i.e. -90

Concentration in Nerve Cell (mmol/L of H2O)


Ion Inside cell Outside cell Equilibrium Potential (mV)
Na+ 15.0 150.0 +60*)
K+ 150.0 5.5 -90*)
ci- 9.0 125.0 -70 Q

Resting M embrane Potential (RMP) = -70mV Q

59. C i.e. O f the acidic pH of vesicle [Ref: Chatterjea 7/e p. 19; Lippincott's 5/e p. 230-31 ]

The low pH breaks the linkage betw een receptor -m acrom olecule (ligand) complex.

Body Fluid / Extra & Intra Cellular Fluid

60. B i.e. 0.4xbody wt [Ref: Ganong 24/e P- 3-5; Guyton 12/SAE p. 47-53]
61. A i.e. Water constitutes 60% of body weight 62. A i.e. Total body water constitutes 60% of body weight
63. B i.e. Phosphorus & Magnesium.. C i.e. Na+ & CP principal ions ECF; D i.e. Kidney tightly regulates Na+, K+, C l-...
64. B i.e. K + & PO 4- 65. D i.e. High Na: K+ ratio is seen 6 6 . C i.e. 28L
67. A i.e. K* 6 8 . B i.e. High Na+; D i.e. High C l"; E i.e. Nearly equal osmolarity in both fluid

• In the average young adult male 18% of body weight is protein, 7% is mineral and 15% is fat. The remaining 60% is
water*). The total blood volume is 8 % of body weight.

• Na+, C l' and HCO 3' are major extra cellular and K +<P O ^ & Mg +2 are major intracellular ions.
• ECF composition is carefully regulated by various mechanisms, but especially by kidneys*)
• Osmolarity (corrected, in mosm/L) and osmotic pressure (mmHg at 37° C) are nearly equal for ECF and ICF*).

ECF ICF ECF: Predominant ions ICF: Predominant ions


Plasma Interstitial Intracellular Cation Na+(most abundant & K+ (most abundant &
fluid fluid (+) osmotically active)*) osmotically active)*) » > Mg++
Osmolarity 282 281 281 » > C a ++
Osmotic 5443 5423 5423 Anion (-) C l» > h c o 3 Phosphate ( P 0 3-3) > » Protein
pressure Overall Na+>C1- > H C 0 3->Ca++ K+> POs4> M g ++> Proteins
224 ■ A Complete Review of Short Subjects

Total Body Water (TBW)= 60% Body Weight (BW) = 42 liters

Intracellular Fluid=28 lit Extracellular Fluid = 14 liters


Is 2/3^ of TBW i.e. 40% - Is l/3rd of TBW i.e. 20% of body w eights = 0.2 x BW
of body weightQ =0.4 x - It is a general term for all the body fluids outside the cell. It consist of:
BWQ interstitial fluid , plasma , lymph, cerebrospinal fluid.
Portion of total body fluid ____________ I__________________ ,
r
which remain within the
In the Vascular system (Plasma) Outside the vascular system (Interstitial
cell membrane
- 25% or 1 /4th of ECF = 3.5 liters fluid)
5% of body w eights (8 % of - 75% or 3/4th of ECF = 10.5 liters
TBW) - 15% of body w eights (25% of TBW)

Composition of ECF & ICF

ECF (Plasma & Interstitial Fluid) ICF


Contains large amounts of Contains large amount of potassium (K+) and phosphate (P O 43') ions plus
sodium (Na+) & chloride (CP) moderate quantities of magnesium (Mg++) and sulfate (S O 4 - ) ionsQ
ionsS, reasonably large Contains only small quantities (low concentrations) of sodium & chloride^.
amounts of bicarbone (HCO 3 ) M ost abundant cation is K+ and anion is P O 43- followed by proteinQ
ions, but only small quantities
of potassium (K+), calcium
(Ca++), magnesium (Mg++), ECF
phosphate (PO43-) and organic O
iNa
acid ions. a r
Most abundant cation is Na+ ! !
and anion is Cl-S K Mg

!P

Cations - Anions

69. B i.e. Total body water 70. C i.e. Inulin 71. D i.e. I125 - albumin for blood volume
72. A i.e. Cr51 [Ref: Ganong 24/e P- 2,3]

Body Fluid/ Volume Method of Measurement


Total body fluid (or TBW) - Deuterium (2H20 , D20 heavy water most commonly used)Q
By Dilution Principle - AminopyrineQ, Antipyrine
Mn : "D A T " - Tritium oxide. (3H20 )
Extracellular fluid volume (ECF) - InulinQ (most accurate measurement), I125 iothalamate
M n : "SIM aN " - MannitolQ, Na22, SucroseQ, Tiosulfate
Plasma volume - Evan's blueQ (T-1824) = Due bound to plasma protein
Mn : [P E A] - Albumin labeled with radioactive iodineQ (I125), Na22, Thiosulphate
Intracellular volume (ICF) Total body fluid - Extracellular fluid (= TBW - ECF)
(Cannot measured directly)
Interstitial fluid volume (Cannot ECF - Plasma volume
measured directly)
Total Blood Volume - Cr?i labelled RBCQ or = Plasma volume
1 - Plematocrit
Red Cell Volume - Is volume occupied by all circulating red cells in the body can be
determined by TBV - PV
- Measured independently by injencting tagged RBCS and measuring the
fraction of red cells that is tagged. A commonly used tag is Cr51 and
Fe59, P32
Cell Physiology « 225

73. D i.e. Radioactive sodium and radioactive labelled albumin

Intestinal fluid volume cannot b e calculated directly. It is calculated by using ECF volume (which is measured by
radioactive Na22) and plasma volume (measured by radioactive iodine labeled albumin)Q.

74. 6 i.e. 18 Litres [Ref: Ganong 24/ep 2]


• Out of 10 gm of injected mannitol, 10% (=lgm ) was excreted. So amount of mannitol distributed in ECF wound be 9
gm. Because at equilibrium, concentration of mannitol in ECF= 50 mg/lOOml, i.e. 50 mg mannitol is distributed in 100
ml of ECF
_ _ . . , • 100 ml „ 0.1 lit 10 „
• So 9 gm mannitol would be distributed in x 9 gm = x 9 gm = —x9 = 18 litQ
50 mg ° 0 .0 5 gm 5

75. A i.e. Normal cardiac output, altered blood volume [Ref: Ganong 22/e P- 2; Sembulingum 3/e P- 39; Chaudhuri 5/e P- 417]

- It is especially useful in excitable cells like neurons, muscle fibres, cardiomyocytes, pancreatic beta cells and bacteria
(giant spheroplasts).

77. A i.e. Anterior hypothalamus 78. C i.e. Urea | 79. C i.e. Due to urea of glucose mainly
80. A i.e. Normal serum level is 134-145 m eq/lit; B i.e. Daily intake is 150 mmol of NaCl; C i.e. extracellular
81. D i.e. Bone [Ref: Harrison 16/e p. 253 - 54; Guyton & Hall 11/e p. 293 - 96; Bailey & Love 24/e p. 56-57]

- Typical western diet consumption provides approximately 150 mmol o f sodium chloride (NaCl) daily ®.
- Normal level of serum sodium is 136 - 145 mmol/L (SI unit) or 136- 145 meq/L (conventional unit)®.
- Na+ - K+ ATP pase pump actively transports Na+ out of cells. As a result, 85- 90% o f all sodium is extracellular® and
ECF volume is a reflection o f total body Na+ content®. And in the same way total body Na+ content is a reflection of
ECF volume.
- Sodium reserve is chiefly stored in stom ach wall®, which makes it alkaline to counter balance the acidic environment.
30-40% sodium is stored in skeletal muscles.
- The normal plasma osmolality is 275 - 290 milliosmoles/kg. To maintain it, normally about 600 mosmols/day solute must be
excreted. And since maximum urine osmolality is 1200 mosmols/kg a minimum urine out put of 500 m L /d is required.
- Osmoreceptor located in an terolateral hypothalmus® is stimulated by hypertonicity. Ineffective osmols i.e. solutes that
do not contribute to shift water across cell membrane, like urea and glucose® do not play a role in stimulating thirst. The
average osmotic threshold for thirst is ~ 295 mosmol/kg. Osmotic adaptation is a defence mechanism of brain cells in
chronic hypo & hyper natremia against large w ater shifts®. It is mediated by initialshifts of K'+ & Na+ f/b synthesis,
import or export of organic solutes (osm olytes) such as inositol, betaine & glutamine®. In chronic hyponatremia brain
cells lose & in chronic hypernatremia gain solutes.
- Sodium is the principal cation content of ECF. The total body sodium is - 5000 mmol, of which 44% is in extracellular fluid
9% is in intracellular fluid and remaining 47% is in bone®.
- 98% of potassium (K+) is intracellular & 2% in extracellular fluid. 3/4 o f total boy K* ( - 3500 mmol) is found in skeletal
mucles®. Normal adult ingests - 1 .0 mmol/kg of K+ in food. Fruit, milk & honey are rich in K+.
226 ■ A Complete Review of Short Subjects

Extracellular Matrix
82. A, B, C, E i.e. Collagen, Laminin, Fibronectin, Elastin
83. C i.e. Integrins [Ref: Harper 28/e p. 527-35; Lippincott 5/e p. 43-51; Lehninger 5/e p. 249-54; Robbin's 7/e p. 103-106]

Extracellular matrix proteins are collagen, elastin. fibrillin, fibronectin, laminin and proteoglycans (GAGs) Mn- "Call
Ela For Last Prose" Whereas cell adhesion molecules present in cytoplasm or cell membrane include integrins,
selectins, cadherins and im munoglobulin family CAMsQ.
Integrins are cell surface adhesion protein that provide linkage between cell outside & inside, between cell- cell and
between cell & extracellular matrix.

84. D i.e. Rhodopsin [Ref: Robbins 7/e p. 103- 06, 956, 54]

Basement membrane is PAS (periodic acid shiff) positive amorphous structure which consist of non fibrillar collagen ☆^
1 |

(mostly type IV), lam inin (most abundant glycoprotein), fibronectin, entactin (nidogen)Q, tenascin and several other
glycoproteins along with polyanionic proteoglycans (mostly heparan sulfate or perlecan).

85. A i.e. Collagen [Ref: Chatterjea: 7/e p. 85-86]

Due to formation of hydrogen & disulphide bonds, peptide chain assumes 3-D secondary structure by way of folding or
coiling leading to formation of

a- Helix P- Pleated sheet Triple Helix Reverse turns /P- Bends

Found in KeratinQ - Silk fibers CollgenQ is rich, in proline - Polypeptide chain of globular protein
- Spider's web & hydroxy- proline & changes direction >2 times, when it folds
- Reptilian claw cannot for a-helix & P k/a P- bends
pleated sheet - Proline is ideally suited for 2nd residue and
It forms triple helix glycine for 3rd residue of reverse turn
86 . D i.e. Memicane [Ref: Albert & Jacobie 2/e vol 2 p.1419-21; The eye: Basic sciences in practice 2/e p. 212; Molecular biology in
Medicine 3/e vol I p. 1018-38]

Interphotoreceptor matrix has SPARC, TIM P and matrix metalloproteinases but no mimecaine (or osteoglycin).

87. A i.e. M etalloproteinases [Ref: Robbins 7/e p. 110-11,103, 312]

Zinc dependent matrix metalloproteinases (MMP) degrade collagen and other extracellular matrix proteins (including t
that of basement membrane)^.

88 . D i.e. IV 89. A i.e. Cytoplasm 90. A i.e. Scurvy; D i.e. Osteogenesis


[Ref: Lipppincott 2/e P-40; Harper's 27/e P-545-49; 26/e P-535-538; Robbins 7/e p. 104]

Collagen show triple helixQ and is never present free inside the cytoplasm^. Scurvy, osteogenesis imperfecta^, Ehler -
Danlos syndrome & Alport syndrome occurs d/t its deficient synthesis.

91. C i.e. Vitreous humor 92. D i.e. Hyaluronic acid 93. D i.e. Hyaluronic acid
[Ref. Harper's 28/e p. 535-38; Lippincott's 5/e p. 158-63; Katzung 10/e p. 547; KDT 6/e p. 600]

- The viscosity and the plastic (thixotropic properties) of the synovial fluid is because of its hyaluronate content.^
- Hyaluronic acid, a glycosaminoglycan is a polymer of repeating disaccharide units (of glucuronic acid and N-acetyl
glucosamine).
Cell Physiology ■ 227

Functions of Glycosaminoglycans

Hyaluronic acid - Viscosity of the sy n ov ial fluid®.


- Tensile strength and elasticity of cartilage and tendons
- Jelly like consistency of vitreou s hum or o f eye®.
Chondroitin sulfate - Tensile strength of cartilage tendons, ligaments and the
walls of aorta*.
Dermatan sulfate - Pliability of skin* and heart valves*.
Keratan sulfate - Found in hom y structures* of body composed of
dead cells - hair, nails, claws, horn, hoofs.
- Also present in cornea*.
Heparin - Anticoagulant

94. A, C, D i.e. Keratin sulphate, Dermatan sulphate, Heparan sulphate [Ref: Gray's 40/e p 974]
f --------------------------------------------------------------------------------------------------------------------------------------------------------------------------
Danaproid is a low molecular weigh (LMW) heparanoid containing chondroitin sulfate, dermatan sulfate and heparan
sulfate (i.e. sulfates of chondroitin, dermatan & heparan)0 . Other heparinoids are ancrod, lepirudin and heparan
sulfate.

95. D i.e. Mucopolysacchridoses [Ref. Harper's 29/e p. 599; Lippincott 2/e P-154; NMS Biochemistry 4/e P-83]
96. i.e Dermatan sulfate D i.e. Heparan sulfate

Uronic acid level is elevated in urine in mucopolysaccharidosis. Dermatan & heparan sulfate are urinary
metabolites accumulated in Hurler & Hunter's syndrome0.

97. B i.e. Keratin [Ref: 1ADVC Textbook & Atlas of Dermatology/H^ /9-10; Lehinger 2/e P 60-65,126- 135]

The elasticity, stability and integrity of stratum comeum of skin is due to disulfide cross linkages betw een the
keratin molecules0

In the stratum corneum layer, keratohyaline granules are present. These granules form protein profilaggrin, which is
degraded into filaggrin (filament aggregating protein), as granular cells transform to keratinized corneum cells. Filaggrin
is a hyaline rich protein which functions as interfilamentous glue to aggregate and align the keratin filaments in
keratinized cells. Keratohyaline also contain cysteine rich proteins, whose disulfide bonds contribute to the chemical
inertness and strengh of the keratinized layer.

98. A i.e Disulphide bond IR ef. L ip p in c o tt’s 2 /e P -1 8 ; R ef: IA D V C T ex tb o o k & A tla s o f D erm a to lo g y /1 st /9 -1 0 ; L eh in g er 2 /e P 60-65, 126- 135/

The keratins are a family of closely related fibrous proteins, whose structure is nearly entirely alpha-helical.They are a
major component of tissues such as hairs and skin; their rigidity is determined by the number of disulphide bonds
between the constituent polypeptide chains.0

Cell Organelle

99. B i.e. Dynenin and kinesin IR ef: C o m p reh en siv e h u m an p h y s io lo g y R. G reg o r/8 6 ; H arper's 2 7 /e 5 8 5 ; 2 6 /e P -5 7 7 ; G a n on g 2 2 /e P- 1 4 ,1 5 1

M icrotubules guide the transport of protein & vesicular material; and 'M otors' (Denin & Kinesin) provide the force
behind such movement0

100. C i.e. Antegrade & Retrograde


101. C i.e. Neurofilament [Ref: The journal of cellular biology: Axonal transport of membranous & non membranous cargoes: a unified
perspective: Anatomy Brown (17 march 2003)]

- Rapid axonal flow in neurons is mediated by dynein, kinesin & lysosome. Whereas, microfilaments » microtubules
& neurofilaments mediate slow axonal transport0.
228 ■ A Complete Review of Short Subjects

- Materials destined for the axon are transported anterogradely, toward the axon tip, and materials destined to return
are transported retrogradely, toward the cell body. This bidirectional transport process, known as axonal transport, is
not fundamentally different from the pathways of macromolecular and membrane traffic that occur in all eukaryotic
cells, but it is remarkable for its scale

102. B i.e. Lipid 103. A i.e. Cytosol 104. B i.e. EMP pathway 105. B i.e. Glycogenolysis
106. B i.e. Biosynthesis of fatty acids 107. B i.e. Lipid 108. C i.e. Rough ER 109. D i.e. Urea cycle
[Ref: Harper 29/e p 308-9,155-55, 211, 207, 549-50,122; Chattarjea 8/e p 6-9, 438; Ganong 22/e P-18; Guyton 11 fe p. 21, Chaudhari
5/e p. 4; Lipponcott 4/e p. 220-225; Harper 27/e p. 100-01, 506]
f-------------------------------------------------------------------------------------------------------------------------------
- Lipid & steroid synthesis occurs in smooth (agranular) cytoplasmic /endoplasmic reticulum, whereas protein

synthesis occurs in rough or granular ER.
- Glycogenesis, glycogenolysis, glycolysis, HMP (PPP) shunt, biosynthesis of fatty acids and denovo purine synthesis
occurs in cytosol (not in mitochondria)Q. Mitochondria is primarily (mainly) involved in TCA (Kreb's /citric acid)
cycle, oxidative phosphorylation (ATP synthesis), electron transport (respiratory) chain, ^-oxidation of fatty acids
(<20 carbon) and ketone body formation (Keto genesis). It is also partly involved in metabolic functions like
gluconeogenesis, urea synthesis (cycle), heme synthesis and denovo pyrimidine synthesisQ.
- Mitochondrial double stranded circular DNA is found in matrix and encodes mitochondrial (mt) r-RNAs (12 S small &
16 S large subunit), 22 t RNAs and 13 protein subunits of respiratory chain (out of total 67). It has high mutation rate
and contains very few untranslated sequences. Although mitochondria synthesize some proteins required for energy
generating function, the structural proteins and other proteins need to carry out cellular respiration are encoded by
nuclear chromosomal DNA.

Sub cellular Sites of M etabolic Pathways

Cytosol (Main / Mitochondria Mitochondria & Nucleus


Primary site) -g-oxidation of fatty acids Cytosol -DNA replication & transcription (mRNA synthesis)
- Glycolysis. (<20 carbons)Q - GluconeogenesisQ
Golgi Apparatus
Glycogenolysi - Electron transport (initially in
- Sorting of proteinsQ for their correct destination
s& (respiratory) chain = ETCQ mitochondria & later
-Synthesize & packaging of complex molecules
GlycogenesisQ -Tricarboxylic acid (Kreb's in cytosol)
(including glycolipids, glycoproteins &
-HM P shunt or citric acid) cycle = TCA - Urea synthesisQ
(PPP lipoproteins) such as hyaluronic acid and
- Oxidative (initial two steps in
pathway)Q phosphorylation (ATP chondroitin sulfateQ.
mitochondira &
- Fatty acid synthesis)Q - Maturation of synthesized proteins, protein sorting,
remaining in cytosol)
synthesisQ -Ketogenesis (ketonebody - Heme (& porphyrin packaging & secretion
- De novo formation) derivatives) synthesis
purine Endoplasmic Reticulum (ER)
Mn BEKT- OX in (Mitochondria —> ._________ I____________ ,
synthesis cytosol —>
Mn- Cytosolic Mitochondria
mitochondria) Rough /Granular Smooth /Agranular
Shunt of Pure, Peroxisome followed by • Ribosomes are >In this ribosomes are absent
- Denovo pyrimidine
Fatty, & Glyco mitochondria attached to the >Function
synthesis (cytosol
synthesis (+ Beta oxidation of very long cytoplasmic side
followed by -Site for steroid syntheisQ
lvsis)Q of membrane
chain fatty acids (> 20 mitochondria) in steroid secreting cells
carbons) Mn- "New Glucose, • Function: -Site of detoxification in
Cytosol f/b
Microsome, Mitochondria urea heme & other cells.
Endoplasmic - Protein
& Peroxisome pyrimidine" -Synthesize lipid
Reticulum (ER) synthesisQ
- Primary bile acid Microsome (“ ER) especially phospholipids
membrane -Initial folding
synthesis (1st regulatory 7- -Hem e catabolism & cholesterolQ
-Cholesterol of
a hydroxylase is (degradation ie -Provide enzymes that
synthesisQ polypeptide
microsomal enzyme; 27 bilirubin production) control glycogen
chain with
hydroxylation forming -Elongation of FA breakdown when
formation of
chenodeoxy cholyl CoA is chains (both saturated glycogen is to be used for
disulfide
mitochondrial step). & unsaturated fatty energy.
bond
- Bile salts are formed by acyl-Co As)
conjugation of primary bile
acids in peroxisome.
Cell Physiology ■ 229

110. B i.e. Golgi apparatus 111. C i.e. Nonpolarized structure 112. B i.e. ER—»Cis-»Trans—»Cell membrane
[Ref: Chatterjea & Shinde 7/e p. 7; Ganong 22/e p. 27- 28; Guyton & Hall 11/e p. 15]

G olgi apparatus serves as a unique sorting & packing deviceQ, that receives newly synthesized proteins, all containing

signal or transit peptide from endoplasmic retinaculum. It rejects proteins with no signal or transit peptide without
processing and that remain as cytoplasmic protein.

113. B i.e. Isolation is done by hydrolysis in alkaline pH & synthesize fat [Ref.Harper 28/e P-193; Chatterjea 7/e P 5-6]
t -----------------------------------------------------------------------------------------------------------------------------------------------------------------------------
- M itochondria is the 'Power house' of cell because enzymes for energy metboism and ATP synthesis are present in
mitochondria^. Mitochondria both import and synthesize proteins^. Isolation of mitochondria is done by
ultracentrifugationQ.Mitochondria has double stranded circular DNAQ.
- M ain pathway for lipogenesis (do not have fatty acid synthesis) occurs in cytosolQ. Fatty acid is synthesized in
cytoplasm o f a cellQ.

114. A i.e. SG O T (AST) [Ref: Vasudevan 6 /e p-268-70; Lehninger 5/e p 631-33, 677-88, 873-74; Herper 28/e p 272-73,
132/35136143147/87/84185,111-12,488,101, 610,319-20, 207,104 Clatchey: Clinical laboratory medicine 2/e p 286-88; Textbook of
Hepatology 3/ep 451; Chatterjae 7/e p 625 .

--------
Glutamate dehydrogenase (GDH) and aparate aminotransferase (AST)/ or serum glutamate oxaloacetate
transaminase (SGOT)Q mitochondrial isoenzyme, are liver enzymes predominantly present in m itochidrial matrix.

• AST (or SGOT) exists in genetically distinct 2 different forms which are named a/t their location: (1) M itochondrial
isoenzyme (80%) is responsible for maximum AST activity of liver a n d ; (2) Cytosolic or cytoplasmic isoenzyme (20%)
is responsible for most of circulating AST activity.
• AST (SG O T) is found in maximum concentration in Heart. ("Mn She G O T Heart") and then in liver, skeletal muscle and
kidney. So elevated levels of mitochondrial AST or mitochondrial AST/Total AST ratio is indicative of necrosis of cells
containing AST (and thus causing cellular leak) eg heart cell necrosis in MI or liver cell necrosis in chronic liver disease.
• SGOT (AST) and SGPT (ALT) both are found in most tissues but their relative amounts vary. Heart muscles are richer in
SGOT w hereas liver is richer in SGPTQ.

115. D i.e. Important component of cartilage [Ref: Harper's 27/e P-558-61; 26/e P-551]

Aggregan is the m ajor proteoglycan component of cartilageQ. It is composed of several glycosa - amino glycans
(hyaluronic acid, chondroitin sulfate and keratin sulfate) and both link and core proteins.

116. A i.e. Galactosyl transferase & G lu c o k in a s e 117. C i.e. Galactosyl transferase


118. B i.e. Glutamic dehydrogenase 119. A i.e. Peroxisomes [Ref: Harper's 28/e p. 410]

^
M arker enzymes of plasma membrane are adenyl cyclase, 5' nucleotidase and Na+ - K + ATPase. Mn "AN NA". Whereas,
marker of golgicomplex is galactosyl transferase^ and mitochondria is glutamate dehydrogenase^.

•74 f f
o rfra S n ' Plasma 5'-NucleotidaseQ
Nucleus DNA Site of chromosomes Adenylyl cyclaseQ
Site of DNA-directed RNA Na+-K + ATPaseQ
synthesis (transcription) Endoplasmic reticulum Glucose-6-phosphataseQ
Mitochond Glutamate Citric acid cycle, oxidative Golgi apparatus
ria dehydrogenase Q phosphorylation
Cis GlcNAc transferase I
Ribosome High content of Site of protein synthesis Medial Golgi mannosidase II
RNA (translation of mRNA into
Trans Galactosyl transferase^
protein)
TGN Sialyl transferase
Endoplas Glucose-6- Membrane-bound ribosomes
Inter mitocho-ndrial ATP synthaseQ
mic phosphatase Q are a major site of protein
membrane
Reticulum synthesis
Synthesis of various lipids
Oxidation of many xenobiotics
(Cytochrome P450)
230 ■ A Complete Review of Short Subjects

Lysosome Acid Site of many hydrolases Serum M ajor Diagnostic


phosphatase Q (enzymes catalyzing Enzyme Use
degradative reactions) Aminotransferases
Plasma Na+-K+ ATPaseQ Transport of molecules in and - Aspartate Myocardial infection
membrane 5'- out of cells aminotransferase
NucIeotidaseQ (AST, or SGOT)
Intercellular adhesion and
- Alanine Viral hepatitis
communication
aminotransferase
Golgi Galactosyl Intracellular sorting of proteins (ALT, or SGPT)
Apparatus transferaseQ Glycosylation reactions Amylase Acute pancreatitis
Sulfation reactions Ceruloplasmin Hepatolenticular degeneration
Peroxisom CatalaseQ Degradation of certain fatty (Wilson's disease)
es Uric acid oxidase acids and amino acids Creatine kinase Muscle disorders and
Production and degradation of myocardial infarction
hydrogen peroxideQ 7-Glutamyl Various liver disease
Cytoskelet No specific Microfilaments, Microtubules, transpeptidase
on enzyme markers Intermediate filaments Lactate dehydrogenase Myocardial infarction
Cytosol Lactate Enzymes of glycolysis & fatty (isozymes)
dehydrogenase Q acid synthesis Lipase Aute pancreatitis
Phosphatase, acid Metastatic carcinoma of the
prostate
Phosphatase, alkaline Various bone disorders,
(isozymes) obstructive liver disease

120. B i.e. Tubulin 121. D i.e. GTP not required 122. B i.e. M icrotubules
[Ref: Harper's 27/e P-584-86; 26/e P-576- 78; 539- 40]

In dividing cells mitotic spindle is formed by microtubules containing a - & p tubulinsQ

★ Ubiquitin is a small protein present in all eukaryotic cells, targets many intracellular proteins for degradation.
* The primary components of basal lamina are three protein laminin, entactin and type IV collagen Q and the GAG heparin
or heparan sulfate. Laminin is a m ajor protein component o f renal glomerular and other basal lam inas 0. Entactin or
nidogen is a glycoprotein containing RGD sequence (Arg - Gly - Asp) is the major cell attachment factor. The negative
charges of heparan sulfate and certain sialic acid containing glycoprotein present in basal lamina repel albumin & most
plasma proteins, which are negatively charged at the pH of blood, (small pore size also prevent excretion of proteins).

Cell Cycle

123. A i.e. G i SG 2M; C i.e. DNA synthesis occurs in S phase; E i.e. Cell division in M phase
124. A i.e. GO - G I - S - G2 -M 125. B i.e. G2; C i.e. S; D i.e. M [Ref: Robbins 7/e P 91, 291; Gray's 40/e p. 20-23]
Sequence of cell cycle is Go Gi S G 2 M ; where 'S ' is the DNA synthesis/replication phase and M is cell division/mitosis
phase. Time taken for S, G 2 and M phases are fixed in duration.
Cell Physiology

NOTES
232 ■ A Complete Review of Short Subjects

NOTES
PHYSIOLOGY

Review Notes with High Yield Text


Chapter 1. CENTRAL NERVOUS SYSTEM: REVIEW NOTES

Higher Function

Association Areas Brain Fuel

The proportion of the various parts of the brain are similar in the brains The brain contains essentially no fuel reserves^ and
of apes and human, but the human brain is largei<2, so the absolute must be continuously supplied with fuel from the
size of the association area is greater^. blood. So it is sensitive to hypoxiaQ.
Association areas: are part of six- layered neocortical mantle of gray Brain tissue normally uses glucose^ as an exclusive
matter that spreads over the lateral surfaces of cerebral hemisphere from fuel, except during starvation
concentric allocortical & juxta cortical rings around the hilum. The most During starvation brain can adapt to use ketone
prominent gross feature of the human brain is immense growth of bodiesQ as an energy source.
three m ajor association areasQ.
i) Frontal in front of premotor area
ii) Parietal temporal occipital - b/w somatesthetic & visual cortex Pattern of Sensory Loss
extending into posterior portion of temporal lobe.
iii) Temporal - from the lower portion of temporal lobe to limbic Cortical lesion Brainstem lesion
system. Speech & other intellectual function are controlled by Patients is able to Loss of pain & temperature
neocortex. recognize all sensation on ipsitateral face and
★ Brain size: Human >Chimpanzee > MonkeyQ. W eight of Human but localizes them opposite side of bodyQ.
brain=1400 gm and Monkey brain = lOOgm poorly (Lateral medullary
★ The ratio b/w brain weight and body weight is more important than - Loss of two point syndrome)^
size of the brain as three species have brains larger than humans (the discrimina-tionQ [Mn: TOP = Temp & Pain
whale, elephant & porpoise) - AsteregnosisQ (loss of Opposite side]
★ The right to left differentiation is seen in chimpanzee and human. In touch sensation)
humans left brain > Right brain - Sensory inattention Spinal cord lesion

Thalam ic lesion Five patterns are seen


Functions & Lesions depending upon whether
Hemisensory loss of all there is:
I ------------------------------------ 1
Somatosensory area I Somatosensory modalities^ - Complete transaction
(SI) area II (SA II) - Hemisection
I I - Central cord section
Primary somatosensory area (SI) is Brodmann's area Lesion of - Posterior column section
1,2, 3 situated in postcentral gyrus. Ablation of SI somatosensory II - Anterior column section
would lead to cortical pattern of sensory loss. Lesion is associated with
in this area produce contralateral impairment of: alteration in pain
TouchQ Sterognosis sensation.
Pressure Barognosis
Proprioception Cu taneographia
Two point
discrimationQ

Limbic System
It is the entire neuronal circuit that controls emotional behaviour & motivational drives and a term that originally was used to
describe bordering structures around basal region of cerebrum.
I - I___________
Functional Anatomy Functions of hypothalamus Functions of other parts
I
From physiological point of view, Posterior Hypothalmus • Hippocampus___________________
hypothalmus, located in the middle Part of Vegetative <5c Behavioural Almost any sensory experience activates
is one of the key (central) element of hypothalmus endocrinal function
some part which intum distribute
limbic system function
236 ■ A Complete Review of Short Subjects

Subcortical limbic structures, Lateral area• Thirst & hunger Stimulation it acts as a channel through which
surrounding hypothalmus are Mn centre increases general incoming sensory signals can initiate
" Amy SAT Perfectly On - Stimulation 1/t level of activity behavioural reactions for different
Portions of Basal Hip" i.e., extreme hunger, 1/t fighting & purposes.
voracious overt - Becomes hyper excitable and gives off
Amygdala
appetite & rage (T B P ) prolonged output signals. So in
Septum area
intense desire to hippocampal seizures psychomotor
Anterior nuclei of
search food (olfactory, visual, auditory, tactile)
Thalmus - Damage causes
Para Olfactory area hallucinations cannot be suppressed as
lost desire for
Portions of Basal ganglia long as seizure persists.
food and lethal
Hippo campus starvation. - Provides the drive that causes
Ventro Stimulation 1/t
consolidation o f long term memory (i.e.
Limbic cortex, surrounds sub • Satiety center
medial tranquility translates short term memory into long
cortical limbic areas. It is a ring of - Stimulation 1/t
satiety, term memory)®. It determines the
cerebral cortex composed of Mn -
decreased eating importance of incoming sensory signal
"CORPUS"
- Destruction 1/t and has a critical decision making role that
-Cingulate gyrus the information is likely to be committed to
voracibus
-Orbitofrontal cortex memory, making the mind rehearse over &
appetite
-Parahippocampal gyrus over the new information until
• Neuroendocrinal
-Uncus permanent storage takes place.
control
- Subcallosal gyrus Mamillary - B/L damage 1/t anterograde amnesia,
• Feeding reflexes
Limbic system & brain stem body such as licking of with lost ability to establish memory
communicate via media forebrain lips & lasting longer then few minutes.
bundle & short pathways among swallowing • Amygdala
reticular formation. The former Dorsomedial G I stimulation
- Corticomedial nuclei concerned with
extends from septal & orbitofrontal nucleus
olfaction
cortex through middle of Perifomical - Hunger Rage
nucleus - Basolateral nuclei is more developed in
hypothalmus to brain stem reticular - T bp
humans which is concerned with
formation. Posterior - tB P behavioural activities
Most anterior & most posterior area hypothalmus - Pupillary - It is the window through which limbic
of hypothalmus stimulates sexual dilation system sees the place of person in the
drive. - Shivering* world®
Fear is produced by stimulation of Arcuate - Neuroendocrinal Fear & - Stimulation can cause all the effects of
hypothalmus & amygdaloid nuclei. nucleus & control punishment hypothalmus puls tonic / clonic /
Amygadala encodes memories that Periventricul reaction rhythmical / circling movements, rage,
evoke fear. arzone escape, punishment, severe pain, fear,
Anxiety is related with anterior end Anterior Hypothalmus reward, pleasure, and even sexual
of temporal lobes activities such as erection, copulatory
Violent range occurs after removal Paraventricular - Oxytocin release® movements, ejaculation, ovulation,
of neocortex, & after destruction of nucleus - Water conservation uterine activity & premature labor.
ventromedial hypothalmic nuclei & Medial preoptic - IBP - Kluver Bucy syndrome d/t bilateral
septal nuclei (with intact cortex) area - iHeart rate ablation o f amygadalaO presents with
Abnormal placidity (calm) - Bladder contraction (1) loss of fear, (2) extreme curosity (3)
produced by b /1 destruction of Posterior - Body temperature regulation tendency to place everything in mouth
amygdaloid nucleus preoptic & - Panting, sweating (4) forgets rapidly and (5) strong sex
Lesions of left insula & putamen 1/t anterior - Thyrotropin inhibition drive so much so that it attempts to
difficult recognition of disgust on hypothlmic copulate even with wrong (immature,
faces or in voices area wrong sex, different species) annuals &
Reward /satisfaction/ approach - Supraoptic - Vasopressin (ADH) release® inanimate objects.__________________
centers are located along the course nucleus • Limbic cortex____________________
o f medial forebrain bundle Punishment /avoidance /aversion /escape - - Anterior temporal cortex : Gustatory &
especially in lateral & centre are found in central gray area surrounding olfactory behavioural association
ventromedial nucleus o f aqueduct sylvius (in mesencephalon), lateral - Posterior orbital frontal cortex damage:
hypothlmus® (Gyton). The most portion of posterior hypothalmus, dorsal midbrain, Insomnia with intense motor restlessness
responsive area is in dopaminergic entorhinal cortex, & periventricular zone of (unable to sit still & moving about
pathway from ventral tegmental hypothalmus & thalmus. continuously)
area to nucleus accumbens®. So it Addiction is associated with reward centre & - Parahippocampal gyri : complex
includes ventral tegmentum, medial particularly with nucleus accumbens. D3 auditory & thought associations derived
fore brain bundle, nucleus accumbens, dopamine receptor blocker reduce and agonist fro Wemike's area
& frontal cortex. increase the rate of self stimulation (reward). - Middle & posterior cingulate cortex:
sensorimotor behaviour association.
Physiology: Central Nervous System ■ 237

I
It has two aspects- sensory & expressive. Sensory aspect The lim bic system or lobe is applied to the part of brain that
of emotions develop in the lim bic systemQ where as consist of a rim of cortical tissue around the hilum of cerebral
expressive em otions develop in the hypothalamusQ (as hemisphere and a group of associated deep structures
well as amygdala) Neuronal circuits exists which connects It was formerly called rhinocephalon because of its relation to
limbic system with hypothalamus eg. Papez circuits olfaction & it is phylogenetically the oldest part of cerebral cortex
(allocortex)
It consists of:
Sensory Expressive 1. Cingulate gyrus
I I 2. Septal nuclei
It is characterized by Expression is the 3. Hippocampal formation - Hippocampus
I) Cognition is an awareness motor side of - Dentate nucleus
of the sensation & usually emotional behaviour 4. Amygdala
its cause (i.e. seeing & and has 2 sub • Cingulate gyrus & hippocampus form a 'C ' which encircle upper
recognizing) eg recognizing divisions part of brainstem
a person who had 1) Autonomic via • Limbic system also contains a fibrous tract median forebrain bundle
previously insulted you. sympathetic or (MFB)
II) A ffect means developm ent parasympathetic • Papez circuit links limbic system to hypothalamus & thalamus and is
o f feelin g* eg. rage system concerned with expressive side of emotions. It consists of:
III) Conation is the desire to 2) Som atic eg act of Fom ix from hippocampus —> m am illary body o f hypothalamus
ta k e action 0 eg. desire to beating —» Anterior nucleus o f thalamus —» Cingulate gyrus —>
beat Hippocampus again.

Sham Rage (= False Anger) Reaction Punishment or Escape Centers

Sham (false) rage is defined as an outburst of inappropriate, Electrical stim ulation of punishm ent center ie thin
uncontrolled (pathological /violent) rage (fear & anger) on mild zone of periventricular nuclei (located adjacent to
peripheral provocation or stimulation. It is seen in decorticate 3rd ventricle), adjoining central grey area of
animals (after removal o f neocortex) and after destruction of m esencephalon and lateral hypothalamus leads to
venteromedial nuclei of hypothalamus & septal nuclei, due to unpleasant (aversive) fear, anger, defence, escap e,
release of punishment centers of brain (esp in periventricular zone terror, punishm ent (even sickness & pain)
of hypothalamus & lateral hypothalamus) from cortical and reactions.
hypothalamic inhibitory control. Most potent areas for punishm ent & escape
It is misnomer, as originally it was thought that the rage attacks in tendencies are found in central grey area
decorticated animals (with fore brain, diencephalic lesions) surrounding the aqueduct of Sylvius in
represent only a physical - motor phenomenon of anger and lack mesencephalon, extending upwards into the
emotional (mental) aspect. But now its proved wrong as animals periventricular zone of hypothalamus & thalamus;
develop unpleasant feelings directed with great accuracy at the whereas less potent areas include amygdale and
source of irritation and develop extremely persistent conditioned hippocampus.
avoidance response. Stim ulation of punishm ent center usually inhibit
Sham rage (like normal rage) can also be produced by strong (in the reward & pleasure centers completely, showing
normal) or mild (in decorticated animals) stim ulation o f that fear and punishment can take precedence over
punishment centers o f hypothalam us Q and shows features of pleasure and reward
sympathetic stim ulation like increased BP, increased heart rate,
dilation of pupils and piloerection.

Rage & Punishment Centers Reward or Pleasure Centers

Rage is an emotional behavior pattern that involves strong If electrical stimulation of any area gives the animal
stim ulation of the punishm ent centers o f hypothalamus & other a sense of reward, then it will stimulate the area by
lim bic structures (especially in periventricular zone of pressing lever again and again (sometimes even
238 ■ A Complete Review of Short Subjects

hypothalamus and in lateral hypothalamus) and cause animal to thousands of time per hour) - ie self stimulatory
behave in a manner that one would expect from an animal being reward induction. When offered the choice between
severely punished. It causes the animal to eating delectable (liked) food as apposed to
(1) Develop a defence posture stimulate the reward center, the animal often
(2) Lift its tail chooses electrical stimulation of reward induction.
(3) Hiss, spit, growl or bite - M ajor reward centers are located along course of
(4) Extend its claws the media forebrain bundle especially in lateral and
(5) Develop sympathetic features like dilated pupils, wide open ventro medial nuclei of hypothalamus. Less potent
eyes & piloerection and reward centers which are secondary to major
(6) Cause an immediate savage (crudely violent) attack even on hypothalamic centers are found in the septum,
slightest provocation. amygdala, certain areas of thalamus and basal
Normally, this rage phenomenon is held in check mainly by ganglia and extending downward into the basal
inhibitory signals from the ventero medial nuclei of hypothalamus. tegmentum of mesencephalon
Portions of hippocampi & anterior lim bic cortex especially in - Lateral nucleus, like many other areas, with weaker
anterior cingulate gyri and subcallosal gyri also help to suppress stimuli giving a sense of reward and with stronger
the rage reaction. stimuli a sense of punishment.

Sexual behavior is Integrating Areas o f Hypothalamus Function


integrated in anterior- Anterior hypothalamus (response to heat), Temprature regulation (anteriorly
ventral hypothalamus®; Posterior hypothalamus (response to cold) hot, posteriorly cold)
plus piriform cortex (in Suprachiasmatic nuclei Control of circadian body rhythm
males only) - Ganong Punishment centers (esp periventricular zone of Defensive reactions (fear, anger,
Sexual drive can be hypothalamus adjacent to 3rd ventricle, lateral rage, punishment, fighting)=
stimulated from several hypothalamus & central grey area of Unpleasant sensation (Aversion)
areas of hypothalamus, mesencephalon) are stimulatory and ventromedial
especially the m ost anterior nuclei of hypothalamus, hippocampi, anterior
and m ost posterior lim bic cortex (esp anterior cingulate gyri and
portions o f hypothalamus® subcallosal gyri) are inhibitory to fear (punishment)
(Guyton) Reward centers locatd along the course of m edial Calmness (Placidity, tameness &
M edial preoptic area forebrain bundle® especially in lateral and tranquility) = Pleasant sensation
(MPOA) or the most ventromedial nuclei of hypothalamus are self (Reward or satisfaction)
anterior portion of stimulatory
hypothalamus is most Appetitive Behavior
important integrative Lateral (superior) hypothalamus® Thirst
nucleus involved in sexual Lateral hypothalamus Hunger (eating) & increased
behavior regulation o f activity level
m ales in a ll vertebrates®. It Venteromedial nucleus®, arcuate & paraventricular Satiety (Decreased eating) &
integrates the male sexual nuclei tranquility
behavior by receiving all Anterior ventral hypothalam us; m ost anterior Sexual drive
indirect inputs from every (medial preoptic area=MPOA) & m ost posterior
sensory organ and sends portions o f hypothalamus®; piriform cortex (in males
efferents to structures that only)
are critical for initiation and Neuroendocrinal control of
patterning of copulation. Preoptic area FSH & LH via Gn RH
Therefore, stimulation of Supraoptic & paraventricular nuclei Vasopressin, Oxytocin
MPOA enhances sexual Paraventricular nuclei TSH (thyrotropin) via TRH
activity in males, whereas its ACTH & P- lipotropin (P-LPH) via
damage consistently impairs CRH
male sexual behavior. Paraventricular nucleus and arcuate nucleus GH via somatostatin & GRH
Arcuate nucleus; (hypothalamus inhibits secretion) Prolactin via PIH & PRH
Dorsal & posterior hypothalamus Catecholamines
Physiology: Central Nervous System ■ 239

Role of Hippocampus and Bilateral Molecular Mechanism of Memory


Lesion
Longterm memory
•Hippocampus is the site where short term
It results from actual structural changes instead of only chemical changes
memory is processed and converted into long term
in following ways.
memory ®, by long term potentiation (LTP).
Hippocampus after processing on basis of reward - Protein synthesis & activation o f genes®. So aquisition of long term
or punishment decides which of the short term memory is prevented, if within 5 minutes the animal is anesthetized,
experience is worthy to be stored long term. given electric shock, subjected to hypothermia or given drug that block
protein synthesis. But there is no respoonse after 4 hours.
•Only long term memory of explicit (declarative)
ty p e, which requires symbolic rehersal in mind - Spatial reorganization (restructuring) in synapses® changes their
(verbalization or symbolic types of intelligence) is sensitivity for transmitting nervous signals by increasing vesicle release
formed in hippocampus. Reflexive (implicit or non­ sites, number of transmitter vesicle, number of presynaptic terminals and
declarative) type of long term memory is not changes in structure of dendritic spines that permit transmission of
formed in hippocampus. So people with stronger signals. So a drug that blocks, DNA stimulation of protein
hippocampal lesion replication will prevent long term memory formation.
- Change in num ber of neuron & their connectivity is often significant
r during first few week, months & even years of life. Even adults use this
Do not have difficulty Have antegrade
mechanism to at least some extent. So soon after birth, there is a
in learning physical amnesia i.e., long
principle of " use it or loose it "that governs final number of neurons &
repetitive skill that do term memory
their synapse (i.e., the recruitment of neurons is from the already
not require symbolic prior to accident
existing pool) - Guyton. But according to Ganong (270) new neurons
rehersing in mind such remains intact
form from stem cells through out life in 2 areas - olfactory bulb &
as type writing & car and they are
hippocampus. (So it is relative answer only).
driving (i.e., implicit / unable to
non declerative/ establish im plicit Intermediate memory
reflexive long term (declarative) type
May last for many minutes to even weeks and will eventually be lost unless
memory) o f long term
activated enough to be become long term. It results from temporary
memory® ________
chem ical &rfor physical changes in either presynaptic terminal or p ost
* Once the short term memory is converted to long
synaptic neuronal membrane in two ways
term it is stored in neo cortex (not hippocampus)®.
- Habituation (negative memory) is d/t progressive closure of Ca++
That's why even after removal of b/1
channels
hippocampus, patients still retain long term
- Facilitation (positive memory) is d/t serotonin release at facilitator
memories prior to removal of hippocampus.
synapse, which acts on its receptor on sensory terminal activating
•Bilateral removal of hippocampal formation
adenyl cyclase & forming c-AMP. The c-AMP activates protein kinase
severely and permanently disrupts recent
which phosphorylates K+ channel protein and blocks the K+
memory. Short term and long term memories
conductance out of terminal. This results in prolonged action potential
(for events that occurred prior to injury) are
& prolonged activation of Ca++ channel.
unaffected, but patient suffers anterograde
amnesia and new long term memories cannot be Short term memory
stored. Therefore, patients remember events It last for few seconds to few minutes, but lasting only as long as person
before the injury but fail to recall new events, continues to think about it. It is caused by
even with multiple exposure, and must be - Continual neural activity in a circuit of reverberating neurons.
reintroduced to their therapists repeatedly. There is - Neurotransmitter chem icals sercetedfrom terminal cause presynaptic
loss of declarative memory involving conscious facilitation or inhibition®. (= changes in level o f neurotransm itter at
recall of personal events, words and their meanings synapse).
and general history. However, these patients retain
im plicit (procedural) memory, the ability to Working Memory
acquire problem solving, association and motor - It is a form of short term memory that keeps information available for
skills and can leam some tasks. These patients if very short periods, while person plans action according to it eg
given a complex task to perform (eg mirror remembering a telephone number while picking up phone and dialing
writing), they will not only improve during 1st it.
training session but will also perform better on - It involves central executive (prefrontal cortex) and two rehearsal
subsequent days despite their denial of having systems: a verbal system for retaining verbal memories and a
any experience with task. visuospatial system for retaining visual and spatial aspects of objects.
240 ■ A Complete Review of Short Subjects

Normal Centres for Sleep

Hypothalmus Brainstem
3 important group of neurons play a critical role in regulation of I
sleep Most conspicuous stimulation are for causing almost
natural sleep is the raphe nuclei in lower half of pons &
1 1 1 in medulla. It is serotonin secreting.
Ventro Lateral Pre Tubero- Suprachiasmatic Some areas in nucleus of tractus solitarius can cause
optic (VL PO) area in Mammilary Nucleus (SCN) sleep. This is termination in medulla & pons for viscerl
anterior Neurons 1 sensoy signals entering by vagus & glossopharyngeal
hypothalamus (TMN) in - Located bilaterally nerves.
1 posterior & above the optic
Brainstem nuclei that
- VLPO neurons lateral chiasma are part of the reticular
activating system
contain inhibitory hypothalamus - These nuclei
neurotransmitters 1 receive
GABA & galanin - These information about
- They project to & hypocretinergic light dark cycle
inhibit TMN and & via special
multiple distinct histaminergic retinohypothalmi
arousal or neurons are c fibers ,1Acetylcholine
;j
wakefullness centre critical for - SCN is responsible
---------- -------
located in brain
stem, thalamus &
ascending
arousal
for circadian
rhythm including
P.,.m m « = = = NREMstoep =

hypothalamus
suggesting that
system &
wake fullness
- TMN are
the sleep wake cycle
& secretion of pineal
hormone melatonin,
TActivation
.JL,
VLPO p la y a key ol the thalamus ol the thalamus J
andqprtax I and^prtta i
executive role in inhibited by ACTH & other
sleep regu lation 0 projections pituitary hormone
- VLPO neurons are from VLPO - SCN have 2 peaks n
selectively of circadian
J[
ntz
activated activity. I AQABA \ f fQABA
conincident with
sleep onset
suggesting that it
- SCN interacts with
VLPO & TMN to
regulate sleep in Hypothalamus with
X
-
acts as a sleep
generating nucleus
VLPO serves as a
relation to
circadian rhythm.
circadian and
homeostatic carters
)
A model of how alternating activity of brain stem and
hypothalamic neurons may influence the different
centre for sleep
states of consciousness.
switch' under the
influence of
biological
(circadian) clock or
SCN

Circadian Rhythm
Suprachiasmatic nucleus (SCN) neurons of anterior hypothalamus represent an intrinsic (biological) circadian clock
(modulator) and demonstrate a spontaneous peak of electrical activity at the same time every 24 to 25 hours. The rhythm is
endogenous and persists even in absence of external environmental cues (i.e. neither stops nor becomes irregular).
Although circadian rhythms are intrinsic (endogenous/inbuilt/or self sustained), they become entrained i.e. synchronized
(adjusted) to the day-night light cycle in the environment. The external environmental cues (factors) that can entrain
(control or synchronized the timing of) circadian rhythm are called zeit gebers (i.e. time givers). Day light is the most
important environmental cue. If not entrained, they become progressively more out of phase as most circadian rhythm are
longer or shorter than 24 hours.
Most circadian rhythms with 23-25 hours periodicity can be entrained to 24 hrs by normal environmental light dark cycle
created by earth's rotation such that the periodicity of clock appears to be environmentally controlled. The entrainment
Physiology: Central Nervous System ■ 241

process is dependent on suprachiasmatic nuclei (SCN) located bilaterally above optic chiasma. Neural inputs are generated
from specialized light sensitive retinal cells that are distinct from rods and cones and carry information about light-dark
cycle via a special neural pathway-the retinohypothalamic fibers to SCN. Efferents from SCN entrain all circadian rhythms
including sleep-wake cycle and secretion of pineal hormone melatonin.
The pineal gland forms a neuroendocrine link between SCN and various circadian rhythm processes. It lies close to
hypothalomns and synthesize melatonin (from serotonic/tryptophan) by rate limiting enzyme N acetyl transferase. The
amount and activity of enzyme vary markedly in cyclic fashion (inhibited by light and stimulated by darkness) which
accounts for cycling of melatonin synthesis and plasma levels. So melatonin may transmit the information that night has arrived.
Entrainment can be disrupted by isolating the subject from the environment (i.e. removing all external cues), putting him
in a constant environment (where there are absolutely no temporal cues) or shifting time zones (jet lag). Circadian rhythms
in constant environment with no external cues (or in constant conditions of light or dark) becomes free running and drift
away slightly from a 24 hour cycle to its original intrinsic rhythm (free running period). F o r s le e p w a k e c y c le i t is slig h tly
g r e a te r th an 24 h o u rs (u p to 25 hours)®.
SCN have 2 peaks of circadian activity and exposure to bright light can either delay (if exposed ju st after dark),
advance/accelerates (just before dawn) or have no effect (if exposed during the usual day time) on onset of sleep period.

Cerebral Dominance or Complementary Specialization of Hemisphere

Dominant (Categorical) Hemisphere (Mostly Left) Non-dominant (Representational) Hemisphere (Mostly


• It is concerned with c a te g o r iz a tio n , s y m b o liz a tio n a n d Right)
s e q u en tia l a n a ly t ic processes® . • It is not less developed or non dominant; instead it is
• It is important for specialized in area of visuospatial (or spatio- tem poral
relations®)
1. L an gu age (w ritten , s p o k e n & ex p ressio n ) fu n ction ®
• It is important for________________________________
2. M a th e m e tic a l & s c ie n tific skills®
1. Recognition & interpretation of music (Music
3. Reasoning
awareness)
4. Right handed control (in right handed persons)
2. Non verbal visual experiences (esp visual
5. Hearing (right ear preference), right visual field,
patterns)
stereognosis (right side) and left side olfaction.
3. Spatial relationships between the person & their
- Planum temporale an area of superior temporal gyrus which is
surroundings (Visua-spatial orientation)
located in th e floor o f lateral fissu re is involved in language
4. Significance of body language & intonations of
related auditory processing and correlates with language
peoples voices and probably many somatic
dominance. It is regularly larger on the left side making it
experiences related to use of limbs & hands.
dominant lobe.
5. Identification of objects by their form and
- Lesions in categorical (which is in most of the
recognition of faces (Recognition of forms,
times left) hemisphere leads to_____________________________
faces& body image)
1. Fluent, nonfluent and anomic aphasia
6. Three dimensional awareness (Memory of
2. Acquired dyslexia (i.e. impaired ability to read and learn) shapes)
3. Patients are depressed & disturbed about their disabilities 7. Art awareness®
4. Dyscalculia (impaired ability to perform mathematical 8. Insight & imagination
calculations) 9. Left hand control (in right handed persons)
5. Loss of ability to thin reasonably (logically) through 10. Hearing (left ear preference), left visual field,
problems. stereognosis (left side) and right side olfaction
6. Lesion in dominant parietal cortex only 1/t loss of higher
order somesthesias eg agraphesthesia (inability to • Lesions in representational hemisphere 1/t
identify characters drawn on palm) and astereognosis 1. Agnosia (i.e. inability to recognize objects by a
(inability to identify an object only by touch)_____________ particular sensory modality even though the
- Understanding the spoken & printed words and expressing sensory modality itself is intact).
ideas in speech and writing (i.e. all language functions) 2. Astereognosis (i.e. inability to identify objects by
depend on dominant (categorical) hemisphere. So feeling them)
damaging wernicke's area in dominant hemisphere (mostly 3. Unilateral inattention & neglect
left) leads to lo s s o f a lm o s t a ll in te lle c tu a l fu n c tio n s 4. Visuo- spatial defects
a s s o c ia t e d w ith lan g u a g e o r v e r b a l sy m b o lis m , su ch a s th e 5. Impaired ability to tell a story, make a joke, to
a b ilit y to rea d , th e a b ilit y to p er fo rm m a t h e m a t ic a l get the point of a joke, & more broadly, to
o p e r a tio n s (i.e. d y s c a lc u lia ) a n d ev en th e a b ilit y to th in k comprehend the meaning of differences in color
th rou g h lo g ic a l problem s® . and inflection of speech.________________________
242 ■ A Complete Review of Short Subjects

★ Hemisphere specialization is related to handedness 6. Dysgraphia or agraphia (i.e deficiency in the


(which is genetically determined). In 96% of right handed ability to write, regardless of ability to read
persons (which is 91% of total population), left 7. Cortical neglect d/t lesion in non dominant
hemisphere is dominant and in remaining 4% the right parietal cortex. In this, patient ignores objects and
hemisphere is dominant. individuals on left side, draws object, that are
★ In 70% of left handers, left hemisphere is dominant, in incomplete on left and denies that his left body
15% right and in remaining 15% thre is no clear parts are his and fails to dress the left side of
lateralization. body. He denies that he has any such difficulty
★ Left handers have above average spatial talents; a (anosognosia). He may respond to touch &
significantly disproportionately large number of pinprick on his left hand but cannot identify
musicians, artists, mathematicians are left handed. objects placed on left hand. The lesion is adjacent
However, dyslexia is significantly more common in left to SI & visual association cortex, areas important
handers who also have slightly but significantly shorter for perception of one's body image and
life span. immediate extra personal space_______________

Aphasia
Is a disorder of understanding, thought and word finding

Broca's aphasia W ernicke's aphasia


[Non flu en t/E x p ressiv e/M otor aphasia] [Fluent/Receptive/Sensory aphasia]
______________ i _____________
L esion site® In ferior fro n ta l gyrus® or Broca's area (Area 44) In wemicke's area (supramarginal gysus) of
immediately anterior to primary motor cortex & the parietal lobe & upper part of temporal lobe.
above sylvian fissure.
Cause d/t occlusion of superior branch o f m iddle cerebral d/1 occlusion of inferior division of m iddle
artery® cerebral artery®._________________________
Comprehension P reserved (except grammer)® Impaired®
Fluency Decreased®: Speech is not fluent, laboured, Preserved or increased: Speech fluent but
dysarthric & interrupted with m any w ord fin din g highly paraphasic & associated with
pause®. It is telegraphic but informative. n eologism (Jargon aphasia)®____________
Naming® Impaired Impaired
Repetition® Impaired Impaired
Neologism® Absent Present
Insight Insight into condition is preserved Not preserved

I 1
Conduction Aphasia Anomic Aphasia Type of aphasia & site of lesion Characteristic Naming Errors
I I Nonfluent (Broca's area) "Tssair"
> A form of fluent When a lesion damage Fluent (Wemicke's area) "Stool" or "choss" (neologism)
aphasia in which angular gyrus in the Fluent (areas 40,41, and 42; "Flair . . . no, swair . . . tair"
patients can speak categorical hemisphere conduction aphasia)
relatively well & have without affecting Broch's & Anomic (angular gyms) "I know what it is . . . I have a
good auditory Wemicke's area, there is no lot of them"
comprehension but difficulty with speech or
cannot put parts of undertanding of auditory
words together or information; instead there is
conjure up words trou ble understanding
< It was throught to be w ritten language or picture
d/t lesions of arcuate because visu al in form ation
fascicu lu s connecting is not processed and
Wemicke's & transmitted to Wernicke's
Brocha's areas. It now area.
appears that it is d/t Most commonly seen in
lesion in and around h ea d trauma®, m etab o lic
the auditory cortex en cep h alop ath y and Pnmary visual cortex (area 17)

(areas 40,41 and 42) A lzheim er's disease®. Pathway taken by impulses when a subject names a visual object
projected on a horizontal section of the human brain
Physiology: Central Nervous System ■ 243

Vision

Stereopsis (Depth of Perception)

I
Binocular Components (True- stereopsis) M onocular Components (Pseudo)
I I
Binocular vision has most important role in depth perceptionQ Relative sizes of objects
This is because each eye views the visual world from slightly Degree one looks down at them
different horizontal positions; so image of both eyes is differ Their shadows
from the other. Objects at different distances from the eyes
Relative movements for moving objects
project images in the two eyes that differ in
(movement parallax)
their horizontal positions, giving the depth cue of horizontal
Overlay of contours
disparity also K/a retinal disparity / binocular disparity or
binasal disparity 0.

Charles Wheatstone discovered stereopsis & invented stereoscope.

Visual Pathway from Retina to Visual Cortex

Retinal Ganglion Cells Lateral Geniculate Body - ► Visual Cortex

These are of two kinds Each Geniculate body contains six well • Each visual cortex has 6
defined layers layers
M cells P cells Six layers divide Lateral geniculate body • Visual cortex receives
(Magno cells) (Parvo cells) into three portions afferents from
Large ganglion Sm all ganglion cells - Magnocellular
cells • Project to the r pathway
Project to the • Parvocellular portion of M agnocellular Parvocellular Interlam inar - Parvocellular
• Magnocellular lateral Geniculate body portion portion portion pathway
portion of • Interlaminar portion of (Layers 1 ,2) Layers 3,4,5, (In between - Interlaminar portion
lateral Lateral Geniculate body 6) layers) • These pathways mainly
Geniculate Receives Receives Receives terminate in Layer 4
body projections projection projection but also in layer 2 & 3
from M cells from 'P' cells from 'P ' cells
Carries signals Caries Caries signals r
O ptic for detection of signals for concerned Magnocellular Interlaminar
chiasm movement depth color vision, with color Parvocellular pathway
L a te ral geniculate and flicker texture vision Layer 4 Layer 2, 3
F rom retina of
From relina of
opposite side
shape and
s a m e side
Dorsal fine detail Blobs & Visual Pathway

Lateral geniculate body (part of thalamus) Layers 2 and 3 of the


is relay station on the visual pathway cortex contain clusters
Ventral (y 2 ,3;4 lying just posterior to the optic tract. It of cells about 0.2 mm in
M a g n o c e llu la r /P a r v o c e llu la r receives fibres from the contralateral nasal diameter, that unlike
p a th w ay / — .p a th w a y hemiretina and ipsilateral temporal neighbouring cells
hemiretina. Gray matter of this body is contain a high
Inferior
P rim a ry visual cortex (a re a 17) split into six laminae. concentration of the
mitchcondrial enzyme
'cytochrome oxidase'.
244 ■ A Complete Review of Short Subjects

Contralateral nasal Ipsilateral temporal


hemiretina hemiretina These clusters have
been named as blobs
Fibres from Fibres from
They are arranged in a
ipsilateral temporal contralateral nasal
m osaic in the visual
hemiretina hemiretina
i | cortex and are
concerned with color
End in lamina End in lamina vision.
2,3,5 ® 1,4,6®.
>The axons from Lateral geniculate nucleus
that form the magnocellular pathw ay end
in layer four (4). Many o f the axons that
Layer 1,2 Layer 3,4,5 and 6
form the parvocellular pathw ay also end
Magnocellular Parvocellular in Layer 4. M ost o f the axons from Lateral
pathway pathway geniculate nucleus thus end in Layer 4 of
l
Signals for detection
I
Signals for clour version,
the visual cortex. Some fibres from Lateral
geniculate body terminate via the inter laminar
of movement, depth texture shape adn fine
and flicker to visual details to visual cortex
pathway. These terminate onto layers '2' & '3'
cortex of visual cortex.
G a n g lio n c e lls ______^ M (M a g n o cells) P (P a rv o cells)
(M c e lls & P cells)

1 1
L a tera l G e n ic u la te b o d y M a g n o c e llu la r L am in as In te rla m in a r R e g io n • < -------- P a rv o c e llu la r L am in as
( 2 R e g io n s) ______ (L a m in a 1, 2)

V isu a l C o rtex
i
Layer 4 L a y e rs 2 , 3
I
L ayer 4
(S u p e rfic ia l la y e r 4c) (B L O B S ) (D eep L a y e r 4 c)

F u n ctio n M ovem ent C o lo u r Shape


L o ca tio n C o lo u r

Spatial Organization Flow of Visual Information of Color Vision

Cells of Retina
Amacrine Cells Muller Cells/ Retinal Ganglion Cells
______Astrocyte______
Transmits signals from rod • Type of glial cells help in Multipolar cells provide connection from bipolar neurons &
bipolars to ganglionic cell & is binding together of neural amacrine cells to neurons in CNS.
therefore essential elements Fires continuously to some degree with or without light
component of rod pathway • Neuroltransmitter uptake The only retinal neurons that always transmit visual signals
Modulates photoreceptive & Remove debris by means of (all or none) action potential. They are of 3
signals, serving to adjust or • Electrical-insulation of types.
maintain the relative colour receptor & neurons Small (< lOfrn) diameter slow velocity (8 m/sec) - W cells
& luminosity inputs under • Store glycogen constitute about 40% of all ganglionic cells and receive most
changing light conditions® • Do not take part in of their transmission from rods. Their dendrites spread widely
viz neurosensory pathway of in inner plexiform, layer, receiving signals from broad fields
- at different times of day light in peripheral rtina W cells are sensitive for detecting directional
- directional movement • Retinal neurons conduct movements in field of vision & are important for much of
detection.(directional their visual signals by crude rod vision under dark conditions.
sensitive) electrotonic graded Most numerous (55%), medium diameter (10-15pm)
Increases visual acuity by conduction (i.e. direct medium velocity (14m/sec) X cells transmitting fine details
lateral inhibition® i.e. inhibit flow electric current not of visual image and colour vision. They have small fields and
firing of cells lateral to main AP). Thus degree of their signals represent discrete retinal location.
neuronal pathway. hyperpolarization is Largest (35 pm) diameter, fastest velocity (50m/sec), least
However, lateral inhibition to directly related to numerous (5%) Y cells. They have broad fields and respond,
enhance visual contract to intensity of illumination like many amacrine cells, to instantaneous (rapid) change in
allow high visual accuracy is visual image- either rapid movement or rapid change in light
primarily a function of intensity sending bursts of signals for only small fractions of
horizontal cells. second.
Physiology: Central Nervous System ■ 245

Visual Cycle
The photosensitive pigm ent in the rods is called rhodopsin or visual purple. It has a peak sensitivity to light at 505nm.
Rhodopsin consists of 11-Cis retinal specifically bound to protein opsin.
Regeneration of rhodopsin requires isom erization of all trans retinal back to 11-Cis retinal.
Trans retinal after being released from rhodopsin is isomerized to 11-Cis retinal which spontaneously com bines with opsin
to form rhodopsin thus com pleting the cycle.
Sim ilar reactions are responsible for color vision in the cone cells
r Forms of vitamin A
I_______
Retinol Retinal Retinoic acid A ll tr a n s re tin o l
A p rim a ry • It is an a ld e h y d e A cid d e riv e d from I
a lco h o l • D e riv e d fro m o x id a tio n o f o x id a tio n o f re tin al. A ll tr a n s r e tin a l
co n ta in in g a b io n a lco h o l. I
o n e rin g w ith an • E x ist in tw o fo rm 11 - C is r e tin a l^
u n sa tu ra ted sid e 1) 1 1 -C is re tin a l
ch ain . -<-------- Opsin
2) 1 1 -tra n s re tin a l
R h o d o p s in

Light -> Opsin


A ll tr a n s r e tin a l

Visual Cycle & Generation of Visual Impulse A ll - tr a n s -re tin o l < -

-In pigment epithelium o f r e tin a , all- trans-retinol is i s o m e r iz e d to 11-Cis- retinol | Iso m e riz a tio n
a n d o x id iz e d to 11- Cis-retinal dehyde. In th e r e tin a th is r e t n a ld e h y d e ( l i ­
In
d s - r e tin a l) r e a c t s w it h (& f u n c tio n s a s th e p r o s th e tic g r o u p o f lysine residue pigm ent < 1 1 - C is - R e tin o l
in th e light sensitive opsin proteins forming h o lo p r o te in rhodopsin (in r o d s ) epithelium
o x id iz a tio n
a n d iodopsin (in c o n e s ).
- T h e p h o to s e n s it iv e p ig m e n t in th e r o d s is c a lle d rhodopsin o r visual purple. It
1 1 - C is - R e tin a ld e h y d e (11 c is r e tin a l) <
h a s a p e a k s e n s itiv it y to lig h t a t 5 0 5 n m . R h o d o p s in c o n s is t s o f 11-Cis retinal
s p e c ific a lly b o u n d to p r o t e in o p s in . A n y o n e c o n e C e ll c o n t a in s o n ly o n e
C o n se n sa tio n L y s in e re sid u e of
ty p e o f o p s in & is s e n s itiv e to o n ly o n e c o lo u r .
p h o to s e n s itiv e
-T h e key to in it ia tio n o f v is u a l c y c le is th e availability of 11-Cis- retinaldehyde o p s in p ro te in
(vitamin A aldehyde) a n d h e n c e v it a m in A w h ic h c o n d e n s e s w ith o p s in to
fo r m r h o d o p s in . S o in v it a m in A d e f ic ie n c y , b o th th e tim e ta k e n to a d a p t to R h o d o p s in o r V is u a l p u rp le (i.e. 1 1-C is re tin a l +
d a r k n e s s & a b ilit y to s e e in p o o r lig h t a r e im p a ir e d . o p sin p ro tein )_________________________________
- W hen rhodopsin is ex p osed to light, the a bsorp tion o f light by rhodopsin
(visu al purple) cau ses p h o to ch em ica l isom erization o f retin ald eh y d e fro m | L ig h t
11 cis retin al to a ll trans r e tin a l , an d a co n fo rm a tion al change in op sin 0.
P h o to rh o d o p s in
T h is p r o c e s s fin a lly r e s u lts in r e le a s e o f a ll tr a n s r e tin a ld e h y d e & o p s in b y
hydrolysis a n d initiation of a nerve impulse th a t is tr a n s m itte d b y o p tic | 4 5 p ic o se c
n e r v e to b r a in .
B a th o r h o d o p s in
- T h e fo r m a t io n o f in itia l e x c ite d fo r m o f r h o d o p s in , b a th o r h o d o p s in o c c u r s P h otoiso m erizatio
w it h in p ic o s e c o n d s o f illu m in a t io n . T h e n th e r e is a s e r ie s o f c o n f o r m a tio n a l | 3 0 n sec n & h ydrolysis o f
visual p u rp le to
c h a n g e s le a d n g to th e formation of metarhodopsin II, w h ic h in it ia te s a L u m ir h o d o p s in
g en erate visual
g u a n in e n u c le o tid e a m p lific a t io n c a s c a d e le a d in g to a lte r e d Na-i- c h a n n e l
| 75p sec im pulse
p e r m e a b ilit y , m e m b r a n e p o la r iz a t io n a n d th e n nerve impulse. T h e fin a l s te p
is h y d r o ly s is to r e le a s e a ll tr a n r e tin a ld e h y d e & o p s in . M e ta r h o d o p s in I
5' G M P CGM P
10 m sec
N a - c h a n n e l c lo sed N a* c h a n n e l o p en

M e ta r h o d o p s in II

M em b ran e H yp er | m in u tes
P o la r iz a tio n
M e ta r h o d o p s in I I I

H y d ro ly sis
G e n e r a tio n A c tiv e
o f n e rv e P h o s p h o d ie s te r a s e In a c tiv e A ll - tra n s - R e tin a l + o p s in
im p u ls e
I-----------
Transducin GTP GDP

Transducin- GDP GTP


246 ■ A Complete Review of Short Subjects

Differences between Rods & Cones

Rods & cones differ in number®, structure, retinal distribution, photochemical molecules, sen sitivity, w av e length, acuity®,
color, synaptic connections & functions.

Feature Rods Cones


Number 90 -1 0 0 millions 3 - 4.5 millions
Density Higher than cones in most of retina except Lower in peripheral retina but increases to
fovea & foveola almost 200 times in fovea. And foveola
(central 300(J.m of fovea) is totally rod free.
Convergence Higher i.e. many rods are connected to one Less convergence
bipolar cells
Visual acuity Low High
Spatial resolution Low High
Vision In dim light (Scotopic) In bright light (Photopic)
Sensitivity (to weak light) High Low
Morphology Outer segment of rods contain ~ 1000- 2000 Cell membrane is invaginated to form
intracellular discs to hold photopigment saccules but have no separate intracellular
discs
Photosenstitive pigment (PP) Rhodopsin (Visual purple) & present in Iodopsin & present in outer membrane of
disc cell
Amount of PP & captured More Less
photons of light
Amplification More (i.e. more C GMP gets hydrolysed Less
for each absorbed photon of light)
Slow response & long integration time Fast response & short integration time
Opsin molecule Only one type achromatic scotopsin that Chromatic, three types of pigment that
has peak sensitivity to light a t a w av e respond maximally to light at w av e
length o f 505 nm® lengths o f 440,535 and 565 nm®
Responsible for colour vision

Colour Vision
It is the capacity of an organism to Theories
distinguish objects b ased on the w ave I_____
lengths (or frequencies)® they reflect or
Young- Helmholtz trichromatic theory of Hering's Opponent
emit.
retinal handling of colors Colour Coding
However, the colour perceived does not I (Processing) theory
depend on wave length alone; as the same It states that colour vision is possible due to I
wave length can change its apparent 3 types of cones, each containing different - It is called into play
color if its intensity is increased or if the photosensitive pigment maximally sensitive during transmission
colour of illuminating light is changed. to 3 primary colors. through visual pathway.
Color perceived also depends on color of The sensation of any color is determined by the Ewald hering proposed
other objects in visual field. Eg. red relative ratio of signals arising from each type of that bipolar cells, ganglion
object is seen as red in green or blue light cone. cells & LGB cells are
but as pale pink or white if the field is Cone type Name Absorb Peak Range responsible for
illuminated with red light. maximal (nm) (nm) phenomenon of
Scotopic (dark) vision by rods is mot or sense opponent colour coding.
sensitive to bluegreen light (440-570nm) Short wave P Blue 440 440-500 - This theory states that
and insensitive to red light; whereas (S) or Blue violet cond^hoto receptors are
phototopic (day light) vision by cones sensitive linked together to form 3
covers the whole range of visible Medium 7 Green 535 450-545 opposing color pairs:
spectrum (380-800 nm) with maximum wave (M) or
blue/yellow, red/ green and
sensitivity in yellow (570-590 nm) with Green
black /white
sensitive
maximum sensitivity in yellow (570- 590 - Activation of one
Long wave P Yellow 565 564-580
nm) region. member of pair inhibits
(L) or Red but also
This shift of maximum sensitivity to longer activity in other. So no
sensitive sense red
Physiology: Central Nervous System ■ 247

wave lengths, which accompanies the This theory is proved by absorption spectra two members of a pair can
change from scotopic to photopic vision of retinal cones using retinal densitometry be seen at the same location.
is k/a purkinje shift. However, it cannot explain phenomenon of That's why we don't
Colours have 3 attributes: hue, intensity complementary after images, in which the experience bluish- yellow
& saturation ("HfS")Q. saturation is extended inspection of one color will 1/t the or reddish green where
degree of freedom from dilution with subsequent perception of its complementary as bluish green
white. Complementary color of any color. For example if one looks at a unique (turquoise) and yellowish
color is the color that when properly red patch for about a minute & then red (orange) are very
mixed with it produces white color. switches the gaze to a homogenous white commonly seen.
Black colour produced by absence of area they will see a greenish patch in white This antagonism between
light is a positive sensation because blind arch. colors occurred in retina as
eye does not see black; rather it sees spectrally opponent
nothing processing (red Vs green
Red (723-647nm), green (575-492) and blue and (yellow vs. blue) and
(492-450 nm) are called primary colors spectrally nonopponent
because mixing of these in various processing (black vs
proportions can produce, white, any white).
spectral & even extra spectral color Mechanism: one colour
purple. cone (eg red) excites the
Each retina has 100 million rods, 3 million ganglion cell by direct
cones & only 1.6 million ganglionic cells. excitatory route through
Thus 60 rods & 2 cones converge on each a depolarizing bipolar
ganglionic cell & optic nerve fiber leading cell whereas, the
from it to brain. In central retina (fovea) opponent color type cone
fewer rods & cones converge on each (eg green) inhibits the
fiber progressively increasing the acuity of ganglion cell by indirect
vision. Central fovea (i.e. foveola) has 35,000 inhibitory route through
slender cones and no rods converging on a hyperpolarizing bipolar
almost exactly equal number of nerve cells.
fibers. 1/t m axim um co lou r vision Retina itself begins to
acu ity Q. differentiate colors by
Peripheral retina has much greater these color contrast
sensitivity to weak light because rods are mechanismQ. There fore
30-300 times more sensitive to light than color analysis begins in
cones and ~ 200 rods converge on a retina & is not entirely a
single optic nerve fiber in peripheral function of brain.
portions of retina. This theory explains
Lateral inhibition (by horizontal & phenomenon of
amacrine cells) provide contrast complimentary after images
detection & enhancement. and why a gray square
on red background looks
green.

Basal Ganglia or Basal Nuclei T T

Structural Organization M ajor Connections o f Basal Ganglia

Basal ganglia is composed of 5 interactive structures on each side Afferents: 2 main inputs to basal ganglia, both of
of brain including__________________________________________ which are excitatory (glutamate) terminate in
- Caudate nucleus, putamen and globus pallidus (= 3 large striatum. They originate from layer V of most
nuclear masses underlying the cortical mantle) and regions of cortex (except primary visual & auditory
- Subthalamic nucleus of diencephalon and substantia cortices) with an important component originating
nigra of midbrain_______________________________________ in motor cortex (corticostriatal pathway) and from
The term striatum (or corpus striatum) is applied to caudate intralam inar nuclei of thalamus (thalamostriatal
nucleus & putamen because of the striated appearance of these pathway).
nuclei. The striations are produced by fiber bundles formed by the Efferents : 2 main outputs of basal ganglia, both of
248 ■ A Complete Review of Short Subjects

a n te r io r lim b o f in te rn a l c a p s u le as it s e p e ra te s the c a u d a te n u cle u s which are inhibitory (GABA ergic); arise from GPi
a n d p u ta m e n and S N pr and project to thalamus. Thalamic neuron
• The putamen and globus pallidus collectively form lenticular inturn give excitatory (glutamate?) projections to
nucleus. prefrontal and premotor cortex thereby, completing a ful
• Globus pallidus h a s tw o p a rts GPi an d GPe. Substantia nigra cortical - basal ganglia-thalamic - cortical loop.
(s u b s ta n c e b lack d / t m e la n in ) h a s 2 p a rts SNpc a n d S N pr Connections within basal ganglia include
N eurotransmitters

• Globus pallidus has 2 parts, an internal and an external segments


(GPi & GPe); both containing inhibitory GABA ergic neuron
• Substantia nigra (= black substance) derives its name from its
content of melanin pigment. It is divided into pars compacta which
uses dopamine neurotransmitter (& melanin; a by product of
dopamine synthesis) and pars reticulate ( S N pr ) which uses GABA
as neurotransmitter.
• Striatum contains at least 4 types of neurons: 95% are medium
spiny neurons using GABA as neurotransmitter. Remaining 5%
are all Aspiny intemeurons differing in size and neurotransmitters
: small (GABA), medium (somatostatin) and large (acetyl
choline).
• Several thalamic nuclei including ventral anterior (VA) and
ventral lateral (VL) nuclei and intralaminar complex are
associated with basal ganglia.
• GABA is always inhibitory neurotransmitter forming negative feed
back loops. Dopamine is mostly inhibitory and sometimes stabilizer.
Glutamate is always excitatory balancing out inhibitory signals
from GABA, dopamine & serotonin inhibitory transmitters.
• Dopamine neurotransmitter of SNpc in nigrostriatal pathway has
an excitatory action on direct pathway has an excitatory action on direct
pathway & an inhibitory action on indirect pathway. This difference in
action of dopamine (modulatory effect) is d/t different types of
dopamine receptors expressed by spiny striatal cells. - ie dopamine is
causing its action by altering the striatal cells response to other
transmitters (& not by generating post synaptic potentials).
Dopamine is a neuromodulator that acts on D l and D2 receptors
on striatal neuron cells participating in direct and indirect
pathways (respectively) by projecting to GPi and GPe respectively
• On the basis of NTs, it can be divided into striosome
Direct & Indirect Pathways (to which limbic system projects) and matrix (to
• Excitatory cortical input to striatum influences output from GPi which cortical projections related to motor control
and S N pr via a direct and indirect pathway. end). Striosome synapse in SNpc to influence
• The overall effect of direct pathway is to enhance motor activity, dopaminergic nigrostriatal pathway.
whereas indirect pathway reduces the activity of motor neurons in
cerebral cortex. • Striatal neurons disoharge before movement occurs
suggesting their role in selecting the probable
movements. Putamen is related to body movement
and caudate nucleus to eye movement.

Eye Movements Limb Movements


Physiology: Central Nervous System ■ 249

Control of Voluntary Movements____________


1 1
Corticobulbar /corticospinal Basal Ganglia Cerebellum
system
•Tracts : • The term basal ganglia applied Functional division
- Corticobulbar: The nerve fibers • to 5 structures on each side of • Vestibulo cerebellum: (flocculonodular
from the motor cortex to cranial brain: lobe)®
nerve nuclei. Caudate nucleus 1 - OldestQ part of cerebellum
_ fStriatum
- Corticospinal: 80% lateral enticuiar r Putamen J - Concerned with equilibrium & learning
corticospinal (that cross the midline mcieus L Globus pallidus induced changes in VOR (vestibulo occulo-
in medulla) + 20% Anterior/ Subthalamic nucleus reflex)
ventral corticospinal tract. Substantia nigra • Spinocerebellum : Receive proprioceptive
• Cortical motor areas • Function: of basal gangliaQ. input from all body as well as copy of 'motor
- Motro Cortex (area4): Where - In planning & programming plan' from motor cortex, By comparing plan
stimulation produces prompt of movementsQ. with performance it sm oothes & coordinates
discrete movement - Convert abstract thoughts into ongoing movements®
- Supplementary motor area: involve voluntary action^. • Neocerebellum: They interact with the
in programming motor sequences motor cortex in planning & programming
- Premotor cortex (area 6): Concerned movement.
with setting posture at the start of PLAN EXECUTE
planned movement.
Basal ganglia
- Posterior parietal cortex: Motor
responses are also produced by /
sal
Cortical
stimulation of somatosensory 1 & II in Idea- ation ■
association Prem otor and Movement
is
areas m otor cortex
parietal lobe
• The corticospinal & corticobulbar
system is the primary pathw ay fo r
\ Lateral
cerebellum v Interm ediate
cerebellum
the initiation o f skilled voluntary
movement Q. Control of voluntary movement

Blood Brain Barrier (BBB) & Blood Cerebrospinal fluid Barrier (BCSFB)
BBB & B-CSF-B exist between the blood & brain interstitial fluid and CSF respectively to uniquely limit the exchange of
substances into the brain. That is why the constituents of CSF are not the same as in ECFs (extracellular fluids) elsewhere in
body.
These barriers exist both at the choroid plexus [between choroid epithelium (CSF) and capillary endothelium (blood)] and at
tissue capillary membranes. The cause of low permeability is presence of light junctions b/w the adjacent endothelial cells.
Because of the sensitivity of cortical neurons to even minor variations of ionic (K+, Ca++, Mg++, H+ etc) concentration changes,
BBB is an additional defence evolved to protect & maintain the constancy of environment of neurons in CNS. It also protects brain
from endogenous & exogenous toxins in the blood & prevents escape of neurotransmitters in general circulation. BBB is immature at
birth and is breached (broken down) in areas of injury, infection and tumors. Tumors develop new capillary that lack light
junction and contact with astrocyte - so the radioactive iodine labeled albumin penetrates tumor rapidly, making it stand out
as an island of radioactivity in the surrounding normal brain. BBB is temporarily disrupted by sudden marked increase in BP
or by IV injection of hypertonc fluids.
Blood-CSF Barrier & Blood-Brain Barriers are highly perm eable to water, CO 2 (carbondioxide), O2 (oxygenh, and m ost lipid
soluble (lipophilic) substances such as free form s o f steroid hormones, alcohol and anesthetics ®; slightly permeable to
electrolytes like Na+ (sodium), K+ (potassium) and Ch (chloride); and almost totally impermeable to plasma proteins, protein
bound forms of steroid hormones, protein antibodies and most non lipid soluble large organic molecules (i.e. impermeable to,
in general, all proteins and polypeptides )®. It slows penetration of some smaller molecules like urea.
Passive diffusion across cerebral capillaries with tight junction is very limited but numerous carrier-mediated & active transport
system move substances out (more freely) as well as into the brain. Many peptide & drugs actually cross the cerebral
capillaries but are transported back rapidly into blood by a multidrug nonspecific, p-glycoprotein - ATP binding
transporter in apical membranes of endothelial cells, that transports various proteins & lipids across cell membranes. So
larger (adequate) amounts of chemotherapeutic drugs, analgesics & opioid can be delivered to brain by inhibiting this
transporter.
250 ■ A Complete Review of Short Subjects

Transporter exist for glucose, thyroid hormone, leptin hormone (control appetite & sympathetic activity) choline, several organic acids,
nucleic acid precursors and basic, neutral and acidic aminoacids at BBB. Diffusion of glucose (the ultimate source of energy for
nerve cells) across BBB would be very slow in absence of glucose transporters like GLUT 1. Two isoforms GLUT 1 (55 K) of
brain capillaries/endothelium and GLUT 1 (45 K) of astroglia, neuron (+ GLUT-3) & microglia (+ GLUT-5) are encoded by a
same gene, but they differ in the extent of glycosylation. Infants with congenital low GLUT1 develop low CSF glucose
concentrations in the presence of normal plasma glucose 1/1 seizures and delayed development.

Circumventricular Organs

They have fenestrated capillaries & because of their permeability they are said to be 'outside - Althoug pineal gland
the blood-brain barrier'. Some of them function as neurohemal organ i.e. area in which and anterior pituitary
polypeptides secreted by neurons enter the circulation. Sites of brain outside the blood brain have fenestrated
barrier i.e. ‘Circum ventricular organ'® include: capillaries and are
outside the BBB, but
M - Median eminence o f hypothalamus® both are endocrinal
S - Subfornical organ (SFO)® glands and are not
o - Organum Vasculosum of lamina Terminalis (OVLT, supraoptic crest) part of brain
A - Area postrema® - Subcommissural
P - Posterior pituitary® (Neurohypolysis)_____________________________ organ closely a/w
[Mn: Marble SOAP] pineal gland &
histologically
- The ease of diffusion (d/t presence of fenestrated capillaries) allow protein bound dyes to
resembling the
stain these areas and function them as chemoreceptor trigger zones (CTZ) in which
circumventricular
substances in the circulating blood can trigger changes in brain function without penetrating
organs does not have
the BBB. Eg area postrema is a CTZ that initiates vomiting in response to chemical changes
fenestrated capillaries
in plasma and increase BP in response to circulating angiotensin II (A II)°. A II also acts on
and high
SFO & OVLT to increase water intake. OVLT has osmoreceptor controlling vasopressin
permeability.
secretion and produce fever in response to circulating interleukin-1 (IL-1).

Cerebellum

3 Layers 4-Deep Cerebellar nuclei


It has a 3 layers - external molecular layer, Purkinje cell layer (that is only Deep cerebellar nuclei separate external
one cell thick) and internal granular layer.____________________________ cerebellar cortex from white matter. There are 4
5-Types o f Neuron Cells deep nuclei: - dentate, emboliform, fastigial &
Cerebellar cortex contains 5 types of neurons - Purkinje, granule, globose0 (Mn: 'DEFG')
basket, stellate and Golgi cells®. The globose & emboliform nuclei are sometimes
Purkinje cells are amongst the biggest neuron in the body have extensive lumped together as interpositus nucleus.
dendrites that extends throughout molecular layer. Their axons are the Afferents
only output from the cerebellar cortex and these generally pass to deep
nuclei®. Its primary afferent inputs, the mossy &
Granule cells are excitatory®. Granule cells receive input from mossy climbing fibres®, send collateral to deep nuclei
fibres & innervate Purkinje cells. Their cell body lie in granular layer & pass to the cortex.
and axon on reaching molecular layer bifurcates to form T ' which are Two main inputs (afferents) to cerebellar
called parallel fibres. (The parallel fibres synapse with many Purkinje cortex- climbing fibres & m ossy fibres are
dendrites) excitatory.
The other 3 types of neurons are inhibitory interneurons. The basket cells Clim bing fibres come from inferior olivary
are located in molecular layer. They receive input from parallel fibres & nuc/ei'Q(propioceptive input to i.o.n. comes from
each project to many Purkinje cells. Their axons form a basket around all over body). Each projects to primary dendrites
the cell body & axon hillock of each Purkinje cell they innervate. of a Purkinje cell, around which it entwines like a
Stellate cells are similar to basket cells but more superficial in location. climbing plant.
Golgi cells are located in granular layer & their dendrite project into the Mossy fibres provide direct propioceptive
molecular layer, receive input from parallel fibres. Their cell bodies input from all parts of the body plus input from
receive input via collaterals from the incoming mossy fibres & Purkinje the cerebral cortex via pontine nucleus to the
fibres. Their axons project to the dendrites of the granule cell. cerebellar cortex. They end on the dendrites of
Physiology: Central Nervous System ■ 251

granule cells in complex synaptic groupings


Transmitters
called glomeruli. The glomeruli also contain the
GABA is secreted by the stellate, basket, golgi and Purkinje cells (GABAa inhibitory endings of Golgi cell.
receptor)
Efferents
Glutamate is secreted by granule cell (only neuron of CNS that has
GABA a receptor containing a-6 subunit). • The output is through purkinje cells and deep
cerebellar nuclei. The output of purkinje cell is
Fundamental Circuit: Summary inturn inhibitory to deep cerebellar nuclei. It is
Clim bing fiber inputs exerts a strong excitatory effect on single Purkinje interesting that the output o f deep cerebellar
cells, whereas Mossy fiber inputs exerts a weak excitatory effect on many nuclei to the thalamus and brain stem is
Purkinje cells via granule cells. alw ays excitatory®
Granule cells via their parallel fibres also excite basket & stellate cells Thus alm ost all the cerebellar circuitry seems to
which inturn inhibit Purkinje cell (Feed-forward inhibition) be concerned solely with modulating or timing
Golgi cells are excited by Mossy fiber collaterals, Purkinje cell the excitatory output o f the deep cerebellar
collaterals & parallel fibres and they inhibit transmission from mossy nuclei to the brain stem & thalmus®
fibres to granule cells • Most of the vestibulocerebellar output (i.e. from
Purkinje cell out put inhibit deep cerebellar nuclei. Nuclei also receive flocculo-nodular lobe) passes directly to the
excitatory inputs via collaterals from mossy & climbing fibres.
brainstem®, but the rest of the cerebellar cortex
projects to the deep nuclei which inturn project
The output from deep cerebellar nuclei to brainstem & thalamus is
always excitatory. Thus almost all cerebellar circuit seems to be to the brainstem. Thus the deep nuclei provide
the only output for the spinocerebellum &
concerned solely with modulating or timing the excitatory out.
neocerebellum.
- Medial portion of spinocerebellar projects to
Lesion leads to
fastigial nuclei & from there to brainstem.
Incordination® of rate, range, force and direction of movements I/t - Ajdacent hemisphere portion of
ataxia, asynergia, astasia, dysmetria, dysarthria, scaning speech, past
spinocerebellum projects to emboliform &
pointing, rebound phenomenon, dysdiadokinesia and nystagmus.
globose nuclei and from there to the brainstem.
Intentional tremors® :unlike the resting tremors of parkinsonism it is - Neocerebelum projects to dentate nucleus &
absent at rest from there to ventrolateral nucleus of thalamus.
Disorder of equilibrium gait & posture
Hypotonia®

The Orexlns (Hypocretlns) System Ghrelin


Orexins were first described in 1998 as a result of a - It is a gastrointestinal hormone synthesized & released mainly by
search for an unknown regulatory peptides in oxyntic cells of stomach (Guyton) / P or D l cells of fundus of
hypothalamus. These are signal substances used both in stomach. It is also produced by the intestine & epsilon cells of
CNS & periphery. pancreas in small amounts.
- Ghrelin acts to stim ulate hunger (appetite)®. So blood levels rise
In the CNS In the Periphery
during fasting, peak just before eating and then fall rapidly after a
• All the orexinergic Orexins and orexin
meal (orexigenic). In this way, it is counter part of hormone leptin,
neurons have their receptors have been
produced by adipose tissue, which induces satiation (i.e. is anti-
origin in the lateral have been found in the
orexigenic) when present at higher levels.
hypothalamus from gastro-intestinal tract
- Ghrelin is also produced in arcuate nucleus of hypothalamus
where they project and in the endocrine
and has marked growth hormone stimulating activity i.e. it stimulates
widely. organs.
GH secretion from anterior pituitary gland by acting directly on
• They are believed to The prominent
receptors in pituitary.
regulate: peripheral effects seen
- G protein coupled ghrelin receptor formerly k/a growth
- Wakefulness and so far include regulation
hormone secretagogue (GHS) receptor is expressed in arcuate
paradoxical sleep, of gastro-intestinal
nucleus, lateral hypothalamus, vagal afferent cell bodies & vagal afferent
- Appetite and food intake motility and hormone
endings through out gastrointestinal tract.
- Endocrine and production and release,
- It has an important role in neurotrophy, particularly in
Autonomic processes. especially in the adrenal
hippocampus and is essential for cognitive adaptation to
gland
changing environment and process of learning.
Orexin (hypocretin) producing neurons occur in
hypothalamus. Orexins A and B increase feeding ★ G ro w th h o rm o n e (GFI) also k / a so m a to tro p ic h o rm o n e or
(appetite) whereas, brains from humans with s o m a to tro p in exerts m u ch of its effects throu gh in term ed iate
252 ■ A Complete Review of Short Subjects

narcolepsy have fewer orexin producing neurons in substances called somatomedins secreted by liver (and to a much less
hypothalamus. The orexins most often act in an extent, other tissues). Because many effects of somatomedins on growth
excitatory manner both via putative pre-, post- and are similar to those of insulin, therefore somatomedins are also called
extrasynaptic mechanisms. insulin like growth factors (IGFs). Out of 4 somatomedins isolated, the
most important is somatomedin C (or IGF-l) and pygmies of Africa &
The orexin system consists of
Levi-Lorain dwarf have congenital inability to synthesize significant
- Orexin - A and Orexin B, two closely related
amount of IGF-l.
neuropeptides, derived from the same gene (common ★ Cholecystokinin (CCK) is secreted by I cells of upper small intestine
precursor) by alternate splicing. (jejunum) and decreases feeding (antiorexigenic / aorexigenic), inhibits
- OX-1 and OX-2, the two orexin receptors. At most of gastric emptying, and stimulates secretion of pancreatic juice rich in
the projection sites both 0X 1 & 0 X 2 receptors are enzymes, contraction of gall bladder, secretion of glucagon & insulin
expressed. Mutation in one of the orexin receptor and small intestine & colonic motality.
genes cause narcolepsy.

Neurotransmitter

Neuropeptide - Y (NP-Y) Neuropeptide (Slowly Acting Transmitters


Neuropeptide Y (found in brain & autonomic nervous system), pancreatic /Growth Factors)
polypeptide (produced by F cells in islets) and polypeptide YY (a • Neuropeptides (unlike small molecule,
gastrointestinal hormone) - are closely related as they all contain 36 rapidly acting transmitters) are not
am inoacids®, end in tyrosine, are amidated at their carboxy terminals and all act on synthesized in the cytosol of presynaptic
at least 2 of the 4 known G protein coupled receptors for these peptides : Y l, terminals. Instead, they are produced in
Y2, Y4 and Y5. However, neuropeptide Y is secreted only by neurons, neuronal cell body by ribosomes as a part of
whereas pancreatic polypeptide & polypeptide YY are secreted by endocrine large protein molecules, which enters ER
cells. and then golgi apparatus. In GA, the
NPY is secreted by (adrenergic, noradrenergic & other) neurons of brain neuropeptide forming protein is
(hypothalamus, periaqueductal gray, medulla) and autonomic nervous system. enzymatically split into smaller fragments
(some of which are either neuropeptide or
Neurons that produce orexigenic substances NPY and AGRP, have their cell
its precursor) and packaged into minute
bodies in arcuate nuclei and project their axons to paraventricular nuclei (of
transmitter vesicles (TV) which are released
hypothalamus) to antagonize melanocortin receptors (MCR-3 & MCR-4) there
into cytoplasm. These TVs are transported
by inhibiting the effects of a-MSH produced by POMC neurons. When energy
by axonal streaming of cytoplasm at a slow
stores of body are low (eg starvation), orexigenic neurons are activated to rate (few cm/day) to tips of nerve fibers,
release NPY, which stimulates appetite and at the same time, firing of POMC where they release neurotransmitters in
neurons is decreased thereby reducing the activity melanocortin pathway, so response to action potentials in the same
further stimulating appetite. (Guyton -870). When injected into manner as for small molecule, transmitter
hypothalamus, NPY increases food intake, and inhibitors of NPY synthesis (SMT). However, these TVs are autolyzed
decrease food intake. NPY m-RNA in the hypothalamus increases during and not recycled & used over & over again
feedin g and decreases during satiety®. NPY exerts its effects through Y-5 (like SMT).
receptor. Knocking out of NPY gene does not produce marked effects on
• Unlike SM Ts, neuropeptides are released
feeding, pointing that other pathways are also involved, but knockout of NPY
in much smaller quantities, are more potent (>
gene in leptin deficient cause them to eat less and spend more energy that
thousand times) and have prolonged action.
have normal NPY gene Leptin, a 167 aminoaicd polypeptide, produced
primarily in fat cells act on hypothalamus to decreases the activity of NPY • Small molecule rapidly acting
neurons and decrease activity of POMC secreting neurons thereby causing transmitters include acetylcholine (Class
anorexia & increased energy consumption. The anorexiant action of leptin is I); epinephrine, NE, dopamine, serotonin,
antagonized by blockade of CB1 receptor (marijuana / cannabinoids increase histamine (II = amines); GABA, glycine,
appetite by acting on CB1 receptor) and inhibition of phosphatidylinositol -3- glutamate, aspartate (III = amino acids);
hydroxykinase (an hypothalamic enzyme activated by leptin). Leptin also and NO (IV).
increases activity of SOCS-3 (suppressor of cytokine signaling-3) in NPY neurons • Slowly acting Neuropeptides are.
and SOCS-3 supresses further leptin receptor signaling, suggesting a turn off
Hypothalamic releasing hormones
mechanism.
(thyrotropin-RH, luteinizing hormone
So NPY (& AGRP) orexigenic neurons increase food intake and reduce and somtostatin / growth hormone
energy expenditure (which includes basal /resting metabolic rate , energy inhibitory factor)
cost of metabolizing & storing food, thermic effect of exercise, and adaptive
Pituitary peptides (ACTH, LH, GH, a-
thermogenesis).
MSH, a-MSH, (3-endorphin, prolactin,
Physiology: Central Nervous System ■ 253

Cotransmitters in autonomic neurons include, VIP (released with actyl thyrotrophin vasopressin, oxytocin).
choline) and ATP & NPY (with norepinephrine). The small granulated - Peptides acting on brain & gut (gastrin,
vesicles in post ganglionic adrenergic neurons contain ATP and CCK, VIP, insulin, glucagons,
norepinephrine (NE) and large granulated vesicles contain NPY. The low substance P, neurotensin, nerve
frequency stimulation promotes release of ATP, whereas high frequency growth factor, brain derived
stimulation causes release of NPY. neuro tropic factor, leucine &
Noradenergic postganglionic sympathetic nerves containing NPY are found methionine-enkephalin)
on many blood vessels and cause vasoconstriction (whereas cholinergic - Others (angiotensin II, bradykinin,
nerves containing VIP and sensory nerves containing CGRPa & substrance P camosine, calcitonin, sleep peptides)
cause vasoconstriction).

★ Large precursor protein (prohormone) propionmelanocortin (POMC) is synthesized in corticotropes of anterior lobe pituitary,
intr mediate lobe, hypothalamus, lungs, gastrointestinal tack and placenta. POMC processing by prohormone convertase (PC) 1 & 2 is
tissue specific and result in production of different peptides in different tissue. Anterior pituitary corticotropes expresses
PCI resulting in formation of N terminal peptide, joining peptide, ACTH, (3-lipotropin (LPH) and a small amount of 13-
endorphin. In intermediate lobe &/or hypothalamus, PC-2 expression 1/t production of CLIP (corticotropin like
intermediate lobe peptide), y-LPH, melanotropins (a-, [J-, and y- M SH) and appreciable quantities |3-endorphins, but not
ACTH from POMC.

Substance - P i t Mammalian Tachyklnlns

It is a polypeptide called tachykinins containing 11 Gene Polypeptide products Receptors


aminoacid residues that is found in intestine, Substance P (SP) / Substance P Substance P (NK-1)
variou s peripheral nerves an d CNS®. All 6 Neurokinn A (NKA) Neurkinin A
mammalian tachykinin members differ at mino gene Neuropeptide K Neuropeptide K (NK-
terminal but have in common the carboxy terminal Neuropeptide a 2)
sequence of Phe - X - Gly- Leumet - NH 2, where X is Neurokinin A (3 -10)
Val, His, lys or phe. NKB gene Neurokinin B Neurokinin B (NK-3)
It is found in -
■* NK 1 & 2 receptors are G protein coupled receptors. Activation of SP
- Primary afferent neuron endings in dorsal horn of
receptor 1/1 activation of phospholipase C and increased IP3-DAG.
spinal cord as mediator at the first synapse in
pathways for pain transmission^. Cutaneous Nature Transmitted Neuro­ Synapse
- Nigrostriatal system where its concentration is pain type by transmitter /project to
proportional to dopamine Fast (mild) Initial, sharp, Ventrolateral G lutam ate® Ventral
- Hypothalmus, where it plays a role in pricking & spinothalic posterior
easily tract lateral (VPL)
neuroendocrine regulation.
Upon injection into the skin, it causes redness & localizable nucleus
swelling, and it is probably the m ed iator released Slow Late, burning Spinoreticular Substance Centrolateral
by nerve fib ers th a t is resp on sible f o r axon reflex® (severe) or dull & pathway P® nucleus of
In intestine it cause peristalsisQ. Centrally active intense, diffuse thalmus
NK-1 receptor antagonist has antidepressant activity & accompanied
in humans by unpleasant
autonomic
feelings

N eurotransm itter - & Neurom odulators

• Excitatory amino acids • Pyrimidine • Amines • Polypeptides


- G lutam ate® - UTP - Dopamine - Substance P, tachykinins
- A spartate® • Purine - NA, Adrenaline - Vasopressin, oxytocin, CRH,
• Inhibitory amino acids - Adenosine - Serotonin GRH, TRH, GnRH,
- Glycine® - ATP - Histamine Somatostatin
- GABA® • Lipids - Endothelin, Enkephain,
Anandamide • Acetylcholine CCK4 & 8 etc
• Gases
- NO, CO
254 ■ A Complete Review of Short Subjects

Thirst Control

• Water intake is regulated by osmolalily of plasma and decrease in ECF volume.


• Body respond to increased plasma osmolality in two ways.
(i) By increasing ADH secretion which decrease water excretion by kidney
(ii) By stimulating thirst centre which increase water intake.
• In both these mechanism osmoreceptors are involved.
(iii)Supraoptic nucleus near anterior hypothalamus causes ADH secretion.
(iv) Preoptic nucleus is centre for thirst stimulation®
• The osmotic threshold for thirst is same or slightly (-5% ) greater than the threshold for increased ADH secretion.
• It is still uncertain whether the same osmoreceptors mediate both effects.
That means centre for ADH secretion and thirst stimulation might be same or stimulation of one can lead to both the
mechanism. Thus with increase in osmolality 1st thing to occur is ADH secretion followed by thirst stimulation.
• Vasopressin secretion is regulated by osmoreceptors, that are situated outside blood- brain barrier & located in
circumventricular organs primarily in OVLT/ su p raop tic crest®

Regulation of Body Temperature

Role of Hypothalamus & Temperature Receptors

• Core body temperature can be maintained normally between 97° & 100°F in an ambient temperature of over 100°C (ie in dry
air environment between 50°-130°F) but only < 50°C in a humid atmosphere. Environmental factors such as temperature,
humidity (amount of moisture), wind movement and even nature o f surroundings have important role temperature regulation.
• Hypothalamus (through temperature regulating centres) integrate all body temperature information obtained from sensory
(afferent) receptors (primarily cold receptors) located in the skin (for peripheral detection of surface temperature) and deep
tissues (eg deep receptors found mainly in spinal cord, abdominal viscera, in and around great veins in thorax and upper
abdomen, extrahypothalmic portion of brain and hypothalamus itself for detecting core body temperature).
• Cold receptors are far more than warmth receptors in skin (almost as many as 10 times) and deep tissues - indicating their primary
concern of preventing hypothermia (low body temperature). Each of the 5 inputs (from skin, spinal cord, extrahypothalmic brain,
hypothalmus & other deep tissues) contributes about 20% of integrated information.
• Anterior hypothalamic preoptic area contains 2/3rd heat sensitive neurons as well as l/ 3 rd cold sensitive neurons and
therefore has the capability to serve as a thermostatic detection and control center of body temperature. These neurons serve
as temperature sensors for controlling body temperature. The heat sensitive neurons increase firing rate in response to rise in body
temperature (2 to 10 fold on 10° C increase), whereas cold sensitive neurons increase their firing rate when the body
temperature falls.
Anterior Posterior
• Bilateral posterior hypothalamic area (approximately at the level of
Hypothalamus Hypothalamus
mammillary bodies) integrate the central and peripheral temperature sensory
Cutaneous Stimulation Shivering
signals (obtained from anterior hypothalamic preoptic area and from elsewhere in
vasodilation & (1/t)
body respectively) to control the heat producing and heat conserving reactions
sweating
of the body.
Hyperthermia Lesion (1/t) Hypothermia
• For each temperature regulating response, there is threshold core
(upto 43°C or (Body
temperature eg 37°C for sweating and vasodilation, 36.8°C for
109.4°F) temperature
vasoconstriction, 36°C for nonshivering thermogenesis, and 35.5°C for
falls towards
shivering. The reflexes activated by warmth are controlled primarily from the
that of
anterior hypothalamus and reflex responses activated by cold are controlled
environment)
from posterior hypothalamus, so —»
Physiology: Central Nervous System ■ 255

Critical Temperature Set Point For Core Temperature Control

• The critical body core temperature set point (of 37.1°C or 98.8°F) in the hypothalamus above which rate of heat loss is greater
than heat production (& sweating begins) and below which the rate of heat production is greater than that of heat loss (&
shivering begins) is determined mainly by the degree of activity of the heat temperature receptors in anterior hypothalamic
pre-optic area. All temperature control mechanisms, continually attempt to bring the body temperature back to this set point.

• Temperature signals from peripheral areas especially from skin & certain deep body tissues (spinal cord & abdominal
viscera) also contribute slightly to body temperature regulation by slightly altering the set point of hypothalamic
temperature control centre

• For example, hypothalamic set point for sweating increases from 36.7°C to 37.4°C when skin temperature falls from 33°C to
29°C. In other words, sweating begins at lower hypothalamic temperature when skin temperature is higher (than when the skin
temperature is low). That's why, sweating is inhibited when skin temperature is low; otherwise the combined effect of low
skin temperature & sweating could cause too much loss of body heat

Similarly cold skin drives the hypothalamic centers to the shivering threshold even when hypothalamic temperature itself is still
on the hot side of normal - ie cold skin anticipates a fall in internal body temperature and prevents this.

Body Heat Producion Body Heat Loss at 21 °C


- Basal metabolic process - Radiation 55-65%
- Foot intake - Conduction 10- 15%
- Muscular activity - Vaporization of sweat 27%
- Respiration 2%
- Urination & defecation 1%
★ Epinephrine &c norepinephrine produce a rapid but short ★ Radiation is heat transfer by infrared electromagnetic
lived increase whereas thyroid hormones produce a slowly radiation between objects not in contact, whereas
developing but prolonged increase in heat production conduction is heat change between objects that are in
★ Sympathetic discharge is increased by feeding & decreases contact. Conduction is aided by convection ie movement
during fasting of molecules away from the area of contact
★ Brown fat is source of considerable heat in infants.

Temperature Regulating M echanism


These reflex and semireflex thermoregulatory responses include autonomic, endocrine, somatic and behavioral changes,
facilitating increase heat loss & decreased heat production (on exposure to heat) or decreased heat loss and increased heat
production (on exposure to cold). Therefore mechanisms activated by

Heat (ie promote heat loss) Cold (ie promote heat gain)
Mostly increased parasympathetic activity Mostly increased sympathetic activity

Increased heat loss Decreased heat Decreased heat loss Increased heat production
production - Cutaneous vasoconstriction - Shivering
- Cutaneous -Anorexia - Piloerection - Hunger
vasodilatation -Apathy and inertia - Curling up - Increased voluntary activity
- Sweating -Inhibition of -Piloerection (horripilation - Increased secretion of
- Increase respiration shivering & or goose pimples) norepinephrine/ epinephrine
- Panting (rapid chemical thermo- - Curling up in a ball -N onshivering thermogenesis
shallow breathing by genesis (universal flexion) posture -T b m r
mouth) in animals
-Sympathetic stimulation
like dogs
-Thyroxine secretion_________
256 ■ A Complete Review of Short Subjects

Physiological changes that Thermoregulation in Cold


occur during Heat [Hypothalamus is responsible for therm oregulation^]
Acclimatization
When a person is exposed to cold, thermoregulation occurs in two phases-
•Cutaneous vasodilatation causes
increase in plasma volume Immediate Delayed
•In creased rate o f sweating® I I
(Approx. two fold) Through autonomic nervous system Through endocrine system
D ecreased renal b lo o d flo w Q (to I I
conserve water as much is lost Increased sympathetic stimulation Stimulates
through sweating) hypothalamus
Cutaneous vasoconstriction Lipolysis® Shivering®
- D im inished loss o f s a lt in sw ea t I
I I I
& urineQ to almost none (d/t Tt r h
W arm b lo o d rem ains in the Generates heat Heat
in creased secretion o f I
core® generation
ald osteron e fo r better TThyroxine
I
con servation o f b od y w ater & I
salt.Y3 Heat not transferred to the
shell tM etabolism
I (BM R)
Heat conserved I
THeat production

★ 'Piloerection means hairs 'standing on end'. Sympathetic stimulation causes the arrector pili muscles attached to the hair
follicles to contract which brings the hairs to an upright stance. This is not important in human beings but in lower animals,
upright protections of hairs allows them to entrap a thick layers of 'insulator air' next to the skin, so that transfer of heat to
the surroundings is greatly depressed'- Guyton l l th/895

Effects of Fall of 1°C Temperature


r
Oxygen Rise In Decrease in
dissociation I I
curve is shifted - Serum K+ Metabolism (by 6.7%)®
to leftd - Acidosis Duration & magnitude of block by
- Duration & magnitude of block at N-m nondepolarizing drugs
junction by depolarize drugs
Lesion of Anterior hypothalamus causes hypertherm ia® (manifestation include cutaneous vasodilation & sweating)
Lesion of posterior hypothalamus causes hypotherm ia® (manifestation include shivering)

Nerve (Axonal) Regerneration


Axonal regeneration is characterized by development & prolongation of g row th cone (short sprouts) from the proximal nerve stump
and its extension along the rows of schwann cell column in the distal nerve stump through intact neurilemma (ie g h o s t tube). A xonal
regeneration occurs in peripheral nervous system but n ot in CNS because of following reasons.

Rate of removal of myelin debris, cytokine release and RAG s Fast an in high Slow and poor
(regeneration associated gents) expression concentration
Schwann cells multiply rapidly & form rows to appropriately In PNS presence of CNS lacks Schwann cells
guide sprouting proximal stump. Dedifferentiated Schwann cells Schwann cells, and single
upregulate production of neurotrophins ie growth promoting Neurilemma oligodendroglial cell
factors eg nerve growth factors (NGF), Ng CAM /Ll. N eurilem m a (endoneural tube) and myelinates many axons
(endoneural tube)® is the outermost cell membrane of Schwann cell growth factors help so it can't form path for
(the nucleus of which lies b/w nyelin sheath & neurilemma) regeneration. growing sprouts.
A strocytic proliferation , F orm ation o f g lia l scar® by astrocytes; Absent Present Q
Activation of microglia; Inflammation and invasion of immune cells
provide inappropriate environment for regeneration.
CNS myelin is axonal growth inhibitor^. M yelin associated Absent Present Q
inhibitors include Nogo, MAG (myelin associated glycoprotein)
OMgp (oligodendrocyte myelin glycoprotein) & trans membrane
semaphorin 4 D
Physiology: Central Nervous System ■ 257

NSAIDs inhibit Rho A (small GTPase protein that prevents axonal repair). Similarly, drugs inhibiting IP3 or PI3
(phosphoinositide 3 kinase) pathway also promote nerve regeneration. Growth cone collapse by myelin associated inhibitors is
prevented by pertussis toxin.

Cerebro Spinal Fluid (CSF) ' f o

CSF Constituents CSF formation® 50-70% is formed by choroids


plexus®, remaining CSF formed
Appearance Clear colourless around blood vessel k ventricular
Proteins 20 - 40 mg/dl® walls®
Sugar 40 - 70 mg/dl® Volume - 150ml®
Chlorides 720-750 mg/dl® Rate of CSF formation - 550ml /day (2.3 ml/hr or 0.38
Cell/mm3 Lymphocytes (0-5)2 ml/min)®
DLC 60-70% lymphocytes Rate of CSF turnover - Approx. 3 time /day®
30-40% monocytes Specific gravity - 1.006-1.007®
0% (None) neutrophils® - PH of CSF - 7.33®
Substance CSF P lasm a conc CSF/plasm a
- CSF pressure - 70-180mm of water (in lumbar
concentration ratio.
area)® and average is 130mm
Mg** 2.2 1.6 1.39
H20 (10mm Hg)
PCO 2 50.2 39.5 1.28
- Ion with maximum - Mg2*®
Creatinine 1.5 1.2 1.25
CSF/plasma ratio
Cl 113 99 1.14 - Minimum CSF/plasma ratio - Cholesterol & Protein®
HCOr 25.1 24.8 1.01 - Equal CSF /plasma ratio is - Osmolality®
O sm olality 289 289 1 with.
Na* 147 150 0.98
Regulation of CSF Pressure by Arachnoid Villi
U rea 12 15 0.80
Inorganic-P 3.4 4.7 0.73 - Rate of CSF formation is nearly constant, so it is seldom a factor
Glucose 64 100 0.649 in pressure control.
K* 2.9 4.6 0.62 - Whereas, arachnoid villi function like valves that allow CSF to
Ca** 2.3 4.7 0.49 flow readily into blood in venous sinuses while not allowing
Uric Acid 1.5 5 0.3 blood to flow backward in opposite direction
Protein 20 6000 0.003
- CSF flow begins when its pressure is 1.5 mmHg higher than the
Cholesterol 0.2 175 0.001
pressure of blood in venous sinuses.
pH 7.33« 7.40
- The valve opens more widely, when this pressure difference is
* A ll ions (N a*, K*, M g**, Ca**, Cl ) are in meq/kg H 2O except
H C O 3 ion w hich is in meq/L. All organic substances (glucose, higher, which in normal conditions never allows CSF pressure
protein, urea, creatinine, uric acid, cholesterol) ^ in o rg an ic - p to rise more than few mm Hg higher than cerebral venous sinus
are in mg/dL w hereas osm olality is in m osm /kg H 2O and pressure.
P C O 2 in m m H g. - Absorption is proportional to pressure. At a pressure of 112 mm
* All +ve ions (N a*, K*, Ca**) are m ore in plasm a except M g* * H2O, which is average normal CSF pressure, filteration &
w hich is m ore in C SF
absorption are equal. Below a pressure of 68 mm H2O,
* All - v e ions (Cl / H CO y) are m ore in CSF.
absorption stops._________________________________________
Enzymes in CSF Causes of Reduced (i.e. between 3-20 mg/dl) CSF Protein
- Creatinine kinase (CK) BB, Enolase and Neopterin are
found in CSF after stroke, global ischemia, hypoxia or trauma - Hyperthyroidism®
and have been used as markers of brain damage. - Benign intracranial hypertension (pseudotumor cerebri)®
- Lactate dehydrogenase (LDH) is increased in bacterial - Infants & children between 6 to 24 months o f age®
(than aseptic) meningitis, intracranial & subarachnoid - Acute water introxication (increased ICP with decreased
hemorrhage (but not in current traumatic tap with intact proteins)
RBC s) .and CNS leukemia/ lymphoma/metastatic - CSF leaks (such as due to trauma, repeated lumbar
carcinomas. puncture® & pneumoencephalography) and removal of
- Adenosine deaminase is increased in meningeal TB (d/t large volumes of CSF (eg for cytological studies).
increased T lymphocytes containing it). Lysozyme is
increased in both bacterial & TB meningitis.

Herniation /Coning
Refers to displacement of brain tissue away from a mass and into a compartment that it normally does not occupy.
[The cranial cavity is separated into compartments by infolding of the dura the true cerebral hemisphere are separated by falx and
anterior & posterior fossa by the tentorium]
258 ■ A Complete Review of Short Subjects

Uncal Central transtentorial Transfalcial Foraminal


Herniation Herniation Herniation Herniation
Displacement of antero medial temporal Symmetric downward movement of upper Displacement Downward
gyrus (uncus) into the tentorial opening 1/t thalamic region through tentorial opening of cingulate forcing of the
Sequential Compression of: 1/t Sequential Compression of: gyrus under the cerebellar tonsil
- Ipsilateral III cranial nv. - Upper midbrain (first) falx and across into the
- Contralateral brainstem (later) - Pons (later) the midline foramen
- Whole brainstem (eventually) - Medulla (finally) magnum.
Physical Sign:
Physical sign:
Early Later Eventually Early Later Eventually
- I/L dilated - Babinski -Tetraparesis - Erratic - Cheyne - Fixed
pupil® sign® - B/L fixed respiration stokes dilated
- Sign of -I/L dilated - Small reactive respiration pupil
supra ten­ hemiplegia pupil pupil - Decorticate - Decerebrat
torial mass - Autonomic - Erratic - B/L extensor rigidity eposturing
lesion disturbance® respiration planter®
- Death

Physiological effects produced by Opiate Receptors

p(m u) K (kappa) 8 (delta)


- Physical dependence Miosis (lower ceiling) - Analgesia
- Miosis® Diuresis® - Respiratory depression
- Constipation® Analgesia®
★ Miosis is a better option as
- Analgesia® Dysphoria®
- Respiratory depression sedation is also a feature of K-
Sedation
- Euphoria® receptor [Miosis is also a
- Sedalion® feature of kappa receptor but
- T ed secretion of GH & prolactin® there is only slight miosis]
★ |i receptor is site of action of
[Mnemonic: P.M CARES] morphine and endorphins.

Thermal Receptors
i— 1---------------------------------- 1
Cold- receptors Warmth (H eat) - receptors Pain receptors
I I I
Cold receptors are inactive at 40° but then steadily increase their firing - Threshold for activation is VR1 (vanilloid
rate as skin temperature falls to about 24°C. As skin (subepithelial) 30°C & they increase their receptor 1 )®
temperature further decreases upto 10°C the firing rate of cold receptor firing rate upto 46°C (i.e. respond to pain
decreases. Below 10°C the receptor are inactive and cold becomes an respond to 30-46°) producing
effective local anesthetic (i.e. respond to 10°C - 38°C). - Warmth receptor, not chemical
Histologically definitive cold receptors are nerve endings of A8 being identified capsaicin &
myelinated nerves, that branches a number of times, the tip of which histologically are temperature
protrude into the bottom surfaces of basal epidermis. So. it is the presumed to be free nerve >43°C
temperature of subepithelial subcutaneous tissue that determine the endings because heat VRL -1 respond
responses. signals are transmitted to temperature >
Cold signals are transmitted mainly by A8 nerve fibres. Some cold mainly by type C nerve 50°C (but not to
sensation may also travel in type C fibres as well. fibres. capsaicin)
There are 4-10 times as many cold sensitive as hot sensitive spots and the ★ In addition to ending in
number in different areas of body varies from 15-20 cold spots/cm2 in lips post central gyrus, fibers
to 3-5 colds spots/cm2 in fingers to < 1 cold spot/cm2 in trunk. from thalamus end in I/L
The receptors for moderate cold is the cold and menthol sensitive insular cortex (true
receptor 1 (CMRI) primary thermal receiving
area)®
Physiology: Central Nervous System ■ 259

Accomodation Adaptation (Desensitization) Electrotonic Conduction


It is a property of It is a progressive decrease in sensory Most dendrites cannot transmit action potentials, because of
nerve, where a receptor's response despite the few voltage gated Na+ channels in their membranes & very
slow ly rising continued presence o f a stimulus 2. high threshold for action potential. But they can transmit
(increasing) When a maintained stimulus of constent signals to soma (within the same neuron) by electrotonic
subthreshold strength is appled to a receptor, the conduction (EC)
stimulus raises frequency of action potential in its EC is direct spread o f electrical current by ion conduction with
(increases) the sensory nerve declines over time. This is in the fluids of dendrites but with out generation of AP.
threshold o f nerve k/a adaptation It is decremental conduction, as the dendritic membrane is
to generate an It is a characteristic of all sensory receptor thin and partially permeable to K+ & Ch ions, making them
action poten tial <3. i.e., when a continuous sensory stimulus is leaky to electric current.
applied, the receptor responds at a high So the synapse that lies near soma has lesser decrement and have
impulse rate at first and then at a far more effect in causing neuron excitation or inhibition.
progressively slower rate until finally the Dendrites can summate all excitatory & inhibitory
rate of action potentials decreases to postsynaptic potentials in the same way as soma can.
very few or often to none at all.

Motor Neurons

Somatic (effector organ = Skeletal ms) Autonomic (effector organ = Cardiac &
smooth muscle)
_L
I 1
UMN LMN (final common pathway) Sympathetic Parasympathetic
Upper motor neuron (UMN) consists of Lower motor neurons (LMN) consists of * Large myelinated fibers originating from
pyramidal cell & their neurons. The anterior horn cells or homologus cells in the giant pyramidal (Betz) cells (found only
pyramidal (corticospinal) system brainstem, their efferent nerve fibers (which in primary motor cortex) are most
consists of the central pathway which pass via the anterior spinal nerve root) impressive fibers in pyramidal tract.
directly link the pyramidal cell in the 5th peripheral nerves to the muscle & the muscle Diameter of cell is 60pm & of fiber is 16
layer of motor cortex with the motor fibers innervated by these nerve fibers & pm (micrometer) with a conduction
neurons in the brainstem & spinal cord. their terminal branches. velocity of 70 m/sec. (most rapid
transmission from brain to cord). Total
Pyramidal system (voluntary Extrapyramidal system (for involuntary fibers in pyramidal tract is 1 million, of
movement) movement: tone, posture & equilibrium) which 3% (34,000) are from Betz cells.
Remaining 97% fibers are <4pm
diameter.

Corticospinal (Pyramidal) Tract


It originates from primary motor cortex (30%), premotor & supplementary motor area (30%) and somatosensory areas posterior to
central sulcus (40%)
It passes through posterior limb of internal capsule (b/w caudate nucleus & putamen of basal ganglia) and then form pyramid
of medulla.

Lateral corticospinal tract (LCT) Anterior /Ventral - corticospinal tract


80% of fibers of pyramidal tract cross in low er medullaQ - 20% fibers do not cross to opposite side in medulla
to opposite side & form LCT. - Do not cross the middle until it reaches the level of muscles it
Concerned with skilled movements and ends directly on motor controls. At this level fiber ends on inter neuron that is
neurons in humans (Ganong). connected to motor neuron of both sides of body (Ganong)
But according to Guyton fibers finally terminate on - Many (if not most) fibers cross to opposite side of body either
interneurons in intermediate regions of cord gray matter in neck or in upper thoracic region (Guyton); and are
(most) > sensory rely neurons in dorsal hom (few) > concerned with control of bilateral postural movements (by
directly on anterior motor neurons that cause muscle supplementary motor cortex).
contraction (very few)
260 ■ A Complete Review of Short Subjects

Difference

F e a tu re U p p e r M o to r N e u ro n L o w e r M o to r M y o p a th y Pyramindal system Extrapyramidal system


(U M N / p yram id al tract) N e u ro n le s io n - It includes motor - Denotes all those portion of
le s io n
cortex + the axons the brain and brain stem,
M u s c le a tro p h y N one M a r k ed ® M ild
descending through that cortribute to motor
(w a s tin g )
F a s c ic u ltio n N one C om m on ® N one
the pyramidal tract control and are not part of
D is tr ib u t io n o f P y ra m id a l o r re g io n a l D istal or P ro x im a l
(PT) or cortico spinal the direct corticospinal
w eak n ess seg m e n ta l tract. pyramidal system.
M u s c le T o n e I n c r e a s e d (S p a s tic )® D ec rea se d lo st N o rm al - Anterior - Basal ganglia
(fla c c id ) / D ecreased corticospinal - Motor nuclei of reticular
C lo n u s P resen t® A b sen t A b sen t - Lateral corticospinal formation of brain stem
D e e p te n d o n E x a g g e ra te d / h y p er L o st N o m a l or - Vestibular nucleus
r e fle x e s (je r k s ) a c tiv e ® / H y p o activ e h y p o a c tiv e - Descending fibers which
B a b i n s k i 's sig n P resen t® A b sen t A b sen t
convey the impulses from
P la n ta r r e fle x E x ten so r® F lex o r F le x o r
these nuclei to spinal cord.
S u p e r fic ia l L o st L o st o n ly w h en N o rm al
- Concerned with - Concerned with control o f
a b d o m in a l r e fle x th e p a rticu la r
voluntary activities posture, tone and
n eu ro n
su p p ly in g the
o f body® equilibrium®
m u sc le is lost.

- Athetosis (i.e. continuous, slow writhing movemets) and chorea (rapid involuntary dancing movements) occur in an
involuntary, disorganized way at the start of voluntary movements. Ballism is involuntary flailing, intense, violent
movements. Akinesia is difficulty in initiating movement & decreaed spontaneous unconscious movements such as swinging
of arm during walking, different facial expressions related to emotional content of speech & thought and multiple gestures &
fidgety actions that occur in all of us. Bradykinesia is slowness of movements.
- Rigidity differs from spasticity because in it motor neuron discharge increases to both the agonist & antagonist muscles and so
passive motion meets with a plastic, dead feeling continuous resistance that has been linked to bending a lead pipe & is therefore
called lead pipe rigidity. Sometimes a series of "catches" (i.e. resistance f/b release f/b resistance) takes place during passive
motion, then it is called cogwheel rigidity, but the sudden loss of resistance as seen in spastic extremity (i.e. clasp knife
rigidity) is absent in EPS lesions. Resting tremors d/t regular alternating 8 Hz contractions of antagonist muscles, disappears
with activity.

Motor Neuron Disease


Is characterized by degeneration of motor neurons, either upper motor neuron or lower motor neuron or both.
I----------------------------------------------------------------------
Types Clinical features of ALS depend on the neurons involved
I----------
Chronic Acute Upper M otor Neurons Lower M otor Neurons
I---------- I I I M anifestations (UMN) M anifestations (LMN)
UMN + UMN alone LMN alone ■Poliomyelitis Corticospinal Tract Low er m otor neurons o f
LMN both predominantly predominantly •Herpes Involvement (UMN o f Limbs) Limbs involved
involved I I zoster - Hyperactive tendon reflexes • Weakness
I Primary lateral Progressive •Coxasackie - Spasticity / spastic - Asymmetric
Amyotrophic sclerosis muscular virus resistance to passive - Insidious
lateral atrophy movements. - Distal to proximal
sclerosis • Progressive wasting and
(most atrophy of muscles <2
common • Fasciculations or
type)® spontaneous twitchings
Corticobulbar Tract Low er m otor neurons o f
Involvement (UMN to bulbar bulbar muscles involved
muscles)
- Dysarthria • Difficulty in chewing
- Exaggeration of emotion - Swallowing
involuntary excess • Movements of face and
weeping/ laughing tongue
(pseudobulbar affect)
Physiology: Central Nervous System ■ 261

Brown - Sequard Syndrome


• It is functional C linical Presentation
hemisection of spinal
• It is important to note that lesion of descending (from cortex to organ) tract like pyramidal
cord usually caused by
tract after crossing over and ascending tracts (from organ to cortex like dorsal column &
spinal cord tumors,
trauma (eg pallet /bullet), spinothalamic tract) before crossing overproduce ipsilateral signs (and vice versa).
degenerative disc disease • This is why, in Brown- Sequard syndrome, hemisection of spinal cord (which at that level
and ischemia. contains already crossed ascending spinothalamic and descending pyramidal tracts and uncrossed
dorsal column ascending tract) produces.
• Normal physiology
/anatomy of tracts_______ Ipsilateral loss Contralateral loss
- Anterior (Ventro) lateral - (on the same side of transaction) (on the opposite side of transaction)
spinothalamic tract crosses •Dorsal column sensations in all dermatomes •Spinothalmic sensations are lost on
the midline more or less at below the level of transaction i.e. contralateral side in all dermatomes
the level of point of entery - Fine (light) touch & pressure begning 1 or 2 segments (Ganong) /2 to 6
itself and hence it ascends - Discrete localization, two point tactile segments (Guyton) below the level of
contralateral along the discrim ination or discriminative touch transaction i.e.
whole length of spinal cord -Proprioception, Join t position sense (static -Pain®
to reach the brain. position sense & kinesthesia or dynamic - Temperature®
proprioception /rate of movement sense) - Crude touch, which is poorly localized,
- Dorsal (posterior) column
-V ibration and sensation of movement may still persist because of partial
- M edial lemniscal tract
(Fasciculus gracilus or against skin transmission in opposite spinothalamic
fasciculus cuneatus) •Corticospinal (pyramidal) tract features tract.
ascends such that it below the level of transaction ★ "B S - POTS"=Brow n Sequard
remains ipsilateral along -M uscle weakness, and spasticity syndrome has Opposite (contralateral)
the entire length of spinal - Clonus, exaggerated tendon reflex loss of pain & temperature or
cord. Its fibers cross over to spinothalamic tract.
opposite side in medulla.
- Descending Corticospinal
(Pyramidal) Tract crosses
the midline to reach
opposite side in medull.
This means that any lesion
in this tract above medulla
will produce, contralateral
whereas, below medulla
(in spinal cord) will
produce ipsilateral signs &
symptoms._______________ T otal lo s s o f all
; s e n s a tio n s a n d
• Segmetal (LMN) signs h y p o to n ic p a ra ly s is

like radicular pain,


muscle atrophy, or loss of L o s s o f touch
' L o s s o f pa in discrimination
deep tendon reflex etc are p lu s te m p e ra tu re vibratory and
j se n s a tio n s , proprioceptive
ipsilateral unilateral. im p a ire d to u ch sensations and
Brown Sequard syndrome ; s e n s e iio n s 9pastic paralysis

Blood Supply o f Spinal Cord C linical Features o f Spinal Vascular Syndromes


Anterior spinal artery: supply anterior 2/3 o f cord
Anterior spinal artery syndrome Posterior spinal
Posterior spinal artery: (a pair): supply posterior 1/3
1) Radicular / girdle pain artery syndrome
of cord [posterior column]
2) Motor signs (Flaccid quadriplegia /
Radiacular arteries: The largest rdicular artery Loss o f
paraplegia below the level of lesion proprioception &
arises from aorta in the lower thoracic or upper
3) All sensory modalities lost except vibration sense below
lumbar region (artery of Adamkiewiez) and major
vibration & position sense. [Posterior the level o f lesion®.
source of blood supply to lower 2/3 o f the spinal cord
column is spared]
4) Impairment of bladder & bowel function
262 ■ A Complete Review of Short Subjects

Conditioned Reflex

Conditioned reflex is a reflex response to a stimulus that previously elicited little or no response, acquired by repeatedly
pairing the stimulus with another stimulus that normally does produce the response.
In Pavlov's Classical Experiment^ -
- Presentation of food - Unconditioned stimulus (US; which normally produce an innate response)
- Ringing of the bell - Conditioned stimulus (C S). After the CS & US had been paired a number of times, the CS is able to
produce the response originally evoked by US. Conditioning of visceral response is called biofeedback.

Autonomic Nervous System (ANS)

• In somatic nervous system (or somato-motor system), the final common pathway linking the CNS to skeletal muscles is
large diameter- rapidly conducting a motor neurons (Aa). However, in ANS the peripheral motor portion of sympathetic
& parasympathetic neurons (supplying smooth muscle, cardiac muscle & glands) are made up of two neurons;
preganglionic and postganglionic neurons.
• The cell bodies of preganglionic neurons are located in the intermediolateral (IML) gray column of spinal cord or the
homologus motor nuclei of the cranial nerves therefore they have a long course °. The cell bodies o f the post ganglionic
neurons are located on or near the body o f viscera therefore postganglionic neurons are very short 0. Each preganglionic
axon diverges to an average of 8 or 9 post ganglionic neurons. Preganglionic: Postganglionic —> 1: 8 or 9Q. So the ANS
output is diffuse. Preganglionic autonomic fibers are myelinated B fiber, whereas postganglionic fibers are unmyelinated
C type. Conduction speed is A>B>C.
• Parasympathetic division is k/a craniosacral (involving 3, 7, 9,1 0 CN & S 2, S3, S4 sacral roots) or cholinergic system (uses
ACh). It stimulates activities that conserve energy & restore body resources such as increased digestion & absorption of food
and decreased heart rate. Ciliary spheno (pterygo) palatine, submaxillary (submandibular) and otic ganglions are
parasympathetic ganglions of 3rd, 7th & 9th cranial nerves.
• Sympathetic (thoracolumbar or adrenergic) system stimulates activities that are required during emergency (fight, fright
and flight) response.

Transmission In Sympathetic Ganglia

The response produced in post ganglionic sympathetic neurons by stimulation of their preganglionic innervations generates
rapid depolarization (Fast EPSPP slow EPSPQ, a late slow EPSP0, and prolonged inhibitory post synaptic potential (slow
IPSPP. Transmission in Sympathetic Ganglia.

Potential Duration Mediator Receptor Special features


Fast EPSP 30 ms Acetyl­ Nicotinic - Rapid
Fast Excitatory Response Slow Responses
1 choline cholinergic depolarization
I
These slow responses modulate & that generates
A rapid depolarization that
regulate transmission through action
generate action potential
sympathetic ganglia potential
Slow IPSP 2 s Dopamine d 2 - d/t increase in
K* conductance.
Fast EPSP Slow EPSP Late Slow Slow IPSP
- Dopamine is
EPSP
secreted by
- Produced - Produced Produced by Produced by
intensely
by acetyl by aectyl GnRH or a dopamineQ (via D2
fluorescent
choline choline® peptide receptor) interneurons
cells (SIF cells)
via acting on a closely within ganglia. The
in ganglia
nicotinic muscarinic resembling it inter- neurons that
Slow EPSP 30s Acetyl­ m2 - d/t decrease
receptor0 receptor Very secrete dopamine are
choline cholinergic in K+
( M i- prolonged the small, intensely
cholinergic) conductance
lasting fluorescent cells (SIF
Late slow 4 min Gn RH Gn RH Late slow EPSP
minutes rather cells) in the ganglia
EPSP has a latency of
than The interneuron is
l-5s and last for
milliseconds. excited by M 2
few minutes.
muscarinic receptor
Physiology.1Central Nervous System ■ 263

Some Drugs and Toxins Affecting Autonomic Activity

Site of Action Compounds That Augment Autonomic Compounds That Depress Autonomic
Activity Activity
Sympathetic • Stimulate postganglionic neurons • Block conduction
and - Nicotine - Hexamethonium (C-6)
parasympathetic - Low concentrations of acetylcholine - Mecamylamine (inversine)
ganglia • Inhibit acetylcholinesterase - Pentolinium
- DFP (diisopropyl fluorophosphate) - Trimethaphan (Arfonad)
- Physostigmine (Eserine) - High concentration of acetylcholine
- Neostigmine (Prostigmin)
- Parathion
Endings of • Release norepinephrine • Block norepinephrine synthesis
postganglionic - Tyramine - Metyrosine (Demser)
noradrenergic - Ephedrine • Interfere with norepinephrine storage
neurons - Amphetamine - Reserpine
- Guanethidine (Ismelin)
• Prevent norepinephrine release
- Bretylim (Bretylol)
- Guanethidine (Ismelin)
• Form false transmitters
- Methyldopa (Aldomet)
Muscarinic Atropine, scopolamine
receptors
a adrenergic • Stimulate a l receptors • Block a receptors
receptors - Methoxamine (Vasoxyl) - Phenoxybenzamine (Dibenzyline)
- Phenylephrine (Neo-Synephrine) - Phentolamine (Regitine)
- Prazosin (Minipress) (block a l)
- Yohimbine (blocks a2)
b adrenergic • Stimulate preceptors • Block Preceptors
receptors - Isoproterenol (Isuprel) - Propranolol (Inderal) and others
(blocks (il & |12)
- Atenolol (Tnormin) and others
(blocks pi) Butoxamine (blocks P2)

_______________ Galvanic Skin Response_______________

- Galvanic skin response (electrodermal response = EDR, skin conductance response = SCR, psychogalvanic response =
PGR) is a measure of electrical resistance of skin as a reflection of changes in emotional or autonomic activity.
- It appears as an increase in electrical conductance (i.e. decrease in resistance) of skin, measured by attaching electrodes to
skin and recording changes in associated moment to moment perspiration & autonomic activity.
- Emotional arousal like fear, anger, sexual feeling, startle response, orienting response, stress, anxiety & knowingly stating a falsehood
(lie) and sympathetic activity as by pinprick or threat of injury produce sweating and increased electrical conductance of
skin 1/1a similar t GSR response. It increases to a maximum after 2 to 10 seconds after stimulation and subsides at about the
same rate.
- It is used in lie detection,behaviour therapy and hypnosis to indicate that client is experiencing emotional arousal.
- GSR is decreased during relaxed states like during yogaQ
Chapter 2. NERVES AND MUSCLES PHYSIOLOGY: REVIEW NOTES

Neurotransmitters

For a substance to be considered NT, it must be present in presynaptical terminal, is synthesized by cell, should be released on
depolarization of terminal and there should be specific receptors for it on the post synaptic terminal (±) it can be divided into

2. Large Molecule NT or
1. Small Molecule NT 3. Gaseous NTs
Neuro-Peptides

A. Acetyl choline Hypothalamic hormones & Nitric oxide (NO)


B. Aminoacids eg CRH, LHRH, TRH, GHRH, Carbon monoxide (CO)
glutam ate, GABA and oxytocin, somatostatin &
glycineQ vasopressin Excitatory aminoacid NT " fo
C. Biogenic monoamines eg Opioid peptides eg
- Aspartate
dopamine, epinephrine enkephalin, dynorphin - GlutamateQ (Major >75%).
( adrenaline), Tachykinins eg substance P,
Mn-"Excited AS Glue"
norepinephrine neurokinin a, B, K Inhibitory Aminoacid NT
(noradrenaline), V IP - glucagon family - GABAQ (major)
serotonin (5 HT or 5 - Glycine (both excitatory & inhibitory).
Insulin, ACTH, CCK, OC-MSH,
hydroxy tryptamine) and Mn "in habited Gay guy"
Motilin etc.
histamine
★ Glycine is inhibitory by increasing Cl" conductance like
D. Purines eg ATP
GABA. It is excitatory when binds to NMDA receptor
and makes them more sensitive to glutamate.

Golgi Tendon - Inverse Stretch Reflex

• Golgi tendon organ is an encapsulated sensory receptor, consisting of a net like collection of knobby nerve endings among
the fasicles of a tendon. There are 3 -25 (usually 10-15) muscle fibers connected to each golgi tendon®.
• Unlike muscle spindles, golgi tendon organs are in series with the muscle fibers, so they are stim ulated by both passive
stretch and active contraction o f muscle®. It is important to note that spindle detects muscle length and changes in muscle
length, wheres golgi tendon detects muscle tension ®. So it is stim ulated when muscle fiber is tensed by contracting or
stretching muscle®.
• Like primary receptor of muscle spindle, tendon organ has both a dynamic (intense response to sudden increase in tension)
and static response® (steady state firing of lesser degree almost directly proportional to muscle tension).
• Impulses from tendon organ are transmitted through large, m yelinated rapidly conducting sensory lb nerve fibers® (~
16 fl dia). This lb fibers end in the spinal cord on inhibitory interneuron, which intum terminate on a - motor neuron
supplying the muscle from which lb originated. They also make excitatory connections with motor neurons supplying
antagonist muscles. Stimulation o f tendon organ causes inhibition o f a-m otor neuron® whereas muscle spindle stimulation
excites a motor neuron.
• The threshold o f golgi tendon is low®. Since more elastic muscle fibers take up much of the stretch, the degree o f stimulation
by passive stretch is not great, and strong stretch is required to produce relaxation®. However, contraction of muscle
regularly stimulate golgi tendon. It is responsible for inverse stretch reflex.

I I
Upto an extent, the strength of reflex contraction - This inhibitory negative feedback lengthening reaction prevents
of muscle is proportional to the magnitude of development of too much tension on the muscle and protects
stretching. However, when tension becomes very tearing o f muscle or avulsion o f tendon®.
high there is sudden relaxation of muscle. This is k/a - Tendon organ equalizes contractile forces of different muscle fibers by
inverse stretch reflex or autogenic inhibition. inhibiting fibers exerting excess tension.
Physiology: Nerves and Muscles 265

b n e r v e fib e r (1 6 p m )

Tendon— M u s c le fib e rs lb fib e r fro m g o lg i \ \ \ (a


le n d o n o rg a n \ \ fro fn
(3-25)
\m u 6 C le
\s p in d le

O rg a n o f go lgi (ra m ific a tio n


o f k n o b b y n e rv e e n d in g s )

M o lo r e n d p la te o n e x tra fu s a l fib e r

Withdrawal Reflex Reciprocal Inhibition Inverse Stretch Reflex


Its a polysynaptic reflex occurring in response It is a phenomenon, in which relaxation of It is sudden relaxation of
to noxious or painful stimuli antagonist muscles occur, when a stretch muscle on development of
The response is flexor muscle contraction & reflex excites one muscle high magnitude of tension.
inhibition of extensor muscles, so that the part Neuronal circuit which causes this It is au togen ic in h ibitory
stimulated is flexed & withdrawn from stimuli reciprocal inhibition is called reciprocal n egative fe e d b a ck
On application of strong stumuli, about 0.2- 0.5 innervation. lengthening reaction®, that
second after flexor & with drawl response in Ia fiber transmit impulse from protects against muscle
one limb, the opposite limb begins to extend, protagonist muscle and cause inhibition tear.
This is called cross extension reflex®. It is a part of motor neuron to antagonist muscle. G olgi tendon organ® is
of withdrawal reflex as extension of the Bisynaptic pathway, collateral from la receptor for inverse stretch
opposite limb can withdraw the entire body fiber passing through inhibitory Golgi reflex.
away from noxious stumuli. bottle intemeuron

Changes in Nerve Fiber after Injury

Degeneration of the distal part (Antegrade) Degeneration of proximal part Stage of Regeneration
(Retrograde)
• Degeneration of distal part (distal to cut) is called • Same as distal generation but • Axis cylinders from stumps
Wallerian degeneration.® upto nearest node o f ranvier grows & sprout out
(nearest collateral)® (pseudopod like structure of
• Within 24 hours histological and chemical changes
fibril)
occur • Changes in the cell body:
- Within 48 hours of injury the • After 2-3 weeks peripheral
• The axon cylinder sw ells up & breaks down into
nissl granules & rough endoneural tube, contain
sm all rodlets.®
endoplasmic reticulum varying no of developing
• Myelin sheath breaks down into oily droplets, It fibril
begins to disintegrate (K/a
occurs 8-32 days after injury.®
chromatolysis). • Medullary sheath begins to
• Cells of the sheath of Schwann - divide mitotically® develop in about 15 days &
- Golgi apparatus fragments
& from cords of cells lying within the endoneurial completed within one year.
tube (neurilemma) —» macrophages remove the - Cells swell
• Repair of cell body (i.e. Nissl
degenerating myelin & axis cylinder —» thus schwann - Neurofibril disappear &
granules & golgi apparatus)
cell cytoplasm gradually fills the neurilemma, this nucleus displaced to cell
—» begins about 20 days
process is completed by 3 months. margin.
after nerve section & is
• Hollow tube of neurilemma remains intact & K/a completed in 80 days.
ghost tube.
266 ■ A Complete Review of Short Subjects

Nerve (Neuron) Action Potential (AP)

Development of AP Resting Membrane Potential (RMP)


Axon hillock is a thickened area of cell body from which originates the long Because of more open K+ channels and
fibrous axon. AP is generated in this segment because the membrane at the greater membrane permeability to K+(in
axon hillock has grater concentration o f voltage gated Na+ channels and comparison to Na+) at rest, the resting
therefore the threshold fo r excitation is low er than the rest o f neurons® membrane potential in neurons is usually
All or none law: Action potential will not develop unless the stimulus about -70 mV, Which is close to
searches a threshold intensity. Once a threshold intensity is reached, a full equilibrium (isoelectric) potential for K+.
fledged action potential is produced. Further increase in the intensity of stimulus
produce no increment or other change in action potential as long as other Phases of AP
experimental condition remain constant and is said to obey "all or none law".
Summation - A single subthreshold stimulus fails to produce an AP. But if a (i) Depolarization of membrane d/1 Na+
second stimulus, which also, is subthreshold, is applied sufficiently quickly influx
after the 1st one, the two stimuli are summated & excitation results. (ii) Firing level - After initial 15 mv of
,_______________ i______________ _
depolarization, rate of depolarization
Temporal Spatial increases. This is firing level.
- When summation is - Two subthreshold stimuli applied (iii)Upstroke phase - rapid depolarization
in relation to time geographically closely but simultaneously phase of membrane. It is due to rapid
flow of Na+ into the cell.
Propagation of AP (iv) Overshoot - part of action potential
An action potential once generated at a point on nerve depolarizes the during which membrane is positive.
adjacent areas &spreads in both direction, newly depolarized area producing (v) Downstroke - Rapid return of
action potential in their adjacent areas. Thus the nerve action potential travels membrane towards its resting potential
with sam e amplitude ® is the repolarization phase or
As the AP propagates, the strength o f AP remain undiminished.Q downstroke. It is produced d/t (a)
An action potential is due to opening up Na+ channels causing Na influx®. closing o f Na+ channels (b) opening o f
Any impulse conducted antidromically up the axons dies out because, post K* channels
synaptical part o f neurons lack chem ical synaptic vesicles® that are needed (vi) Undershoot - Membrane potential
for synaptic transmission. Therefore an impulse arriving at the post synaptic becomes more negative to form small
membrane cannot release synaptic mediator. Axons can conduct impulse in but prolonged after hyperpolarization.
both direction. Dendrites are found on the axon's endings and so behave in
the same way as the axons i.e. bi-directional.
Velocity of Conduction
Conduction of nerve is an active self propagating process and impulse moves at
a constant amplitude & velocity ®. Conduction of nerve impulses, although
rapid, is much slow er than that o f electricity,®
Nerve fibers with a greater diam eter & with the myelin sheath conduct
faster,®
New action potential is fa stest in A type o f fibers® (as it is myelinated & has a
greater diameter).
Membrane Capacitance
Myelination of nerve decreases membrane capacitance 50 folds & allow
depolarization to occur very fast
A decrease in capacitance increase the speed o f action potential propagation®
Orthrodromic Conduction A ctio n p o te n tia l in a n eu ro n re co rd ed w ith o n e
Transmission at synapse is alw ays in one direction (orthodromic) only because e le c tro d e in s id e th e cell
the neurotransmitter substance responsible fo r conduction is present in vesicles
o f presynaptic neurons only®. The p ost synaptic neuron does not contain any
neurotransmitter®. It has protein molecules embedded in it which serve as
receptor for the neurotransmitter.
Susceptible to
The relative susceptibility of nerve fiber to
Hypoxia Pressure Local anaesthetics
B> A> C® A> B> C® C> B >A®
Physiology: Nerves and Muscles ■ 267

Resting Membrane Potential (RMP) Electrotonic potential (EP)

• Neurons have higher K+ concentration inside whereas higher Na+ • Electrotonus is development of local,
concentration outside the cell. This concentration gradient is nonpropogating electrotonic potential due
established by Na+K+ATPase which pumps Na+ ions out to exterior to application of subthreshold current to a
and take K+ ions inside (both against electrochemical & concentration nerve.
gradient). • A cathode current produces
• Because of this concentratioin gradient, there is passive diffusion of K+ hypo(de)polarization (ie the interior of cell
out of the cell and Na+ inside the cell. Because of higher permeability, the becomes less negative or more positive) called
K + can cross the membrane about 100 times faster than Na+. So the catelectronic potential. Conversely anodal
amount of K+ exiting out of the cell is much-much greater than the current produces hyperpolarization
amount of Na+ entering into the cell. anelectronic potential ie cell interior
• So the RMP usually between -70mv (to -95mv) is close to equilibrium becomes more negative.
(isoelectric) potential for K+Q. • At 7-15mv of depolarization (ie potential of
• During RMP The interior of cell is negatively and exterior is positively minus -55mv ) the threshold firing potential
charged. is reached and action potential develop.
- I f depolarization occurs rapidly, the opening
Accommodation of Na+ channels overwhelms the repolarizing
force and action potential result —» nerve
Only the current or stimulus which rises to its peak intensity rapidly impulse generates.
causes the action potential to develop^. The current which rises to its - I f the induced depolarization is produced
peak very slowly does not produce AP, although, at peak level, its gradually slow ly, the opening of K+ channels
strength may be adequate. This is called accommodation & occurs balances the gradual opening of Na+
because the activation or opening up of Na+ channels do not develop with channels and an action potential does not
slowly rising current strength. occur.®

Dendrite

Morphology R e c e p to r zon e: Functions of dendrites


I G ra d e d e le c tro g e n e sis I
• The neuron has 5-7 process called dendrites S ite origin of • The sites of current sources or sinks
co n d u cted im pulses
that extends outwards from the cell body and that electronically change the
arborise extensively membrane potential at initial
• Particularly in the cerebral and cerebellar cortex, segment; i.e. they are merely
the dendrites have small knobby projections Axon: extensions of the soma that extends the
called dendrites spine All or no ne area available for integration. The
transm ission effects of discharge at individual
• Neuron generally have 4 zones : synaptic endings can be excitatory or
1. Receptor /dendritic zone, where multiple local inhibitory and summation of all the
potential changes generated by synaptic excitatory and inhibitory effects in
N e rv e endings:
connections are integrated. S e c re tio n of
dendrites determine whether an
2. A site where propogated action potential are syn aptic tran sm itter action potential is generated.
generated (the initial segment in spinal Functional organization of • Very rarely propagated action potential
m otor neuron, the initial node ofR an v ier in neurons. Non-conducted local may develop in dendrite (It is important
cutaneous sensory neuron®.) potentials are integrated in the to know that most dendrites can not
3. Axonal process that transmits propagated receptor zone., and action generate and transmit action potential
impulse to nerve endings potentials are initiated at a site because their membrane have
4. The nerve endings, where AP causes release close to the receptor zone relatively few voltage gated Na+
of synaptic transmitters (arrow). The action potentials channels, so that their threshold for
• Most synapses are between terminal buttons are conducted along the axon to excitation is too high for AP to occur.
(synaptic knobs) of pre synaptic cell & the nerve endings, where they
dendrites of post synaptic cell. In cerebral cortex • Malleability of dendritic spine (i.e.
cause release of synaptic
98% of synapses are on dendrites & only 2% appear, change & even disappear
transmitters.
are on cell bodies. In spinal cord the proportion over a time scale of minutes & hours)
of endings on dendrites is less in 80% and protein synthesis in dendritic
268 ■ A Complete Review of Short Subjects

On average, each neuron divides to form over Presynaptic cell spine alters the effects of input from
2000 synaptic endings and, since human CNS M icrotubules individual glutaminergic synapses
has 10" neurons, it follows that there are about M itochondria (NMDA & AMPA receptors) on the
2 x 1014 synapse. C lear vesicles dendritic spine. These selective
changes mediate learning and long
Transmission in synapses may be very rarely P ostsynaptic Active term p oten tiation (LTP)®.
d en sity zone
electrical (low resistance bridge through which • The AP is generated in the initial
ions passes easily) e.g. in lateral vestibular segment (the p ortion o f axon a t and
Postsynaptc cell
nucleus or chemical (mostly). ju st beyon d the axon hillock®), and
D endritic spine
its discharge is propagated in two
Synaptic cleft is 20-40nm® wide. Across the A xodendritic directions : down the axon and back
D endrite
cleft are many neurotransmitter receptors in into soma. Retrograde firing of the
postsynaptic membrane in thickened zone A xodendritic soma in this fashion probably has
called postsynaptic density. Similarly active value in "wiping the slate clean" for
zone containing many proteins & rows of subsequent renewal of the interplay
calcium channels is seen as thickened A xosom atic of excitatory & inhibitory activity on
membrane in presynaptic membrane. the cell.
Ca++ that triggers exocytosis of transmitter • They can transmit signals by electronic
from synaptic vesicle enters the presynaptic A xo-axonal
conduction down the dendrite to soma
neurons (active zone) and transmitter release but there is decrement of postsynaptic
A xodendritic, axoaxonal. and
occurs starts in 200-500/j s . The process by axosom atic synapses. Many potential during transmission due to
which synaptic vesicle fuse with the cell presynaptic neurons term inate on thin & permeable dendrite membrane
membrane involve V-snare protein dendritic spines, as shown at the that leak K+ & Cl
synaptobrevin in vesicle membrane locking top, but som e also end directly on
the shafts of dendrites. Note the
• Current flow to & from the active
with the t-snare protein syntaxin in the cell
presence of clear and granulated synaptic knob on dendrites produce
membrane. The organization of synapses
synaptic vesicles in endings and w av e activ ity in EEG®.
depends on neurexins protein bound to
clustering of clear vesicles at active
membrane of presynaptic neuron.
zones.

★ Several toxins which block neurotransmitter release are zinc-endopeptidases that cleave & hence inactivate proteins in the
fusion-exocytosis complex. Tetanus toxin & botulinum toxin- C, D, F, G act on syntobrevin & botulin toxin C acts on syntaxin.
Botulinum toxin A & B act on SNAP-25. Clinically, tetanus toxin causes spastic paralysis by blocking presynaptic
transmitter release in CNS and botulism causes flaccid paralysis by blocking the release of acetyl choline (ACh) at
neuromuscular junction.

Nlssl Body (or Nissl Granule or Tigroid Body)

Named after Franz Nissl, a German neurologist Nissl body (or Nissl granule or tigroid body) is a large granular body
found in the soma, (or perikaryon, or cyton, or cell body, the bulbous end of a neuron, containing the cell nucleus)^
and dendrites of neurons, though not in the axon or axon hillock.

These granules are rough endoplasmic reticulum (with free ribosomes) and are the site of protein synthesis involved in the
synthesis of neurotransmitters such as acetylcholine.

Nissl bodies can be demonstrated by a method of selective staining developed by Nissl (Nissl staining), using
an aniline stain to label extranuclear RNA granules. This staining method is useful to localize the perikaryon / cell
body, and dendrites of neurons. Due to RNA's basophilic ("base-loving") properties it is stained blue by this method.

Nissl bodies show changes under various physiological conditions and in pathological conditions they may dissolve and
disappear (karyolysis).
Physiology Nerves and Muscles ■ 269

Electric Events In Post Synaptic Neurons

Excitatory Post-Synaptic Potential (EPSP) Inhibitory Post Synaptic Potential (IPSP)


Excitatory post synaptic potential (EPSP) or Hyperpolarization of postsynaptic membrane makes it resistant
generator (receptor) potential (which is EPSP in to stumuli (i.e. it decreases the cell's excitability or probability of
Pacinian corpuscles) is a nonpropagating firing the AP) and is k/a inhibitory post synaptic potential
localized depolarizing potential, the magnitude (IPSP).
o f which is proportional to the intensity o f Like EPSP, IPSP peak 11.5ms after stimuli and decrease
stimulus (i.e. the amount o f transmitters released exponentially.
by the pre synaptic neurons)®. So EPSP or Produced by opening of C l channels. CP influx increases
GP(RP) are graded potentials rather than "all or intracellular negativity and leads to hyperpolarization. Also
none" action potential. produced by K+efflux and closure of Na+ or Ca++ channels.
Small depolarization (hypopolarization) Decreased excitability during IPSP (hyperpolarization) is due to
changes in membrane of post synaptic cells are movement of membrane potential away from the firing level. So more
termed excitatory (=EPSP), If they increase the depolarization (excitatory) activity is required to reach firing level.
cell's excitability (ie probability of firing AP). The membrane potential, at which there is no EPSP (or EPSC =
EPSP begins about 0.5 ms after the afferent excitatory post synaptic current), or the postsynaptic potential
impulse reach the spinal cord; peaks 11.5ms after disappears is k/a reversal potential. When the membrane
the stimulus and declines exponeatially. potential is at equilibrium potential for chloride Eci= -70 mV),
Produced by opening of N a+ or Ca++ channels® the EPSP disappears and at mores negative membrane potentials, it
producing an inward current. EPSP is localized, becomes positive (=reversal potential). This proves that IPSP is d/t
nonpropogated, inscribed and summated. CHnflux.
EPSP's are directly proportional to the strength Synaptic Delay
o f initial stimuli (amount o f transmitter released When an impulse reaches the presynaptic terminals, an interval of
by the presynaptic neurons)®. at least 0.5 ms, occurs before a response is obtained in the
It does not obey all or none law. Stronger the postsynaptic neuron. It is because of time taken by synaptic
stimulus greater will be EPSP. Strictly localized, mediators to be released & to act on receptors on membrane of post
that It is not self propagating® (unlike AP). It synaptic cell.
develops as a result of entry of all cations inside Synaptic delay across one synapse is 0.5ms; so it can determined
the cell, it is forerunner of an AP. that the reflex pathway is mono or poly-synaptic.
Excitatory
presynaptii

Increased permeability of terminal


excitatory axon to C’ and K ions
causing C l' influx and K* efflux GABA
(by voltage gated K+ chanels)
Inhibiting neuron
release inhibitory
j v ^ [ _ substance (GABA)
on to the outside of
synapses \ presynaptic nerve
^ fibrils before their
The cancel much of the excitatory own endings terminate
effect of positively charged sodium on post synaptic neuron
ions that also enter the terminal
fibrils when an action potential Post synaptic
| arrives__________________________ neuron

Presynaptic axo-
axonal indirect inhibition
Decreased size of action potential
reaching the excitatory ending

Decreased neurotransmitter
(excitatorv) release

Decreased Ca* entery and


consequently decreased amount
of excitatory neurotransmitter Decreased or abolished
release from excitatory pre excitatory post synaptic
synaptic nerve ending 1/t potential (E P S P )
decreased EPSP
270 ■ A Complete Review of Short Subjects

Synaptic Inhibition

Preaynaptic Inhibition Postsynaptic Inhibition itenshaw Cell Inhibition


• This is also called • This is also called direct • Negative - feed back innwuion o j a spinal m otor neuron
indirect inhibition. (reciprocal) inhibition via an inhibitory intemeuron (Renshaw cell).®
• It occurs d/t failure o f because it is not a Interneuron releases glycine which is inhibitory in
presynaptic axon consequence of previous function.
terminals to release the discharge of post synaptic • Renshaw Cell Inhibitory System is located in the ventral
excitatory neuron. Where as indirect horns of the spinal cord in close association with the
neurotransmitter inhibition is d/t effects of motor neurons. Impulse generated in motor neuron activate
substance Q previous post synaptic the inhibitor interneurons (Renshaw cell) to secrete inhibitory
• It occurs by axo - axonic neuron discharge (eg. mediator & this slows /stops the discharge of motor
transm issionfi Postsynaptic cell can be neuron.
refractory to excitation
• Mechanism:
because it has just fired & is
- GABA mediated increase
in refractory phase).
in CT conductance
• It occurs d/t release o f an
causing CT influx
inhibitory neuro­
- Opening of voltage gated transmitter causing post­
K+ channels causing K+ synaptic neuron to
influx hyperpolarize (IPSP)®.
- It causes decrease in • M echanism:
EPSPQ d/t decreased AP,
- Inhibitory neurotransmitter
decreased Ca++ entry &
decreased neuro - (GABA & Glycine) acts on Negative feedback inhibition of a spinal motor neuron via
transmitter release fi postsynaptic membrane & an inhibitory neuron (Renshaw cell)
- Direct inhibition of voltage produce inhibitory post
gated Ca++ channel synaptic potential (IPSP)
Feed Forward Inhibition
opening (necessary for Antagonistic drug:
exocytosis of Strychnine®
Seen in the cerebellum ,® stimulation of basket cells produced
neurotransmitter release) • Occurs in myotactic stretch
IPSP, in the purkinje cells. However the basket cells & the
- Direct inhibition of reflex. For example if a
purkinje cells are exited by the same parallel excitatory
neurotransmitter release skeletal muscle is
input.
(independent of Ca++ stimulated to contract by
influx mechanism) sudden stretch the Quadriceps

• Antagonistic drue: opposing muscles are (extensor)

inhibited (reciprocal
- Picrotoxin® inhibits
inhibition). Afferent axons
presynaptic inhibition
from homonymous muscle
- L ocal anaesthetic do not secrete excitatory
affect presynaptic glutamate at their
inhibitionfi terminals. Direct
• Occurs in sensory inhibition is achieved by
pathways where adjacent some of the branches of
nerves often mutually afferent nerve that synapse
inhibit one another with inhibitory Golgi
Presynaptic Facilitation j S:
which minimizes bottle intemeuron which
sideways spread & secrete glycine to inhibit Presynaptic Facilitation is produced when the AP is
mixing of signals in antagonist motor neuron. prolonged and calcium channels are open for a longer
sensory tracts. periodQ. Serotonin released at an axonaxonal ending
increases c-AMP & resulting phosphorylation of K+
channel closes the channel, slowing repolarization &
prolonging the AP.
Physiology: Nerves and Muscles ■ 271

Transmission In Sympathetic Ganglia: Fast & Slow Responses of Postganglionic Neurons In


________________________________ Sympathetic Ganglia_________________________________

Potential Duration Mediator Receptor Special features


FastEPSP 30 ms Acetylcholine Nicotinic cholinergic - Rapid depolarization that generates
action potential
Slow IPSP 2s Dopamine d2 - d/t increase in K+ conductance.
- Dopamine is secreted by intensely
fluorescent cells (SIF cells) in ganglia
Slow EPSP 30s Acetylcholine M2 cholinergic - d/t decrease in K+ conductance
Late slow 4 min G nRH G nRH Late slow EPSP has a latency of l-5s and
EPSP last for few minutes.

Sensory Information

Law of Projection Localization Muller's Law of Specific Nerve


energies
- States that, no matter where A single sensory axon & all its peripheral branches are
a particular sensory pathway termed sensory units . The receptive field of a sensory The sensation evoked by specific
is stimulated along its course unit is the spatial distribution from which a stimulus sensory pathways are discrete
to the cortex, the conscious produces a response in that unit. from sense organ to cortex.
sensation produced is The size of receptive field for light touch can be States th a t, no matter how or
referred to the location of the measured y two point threshold test and it tests the where a particular sensory pathay
receptors. integrity of dorsal column (medial lemniscus) system, for particular sense organ is
■Phantom limb is an example the central p a th w a y fo r touch and proprioception® . The stimulated along its course to the
of law o f projection®. The minimum distance between the two caliper poins that cortex, the sensation evoked is
patient complains of pain & can be perceived as separate points of stimulation is that for which the receptor is
proprioceptive sensation in called two points discrimination threshold and is a specialized.
the absent limbs, because the mesure of tactile acuity.
For example, if sensory nerve
end of the nerves cut at the
Information from sensory neurons whose receptors are from a pacinian corpuscle in hand
time of amputation are at the peripheral edge of the stimulus is inhibited is stimulated by pressure at elbow
stimulated by pressure on
compared to information from the sensory neurons at or by irritation of tumor in
them. The impulses the center of stimulus. This phenomenon is k/a lateral brachial plexus or by stimulation
generated in these nerve
inhibition and it enhances the contrast between the of appropriate fibers of dorsal
fibers came from sense center & periphery of a stimulated area and increases columns of spinal cord, thalamus
organs in the amputated the ability of brain to localize a sensory input or post central gyrus of cerebral
limb, thus the sensation felt
L ateral in hibition is resp on sible f o r lo ca liz a tio n o f a cortex, the sensation produced
are projected to where the
stim ulus site and tw o p o in t discrim ination® . would be touch.
receptors used to be.

Judgement of Stimulus Intensity

Weber - Fechner principle - Detection of ratio of stimulus strength Power Law


It states that gradation of stimulus strength are discriminated approximately in proportion to States that a person
logarithm of stimulus strength. In other words it emphasizes that greater the background interprets changes in
sensory intensity, the greater an additional change must be for the psyche to detect the change. intensity of sensory stimuli
For example a person already holding 30gm weight can barely detect an additional 1 gm approximately in proportion
change and similarly when holding 300 gm can barely detect 10 gm change (the ratio to a power function of the
remained same in both) actual intensity.
Weber-Fechner law: states that the m agnitude o f the sen sation fe lt is p ro p o rtio n a te to
the log o f intensity o f the stimulus® Interpreted signal strength
= K x (Stimulus - k)y
Interpreted signal strength =
Log x stimulus intensity + Constant
272 ■ A Complete Review of Short Subjects

• There are three methods by which brain detects intensity of a sensory stimulus (differentiates (exponent y and the constant
b/w two stimuli of different - intensities). If a greater or lesser pressure is applied to K & k are different for each
skin there will be variation (increase or decrease respectively) in type of sensation)
- the receptor potential in the mechanoreceptor®. • When plotted on a graph
- the frequency o f the action potentials® generated by the stimulus in a given receptor. using double logarithmic
- the number o f receptors activated by the stimulus®. coordinates, a linear relation
• It now appears that a power function more accurately describes this ratio can be attained between
r = k s4 interpreted stimulus strength &
where R = senstion felt, actual stimulus strength over a
S = intensity of stimulus and K & A = Constants large range for almost any
• This is quantitatively accurate only for higher intensities of visual, auditory & cutaneous type of sensory perception.
sensory experience & applies only poorly to most other types of sensory experiences.
• Frequency of action potential generated in sensory nerve fiber is also related to intensity
of initiating stimulus by a power function.

Miniature End Plate Potential

Miniature end plate potential does not result in firing of muscle fibre
Its about 0.5 mv in amplitude.
- The size of quanta o f Ach released varies directly with Ca2* concentration & inversely with M g2* conc. At the end plate.
When a nerve impulse reaches the ending, the number of quanta released increases by several order of magnitude, and the
result is the large end plate potential that exceeds the firing level of muscle fiber.

Generation of Action Potential In Postsynaptlc Neuron

• The axon (devoid of Nissl granules) starts from a short pyramidal


shaped part of cell body k/a axon hillock. In myelinated axons, the
part of axon between axon hillock and beginning of myelination is
k/a in itia l seg m en t. So initial segment is the portion o f axon at and just
beyond the axon hillock.
• In m y e lin a te d m o t o r neu ron s, th e u n m y lin a ted in itia l seg m en t is th e
s ite o f in itia tio n o f p ro d u c tio n o f a c tio n p o t e n t ia l (fu ll fle d g e d ) an d
p r o p a g a t io n o f n erv e im pulse® . In cutaneous sensory nerves, the site
of initiation of nerve impulse is I 91 node of Ranvier.
• Action potential begins in initial segment and not adjacent to
excitatory synapses of soma because the initial segment membrane
has 7 times greater concentration o f voltage gated sodium channels as does
the soma. So the initial segment has lo w e s t th r e s h o ld o f excitation ® .
The excitatory post synaptic potential (EPSP) that will elicit an action
potential in initial segment is + 10 to + 20 milivolts whereas > + 30 to +
40 mV is required for soma.
• When the 10 - 1 5 m V o f depolarization sufficient to reach firing level is
reached and impulse is propogated in two directions: down the axon &
back into the stoma. Retrograde firing of soma has value in wiping the
state clean fo r subsequent renewal of interplay of excitatory &
inhibitory synaptic knobs.
Physiology: Nerves and Muscles 273

Transmission At Neuromuscular J u n c tio n ^ 7 Physiological Anatomy of NM Junction


I-----------------
Presynaptic Nerve Ending Presynaptic Nerve Post synaptic Motor
Nerve impulse reach NM junction —» Increased Ca++ Ending (End feet or End Plate (Sarcolemma)
permeability of presynaptic neural membrane d/t opening of Terminal Button) - Depress or invaginates to
voltage gated Ca++ channels —> Influx of calcium from - Contains many - Contain form synaptic trough or
synaptic space to interior of presynaptic nerve ending —> mitochondria which large synaptic gutter in which
Calcium ion attracts acetyl choline (ACh) vesicles and draws provide ATP for quantities nerve end feet fit in
/fuse them to neural membrane adjacent to dense bars leading to synthesis of ACh in of actyline - Subneural clefts are
a m arked increase in exocytosis/release o f ACh in synaptic cleft® cytoplasm choline neumerous small folds of
I - Contain many small esterase muscle membrane
Synaptic Cleft clear synaptic vesicles enzyme increasing the surface
that rapidly absorb. which area.
ACh diffuses through synaptic space and binds to nicotinic
ACh destroys - Contain nicotinic (Nm)
cholinergic (Nm) receptors in the post synaptic motor end
-Contain voltage gated ACh cholinergic receptors
plate. (ACh receptors are concentrated at the top of subneural
Ca++ channels
clefts = junctional folds) —» Binding of ACh to ACh receptors
R elease V esicles
(which are ligand gated sodium channel) increases Na+ and K+ sites
conductance of muscle cell membrane.

In Post Synaptic Motor End Plate (Muscle Membrane) Dense bar


Influx of positively charged Na+ ions into the muscle fibre Calcium
creates a local positive potential change inside the muscle fiber channels
Basal lam ina and
i.e. end plate potential (EPP) -> Current sink created between
a cetylcholinesterase
depolarized end plate (with EPP) and adjacent muscle plasma Acetylcholine
membrane on (both sides) initiates an action potential (ie receptors
depolarizes the adjacent muscle membrane to its firing level) —» voltage activated
Action potentials are generated on either side of end plate and Na* channels
are conducted away from end plate in both directions along the
Junctional folds
muscle fiber —> AP (depolarization) spreads to all parts & to D H P receptor

interior of muscle fibers by way of T (transverse) tubules


(communicate externally with ECF, contain ECF and are actually
internal extension of cell membrane) —» As AP reaches T-tubules,
the voltage change is sensed by DHPR (dihydro pyridine
- C a + + release
receptors) on T tubule —» DHPRs are physically linked to (raynodine
calcium release (ryanodine receptor) channels in the adjacent channel)

sarcoplasmic reticular (SR) cistemae. AP in T-tubule cause


- Calsequestrin
conformational change in voltage sensing DHP receptors,
which unlocks (opens) the Ca-1"1- release channels in terminal i S R
cistern of SR and permitting Ca++ to rapidly diffuse into the
Action potential
sarcoplasm (thick & thin filament). The calcium induced Ca++ (depolarization)
Repolarization

release quickly amplifies the release of Ca++ —>Binding of Ca++


to troponin C, uncovering myosin binding sites on actin —» (T ) M o to r n euron
a ctio n p o le tia t

Formation of cross linkage between actin and myosin and


S yn a p tic vesicle
sliding of thick on thin filaments producing movement. ) C a en te rs co n ta in in g A C H
v o lla g e -g a le d
I ch a n n e ls ) P ro p a g a te d A P
in m u scle pla sm a
Acetyl choline degradation in synaptic cleft: ACh is rapidly m em b ra n e

removed by acetyl choline esterase, which is present in high Sr* **> _


concentration in synaptic space. This prevents continued muscle
re-excitation after muscle fiber has recovered from initial AP.
Summary of Muscle Relaxation
Calcium ions are pumped back from sarcoplasm (myofibrillar
fluid) into sarcoplasmic reticulum by an ATP dependent Ca++
pump, concentrating Ca++ ions about 10 thousand folds inside the
tubules. Calsequestrin protein that can bind upto 40 times more
Ca++is found inside SR -> Decreased Ca++ ion concentration in
( § ) Lo cal cu rre n t be tw ee n
sarcoplasm and release of Ca++ from troponin —» Cessation of d e p o la rize d e n d p la te an d
a d ja c e n t m u s d e p la s m a
interaction between actin & myosin causing muscle relaxation^. m e m b ra n e
274 ■ A Complete Review of Short Subjects

Sacrolemmal Proteins Perlecan

Sarcolemma is the cell membrane of muscle fiber. It contains several integral - Perlecan is a large
(transmembrane) & peripheral proteins which form complexes. Defect of these proteins & multidomain
complexes results in muscular dystrophies. proteoglycan, which is
1 synthesized by both
Proteins Proteins & glycoprotein complexes vascular endothelium
I___ I & smooth muscle cells
I------------- 1
Integral (Transmem-brane) Peripheral - Integrin complex & deposited in
protein proteins localizes - Dysferilin - caveolin complex extracellular matrix.
I to the - Dystrophin- dystroglycan - - It binds to & cross link
- Dystroglycans (DG)® cytoplasmic side sarcroglycan complex adds many extracellular
(jl-dystroglycan) I strength to muscle by providing a matrix components &
- Sarcoglycans (SG)® (a, p, y, 8) - Dystrophin® scaffolding for the fibrils & cell surface molecules.
- Caveolin- 3 - Dysferlin connecting them to extracellular - It is a key component
- Integrins (|3, a7) - Calpain environment. of vascular ECM. It
promotes growth
factor (FGF2) activity,
stimulate endothelial
The dystrophin-glycoprotein complex. Dystrophin growth & regeneration
connects actin to the two members of the and maintain
dystroglycan (DG) complex, a-and P-dystroglycan, endothelial barrier
and these in turn connect to the merosin subunit of function.
laminin in the extracellular matric. The sacroglycan - In smooth muscle it is
(SG) complex of four glycoproteins, a-, P-, y- and 8- a potent inhibitor of
sacroglycan, are associated with the dystroglycan cell proliferation and
complex. maintains vascular
homeostasis.

Mechanism of Contraction of Skeletal Muscle


• Mechanism of contraction: when Ca2+ are released during the contraction process, the troponin complex undergoes a
confirmational change that in some way shifts the tropomyosin molecules into the groove between the two actin strands.
This uncovers the active sites on actin thus allowing myosin to bind the actin & contraction proceed.
Myosin—[

Troponin

Active site
Ca++
myosin
binding

Actin Tropomyosin (covers Ca binding shifts tropomyosin


active sites of actin laterally uncovering active sites
where myosin and allowing actin-myosinbinding
head binds)

In resting skeletal muscle tropom yosin (a long filam entous protein) covers the active sites o f actin filam ent where myosin
head binds to actin ®. So that the attraction cannot occur between actin and myosin filaments to cause contraction.
Initiation of muscle contraction occurs, when Ca** binds toroponin C®. Binding causes lateral displacement of tropomyosin
into the groove between two actin filaments. This uncovers active sites on actin thus allowing myosin head to bind the
actin and contraction proceeds.
7 myosin binding sites (on actin) are uncovered fo r each molecule o f troponin that binds a calcium ion®.
Physiology; Nerves and Muscles ■ 275

» An action potential (AP) develops on sarcolemma (cell membrane)—> AP proceeds & enters the T tubule —>comes in
contact with the cistem (dilated sarcoplasmic reticulum close to T tubules, store Ca++) —» release of Ca2+ -» combine with
troponin C—> this leads to loosening of tie between troponin & actin molecules —> this causes release of tropomyosin from its fixed
position —» myosin binding sites of actin are uncovered —> myosin heads get attached with actin -4 contraction
^ Troponin Active site
S ile fo rC a binding ^
• In the contracted state.
I) H zone is greatly narrowed or disappears®
II) Width o f I band is reduced.®
III) Width o f A band remains unchanged.®
IV) The muscle fiber as a whole shortens.

A= Showing double helix of actin (resting state) polymer.


B = Changes seen with the onset of contraction
C = Actin monomer

Site for binding


Actinin - Binds actin to Z line with myosin head
Titin - Connects Z line to M line and provides
the scaffolding for sarcomere
Desmin - Binds Z line to plasma membrane Site for attachment
with troponin

★ Depolarization of T tubule membrane activates the sarcoplasmic reticulum via dihydropyridine receptors (named after
drug which blocks them). They are voltage gated Ca++ channels in the T-tubule membrane. In cardiac muscle Ca++ influx via
these channel triggers Ca++ release from sarcoplasmic reticulum. But in skeletal muscle, Ca++ entery by this route (from
ECF) is not required for Ca++ release. Here these receptor serves as voltage sensor & trigger that unlocks Ca++ release from
sarcoplasmic reticulum (SR). The Ca++ channels of SR are not voltage gated & k/a ryanodine receptor (locked in open
position by plant alkaloid ryanodine).

Slow and Fast Twitch Fibers


- Type I slow twitch red muscle fibers are characterized by smaller (lesser) diameter, number, force produced, velocity of
contraction and fatigability. It is innervated by smaller nerve fibers with lesser conduction velocity. Slow twitching muscle
fibers derive energy primarily from oxidative phosphorylation and therefore their mitochondrial content, capillary (vessel)
density and amount of myoglobin (responsible for red color) are high. Their glycolytic capacity, calcium pumping capacity
of sarcoplasmic reticulum and myosin ATPase activity (rate) is lesser in comparison to fast twitch muscles.
- Type II fast twitch white muscle fibers are characterized by large diameter and more num ber of fibers for higher force &
velocity of contraction. It derives energy primarily from glycolytic pathway and therefore they have large amount of
glycolytic enzymes but have less extensive blood supply, fewer mitochondria and deficit of red myoglobin (responsible for white color) as
oxidative metabolism is of secondary importance. Fast fibers have extensive sarcoplasmic reticulum for rapid release of
calcium ions, greater SERCA 1 (than SERCA2) activity, and higher & quicker calcium pumping & reuptake capacity of SR
(responsible for faster contraction & relaxation). Fast fibers begin to develop tension at higher Ca++ concentration (than
slower fibers) because of 2 low affinity calcium binding sites in troponin C (as compared to 1 in slow fibers)
* SERCA 2 is greater in slow twitch & cardiac muscles

Smooth Muscles

Types: Two M echanical Properties


• Unitary (Visceral) - Occur in large sheets, with many low It can contract & relax continuously without any obvious
resistance gap junctional connections between individual stimulation, independent o f its nerve supply® i.e. it has
muscle cells intrinsic rhythm.
- Nerve fibers on fewer cells, with excitation spreading to • Length tension relationship
other cells by gap junction i.e. it functions in a syncytial - Which is characteristic of skeletal & cardiac muscles does
fashion. not exist in visceral smooth muscles^.
- Found in wills of hollow viscera such as alimentary tract - When stretched, smooth muscle initially contrast & exerts
including gall bladder & ducts of digestive glands, ureter, tension, but then it gradually relaxes & adopts the new
urinary bladder, uterus and blood vessels. length with no increase in tension; a phenomenon k/a
• M ultiunit: - Made up of individual units with few (no) gap plasticity or stress relaxation and is d/1 latch- bridges
junctions. formation.
276 ■ A Complete Review of Short Subjects

- Each multiunit muscle cell has on passant endings of nerve - On removing the stretching force, the muscle do not return
fibers to its previous size until active contraction is induced.
- Found in intrinsic muscles of eye (cliary body & iris), piloerector • Slow & prolonged contractile response:
muscle and in precapillary sphincter. So blood vessel has both - Latent period (& therefore excitation contraction coupling) is
type of smooth muscle. much longer than that of skeletal & cardiac muscle
500 ms delay in smooth muscles as compared to 10 ms delay
Structural Differences
in skeletal & cardiac muscles.
• Lacks visible cross striations (unlike cardiac & skeletal
muscles) Mechanism of Contraction
• Gap junctions & functional syncytial behaviour is seen in
unitary smooth muscles
D ifferent source of calcium
• Sarcomere arrangement is absent. Instead of z lines (of skeletal
As in skeletal & cardiac muscle, Ca++ plays a important role
muscle), there are free floating dense bodies which contain
in initiation of contraction but the source of Ca++ increase is
aactinin (that bounds it to actin filaments). Some of them much different
are attached to cell membrane Unlike skeletal & like cardiac muscle m ost o f the calcium
• Thick filaments are fewer than thin filaments and contain which enters sarcoplasm comes from ECF and very little
myosin. Unlike skeletal muscle there is no troponin in thin comes from sarcoplasm ic reticulum®. So the source of Ca++
filaments, which are made up of actin & tropomyosin. are:____________________________________________________
Instead a Ca++ binding calmodulin protein is present. 1. Influx through voltage or ligand gated plasm a
• Sarcoplasmic reticulum is present but less extensive. There are membrane Ca** channels®
no T tubules, contain few mitochondria and depends, to a large 2. Efflux from intracellular stores through RyR
extent, on glycolysis for their metabolic needs. Caveolae 3. Efflux from intracellular sarcoplasm ic stores through
(small invaginations of cell membrane) are present. IP3 receptor Ca** channels® _________________________
•No neuromuscular junction with motor end plates (as in skeletal
muscles); instead the terminal axon loose their myelin . Phosphorylation of myosin is essential for its binding to
sheaths and branch into fine diffuse endings with actin
enlargement along axon k/a varicosities, from which the Lack of troponin in smooth muscles prevents Ca++ activation
neurotransmitter is released onto the interstitial fluid few via troponin binding. Rather Ca++ binds to calmodulin and
nm away from cell membrane. resulting complex activates calmodulin dependent myosin
light chain kinase (MLCK). This enzyme catalyzes the
Electrical Properties
phosphorylation of myosine light chain on serine at position
• Less excitable (i.e. higher stimulation threshold and longer 19
chronaxie) than skeletal muscle Myosin LC20 phosphorylation is a must and increases
• A unique property of visceral smooth muscle is myosin ATPase activity® resulting in binding of myosin to
depolarization when it is suddenly stretched; the resulting actin and contraction . However, phosphorylation &
action potential 1/1 contraction. dephosphorylation of myosin also occur in skeletal muscle
• Variable RMP & shifting pacemakers. but phosphorylation is not necessary for activation of
- Unitary muscle is characterized by instability of its resting ATPase
membrane potential which is generally low ranging from - 2 0 • Dephosphorylation & Latch bridges
to - 60 mV, and is unstable with no true resting value. Myosin LC is dephosphorylated by myosin light chain
- Spontaneous discharge of impulse is organized into basic phosphatase but it does not necessarily l/t relaxation &
slow wave rhythm of membrane potential, (slow sine wave fin al detachment® of myosin cross bridges from actin until
like fluctuations) the intracellular concentration of Ca++ falls below a critical
- Unlike cardiac muscle, there is no fixed location of pacemaker level.
and several shifting pacemakers can exist at one time. This continued attachment of myosin cross bridges to actin
• Excitation may or may not be preceeded by AP even after dephosphorylation creates latch bridges
- Variable types of action potentials: Latch bridges do not m ake significant cycles o f attachm ent
- AP is generally low amplitude about 60 mV (as compared & detachment® between actin & myosin so do not require
with 120 mV) of skeletal muscle); there may be no reversal of much ATP® and so maintain prolonged contractions without
polarity (overshoot) expenditure of much energy.
- AP may take form of spikes (as in skeletal muscle) or exhibit - If epinephrine or NE is added, the membrane potential
a prolonged platue phase during repolarizaion as in cardiac becomes larger, the spikes decrease in frequency & muscle
muscle) relaxes. Whereas acetylcholine addition 1/t decreased
- Unlike skeletal muscles (which depolarize by opening membrane potential, spikes become more frequent, muscle
voltage gated Na channel) sm ooth muscles have few Na becomes more active with an increase in tonic tension &
channels and they open voltage gated Ca** channels to let number of rhythmic contractions.
in Ca** from ECF into cell to cause AP®.
Physiology: Nerves and Muscles ■ 277

Calmodulin Calcium Binding Proteins


• It has 148 amino acid residues & 4 Ca++ binding domains. It's 115th aa. Troponin
is trimethylated. It is extensively conserved & found in plants & Calmodulin
animals both. Calmodulin is involved in contraction o f sm ooth Calbindin
muscle.® It helps in initiation of contraction, not by, uncovering active
site on actin molecule but by activating the myosin cross bridges. ★ Calcineurin: is calmodulin activated
• Is a Ca2* binding protein® present in many cells notably in sm ooth protein, a phosphatase that inactivates
muscles and brain cells®. Ca2+ channels by dephosphorylating
• Is a counterpart of Troponin. (Troponin is present in skeletal muscles) them. It plays a role in activating T-
• Calmodulin binds with Ca2+ & capable of activating 5 different
cells.
calmodulin- dependent kinases ®. They are:
(i) M yosine light chain kinase® - it phosphorylates myosin head
conferring with the capability of binding with actin filament. This
brings about contraction in smooth muscle.
(ii) P hosphorylase kinase®
(iii) Ca2+/Calmodulin kinase I & II: concerned with synaptic function
(iv) Ca2+ /Calmodulin kinase III: concerned with protein synthesis

Factors Responsible For Grading of Muscular Activity

Number of motor units With minimal voluntary activity only a few motor units discharge. With increasing voluntary
(Recruitment) effect more and more units are brought into play - 'Recruitment phenomenon'.
With increasing motor units, force of muscle contraction increases
Frequency of Contraction Frequency of discharge in the individual fibre play a role. Tension developed during a tetanic
(Summation) contraction (with rapidly repeated stimulation) is greater than during individual twitches
Large motor units Force of contraction depends on length of muscle - starling's law

Malignant Hyperthermia

This is a life threatening event characterized by uncontrolled release o fC a *2 ions from the sarcoplasm ic reticulum®.

Association Clinical features Triggering agents Treatment


With mutation in the gene • Severe • Succinyl choline • Immediate cessation of
Ry - Ri gene encoding for hyperthermia® • Halogenated anaesthesia
the skeletal muscle • Contracture, rigidity anaesthetic • /.V. dantrolene®
ryanodine receptor • Tachycardia - Halothane • Procainamide (because of
• Metabolic acidosis - Isoflurane possibility of VF)
- Sevoflurane • Rapid cooling
• 100% oxygen
Chapter 3. RESPIRATORY SYSTEM: REVIEW NOTES

Systemic Arteriole Systemic Venule Interpretation W hat Happens in Systemic


Capillaries
Oxygenated blood with more PO 2 Deoxygenated blood with more - Decrease in O 2 & increase in CO 2 content
PCO 2 - O 2 dissociation curve shift to rights
- CO 2 dissociation curve shift to leftQ
- Less H + in RBC - More H+ ion in RBC - Increase in HCO 3 in plasma & RBC
- Less HCO 3 ion in plasma & RBC - More HCO 3 ion in plasma & RBC - Increase H + & CI" content in RBCQ
- Less Cl" content of RBC - More CT content of RBC - PH decrease®
- pH 7.40 - pH 7.36
Small amount of fluid is filtered out - Because of more osmotically - H em atocrit increases by 3%®
and returns via lymphatics (which active ions (CT, HCO 3 ) RBCs - Protein concentration increases®
decreases its hematocrit)
swell up by taking water
(increases hematocrit)
★ In lung all these effects are reversed in pulmonary capillaries.

P50
• P 50 is the index of affinity of hem oglobin for O 2 . H igher the P 50 , the lower the affinity of hem oglobin for O 2 .
• Under normal condition when: 1) Hb is norm al 2) PaCC>2 is 110 mm Hg 3) Tem perature is 37°C and 4) 2,3 DPG is 15
pmol/gm of Hb. The value o f P 50 is 25 mm Hg ~ 3.6 KPa®. (1 mm Hg = 0.14 KPa)

Alveolar-Capillary Barrier (ACB)

• ACB is the structure through which the gases (like O 2 &: CO 2) must diffuse from alveoli to reach pulmonary capillary blood. It
includes
- Alveolar surfactant, alveolar epithelium and basement membrane
- Capillary endothelium, plasma
- RBC membrane, intra erythrocyte fluid and hemoglobin
• In healthy individuals, the distance b /w alveolar air and pulmonary capillary blood is about 0.3pm.

Capillary Transit Time (CTT)


- It is the time that the RBC spends in pulm onary capillary bed (or time taken by blood to flow from pulm onary artery to
pulm onary vein.)
- CTT is about 0.75 seconds, whereas gas exchange (for O 2 & CO 2) between alveoli and blood is complete (i.e. equilibrium is
attained) within 0.25 seconds, which means flow rate is adequate for full Oxygenation and CO 2 transfer. Thus O 2 & CO 2
transfer is normally perfusion limited.

CO Transfer Factor (TLco) and Diffusion Capacity of Lung (Dlcq) for CO


Carbon monoxide transfer factor ( T lc o ) term is used for diffusion capacity of CO ( D lc o ) because the measurement does not solely
reflect the diffusion properties of lung (ie area and thickness of blood gas barrier). But it is also dependent on volum e of blood in
pulm onary capillaries and alveolar volum e. T l c o is product of (=) alveolar volume and (X) transfer coefficient ( K c o ) . Diffusion
capacity of O2 (DL02) is 1.23 times of Dlco- The average D l c o is 17 m l/m in /m m Hg; so the D lo 2 is 1.23 times of this i.e. 21
m l/m in /m m Hg. Causes of
Increased T l c o and Kco Decreased T l c o and Kco Decreased T l c o but increased Kco
Factors which increase number of RBC Factors which decrease number of RBC Factors that cause increased density of
that come in contact of CO such as that come in contact of CO such as anemia, blood per unit lung volume eg.
polycythemia, left (L) to right (R) R—»L shunt, pulm onary em bolism , extrapulmonary restriction like
shunt®, alveolar hemorrhage, alveolar prim ary pulmonary hypertension, thoracic cage deformity, respiratory
inflammation, exercise & asthma. interstitial lung disease, lymphangitis, muscle weakness, pleural disease.
obstructive airway disease.
Physiology: Respiratory System ■ 279

Oxygen Transport In Blood


Oxygen is carried in blood in 2 forms : dissolved in plasma and chemically combined with hemoglobin
Oz Chemically Bound to Hemoglobin O2Dissolved in Plasma
O 2 forms an easily reversible combination with hemoglobin (Hb) - Only 3% of O 2 is transported in dissolved state in water
causing conformational change in deoxy Hb (tense /T state) to of plasma and cells. This obey's Henry's law i.e. amount
give oxyhemoglobin (relaxed/R state). 97% of O 2 in blood is dissolved is proportional to partial pressure.
transported in this chemical combination. - Amount of O 2 dissolved in plasma is the linear
The maximum amount of O 2 that can be combined with Hb is function of P02 (and = 0.003 ml Oz/dL blood/ mmHg P 02
called O 2 capacity, indicating that all available O 2 binding sites = 0.003 ml O 2/IOO ml blood /mmHg P 02 i.e. for each mm
of Hb are occupied by O 2. Hg P02 , there is 0.003 ml O 2 dissolved in 100ml or IdL of
The amount of O 2 carried by Hb increases rapidly upto P 02 of blood). Because of its dependence on P 02 , this is the
about 50 mmHg (83.5%) but above that the curve becomes much fraction of carried O 2 which alters with baricity (partial
flatter reaching 97.5% saturation at lOOmgHg. So the amount of pressure) of inspired O 2 and forms the basis of
O 2 bound to Hb does not increase (alter) in a person breathing hyperbaric O 2 therapy.
100% O 2 under hyperbaric conditions because the Hb has - Normal arterial blood with PO 2 of lOOmmHg contains
already become fully saturated at much lower P 02 (lOOmmHg). 0.3ml 0 2 pr dl (100ml)
When the blood is equilibrated with 100% O 2 (at normobaric - By utilizing this principle in hyperbaric (100%) O 2
conditions ie P02 760mmHg), the normal Hb becomes 100% therapy, raising the inspired PO 2 to 3atm (=3 x 760
saturated. When fully saturated each gram of normal Hb mmHg) or 4atm (=4 x 760 mmHg) increases the amount
contains 1.39 ml of O 2. However, normally some of the of plasma dissolved O 2 in arterial blood to about 6
hemoglobin is in form of inactive Hb derivatives such as ml/lOOml and 9ml/100ml respectively.
methemoglobin that cannot combine with O 2, This lowers the Atmospheric pressure Amount of O 2 dissolved/100
measured value (in vivo) to 1.34 ml of O 2 per gram of Hb (atm) (P02 ) o f inspired ml in plasma
(practically). 02
O 2 saturation of Hb (Sat) is the percentage of available binding 1 atm 0.003 x 760 = 2.28 mlQ = 2 ml
sites that have O 2 attached and is given by 2 atm 0.003 x 760 x 2 = 4.56 mlQ = 4 ml
0 2 combined with Hb 3 atm 0.003 x 760 x 3 = 6.84 mlQ = 7ml
S a 0 2 = Sat = xlOO
0 2 Capacity 4 atm 0.003 x 760 x 4 = 9.12 mlQ = 9 ml

- The total amount of O 2 bound to Hb is a function of Hb concentration and percentage of Hb saturation (Sat).

0 2 bound toHb = 1.39x H b (g m / 100m l)x^ - (for pure Hb)


2 100
(practically invivo)
=1.34xH bx—
100
Because Hb is 100% saturated on breathing 100% O 2 in normobaric conditions (P02 = 760mm Hg = latm ) and normally
blood has 15 gm Hb/100 ml

0 2 capacity = 0 2 bound to Hb= 1.39x15 = 20.8ml 0 2 /100ml blood

100
or = 1.34xl5x— =20.1mlO,/lOOmlblood
100 2
The O 2 saturation of arterial blood at the end of pulmonary capillaries with P 02 of 100 mm Hb is -97.5% whereas that of
mixed venous blood with Po2 of 40mm Hg is -75% . Due to physiological shunting the blood in systemic arteries is only
97% saturated. Therefore arterial blood contains 19.8 mL of Ch/dL: 19.5 mL bound to Hb and 0.29 mL in solution And
venous blood contains 15.2 ml of O 2 / d l: 15.1 mL bound to Hb and 0.12 mL in solution.
Gas content of blood PO2 % Sat Dissolved Ch (mL/dL)
m L /d L of Blood containing 15g of Hemoglobin (mm/Hg) ofHb(SaOz)
Arterial Blood (PO2 95 mmHg; Venous Blood (P02 40 mm 1° X 13.5 0.03 X
PCO2 40 mm Hg; Hb 97% Hg; P C 02 46 mm Hg; Hb 20 35 0.06
saturated) 75% saturated) 30 57 0.09
40 75 0.12
Gas Dissolved Combined Dissolved Combined
50 .Difference 83.5 0.15 , Table of 3
O2 0.29 19.5 0.12 15.1 f
60 of 10 89 0.18
co2 2.62 46.4 2.98 49.7 70 92.7 0.21
n 2 0.98 0 0.98 0 80 94.5 0.24
90 96.5 0.27
100 ^ 97.5 0.30
280 ■ A Complete Review of Short Subjects

★ For calculating the total O 2 concentration of blood (both in Hb & plasma)

Total O 2 content in blood = Hb bound O2 + Plasma dissolved O2


Sat
1.39(o rl.34) x H b x + 0.003 PQ2
100

____________________ Oxygen- Hemoglobin Dissociation Curve_________________j .


The relationship between partial pressure of O 2 and the percentage saturation of hemoglobin with O 2 can be explained
graphically. And the graph is called O 2- hemoglobin dissociation Curve. It represents the affinity of hemoglobin for O 2.

Norm al Curve Factors affecting affinity of Hb for O 2 and causing the curve to
- S- shaped or sigmoid shaped® due to
interconversion of hemoglobin from low 1 ' 1
affinity tightly bound Tense configuration (T Left Shift R ight Shift
state) to high affinity released configuration (R • This means the affinity of O 2 to Hb • This means the affinity of O 2 to Hb
state) as more O 2 molecules are bound. is increased and O 2 is bound more is decreased which favours release
- Relaxed configuration exposes more tightly to hemoglobin®. So low er of oxygen to tissue®. So higher PO 2
oxygen binding sites & significantly partial pressure of oxygen (PO 2) is (partial pressure of O 2) is required
increases O 2 affinity. required to bind a given am ount of to bind a given amount of oxygen
- Combination of the first heme with O 2 oxygen (i.e. P50 decreases)®. (i.e. P 50rises).
increase the affinity of second heme for O 2 • This happens in lungs where O 2is • This happens in peripheral tissues
(due to TR interconversion) and bound to Hb and muscles where O 2is delivered.
oxygenation of second increases the • Factors leading to left shift are • Factors 1 /t right shift are
affinity of the third and so on. - A lkalosis o r tpH ® - Acidosis or 4pH®
★ P50 - The p artial pressure o f O2 a t which - 4 PCO 2Q (CO 2content of blood) - T PCO ®2 (CO 2 content of blood)
Hb saturation is 50% ®. The normal - 4 Temperature® (H ypothermia) • T Tempeature® (H ypertherm ia
value is 26.6 mm Hg - 4 2 - 3 Biphosphoglycerate (2 ,3 - T 2 - 3 BPG®
★ M yoglobin has hyperbolic O2' binding BPG)
curve. ®

2 ,3 Bisphosphoglycerate (2,3- BPG) or 2 ,3 Diphosphoglycerate 2,3- DPG) ^

• It is most abundant organic phosphate in RBCs where its concentration is approximately that of hemoglobin. It is synthesized
from glycolytic intermediates 3-phosphoglyceraldehyde by means of Rapoport- leubering shunt.
• It is highly charged anion that binds to the /3 chains of deoxy hemoglobin. One mole of deoxyhemoglobin binds to one mole of 2 ,3
BPG. A single molecule of 2 , 3 - BPG binds to a positively charged pocket (cavity) formed by two /3-chains, in the centre of
deoxyhemoglobin. Positively charged aminoacids of pocket form ionic bonds with negatively charged phosphate group of 2,
3- BPG.
• It decreases O2affinity of hemoglobin by binding to deoxyhemoglobin but not to oxyhemoglobin and stabilizing the taut
conformation of deoxy hemoglobin.
Oxyhemoglobin + 2 , 3 - BPG > Deoxyhemoglobin + O 2
(H b02) (Hb - 2, 3 - BPG)
• In this equilibrium, an increase in the concentration of 2 ,3 - BPG shifts the reaction to right causing more oxygen to be
released.
• Hemoglobin from which 2 ,3 - BPG is removed has a high affinity for oxygen. However, the presence of 2 , 3 - BPG significantly
reduces the affinity of hemoglobin for oxygen, shifting the oxygen dissociation curve to right. This reduced affinity enable
hemoglobin to releases oxygen efficiently at the partial pressure found in tissues.
• Role of 2, 3 - BPG in stored & transfused blood:
Storing blood in acid- citrate - dextrose leads to a decrease of 2, 3- BPG in RBC. Hemoglobin deficient in 2, 3 - BPG displays
an abnormally high O2affinity & fails to unload its bound O2 in tissues properly and thus acts as an oxygen trap rather than as
oxygen transport system. However, transfused RBCs are able to restore their depleted supplies of 2 ,3 - BPG in 24-48 hours;
this may be a serius issue in critically ill patiens. This decrease in 2 , 3 - BPG can be prevented by adding inosine
(hypoxanthine - ribose) to the storage medium®. Uncharged inosine molecule enters RBC, releases its ribose moiety, gets
phosphorylated and enters HMP pathway to become 2 , 3-BPG. Hypoxanthine is formed during utilization of inosine to
maintain or increase 2 ,3 - BPG content. Thus w hile inosine increase / maintain 2 , 3 - BPG content, hypoxanthine does not®.
• In chronic hypoxia & anemia the concentration of 2 ,3 - BPG increases whereas, when pH falls (or is low), 2 3- BPG
concentration falls because acidosis inhibits red cell glycolysis.
Physiology: Respiratory System 281

Factors
I___
I 1
Increasing 2 ,3 - BPG Decreasing 2 ,3 - BPG

Chronic hypoxia eg
- Obstructive pulmonary emphysema
- High altitudes
- Cyanotic congenital heart disease
Anemia eg sickle cell Hb
Acidosis ( i pH)
Exercise (in non athletes)
Harmones
- Thyroid hormone Glycolysis
- Growth hormone Glucose
- Androgens
Pregnancy
Rejuvenating chemical agents Glucose 6 P 0 4
InosineQ
Dihydroxyacetone I
3- phosphoglyceraldehyde
Pyruvate
Phosphate
Phosphoenol, pyruvate 1,3- Biphosphoglycerate
I 2 .3 BPG Mutase

1 O2 affinity of Hb
T 0 2 delivery to tissue (Right shift)
^ Phospho
glycerate X2 .3 - BPG
kinase
3- Phospho glycerate 2 . 3 - B P G - Phosphatase

i
Pyruvate

Lung Volume Lung Capacities


Tidal 500 mlQ Is the air that moves into the lung Inspiratory capacity I 3800 ml. Total amount of air that can
volume with each normal inspiration or C=TV+ IRV be breathed in.
(T.V) the volume of air that moves out
of lung with each expiration
Inspiratory 3300 mlQ The air inspired with a maximal Vital capacity(VC) 4800 ml. Maximal amount of air that
reserve inspiratory effort in excess of =TV+ IRV+ERV can be expelled out force
volume tidal volume fully after a maximal (deep)
(IRV) =IC + ERV inspiration
Expiratory 1000 mlQ The air expelled with a maximal Functional residual 2200 ml. It is the volume of air
reserve expiratory effort in excess of tidal capacity remaining in the lung after
volume volume FRC=ERV+RV normal expiration (after
(ERV) normal tidal expiration)Q
Residual 1200 mlQ The amount of air remaining in Total lung capacity 6000 ml. The amount of air present
Volume the lungs even after forced (TLC) (4.2-6 litQ) in the lung after a maximal
(RV) expiration =TV+IRV+ERV+RV inspiration. This is the
maximum volume to
=VC + RV w hich the lungs can be
=IC + FRC expandedQ
282 ■ A Complete Review of Short Subjects

Respiratory minute 6L/minQ D e ad space


volume (rest):
Volum e (L)
Alveolar ventilation (rest): 4.2 L/minQ
Maximal voluntary 125-170
ventilation (BTPS) L/minQ TLC

I Resling
ERV 1 expiratory level 1
Timed vital capacity: 83% of total in Is; FRC
Maximal
97% in 3 s RV | expiratory level
Work of quiet breathing: 0.5 kg-m/minQ
Maximal work of breathing: 10 kg- IR V = Inspiratory reserve volum e T V = Tidal volum e
E R V = Expiratory reserve volum e R V = Residual volume
m/breathQ

Volume (L) FEViQ: (Forced expiratory volume in 1st


second) It is the fraction of vital capacity expired
Men Women
during the first secondQ of forced expiration.
r IRV 3.3 1.9 T
Inspiratory capacity FEVi also k/a timed vital capacity.
Vital Capacity -j TV 0.5 0.5
- In normal person FEVi = 70-85% (70-85% of air
I ERV 1.0 0.7
J - Functional residual is expelled in the 1st second of measurement of
RV 1.2 1.1
capacity VC).
Total Lung Capacity 6.0 4.2 - FEVi is more sensitive index than VC because
Lung volumes and some measurements related to the mechanics of in the early stages of many chronic diseases
breathing. The diagram at the upper right (e.g. Emphysema), the VC may remain within
represents the excursions of a spirometer plotter against time. normal limits but the FEVi begins to fall from
the very beginning.

Maximal Mid Expiratory Flow Rate

The average expiratory flow rate during the middle 50% of the VC. [Forced expiratory flow (FEF) between 25 and 75% of the
VC]. MMFR is also called FEF 2s-75%- Also know

Pulmonary Ventilation /Respiratory 6000 mlQ (6L) = 500x 12 The amount of air breathed in and out of lungs
M inute Volume (RMV) every minute. It is product of tidal volume (TV)
(RM V = TV x RR) and respiratory rate (RR). RMV = TVX RR
Maximal Voluntary Ventilation (M W ) 125-170L/min The maximum amount of air which can be
Maximal breathing capacity breathed in and out of lungs by forceful
respiration in one minute.
Alveolar ventilation = 4.2 liters The amount of air utilized for gaseous exchange
(TV - Dead space volume) x RR. every minute.
Total dead space/ physiological dead • Anatomical dead space - The conducting part where gaseous exchange does
space= Anatomical dead space + Alveolar not take place. It is volume of respiratory tract from nose up to terminal
dead space bronchiole. It is 150ml.
* In normal adult • Alveolar dead space - Those alveoli which are non functioning and those
Physiological dead space = Anatomical which do not receive adequate blood flow.
dead spaceQ
Closing volume The lung volume, at which, no further air can be expelled by expiratory effort.
Physiology: Respiratory System ■ 283

Closing Volume (CV) and Functional Residual Capacity


Closing Capacity (CC) (FRC) or Relaxation Volume
CV is volume below which the terminal airway connected to alveoli gets closed and It is volume of air remaining in
hence the gas cannot come in or out of alveoli. lungs after normal (tidal)
In young subjects, the closing volume is about 10% of vital capacity (V C). It increases exhalation, and is determined by
with age and becomes equal to about 40% of VC (i.e. = FRC) at the age of 65 years. the balance b/w the elastic recoil
Relatively small amounts early disease in the small airways apparently increases the pressure generated by lung
closing volume. parenchyma to become smaller
Closing capacity is closing volume plus residual volume. CC = CV + RV. CC is (inward recoil) and the pressure
higher in infants than in adults because of decreased elastic recoil, of lungs. generated by chest wall to become
larger (outward recoil). At FRC,
Single Breath N 2 W ashout Test to measure CV transmural pressure across
From mid inspiration, the subject takes a deep breath (vital capacity single breath) of respiratory system (chest wall +
pure (100%) O 2 and then exhales steadily. The changes in N2 concentration of expired air lung) is zero (O). It is positive at
(at lips) is continuously measured during subsequent full exhalation. greater & negative at smaller
4 phases of N 2 conc. can be recognized one after another. volumes.
1. Phase 1 or initial gas exhaled is the gas that filled the pure dead space. It contains no FRC decreases when the chest w all
N2 because inhaled gas is 100% O 2. muscles are w eak, (eg paralysis) or
2. Phase II: Mixture of dead space & alveolar gas is exhaled (with increasing N 2 conc). the elastic recoil o f lung (i.e.
The volume of dead space is volume of gas expired from peak inspiration to mid tendency o f alveoli to collapse) is
portion of phase II. more eg hyaline membrane disease Q
3. Phase III: Pure alveolar gas is exhaled (with constant N 2 conc). Phase III terminates (i.e. lung elastic recoil > chest wall
at (ie transition from phase III to IV ) is closing volume (CV). The CV is the lung muscle force). Disease characterized
volume above RV (residual volume) at which the airways in lower dependent parts of by low FRC include pulmonary
lungs begin to close off because of the lesser transmural pressure in these areas. In most edema, pneumonitis & RDS/HMD.
normal persons phase III also has a slight positive slope indicating a gradual increase in In hyaline membrane disease, FRC
the proportion of expired gas coming from relatively N2 rich upper portions of lung. usually decreases below closing
4. Phase IV: At the end of expiration, an abrupt increase in N2 conc. is seen signaling volume Q.
closure of airways at the base of lung. This is caused by preferential emptying of apex, When closing capacity exceeds FRC
which has relatively high concentration of N 2. The gas in upper lung is richer in N2 (FRC < CC), some alveolar
than lower dependent portions because the upper lung alveoli are more distended at the segments are collapsed during a
start of inspiration and therefore the N 2 in them is less diluted with 100% O 2 inspired portion of tidal breathing. This
from mid inspiration (because during a VC breath of 100% O 2, apex expands less and decreases V/P ratio and may cause
therefore previously occupied N2 there is less diluted with O 2). hypoxia and hypercarbia. And
when the CC > FRC + TV. (i.e. C C
TLC RV exceeds FRC plus tidal volume),
lung segments will be collapsed
during inspiration and expiration of
tidal breathing. This further
exaggerate V/P mismatch and 1/t
complete atelectasis. So respiratory
.0 therapy is designated to increase the
IS lung volume towards normal FRC.
c
<D Positive end expiratory pressure
O
c
o (PEEP) and continuous positive
o
airway pressure (CPAP) raise FRC
to a level above CV/CC in diseases
associated with alveolar collapse; So
the terminal airways remain open
throughout the respiratory cycle.
In presence of airway obstruction,
L u n g v o lu m e (I) FRC increases b/o premature
airway closure, which traps air in
the lung.
284 ■ A Complete Review of Short Subjects

Pulmonary Function Test Obstructive Restrictive Lung Examples


Measurements Lung Disease Disease (RLD)
(OLD) I
Forced vital capacity (FVC) Decrease Decrease Obstructive Restrictive lung
FEV I (Forced expiratory volume Decrease Decrease lung disease disease
in 1 second) AsthmaQ I. Parenchymal
Vital ca p acity (VC) Decreased Decreased BronchiectasisQ - Sarcoidosis
FEVI /FVC Decreased(<72%) Normal to Bronchiolitis Pneumo coniosis
Increased Cystic fibrosis - Idiopathic pulmonary fibrosis
FEF 25-75% (Forced expiratory Decrease Normal to Increase COPD i.e. Drug / Radiation interstitial lung
flow rate from 25% to 75% of chronic disease
VC) or average mid maximal bronchitisQ & II Extraparenchymal
expiratory flow rate (AMMEF) Emphysema^
Slope of FV (flow-volume) loop Decrease Normal to Increase
PEFR (Peak expiratory flow rate Decrease Normal Mnemonic - Neuromuscular Chest wall
i.e. greatest flow rate achieved "A BC D E" - D iaphragm atic - K y p h o sco lio sisC
during expiration in FV loop) palsy® - Obesity
FEF 50 (Vmaxso) and FEF 75 (Vmax75) Decrease Normal - GB syndrome - Ankylosing
i.e. instantaneous flow rate in FV - Muscular spondylitis
curve at which 50% & 75% of the dystrophy
VC remains to be exhaled - Cervical spine
respectively injury
Residual Volume (RV) Increased Decreased
D iffusion capacity Normal (1 in Decreased
emphysems)
Total lung capacity (TLC) Normal to Decreased
Increased

Tests for Pulmonary Compliance


Ptethysmography Helium D ilution Technique
- It is a technique to study the varations in size or volume r 1 (HDT)
of a part of body such as limb. .. .y. This technique to measure lung
- Plethysmograph (a water tight chamber) is the volumes (RV, FRC and TLC) is
instrument used and the limb (eg forearm) is sealed in it. simpler, cheaper but often less
Changes in the volume of forearm (because of changes accurate especially in lung
in the amount of blood and interstial fluid it contains) disease.
change the volume of plethysmograph and displace In normal individuals FRC
water in it. This displacement is measured with a measured both by helium
volume recorder. dilution and body
Venous Occlusion Plethysmography plethysmography are same. But
- On occluding the venous drainage of a limb, the rate of increase in the volume of the this is not true in lung disease.
limb is a funtion of arterial blood flow. FRC measured by HDT
Body Plethysmography (BP) measures the volume of gas
- All types of plethysmograpy are based on Boyle's law of gas which states that at that communicates with the
constant temperature______________________________________________________ airways, whereas BP measures
1. Pressure multiplied by volume is constant or the total volume of gas in the
2. Volume o f a sam ple o f gas is inversely proportional to the pressure o f that lung at the end of normal
gasQ i.e. Pi Vi = P2 V 2____________________________________________________ exhalation. If a significant
- In body plethyomography, subject sits in an air tight box k/a body box (or whole body amount of gas is trapped in the
plephysmograph) and breaths through a mouth piece that is connected to a flow sensor diseased lung b/o premature
(pneumotach /pneuotachograph); pneumotachometer is used to measure air flow and a closure of airways, the FRC
mouth pressure transducer with shutter measures the alveolar pressure. determined by BP will be
- Body plethysmography is used in measurement of thoracic (lung) volume including greater than that is measured
total lung capacity (TLC) and residual volume (RV) and airflow resistance. by HDT.
In HDT, a known concentration
Physiology: Respiratory System ■ 285

(Ci) of an inert gas like helium


M easurement of lung volumes Measurement o f airway is added to a box of known
- The subject sits in a an airtight box and breaths normally resistance volume (Vi). The box is then
through a mouth piece for few minutes and then subject Airway resistance is the connected to an unknown (V2)
makes panting respiratory efforts (i.e. breaths rapidly pressure difference volume to be measured (in this
with maximum force) against a closed mouth piece between the alveoli and case the lung volume). After
(glottis). the mouth per unit of adequate time for distribution
- By measuring the volume (A V) and pressure changes in airflow. of inert gas, the new
the box (PI and P2 are pressure change of box at mouth), Using body concentration of inert gas in
the volume of lungs is calculated (V = FRC). PI x V = P2 plethysmography box (C2) is measured
x (V - AV) alveolar pressure is The change in concentration of
calculated by measuring inert gas is used to determine
During (Maximum) Expiration During Inspiration change in volume of box the new volume (Vi + V2) in
Lung volume decreases (with Alveolar gas (volume) (A V) (which is a which the inspiration gas is
increase in pressure) but the expands (increases) with reflection of change in distributed.
volume of gas in chamber decrease in pressure but the
alveolar volume).
increases (with decreases in volume of gas in box C, x V , = C 2x ( V , + V 2)
The difference b/w
pressure) decreases with increase in
alveolar and mouth

•Simply stating, the pressure difference between alveolus and atmosphere is responsible for airflow into and out of alveolus
(i.e. inspiratory & expiratory airflow). And pressure difference b/w alveolus (expanding lung) and intrapleural /or outside
the lung (compressing the lung) is responsible for expansion & compression of lung.
•Before inspiration begins, airway (alveolar) pressure is everywhere zero (0 i.e. equal to atmospheric pressure). In the
absence of pressure gradient b/w atmosphere and airways, there is no airflow. But at the same time (before inspiration)
intrapleural pressure is - 5cm H20 because of elastic recoil of lung- in other words there is a pressure of 5 cm of water
holding the airway open. The negative pressure in pleural space relative to atmospheric pressure is created by inward elastic
recoil pressure of lung and it acts to pull the lung away from the chest wall.
•Both the thoracic cage and the lungs are elastic structures. Being elastic structure they both tend to recoil, but in opposite
direction:
i. The thoracic cage has inherent tendency to expand, while trying to expand, it draws the parietal pleura with it (because
parietal pleura is strongly attached to inner side of the chestwall)
ii.The lungs have an inherent tendency to collapse because of their elasticity and surface tension of thin layer of fluid lining
the alveoli.
•As the visceral pleura is strongly attached with the lungs, the pull in the opposite directions of the visceral and parietal
pleura, creates a negative intra- pleural pressure. Normal intrapleural pressure = - 4 to -5 mm HgQ.; stronger the inspiration,
more -v e is the intrapleural pressure
■Because of inherent elastic tendency of lungs to collapse, a negative force is always required on the outside of lungs to keep
them expanded. And this is more so because in no airflow stage of breathing cycle the airway/alveolar pressure is '0' so only
negative pressure outside the lung can prevent their collapse. Because the normal collapse tendency of lungs is about - 4
mmHg, the pleural fluid pressure must always be at least as negative as -4 mmHg to keep the lungs expanded. The basic
cause o f this negative intraleural pressure is lym phatic pumping o f fluid®.
■As the inspiration starts, the diaphragm and chest wall muscles contract and cause downward movement of diaphragm and
upward & outward movement of rib cage. So with the onset of inspiration, both intra pleural and alveolar pressure fall by 2
cm water and flow of air begins. Because of pressure drop along the airway, the pressure inside is -2 cm of water (responsible
for inward airflow), and there is -7cm of water intrapleural pressure holding the airway open. So the intra pleural pressure
also falls during inspiration and becomes more negative. This decrease equals the lung elastic recoil, which increases as the
lung inflates.
286 ■ A Complete Review of Short Subjects

■Two reasons of intrapleural pressure fall during inspiration are - (1) as the lung expands its elastic recoil increases (2) and the
pressure drop along the airway is a/w a further fall in intrapleural pressure. The extent of alveolar pressure fall depends on
flow rate and the resistance of airways.
At end inspiration air flow stops when alveolar pressure and atmosphere pressure become equal (O); but the airway
transmural pressure (i.e. = airway pressure - pleural pressure) expanding the lung is 8 cm of water. In other words
intrapleural pressure at end of inspiration is -8cm of water.
On expiration (exhalation), the diaphragm moves higher into the chest, pleural pressure increases (i.e. becom es less
negative) and alveolar pressure becomes positive, the glottis opens and air again flows from alveolus (higher positive
pressure) to lower atmospheric (O) pressure. The driving force for exhalation is the sum of elastic recoil of lung and pleural
pressure.
At the onset of forced expiration, both intrapleural pressure and alveolar pressure increases by 38 cm H2O and becomes
positive.

Pressures In & Around Lung


The pressure difference between the air spaces (alveolar pressure = Pa) and pressure surrounding the lung (i.e. pleural
pressure = Ppi) is called transm ural pressure across lung or transpulm onary /trans lung pressure (P l) P l = P a - P pl The
lung requires positive transpulmonary pressure (i.e. relatively more alveolar or lesser pleural pressure) to increase its volume
and the lung volume increases with increasing P l. The lung has smallest size at P l = O.
Similarly the transmural pressure across chest wall (Pw) is difference b/w pleural pressure
(P pl) and pressure surrounding the chest wall (= body surface pressure; Pb) P w = P p l - P b
The pressure across respiratory system (Prs) is sum of PL and PW.

Prs =P l + Pw = (P a - Ppi.)+(PpL-Pb)=PA-Pb
At resting volume of lung (FRC), the elastic recoil of lung acts to decrease lung volume, but
t&°„
I
this inward recoil is offset (balanced) by the outward recil of chest wall, which acts to increase
lung volume. At FRC, these forces are equal and opposite and the muscles are relaxed. When
the chest is opened as in thoracotomy (surgery) the lung recoils until transpulmonary
pressure is O and chest wall increases in size.
The transmural pressure across respiratory system (Prs) at FRC is O. At TLC (total lung
capacity) both lung pressure and chest wall pressure are positive and they both require
positive transmural distending pressure. The resting volume of chest wall is volume at
which transmural pressure for chest wall (Pw) is 0, and it is approximately 60% of TLC. So at
volume > 60% of TLC, the chestwall is recoiling inwards and positive transmural pressure is
needed, whereas at volumes < 60% of TLC, the chest wall tends to recoil outward.
The intrapleural pressure is less negative at base (-2.5 cm H 2O) than at apex (-10 cm H2O) at
the start of inspiration. More negative intrapleural pressure at apex holds the lung in a more
expanded position; and so further increase in volume per unit increase in intra pleural pressure are smaller than at the base
because the already expanded lung is stiffen Ventilation is consequently lesser at apex and greater at base. Blood flow (perfusion)
is also greater at the base than apex. But the ven tilation /perfusion ratio is lo w a t b a se an d high a t apex because the relative
change in perfusion is greater than the relative change in ventilation.
Physiology: Respiratory System ■ 287

Examples

- For example at the start of expiration, pressure inside alveolus (PA) is O (i.e. no air flow) and pleural pressure (Ppi) is - 30 cm
H20 . S o transpulmonary pressure (P l) of + 30 cm ( P l = P a - Ppi = O - (-30) = + 30) of water is holding the alveoli open.
Because there is no flow , the pressure inside airways (Paw) is also O and similarly + 30 cm H20 of transairway pressure holds
the airway open (Pia = Paw- Ppi).

With the contraction of expratory muscles both pleural and alveolar pressure rises + 90 cm each and becomes + 60 cm H20
and + 90 cm H20 respectively. So the P l remains same but because of higher alveolar pressure (in comparison to atmosphere)
airflow begins.
Because of gradual decrease in lung volume (decreasing driving pressure) and expiratory airflow resistance, there is gradual
dissipation of airway pressure (Paw)- And at equal pressure point (i.e. the point at which the pressure inside the airways
equals the pressure outside the airways) the airways become compressed (dynamic airway compression).

Forced Expiration, Equal Pressure Point & Dynamic Airway Compression

- Alveolar pressure ( P a ) is the sum of pleural pressure ( P p l ) and elastic recoil pressure (Pei). PA = Ppi + Pei ; and this is the
driving pressure for expiratory gas flow. Because alveolar pressure exceeds atmospheric pressure (during expiration), gas
begins to flow from alveolus to mouth, through open glottis. As gas flows out of alveoli the transmural pressure across the
air way (Pta) decreases. In otherwords, there is a gradual decrease in airway pressure (Paw) from distal (alveoli) to proximal
(trachea) respiratory tract. This gradual pressure dissipation is caused by
i. Expiratory airflow resistance (frictional pressure loss); the major site of resistance along bronchial tree is large bronchi
(Berne & Levy) / medium sized bronchi (John West).
ii. As the overall cross sectional area of the airways decreases towards the trachea, gas velocity increases. This acceleration of
gas flow decreases the pressure (Paw) and can make flow turbulent (increasing resistance).
iii. As air moves out of lung, the lung volume decreases; which intum decreases the driving pressure (alveolar pressure -
intrapleural pressure) and the airways become narrower (increasing resistance).
- The positive transpulmonary (P l) and transairway pressure (Pta) hold the alveoli and airway open. Because Pl = Pa - Ppi
and Pta = Paw- Ppi; it also means that alveolar pressure (PA) or airway pressure (Paw) greater than pleural pressure hold the
alveoli or airways open. In other words, a negative intrapleural pressure (Ppi) is required to keep the airways stretched open.
- Equal pressure point is a point in respiratory tract between mouth and alveoli at which the pressure inside the airways
equals the pressure that surrounds the airways. So the airway proximal to equal pressure point (towards mouth) become
compressed because the pressure outside the airway exceeds the pressure inside the airway. This is called dynamic
compression of airways.
- Just beyond the (proximal to) equal pressure point the transairway pressure (Pta) becomes negative. Because Pla = Pressure
in the airway expanding it (Paw) - Pressure around the airway compressing it (i.e. pleural pressure Ppi) Pta = Paw - Ppi
That is why no amount of expiratory effort will increase the flow further because the higher pleural pressure (which rises
with increased expiratory effort) also tends to collapse the airway at the equal pressure point (EPP), just as it tends to
increase the gradient for expiratory gas flow (by increasing the pressure gradient b/w alveoli and atmosphere).
That is how dynamic airway compression limits air flow in normal subjects during a forced expiration and the airflow is
independent of total driving pressure. In other words during dynamic compression, flow is determined by
transpulmonary pressure ( P l = PA - Ppi) or alveolar pressure minus pleural pressure (not mouth pressure). Hence the
expiratory flow is effort independent, flow limited and has greater airway resistance (than during inspiration).
Equal pressure point is dynamic (not static). In normal lungs (without disease), the EPP occurs in airways that contain
cartilage and thus they resist collapse. As expiration progresses (i.e. lung volume and elastic recoil pressure decreases), the
EPP moves distally, deeper into the lung, closer to the alveoli. This occurs because the resistance of airways rises (d/t
decrease in radius) as the lung volume falls, and therefore the pressure within airways fall more rapidly with distance from
the alveoli. This resistive drop in airway pressure is greater in diseased lung (with airway obstruction secondary to mucus
accumulation and inflammation). As a result EPP occurs in small airways that are devoid of cartilage causing them to
collapse (premature airway closure). The major site of increased resistance in patients with COPD is in airways < 2mm dia.
Dynamic airway compression may occur in diseased lung at relatively low expiratory flow rates, thus reducing exercise
ability. DCA is exaggerated in emphysema because of reduced lung elastic recoil and loss of radial traction on airways.
288 ■ A Complete Review of Short Subjects

Airway Resistance
Highest airway resistance is in medium sized /large bronchi. The smallest airways contribute very little to overall total
resistance because in smaller airways
1. With increase in effective cross sectional area the airflow velocity
decreases substantially and flow becomes laminar and
2. The airway exist in parallel rather than in series. The resistance of
airways in parallel is the inverse of sum of individual resistances
J _ _ _ L + _!_ 1 whereas in series it is R t0 ia i = R i + R 2+ ....R n

, R /o la l ^2 J
Normal airway resistance is 1 cm H20/L. sec. It decreases as lung
volume rises because the airways are pulled open and similarly
resistance increases with decreasing lung volume. Density and
viscosity of inspired gas affect the airway resistance. In deep under
water /Scuba diving the increased pressure raises gas density and airway resistance. Whereas breathing a low density gas
oxygen & helium mixture reduces air way resistance.
Resistance of airways (Raw) is low during lam inar flow and higher during turbulent flow. During laminar flow it can be
calculated by Poiseuille's law ie is
8 x L e n g t h o f tu b e (l) x G as v is c o s ity On)

71 x (R a d iu s o f tu be or r)

Increased Raw Decreased Raw


- High (increased) lung volume^
Low (decreased) lung volumeQ
Forced expirationQ - During inspiration & at TLC (minimum)
More dense or viscous inspired gasQ eg when scuba - Less dense /viscous inspired gasQ eg the helium oxygen
divingQ mixture
Turbulent airflow - Luminar airflow
■Airway in series (ie longer length) - Parallel airways (smaller length)
Less effective cross section (radius) eg medium & large - Increased effective cross sectional area (radius) eg small
bronchi airways

Work Load of Breathing Work Capacity


It includes work done by respiratory muscles for Capacity to do work (work capacity of
- Stretching the elastic tissues of chest wall & lung (Elastic work, 65%) and all muscles skeletal & cardiac is greatly
- Moving inelastic tissues (Viscous resistance, 7%) and moving air through decreased in hypoxia & at high altitudes.
respiratory passage (Airway resistance, 28%) known as non elastic work Work capacity is reduced in direct
(35%). proportion to the decrease in maximum
Work of breathing can be calculated from relaxation pressure curve (by rate of O 2 uptake that the body can
pressure X volume = g/cm2 x cm1 =gxcm) - achieve®.
The elastic work required to inflate the whole respiratory system (ABCA) < Acclimatization increases work capacity
the amount of elastic work required to inflate (increase volume of) lungs Person Work capacity
(ABCDEA) because part of work comes from elastic energy stored in thorax. (% of normal)
The elastic energy (work) lost from thorax (AFGBA) is equal to gained by Unacclimatized 50
lungs (AEDCA) Low lander 68
The pressure volume curves that the lung follows during inflation & acclimatized for 2
deflation are different. This behaviour is k/a hysteresis. The lung volume at months
any given pressure during deflation is larger than during inflation. Pressure Naturally 87
volume curve of lung is nonlinear, with the lung becoming stiffer at high acclimatized
volumes. native living at
In quiet breathing, when pressure is plotted against volume, it forms a 1300 feet but
hysteresis loop rather than a straight line because intrapuleural pressure working at 1700
changes 1/t lung volume changes during inspiration & expiration. Area feet
AXBYA represents non elastic work done required to over come lung
viscosity & airway resistance
Physiology: Respiratory System ■ 289

Resistance to air movement is relatively small during quiet breathing^.


When air flow becomes turbulent during rapid breathing the energy Naturally acclimatized native person can
required to move air is greater than when the flow is laminar. Total work of achieve a daily work output even at high
quiet breathing is 0.3 - 0.8 kg- m /min, which rises markedly during altitude almost equal to that of low
exercise^, and in diseases such as emphysema, asthma, & CHF. lander at sea level, but even well
When over stretched, respiratory muscles contract with less strength, acclimatized low lander can almost
become fatique & fail (d/t length- tension relationship) leading to in never achieve this.
adequate ventilation

R e la x a tio n p re s s u re c u rv e (P ) o f c h e s t

T ra n s m u ra l p re s s u e (c m H 20 )

Compliance (Stretchability) / Elasticity


Change in lung volume per unit change in distending (airway) pressure of lung (if enough time is allowed to reach
equilibrium) is k/a lung compliance. So the volume change per unit pressure change or (volume change resulting from 1 cm
H2O change in pressure) or the slope of pressure-volume curve is k/a compliance.
It is normally measured in the pressure range where the relaxation pressure curve is steepest. Because of hysteresis caused
by surfactant, the deflation P-V curve is used for measurement. And lung compliance is the slope of line b/w any two points
on deflation limb of pressure volume loop.

^ AV AV
C l = ----- =
AP
A P P>

To measure pressure change (or pleural pressure change in lungs) esophageal balloon connected to pressure transducer is
used, which is an excellent surrogate marker for pleural pressure. Esophageal pressure is not identical to intrapleural
pressure but reflects its pressure change fairly well. However, the measurement is not reliable in supine position b/o
interference by the weight of mediastinal structures.
Unit of compliance (i.e. stretchability or elasticity) is mL (or liter) per cm H 2O. The compliance of normal human (both) lung
is O.lL/cm H 2O (200 mL/Cm HzO,)Q which means every time the transpulmonary pressure increases 1 cm H 2O, the lung
volume will expand 200 ml (0.2 L) after 10-20 seconds.
Lung compliance (Cl) is a measure of the elastic properties (or stretchability) of lung and hence its total capacity (TLC). It is a
measure of how easily the lung is distended. High value refers to a lung that is readily distended and low compliance means a
stiff lung that is not easily distended.
Lung's elastic behaviour (i.e its tendency to return to its resting volume after distension) or compliance is due to geometrical
(interwoven) arrangement of elastin and collagen fibers and elastic forces caused by surface tension. The compliance is a
static measure of lung and chest recoil. Whereas resistance of lung & chest (i.e. pressure difference required for a unit of air
flow) is a dynamic rather than static measurement that takes into account the resistance to airflow in airways.
However, compliance also depends on lung volume; and patient with one lung has approximately half compliance of normal (i.e.
half AV for a given AP). Expiratory compliance (measured during deflation) is slightly greater than inspiratory compliance
(measured during inflation). The compliance of respiratory system (combined lung & thorax) is exactly half of the
compliance of lung alone (i.e. 110 ml/cm H 2O).
290 ■ A Complete Review of Short Subjects

Com pliance of a lung depends on its size (volume). Sim ilarly lung is stiffer (less distensible or has sm aller com pliance) at
high lung volumes (and high expanding pressures) as show n by the flatter slope of curve. For this reason compliance per
unit volume of lung (k/a specific compliance) is measured to know intrinsic elastic properties of lung tissue.

T ra n s lu n g p re s s u re (c m H 20 )

Translung pressure (cm H20 )

Hypoxemia & Hypoxia

Hypoxemia means low arterial oxygen tension in blood (i.e. low Pacu)- Arterial hypoxemia is defined as an arterial partial
pressure of O 2 (i.e. Pace) < 80 mmHg in an adult who is breathing room air at sea level.
Hypoxia is O 2 deficiency at the tissue level to carry out norm al metabolic functions and often occurs when Pao 2 is less than
60 mmHg. Anoxia means no O 2 at all left in the tissue.
Hypercapnia is an increase in partial pressure of arterial CO 2 (Paco 2 ) above the normal range (40 + 2 mmHg) and
hypocapnia is abnorm ally low Paco 2 (< 35 mmHg).

4 Type of Hypoxia

- Hypoxic hypoxia, which occurs in high altitude & lung /respiratory diseases, is best diagnosed by m easuring partial
pressure of arterial O 2 (P a 0 2); and P a 0 2 is low (J.) in it. In anemic hypoxia, there is decreased O 2 carrying capacity of blood
d/t quantitative or qualitative deficiency of hemoglobin. So PaC>2 is normal but O 2 content is decreased.
- Both stagnant and histotoxic hypoxia are best diagnosed by arterio-venous O 2 difference, w hich increases in stagnant and
decreases in histotoxic hypoxia. In stagnant (ischem ic) hypoxia both PaC>2 and Hb are normal but the blood flow to the
region is low so O 2 content is exhausted. W hereas in histoxic hypoxia tissue can not utilize available O 2 .
- Each hemoglobin (Hb) molecule can bind upto & 4 0 2 atoms and each gram of Hb can bind upto 1.34 mL of 0 2 (= 100%
saturation). Oxygen saturation (S 02 ) refers to the am ount of 0 2 bound to Hb relative to the m axim um amount of O 2 that can
bind to Hb. At 100% O 2 capacity 4 molecules of 0 2 occupy the 4 heme groups of Hb; at 75% S 0 2 3 of 4 heme grous are
occupied.
- O 2 content in blood is sum of dissolved 0 2 and O 2 bound to Hb. O 2 content is decreased in hypoxic and anem ic hypoxia
(i.e. in presence of CO 2 , CO and anemia)
- Alveolar and arterial C O 2 are equal but alveolar & arterial O 2 are not. N orm ally alveolar 0 2 (Paco) is slightly greater than
arterial 0 2 (Pa02). This difference is called alveolar-arterial PO 2 difference (A a D 0 2). It is not caused by im perfect gas
exchange but is produced by sm all number of veins that bypass the lung and empty deoxygenated blood directly into the arterial
circulation. Eg thebesian vessels of left ventricular m yocardium drain into left ventricle (rather than into coronary sinus) and
few bronchial & m ediastinal veins drain into pulm onary veins. This venous admixture (anatomic shunt) decreases arterial
Po 2 - A a D 0 2 is < 15 mmHg in norm al individuals and rises ~ 3 mm Hg per decade. Hence < 25 mmHg is the upper normal
limit. An increased A a D 0 2 is hallmark of abnormal O 2 exchange.
N = N o rm a l, A = A lv e o la r , a = a rteria l, v = v e n o u s , A aDo2 = A lv e o la r a rte ria l O2 p a rtia l p re s su r e d iffe re n c e C ao2 /C vo 2 = C o n c e n tr a tio n o f O2 in arte ria l
& v e n o u s b lo o d ; Sao2 = S a tu ra tio n o f O2 in a rteria l b lo o d .
Physiology: Respiratory System ■ 291

Type of C au ses A m o u n t o f 0 2 A rte rio C on ten t/c S a t u r P a r tia l P re s s u re (P) Pao2 r e s p o n s e


H y p o x ia v e n o u s o n c e n tra t a tio n to 1 00 % 0 2
(P a o 2- io n Sa02 a d m in is tr a tio n
Pvo2) A lv e o la r A rte ria l A aD o2 V enou s

C o n tin u e d

D e liv e r e d
d iffe r e (P ao2-

U sed
n ee Go Cvo2 P a o 2)
2

P A o 2 PAC02 P a o 2 P a C o 2 Pvo2
H yp oxic/ A n H y p o v e n tila tio n 4 T T N T
o x ic D e c re a s e d F i0 2 i T T N T
H y p o x ia o r H elp fu l
D if f u s io n Im p a irm e n t N N N T T
H y p o x e m ia
V e n tila tio n -P e r fu s io n Low N - V t/ N T/n T T
m is m a tc h (V/Q lo w )
(L ev e l =
Lung) A n a to m ic s h u n t N N N t LV L im ite d
1r ' 1f f r
P h y s io lo g ic a l s h u n t N N N T LV v a lu e

A n e m ic A n e m ia (L o w H b ) C O
H y p o x ia P o is o n in g L im ite d
(L ev e l = I N N - 1 i N N N N N N i LV v a lu e
B lo o d )
H y p o p e rfu s
io n / Isch e m i V a s c u la r d ise a se
c/ S ta g n a n t
H y p o x ia N Low N T N i N N N N N N I LV L im ite d
(L e v e l= C ir c C a r d io g e n ic sh o c k v a lu e
u la tio n )

H is to to x ic C y a n id e p o is o n in g
H y p o x ia S o d iu m a z id e
(L ev e l = I n h ib it io n o f N N Low 4- N T N N N N N N t O f n o v a lu e
T is s u e ) c y to c h ro m e o x id a s e
B e r ib e r i

Effects of Hypoxia

- Hypoxia causes production of transcription factors (hypoxia inducible factors; HIFS) made up of a and P subunits. In
normally oxygenated tissues, the a subunit is rapidly ubiquitinated & destroyed. Where as in hypoxic cells a- subunit
dimerizes with P subunit and activate genes that produce angiogenic factors & erythropoietin. The stimulus for secretion of
erythropoietin is fall in O2 tension in the tissues sensed by kidneys and liver.
- Brain is affected first. Less severe hypoxia 1/t variety of mental aberrations like impaired judgement, drowsiness, dulled pain
sensibility, excitement, disorientation, loss of time sense, headache, anorexia, nausea, vomiting, tachycardia (like those
produced d/t alcohol intake). Whereas severe hypoxia 1/t hypertension, increased rate of ventilation (b/o hypoxia of
carotid chemoreceptor). And because anemic hypoxia do not stimulate peripheral chemoreceptor, hyperventilation is not
seen in it
- Hypercapnia and to lesser extent hypoxia cause dyspnea.
- Cerebral blood flow :
i. Increases if Pao 2 decreases
ii. If -I Paoi is associated with hyperventilation: cerebral blood flow falls & hypoxia intensified
- Synpathetic discharge: Increases® by directly stimulating the vasomotor center.
- Lactic acidosis® : In hypoxia a large proportion of pyruvate is reduced to lactic acid. (Glucose is broken down to pyruvate
under anaerobic condition & generates ATP)
- 2, 3 - DPG : IncreasesQ, which cause a right shift of 0 2- dissociation curve.
- Cardiac outputQ; Increases due to diffuse vasodilation (4 P02 cause vasodilation). Dilatation - » iB P —> THeart rate.Q
292 ■ A Complete Review of Short Subjects

_________________ Cyanosis_________________
It is bluish discolouration of skin & mucous membranes resulting from an increased quantity of reduced (deoxygenated)
hemoglobin. The deoxygenated hemoglobin has an intense dark blue purple color that is transmitted through the skin.
In general cynosis becomes apparent when the mean capillary concentration of reduced Hb exceeds 4gm/dL (40gm/L)® Harrison
/5gm per dl (Ganong & Guyton). It is absolute rather than relative quantity of reduced hemoglobin that is important in
producing cyanosis. Thus in a patient with severe anemia the relative amount of reduced Hb may be very high. However
since the total conc. of Hb is reduced, the absolute quantity of reduced Hb may still be small, and therefore patients with
severe anem ia and even marked arterial desaturation may not display cyanosis®
Definite cyanosis appears whenever the arteria blood contains >5gms of deoxygenated Hb in each 100 m illiliters of blood.
So a patient with anemia almost never becomes cyanotic because there is not enough hemoglobin for 5 grams to be
deoxygenated in 100 ml of arterial blood. Conversely in patients with polycythemia vera (excess RBCs) the great excess of
available Hb that can become deoxygenated leads frequently to cyanosis, even otherwise normal conditions.
Cyanosis is not a reliable sign of mild desaturation (i.e. early hypoxia) because its recognizition depends on so many
variables, such as lighting conditions and skin pigmentation. Cyanosis is a late sign of hypoxia and indicates the condition is
marked (profound). So it is never advisible to wait for cyanosis to appear to diagnose hypoxia. Similarly absence o f cyanosis
does not alw ays indicate adequate airw ay and absence o f hypoxia ®, because if the percentage saturation is high or in cases of
anemia, cyanosis does not appear.
A cherry red flush rather than cynosis is caused by COHb (carboxy hemoglobin)®

Central Cyanosis Peripheral Cyanosis


Central cyanosis is caused by inadequate oxygenation of systemic arterial blood due to • It is caused by slowing of
decreased arterial oxygen saturation or hemoglobin abnormalities. blood flow & abnormaly great
Central cyanosis is charachterized by cyanosis affective both mucous membranes and skin. extraction of O2from
normally saturated blood.
Seen if Causes • It results from
- Mean reduced Decreased arterial 0 2 saturation (S a 0 2) vasoconstriction or
(deoxygenated) decreased peripheral blood
1.Decreased - High altitude
hemoglobin flow, reduced cardiac
atmospheric pressure
>4/5gm/dl® output or vascular
2.Impaired pulmonary - Impaired O 2 diffusion occlusion
- Oz saturation fa lls < function - Alveolar hypoventilation • It is characterized by
85%® (In dark skinned
- V/Q mismatch cyanosis of skin alone &
detected when <75%)
3.Anatomic shunts - Pulmonary A-V fistula sparing of mucous
■M ethhemoglobin 2 membranes (esp. oral
1.5gmldl® - Congenital heart disease (some)
cavity).
- Multiple small intrapulmonary
- Sulfhemoglobin <> 0.5 • Causes are-
shunts
gm/dl® - I Cardiac output
4.Hb abnormality (with - Methhemoglobinemia (hereditary
- Arterial / Venous
low affinity for O 2) acquired) obstruction
- Sulfhemoglobinemia (acquired) - Redistribution of blood
- C arboxyhem oglobin em ia (not true flow from extremities
cyanosis)® - Cold exposure

Methemoglobinemia
In Methemoglobinemia, iron within hemoglobin is oxidized from the ferrous (Fe++) to ferric (Fe+++) form, resulting in the
inability to transport oxygen and carbon dioxide.
It may result from exposure to oxidizing agents such as nitrites, nitrates®, aniline dyes, and medication including local
anesthetics benzocaine, lido caine, prilocaine, phenazopyridine hydrochloride, sulfonamides®, chloroquin, inhalant in
cyanide antidote kit, nitrogen oxide, nitro & nitroso - hydrocarbons, primaquine - type antimalarials®, etc.
Methemoglobinemia >25% causes symptoms of cerebral ischemia, and levels >60% are usually fatal.
Congenital methemoglobinema arises from HbMl w ata ( a 87 Hlx T y r) mutation® in globin or from mutation that impair
enzyme that reduce methhemoglobin to hemoglobin (eg. methemoglobin reductase, NADP diaphorase)®.
IV methylene blue® (lmg/kg) is effective emergency therapy for >30% methemoglobinemia, symptomatic hypoxia or
ischemia (but contraindicated in G6PD deficiency). Milder cases & follow up of severe cases are treated by oral methylene
blue or ascorbic acid® (300 - 600 mg/day). Severe & refractory cases require exchange transfusion & hyperbaric oxygen®.
Physiology: Respiratory System 293

Abnormal Respiratory Pattern


Cheyne- Stokes respiration Rhythmic alteration of apnea and hyperapnea due to
Opium poisoning anoxia, (c/b periods of waxing and waning
Uremia tidal volume separated by period of apnea). It is
- CCF a type of periodic breathing.
Hypoxia
Biot's respiration Type of periodic breathing, that consists of one
Meningitis ( TlCT) or more large tidal volumes separated by
periods of apnea i.e. irregularly irregular
respiration
Kussumal's respiration Rapid & deep breathing (Air hunger)
Diabetic ketoacidosis
Uremia
O ndine's curse Loss of automatic respiratory control with
destruction of involuntary neural pathway.

Reduced Arterial 0 2 Tension


Decreased inspiratory O 2 D efect in oxygenation D efect in
tension (VQ mismatch)=Type I failure Ventilation = Type II failure
High altitude
Defect in Defect in lung Obstruction Hypoventilation
parenchyma vasculature - COPD - Decreased respiratory
- Pneumonia - Right to lung drive from CNS
- ARDS shunt - Weakness of
respiratory muscles

Carbon Monoxide (CO) Poisoning


Physiological Role of CO Reduced 0 2 content but Normal PO 2
Small amounts of CO is formed in body and functions as g a seo tran sm itter Even though 0 2 content of blood is
v a s o d ila to r (along w ith NO and HiSjO. greatly reduced in CO poisoning, the P O 2
Causes Anemic (not Histotoxic) Hypoxia o f b lo o d m ay be n orm alQ. This makes it
CO is toxic because it reacts with hemoglobin (at the same point as does O 2) to especially dangerous, because the blood is
form carbon monoxyhemoglobin (carboxyhemoglobin or COHb) and therefore bright red and there are no obvious signs
displace O 2 from Hb thereby decreasing the O 2 carrying capacity of blood. of hypoxemia (such as cynosis). Also, P 0 2
CO poisoning causes anemic hypoxia because the amount of Hb that can carry 0 2 is not reduced, and feedback mechanism that
is reduced but the total Hb amount of blood is unaffected by CO. usually stimulates increased respiration rate
Left Sh ift o f O 2 Dissociaton Curve in response to lack of 0 2 ( l P 0 2) is absent.
COHb can n ot ta k e up O 2; lib era tes CO very slo w ly an d sh ifts d issocia tion Clinica Toxicity
curve o f rem aining H bO i to leftG, decreasing the amount of O 2 released. This is Because the brain is the fir s t organ
why an anemic person who has 50% of normal amount o H b 0 2 may be able to a ffected <2, the person may become
perform moderate work whereas an individual whose Hb is reduced to same disoriented & unconscious before
levels because of COHb formation is seriously incapacitated. becoming aware of danger.
High (250 times) A ffinity for Hb The amount of COHb formed depends on
The affinity of Hb for CO is 210 times (Ganong) / 250 times (Guyton) its affinity duration of exposure to CO as well as
for O 2. So CO-Hb dissociation curve is almost identical to O 2 Hb dissociation concentration of CO in inspired air and
curve except that partial pressure are at a level of 1/250. So a CO partial alveolar ventilation. Death results when
pressure of only 0.4 mm Hg in alveoli (1/250 that of normal alveolar 0 2 100 mm 70-80% circulating Hb is converted to
Hg P 0 2), allows the CO to compete equally with 0 2 for combination with the Hb COHb. CO is also toxic to cytochromes in
and causes half the Hb in blood to become bound with CO instead of with 0 2. tissue but at 1000 times of lethal dose; so
Therefore CO pressure of only 0.6 mmHg can be lethal. tissue tox icity p la y s no role in clin ical
T ransport an d toxicity o f carbon m on oxide (CO) is lim ited to its diffusing CO p oison in g Q.
ca p acity becau se hem oglobin com bin es w ith this g as s o rap id ly 0 that its partial Treatment
pressure never has time to build up and is essentially zero in blood. The affinity The treatment is hyper baric pure oxygen
of Hb for CO is 210 greater than its affinity for 0 2. and 5% CO20
294 ■ A Complete Review of Short Subjects

Oxygen Toxicity (Poisoning)

Mechanism Types Effects


Once the alveolar PO 2 rises 1) Acute toxicity results CNS (Bert) effect is acute manifestation (minutes to an
above a critical level (>2 from extremely high tissue hour) of exposure to hyperbaric O 2, which results in
atmospheric PO 2) hemoglobin - PO 2that occurs when 0 2 - Hyperirritability f/b increased muscular twitching®
oxygen buffer mechanism is no is breathed at very high - Ringing in ears
longer capable of keeping the alveolar 0 2 pressure for a - Conuilsions w ithout warning®
tissue PO 2 in normal safe range short period. The nervous - Dizziness, irritability, disorientation®
20-60 mmHg, and tissue PO 2 can tissue is most susceptible - Coma & death
rise to hundreds or thousands of because of its high lipid Pulmonary (Smith) effects, is usually d/t chronic
mm Hg. content. exposure but may also be seen in acute.
This results in production of 2) Chronic toxicity results - Tracheobronchial irritation & pulmonary edema® are
oxygen free radicals (e.g. from breathing relatively first presentations
superoxide anion, singlet lower concentration of O 2 - Lung passage congestion
oxygen, & peroxide radical) in but for prolonged - Atelectasis
very high amount which periods. A person can be Retinal effects
completely swamp the enzyme exposed to 1 atm pressure - Retrolental fibroplasia in neonates®
system to remove them. of 0 2 for indefinite period - Ratinal damage® d/t angiogenesis causing
Free radicals oxidize poly without developing CNS disturbance of vision
unsaturated fatty acids present in toxicity. But it 1/t Burnt out tissue - i.e., d/1 high metabolic rate and
cell membranes (mainly) and some pulmonary toxicity after excess heat production, cytochrome system & finally
enzymes (proteins) of cellular only about 12 hours tissue is damaged
metabolism.

Physiological & Pathological Effects of Smoking


Effect are related to number of cigarettes smoked, smoking duration, amount and type of smoke inhaled. The effects are
produced by nicotine & carbon monoxide. It

Increases Decreases
- Glucose (from with in minute to an hour) - HDL®
- Insulin (after an hour) - Blood partial pressure of 0 2
- Growth hormone (GH), within 30 minutes) - Immune response, IgA, IgG and IgM
- Vasopressin, and IgE - Uterine cancer among post menopausal women
- Free fatty acid, cholesterol, Beta lipoprotein and , - Expiratory airflow
triglycerides
- Thrombogenic balance of prostacyclin & thromboxane A2, No association
RBC, hematocrit, leukocytes, & fibrinogen 1/t alteration in
platelet aggregation & survival that produce thrombosis. - Post menopausal breast cancer
- RBC, carboxyhemoglobin and hematocrit® - Endometrial cancer

- Endothelial (blood vessel) damage 1/1 increased


permeability
Increased risk & extent of advanced atherosclerotic
- Cancers of lung, oral cavity, pharynx (naso/oro/hypo),
vascular disease in peripheral as well as coronary
larynx, naal cavity, paranasal sinuses, esophagus, stomach,
arteries causing claudications, amputation, occlusive
pancreas, lieve, kidney (body and pelvis), ureter, urinary
cerebro vascular disease (stroke), abdominal aortic aneurysm,
bladder, uterine cervix, and myeloid leukemia.
myocardial infarction and failure of vascular reconstruction
- Smoking may play a role in increasing colorectal &
(graft occlusion).______________________________________
premenopausal breast cancer.
- COPD, chronic mucous hyper plasia, chronic productive
cough, pulmonary emphysema.
Physiology: Respiratory System ■ 295

Respiratory Failure

Respiratory failure is defined as a disorder wherein lung function is inadequate to meet the metabolic demands of the
individual and is unable to maintain normal arterial gas level in the blood.

Type Characteristic Features Causes


RF-I Represents failure of Results from failure in exchange of respiratory gases (mainly O 2) at the alveolar
oxygenation, and is capillary junction (i.e. alveolar capillary block syndrome) as a result of disease of
characterized by dysponea and lung parenchyma. There is thickening of alveolar or capillary wall resulting in
secondary hyperventilation 1 /t ventilation -perfusion (V-Q) mismatch. Only O 2 transfers is affected because C 0 2 is
- Hypoxemia (Pao 2- decreased; 20 times more diffusible. So causes are
< 60 mm Hg) I. Parenchymal (interstitial) lung diseases
- Increased alveolar - arterial - which thicken alveolar - capillary membrane eg. asbestosis, sarcoidosis,
Oxygen gradient*? (Pa-s02 > 15 pneumoconiosis, berylliosis, diffuse interstitial fibrosis, infiltrative lung disease
mm Hg) like malignancy & granulomatosis.
- Normal or decreased Paco 2 (S - Which separate A-C membrane like pulmonary (interstial) edema (in cardiac
40 m m Hg) i.e. respiratory failure) and exudates (pneumonitis or pneumonia)Q and (resultant) pulmonary
alkalosis fibrosis.
II. Mixing of venous blood with arterial blood like right to left shunt G.
III.V entilation - Perfusion mismatch Q: In emphysema surface area for diffusion
decreases (i.e. poorly ventilated alveoli increase).

Mn: "R A P E -V IP " = "R ight to left shunt, Alveolar (pulmonary) edem a, parenchymal
disease (pneumonia), emphysema - ventilation perfusion m ism atch".
R F II Represents failure (defect) in I. Decreased central respiratory drive to breathe
ventilation and is characterized - Drug like morphine, sedative & anesthetics overdose.
by hypoventilation 1/t - Brain stem injury, bulbar poliomyelitis®, hypothyroidism, sleep disordered
- Hypoxemia (Pao2 decreased; < breathing.
60 mmHg) II. Respiratory muscle weakness
- Normal alveolar-arterial - N eurom uscular disorders like myasthenia gravis, GB syndrom e, bulbar
oxygen gradient (P a-302 < 15 poliomyelitis, ALS.
mmHg) - M yopathy, polymyositis, electrolyte derangement.
- Hypercapnia (Paco 2 > 40 mm III. Obstructive lung disease
Hg) i.e. respiratory acidosis Q - Acute obstruction like foreign body, laryngeal edema, bronchospasm, asthma
- COPD (esp during acute exacerbation) like chronic bronchitis, emphysema,
interstial lung disease.
IV. Increased load on respiratory system d/t
- Resistive load eg bronchospasm
- Reduced chest wall compliance eg. pleural effusion, pneum o/fibro - thorax,
abdominal distension (ascitis), rib cage disorder (kyphoscoliosis)®, ankylosing
spondylitis, flail chest.
- Reduced lung compliance eg. atelectasis, lung resection, alveolar edema
(ARDS)*?, PEEP (positive end expiratory pressure).
- Increased minute ventilation requirements eg pulmonary embolism with
increased dead space, sepsis.
R F III Also called peri-operative Occurs as a result of atelectasis and atelectasis is comm on in perioperative period
respiratory failure
R F IV Result of hypoperfusion of Occurs in patient with shock
respiratory muscles

Effects of Hypercarbia/ Hypercapnia (Increased CO2 content of Blood)

Respiration C.V.S CNS Eye


Dyspnea - Tachycardia ”1(Due to sympathetic - Headache Mydriasis
- Hypertension J over activity) - Dizziness (due to sympathetic
- Loss of consciousness activation)
296 ■ A Complete Review of Short Subjects

Central (Neural) Control of Breathing

Medulaiy Respiratory Centers ' fa

Main components of respiratory control pattern generators responsible for automatic respiration are located in medulla and
inherent rhythmicity of breathing originates in medulla. Therefore periodic breathing is maintained in animals with brain
transaction b/w pons & medulla. Medulla has.
1. Pre-Botzinger Complex (Pre-BOTC)
- Spontaneous automatic rhythmic respiration is initiated by a small group of synaptically coupled pacemaker cells in the
pre-Biotzinger complex (Pre-BOTC) situated on either side of medulla between nucleus ambiguous and lateral reticular
nucleus.
- Pre-BOTC neurons discharge rhythmically, producing rhythmic discharges in phrenic motor neurons, which can be
abolished by section b/w the two (pre BOTC & phrenic motor neurons).
- Pre- Botzinger complex also contact hypoglossal nuclei, and the tongue is involved in the regulation of airway resistance.
- Pre-BOTC neurons discharge rhythmically even in in-vitro brain slices. However, hypoxia causes gasping and addition of
cadmium causes occasional sig h -like discharge patterns.
- Pre BOTC neurons have NK1, p-opioid and 5-HT4 receptors; and in vivo substance P stimulates, opioids inhibit respiration
and treatment with 5HT4 agonists block the inhibitory effect of opiates on respiration, with out inhibiting their analgesic
effect.
- Dorsal & ventral group of respiratory neurons involved in generation of respiratory pattern (however, lesions of both do not
abolish respiratory activity), project to pre-BOTC.
2. Dorsal respiratory group (DRG).
- DRG is composed of neurons located mostly in nucleus tractus solitarius and adjacent reticular substance in dorsomedial
region in medulla.
- DRG constitute the initial intracranial processing station for afferent inputs from 9th & 10th cranial nerves, which originate
from peripheral chemoreceptors, baroreceptors and several other receptors of airways and lungs.
- DRG is chiefly a/w inspiration and cause rhythmic inspiratory discharge. Action potential in inspiratory neurons begins
weakly, and increase steadily in a ramp like (crescendo) manner for about 2 seconds. Then it ceases abruptly for 3 seconds,
which turns off inspiratory muscles (diaphragm) and allows elastic recoil of lung & chest wall to cause expiration.
3. Ventral respiratory group (VRG)
- Located bilaterally about 5mm anterior & lateral to DRG, in ventrolateral region of medulla. It contains 3 group of both
expiratory and inspiratory neurons (stimulating both)._________________________________________
(Rostral) Active during expiration
Nucleus retro fascialis
(Caudal) Rostrally (cranially) located cells are active during inspiration
Nucleus retroambiguus (NRA) and caudally located cells are active during exhalation.
Nucleus paraambiguus (NPA) Has both inspiratory & expiratory neurons that travel in
vagus nerve to laryngeal & pharyngeal muscles.
- VRG neurons remain almost totally inactive during normal quiet respiration but act as an over drive mechanism when high
levels o f ven tilation are required esp during heavy exercise (when they especially provide powerful expiratory signals to
abdominal muscles.)
- So, normal quiet breathing is caused only by repetitive inspiratory signals from DRG and VRG contributes extra respiratory
drive for increased (greater than normal) pulmonary ventilation. Inspiration begins with abrupt increase in discharge from
NTS (DRG), NRA (Cranial cells) and NPA.
★ The fibers concerned with expiration converge primarily on the internal intercostals motor neurons in the th oracic sp in al cord.®

Pontine and Vagal Influence 'fy '

Spontaneous rhythmic discharge of medullary neurons is modified by neurons in pons and afferents in vagus (from receptor in
airways & lungs). Pons has 2 centers.
1. Pneumotaxic center (PC)
- Located in medial parabrachial & Kolliker-Fuse nuclei of upper dorso-lateral pons, and contain neurons active during
inspiration & neurons active during expiration. It transmit signals to inspiratory area.
- It may have a role in switching between inspiration and expiration and in fine tuning of respiratory rhythm (because
normal rhythm can exist in the absence of this center)
- It controls the switch off /(turn off point of inspiratory ramp (of DRG), thus inhibiting inspiration. It regulates inspiration
volume by controlling the duration of filling phase of lung cycle primarily, which has secondary effects on respiratory rate.
Physiology: Respiratory System ■ 297

- Primary function of PC is to limit inspiration (volume) and secondarily increasing the rate of breathing. So stimulation or
strong signal 1/t shallow & rapid breathing.
- When pneumotaxic signal is weak (or damage to PC) causes deep (greater tidal volume) and slower respiration (in intact
vagi animals); and prolonged inspiratory spasms resembling breath holding (apneusis) when the vagi are also cut.
Stretching of lungs during inspiration initate impulses in afferent pulmonary vagi which inhibit inspiratory discharge. That's
why depth of inspiration increases after vagotomy.
2. Apneustic Center
- Located in lower pons, have an excitatory effect on inspiratory area of D RG , tending to prolong the ramp action potential. It
is normally inhibited by pneumotoxic center & vagus.
- Named so, because section just above this cause apneusis (prolonged inspiratory gasps) interrupted by transient expiratory
effects®._______________________________________________________________________________________________________________
_________________________________ Emotional-Limbic & Cortical-Voluntary Control.__________________________________
- Cortex can override the functions of brain stem within limits if voluantry control is desired, eg by voluntary hyperventilation
decreasing Pco 2 and increasing arterior pH.
- Ondines' curse is disruption o f automatic control without loss of voluantry control of respiration. So respiration occurs
only in awake state (not during sleep)1?.
- Limbic system & hypothalamus alter pattern of breathing in emotional states such as fear & rage.

Chemical Control of Respiration


Central - Chemoreceptors
- Pneumotoxis
- Pneumotaxic center, and dorsal & ventral respiratory group of neurons are not affected directly by
changes in blood CO 2concentration or H+ ion concentration in CSF
- A chemosenstive area, located bilaterally, only 0.2 mm (Guyton)/200-400mm (John West)
beneath the vental surface of medulla1?, is highly sensitive to changes in either blood Pco 2
or hydrogen ion concentration (in CSF and interstial fluid), and in turn excites other
portions of respiratory center.
- Central chemosensitive area of respiratory center, that mediate hyperventilation produced
by increase in arterial Pco2 after the peripheral chemoreceptors (carotid and aortic bodies)
are denervated are located in ventral surface of medulla oblongata in the vicinity of exit of
9th & 10lh CN (k/a medullary chemoreceptor). Additional chemoreceptors are located in
vicinity of nuclei of solitary tract, locus ceruleus and hypothalamus.
(R) Rostral & Caudal (C) Chemosensitive area
I in ventral surface of medulla
I 1
Primary Indirect Stimulus Primary D irect Stimulus
is CO 2 concentration (Pco2>in Blood is H+ ion concentration (or pH) in CSF and ECF._______________________
- Although CO 2 has very little direct effect in - Central chemoreceptor is surrounded by brain ECF (interstitial fluid)
stimulating chemosensitive area, it does have a and CSF and separated from blood by BBB, which is relatively
potent indirect effect®. im permeable to H+ and H C O 3 ' (penetrate slowly) although CO 2
- Carbon dioxide passes through blood brain diffuses across it easily.
barrier alm ost as if the barrier does not exist®. - Blood brain barrier is not very permeable to H+ ion. So, changes in H+
So whenever, the blood PCO 2 increases, so does ion concentration in blood have very less effect in stimulating the
the P C 0 2 of CSF and interstitial fluid of medulla. chem osensitive neurons ® than do changes in blood CO 2, even though
Here CO 2 reacts with water to form new C 0 2 is believed to stimulate these neurons secondarily by changing
hydrogen ions. Thus, paradoxically, more H+ hydrogen ion concentration.
ions are released in respiratory chemosensitive - H+ ion concentration in CSF and interstitial fluid® (not in blood) is the
sensory area of medulla when the blood CO 2 only important direct stimulus. An increase in H+ ion concentration
concentration increases than when the blood stimulates ventilation, whereas a decrease inhibits it.
hydrogen ion concentration increases. - CSF with normal pH 7.32 has much lower buffering capacity than blood
298 ■ A Complete Review of Short Subjects

- That's why, respiratory center is stim ulated b /o much less protein content. Therefore, the change in CSF pH for a
very strongly (but indirectly) by changes in given change in Pco 2is greater than in blood. Similarly, the CSF pH
b lood CO2 concentration ®. Whereas, blood pH returns to its near normal value more promptly than does blood pH by
(H + ion concentration) has a little effect (b /o renal compensation. That's w hy CSF pH has more im portant effect on
relative impermeability of BBB to H + ion) ventilation than blood Pco 2- And for the same reason a person with
- Changes in Pco 2 of blood and CSF have only long standing CO 2 retention ( d/ t chronic lung disease or exposure to 3%
minor effects on respiration as long as H+ ion CO 2 for few days) has nearly norm al CSF pH and, therefore, an
concentration in CSF is held constant. Thus the abnormally low ventilation for his arterial Pco 2-_________________________
effects of CO 2 on respiration are mainly d / t its
movement into CSF and brain ECF, where it
increases H+ ion concentration and stimulates
central chemoreceptors sensitive to H+ ion. Thus
the CO 2 level in blood (Pco 2> indirectly
regulates ventilation by its effet on pH (H+ ion
concentration) of CSF.
- A change in blood CO 2 concentration has a
potent acute effect on controlling respiratory drive
but only a w eak chronic effect Q after few days of
adaptation. This results from renal readjustment H+ ion stimulates chemosensitive area, but CO 2gives rise to most of the
of H+ ion conc. towards normal, by increasing H + ion
blood bicarbonate. But more importantly these O2 has no Central role
Changes in O 2 concentration have virtually no direct effect on
HCO 3 ions diffuse through blood brain &
respiratory center® (although do have an indirect effect, acting through
blood - CSF barrier and combine directly with the peripheral chemoreceptor.)
H+ near respiratory neurons.

Peripheral Chemoreceptors
• Carotid bodies (located at bifurcation of common carotid arteries) and aortic bodies (located above & below aortic arch) are
the peripheral chemoreceptors; an these respond to decrease in arterial P 02 and pH, and increase in arterial Pco 2.
• In humans carotid bodies are most important; and the carotid but not the aortic bodies respond to a fall in arterial pH.
• They are unique in that their sensitivity to changes in arterial P0 2 begins around 500 mmHg, have non linear; relatively little
response until P 02 is reduced below 100 mmHg. M aximum response occurs below P 02 of 50 mmHg.
• Peripheral chemoreceptors are responsible for all response to arterial hypoxemia (P 02) and blood pH whereas, less than 20%
of ventilatory response d / t Pco 2 can be attributed to it. However, their response is more rapid and is useful in matching
ventilation to abrupt changes in Pco 2-
• If the peripheral chem o receptor is denervated the response to

- Arterial hypoxemia (P 0 2 ) and pH is com pletely abolished®


- H yper capnia (arterial PC02) is reduced by 30%.
• Carotid bodies contain glomus cells of 2 types surrounded by fenestrated sinusoidal capillaries
- Intense fluorescent type I or glomus cells are closely associated with cup like endings of the afferentnerves. It resemble
adrenal chromaffin cells & have dense core granules containing catecholamines that are released upon exposure to hypoxia &
cyanide. The cells are excited by hyoxia & principle transmitter appears to be dopamine which excites the nerve endings by D2
receptors
- The type II cells are glial cells, and each surrounds 4- 6type I cells. Their function is probably
sustantecular
- Afferents from carotid body ascend to medulla via carotid sinus & glossopharyngeal nerves and
fibers from aortic bodies ascend in the vagi
- Type I glomus cells have O 2 sensitive K+ channels; whose conductance is reduced with
hypoxia —» 1/t J. K> efflux —>depolarizing the cell & causing Ca++ influx via L type Ca2+
channels —> triggers AP & excitation of afferent nerve endings.
• The smooth muscles of pulmonary arteries contain similar O2 sensitive K* channels, which
mediate the vasoconstriction caused by hypoxia®. This is in contrast to systemic arteries, which
contain ATP dependent K* channels that permit more K+ efflux with hypoxia 1/t vasodilation
(instead of vasoconstriction)
• The blood flow in each 2mg carotid body is ~ 0.04 ML/min or 2000 ML/lOOgm/min. Because of
enormous blood flow the O 2 needs of the cells can be met largely by dissolved O 2 alone
- Therefore, the receptors are not stim ulated in conditions such as anemia & carbon monoxide
Physiology: Respiratory System ■ 299

poisoning ®, in which amount of dissolved O 2in the blood reaching the receptors is generally normal even though the
combined O 2in the blood is markedly decreased.
- The receptors are stimulated when the arterial PO2is low or when, because of vascular stasis the amount of O 2delivered to receptor
per unit time is decreased.
- Powerful stimulation is produced by cyanide Q; which prevents O 2utilization at the tissue level
- Nicotine & lobeline activate chemoreceptors.
- Infusion of K* increases the discharge rate in chemoreceptor afferents and since the plasma K+ level is increased during
exercise, the increase m ay contribute to exercise induced hyperpnea.
• The response of aortic bodies are similar but of lesser magnitude. In humans in whom both carotid bodies have been
removed but the aortic bodies left intact, the response are essentially the same as those following denervation of both
carotid & aortic bodies in animals: little change in ventilation at rest, but the ventilatory response to hypoxia is lost lost &
ventilatory response to CO2 is reduced by 30%®

Response o f Chemoreceptors to hypoxemia

- The peripheral chemoreceptors are sensitive to hypoxia but central chemoreceptors are not. These respond to P02 rather
than oxygen content of arterial blood. For the same reason conditions with low O 2content but normal P 02 like CO poisoning,
anemia & methaemoglobin do not stimulate receptor (or cause reflex hyperpnoea)
- Body's mechanism for detecting O 2 lack is relatively insensitive (cause no significant hyperpnoea until O 2conc < 8% or P02 <
60 m m H g or Hb saturation < 70%) because reduced oxygenation of Hb causes it to bind H + and a fall in [H+] inhibits
respiration, thereby countering the stimulatory effect of O 2 lack. The receptors are non adapting and continue to fire impulses
as long as the P 02 is low, even after death of all tissues, and are responsible for last gasp of a dying man, hence they are called
ultimum moriens.
- If oxygen is administered to hypoxaemic patient, who is unresponsive to high Pco 2, it could be dangerous, because the rise
in P02 also removes the hypoxic stimulus to respiration and the patient may die of CO 2 retention and com a (CO 2 narcosis).

Response to Hypercapnia
- Both peripheral & central chemoreceptors are sensitive to CO 2 but peripheral receptors are not (or less) important in control
of respiration at physiological range, because they are less sensitive (have higher threshold).
H ow ever peripheral chemoreceptors respond rapidly within 1 second unlike central receptors which respond after a delay of
some minutes (b /o time required for CO 2 to enter CSF and produce H+).

Nonchemical Responses Mediated by Pulmonary Receptors and Reflexes ★ Bronchial C fiber receptors
located in the airways and
innervated by nonmyelinated
Receptor Type Location Impulse travel Stimulus Response /Reflex
fibers are activated by chemicals
in
(notably bradykinin) injected
(i.e. Vagal
into bronchial artery. Their
innervation)
stimulation gives rise to
Slow ly adapting Airway M yelinated Lung inflations - Inspiratory time tachypnea, bronchoconstriction, and
Stretch Receptor smooth vagal fibers shortening airway secretion. Along with type
(pulmonary or muscle (?) - Hering-Breuer ] (juxta pulmonary capillary)
bronchial stretch inflation & deflation receptors, they contribute to air way
receptors) reflexesQ defence reflexes.
★ Receptors in nose & upper
- Bronchodilation
airway (nasopharynx, larynx &
- Hyperpnea trachea) responding to
-Tachycardia mechanical & chemical
Rapidly adapting Airway M yelinated -Lung - Increased airway stimulation 1 /t sneezing,
coughing, broncho constriction and
Irritant Receptors epithelial vagal fibers Hyperinflation resistance i.e.
Q aryngospasm.
(type of cells broncho constriction
★ Impulses from jo in t & muscle
s tre tch / mechano- - Exogenous & (eg asthama)
receptors of moving limb
receptor) endogenous - Mucus secretion stimulates ventilation during
substances eg - Cough early exercise.
histamine and ★ Gamma system or muscle
- Hering-Breuer
prostaglandins spindle of thoracic intercostal
deflation reflex
muscles and diaphragm sense
300 ■ A Complete Review of Short Subjects

J (Juxta- In Unmyelinated - Lung Hyper - Apnea (d/t intense elongation of these muscles and
capillary/alveolar) pulmonary (pulmonary inflation^ stimulation) f/b reflexly control strength of
Receptors interstitial vagal) (large) rapid, shallow contraction. For example
dyspnoea is sensed by unusually
(Juxta pulmonary space close C_fibers - Exogenous & breathing
large respiratory efforts required
capillary receptor to blood endogenous (tachypnea) and
to move the lung.
or pulmonary C capillaries of agents eg dyspnea
★ Pain cause apnea followed by
fiber receptor) - Pulmonary capsaicin, - Broncho constriction hyperventilation. Heating of skin
circulation bradykinin, - Mucus secretion (or fever) result in
- Bronchial serotonin, hyperventilation (partly d/t
- Brady cardia
circulation histamine, stimulation of hypothalamic
- Hypotension
prostaglandins, thermorecep tor).
phenyl (J-pulmonary
★ Arterial (aortic & carotid sinus)
diguanide chemoreflex) baroreceptor stimulation by
- Pulmonary increased BP causes reflex
edema, hypoventilation or apnea;
embolism & whereas decreased BP 1/1
congestion^ hyperventilation (thereby
enhancing venous return
follow ing severe hemorrhage).

Respiratory Reflexes & Receptors

Head's Paradoxial Hering-Breuer Reflex & Stretch Receptors J-Receptor Reflex


Reflex - J (juxta capillary) receptors, are
Slowly adapting pulmonary stretch receptors lie within
■Vigorous inflation of nerve endings of unmyelinated C
airway smooth muscle. They discharge in response to lung
the lung reflexly fibers of vagus. They lie in alveolar
distension and show little adaptation i.e. their activity is
induces a contraction walls close to pulmonary capillaries
sustained with lung inflation. The impulse travels in large
of the diaphragm, and are stimulated by hyper
myeliated fibers of vagus nerve to DRG neurons and is
further augmenting inflation of lungs, intravenous or
responsible for Hering-Breur reflex.
the size of breath. intracardiac administration of
Their stimulation affects inspiration in much the same way
This activity is chemicals such as capsaicin,
as signals from pneumotaxic center i.e. when the lungs become
mediated by lung bradykinin & serotonin.
over stretched (overly inflated), stretch receptors activate
irritant receptor.Q - They are specially stimulated in
appropriate feed back response that switches off the
- Rapidly adapting pathological states when pulmonary
inspiratory ramp and thus stops further inspiration. This is
pulmonary stretch capillaries become engorged with
k/a Hering Breuer inflation reflex; and like pneumotaxic
(mechano) receptor em boli (or blood) or when
center, this reflex increases rate o f respiration.
also k/a irritant pulmonary edema or pulmonary
Hering Breuer reflexes were once thought to play a major
receptors lie between congestion occurs in left heart
role in ventilation by determining the rate & depth of
airway epithelial failure, congestive cardiac failure
breathing by modulating the switch off mechanism in
cells of trachea & and interstial lining disease.
medulla. Eg bilateral vagotomy, which removes input from
large airways. They - The impulse pass in vagus nerve and
these receptors cause slow, deep breathing in most animals.
are stimulated by result in apnea (d/t intense
•However, more recent work indicates that H-B reflexes are
noxious stimuli like stimulation) followed by rapid,
largely inactive in adult humans (not animals) until tidal
gases, smoke, inhaled shallow breathing and dyspnea (i.e.
dust and cold air in the volume exceeds 1/1.5 liters, as in exercise. Eg transient
sensation of difficulty in breathing),
inspired air. The bilateral vagal block by local anesthesia in awake human
bradycardia, hypotension, broncho
impulses travel up in does not change either breathing volume or rate. Therefore
constriction and mucus secretion
myelinated fibers of this reflex protect against excessive lung inflation rather
(pulmonary chemoreflex). A similar
vagus and reflex than an important ingredient in normal control of
response is produced by stimulation
effects include ventilation, in adult humans. Some evidence show that
of receptors in heart (Bezold-Jarish
broncho constriction these reflexes may be more important in newborn babies.
reflex or coronary chemoreflex). The
(increased airway physiological role o f this reflex is
resistance), H-B Inflation reflex H-B Deflation reflex uncertain but probably occurs in
coughing, mucus When lung becomes Excessive deflation of pathological states like pulmonary
production and overstretched, the stretch lung causes reflex congestion, pulmonary edema and
hyperpnea (John receptors located in the inhibition of expiration, pulmonary em bolism Q.
Physiology: Respiratory System ■ 301

west)/ reflex apnea bronchi and bronchioles it may serve to protect Bronchial C -fiber Reflex
(Berne). They may send signals to respiratory against lung collapse Similar reflex in response to chemicals
have a possibl role in neurons. These signals (atelectasis). In other injected into bronchial circulation.
acute attack of switches off the inspiratory words, HB- deflation
asthma. center and stops further reflex is decrease in Arterial-Baroreceptor Reflex
inspiration. It is not activated duration of expiration
Fall in BP increases both vasomotor
Protective Reflex until the tidal volume produced by marked
center (VMC) activity (Ting BP) and
During swallowing, increases to 1.5 litres. deflation of lungs.
pulmonary ventilation. Conversely
when food enters the ■Excessive inflation of lungs If lungs are greatly
increased BP cause reflex
oropharynx, there is a stimulate the stretch deflated impulses from
hypoventilation or apnea and I BP
reflex in hibition o f receptors, causing a receptors, travel up via
through stimulation of aortic &
respiration® and protective reflex nhibition vagus to stimulate
carotid sinus baro receptor.
closure of glottis, to of inspiration. In other inspiration. It comes into
prevent bolus from words, HB inflation reflex is play in conditions such
an increase in duration of as atelectasis. Pain & Temperature Reflex
entering airways.
expiration, produced by Pain causes apnea f/b
steady lung inflation. The hyperventilation; and heating of skin
main reflex effect is slowing may result in hyperventilation
of respiratory frequency d/t Joint & Muscle receptor; Muscle
an increase in expiratory spidle gamma system reflex
time.

Surfactant

Production and Uptake

- Surfactant is produced by type II alveolar epithelial cells (or pneumocytes), which develop to the end of 6 th month (24th
week) of gestation.
- Synthesis of surfactant begins at 16-18/20/24 weeks (a/t various sources). It is stored in alveolar cells until 26 weeks when
release on the alveolar surface occurs, and amount peaks at 35 weeks of gestation particularly during the last 2 weeks before
birth.
- Type II pneumocytes are rich in mitochondria, ER, and lam ellar bodies (i.e. membrane bound organelles containing whorls of
phospholipids). Secretion of surfactant into the airways occur via exocytosis of lam ellar body. Tubes of lipid called tubular
myelin is formed from extruded lamellar bodies which intum for phospholipids film. So tubular myelin is a precursor stage of
surfactant as it is initially secreted from type II cells and has not yet spread.
- Padrenergic agonist, purinergic agonists, activators of protein kinase C and leukotrienes stimulate exocytosis of surfactant.
- Major routes of clearance of pulmonary surfactant are reuptake by type II cells, absorption into lymphatics, and clearance by
alveolar macrophages.

Surface Tension
- When water forms a surface with air, there is a strong attraction between its molecule on surface, which is always attempting to
contract the surface. This tight contractile membrane of surface holds the water drop together.
- On inner surface of alveoli, the water surface is also attempting to contract. This attempts to force air out & cause alveoli to
collapse. This elastic contractile force of lung is called the surface tension elastic force.
- Surface tension is a force caused by water molecule at the air-liquid interface (in the alveoli) that tends to minimize surface
area, thereby making it more difficult to inflate the lung. Higher pressure is required to inflate the lung with air than with
saline because saline abolishes surface tension.
- Surface forces in a sphere attempt to reduced the area of surface and generate pressure with in sphere. According to Laplace's
law, the amount of pressure generated in alveoli (P)

p _ 2 x surface tension (T)


Radius o f alveolus (r)

- P is inversely proportional to radius of sphere. So surface forces in smaller sphere generate higher pressure than those in larger
sphere. As a result air moves from higher pressure (smaller sphere) to lower pressure (larger sphere) causing the small sphere
to collapse and large sphere to become overdistended.
302 ■ A Complete Review of Short Subjects

Effect of Surfactant

- Surfactant is a surface active agent which breaks this surface tension elastic force o f water molecules in alveoliQ. The
reduction in surface tension is greater when the film is compressed because the molecules of DPPC are then crowded closer
together and repel eachother more. Surfactant lowers surface tension more in smaller spere than in larger sphere (i.e. in
lungs small surface area is a/w small surface tension). The net result is that the pressure in small and large sphere (alveoli) is
similar, the tendency for small alveoli to empty into large alveoli is reduced
and alveoli are stabilized (i.e. do not collapse).
- Surface tension of saline etc and its lowering by detergent etc is independent of
area whereas reduction of surface tension by surfactant is area dependent.
The surfactant thins out when alveoli are large and stretched during
inspiration, but packs closely as the radius reduces during expiration. Because
of surfactant small surface area is a/w small surface tension in lungs. Therefore
the surface tension falls as the alveolus deflates.
- B/O surfactant there is an increase in surface tension at high lung volumes and a decrease at low lung volumes. So lung can
maintain alveoli at many different volumes. Otherwise, small alveoli would empty into large alveoli. As a result, the
transmural pressure required to keep an alveolus inflated increases as lung volume (and transpulmonary pressure) increases
and it decreases as lung volume decreases.
- Stability o f alveoli is mainly the function o f surfactantQ which regulates the alveolar size by regulating surface tension and
thus preventing their collapse at the end of expiration. As the alveoli increase in size (at the end of inspiration), the surfactant
becomes more spread out over the surface of liquid allowing surface tension to increase. This effectively slows the rate of
increase of alveoli preventing over expansion and helping all alveoli in lung to expand at the same time.
- Negative intrapulmonary pressure, residual air and compliance mainly affect the inflation and deflation of lungQ. They do
have some effect on stability but most important factor remains surfactant as the maximum work of breathing is done to
oppose surface tension.

Function

- DPPC is the major component and it decreases surface tension.


- Type II pneumocytes occupy only 5% of alveolar surface whereas surfactant must spread and form film over entire alveolar
surface. PG, SP-B, SP-C and calcium have spreading properties, required for film formation.
Mutation of SP-C gene is a/w familial interstial lung disease.
- SP-A is a large glycoprotein and has a collagen like domain in its structure. It is involved in regulation of surfactant uptake by
type II pneumocytes. SP-A and SP-D (collection proteins) are involved in innate immunity in conducting airways and alveoli.
- Reduces surface tension in alveoli, which in turn increases the com p lian ce o f lungQ and reduces the work of expanding with
each breath.
- Alveolar size regulation and stability of alveoli is promoted, which prevents alveolar collapse (or alveolar atelectasis) at
the end of expiration.
- Prevents transudation of fluid (& pulmonary edema) by keeping alveoli dry. Because it counter 20 mmHg force of surface
tension favouring transudation of fluid from blood into alveoli.
★ Knocking out GM-CSF gene 1/t presence of excess surfactant proteins & lipids in mice (resembling pulmonary alveolo
proteinosis of humans)
★ Mucin is a glycoprotein which is secreted by mucous secreting bronchoalveolar Ca.

Deficiency Occurs with Mechanism of Decreasing Surface Tension


Occlusion of main bronchusQ Dipalmitoylphosphatidyl choline, along with other
Occlusion of one pulmonary arteryQ phospholipids reduce surface tension by not
Cigarette smokersQ dissolving uniformly in the fluid lining the alveolar
Long term inhalation of 100% O 2. surface. The hydrophilic head dissolves in fluid and
Cardiac surgery involving use of pump oxygenator. 2 hydrophobic tails face the alveolar lumen. Instead,
Deficiency of surfactant 1/t infant respiratory distress syndrome part of molecule dissolve, while the remainder spreads over
(IRD S) /hyaline membrane disease, patchy atlectasis, and the surface of water.
pulmonary alveolar proteinosis. Exogenous bovine surfactant The surface tension of this nonuniform surface
(beractant) through endotracheal tube & mechanical ventilation is containing surfactant is 5-30 dynes/cm, which is - 1/12
used for management in IRDS. to 1/2 of surface tension of pure water (72 dynes/cm).
RDS is common in diabetic (4- insulin) mothers, low thyroid Whereas, alveoli without surfactant produce 50 dyne
harmone and low glucocorticoid levels and in premature babies. /cm surface tension.
Physiology: Respiratory System ■ 303

Hyaline Membrane disease / Infant Respiratory Distress Syndrome (IRDS)


When surfactant is insufficienct, in premature infant, the air-water (blood) surface membrane tension becomes high, bringing
great risk that alveoli will collapse during expiration, In RDS, the partially collapsed alveoli contain fluid with high protein
content, hyaline membranes and lamellar bodies.
____________I_____________________ .
I
Alveolar collapse Prolonged immaturity of Pathophysiology
(atelectasis) ENaCs (Epithetial Na+ Decreased A lveolar surfactant® — ►Increased alveolar surface tension
d/t high surface Channels) 1/t retention of - Hypoxemia I
tension fluid in lungs______ - CO2 retention ■*— Collapse /Atelectasis
High surface tension - During fetal life, Cl- is - Acidosis
in the alveoli makes secreted with fluid by I
it very difficult for the pulmonary epithelial Pulmonary hypoperfusion
the lungs to expand cells
in child bom - At birth there is a shift to I--------------
deficienct in Na+absorption by these Endothelial damage Epithelial damage
surfactant, because it cells Via ENa Cs and
I_____
would require fluid is absorbed with i
intrapleural pressure Na+ Plasma leak into alveoli
as high as -20 to -30 - Immaturity of these _________ L_________
mm Hg to overcome channels 1/t retention of Fibrin + Necrotic cells®
the surface tension. fluid in lungs. I
Hyaline membrane.

Non-Resplratory - Metabolic Functions of Lungs

Fate of biologically active substances metabolized m lungs As Blood reservoir


Synthesized & used - Surfactant® (DPPC) Because lung is the only organ except the heart that
in lungs - Proteins collagen & elastin of framework receives the whole circulation and has uniquely
- Carbohydrate mucopolysaccharide of mucous remarkable ability to reduce its pulmonary vascular
Synthesized or • Histamine resistance by recruitment and distension, it acts as
stored & released • Arachidonic acid metabolites a reservoir for blood.
into the blood. - Prostaglandins & Thromboxane A2 (by Blood filtration
Mn-"HAK" cyclooxygenase) Pulmonary capillary bed acts as a blood filter and
- Leukotrienes including SRSA i.e. slow reacting remove small thrombi (dots), bubbles, detached
substance of anaphylaxis (by enz. lipooxygenase) cells and prevent them from entering systemic
• Kallikrien circulation.
Activated in lungs - Angiotensin I is converted to angiotensin II by
Fibrinolysis and Anticoagulation
ACE.
Lung is rich in mast cells, which secrete
Partially removed - Norepinephrine (up to 30% is taken up by lungs)
anticoagulant heparin. And pulmonary
from the blood - Bradykinin (80% inactivated by ACE)
endothelium is rich in plasmin activator, which
Mn-"No-BAAPS" - Acetylcholine
converts plasminogen into plasmin and promotes
- Adenine nucleotide
fibrinolysis. This is how lungs delay clotting and
- Protaglandins Ei, E2 and F201 (almost completely
dissolve small clots (fibrinolysis).
inactivated)
- Serotonin (almost completely removed by Mucosal Immune System
uptake & storage in platelets and released during Lungs contribute to defense against infection by
anaphylaxis) secreting immunoglobulin A (IgA). It also has
Passed through - Angiotensin II other defense mechanism like- filtration, epithelial
lungs without being - Adrenaline (epinephrine) barrier, mucociliary clearance, MALT, innate &
metabolized - ADH (Vasopressin) adaptive immune cells (like alveolar macrophage,
(without significant - Prostaglandin (Ai, A2) NK cells, NK/T cells, dendrite cells, P lymphocytes,
gain or loss of - Prostacyclin (PGI2) TCRap and TCR y5 lymphocytes providing
activity) - Oxytocin humoral & cell mediated immunity)
Mn-"DOPA" - Dopamine & histamine Acid Base Balance, Sodium balance & cooling
304 ■ A Complete Review of Short Subjects

Pulmonary Neuro-Endocrinal Cell (PNEC) System


It is more numerous in fetal/neonatal lung (than adults) and produce amines (eg dopamine, 5 HT or serotonin) and
neuropeptides (eg bombesin/GRP, CGRP and calcitonin). Chromogranin A immunohistochemical marker is most sensitive in
identification. PNECS has dual innervations from vagus and P/calcitonin gene related gene peptide IR nerve fibers. It
consists of
Solitary /Cluster PNECs Pulmonary Neuro Endocrinal Bodies (PNEB)
PNECs (and their secretion) are involved in lung development like Because of membrane bound O 2 sensing
branching morphogenesis, direct mitogenic effects on epithelium and complex (ie O 2 sensing K+ channels and
promote differentiation of adjacent epithelial cells. protein NADPH oxidase), PNEB acts as airway
Serotonin released from PNEC (eg Kultschitzky cells)/ PNEB is O 2 sensor especially during adaptation to
involved in lung fluid production and clearance and help in transition extrauterine life.
form fetal to extrauterine life. Hypoxia (J/PO2) activates O 2 sensor 1/1 Ca++
PNEC/PNEB hyperplasia is seen in disorders like broncho pulmonary dependent exocytosis of amines & peptides from
dysplasia, central hypoventilation syndrome, SIDS, NE hyperplasia of dense core vesicles (DCV=storage site).
infancy, CF and pediatric asthma.
Acid - Base Balance Calculation of pH: Handerson - Hasselbach Equation showing
- The pH of arterial plasma is 7 .4 0 and that of venous effect of CO2 & HCOs- concentration
plasma is slightly lower. A decreased pH below 7.4 is pH=pKA + [H C C V ]
acidosis and an increase in pH above is k/a alkalosis. log [ c o 2]
However, 0 .0 5 pH variations are without untoward Because CO 2 follows Henry law, its concentration (in mmol/lit)
effects. Normal pH range for adult is 7 .3 5 - 7 .4 5 and is can be replaced by solubility (a) x partialpressure (Pco2). And
maintained by lung, kidney and 3 main chemical buffer at body temperature (37°C), a = 0.03 and pKA= 6.1; so
systems in blood i.e. plasma proteins, hemoglobin and
PH :=6.1 log [HCCV
carbonic acid-bicarbonate system. + 0.03 x PC02
- When acid-base disturbance develops, body uses same
Because normal H C 03 concentration in arterial blood is 24
defence to compensate the pH change.
mmol/lit and Pco 2 = 40 mmHg.
- First line of defence against acid-base disorder is
pH= 6.1 Log 24 ' = 6.1 + log 20= 6.1 + 1.3 =
instantaneous ECF & ICF buffering. Metabolite disorder
+ 0.03 x 40 7.4
are buffered in both ICF & ECF compartments whereas,
all buffering in respiratory disorders occurs in ICF. - This shows the effect of Pco 2 (which is determined by lungs)
- Bicarbonate buffer system (main), Pi and plasma proteins and HCO3’ concentration (determined by chiefly by kidney.)
are ECF buffers. Bone Ca++ is additional source of ECF 4 Types of Add-Base Disturbances
buffering. Intracellular buffering involves movement of The ratio of bcarbonate/Pco 2 can be altered in 4 ways: both
H+ into & out of cell. The H+ is titrated inside the cell by HCO 3- and Pco 2 can be lowered or increased
H C O 3 - , Pi and histidine group of hemoglobin (protein).
I. Respiratory Acidosis is caused by Ted Pco 2 (hypercapnia
- Lungs are the second line of defence; ventilation rate caused by hypoventilation), which decreases HCO 3-/PC02 ratio
determines Pco 2- The lungs excrete over 10,000 mEq of and thus pH. It is compensated by intracellular (ICF) buffers
carbonic acid per day compared to with <100 mEq of and increased net acid excretion (NAE) by kidney i.e. kidney
fixed acids by kidneys. excreting a more acid urine by secreting H+ ions in form of
- Third (final) line of defence is kidneys, which make NH4+ or H2PO 4-; and reabsorbing HCO 3 (conserving
appropriate adjustments in the excretion of H C O 3 - and bicarbonate).
net acid. However, this may take hours to days (time
II. Respiratory Alkalosis is caused by lP c o 2 (hypocapnia),
required to increase synthesis of PCT enzymes involved
which increases HCO3" /Pco2 ratio and thus elevate pH. A T
in synthesis of NH 4+)
Pco 2 is caused by hyperventilation (eg at high altitudes). It is
- Com pensation of respiratory acidosis /alkalosis:________
compensated by ICF buffer and J.NAE (or increased HCO3"
• 10 mmHg increase in Pco2, increases HCO 3' level 1 excretion) by kidney.
mmol/L in acute and 4 mmol/L in chronic (>24hrs) stage. III. M etabolic Acidosis is caused by THCO 3 (d/t accumulation
However, HCO3- does not increase above 38 mmol/L of acids as in diabetic ketoacidosis, or tissue hypoxia 1/t lactic
• 10 mmHg decrease in Pco 2 decrease 2 mmol/L H C O 3 acidosis). The ratio of H C O 3 /PC02 and so pH falls. It is
level and 7 mmol/L H+ or increase 0.1 PH in acute stage. compensated by ICF & extracellular (ECF) buffers, lungs
Whereas in chronic stage 10 mmHg decrease in Pco 2 (hyperventilation that i PC02) and T renal NAE.
cause 4-5 mmol/L drop in H C O 3 " and 3 mmol/L drop in
IV. M etabolic Alkalosis is caused by THCO3" (d/t alkali
H C O 3 " and 3mol/L fall in H+ or 0.03 rise in pH
ingestion or prolonged vomitting) and thus raising
- Compensation of metabolic acidosis /alkalosis: HCO3 /PC02 ratio and pH. It is compensated by ICF & ECF
1 meq/L decrease/increase in HCO 3- cause 1.2 mmHg buffers, lung (hypoventilation) and I renal NAE.
decrease/0.7 mmHg increase in Pco 2 rspectively.
Physiology: Respiratory System ■ 305

High Altitude
The composition of air stays the same, but the t o t a l b aro m etric pressure falls® with increasing altitude, therefore P O 2also
falls® . Hypoxic hypoxia is a problem in normal individuals at high attitude.

M ountain sickness A cclim atization


• Condition characterized by the ill effects of hypoxia at An acclimatized person is a person who has developed
high altitude. This syndrome develops 8-24 hours after compensatory mechanism so that hypoxia causes lesser
arrival and lasts 4-8 daysQ. effects on the body.
• L o w PO 2stim u lates the arterial chem oreceptors® I
leading to an immediate increase in pulmonary ventilation 1. Increased pulmonary ventilation (Hyper ventilation)
—> This blows off CO 2 (TPCO 2 & TpH) -» Inhibits the Hypoxic stimulation of central chemoreceptor (carotid and
brain stem respiratory center —>R esp ira to ry distress® aortic) increases alveolar ventilation
- Increase in pulmonary ventilation blow s o ff CO 2, i
Pathophysiology Symptom PCO 2 and there by increases pH & inhibits central
chemoreceptors (respiratory center). However with
- Expansion of gases in the git- - Nausea, vomiting
acclim atization over 2-5 days, CSF begins to expel
- PulmonaryQ edema - - B reath lessn ess H C O 3 - & pH-i- es, this stimulates central
(d/t Ted p u lm on ary ca p illa ry P ulm on ary
chemoreceptors of respiratory center
p erm ea b ility an d n orm al left hypertenson®
2 Polycythem ia and Ted Hb i
a tr ia l pressure)®
3 Ted diffusion capacity
4 Circulatory system show
- Cerebral edemaQ - - Headache
- Increased cardiac output (30%) immediately, which
[low PO 2 causes vasodilation in - Irritability
becomes normal with in few weeks as blood
brain & if cerebral auto - Insomnia
hematocrit increases.
regulation does not - Weakness - Increased (systemic) tissue capillarity (or angiogenesis)
compensate, T in capillary
and capillary permeability.
pressure favours T ed
5 Cellular - Increased mitochondrial oxidative enzymes
transduation of fluid into
brain tissue] 6 2-3 D PG rises within RBC, shifts the O2 ' dissociation
curve to the right.® So O 2 unloading to tissue becomes
easier.

Compensatory Mechanism Involved In Acclimatization


Ventilatory Response Blood Response Compensatory
Changes In Tissue
- The initial ventilatory response is small as alkalosis tends to Erythropoietin secretion
counteract effect of hypoxia. However ventilation increases over increases & then falls - Mitochondria increase
next 4days (d/t I CSF PH). After 4 days, ventilatory response begins after 4 days when in number
to decline slowly but it takes years of residence at higher altitudes ventilatory response
- Myoglobin increases,
for it to decline to the initial levels. increases & arterial
which facilitates the
- The resp iratory a lk a lo s is produced by hyper ventilation sh ifts O 2- PO 2 rises
movement of O 2 in
H b d isso cia tio n curve to the left®, but a concomitant in crease in Increase in RBC begins tissue
R B C 2, 3, BP G tends to d ecrease the O 2a ffin ity o f hem oglobin® in 2-3 days and is
- Cytochrome oxidase
- The net effect is a small increase in P 5 0 . The decrease in O 2 affinity sustained as long as
content increases
makes more O 2 available to tissue. However the value of increase in the individuals remain
P 5 0 is limited because when the arterial P O 2 is markedly reduced, the
at high altitude
decreased O 2 affinity also interferes with O 2 uptake by Hb in lung.
- Hyperventilation cause respiratory alkalosis d/1CO 2 washout.
306 ■ A Complete Review of Short Subjects

Decompression Sickness
(Compressed Air Sickness, Dysbarism, Air Sickness, Bends, Diver's Paralysis, Caisson's Disease)

The disorder occur when a person returns rapidly to normal surroundings (atmospheric pressure) after exposure to high atmospheric
pressure, as in deep sea.
Because of Ted barometric pressure at deep sea, compression occurs. This compression reduces the volume of gases in the
body.
Due to sudden return to atmospheric pressure, the N 2 is decompressed and escapes from the tissue. Being a gas it forms
bubbles, the bubbles obstruct the blood flow producing air embolism.
Aviators*3 may develop decompression sickness if they are suddenly decompressed at very high altitudes as a result of
cabin failure.
- It manifest as bends (joint pain most commonly in knee), staggers (vertigo d/t formation of bubbles in vestibular apparatus),
chocking sensation, paresthesias accompanied by rashes & itching.
★ Mountaineer suffer from mountain sickness due to decreased barometric pressure.

Effects of High partial pressure of gases on body

N2 Narcosis Oxygen Toxicity

At high tissue concentrations, N 2 and other rare ga9es - Breathing high concentration (or very high alveolar O 2
such as helium & argon behave as anesthetic gases pressure) causes lung damage, nausea, muscle twitching,
Mild N2 narcosis sets in at round 120 feet below sea dizziness, irritability, disorientation, disturbances of vision
surface and individual feels like one intoxicated with and brain seizure followed by coma.
alcohol & is in high spirits exhibit a joviality & loses his - Above a critical alveolar PO 2(> 2 atmP), the hemoglobin - O 2
cares a phenomenon referred as raptures of death. buffering mechanism fails & tissue PO 2 rises to hundreds or
Manual dexterity is maintained but intellectual thousands of millimeters of mercury
function is impaired. - At these high levels amount of toxic oxidizing free radicals
At 150-200 feet, diver becomes drowsy; at 200-250 feet (such as superoxide free radical O2& peroxide free radical in form
strength decreases & becomes too clumsy to perform of hydrogen peroxide) swamp the enzyme systems (peroxidases,
required work. Beyond 250 feet (8.5 Atm. Pressure) catalses, and superoxide dismutases) designed to remove them
becomes almost useless. - In excess, free radicals oxidize the poly unsaturated, fatty
At high concentrations, nitrogen behaves as anesthetic acids of cell membrane and some cell enzymes. The nervous
gases and show increased solubility in lipid (fatty) tissues with its high lipid content is particularly vulnerable. There
substances o f neuronal membrane and m akes them less fore acute lethal O 2 toxicity is caused by brain dysfunction.
excitable by altering ionic conductance®.
Chapter 4. CARDIOVASCULAR SYSTEM: REVIEW NOTES

Conducting System of Heart

Anatomy Blood supply Histology Innervation

• S.A node is located at the Part Artery • It is composed for the • SA node develop from
junction o f superior venacave SA Node Atrial most part of m odified structures on the right
with right atriumQ just deep to branches of cardiac muscle Q that side of embryo AV node
the epicardium, near the - Right has fewer stria tions from structures on left
superior end of sulcus terminalis coronary and indistinct side. This is why
• AV node is located in the right artery boundaries. - SA node is supplied by
postero-inferior region of interatrial (65%) e • The SA node and to a right vagus & rt.
septum near the opening of - Left coronary lesser extent AV node Sympathetics
coronary sinus artery (35%) also contain small - AV node is supplied by
• There are three bundles of atrial round cells with few left vagus & It.
fibers that contain purkinje type AV - Right organelles which are Sympathetics
KT—J —
Node
fibers and connect the SA node coronary connected by gap • On each side, most
to AV node artery 0 junctions. These are sympathetic fibers come
- Anterior internodal tract of actual Pacemaker cells from the stellate
Bachman Bundle - Right also called P cells ganglion.
- Middle internodal tract of of His coronary • The atrial muscle fibers • Noradrenergic fibers are
Wenckebach arteryQ are separated from epicardial whereas vagal
- Posterior internodal tract of those of ventricles by a fibers are endocardial
Thorel RBB - Left
fibrous tissue ring, and • Reciprocal inhibitory
• Conduction also occurs through coronary
normally the only effect occurs b/w
atrial myocyte but it is more artery Q
conducting tissue sympathetic &
rapid in these bundles between the atria and parasympathetic
LBB - Left coronary
• AV node is the only conducting ventricle is the bundle system.
artery
pathway between atria & of His. - Neuropeptide Y
(mostly)
ventricles secreted from
- Right
• Bundle of His gives off a left noradrenergic endings
coronary
bundle branch (LBB) and inhibit acetyl choline
artery
continues as right bundle branch release.
(rarely)
(RBB). The LBB divides into an - Ach inhibit
anterior and posterior fascicle norepinephrine release.
• The branches & fascicles run
subendocardially and come into
contact with purkinje system
whose fibers spread to all parts
of ventricular myocordium

Action Potential in Cardiac Ventricular 1 pjT Difference b/w AP of Cardiac & Skeletal Muscles
Muscles
Cardiac muscles have prolonged AP (Action potential) and a platue
- Resting membrane potential is much more negative phase, whereas skeletal muscles do not because
i.e. -90 m ili volts in ventricular muscle fibers instead
of -55 milivolts in the nodal fibers. 1. AP of skeletal muscle is caused and almost entirely by sudden
- Action potential in cardiac muscle fibers averages opening of fast Na+ channels for a very short period. At the end of
about 105 mili volts i.e. the intracellular potential rises this abrupt closure, repolarization occurs and AP is over in another
form a very negative -90 mV between beats to slightly thousandth of a second. Whereas AP in cardiac muscle is caused
positive +20 mV during each beat. by same fast Na+ channels as well as slow Ca+ (Na+ - Ca++)
channels which are slower to open and more importantly remain
308 A Complete Review of Short Subjects

- 3 type of membrane ion channels in cardiac muscles open for several tenths of a second (i.e prolonged duration) thereby
responsible for causing the voltage changes include maintaining prolonged period of depolarization, causing the
platue phase.

Channel Effect
2. In skeletal muscles, Ca++ ions for contraction is derived from
intracellular sacroplasmic reticulum. Whereas, in cardiac muscles the
Fast Na+ Rapidly opening of fast sodium Cfl++ that enters during plate phase activate the muscle contractile
Channel channels for 10,000 ths of a second) process.
causes rapid influx of positive sodium
ion (Na+) to the interior of cardiac 3. Immediately after the onset of AP, the permeability of cardiac
muscle fiber (Na+current, I n3). It is muscle fiber for K+ decreases about 5 folds as a result of excess
responsible for rapid upstroke spike of Ca++ influx through Ca++ channels. Therefore, greatly decreased
action potential or rapid depolarization outflux of positively charged K+ ions during AP platue in cardiac
phase O. muscles prevents early return of AP voltage to its resting levels.
This greatly decreased K+ permeability is not seen in skeletal
Initial rapid repolarization phase lis muscles.
caused by inactivation of Na+channels.
D ifference b/w AP of Cardiac Muscle & Sinus Nodal Fibers
Slow Ca++ Slower opening of slow calcium (or
sodium-calcium) channels for about 0.3 •AT the less negative (- 55 mV) RMP of nodal fibers the fast Na+
(Na+ - Ca++) second (i.e longer duration) causes large channels mainly have already become inactivated (or blocked).
influx of both Ca++ and Na+ positive Therefore only the slow Na+ channels can become activated (open)
Channels ions to interior of cardiac muscle fibre. and cause AP.
This (Ca++ current/influx, lea mainly) is
•So AP is slower to develop in nodal fibers than ventricular fibers.
responsible for maintenance of
Similarly return of potential to its negative resting state occurs
prolonged period of depolarization or
slowly as well rather than the abrupt return that occurs in
platue phase (2) of ventricular AP.
ventricular muscles.

Potassium Rapid diffusion of large amounts of


(K+) positive K+ ions in outward direction
Channels (efflux) from the cardiac muscle fibre
(Ek+, K+ current) immediately returns
the membrane potential to its resting
levels, thus ending the AP

Mechanical Events In Cardiac Cycle (= 0.80 sec)


Both (left & right) sides of heart have similar but slightly asynchronous events. Right atrial systole and right ventricular ejection
begins before that of left. Whereas, right ventricular contraction starts after that of left. Events include
Physiology: Cardiovascular System ■ 309

Ventricular Systole = 0.27 sec D iastole = 0.53 sec duration


It has 2 phases which are It has 3 phases which are
1. Isovolumetric (Isovolumic, 1. Early diastole = 0.12 sec
Isometric) Contraction = 0.05 sec In 0.04 sec of protodiastole the ejection stops and aortic & pulmonary valves
Both AV (atrioventricular) valves ie close.
m itral (bicuspid) and tricuspid In 0.08 sec of isovolumetric ventricular relaxatioin, ventricle relax as closed
valves asw ell as aortic and cavity with both the semilunar & AV valve closed until the pressure in ventricles
pulmonary valves' are closed Q falls below the pressure in the atria.___________________________________________
during this phase. Ventricle 2. Late Diastolic Ventricular Filling
contracts as a closed chamber with 3. Atrial Systole (Contraction) Ventricular Filling.
both the semilunar & AV valve In both of these pheses AV valves are open and sem ilunar (aortic &
closed until the pressure in left and pulmonary) valves are closed.
right ventricle exceeds the pressure
in the aorta
2. Ventricular Ejection=0.22 sec
When aortic and pulmonary valves
open Q, the blood flows out of
ventricle and ventricle ejection
begins. Because the end diastolic
ventricular volume is 130ml and
end systolic ventricular volume is
50 ml. The ejection fraction (or %of
EDVV that is ejected with each
stroke) is -65% .EF is a useful
measure of ventricular function.

Vector Analysis of ECG


• At a given time (instant) during the cardiac cycle, heart current flows in a particular
direction. A vector (arrow) points in direction of the electrical potential generated by
the current flow, with the arrowhead in the positive direction. The length of arrow
is drawn proportional to the voltage of the potential. This is called resultant vector
in the heart at a given instant.
• At the instant of partial depolarization of ventricles (septum & apical endocardial
walls shown by negative signs and shaded area), the electrical current flows
between the depolarized areas inside the heart and non-depolarized areas on the
outside of heart (shown by long elliptical arrows). Overall,/or more current flows
Instantaneous Mean Vector through the
downward from the base of ventricle toward the apex than in upward direction. So
partially depolarized ventricles
instantaneous mean vector (i.e.) the summated vector o f generated potential at
-90 °
this particular instant) during septal depolarization is represented by the long ♦270 “

black arrow drawn through the centre o f the ventricles in a direction from base
tow ard apex Q.
• Directions of vector: When a vector is exactly horizontal and directed towards the
person's left side, the vector is said to extend in the direction of 0 degree . From
this 0° reference point, the scale of vector direction rotates clockwise. So when the
vector extends from above to vertically straight downward, it is in the direction of
90°. When it extends from person's left to right the direction is 180°; and when it
extends straight upwards, it has a direction of -90° (or + 270°). Direction of Vector
• M ean Q RS vector: is the average direction of the vector during spread of the depolarization wave through ventricles. In a
normal heart it is about +59° which means that during most of depolarization the apex of heat remains positive with
respect to the base of heart.
310 ■ A Complete Review of Short Subjects

• If it is assumed that three ECG electrode locations form the points of an equilateral triangle (Einthoven's triangle) and
the heart lies in the centre of this triangle; the vector at any given movement in two dimensions of frontal plane can be
calculated from any two standard limb leads.
• Mean Q RS vector (electrical axis of heat) is plotted by using the average QRS deflection in each lead (or by measuring
the net differences between the positive of negative peaks of QRS). The normal direction of mean QRS vector is -30° to
+110°. Left axis deviation is present if calculated axis falls to left of -30° and may be d/t left ventricular hypertrophy.
Right axis deviation is present if axis falls to the right of +110° and suggests right ventricular hypertrophy.

Normal Electrocardiogram (ECG)


- P wave reflects atrial depolarization before atrial contraction begins.
Q RS complex (wave) reflects ventricular depolarization before 1
contraction. In other words, QRS complex is caused by potentials
generated when the depolarization wave spreads through the
ventricles, but before ventricular contraction begins. Therefore, both P
waves and QRS complex are depolarization waves.
- T wave represents ventricular repolarization. It is caused by potentials 05
generated as the ventricles recover from the state of depolarization. This
process normally occurs 0.25- 0.35 seconds after depolarization in E
ventricular muscles. T wave is also k/a repolarization wave. 0
- Current flows from one part of ventricle to another part ( and therefore
to the surface of body) to produce ECG only when the muscles is
partly polarized and partly depolarized. No ECG is recorded when the
ventricular muscle is either completely polarized or completely
depolarized. ~°5
- Depolarization must spread through the muscle before contraction of muscle
can occur. So the P wave occurs at the beginning of contraction of atria 0 0 .2 0 .4 0 .6
Time (s)
and the Q RS complex occurs at the beginning of contraction of
ventricles. The ventricles remain contracted until after repolarization
has occurred i.e. until after end of T wave.
- Atria repolarize about 0.15 to 0.20 seconds after termination of P wave i.e. approximately during QRS complex. Therefore
atrial repolarization wave (atrial T wave) is usually masked by much larger QRS complex (of ventricular depolarization).
- Ventricular repolarization begins about 0.20 seconds after the beginning of depolarization (QRS complex) in some fibers and
takes as long as 0.35 seconds in others. Thus, ventricular repolarization (T wave) is prolonged process extending over 0.15
second, but the voltage of T wave is lesser than the voltage of QRS complex, partly b/o its prolonged length.
- P-Q or PR interval: The time between the beginning of P wave and the beginning of QRS complex is called PR interval (when Q
wave is absent) or more precisely P-Q interval. It is a measure of the time from the onset of atrial activation (ie beginning of
electrical excitation of atria) to the onset of ventricular activation (or excitation).
- The PR interval measures the time between the atrial and ventricular depolarization, including the physiological delay imposed by cells
in AV junction area. PR interval is normally 0.16 second (0.12 - 0.20 second). It shortens as the heart rate increases.
- Q T interval (or period of electrical systole of ventricles) is closely correlated with the mean action potential duration of
ventricular myocytes. Contraction of ventricles lasts from the beginning of Q wave (or R wave, if Q wave is absent) to the
end of T wave: an interval k/a QT interval. Normally QT interval is 0.4 second, but is inversely related with the heart rate
because the duration of myocardial cell action potential varies inversely with the heart rate. A heart rate related (corrected)
QT interval (QTc) can be calculated as Q tV r - R and normally is ^ 0.44 second.

_________ Cardiac Cycle : Pecualirities_________ Duration (in In Cardiac Muscle Skeletal


- Cardiac muscle contract & repolarize faster at higher heart rates (HR) and seconds) of at HR Muscle
the duration of systole decreases mainly d/t a decrease in the 75/min 200/min
duration of systolic ejection. Cardiac cycle 0.80 0.30 -

- However, the duration of systole is much more fixed and the Systole 0.27 0.16 -
diastole is shortened to a much greater degree Action potential 0.25 0.15 0.007
- Only during diastole, coronary blood flows to the endocardial Absolute 0.20 0.13 0.004
portion of left ventricle and heart muscles rest. Most of the refractory period
ventricular filling occurs in diastole. So at very high heart rates Diastole 0.53 0.14 -
(>180/min) filling may be compromised decreasing cardiac output
per minute.
Physiology: Cardiovascular System ■ 311

- Because of prolonged action potential, cardiac muscles cannot


contract in response to a second stimulus until near the end of initial
contraction. That's why cardiac muscles cannot be tetanized like
skeletal muscles. The highest rate at which heart can contract is
400/min (theoretically) but AV node will not conduct >230
impulses/min b/o its long refractory period. Hence ventricular rate
> 230/min is only seen in paroxysmal ventricular tachycardia.
- Exact clinical measurement of isovolumetric ventricular contraction
is difficult but simultaneous recordings of ECG, phonocardiogram
and carotid pulse can easily measure
1. Total Electromechanical systole (Q-S2 interval)
It is period from onset of ventricular depolarization (activation) or
QRS complex to the closure of aortic valves, as determined by the
onset on second heart sound (S2)
2. Pre-ejection Period (PEP)
It is difference between QS2 and LVET and represents the electrical
and mechanical events preceding systolic ejection
3. Left Ventricular Ejection Time (LVET)
It is the period between the begining of carotid pressure rise to
dicrotic notch (incisura)
- Normal PEP /LVET ratio is 0.35; which increases without change in
QS2 when left ventricular function is compromized

Period Definition Measured by


Electromechanical systole Period from the beginning of QRS complex to Electrocardiogram (for QRS complex)
(QS2)q the closure of aortic valve (2nd heart sound). Phonocardiogram (show the 2nd heart sound.
Left ventricular ejection Period from carotid pressure rise to the Carotid transduced (measures carotid
time (LEVT) dichrotic notch pressure rise k dichrotic notch)
Pre- ejection period (PEP) Time for electrical as well as mechanical Is the difference between QS2 and IVET.Q
events that precede systolic ejection
❖ PEP/LVET is normally 0.35 seconds. When left ventricular performance is compromised it increases without a change in QS2.

Fourth (Atrial) Heart Sound (S4)

• It is a low pitched presystolic sound which can almost never be heard with a stethoscope (& ear) because o f its weakness
and very low frequency - usually <20 cycles/second (HJQ. It is caused by in thrashing o f blood into ventricles (ventricular
fillin g , which initiates vibration) when the atria contractsQ.
• S 4,sometime becomes audible, when atrial pressure is very high or ventricle is stiff (diminished ventricular compliance)
increasing the resistance to normal filling. When audible it is best heard with bell of stethescope, at left ventricular apex in left
lateral position and accentuated by exercise (isotonic or isometric) in the supine position.
• Pathologically S4 may be seen in
Pulmonary stenosis & hypertension (right S 4 )
Aortic stenosis & systemic hypertension
Ischemic heart disease - HOCM - Mitral regurgitation
Advanced age (physiological)

___________________ Ventricular Extrasystole/ Premature Beat___________________


In abnormal conditions, the His- Purkinje fibers or the myocardial fibers may discharge spontaneously, that result in a beat
that occurs before the expected next normal beat & transiently interrupts the cardiac rhythm.
- They have bizarrely shaped prolonged QRS Q because of slow spread of impulse from focus to rest of the ventricle
- The P wave is usually buried in the QRS complex-
- Tendency to be followed by compensatory pause Qbecause when the normal impulse (of SA node) reaches ventricle, they
are still in the refractory period following depolarization from the ectopic focus.
- They do not interrupt the regular discharge of SA node (whereas atrial premature beats interrupt normal rhythm)
They are not strong enough to produce radial pulse .Q
- Ventricular premature beats, in the absence of ischemic heart disease, is usually benign.Q
312 ■ A Complete Review of Short Subjects

Cardiac Motion and Imaging

- Radiological imaging by CT and MRI can detect coronary artery disease in early stage. However, the use of these tools to
cardiac imaging is severely limited because of motion artifacts produced by constantly in motion heart completing an entire
cycle of contraction and relaxation in about 1 second.
- So radiological cardiac imaging requires a 'window' o f minimum cardiac m otion to reduce m otion artifacts and achieve
meaningful images. The phase of minimum cardiac motion is although variable and depends on several factors especially
heart rate and the chamber (i.e. right /left - atrium or ventricle) to be specifically imaged. However, the phase of minimum
cardiac motion (= optimum phase for cardiac & coronary vessel imaging).

Mid diastole (i.e. diastasis just Late Systole Phase of Relatively


before atrial systole) Rapid Cardiac Minimum Cardiac
At high heart rates; because the Motion Motion
In persons with low / duration of diastole is shortened - Early systole - Mid diastole
intermediate heart rates (i.e. at & mid diastolic cardiac motion - Mid systole (diastasisQ)
normal HR/ Cardiac cycles)______ increases as the heart rate - Early diastole - Early diastole
4 phases of diastole are - isometric increases, whereas the duration of (during rapid (during
relaxation, early rapid filling, systole is less affected by changes filling) isovolumetric
diastasis and atrial systole. in heart rates. - Late diastole relaxation)
(atrial systole) - Late systole

Measurement of Cardiac Output


Direct Fick Method Indicator Dilution Method
■Direct Fick method for measuring cardiac output -Indicator (dye) dilution method is based on Stewart - Hamilton
is based on oxygen Fick principle which states principle which states that when an innocuous (harmless) dye (indicator
that the amount of a substance taken up by any or radioactive isotope) that stays in blood stream during the test, is
organ (or whole body) per unit of time is equal to injected into venous system (eg arm vein); the cardiac output is equal to
arterial level of substance minus the venous level amount of dye injected divided by its average concentration in arterial
(A-V difference) times of blood flow. However, blood after a single heart circulation (determined after extrapolation).
the arterial blood should be the sole source of - Cardiogreen dye is usually injected as bolus in arm vein. The log of
substance taken up. indicator concentration in serial arterial samples is plotted against time as
•Cardiac output is determined by measuring the the concentration rises, falls and the rise again indicating recirculation
amount of O 2 consumed by the body in a given of dye. Therefore it is necessary to extrapolate the early down slope of
period (such as O 2 absorbed per minute by curve to zero point (abscissa), giving the time for first passage of
lungs in ml/mn) and dividing this by A-V indicator through the circulation (Extrapolated time - concentration
difference (ie arterio-venous O 2 difference) curve). The cardiac output for that period is calculated & then converted
across the lungs (in ml/lit) to output per minute.
- In thermodilution technique, cold saline is used as an indicator because
O2 C on su m p tion O2 ab sorb ed /m in b y lun gs of 2 important advantages (1) complete innocuous nature of saline, (2)
C ard iac _____ m l/m in _____ ________ (m l/m in)________ recirculation is not an issue since cold is dissipated in tissue making
o u tp ut = A terio ven o u s O2 = A terio v en o u s O2 difference
repeated determinations easy.
(L/m in) differen ce (A02-V02) across lu n gs (m l/L o f blood)
-Through a double lumen catheter, saline is injected into the right atrium (via
smaller arm of catheter) and temperature change is measured in pulmonary
■Rate of O 2 absorption is measured by oxygen artery, using a thermistor (via longer side of catheter). The temperature
meter measuring the rate of disappearance of O 2 change is inversely proportional to the extent that the cold saline is diluted by
from respired air. A 02 can be measured from any blood ie the amount of blood flowing through the pulmonary artery.
systemic arterial blood, where V 02 is measured Amount of indicator injeced(E)
from pulmonary artery (or right ventricle) Cardiac Flow (F) = Instananeous concentration of indicator in
arterial blood i.e. Cdt
Physiology: Cardiovascular System ■ 313

Myocardial (Heart) Contractility


Myocardial contractility has major influence on stoke volume. The relationship between ventricular stroke volume and end diastolic
volume is called Frank-Starling curve. This (length-tension )curve shifts upwards and to the left as contractility is increased (eg
in sympathetic stimulation). Wheareas, the curve shifts downwards and to right when contractility decreases eg in vagal
(parasympathetic stimulation).

Intrinsic Factor Extrinsic Factor


Length of activation^ = Length - Adrenergc neural stimulation /Sympathetic stimulation^
Tension relation in cardiac muscle. - Inotropic effect is smaller, maximum in atria. It becomes important
- Most important factor when the heart is increased as it permits adequate diastolic filling.
- Frank- starling law: "Energy of Cardiac rate and rhythm changes (force- frequency relation) eg post
contraction is proportional to initial extra systolic potentiation d/t increased availability of intracellular Ca+
length of cardiac muscle fiber." Sympathetic & parasympathetic impulses^.
Length of muscle fiber (preload) is Circulating catecholamines
proportionate to end diastolic blood
Hypoxia, hypercarbia, acidosis
volume (preload)Q.
Digitalis & other inotropic agents

Medullary Centres regulating Blood Pressure

Vasomotor Centre (VMC)

• Collection of glutaminergic neurons in superficial medullary reticular formation, known as Ci neurons, serves as pressure area
of VMC. Cell bodies of C l cells lie in ventrolateral region of upper half of medulla.
• These neurons are spontaneously active day & night, tonically discharging impulses (in rhythmic manner)® to the heart and
blood vessels through lateral reticulospinal tract
• These reticulospinal tracts synapse with preganglionic neurons of sympathetic nervous system located in
intermediolateral grey column of spinal cord (thoraco lumbar sympathetic out flow)
• Post ganglionic sympathetic neuron secrete nor adrenaline thereby increasing sympathetic - Vasoconstriction
activity and causing tonic vasoconstrictor & cardioaccelerator activity resulting i n ----------------------- -TBP
• Regulation of VMC activity - TCardiac output
1. Baroreceptor reflexes: - TStroke volum e
Baroreceptors are stretch receptors located in walls of aortic arch & carotid sinus, they are tonically
active & exert tonic inhibition o f VMC and facilitation o f CVC®.

Cardiovagal Centre (CVC)

• It refers to nucleus ambiguous which lies lateral to VMC (medulary reticular neurons)
• It receives afferents via Nucleus tractus solitarus (NTS) & in turn sends parasympathetic impulses to heart via the vagus,
so cardiovagal centre
• The neurons in CVC do not discharge rhythm ically (i.e. are not tonically active)® - Vasodilation
• The discharge increases parasympathetic activity there by resulting in --------------------------------- - TBP
- ^Cardiac output
• A pecularity of sympathetic system, which is not found in parasympathetic system, is the fact - I Heart rate
that entire thoraco lumbar outflow is always activated as a unit, whereas the parasympathetic
stimulation is very discrete, affecting specific muscles or glands.
314 ■ A Complete Review of Short Subjects

I n h ib ito ry in tem eu ro n s I C erebral C ortex I N u cleu s tractus so litaru s


R esp irato ry centre C N S (N TS)
ischem ic resp on se (h yoxia & C ortico H y p o th e lm i
hypercapn ia) D e sce n d in g c p a th w a y

H ypthalm us

C a rd io v a g a l C entre
V aso m o to r C en tre (V M C ) ■ve Baroreceptors (aortic +ve (N u cleu s am b igu ou s)
Pressor A rea < ---------
arch, carotid sinus)
^ s^ + v e

C h em orecep to rs
Lateral R eticulospinal Tract (aortic & carotid bodies and
m edulla oblongate)

P rega n glio n ic S ym p ath etic


N eu ro n s (In term ed iolateral grey
colum n)
T i to L3

P arasym path etic im pu lses


Post ganglion ic sym pathetic (via V ag u s)
neuron (Secrete norodrenaline)

(+ve)
H eart & Blood vessels (-ve)

V aso co n striction ,
V a so d ila tio n , I B P , i C O ,
Tb p, tco, Tsv
iH R

Vasomotor Control
Main control of BP is exerted by a group of neurons in medulla oblongata K/a Vasomotor area (center).
Afferents to vasomotor area Efferents from vasomotor area
Direct Stimulation • Sympathetic outflow:
C 02 - Neurons that mediate increased sympathetic discharge project to
Hypoxia sympathetic preganglionic neurons in intermedio lateral gray column
Excitatory inputs (IML) of spinal cord. Cell bodies of these neurons are locate in rostral
From cortex via hypothalamus ventrolateral medulla (RVLM)l The excitatory transmittor is glutamate.
From pain pathways and muscles - It causes increase in heart rate (chronotropic effect) & force of cardiac
From carotid and aortic chemoreceptors contraction (ionotropic effect), Vasoconstriction, T BP, tH R , Tstroke
Inhibitory inputs volume and T cardiac output
From cortex via hypothalamus • Parasympathetic outflow:
From lungs - Neurons of vagal fibers arise in dorsal motor nucleus of vagus &
From carotid, aortic, and cardiopulmonary nucleus ambiguous.
baroreceptors____________________________ - It causes vasodilation, fa ll in BP, and increase in storage o f blood in
Baroreceptors are stretch receptors in the venous reservoirs, decrease in heart rate and cardiac outputQ
walls of heart & vessels eg carotid sinus &
aortic arch, walls of atria, pulmonary veins &
pulmonary circulation. Carotid sinus is a
small dilation of internal carotid artery just
above the bifurcation of common carotid.
Receptor are also located in walls of right
and left atrium at the entrance of SVC and
IVC and pulmonary veins as well as in Nucleus
ambiguua
pulmonary circulation. In low pressure part
of circulation, these are K/a
Cardiopulmonary receptors. These are
stimulated by distension or pressure rise in
structure (total cardiovascular output)^.
Physiology: Cardiovascular System ■ 315

1 The afferent from carotid body pass through carotid sinus branch of glossopharyngeal (9lh) nerve and from aortic arch through
vagus nerve. Both are called buffer nerves. Most of them end in nucleus tractus solitarius (NTS). Excitatory glutaminergic
projections extends from
NTS to caudal & intermediate ventrolateral medulla (CVLM & IVLM), where they stimulate GABA secreting inhibitory neurons
that projects to rostral ventrolateral medulla (RVLM)
NTS to vagal motor neurons in the dorsal motor nucleus & ambiguous
Thus increased baroreceptor discharge inhibits tonic discharge of vasoconstrictor nerves & excites vagal innervation,
producing - vasodilation, venodilation, a drop in BP, brady cardia and a decrease in cardiac output®.

Cushing’s Reflex (Phenomenon)

Etiology Pathophysiology
- It is a special CNS ischemic response - When the ICT or CSF pressure rises to equal the arterial pressure, it compresses the
that results from increased whole brain as well as the arteries in the brain and cuts off the blood supply to the
intracranial tension (or increased CSF brain, thereby activating the CNS ischemic response
pressure around the brain in cranial - Compromized blood supply, local hypoxia & hypercapnia of RVLM (rostral ventro
vault). lateral medulla) neurons or vasomotor area increases their discharge and evoke
- Causes of increased ICT may be space Cushing's reaction raising systemic arterial pressure.
occupying lesions such as -The resultant rise in systemic arterial pressure (d/t ischemic stimulation of
vasomotor regions in mdeulla) tend to restore the blood flow to medulla, thereby
1. Brain tumor® protecting the vital centers of brain from loss of nutrition if ICT rises enough to
2. Cerebral abscess compress cerebral arteries.
3 .Head trauma® with associated intra - The rise in BP is proportional to increase in intracranial tension (over a considerable
cranial bleeding eg epidural/ range) and causes a reflex decrease in heart rate via the arterial baroreceptors. This
subdural/ cerebral-haemorrhage is why bradycardia rather than tachycardia is characteristically seen in patients with
increased ICP (intra cranial pressure)

Effects on Heart

Sympathetic & Denervation Exercise


Parasympathetic Innervation

Vagal (parasympathetic) •At rest, there is a moderate - Cardiac muscle has the unique property of contracting &
fibres are mainly distributed to amount of tonic discharge repolarizing faster when the heart rate is high
atria & not much to ventricles. in cardiac sympathetic - Cardiac muscle cannot be tetanized as it has prolonged
This explains the fact that the nerves, whereas vagal tone action potential during which muscle is refractory & will
main effect of vagal stimulation (tonic discharge) is of good not contract in response to second stimulus until near the
is to decrease heart rate rather amount. end of initial contraction.
than to decrease the strength ■So when vagi are cut (or - The duration o f systole is much more fixed than that o f
of heart contraction greatly parasympatholytic drug eg diastole and when the heart rate is increased (eg. in
(which is dependent on atropine is given) the normal exercise), diastole is shortened to a much greater degree.®
ventricle) resting heart rate rises
Duration of At HR At HR
Sympathetic (noradrenergic) (from 70) to 150-180
75/min 200/min
fibres are distributed to both beats/min due to
Cardiac cycle 0.80 0.30
atria & ventricle. So its unopposed sym pathetic
Systole 0.27 0.16
stimulation has a strong effect tone.®
If both noradrenergic & Diastole 0.53 0.14
on both heart rate (chronotropic)
& force of contraction cholinergic systems are Action potential 0.25 0.15
(ionotropic) blocked, the heart rate is Absolute refractory period 0.20 0.13
-100 beats / min. Relative refractory period 0.05 0.02
316 ■ A Complete Review of Short Subjects

Effects of Assuming Supine (Lying down) Posture

I------------------------------------ 1----------------
Cerebral blood flow Pulmonary blood flow Cardiovascular system
I
I I
Onlying On standing
Auto regulated In standing posture - Stroke volume
extremely well & apex is less perfused - Cardiac output
maintained at than base® Venous return
normal levels - Central blood pool Increase Decrease
On lying down
between arterial Central venous pressure
blood flo w to lung
pressure limits of 60- Arterial BP
apex increases®
140 mmHg - Abdominal & limb flow
Heart rate
- Peripheral venous pooling
Total peripheral resistance Decrease Increase
- Abdominal & limb resistance
- Small vein pressure
Gravity has marked effect on pulmonary circulation. In upright position in upper parts o f lung (apex) the blood flo w is lesser
than the base ®. In middle portion & upper lung blood flow is determined by pulmonary artery - alveolar pressure difference, and
pulmonary veins whatever amount of blood the constrictions (produced by alveolar pressure) lets flow into them (Water fall
effect). In lower lung alveolar pressure is lower than pressure in all parts of pulmonary circulation, so blood flow is
determined by pulmonary artery - pulmonary vein pressure. On lying down blood flow to lung apex increases
On standing, at head level arterial and jugular venous pressure falls 20-40 mmHg and 5-8 mmHg respectively reducing the
drop in perfusion pressure (arterial - venous pressure). Cerebral vascular resistance is reduced (d/1 4jV P 1/1 4lCP that
presses the vessels). There is only 20% decrease in cerebral blood flow® d/t autoregulatory mechanism activated by TPCO 2
(partial pressure of CO 2), and 4in pH and PO 2. However, the amount of O2 extracted increases and so the cerebral O 2
consumption is about the same in supine & upright position. (Ganong).
Venous return must equal cardiac output overtime as vascular system is a closed circuit.
Cerebral blood flo w is autoregulated extremely w ell without significant change in cerebral blood flo w between mean,
arterial pressure limits o f 60-140 mmHg® (Guyton).

Sequence of events on Effect on Cardiovascular system Compensatory Mechanism


Assumption of Erect
Posture
Pooling of blood in Arterial blood 4 es The reduced C.O together with decreased arterial
lower extremity pressure pressure in the brain (d/t force of gravity) can cause
I cerebral ischemia. This can cause syncope. This is
Insufficient return of Central venous Jes® prevented by the following mechanism
blood through inferior pressure As the C.O falls
venacava (4 in central BP falls
venous pressureP Abdominal & 4 es by 25% 4
4 limb flow Stretching on the baroreceptor of
Reduced preload Aortic sinus decreases
4 Cardiac output 4es by 25%Q 4
iStroke volume Vosomotor center active
4 Stroke volume ■les by 40%® (Normally firing from Aortic & carotid
iCardiac output baroreceptor keeps the V.M.C inhibited)
Heart rate T es
4
Sympathetic system activated
4
Tachycardia + Vasoconstriction
( fH eart rate & T peripheral resistance)®
4
B.P restored (syncope avoided)
Physiology: Cardiovascular System ■ 317

Diastolic Pressure
Systolic pressure is peak pressure reached during systole, similarly, diastolic pressure refers to lowest pressure during
diastole
Elsticity o f aorta and large arteries is mainly responsible fo r origin & maintenance o f diastolic pressure (by wind kessel
elastic recoil effect)®. Because the elasticity is
__________________ l__________________
I--------------------- 1
Higher in younger subjects Lower in old persons
I I
- Diastolic pressure is maintained - Diastolic pressure is decreased
- Pulse pressure is narrow - Pulse pressure is widened

Measures of Venous Return & Degree of Filling of Veins


Mean Circulatory Filling Pressure (MCFP) Mean Systemic Filling Pressure (MSFP)
It is the pressure that would exist every where in the - It is slightly different from the MCFP. It is the pressure
circulation if the heart pumping is stopped® (by shocking measured every where in the systemic circulation after blood flow
with electricity to cause VF or in any other way), and the has been stopped by clamping the large blood vessels at the
pressure is allowed to equilibrate through out the system. heart, so that the pressure in the systemic circulation can be
The normal value is 7 mm Hg at (5 lit blood volume). An measured indepently from those in pulmonary circulation.
increase in Blood volume increases the MCFP, while - It is almost impossible to measure in living animals. Although
hypovolaemia (as in dehydration & haemorrhage) it is im portant pressure fo r determining venous return®
decreases it - It is alm ost alw ays nearly equal to MCFP® because the
At 4 litre blood volume, the MCFP is close to 'O ' and this pulmonary circulation has only 1/10 as much blood & <
volume is k/a unstressed volume o f circulation. The 1/8 as much capacitance as the systemic circulation.
MCFP increases almost linearly with increase in volumes - The normal MSFP is 7 mmHg venous return curves
Strong sympathetic stimulation constricts all systemic demonstrate that greater the MSFP (eg 14), the greater the
blood vessels, larger pulmonary vessels & even chambers tightness with which circulatory system is filled & the more
of heart, therefore, decreasing the capacity of system. So the venous return curve shifts upward & to the right. And
that at each level of blood volume MCFP is increased. At vice versa. In other words, the greater the system is filled
normal blood volume maximal sympathetic stimulation & the easier it is for blood to flow into the heart. The less the
inhibition cause MCFP to rise / decrease to 17 mm Hg & 4 filling, the more difficult it is for blood to flow into heart.
mm Hg respectively.

Blood Pressure Methods of Measurement of BP

Spurious Falsely Direct Intraarterial Indirect


high low method - By sphygnomanometer (External pressure required
I I
The artery is to cause occlusion of blood flow, indicates the
Conditions Auscultatory gap: During catheterized & pressure inside the vessels)
where measuring of BP (in connected to a - Giver falsely high reading ®: the pressure induced in
more hypertensive individuals) mercury manometer the cuff, is dissipated in the tissue of arm (or thigh),
pressure is the sound of Korotkoff or pressure so at a pressure equal to arterial pressure, the arteries
required in sometimes disappear at transducer. fail to close. Thus pressure has to be raised further for
the cuff to pressure well above the
Measures accurate that.
obliterate diastolic pressure, then
the artery reappears at lower BP® ★ K o rotkoff sound is d/t arterial turbulence.®
pressure (Auscultatory gap) BP & VR Measurement
- Small
cuff® to again disappear at the
The overall blood flow in circulation of adult at rest is about 5 liter/min which
diastole.
- Obesity® equals to cardiac output because it is the amount of blood pumped by heart per
To avoid it, palpatory unit time. So
- Thick method of cuff is used
calcified Blood pressure = Cardiac output x peripheral resistance^ or Blood flow
until the radial pulse is (Q) x resistance (R)
vessels®
obliterated.
Venous retumQ = MSFP = Mean systemic filling pressure
M SFP-RA P RAP = Right atrial pressure
RVR RVR = Resistance to venous return
318 ■ A Complete Review of Short Subjects

Baroreceptors Stimulation of Baroreceptors


Stretch receptors in the walls of the heart & blood vessels (in adventitia) TBP —> Baroreceptors stimulated
1
Carotid sinus Aortic arch
Inhibits Vasomotor center Vagal (Parasympathetic)
Small dilatation of the In the wall of arch of aorta
( i Sympathetic activity) stimulation
internal carotid artery just
above the bifurcation of + R ev erse
1 1
common carotid artery o c cu rs w h en
BP d e cre a se s
• Afferent: • A fferen t: Vagus (X CN) =
Glossopharyngeal (IX Aortic depressor nerve - Vasodilation - J.BP (Hypotension)Q
CN) = Carotid sinus • Efferent:- Sympathetic - Venodilation - Bradycardia®
nerveQ - Parasympathetic - 4CO (cardiac output)
• E fferen tS y m p a th etic (vagal) - W hen carotid sinus nerve is sectioned. Loss of
- Parasympathic inhibitory impulse from carotid sinus nerve -»
(vagal) Medullary vasomotor center activated —> T
sympathetic discharge -> Vasoconstriction ( tBP),
Th r & Tco.
Most afferent fibers reach nucleus of tractus solitarius (NTS) and - The baroreceptors do not fire when mean BP is
secrete excitatory neurotransmitter glutamate. Excitatory (glutamate)
below 30 m m H g (fire m axim ally at mean BP of 150
projections from NTS extend to vagal motor neurons in nucleus
m m H g). Carotid sinus nerve and vagal fibres from
ambiguus and dorsal motor nucleus. Excitatory projections from NTS aortic arch com m only called Buffer nerves.
also reach to CVLM (caudal ventrolateral medulla) where they - Severin g on ly carotid sinu s nerve w ill not lead to
stimulate GABA secreting inhibitory neurons inhibiting RVLM ( rostral decrease in BP becau se aortic arch baroreceptors are still
ventrolateral medulla) and ultimately sympathetic activity. So fu n ction al. I f both are severed the BP rise to m ore than
b aroreceptor stim u lation , stim u lates p arasy m p ath etic and decreases 200-300 m m Hg.
sym pathetic activity®.

Effects of Acceleration During Exposure to Positive ‘G’


The effects o f gravity on the circulation are multiplied during Arterial pressure in the head is reduced, but so are
acceleration or deceleration. venous pressure and intracranial pressure and this
Force acting on the body as a result of acceleration is commonly reduces the decrease in arterial blood flow that
expressed in 'g' units, l'g ' being the force of gravity on the would otherwise occur.
earth's surface. Cardiac output is maintained for a time because
Positive 'g ' (Black outs) Negative 'g' (Red outs) blood is drawn from the pulmonary venous
Is the force due to Is the force due to reservoir and because the force of cardiac
acceleration acting in the long acceleration acting in the contractions is increased.
axis of body from head to foot long axis of body in opposite At accelerations more than '5g' however vision fails
During exposure to positive direction. (black outs) and unconsciousness follows almost
'g' blood is thrown into lower During exposure to negative immediately.
part of the body, and 'g' blood pools into the
consequently there is upper part of the body, and
decrease in cerebral arterial consequently there is an
pressure. increase in cerebral arterial
pressure.
Positive 'g ' creates a tendency Negative 'g ' creates a tendency
for for
Decreased cardiac output Increased cardiac output
Decreased cerebral arterial Increased cerebral arterial
pressure pressure
- intense congestion of head
/ neck vessels
- echymosis around eyes
- severe throbbing head
ache Negative G: is a state of weightlessness, it occurs when
Black out and 'Red out' and mental airplane descends very rapidly
unconsciousness confusion
Physiology: Cardiovascular System ■ 319

Endothelins

This 21 aminoacid polypeptide structurally resembles sarafotoxins (b) found in venom of Israeli burrowing asp, snake. It is
of 3 types endothelin 1 (ET1), endothelin 2 (ET2), & endothelin 3 (ET3); all encoded by a different gene.
Endothelin 1 produced in endothelial cells is one o f the m ost potent vasoconstrictor®.

Endothelin 1 _____ > Big endothelin 1 (39 aminoacid endothelin converting enzyme (E C E ) ^g rp j

gene prohormone of only 1 % activity) Trp-V al bond clevage

2 types of endothelin receptors, both of which are coupled to phosphotipase C via G proteins are ETa an ETb. The ET a receptor
is specific for endothelin 1 and mediates vasoconstriction
ETb receptor responds to all three endothelins & is coupled to Gi. It may mediate vasodilation and developmental effects.

Secretion Function

Big ET 1 & ET1 are secreted in blood only in small ET1 is found in endothelial cells, brain & kidney. ET -2 is mainly
amounts, and mostly they are secreted into the media produced in kidney & intestine. ET3 is found in brain (high conc),
of blood vessels and act in a paracrine fashion to blood, kidney and gastrointestinal tract.
regulate vascular tone. ET1 is not stored in secretory Endothelins produced by both astrocytes & neuron are abundant in
granules. Factors regulating its secretion are brain in early life. They are found dorsal root ganglia, ventral horn
cell, cortex, hypothalmus, & cerebellar purkinje cells. They regulate
transport across blood brain barrier®.
Endothelin produces mesengial cell mediated decrease in GFR
1 (glomerular filteration rate)®.
Stimulators Mn Inhibitors It plays a role in closing ductus arteriosus at birth.
1 1 ET1 is primarily a paracrine regulator of vascular tone®. It is
Angiotensin II A tense Prostacyclin vasoconstrictor, bronchoconstictor, ionotroic (cardiac
Hvpoxia Hypoxic pge2 stimulant)®, which stimulates aldosterone secretion & atrial
Shear stress Stress ANP natriuretic peptide release.
Insulin In NO ET1 is implicated in pathogenesis of congestive heart failure,
Growth factors Growing myocardial infarction, coronary vasospasm and hypertension.
Cate cholamines Cat Mn- "Cyclic Deletion of both alleles o fE T l gene results in craniofacial
Thrombin Throbs PAN" abnormalities, Hirschsprung's disease (megacolon) & death d/t
HDL Her respiratoryfailure at birth.

Fluid Filtration Across Capillaries


Rate of filtration along a capillary depends on starling forces, i.e.,

- Hydrostatic pressure gradient (hydrostatic pressure - Osmotic pressure gradient (colloid osmotic pressure
in capillary minus hydrostatic pressure in interstitial of plasma minus colloid osmotic pressure of interstitial
fluid) fluid.)
- Capillary (hydrostatic) pressure (Pc) tends to force - Capilary plasma colloid osmotic pressure ( 71c ) tends
fluid outward through capillary membrane to cause osmosis of fluid inward through capillary
membrane
- Interstitial fluid (hydrostatic) pressure (Pi) tends to - Interstitial fluid colloid osmotic pressure ( 71,) tends
force fluid inwards through capillary membrane when to cause osmosis of fluid outward through the capillary
Pi is positive but outward when Pi is negative
membrane.
320 ■ A Complete Review of Short Subjects

If the sum of these forces, the net filtration pressure (NFP) is positive (+), there will b e fluid filtration (outward movement)
from capillaries. And if NFP is negative (-ve), there will be net fluid absorption from interstitial space into capillaries
(inward movement). At arteriolar end of capillary, NFP is +ve & at venular end, it is -ve.

NFP = Hydrostatic pressure gradient - Osmotic pressure gradient


NFP = (Pc - Pi) - ( Kc - Ttj)
NFP = Pc - Pt - Xc + Xj

I
Rate of fluid Alteration is also dependent on number & size of pore in each capillary as well as number of capillaries in
which blood is flowing. These factors are expressed as capillary filteration coefficient (K<).
Fluid movement / Filteration = K f x NFP

Arteriolar \ Venule

End| I/ i _____________ t _ ) ▲

I i t T
Factors Regulating Cerebral Blood Flow (CBF)

Intrinsic factors Extrinsic factors


Cerebral metabolism rateQ proportionally increase CBF A rterial PCO 2is most important extrinsic
Cerebral perfusion pressure (CPP) factor causing immediate effect by changing pH
CPP = Mean arterial pressure (MAP) - Intracranial pressure (ICP) or of CSF & tissue. CBF is directly proportional
central venous pressure (CVP), which ever is greater. CPP is primarily to arterial PCO 2b/w 20-80 mmHg.
dependant on MAP (or BP) as ICP is usually <10 mmHg. CPP is normally A rterial PO r. Only marked changes alter CBF
80-100 mmHg. Autoregulation maintains a steady CBP as CPP may vary Hyperoxia cause only minimal decrease (-10% )
between 60-160 mmHg. Decrease in CPP cause cerebral vasodilation, & in CBP, whereas severe hypoxia (PO 2<50
elevation results in vasoconstriction. mmHg) profoundly increase CBF
Blood viscosity fall l/t rise in CBF & vice versa Tem perature: Hypothermia of 1 °C decreases
B lo o d v iscosity fall l/t rise in CBF & vice versa CBF 5-7% whereas pyrexia has reverse effect.
A uton om ic system
Substance released fro m astrocytes

Arteriovenous Anastomosis (Shunt Vessels)


• Shunt vessels are short muscular vessels that connect arterioles and venules, by passing the capillaries®.
• Like other vascular smooth muscles the shunt vessels are under the control o f sym pathetic nervous system (& receive
sym pathetic innervation)®
• Uneven distribution : shunt vessels are mainly concentrated in tissue with a small surface area / volume ratio eg fingers,
toes, ears, & nose.
• Shunt vessels are found in skin and nasal mucosa especially in
- Hands : Palm & fingers
- Feet : Sole & toes
- Face : Ear, Nose & lips

• No nutritive only thermoregulatory function: Nutrition is provided by capillaries since capillaries deliver O 2 to tissues.
Because shunt vessels bypass capillaries, these have no nutritive function.
However shunt vessel have important role in thermoregulation. These provide an important site o f body heat transfer
between skin & blood stream. By dilating in cold conditions and directing large volumes of blood through these vessels, it
provides a valuble mechanism to prevent tissue freezing and cold damage to the extermilies®
Physiology: Cardiovascular System ■ 321

Laminar (Streamline) Flow Turbulence


A streamline flow is also K/a laminar flow because - Turbulent flow is a chaotic flow with irregular motions and flows in all
it moves in layers (or lamina). So a dye carefully directions; it forms eddies or whirlpools and fluid elements do not remain
introduced into a given lamina (layer) remain in that confined to definite lamina, but rapid, radial mixing occurs.
lamina as the fluid moves longitudinally along the - Turbulent flow offers more resistance than laminar flow; so greater
tube. pressure is required to force a given flow of fluid through the same
In laminar flow the layer touching the wall of tube tube when the flow is turbulent than when it is laminar. In turbulent
adheres to it and hardly moves b/o friction, while flow, pressure drop is approximately proportional to the square of
the concentric layer or lamina next to it shear/slide flow rate whereas in laminar flow, the pressure drop is proportional
against this motionless layer with less friction. In this to the square of flow rate whereas in laminar flow, the pressure drop
way the inner lamina moves faster than laminae on is proportional to the first power of the flow rate. So to produce same
their outside with the result that the portion of fluid flow a pump like heart must do considerably more work if
at the center moves fastest. This is why the shape of turbulence develops.
progressing front is parabola. - Laminar flow occurs at velocities upto critical velocity, at or above
The velocity at the center of stream (in stream line which the flow becomes turbulent, and creates sound. Turbulent flow
flow) is maximal and equal to twice the mean accounts for development of heart sounds, murmurs a/w valvular
velocity of flow across the entire cross section of the heart disease, Korotkov sounds heard during the measurement of
tube. Flow of blood in vessels is normally laminar arterial blood pressure, bruits heard over arteries constricted by
(stream line), which means the layer in the center of atherosclerotic phaque and functional cardiac murmurs heard in
stream has highest velocity & peripheral layer (near patients with hyper dynamic circulation (as thyrotoxicosis & severe
blood vessel wall) has lowest. Streamline flow is chronic anemia).
silent. Therefore no sounds are heard with - Turbulence is more common in anemia because of reduced viscosity
stethoscope in normal arteries. and high flow velocities a/w high cardiac output.
Average velocity of flow is inversely proportional to - Turbulence is usually accompanied by audible vibrations. Blood
the total cross section area of that vessel. Therefore clots and thrombi are more likely to develop in turbulent flow than
the average velocity of blood is high (33 cm/s) in in laminarflow.
aorta (CS 2.5 cm2), declines steadily in smaller Critical Velocity
vessels and is lowest in the capillaries (0.3 mm/s),
Critical Velocity (Vc) at which turbulence occurs is proportional to the
which have 1000 times the total cross sectional area
viscosity (q) of fluid, but inversely proportional to the density of fluid
of aorta (i.e. 2500 cm2).
The average velocity of blood flow increases again in (p) and radius of tube (r) i.e
veins and is relatively high in venacava, although Kq
Vc =
not so high as in the aorta. So the cau se o f lam in ar
pr
flo w in sm a ll v essels is large area o f cross section
and less average velocity®. Probability of Turbulence
Probability of turbulence is expressed by Reynolds number (i.e. ratio
of inertia to viscous forces)
Reynolds number (Re) =
n
p = density of flu id ; D ■diameter of vessel
V = velocity of flow ; Tj = Viscosity of fluid

- The higher the valu e o f Re, the g reater the p ro b a b ility o f turbulence®.
I f R e is < 2000, th e flo w is usually n ot turbulent. W hen Re > 3000,
turbulence is a lm o s t a lw a y s present®. For RE / NR between 2000
and 3000, the flow will be transitional between laminar and
turbulent.
- So high flu id den sities, large tube diam eters, high flo w v elo cities and
low flu id v isc o sity p red isp ose to turbulence®. Constriction of arteries
increases velocity of blood flow and thus produces turbulence. In
addition to these, abru pt v ar iatio n s in tube dim ension s or
irregularties in the tube walls may produce turbulence.
- Re for most biological fluids including blood is 1000. Laminar flow is
disturbed at branching of arteries and turbulence may be produced
because at the sites of branching Re > 200-400 (instead of >3000) can
cause turbulence.
322 ■ A Complete Review of Short Subjects

Pulmonary Circulation

- Pulmonary circulation begins with main pulmonary artery, which receives the mixed venous (deoxygenated) blood
pumped by the right ventricle. Pulmonary arteries branch and accompany the airways as far as terminal bronchioles. Beyond
that they break up to supply the ca p illa ry bed which lies in walls of alveoli, oxygenate blood and convey this oxygenated
blood to small pulmonary veins that run between the lobules and eventually unite to form the 4 large veins which drain into
the left atrium. Vessels can be divided into.

Alveolar vessels (i.e. pulmonary capillaries & slightly Extra-alveolar vessels (i.e. all the arteries and veins that run
larger vessels in the corner of alveolar walls) are through the lung parenchyma) are exposed to pressure
exposed to alveolar pressure (i.e. very close to considerably less than alveolar pressure (even less than intraplural
atmospheric pressure) as they are virtually surrounded pressure). Their caliber is remarkably affected by lung volume
be gas. The pressure difference b/w inside & outside because this determines the expanding pull of parenchyma on
of capillary is called transmural pressure. When their walls; and are pulled open by radial traction of surrounding
alveolar (out side) pressure rises above the pressure parenchyma. The very large vessels near the hilum that are
inside, the capillaries collapse. outside the lung parenchyma are exposed to intrapleural
pressure.
Mean = 15 Mean - 100
- With a surface area of 70-80 m2 which is nearly as large as alveolar
surface area, the pulmonary capillary bed is the largest vascular
bed in body. The network of capillary is so rich that it is considered
as a sheet of flowing blood interrupted in places by small vertical posts,
rather like an under ground parking.
- The pressures in the pulmonary circulation is very pulsatile and
remarkably low (mean, systolic & diastolic pressures in main
pulmonary artery are 15, 25 & 8 mmHg, respectively) whereas the
mean pressure in aorta is 100 mmHg - about 6 times more than in
the pulmonary artery. The pressures in the right & left atriums are 2
and 5 mmHg that are not very dissimilar. So the pressure
differences, from inlet to outlet in circulations are

Pulmonary Circulation Systemic Circulation


I I
15 - 5 = 10 mmHg 100 - 2 = 98 mmHg

- In striking contrast to the systemic circulation, where the arteries have thick walls and the arterioles in particular have
abundant smooth muscles, the walls of pulmonary artery and its branches are remarkably thin & contain little smooth
muscle (so that they are easily mistaken for veins)
- The lung has to accept whole of the cardiac output at all times, therefore it requires stretchable arteries with thin (less muscular) walls.
Whereas, it is not concerned with directing blood from one region to another (except in localized alveolar hypoxia) so its arterial
pressure is as low as is consistent with lifting blood to the top of lung (only). This arrangement keeps the work load of right heart as
little as is feasible for efficient gas exchange to occur in the lung. Similarly high pressure muscular arterial systemic
circulation help to regulate the adequate supply of blood to organs which may be for above the level of heart (eg outstretched
arm).
- The pressure within pulmonary capillaries lies about half way between pulmonary arterial & venous pressure and varies
considerably throughut the lung d/t hydrostatic effects. However the distribution of pressure along pulmonary circulation is far
more symmetrical than in systemic circulation, where most of the pressure drop is just upstream of capillaries.
- The total pressure drop in pulmonary circulation from pulmonary
artery to left atrium is only 10 mmHg, against about 100 mmHg ( Input-Output) Pressure
for systemic circulation (b/w aorta & right atrium). As the blood Vascular resistance = ----------------------------------
Blood flow
flow through the two circulations is virtually same (~ 6 liters /
min), so the pulmonary vascular resistance is only one tenth® = [(15 - 5) /6 = 1.7 mm Hg x min / lit] that of systemic
circulation, where the high resistance is largely caused by very muscular arterioles.
- Extraordinarily (10 times) smaller normal pulmonary vascular resistance even decreases when the pressure (flow) within it
rises i.e. an increase in either pulmonary venous or arterial pressure causes pulmonary vascular rsistance to fall. This
decrease in pulmonary vascular resistance eg during exercise occurs d/1 passive process of
Physiology: Cardiovascular System ■ 323

____________________ Recruitment___________________ ________________________Distension_______________________


- Under normal resting conditions of cardiac output, - Under normal resting conditions of cardiac output, perfused
some pulmonary capillaries are either closed or open pulmonary capillaries are not distended to their maximum
but with no blood flow (i.e. they remain unperfused). caliber /diameter.
- As the cardiac output is increased and pulmonary - As the cardiac output & pulmonary blood flow increases to
blood flow & pressure begins to rise, these previously much higher level or when the lungs fill with blood, as it does
closed / unperfused capillaries begins to open. in left heart failure, which is a/w elevated left atrial pressure
- This recruitment establishes new parallel pathways for the internal pressure the internal pressure or transmural
blood flow, thus lowering the overall pulmonary pressure gradient of already perfused/open pulmonary blood
vascular resistance. vessel increases causing distension of vessels.
- Recruitment is a passive process of increasing the - D istension is chiefly a passive change in shape of capillaries
number of open parallel capillaries that were from near flattened to more circular. This increase in caliber,
previously not utilized in the resting state. This is the widening or distension causes fall in pulmonary vascular
chief mechanism of fall in pulmonary vascular resistance and is the predominant mechanism at relatively
resistance that occur as the pulmonary artery pressure high vascular pressure.
is raised from low levels.
'///////
y rzzzzzz :
7ZZZZZZ
R zzzzzz
4zzzzzz 7' Z Z Z Z Z Z
fz z z z z z z V/V/V/V W / / / / A

N orm al R e cru itm e n t D is te n s io n N orm al R ecruitm ent D istension

Passive processes of recruitment and distension lower pulmonary vascular resistance (and maintain pulmonary artery
pressure) over a wide range of cardiac output. Lung volume also determine the pulmonary vascular resistance (which
increases both at high and low lung volumes). Lung collapse (low lung volume) increases pulmonary vascular
resistance b/o reduced caliber/collapse of extra alveolar vessels. Whereas, vascular resistance increases with large lung
inflations (large lung volumes) because of reduced caliber of capillaries d/t stretching and thinning of alveolar walls.
Pulmonary vascular resistance increases with alveolar hypoxia (d/1 constriction of small pulmonary arteries) and drugs
causing contractionof smooth muscle of extra alveolar vessels (eg serotonin, histamine and norepinephrine). Whereas
acetylcholine & isoproterenol relax smooth muscle in pulmonary circulation.
Because pulmonary circulation is low pressure, low resistance system, it is influenced by gravity much more
dramatically than the systemic circulation is. For every 1 cm increase in height above the heart, there is 0.74 mmHg
decrease in hydrostatic pressure. This effect of gravity on blood flow affects arteries and veins equally. This is why in
upright position blood flow increases from apex of lung to the base, where it is greatest. And in supine position it is less
in anterior (upper most) and greater in posterior (lower) regions.
The blood volume in the lungs is ab o u t 450 ml, 9% o f to ta l b lo o d volum e. 70ml in pulmonary capillaries &
remainder is divided equally b/w arteries and veins. Blood flowing to the lungs through bronchial arteries is 1-2% of total
cardiac output. Pulmonary resistance is about one- seventh of systemic vascular resistance. In the lung the capillary
volume at rest is 70ml which may approach 200ml during exercise.
- Pulmonary capillary pressure is ~ 10 mmHg & oncotic pressure is 25 mHg, so an in w ard - directed pressure g rad ien t o f
a b o u t 15 tntnHg® keeps the alveoli free of all but a thin film of fluid. Mean pressure is about 15 mm Hg (in systemic 90
mm Hg)
- V entilation / perfusion ratio fo r w h o le lung a t rest is 0.8® (4.2 L/min v en tilatio n / 5.5 L/min b lo o d flo w ). V entilation
as w ell as perfusion declines in a lin ear fa sh io n from the b a se to ap ex o f the lungs® in upright p osition . So,
v en tilation as w ell as perfusion is m ore in low er lobes®. H ow ever, v en tilation / perfusion ra tio s are high in upper
portion s o f lung (highest a t apex)®, which accounts for the predilection of tuberculosis in this area.
- On lying down, pulmonary blood volume increases by upto 400 ml, which is discharged into general circulation when
the person stands up. This shift decreases v ita l ca p acity in supine position® & is responsible for the occurrence of
orthopenia in heart failure.
- P ulm onary sm all arteries & arterioles con strict in response to hypoxia® (especially when 0 2 conc. falls below 70% of
normal or <73 mm Hg P 0 2). This is o p p o site to the effect ob serv ed in sy stem ic vessels®, which dilate rather than
constrict in response to low 0 2 levels. This distributes blood flow where it is most effective (i.e., in most aerated areas).
So the pulmonary blood is distributed in proportion to their alveolar oxygen pressures. The vasoconstriction occur d/t
324 ■ A Complete Review of Short Subjects

presence of K+ gated channels in pulmonary vessels.


- Gravity has a marked effect. So at apex there is low er in trav ascu lar pressure, less b lo o d flo w , less v en tilation and
perfusion® and greater transmural pressure, more negative intrapleural pressure and larger alveoli as compared to
base
- P ulm onary veins are co lla p sed in the upper & m iddle lung lobes. They are disten ded in the lo w er lobes.® This is
because of effect of gravity. There is marked pressure gradient in the pulmonary arteries (/veins) from the top to the
bottom of lungs (maximum arterial pressure in the base of lung). In the upper & middle lobes the pressure in the
pulmonary veins is below the atmospheric pressure in the alveoli, this causes these veins to collapse. And pressure in
the basal lobes is high enough to keep them distended.
- 30 cm of lung height 1/t 23 mm Hg pressure difference b/w highest & lowest points of lung, about 15 mmHg of which is
above the heart and 8 below i.e., pulmonary artery pressure is 15 mm Hg lower in uppermost & 8 mmHg higher in
lowermost portions of lung than the PAP at level of heart
- The capillaries in alveolar wall are distended by blood pressure inside them and compressed by alveolar air pressure
on their out sides. This results in three lung zones
I---------------------------- I
Zone 1 : No Blood flow Zone 2 ; Intermittent Blood flow Zone 3 : Continuous
- It is a zone where pulmonary arterial In upper third of lung (or zone 2), pulmonary arterial Blood flow
pressure (Pa) is so low that it can be pressure is greater than P a , which is also greater than - In zone 3, Pa >PV>
exceeded by pulmonary alveolar pulmonary venous pressure (Pv) i.e. Pa > Pa > Pv- Pa
pressure ( P a ). The capillaries collapse b/o This is because the arterial pressure increases with - Continuous blood
greater external P a , and blood flow gravity but the alveolar pressure remains same in flow because the
ceases. whole lung. alveolar capillary
- No blood flow during all part of cardiac Intermittent blood flow occurring only during pressure remains
cycle because capillary pressure is < pulmonary arterial pressure peaks in systole greater than
alveolar air pressure during whole Systolic arterial pressure > alveolar air pressure; but alveolar air
cardiac cycle. diastolic arterial pressure < alveolar air pressure pressure during
- No gas exchange takes place and become So the blood flow is determined by the difference the entire cardiac
p art o f p h y sio lo g ica l (alveolar) d ead b/w arterial and alveolar pressure not the usual cycle
space® arterial venous pressure difference. Indeed venous - Found in lower
- Under normal conditions this zone does pressure has no influence on flow unless it exceeds areas of lung
not exist; however during positive Pa-
pressure mechanical ventilation (when Because Pa > Pv, it partically collapses capillaries
P a increases) and in severe blood loss causing damming /Starling resistor /Sluice or water
(when Pa decreases sufficiently) zone 1 fall effect.
can be established.

Bronchial Circulation

- Bronchial arteries arise directly from aorta and p rov id e nourishm ent /oxygenated b lo o d to lung parenchyma®. Bronchial
arteries are usually 3 in number, accompany the bronchial tree and divide with it.
- The main function of bronchial artery is to provide oxygenated systemic blood & nutrition to lungs i.e. walls of conducting
airways (bronchi, bronchioles) down to and including respiratory bronchioles, blood vessels (eg pulmonary arteries), nerves,
lymph nodes and most of the visceral pleura.
- Bronchial circulation receives 1-2% of total cardiac output, which increases as much as 10 to 20% (of CO) in cystic fibrosis.
Approximately 1/3 of blood returns to right atrium via bronchial veins; whereas the remainder drains into the left atrium
via pulmonary veins. The erosion of inflamed tissue into these vessels secondary to bacterial infection is responsible for
hemoptysis.
- B ron chial circulation causes venous admixing® (i.e. mixing of un-oxygenated bronchial venous blood directly into
oxygenated pulmonary venous blood) as its 2/3rd blood returns through pulmonary veins.
- Gaseous exchange (i.e. removal of CO 2 from deoxygenated blood and mixing of O 2 = oxygenation) takes place in pulmonary
circulation of lung.
Physiology: Cardiovascular System ■ 325

Myocardial Oxygen Consumption


% In c re a s e In M V O ,
• Basal O 2 consumption (determined by stopping the heart and artificially a b o v e r e s tin g v a lu e

maintaining coronary circulation) is 2 mL/100 gm /min, which is considerably higher


than that o f skeletal muscle®. O2 consumption by beating heart is 9 mL/100 gm /min
at rest, which is the highest per unit m ass consumption o f Oi®.
• Due to very high O 2 extraction, the arterio - venous O2 difference (ML/L) is also
maximum in heart. So increase in O 2 demand requires increase in coronary blood
flow otherwise ischemia develops.
• Factors that enhance magnitude b /or rate of tension development will increase
myocardial O 2 demand. So increasing heart rate increases O2 demand by increasing
rate (number o f tension cycles developed/minute); whereas increasing inotropy
(contractility) by increasing both magnitude & rate o f tension developed °.
• So myocardial O 2 consumption is prim arily determined by intram yocardial
tension, contractile state & heart rate®.
% in c rea se o f e a ch fa c to r a b o v e re stin g ve lu e

Myocardial O 2 consumption a Heart rate, Contratility, Intram yocardial tension®.


a Venticle work/ beat
a Stroke volume x Mean arterial pressure®

• Left ventricular stroke work is 7 times greater than that of right ventricle since the aortic pressure is 7 times greater than the
pulmonary artery pressure. Theoratically, a 20% increase in stroke volume without a change in arterial pressure and a 20%
increase in arterial pressure without a change in stroke volume should produce the same increase in O 2 consumption But
because of La plac relationship, pressure w ork (i.e, increase in after load or mean arterial pressure) produces a greater
increase in O2 consumption than volume w ork (i.e., increase in preload ® or stroke volume). This is why angina is more
common in AS than AR (aortic regurgitation). According to La place law

Intramyocardial tension (T) a Intraventricular pressure (P) x ventricular radius (r)

Factors determining (directly proportional to) myocardial oxygen demand

1
Heart Rate Contractility of ventricles (Inotropy) Intramyocardial wall Tension
Demand increases & decreases Demand increases & decreases with Wall tension is directly proportional to
with HR. contractility. Intracavitatory pressure (P i.e. after load) &
(i.e. M OC oc HR) (i.e. M OC oc Contractility and Ventricular Radius (R i.e. preload)
ionotropy)
It is inversely proportional to wall thickness (h)
Intramyocardial wall Tension
PR
~ 2/T
• Energy (ATP or O 2) expandature: Basal metabolism of heart tissue normally accounts for ~ 25% of myocardial ATP use (and
therefore same is the myocardial oxygen demand in resting individual). In basal metabolism energy is consumed in cellular processes
other than contraction eg energy dependent ion pumping. The energy spent during isovolemetric contraction (internal
cardiac work) phase of the cardiac cycle accounts for maximum portion (-50% ) of total myocardial oxygen (or ATP)
consumption despite the fact that heart does no external work during this period. It is during the ejection phase of the
cardiac cycle that the heart actually perform external cardiac work but the energy (ATP) or 0 2 consumed is only 25%
M yocardial O 2 / Energy (ATP) consumption (Demand)

Basal M etabolism M uscle Contraction

25% 25% 50%


326 A Complete Review of Short Subjects

• Energy required for cardiac contractions derives almost entirely from aerobic metabolism, and unlike skeletal muscle the
heart cannot be sustained on anaerobic metabolism for any length of time, & cannot incur an 0 2 debt. This is because energy
liberated under anaerobic metabolism, constitutes an insignificant 1% of total energy need & is not adequate to sustain
ventricular contractions. The efficiency (i.e. work done to energy expenditure) of normal heart is 20- 25%. The substrates for
cardiac metabolism are:
Fatty acid (provides 60% of energy)
Carbohydrates (lactate & glucose) provide 35% of energy
Ketones & amino acids provide 5% of energy
• Heart contains highest concentration of aspartate amino transferase (AST) then any other tissue in body & this enzyme is
released into circulation in death of any part of myocardiam.

Control of Blood Flow in Splanchinlc Circulation

Splanchnic circulation - The whole amount of blood that passes through the liver and open into the inferior venacava via
hepatic veins. Splanchnic vessels contain 20-30% o f total blood volume Q.

Sympathetic stimulation Gastrointestinal hormones Chemical control


Venoconstriction with Vasodilatation produced by: Vasodilatation & increased blood flow d/t
decreased blood flo w Q Gastrin Local lack of O 2
Arteriolar constriction - CCK-PZ Excess of CO 2
5 Hydroxytryptamine

Denervated Heart Graft


X
I
Autonomic regulatory Ted Sensitivity to catecholamines
mechanism absent
I ★ The cardiac transplant recipient
Vagal tone, carotid reflex & because of T (J- receptor density & can achieve about 70% of the
sinus arrhythmia absent loss of NE uptake in postganglionic, maximal output expected for his
I sympathetic neurons or her age, easily sufficient for the
Higher resting heart rate I stress of everyday.
Adequate cardiac response to exercise
& stress Q

Bain bridge reflex: Rapid infusion of blood or saline in anaesthelized animals


produces a rise in heart if the initial heart rate is low.
Infusion of fluids in person with transplanted heart increases the rate of recipient's
atrial remant but fails to affect the rate of the transplanted heart.

Erythropoiten Production Hormone Stim ulating Erythropoisis


Vasopressin
Testosterone
Stimulator Inhibitor
Cortisol
Local hypoxia Estrogen
Anterior pituitary hormone eg TSH , ACTH & Prolactin^
Prostaglandin E2
Adenosine
Norepinephrin
Thyroxine
Androgen
Physiology: Cardiovascular System ■ 327

2,3 DPG & Hemoglobin

2.3 DPG is present in high concentration in erythrocytes (HbA> HbA 2)


2.3 DPG g reatly reduces the affin ity o f Hb to 0 2 Q (shifts (^-dissociation curve to right)
Fetus H b (HbF) con sists o f o2V2, this tetram er h as much low er affin ity fo r 2-3 DPG than ad u lt Hb® & Corresponding a
higher affinity for O 2.

Viscosity of Blood Depends on

Hematocrit Composition of the plasma


Percentage of the volume, of blood occupied by red Depends upon the presence of plasma proteins such as
blood cells. Viscosity is directly proportional to immuno globulins & the resistance of the cells to deformation.
hematocrit. (viscosity is disectly proportional to composition of the plasma)
,____________ I________ ,_________________ 1_____________
I
Increased Decreased Increased Decreased
1 1 1 1
Polycythemia Anemia Multiple myeloma Hereditary spherocytosis

Neutrophils
_____________ Biochemical Features___________ Some important enzymes and proteins of neutrophils
- Active glycolysis Enzyme or Reaction Catalyzed or Comment
- Active pentose phosphate pathway Protein Function
- Moderate oxidative phosphorylation Myeloperoxidase H20 2 + X ' (halide) + H+ -> Responsible for the green color pus
- Rich in lysosomes and their degradative enzymes (MPO) HOX + H20 Genetic deficiency can cause
- Contain certain unique enzymes (e.g. (where X ' = Cl , HOX = recurrent infections
myeloperoxidase and NADPH-oxidase) and hypochlorous acid) H20 2-MP0-Halide. System is most
proteins efficient bactericidal system in
- Contain CD 11/CD 18 integrins in plasma neutrophils
membrane___________________________________ NADPH-odixase 2 0 2 + NADPH -> 2 0 + Key component of the respiratory
Neutrophils NADP + H+ burst
Deficient in chromic granulomatous
disease
Primary granules Secondary granules
Lysozyme Hyrolyzes link between N- Abundant in macrophages
• Appears at • Appears at
acetylmuamic acid and N-
promyelocytic stage myelocytic stage
acetyl-D-glucosamine
• Cotain following • Contain following found in certain bacterial
enzymes enzymes cell walls
- M y elop erox id ase0 - Lactoferrin Defensins Basic antibiotic peptides of Apparently kill bacteria by causing
- Hydroxylase - Lysozyme 20-33 aminoacids membrane damage
- Lysozyme® - Alkaline Lactoferrin Iron-binding protein May inhibit growth of certain
- Elastase phosphatase bacteria by binding iron and may be
- C athepsin GQ - Plasminogen involved in regulation of
- Defensin activator proliferation of myeloid cells
- Phospholipase A 2 - Phospholipase A 2 CD 11 a/CD 18, Adhesion molecules Deficient in leukocyte adhesion
• Arneth count: counting of lobes in the CD1 lb/CD 18, (members of the integrin deficiency type I (MIM 116920)
CD1 lc/CD182 family)
neutrophils.
Receptors for Fc Bind Fc fragments of IgG Target antigen-antibody complexes
fragments of IgGs molecules to myeloid and lymphoid cells,
eliciting phagocytosis and other
responses
★ Lysome is not found in erythrocytes, prokaryotes & virus. PH inside the lysosome is lower than that of cytoplasm. A cid
p h o s p h a ta seQ is the marker enzyme of lysosome.
328 A Complete Review of Short Subjects

Natural Killer (NK) Cells


Natural K iller (NK) cells are large, granu lar lym phocytes, w hich k ill virus in fected cells an d cancer cells by cell m ed iated
immunity®. They release perforin (perforates cell membrane) and granzyme (cause apoptosis). NK cells differ from
cytotoxic T cells in that they lyse sensitive cell targets on first contact (without a prior antigen sensitization). They d o n ot
require thym ic processing® and do not express surface T cells antigen receptors.
NK cells make 10-15% of leukocytes in blood. NK cells also play important role in antibody dependent cellular cytotoxicity
(ADCC). In ADCC, Fc receptor of NK cell bind to antibody attached to target cell. This provides NK cell another
opportunity to inhibit the replication of viruses and intracellular bacteria.
NK cells have 2 types of cell receptors: (1) lectin like NK cell receptor that binds to protein not carbohydrate and (2) KIRs
(killer immunoglobin like receptors which recognize MHC class I molecules HLA-B or HLA-C. NK cells turn into
lymphokine activated k iller cells (which is more efficient in killing) on exposure to lymphocytic secretions like IL2 and

Clotting Factors
Factor Names Clinical Syndrome
I FibrinogenQ Afibrinogen
II ProthrombinQ Hypoprothombinemia
(hemorrhagic
First step in intrinsic system is Extrinsic system is tendency in liver
activation of factor 12 which is triggered by release of disease)
catalyzed by high molecular tissue factor or tissue III Thromboplastin
weight kininogen (HMW K) and thromboplastin (TPL) a IV Calcium
Kallikrein. It occurs in vivo (body) lipoprotein - phospholipid V Proaccelerin, labile factor, Parahemophilia
when blood is exposed to collagen mixture from traumatized accelerator globulin
fibers underlying the tissue, which is a VII Proconvertin, SPCA, stable Hypoconvertinemia
endothelium in blood exposed to proteolytic enzyme & factor
collagen fibers underlying the activates factor 7. V III Antihemophilic factor Hemophilia A (classic
endothelium in blood vessels Extrinsic pathway is (AHF), antihemophilic hemophilia)
and can be brought about in vitro inhibitor by tissue factor factor A, antihemophilic
(test tube) by exposing the blood pathway inhibitor, which globulin (AHG)
to electronegative charged forms a quartemary IX Plasma thromboplastic Hemophilia B
wettable surface such as glass & structure with TPL, factor
component (PTC), (Christmas disease)
collagen fibers. 7a & factor 10a
Christmas factor,
HMWK TPL (Tissue thromboplastin)=(^) antihemophilic factor B
Kallikrein TFI (Tissue factor X Stuart-Prower factor Stuart-Prower factor
pathway inhibitor) deficiency
(12 XI Plasma thromboplastin PTA deficiency
antecedent (PTA),
antihemophilic factor C
XII Hageman factor, glass Hageman trait
factor
XIII Fibrin stab ilizin g fa ctor,
L aki-L oran d factor®
© HMW- High-molecular-weight
Prothrombin
K kininogen, Fitzgerald
activator (5 ^ P L , Ca++
factor
Pre-Ka Prekallikrein, Fletcher
Prothrombin ( ? ) ---------------- ■---- ►ThrombinQ
factor
Fibrinogen ( ? ) ------- ------------ 1----- ► Fibrin Ka Kallikrein
PL Platelet phospholipid
J ©
Common Pathway Stable cross
linked fibrin
★ P T T is p ro lo n g e d w ith d e fic ie n c y o f fa c to rs : 8, 9 , 1 1 ,1 2 , 2 , 1 , P r o th r o m b in ®
★ P T is p ro lo n g e d w ith d e fic ie n c y o f : 5, 7, 2 , 1 , P r o th r o m b in ®
★ E x cep t fo r th e first tw o ste p s in in trin sic p a th w a y , c a lciu m io n s a re re q u ired fo r p ro m o tio n o r a c ce le ra tio n o f all b lo o d clo ttin g re a ctio n s
Physiology: Cardiovascular System 329

Features Hemophilia A Hemophilia B Clotting Factors Involved in


(Classical (Christmas I--------------
hemophilia) disease) Intrinsic Pathway Common Extrinsic Pathway
_______ Only_______ Pathway Only_______
Defect Deficiency of D eficiency o f
coagulant fa c t o r IX®
12 (XII) 10 (X) - 7 (VII)
subunit of factor
11 (XI) 5 (V) - Tissue
VIII i.e. VIIIC
9 (IX) 2 (II) thromboplastin (III)
factor
V III=V IIIC + 8 VIII 1(1)
VWF Prekalle krein 13 (XIII)
HMW Kininogen
Pathway of Instrinsic Intrinsic pathway
coagulation pathway
affected APTT PTT
Clotting Time T T Measures time required to Measures time required to
PTT T T generate thrombin & fibrin generate thrombin & fibrin
polymer via intrinsic & polymer via extrinsic &
PT N N
common pathway common pathway
BT N N Reflects activity of________ Reflects activity of
1 2 ,1 1 ,9 ,8 + 10, 5, 2,1 7 + 10, 5, 2,1

Anticlotting Mechanisms in Normal Vascular System


Prevention of Clotting Lysis of Blood Clot
r i I
Endothelial Surface Circulating Anticoagulants in Plasmninogen (Fibrinolytic) System
Factor Blood Activated proein C (APC) inactivates
• Smoothness of • Antithrom bin action of Fibrin and inhibitors of t-PA, increasing the
Endothelial cell surface Anti thrombin III (AT III) formation of plasmin (fibrinogen) from
prevents contact activation of
- Most important anticoagulants in blood are plasminogen.
intrinsic clotting system those that remove thrombin viz fibrin and anti Large amounts of plasminogen is trapped
• Glycocalyx (Mucopoly thrombin III or antithrombin- heparin cofactor in the clot, which is activated after few
sacchride) layer on - During clot formation, 85-90% of thrombin days by tissue plasminogen activator (t-
Endothelium (formed from prothrombin) becomes adsorbed PA) a very power full activator which is
(mucopolysaccharide) to the fibrin fibers. This prevents spread of very slowly released by injured tissue &
layer repels clotting factor thrombin into remaining blood &, hence endothelium. Other activators are
& plateletts prevents excessive spread of clot. urokinase type plasminogen activator
• Thrombomodulin, a - Antithrom bin III, a circuiting protease inhibitor (U-PA) and thrombin.
thrombin binding protein, that binds to serine protease, combines with the Plasminogen (profibrinolysin), when
expressed in all remaining thrombin that does not adsorb to activated becomes plasmin
endothelial cells, except fibrin (in 12-20 minutes) (fibrinolysin). Plasmin is a proteolytic
those in crebral • Heparin, is a mixture of highly negative sulfate enzyme (like trypsin), which digest fibrin
microcirculation®. It polysaccharides with molecular wt. ~ 15-18 & fibrinogen with the production of
prevents clotting in 3 ways thousand. It is produced in large quantities by fibrinogen degradation products (FDP) that
- By removing thrombin basophilic mast cells located in pericapillary inhibit thrombin. It also lyses prothrombin,
(activator of factor 5 & 8) factor V, VIII and XII.
connective tissue. (Basophils also secret small
- Thrombin- thrombomodulin amounts). However, its concentration in blood is Recom binant t-PA (Altepase) and
complex activates protein low and by itself it has little or no anticoagulant Streptokinase
C that acts as an anti (bacterial enzyme) are used as fibrinolytic
property. But
coagulant by inactivating - Its binding with antithrombin III, increases agents in MI.
activated factor V & VIII effectiveness of anti thrombin III 100 - 1000 Annexin II, forms a platform on endothelial
- Activated protein C times. As a result heparin - antithrombin III cells
(APC) also inactivates on which components of fibrinolytic
complex removes free thrombin from blood
inhibitor of tissue system interact, producing fibrinolysis.
almost instataneously
plasminogen activator, Annexin V forms a shield around
- Heparin- antithrombin III a ls o rem oves
increasing the formation of (inhibits) a c tiv a ted clottin g fa c to r s IX, X, XI, phospholipids involved in coagulation has
plasmin an anti thrombotic effect.
XII®.
330 ■ A Complete Review of Short Subjects

L y ses fib r in , f ib r in o g e n ,
fa c to r V , V I I I , IX a n d p r o t h r o m b in

i
F D P ( i n h i b i t th r o m b in )

ABO Blood Group system

Highly immunogenic ABO blood group antigen is due to complex G e n o B lo o d F r e q u e n c y A g lu tin o - A g g lu tin in s P la sm a
oligosaccharide (glycoprotein) found on RBC membrane. It is ty p e Type (% ) in U S gens (a n tib o d y ) a g g lu tin a te s
inherited as medelian autosomal dominants'? (A n tig e n s) in p la sm a R B C ty p es

Alleles of ABO gene are located on long arm of chromosome 9 (and OO O 47 - A n ti A A ,B , A B


of Rh genes on chromosome 1). One of the 3 gene types (A, B, O) A n ti B
determines blood group. OA/ A 40 A A n ti B B, AB
Individuals can be divided into 4 major blood group types on the AA
basis of antigens (or agglutinogens) present on membrane of RBC.
OB/ B 10 B A n ti A A, AB
They are called agglutinogens as they are responsible for blood cell BB
agglutination in transfusion reactions. Type A individuals have A
AB AB 3 A&B - None
antigen, type B have B, type AB have both A & B, and blood group
type O have neither antigen.
A and B antigens are complex oligosaccharides that differ in their
terminal sugar moietyQ. H antigen is present in all blood types. H Fucose |— Galactose
gene coding fucose transferase that adds a terminal fucose, forms H
antigen. Individuals with type A express a second transferase that N-acetylgalactosamine
H antigen
catalyzes placement of a terminal N-acetylgalactosamine on H (fucose
antigen, whereas type B (individuals) express a second transferase Galactose transferase)
that places a terminal galactose. Individuals with AB blood group
have both and with O blood have neither transferases (i.e only H
Glucose
antigen persists in O group).
Plasma antibodies against RBC agglutinogens are called 00 00
1 1 Ceramide 1 1
agglutinins. Antigens very similar to A and B are very common in 11
intestinal bacteria and possibly food to which new borns are OO Lipid Bilayer q q

exposed. These substances initiate the development of Anti A &


Anti B antibodies. Therefore, infants rapidly develop antibodies
| N-Acetyl galactosamine ] [ Galactose
against the antigens not present in their own cells. So type A blood
group individual develop anti B, type B develop anti A, type O
develop both, and type AB individual develop neither. H H
Antigen Antigen
Immediately after birth, the quantity of agglutinins in the plasma is
almost zero. An infant begins to produce agglutinins, 2 to 8 months A- Antigen B-Antigen
after birth and maximum litre is reached at 8 to 10 yrs. of age. Then it (b y N -ace tyl (b y G a la c to s e
gradually declines throughout the remaining years of life. g a la c to s a m in e tra n s fe ra s e )
tra n s fe ra s e )
Anti A and anti B antibodies are isohaemagglutinins (iso
antibodies) as they are produced by an individual against antigens
Physiology: Cardiovascular System 331

produced by members of same species (isoantigens). Landsteiners Law in relation to agglutinogen


Antibodies are usually IgM type and cannot cross placenta. O type (antigens) and agglutinins (antibodies)
individual can produce IgG type antibodies.
- If an antigen is present on RBC membrane the
The basic difference between ABO blood group system and other
corresponding antibody (agglutitin) must be
blood group systems (such as Rh, Kell, Duffy, MNSs blood groups),
absent in plasma.
which makes the ABO group so important is that - preformed ABO
antibodies are present in persons serum when his RBCs lack the - If an antigen is absent on RBC membrane, the
corresponding antigen (ie. anti B antibody would be present in a person corresponding antibody (agglutinin) must be
of type A and type O blood groups. These two blood groups do not have the present in the plasma.
'B' antigen on the RBCs). This is not the case with other blood groups.
Preformed antibodies are not present.
They are formed only after an exposure to the antigen, for example Rh n egative p erson do n ot h a v e an ti- Rh an tib od ies, these
antibodies are formed only after an exposure to Rh positive blood (by a blood transfusion, i.e. this makes the first blood transfusion safe
even if mismatched). These preformed antibodies, rapidly destroy the RBCs of any mismatched blood transfusion.
"In clinical transfusion practice, the ABO blood groups are the most important and can never be ignored in red cell transfusion, because
individuals who genetically lack any antigen, have antibodies against the red cell types that they have not inherited. These antibodies can
destroy red cells rapidly in circulation". But the question arises - why are these agglutinins (antibodies against AB antigen)
produced in people who do not have the respective agglutinogens (A or B antigens) on their RBCs? The answer is that - Small
amounts of group A and B antigens enter the body in food, in bacteria, and in other ways, and these substances initiate the
development of the anti-A and anti-B agglutinins
Chapter 5. DIGESTIVE SYSTEM: REVIEW NOTES

Body Fluid K+ ion conc (meq/L) Total secretion pH


(L/day)
Saliva 25-30 (low flow) 1 (-1.5) 6 -7
15-20 (high flow)
Stomach 10 (may be upto 32) 1.5 (-2.5) 1-3.5
Pancreatic 4.5 (same as plasma 1 8 -8 .3
Bile 5- 6 (in liver bile) 1 7.8
12 (in gall bladder bile)
Small intestine 10-12 (in ileum) 1.8 7.5-8
Large intestine 75 (rectum)® 0.2 7.5-8
Brunner's gland - 0.2 8-8.3
secretion

Digestive Enzymes

Activator Proenzyme/ Enzyme Source


Trypsin (activates - Chymotrypsinogens 1
only pancreatic /chymotrypsin®. Salivary gland - a amylase • Exocrine pancreas
enzymes) - Pro carboxypeptidase A/ Lingual glands - lipase - Trypsin®
carboxypeptidase A Stomach - Chymotrypsin®
- Procarboxypeptidase B/ carboxy - Pepsinogen® - Elastase
peptidase B - Lipase - Pancreatic lipase®
- Pro-colipase/colipase Cytoplasm of mucosal cells - Bile salt-acid lipase
- Pro-elastase/elastase - Peptidases - Colipase
- Pro-phospholipase Intestinal mucosa - Cholestery ester
A 2/phospholipase A 2 - Enteropeptidase hydrolase
Enteroeptidase - Trypsinogen/Trypsin® - Aminopeptidase - Pancreatic a amylase
HC1 - Pepsinogens /pepsin® - Carboxypeptidase - Carboxy peptidase A and
ci- - Salivary and pancreatic a amylase - Endopeptidase B
- Dipeptidase - Phospholipase A 2
★ Amino / Carboxy / endo - peptidases cleave amino - Maltase, lactase - Ribonuclease
terminal / carboxy terminal / mid portion of - Sucrase, Ce dextrinase - Deoxyribonuclease.
polypeptides respectively. - Trehalase
- Nuclease

Proprotelns (Proenzymes or Zymogens)


• Zymogen is an inactive precursor of enzyme /protein that require selective proteolysis for activation. Selective
proteolysis involves one or more highly specific proteolytic clips that may or may not be accompanied by separation of
resulting peptide. So selective proteolysis often result in conformational changes that create the catalytic site of an enzyme or it
unmasks the active site of an enzyme by removing small region of peptide chain (by hydrolysis of specific peptide bond).
• Enzymes needed intermittently but rapidly often are secreted in an initially inactive form since new synthesis and secretion
of required proteins might be insufficiently rapid to respond to a pressing pathophysiological demand such as clot
formation, clot dissolution, tissue repair etc. So proenzymes facilitate rapid mobilization of activity in response to
physiological need.
• Proteolytic digestive enzymes are known as proteases. These are secreted as inactive zymogens. The synthesis of enzymes as
Zymogens protects the cell from being digested by its own product (autodigestion). Proteases are of two types
Physiology: Digestive System ■ 333

Trypsin can activate several proenzymes


I
Exopeptidase Endopeptidase 1. Chymotrypsinogen to chymotrypsin
I I 2. Procarboxypeptidase A/B to
Hydrolyse peptide bonds one at a time Hydrolyse peptide bonds in
carboxypeptidase A/B
from the ends of polypeptide chain midportion between specific
3. Proaminopeptidase to aminopetidase
__________ I_____ aminoacids. eg. are Pepsin, Elastase,
I 4. Proelastate to elastase
Trypsin, Chymotrypsin
Carboxypeptidase Aminopeptidase
HCIQ
(in pancreatic (in small Pepsinogen ► Pepsin (in stomach)
huice) intestine) (zymogen)
Cleaves Cleaves
EnterokinaseQ
aminoacid from aminoacid from Trypsinogen-------------► Trypsin (in
carboxy terminal amino terminal (zymogen) small intestine)

Most of clotting factors (factor 2 ,7 ,9 ,1 0 ,1 1 ,1 2 ) are zymogens of serine proteases. Similarly tissue plasminogen activator
(t-PA) and angiotensinogen are also screted as zymogens.

Trypsin
It is a proteolytic enzyme (proserine-proteinase) involved in digestion of proteins - chymotrypsinogen, procarboxypeptidase A
&c B, procolipase, prophospholipase A2, and proelastase. It is secreted in inactive form trypsinogen

Activators of trypsin Inhibitors of trypsin


Active site which is histidylserine residue is unmasked in the Alpha (a) 1 antiproteinase or Alpha (a) 1 antitrypsin Q
process of activation. It is brought about by - Di isopropyl fluro phosphate (DFP)
- EnterokinaseQ, a glycoprotein enzyme of intestinal juice Egg white (containing water soluble mucoprotein)
at a pH of 5.5 Human and bovine colostrums
- Autocatalytically by trypsin once it is formed at a pH of Raw syoabean
7.9 a l antitrypsin is synthesized by hepatocytes and
- Ca++ is also required. macrophages & is principal serine protease inhibitor that
Enteropeptidase Inactive inhibits trypsin, elastase etc by forming complexes with
(Enterokinase) Hexapeptidase them. Its deficiency has role in emphysema mainly in
Trypsinogen
subjects with zz genotype. Methionie (residue-358) of ai
Ca++
Autocatalytic Active Trypsin antitrypsin is involved in its binding to proteases.
Smoking inactivates it by converting it to methionine
Chymotrypsinogen [Trypan h Chymotrypsin sulfoxide. So it cannot neutralize proteases, This is
Procarboxypeptidase A/B ■Carboxypeptidase A/B particularly devastating in Pizz phenotype who already
Proaminopeptidase Aminopeptidase have low levels of a l antitrypsin.
Proelastase Elastase

Regulation (Phases) of Gastric Secretion


Gastric secretion & motality are regulated by neural & humoral mechanism and has 3 phases
Neural components are local autonom ic reflexes involving cholinergic neurons and parasym pathetic impulses from CNS
(by vagus nerve)®. Vagal stimulation increase gastrin secretion by release of GRP.
Humoral components are hormones such as gastrin, GRP and acetylcholine

Cephalic Phase Gastric Phase Intestinal Phase


- M ainly mediated by vagal •Once the food enters the stomach it excites - Once the food enters upper
(parasympathetic) inputs originating Gastrin mechanism (predominantly portion of small intestine (esp
from the dorsal vagal complex in responsible for secretion) because of meal duodenum) it causes small
brain. Vagal outflow to stomach cause constituents amounts of gastric juice
GRP & acetylcholine release, there by Lung vago-vagal reflex (form stomach to secretion because of gastrin
initiating secretion. brain to stomach) and local enteric reflexes released from duodenal
It results from sight, smell, thought, or are activated by distension of stomach & mucosa® (Guyton)
taste of food even before food enters stretch receptor activation. These further - Although small intestine has
stomach amplify secretion. gastrin containing cells
334 ■ A Complete Review of Short Subjects

Neurogenic signals of gastric secretion • Meal buffers gastric acidity which otherwise is a instillation of amino acids
arise in cerebral cortex, & in appetite inhibitory feed back signal to shut off secretion directly into duodenum does
centers o f amygdala and hypothalmus. secondary to release of somatostatin ( which not increase circulating gastrin
The efferent fibers travel through vagus inhibits G, ECL cells and parietal cells ). levels.
nerveQ (dorsal motor nuclei). • Receptors for local enteric reflex are in wall of Fats, carbohydrates & acid in
This parasym athetically (vagally) stomach & mucosa and respond to stretch and duodenum inhibit gastric
mediated phase can be conditioned. chem ical stimuli mainly amino acids & products motality, acid and pepsin
Tension, anger & hostality increase o f protein digestion. The fibers from receptor secretion via neural &
gastric secretion whereas fear & reach submucosal plexus, which synapse on hormonal (peptide YY)
depression decrease it. postaganglionic parasympathetic neurons that mechanisms. That's why
It accounts for 20% of gastric end on parietal cell & increase acid secretion gastric acid secretion is
• It accounts for m ost (70%) o f gastric secretion Q increased proportionately to
secretionQ (Guyton) / i to — ac^
3 2 (~ 1500ml) the amount of small intestine
secretion (Ganong) removed.

Factors Affecting Intestinal Motality

Hormonal Neural Physical & chemical

- Gastrin, CCK, insulin, motilin, substance P Parasympathetic (cholinergic) - Distension o f gut increases
and serotonin enhance intestinal motalityQ. stimulation (Ach) enhances gastric & motality^
Whereas, secretin, neurotensin and intestinal motalityQ - Chemical stimulation by
glucagon inhibits small intestinal Sympathetic stimulation inhibit gastric acid, protein products &
motalityQ. & intestinal motality. fatty acids alter motality by
- Stomach emptying (motality) is inhibited by increasing secretin and
Gastro-enteric, gastro-colic, duodeno-
cholecystokinin (CCK), secretin & gastric colic reflexes increase motality, while CCK secretion.
inhibitory peptide (GIP) whereas gastrin entero-gastric and colo-ileal reflexes
increase stomach motality. inhibit motalityQ.

Intestinal motality is dependent of gastric motality; as explained below


- Peristalitic activity of small intestine is greatly increased by distension of stomach wall (gastro enteric reflex) and duodenal wall.
- Chyme is sometimes blocked at ileocecal valve for several hours, until the person eats another meal, which initiates gastro-
ileal reflex. This intensifies perstalsis in ileum and forces the remaining chyme through ileo-cecal valve into cecum (large
intestine)
- Gastro- colic & duodeno- colic autonomic reflexes result from distension of stomach and duodenum. These facilitate appearance
o f propulsive mass movements after m eals, in large intestine Q.
- Entero- gastric reflex inhibits stomach secretions and motality (by inhibiting pyloric pump propulsive contractions & increasing
tone of pyloric sphincter) based on the signals from small intestine and colon.
- Reflexes from colon inhibit emptying of ileal content into the colon via colono-ileal reflex.

Gastrin

Gastrin is a hormone which is produced by G-cells in the lateral w all o f glands in the antral portion o f gastric m ucosa Q. It is
also found in pancreatic islets in feta l life, gastrinom as o f pancreas, and pituitary gland Q, hypothalamus, medulla oblongata
and in vagus & sciatic nerves.
Physiology: Digestive System ■ 335

Actions Factors affecting Gastrin Secretion


• Stimulation o f g astric a c id and pepsin Any stimuli that affect gastrin seretion also affects acid secretion from the
secretion ® stomach _____________________ | ______________________
• Stimulates growth of mucosa of stomach,
Stim uli that increase gastrin Stim uli that inhibit
small & large intestine (trophic action).
(& HC1) secretion gastrin secretion
• P rom otes g astric emptying®
- Oligo-Pep tides and amino Luminal - Acid®
• Stimulation of Insulin secretion.
acids in gastric lumen - Somatostatin®
★ T otal acid secretion depends upon the
- Distention®
fu n ction al p a r ieta l cell mass®.
- Cau ® Blood borne - Secretin®, GIP, VIP®,
★ Acid secretion is stimulated by
- Epinephrine glucagons, Calcitonin ®
- H istam ine Via H 2receptor®.
- Increased vagal discharge Neural
- Acetylcholine via M 3 receptor.
via bombesin GRP
★ H2 blocker eg ranitidine decreases gastric
[Gastrin Releasing
acid secretion; But proton pump inhibitor
Polypeptide -
like omeprazole blocks gastric acid
Noncholinergic]
secretion.

Factors Regulationg Gastric Emptying


Increase Emptying (Gastric Factors)
- Gastric food volume distending
Decrease Emptying (Duodenal Factors)
- Enterogastric nervous reflexes

(stretching) the stomach wall elicit - Hyperosmolarily of duodenal content (chyme) decreases, whereas
local myentric reflexes thereby hypoosmolarity increases emptying of gastric content
accentuating pyloric pump & - Hormonal feed backs involving cholecystokinin (CCK; mainly d/t protein &
inhibiting pyloric sphincter. Distension fat entering duodenum)®, secretin (b/o acidic chyme)®, gastric inhibitory
by water quickens emptying more peptide(GIP; weak), peptide YY (messenger) and somatostatin (±)
then distension by solids - Carbohydrate rich food leaves stomach in few hours. Protein rich food leaves
Gastrin mild to moderately enhances more slowly & em ptying is slo w est a fte r f a t rich meal®. Carbohydrate
pyloric pump. (fastest) - * protein -» Fat (slowest emptying)

Amphipathic Bile Acids: Micelles


• Bile acids are synthesized from cholesterol and bile salts are sodium & potassium salts of bile acid.
• Bile salts have am p h ip ath ic nature®, i.e. they have both hydrophilic and hydrophobic domains. Therefore they tend to form
cylindrical disk called micelles, with the hydrophilic surface facing out and the hydrophobic surfaces facing in.
• L ipids co llec t w ithin the m icelles. B ile acid s h a v e detergent action on lip id s d /t am p h ip ath ic nature®.
• The micelles play are important role in keeping lipids in solution & transporting them to the brush border, where they
absorb.

Proteins Synthesized by Liver


Protein Function
Albumin Ormotic regulator; and carrier protein binding hormone, aminoacids,
sterioids, vitamins, & fatty acids
a-fetoprotein Osmotic regulator; binding and carrier protein for hormones, amino acids
(although functions are uncertain but because of its structural
homology to albumin these functions are assigned)
oti-antiprotease Trypsin & general (serum & tissue) protease inhibitor
a 2- macroglobulin Serum endoprotease inhibitor
Antithrombin III Protease inhibitor of intrinsic coagulation system (1:1 binding)
Angiotensinogen Precursor to pressure peptide angiotensin II
Apolipoprotein B Lipid carrier assembly of lipoprotein particles
Antithrombin C, Protein C Inhibition of blood clotting
Coagulation factors II, V II, IX, X Blood clotting
Ceruloplasmin Transport copper (6 atoms Cu/mol)
C- reactive protein® Uncertain, tissue inflammation, bind complement C lq
Fibrinogen Precursor to fibrin in hemostasis
H aptoglobin® Bind & transport cell free hemoglobin (1:1 binding)
336 ■ A Complete Review of Short Subjects

Hemopexin Binds to prophyrin, particularly heme for heme recycling.


Insulin like growth factor I Mediator of anabolic effects of growth hormone (binds to IGF-1
receptor)
Orosomucoid Uncertain, ± role in inflammation
Sterioid hormone binding globulin Carrier protein for steroid in blood
Thyroxine binding globulin Carrier protein for thyroid hormone in blood
Trans- thyretin (Thyroid binding Carrier protein for thyroid hormone in blood
pre albumin)
Transferrin Transport of iron
★ M ost o f other plasm a proteins except antibodies (yglobulins or immunoglobins) are synthesized in liver®.

Enterogastrlc Reflex Postvagotomy Diarrhoea


When the chyme enters the intestine, the gastric muscle is inhibited Truncal vagotomy is associated with climically
and the motility stops leading to stoppage of gastric emptying - significant diarrhoea in 5-10% of patient.
enterogastric reflex. This reflex involves the vagus nerve. Factors that Factors responsible for diarrhoea are:
stimulate enterogastric reflex - Intestinal dysmotility (Irregular peristalsis)®.
1. Degree of distention o f duodenum® - R apid gastric emptying®.
2. Any degree of irritition of duodenal mucosa - Bile acid malabsorption
3. The degree of acidity o f duodenal chyme®. - Bacterial over growth due to decreased gastric
4. The degree of (high) osm olarity o f chyme® acid secretion®.
5. Presence o f breakdown products o f protein and fat®. - Fat malabsorption
★ Chyme - Peristaltic movements of stomach mix the food bolus with - Exaggerated gastrointestinal peptide response.
gastric juice and convert it into semisolid material.

Motality of Colon BER of Colon


- Basal electrical rhythm that governs
M ixing & Retarding Movements High Amplitude (Intensity) the rate & origin sites of smooth
Propagating Contractions or Mass muscle contraction in small intestine
Consistent with its primary function (of
Action Contraction does not cross the ileocecal valve to
water & electrolyte absorption), the two
In healthy individuals colon engages continue into the colon. So the
predominant motality patterns of large
in high intensity (amplitude) motality movements of colon are cordnated
intestine are directed not to propulsion of
pattern that sweesps along the length of by the BER of colon.
colonic content but rather to retarding
(slowing) their movement and mixing of large intestine from cecum to rectum - The frequency of BER/wave, unlike
contents. Both originate in response to lo c a l' It occurs about 10 times per day and in small intestine, increases along the
conditions eg distension. Two types occurs only in the colon colon, from about 2/min at the
include As a result of both local influences & ileocecal valve to 6/min at the
long reflex arcs, there is simultaneous
contraction of smooth muscles over large Transit Time In Small & Large
Short Duration Long Duration (20-60 Intestine
confluent areas. They move exclusively
(8 sec) Stationary sec) Peristalitic waves Test meal / undigested Transit
in aboral direction and are designed
Pressure Waves I food reaches time
to clear the colon of its contents.
or Segmentation Produced by teniae coli 1st part of test meal reach 4 hr
Contractions It moves colonic content from one
and may propogate over cecum
I portion to another and then into
short distances in both Colon entery of all 8-9 hr
Originating in rectum - and rectal distension initiates
directions ie orally as undigested portion
circular muscles well as aborally. defecation reflex. Although can be
associated with defecation it does not Traverse the first third 6 hr
necessarily result in defecation. (ascending) colon
Traverse the second 9 hr
Primary Fuction of Large Bowel
third of colon (ie
- Large intestine (colon) works as a reservoir for the residual meal that cannot be transverse colon)
digested or absorbed. The major primary role of colonic epithelium is to either Reach the terminal part 12 hr
absorb or secrete electrolytes & water rather than nutrients. Secetion (confined to of colon (sigmoid colon)
crypts) maintains the sterility of crypts. From sigmoid colon to Much
- By absorbing 90% of fluid, it converts 2L of isotonic chyme that enters colon to 250 anus slower
ml of semisolid feces. However, colon has 3 times more (4 to 6L) reserve capacity for * 70% matter reach stool in 72 hrs but
fluid absorption; and diarrhea results only after exceeding this. total passage takes >1 week
Physiology Digestive System ■ 337

Defecation Reflex

• There are two types of defecation reflex. In a similar way, when the rectum is inflated
1. Intrinsic reflex mediated by local enteric nervous system with gas (CO 2 ) in sig m oid oscop y , nerves are
2. Parasympathetic defecation reflex mediated by sacral segment of the spinal stimulated and causes increased peristalsis in
cord. the descending colon, sigmoid (d istal
• In both these types, the reflex is initiated by distention of the rectum by the colon)® and rectum.
feces and causes increased peristalsis in the descending colon, sigmoid colon
and rectum. This forces the feces toward the anus.

Small-intestinal water transport. Typical quantities (in by L-cell, secretin by S- cell, motilin by M-
volume per day) of fluid entering and leaving the small cell and Neurotensin by N- cell,
intestinal lumen in healthy adult are given

Site: Sm all intestine Colon


Substance Absorbed Upper (Proximal Jejum um mainly Mid Lower
duodenum in most cases except (distal
NaCl & H CO j ) ileum)
1. Na+ 3+ 2+ 3+ 3+
2. Cl- 3+ 2+ + +
3. Ca++ 3+ 2+ + 7
4. Long chain fatty acid (absorption & 3+ 2+ + 0
conversion to triglycerides)
5. Water soluble & fat soluble vitamins except 3+ 2+ 0 0
B 12
6. Iron (Fe++) 3+ + + 7
7. SO 4- 2+ + 0 7
8. Pyrimidines (thymine & uracil) + + 7 7
9. K+ + + + Secretion
10. Sugars (glucose, galactose etc). 2+ 3+ 2+ 0
11. Aminoacids 2+ 3+ 2+ 0
12. Antibodies (in newborn) + 2+ 3+ 7
13. Bile salts + + 3+
14. Vitam in B 12 0 + 3+ 0
15. Betaine, Dimethylglycine, sacrosine + 2+ 2+ 7
338 ■ A Complete Review of Short Subjects

Lipid (Fat) Digestion Fat Absorption


Major fat absorption takes place in upper
In Stomach: By Lingual & Gastric In Small Intestine: By Pancreatic
sm all intestine (mainly jejunum &
Lipase Lipase, Colipase & Bile Salts
duodenum) except short chain fatty acids
Lingual & gastric lipases are of Most fat digestion begins in the which is absorbed in colon®.
little quantitative significance for duodenum. Enzyme involved are: Short chain FAs (SCFAs) are 2-5 carbon
fat digestion^ except in cases of 1. |Pancreatic Lipase weak acids produced in colon by action of
pancreatic insufficiency. However, colonic bacteria on complex carbohydrates,
It acts on fats that have been
they may produce FFAs that causes resistant starches & dietary fibers that
emulsified. It hydrolyzes 1 and 3
release of CCK by singaling to most escape digestion in upper gitract. 60% o f
bonds of TG (triglycerides) with ease
distal parts of GI tracts. SCFAs is acetate, 25% propionate & 15%
but bond 2 at a very low rate, so
Lingual lipase: butyrate® withan average 80 mmol/I
producing FFA (free fatty acids) and 2-
Secreted by Ebner's glands on the luminal concentration. SCFAs contribute to
monoglycerides (2-monoacylglycerol).
caloric intake, exert trophic action on coloic
dorsal surface of the tongue.
2. pancreatic-Colipase epithelium, combat inflammation and maintain
It is an acid- stable lipase.
A protein, which is secreted in acid-base balance (as are absorbed in
It is active in stomach & can digest exchange for H+ ion).
pancreatic juice. Secreted in inactive
as much as 30% of dietary In enterocytes, fate of FA depends on their
form & activated by trypsin.
triglyceride size. <10-12 carbon atom FAs are water
It facilitates the activity of lipase by
soluble enough to pass through enterocytes
Role of Bile Salts: Formation of binding to - COOH terminal and
unmodified into the portal blood. They
Micelles opening (benting back) the lid covering
circulate as free (unesterified) FAs. But >10-
Fats are finely em ulsified in the the active site of lipase. It also allows
12 carbon FAs are insoluble and are
small intestine by the detergent lipase to remain associated with
reesterified to TGs in the enterocytes.
action o f bile salts®, lecithin, droplets of dietary lipids
Normal fecal fat excretion is less than 6 gm
monoglycerides and 3. Bile Salt Activated Lipase: /day, more than 6 gm /day indicates
phosphatidylcholine. Cholesterol Esterase malabsorption®. Steatorrhea is defined as
Lipids & bile salts interact It is 10-60 times less active, but unlike stool fat > 7gm/day®.
spontaneously to form micelles. pancreatic lipase, it catalyzes the
Micellar formation solubilize the hydrolysis of cholesterol esters,
lipid & provides a mechanism for eastrers of fat soluble vitamins &
their transport to the enterocytes. phospholipids as well as triglycerides.

Vitamin B / Cyanocobalamine Iron Absorption

Absorption I Causes of Deficiency A lm o st all iro n a b so rp tio n o c cu rs in d u o d e n u m a n d u p p e r je ju n u m


( p r o x im a l s m a l l in tes tin e)® .
V ita m in B 12 n o rm a lly b in d s to - D u e to in a d e q u a te
in t r in s ic fa c t o r ® ( s e c r e t e d b y d ie ta ry in ta k e is v ery rare. M o st o f th e iro n in d ie t is in fe rric fo rm (F e3*) b u t it is f e r r o u s (F e*2) t h a t
p a r i e t a l o r o x y n t ic c e l l s o f g a s t r i c - P ro d u ce d b y i s a b s o r b e d ® . T ra n sp o rt o f F e 2* in to en te ro c y te o c cu rs v ia D M T 1.
m u c o s a )® G a s tr e c to m y ® , w ith
C o m p le x is tak en u p b y cu bilin ® . re m o v a l o f in trin sic
Increased by Decreased by
(H ig h a ffin ity a p o lip o p ro te in ) in fa c to r p ro d u c in g tissu e • A c id s > • A lk a lies
th e d i s t a l ileum ® . - In p e r n i c io u s a n e m ia ® . • A s c o r b ic a c i d • P h osp h ates®
T h is trig g ers a b so rp tio n o f (V it 0 ® R ed u ces • P h y t a t e s ® (in m aize,
co m p le x b y en d o cy to sis. • A m in o acid > F e3* w h ea t)
★ D e fic ie n c y ca u ses
In ilea l e n te ro c y te s, B i 2 is co n ta in in g in to • T e tra cy clin
m e g a l o b l a s t i c a n e m ia ® .
tra n s fe rre d to t r a n s c o b a l a m i n e I I S H - g ro u p F e 2* • P re se n c e o f o th e r fo o d in
( t r a n s p o r t e r p r o tein )® . • M ea t sto m a ch

Role of Colonic Microflora (Enteric Bacterial Ecosystem)


L a r g e in t e s t in e c o n t a in s a u n iq u e b io lo g ic a l e c o s y s t e m c o n s is t in g o f tr illio n s o f c o m m e n s a l (n o n p a t h o g e n ic ) b a c te r ia th a t e n g a g e in a
life lo n g s y m b io tic r e la t io n s h ip w it h th e ir h u m a n h o s t. T h is c o lo n ic e c o s y s t e m is e s t a b lis h e d s h o r tly a ft e r b ir t h a n d r e m a in s r e m a r k a b ly
s t a b le u n le s s d is t u r b e d b y a n tib io tic s . It is h ig h ly u n u s u a l r e la t io n s h ip a s o th e r b o d y c o m p a r tm e n ts a r e la r g e ly s te r ile . It is in v o lv e d in
Physiology: Digestive System ■ 339

Fermentation & Digestion of Complex Carbohydrate Detoxification


- Detoxify xenobiotics (eg drugs) & dietary carcinogen (but may also
- Colonic bacteria metabolize meal components that are
generate toxic or carcinogenic compounds from dietary substrates.
not digested by host enzymes and make their products
available to the body via fermentation. It provides
Antimicrobial
advantage to the host in that the colonic bacteria are
capable of performing metabolic reactions that do not take - Gut flora limits the growth & invasion of pathogenic
place in mammalian cells. microorganisms by
1) Functioning as a physical barrier to prevent attachment of pathogens.
- Carbohydrates that humans cannot digest without
2) Forming microbicidal substances,
bacterial help include complex carbohydrate, resistant
3) Triggering gene expression in epithelium that counteracts the
starches, dietary fibers, mucin (moucous) produced by
adverse effects of pathogens
gut and oligosaccharides / sugars that the body failed to
digest /absorb d/t some pathology eg lactose in lactose
Prevent Allergy & Provide Immunity
intolerance
- Gut flora is not essential for life as animals raised in germ free
Formation of Short Chain Fatty Acids (SCFAs) conditions apparently develop normally. However, in these animals
the m ucosal immunity is immature and intestinal ep ith elial cells
- Colonic bacteria form SCFAs by femanting carbohydrates differentiate more slowly®
(starches, fibers). SCFAs are 2-5 carbon weak acids with
- Colonic microflora trains immune system preventing allergy &
80mmol/L average normal concentration in lumen. About
providing immunity.
60% of this is acetate, 25% propionate and 15% butyrate.
- SCFAs are absorbed & metabolized to make a significant Synthesis of Vitamin K & Biotin
contribution to total calory intake - butyrate is a critical
energy source for colonocytes. Substrate Enzymes Products Disposition
- SCFAs excert a trophic effect on colonic epithelium
Endogenous Substrates
cells. The colonic epithelium turns over rapidly even in
health, limiting the genetic damage that might otherwise Urea Urease Ammonia Passive absortion or
be caused by exposure to toxins in lumen. However, this excretion as
also increases the risk of malignancy. The colonic flora ammonium
(via SCFAs) is involved in promoting development of Bilirubin Reductases Urobilinogen Passive
normal colonic epithelium and in stimulating its Stercobilins reabsorption
differentiated functions. Butyrate regulates the Excreted
expression of specific gene in colonic epithelial cells and
Primary bile Dehydroxylase Secondary Passive
may suppress the development of a malignant
acids s bile acids reabsorption
phenotype. Expression of SMCT1 (sodium
monocarboxylate transporters 1) eg SLC5A8 may be Conjugated bile Deconjugases Unconjugate Passive
reduced in colon cancers, thereby reducing butyrate acids (primary d bile acids reabsorption
uptake - which may contribute to malignant or secondary)
transformation. So SCFAs increase the growth of intestinal Exogenous Substrate
epithelial cells and control their proliferation & differentiation
Fiber Glycosidases Short-chain Active absorption
(ie promoting normal development & stimulating fatty acids Excreted in breath
differentiation) Hydrogen, or flatus
- SCFAs combat inflammation (prevent inflammatory CO2 and
bowel disease) maintain acid -base equilibrium (absorbed in methane
part by exchange for H+) and promote absorption of Na+,
Amino acids Decarboxylase Ammonia Reabsorbed or
Mg*+, Ca++ and Fe++
s and and excreted (ammonia)
Metabolism of Exogenous & Endogenous Substances deaminases bicarbonate as ammonium
- Gut flora forms secondary bile acids and deconjugate any Cysteine, Sulfatases Hydrogen Excreted in flatu
bile acids that have escaped uptake in terminal ileum methionine sulfide
- Contribute to formation of intestinal gas.
340 ■ A Complete Review of Short Subjects

Diet Transit Time in Small


Intestine & Colon
Estimated Recommended Dietiy Allowance (RDA) Adequate Intake (Al)
• The first part of a test meal reaches
Average
It is based on the caecum in about 4 hours, all of
Requirement It is the average daily dietry intake that is
estimates of nutrient the undigested portions have
(EAR) sufficient to meet the nutrient requirement of
intake, by a group (or entered the colon in 8-9 hours.
It is an average nearly all (97-98%) individuals in a life stage
groups) of apperantly • On average, the first remnants of
& gender group.
daily nutrient healthy individuals, the meal reach the hepatic flexure
intake estimated to RDA is based on EAR and is statistically set that are assumed to in 6 hours, the splenic flexure in 9
meet the at 2 standard deviations above EAR® be adequate hours & the pelvic colon in 12
requirement of RDA is neither equal to Al nor to hours.
At is set instead of
50% of healthy recommended minimum requirement0 for • From the pelvic colon to the anus,
RDA if sufficient
individuals in transport in much slower®.
healthy individuals. Rather, it is intentially scientific evidence is
particular life stage • As much as 25% of the residue of a
set to provide a margin of safety for most not available to
& gender. test meal still be in the rectum in 72
individuals. In fact for many individuals calculate the EAR on
RDA would be in excess (but not injurious) RDA hours.
of their needs. So, the transit time of colon is the
longest<3.

Dietary Fibre
Defined as all ingested food that reaches the large intestine in an essentially unchanged form.

Advantage Components Category


>Contributes bulk by absorbing water 10-15 Consist of all plant cell wall It is of two types
times of its own weight (provides a larger components that cannot be I -------------1
volume of indigestible material to the colon) digested by animals own Insoluble fiber Soluble fiber
and thus hastens passage through the gut, enzymes. These include I I
decreasing the transit time®. High fiber diet ,________ I________ , - It promotes - It is dissolved in
aids water retention during passage of food movement of water to form a
Cannot be Neither material through gel like material
along the gut, producing larger & softer feaces;
hydrolysed or digested digestive system -Oat, flax seeds,
there by reducing stool - transit time.
digested (by nor & increase stool peas, beans,
>Reduces incidence of®:
endogenous fermented
- Diverticulitis, constipation hemorrhoids bulk apple, cirtus
enzymes) but is I
- Cancer o f colon® -Whole wheat fruits, carrots,
fermented by GI Lignin® flour, wheat
- DM (diabetes mellitus) barley &
bacteria bran, nuts, & psyllium are
- Coronary artery disease
I vegetables are good source.
i Soluble fiber low er total blood cholesterol
levels by lowering LDL or bad cholesterol - Cellulose good source - Pectins; gums
level®. It also slow s the absorption o f sugar - Hemicellulose - Cellulose, and oliginates are
and reduce the risk o f developing type 11 - Pectin hemicellulose & soluble
diabetes®. Soluble fibers found in legumes & - Pentosans lignin are
fruits eg gums & pectin, lower blood cholesterol - Gums & insoluble
level; and slow stomach emptying and thus delay aliginates
the post prandial rise in blood glucose so it is
beneficial in diabetes.

Glycemic Index

Glycemic index is a measure of a food's ability to elevate blood sugar level. The indexing is done by comparing a foodstuffs digestion
rate to that of glucose, with a glycemic index of 100. The higher the glycemic index, the faster will it enter the blood and increase
the blood sugar level.
Glycemic index is used to lable the food as good and evil. Foods with a high glycemic index of 60 and above are considered
unhealthy as they break down quickly and spike blood sugar level. Low glycemic foods enter the blood stream slowly and
provide sustained long term energy (maintain stable blood sugar levels)
Physiology Digestive System ■ 341

High glycemic food M oderately G ly cem icfo o d LowGlycemic food


Glucose 100 Orange Papaya 58 Apple 37
Baked potato 85 White rice 58 Yoghurt 33
Com flakes 84 Brown rice 55 Skimmed milk 32
Honey 73 Popcorn 55 Kidney beans 27
Watermelon 72 Sweet potato 54 Peanuts 14
White bread 70-72 Mango 55
Table sugar 65 Banana 50
Ice Cream 61 Carrots 49
Orange 44

Adipose Tissue

• F atty acid stored in a d ip o se tissu e in fo rm o f neutral TAG, serve as the body's m a jor fu el storag e reserve0. It is highly
concentrate source of energy as it is highly reduced, largely anhydrous and yield 9 k cal/gm energy (as compared to 4 k cal/g of
protein & carbohydrate).
• Fat stores constitute 75% of energy reserve and protein stores 25%. Glycogen is the smallest resource of stored energy. Metabolic
fuel stores in 70 kg man are

F a t: 15 kg = 135,000 K cal > Protein : 6 kg = 24,000 k cal > Glycogen: 0.2 kg = 800 k cal.

Acceptable Macronutrients Distribution Ranges (AMDR)

• M ain source o f energy in hum ans is carbohydrate. Macronutrient Range (% of energy) KC1 gm
The p rim ary role o f dietary ca rb oh y d rate is to Carbohydrate (No less than 130 4 5 -6 5
p rov id e enegy®. gm/day)
• The carbohydrates are not essential nutrient, because Protein 10-35 4
the carbon skeletons of amino acids can be converted Fat 20-25 9
into glucose by gluconeogenesis. However the - n-6 only unsaturated FA 5-10
absence of dietary carbohydrates 1/1 ketone body - n-3 Polyunsaturated FA 0.6-1.2
production, protein degradation & gluconeogenesis. Carbohydrate 45-65 4
• Recommended dietary allowance (RDA) for No less than 130 gm/day
carbohydrate is set at 130 gm/day for adults & - Added sugars
children, based on the amount of glucose used by - Fiber in men : 38 gm
carbohydrate dependent tissues, such as brain and - in women: 25gm
RBC.
• Adults should consume 45-60% of their total calories Quality of protein : Protein digestibility corrected aminoacid
from carbohydrate and added sugars represent no Scoring (PDCAAS)
more than 25% of total energy.
• Very low carbohydrate diets, providing only 20 Source PDCAAS value
gm/day, but permitting unlimited consumption of Aninial protein 1
Egg
fat and protein promote weight loss because of high Milk 1
energy (ATP) cost of gluconeogenesis. Since there is a Beef / Poultry /Fish 0.82- 0.92
continual demand for glucose, there will be a Gelatin 0.08
considerable amount of gluconegenesis from amino
Plant protein Soyabean 1
acids; the associated high ATP cost must then be met
Kidney beans 0.68
by oxidation of fatty acids.
Whole wheat bread 0.40
• Glycogen is the only carbohydrate in options
Chapter 6. EXCRETORY SYSTEM: REVIEW NOTES

Renal Medullary and Cortical Structures

Renal corpuscle (Bowmen's capsule & glomerulus), Difference


convoluted proximal part of proximal tubule, distal part
of thick ascending limb of loop of Henle, juxta Cortical Juxtamedullary
glom eru lar apparatu s (i.e. m acula den sa o f DCT, JG cells Nephron Nephron
o f afferen t arterioles & lacis cells)®, distal convoluted Over all % 85% nephrons 15% nephrons
tubule (DCT), collecting segment and cortica collecting Loop of Henle Short Long
ducts as well as interlobar, arcuate, interlobular vessels,
afferent arteriole, glomerular capillaries, efferent Peritubular Short Long & form
arteriole and peritubular capillaries are found in renal capillary network hair pin loops
cortex. (vasa recta)
Whereas, straight part of proximal tubule, thin O 2 extraction Very less Large
descending & ascending lim b and proxim al p art o f thick P02 50 mm Hg 15 mm Hg
ascending lim b o f lo o p o f Henle; m edullary collectin g Blood flow Large Less
duct as w ell as ascending & descending v asa recta® lie
Renal capsule
in renal medulla (pyramid)
Juxlaglom rular apparatus
Juxtaglom erular cells and |
Macula densa I

Renal corpuscle | Connecting


B ow m an’s capsule and j sem enl

Glom erulus I
1 Cortex

Distal convoluted H
tubule

Proximal Convoluted
tubule 1 straight
Collection duct

Thin lim b o f — 4— Thick limb of


loop of henle loop of henle } Medulla

Loop of henle

S c h e m a tic d ra w in g o f a ju x ta m e d u lla ry n e p h ro n s, th e lo o p s o f H en le
a re sh o rt, an d reach o n ly th e o u te r m ed u lla.

Nephron & Glomerular Filtration


Each human kidney has about 1 - 1.3 m illin nephrons®. The length of nephron including collecting ducts ranges from 45-
65mm. The human PCT is about 15 mm long® & 55 jJm diameter; DCT is about 5 mm long® and collecting ducts are 20 mm
long.
The nephrons with glomeruli in outer portions of renal cortex have short loops of Henle (cortical nephrons). Whereas,
about 15%® (Ganong) / 20- 30% ® (Guyton) nephrons have glomeruli in the renal cortex near the medulla (Juxta­
medullary nephrons). These have long loops of Henle extending down into medullary pyramids.
Total area of glomerular capillary endothelium across which filtration takes place is 0.8m2. Pore size of endothelium of
glomerular capillary is 70-90nm diameter. Filtration slits formed by pseudopodia of podocytes (epithelial cells) is 25 nm.
Functionally glomerular membrane permits neutral substance upto 4 nm diameter freely and excludes substance > 8nm totally.
Physiology: Excretory System ■ 343

Measurement of Renal Plasma Flow (RPF)


To measure renal plasma flow, a substance, which is filtered and secreted but not reabsorbed, should be used. Para aminohippuric
acid is such a substance and its clearance indicates renal plasma flow.
r
U pah - conc of PAH in urine
U,PAH
Renal plasm a flo w 0 = P pah - conc of PAH in plasma
V - urine flow

At low plasma concentration o f PAH, it is almost completely cleared from the plasma by a combination of glomerular
filtration and tubular secretion and indicates renal plasma flow.
When plasma concentration o f PAH are elevated beyond 30m g/dl; the secretary mechanism becomes saturated, and the
transport maximum (TM) is reached, so the clearance of PAH from plasma is decreased giving a false low value of RPF.
Afferent Efferent
arteriole arteriole
Renal Blood Flow

• Kidney receives 1.2 -1 .3 liters o f blood / minute or 25% o f


cardiac output at rest®.
• Glomerulus receives capillaries from afferent arterioles
- Afferent arterioles are short straight branches of inter lobar
arteries originating from renal artery.
- Each afferent arteriole divides into multiple capillary
branches. 20 -50 such fenestrated capillary loops form
glomerulus (glomerular vessels)
- The loops rejoin to form another efferent arteriol, which then
supplies the tubular part of nephron before emptying into
renal veins.
Basic kidney ptocessos mni c o m
• As the afferent arteriole enters glomerulus, the cells of tunica uunfr Unnary excretion rale ol a m mo -ale at
which the substance is Miterect ' • rate p.us me
media (smooth muscle layer) become more numerous & rate- at which it is secreted l?om in.*
th e tirbules
contin rennin granules. These are called juxta (i.e. next to)
glomerular cells. At the site of origin of distal convoluted
tubule (DCT), which is in close contact with the juxta
glomerular cells of afferent arteriole, the cells of DCT become
taller & larger, with more prominent nuclei than the other
cells lining the tubule, and are called macula densa. The
macula densa functions together with ' JGC, hence they are
jointly referred to as the juxta glomerular apparatus. By
tubuloglomerular feed back macula densa stimulate JGC to
release rennin. The mediator is NO.

Determination & Regulation o f GFR


GFR, like renal blood flow, remains constant over a wide range
of perfusing pressure, from 80 - 180 mmHg d/t autoregulation.
Factors affecting GFR are those affecting fluid exchange across
capillaries & are governed by Starting's force. GFR is
determined by changes in

1
Net Filtration Pressure Glomerular Capillary Filtration C oefficient (Kf)
It is sum of hydrostatic and colloid Glomerular capillary filtration coefficient (Kf) is a product of (1) hydraulic
osmotic pressures (forces) across conductivity or perm eability and (2) filtering surface area of glomerular
glomerular capillary membrane. The capillaries
forcing It can be measured indirectly (not directly) by this equation
GFR 125ml/min
K ,= = 12.5 ml/ min/ mmHg
Net filtration pressure 10 mmHg
344 ■ A Complete Review of Short Subjects

1. Favouring filtration - K f /100 gms weight for kidney is 4.2ml/min/mmHg (about 400 times higher)
than average (0.01) of other capillary system of body. This extremely high Kf value
- Glomerular hydrostatic of glomerular capillaries is responsible for their rapid rate of fluid filtration
pressure (Pc) inside glomerular - GFR is increased by raising K f and decreased by reducing Kf, but changes in Kf
capillaries (60 mmHg) donot provide a primary day to day mechanism for regulation of GFR
- Bowman's capsule colloid - Chronic uncontrolled hypertension and DM gradually decrease Kf by increasing the
osmotic pressure (hb) of proteins glomerular capillary basement membrane thickness.
outside glomerular capillaries
(OmmHg)_____________________

2. Opposing filtration

1. Glomerular Capillary Hydrostatic Pressure (Pg) 1. Bowman's Capsule Hydrostatic Pressure


• Changes in Pg is the primary means for physiological regulation of (Pb>
GFR. Increase in Pg increases GFR whereas, decrease in P c reduces GFR. Increased P b i.e. intra (Bowman's) capsular
• P c is determined by (1) arterial (blood) pressure (2) afferent arteriolar
pressure (m.c. d/t ureteric obstruction) oppose
resistance and (3) efferent arteriolar resistance hydrostatic filtering pressure & filtration
thereby decreasing GFR.
• Increase in BP increases hydrostatic pressure & GFR whereas decrease
in blood pressure decreases HP & GFR
2. Glomerular Capillary Colloid Osmotic
Pressure (jig )
• Change in caliber of afferent & efferent arteriole alter GFR as follows
- Glomerular capillary colloid osmotic pressure
- Afferent arteriolar constriction caused by sympathetic stimulation or
or glomerular oncotic pressure (plasma oncotic
thromboxane A2 reduces the hydrostatic pressure & GFR
/osmotic pressure) is exerted by proteins in
- Efferent arteriolar constriction caused by angiotensin II or ANP blood plasma and tries to pull water into the
increase HP & raises GFR circulation (ie capillary). It is unimportant and
- Afferent arteriolar dilation by ANP (atrial natriuretic peptide) increase normally opposes filtration.
HP & GFR - Glomerular filtrate is essentially protein free
• Auto regulatory mechanisms buffer effects of BP and maintain relatively because glomerular capillaries are relatively
constant glomerular pressure impermeable to proteins. This results in alm ost
• Efferent arteriolar constriction has a biphasic effect on GFR ie at zero (or very low) colloidal osm otic pressure of
moderate constriction, GFR increases slightly, whereas with severe glomerular filtrate®. Therefore glomerular
constriction (> 3 fold increase in efferent arteriolar resistance) GFR oncotic pressure is much higher than that o f
decreases. This is caused by nonlinear increase in colloid osmotic glomerular filtrate® in Bowan's capsule®.
pressure by Donnan effect exceeding the P c - - Conditions causing fall in plasma protein
concentration such as nephrotic syndrome and
2. Bowman's capsule Colloid Osmotic Pressure (its) cirrhosis decrease glomerular plasma osmotic
pressure.
Glomerular filtrate in Bowman's capsule has nil proteins & so almost
zero pressure.
★ Fluid emerging from loop of Henle and entering the distal tubule is always hypotonic. This is because of two factors
- Ascending loop of Henle & initial part of DCT are impermeable to water
- Ascending loop of Henle & DCT actively transport solutes out of tubule into interstitum.
Physiology: Excretory System ■ 345

Filterability Measurement of GFR

As the plasma volume is 3 liters and GFR is 180 L/day GFR can be measured by measuring the excretion & plasm a level
(7.5L/h or 125 mL/min) the entire plasma is filltered & o f a substance that is freely filtered through glomeruli and neither
processed about 60 times each day. secreted not reabsorbed by the tubules®.

Filterability of 1 means that the substance is filtered as


freely as water. Electrolytes such as sodium (N a*), and Requirement of substances Substances used
sm all organic compounds such as glucose, inulin are used to measure GFR
freely filtered® (filterability = 1). - Filtered freely - Inulin
Filterability of solutes is inversely related to their size. As - Neither reabsorbed, nor (standard)®
the molecular weight of molecule approaches that of secreted by tubules - Creatinine
albumin, the filterability rapidly decreases, approaching - Non toxic
zero - Not metabolized by body.
0.75 filterability means that the substance is filtered only
75% as rapid as water. M yoglobin (F = 0.75) and albumin Clearance of Substance, Tubular Secretion & Reabsorption
(F = 0.005) are n ot freely filtered®.
• Renal plasma clearance is the volume of plasma from which a
Glomerular filtrate is protein free, and devoid o f any substance is completely removed by kidney in a given amount of
cellular element, including RBC®. The concentration of time. Therefore clearance (GFR) of substance X =____________
other constituents including most salts & organic molecules, Ux (concentration of X in urine) multiplied by V
are similar to the concentration in plasma (i.e., are freely (urine flow per unit of time)
filtered). Exception to this generalization are few low
______________ Px (arterial plasma level of X)______________
m olecular weight substances, such as calcium & fatty
acid®, which are partially bound to plasma proteins and • If a clearance of a substance is greater than GFR, then an
thus not filtered freely. additional tubular secretion (of substance from blood into tubule)
must be present® but it doesn't mean that if there is tubular
Neutral substances with molecular diameter of < 4
secretion, the clearance of a substance will always be more than
nanometers m e filtered freely and filtration of substance >
GFR, as there may also be tubular reabsorption of substance.
8 nm approaches zero, as the pores of glomerular
membrane are of ~ 8 nm (80 angstroms) size. Between 4 - • Clearance or amount of substance excreted (depends on)
8 nm, filtration is inversely proportional to diameter.
However, negatively charged large m olecules are filtered
less easily than positively charged m olecules o f equal Clearance = G FR Clearance > GFR Clearance < GFR
m olecular size®, and even neutral molecule are filtered I I
much more easily than negatively charged molecules of There is net
No net tubular - There is net
same size. This is because, the negative charges of tubular
secretion or tubular
basement membrane & podocytes restrict and repel secretion of
reabsorption reabsorption of
large negatively charged and attract positively charged substance
Eg Inulin substance
molecules^. This explains, only 0.2% filtrability of
Eg PAH
negatively charged albumin with an effective diameter of - Eg Glucose
~ 7 nm.
The tubular fluid is hyper tonic in descending limb and
Inulin (a polymer of fructose found in dahalia tubers, hypotonic in ascending limb o f loop o f Henel® because
MW 5200) that is freely filtered, neither reabsorbed nor - Thin descending lim b is highly permeable to water, so water
secreted in tubules, and creatinine (that is freely filtered, is reabsorbed (moved out) making tubular fluid hypertonic.
secreted & reabsorbed in some amount) are used to - Thin ascending lim b is permeable to Na* & Cb which is moved
measure glomerular filtration rate (GFR). out of tubular lumen making tubular fluid hypotonic.

___________________________ Transport Maximum (Tm)_______________^ _________


For most substance that are actively reabsorbed or secreted there is a maximum rate a t which the solute can be transported®
after k/a transport maximum. This limit is d/t saturation of specific transport system.
Tm fo r kidney in practice is less than calculated value (i.e.,threshold o f substance is less than its Tm) because, not all nephrons
have the sam e Tm and som e o f the nephron excrete substance before others have reached their Tm®. It is important to know that
overall Tm of substance for kidney is reached when all nephrons have reached their capacity to reabsorb or secrete that
substance.
346 ■ A Complete Review of Short Subjects

Tm for glucose in normal adult is -375 mg/min®, whereas its filtered load is only 125mg/min (GFR x Plasma glucose = 125
ml/min x 1 mg/ml). With large increase in plasma glucose &/or GFR, which increase the filtered load above 375 mg/min, the
excess glucose filtered is not reabsorbed & passes into urine. However, even when the plasma glucose concentration rises
above 200 mg/lOOml (2mg/ml) (so filtered load is 250 mg/min, which is less than Tm), a small amount of glucose appears in
urine. This is known as threshold of glucose. So renal threshold of substance is the plasma level at which it first appears in
urine in more than the normal minute volumes (amount).
Predicted renal threshold for glucose would be 300mg/dl (i.e. 375 mg/min Tmc divided by 125 ml/min GFR). However, the
actual renal threshold is about 200 mg/dl (of arterial plasma) or 180 mg/dL (of venous). This deviation between ideal & actual
curves is k/a splay. This (less than predicted) actual renal threshold is obtained as not all nephrons have identical Tm.

I I 1
Tm for substanes that are Tm for substances that are Substances that do not exhibit Tm

Substances that are passively reabsorbed do not


GlucoseQ 375 mg/min P a ra -a m in o 8 0 m g / m in
h ip p u r ic acid demonstrate Tm, because their rate is determined by
Lactate 75 mg/min
electrochemical gradient, permeability, & time that solute
C r e a tin in e 16 m g / m in
Plasma 30 mg/min remains within tubule eg. are urea, chloride®, water
protein Some actively transported substance also have
electrochemical gradient- time transport, e.g. is Na
UrateQ 15 mg/min
reabsorp tion in PCT®. However in DCT (where
Amino 1.5 mM/min
epithelium has much tighter junction & so transporting
acids®
only small amounts of Na+) sodium reabsorption exhibits
Phosphate® 0.10 mM/min
Tm.
Sulfate® 0.06 mM/min

Proximal Tubular Reabsorption

• PCT epithelial cells favour Changes in Concentrations


reabsorption because of 1 0 0 .0 -
The total solute concentration (i.e., osmolarity)
- large number of mitochondria 5 0 .0 -
remains same all along PCT, but
PAH
supporting active transport.
2 0 .0 - - Concentrations of few organic solutes such
- extensive brush border on luminal side o 10 0 -
as glucose, amino acids, and bicarbonate decrease
/
& extensive labyrinth of intercellular ra
/ oS markedly along the length of PCT because
c 5 0 -
& basal channels providing 8
c r j k t> Cl these are more avidly reabsorbed than
extensive surface area °
CD
2 .0 -
water.
E
\ \
CD 1 0 "
- loaded with protein carrier molecules Q. \ \
and N A ^
- Concentration of Na* remains relatively
i» N
1 0 .5 0 -
• In first half of PCT, sodium is \\o ''
constant, (despite of its high reabsorption)
reabsorbed by cotransport along with 1
.0
0 .2 0 • throughout PCT because water permeability
3 W'co
f e ' '
keeps pace with high Na+ reabsorption.
glucose, amino acid, bicarbonate & other 0 .1 0 -

organic ions, leaving behind a 3 \\\T3 - Concentration of K+, & Cl" also remain
0 .0 5 -
|\VS-
solution that has a higher «\v% constant
0 .0 2 -
oo-\w
v/
concentration of CL ^ \\y
P r o x im a l Loop of C o lle c t io n ! - Solutes that are less permeant & also not
• But in second half of PCT, little tu b u le h e n le lu b u le
actively reabsorbed such as PAH > creatinine
glucose & amino acid remains to be > Inulin > urea increase in concentration
absorbed. So Na* is now reabsorbed along PCT.
with C l ions.
Physiology: Excretory System ■ 347

Reabsorption of Water

Obligatory Reabsorption Role of Vasopressin (ADH)

Absorption of water in proxim al ADH or vasopressin is key regu lator o f w ater ou tput ®. V asopressin acts on the
tubules secon dary to N a+ collecting ducts and enhances their perm eability to water. W hen vasopressin is
reabsorption® absent the collecting duct epithelium is relatively im perm eable to water and only 2%
Absorption of water in renal tubules of w ater is reabsorbed. H ow ever by the time water reaches collecting duct, a
independent of plasma osmolality m axim um of 60-70% has a lread y been re- a bsorb ed in the proxim al tubules.
(irrespective of water balance)
Feature % of water reabsorbed in % of water reabsorbed in
Filtered water is reabsorbed
absence o f vasopressin presence of vasopressin
isoosmotically & is in depen den t o f
Proxim al tubule 60-70%® 60-70%®
A D H levels.
Loop of Henle 15% 15%
Distal tubule 5% 5%
Facultative Reabsorption Collecting duct 2% 10%®
Thus the g rea test fra ctio n o f filter ed w ater (60-70%) is reabsorb ed in p rox im al tubule
- Absorption of water in distal tubules
an d collectin g duct in presence o f irrespective o f presence or absen ce o f vasopressin® .
ADH® Alteration in Water M etabolism by Vasopressin
- Absorption of water depends on
variation in plasm a osm olality
GFR % of filtered Urine Urine Gain /Loss of
- Norm ally DCT and collecting ducts
(ml/min) water volume concentration water in
are not perm eable to water but in
reabsorbed (L/d) (mosm /kg H 2O) excess of
presence of ADH, these segm ents solute (L/d)
becom e permeable. Urine 125 98.7 2.4 290
isotonic to
plasma
★ T hick ascending lim b o f H enle &
early d ista l tubule are both Vasopressin 125 99.7 0.5 1400® (alm ost 5 1.9 gain
im perm eable to w a ter ® & no present time of plasma)
reabsorption occurs irrespective of No 125 87.1 23.3 30 20.9 loss
w ater balance. vasopressin
(Diabetes
insipidus)

Sodium Reabsorption Bicarbonate (H C 03) Reabsorption

Percentage Site Segment % HCO3 reabsorbed % Na+


60% P roxim al tubule® (by N a+ - H + exchange) reabsorbed

30% Thick ascending limb (by N a+ - 2C1+ - K + Proximal tubule 80-90%® 60%®
cotransport) Loop of Henle (thick) 10% 30%
7% Distal convoluted tubule (by Na - - Cb Distal tubule & Rem ainder 10%
cotransport) collecting duct.
3% C ollecting duct, (via ENaC channels)
348 ■ A Complete Review of Short Subjects

Permeability and Transport In Various Segments of Nephron

Segments Permeability Active Osmolality • At the end o f proxim al tubule 60-70% o f filtered
H2O Urea NaCl transport solute is reabsorbed & also 60-70% o f filtered w ater
of Na+ is reabsorbed®. Thus in the proximal tubule, water
• Proximal tubule P e rm e a b le P e rm e a b le P e rm e a b le P re se n t I s o s m o t ic ® moves passively out of the tubule, along osmotic
• Loop of Henle gradients set up by active transport of solutes and
- Thin descending 4+ + + 0 H y p e rto n ic ®
Iso tonicity is maintained®.
limb • Intratubular content in DCT is Hypotonic® This is
- Thin ascending 0 because of two factors:
+ 4+ 0 H y p o to n ic
limb a) The distal tubule, particularly its first part, is
- Thick ascending 0 relatively impermeable to water.
± + 4+ H y p o to n ic
limb b) The ascending limb of loop of Henle and the
• Distal + + + 3+ H y p o to n ic ® DCT have a strong & active system of
convoluted reabsorption of solutes. Active transport of Na+,
tubule co- transport of K+ /Cb occurs out of thick
• Collecting ascending limb, thereby diluting the contents of
tubule DCT.
- Corticle tubule 3+* 0 + 2+ Iso to n ic Thus while most of the solutes are reabsorbed, water
- Outer medullary 3+* 0 ± 1+
remains behind, causing dilution of tubular fluid.
portion • The segments of nephron that are impermeable
- Inner medullary to water: Thin ascending limb®, Thick ascending
3+* 3+ ± 1+ H y p erto n ic
portion limb®, 1st part o f distal tubule (DCT)®__________
* Indicates presence o f vasopressin®.

Sodium Dependent Glucose Transporter (Symport)

- Since the intracellular Na+ concentration is low in intestinal & renal cells, Na+ moves into the cells along its concentration
gradient.
- Glucose moves with the Na* & released into the cell.
- The Na+ is then transported to lateral intercellular spaces & the glucose is transported by GLUT-2 into the interstitium &
then into capillaries.
- This is an example of secondary active transport i.e. the energy for glucose transport is provided by the active transport of Na* out
of cells.

Na* Transport

Sodium dependent glucose transporter Transport proteins involved in the movement of Na+ & Cl- across
(symport) in intestine renal tubular cells
I I
Site Apical Transporter Fuction
P ro x im a l N a + / g lu c o se C T N a+ u p ta k e , g lu co se u p ta k e
T u b u le N a+/Pi C T N a+ u p ta k e , Pi u p ta k e
N a+ a m in o a c id C T N a+ u p ta k e , a m in o acid u p ta k e
N a + / la c ta te C T N a+ u p ta k e , la c ta te u p ta k e
N a+/ H + exch ang er N a+ u p ta k e , H e x tru sio n
C l-/ b a se e x c h a n g e r C l- u p ta k e
T h ic k N a+ -K + -2C 1- N a+ u p ta k e , C l- u p ta k e , K+
a scen d in g u p ta k e
lim b N a+ / H + exch ang er N a+ u p ta k e , H + e x tru sio n
K + c h a n n e ls K + e x tru sio n (recy clin g )
D istal N aC l CT N a+ u p ta k e , C l- u p ta k e
c o n v o lu te d
tu b u le
C o lle c tin g N a+ c h a n n e l (E N ac) N a+ u p tak e
d u ct
Physiology: Excretory System ■ 349

Also know:
- Glucose mechanism also transport glactose®.
- Fructose utilizes different mechanism, it is
transported by fa cilita ted diffusion®.
- Glucose is incorporated in ORS to facilitate
Na* absorption®.

Mechanism of Na+ absorption in the Proximal Tubule

Loop of Henle

Thin descending lim b Thin ascending lim b Thick ascending lim b


•This segment is highly perm eable to water.® Fluid becomes more dilute in In this segment, a carrier
•As this segment passes through inner medullary this segment because of co- transports one Na*, one
interstitium (which is increasingly hypertonic) movement of Na* and CT out of K* and 2 Cl® from the
more water is reabsorbed m aking tubular flu id tubular lumen. tubular lumen into the
hypertonic® tubular cells.
This is an example of
In terstitial Tubular
fluid R en al tubu le cell lum en secondary active transport
; the Na* is actively
transported® from the cells
N a '^ - p T -
into interstitium by Na* -
'• = V r K *
“ K
+ 7 -------- N a ’ K* AT pase in the
c\‘ + . __ - — 2? '
Barttm ■4— K*
basolateral membrane,
-L _ keeping intracellular Na+
ROMK ROMK

K "*-_____r > K '


low.
The K* diffuses back® into
tubular lumen via ROMK.
NaCl transport in the thick ascending limb of the loop of Henle. The Na+ - K+-2C1- The CT moves into
cotransporter moves these ions into the tubular cells by seconday active transport. Na+ interstitium® via CIC - Kb
is transported out of the cell into the interstitium by Na+-K+ ATPase in the basolateral channels®. (Barttin)
membrane of the cell. Cl- exits in basolateral CICKb Cl- channels. Bartin, a protein in
the cell membrane, is essential for normal CIC-Kb function. K+ moves from the cell to

Tubulo Glomerular Feedback Macula Densa

It is a feedback from the renal tubules, to It is specialized group o f epithelial cells in the distal tubule® tht
the glomerulus to regulate the GFR, in an comes in contact with the efferent and particularly afferent
attempt to ensure constant sodium arteriole®.
chloride delivery to the distal tubule. Macula densa cells contain golgi apparatus which are intra cellular
The sensor for this response is macula secretory organelle directed towards the arterioles.
densa It is the short segment of renal tubule at the end of thick ascending limb
The m acula densa senses the changes in & continues into distal tubule. It is located at the joint where afferent
sodium chloride concentration. arteriole enters glomerulus and efferent arteriole leaves, and thus at
The Na+ and Cl- enter the macula densa this location the tubule is in contact with both the afferent and
cells via the Na-i- - K+ - 2C1- cotransporter. efferent arterioles (as depicted in figure)
Juxta glomerular apparatus consists of macula densa (in DCT) & JG
cells in afferent & efferent arterioles
350 ■ A Complete Review of Short Subjects

4 Arterial pressure

I
4 Glom erular hydrostatic pressure 4 "

Glom erular
epithelium 4 GFR
TProximal I
NaCl —
4 Macula densa NaCl
reabsorption
juxtaglom erular
'cells
Effere Afferent
arteriol T Renin
arteriole
4
T Angiotensin II
. { Macula ^ ,, Internal
densa
O' elastic 4
— T Efferent arteriolar 4 Afferent arteriolar —
Smooth ■ lamina
m uscle Qjstal Basem ent resistance resistance
fiber tuble m em brane M acu la densa fe ed b ack m echan ism fo r autoregulation o f glom eru lar
hydrostatic pressure and glom eru lar titration rate (G F R )

Reabsorption / Secretion

______________ Na+_____________ K+ ___________Glucose__________ Aminoacids


• Na* resorption occurs in all • Much of the filtered K + is • Glucose and bicarbonate are • Am ino acid
segments o f nephron excepting reabsorbed in PCT®. reabsorbed along with Na* reabsorption is
descending limb o f loop o f • Som e K * is secreted in thin in the early portion o f most marked in
Henle®. descending limb of LH. proxim al tubule®. the early
• Na+ is actively transported into • Again K* is reabsorbed by thick • Glucose and N a+ bind to the portion of PCT.
interstitium by Na+ K+ ATPase ascending limb of LH by N a+- K + common carrier SGLT - 2 in • The main
• Reabsorption - 2CL cotransporter. the luminal membrane and carrier in the
- PCT - 60% (Major portion)® • In Distal tubule K* is secreted glucose is carried into the luminal
- Thick ascending limb LH -30% [In exchange of each Na+ cell as Na+ m oves down its m embrane co -
- DCT-7%® reabsorbed either one K + or H + electro chem ical gradient transport Na*
- CT-3% are secreted]

Acidification of Urine
Removal of H + ions (generated by m etabolism ) is done by renal tubules, therefore kidney failure produces renal or metabolic
acidosis. H + ions are extruded by renal tubules by H* ion secretion or proton secretion, this occurs in two sites:

Proximal tubule (PT) Collecting duct


In both PT & distal tubule within the lining cells of tubules, CO 2 & H 2O in presence of carbonic anhydrase®, produce H CO2 3
which splits into H+ & H C O 3
• In PT, H + ion is secreted into the lum en on exchange • In CD, H+ ion is secreted actively by the help of proton pump.
• of one N a+ by help of Na* H* exchanger. • Within the lumen of CD, H+ ion react with N a 2 H P O 4 (alkaline
• The H+ ions within the lumen are utilized to produce sodium phosphate, because virtually all HCCV are reabsorbed
• H 2 C O 3 which subsequently breaks down into H 2 O in PT) & N aH 2 P O 4 (acidic sodium phosphate) is produced
& C O 2 . So very little free H* ions appear. and one N a+ is released which is reabsorbed. Acid sodium
Therefore the pH o f the fluid in PT does not fallQ. phosphate m akes the urine acid®.
• H+ concentration can be Ted as much as 900 folds in CD
Physiology: Excretory System ■ 351

Syndrome of Inappropriate ADH Secretion (SIADH) Concentration & Dilution of Urine

Syndrome of inappropriate ADH secretion (SIADH) is When the urine is concentrated, solute are excreted in
characterized by relatively excessive ADH levels or plasma ADH excess of water. Conversely, when the urine is dilute,
levels that are elevated above what would be expected on the basis water is excreted in excess of solutes. Renal clearance is
of body fluid osmolality, blood volume and blood pressure - the rate at which plasma is' cleared of solutes (osmotic
hence named inappropriate ADH secretion. Patients with SIADH clearance) & water (free water clearance).
retain water and their body fluids (eg serum) becomes
progressively hypoosmotic. In other words, decreased excretion of
excess water results in reduced (low) plasma osmolality (ie Osmolar Clearance Free Water
hypoosmotic body fluids), hyponatremia (reduced plasma Na+) (Cosm) Clearance (CH2O)
but the urine is hyper-osmotic (more concentrated) and contain It is the volume of plasma cleared • It is the volume
increased urinary Na+ than would be expected on the basis of low of solutes each minute i.e., the rate of plasma cleared
body fluid osmolality. at which plasma is cleared of of solute free
SIADH is characterized by low plasma osmolality, hyponatremia solute. water each
(decreased plasma /serum Na*), inappropriately high urine It is calculated by________________ minute (per unit
osmolality and significant amounts o f sodium in urine (Turinary _ R ate o f o sm o la r c le a r a n c e time).
Na*) "sm P lasm a osm alarity {P„„„)
• Rate of free
In SIADH, restricted wate intake is necessary to prevent cellular
water clearance
Urine osmolarity(UnJm)x Urine flow rale (V)
overhydration. D eclom ycin, an agent causing reversible nephrogenic
represents the
diabetes insipidus (ie ADH deficiency 1/t dehydration, polyduria &
polydipsia), may also be useful. ★ Diluting / concentrating ability of rate at which
kidney may be impaired d/t solute free
Bartter & Schwartz clinical criteria for diagnosis of SIADH include
- inappropriate ADH secretion water is
1. SIADH is most common cause of euvolemic-hypoosmolality.
Euvolemia is defined clinically on the basis of absence of - inability of distal tubule, collecting excreted by
tubule, and collecting ducts to kidney.
signs of hypovolemia (ie tachycardia, orthostatic or postural
hypotension decreased skin turgor & dry mucous membranes respond to ADH (nephrogenic • It is calculated
) or hypervolemia (generalized edema, ascites etc). Other diabetes insipidus) and / or as the difference
causes of euvolemic hypoosmolality such as hypothyroidism, - impairment of counter - current between water
hypocortisolism (pituitary ACTH deficiency or Addison's mechanism, which depends on excretion (urine
hyper osmotic medullary flow rate) and
disease) and diuretics must be excluded.
interstitium. osmolar
2. Hypoosmolality (decreased effective osmolality) of ECF or
serum (<275 mOsm/ kg H2O) and hyponatremia (reduced ★ Furosamide, like other loop clearance
diuretics, act by inhibiting Na* -
plasma Na*); however, pseudo hyponatremia or hyper
glycemia must be excluded. K+ -2C1' symporter in thick
ascending loop of Henle. It C h 20 = V - C„sm
3. Inappropriately high urine osmolality (concentration) ie >
abolishes the corticomedullary C||20= y _ (U XV)
' -"I”----'
100 mOsm/kg H 2O. It does not mean that urine osmolality is
greater than that of plasma. It only means that the urine is less osmotic gradient and blocks (p. )

than maximally diluted. And urine osmolality is not negative as well as positive free • It can be
necessarily elevated inappropriately at all levels of plasma water clearance.
osmolality.
4. Elevated urinary sodium excretion (Turinary Na+) on a
normal salt & water intake. It is present in most patients but is 1 1
Positive Zero Negative
neither 100% diagnostic nor its absence rules out the
It means urine osmolarity < Kidney is Urine
diagnosis
plasma osmolarity indicating. producing osmolarity is
- Kidneys are forming dilute urine greater than
Nephrogenic Syndrome of Inappropriate Antidiuresis urine isosmotic plasma
- Water is being removed to plasma osmolarity
Nephrogenic syndrome of inappropriate antidiuresis (NSIA) occurs from plasma by kidneys in indicating
d / t gain of function (activating) mutations is V2recep tor gene resulting excess of solutes water
in its constitutive activation even in the absence of ADH. It present with - Water free of solutes (= free conservation
same laboratory findings as those of SIADH including hypoosmolar water) is being lost from
serum, hyponatremia (iserum Na+), hyperosmolar urine and Turine body
Na*. But unlike SIADH (where high ADH levels are responsible for - Plasma is being
water retention by kidneys) patients with NSIA have undertrevels concentrated
levels of ADH in their plasma.
352 ■ A Complete Review of Short Subjects

Differential Diagnosis of ECF Hyponatremia Actions of ADH (on Kidney)


Primary action of ADH on kidneys is to increase the
Findings Type I Type II Type Type IIB permeability of collecting duct to water and to
Hypervolemic Hypervolemic IIIA- Euvolemic increase permeability of medullary portion of
Euvolemic (SIADH) collecting duct to urea. It stimulates reabsorption of
History / CHF, Salt & Water ACTH- Infections NaCl by thick ascending limb of Henle's loop, distal
Cause Cirrhosis, loss cortisol & tubule Si collecting duct
Nephrosis deficiency neoplasms ADH exerts its antidiuretic effect by binding &
, Nause & of brain, activating V2 (vasopressin 2) receptors on
vomiting antitumor basolateral membrane of principal cell of late distal
drugs, tubule & collecting duct. V2 receptor is coupled to
pulmonary adenyl cyclase via stimulatory G protein (Gs). So
diseases & ADH binding to V2 receptor increases intracellular c-
carcinoma AMP —>activates protein kinase A (PKA) ultimately
of lung resulting in insertion of vesicles containing
(esp small aquaparin 2 (AQP 2) water channels into the apical
cell type) membrane of cell, as well as synthesis of more AQ
Hypervolemia Present Absent Absent Absent P2. With the removal of ADH, AQP2 water channels
(edema, are reinternalized into the cell and apical membrane
ascitus) again becomes impermeable to water. This shuttling
Hypovolemia + + + Absent (translocation) of water channels into Si out of apical
(postural membrane controls its permeability to water. Because
hypotension) the basolateral membrane is freely permeable to water (d/t
BUN, High- normal Low-normal AQP3 & 4 channels), any water that enters the cell
Creatinine, through apical membrane thereby results in net
Uric acid absorption of water from tubule lumen.
Serum K+ Low -normal Normal ADH exerts its antidiuretic effect by activating V 2
Plasma Renin High Low receptors and involves rapid translocation of
activity aquaporins (protein water channels) from endosomes
Urinary Na+ Low High® (where they are stored inside the cell) to luminal
Serum Low normal membranes of principle cells of collecting tubules and
High normal Normal
albumin duct. It is important to mention that Via receptors
mediate vasoconstriction by ADH. And V ib ( V 3)
Serum Normal - high Low Normal
receptors are unique to pituitary, where they mediate
cortisol
increased ACTH secretion from corticotropes.
V I receptor is present on vascular smooth muscle
and mediates vasoconstrictor response to ADH
(accounting for its vasopressin name).
Potassium (K*) Hyperkalemia In Metabolic Acidosis
x:
1 In acidosis H+ concentration is increased in
K+ concentration Factors affecting K + concentration
plasma (4, PH)
r
Potassium is the main Shift K+ into Shift K+ out of I----------------------- 1------------------------1
This reduces To maintain Na+ is
intracellular cation®. cells cells
A bout 98% o f to ta l body the activity of pH H+ are reabsorbed in
(D ecrease (Increase
potassiu m is contained inside® extracellu lar extracellu lar Na+ K+ pumped inside distal tubules
the cell (150mg) & only 2% in K*)® K+)® ATP pump the cells & to in exchange
the extracellular fluid. • Insulin® • Insulin I maintain with H+
This ECF/ ICF gradient of • Aldosterone® Increased electroneutralit (Generally Na+
deficiency (DM)
extracellular K+ y is reabsorbed
potassium is responsible for • fi-adrenergic • Aldosterone
(Hyperkalemia intracellular and K+ is
neuro- muscular function and stim ulation® deficiency
• A lkalosis® ) K+ is pumped secreted)
made possible by activity of • Acidosis®
• D iarrhoiea® out. 1
N a+ K* pum p on the cell • fi- blocker®
membrane®. Mn: "In A J. K + excretion is
• Tissue injury®
BAD " Increased decreased®
A bout 3/4 th (109mg) o f to ta l (cell lysis)
extracellu lar (Hyperkalemia
body potassiu m (150mg) is • Strenuous
K- )
present in sk eleta l muscle)® excercise®
Physiology: Excretory System ■ 353

Buffer in Blood Principal Buffers in Body Fluids

1) Plasma proteins
Blood H 2C O 3 H* +HCO3 -Q
In the blood, proteinsQare effective buffers because both
H Prot H *+ P ro f© Q
their free carboxyl and amino groups dissociate
HHb ^----- - H ++ HbQ
2) Hemoglobin
Interstitial fluid H 2C O 3 H*+ H C O 3 -
Imidazole group of histidine residue in hem og lobin Q
Intracellular fluid Hprot v N H ++ Prot ©
provides an im portant buffer system
H 2P 0 4 H ++ H P0 4 2-Q
3) Carbonic acid - bicarbonate system
Handerson - H asselbach equation for this system is ★ Phosphate buffer is m ainly present in intracellular
pH = pK + log [H C 03 ] fluid Q. Phosphate concentration is too low in plasma
h 2c o 3 for this system to be a quantitatively im portant buffer.

Siggard - Anderson Curve Nomogram

Siggard - Anderson curve 110


nomogram is used to plot 100 40 4^ ^ 0 65
the acid -b ase 90 q 10 \ 2 5 ^ V 7 0
characteristic of arterial 80 r \
blood and is helpful in \ oua / H e m o g lo b in ^
.80
70
clinical situation. PCO 2is \y ' (g /d L ) ^ " ^ B u ffe r b a s e

plotted on the vertical axis 60 25 ^ \ C 0 2 titra tio n lin e ( m e q /L )

and pH on the horizontal \ ^ o f n o rm a l b lo o d

axis. 50 \ \
Vertical line through pH \ \ S ta n d a r d b ic a r b o n a te
\ ( m e q /L )
7.40: o) 40
I 10 1* 2 Q < ^ \2 5 3 0 ^ v ^ 4 0 50
- Any point to left of this E 35 C 0 2 titra tio n lin e \ ** -5 \ +10 '
E s o lu tio n c o n ta in in g +15
line indicates acidosis 30 N aH C 0 3, 1 5 m e q /L , X
- Any point to right a n d n o b u ffe rs + 20 "
25 B ase excess
indicates alkalosis
( m e q /L )
Horizontal line through 17.
20
PCO 2 of 40 mmHg:
Position of point above or
below this horizontal line 15
defines the effective
degree of hypoventilation
or hyperventilation.
10 I 1 I I I I I I 1 I I I I I I I 1 1 I I I I L I I I I I I I I I I I I I I I I I I I I I I I I I I I l 1 I I 1I I I I I I H I I I 1 I I I I I I I I I LI I Ll I I H I I I 1 I I

6.9 7.0 7.1 7.2 7.3 7.4 7.5 7.6 7.7 7.8
pH

Siggard-Anderson curve nomogram


354 ■ A Complete Review of Short Subjects

__________________________ Renin Angiotensin System

A fall in BP is sensed by intra Angiotensinogen /Renin substrate ACE is dipeptidyl carboxy


renal baroreceptor Glycoprotein with 453 amino ac (aa) peptidase that split off
(juxtaglomerular /JG) cells in Synthesized in liver0 with a 32 aa sequence that is histidyl- leucine bond
afferent arterioles of renal removed in ER. from angiotensin I forming
glomeruli Ted by angiotensin II, cytokines, estrogen, thyroid angiotensin II
JG cells release a glycol protein hormone& glucocorticoids It inactivates bradykinin.
hormone renin, with enzyme So increased tissue
function (mol wt 37, 326) Renin Renal bradykinin produced by
Renin is an aspartyl (acid) Leucine- valine bond ACE inhibition acts on (32
protease. The molecule is made ■1 receptor to produce the
up of 2 lobes (domains) between Angiotensin I cough that is an annoying
which the active site - Decapeptide i.e. contain 10 aa. side effect
(containing 2 aspartic acid - No established action (i.e. physiologically ACE is located in
residues at positions 104 & 292) inactive endothelial cells of
is located in deep cleft. - Aminopeptiase act on it to produce (des- Asp) pulmonary (lung)
Preprorenin (406 aa) is angiotensinl which can be converted directly to capillaries (& partly in
converted to prorenin (383aa) angiotensin III by action of ACE kidneys and other organs).
to active renin (340 aa) after It is an ectoenzyme
removal of 23 & 43 aa residues Pulmonary Angiotensin A existing in 2 forms:
from aminoterminal endothelium Converting enzyme ' r somatic (found throughout
respectively. (ACE) the body) & germinal
Prorenin is inactive & is Histidyl-leucine (found solely in
secreted by other organs eg bond spermatozoa & postmeiotic
ovary. Rennin splits Leu-Val spermatogonic cells)
bond & has helf life of < 80 min. Angiotensin II/ Angiotonin /Hypertensin Both forms have single
After nephrectomy the - Octapeptide i.e. contain 8 aa transmembrne domain & a
prorenin level in circulation is - Metabolized very rapidly & half life is 1-2 min short cytoplasmic tail.
only moderately reduced or -Metabolized by RBC & other tissues However somatic ACE is
may actually rise, but the active - Removed from circulation by trapping mechanism 170 KDa protein with 2
rennin level falls to essentially in vascular beds of tissue other than lungs________ homologous extracellular
zero. Which means, very little A n g io te n s in a s e doman & catalytic sites
prorenin, is converted to rennin (A m in o p e p tid a se rem ov in g (containing Zn++)
in circulation & active rennin a sp a rtic a c id resid u e from
Germinal ACE is 90 kDa
a m in o term in al b y sp littin g
is product primarily, if not protein with 1 extra
A S P - A rg b o n d )
exclusively of kidneys cellular domain & active
Angiotensin III
Plasma rennin activity (PRA) site (Zn++)
- Hepta peptide i.e. contain 7 aa
measured by immunoassaying Both forms are formed
- Physiologically active has 40% of pressor & 100%
the amount of angiotensin I from a single gene.
of aldosterone stimulating activity of A II________
generated, may be low in Knocking out ACE gene
deficiency of angio tensinogen b A m in o p e p tia se (sp littin g lowers BP & fertility in
rennin. To avoid this, A rg -V a l b o n d & re m o v in g
males but not in females.
exogenous angiotensinogen is A r g in in e fro m am in o
te rm in al)__________
added & plasma rennin
Angiotensin IV
concentration (PRC) rather
- Hexapeptide i.e. contain 6 aa
than PRA is measured.
Has some activity
Physiology: Excretory System ■ 355

Actions of Angiotensin II

I I I
B lood V essles (A T ia B rain (A T1a receptors) A drenal Cortex • D irect effect on PCT
receptors) I (A T ib receptors) to increase Na+
I It does not penetrate blood brain barrier, but I reabsorption by
Potent arteriolar (Vaso) Increase secretion increasing Na+ - H+
triggers responses by acting on circum
constrictorQ causing ventricular organs (out side BBB) o f aldosterone 0 exchange
which • C ontraction of
Rise in systolic & • Area postrema is prim arily responsible for
- Prom otes Na+ & m esangial cells with
diastolic BP pressure potentiation. Angiotensin II acts
w ater resultant decrease in
Rise in total peripheral here to decrease the sensitivity of baro reflex
reabsorption Q GFRQ
resistance • D ip so g en ic effect (ie. Increase thirst or
- Expands ECF • D irect action on post
Its pressor activity is w ater in take) is d/t action of subfornical
(1/t TCO) gan g lio n ic
decreased in cirrhosis & organ (SFO) & organum vasculosum of
sym pathetic
in Na+ depleted lamina terminalis (OVLT). - Facilitates K +
neurons facilitates
individuals. Because • Angiotensin sensitive thirst centre is in excretion
release of
circulating angiotensiin anterior hypothalus (periventricular optic norepinephrine
II is increased in those recess) w hereas osm om eteric thirst centre
conditions, whch down is in m edian anterior hypothalamus.
regulates angiotension • T A ntid iuretic horm one (ADH) or
receptors in vscular vasopressin) from posterior pituitary
sm ooth muscle
• T a c th
Chapter 7. ENDOCRINE SYSTEM: REVIEW NOTES

Receptors and Second Messengers


Receptors Receptors with Intrinsic Ion Channels
Group II: Cell membrane Group I (Lipophilic) These cell surface Receptor Acts Through
> Membrane receptors are Hormones that bind to receptors enclose ion - Muscarinic - G- protein
specific to protein, Intracellular Receptors selective channels receptor, opioid coupled
peptides & I (Na+, K+, Ca2+, Ch) and GABA b receptor
catecholamines - Hormone diffuses through within their molecule. - M l, M3 - IP3-DAG
' Involved in molecular plasma membrane of all cells Agonist binding opens (muscarinic) - CAMP
transportQ e.g. large but only encounter their the channels & causes - M2 muscarinic, - TK+, Ca+4
molecule by endocytosis specific, high affinity depolarization /
Opioid |i ,8
or exocytosis. intracellular receptors in target hyperpolarization
- GABA b
> Binding of molecule to cells. depending on the ion
receptor gives specificity - Steroid receptor superfamily: that flows through.
to process.® are soluble DNA binding This includes:
> Involved in transduction proteins that regulate the - Nicotinic cholinergic
of signal (after binding of trancription of specific genes.
receptor®
ligand with receptors) that - Glutamate receptor
Hormone receptor complex first
influences intracellular - Glycine receptor
undergoes an activation
process. reaction and is the signal for
- GABAa receptor®
- Activation of - 5 HT 3 receptor
group I hormones
intracellular enzymes • ligand - receptor complex
- Alteration of directly provides the signal to
permeability of specific genes whose rate of
membrane result in transcription is there by
opening of ion channels. affected.

Hydrophobic (lipophilic) steroid hormones eg glucocorticoid diffuse across plasma membrane Thyroid hormones &
and encounter their cognate receptor in the cytoplasm of target cells. Hormone (ligand)- retinoic acid directly enter
receptor binding activates receptor by resulting in conformational change in the receptor leading the nucleus where their
to dissociation of heat shock protein (hsp) 90 (from receptor). This activated receptor hormone cognate (hetero dimeric)
complex moves (translocates) from cytoplasm to nucleus and the nuclear localization sequence receptors are already bound
of receptor binds with high affinity to a specific DNA sequence called hormone response to appropriate hormone
element (HRE). The DNA bound-hormone-receptor serves as a high affinity binding site for co­ response element (HRE).
activator proteins enhancing transcription. Example includes Steroidal hormones like However this DNA bound
- Glucocorticoids & receptor fails to activate
mineralocorticoids® transcription because it
- Androgens (testosterone), exists in a complex with a co
estrogen & progestins® repressor. The association of
- Retinoic acid (retinoids) ® hormone (ligand) with
- Vit D3 (1 ,25-(OH)2 D3) ® receptor results in
dissociation of the co­
repressor (s). The liganded
receptor is now capable of
binding co-activators
resulting in recruitment of
RNAP II & GTFs and
activation of gene
transcription.
Includes Thyroid (T3 & T4)
hormones®
Physiology: Endocrine System ■ 357

Classification of Hormones by Lipid/Water Solubility

Group I Hormones Feature Group 11 Hormones


Are lipophilic (i.e. lipid or steroid soluble)® Solubility Are hydrophilic (i.e. water soluble)
Being insoluble in water, they are transported in Transport Carrier protein is not required as they
blood bound to a plasma protein; their association (Carrier) protein themselves are water soluble
with carrier protein (commonly globulin)
circumvents the problem of solubility
They are not stored & secreted on demand (except Synthesis & They are synthesized in endoplasmic
thyroid hormones; thyroxine stores can last for storage reticulum (ER), like all proteins and stoed in
several weeks) granules or vesicles.
Prolonged (hours-days) d/t transport protein Plasma half life Short (minutes)
(ti/2)
In target cells, they easily penetrate lipophilic Site of action & Being lipid in soluble, they act on plasm a
plasma membrane to act intracellularly in cytosol Receptor membrane receptors®, and do not enter the cell
or nucleus to act.
Slower in onset and prolonged in duration. Their Action Rapid in onset and shorter in duration (bcoz
actions are terminated in liver. cell entery is not required)
Hormone -receptor complex undergoes activation Mediator or Intra Use intracellular second messengers like c-
reaction and moves into the nucleus to bind with cellular messenger AMP, c-GMP, IP3-DAG, calcium -calmodulin
high affinity to a specific DNA sequence k/a and kinases etc.
hormone response element (HRE).
All steroid hormones & thyroid (T3r T4) Examples - Catecholamines (adrenaline &
hormones® noradrenaline)
- Protein (glycoprotein) and poly peptide
Hormone HRE DNA sequence hormones which includes all hormones of
Glucocorticoids GRE GGTACA NNN TGTTCT
Mineralocorticoids MRE •<-------- --------► 1. Hypothalamus (except PIF)
Progestins PRE 2. Anterior & posterior pituitary
Androgens ARE 3. Pancreatic & parathyroid hormones
Estrogens ERE AGGTCA — TGA/TCCT 4. Gastro intestinal hormones
•<------- -------------► 5. Calcitonin (from thyroid,) renin®, &
Thyroid hormones TRE erythropoietin, (kidney), angiotensin, atrial
Retinoic acid (retinoids) RARE AGGTCA N3,4,5 AGGTCA natriuretic peptide/ANP (heart), leptin
Vitamin D VDRE ---------- ► ---------- ►
(from adipocyte)
cAMP* CRE TGACGYTCA
★ Not of group I

________ Adenyl Cyclase - Cyclic-AMP Pathway________ Toxin Mode of Action


Pertussis Toxin Inhibits adenyl cyclase (through
• 5 components are involved in the mechanism by which
Gi a subunit)
ligands bring about changes in the intracellular concentration
Heat stable Persistent activation of gaunylate
of C- AMP
enterotoxin (ST) by cyclase & elevation of
- Catalytic unit adenyl cyclase (a trans membrane protein
enterotoxigenic intracellular Cyclic -GMP
crossing membrane 12 times) which catalyzes the conversion
E.Coli
o f ATP to CAMP®
- Cholera Toxin ADP ribosylation (activation) of
- Stimulatory & Inhibitory receptors
- Heat labile Gsa subunit 1/t persistant
- Stimulatory (Gs) & Inhibitory (Gi) - G proteins that link the
enterotoxin (LT) chronic activation of adenyl
receptor to the catalytic unit
produced by cyclase -> elevated c-AMP ->
• Ligand binding to stimulatory receptor (Gs a)- subunit
enterotoxigenic activates PKA ->
stimulates; whereas binding to inhibitory receptor (Gi -a
strains of E.Coli Phosphorylation of CFTR &
subunit) inhibits adenyl cyclase.
Na+ - H+exchanger - 4- Na+
• 2nd messenger of this pathway is C- AMP (cyclic adenosine 3',
absorption & TCP secretion -»
5' - monophosphate) formed from ATP by action of enzyme
watery diarrhoea
adenyl cyclase & converted to physiological inactive form (51
AMP) by phosphodiesterase. Methyl xanthines such as
theophylline <&caffeine inhibit phosphodiesterases hence
358 ■ A Complete Review of Short Subjects

augm ent horm onal effect mediated by C-AMP. Cholera Toxin

C- AMP activ ates protein kin ase A (PKA) w hich


p h osp h ory lates protein, changing their con form ation &
altering their activ ity Q. In addition active catalytic subunit of
PKA moves to nucleus & phosphrylates C-A M P responsive
elem ent b in d in g protein (C REB). This transcription factor
then binds to DNA & alter transcription. Uses NAD’ as
donor

Stimulatory Adenylyl
receptor cyclase

ATP
Adenyl Cyclase
Chronic/Persistent
Activation Tc-AMP

4
Protein kinase A (PKA) activation
Cytoplasm ATP cA M P ► 5 'AMP
I
r— Phosphorylation of — n
I
Protein kinase A IC FT R I | Na*-H* Exchange |

I i
Phosphoproteins C l secretion TNa* Absorption

i I
Physiologic effects
Watery (Liquid) Diarrhea

Nuclear Receptor
• Steroid horm one receptors are monomeric phosphoproteins. Each receptor Group I lipophilic horm ones cross plasma
has 6 codomains (A to F). m embrane of all cells but encounter intracellular
- All have centrally located D N A - b in d in g dom ain (D B D ) or C- dom ain receptor in target cells.
containing 2 zinc fin ger binding m o tifs Q that allows receptor to bind Horm one (ligand) receptor com plex first
w ith high affinity to a DNA sequence called hormone response element undergoes an activation reaction in 2 ways
(HRE), either as homodimer, as heterodimer (retnoid X receptor, RXR or as
monomer. G lucocortitosteroid R e t in o id s th y r o id
- M ultifunctional ligand b in d in g dom ain (LBD ) in carboxy terminal half encounters their hormone go directly
(E domain) of receptor. It binds to horm on es (steroid), or m etabolitesQ receptor in cytoplasm into nucleus
and also m ediate binding of heat shock proteins (hsp), dimerization, nuclear 1 I
localization, and transactivation. Its binding with In this case the
- At carboxy (C) term inal, transcription activation function (AF 2 ) of F receptor results in receptor is already
domain interacts w ith coactivators and facilitate trans activation. dissociationof heat shock bound to HRE (here
- Hightly variable hinge region (domain D) seperates DBD from LBD and protein 90 (hsp 90) from thyroid hormone
provides flexibility to receptor to assume different DNA binding the receptor response elem ent =
conform ations. I TRE or retinoic acid
- Highly variable am ino (N) term inal contain another transactivation This activated receptor response elem ent =
domain - AFi (at domain A/B). It is also k/a modulator region as it is - ligand complex RARE) but fails to
involved in activating transcription. It is responsible for receptor isoforms moves into nucleus activate transcription
that share same DBD & LBD but exert different response because of and binds to hormone b/o presence o f co
association of various coregulators. response element (HRE), repressorG.
eg glucocorticoid / i
R e ce p to r S te ro id class R e tn o id X re ce p to r O rp h an
H e te ro d im e r H o^nocjim er steroid etc. response Binding of ligand
B in d in g H o m o d in ers
OO O □ elem ent depending on with this receptor
L ig a n d S tero id 9 -C is R e tin o ic acid + (X) N o t kn o w n ligand results in dissociation
D N A ele m e n t In v erted re p ea t D ire c t re p ea t D irect re p ea t I of co-repressor
-» «- -> -> -» -» DNA bound- ligand I
E x a m p le G lu c o c o rtic o id s , T h y ro id h o rm o n e , retin o ic C O U P - TF; TR- receptor complex Activated ligand
H o rm o n e m in e ra lo c o rtico id a cid , v ita m in D , & 2, G E N 8, H N F - serves as binding site receptor complex
re ce p to r for s, e stro g e n , m e ta b o lite lig a n d s eg. P P A R 4, T L X
for transcriptional co binds coactivators
a n d ro g e n , & a , fi & y, F x r , p x r /
p ro g e stin
regulatos (coactivators) resulting in activation
S X R , L X R , an d C A R that enhance gene of gene transcription.
transcription.__________
Physiology Endocrine System ■ 359

] [ JL x l _
A/B c D
N - AF-1 DBD Hinge LBD AF-2 -c
Am ino Term inus Central Domain Hinge Region Carboxy Term inus

Structure of N uclear (Androgen) Receptor

Nuclear Receptor

I
Amino (N) terminal Central (C) domain Hinge region (D domain) Carboxy (C) terminal
I I I I
Highly variable region Centrally located, most Highly variable regions - Ligand binding domain (LBD) also
responsible for conserved DNA binding Provides flexibility to k/a E-domain binds to horm ones or
receptor isoform domain (DBD) receptor to assum e m etabolites selectively & thus
Containing containing 2 zinc different DNA binding specifies a particular biological
transactivation domain motifs®. conform ation response.
AF-1 (at domain A/B) Binds DN A sequence - Transcription activation function
It is modulator region, called hormone (AF2) of F-domain interacts with
as is involved in response element coactivators & facilitate transactivation.
activating transcription (HRE)

G- Protein
G- Protein is nucleotide regulatory protein that bind GTP and translate a signal to a biological effect inside the cells.

Sm all G Protein Large Heterotrimeric G proteins


Involved in many cellular function. 1These couple cell surface receptors to catalytic unit, that catalyze the intracellular
They are related to product o f ras formation of second messengers or couple the receptor to ion channels.
proto-oncogene® 1Are made up of 3 subunits - a, fi, and y®. The a unit is bound to GDP and it separates
- Rab family : regulates rate of vesicle from Py subunit, when a ligand binds to G protein coupled receptor. The intrinsic
traffic betw een endoplasm ic GTPase activity o f a subunit converts GTP to GDPQ and this leads to reassociation of
reticulum , golgi apparatus, a with the Py subunit.
lysosom e, endosom es & cell ’ All the hetrotrimeric G-protein coupled receptors are proteins that span the cell
m em brane membrane seven times (serpentine receptors)®
- Rho /Rac fam ily : m ediates ■Can be divided into 5 families - Gs, Gi, Gt, Gq and Go
interaction b/w cytoskeleton & ■Cholera toxin stimulates binds Gs a subunit and adenyl cyclase; whereas pertusis toxin
cell m embrane bind Gi a subunit and inhibit adenyl cyclase.
- Ras family : regulates growth by ' There are three major effector pathway through which G- protein coupled receptor
transm itting signals from the cell
function:
m em brane to nucleus i) Adenyl cyclase : c-AMP pathw ay (Via Gs & Gi) ®
ii) P hospholipase C : IP 3 - DAG pathw ay (Via Gy)®
iii) Channel regulation:

G Protein Coupled (Linked) Receptor (GPCRs)


N-lerminal
These cell surface receptors bind to the ligand at the cell
surface®. And have 7 transmembrane segments that loop
in and out o f the cell membrane®. So the receptor is also
k/a seven transmembrane domain (7TM) receptors, S erpentine (7 pass)
7 m em brane
heptahelical or serpentine receptor. spanning re ceptor
Part of receptor that protudes into cytoplasm M yristolylated
(cytoplasm ic tail of receptor) is coupled to G protein that group

has 3 different subunits (hetrotrim eric) - a , P, y. This G


protein has an ability to bind guanosine nucleotides. No horm one (H ) - Inactive effector (E) Horm one (H) bound - A ctive effector
360 ■ A Complete Review of Short Subjects

- In resting (inactive) sta te the a , P, y subunits fo rm a Type/Class Effector (E) Stimulus (H) Effect
com plex th at binds GDP on a subunit®. of G protein
When receptor gets activated (after ligand binding) it
GS Gts Adenyl cyclaseT P-adrenergic & Glyconeogenesis,
causes this GDP bound trimeric G protein complex to Cardiac Ca++/Na+/Cr Glucagon glycogenolysis,
associate with cytoplasmic tail of receptor and to exchange channels? lipolysis
GDP by GTP. This exchange causes a subunit to
a-oit Adenyl cyclese T Odorant Olfaction
d issocia te from com bined f)& ysubunits. This sep arated
Gi a -i-1,2 ,3 K+ channels T a 2adrenergics
a subunit brings a b o u t b io lo g ic a l effect®. The P & y
Adenyl cycleasel & Decreased heart
subunits do not separate from each other and activates
acetylcholine rate
variety of effectors. (So dissociation of subunits 1/t
action). Ca++channels 1 m2
Cholinergics
- When ligand is removed from receptor, the intrinsic
G TPase activ ity o f a subunit in activ ates its e lf by do K+ channels T Endorphins, Neuronal
converting its bound GTP in to GDP®. opioids electrical activity
• In the absence of hormone, the heterotrimeric (a,p,y) G at C-GMP- Light Vision
protein complex is in an inactive GD P bound form phosphodiesterase T
anchored to plasma membrane through prenylated Gq Ctq Phospholipase C6piT Oi adrenergics 1 Muscle
groups on P y subunits and by myristoylated groups on & contraction & 1
a-subunit, but not associated with the receoptor. Ml BP
Binding of hormone (H) to receptor causes cholinergics
conformational change of receptor and activation of G an Phospholipase CP2 ? Oi adrenergics nI Muscle
protein complex d/1 exchange of GDP with GTP on a contraction &
subunit. This dissociates a-from p,y subunit so that a- IBP
subunit binds and activates the effector (E). E can be (all Gl2 an Cl- channel ? ?
are T)

- Tadenyl cyclase, Ca++/Na+/Cl- channels for a-s Ligands for GPCRs


- TK+ channels for cti and do
- T Phospholipase CPi for ctq and phospholipase Cp2 for a n Neurotransmitters Tachykinins Arachidonic Acid
- T C GM P phosphodiesterases for ott and CD channel for ctu - Adrenaline2 - N e u ro k in in A derivatives
• Depending on the coupling of ligand (hormone) to inhibitory G - Noradrenaline0- N e u ro p e p tid e K - T h r o m b o x a n e A 2
- Dopamine0 - S u b sta n ce P
protein(Gi) or stimulatory G protein (Gs) receptors, these can either
- 5 -H y d ro x y try p tam in e Other Ligands
decrease or increase the activity of intracellular enzyme, (so it is not Peptides - Endothelins
- H ista m in e
dependent on a subunit). - A c e ty lc h o lin e (A C H ) - A n g io te n sis II - Platelet
• Mechanism of action: Ligand (photon, protein, amine hormone, & - A d e n o s in e - A rg in in e activating factor
neurotransmitter) binding —» conformation change & activation of G - O p io id s v a so p re ssin - C a n n a b in o id s
- O x y to c in - O d o ra n ts
protein exchange of GDP for GTP and dissociation between a- Glycoprotein -
- V IP , P T H , G R P , - L ig h t
and P ysubunits which interact with effectors. Hormones
TRH - T a sta n ts
• a - subunit act on Na, Ca, K- channels and phospholipse - C (PLC). - LH , F S H , h C G
- TSH
P y subunit acts on adenyl cyclase, phospholipase A2, DLC, Ca -
ATPase etc. Catecholamines (adrenaline, noradrenaline &
- Adenyl cyclase - cAMP pathway activation (i.e. adenyl cyclase dopamine) act through G protein coupled receptors.
activation 1/t T cAMP) causes stimulatin of cAMP dependent a { adrenargic catecholamine use phospholipid C(IPa
prtein kinase (protein kin ase A) w hich p h osp h ory la tes the - D A G ) and a 2 & (3 adrenergic catecholamines use C-
receptor®. AM P (adenyl cyclase) second messenger system.
- G protein coupled receptor kinase are protein kinase that Hetrotrimeric GTP binding proteins (G- proteins) are
phosphorylates only active GPCRs cell membrane proteins that have seven
- Phosphorylation of receptor 1/t translocation & arrest linking. transm em brane segm ents th a t lo o p (pass) in & ou t o f
• GPCRs are found only in eukaryotes, including yeast, plants & cell membrane®, (serpentine protein)
choanoflagellates. These are grouped in 6 classes: A (rhodopsin Cytoplasmic tail is coupled to GTP that has 3 part
like); B (secretin receptor family); C (pheromone/metabotropic (trimeric) - a, P, ysubunits.
glutamate); D (Fungal mating pheromone receptor); E (cyclic AMP
receptor); and F (frizzled or smoothened).
Physiology Endocrine System ■ 361

Second Messenger

- Binding of l sl messenger (ligand) to cell surface receptors 1/1 a short lived increase (or decrease) in concentration of low
molecular weight intracellular signaling molecules which relay & amplify the strength of signal received. These are k/a second
massengers.
- Second messengers (concept proposed by E.W. Sutherland) are intracellular signaling molecules mediating intracellular
hormone/ ligand (1st messenger) function.
-Hormones or drugs require receptors for their action. Some receptors are directly attached to effector molecules and some receptors
are coupled to effector by second messengers. Second messenger are intermediary molecule that intervene between the original
message (Hormonal signal) and the ultimate effect on the cell (intracellular metabolic process).
-Hormone combines with receptor —> Receptor changes shape —>Combines with Gs protein —> Gs protein release GDP & binds
GTP —> Adenylcyclase activates —>ATP converted to c-AMP —> TcAMP (2nd messenger) —> Activates proteinkinase —>
Phosphorylates proteins —>Inactivate or activates enzyme.
-Thus, the only direct effect that a ligand / hormone / drug (1st messenger) has on cell is to activate a single type of membrane
receptor. The intracellular signaling molecule ie second messenger does the rest.
-Second messangers respond differentially to a hormone or drug because of receptors but cells respon d to secon d m essangers
d ifferen tially becau se o f d ifferen t en zym atic com position® . Cell respond differentially to second messenger because the enzyme present
in cells respond differentially to increase or decrease in cAMP.
-Second messengers like cAMP, cell membrane phospholipids (DAG) regulate activity of protein kinase enzyme (which
phosphorylates substrate protein), thereby modifying enzyme activity. The diversity of protein kinases present within a cell
provides for complex regulatory circuits which control numerous growth & metabolic processes.
-Types & examples of second messengers include

1) Cyclic neucleotides eg C- AMP and C- GM P ★ DAG & phosphatidylinositols are


2) Diacyl glycerol (DAG) & Inositol 1,4, 5- triphosphate (IP 3) hydrophobic (water insoluble),
3) Calcium & calmodulin whereas IP3, Ca++, cAMP & cGM P
4) Protein kinase such as PKA, PKC, and Ca++ - Calmodulin (CaM) - Kinase are hydrophilic (water soluble).
5) Gases eg NO (nitric oxide),CO & H 2S___________________________________

Classification of Hormone (Ligand = 1st Messenger) by Mechanism of Action

Bind to Intra­ Bind to Cell Surface Receptor and Act via Second Messengers
cellular
Receptors
Adenylyl Guanylate Cell M embrane Phospholipid Second Protein Tyrosine
• Nuclear cyclase cAMP Cyclase- M essenger System Kinase
Receptors: (2 nd messenger) cGM P (2nd • Hormones activate transmembrane receptors -Insulin, Insulin like
Thyroid system messenger) that activate phospholipase C. It catabolizes growth factors (IG F)-l
hormones ( T 3 &
System
ACTH, breakdown of cell membrane phospholipids & II
T 4)Q
Angiotensin II ■ANP (atrial esp PIP 2, (phosphatidyl inositol - EGF, PDGF, NGF, FGF,
• Cytoplasmic (epithelial cells), natriuretic M CSF
biphosphate) into 2 second messengers IP3
Receptors ADH peptide) (inositol triphosphate) & DAG (diacyl Calcium - Calmodulin
T hyroid Calcitonin, NO glycerol). IP3 mobilizes calcium ions from 2nd M essenger system
Hormone® Catecholamines ■Muscarinic mitochondria & ER which has its own 2nd
- Retinoic acid (Beta-recep tors), cholinergic messenger effects. Whereas DAG activates Normal calcium ion
■Androgen CRH agents & protein kinase C (PKC) enzyme which concentration in cells (1 0 8
Estrogen® Glucagon, FSH, oxytocin phosphorylates proteins. In addition lipid to lO'7 mol/L) when rises
•Progesterone HCG, LH (Tayler) portion of DAG is arachidonic acid (a to 10' 6 to 10 ' 5 mol/L
G lucocorticoid® PTH precursor of prostaglandin). because of entery of Ca++
M ineralo- (parathyroid into cells activate
• Hormones using phospholipase C: IP3 /
corticoid® hormone), TSH , calmodulin system. It
DAG or IP3- calcium & DAG - PKC 2nd
- Vitamin D 3 (1, Vasopressin (V2 activates myosin light
messengers include
25 (OH )2 D 3 receptor chain kinase to stimulate
- Acetyl choline, a l Adrenergic, Angiotensin
epithelial cells) II (vascular smooth muscle) contraction in smooth
Secretin, - Oxytocin Gn RH. GHRH. TRH, Vasopressin muscle & other cells.
Somatostatin (VI receptor vascular smooth muscle)
362 ■ A Complete Review of Short Subjects

★ The in sulin, ep id erm a l g r o w th fa c t o r (EG F) a n d IG F -1 recep to rs h a v e in trin sic p r o te in ty ro sin e k in a s e activ ities® located in
their cytoplasmic domains.
★ Role of cGMP in insulin: in su lin a c tio n in certa in tissu e m a y b e m e d ia te d throu gh cGM P® which activates protein kinase,
which in turn phosphorylates certain enzymes to modulate their activities. [Shinde Chatterjea 5th/P-200]

Calmodulin Calcium Calmodulin Complex


Calmodulin is the most widely distributed small calcium
binding protein. Activates Calmodulin - Regulate various microfilament
Calmodulin has 4 calcium binding sites and full dependent protein kinase mediated process in non
occupancy of these sites induces marked conformation contractile cells including
a. M y o sin e lig h t ch a in kinase®
changes that allow calmodulin to activate enzymes & ion - Cell motility
b. p h o s p h o r y la s e kin ase®
channels. - Cell conformation changes
c. Ca2+ / calmodulin kinase I
Calmodulin + Ca +2 —» activated calmodulin • - Mitosis
d. Ca2+ / calmodulin kinase II,
Activation of enzymes & ion channels. - Endocytosis
III

Parathyroid Hormone (PTH)


Regulation of Synthesis & Secretion Biological Effects
■Synthesis & secretion of parathyroid hormone PTH is a 84 a m in o a c id lin ea r p o ly p e p tid e horm one® . Bone and kidneys
(PTH) are regulated primarily by serum ionized are two main target for organs affected by PTH. PTH cell surface
calcium concentration sensed by ca lciu m sen sin g receptors are coupled to adenyl cyclase & c-AMP is the intracellular
re c ep to r (C aS R ) lo c a te d on c h ie f c e ll o f mediator of PTH action.
p a r a t h y r o id g la n d . When ionized serum Ca++
1. Bone: PTH acts directly on bone to increase bone resorption (T
levels decrease below 1.3mM (eg d/t prolonged
calcium & phosphate release in plasma).
low calcium diet), PTH synthesis and secretion
increases (to restore Ca++ levels). Whereas when - PTH has a rapid and a slow phase of bone resorption.
extracellular serum Ca++ increases above 1.3 mM, I. Rapid phase begins in minutes & increases progressively for several hours. It
PTH decreases & serum calcium diminishes. Even results from activation of already existing bone cells (mainly osteocytes &
with very high levels of serum calcium a low rate osteoblasts) to promote calcium & phosphate absorption (bone
of PTH synthesis & secretion persist. osteolysis) in 2 areas: (1) From bone matrix in vicinity of osteocytes and
■When hypocalcemia results from PTH deficiency, (2) From bone surface in vicinity of osteoblasts. Osteoblasts &
PTH levels are low, whereas in hypocalcemia d/t osteocytes have receptor proteins on cell membrane for binding PTH.
vitamin D deficiency, PTH levels are increased as a Long processes of osteocytes & osteoblasts form extensive osteocytic
compensatory response to hypocalcemia. In membrane system containing osteocytic membrane pumps that pumps
h y p e r c a lc e m ia due to malignancy & vitamin D calcium from bone fluid to ECF (when activated by PTH).
intoxication, PTH concentrations are low, whereas in II. Slower phase requires several days to weeks to become fully
primary hyper parathyroidism (adenomatous or developed. It results from proliferation of osteoclasts, followed by
hyperplastic parathyroid glands), PTH levels are greatly increased osteoclastic resorption of bone itself, not only
elevated even though hypercalcemia persists. absorption of calcium phosphate salts from bone. PTH stimulates
•PTH synthesis is stimulated by ^-adrenergic osteoclastic activity through an indirect mechanism because
agonists & magnesium (minor) whereas osteoclasts do not have membrane receptor proteins for PTH. PTH
phosphate has no affect. PTH synthesis is binds to receptors on osteoblasts (& osteocytes) & cause them to release
impaired in severe hypomagnesemia. secondary signals (cytokines) including osteoprotegerin ligand
(OPGL) = RANKL (receptor activator of nuclear factor kB ligand).
OPGL transforms proosteoclastic cells into mature osteoclasts. The
V it D P rim a ry stimulation of osteoclastogenesis requires M-CSF (macrophage colony
D e fic e n c y H y p e rp a ra th y ro id is m stimulating factor) and RANK ligand surface proteins on osteoblasts.
Bone Rapid Phase Slow Phase
Resorption
| N o rm a l |
V ita m in D intox ica tio n
Begins Minutes to hours Days to weeks
H y p o -P T H m a lig n a n c y Results from Direct activation of PTH indirectly stimulates
preexisting osteoblasts osteoclastogenesis
& osteocytes (transformation of prosteoclast
ISerum C lT
(osteocytic membrane into mature osteoclasts) through
system) by PTH OPGL (RANK ligand) & M. CSF
cytokines released by
osteoblasts.
Physiology: Endocrine System ■ 363

Hormone Involved in Calcium Metaboism Mechanism Absorption of calcium Osteoclastic resorption of bone
phosphate salts from
bone matrix near
Features PTH V itam in D Calcitonin
osteocytes & bone
Stim ulus 4 serum Ca2+ 4serum Ca2+ T serum
surface near osteoblasts
for 4serum P Ca2+Q
Estrogen stimulate osteoblasts to produce osteoprotegerin (OPG) or
secretion T pth
osteoclastogenesis inhibitory factor (OCIF), a cytokine which acts as a
Action on: T resorption QT resorption 4
decoy (fake) receptor binding to OPGL & preventing OPGL from binding
- Bone resorptionQ
its receptor, thereby inhibiting osteoclastogenesis and bone resorption.
- Kidney -4p -T P
Vitamin D and PTH stimulate osteoclastogenesis (production of mature
reabsorption reabsorption
osteoclasts) by dual action of inhibiting OPG production and stimulating
Q (T urinary -T Ca2+ OPGL formation.
CAM P) reabsorption
2. Kidney: PTH decrease calcium excretion (or increase absorption) in
- T C a2+
late distal tubule and increase excretion (or decrease reabsorption) of
reabsorption
phosphasphate in proximal tubule
- Intestine T Ca2+ -T C a2+
Normally 98-99% of filtered Ca++ is reabsorbed in kidneys: about 60%
absorption absorption
from proximal tubule, 25% from ascending loop of Henle and small
(via activation (calbindin
extent in distal tubule.
o f vit D) D- 28k)
Distal tubular reabsorption of calcium depends on TRPV5 (transient
-Tp
receptor potential vanniloid type 5) channels, whose expression is
absorption
regulated by PTH. The increased Ca++ absorption due to PTH occurs
Overall
mainly in late distal tubules, collecting tubules, early collecting ducts &
effect on:
ascending loop of Henle (± ). Although Ca2+ excretion is increased in
- Serum Tq T 4
hyper parathyroidism because increase in amount filtered overwhelms the
C a2+
effect on reabsorption. PTH increases plasma Ca++ levels.
- Serum P 4q T
Normally 85- 90% of filtered phosphate (Pi) is reabsorbed mainly through
active transport in proximal tubule involving 2 sodium dependent
phosphate cotransporters Na- Pi II a & Na Pi II c. PTH decreases
•1-Serum Calcium
phosphate reabsorption in proximal tubule by powerfully inhibiting Na
- Pi Ila causing it to internalize & degrade. So PTH decreases plasma
T pth
phosphate leve by this phosphaturic action.
______ I________
1 PTH also increases rate of reabsorption of M g++ (magnesium) and
Kidney T Bone Resorption hydrogen ions (H+) while decreases reabsorption of sodium (Na+),
potassium (K+) and aminoacids and result in mild metabolic acidosis
-TC a++ T 1,25- by osteocytic/
Reabsorptin osteoblastic 3. In testine: PTH increases absorption of Ca++ (mainly) & phosphorus
(OH ) 2 d 3 (+) indirectly via vitamin D 3 (1, 25 dihydroxycholecalciferol)
in distal membrane system
formation
tubue (rapid) and ■Calcium is primarily absorbed from upper small intestine. Ca++
osteoclastic absorption is facilitated by proteins and inhibited by phosphates & oxalates.
-4 p
resorption (slow) ■Both PTH and hypophosphatemia (produced d/t phosphaturic action of
Reabsorptio
n in
proximal
1 T PTH) stimulate 1 a-hydroxylase activity in kidney to increase formation
of 1,25, (OH)z D 3. This active vitamin D then increases gastrointestinal
Intestine T Calcium & absorption of calcium (mainly) and phosphorus. So PTH indirectly
tubule
T Ca++ & P in plasma promotes intestinal absorption via activation of vitamin D.
Ca++ is transported across brush border of intestinal epithelium via TRPV
Tp
6 channels and binds to intracellular calbindin D 9K protein, which
4 absorption
sequesters the absorbed calcium so that it does not disturb epithelial
T Ca++ & 4 p signaling processes that involve calcium, thereby delivering it to
in plasma basolateral membrane of epithelial cells. From here Ca++ is transported
1 into blood stream by NC X 1 (Na+/Ca++ exchanger) or Ca++ dependent
ATPase. Normally phosphate (Pi) is absorbed, in duodenum & small
TPlasma Calcium & 4 Plasma Phosphorus = Net
intestine by N aP i l ib transporter that takes advantage of low intracellular
Effect
Na+concentration established by NaKATPase on baso lateral membrane
of intestinal epithelial cells to load Pi against its concentration gradient.
Intestinal absorption of Ca++ & phosphate is increased by 1,25 vitamin D 3 .
364 ■ A Complete Review of Short Subjects

Calcium Metabolism
• V it D is really a hormone, since skin is the major source of Vitamin D. And only
w h en su n ex 1
p o s u r e i s i n a d e q1u a t e a d /
i e t a r y s o u r1c e is r e q u i r e d . T h e a d e q1u a t e
Cn
alcium
V] intake
intake recommended for adults it 200IU (5fig) if sun exposure is adequate,
and 400 - 800 IU/day in the absence o f sunlight ®. Osteomalacia is associated
with intakes <100 IU/day. Maximum (upper) limit of dietary intake of vit D is
2000 IU (50jug) /day in adults is now recommended d/t concerns about
potential toxic effects.
• Vitamin D from plant source is in form of vitamin D2, whereas from animal
source is vitamin D3. Both have equivalent biological activity and fate. Vit D
enters circulation, whether absorbed from intestine or synthesized
cutaneously, bound to vitamin D binding protein, an a-globulin synthesized in
liver. O v e rv ie w o f ca lciu m e x c h a n g e b e tw e e n d iffere n t
• In Skin - Vitamin D3 (Cholecalciferol) is formed as a result of irradiation of 7 tissu e co m p a rtm e n ts in a p e rso n in g e stin g 1000 m g
dehydroxy cholesterol by UV rays from sun. o f c a lciu m p e r d ay . N o te th at m o st o f th e in g ested
• In Liver - Conversion of cholecalciferol to 25- hydroxy cholecalciferol® c a lciu m is n o rm a lly elim in a te d in th e fe ce s, a lth o u g h

occurs. 25(OH)vitamin D is the major circulating and storage form of vitamin th e k id n e y s h a v e th e c a p a c ity to e x c re te larg e
a m o u n ts b y re d u c in g tu b u la r re a b so rp tio n of
D. 88 % of 25(OH)D circulates bound to vitamin D binding protein, 0.03% is
calciu m .
free and rest circulates bound to albumin. The half life of 25(OH)D is 2-3
weeks, which is greatly reduced when vitamin D binding protein levels are
reduced as in nephritic syndrome.
• In Kidney - Formation of 1 , 25 - dihydroxy cholecalciferol takes place in proximal tubules o f kidney ® which is the active
form of vitamin - D.
• l a hydroxylase is also present, so 1,25 (OHh D3 also formed in placenta, in Keratinocytes o f Skin & macrophages and
granulomas o f sarcoidosis, tuberculosis, berylliosis as w ell as lymphomas®. But in these pathological granulomas, interferon y
and TNF induce enzyme activity and these are not regulated by calcium and 1,25 (OH )2D feed back. Therefore hypercalcemia may occur
b/o elevated 1,25 (OH)2D, which is treated by glucocorticoids, ketoconazole or chloroquin.
• In sarcoidosis pulmonary macrophage® produces 1, 25 (OHh D3 due to apparent stimulation of 'Y' - interferon.
• In Ca lung plasma Ca2+ level increased d/t ectopic production o f PTHrP (Parathormone related peptide)®.

Calcium (Ca**)
• The body of young adult contains ~ llOOgm Ca++ and Pi Transport in Small Intestine
(27.5mol) of Ca; 90% of it is in skeleton. The 200 mg calcium is absorbed per day form p r o x im a l s m a ll in testin e (ie
plasm a calcium, is norm ally about lOgm IdL d u od en u m a n d jeju n u m ) b y a h o r m o n a lly reg u la ted (ie s tim u la te d by
(5m eq / L, 2.5 m m o l / L)®, of which 50% is 1,2, d ih y d r o x y v ita m in D) tran s c e llu la r ro u te a n d b y a p a s s iv e
ionized. The rem ainder is bound ionically to p a r a c e llu la r route®.
negative charged proteins (predom inantly Ca ++ m ovem ent from GI lumen into the enterocyte (across brush border
album in & im m unoglobins) or loosly of intestinal epithelial cells) is favoured by chem ical/electrical gradients
com plexed with phosphate, citrate, sulphate. I t and is facilitated by apical epithelial calcium channels k/a TRPV
is th e fr e e io n iz e d calciu m , th a t is a v it a l (transient receptor potential vanilloid) type 6 and 5. In cytoplasm,
s e c o n d m essenger® and necessary for b lo o d calbindin D?k protein binds to (sequesters) the absorbed calcium, which
co a g u la tio n , n erv e con d u ctio n a n d m u scle maintains low cytoplasm ic C a*+ levels (thus preserving the favourable
con traction ® . The level of plasma calcium at lumen to enterocyte C a++ gradient) so that it does not ham per epithelial
which fatal tetany occurs is still above the level signaling processes that involve calcium. Calbindin 9k shuttles Ca+* from
at w hich clotting defect would occur. apical to basolateral membrane. C a++ is transported across the basolateral
• Since the extent of calcium binding by plasma m em brane into blood against an electrochem ical gradient by either
protein is proportionate to the plasm a protein plasma membrane calcium ATPase (PMCA), a Ca++ dependent
level, it is im portant to know the plasma protein ATPase or Lum inal
1 . (A pical) side _
Serosal
level when evaluating the total plasma calcium. sodium/calcium tow ards G IT ^ Ca« -C a lb in d in 9K
(B as sola lera l)
S id e tow ards blood
An algorithm to correct for protein changes exchanger (3Na+/lCa++ p| iA_
adjusts the total serum calcium (in mg / dL) antiporter = NCX1). 1, C a "—( J g |^ 2 ) > ( c a “ ') jc B — Ca**

upward by 0.8 times the defecit in serum albumin (in 25 dihydroxy vitamin D TRPV 6/5 / <N C X 1 [►Ca**
3N a+
Ca** -C a lb in d in 9K
gm/dl) or by 0.5 times the defect in serum stim ulates all P a r a c e llu la * -.. _ — —------- : _ ' — q 3~

immunoglobin (in g/dl). So corrected calcium com ponents of Ca++


concentration absorption in intestine. Intestinal Epithelial Cell
Physiology: Endocrine System ■ 365

= Total calcium level (mg/dL) + 0.8 mg/dL - Phosphate (Pi) absorption by jejunum remains constant at 70% and is
(0.2MM) for every decrement in serum albumin under control of 1, 25 dihydroxy vitamin D. Transcellular Pi absorption
of 1.0 gm/dL (below the reference value of 4.1 across apical bursh border mediated by Na+ -Pi cotransporter (NPT2) is
gm/dL of albumin). the limiting step.
= Total calcium level (mg/dL) + 0.5 mg/dL for Calcium Transport Along Nephron
every decrement in serum immunoglobulin of
99% of filtered Ca++ is reabsorbed by nephron. About 1% ie 200
lgm/dL.
mg/day is excreted in urine which is equal to net amount daily
• PH affects the calcium level, for example tetany absorbed in git.
appear at much higher total Ca++ levels, if
Part of % of Ca++ M echanism
patient hyperventilates, increasing plasma pH.
Nephron Reabsorbed
Plasma proteins are more ionized when the pH
PCT 60-70% of - Passive paracellular path between cells
is high, providing more protein anion to bind
total across tight junction accounts for 80%
with calcium thus reducing effective amount of
reabsorption. It occurs via solvent drag in
ionized calcium. In same way a c id o s is in c rea se s
entire PCT and by positive luminal voltage
io n iz ed C a*+ levels®.
(i.e. diffusion) in distal half PCT.
• Intestinal absorption of calcium involves both - Active transcellular path accounts for 20%
active (transcellular) and passive (paracellular) of total PCT reabsorption.
mechanism. Passive absorption is nonsaturable Loop of 20% - Via both paracellular and trans cellular
& approximates 5% of intake, whereas active Henle paths by same mechanism as of PCT but
mechanism controlled by 1, 25, (OH )2 D, (cortical thick with one difference i.e. Ca*+ is not reabsorbed
normally ranges from 20- 70%. A c tiv e ascending by solvent drag as TAL is impermeable to water.
a b s o r p tio n occu rs m a in ly in p r o x im a l s m a ll limb)
b o w e l (du oden u m & p r o x im a l jejunum )® . DCT 9% - Exclusively active transcellular because the
O p tic a l r a te s o f C a *+ a b s o r p tio n req u ires Collecting <1% tubule lumen (DCT) is electrically negative
g a s t r ic acid® . This is especially true for weakly duct with respect to blood so the Ca++ is
dissociable calcium supplements as CaCCb. reabsorbed against electrochemical
• Calcium interchange between plasma and gradient.
rapidly exchangeable reservoir of bone is - Mutations in claudin 16, a protein component of tight junction in TAL
500mmol per day. Whereas between plasma (thick ascending limb of Henle's loop) cells, causes fam ilial
and stable pool of bone is only 7.5mmol/day. A hypomagnesemic hypercalciuria b/o enhanced excretion of Ca++ &
large amount of calcium is filtered in kidney but Mg++ (d/t fall of passive paracellular absorption of these ions).
98-99% of filtered calcium is reabsorbed - 60%
of which occurs in PCT and remainder in _________________ Calcium Sensing Receptors (CaSR)_________________
ascending loop of Henle (25%) and distal Calcium sensing receptors (CaSRs) are present in PTH secreting cells of
tubule (small extent). parathyroid glands, calcitonin secreting parafollicular cells in thyroid
gland and calcitriol (Vit D) producing cells of proximal tubules (i.e. in
the cells producing hormone involved in regulation of serum calcium).
Total diffusible calcium 1.34 CaSRs are also present in th ic k a scen d in g lim b o f lo o p o f H en le a n d d is t a l
- Ionized (Ca++) 1.18 tu b u le (DCT)® to directly regulate Ca++ homeostasis. Small intestine
- Complexed to H C O 3, citrate 0.16 cells lack CaSRs
etc Autosomal Dominant
Feature AD Fam ilial Hypocalciuric
Total nondiffusible (protein 1.16 Hypercalcemia (FHH) Hypocalcemia
bound) calcium 0.92 Activating mutation in CaSR
Cause Inactivating mutation in CaSR
- Bound to albumin 0.24 Clinic - Hypercalcemia d/t deranged - Hypocalcemia d/t
- Bound to globulin Features Ca++ regulated PTH secretion deranged Ca++ regulated
Total plasma calcium 2.50 (i.e. PTH is elevated at all PTH secretion (i.e. PTH
plasma Ca++ levels). level is decreased at any
- Hypocalciuria d/t enhanced level of plasma Ca++)
Ca++ reabsorption in TAL and - Hypercalciuria d/t
DCT as a result of elevated PTH decreased PTH levels and
and defective CaSR defective CaSR.
366 ■ A Complete Review of Short Subjects

Parathyroid Related Peptide and Features of Hypocalcemia


Hypercalcemia
Symptoms Signs
Hum oral hypercalcemia o f malignancy is most often 1) N u m bn ess a n d tin g lin g o f 1) C h o v ste k 's ®: Contraction of
associated with overproduction o f PTHrP. circu m o ra l reg ion & th e tip s o f facial muscles in response to
PTHrP is structurally related to PTH & binds to fin g ers a n d toes®. tapping the facial nerve anterior
the PTH receptor producing features similar to 2) E x ten siv e s p a s m o f s k e le t a l to the ear.
hyperparathyroidism (hyper cacemia) m u scle c a u se s cra m p s & 2) T rou sseau 's sign®- Capal spasm
Mutation in Ras oncogene stimulates PTHrP tetany® . occurring after occlusion of
production. 3) C a r p o p e d a l spasm ® occurs. brachial artrery with a blood
HHM occurs in about 5% of patient with cancer. 4) Laryngospasm with stridor. pressure cuff for 3 minutes.
HHM is most common in cancers of lung®, 5) H y p e ra c tiv e ten d o n reflex® 3) Erb's sign®- Muscle contraction
breast® , head & neck, genitourinary tract, 6) P r o lo n g a tio n o f Q T interval® . can be produced by application
esophagus, m u ltip le m y elom a® & lymphoma. 7) Increased intracranial pressure of sub threshold electrical
Many bone metastasis eg breast, multiple occurs in some patient with stimulation.
myeloma produce PTHrP leading to local long standing hypocalcemia. 4) P e r o n e a l sign® - Tapping over
osteolysis and hyper calcemia. Papilloedema is associated with the peroneal nerve at the neck of
these cases fibula will produce planter
flexion & adduction of the foot
(pedal spasm).__________________

Calcium Metabolism In Mammalian Cells IP3 & Ca+

Concentration of intracellular Ca2* is 100 nmol/L and I n o s it o l tr ip h o s p h a te (IP3) d iffu s es to th e e n d o p la s m ic reticu lu m ,


it is a b o u t 12000 tim es le ss th an t h a t in ex tra c ellu la r w h ere it triggers th e r e le a s e o f ca lciu m in to cy top lasm ® . IP 3 is the
fl u id .0 major second messenger that causes release of Ca++ from ER
Ca2+ is stored in endoplasmic reticulum and through the I P 3 receptor activation (a ligand gated channel). So one
mitochondria & can be released from them to replenish second messenger (IP3) causes release of another second
cytoplasmic Ca2+ messenger (Ca++)Q.
Calcium binding proteins bind cytoplasmic Ca2+ and Inositol triphosphate and diacylglycerol (DAG) both of which are
when activated in this fashion, bring about a variety second messengers, are associated with Gq subset of G protein
of physiological effect. mediated receptors.
Ca2+ enter cell via (i) Voltage gated Ca2+ channels
(ii) Ligand gated Ca2+ channels (T & L types) (iii) Hormones
Store operated Ca2+ channels (SOCCs) 1
It is transport out of cell by (A ctiv e transport)® Activated receptors
(i) C a2* H* A T P a se antiport® : cause 2H * in sid e &
C a 2* outside® of membrane at the cost of Gq protein
hydrolysis of ATP.
jstim ulate
(ii) N a* C a2* a n tip o r t ®: It is a counter transport
causes 3 N a* in sid e & 2 C a2* ou tside® of Activation of membrane bound Phospholipase C
membrane.
(iii) C a - Mg ATPase Cleaves membrane bound phosphatidyl inositol into
Calcium movement into the internal stores is through ____________________ I_________________,
the action of sarcoplasmic or endoplasmic reticulum
Ca++ ATP ase (SERCA pump)Q. I n o s ito l 1,4,5, trip h osp h ate® Diacylglycerol
Calmodulin is Ca2+ binding protein involved in 3
(IP ) (DAG)
contraction of smooth muscle.
I
* Passive transport is when molecules traverse Acts on endoplasmic Activated protein
membrane by electro mechanical gradient reticulum kinase C
(Passively). Uniport is form of transport where only
one substance is transported. Symport is binding of
Releases C « 2+
I
Phosphorylated protein
more than one substance to transport protein &
substances are transported across the membrane
together. Antiport is the transport exchange one
substance for another. Intracellular effects
Physiology: Endocrine System ■ 367

Endocrine Pancreas Exocrine Pancreas


- Although scattered through-out the pancreas, islets are more - Pancreas secretes its exocrine products as
plentiful in tail. All 1-2 million human islets have copious blood enzymatically inert proenzymes both to prevent self
supply. Blood from islets, like that from gastrointestinal tract (but digestion® & to efficiently utilize the enzymes within the
unlike that from any other endocrine organs) drain into hepatic lumen of duodenum. Trypsin is a powerful pancreatic
portal vein. proteolytic enzyme which would digest the pancreatic
- On the basis of staining & morphological properties islets cells can cells, if not produce as proenzyme (inactive form) -
trypsinogen®. Trypsimogen is activated in the
Islets cell type Secretion % of total endocrine volume intestinal tract by enzyme called enterokinase®.
A cells Alpha (a) Glucagon® 10 -20% - The self digestion of pancreas is prevented by:
B cells Beta (fi) Insulin® & 60 -75% (most abundant) 1) The majority of the enzymes are synthesized as inactive
amylin proenzymes® (with the exception of amylase & lipase)
D cells Delta (5) Somatostatin® 5% 2) The enzymes are sequestered in membrane - bound
F cells PP cells Pancreatic < 5% overall, but ~ 80% in zymogen granules in the acinar cells.
polypeptide® posterior head & uncinate process 3) Activation of proenzymes requires conversion of
D1 cells VIP - inactive trypsinogen to active trypsin by duodenal
- Most abundant B (0) cells are usually located in center of each islet, enteropeptidase (enterokinase)
which tend to be surrounded mainly by many A (a) cells and few if 4) Trypsin inhibitors including serine protease inhibitor
any F cells in outer rim in tail, body, and anterior & superior part o f
K azal type 1 are present within acinar and ductal
secretion®
head o f pancreas. Whereas, posterior part of head (including
5) Trypsin contains a critical self- recognition cleavage
uncinate process) of pancreas have large number of F cells & few A
site that allows trypsin to inactivate itself.
cells.
6) Lysosomal hydrolases are capable of degrading
- A cell (glucagon) rich islets arise embryologically from dorsal
zymogen granules when normal acinar secretion is
pancreatic bud whereas, F cell (pancreatic polypeptide) rich islets
blocked.
arise from ventral pancreatic bud.
7) Acinar cells are remarkably resistant to the action of
- In B cells granules, insulin forms different size & shape polymers or
trypsin, chymotrypsin & phospholipase A 2.
zinc (Zn) aggregates / complexes (so these are heterogenous) in
★ Pancreatic enzymes require alkaline medium fo r
different species. A cell and D cell granules are relatively
their activity so bicarbonate secretion w ill help in
homogenous (uniform).
their activity®.
Pancreatic Juice (Exocrine Juice)
Features Stim ulant For Secretion
Isoosm otic with blood Secretin - Produced by duodenal mucosa®
plasma® - Stimulated by acidity® of chyme
Volume = ~1500ml/day® - Stimulates a bicarbonate® rich pancreatic secretion.
PH = ~ 8 (alkaline) Cholecystokinin - Produced by jejunal® mucosa.
Rich in enzymes® (about (CCK) - Stimulated by fat® in chyme
20 different enzymes are - Stimulates enzyme rich® pancreatic secretion
secreted by pancreas) Acetylcholine - Acts on acinar cells via phopholipase C.
- Causes discharge of zymogen granules
Vagal stimulation - Stimulates enzyme rich® secretion

Starvation/Fasting
Effects on Carbohydrate Metabolism Hormone Changes
• During first few days - Glycogenolysis® provides the major & immediate source of Insulin Decreased
glucose, after which glucose level begins to fall. Glucagon Increases®
• Hypoglycemia depresses insulin® secretion & thus increase gluconeogenesis®. Growth hormone - Increase®
• Gluconeogenesis maintains blood sugar but at subnormal level. Thus glucose
tolerance is decreased®.
• Antagonistic hormones are increased®:
- Glucagon increases® (tend to increase blood glucose)
- GH level increases® (tend to increase blood glucose)
• After 2 days of fasting, the major substrate for energy product will be stored fat®, (as
glycogen reserve is exhausted in first 2 days).
• Catabolism of fat will load to increase in FFA in blood®.
368 ■ A Complete Review of Short Subjects

Insulin
Nobel Prizes Structure and Synthesis

• Nobel prize for isolation of insulin was given to Insulin has tw o interchain (A 7- B7, & A20 - B19) and one intrachain
Frederick G. Banting and J.J.R . MacLeod in 1923. (A6 - A ll) disulfide bridges w ith in AB hetrodimeric structure®.
Banting immediately announced that he would Insulin is synthesized as pre-pro-hormone (MW 11, 500), which makes
share his prize with Charles Best; Macleod shared it difficult to synthesize insulin in laboratory even after synthesizing
his with J.B . Collip. A, B chains. This pre prohormone is directed by hydrophobic 23
• Fredrick G Sanger got Nobel prize in 1958 for aminoacid pre or leader sequence into cisternae of endoplasmic
sequencing insulin structure (ie for determining the retinaculum, which is then removed to form 9000 MW proinsulin.
com plete am inoacid sequence o f both polypeptide Starting from amino terminal, the chain sequenc of proinsulin is.
chains o f bovine insulin)® B chain Connecting (C) peptide A chain
• Fredrick Sanger again with Walter Gilbert got Nobel
Proinsulin molecule undergoes on initial cleavage by trypsin like
prize in 1980 for sequencing of large nucleic acid
enzyme followed by serries of several cleavages by carboxypeptidase
like enzyme resulting in form ation o f equim olar amounts o f mature
hetrodimeric (AB) insulin and C - peptide®. (1 :1 ratio)®.
Pork insulin differs from human insulin by only one amino acid
residue®. It also has low antigenicity.
Human insulin produced in bacteria by recombinant DNA technology
is used to avoid antibody formation.

Factors Affecting Insulin Secretion Stimulators Inhibitors


(T Insulin serection) (4 Insulin secretion)
- Plasma glucose concentration is the most important regulator of insulin 1 1
secretion, and glucose acts directly on pancreatic B cells. The response to - Increased blood levels of
hyper glycemia is biphasic i.e. a rapid but short lived initil spike followed by glucose, mannose, amino - Fasting and
more slowly developing prolonged increase (second phase). Glucose enters 13- acids (esp leucine & decreased blood
cells via GLUT-2 transporters and is phosphorylated by glucokinase then arginine), free fatty glucose levels
metabolized to pyruvate in cytoplasm. Pyruvate enters mitochondria form acids, P-keto acids (eg
- 2 deoxyglucose
ATP by oxidative phosphorylation and also cause increase in intracellular aceto-acetic acid).
and
glutamate (via TCA cycle). - Obesity, insulin mannoheptulose
Regulation of Insulin Secretion by plasma glucose resistance
- Somatostatin
- Intestine (gut) hormones
I------------- - Leptin, Galanin
Rapid short lived Slow, Prolonged increase in secretion (GIP=gastrointestinal
increase in Glutamate act as intracellular 2nd messenger on peptide, GLP=glucagons - Insulin
secretion second pool of secretory granules. It primes secretory like peptide 1 [17-36], - a Adrenergic
ATP enters granules for secretion, maturing them by decreasing gastrin, secretin, stimulators
cytoplasm, their pH. This commits granules to the releasable form cholecystocynin = CCK) (epinephrine,
inhibiting ATP producing prolonged 2nd phase of insulin release. - Glucagon, GH (growth norepinephrine)®
sensitive K+ hormone) and - (3-Adrenergic
Glucose
channels & glucocorticoids® blockers
reducing K>
efflux. This
m G
Glucokinase
- Parasympathetic
stimulation and acetyl
(propranolol)
- K+ depletion
depolarize B cells Gluocse-P
IOCS* choline (Ach)
and Ca++ influx via - Cyclic AMP and various - Diazoxide
voltage gated Ca++ i substances generating c- - Thiazide diuretics
V f
channels cause AMP - Phenytoin
exocytosis o f
readily releasable
1/ Citric acid
► Insulin - |3-Adrenergic stimulation - Alloxan
cycle (fors) - Micro tubule
p ool o f insulin - Sulfonylureas (glyburide,
/ \ inhibitors
containing tolbutamide)
secretory granules, - Theophylline
producing initial
spike of insulin
secretion.
Physiology: Endocrine System ■ 369

Glucose, mannose, amino acids (esp arginine & leucine) and P-keto acids (eg acetoactic acid) provoke greater insulin
secretion, when administered orally (than that by parenteral /iv route). This is b/o gut /gastrointestinal hormones - gastric
inhibitory peptide (G1P), glucagons like peptide (G LP)-l (7-36)/entero glucagons, secretin, gastrin & CCK.
Glucagon, growth hormone (GH), glucocorticoids, P-adrenergic stimulation, parasympathetic stimulation (acetylcholine)
and C-AMP increase insulin secretion. Whereas, a-adrenergic stim ulation (by epinephrine & norepineprine), somatostatin®,
leptin & galanin decrease insulin secretion.
So catecholamines (epinephrine & nor epinephrine) have a dual effect on insulin secretion. They decrease it via a2
adrenergic receptors and increase it via P-adrenergic receptors. But the net effect is inhibitory. However, if catecholamine
are administered after a-blockage, it stimulates insulin secretion d/t unopposed P-action.

Effect of Insulin on K+ and PO 43- Factors Influencing K+ D istribution

Insulin increases the transport of potassium (K+) from plasma Shift K + into cells Shift K + out o f cells
into cells by stimulating the Na+-K+ - ATPase pumps in cell (decreae extra (increase extracellular
membranes of muscle, liver and adipocytes. This results in lowering cellular potassium ) potassium ) =Cause Hyper
o f plasm a (extracellular) K+ concentration i.e. hypokalemia®. = Cause kalem ia
Infusions of insulin (and glucose) significantly lower the plasma H ypokalem ia
K+ levels in normal individuals and are used for temporaty relief - Alkalosis® - Acidosis®
of hyperkalemia in patients with renal failure, waiting for dialysis. - Aldosterone - Aldosterone deficiency
Hypokalemia often develops when patients with diabetic acidosis - P-adrenergic (Addion's disease)
are treated with insulin. stimulation - P adrenergic blockade
Insulin induces rapid entery of glucose into cells, a process which is - Insulin® - Insulin deficiency
followed by phosphorylation reaction in glycolytic pathway, lowering (diabetiese mellitus)
the intracellular concentration of inorganic phosphate, and therefore - Cell lysis
promotes phosphate ( P O 3 ^ ) entry into cells. This is how insulin 1/t - Sternuous exercise
hypophosphatemia (decrease plasma /ECF P O 4 3 ) - Increased extracellular
fluid osmolarity

Insulin: Summary

Embryology Principal Actions Effects on Various Tissue


Insulin containing granules Rapid (seconds) Adipose tissue
can be identified in the human - Increased transport o f glucose, amino acids, - Increased glucose entry
fetal pancreas by 9 to 10 weeks and K+ into insulin-sensitive cells® - Increased fa tty acid synthesis and
and detectable in fe ta l plasm a Intermediate (minutes) lipogenesis®
by 12 weeks®. Glucagon - Increased glycerol phosphate synthesis
- Stimulation of protein synthesis
secretion begins by 8lh week. - Increased triglyceride deposition
- Inhibition of protein degradation
Thyroxin secretion begins by - Activation o f lipoprotein lipase®
- Activation o f glycolytic enzymes and
10th to 22th week. increased clearance of VLDL & chylomicron.
glycogen synthase®
- Inhibition o f horm one-sensitive lipase®
- Inhibition o f phosphorylase and
Substance w ith Insulin like inhibits lipolysis
gluconeogenic enzymes®
Activity - Increased K+ uptake
• Insulin Delayed (hours)
• Proinsulin - Increased in mRNAs for lipogenic and other M uscle
• Nonsuppressible insulin-like enzymes - Increased glucose entry
activity (NSILA) - Increased glycogen synthesis
- Low-molecular-weight Insulin Stim ulates lipogenesis by - Increased amino acid uptake®
fraction - Increasing acetyl - C o A carboxylase - Increased protein synthesis in
IGF-I activity®, (most important enzyme in ribosomes®
IGF-II regulation of lipogenesis). This allosteric - Decreased protein catabolism
- High molecular-weight enzyme is activated by citrate, which - Decreased release of gluconeogenic
fraction (mostly IGF bound increases in concentration in well fed state amino acids
to protein) and is an indicator of a plentiful supply of - Increased ketone uptake
acetyl CoA. - Increased K+ uptake® _____________
370 ■ A Complete Review of Short Subjects

Liver
Insulin causes hypoglycemia. - Increasing transport o f g lu cose into cells® - D ecreased ketogenesis®
Hormones causing like adipose tissue. - Increased protein synthesis®
hyperglycemia (opposite - Increasing a v a ila b ility o f pyru vate fo r fa tty - Increased lipid synthesis
action) acid syn thesis and g ly cerol 3 p h o s p h a te fo r - Decreased glucose output due to
- Catecholam ines® easterification® of newly formed fatty acids. decreased gluconeogenesis; increased
(epinephrine, - A ctiv ates p yru vate dehydrogenase in ad ip ose glycogen syn thesis; and increased
norepinephrine) tissue bu t n ot in liver®. - Glycolysis®
- Glucagon® - D ecreases in tracellu lar C- AMP level® - thus - Inhibits glycogenolysis Si glucose uptake.
- G row th hormone®, ACTH inhibitibiting lipolysis in adipose tissue and General
- Glucocorticoids thereby reducing concentration o f p la sm a - Increased cell growth
- T hyroid hormones® fre e fa tty acid and thus long chain acety l
CoA®, an inhibitor of lipogenesis.
★ Vagal stimulation causes stimulation of insulin secretion but only when blood sugar level is high.
* Glucagon: In the liver, it acts via Gs to activate adenyl cyclase & increase intracellular cAMP. An increase in ratio of insulin to
glucagons implies a decreased concentration of glucagons & hence decreased ratio o f c AMP®.

Insulin
1
Mechanism of Action Receptor
Insulin + insulin receptor Insulin receptor has two subunits a- and P, which are
4 then assembled into a tetramer linked by disulfide bond.
A ctivation o f the receptor- tyrosine kin ase activity®
4 a - Subunit P - Subunit
Phosphorylation of insulin receptor and insulin receptor c| I
substrates (IRSs) Is extracellular Is transmembranous protein
4 Function is to Its function is sign al tranduction®
Activation of other protein kinase & phosphatase. bind insulin® The cytoplasmic portion has
4 tyrosine kin a se activ ity
Biological effects of insulin (enzym eatic receptor)®

Metabolism Increased Decreased Secretion


Carbohydrate - Glycolysis Gluconeogenesis
A Polypeptide hormone
Glycogenesis Glycogenolysis
(glycogen (glycogen I
synthesis) degradation) Is secreted by
- Glucose uptake 4
- HMP shunt P cells of the pancreas
Protein Protein synthesis Protein degradation
Fat Lipogenesis Lipolysis Secretion is stimulated Secretion is in hibited
(fat synthesis) ketogenesis by by®
4 4
T Blood glucose T Epinephrine
Physiology: Endocrine System ■ 371

Effect of Insulin On Metabolism of

Carbohydrate Lipids
Net effects is - To J- blood glucose level Tglycogen
storage® • Enhanced u p take o f g lu cose in ad ip ose cells v ia GLUT 4
tran sporter and in creased K* uptake®

Increases Decreases • Insulin increases lipogenesis (i.e. fatty acid glycerol phosphate
and triglyceride synthesis®) and decreases lipolysis (and so I TG
Glycogenesis in liver & • G luconeogenesis in
muscle by activating
degradation, J-glycerol, J-free fatty acid level and i ketogenesis®)
liver by inactivating
enzyme glycogen following enzymes: • Fatty acid synthesis is increased by activating following enzymes
synthetase® - Pyruvte carboxylase® - A cetyl COA carboxylase® (rate limiting enzyme)
Glycolysis: TUtilization of - Phosphoenol - ATP citrate lyase (T acetyl COA synthesis from citrate)
glucose for providing pyruvate - Pyruvate dehydrogenase complex (T availability of acetyl COA
energy, by inducing the carboxykinase from pyruvate)
synthesis of following - Fructose 1, 6, - Fatty acid synthase
enzymes biphosphatase - Malic enzyme
Phosphofuctokinase® - Glucose - 6 - • Triglyceride synthesis is increased by
Pyruvate kinase® phosphatase - A ctivatin g lip op rotein lipase® which hydrolyses TG from VLDL
Glucokinase • Glycogenolysis in & chylomicrons and release FFA (pool 2) used for TG synthesis.
HMP shunt: by inducing: liver by inactivating - Increased FA synthesis provide acyl CoA (pool 1)
Glucose -6 P O 4 following enzyme
• Lipolysis is decreased by
dehydrogenase - Glycogen
- Activating phosphoprotein phosphatase and phosphodiesterase (both
6 Phosphogluconate phosphorylase
inactivate TG lipase)
dehydrogenase Glucose- 6 phosphatase
- Inhibiting hormone sensitive lipase
Protein • In hibitin g horm on e sen sitiv e lipase® (principal action of insulin in
1__________ adipose tissue, which reduces release of FFA & glycerol)
• Decreased p oxidation, cholesterol synthesis and ketogenesis d/t
Increase protein Decrease protein
decrease plasma FFA levels by decreasing HMG CoA synthetase
synthesis by increasing degradation by decreasing
RNA polymerase transaminases &
deaminases

Glucose Transport In Various Tissue

Active transport Facilitated transport


Energy requiring process that Glucose movement follows a conc.
transport glucose 'against' gradient, i.e. from high glucose conc. to a
concentration gradient lower one.
Insulin sensitive - - Muscle: skeletal & cardiac muscle®
- Adipose tissue®
Insulin insensitive Epithelia of - Erythrocytes®
- Intestine - Leukocytes
- Renal tubules® - Lens of eye
- Choroids plexus - Cornea
- Liver
- Brain
❖ Insulin increases entry of glucose into liver cells, but by an indirect mechanism®.
372 ■ A Complete Review of Short Subjects

Inhibitors & Toxins Absorption of Hexose in Intestine


• Sodium dependent glucose transporter (SGLT or Na+ glucose • Hexoses are rapidly and completely absorbed before the
co transporter) is inhibited by phlorizin® whereas, phloretin meal reach terminal ileum
inhibits sodium (Na+) independent glucose transporter (GLUT • Transport of most hexoses (glucose & galactose) is
2) in intestine®. dependent on Na+ ions in intestinal lumen; a high Na+
• Fish neurotoxin tetrodotoxin (applied externally) block Na+ concentration facilitates and a low concenteration inhibits
channels® (esp early membrane voltage dependent Na* current; sugar influx into the epithelial cells.
hence'm ' gates controlling Na+ channels externally) where as • This is because Na+ & glucose share a same cotransporter
enzyme pronase (applied internally) block voltage dependent or symport, the sodium dependent glucose transmporter
inactivation ofN a* current (hence 'h' gates controlling Na+ (SGLT), Na+ glucose cntransporter). These (SGLT 1 & 2)
channels from inside). resemble glucose transporters responsible for facilitated
• Tetraethyl ammonium or 4 aminopyridine block potassium diffusion in that they cross cell membrane 12 times and
channel® (esp delyed voltage dependent increase in K+ have their - COOH & - NH2 terminals on cytoplasmic
permeability and hence 'n' gate). side of membrane. However , there is no homology to
• Ouabain that comes from adrenal glands is a digitalis like glucose transporter (GLUT). S G L T -1 is responsible for
steroid and a Na+ - K+ ATPase inhibiting factor®. It causes uptake of dietary glucose from gut where as SGLT-2 is
- Natriuresis and raises rather than lowers blood pressure responsible for glucose transport out of renal tubules.
- Inhibition of iodide trapping (active transport of iodide in • As the intracellular Na+ conc. is low in intestinal cells (as
follicular cells) in thyroid it is in other cells), Na+ moves into the cell along conc.
- Black most of calorigenic action of thyroid hormone gradient and glucose is contransported with it. From here
• Vandate inhibits Na+ - K+ - ATPase pump when present inside glucose is transported by GLUT -2 into interstitium (& thence
the cell. to capillaries), and Na+ into the lateral intercellular space.
• Azide, Cyanide and H2S block site III (complete IV) of electron • So glucose transport is a secondary active transport i.e.
transport chain and inhibit terminal transfer of electrons to the energy for glucose transport is indirectly provided by
molecular O 2 active transport of Na* out of cell, which maintans the
Enterocyte cytosol SGLT 1 Intestinal lum ber concentration gradent (so more Na+ & consequently more
glucose enter)
• SGLT - 1 transports glucose & galactose. But fructose
utilizes Na+ independent GLUT -5 for its facilitated
diffusion from intestinal lumen into enterocytes and
GLUT- 2 for transport outside enterocytes into
interstitium
• Like glucose reabsorption in PCT o f kidney, insulin has
no (little) effect on intestinal absorption (transport) of
sugars®. Neither process require phosphorylation, and
both, are normal in diabetes but depressed by drug
phlorizin®.
CLUT2 CLUT5

Insulin Resistance in O besity


It is caused by decreased ability or inability of insulin to
1. Increase GLUT-4 Mediated Uptake of 2. Repress Hepatic Glucose Production by 3. Repress Hormone Sensitive
Glucose by Skeletal Muscle Glycogenolysis & Gluconeogenesis Lipase (HSL) or Increase LPL
in Adipose Tissue___________
Increased burden of glucose in skeletal - Decreased repression of these pathways r/i High HSL & low LPL are
muscle with decrease uptake promote increased burden of intrahepatic glucose that is major factors in dyslipidemias
lipogenesis and excessive TG converted to FFAs & TG. The influx of glucose a/w insulin resistance &
accumulation (lipotoxicity). and FFAs exceed the ability of liver to pakage diabetes.
By products of FA & TG synthesis like lipids into VLDL for secretion & transport to Increased paracrine
diacylglycerol and ceramide adipose tissue. So it stores increasing amounts diabetogenic factors in adipose
accumulate and stimulate signaling of TG causing hepatic steatosis (fatty liver or tissue like TN Fa, resist the
pathways (eg protein kinase C- lipotoxicity) that progress to nonalcoholic actions of insulin.
dependent pathway) that antagonize steatotic hepatitis (NASH).
signaling from the insulin receptor (IR) - Cytokine TN Fa produced by adipose tissue
or IRS (insulin receptor substrate) antagonize insulin signaling pathway.
protein.
Physiology: Endocrine System ■ 373

Stress Syndrome: CRH - ACTH - Cortisol Axis

• In maintenance of internal hom eostasis (ie avoiding stress) and reproduction p r o c e s s e s , m u ltip le h o r m o n e s c o o p e r a te to
b rin g a b o u t o rg a n iz e d b io c h e m ic a l & p h y s io lo g ic a l re s p o n s e s .
• The primitive signal of glucose (substrate) lack is e x p a n d e d to a b r o d e r signal of fright or stress a n d e v o k e s a c o o r d in a te d
n e u r a l a n d e n d o c r in e r e s p o n s e . A n y s tre s s fu l e v e n t eg . s u r g e r y , i n f e c t i o n , b u r n trig g e rs a h y p e r m e ta b o lic s ta te in w h ic h a
re s tin g e n e r g y e x p e n d itu r e is in c r e a s e d a lo n g w ith b o d y te m p e r a tu r e .
• CRH-ACTH - cortisol axis is central to th is in te g r a te d re s p o n s e to s tre s s .

Features of Stress Syndrome

- Corticotropin releasing hormone (CRH) is increased in response to noxious environm ental stim uli like pain, trauma,
hypoglycemia, infection, surgery, hypovolemia. V arious nervous im pulses using several neuro transm itters (NTs) like
norepinephrine, serotonin, acetylcholine, dopam ine, and G A BA reach CRH neurosecretory cells and cause release o f CRH
- In response to CRH , both A C TH and P endorphins (secreted p-LPH & m ature P-endorphin) are released in equim olar
quantities P-endorphin produce central analgesic response.
- Glucagon & catecholamines (epinephrine & norepinephrine) are increased, stimulating glycogenolysis (glucose
production in liver) for im m ediate use by critical organs such as the brain.
- Cortisol production is increased in adrenal cortex w hich induces gluconeogenesis (glucose production from
noncarbohydrate source like m uscles & other bod y proteins)
- Glucogon, epinephrine, ACTH and growth hormone (GH) stim ulate lipolysis and free fatty acids (FFA) & ketones are
released into the circulation to be used as an alternative m etabolic fuel.
- Insulin is suppressed® that results in decrease in glucose utilization.
- O ne form of stress that is cold, specially in neonate causes increased TRH secretion due to depressed m etabolism of
neurons in pre- optic area by cooling w hich release inhibitory discharge, perm itting TRH secretion.
- ADH (vasopressin), angiotensin & aldosterone synthesis is increased enhancing salt water retention
- So increased glucose production (by glycogenolysis & gluconeogenesis) and decreased utilization of glucose results in
hyperglycemia. There is m arked increase in release of am inoacids from tissue protein, particularly from skeletal m uscle for
gluconeogenesis. H ow ever, synthesis of specific proteins m ay increase
- These m etabolic changes are coordinated w ith increased sympathetic nervous system activity causing peripheral
vasoconstriction and increased blood pressure.
374 ■ A Complete Review of Short Subjects

Stress

Any change in the environment that changes or threatens to change an existing optimal steady state.

1
TACTH I/tT glucocorticoids; which acts by • Activate sympathetic system 1/1
- Maintenance of vascular reactivity to catecholamines TCatecholamines
- Also necessary for catecholamines to exert their full • Increase glucagon and decrease anabolic
FFA - mobilizing action, and FFA are important hormones e.g. insulin & testosterone
emergency energy supply
i 1
Gluconeogenesis & Lipolysis

★ Hypophysedomized animals or adrenaledomized System Other effects of stress


animals treated with maintenance dose of Endocrinal - T ca ta b o lic horm on es eg catach olam in es,
glucocorticoids, die when exposed to the same co rtiso l & glucagon®
stress. However sympathectomized animals - 4 anabolic horm ones eg insulin &
tolerate stress with relative impunity. testosterone
+ Increase in ACTH secretion to meet emergency CVS Hypertension, tachycardia,? CO (4 in LVF)
situation are mediated almost exclusively through &T t p r
hypothalamus via release of CRH ( a polypeptide Hematology Hypercoagulability, Tplatelet adhesiveness,4
produced by neurons in paraventricular nuclei,
fibrinolysis
secreted in median eminence & transported in
Im munology Leukocytosis, Lym hopenia
portal- hypophyseal vessels to anterior pituitary
where it stimulates ACTH secretion. Inputs from
su prachiasm atic nuclei provides the drive for
diurnal rhythm®

Melatonin

Synthesis Regulation of secretion


__________ I____________
Melatonin & the enzyme responsible for its In humans & all other species, Exposure to darkness
synthesis from serotonin are present in m elaton in syn thesis & secretion I
pineal parenchymal cells. are in creased during the d ark Retinohypothalamic fibres
period® of the day & maintained I
Serotonin at a low level during day light Supra ch ia sm a tic nucleus in
hours. h y p oth alam u s activated® .
N- A cetyl transferase® The diurnal variation in secretion is I
+ Acetyl COA brought about by training of the Preganglionic sympathetic neurons
N - Acetylserotonin hypothalamus to light- dark cycle in spinal cord
H ydroxy in dole -O - via retino-hypothalamic nerves. I
m ethyl transferase® Hypothalamus causes Norepinephrine release from
+ S - Adenosyl norepinephrine secretion by postganglionic sympathetic nerves
______________ . . methionine (SAM) postganglionic sympathetic nerves I
Melatonin (nervi conarii) that innervate pineal Tintracellular cAMP in pineal gland
(N - Acetyl -5 - methoxytryptamine) gland. I
Norepinephrine acts Via f3 - TN - A cetyl tran sferase activity®
receptors in pineal gland to increase I
intracellular c AMP. cAMP fH ydroxyin dole - O - m ethyl
increases N-acetyltranferase transferase®
activity. Thus increases melatonin 4
synthesis & secretion. TMelatonin
Physiology: Endocrine System ■ 375

Somatomedins

Somatomedins are polypeptide growth factors, secreted by liver and other tissues, which mediate the effects of growth hormone
(somatotropin or STH) on growth, cartilage and protein metabolism.
Liver (and to a much lesser extent, other tissues) form somatomedins, when stimulated by somatotropin (or growth hormone).
That is why gwoth hormone supplied directly to cartilage chondrocytes cultured outside the body fails to produce
proliferation & enlargement of chondrocytes whereas, growth hormone injections into intact living animal does cause it.
Somatomedins are also k/a insulin like growth factors (IGFs) because many of its effects on growth are similar to effects of
insulin on growth.
At least 4 types have been isolated 3 of which are
Somatomedin A or insulin like growth factor 2 (IGF2) or multiplication stimulating activity (MSA)
- Somatomedin B derived from vitronectin
Somatomedin C or IGF 1 is the most important type & its concentration closely follows the rate of growth hormone
secretion
The principal & in humns probably the only circulating somatomedins are IGF1 & IGF-2. these factors closely relate to insulin,
except that their C chains are not separated and they have an extension of A chain called the D domain.
The activity that is not suppressed by antiinsulin antibodies is called non suppressible insulin like activity (NSILA). Most,
if not all, of this activity persists after pancreatectomy and is d/1 IGFland IGF II. However because of 100 fold weaker binding to
insulin receptor the insulin like activity of IGF 1 & 2 is weak enough to prevent diabetes mellitus after pancreatectomy. Insulin
also binds to IGF 1 receptors whereas IGFI & II bind to all 3 i.e. insulin, IGFI & IGF II receptor

r
Comparison of insulin & IGF Substances with insulin like activity in plasma
Insulin IGF-I IGF-II - Insulin
Name Somatomedin C Multiplication - Proinsulin
stimulating activity - Non suppressible insulin like activity (NSIL)
(MSA) d/t
No. of amino 51 70 67 1. Low molecular weight fraction i.e. small
acids amounts of free plasma IGF 1 and IGF2
Source Pancreatic (3 Liver, & other Diverse tissue
cells tissues 2. High molecular weight fraction i.e. large
Level Glucose Growth hormone amounts of IGFs bound to proteins
regulated by after birth,
nutritional status
Plasma 0.3-2 ng/mL 10-700 ng/mL 300 -800 ng/ml 5 characteristic features of somatomedin
levels peas at puberty
Secretion is independent of growth hormone
Plasma Absent Present Present before birth but is stimulated by growth
binding hormone after birth. Its concentration rises
proteins during childhood, peaks at puberty & then
Role Metabolism Skeletal & Growth during fetal declines to low levels in old age.
control cartilage growth development It has pronounced growth stimulating activity
Characteristics of IGF- II (epiphyseal growth & protein synthesis)
Hormone relaxin (both isoforms) resemble IGF-II Stimulates sulfate incorporation into cartilage
Largely independent of GH & plays a role in growth of fetus before birth. proteoglycan chondroitin sulfate®
If over expressed in fetus, growth of organs esp tongue other Insulin like effects on adipose tissue
muscles, kidney, heart & liver is disproportionate. (antilipolytic) and muscles
In adults IGF II gene is expressed only in choroids plexus & meninges Mitogenicity for fibroblasts
IGF II receptor has distinct structure & is involved in the intracellular IG I-1 receptor is similar to insulin receptor &
targeting of acid hydrolyses & other proteins to intracellular uses same intra cellular signaling pathways.
organelles

• Pygmies of Africa & Levi-lorian dwarf have a - congenital inability to synthesize somatomedin C in adequate amount.
Therefore, even though their plasma concentration of GH is either normal or high, they have diminished amounts of
somatomedin C 1/t small statures.
376 ■ A Complete Review of Short Subjects

• IGF I & II are tightly bound to proteins (IGF binding protein 3 or IGFBP3 accounts for 95% binding ) in plasm a, at least for IGF I
this prolongs their half life in circulation to about 20 hours. W hereas the growth horm one weakly binds to protein & has a
short half life of 20 minutes.
• It seems that GH & som atom edins can act both in cooperation & independently to stim ulate pathw ays that 1/t growth. GH
prom ote growth of muscle & bone. Its metabolic actions are counter regulatory to insulin as it induces glycogenolysis &
gluconeogenesis in liver to increase blood glucose and mobilizes free fatty acids (i.e. lipolysis). It inhibits glucose uptake by muscle
& fat cells, so more glucose is provided to brain & heart. Sim ultaneously, GH stim ulates lipolysis in fat cells providing free
fatty acids to heart & active skeletal muscle. But IG F I structurally sim ilar to insulin is capable of increasing glucose utilization
& decreasing lipolysis. These effects are opposite to those of acute, antiinsulin actions of GH.
G row th H orm one
1 1 1 i 1
Na+ retention D ecreased Lipolysis Protein Epiphyseal growth
in su lin synthesis
sen sitivity

IG F -1

Insulin like Antilipolytic Protein synthesis Epiphyseal


activity activity growth

Posterior Pituitary Hormones: ADH & Oxytocin

- Posterior pituitary gland (or neurohyophysis) is composed mainly of supporting, non (hormone) secreting glial like cells k/a
pituicytes which act simply as a supporting structure for terminal nerve fibers (& nerve endings) from nerve tracts that originate
in the supraoptic and paraventricular nuclei of hypothalamus. These tracts pass to posterior pituitary through the pituitary
stalk (hypophysial stalk).
- Hormones of neurohypophysis are actually synthesized in cell bodies (soma) of magnocellular (large) neurons in the
supraoptic and paraventricular nuclei of hypothalamus
- ADH (vasopressin) is formed primarily (5/6th part) in supraoptic nuclei® and oxytocin is formed mainly (5 /6 * part) in
paraventricular nuclei®. However, each of these nuclei can synthesize about l f 6 th as much
of the second hormone as of its primary hormone. In other words hypothalamus contains 2 types of magnocellular neurons -
some making ADH (predominantly found in supraoptic nuclei) and others making oxytocin (mainly found in paraventricular
nuclei); however both types of neuron (cells) found in both nuclei.
- Like other peptide hormones, neurohypophyseal hormones are also synthesized as part of larger precursor molecules. The
precursor for oxytocin, prepro-oxyphysin contains a 19 aminoacid (aa) signal /leader sequence f/b oxytocin (9 aa) and
neurophysin I (93 aa). Prepro-pressophysin, the precursor for arginine vasopressin is similar but larger molecule that contains
19 aa signal (leader) sequence f/b vasopressin (9 aa), neurophysin II and a 39 aa glycopeptide. The precursor molecules are
synthesized in ribosomes of cell bodies of magnoneurons. Their leader sequence is removed in endoplasmic reticulum, packaged
into secretory granules (or Herring bodies) in the golgi-apparatus and then are transported down the axon in these granules
in combination with carrier protein (neurophysins) by axoplasmic flow to the nerve endings in posterior pituitary, requiring
several days to reach the gland.
- During transport, cleavage of precursor molecules occur and the storage granule in the nerve endings contain free vasopressin
or oxytocin® and corresponding neurophysin (+ glycopeptide in case of vasopressin). All these products are secreted in blood
(Ganong)/Both neurophysin & hormone are secreted together, but because they are only loosely bound to each other, the
hormone separates almost immediately (Guyton). So vasopressin & oxytocin are typical neural hormones i.e. hormones
secreted into the circulation by nerve cells (neurosecretion)
- Oxytocin & vasopressin secreting neurons also generate & conduct action potentials, which on reaching to their endings trigger
release of hormones from storage granules by Ca*+ dependent exocytosis. At rest these neurons are silent or discharge at low,
irregular rates (0.1-3 spikes/s)
- If the pituitary stalk (Containing axons of magnocellular neurons) is cut above gland but the entire hypothalamus is left
intact, the posterior pituitary hormones continue to be secreted normally, after a transient decrease for a few days; however they are
then secreted by cut ends of fibers within the hypothalamus and not by nerve endings in posterior pituitary (Guyton-928). (Transient)
Decrease of ADH 1/t polyuria & diabetes insipidus. And similarly decrease of hypothalamic prolactin inhibiting hormone
(PIH), dopamine leads to an increase in circulating prolactin causing galactorrhea®.
Physiology: Endocrine System ■ 377

- Stimulation of vasopressin secreting neurons causes an initial steady increase in - Stimulation of oxytocin secreting neuron
firing rate f/b a prolonged pattern of phasic discharge /phasic bursting, in causes a synchronous, relatively short,
which periods of high frequency discharge alternate with periods of electrical high frequency discharge causing release
quiescence. In different neurons this phasic bursting is non-synchronous, well a pulse of oxytocin and consequent milk
suited to maintain a prolonged increase in the output of vasopressin. ejection in post partum females.

- Both ADH & oxytocin are polypeptides, each containing 9 amino acid with almost identical sequences except that in
vasopressin, phenylalanine, & arginine replace isoleucine and leucine of oxytocin in 3rd & 8lh positions respectively.
Vasopressin is arginine vasopressin (AVP) in most mammals, lysine vasopressin in hippopotami & most pigs (in it arginine
is replaced by lysine) and mixture of both arginine & lysine vasopressin in some pidgs & marsupials.

1 2 3 4 5 6 7 8 9
Cys—Tyr- Phe(V> -Gin—Asn—Cy s—Pro- •Arg (AVP)—
lie (O) Lys (LVP)
Leu (0)
- S ------- ---------------S -

+ V asopressin plays im portant role in BP regulation in hemorrhage & dehydration. Conivaptan (YM 0 8 7 ), a drug w ith both Via &
V 2 antagonist action is approved for treatm ent of hyponatrem ia.

- Vasopressin & oxytocin are also synthesized in the gonads & adrenal cortex and oxytocin is present in thymus. Circulating
vasopressin is rapidly inactivated (t Vi = 18 min) m ainly in liver & kidney.
• Control of vasopressin (ADH) secretion
- Hyperosmolarity or increased effective osmotic pressure of plasma (> 285 m O sm /kg) is the most sensitive & im portant
stim ulator o f ADH secretion. At 285 mOsm /kg effective osm otic pressure of plasm a, vasopressin is at or near the limits of
detection but below this is undetectable and when pressure is increased above 285, vasopressin secretion increases. So
osm olality of plasma is maintained very close to 285 m O sm /L in norm al individuals.
- V asopressin secretion is regulated by Osmoreceptors located in anterior hypothalamus. They are outside the BBB in
circum ventricular organs prim arily the organum vasculosum of lamina terminalis (OVLT). Osm otic threshold for thirst is same
or slightly greater than the threshold of ADH secretion (Ganong) Osm oreceptors (neurons) are located in the vicinity ofA V 3V
(antero-ventral region of 3rd ventricle) region and supraoptic nuclei. S u bfornical organ (SFO) is a structure at the upper part and
OVLT is a structure at the inferior part of AV3V. Between these two organs is median preoptic neucleus, w hich has m ultiple
nerve connections w ith SFO, OVLT, supraoptic nuclei and blood pressure control centers in medulla. Electrical stim ulation or
stim ulation by angiotensin II o f AV3V region can alter ADH secretion, thirst & sodium appetite (Guyton).

All 3 types of vasopressin receptors are G protein coupled

ADH Receptor Site of action Function 2nd messenger


V1A V ascular sm ooth Potent vasoconstrictor (invitro), relatively large amount is
muscle® needed to raise BP in vivo (b/o -ICO caused by action in brain) Phosphatidyl-inositol
Area postream of Decrease in cardiac output (CO) hydrolysis to increase the
brain intracellular
Brain stem & Neurotransmitter & involved in cardiovascular ( i CO) control Ca++ concentration
spinal cord
Liver Cause glycogenolysis
V IB = v 3 Anterior pituitary Increased ACTH secretion by corticotropes
V2 P rin cipal cells o f Exerts an tidiu retic effect® b y insertion of water channels, a Activation of adenyl
collectin g duct® protein k/a aquaporin-2 into the apical (luminal) membranes cyclase; acts through Gs to
of principle cells of collecting ducts. These channels are stored in crease C-Amp®
in endosomes inside the cells and ADH causes their rapid
translocation to luminal membranes. The movement of water
across membranes by simple diffusion is augmented by
aquaporins (13 i.e. AQPO to AQ P12 channels in almost all
tissues have been identified.)
378 ■ A Complete Review of Short Subjects

- ADH secretion is stimulated by either an increase is ECF osm olarity or decrease in effectiv e b lo o d volum e. However, ADH
secretion is con siderably m ore sen sitive to sm all changes in osm olarity® than to similar changes in blood volume. For
example, 1% Tin osmolarity or 10% Tin blood volume is required to ?ADH level.
- With severe decrease (>10%) in blood volume, cardiovascular reflexes (i.e., arterial baro receptor reflex & cardio pulmonary
reflex) play a major role in stimulating ADH secretion.
- The usual d ay to d ay regulation o f ADH secretion during sim ple dehydration is effected m ainly by changes in p la sm a
osm olarity® . However, decreased blood volume greatly enhances response.

Factors Affecting ADH Secretion

Increasing ADH (Vasopressin) Secretion Decreasing ADH (Vasopressin) Secretion


-H yper osm olarity® (Tplasma osmolarity) - Hypo-osmolarity (Tplasma osmolarity)
-Hypovolumia (Tblood /ECF volume) - Hypervolumia (Tblood / ECF volume)
- Hypotension (TBP) - Hypertension (?BP)
- Drugs : Clofibrate, Carbamazepine, Angiotensin - Drugs: A lcohol®
II Clonidine (antihypertensive drug)
- Nausea (100 times increase in ADH) Haloperidol (dopamine blocker)
-Vomiting
- Surgery, exercised,emotion, stress, pain

Relationship between T4 & T3

T4 T3
- Thyroid secretes more T4 than T3. - T3 is 5 times more potent than T4
- T4 is major circulating hormone. - T3 acts faster than T4.
- T4 is more tightly bound to plasm a protein (15 times)® - Tj is more avidly bound to nuclear receptors® than T4.
- H alf life o f T4 (6-7 days) is more® than T3 - About 1 /3 of T4 is converted to T3 in peripheral tissue.
(1-2 days).
It is believed that T3 is active hormone & T 4 is only transport form.

Effects of Thyroxine on Thyroxine Releasing Hormone


Oxidative O 2u p take o f a ll tissue®, excepting
- T3 stimulates in crease in TRH TRH stimulation testing
metabolism brain & testis. (hypothalamic - is useful in diagnosis of:
- Metabolism of all tissues (except brain & testis) is increased tripeptide) - Hyperthyroidism
i.e. fBMR®. stimulates - R ecurrent acrom egaly®
- T4 & T 3 causes calorigenesis (excessive colory production) release of:
Carbohydrate - T 3 increases gluconeogenesis - Gonadotropin secreting
- TSH tumour
metabolism - T 3 increase glycogenolysis
- Prolactin - Cases where
- These two factors causes elevation of blood sugar level.
Lipid - T3 reduces serum cholesterol by increasing its metabolism. - GH® documentation of
metabolism - T3 increase lipolysis® (adrenaline mediated) prolactin deficiency is
- Lipolysis result in increase FFA. necessary.
Protein T 3 enhances both catabolism & anabolism of proteins. But under
metabolism physiological condition anabolism> catabolism. Na* / 1' Symporter (NIS)
Skeletal - T 3 help growth of the bone, it does not act directly on bone cells.
system - T3 potentiate effect of a) GH,b) insulin like grow factor (IGF) The thyroid cell membrane facing the
on bone —>Bone growth capillaries contain a symporter that transports
- T 3 helps in bone maturation also. Na+ & F into the cells against the electro
CNS - Required for growth & development of brain during fetal & chemical gradient for F.
post natal period NIS is also present in:
- T3 is required for myelination of nerve fibers in brain - S alivary gland® - Gastric mucosa
CVS - T3 in creases m y cocard ial con tractility and h eart rate®. - M am m ary gland® - Placenta
(Plsympathetic receptor increase under the influence of T3) - Ciliary body of eye - Choroids plexus
Temperature - T3 causes calorigenesis — heat production but their uptake is not affected by TSH.
regulation
Physiology: Endocrine System ■ 379

Actions of Glucocorticosteroids (GC)


M etabolic Actions Permissive action on
It is an anti-insulin hormone that is catabolic to peripheral tissue & anabolic to liver. catecholamines
____________________ I______________ • D espite no direct bronchodilator or
I vasoactive property, GC is necessary
Carbohydrate M etabolism Lipid metabolism
for bronchodilation & vasoconstrictor
I I
action of catecholamine (in maintaining
It causes hyperglycemia (i.e. plasma - Increase lipohysis and liberate free
BP). It inhibits enzyme COMT
glucose level is increased and that is fatty acids (FFA) and glycerol, both of
(catecholamine-o-methyl transferase)
why it is called glucocorticoid) and which are increased in plasma
w hich inactivates catecholam ines.
makes diabetes worse by - Decrease esterifcation, I
• Sim ilarly GC is necessary for
1. Increased gluconeogenesis in liver triglyceride formation and I a calorigenic effects of catecholamines and
- as a result of enhanced hepatic glycogen P. glucagons.
catabolism of aminoacids (d/t - GC & thyroid hormones are • Osteoporosis®
induction of aminotransferases, facilitatory or permissive to other • Facilitate renal excretion of waterO
trypthophan dioxygenases / lipolytic hormones eg. epinephrine, • Glucocorticoids inhibit intestinal
pyrrolase alanine / tyrosine norepinephrine, glucagons, ACTH, absorption & enhance renal excretion
transaminase) which make more TSH, GH, a and ft melanocyte of Ca++Q
substrate available stimulating hormone (MSH) and • Antiinflam m atory action is d/t
- induction of key enzymes of vasopressin. Many of these activate decreased
gluconeogenesis e.g. pyruvate hormone sensitive lipase. - perm eability of capillary
carboxylase, PEP carboxykinase, fructose - release of lysosom al hydrolase (d/t
1-6 biphosphatase & glucose 6 Protein m etabolism stabilized lysosomal m em brane) &
phosphosphatase - Overall it causes negative mitrogen interleukin I from granulocytes.
2. Decreased glucose uptake & balance. - Form ation of bradykinin, PGs, PGI2,
utilization in peripheral tissue, except - In extrahepatic tissue (esp. muscles) Tx & leukotrienes (by inhibiting
the brain and heart. proteolysis (protein breakdown) is phospholipase A2)Q
3. Decreased glycolysis in peripheral increased 1/t increased availability • GC decrease number of eosinophils
tissue of amino acids in plasma. (by increasing their sequestration in
Glycogen deposition (or increased - It promotes amino acid uptake by liver spleen and lung), basophils and
glycogen synthesis) in liver enhancing gluconeogenesis & lym phocyte; reduce the size of
- as a result of increased glucose protein synthesis here. lym ph node & thym us; and reduce
formation from amino acids & - Inhibits amino acid uptake & 4- secretion of cytokines. It increases
glycerol (i.e. gluconeogenesis) and protein synthesis in peripheral num ber of RBC, platelet &
- Stimulation of glycogen synthase & tissue. neutrophils.
protein phosphatase 1 enzyme

Metabolic Actions Of Glucocorticoid


It is a catabolic hormone. Their function appears to be oriented to maintain blood glucose levels during starvation - so that
brain continues to get its nutrients.

I--------------------
Carbohydrate metabolism Protein metabolism Fat metabolism
- Promote glycogen deposition in -C au sep rotein breakdow n® & ■Prom ote lipolysis® (d/t glucagons, GH,
liver and promote aminoacid mobilization from adrenaline & thyroxine) and C-AMP induced
gluconeogenesis® peripheral tissues causing muscle break down of triglycerides i.e.is ketogenic
-They in h ibit glu cose wasting, lympholysis, loss of osteoid -Redistribution of body fat occurs.
u tilization by peripheral from bone and thinning of skin Subcutaneous tissue over extremities loses fat
tissue®, resulting in hyper -The amino acids are used up in which is deposited over face, neck & shoulder
glycemia, resistance to insulin gluconeogenesis —>excess urea is - moon face, fish mouth, buffalo hump This is
and a diabetes like state produced —» negative nitrogen because peripheral adipocytes are less
balance sensitive to insulin
In stress vasopressin® also increases Glucocorticoids
In adrenal insufficiency, the blood glucose is normal as long as food intake is maintained, but fasting precipitates
hypoglycemia & collapse. This is because glucocorticoids are necessary for glucagons to exert its gluconeogenic action
during fasting.
380 ■ A Complete Review of Short Subjects

Circadian (Diurnal) Rhythm of ACTH, Cortisol & Eosinophil

Suprachiasmatic nuclei (SCN) of hypothalamus is the biological clock


ACH - Maximum in
responsible for diumal (circadian) rhythm of ACTH, which intum leads to
early m oming
diurnal rhythm of cortisol (glucocorticoid) secretion. In humans, ACTH
- Least in
irregular bursts (episodes of high secretion) are most frequent in early
evening
mom ing and least frequent in the evening. Increased ACTH secretion is not Glucocorticoid
d/t stress of getting up in the morning as these bursts occur before waking
up. Even if the length of day is experimentally lengthened to >24 hrs ie if the
person is isolated and day's activities are spread over more than 24h, the I _Max= Evening
adrenal cycle also lengthens, but the ACTH burst still occurs during period of Eosinophil
sleep. The circadian rhythm of ACTH secretion is present in patients with Min = Morning
adrenal insufficiency receiving constant dose of glucocorticoids.
Cortisol decrease the number of circulating eosinophils by increasing their sequestration in the spleen & lung. So the
dium al variations in blood eosinophilic counts is inversely related to the circadian cortisol levels®. The eosinophilic
concentration in blood is < 450 / mL usually. A reciprocal relationship to plasma cortisol, causes the diurnal eosinophilic
concentrations to vary ~ 2 fold (40%) with eosinophilic count lowest in m oming and peaking in evening.

Aldosterone

Actions Regulation of secretion


• Increase the reabsorption ofN a* from the Increased Renin - angiotensin system is the principal regulator &
urine, sweat, saliva & the contents of the greatly increase aldosterone secretions.
colon. - Renin is secreted from juxtaglomerular cells that surround afferent
• Retention of Na+ in ECF expands ECF arteriole. Angiotensin II, is formed by the action of rennin, and it
volume. stimulates aldosterone secretion
• Facilitation of K * extrusion by the renal - Factors (1 ECF volume, hypotension) that increase renin secretion &
tubular cells. angiotension II formation that stimulates aldosterone secretion
[aldosterone causes Na conservation + K + Increased K+ (not Na+) ion concentration in ECF greatly increase & by
excretion ] far most important electrolyte in regulation of aldosterone secretion.
• In the kidney aldosterone act primarily on Increased Na+ ion concentration in ECF very slightly decrease aldosterone
principal cell (P cells) o f collecting duct. secretion.
ACTH is necessary for aldosterone secretion but has little effect in
Secretagogue Intracellular mediator controlling the rate. A small amount of ACTH secreted is enough to
ACTH Cyclic AMP, protein kinase A produce normal amount of aldosterone but total absence can significantly
Angiotensin II Diacylglycerol, protein kinase C reduce aldosterone secretion.
K+ Ca2+ via voltage-gated Ca2+ Endothein® & also epinephrine, nor epinephrine cause vosoconstriction of
channels efferent & afferent arteriole —»T GFR & i renal blood flow. It causes Ted
level o f plasm ANP, rennin & aldosterone®.
Stim uli that increase aldosterone secretion and

Glucocorticoid secretion also increased Glucocorticoid secretion unaffected


- Surgery - High potassium intake
- Anxiety - Low sodium intake
- Physical trauma - Constriction of inferior vena cava in thorax
- Hemorrhage - Standing
- Secondary hyperaldosteronism (in some cases of
congestive heart failure, cirrhosis, and nephrosis)
Physiology: Endocrine System ■ 381

Excess Mineralocorticosteroid (Aldosterone)

1
Increased ECF and arterial pressure but very small effect on Hypokalemia, M uscle weakness & alkalosis (Mild)
plasma Na+ concentration

A ldosterone cause K+ secretion into tubules


A lthough aldosterone has a potent effect in decreasing rate of inexchange for sodium reabsorption in principal
Na+ ion excretion, but plasm a Na* level is elevated only slightly cells of renal collecting tubules
if a t all because w ater is retained with osm otically active Nat­
A ldosterone causes hypokalemia not only by
ions®
causing loss of K+ ions from ECF into urine but also
Consequently ECF volume is expanded and blood pressure rises® stim ulating transport o f K + from ECF into cells.
When ECF expansion (& tB P ) passes a certain point, Na+ and Severe muscle weakness develops (at K+ < 2)
water excretion is usually increased (called pressure natriuresis because of altered electrical excitability of nerve &
& pressure diuresis respectively) inspite of continued action of muscle fiber m em brane
m ineralocorticoids on renal tubules. This aldosterone escape
A ldosterone also causes secretion of H+ ions in
phenomenon is due to increased secretion of ANP.
exchange for N a+ in intercalated cells o f cortical
Because of this escape, m ineralocorticoids do not produce edem a collecting tubules. This leads to decreased H+ ion
in norm al individuals & in hyper aldosteronism concentration in ECF causing mild alkalosis.
I__________________________ ________________________ I

So excess m ineralocorticoid leads to hypokalemic (K* depletion) alkalosis with weakness & tetany and Na' & water retention usually
with out edema but with hypertension & polyuria. It m ay be d/t

I
Primary hyperaldosteronism: Conn syndrome Secondary hyperaldosteronism

D/t prim ary adrenal disease such as adrenal hyperplasia (u/1 or Caused by cirrhosis, heart failure, nephrosis
b/1), adenom a of zone glom erulosa, adrenal carcinom a or GRA High plasm a rennin activity®
Renin secretion is depressed®

AME Syndrome & 11 p HSP type 2

Invitro (in lab), the m ineralocorticoid receptor has m uch higher affinity for glucocorticoids than the glucocorticoid receptor
does, & glucocorticoids are present in large am ount in body (vivo). But binding of glucocorticoids to mineralocorticoid
receptor (& so production of m ineralcorticoid effects by glucocorticoids) is prevented by presence of 11 P hydroxyl steroid
dehydrogenase tye 2 enzym e in m ineralocorticoid sensitive tissues. This enzym e leaves aldosterone untouched, but converts
cortisol to cortisone & corticosterone to its 11 oxy derivatives. These 11 oxy derivatives do not bind to receptor.

If 11 p hydroxysteroid dehydrogenase type 2 is congenitally absent or inhibited by prolonged ingestion of licorice


(containing glycyrrhetinic), cortisol has marked m ineralocorticoid effects resulting in apparent mineralocorticoid excess
(AME syndrome.
Patient with AM E have clinical picture of hyper aldosteronism because cortisol is acting on their mineralo corticoid
receptors, & their plasma aldosterone levels and plasma rennin activity is low.

Ovarian Reserve

It refers to the size of resting /nongrowing /primordial follicle population, w hich reflects the quantity (no) of growing follicles and
quality o f oocytes within it. So ovarian reserve determ ines the functional reproductive potential of ovary.
Under the influence of FSH and LH, there is rapid grow th of several follicles in ovary, during first few days of each menstrual
cycle. But after a week only 1 follicle begins to outgrow all the others /which involute by a process called atresia and thus
preventing m ore than one child from developing with each pregnancy).
O varian reserve (& capacity of ovary to produce eggs) decline with advancing age. Best predictor of ovarian reserve is age.
Tests that can determine ovarian reserve include
382 ■ A Complete Review of Short Subjects

1
FSH level on day 3 FSH level on day 10 after dom ifene citrate Serum-Inhibin B
■FSH levels are (antiestrogen) challenge test levels
elevated in an ■Inhibin B is secreted by
It is a provocative test 2-3 times more sensitive than day 3 FSH level. So
attempt to force the granulosa cells of ovary
it is the best screening test which unmask patients which might be
aging ovary to predominantly during
missed with a day 3 FSH level test.
respond. Cycle day 3 follicular phase (i.e. by
FSH levels are used In this test after measuring FSH & estrogen levels on cycle day 3 follicles) to inhibit FSH
for assessment (following the onset of menses), an antiestrogen clomiphene citrate secretion by pituitary.
because at this time 100 mg is given from day 5 to 9 and repeat estimation of FSH and
■So ovary with decreased
follicles produce low estrogen levels are done on day 10.
reserve will secrete less
levels of estrogen (& Being antiestrogen, clomiphene inhibits negative feedback of estrogen inhibin B (i.e. decreased
inhibin B) which has on hypothalamus, thus hypothalamus perceives false signals that serum inhibin B levels)
negative feedback on patient does not have enough estrogen. This induces pituitary to rise and thus leading to higher
FSH FSH & LH which inturn initiates follicular growth to produce more FSH levels.
High FSH indicates estrogen. Clomiphene can exert this effect only when ovarian reserve or
It is less sensitive than FSH
poor ovarian reserve estrogen level is low.
level tests.
and < 10 IU/mL FSH Positive test indicate low ovarian reserve & poor chance to conceive.
on day 3 predict Ultrasound scanning to
adequate ovarian FSH level Result count number of antral
oocyte reserve. follicles is more accurate
< 15 Good
than estimating ovarian
> 15 < 20 Border line volume.
>20. Poor

Estrogens
Structure and Synthesis Effects

- The naturally occurring estrogens - Estrogen ■Increased HDL and - Many studies show that hormone
are 17- (3 - estradiol, estrone & induced growth plasma triglyceride replacement therapy (HRT) has no highly
estriol requires level (d/t lipolysis deleterious effect in patients with NIDDM or
- They are secreted primarily by continuous & increased TG with impaired glucose tolerance test. Infact HRT
granulose cells of ovarian follicles, replenishment synthesis) significantly lowers H M jc levels in NIDDM.
corpus luteum and the placentaQ of energy, Reduced - It was progesterone component of OCP (with
- Their biosynthesis depends on mainly generated cholesterol & LDL higher doses) that produces abnormal glucose
arom atase enzyme ® which converts by glycolysis Estrogen causes fa t tolerance tests with elevated insulin level or
testosterone to estradiol. - It regulates to be stored in worsening o f NIDDM.
- They are also formed from GLUT I buttocks, thighs & - At physiological levels, estrogen and
aromatization of androstenedione expression and hips in women.® In testosterone apparaently increase the insulin
in the circulation. induction o f post menopausal sensitivity® and increase insulin dependent
A n d r o s te n e d io n e ; ^ T e sto stero n e glycolytic women with metabolic processes eg tissue glucose uptake
enzymes® decreased estrogen & lipid synthesis (where as progesterone has
Aromatase > A rom ataseQ \ ■It promotes production, fat the opposite effect). However outside the
energetic migrates to physiological window these steroids may
E stro n e E stra d io l capacity of abdomen (central prom ote insulin resistance.®
mitochondria obesity). Men store - Post menopausal women who take estrogen
by maximizing fat in abdomen d/t replacement therapy (ERT) are less likely to
E strio l
aerobic sex hormone develop diabetes; and if they do have disease,
glycolysis®. difference. are able to maintain good blood sugar
control.
Physiology: Endocrine System ■ 383

Sertoli Cells (S C)
SC are large, complex glycogen containing cells that lines the wall of seminiferous tubules (of testis) along with primitive germ
cell. They all contact basal lamina and stretch all the way to lumen i.e. they extend from basal lamina to lumen where their apical
plasma membranes form complex recesses which envelop spermatids & spermatozoa until the latter are mature enough for
release.
SC form a major cellular component of tubule before puberty & in elderly. They are supporting, non spermatogenic cells 0 of
seminiferous tubules.
Long cell process of SC extends between spermatogonia in basal compartment (near basal lamina) and between
spermatocytes in adluminal (near lumen) compartment
Its nucleus is euchromatic, irregular or pear shaped, contains 1-2 prominent nuclei, & is usually aligned perpendicularly to the
basal lamina. The cytoplasm is rich in lysosome, consistent with its phagocytic action
SC have high affinity FSH receptors0 that bind FSH & stimulates production of ABP which concentrates testosterone.
Spermatogenesis requires presence of FSH & high intertesticular levels of testosterone.
Functions of sertoli cells are

Germ cells must stay in contact with SC to I M ullerian Inhibiting (Regression) Factor
survive 0 & this contact is maintained by Occluding tight junctions / Substance (M TS). SC secrete MIS which
cytoplasmic bridges between sertoli cells cause regression o f Mullerian
SC provide trophic mechanical support to forms the BTBQ (paramesonephric) ducts in m ales during
germ cells through out the stages of BTB protects the developing feta l lifeQ.
spermatogenesis spermatogenic cells from Inhibin . SC are under control of FSH
M aintenance or Nourishment: SC are rich blood borne noxious agents & from anterior pituitary; and in turn secrete
in glycogen and provide a source of immunological attack. inhibin which by negative feed back
nourishment to spermatogenic series Steroids (eg androgen from inhibits FSH releaseQ
Phagocytosis of excess cytoplasmic interstitial cells of Leydig) Contain Enzyme Aromatase (C Y P -19)
fragments (residual body) during and maturing germ cells can which converts androgens into oestrogens
spermiogenesis penetrate this barrier with
Release of spermatozoa: SC participate in ease while passively Exocrine functions
release of sperm by extruding axial diffusing macromolecule & Synthesis & seretion o f Androgen Binding
components of spermatid while holding & small ions, are prevented protein (ABP): ABP binds and
phagocytizing lobules of their cytoplasm from passing from the concentrates the androgens
Play important role in controlling blood into lumen (testosterone)Q secreted by Leydig cells. It
sperm atocyte & sperm atid differentiation & causes great concentration of testosterone
m aturation 0 (Gray's - 1267) in seminferous tubules. A high
concentration of androgens is essential for
Late spermatids the production & maturation of
Early spermatids
spermatozoa
Secretion of seminal fluid: SC secrete a
Spermiogenesis
fluid into seminiferous lumen rich in K+ ,
Meiosis
HCO 3 , androgen, oestrogen, inositol &
glutamic and aspartic acid but low Na+, Cl-
Spermatogonium (like ICF), —protein, glucose. It contains
androgen binding protein, steroids
(testosterone) and inhibin. Maintenance of
its composition depends on BTB and it is
necessary for onward transport of
spermatozoa
Secretion of H-Y antigen

★ S e rto li cells h a v e r e c e p t o r s f o r F S H , a n d te s t o s te r o n e Q . A fte r c o m b in in g w ith F S H , se rto li c e lls stim u la te s the f i r s t h a l f o f


s p e r m a t o g e n e s i s Q. S u b se q u e n tly testosterone - sertoli cell b in d in g c a u se s d e v e lo p m e n t o f la st h a lf o f sp e rm a to g e n e sis . T e sto ste r o n e is
c o n v e rte d to 5 a d ih y d ro te sto ste ro n e b y se rto li cells.
★ Interstitial cells of Leydig are lipid containing cells present in between the seminiferous tubules in testis.
Leydig cells secrete testosteroneQ into the bloodstreem.
384 ■ A Complete Review of Short Subjects

Onset of Puberty

• Puberty begins earlier in girls. Because both sexes show a similar increase in GnRH and LH at the onset of puberty, whereas
FSH secretion shows sexual diamorphism i.e., its secretion varies between both sexes in childhood & early puberty. It has been
suggested that differences in the tim ing o f on set o f puberty b/w b oth sexes is due to differences in FSH seretion®.
• FSH secretion is under regulation of FSH regulatory p ep tid es (& GnRH). So the FSH regulatory p ep tid es w hich stim u late
FSH secretion cause early on set o f puberty w h ereas puberty is d elay ed by p ep tid es th a t in h ibit FSH secretion®. FSH -
regulatory peptide

W h ic h in h i b i t F S H s e c r e tio n & d e la y W h ic h s tim u la te F S H s e c r e tio n &


p u b e rty c a u se e a r ly o n s e t o f p u b e rty

- Inhibin® Activin A®
- Follistatin®

★ So T a c tiv in - A c a u se s e a rly p u b e rty a n d T in h ib in & T fo llista tin d e la y p u b e rty .

Spermatogenesis

• Spermatogenesis occurs in germinal epithelium o f seminiferous tubules®


• Epididymis reabsorbs 99% o f testicular flu id and concentrates and stores spermatozoa®. After a 18-24 hour stay in
epididymis sperms develop the capability o f motility® which is important for fertilization in vivo. But fertilization occurs in
vitro even if an immotile spermatozoa from head of epiddymis is microinjected directly into an ovum.
• Out of 120 million sperms formed each day by both testes, only a sm all quantity can be stored in epididymis but m ost are
stored in vas deferens®.
• Progressive motality (i.e. ability to move forward) is acquired in epididymis by activation of Cat sper protein localized to
principal piece of sperm tail. This protein is a Ca++ ion channel that permits cAMP generalized Ca++ influx.
• Ejaculation involves P2 x receptor ligand gated cation channels (that respond to ATP) mediated contraction of vas deferens
• Normal motile, fertile sperms are capable of flagellated movement at 1- 4 mm/min velocity. Its activity is greatly enhanced in a
neutral & slightly alkaline medium and with increasing temperature (which also increases metabolism rate & so decreases
life span). Mildly acidic medium decreases activity & strongly acidic medium causes rapid death.
• Although sperm can live for many weeks in male genital ducts, once ejaculated their maximum life is 1-2 days in fem ale genetal
tract®. When frozen at temperatures below - 100°c, sperms can be preserved for years.
• Seminal vesicle provide the bulk (60%) volume o f semen® that flushes the spermatozoa & prostatic secretion out of urethra.
It is rich in fructose, vitamin C (citric acid), K+, fibrinogen, prostaglandins and flavins (giving yellow color). Prostaglandins aid
fertilization by
Making female cervical mucus more receptive to sperm movement
Causing backward reverse peristaltic contractions in uterus & fallopian tubes to move the ejaculated sperm towards the
ovaries.
• Prostatic fluid is rich in calcium, citrate ion, phosphate ion, a clotting enzyme and a profibrinolysin and it provides slightly
alkaline medium by neutralizing the acidity o f other seminal fluids & vogina®. It is important to note that fluid of vas
deferens is acidic d/t presence of citric acid & metabolic end products of sperm and so are vaginal secretions (pH 3.5- 4).
Sperms are not optimally motile until pH of surrounding fluid is 6- 6.5 Spermatozoa express olfactory receptors & ovaries
produce odorant molecules (chemotaxis)
• After ejaculation spermatozoa undergo two processes in female genital tract to acquire the capability of fertilization.
Physiology: Endocrine System ■ 385

Capadtation Acrosome Reaction

- It is a period of conditioning o f sperms in fem ale reproductive - When ovum is released , it still carries with it
tract particularly in isthmus o f uterine tubesQ multiple layer of granulose cells. Before a sperm
- It normally requires 1-10 hours and 1/t some changes that are can fertilize the ovum, it must dissolute the
granulose cell layer and thick covering of ovum zone
1. Uterine & fallopian tube fluids wash away the inhibitory factors that
pellucida.
suppress sperm activity in male genital ducts
- Hyaluronidase stored in acrosome of sperm
2. Sperm gradually lose much of their excess cholesterol that has been
depolymerizes the hyaluronic acid polymers in
continually added to the cell membrane covering the sperm
intercellular cement that hold ovarian granulose
acrosome, toughening this membrane & preventing release of its
cell together.
enzymes. So loss of cholesterol weakens the membrane at the
head of sperm. - When sperm reaches zona pellucida, its anterior
membrane binds with receptor proteins and then
3. Increased entery of Ca++ ions d/1 increased membrane permeability,
rapidly the entire acrosome dissolves releasing all
changes the activity of flagellum, giving it a powerful whiplash
the acrosomal enzymes. Within minutes, these
movements in contrast to its previously weak undulating
enzymes open a penetrating pathway through zona
motions.
pellucida to inside the ovum
4. Ca++ ions change acrosomal cell membrane, so that it can release
its enzyme rapidly & easily during acrosomal reaction.

Hormones Stimulating Spermatogenesis

Testosterone (Androgen) Follicle stimulating Estrogens Leuteinizing Growth Hormone


I Hormone (FSH) I Hormone (LH) (GH)
Secreted by Leydig cells I Formed from testosterone I I
located in interstitiumof - Secreted by anterior by sertoli cells when they - Secreted by Promotes early
testis is essential for growth & pituitary gland and are stimulated by FSH anterior pituitary division of
division of testicular germinal acts on sertoli cells Is probably essential for gland & spermatogonia
cells, which is the first stage to facilitate the last spermiogenesis stimulates leydig In its absence, as in
in forming sperm stages of spermatid Estrogen content of fluid cells to secrete pituitary dwarts,
The stages from maturation i.e. the in rete testis is high & the testosterone spermatogenesis is
spermatogonia to spermatids conversion of walls of rete contain - After severely deficient or
appear to be androgen spermatids to sperm numerous ER a esterogen hypophysectomy, absent, thus causing
dependent (the process of receptors, In this region, injection of LH infertility.
Maturation of spermatids to spermiognesis) fluid is reabsorbed & produces a high
spermatozoa depends on - Promotes the spermatozoa are local
angrogen acting on sertoli production of concentrated. If this does concentration of
cells in which the developing androgen binding not occur, the sperm angdrogen in
spermatozoa are embedded. protein (ABP) entering the epididymis testes, & this
I______ I are diluted in large maintains
I spermatogenesis.
volume of fluid, &
FSH & androgens maintain the gametogenic function of
infertility results.
testis

Testosterone & Male Sterility


The fact that intratesticular levels of testosterone need to be greater than 100 folds higher than circulating levels of
hormone to maintain normal rates of spermatogenesis; however, it is the circulating levels of testosterone that provide the
negative feedback to pituitary & hypothalamus is an important loophole in male reproductive axis.
This means that exogenous testosterone (T) administration can raise circulating levels of T. sufficient to inhibit LH but not
sufficient to accumulate in testis at required concentration for normal spermatogenesis. However, decreased LH levels will
further diminish intratesticular production of testosterone by Leydig cells resulting in reduced spermatogenesis.
This is the cause for sterility in cases of prolonged testosterone adm inistration or steroid abuse in men 0. And this loophole
is recently being researched as a possible strategy for developing male oral contraceptives.
386 ■ A Complete Review of Short Subjects

Contraction Stress Test (or Oxytocin Challenge Test)

It is an invasive method to assess fetal well being during pregnancy, when there is alteration in FHR in response to uterine
contraction. It suggests fetal hypoxia.

□ Procedure Indication Contraindication


Oxytocin is infused and the alteration in FHR during uterine . IUGRQ • Compromised fetusQ
contraction are noted. • PostmaturityQ • Previous history of
• Hypertensive caesarean section^
□ Interpretation disorder of • Complication likely
P o sitiv eQ - P ersisten t la te d eceleration o f FHRQ pregnancyQ to produce preterm
- It is associated with Ted incidence of IUD, fetal • DiabetesQ labourQ.
distress & low Apgar score. . APHQ
N egativeQ - No late deceleration or significant variable
deceleration
- associated with good fetal outcome.

Congenital Adrenal Hyperplasia

Types M ain function of Mineralo Gluco Androgen Features


enzyme cortcoid corticoid
(M) (G)
21- OH- Glucocorticoid & I u T • HypotensionQ
deficiency Mineralocorticoid (salt losing virilization)
production • VirilizationQ in females
• Precocious puberty in femalesQ.
11 Glucocorticoid T u T • Hypertension^
hydroxylase- production • VirilizationQ in females
deficiency • Precocious puberty in femalesQ.
17- Glucocorticoid & T I 11 • Male hermaphroditeQ
hydroxylase Androgen • Primary amenorrhea in femalesQ.
deficiency production • HypertensionQ
3 -P - Mineralcorticoid & I I T • Male hermaphrodite.
hydroxylase glucocorticoid
deficiency production.

Summary

I
Hypertension (is seen in) M ale Pseudohermaphroditism Female Pseudohermaphroditism
I (present in) (present in)
17 hydroxylase deficiency I I
11 Hydroxylase deficiency 17 hydroxylase deficiency 21 hydroxylase deficiency
3 - hydroxylase deficiency 11 hydroxylase deficiency
Physiology: Endocrine System ■ 387

Amniotic Fluid: Composition & Volume

Liquor amnii is derived from filtration (from maternal vessels) across amniotic membrane Water = 98-99%
(most important in early stages so AF resembles blood plasma), filtration from fetal Solid = 1-2%
vessels via umbilical cord, secretion from amniotic epithetiun,/efa/ urine & fetal lung - Organic solids include lipids
secretions (50mg%), NPN (30mg%),
Anount of AF increases upto 6-7 months and then diminishes slightly to become <1 lit at glucose (20mg%), uric acid
the end of pregnancy. Deficiency of AF is oligohydroamnios and usually result from (4mg%), protein (0.3 mg%),
premature rupture of membrane, uteroplacental insufficiency or urinary track hormones (eg prolactin,
malformations like renal agenesis. Absent AF is ahydroamnios. Oligohydroamnios at insulin & renin), IGF-BP1,
< 20 weeks may cause pulmonary hypoplasia. fibronectin and urea.
Polyhydroamnios is > 2 lit AF. It may occur d/t reduced fetal swallowing (owing to - Inorganic solids like Na+, Cl"
anencephaly, oesophegeal / upper gi atresia, compression d/t cong. diaphragmatic and K+.
hernia) or decreased AF absorption and increased fetal urination.

______________________ Placentral Hormones and Pregnancy______________________

Endocrine function begins with the onset of implantation, when syncytiotrophoblasts start secreting the LH like protein hCG
(human chorionic gonadotropin), which prevents luteolysis (i.e. maintains the viability of corpus luteum) and thus maintains
a high level of luteal derived progesterone production during the first 10 weeks. By this time (8-10 weeks), the
syncytiotropholasts aquire the ability to make progesterone at sufficient levels to maintain pregnancy independently of
corpus luteum (- referred to as luteal - placental shift).

hCG has a a glycoprotein subunit (a-GSU) and a hormone specific (3-sub unit ((3-hCG). Antibody used to detect hCG in
pregnancy tests and laboratory assay detect the fi subunit®. Being similar to LH, hCG binds with high affinity to LH
receptors. But the (3 subunit of hCG is longer than that of LH & contains more sites for glycosylation, which greatly increase
the stability (ti /2 = 24-30 hours) allowing it to rapidly accumulate in maternal circulation such that the hCG is detectable
within maternal serum within 24 hours of implantation.
Maternal serum hCG level doubles every 2 days for first 6 weeks & peak at about 10 weeks, then decline to a constant level at
about 50% of the peak value. This rapid increase is responsible fo r the nausea o f morning sickness® a/w early pregnancy.
Small (1-10%) amount of hCG enters fetal circulation and stimulate fetal leydig cells (to produce testosterone before fetal
gonadotropic axis is fully mature) and fetal adrenal cortex.

Human placental lactogen (hPL) also k/a human chorionic somato mammotrophin (hCS) is a protein hormone
(structurally similar to GH and prolactin /PRL) that is produced in syncytiotrophoblast. It can be detected within
syncytiotrophoblast by 10 days after conception & in maternal serum by 3 weeks of gestation. Maternal level rises
progressively during gestation (related to size of placenta). It is protein anabolic & lipolytic (like GH) and stimulates
mammary gland growth & development (like PRL). Its antagonistic action to insulin is the major basis for diabetogenicity
(insulin resistant state or accelerated starvation) of pregnancy. Mammary gland development in pregnancy results from
action of hPL, PRL, estrogen & progestins. Both hPL & PRL act as fetal GHs & stimulate production of fetal growth
promoting hormones insulin like growth factors (IGF) I & II.
Placenta (syncytiotrophoblast) also produce progesterone, estrogen and corticotropin releasing hormone (CRH). Progestore
production by placenta is largely unregulated (i.e. produces as much as the supply of cholesterol & levels of CYP 11 A l & 3(3-
HSD allow). Placental steroidogenesis differes from that in adrenal, cortex , ovaries & testis in that cholesterol is transported
into the placental mitochondria by mechanism that is independent of St AR protein. This first step of steroidogenesis is not
regulated, rate limiting step in placenta as it is in other steroidogenic glands. So fetus with inactivating mutation in StAR
protein will develop hypogonadism, lipoid congenital adrenal hyperplasia but will produce normal progesterone levels by
their placenta. Progesterone production by placenta does not require fe ta l tissue and are largely independent o f fe ta l health®
(i.e. cannot be used as a measure o f feta health)®.
388 ■ A Complete Review of Short Subjects

Nitric Oxide (NO) / Endothelium Derived Relaxing Factor (EDRF)

I 1
Synthesis and Metabolism 3- isoforms or subtypes of NOS are

NOS- Found in Activated by Feature Result of gene


- Nitric oxide (NO) is synthesized from L-arginineQ in a
Subtype Knock out in
reaction catabolized by cytosolic enzyme nitric oxide
Mice
synthase (NOS)Q. Details of the reaction are as following.
NOS-1 = n- Nervous Ted Ca2+- Pyloric stenosis,
Substrates Coenzymes Enzyme Mechanism Products NOS (neural) (increased) calmodulin resistance to
L- FMN Cytosolic 5e- Citrulline (Constituti tissue intracellular
dependent, vascular stroke,
ArginineQ FAD NOS oxidation of NO ve calcium calcium constitutiv aggressive
Oxygen Thiol (Ganong) (e/n/i) amide dependent, concentration e enzyme sexual
( 0 2) BH4 (tetra isoform. nitrogen of neuronal- activate NOS (i.e behaviour
NADPH hydro arginine NOS) n & e. synthesized (males)
biopterin)
So vasodilator at a constant indicating its
NADPH,Heme rate role in smooth
agents like
(Lippincot).
acetyl choline regardless muscle
(ACh), of relaxation.
NOS-3 = e- Endotheli bradykinin, demand). Elevated mean
NOS al cells Shear stress, NOS- n & e blood pressure
VIP, substance constantly
Ach bradykinin L-Arginine + 0 , + NADPH Sildenafil citrate (Constitutiv (hypertension)
Shear stress VIP (viagra) inhibits P and prodce low
Substance P C ° " ^ [N O S i] | E Jg ? phosphodiesterase and e, calcium indicating its
histamine via levels of NO.

t
Histamine via H ,- slows down break down
receptor
f NADPH, Heme
ofcG M P dependent, role in
Serotonin L-Citrulline + NO + NADP H1 receptors
endothelial maintaining
which act via
NOS) normal BP in
Vasodilator nitrates such as T Soluble endothelium
nitroglycerine and NO Q - * cytoplasmic
nitroprusside guanylyl
activate NOS
cydase I Phospho
cGMP
dle9terase by T calcium
J NOS-2 = i- Immune Not activated by Ca2+ (i.e. More
Smooth muscle relaxation Smooth Inactivate NOS cells calcium independent susceptible to
c-GMP
muscle cell
(Inducible, including enzyme) but is induced by certain types of
calcium macropha inflammatory cytokines, infection.
- NO is inhibited by hemoglobin and other heme proteins, independe tumor necrosis factor a-
&e>
which bind it tightly nt, immune monocytes (TNFa), yinterferon,
- NO has very short half life (ti/2) in tissues (3-10 seconds) NOS) t bacterial lipopolysaccharide
because it reacts with oxygen and superoxide and then neutrophil and bacterial endotoxins
converted into nitrates and nitrites including peroxynitrite s& (inducer vary with cell
(ONOO~), a reactive nitrogen species (RNS), which hepatocyte type). Induction results in
decomposes to form highly reactive OH* radical. large amount of NO
being produced over
hours or days.

Functions

• Smooth M uscle relaxation (of vessels & gi tract mainly).


- NO synthesized by e NOS in endothelial cells, diffuses to vascular smooth muscle and activate cytosolic guanylate (guanylyl)
cyclase (catalytic action), resulting in rise of c-GMP levels. Cyclic GMP activates protein kinase G, which phosphorylates Ca++
channels, causing decreased enetry of Ca2+ into smooth muscle cells. This decreases the calcium - calmodulin activation of
myosin - light chain kinase, thereby decreasing smooth muscle contraction and favouring relaxation.
- Acetyl choline (Ach), bradykinin, vasoactive intestinal peptide (VIP), substance P, serotonin, histamine via Hi receptors
and shear stress (d/t rapid blood flow in arteries & arterioles) act indirectly via endothelium cells to produce endothelium
derived relaxing factor (EDRF) or NO (by increasing Ca++ concentration) and causing vasodilation. That is why vasodilation
caused by Ach etc can be abolished if endothelium cells are stripped from underlying smooth muscle. However, adenosine,
atrial natriuretic peptide (ANP), and histamine via H2 receptors produce relaxation of vascular smooth muscle that is
independent of endothelium.
- When flow to a tissue is suddenly increased by small arteries and arteriolar dilation (d/t local mechanism like O 2 deficiency),
Physiology: Endocrine System ■ 389

the large arteries to tissue also dilate d/t shear stress or viscous drag of blood against the vascular walls of arterioles relasing
NO.
Vasodilator nitrates like nitroglycerine & nitroprusside are metabolized to produce NO (endogenous nitrovasodilator),
causing vascular smooth muscle relaxation and lowering BP.
Resistance vessels (small arteries & arterioles) are in continous state of No mediated vasodilation. NO also controls resting
vasodilator tone of in central veins but not in peripheral veins.
Inhibitors of NO synthase 1/1vasoconstriction and elevation of BP, indicating its importance in maintaining BP in vivo.
Inhibitors of NO synthase 1/t vasoconstriction and elevation of BP, indicating its importance in maintaining BP in vivo.
Release of NO with consequent vasodilation and engorgement of corpora cavernosa 1/t penile erection. This accounts for
efficacy of sildenafl citrate (viagra) which slows the break down of cGMP by inhibiting phosphodiesterase (used for erectile
dysfunction)
NO, VIP, PACAP and ATP are inhibitory neurotransmitters of enteric motor neurons that mediate gastrointestinal smooth
muscle relaxation. Low level of NO is involved in causation of pylorospasm in infantile hypertrophic pyloric stenosis®.
Inhibits adhesion, activation and aggregation of platelets
Products of platelet aggregation cause NO release and resulting vasodilation keeps blood vessels with intact endothelium
patent. Whereas endothelial damage produces vasoconstricton and platelet aggregation.
NO is involved in vascular remodeling, angiogenesis and pathogenesis of atherosclerosis. It is seen that some patients of heart
transplant develop an acclerated form of atherosclerosis in the vessels o transplant.
Constitute part of primitive immune system by mediating macrophage bactericidal activity
Induced by inflammatory mediators i-NOS produce NO in large amounts which decomposes to, highly bactericidal OH*
radical.
Mediates tumoricidal & cytotoxic actions
Gaseous neurotransmitter in brain & peripheral autonomic system.
NO is produced by nerve terminals in granular cell layer of cerebellum and hippocampus and seem to be involved in long
term behaviour and potentiation of memory.
Gaseous neurotransmitter NO differs from other small molecule transmitters in that it is neither preformed & packaged /stored
into synaptic vesicles in pre synaptic terminals nor released by exocytosis. Instead, it is synthesized almost instantly as needed, is highly
permeant & simply diffuses out of presynaptic terminals to neighbouring cells (eg post synaptic neurons) over period of
secons. Their synthesis is triggered by depolarization of nerve terminal (Ca++ influx).
There are no specific reuptake mechanisms nor do they undergo enzymatic destruction, so their action appears to be ended by
diffusion or binding to superoxide anions or scavenger proteins,
NO usually does not alter greatly membrane potential but instead changes intracellular metabolic functions that modify
neuronal excitability for seconds/minutes / or longer in post synaptic neuron.
NO is a neurotransmitter at synapse b/w inhibitory motor neurons of enteric nervous system and gastrointestinal smooth
muscle cells - causing smooth muscle relaxation in gitract®.
NO has role in neurotoxicity.
May have role in skeletal muscle relaxation
Chapter 8. GENERAL PHYSIOLOGY: REVIEW NOTES

Resting Energy Basal Metabolic Rate (BMR)


Expenditure (REE) The metabolic rate determined at rest, at least 12 hours after the last meal after a night
or of restful sleep, at an ambient temperature of about 25°C (68°-80°F) and the subject is
Resting Metabolic Rate free from any illness (physical or psychic factors that cause excitement). It represents
_________(RMR)_________ the energy required for normal body function. No sternous activity is performed for at
least 1 hour before test & no physical activity is permitted during the test. BMR is
It is amount of energy required to
expressed as K cal energy consumed per m 2 body surface area per hour Q. BMR is
maintain essential physiological
directly proportional to total body surface area
functions of body to maintain life.
From 50% to 70% of daily energy expenditure in sedentary individual is attributable to
REE is energy expanded by body
BMR until very old age. Factor affecting BM R
during resting state without fasting,
Primary factor affecting (and most closely related to) BMR & REE is lean (fat-free)
whereas, BMR is the energy
body mass, which gives an estimate of metabolically active tissue (i.e. muscle & internal
expanded during resting & fasting
organ). Body surface area is a reasonable determinant of lean body mass. Katch - Me
state
Ardle formula is used to calculate BMR is
So REE is slightly higher than BMR
but the factors that influence REE B M R = 3 70 + (21.6 X L e an b o d y m a ss in kg)
are essentially the same factors that
influence the BMR. Other factors affecting BMR & REE are

REE = BM R + Energy used Factor Increasing (T) BMR & REE Decreasing (1) BMR & REE
in digestion of food A ge E a rly a g e (i.e. n e o n a te s & c h ild ren ) M o st o f th e d e clin e in B M R w ith in c r e a s in g
- Recently ingested food increase the a g e (i.e. a d u lts & old ag e ) is re la ted to
metabolic rate because of their re p la ce m e n t o f m e ta b o lic a lly a c tiv e m u scle
m a ss w ith a d ip o se tissu e, w h ich h as a lo w e r
specific dynamic action (SDA).
ra te o f m eta b o lism .
The SDA of a food is the obligatory
Sex M a le s F e m a le s (b / o lo w e r p e rce n ta g e o f m u cscle
energy expenditure that occurs
m a ss & h ig h e r p e rce n ta g e o f a d ip o se tissu e).
during its assimilation into the G r o w th P re g n a n cy , la c ta tio n , g r o w in g age
body as a result of different & re p ro d u ctio n .
chemical reactions associated with F e e d in g R ecen t in g e s tio n o f fo o d esp d iet F a s t in g , s t a r v a t i o n , m a ln u t r i t io n ( J u p t o 40%
digestion, absorption & storage of co n ta in in g p ro tein s. d u r in g p r o l o n g e d s t a r v a t i o n )Q
food in body. This is called E n v ir o n m e n ta l C o ld o r v e ry h o t (u n c o m fo rta b le ) H o t c o m fo rta b le c lim a te
thermogenic effect of food te m p e ra tu re tem p eratu re .
because these processes require C u rv e re la tin g B M R to te m p era tu re
i.e. 'U ' sh a p e d i.e. B M R rise s w h en
energy & generate heat. It accounts
e n v iro n m e n ta l te m p era tu re is b o th
for 8% of total daily energy SDA is
b elo w o r h ig h e r (ab o v e) b o d y
5% fat, 6% for carbohydrate and 30% te m p era tu re
for proteins (i.e. it takes 30 K cal to Body F e v e r raise s B M R (14% fo r each
assimilate amount of protein te m p e ra tu re d e g re e celsiu m elev atio n ).
sufficient to raise metabolic rate H o rm o n e s T h y r o x in e (k ey re g u lato r),
100 K cal. c a te c h o la m in e s (e p in e p h rin e &
- The primary factor affecting REE n o re p in e p h rin e ), G ro w th h o rm o n e ,
& BM R is fa t free lean body m ass Q m ale sex h o rm o n e s. E g h y p e r
t h y r o id is m o r t h y r o t o x i c o s i s 0 .
(i.e. metabolically active muscle &
O th e rs S m o k in g , C a ffe in e , m u s c u la r S lee p (falls - 10% ), O b e s i t y d (d / t h ig h fa t /
internal organs). Fat (adipose
e x e rc is e (e x e rtio n ) d u rin g o r ju st m u sc le ratio ). S k e le ta l m u s c le s ev en u n d e r
tissue) is metabolically inert & b e fo re m e a su re m e n t, em o tio n a re stin g c o n d itio n s , a c co u n t fo r 20 -3 0 % o f
does not contribute to BMR or h y p e ra ctiv ity & stress. BM R
REE. Body surface area is a H e ig h t, T a ll thin S h o rt o b ese
reasonable determinant of lean w e ig h t, B M I B o d y su rfa c e area (B SA ) can b e sim ila r in b o th b u t B M R d iffers. B M R h a s h ig h e s t
body mass. c o rre la tio n w ith le a n b o d y m a ss (L B M ) > B o d y m a ss > B S A > B o d y w e ig h t >
- In determining metabolic rate, O 2 BM I > H e ig h t (r is 0.95, 0 .8 5 , 0.84, 0 .8 0 , 0.64 & 0 .5 7 re sp e c tiv e ly )
|BMR = 3 .5 2 W l,75|is h ig h e r th an it w o u ld b e if it w a s d u e so le ly to b o d y are a th at is
consumption is measured & then
B M R = 3.5 W " "
converted to energy production by
multiplying 4.82 Kcal/ L of O2
consumed.
General Physiology ■ 391

Age Related Physiological Changes

1. General Proportion of body fat increases 6. Hematological - T- lymphocyte function deteriorates


2. Respiratory Compliance decreases® - Immune mechanism deteriorates
3. CVS BP increases® as arterial elasticity 7. Renal ■ GFR decreases®
decreases. - Clearance decreases
4. Endocrine ■ Glucose tolerance decreases® 8. Genital - Prostate enlargement®
- ADH secretion increases - Vaginal mucosal atrophy®
5. GIT - Gastric acidity decreases® 9. Bone Osteoporosis® (particularly in
■ Colonic m otility decreases® females)
- Increased chances of constipation 10. Brain Brain atrophy leading to amnesia

Exercise

M uscle blood flow Circulatory changes Ventilation Temperature


Sometimes increases at or • Heart rate rises with just the thought With the onset of exercise, ventilation - Body
even before the start o f of performing muscle contraction increases abruptly follow ed after a temperature
exercise probably d/t neurally even if muscle contraction is brief pause by a gradual increase®. rises® &
m ediated sym pathetic prevented by local infusion of When exercise ceases, it decreases hypothalmic
vasodilation®. neuromuscular blocker) and is abruptly followed after a brief pause centers that
When a muscle contracts, if it probably d/t psychic stimulation of by a gradual decline. With moderate control heat
develops >10% of its maximal medulla oblongata. exercise the depth of respiration is dissipating
tension, the blood vessels in it mostly increased, which is mechanism
• Cardiovascular response depends
are compressed. And if tension accompanied by an increase in are
on type of muscle contraction
>70% blood flow is completely respiratory rate when the exercise is activated
stoped. However, b/w Feature Isometric Isotonic more strenuous - Increased
contractions blood flow is sweat
Heart rate Increase Prompt
increased as much as 30 folds production &
(less) increase
Once exercise has started its
local mechanism maintaining Cardiacout Increase Increase vaporization
high blood flow include fa ll put (less) is the major
in tissue POi, and a rise in path for
Systolic Sharp rise in Rise
tissue PCO 2, temperature & heat loss
BP both moderately Abrupt increase a t start is d/t
k +q - Some heat is
Diastolic Unchanged psychic stumuli & afferent impulses
K+ depleted individuals show BP from proprioceptors in muscles, radiated
or decrease
a lesser increase in flow tendons, & joints®. from skin
during early phase of exercise Stroke Little Marked
Gradual increase is humoral even and
and a greater tendency of volume change increase increased
though arterial PCO 2, PO 2 and pH
exertional disintegration remain constant during m oderate ventilation
Peripheral Increased Decreases
(rhabdom yolysis) o f resistance d/t d/t exercise®. Increase body temperature, cause heat
muscles®. compression vasodilation loss in
T ed plasm a K+ levels, increased
Dilation of arteriole cause of blood in exercising expired air.
neuronal sensitivity to CO2 ® (that
increase in number of open vessels & muscle. detect fluctuations) and probably O2
capillaries, & increased cross reduced
may play role.
sectional area of vascular bed blood flow
Gradual decrease after exercise does
but the flow velocity & overage in
not reach basal levels until O 2 debt is
distance between blood and contracting
repaid. And the stimulus to
active cells (i.e., the distance O 2 muscles
ventilation is elevated arterial H*
& metabolites must diffuse) is
Cardiac More Less concentration d/t lactic acidemia. (It is
greatly reduced.
work load neither arterial PCO 2 which is normal
O2 dissociation curve shifts
or low, or arterial PO 2 which is
to the right® d/t
>Maximum heart rate increase during normal or high).
T temperature & I pH, so exercise decreases with age. >In vigorous exercise, more lactic acid
that more O 2 is given up by ' Lymph flow and venous return is is produced causing metabolic acidosis,
blood. 2-3 DPG in RBC is
392 ■ A Complete Review of Short Subjects

increased ® and this further greatly increased which is buffered by more production
decreases O 2 affinity of • After exercise BP may transiently of CO? & hyperventilation. Alveolar
hemoglobin. drop to subnormal levels because and arterial CO 2 change relatively
Artero - venous O2 difference, accumulated metabolites keep little (iso capnic buffering) and
O2 consumption, energy vessels dilated for a short period. alveolar PO2increase. With further
output is increased®. However, BP soon returns to lactic acidosis & hyper ventilation
Rhythmic exercise normal pre exercise levels. Heart rate alveolar & arterial PCO 2 falls.
returns to normal more slowly. The PO 2of blood flowing into
l 'i j i h i h i j r ,i pulmonary capillary falls, so that
• Cardiac output may exceed upto 35
alveolar - capillary PO 2is increased and
L/min during isotonic exercise,
more O 2 enters the blood. The
which is proportional to O 2
lnHHlilii !!!!1 amount of CO 2 removed from blood
Calf •
consumption. And the regional blood
is increased.
flow is

1
Decreased in Maintained in Increased in
I I I
Inactive skeletal muscle - Brain® (750 ml/min) - Active skeletal muscle
Kidney - Skin (500 ml/min) - Heart (coronary blood flow)
Liver - Lung
Gastrointestinal etc.

Control of Blood Flow through Skeletal Muscles

The tremendous increase in muscle blood flow that occur during exercise (muscle activity) is caused primarily by local
m etabolites (chemical effects) acting directly on muscle arterioles to cause vasodilation®

The main cause of increased blood flow to Primary Regulation: Local M etabolites (Chemical Effects)
exercising muscle is vasodilation due to local
metabolites®. The most important local chemical Reduction of oxygen in muscle tissue is most important local
effect is reduction of O2 in muscle tissue. Increased chemical effect. O 2 deficiency causes release of vasodilator
arterial pressure above normal, f) (beta) substances eg.
adrenergic receptor mediated vasodilation in - Adenosine (most important but experiments have shown that it
exercising muscle and a (alpha) adrenergic has decreased effect on vasodilation in exercising muscles)
receptor mediated vasoconstriction in resting - Potassium (K+) ion
muscles are other factors responsible for increased - Adenosine triphosphate (ATP)
blood supply in muscles during exercise. - Lactic acid
Increased (not decreased) p2 adrenergic - Carbon dioxide (CO2)
stimulation (in humans) and cholinergic Associated Regulatory Factor: Increased Sympathetic Tone
stimulation (in some animals like dogs, cats) are It is responsible for circulatory changes that support increased blood
responsible for skeletal muscle vasodilation flow to muscle eg.
whereas, a i adrenergic stimulation cause skeletal - T Heart rate and BP
muscle vasoconstriction. Epinephrine excites more - Vasoconstriction of peripheral vessels (other than vessels in active
of p receptors (=vasodilation), whereas a muscle that are dilated d/t local metabolites)
vasoconstrictor receptors are excited especially by - Venoconstriction of veins & capacitance vessels to increase filling
norepinephrine. pressure.

Physiological Changes in Exercise


- With the onset of exercise, there is an abrupt increase in ventilation®; followed after a brief pause by further more gradual
increase. The increased stimulation is presumably due to psychic stimuli and afferent impulses from proprioceptors® in
muscles, tendons and joints.
- During muscular exercise, increase in O 2 consumption is proportionate to the energy expended, and all the energy needs are
met by aerobic process. With more muscular exertion, aerobic resynthesis of energy stores cannot keep pace with their
utilization and lactate is produced (d/t anerobic breakdown of glucose):
General Physiology ■ 393

With vigeourous exercise buffering of increased amount of lactic acid, liberates CO 2 and this further increases ventilation.
With further accumulation of lactic acid, the Tin ventilation outstrip CO 2 production and alveolar Pco 2 falls as does arterial
Pco 2q- The decline in Pco 2 provides respiratory compensation for metabolic acidosis
Arterial pH and H C O -3 decreaseQ with severe exercise due to lactic acidosis
Strenuous exercise can cause Hyperkalemia^ by releasing K+ from skeletal muscle.

Respiratory CVS M etaboic During Exercise


Ted O 2 uptake & HRT edS T ed glucose The contractile muscles need more O 2S for energy
Consumptions Stroke volumeT uptake production.
Ted ventilation edQ T ed insulin This need is meet by increasing the blood supply to the
T ed alveolar - capillary BPT ed sensitivity muscles. Cardiac activity increases to increase the
PO 2 gradient Muscle blood T ed K +S stroke volum es. Capillary bed of the contracting
T ed CO 2 excretions flow -T ed in muscle dilate and many previously closed capillaries
T PaC02° isotonic exercise open up.
T Pa02Q -Ted in isometric In contracting muscle the amount of O 2 extracted from
i ed pH, HCO j S blood increases (3 fold increase in O 2 extraction from
each unit of blood)S. This is because of right shift of
Hb - O 2 dissociation curve.

Energy Sources & Metabolism During Muscle Contraction

Energy derived anaerobically in 100 meter race of (10s) is 85% whereas in long distance run (of 1 hour) is 5% only. After
exercise, extra O 2 is consumed to remove the excess lactate, replenish phosphocreatine and ATP stores and replace O2 that was
released by myoglobin. This O 2 debt is measured by determining O 2 consumption after exercise until a constant basal
consumption is reached and substeracting the basal consumption from total. It may be 6 times of basal consumption. At rest
and during light exercise, muscle utilizes lipids in the form of free fatty acidQ as their energy source. As intensity of exercise
increases carbohydrate^ becomes the predominant component in muscle fuel mixture. There are three important metabolic
system that supply energy for muscle contraction.

Aerobic System Anaerobic System

Phosphagen System Glycogen Lactic Acid System


Glucose
During exercise p h osp h ory l creatine® If O 2 supply is insufficient, the
Fatty acid +o 2 c o 2 + h 2o
is hydrolyzed to creatine and pyruvate formed from glucose
Amino acid + Urea
phosphate groups, forming ATP from does not enter TCA cycle but is
+ ATP
ADP reduced to lactate.
Phosphocreatine----- > Creatine + PO 4
Glycogen + 1ATP
ADP— t * ATP
1 ! * 2 Lacticacid + 4ATP
PO 4 <......................................

Feed Back Mechanism


There are 2 type of feedback mechanisms:

Positive feedback mechanism Negative feed back mechanism


• Better known as 'Vicious cycle' In this if some factor become excessive, a control system
• It is often of a milder degree & can therefore be overcome initates negative feedback, which consist of series of
by stronger negative feed back control of the body and changes that return the factor towards a certain mean
vicious cycle fails to develop. value.
• Examples are : [Mnemonic : CLASP] » Examples are______________________
C - Clotting bloodQ - ACTH secretionQ
- Ca2+ entry into sarcoplasmic reticulumQ - Aldosterone - K+ secretion
L - LH surge during ovulationQ - Glucose regulation
A - Action PotentialQ - GH secretionQ
S - ShockQ - BP regulationQ
P - ParturitionQ
394 ■ A Complete Review of Short Subjects

H aptoglobin
• It is a plasma glycoprotein that binds extracapsular hemoglobin (Hb) in a tight non-covalent complex. Its amount ranges from
40m g -180 mg of hemoglobin binding capacity / deciliter.
• Only 10% of degraded hemoglobin is released in circulation (i.e. extra corpuscular) the remaining 90% is present in old
damaged RBCs, which are degraded by histiocytic system.
• Free Hb (molecular mass 65KDa) passes through glomerulus and precipitate in tubules. However, after binding with
haptoglobin (90 KDa) the Hb-Hp complex (155KDa) becomes too large to pass through glomerulus. So the function of
haptoglobin is to prevent loss of free hemoglobin into kidney and conserve the valuable iron present in hemoglobin, which
would otherwise be lost to the body.
• Humans have 3 polymorphic forms, k/a Hp 1-1, Hp 2-1, Hp 2-2. Low levels of haptoglobins are found in patients with
hemolytic anemia. This is explained by the fact that where the life of Hp is 5 days, the tVi of Hb-Hp complex is 90 minutes,
the complex being rapidly removed by hepatocytes. So when the hepatoglobin is bound to Hb, it is cleared from plasma
about 80 times faster than normally.
• Hb is an acute phase protein & is elevated in variety of inflammatory states.
• Haptoglobin related protein binds hemoglobin; hemopexin binds free heme; and albumin binds metheme (ferric heme)

H istam ine
Synthesis & Catabolism Receptors and Functions
Histamine is formed by decarboxylation of aminoacid It has three types of receptor - H I, H2 and H3. The HI
histidine. receptor activate phospholipase C, & H2 receptors
H is t id in e increase intracellular C-AMP. Most, if not all of the H3
Histamine receptors are presynaptic and they mediate inhibition of
d e c a r b o x y la s e Q
release of histamine & other neurotransmitter via G
Histamine protein
N-Methyl
Its major actions are
Transferase
I-------------------------1-------------------------1
Diamine oxidase / Methylhistamine CNS Gastric • Marked
(histaminase) | Monoamine oxidase 1 mucosa vasodilation of
| (MAO) 1 small blood
Imidazole acetic acid Methyl imidazole acetic acid - Arousal® Increased vessels
■Histamine is mainly found in mast cells & basophils. Heparin - Sexual behaviour gastric acid • Visceral smooth
containing mast cells are plentiful in anterior & posterior lobes of - Sensation of itch secretion® muscle
pituitary glands. - Regulation of secretion (via H2 constriction in
•It is also found in enterochromaffin like (ECL) cells of gastric of some anterior receptor) airway,
mucosa and histaminergic neurons of brain (non mast cell pituitary hormone (broncho-
histamine). Histaminergic neurons have their cell bodies in - Blood pressure constriction)
tuberomammillary nucleus of posterior hypothalamus, and their - Drinking uterus and
axons project in all parts of brain. - Pain threshold intestine

Lewis Triple Response Axon Reflex


When a pointed object is stroked very firmly over the skin, a - When an area of skin is stroked, sensory nerve fibres
characteristic 3 part response develops. Its features are- arising from the area are stimulated and carry the
1.R ed reaction (red line)® is due to transient capillary impulse to respective spinal cord segment as usual.
engorgement (vasodilation) caused by histamine® released However durig this journey a branch of this nerve
from damaged tissue. Histamine relaxes precapillary sphincter gets activated and the impulse returns back to
thus 1/t capillary dilation. cutaneous arteriole (without reaching the spinal
2. Flush or flare® is spreading of redness beyond the injury (red cord) causing this arteriole to dilate. This is one of the
line). It is due to arteriolar dilation m ediated by axon reflex® few reflex in out body which does not involve CNS.
(may be a indirect effect of histamine) -So it is a response in which impulses initiated in
3. Wheal® is local edema and elevation of skin due to increased sensory nerves by the injury are, conducted
perm eability o f capillaries & post capillary venules with antidromically down other branches of sensory
consequent extravasation o f fluid®. Capillary permeability is nerve fibers.
increased due to histamine & other mediators. -Substance P is the transmitter released at central
• Triple response is present after total sympathectomy®. The termination of sensory C fiber neurons. And
flare is absent in locally anesthetized or denervated skin after substance P & CGRP are present in all parts of
sensory nerves have degenerated; but it is present immediately neuron. Both dilate arteriole (p rod u ce flare) and
after nerve block or section above the site of injury. substance P causes extravasation of fluid (wheal) also.
General Physiology ■ 395

Kinin System Roles of Inflammatory Mediators

Vasodilation - NO (nitric oxide)


M n-"N o pH" - Prostaglandins E l, E2, D 2 ,12
Factor XII (Hageman factor)
- Histamine
Vascular leakage - Vasoactive amines (histamine,
(increased serotonin)
permeability) - Bradykinin
M n-"in, ine & - Araphylatoxins (C3a & C5a) through
ene + P" liberating amines
- Leukotrienes C4, D4, E4
- Substance P
- PAF
Chemotaxis, - C5a
leukocyte - Leukotrienes B4
recruitment & - Chemokines
activation - il -l t n f
- Bacterial products
- HETE, lipoxins
Fever - IL-1,TN F
- Prostaglandins
Pain - Prostaglandins
- Bradykinin
C3a <-------------------C3 Tissue damage - Neutrophil & macrophage lysosomal
enzymes
- NO
- Oxygen metabolites
Vasoconstriction - Thromboxane
- Leukotrienes C4, D4, E4
Chapter 9. CELL PHYSIOLOGY: REVIEW NOTES

Composition of Cell Membrane / Plasma Membrane

•Cell membrane is composed of proteins > lipids > carbohydrate^. It has 3 classes of amphipathic
lipids: phospholipids > cholesterol (steroid or sterol) > glycol (sphingo) lipidQ.
•Major membrane phospholipids are: lecithin (phasphatidylcholine), phosphatidyl ethanolamine,
phosphatidyl inositol, phosphatidyl serine and sphingomyelin^.
•Glycolipids include cerebrosides (galactosyl & glucosyl ceramide) and ganglioside.
•Oleic acid is most abundant unsaturated fatty acid. Triglycerides are absents and cholesterol
confers a stiffening & strengthening effect.
•Carbohydrate in plasma membrane is predominantly present as glycoprotein but also as glycolipids
in small amount. These oligosacchrides give cell an identity (and so are responsible for blood groups
& transplant rejection).

Function of Cell Membrane Components

Phospholipid

Along with Membrane Caveolae


cholesterol Lipid Rafts (Lipid raft + Caveolin-1)

Maintains -Are specialized dynamic areas of exoplasmic (outer) leaflet of lipid bilayer They are flask shaped
fluidity of enriched in cholesterol, sphingolipids (sphingomyelin), indentations of the cell membrane
membranes glycosphingolipids (ganglioside, GMi), saturated phospholipid and facing the cytosol in the region of
As membrane certain integral proteins like GPI (glycosylphosphatidyl inositol) anchored lipid raft. Mostly caveolin-1
fluidity proteins (in outer leaflet) and acylated/prenylated proteins (inner leaflet). protein is involved in their
increases, its -Membrane lipid raft microdomains are some what thicker, more ordered & formation form lipid raft.
permeability tightly packed than the remainder of bilayer membrane but float freely in the Proteins detected in coveolae form
to water & membrane. Lipid rafts contain 3 to 5 times more cholesterol and 50% more various component of signal
small sphingolipids compared to surrounding plasma membrane transduction system eg insulin
hydrophilic receptor, & some G proteins,
-They compartmentalize (or cluster) certain components of signal
molecule folate receptor, endothelial nitric
transduction and other cellular processes by serving as organizing centers;
increases. oxide synthase (eNOS).
close clustering may increase the efficiency. They also influence membrane
fluidity, protein trafficking, receptor trafficking, neurotransmission, G Caveolin protein is a dimer; each
protein signaling, endocytosis & binding of viral pathogens. Defective GPI monomer is anchored to the inner
anchors cause paroxysmal nocturnal hemoglobinuria (PNH). leaflet of plasma membrane by 3
P palmitoyl molecules.
/ G P I linked protein
Caveolae mediates transcytosis of
Lipid raft 4 O uter macromolecules (IgA) from
I (ch. S pL) j Inner leaflet luminal side, endocytosis of
cholesterol containing
O ulside
lipoproteins and fusion &
budding of viral particles.
^ ^ C a v e o l i n dim er

Inside
Cell Physiology ■ 397

Protein

Every membrane possesses a different complement of protein. [Cell to cell variation of membrane protein]

Integral/Transmembrane Proteins Peripheral (Extrinsic) Proteins


•They interact with the phospholipids and require the use of detergents for their >Peripheral proteins present embedded
solubilization. in either the inner or outer leaflet of
- gen erally span the lipid bilayer® the lipid bilayer.
- distributed asym m etrically across the lip id bilayer® ■They do not interact directly with the
- are usually g lo bu la r protein s ® phospholipids in the bilayer and do
- they are amphipathic in nature not require use of detergents for their
- Integral proteins are amphipathic in nature and contain both hydrophobic and release
hydrophilic regions. •They are weakly bound to the
- H ydrophillic regions are lo ca ted a t the ends w h ile the transm em brane region i.e. the hydrophilic region of specific integral
region th a t traverses the core o f the b ila y er is com p osed o f a stretch o f h y d rop h ob ic proteins by ion ic in teraction or
am ino acids.® hydrogen bonds.®
- Integral protein are in Vander W alls contact with hydrophobic region of membrane.

Lipid Content Of Membranes

______________ ________ Phospholipid______________________ Glycosphingolipid Sterols


- Phosphoglycerides are more common and consist of glycerol Are sugar containin g lipid® built on a Mainly cholesterol
+ fatty acid (usually even numbered mostly 16,18 - are back bone of ceramide. They include
unbranched. & canbe saturated or unsaturated ) + phosphate g a la c to s y l - an d g lu cosy l ceram ide
(phosphoric acid) + nitrogenous or nonnitrogenous base (cerebrosides) an d ganglosides®
(choline, ethanolamine, serine & inositol)
- Sphingomyelin : glycerol is replaced by sphingsine.________

Fluidity of Membranes & Transition Temperature (Tm)


T he fluidity of membranes, are highly depen dan t upon the lipid composition of the membrane. In a lipid bilayer, the
h ydrophobic chains o f the fatty acids can be highly aligned or ordered to prov id e a rather stiff structure. As the
tem peratu re increases, the hydrophobic side chains undergo a transition fro m the ordered state ( m ore g el-like or crystallin e
phase) to a disordered one, taking on a m ore liquid like or flu id arran gem en t. T he tem perature at w hich the structu re

— Polar head group


A queous

mum
H y d ro ­
p h ilic

muni
H y d ro -
Apolar
hydrocarbon tails p h o b ic

H y d ro ­
ph ilic

Aqueous
D ia g ra m m a tic re p re se n ta tio n o f a p h o sp h o lip id o r o th e r D ia g ra m o f a se c tio n o f a b ila y e r m e m b ra n e fo rm e d from
m e m b ra n e lipid. T h e p o la r h e a d g ro u p is h y d ro p h ilic , an d the p h o sp h o lip id m o le c u le s. T h e u n sa tu ra te d fa tty acid tails are k in k ed
h y d ro ca rb o n tails a re h y d ro p h o b ic . T h e fa tty a c id s in th e tails an d lead to m o re sp a c in g b e tw ee n th e p o la r h ead g ro u p s; h e n ce to
are sa tu ra te d (S) o r u n sa tu ra te d (U ); th e fo rm e r a re u su a lly m o re ro o m fo r m o v e m e n t. T h is in tu rn re su lts in in crea sed
a tta c h e d to ca rb o n 1 o f g ly c e ro l a n d th e la tter to c a rb o n 2. N ote m e m b ra n e flu id ity.
th e k in k in th e tail o f th e u n sa tu ra te d fa tty acid (U ), w h ich is
im p o rta n t in c o n fe rrin g in c re a se d m e m b ra n e flu id ity .
398 ■ A Complete Review of Short Subjects

Cholesterol modifies the fluidity of membranes. At temperature

I------------------------------------------- 1
Blow Tm Above Tm
It interferes with the interaction of hydrocarbon tails It limits disorder because it is more rigid than the
of fatty acids and thus increases fluidity hydrocarbon tails of the fatty acids and cannot move in the
membrane to the same extent, thus limiting fluidity.________
• At high cholesterol: phospholipid ratio, Tm are altogether indistinguishable
• The fluidity affects the membrane functions. It increases permeability to water and other small hydrophilic molecules, and lateral
mobility of integral proteins.
• The fluidity of the plasma membrane is highly dependant upon the lipid composition of the membrane.
• Saturated fatty acids: Increase the transition temperature and hence decrease the fluidity.
• Unsaturated fatty acids (PU FA ) : Decrease the transition temperature and hence increase the fluidity o f membrane.

Properties of Cell Membrane


• Membranes are asym m etrical Q sheet like enclosed structure with distinct out and inner surfaces. This asymmetry is
attributed to
irregular distribution of protein with in the cell membranes
external location of carbohydrate attached to membrane protein, (inside - outside asymmetry)
- specific enzymes are located exclusively on the outside or inside of membranes, (inside - outside asymmetry)
gap junction synapses and tight junctions occupy only smaller regions of membrane and generate local (regional)
asymmetry there is inside - outside (transverse) asymmetry of the phospholipids
• The choline- containing phospholipids (phosphotidylcholine or lecithin & sphingomyelin) are located mainly in the outer
molecular layer; the aminophospholipids (phosphotidyl serine & phosphotidylethanolamine) are preferentially located in
inner leaflet. There must be limited transverse mobility (flip- flop) of the membrane phospholipid to maintain this transverse
asymmetry. Phospholipids exhibit an extraordinarily slow rate of flip flop. When certain membrane proteins such as
erythrocyte protein glycophorin are inserted artificially into synthetic bilayers, the frequency of phospholipid fillip -flop
increases as much as 100 folds.
• Membranes are composed of - lipids, proteins and carbohydrates and membranes are dyanic structure
Major lipids in mammalian membranes are phospholipids (phosphoglycerides > sphingomyelin), glycolipids &
cholesterol.
Membrane lipids are am phipathetic i.e. contain both hydrophobic & hydrophilic regions Q.
Membrane lipids are bilayer
Proteins are associated with lipid bilayer (+ covalent linkage) and may be integral or peripheral in location.
Ion channels are transmembrane proteins that allow selective entry of various ions; ionophores are molecules that act as
membrane shuttle for various ions and aquaporins are proteins that form water channels in certain molecules.
• Oleic acid is most abundant unsaturated fatty acid in animal membrane lipids. Sphingolipids are found in membranes
specifically in tissues of nervous system. Degree of unsaturation determines fluidity of membrane and cholesterol helps to
regulate it.

Osmosis, Osmolarity, Osmolality, Osmotic Pressure, Tonicity & Oncotic Pressure


Osmosis M ole and Osmole
- Osmosis is the diffusion A mole is molecular weight of a substance in grams eg MW of glucose is 180; its mole is
/movement of water (solvent) 180 gm and NaCl is 58.5gm. An Osmole (Osm) is gram MW (mole) of substance divided by
through a semiermeable numer of osmotically active freely moving particles that each molecule liberates in solution. For
membrane (permeable to biological solutions, the milliosmol (mOsm i.e. 1/1000 of 1 Osm) is used. A molecule of
solvent, but not to solute) from glucose (nonionizing) compound gives only one active particle whereas 1 molecule of
an area of high water potential ionizing NaCl yields 2 and CaCb or Na2S 0 4 yields 3 molecules. Therefore 1 mole of
/activity/concentration to low glucose yields 1 osmole of glucose; 1 mole of NaCl yields 2 osmoles (Na+ and C l'); and 1
water potential /thermodynamic mole of CaCl2/Na2SC>4 yields 3 osmoles. (Ca2+ & 2C1" /2Na+ & SO 42 )
activity/concentration (i.e. from
Osmole (in place of grams) unit is used to express concentration of solution in terms of
lower solute concentration to
number of particles. 1 Osmole = 1 gram molecular weight of osmotically active solute. So
higher solute concentration).
for glucose 180 grams = 1 gram MW = 1 Osmole whereas, for sodium chloride 58.5 grams
So osmosis is net diffusion of
= 1 gram MW = 2 Osmoles (as NaCl dissociates into 2 osmotically active particles).
water across a selectively
Cell Physiology ■ 399

permeable membrane down Osmolarity & Osm olality


their concentration (chemical)
gradient. Because presence of Osmolarity is numbers of osmoles per liter (in 1 liter) of solution (plasma), whereas
solute decreases the osmolality is numer of osmoles per kilogram of solvent (water). (Mn "Both Osmolar and
concentration of water on that liter end in r")Q. For dilute solutions such as body fluids the difference between osmolality
side (when compred to equal and osmaolrity is insignificant. Although, strictly speaking, it is osmolality (Osmoles per
volume of pure water). Hence in kg of water) that determines osmotic pressure. However, because of difficulty of
osmosis water moves from measuring kgs of water in a solution, osmolarity (i.e. osmoles per liter) is used.
solution of lower solute Measurement of osmolarity are dependent on temperature because the volume of solvent
concentration (which also varies with temperature while osmolality is not. For this reason osmolality is preferred
means higher water term for biological systems.
concentration) to higher solute Like vapour pressure lowering, freezing point depression and boiling point elevation,
concentration (= lower water the osmotic pressure depends solely on num ber of molecules in that solution. It is not
concentration). dependent on type or size of molecule, their mass or chemical nature (eg valence)- i.e. it
- Osmosis can be opposed by is fundamental colligative property o f solutions. The reason for this is that each particle in a
applying a pressure in the opposite solution, regardless of its mass, exerts, on average, the equal amount of pressure against
direction of osmosis (i.e. on the membrane (i.e. large particle with greater mass have slower velocities & smaller
solution containing lower water particles have higher velocities in a way that both have almost equal kinetic energies).
/ higher solute concentration).
A solution that has 1 osmole of solute dissolved in each kilogram of solvent is said to have
The precise amount of
an osmolality of 1 osmole/kg. In body fluids solvent is water and the density of water is 1,
hydrostatic pressure necessary
so osmolal concentration can be expressed as osmoles/kg of water. 1 milliosmole /kg
to prevent the osmosis or
means 1/1000 osmoles dissolved in 1 kg of solvent. Similarly 1 osmole of solute dissolved
osmotic flow of solvent (water
in each liter of solution is said to have osmolarity of 1 osmole/lit. Total osmolarity
in most cases) is called osmotic
(osmolality) of each of the three compartments of body fluid is 300 m illi osmoles per liter
pressure. It is an indirect
(or kilogram of H 2O) with the plasma being about 1 mOsm/L greater than that of interstitial
measure of solute and water
and intercellular fluids.
concentration of a solution. The
higher the osmotic pressure of a However, the corrected osmolar activity of body fluids is 281 (Guyton) /290 (Ganong)
solution, the higher the solute mOsm/L with plasma being about 1 mOsm/L greater. The reason for the difference is that
and lower the water many of the ions in body fluids eg Nad are highly attracted to one another consequently, they
concentration of that solution cannot more entirely unrestrained in fluids and contribute some what less than 2 mOsm of
(and viceversa) osmotically active particles per liter.
Na+ and its accompanying anions esp CF & HCOj- contribute all but 20 osmoles per liter in
Osmotic Effectiveness
ECF. Although concentration of plasma proteins is large when expressed in grams per
- Although a homogenous liter, they usually contribute < 2mOsm/L b/o their high molecular weight. The major non
solution contains osmotically
electrolytes of plasma - glucose & urea are in equilibrium with cells and contribute to
active particles and can be said 5mOsm/L of osmolarity each.
to have an osmotic pressure, it
can exert an osmotic pressure
Concentration (Osmolarity in mOsm/L H2O)
only when it is in contact wit
Substance ECF Intracellular Fluid
another solution across a
Plasma Interstitial (ICF)
membrane permeable to the solvent
Na+ 142 139 14
but not to the solute.
■For a molecule (substance) to K+ 4.2 4 140
exert and maintain a stable M g++ 0.8 0.7 20
osmotic pressure, it must be ci- 108 108 4
effectively confined to one side of HCO 3 24 28.3 10
the membrane (i.e. must not cross Phosphocreatine - - 45
the membrane). For a molecule Camosine - - 14
that can freely cross the cell Protein 1.2 0.2 4
membranes, like urea, can not Glucose 5.6 5.6 -

sustain a stable osmotic effect. Urea 4 4 4


Such a substance causes only Total 301.8 300.8 301.2
transient osmotic effects and is Corvated 282 281 281
said to be osmotically Total Osmotic pressure 5443 5423 5423
ineffective (ineffective (mmHg) at 37°C
osmole). Similarly a substance
like glucose that is metabolized
400 ■ A Complete Review of Short Subjects

is also osmotically ineffective Tonicity


because its concentration is not •Tonicity is used to describe osmolality of a solution relative to plasma and its effect on volume
sustained (d/t metabolism). o f cell. The tonicity depends on concentration of impermeant solutes. Solutions that have
Although glucose is not freely the same osmolality as plasma are said to be isotonic, and these solutions do not change
diffusible b/o large size. the volume of a cell (Osmolarity = 282 m Osm/L or osmolality = 282 mOsm/kg of H 2O).
- So osmotically effective Solutions with greater osmolality (>282) are called hypertonic and water will diffuse out
substances are neither freely of cell into ECF, concentrating the ICF, shrinking the cell and diluting ECF. Solutions with
permeable nor metabolized eg lesser osmolality (< 282) are hypotonic and water will diffuse into cell causing dilution of
plasma proteins and various infra cellular fluid and cell swelling while ECF is concentrated until both solutions have
complex polysaccharides like about the same osmolality. Similarly solutions with same, higher or lower osmolarity
dextrean, sucrose. Sodium respectively, compared with normal ECF, without regard for whether the solute permeates
chloride is osmotically effective the cell membrane are called isosmotic, hyperosmotic and hyposmotic.
because even though Na+ & Cl-
ions can diffuse through cell Measurement of Osmotic Pressure (n)
membranes Na+ is maintained 1. By measuring pressure that must be applied to prevent water from entering the solution
extracellularly b/o sodium with higher solute concentration.
pumps in cell membrane. 2. By freezing point depression: 1 mol per liter of an ideal solution depresses the freezing
- The clinical significance of point of water by 1.86°C. So the numer of milliosmoles per liter in a solution equals the
osmotic effectiveness (in freezing point depression divided by 0.00186.
management of hypovolemia) is The average freezing point for human plasma is - 0.54°C, which corresponds to an
that a solution used to restore osmolality of 290 mOsm/kg. This is equivalent to an osmotic pressure of 7.3 atm against
and maintain circulating blood pure water.
volume must be one whose 3. By Van't Hoff's equation (Law)
active constituents remain g (T1 CRT where 7t = osmotic pressure; c = reflection or osmotic coefficient which is O
within the circulation or at least, for freely permeable substance like urea and 1 for totally impermeable like sucrose; r| =
ECF (i.e. neither metabolized number of dissociable particles per molecule eg 1 for glucose & 2 for NaCl; C or AC = total
nor diffuses into cells). Hence (or difference in) solute concentration (mol/L); R = natural gas constant (62 mm Hg x
although 5% glucose solution is 1/mmol x °K or 0.082 atmL/mol °K) and T = absolute temperature (°K) for body fluid it is
isotonic (isomotic) with plasma 37°C (= 273+37 = 310° Kelvin). In few books 7t = CRT where C is solute concentration in
when initially infused osmoles/L (i.e. it includes both n & C of this formula).
intravenously, but cannot be So somotic pressure (rt) for totally impermeable (g = 1), non-dissociable (n = 1) molecule at
used to treat hypvolemia 1 mol/L concentration in body fluids (i.e. T = 310°K) is
because glucose is metabolized, 7t = l( lx 1 x 0.082 x 310) = 25.4 atm = 19300 mm Hg Because 1 atm = 760 mmHg
so the net effect is that of
And similarly for 1 milimole /L solute concentration which is equal to 1/1000 mole/L. The
infusing a hypotonic solution.
7t would be 2.54 x 10 2 atm or 19.3 mmHg. This means, fo r each m iliosm ole concentration
But 0.9% saline solution (NaCl)
gradient across the cell membrane 19.3 mmHg or 0.0254 atm osm otic pressure is exertedQ.
can be used as it remains
Or in other words, at normal body temperature (37°C) a concentration of 1 osmole per
isotonic because there is no net
liter will cause 19300 mmHg (or 25.4 atm) osmotic pressure in solution.
movement of osmotically active
Because concentration of body fluid is 300 mili Osmole/liter so the (7t) should be = 19.3 x
particles into cells and the
300 = 5790 mmHg. However, actually it is 5500 mmHg (0.93 times of calculated value)
particles are not metabolized.
because due to attraction between few ions the corrected effected osmolarity is about 280
■Similarly 300 mmoI/L solution
mOsm/L (0.93 times of calculated value).
of sucrose & urea both are
■0.9% (=0.9grams/100 mililiters = 9gm/L) NaCl is isotonic to plasma. Because the MW of
isosmotic (i.e. have same
NaCl is 5 8 . 5 , the molarity of 0 . 9 % solution is 9g/L + 5 8 . 5 g/mol = 0 . 1 5 4 mol/L; and the
osmolality) to intracellular fluid
osmolarity is 0 . 1 5 4 x 2 = 0 . 3 0 8 Osm/L = 3 0 8 mOsm/L (because 1 molecule provides 2
(i.e. 300 m Osm/ Kg H20 ) but
particles). Now because each mOsm/L of body fluid exerts 19.3 mmHg of osmotic
cells (eg RBCs) placed in urea
pressure (at 37°c); the osmotic pressure of 0 . 9 7 c NaCl would be 3 0 8 mOsm/L x 1 9 . 3
solution swell & burst and those
mmHg/mosm/L = 5 9 4 4 mmHg. However this value is higher than the actual because the
in sucrose solution maintain
osmotic reflection coefficient of NaCl is 0 . 9 3 . Therefore the actual osmolarity of 0 . 9 7 o Nacl
their nomal volume. So
is 0 . 9 3 x 3 0 8 = 2 8 6 mOsm/L
effectively sucrose solution is
isotonic but urea solution is Oncotic Pressure
hypotonic b/o high permeability - Oncotic pressure is the osmotic pressure (71) generated by large molecules (esp proteins) in
of urea (d/t urea uniporters in solution. It does not conform to van't Hoff's law most probably because of size & shape of
cell membrane) and protein molecule. Proteins exert 26-28 mm Hg of oncotic pressure in human plasma, which
impermeability of sucrose. although appears to be small (28 mm Hg - 1.4 mOsm/kg H 2O) is an important force
involved in fluid movement across capillaries.
Cell Physiology ■ 401

Fick’s Law of Diffusion

Fick's law of diffusion explains passive diffusion along concentration gradient Q. The magnitude of diffusing tendency is
directly proportionate to the cross sectional area across which diffusion is taking place and the concentration gradient or
chemical gradient, which is the difference in concentration of the diffusing substance divided by the thickness of boundary.
Thus

Net rate of diffusion (J) = a Area (A) and concentration gradient A C'
,AX

AC Where D = Diffusion coefficient and


J = - DA Minus (-) sign = Indicates the direction of diffusion when
AX
molecules move from higher to lower concentration, I ^ |is
VAX J
Flux = - P A A C.
negative, so multiplying it with - DA gives a positive value.
A C = Concentration inside (Ci) - Concentration outside
Flux = - PA (Ci-C„) Since thickness of biological membrane ( A x) is constant at 1CH
cm, D/x simplifies to permeability coefficient (P).

The net flux of solute is from high to low concentration. And the time required for equilibrium by diffusion is
proportionate to the square o f diffusion distance.

Resting Membrane Potential (RMP) Established by

Selective permeability of cell membrane Concentration (mmol/L) Sodium-Potassium Pump


I---------------- Ion Intracellular Extracellular Equilibrium
K+ diffusion potential Na+ Fluid Fluid Potential »Sodium & potassium ions
>K+ is highly permeable diffusion (mV) are activately transport in
at resting condition (all potential Na+ 12 145 +65 opposite direction across
the K+leak channels are Most of Na+ K+ 155 4 -95Q the cell membrane by
opened in resting channels H+ 13 x 10-5 3.8 x 10-5 -32 electrogenic pump called
condition). are closed Cl- 3.8 120 -90Q Na+ K + - pump.
>K* efflux is much greater at rest, so H C 03- 8 27 -32 »It pumps 3Na+ outside
than Na* influx 0 only very fo r each 2 K* pumped to
A 155 0
because it is 1 0 0 times little inside o f membrane .Q i.e.
Membrane potential = -90 mvQ
more permeable. amount of pump creates some
>Net outw ard diffusion Na+ diffuses negativity inside and
ofK + m ake interior o f into cells. positivity outside i.e. it
cell more negative and is electrogenic
is believed to be the » But Na+ - K+ pump
main mechanism contributes no more than
responsible fo r RMPQ. -5mv to RMP.
This is supported by
effects of
I
Increasing Extracellular K+ concentration Decreasing Extracellular K+ - Changes in internal concentration of K+
- Lowers RMP and negatively o f concentration cause change in RMP as predicted by
membrane Q because increase in [K+] Increases magnitude o f RMP Nernst equations
reduces the concentration gradient which and negativity o f - Alteration in Na+ & CF concentrations
causes the net outward diffusion of K+ membrane Qby increasing do not produce comparable results.
- It depolarizes the cell concentration gradient
It hyperpolarize the cell
402 ■ A Complete Review of Short Subjects

K* Balance K+ Excretion
• K + is the m ajor intracellular c a tio n V The normal plasma K 1 > Renal excretion is the m ajor route of
conc. is 3.5- 5 mmol/LQ, whereas that inside the cell is ~ 150 m elimination. The filtered load of K+ (GFR x
m ol/L. Therefore the amount of K + in the ECF (30-70 m m ol) Plasma conc. = 180 L/d x 4mm ol /L = 720 mmol
constitukes < 2% of total body K+ content (2500- 4500m m ol). /d) is 10 -20 folds greater than the ECF K+
The ratio of ICF to ECF-K+ conc. (normally 38:1) is the principal content
result o f the resting m em brane p o ten tia l Q and is crucial for > Some 90% of filtered K+ is reabsorbed by
neurom uscular function®. proxim al convoluted tubule & loop of Henle.
• The passive outward diffusion of K + is quantitatively the most P roxim ally the K+ is reabsorbed p assiv ely w ith N a *
im portant factor that generates the resting m em brane potential. & w ater, whereas the luminal N a*- K * - 2CI
N a+ K*- ATP ase pump activ ely transports K + in & N a+ out o f cotran sp orter mediates K + uptake in the thick
the cell in a 2:3 ra tio ®. The activity of pump is stimulated by ascen din g lim b o f loop o f H enle
in creased in tracellu lar N a + con c and inhibited by digoxin toxicity,
< There fore Kr delivery to distal nephron [distal
C H F, C R F
Extracellular lUncl Intracellular llwcl
convoluted tubule & cortical collecting duct
(CCD)] approxim ates dietary intake
• Net distal K + secretion or reabsorption occurs in
the setting of K + excess or depletion
respectively. The cell responsible for K+
secretion in late distal convoluted tubule (or
connecting tubule) & CCD is the principal cell.
Virtually a ll regulation o f renal K * excretion
and to ta l bod y K* balan ce occurs in d ista l
Electrolyte com postion of human body fluids. Note that the nephron®
values are in meq/L of water, not of body fluid. <The rate of K * secretion is p rop ortion ate to the rate
o f flo w o f tu bu lar flu id through the distal portion
of nephron, because with rapid flow there is less
A c tiv e tra n s p o rt
2K* opportunity for the tubular K + conc to rise to a
, O u a b a in
value that stops further secretion. In collecting
^5 duct N a+ is generally reabsorbed & K + is
3N a\ secreted. There is no rigid one for one exchange
3Na‘
ATP & much o f the m ovem ent o f K + is p a s siv e<2,
A D P + Pi
However there is electrical coupling in the sense
Ca"
t Na , that intracellular migration of N a+ tends to
Na' 15 m e q /L lower the potential difference across tubular cell
K' 150 -
& this favours m ovem ent of K> into the tubular
H* CP 7 -

K'
lumen. Since Na+ is also reabsorbed in
association with H + secretion, there is
com petition for the N a+ in the tubular fluid.
And K* excretion is decreased when am ount of
Vm = -7 0 mV N a* reachin g distal tubule is sm all & when H*
secretion is increased.
Na'
N a ' 1 4 0 m e q /L ■Potassium secretion is regulated by two
K' 4 - physiological stimuli - aldosterone &
CP 105 -
hyperkalem ia

Diffusion & Active Transport

Process in which solute particles move from a more concentrated environm ent to a less concentrated one in order to bring a
uniform concentration throughout.
Cell Physiology ■ 403

Sim ple D iffusion Facilitated D iffusion Active Transport


Molecules can passively traverse When carrier protein (transporter
When carrier proteins move substances against
the bilayer down electro protein) move substance in the
their chem ical or electrical gradient by expenditure
chemical gradient. direction o f their chem ical or
o f energy.®
• Energy is not required electrical gradient® without any
eg. Na* - K * pump®
energy expenditure (energy is not
required)®. Type of Carrier Protein
Facilitated diffusion is a form o f a) Uniport: when they transmit only one substance
Rate o f transport passive (no energy) transport that b) Sy m p o rt: binding of more than one substance to
is proportionate
to concentration
occurs along an electro chemical the transport protein & the substances are
gradient gradient and requires no energy®. transported across the membrane together e.g.
The rate o f transport is not Na* glucose sym port in intestine.®
alw ays proportionate to the
C o n c e n t r a t io n c) Antiport: transport protein exchange one
concentration gradient because it
Simple Diffusion substance for another, e.g.in Na+ K+ Atpase.
is m ediated through carrier
proteins®. Saturation Kinetics
At lower concentrations before • While in other transport mechanism, the rate of
T r a n s p o r t M a x im u m the carrier protein becomes diffusion approaches a maximum, called Vmax, as
V m ax saturated (or before Vmilx is the concentration of substance increases and then
reached), the rate of facilitated pleateu's off i.e. attains saturation at a particular
R a t e o f tr a n s p o r t is
p r o p o r t io n a l to diffusion is proportionate to level. This is called saturation kinetics
c o n c e n tr a tio n
g r a d ie n t o n l y a t
concentration gradient whereas, at mechanism®.
l o w c o n c e n t r a t io n
higher concentrations when • Saturation kinetics will be seen in any transport
carrier protein becomes saturated process which involves carrier proteins eg.
C o n c e n t r a t io n
(Vmax is reached) the rate of facilitated diffusion and active transport®.
facilitated diffusion becomes - The rate at which a substance can be transported
Facilitated Diffusion constant. by these mechanism depends upon the rate at
Hormones regulate facilitated which the carrier protein can undergo
diffusion by changing the number conformational change between its active state &
of transporter available, e.g inactive state.
Insulin increases glucose - So the rate at which molecules can be transported
transport in fa t & muscle®. by this mechanism can never be greater than the
- Glucocorticoids increases rate at which the carrier protein undergoes
transport of amino acid into liver. conformational change.
- This limits the rate of transport in these process.

Ionophores Transport Across Membrane


Certain microbes synthesize small organic molecules called ionophores,
l I
that function as shuttle for the movement of ions across membrane.
Sm all uncharged - Charged molecules
X X (neutral) molecules
Structure Types Characters - Larger uncharged
I (neutral) molecules
Ionophores a) M obile ion carriers: Like >Ionophores are
P ass freely by sim ple - Some small
contain valinomycin (refer uncoupler of termed because of
passive diffusion down uncharged (neutral)
hydrophilic oxidative phosphorylation) their ability to
an electrochem ical molecules, through
centers that b )Channel formers: Like complex
gradient®.
bind specific gramicidin, Nigericin specification &
+ Active transport is
ions & are Nigricin: acts as an ionophore facilitate their - Channel proteins or
against electrochemical
surrounded for K+ but in exchange for H+, transport through prions® (are
gradient and requires
by _ therefore abolishes pH gradient biological transmembranous beta
energy; whereas
peripheral across the membrane. membrane. barrel proteins that act as
passive diffusion is
hydrophobic Valinom ycin & Nigricin: If •The properties are pores through which
always down
regions. present, both membrane d/t their molecules can diffuse, by
electrochemical
potential & pH gradient are lipophilic simple passive diffusion)
gradient and does not
abolished & phosphorylation is character® which - Carrier proteins allow
require energy. fecilitated passive diffusion
completely inhibited i.e. allow penetration
conversion o f ADP to ATP is of lipid as well as active transport.
inhibited® membranes.
404 ■ A Complete Review of Short Subjects

Transport Across Cell Membrane


Exocytosis Endocytosis

Vesicles containing material for export are ticketed to the cell - It is the reverse of exocytosis and may be phagocytosis
membrane, where they bound via the v- SNARE /f- SNARE (cell eating) & pinocytosis (cell drinking)
arrangement - In endocytosis two non cytoplasm ic side o f
The area of fusion then breaks down, leaving the content of membranes fuse®
vesicle outside & cell membrane intact. This is Ca++ - It 1/t removal of cell membrane preventing cell to
dependent process. enlarge and thus maintaining the surface area of cell.
In exocytosis, cytoplasm ic sides o f tw o membranes fuse®
Endocytosis a
and there is net increase in surface area of cell (d/t adding of
extra membrane) • • *# •
,«•• .
I • j
a .....

t / .• * ! \
V- ;
Cytoplasm

Features Exocytosis Endocytosis


D efinition The process by which the substances are expelled from The process by which the substance enter the cell
the cell. It is the extrusion of cell bound vesicles, and requires without actually passing through the membrane. It
Ca2* and energy® requires: Energy, Ca2* & contractile element in the cell®.
Mechanism The inner membrane of vesicle fuses with the outer When segment of plasma membrane in vagina tes
plasma membrane, while cytoplasmic side of vesicle fuses enclosing a minute volume of ECF & its contents. The
with the cytoplasmic side of plasma membrane. Thus the invagination is pinched off, leaving the engulfed
contents of vesicles are externalized. material in the membrane- enclosed vacuole & the cell
membrane intact.
Types Secretion from the cell occurs Via two pathway Endocytosis
1 ‘ 1
Constitutive Nonconstitutive Phagocytosis Pinocytosis
Prompt transport of Protein from the golgi
proteins to cell apparatus initially enters ,------------------------------------- > -i
membrane in vesicles secretory granules, where Fluid phase Receptor mediated
with no processing. processing occurs. pinocytosis absorptive pinocytosis

Endocytosis
Pinocytosis Phagocytosis
It is a property of all cell. • Occurs only in
It is cellular uptake yf fluid & fluid contents.Q specialized cell
such as
Fluid phase pinocytosis Absorptive pinocytosis macrophage &
Is a nonselective process in Is a receptor mediated selective process primarily responsible for the granulocyte.
which the uptake of solute uptake of macromolecules. The vesicles formed during absorptive • Involves
is simply proportionate to its pinocytosis are derived from invaginations (pits) that are coated on ingestion of
concentration in the the cytoplasmic side with a filam entous m aterial called clathrin.® large particles
surrounding extracellular The protein dynamin which binds & Hydrolysis GTP, is necessary such as viruses,
fluid. for the pinching off of clathrin coated vesicles from the cell surface bacteria, cells or
(also called pinclmse) debris

Fate: Most endocytic vesicles fuse with primary lysosomes (early endosome) to form secondary lysosomes which contain
hydrolytic enzymes. & therefore are specialized orgenelle for intracellular disposal.
Cell Physiology ■ 405

★ Clathrin®: Clathrin molecules have the shape


of triskelion (three legs radiating from central
hub). As endocytosis progresses, the clathrin
molecule form a geometric array that
surrounds the endolytic vesicle. Once
complete vesicle is formed, it falls off & recycle C la th rin m o le c u le on th e su r fa c e o f an e n d o c v to tic v e sicle . N o te th e c h a ra c te ristic
to form another vesicle. trisk elio n sh a p e a n d th e fa c t th a t w ith o th e r c la th rin m o le c u le s it fo rm s a n et
su p p o rtin g th e v esicle.

Intercellular Communication
(Transmission of regulatory signals or chemical messenger between cells)

W ithout entering ECF (extra cellular fluid) Through ECF


Neural synaptic signaling
Gap Junction
Neurotransmitter released from
At gap junction, the intercellular space narrows from 25 nm to 3 nm. presynaptic neuron cross a narrow
Its unit connexons (an aqueous hemi channel) is made up of 6 synaptic cleft (ECF) to act on post synaptic
transmembrane protein subunits called connexins. Each connexin has 4 neuron
membrne spanning regions. Paracrine & autocrine signaling products
Connexon surround a channel, which when lined up with the channel of of cell diffuse in the ECF to affect nearby
corresponding connexon in adjacent cell, permits the substance to pass neighbouring cell (paracrine) or the same
between the cells w ithout entering extracellular fluid®. all (autocrine).
Diameter of channel is 2 nm, which permits the passage of ions. Sugars, Endocrine signaling
amino acids & other solutes with molecular wt upto 1000. These pores are Hormones secreted by endocrine organs
much larger than those formed by ligand or voltage gated channels & thus reach target cells via circulating body fluid
permit rapid propogation of chemical messenger & electrical activity from (i.e. blood or ECF)
cell to cell. Hormones bind on target cell to
The connexons keep the adjacent cell membranes at a fixed gap of 2-4 nm, intracellular receptors or extracellular
hence the name gap junction. receptor (present in ECF). And G protein
X linked Charcot Marie Tooth disease (peripheral neuropathy) is a/w coupled receptors are extracellular
mutation of connexin gene. receptor.
By means of gap junction myocardial & visceral smooth muscle cells have a
syncitial behaviour and conduct impulse from cell to cell.

Ligand
Endosomes and their role in receptor Plasma membrane
* * ••
mediated Absorptive Pinocytosis • FfJ'cfept&r L♦ • J |V
k •* .! . i
Coated
These are transient cellular structures involved in the pit
transport of macromolecules from the exterior of cells to its tJlathrin’* .
interior.
Coated fi .'-‘. V * ‘. • I - ’
They do not contain hydrolytic enzymes, are less dense \ v e s ic le ^ ^ ’ • | ' *•
than lysosomes and have an internal pH of 5.0.
.•***•* * •
Endosomes receive the receptor-ligand complex by fusing . * *. ‘
with the 'coated vesicles' \m ••
(Coated vesicles consist of Receptors, Ligand
(macromolecule) and a layer of clathrin protein)
On fusion, the coated vesicles discharge their
macromolecules into the interior of endosomes.
The low pH within the endosome, breaks the linkage
between receptor-m acromolecule complex and releases the
receptor, the m acromolecule, clathrin, and membrane
fragments®.
406 ■ A Complete Review of Short Subjects

Extracellular Matrix
Cells are surrounded by extracellular matrix often referred to as connective tissue. It can be organized into 2 forms: interstitial
matrix & basement membrane. It consists of 3 major classes of molecules

Structural Fibrous Proteins Gel of Specialized


I Proteoglycans Glycoproteins
Collagen & I
Fibrillar (type I, 2, 3, 5, & 11) collagens are most abundant and found in glycosaminoglycans • Fibronectin is an
interstitial matrix (which also contain non fibrillar collagens) (GAGs) important glycoprotein of
B asem en t m em brane con tain s non fib r illa r (m ainly type 4) collagen® extracllular interstial
P rovide tensile strength. Scurvy affects its triple h elical structure®. - Hyaluronic acid matrix, also found in
Deficiency 1/t Ehler Danlos synsrome, Alport syndrome, epidermolysis bullosa, - Chondratin sulfate soluble form in plasma,
osteogenisis imperfecta, chondrodysplasia and Menke's disease - Keratin sulfate which is involved in cell
Scurvy affects structure of collagen - Heparin adhesion & migration.
Elastin - Heparan sulfate • Laminin is a m ost
Confers exten sibility and recoil® to tissues such as skin, blood vessels, lung, - Derma tan sulfate abu ndant glycoprotein
ligament & uterus. o f glom eru lar & other
Deficiency 1/1 W illiam 's Beuren syndrome and supravalvular aortic b a s a l lamine® that has
stenosis binding sites for type IV
Fibrillin collagen, heparin &
In elastic fibers, central core of elastin is surrounded by peripheral network integrins
of microfibrils, that consists of 350 KD glycoprotein -fibrillin
Serves as scaffold for deposition of elastin & assembly of elastic fibers.
Defects results in abnormal elastic fibers in Marfan's syndrome. Mutation in
fibrillin 1 gene present in Microfibrils cause Marfan syndrome whereas,
mutation in fibrillin 2 cause congenital contractural arachnodactyly but not
Marfan syndrome.

Cell Adhesion Molecule (CAM)


• Stored in cytoplasm or located in cell membrane as receptor and priovide interaction between the same cells (homotypic
interaction) or different cells types (hetrotypic interaction).
• Classified into 4 main families immunoglobulin family CAMs, cadherins, integrins & selectins. Some other secreted adhesion
molecules are - SPARC (secreted protein acidic, and rich in cysteine or osteonectin), thrombospondins (both inhibiting
angiogenesis), osteopontin (regulates calcification) and tenacin
• Cadherin is calcium dependent adhesion protein providing homotypic interaction and forming zonula adherens & desmosomes.
Integrins is a super family of surface proteins present on wide variety of cells that links the outside of cells to their inside there
by helping to integrate response of cells eg. movement, phagocytosis. It provide adhesion both b/w cell & ECM (by binding
to matrix proteins fibronectin & lam inin as well as to other cells.
• Cadherin & integrins link the cell surface with the cytoskeleton (inside of cells) through their binding to actin.
P & a catenin link cadherin with cytoskeleton.

Interphotoreceptor - Matrix Mlmecaine/Osteofllycln/Osteo Inductive Factors


It is viscous biological glue that contributes to - It is a small leucine rich proteoglycans (SLEPs) that is abundant in
retinal adhesion and extends from outer extracellular matrix of bone, cartilage and connective tissue.
(external) limiting membrane to the surface of - It is important for collagen fibrillogenesis, cellular growth,
retinal pigment epithelium, (i.e. including layer of differentiation and migration and can induce ectopic bone formation,
rods & cones). It contains. - BMP-1 converts mimecan from precursor to mature form &
• Interphotoreceptor Retinal Binding Protein potentiates it to modulate the formation of type 1 collagen
(IRBP) fibrillogenesis
• Proteoglycans - Glycosaminoglycans - In atherosclerotic lesions, mimecan mRNA is downregulated in
Sulphated / Non sulphated media & upregulated in activated endothelium and neointima,
chondroitin whereas protein is accumulated in front edge of migrating smooth
Hyaluronic Acid muscle.
Cell Physiology ■ 407

Fibronectin
Matrix Metalloproteinases (MMP)
Sialoprotein associated with rods and cones
(SPARC) A family of enzyme, having more than 20 members. These are involved
(SPARC / Osteonectin / BM - 40) in degradation o f basement membrane & ECM.
Intercellular Adhesion Molecule I (IC A M -1) ._________________I___________
I
Hyaluronic Acid Receptor (CD44 antigen)
Celatinases /Collagenases Stromelysins
Lysosomal enzymes
(MMP2 & 9) (MM P 3 , 1 0 & 1 1 )
Matrix m etalloproteinases Degrade amorphous collagen & Degrade amorphous collagen,
Tissue Inhibitors o f fibronectin (of basement fibronectin, laminin & proteoglycan
M etalloproteinases (TIMP) membrane) (found in basement membrane)

Collagen

It is the m ost abundant & im portant protein Q in animal world providing extracellular framework for multicellular organisms.
In addition a number of proteins eg. Cic\ complement, pulmonary surfactant proteins SP- A and SP-D have collagen like domains in
their structure and sometimes referred to as non collagen collagens

Types Of Collagen Based on

Structure Location

Class Type Type Genes Tissue


Fibril-formin I, II, III, V and XI I COL1A1 Most connective tissues, including bone
Network-like IV, VIII, X II COL2A1 Cartilage, invertebral disc vitreous humor
FACITs IX, XII, XIV, XVI, XIX
III COL3A1 Extensible connective tissues such as skin,
Beaded VI lung, and the vascular system (hollow organ)
filaments
IV COL4A1- Basem ent membranes Q
Anchoring VII
COL4A6
fibrils
V COL5A1- Minor components in tissues containing
T ransmembrane XIII, XVII
COL5A3 collagen I
domain
Others XV, XVIII VI COL6A1- Most connective tissues
COL6A3
VII COL7A1 Anchoring fibrils at dermo-epidermal
junction
+ Just go through this chart and don't get confused by VIII COL8A1- Endothelium, other tissues
the genes & types as both are usually similar (i.e. COL8A2
type l=gene 1)
IX COL9A1- Tissues containing collagen II
★ Only remember those points which are dark & bold
COL9A3
X COL10A1 Hypertrophic cartilage
XI COL11A1, Tissues containing collagen II
COL11A2,
COL2A1
XII COL12A1 Tissues containing collagen I
XIII COL13A1 Many Tissues
XIV COL14A1 Tissues containing collagen I
XV COL15A1 Many Tissues
Endostatin forming collagen, in
XVI COL16A1 Many Tissues
endothelial cells.
XVII COL17A1 Skin hemidesmosomes
XV III COL18A1 Many Tissues (eg Liver, Kidney)
XIX COL19A1 Rhabdomyosarcoma cells
408 ■ A Complete Review of Short Subjects

Structure Post translational modifications Diseases


- All collagens have triple helical - Collagen is synthesized on
Disease Gene /Enzyme
structure® ribosom e® in precursor form- pre
Ehler d an los Syndrome®
- A striking feature is occurrence of procollagen, which contains a leader
glycine at every 3rd position of triple or signal sequence that directs it into - type IV CoL- 3A1
helical portion of a- chain. the lumen of en d op lasm ic - type VI Lysyl
- This repeating structure (Gly- X- reticulum® hydroxylase
- type VII AR
Y)n® is an absolute requirement for - In ER leader sequence is removed, Procollagen -N
the formation of triple helix hydroxylation of proline & lysine proteinase
- Proline and hydroxyproline® (formed and glycosylation of hydroxytysine COL- 1A1 & 1A2
in procollagen takes place - type VII AD
by hydroxylation of proline by prolyl
hydroxylase whose cofactors are vit C - Procollagen contains extension - O steogenesis COL - 1A1& 1A2
& a ketoglutarate) confirrigidity to peptides® (Polypetides of 20- 35 imperfecta®
collagen. These acquire 100 of X & 100 Kda) at its carboxy & amino (type 1)
of Y positions. terminal ends (not present in
- Osteoporosis
- Lysine in Y position is also modified mature collagen)
- Extension peptide contain cysteine - Severe COL 2A1
to hydroxy lysine through the action
residue. While amino terminal forms chondrodysplasia
of lysyl hydroxylase and similar
cofactors. only infra chain disulfide bonds, the - Osteo arthritis
- Galactose or galactosyl glucose is carboxy terminal form both intra - Schmid metaphysical COL- 10A1
added to hydroxylysines through an chain & inter chain disulfide bond. chondrodysplasia
O- glycosidic linkage, a glycosylat These assist in triple helix
- Alport Syndrome (AR COL -4A 3 & 4A6
ion site that is unque to collagen®. formation.
& X- linked)
- It is further stabilized by formation of - After formation of triple helix, no
further hydroxylation or - Epidermolysis COL - 7 A 1
covalent cross links both within and
glycosylation take place bullosa dystrophic
between the triple helical units
through the action of lysyl oxidase®, - Following secretion by the way of - M enkes disease Lysyl hydrxylase
a Cu dependent enzyme that golgiapparatus® and packed into
oxidatively deaminates the E- amino secretory vesicles®, extracellular ★ Scurvy also affects the structure of collagen
groups of lysine & hydroxy- lysine enzymes called procollagen ★ Marfan syndrome is d/t mutation in the
yielding reactive aldehydes. aminoproteinase and procollagen gene for fibrillin, a protein present in
- Several collages (eg. Type IV) are carboxy proteinase remove extension microfibrils
characterized by interruptions of peptides at the amino and carboxy ★ Hunter syndrome is a mucopoly
triple helix with stretches of protein terminal ends respectively, saccharidosis d/1 def. Of enzyme iduronidase
lacking Gly-X-Y repeats. This results in resulting in formation of collagen. that is involved in degradation of
areas of globular structure glycosaminoglycans
interspersed in triple helical
structure.

Proteoglycans & Glycosaminoglycans

- Proteoglycans are protein that contain covalently linked glycosaminoglycans. Examples are syndecan, betaglycan, serglycan,
perlecan, aggrecan, versican, decorin, biglycan & firomodulin.
- The protein of PG that bound covalently to glycosaminoglycans are called core proteins. It is synthesized in endoplasmic
reticulum and few GAG linkage also form here. Most of later GAG chains & modification occur in golgi apparatus.
- An O- glycosidic bond between xylose (Xyl) & ser is unique to proteoglycans.
- The amount of carbohydrate in PG is usually much greater than found in glycoproteins & may be upto 95%
- Aggrecan is found in cartilage®
- Glycosaminoglycans (GAG) is an unbranched polysacehride made up of repeating disaccharides, one component of which is
always an amino sugar, either D- glucosamine (Glc-N), or D- galactosamine (Gal-N). The other component (except in case of
keratan sulfate) is uronic acid either L-glucronic acid (Glc-UA) or its 5'epimer, L- iduronic acid (Id- UA)
- 7 types of GAGs are - hyaluronic acid, chondroitin sulfate, keratan sulfates I & II, heparin, heparan sulfate & dermatan sulfate
- With exception of hyaluronic acid, all GAGs contain sulfate groups, either as O- esters or N- sulfate ( in heparin & heparan
sulfate). Hyaluronic acid affords another exception because there is no clear evidence that it is attached covalently to protein
Cell Physiology ■ 409

M ajor proteins of Glycosaminoglycans

GAG Sugars Sulfate Linkage of protein Location


HA GlcNAc, GlcUA Nil No firm evidence Synovial fluid, vitreous humor,
loose connective tissue
CS GalNAc, GlcUA GalNAc Xyl-ser; assoiated with Cartilage, bone, cornea
HA via link proteins
K SI GlcNAc, Gal GlcNAc GlcNAc-Asn Cornea
K SII GlcNAc, Gal Same as KS I GalNAc-Thr Loose connective tissue
Heparin GlcN, IdUA GlcN Ser Mast cell
GlcN
IdUA
Heparan sulfate GlcN, GlcUA GlcN Xyl-Ser Skin fibroblasta, aortic wall

Dermatan sulfate GalNAc, IdUA, GalNAc Xyl-Ser Wide distribution


(Glc.UA) IdUa

_________________________Mucopolysaccharidoses (MPS)

Etiopathogenesis Type of M PS

• Degradation of GAGs is carried Type Syndrome Enzymatic Defect Accumulated /Urinary


out by a battery of lysosomal metabolite
hydrolases. These include- I Hurler or a - L- Id u ron id aseQ - D erm atan su lfa teQ
endoglycosides, Scheie - Hepran su lfa teQ
exoglycosidases, & sulfatase II Hunter Iduronate sulfatase
• Deficiency of one of the lysomal III A Sanfilippo A Heparan sulfate N- sulfatase
hydrolases involved in (sulfamidase)
degradation of one or more of III B Sanfilippo B a-N- Acetyl glucosaminidase H eparan s u lfa teQ
GAGs 1/1 m e Sanfilippo C Acetyl transferase
mucopolysaccharidoses. HID Sanfilippo D N- Acetylglucosamine 6
• This results in accumulation of sulfatase
GAGs in various tissues IV A M oriquio- A Galactosamine -6 - sulfatase - Keratan sulfate
including liver, spleen, bone, - Chondroitin -6 sulfate
skin & CNS. IV B M orquio -B P~ Galactosidase - Keratan sulfate
• GAGs are composed of VI Maroteauxlamy N- acetyl galactosamine 4- Dermatan sulfate
repeating disaccharide unit sulfatase (aryl sulfatase p
with either VII Sly p- Glucronidase - Dermatan sulfate
- amino sugar = d - glucosamine or - Heparan sulfate
or D- galoctosamine - Chondroitin 4/6- sulfate
- acid sugar = D- glucoronic acid
L- iduronic acid
• There is increased excretion o f M ucolipidosis (MLS)
accum ulated GAG, D- glucronic I Sialidosis Sialidase (neuraminidase) Glycoprotein fragments
acid or L- iduronic acid in
II I- Cell disease UDP- N- acetyl Glycoprotein fragments
urine Q.
glucosamine: glucoprotein-
N- acetylgluco
samininylphospho
transferase (Acid hydrolases
thus lack phosphomannosyl
residues)
III Pseudo Hurler As with ML-II but Glycoprotein fragments
Polydystrophy deficiency is incomplete
★ M ucolipidosis (MLS) denote diseases that combine features common to both mucopolysaccharidoses & sphingolipidoses.
410 ■ A Complete Review of Short Subjects

Keratins
Are fibrous insoluble proteins of animals derived from ectodermal (skin) cells. They include the structural protein elements of
skin (leather is almost pure keratin) as well as the biological derivatives of ectoderm, such as hair, wool, scale, nails, quills,
hoofs, horn & silk. There are two classes of keratins.

____________________ q- Keratins___________________ _________________________ P- Keratins________________________


- Are relatively rich in cystine residues and thus - Contain no cysteine or cystine but are rich in aminoacids with
contain many disulfide cross bridges small side chains particularly glycine, alanine, and serine
- They contain most of the common amino acids - Are found in the fibers spun by spiders and silkworins and in
- Include hard & brittle p rotein s containing high scales, claws and beaks of reptiles and birds.
conten t o f cystine (-22% ) and so higher number o f - An important difference is that the a- keratins stretch when
disu lfide bonds. Eg- n ails and horn®; as well as the heated; hair for example stretches when exposed to moist heat
softer, m ore flex ib le keratin s o f skin, h air and w ool, but contracts to its normal length on cooling. The p- Keratins
w hich contain ab o u t 10-14% o f cystine (lesser do not stretch under these conditions.
d isu lfide bonds)®.________________________________

Molecular Motors
Molecular motors are ATPase that move proteins, organelles, and other cell parts (their cargo) to all parts of the cells.

Those producing motion Those producing motion


along microtubules.Q along actin
I------------------------------' 1 '
Dyneins Kinesin Myosin
- moves the material towards the minus end - Moves the materials towards
of microtubules the plus end of microtubules.
- Involved in the movement of cilia & flagella

Axonal Transport

Fast Slow
i
— I—
Membranous cargoes Fast molar Nonmembranous cargoes Cytosolic proteins Slow motor protein
I proteins I I I
Golgi derived vesicles > I - M icrofilam ents, Clathrin, synapsin, - Neurofilament
E ndocytic vesicles, Dynein® cy to so lic proteins® » calmodulin, creatine kinase, protein & tubulinQ
lysosom es, Kinesin® - M icrotubules, aldolase & enolase - Myosin & dynein
A utophagosom es® > Vimentin N eurofilaments® Cytoskeletal proteins (?)
Mitochondria - Ribosomes, m-RNA Spectrin, tau, & dynactin

Axonal Transport of Membranous & Non membranous Cargoes


• It is now known that intracellular transport is coordinate by molecular motor proteins that bind cargoes and convey them in
a particular direction along cytoskeletal polymer tracks. This includes every type of membranous organelle and transport
vesicle (membranous cargoes), as well as nonmembranous cargoes such as cytoskeletal polymers, cytosolic protein
complexes, ribosomes, and messenger RNAs.
• Membranous organelles move most rapidly, in the fast components of axonal transport, whereas nonmembranous
cytoskeletal polymers and cytosolic protein complexes move more slowly, in the slow components. Recent studies suggest
that slow axonal transport is generated by fast motors and that the slow rate is due to rapid movements interrupted by
prolonged pauses.
• Materials destined for the axon are transported anterogradely, toward the axon dp, and materials destined to return are
transported retrogiadely, toward the cell body. This bidirectional transport process, known as axonal transport, is not
fundamentally different from the pathways of macromolecular and membrane traffic that occur in all eukaryotic cells, but it is
remarkable for its scale
Cell Physiology ■ 411

• Membranous organelles are the principal cargoes of fast axonal transport. The many proteins, lipids, and polysaccharides that
move along the axon at fast rates do so by virtue of their association with one or more subclasses of organelle or vesicle, either
because they are sequestered within its lumen, embedded in its membrane, or bound to its surface.
• Both neurofilaments and microtubules move at fast rates, approaching the rate of movement of membranous organelles, but
the average rate of movement is slow because the movements are both infrequent and bidirectional. Thus, the overall speed
and direction of neurofilament and microtubule movement is a temporal summation of anterograde and retrograde
movements and pauses, perhaps not fundamentally dissimilar from the behavior of mitochondria in axons described above.
As is the case for mitochondria, the slow overall rate of movement of neurofilaments and microtubules suggests that these
structures move with a low duty ratio, spending most of their time not moving.
• The rapid rate of movement of neurofilaments and microtubules in axons indicates that they are transported by fast motors,
perhaps similar or identical to motors that move membranous organelles. Several lines of evidence suggest that dynein may
transport axonal microtubules anterogradely, perhaps relative to the microfilament matrix, and that dynein and kinesin may
transport axonal neurofilaments bidirectionally along microtubules by the same mechanism that is thought to move vimentin
along microtubules in nonneuronal cells
• Cytoskeletal proteins have been the exclusive focus of studies on slow axonal transport in recent years, but it is important to
remember that several hundred other proteins also move in this rate group, representing the entire spectrum of cytosolic
proteins that comprise axoplasm. Some examples include proteins involved in vesicle dynamics such as clathrin and
synapsin; regulatory proteins such as calmodulin; metabolic enzymes such as creatine kinase, aldolase, and enolase;
cytoskeletal proteins such as spectrin, tau, and dynactin; and motor proteins such as dynein and myosin
• Cytoskeletal polymer tracks are made up of intracellular network of filamentous structure. All eukeryotic cells have 3 types
of filaments - Microfilaments (actin filaments)
- Microtubules (tubulin filaments)
- Intermediate filaments has 4 classes: keratins, neurofilaments, lamins and vimentin like proteins

Golgi Complex (Apparatus) / Dictyosome

• It is a collection of smooth membrane enclosed sacs (cisterns) that are stalked like dinner plates, usually near the nucleus. It
has membrane similar to those of smooth ER. It is usually composed of >4 stalked layers of >6 sacs in each apparatus
• It is a polarized structure with a cis (proximal) and trans (distal) sides 0.

Cis side of GC receives or transpost In median part of golgi lumen, post Trans side of GC release
endoplasm ic reticulum (ER) vesicles 0 translational modification takes place to proteins in secretory vesicle Q,
containing newly synthesized form glycoprotein & lipoprtein by adding which shuttle to cell surface &
proteins carbohydrate & lipid moeity respectively lysosome

★ Presence of SNAREs (Soluble N-ethylmalemite - sensitive


factor attachment receptor) protein determine, Transpon ER- Secretory vesicles
where inside the cell vesicle would go. V (for vesicles Secretory granule
vesicle) SNAREs interact with t (for target)
SNAREs. In this way vesicles are ticketed for
specific loci (eg cell membrane , GC, lysosome) by
particular molecule such as mannose- 6 -
phosphate. Nucleus
•4
• o• C e ll m e m b ra n e
o
★ Initial glycosylation o f proteins occurs in ERQ by •oo
attachment of preformed oligosaccharides but o°
these oligosaccharides undergo modifications to form >o°
variety of different carbohydrate moieties in golgi GC
apparatus.
★ Lysosome is present in all animal cells except RER
C is-cn d T ran s-cn d
RBC <2. It has pH (~5) lower than that of cytosol. L Polarized GcJ
Acid phosphatase is its marker
412 ■ A Complete Review of Short Subjects

Enzymes & Metabolic Functions of Mitochondria

Outer Membrane Inter membrane Inner Membrane Soluble Matrix


I space (b/w I I
outer & inner
• For activation & • Relative impermeability of inner • Pyruvate dehydrogenase
membrane)
transport of fatty acids membrane necessitates exchange • Enzymes of TCA (citric acid
(FA) I transporters cycle)
- Acyl Co A synthetase • Creatine - Phosphate & adenine nucleotide - Citrate synthase
(thiokinase) activates kinase® (of transporter
- Aconitase
free FA creatine - Dicarboxylate & tricarboxylate
phosphate - Isocitrate dehydrogenase
- Carnitine palmitoyl transporter
shuttle - a-Ketoglutarate (a-
transferase 1 - Pyruvate symport
facilitating oxaloglutarate) dehydrogenase
(transports longchain - a-ketoglutarate and
transport of - Succinate thiokinase (succinyl
FA as carnitine glutamate/aspartate transporter (of
high energy Co A synthatase)
derivatives) Mallae shuttle)
phosphate - Fumarase
• Biosynthesis of from - Glycerol 3 phosphate dehydrogenase
triacyglycerol & - Malate dehydrogenase
mitochondria (of Glycerophosphate shuttle) on outer
phospholipids ) in active surface • Anplerotic reactions (by which
- Glycerol phosphate acyl tissues such cycle intermediates are
- Energy linked proton tranlocating
transferase as heart & siphoned off in other
transhydrogenase
pathways or replenished)
• Monoamine oxidase skeletal - Carnitine palmitoyl transferase 2 and
(MAO) for muscle - Aspartate amino transferase
carnitine acyl carnitine translocase
neurotransmitter (SGOT)®
• Adenyl (in transport of long chain of FA)
metabolism - Glutamate dehydrogenase®
(adenylate • Ketone body formation
• Kynureninase (tryptophan kinase) • Both in urea cycle
- D (-) -3- hydroxybutyrate
metabolism) • Nucleoside dehydrogenase • Urea cycle enzyme
• Phospholipase A 2 diphosphokina • Enzymes of oxidative - Carbamoyl phosphate
• Nucleoside di se phosphorylation or respiratory chain synthetase 1®
phosphokinase • Sulfite oxidase which include - N-acetylglutamate synthase
• Cytochrome bs & - Electron transferring flavoproteins - Ornithine transcarbamoylase®
cytochrome bs reductase eg complex I (NADH • Alcohol metabolism
• Uncoupling proteins dehydrogenase complex II - ALDH2
• Dihydrorotate (Succinate dehydrogenase)
• Glutaminase
dehydrogenase (of - CoenzymeQ (ubiquinone)
• Enzymes of P oxidation
pyrimidine metabolism) - Electron transferring cytochromes
• Heme synthesis (part)
eg Complex III (cytochrome b Ci)
- ALA (aminolevulinate)
cytochrome c
synthase
Complex IV(Cytochrome aas or
oxidase)
- ATP synthase (F1/F0 ATPase or
complex V for ATP synthesis)

Mitochondria both import and synthesize proteins

13 proteins are encoded by mitochondrial genome & synthesized in that orgenelle using its own protein synthesizing system
Majority (about 100) are encoded by nuclear gene on cytosolic polyribosome and imported to mitochondria
Cell Physiology 413

Location of Liver Enzymes

Plasma (cell) membrane (bound) Cytosolic or Cytoplasmic Mitochondrial Matrix

- 51 N u cleotidase (or N u cleotide - Lactate dehydrogenase (LDH) - G lu tam ate dehydrogenase (GDH)®
p h osp h atase, NTP)® - SGPT (Serum glutamate pyruvate - SGOT (serum glutamate
- Gamma Glutamyl Transferase (or transaminase) or ALT (alanine oxaloacetate transaminase) or AST
Transpeptidase = GGT) amino transferase) (aspartate aminotransferase) -
- Alkaline phosphatase (ALP) - SGO T (AST) cytosolic isoenzyme m itoch on d rial isoenzyme®

Major Proteins of

Cartilage____________________ | |
________________________________Bone

Collagen Protein Non Collagen protein Collagen Non Collagen Protein


M n-"ACP" protein
• C ollagen type • Anchorin C-2 • C ollagen • Bone specific
2® (90-98% of • Chondronectin type 1® - Osteocalcin
all articular • Proteoglycans (90% of - Bone sialoprotein
cartilage - Aggrecan® (major total bone - CSPG III
collagen) proteoglycan) protein) • Non specific
• Collagen - 5, 6, - Non - aggregating • Collagen - Bone morphogenetic proteins (BMPS)
9,10,11 large proteoglycan type-5 - Osteopontin
- DS - PGI (biglycan) - Osteonectin (Bone SPARC Protein)
- DS - PG2 (decorin) - Proteoglycans C S-PG1 (biglycan) & CS - PG2
(decorin)
- Plasma proteins

Cytoskeleton

The cell cytoplasm is not a sac of fluid. Essentially all eukaryotic cells contain three types of filamentous structures (responsible
for cell sh ap e change)®.

__________ I
Actin (Micro) Inter mediate Filaments Micro tubules
Filaments (10-12 nm in diameter) (25 nm in diameter)
(7- 9.5 nm in diameter)
G - actin (present in Most are relatively stable components - Are necessary fo r form ation and function o f the m itotic
most cells) of cytoskeleton not under going rapid spindle® and thus are present in a ll eukaryotic cells.
spontaneously assembly & disassembly and not - Involved in intracellular movements o f endocytic &
polymerizes to form a disappearing during mitosis, as do exocytic vesicles®, and form the major structural
double helical F- actin actin and many microtubular component of cilia &flagella.
filaments in filaments. As important exception is
laminin, which subsequent to - In axons & dendrites they maintain structure & participate
appropriate conc. of
Mg++ and KCl phosphorylation, disassemble at in axoplasmic flow of material.

F actin form mitosis and reappear when it - Microtubule consists of 13 longitudinally arranged
microfilaments that terminates. profilaments, each consisting of dimers of a & ft tubulin. +
exists as bundles of Organizing center, located around a pair of centriole, y-
tangled appearing tubulin, GTP and MAPsQ (microtubule associated
meshwork proteins eg. tau ) play important role in assembly &
underlying plasma stabilization
414 w A Complete Review of Short Subjects

membrane referred to Proteins Distributions Microtubules are in state o f dynamic instability®


as stress fibers. constantly assembling and disassembling. They exhibit
Keratins
Stress fibers polarity (plus & minus ends)® this is important in their
Type I (acidic) Epithelial cells, hair, growth & their ability to direct intracellular movements.
disappear as cell
motality increases or Type II (basic) nails Some drugs eg colchicines, vinblastin, paclitaxel &
upon malignant Vimentin-like griseofulvin® bind to micro tubule and interfere with their
transformation of assembly or disassembly
cells by chemical or Vimentin Various
onchogenic viruses. mesenchymal cells
Muscle Protein Function
Desmin
Dynamin (use Involved in endocytosis
Glial fibrillary Glial cells
GTP)
acid protein
Kinesin In Axonal transport to move vesicle
Peripherin Neurons
(use ATP) down the axon to wards the +ve
Neurofilaments end of microtubule formation
Low (L), Neurons Cytosolic Dynein In axonal transport flow of
medium (M), material towards negative end
(use ATP)
and high (H)
Axonemal Power ciliary flagellar movement
Lamins
Dyneins
A, B and C Nuclear lamina

Cell Cycle

- Cell cycle is the period of time between Phase State Description


the birth of a cell and its own division to
GO Gap 0 Quiescent Cells that retain the capacity for
produce two daughter cells. It can be
(Senescent or proliferation, but which are no longer
divided into 4 distinct phases, which
Resting) Phase dividing have entered resting (quiescent)
are G l, S, G2, M. The combination of
phase, even though they may be quite
G l, S & G2 phases is k/a interphase
active physiologically. Growth factors can
and M phase is the mitotic phase.
stimulate quiescent cell to leave Go and re
- The duration of cell cycle and its stages enter the cell cycle.
varies greatly between cell types. The
Gl Gap 1 Presynthetic phase In this phase cell responds to growth factors,
time taken fo r S, G2 and M phases are
irreversibly directing cells to start another cell
sim ilar fo r m ost cell types®, occupying
cycle. During G l, most of the molecular,
about 6-8, 2-4 and 1-2 hours respectively
machinery required to complete another
whereas the duration of Gj show s
cell cycle is generated. Proteins encoded by
considerable variation® as short as 2
certain tumor suppressor genes (eg Rb)
hours in rapidly dividing cells i.e.
block the cell cycle in G l check point.
embryonic tissue or longer than 12 hours
in some adult tissue. S Synthesis DNA synthesis DNA synthesis (or replication o f genome)®
(replication) occurs during S phase, at the end of which
- Gi phase - is m ost variable® because, in
the DNA content of cell has doubled.
this phase cells are not committed to
DNA replication, can enter resting state or G2 Gap 2 Premitotic During G2, cell prepares for division;
progress to next cell division. In mammals cytoplasmic enlargement & DNA repair can
because of the time required for a cell to occur; phase ends with onset of
progress of S - phase through mitosis chromosome condensation and breakdown
from beginning is typically 12-24 hours of nuclear membrane
irrespective of duration of Gi phase;
M M itosis Cell division Cell growth stops and cell divides into 2
almost all the variation in proliferating
(mitosis) daughter cells with identical copies of
rates are attributable to the amount o f
parent cell genome
time spent in G1/G 0 state®.
Cell Physiology 415

NOTES
416 ■ A Complete Review of Short Subjects
BIOCHEMISTRY

MCQs with Answers and Explanations


Chapter -1 CARBOHYDRATE METABOLISM

QUESTIONS

Glycolysis A. Enolase □
B. Phosphofructokinase □
1. Enzyme catalyzing reversible step in glycolysis is /are: C. Pyruvate kinase □
A. Phosphofructokinase (PGI 14,10) □ D. Glyceraldehyde-3-phosphate dehydrogenase □
B. Enolase □ E. Hexokinase □
C. Pyruvate kinase □ 10. In glycolysis,the first commited step is catalysed by :
D. Phospho-glyceromutase □ A. 2 ,3 DPG (AIIMS 07) □
E. Glyceraldehyde-3-phosphate-dehydrogenase □ B. Glucokinase □
2. Enzyme responsible for com plete oxidation of glucose C. Hexokinase □
to CO 2 to w ater is present in (AIIMS 15,07) D. Phosphofructokinase. □
A. Cytosol □ 11. The rate-lim iting enzyme in Glycolysis is : (AI 02)
B. Mitochondria □ A. Phosphofructokinase □
C. Lysosomes □ B. Glucose-6-dehydrogenase □
D. Endoplasmic reticulum □ C. Glucokinase □
3. About glycolysis true is: (PGI 09) D. Pyruvate kinase □
A. Occurs in mitochondria □ 12. Glycolytic enzymes (s) inhibited by fluoride:
B. Complete breakdown of glucose □ A. Hexokinase (PGI 08) □
C. Conversion of glucose to 3C units □ B. Aldolase □
D. 3 ATP's are used in anaerobic pathway □ C. Enolase □
4. Fluroide ions act by inhibiting - (PGI 05) D. Pyruvate Kinase □
A. Enolase □ E. Phosphofructokinase □
B. Hexokinase □ 13. Fluoride, used in the collection of blood samples for
C. Cytochrome oxidase □ glucose estimation, inhibits the enzyme: (AI 05)
D. Carbonic anydrase □ A. Glucokinase. □
5. In glycolytic pathway, products formed are B. Hexokinase. □
A. Fructose-2, 6-Biphosphate (AIIMS 12) □ C. Enolase. □
B. Fructose-1, 6-Biphosphate □ D. Glucose-6-phosphatase. □
C. Glyceraldehyde-3-Phosphate □ 14. Fluoride inhibits which enzyme (AI 03)
D. All of the above □ A. Pyruvate kinase □
6. Compound that joints glycolysis with glycogenesis & B. Succinyl dehydrogenase □
glycogenolysis (Jipmer 04, WB-03, Assam 05) C. Enolase □
A. Glucose 1, 6 bi phosphate □ D. Aldolase □
B. Glucose 1 P O 4 □ 15. Anticoagulant used to estimate glucose from a sample
C. Glucose 6 P O 4 □ sent from PHC is: (AIIMS 11, 06)
D. Fructose 1, 6 bi phosphate □ A. EDTA □
7. Peroxidase enzym e is used in estim ating B. Calcium oxalate □
A. Haemoglobin (AIIMS 07) □ C. Potassium oxalate + NaF □
B. Ammonia □ D. Sodium citrate □
C. Creatinine □ 16. Which of the follow ing anticoagulant used in
D. Glucose □ estimating blood glucose prevents glycolysis -
8. W hich of the follow ing enzym es catalyze the A. Oxalate (DNB 06) □
irreversible steps of glycolysis? (AIIMS 13) B. Citrate □
A. Glucokinase, phosphofructokinase, pyruvate C. NaF □
carboxylase □ D. Heparin □
B. Hexokinase, fructosel, 6 biphosphatese, pyruvate 17. Cancer cells derive nutrition from (AIIMS 01)
kinase □ A. Glycolysis □
C. Glucokinase, phosphofructokinase, pyruvate B. Oxidative phosphorylation □
kinase □ C. Increase in mitochondria □
D. Enolase, fructose 1,6 biphospahatase, D. From a fast food joint □
phosphofructokinase □ 18. Insulin acts on which enzyme in glycoysis- (AI 03)
9. Key glycolytic enzym es for irreversible step (s) in A. Glucokinase □
glycolysis is/are: (PGI 12, 08) B. Hexokinase □
420 ■ A Complete Review of Short Subjects

C. Glucose 6-Phosphatase □ 28. Malate shuttle is important in (AIIMS 04, AI 08)


D. Adenylate kinase □ A. Glycogenolysis □
19. True statements about Glucokinase is/are: (PGI 03) B. Glycolysis □
A. Km value is higher than normal blood sugar □ C. Gluconeogenesis □
B. Found in liver □ D. HMP shunt □
C. G-6P inhibit it. □
PDH Complex
D. Has both Glucose 6 Phosphatase and Kinase
activity □ 29. Congenital lactic acidosis may occur d/t defect in
E. Glucose enter into cells through GLUT-2 □ A. Pyruvate decarboxylase (AI 10) □
20. After overnight fasting, levels of Glucose transporters B. PDH complex □
are reduced in (AIIMS 10, AI 09) C. Transketolase □
A. Brain cells □ D. a-ketoglutarate dehydrogenase □
B. Hepatocytes □ 30. First substrate of Kreb's cycle is: (AIIMS 07)
C. Adipocytes □ A. Pyruvate □
D. RBCs □ B. Glycine □
21. Insulin dependent entry of glucose is seen in- C. HC1 □
A. Liver (N B E P 1 5 )n D. Lipoprotein □
B. Brain □ 31. Coenzymes for pyruvate dehydrogenase are A/E
C. Heart □ A. TPP (Jipmer 03, WB 02, KA 0 1 ) 0
D. Kidney □ B. CoA □
22. Glucose is transported in pancreas through which C. Lipoic acid □
receptor- (NBE P 15) D. FAD □
A. GLUT 1 □ E. NADPH □
B. GLUT 2 □ 32. Pyruvate dehydrogenase complex has all enzyme
C. GLUT 3 □ components except (SG PG I04, AMU 05, UP 05)
D. GLUT4 □ A. Decarboxylase □
23. Within the RBC, hypoxia stimulates glycolysis by B. Transacetylase □
which of the following regulating pathways? (AI 07) C. Dehydrogenase □
A. Hypoxia Stimulates pyruvate dehydrogenase by D. Phosphatase □
increased 2,3 DPG □ 33. Pyruvate dehydrogenase contains all except -
B. Hypoxia inhibits hexokinase □ A. Biotin (NBE P 1 3 ,1 4 )0
C. Hypoxia stimulates release of all Glycolytic enzymes B. NAD □
from Band 3 on RBC membrane □ C. FAD □
D. Activation of the regulatory enzymes by high PH □ D. CoA □
24. TCA cycle does not take place in - (JIPMER 02, AI 05) 34. Which of the following vitamins does not pariciate in
A. Hepatocyte □ oxidative decarboxylation of pyruvate to acetyl CoA-
B. Osteocyte □ A. Thiamine (NBE P 15) □
C. Neuron □ B. Niacin □
D. Erythrocyte □ C. Riboflavin □
25. Mature RBC contains all except - (AIIMS 04) D. Biotin Cl
A. Enzyme of HMP shunt pathway □ 35. Pyruvate can be converted directly into all the
B. Enzyme of TCA cycle □ follow ing except: (AI 02)
C. Glycoytic enzymes □ A. Phosphoenol pyruvate Cl
D. Pyridine nubleotides □ B. Alanine Cl
26. The end product of one stage fermentation is - C. Acetyl Co A Cl
A. Formic acid (JIPMER 05, PGI 04) □ D. Lactate Cl
B. Pyruvate □ 36. Arsenic inhibits all except (AI 009, UP 10)
C. Lactate Q A. PDH □
D. Ethanol □ B. Lipoic acid Cl
C. a-KG dehydrogenase Cl
Shuttle Systems D. Enolase Cl
37. Thiamine deficiency causes decreased energy
27. During gluconeogensis reducing equivalents from production because (AI 11, AIIMS 08)
mitochondria to cytosol are transported by A. It is required for the process of transamination
A. Malate (AIIMS 0 2 ) 0 B. It is co-factor in oxidative reduction
B. Aspartate d C. It is co-enzyme for transketolase in pentose
C. Glutamate Cl phosphate pathway
D. Oxaloacetate Cl D. It is co-enzyme for pyruvate dehydrogenase.
Biochemistry: Carbohydrate Metabolism ■ 421

Citric Acid Cycle 47. W hich of the follow ing substance binds to acetyl CoA
and condenses OAA, inhibiting the TCA cycle:
38. Oxaloacetate + Acetyl CoA Citrate + CoASH A. Arsenite (AIIMS 10) □
This reaction is: (AIIMS 04) B. Fluoroacetate □
C. Malonate □
A. Reversible □
D. Fumararte □
B. Irreversible □
48. M alonate competetively inhibits- (AIIMS 05)
C. Can be reversed by catalase □
A. Fumarate dehydrogenase □
D. Competitive □
B. Succinate thiokinase □
39. In TCA cycle of tricarboxylic acid, which is first
C. Aconitase □
formed (Assam 04, WB 01, Bihar 02)
D. Succinate dehydrogenase □
A. Isocitrate □
49. Number of ATPs produced from adipose tissue from 1
B. Citrate □
NADH (NAD+/NADH) through respiratory (PGI 11)
C. Succinate □
A. 0 ATP □
D. Fumarate □
B. 1 ATP □
40. In TCA cycle, citrate is converted into, after losing a
C. 2 ATP □
molecule of H20 - (DNB 09, UP 01)
D. 2.6 ATP □
A. Isocitrate □
E. 3 ATP □
B. Cis-aconitate □
50. A TP'S are formed in the follow ing steps of kreb's
C. Oxaloacelate □
cycle: (PGI 06)
D. Glutarate □
A. Isocitrate dehydrogenase □
41. Which of following inhibitor in TCA cycle acts by
B. Succinate dehydrogenase □
blocking citrate - (KA 03, TN 02)
C. Succinate thiokinase □
A. Fluroacetate □
D. Malate dehydrogenase □
B. Arsenite □ 51. In TCA cycle, C 0 2 is released from: (PGI 08)
C. Malonate □ A. Thiokinase □
D. None of the above □ B. Isocitrate dehydrogenase □
42. Fluroacetate inhibits - (AP 01, CMC 09, JIPMER 08) C. Citrate dehydrogenase □
A. Citrate synthetase □ D. Alpha ketoglutrate □
B. Aconitase □ 52. Proper functioning of Kreb cycle does not occur with
C. Succinate dehydrogenase □ deficiency of: (PGI 2K)
D. Alphaketoglutarate dehydrogenase □ A. Thiamane □
43. Which one of the follow ing is the correct sequential B. Riboflavin □
order in which the given enzymes of Kreb's cycle are C. Vitamin E □
formed after a m olecule of acetyl CoA enters the cycle- D. Vit K □
A. Citrate, Oxaloacetate, Ketoglutarate (DNB 11) □ 53. W hich among the follow ing controls is anallosteric
B. Ketoglutarate, Oxaloacetate,Citrate □ inhibitor of TCA cycle: (PGI 09)
C. Citrate, Ketoglutarate,Oxalocetate □ A. Pyruvate dehydrogenase □
D. Oxaloacetate, Ketoglutarate,Citrate □ B. Keto glutarate dehydrogenase □
44. Specific poison for succinate dehydrogenase is - C. Isocitrate dehydrogenase □
A. Cyanide (Delhi 03, Raj 04, Rohtak 04) □ D. Malate dehydrogenase □
B. Malonate □ 54. In vivo control of citric acid cycle is effected by:
C. Arsenite □ A. Acetyl CoA (PGI 01) □
D. Fluoride □ B. Coenzyme A □
E. Malate □ C. ATP □
45. What is the substrate for the only physiologically D. Citrate □
irreversible reaction in the Kreb's Citric acid cycle- E. NADH □
A. Citrate (NBE P 13) □ 55. O f the follow ing groups of amino acids produce
B. Alpha ketoglutarate □ common component of the TCA cycle is:
C. Succinate □ A. Alaine, isoleucine, leucine, lysine (PGI 09) □
D. Malate □ B. Serine, asparagines, glycine, glutamate □
46. Intermediate metabolite in TCA cycle are A/E: C. Isoleucine, valine, methionine □
A. Pyruvate (PGI 06) □ D. Prolone, leucine, tryptophan □
B. Isocitrate □ 56. Substrate level phosphorylation is catalysed by :
C. Oxaloacetate □ (AIIMS 10,14; NBE P 15,14)
D. Malonate □ A. Succinate dehydrogenase □
E. Nitric oxide □ B. Succinate Thiokinase □
422 ■ A Complete Review of Short Subjects

C. Malate dehydrogenase □ 66. Riboflavin is a constituent of- (NBE P 15)


D. Hexokinase □ A. FMN □
57. Substrate level phosphorylation is seen in the B. NAD □
conversion of: (NBE P 15,14; AI 09) C. PLP □
A. Acetoacetate to a-keto glutarate □ D. THF □
B. Succinyl CoA to Succinate □ 67. In oxidative phosphorylation, the ATP production and
C. Fumarate to malate □ respiratory chain are linked by: (AI 02)
D. Succinate to fumarate □ A. Chemical methods □
58. Substrate-level phosphorylation seen in reaction B. Physical methods □
catalysed in: (PGI 15,13,08) C. Chemiosmotic methods □
A. Succinate dehydrogenase □ D. Conformational changes □
B. Alfa-ketoglutarate dehydrogenase □ 68. About oxidative phosphorylation true is:
C. Succinyl CoA thiokinase □ A. Generation of ATP's (PGI 07) □
D. Malate dehydrogenase □ B. Generation of ADP's □
E. Cis-aconitase □ C. Utilization of ATP's □
D. Utilization of NADP □
High Energy Compounds 69. ATP is generated in ETC by- (NBE P 15)
59. Im m ediate energy supply for m uscle contraction- A. Na + k+ ATPase □
A. GTP (NBE P 15) □ B. Na + Cl-ATPase □
B. ATP □ C. Fo Fi ATPase □
C. Creatine phosphate □ D. ADP Kinase □
D. Fatty acid □ 70. W hich com ponent transfers four protons:
60. Im m ediate source of energy is- (NBE P 15) A. NADH-Q Oxidoreductase (PGI 10, 09) □
A. Cori's cycle □ B. Cytochrome -C oxidase □
B. HMP □ C. Cytochrome C - Q oxidoredictase □
C. ATP □ D. Isocitrate Dehydrogenase □
D. TCA cycle □ E. Succinate Q Reductase □
61. W hich energy molecule gives 10.5 kcal/m olecule- 71. In ETC, Oxidative phosphylartion (ATP form ation) is
A. ATP (NBE P 15) □ regulated by: (PGI 11)
B. GTP □ A. NADH Co-Q reductase □
C. Creatine phosphate □ B. Cytochrome C oxidase □
D. Glucose-6=phosphate □ C. Glutathione reductase □
62. W hich of the follow ing is high energy phosphate bond D. Isocitrate dehydrogenase □
(produce ATP on hydrolysis): (NBE P 15,13;PGI 15,11) E. Co-Q-Cytochrome C reductase □
F-6-phosphate
A. □ 72. True about ATP synthase is all except: (PGI 05)
B.
Creatine phosphate & Carbamoyl phosphate □ A. One rotation produce 3 ATP □
C.
Glucose-1-phosphate □ B. Fo unit functions as proton channel □
D.
Glucose-6-phosphate □ C. a and p subunits rotate □
63. High energy phosphate com pounds are: (PGI 12,10) D. Bent axle is formed by e and y subunits □
A. ATP □ 73. Internal respiration are: (PGI 03)
B. ADP □ A. Exergonic and anabolic □
C. Creatinine phosphate □ B. Exergonic and catabolic □
D. Acetyl CoA □ C. Endergonic and anabolic □
E. Glucose 1 phosphate & glucose 3 phosphate □ D. Endergonic and catabolic □
E. Cytochrome-C reductase involved □
Respiratory Chain and Oxidative Phosphorylation 74. M itochondrial m em brane protein contain transporter
of:
64. Am ong the following, the m axim um redox potential is
for:
A. NADH (PGI 03) □
A. NADH/NAD (PGI 08, 05) □ B. Acetyl COA □
B. Succinate/Fumarate □ C. NADPH □
C. Ubiquinone □ D. ATP □
D. Fe+3/ Fe+2 □ 75. Inem al respiration are: (PGI 03)
65. W hich vitam in is involved in Redox reactions- A. Exergonic and anabolic □
A. Pyridoxin (NBE P 15) □ B. Exergonic and catabolic □
B. Biotin □ C. Endergonic and anabolic □
C. Folic acid □ D. Endergonic and catabolic □
D. Riboflavin □ E. Cytochrome-C reductase involved □
Biochemistry: Carbohydrate Metabolism ■ 423

76. Cyanide is toxic because it - (DNB 14, PGI 04) 86. During phagocytosis, the m etabolic process called
A. Inhibits cytochrome oxidase □ respiratory burst involves the activation of:
B. Forms cyan meth Hb □ A. Oxidase (AI 06) □
C. Inhibits ATP carrier in mitochondria □ B. Hydrolase □
D. Inhibits Na-K-ATP ase □ C. Peroxidase □
77. Cytochrome oxidase in oxidative phosphorylation is D. Dehydrogenase □
inhibited by: (PGI 06, 02,14) 87. Cytosolic Cytochrome C mediates (AIIMS 06)
A. CO □ A. Apoptosis □
B. H2S □ B. Electron transport □
C. Cyanide □ C. Krebs cycle □
D. Amobarbital □ D. Glycolysis □
E. Rotenone □ 88. RQ of a person on pure carbohydrate diet w ill be:
78. Cyanide affects respiratory chain by- (NBE P 15) A. 1 (AIIMS 16) □
A. Non-competitive reversible inhibition □ B. 0.7 □
B. Competitive reversible inhibition □ C. 1.2 □
C. Suicide irreversible inhibition □ D. 1.5 □
D. Non-competitive irreversible inhibition □
79. The electron flow in Cytochrome C oxidase can be Glycogen Metabolism & Storage Disease
blocked by: (AIIMS 14, 06)
A. Rotenone □ 89. Branching enzyme is found in- (NBE P 15,13)
B. Antimycin-A □ □
A. Glycogenesis
C. Cyanide □ B. Glucogenesis □
D. Actinomycin □ C. Glycogenolysis □
80. Cellular oxidation is inhibited by : (AI 09; NBE P 13) D. Glycolysis □
A. Cyanide □ 90. M ain enzyme for glycogen metabolism- (NBE P 15,14)
B. Carbon dioxide □ A. Glucose-6- phosphatase □
C. Chocolate □ B. Glycogen synthase □
D. Carbonated beverages □ C. PFK-1 □
81. The specialized mammalian tissue/organ in which fuel D. None of the above □
oxidation serves not to produce ATP but to generate 91. In glycogen metabolism, some metabolically important
heat is: (AI 06) enzymes found in the liver are converted from their
A. Adrenal gland □ inactive dephosphorylated state to active
B. Skeletal muscle □ phosphorylated state. Which of the follow ing is true?
C. Brown adipose tissue □ (AIIMS 12)
D. Heart □ A. Always activated by c AMP dependent protein
82. Dinitrophenol causes: (AI 07) kinase □
A. Inhibition of ATP synthase □ B. Commonly seen in fasting state than in fed state □
B. Inhibition of electron transport □ C. Catecholamines directly cause their
C. Uncoupling of oxidation and phosphorylation □ phosphorylation □
D. Accumulation of ATP □ D. ATP activates phosphorylase □
83. Which of follow ing prevent formation of ATP by 92. Adrenaline acts on which enzyme in glycogenolysis-
blocking movement of ADP across mitochondrial A. Glucokinase (AI 09) □
membrane: (AIIMS 10) B. Hexokinase □
A. Atractyloside □ C. Phosphorylase □
B. Rotenone □ D. Glue diphosphatase □
C. Oligomycin □ 93. The m ajor fate of glucose-6 Phosphate in tissue in a
D. Ouabain □ well fed state is: (AI 07)
84. Atractiloside act as- (NBE P 15) A. Hydrolysis to glucose □
A. Uncoupler □ B. Conversion to glycogen □
B. Inhibitor of oxidative phosphorylation □ C. Isomerization to fructose 6 phosphate □
C. Inhibitor of complex 1 OF ETC □ D. Conversion to ribulose 5 phosphate □
D. Inhibitor of complex III of ETC □ 94. In the fed state, m ajor fate of Glucose 6- P O 4 in tissues
85. Natural uncoupler is- (NBE P 15) is (AIIMS 03)
A. Thermogonin □ A. Storage as Fructose □
B. 2,4 nitrophenol □ B. Storage as Glyceraldehyde 3 -PO 4 □
C. 2,4 Dinitrophenol □ C. Enters HMP shunt via Ribulose 5 -PO 4 □
D. Oligomycin □ D. Storage as glycogen □
424 ■ A Complete Review of Short Subjects

95. I n m u s c le , p h o s p h o r y la s e b i s in a c t iv a t e d b y : C. Pompe's disease □


A. cAMP (AIIMS 08) □ D. Anderson's disease □
B. Ca ions □ E. Fabry's disease □
C. Glucose □ 105. An infant has hepatorenomegaly, hypoglycemia
D. ATP □ hyperlipidemia, acidosis & normal structured glycogen
96. T r u e r e g a r d in g g ly l c o g e n o ly s is i s /are A/E (PGI 08) deposition in liver. What is the diagnosis:
A. PPi is a dephosphorylating enzyme □ A. H ers disease (PGI 01) □
B. Vasopressin increases it □ B. Von Gerke's ds □
C. Oxytocin decreases it □ C. Cori's ds □
D. Ca++ is a synchronizerand allosteric activator □ D. Anderson's ds □
E. 5 subunit is catalytic □ E. Pompe's ds □
97. P h o s p h o r y la s e b is m a in t a in e d i n a n in a c t iv a t e d s ta te 106. A person with von G ierke's disease has ketosis. All are
by (AI 08) true for ketosis in this patient except: (AIIMS 13)
A. ATP □ A. Lactic acidosis □
B. c AMP □ B. Hypoglycemia □
C. Calcium □ C. Fat mobilization is less □
D. Insulin □ D. Hypertriglyceridemia □
98. W h ic h o f t h e f o l l o w in g i s a d e b r a n c h i n g e n z y m e - 1 0 7 .T h e c a u s e o f h y p e r u r ic e m ia a n d g o u t i n g lu c o s e - 6-
A. Glycogen synthetase (AIIMS 15,14,10) □ p h o s p h a ta s e d e fic ie n c y is : (AIIMS 01)
B. Glucose-6-phosphatase □ A. More formation of pentose □
C. Amylo (1,6) glucosidase □ B. Decreased availability of glucose to tissues □
D. Amylo 1,4, - 1 , 6 transglycosylase □ C. Increased accumulation of sorbitol □
99. S e q u e n c e o f e v e n ts i n g l y c o g e n o l y s i s ? □ D. Impaired degradation of free radicals □
A. Phosphorylase, Glucan transferase, Debranching, 1 0 8 .G ly c o g e n s t o r a g e d is e a s e s i n c lu d e a l l th e f o l l o w in g
Phosphorylase (PGI 14,08) □ e x c e p t:
B. Debranching, Phosphorylase, transferase, A. Von Gierke's disease (PGI 06) □
phosphorylase □ B. Fabry's disease □
C. Transferase, phosphorylase, Debranching, C. Me Ardle's disease □
phosphorylase □ D. Fragile syndrome □
D. Any of the above. □ E. Krabbe's disease □
lOO.In w h i c h o f t h e f o l l o w in g t is s u e s , i s g ly c o g e n 1 0 9 .C o ris d is e a s e i s d u e to d e f e c t i n - (NBE P 15)
in c a p a b l e o f c o n t r i b u t i n g d ir e c tly to b l o o d g l u c o s e ? A. Branching enzyme □
A. Liver (A I0 8 )D B. Debrandhing enzyme □
B. Muscle □ C. Myophosphorylase □
C. Both □ D. Hepatic phosphorylase □
D. None □ 110.Which among the follow ing is not a cause of fasting
1 0 1 .T h e e n z y m e n o t p r e s e n t i n t h e s k e l e t a l m u s c le s OR hypoglycemia? (AI 11)
M u s c l e c a n n o t m a k e u s e o f g ly c o g e n f o r e n e r g y A. Glucagon excess □
(AIIMS 08,15; PGI 14,12,06)
b e c a u s e o f d e fic ie n c y o f B. Glucose 6 phospatase deficiency □
A. Creatinine phosphokinase □ C. Uremia □
B. Hexokinase □ D. Glycogen synthase deficiency □
C. Phosphofructokinase □ 111.Metabolites in HMP shunt are all except
D. Glucose 6 phosphatase. □ A. Glycerol-3 phosphate (AI 05) □
102.W h ic h o f th e f o l l o w in g d o e s n o t c r o s s t h e c e ll B. Sedoheptulose-7 phosphate □
m em b ra n e C. Glyceraldehyde-3 phosphate □
A. Glucose-6-phosphate (AIIMS 02) □ D. Xylulose-5 phosphate □
B. Glucose □ 112.NADPH is produced by OR The major source of
C. Nitrous oxide □ NADPH for fatty acid synthesis:
D. Carbon monoxide □ A. Glycolysis (AIIMS16,13,08; NBE P 14,13) □
103.M u s c l e s a r e n o t in v o lv e d i n w h ic h g ly c o g e n s to r a g e B. Citric acid cycle □
d is e a s e : C. Hexose monophosphate shunt □
A. I (PGI 12, 07) □ D. Glycogenesis □
B. II □ 113.W hich is NOT a product of Pentose phos-phate
C. Ill □ pathway (AIIMS 13,10)
D. IV □ A. Glyceraldehyde 3 -PO 4 □
104.G 6 p h o s p h a t a s e d e f i c i e n c y s e e n in : (PGI 06, 09) B. NADPH □
A. Von Gierke's disease □ C. Sedoheptulose P O 4 □
B. Taysach's disease □ D. CO 2 □
Biochemistry: Carbohydrate Metabolism ■ 425

1 1 4 .N A D P H is generated by the action of (DNB 13,10) 124.R e g a r d i n g H M P s h u n t a ll o f t h e f o l l o w in g a re tru e


A. Glucose 6 Phosphate dehydrogenase □ e x c e p t: (AI 07)
B. Glucose 1 phosphate dehydrogenase □ A.Occurs in the cytosol □
C. Glucose 1.6 diphosphate dehydrogenase □ B.No ATP is produced in the cycle □
D. All of the above □ C.It is active in Adipose tissue, Liver and Gonads. □
115.HMP shunt is of great importance in cellular D.The oxidative phase generates NADPH and the Non
metabolism because it produces - oxidative phase generates pyruvate. □
A. ATP (NBE P 15; UP 04, WB 05) □ 125.1n G - 6 P D d e f i c i e n t p a t i e n t h a e m o ly s is i s d u e to
B. ADP □ d e c re a se in
C. Acetyl CoA □ A. H+ (AIIMS 05) □
D. NADPH □ B. TPP □
1 1 6 .D e h y d r o g e n a s e s o f HMP s h u n t a r e s p e c if i c f o r - C. NADH □
A. FAD (AIIMS 14; DNB 15) □ D. NADPH □
B. NAD □
Calvin Cycle, Cori & Cahill Cycle
C. NADPH □
D. FMN □ 1 2 6 .E n z y m e s o f C a l v i n c y c le a r e : (PGI 10)
117.Which one o f the follow ing enzymes use NADP as A. G-6-PD □
coenzyme- (DNB 14,12) B. Sedoheptulose-1-7- bisphosphatase □
A. Glyceraldehyde-3-phosphate dehydrogenase □ C. Glyceraldehyde 3 P O 4 dehydrogenase □
B. Lactate dehydrogenase □ D. Phosphoribulose kinase □
C. Glucose-6-phosphate dehydrogenase □ 1 2 7 .1 n c re a s e d l e v e l s o f a l a n i n e i n s e r u m a f t e r f a s t in g

D. Beta hydroxy acyl CoA dehydrogenase □ su g g e st: (AIIMS 11)


E. Mitochondrial isocitrate dehydrogenase □ A. Increased release of alanine from muscle □
11 8 .S t e p in HMP p a th w a y r e q u i r i n g TPP B. Reduced amino acid utilization for
A. G6 PD (AP 02, TN 01,KA 09) □ gluconeogenesis □
B. 6 Phoshogluconate dehydrogenase □ C. Break in continuity of plasma membrane resulting in
C. Transketolase □ leakage of amino acids □
D. Transaldolase □ D. Decreased uptake of alanine by liver □
1 2 8 .D u r in g p r o lo n g e d s t a r v a t io n , th e r a te o f
1 1 9 .T h e m a in p r o d u c t s o f HMP s h u n t a re a ll e x c e p t -
g lu c o n e o g e n e s is d e p e n d s o n (AIIMS 11)
A. 6-NADPH (Raj 02, Delhi 03, Punjab 03) □
A. Increased alanine levels in liver □
B. 2-Fructose-6-P04 □
B. Decreased cGMP in liver □
C. l-Glyceraldehyde-3-po4 □
C. ADP in liver □
D. 1-C 02 □
D. Decreased essential fatty acids in liver □
120.Which of the follow ing enzyme is a constituent of
129.T r a n s a m in a t i o n r e a c t i o n is (PGI 10)
HMP shunt- (UP 04, BHU 05)
A. Net deamination with splitting of N H 3 □
A. Glucose-6-phosphatase □
B. a-ketoglutarate is NH2 donor □
B. Hexokinase □
C. Transaminase enzyme & pyridoxial P O 4 binding is
C. G-6-P dehydrogenase □
covalent □
D. Phosphorylase □
D. Glutamate is formed □
121.Sites where HMP shunts can occur include - (PGI 05)
E. Non reversible pingpong reaction □
A. Liver □
1 3 0 .T ru e a b o u t a m in o g r o u p o f a m in o a c id is (PGI 09)
B. WBC □
A. Accepted by a-ketoglutarate □
C. Lactating mammary gland □
B. Funneled to L-Glutam ate □
D. Testes □
C. Prevents oxidative breakdown □
E. All □
D. Oxidative deamination is the l sl step to remove it □
122.W h i c h o f t h e f o l l o w in g m e t a b o l i c p a t h w a y s d o e s n o t
E. SGPT / SGOT transfer it □
g e n e r a te ATP: (AI 08)
131.T r u e a b o u t g lu t a m a t e d e h y d r o g e n a s e i s A/E
A. Glycolysis □
A. Liver mitochondrial enzyme (AMU 09, UP 10) □
B. TCA Cycle □
B. Use both NAD+ or NADP+ coenzyme □
C. Fatty Acid Oxidation □
C. Inhibited by ADP & activated by GTP □
D. HMP Pathway □
D. Reversible oxidative deamination □
1 2 3 .T h e m o s t c o m m o n e n z y m e d e f i c i e n c y i n m a n i s -
1 3 2 .G lu c o n e o g e n e s is f r o m l a c t a t e n e e d s a ll e x c e p t-
A. Glucose-6-phosphate dehydrogenase □ A. Transport oflactate from muscle to liver □
B. Glucose-6-phosphatase (AP 08, AI 06) □ B. Conversion of lactate to pyruvate (NBE P 15)0
C. Hexokinase □ C. Transamination of pyruvate to alanine □
D. Glucose-1.6-diphosphatse □ D. None of the above □
426 ■ A Complete Review of Short Subjects

1 3 3 .D u r in g s t a r v a t io n , m u s c le u s e s - (NBE P 15) C. Alanine & lactate both canserve as substrate □


A. Fatty acids □ D. Glycerol is not a substrate □
B. Ketone bodies □ 143.W h ic h o f t h e f o l l o w in g e n z y m e s is/are in v o lv e d i n
C. Glucose □ n e o g l u c o g e n e s i s e x c e p t: (PGI 15,13,08)
D. Proteins □ A. Phosphoglycerate kinase □
B. Fructose 1, 6-biphosphatase □
Gluconeogenesis C. Phosphoglucomutase □
D. Pyruvate carboxylase □
E. Isomerase
1 3 4 .G lu c o s e c a n b e s y n t h e s iz e d ( b y g l u c o n e o g e n e s i s ) fr o m
1 4 4 .E n z y m e to b o t h c o m m o n i n g l u c o n e g e n e s i s a n d
a ll except(AIIM S 15,14,12; AI 10; NBE P 15,14, PGI 14,13)
A. Aminoacids eg alanine □ g ly c o l y s i s p a t h w a y i s : (NBE P 13; AI 08)
A. Phosphofructokinase □
B. Glycerol from adipose tissue □
B. Fructose 2,6-biphosphatase □
C. Acetoacetate / Fatty acids/Palmitate/GH □
C. Hexokinase □
D. Lactate/Lactic acid/ Pyruvate □
D. Glucose 6 phosphatase □
135.W h ic h is n o t k e t o g e n ic - (NBE P 15)
145.W h ic h o f t h e e n z y m e o f g ly c o l y s i s is u s e d in
A. Leucine □
g lu c o n e o g e n e s i s - (AIIMS 13; NBE P 15,14)
B. Lysine □
A. Glucokinase □
C. Methionine □
B. PFK □
D. Tryptophan □
C. Pyruvate kinase □
136. Which is not Glucogenic- (NBE P 15)
D. Phosphotriose isomerase □
A. Arginine □
146.P r o d u c ts f o r m e d f r o m a lc o h o l a n d n o t i n t e r m e d ia t e s o f
B. Histidine □
TCA c y c le / g ly c o ly s is : (PGI 06)
C. Glycine □
A. Acetaldehyde □
D. Lysine □
B. Pyruvate □
137.A11 a r e u s e d i n g lu c o n e o g e n e s i s e x c e p t- (NBE P 15)
C. Lactate □
A. Oleate □
D. Oxalate □
B. Succinate □
E. Malate □
C. Glutamate □
147.1n w e ll f e d s t a t e g lu c o n e o g e n e s i s i n l iv e r i s i n h ib i t e d
D. Aspartate □
by (AIIMS 11, DNB 12)
1 3 8 .T ru e a b o u t Acetyl CoA: (PGI 15,11)
A. Protein breakdown in muscle □
A. Precursor for synthesis of cholesterol and other
B. Alanine content in liver □
steroids. □
C. ADP level □
B. Form ketone bodies □
D. cGMP □
C. Starting material for synthesis of fatty acid □ 148. Which of these is most effective for gluconeogenesis?
D. Arise from glycolysis □ A. Citrate stimulation of acetyl ca rb o x y la se ! AIIMS16) □
1 3 9 .A c e ty l C o -A a c ts a s a s u b s tr a te f o r a ll th e e n z y m e s
B. Acetyl-CoA stimulation of pyruvate carboxylase □
e x c e p t:
C. Fructose-2,6-biphosphate stimulation of
A. HMG-Co A synthetase (AIIMS03) □ phosphofructokinase-2 □
B. Malic enzyme □ D. Fructose-1,6-biphosphate stimulation of
C. Malonyl CoA synthetase □ phosphofructokinase-1 □
D. Fatty acid synthetase □ 1 4 9 .G lu c o n e o g e n e s is i n F a s t in g s t a t e i s in d ic a te d
140.A c e t y l C o A c a n b e c o n v e r te d in t o a l l o f th e f o l l o w in g . by (AIIMS 12)
E xcep t (AI 09, AIIMS 11,10) A. Citrate activation by acetyl Co-A carboxylase □
A. Glucose □ B. Pyruvate Carboxylase activation by Acetyl CoA □
B. Fatty Acids □ C. Fructose 1,6 bisphosphate activates Pyruvate
C. Cholesterol □ Kinase □
D. Ketone bodies □ D. Fructose 2 ,6 bisphosphate activates PFK-1 □
1 4 1 .T ru e a b o u t g l u c o n e o g e n e s is : (PGI 13) 150.A genetic disorder renders fructose 1,6 - biphosphatase
A. Prevent hypoglycemia during prolonged fasting □ in liver less sensitive to regulation by fructose 2,6 - bi­
B. Occur in both muscle and liver □ phosphate. All of the follow ing metabolic changes are
C. Fructose 2,6-biphosphate stimulate it □ observed in this disorder except: (AI 04)
D. Excess of acetyl co A cause stimulation □ A. Level of fructose 1,6-biphosphate is higher than
E. Carbon skeleton of amino acid is involved in normal □
gluconeogenesis □ B. Level of fructose 1,6 - biphosphate is lower than
1 4 2 .T ru e a b o u t g lu c o n e o g e n e s i s - (NBE P 15) normal □
A. Occurs mainly in muscle □ C. Less pyruvate is formed □
B. It is reverse of glycolysis □ D. Less ATP is generated □
Biochemistry: Carbohydrate Metabolism ■ 427

1 5 1 .P h o s p h o -d e p h o s p h o r y la t io n o f p h o s p h o f r u c t o k i n a s e 160.A f e w d a y s a f t e r b i r t h , a c h ild b e c o m e s l e t h a r g i c ,
an d fru c to se 1 , 6, b i p h o s p h a t a s e b y f r u c t o s e 2, 6, r e f u s e s f e e d s a n d v o m i t s d u r in g b r e a s t f e e d i n g . O n
B ip h o s p h a t e r e g u la t io n i s s e e n i n : (PGI 03) e x a m i n a t i o n h e i s p o t b e l l i e d . B e n e d i c t 's t e s t is
A. Brain □ p o s i t i v e i n u r in e . W h a t i s t h e l i k e l y r e d u c in g s y b s t a n c e
B. Liver □ p r e s e n t i n u r in e : (AIIMS 10)
C. Adrenal Cortex □ A. Glucose □
D. RBC □ B. Fructose □
E. Muscle □ C. Sucrose □
1 5 2 .G ly c o n e o g e n ic c a p a b il i t y o f c e l l i s d e te r m in e d b y th e D. Galactose □
p r e s e n c e o f: (PGI 05) 1 6 1 .G a la c to s a e m ia c o m m o n ly is d u e to d e f i c i e n c y o f -

A. Pyruvate dehydrogenase □ A. Galactose-1 phosphate uridyl transferase □


B. Glucose 6 phosphatase □ B. Galactose-1 phosphatase (PGI 05) □
C. Pyruvate carboxylase □ C. Glucose-1 phosphatase □
D. Glucose -6 phosphatase □
D. Fructose 1-6 biphosphatase □
162.A n e w b o r n i n f a n t r e f u s e s b r e a s t m i l k s i n c e t h e 2 n d
E. Pyruvate carboxykinase □
d a y o f b i r t h , v o m it s o n f o r c e - f e e d i n g b u t a c c e p t s
153.W h ic h o f th e f o l l o w in g e v e n t s d o e s n o t o c c u r w h e n
g lu c o s e - w a t e r , d e v e lo p s d ia r r h e a o n t h e th ir d d a y , b y
(AI 12)
c o n c e n t r a t i o n o f g lu c o s e i n t h e l i v e r d e c r e a s e s :
5 t h d a y s h e i s ja u n d i c e d w ith l iv e r e n la r g e m e n t a n d
A. Inactivation of phosphofructokinase-2 (PFK-2) □
e y e s s h o w s i g n s o f c a ta r a c t. U r in a r y r e d u c i n g s u g a r
B. Activation of Fructose Bisphosphatase-2 (Fructose 2,
w a s p o s i t i v e b u t b l o o d g lu c o s e e s t im a t e d b y g lu c o s e
6 Bisphosphatase) □ o x id a t io n m e th o d w a s f o u n d lo w . T h e m o s t l i k e l y
C. Increased levels of Fructose 2 ,6 - Bisphosphate □ ca u s e is d e fic ie n c y o f : (AIIMS 03)
D. Increased levels of Glucagon □ A. Galactose-l-phosphate uridyltransferase. □
1 5 4 .A fte r 50 g m o f g lu c o s e i s f e e d o r a ll y : (PGI 08) B. Beta galactosidase. □
A. Decrease ketone body production □ C. Glucose- 6 -phosphatase. □
B. Increased lactate production upon exercise □ D. Galactokinase □
C. Decreased gluconeogenesis □ 1 6 3 .T ru e r e g a r d in g g a la c to s e m ia : (PGI 01)
D. Increased gluconeogenesis □ A. Mental retardation occurs □
1 5 5 .P F K -1 i n h i b i t o r - (NBE P 15) B. Absent disaccharidase in intestine □
A. AMP □ C. Defect in epimerase □
B. Citrate □ D. Defect in galactose 1 PO 4 uridyl transferase □
C. Glucose 6 phosphate □ 1 6 4 .R e d u c in g s u g a r i n u r in e is s e e n in : (PGI 15,13)
D. Insulin □ A. Fanconi anemia □
Fructose Metabolism B. Lactose intolerance □
C. Galactosemia □
156.False about hereditary fructose intolerance is:
D. Phenylketonuria □
(AIIMS 08)
E. Salicylate poisoning □
A. Deficiency of fructose 1- phosphatealdolase. □ 165.A p a t i e n t h a s n o r m a l b l o o d g lu c o s e l e v e l a s e s t im a t e d
B. Accumulation of fructose 1 - phosphate intissues □ b y G l u c o s e - O x i d a s e P e r o x id a s e m e th o d , s h o w s
C. Hyperglycemia □ p o s i t i v e B e n e d ic t s t e s t i n u r in e . W h ic h o f th e
D. Liver & kidneys are involved. □ f o l l o w in g i s l i k e l y c a u s e ? " (AIIMS 16)
1 5 7 .F r u c to k in a s e is n e c e s s a r y f o r : (PGI 08) A. Fructosemia □
A. Fructose-l-PCh □ B. Galactosemia □
B. Ructose 1,6 diphosphate □ C. Denaturation of glucose □
C. Fructose 6 phosphate □ D. Faulty tests □
D. Glyceraldehyde □ 1 6 6 .T h e m o l e c u l e , w h i c h is t h e i n t i a t o r o f c a ta r a c t
15 8 .A n e n z y m e in v o lv e d i n t h e c a t a b o li s m o f f r u c t o s e to f o r m a t io n i n t h e e y e l e n s a n d w h o s e 1- p h o sp h a te
p y r u v a te i n th e l iv e r is : (AI 03) (AIIMS 05)
d e r iv a tiv e i s r e s p o n s i b l e f o r l i v e r f a il u r e , is :
A. Glyceraldehyde-3-phosphate Dehydrogenase □ A. Sorbitol □
B. Phosphoglucomutase □ B. Mannitol □
C. Lactate-dehydrogenase □ C. Inositol □
D. Glucokinase □ D. Galacticol □
167.A c h il d p r e s e n t s w i t h h e p a to m e g a ly a n d b i l a t e r a l
Galactose Metabolism & Intolerance l e n t i c u l a r o p a c it i e s . D e f i c i e n c y o f w h i c h o f th e
159.Enzyme defect in galactosemia- (NBE P 15) f o l l o w i n g e n z y m e s w i l l n o t c a u s e s u c h f e a tu r e s .

A. Uridyl transferase □ A. Galactose-l-phosphate uridyl transferase □


B. Galactokinase □ B. UDP-galactose-4-epimerase (AIIMS 11) □
C. Epimerase □ C. Galactosekinase □
D. All of the above □ D. Lactase □
428 ■ A Complete Review of Short Subjects

168.A y o u n g m a n f in d s th a t e v e r y t im e h e e a ts d a ir y Carbohydrate Classification


p r o d u c ts h e f e e l s v e r y u n c o m f o r t a b le . H is s t o m a c h
b e c o m e s d is te n d e d . H e d e v e lo p s g a s a n d d ia r r h e a
171. Which of the following are enantiomers:
fre q u e n tly . T h e s e sy m p to m s d o n o t a p p e a rs w h e n h e
A. D-Galactose & L-Glucose (PGI 11, DNB 12) □
e a ts f o o d s o t h e r th a n d a ir y p r o d u c ts . W h ic h o f th e
B. d-Galactose & 1-Glucose □
f o l l o w in g is t h e m o s t l i k e l y e n z y m e i n w h i c h th is
C. D-Mannose & L-Mannose □
y o u n g m a n is d e f i c i e n t ? (AI 04)
D. d-Mannose & 1-Mannose □
A . a - a m y la s e □
E. D-glucose and L glucose □
B. | 3-g alactosid ase □
172. Epimer combination (s) is /are
C . a - g lu c o s id a s e □
A. D-glucose & D-fructose (PGI 14,10,09) □
D . S u cra se □
B. D-mannose & D-talose □
C. D-glucose & D-mannose □
Alcohol Metabolism D. D-glucose & D-gulose □
E. D- galactose & D-glucose □
169.1n c h r o n ic a l c o h o l i s m t h e r a te l i m i t i n g c o m p o n e n t f o r 1 7 3 .E p im e rs o f g lu c o s e - (NBE P 15)
a l c o h o l m e t a b o l i s m e x c lu d in g e n z y m e s i s / a re : A. M a n n o se □
A. N A D P (PGI 08)□ B. G ly c e ra ld e h y d e s □
B. NAD □ C. F ru c to se □

C. N A D PH □ D. None □
D. FA D H □ 174. Inulin like fructans are used as prebiotics as they are
E. FA D □ non digestible. Resistance to digestion in the upper GI
170.A11 a re u s e d i n th e d ia g n o s is o f d ia b e t e s m e llit u s tract results from: (AI 10)
e x c e p t: (AI 11, AIIMS 10) A. Absence of digestive enzyme inthe upper GIT □
A. FBS □ B. Beta configuration of anomeric C2 □
B. H b A lc □ C. Low pH of the stomach □
C. D -x y lo s e a b s o r p tio n te s t □ D. Presence of alpha-osidic linkages □
D. O GTT □
Biochemistry: Carbohydrate Metabolism ■ 429

ANSWERS & EXPLANATIONS:

Glycolysis

1. B, D, E i.e. Enolase, Phospho-glyceromutase, Glyceraldehyde-3-phosphate-dehydrogenase


2. B i.e. Mitochondria [Ref: Harper's 28/e P-149-53; Lippincott 5/e p. 96-103; Vasudevan 6/e p. 93-97; Chatterjea 7/e P-306-10]
3. C i.e. Conversion of glucose to 3 C units

Enzymes of glycolysis - a process which converts 6 carbon glucose to 3 carbon unit pyruvate & lactateQ - are
present in cytoplasm. Whereas complete oxidation of glucose (to CO2 & H2O) requires mitochondrial enzymes (of
TCA cycle).
Out of total 9 enzymes used in glycolysis
3 enzymes - hexokinase, phosphofructokinase (PFK-1) and Pyruvate kinase are used in irreversible steps.
6 enzymes - phosphohexose isomerase, aldolase, glyceraldehydes 3 phosphate dehydrogenase, 1,3- biphospho
glycerate kinase, phosphoglyceromutase and enolase are used in reversible stepsQ.
2 enzymes used in energy utilizing steps are - hexokinase (using 1 ATP) and phosphofructokinase /PFK-1 (using 1
ATP). This energy consumption is for 1 molecule of glucose.
3 enzymes used in energy producing steps are - glyceraldehydes 3phosphate dehydrogenase (producing 2 NADH =
5 ATP), 1,3 - biphospho glycerate kinase (producing 2 ATP), and pyruvate kinase (producing 2 ATP). This energy
production is for 1 molecule of glucose or 2 molecules of glyceraldehyde 3-P.

4. A i.e. Enolase 5. D i.e. All of the above 6. C i.e. Glucose 6 PO4


[Ref: Harper's 28/e P-149-50,583; Lehninger 5/e p. 150-53,582-84; Vasudevan 6/e p. 93; Chatterjea 7/e p. 306-9]

Glucose 6 phosphate is an important compound that joins several metabolic pathways viz. glycolysis, glycogenolysis,
glycogenesis, gluconeogenesis and pentose phosphate pathwayQ.

7. D i.e. Glucose [Ref: Enzymes in industry & medicine Arnold: New studies in biology;
http://www.ncbe.reading.ac.uk/ncbe/protocols /pracbiotech/ pdf/glucdet.pdf]

Oxidase - peroxidase enzyme system is used to determine glucose in urine & bloodQ.

8. C i.e. Glucokinase, phosphofructokinase, pyruvate kinase


9. B i.e. Phosphofructokinase; C i.e. Pyruvate kinase; E i.e. Hexokinase
10 . D i.e. Phosphofructokinase [Ref: Harper's 29/e P-170-77; Lehinger 3/eP-541]
f■ —1 ■s^T”
The irreversible reaction catalysed by PFK -1
(Phosphofructokinase) is the step that commits a GlucokinaseQ
Glucose Glucose -6 - PO4

HexokinaseQ
- A C o m m itt e d s te p is an irreversible reaction which
commits the product for a particular metabolism.
There are three steps in glycolysis that are PhosphofructokinaseQ
physiologically irreversible. Fructuse -6 - Phosphate Fructose 1,6 - biphosphate
- Thus here the reaction Glucose —» Glucose 6 P O 4
appears to be the l sl committed step Phosphoenol pyruvate Pyruvate
But the product formed (i.e. Glucose - 6- P O 4) is not Pyruvate kinaseQ
particularly committed for glycolysis. It is only
committed for metabolism
inside the cell (can enter into many pathways, glycolysis being one of them), so its not the 1st committed step.
- The next irreversible reaction is fructose “ ►Fructose 1, 6- biphosphate T h i s r e a c t i o n c o m m it s th e c e l l to c h a n n e l
g lu c o s e i n t o g ly c o l y t i c p a th w a y Q . Thus this reaction is 1st committed step of glycolytic pathway.

- Glucose 6 P O 4can flow either into glycolysis or through alternative oxidative pathways. The irreversible reaction
catalysed by P F K - 1 is the step that commits a cell to channel glucose into glycolysis.
- 3 enzymes - h e x o k i n a s e ( g l u c o k i n a s e ) , p h o s p h o f r u c t o k i n a s e (P F K -1 ) a n d P y r u v a te k i n a s e are used in i r r e v e r s i b l e
ste p s.
430 ■ A Complete Review of Short Subjects

11. C i.e . G l u c o k i n a s e > A i.e . P h o s p h o f r u c t o k i n a s e [Ref: Harper's 28/e p. 150,152,166-65; Lehninger 5th/e p.578-81 ]
• Glycolysis is regulated at 3 steps involving irreversible exergonic reactions
i.e. the reaction catalyzed by: h e x o k i n a s e (o r g l u c o k i n a s e ) ,
p h o s p h o f r u c t o k i n a s e (P F K -1 ) and p y r u v a te k i n a s e . The phosphofructo
kinase reaction may be considered to be functionally irreversible under
physiological conditions; it is both inducible & subject to allosteric
regulation and has a major role in regulation of rate of glycolysis - Harper
• For glycolysis, PFK - 1 was considered the rate limiting enzyme,
because it was know to be closely regulated by fructose 2, 6
biphosphate & other allosteric effectors. But experiments suggest that
henokinase IV (glucokinase) & PFK-1 both contribute to setting the flux
(rate) through pathway (hexokinase more than PFK-1). Flux control
coefficient contradicted the conventional wisdom; it showed that hexokinase, not PFK-1 is m ost influential in setting
the flux (rate) through glycolysis Q. - Lehninger

12. C i.e . E n o la s e 13. C i .e . E n o la s e 14. C i .e . E n o la s e


[Ref: Harper’s 27/e P-152-53; Chatterjea 7/e P-308-10] ^_A_,

F o r b lo o d g lu c o s e e s tim a tio n in th e la b o ra to ry , Mg++ / Mn+


f lu o r id e i s a d d e d t o t h e b l o o d t o p r e v e n t g ly c o l y s i s 2 Phosphoglycerate Phosphoenolpyruvate
b y t h e c e l l s b y i n h i b i t i n g 'e n o l a s e ' s o t h a t b l o o d Enolase Q
g l u c o s e i s c o r r e c t ly e s t im a t e d .^

Fluoride0

15. C i.e . P o t a s s iu m o x a la t e + N a F 16. C i.e . N a F [Ref: Chattergea & Shinde 7/e p. 308-10; Harper's 27/e p. 152-53]

The anticoagulant mixture used to estimate blood glucose from a sample contains anticoagulant & glycolytic inhibitor as
there is loss of 1-2% of glucose /hour due to glycolysis of RBCs.
I I
A n t ic o a g u la n t G ly c o ly tic in h ib ito r
I I
Potassium oxalate0 Sodium fluoride (NaF)Q

17. A i.e . G l y c o l y s i s [Ref: Harper's 27/e P-151; 26/e P-136, Lehinger 3/e P541 ] .

( -----------------------------------------------------------------------------------------------------------------------------------------------------------------------------k*
C a n c e r c e lls derive nutrition from glycolysis 0 . C a n c e r c e lls a r e m u ltip ly in g a t a r a p id r a te f o r w h ic h t h e y n e e d e n e rg y .
B u t th e y la c k a n e x te n s iv e c a p illa r y n e tw o r k to s u p p ly th e c e lls w ith O 2. S o th e tu m o r c e lls h a v e t o d e p e n d o n a n a e r o b ic
p ro cess - Glycolysis f o r A T P p r o d u c tio n

R o l e o f g ly c o l y s i s i n c a n c e r th e r a p y

In fast growing cancer cells, rate of glycolysis is very high, produces more pyruvic acid (PA) than TCA cycle can handle.
Accumulation o f pyruvic acid leads to excessive formation of lactic acid producing Lactic acidosis.

18. A i.e . G l u c o k i n a s e Ref: Lehninger 5/e 591, 583

I n g l y c o l y s i s in s u lin acts on hexokinase IV (i.e. glucokinase) and hexokinase 7/2 b u t n o t o n o t h e r (I & III) h e x o k in a s e

19. A i .e . K m v a lu e is h i g h e r t h a n n o r m a l b l o o d s u g a r ; B i.e . F o u n d i n l iv e r ; E i.e . G lu c o s e e n te r in t o c e l l s th r o u g h G L U T - 2


20. C i .e . A d ip o c y te s 21. C i .e . H e a r t 22. B i.e . G L U T 2
[Ref: Lippincott 4,h/ed pg-312; Lehninger 5th/ed pg-391-93, 608, 441-42, 583-85; Chatterjea 6/e P-271; Vasudevan 3/e 73, 74]

I n s u l i n s e n s i t iv e ( d e p e n d e n t ) g l u c o s e t r a n s p o r te r , G L U T - 4 th a t a r e fo u n d in adipose tissueQ a n d s k e le ta l m u s c le s a r e '


reduced in fasting state, when the insulin levels are low.
Biochemistry: Carbohydrate Metabolism ■ 431

G l u c o s e T r a n s p o r te r s

Insulin dependent (sensitive) Insulin insensitive (not dependent)


Facilitated transport 1 1
I A c t iv e tr a n s p o r t F a c ilit a t e d t r a n s p o r t
- Adipose tissue® GLUT-4 1 1
- Skeletal Muscle - Intestine RBC, Placenta, BBBQ, Colon, GLUT-1
- Heart epithelium Kidney
- Renal tubules Pancreas ((J-cells)Q, Liver, GLUT 2
1 - Choroid plexus Small intestine, Kidney
Decrease in fasting Brain, Placenta, Kidney GLUT 3
Intestine, Testis, Kidney, GLUT 5
Sperm (fructose transport)
Leukocyte, Spleen GLUT6
Lens
Cornea
Liver GLUT7

23. C i.e. Hypoxia stimulates release of all glycolytic enzymes from Band 3 on RBC membrane.
[Ref: Ganong 533, 670, 720, 7 3 3 ,14f; http:Iliuww.jbc.org/cgilcontent[abstract126617/4106;
http://urwio.pnas.org/cgi/content/full/102l7/2402]

• RBC membrane cytoskeletal protein spectrin is anchored to transmembrane anion exchanger protein Band 3 by
protein ankyrin & protein 4.2. Another cytoskeletal protein actin is attached to transmembrane glycophorin C by
protein 4.1. Tropomyosin, tropomodulin, adducin and 4.9 are other proteins.
• Band 3, is a multifunction RBC transmembrane protein, which is important for its cytoskeletal structure, cell shape,
anion exchange activity and glycolysis®. Band 3 is responsible for chloride shift in RBC®. Hypoxic deoxygenation of
hemoglobin causes Band 3 tyrosine phosphorylation and thereby stimulates glycolysis by releasing glycolytic
enzymes from band 3 on RBC membrane®.
• In kidney, I cells contain Band 3, an anion exchange protein in their basolateral cell membrane, which may function as a
Ch - H C O j exchanger for the transport of HCOf to interstitial fluid.
• In animal cells, principal regulators of intracellular pH are HCO/ transporters such as Band 3 ci~ - HCOj exchanger,
3Afa+ - HCO/ cotransporters and a K + - HCO/ cotransporter.

24. D i.e. Erythrocytes 25. B i.e. Enzymes of TCA cycle

Erythrocytes (RBC) lack mitochondria (so TCA cycle enzymes) and completely rely on glucose as their metabolic fuel.
They metabolize it by anaerobic glycolysis^.

26. D i .e . E th a n o l [Ref: Harper's 27/e p. 151, Lippincott's 4/e p. 96]

1------------------------------------------------------- r -
A e r o b ic g l y c o l y s i s i n c e l l s w i t h A n a e r o b ic g l y c o l y s i s in cells that in yeast & some other
F e r m e n t a t io n
m it o c h o n d r ia & a d e q u a te l a c k m it o c h o n d r i a (e g R B C s ) o r microorganisms
o x y g e n s u p p ly d e p r iv e d o f s u f f i c i e n t O 2 I
I I Ethanol® is the end product as pyruvate is
Pyruvate® is the end product and Lactate® is the end product as decarboxylated & reduced into it
NADH enters oxidative NADH formed in glycolysis is
phosphorylation for ATP reoxidized by reducing pyruvate to
production lactate.

Shuttle Systems

27. A i .e . M a la t e 28. B i.e . G l y c o l y s i s


[Ref: Harper's 28/e P-155; Vasudevan 6/e p. 228; Chatterjea 7/e P-321-22]
432 ■ A Complete Review of Short Subjects

Mallate shuttle is a system by which reducing equivalents (NADH) produced in glycolysisQ are transferred from cytosol
to mitochondria to yield energy.____________________________________________________________________

NADH produced in the glycolysisQ , is extramitochondrial where as the electron transport chain, where NADH has to be oxidized
to NAD+ is in the mitochondria (Oxidation of 1 NADH yields 3 ATP/2.5 ATP). NADH is impermeable to mitochondrial
membrane, so to enable transfer of NADH inside mitochondria Shuttle system is utilized.

PDH- Complex

29. B i.e . P D H c o m p le x 30. A i.e . P y r u v a te 31. E i.e . N A D P H


32. D i.e . P h o s p h a t a s e 33. A i.e . B io t i n 34. D i.e . B io t i n [Ref: Lippincott's 5/e p. 109-12]

Pyruvate, the end product of aerobic glycolysis is transported into mitochondria by specific pyruvate transporter, where it is
converted to acetyl CoA by pyruvate dehydrogenase (PDH) complexQ. It uses niacin (as NAD), riboflavin (as FAD) and
thiamin (as TPP)Q.

Pyruvate Dehydrogenase (PDH) Complex

It is not the part of TCA cycle, but is the major source of acetyl CoA - a 2 carbon substrate for cycle. The i r r e v e r s i b i l i t y of PDH
reaction precludes the formation of pyruvate from acetyl CoA, and explains why glucose cannot be formed from acetyl
CoA by gluconeogenesis. PDH complex is a multimolecular aggregate of 3 enzymes components and 2 regulatory
enzymes.
I---------------------------------------------------------------------------------------1
Component enzyme Coenzyme Regulatory enzyme Activator of Action
enzyme
Ei = Pyruvate Thiamine
dehydrogenase pyrophosphate PDH kinase ATP, Acetyl Phosphorylates & inhibits
(decarboxylase) (TPP)Q CoA, NADH (tum.off) E l
(c-AMP-
E2 = Dihydrolipoyl Lipoic acid & CoAQ independent)
trans acetylase
PDH phosphatase Ca++ Dephosphorylates or
E3= Dihydrolipoyl FAD, & NAD+Q
activates (turn on) Ei
dehydrogenase
★ Pyruvate is a potent inhibitor of PDH kinase therefore it activates El.
★ Acety CoA & NADH formed in this reaction allosterically inhibit
active form of Ei
PDH complex ★ S o e n e r g y p r o d u c e d i n c o m p le t e a e r o b ic o x id a t io n o f 1 g lu c o s e
(inactive) - H ,0
m o l e c u l e i s 3 0 -3 2 A T P (new concept)
Pyruvate ©
T
Phosphatase © c a A e r o b ic g l y c o l y s i s P D H c o m p le x r e a c tio n C ir t ic A c id C y c le
ATP "I
A c e ty l C o a H © I I
NADH J / 5-7 ATPQ 2NADPH or 5 ATP X 2 10 A T P
PDH complex
(active) i.e. 2 0 A T P
© ★ Each glucose molecule gives rise to 2 pyruvate molecules & so 2 TCA
cycles occur.
NAD+ NADH + H+
- X-linked dominant deficiency in E l component of PDH complex, although
|PDH complex| rare, is the most common cause of congen ital lacticacidosisQ . This
Pyruvate -— * Acetyl CoA
enzyme deficiency results in inability to convert pyruvate to acetal
CoA *02
CC CoA, causing pyruvate to be converted to lactic acid via lactate
dehydrogenase. This mainly affects brain which relies on TCA cycle for
most its energy & is particularly sensitive to acidosis.
- L e ig h s y n d r o m e (subacute necrotizing encephalomyelopathy) occurs d/t
defects in mitochondrial ATP production as a result of mutation of
PHD complex, electron transport chain or ATP synthase. It can affect both
nuclear & mitochondrial DNA.
Biochemistry: Carbohydrate Metabolism ■ 433

35. A i.e . P h o s p h o e n o l P y r u v a te [Ref: Harper's 28/e P -155,165-67; Lippincott 5/e p. 105-103,118; Shinde & Chatterjea 6/e P-305]

P y r u v a t e c a n b e d ir e c tly c o n v e r te d to a c e t y l C o A (b y o x id a tiv e d e c a r b o x y la tio n ) , o x a lo a c e t a te (b y c a r b o x y la t io n ) , f t


But pyruvate can not be directly converted to
a l a n i n e ( tr a n s a m in a tio n ); a n d l a c t a t e (in a n a e r o b ic c o n d itio n s ).
phosphoenol pyruvate (a reaction o f gluconeogenesis)®. However, phosphoenol pyruvate can be directly converted to
pyruvate (in glycolysis)®.

36. D i.e . E n o la s e

A r s e n it e ( t r iv a le n t f o r m o f a r s e n ic ) f o r m s a s ta b le c o m p le x w ith th io l (-S H ) g r o u p o f l ip o i c a c id , m a k in g i t u n a v a ila b le f t


to s e r v e a s a c o e n z y m e . S o a r s e n ic p o is o n in g is p r im a r ily d / t inhibition o f enzymes that require lipoic acid (lipoate) as a
coenzyme including E2 o f PDH complex, a-K etoglutarate dehydrogenase and branched chain a-ketoacid dehydrogenase®

37. D i.e . It i s c o e n z y m e f o r p y r u v a te d e h y d r o g e n a s e

f t
thiamin requirement
T h i a m in ( v it B i) h a s c e n tr a l r o le in e n e r g y y ie ld in g m e ta b o lis m e s p e c ia lly o f c a r b o h y d r a te s . S o
increases in excess in take o f carbohydrates and its deficiency leads to decreased energy production®. H a m p e r e d o r
d e c r e a s e d f u n c t i o n o f p y r u v a te d e h y d r o g e n a s e e n z y m e ( w h ic h f o r m s a c e ty l C o A fr o m p y r u v a te ) a n d a - k e t o g l u t a r a t e
d e h y d r o g e n a s e e n z y m e (o f T C A c y c le ) in w h ic h th ia m in a c ts a s c o e n z y m e is t h e m a in c a u s e o f t h is d e c r e a s e d e n e r g y
p r o d u c tio n . D e c r e a s e d P D H f u n c tio n a ls o l/t pyruvic and lactic acidiosis®.

Citric Acid Cycle

38. B i .e . I r r e v e r s ib l e

F o r m a t io n o f C it r ic A c id f r o m A c e t y l - C O A a n d O x a l o a c e t a t e (O A A ) i s 1 st a n d i r r e v e r s i b l e s t e p o f T C A , g i v in g 7 .8 K c a l |
( e x e r g o n ic r e a c tio n ^ )

Citrate synthase
A c e t y l C O A + O A A + H 20 C it r ic acid Q + C O A - S H
1st s te p o f T C A c y c le .
An irreversible r e a c tio n , g iv e s 7 .8 K c a l ( e x e r g o n ic re a c tio n )

39. B i.e . C it r a t e 40. B i .e . C is a c o n ita t e 41. A i.e . F lu r o a c e t a te


42. B i .e . A c o n ita s e 43. C i .e . C it r a t e , k e t o g l u t a r a t e , o x a lo a c e t a te 44. B i.e . M a l o n a t e
45. B i .e . A l p h a k e t o g l u t a r a t e 46. A i .e . P y r u v a te , D i.e . M a l o n a t e , E i .e . N i t r i c o x id e 47. B i.e . F lu o r o a c e t a t e
48. D i.e . S u c c i n a t e d e h y d r o g e n a s e 49. D i .e . 2 .6 A T P
50. A i.e . Is o c itr a te d e h y d r o g e n a s e ; B i.e . S u c c in a t e d e h y d r o g e n a s e ; C i.e . S u c c in a t e t h io k in a s e ; D i.e . M a la t e d e h y d r o g e n a s e
[Ref: Lehninger 5lhle p. 630; Harper's 28je P-143-46; Chatterjea 7/ep. 316-19; Lippincott's 5/e p. 109-13; Vasudevan 6/ep.219J

E n e r g y p r o d u c in g e n z y m e s in K r e b 's (T C A ) c y c le a r e - a - k e to g lu ta r a te d e h y d r o g e n a s e , M a la t e d e h y d r o g e n a s e , f t
Is o c itr a te d e h y d r o g e n a s e , S u c c in a t e d e h y d r o g e n a s e a n d S u c c in a t e th io k in a s e .
M n - A K G M I S (- d e h y d r o g e n a s e ) is S T (s u c c e s s fu l te a c h e r )
O n e N A D H m a y p r o d u c e 2 .5 A T P a n d 1 F A D H m a y p r o d u c e 1 .5 ATP through respiratory chain®.

51. B i .e . Is o c it r a t e d e h y d r o g e n a s e

2 m o le c u le s o f c a r b o n d io x id e a r e r e le a s e d in T C A c y c le . O n e b e f o r e a n d o n e a f te r a - k e to g la ta r a te . E n z y m e s u s e d a re
- isocitrate dehydrogenase and a ketoglutarate dehydrogenase® r e s p e c tiv e ly .
434 ■ A Complete Review of Short Subjects

52. A i.e. Thiamine; B i.e. Riboflavin [Ref: Harper's 28/e P-146] Acetyl CoA (C2)
53. C i.e. Isocitrate dehydrogenase [Ref: Debajyoti Das 101]

The enzyme of TCA cycle having allosteric inhibitor


is isocitrate dehydrogenaset®.
The allosteric activator is ADP and allosteric inhibitor
is ATP and long chain aceyl COA

54. C i.e. ATP, E i.e. NADH


55. C i.e. Isoleucine, Valine, methionine
[Ref: Harper’s 28je P-147, 248; Lehninger 5/e p. 861]

Valine, Isoleucine and Methionine® produce


succinyl COA which is an important component of
TCA cycle. [Mn: Successful VIM ]

56. B i.e. Succinate Thiokinase 57. B i.e. Succinyl CoA to succinate 58. C i.e. Succinyl CoA thiokinase

Substrate & Oxidative Phosphorylation

There are two types of phosphorylation of ADP to ATP


_________________________ I_______________________
X
Substrate level phosphorylation Oxidative Phosphorylation at Respiratory Chain Level
Is production of ATP 'at the
Generation of High Energy Phosphate In Catabolism of Number of (P)
substrate' without the
Glucose Formed per Mole of
participation of electron transport
Pathway Reaction Catalyzed by Method of ~ Glucose
chain
Formation of ATP does not Production
require ATP Glycolysis Glyceraldehyde-3- Respiratory chain 6*
Respiratory chain does not phosphate dehydrogenase oxidation of 2 NADH
participate Phosphoglycerate kinase Phosphorylation at 2
It is not affected by dinitrophenole substrate level
Pyruvate kinase Phosphorylation at 2
In TCA cycle substrate level
10
Succinate Allow for consumption of ATP by reactions catalyzed by -2
thiokinase0
hexokinase and phosphofructokinase Net 8
Succinyl ►Succinate
CoA Pyruvate dehydrogease Respiratory chain 6
oxidation of 2 NADH
In Glycolysis Isocitrate dehydrogenase Respiratory chain 6
oxidation of 2 NADH
Phosphoglycerate
Kinase®
a-ketoglutarate Respiratory chain 6
1,3- biphos- * 3 phospho dehydrogenase oxidation of 2 NADH
Phoglycerate glycerate Citric acid Succinate thiokinase Phosphorylation at 2
cycle substrate level
Pyruvate
Succinate dehydrogenase Respiratory chain 4
Kinase® oxidation of 2 FADH2
Phosphoenol-----------►(Enol) Malate dehydrogenase Respiratory chain 6
Pyruvate pyruvate oxidation of 2 NADH
Net 30
Total per mole of glucose under aerobic conditions 38
Total per mole of glucose under anaerobic conditions 2
Biochemistry: Carbohydrate Metabolism ■ 435

★ It is assumed that NADH formed in glycolysis is transported into mitochondria via the malate shuttle. If the
glycerophosphate shuttle is used, only 2 ~ (P) would be formed per mole of NADH, the total net production being 26
instead of 38. The calculation ignores the small loss of ATP due to a transported of H+ into the mitochondrion with
pyruvate and a similar transport of H+ in the operation of the malate shuttle, totaling about 1 mol of ATP. Note that
there is a substantial benefit under anaerobic conditions if glycogen is the starting point, since the net production of
high-energy phosphate in glycosis is increased from 2 to 3, as ATP is no longer required by the hexokinase reaction.

High Energy Compounds

59. C i.e. Creatine phosphate 6 0 . C i.e. ATP 61. C i.e. Creatine phosphate 62 . B i.e. Creatine phosphate & Carbamoyl..
63 . A, C, D i.e. ATP, Creatinine phosphate, Acetyl CoA [R ef: V asu d ev an 6 /e p. 2 2 5 -2 6 ; H a rp er 2 9 /e p. 5 3 0 -4 0 ; S a ty a n a ra n 3 /e 2 2 2 -2 3 ]
/ 1
- ATP, GTP, UTP, creatine phosphate and CoA derivatives (like acetyl/succinyl /fatty acyl /HMG-CoA)Q and
carbamoyl phosphate are high energy compounds. Wherease, ADP, glucose 1 or 6 phosphate, fructose 6 P 0 4 and
glycerol 3 P 0 4 are low energy compounds.
L- Creatinine phosphate gives 10.5 KCal/moleculeQ._________________________________________________________________

Respiratory Chain & Oxidative Phosphorylation

64 . D i.e. Fe2+ 65. D i.e. R iboflavin 66 . A i.e. FMN


[Ref: Lippincott 5/e p. 76-77; Harper 28/e p. 99,103-5; Chatterjea 7/e p. 133-129; Lehninger 5/e p. 708-18-515]

- Because electrons tend to flow spontaneously from carries of lower redox potential (E°) to carries of higher redox
potential. The order (sequence) of electron carriers in ETC of mitochondria (and so the increasing order o f redox
poten tial) is______________________________________________________________________________________________________
Substrate -> NADP+ /NADPH -> NAD+/NADH -> NADH dehydrogenase (FMN) /NADH dehydrogenase (FMNH2)
-> FAD/FADH 2 -» Ubiquinone or Coenzyme Q —>Fe3+ / Fe2+ in cytochromes b - » Ci -» C - » a -» a3 -» O 2____________
(Mn: " N F Q b C i c a a 3 0 2")
Riboflavin (vitamin B2) in form o f FAD /FMNQ and niacin in form of NAD+/NADP+ function as coenzymes in
Redox (oxidation & reduction) reactions^.

67. C i.e. Chemiosmotic methods 6 8 . A i.e. Generation of ATPs 69. C i.e. Fo Fi ATPase
70 . A i.e. NADH-Q Oxidoreductase; C i.e. Cytochrome C-Q oxidoreductase
71. A, B, E i.e. (NADH Co-Q reductase, Cytochrome C oxidase, Co-Q-Cytochrome C reductase)
/ \

M itchell's chemiosmotic hypothesis links the respiratory (i.e. electron transport) chain to ATP production (i.e.
oxidative phosphorylation)^. It explains how the free energy generated by the transport of electrons from NADH to
molecular oxygen (in respiratory chain) is used to produce ATP from ADP + Fi (from proton pumps).
• For each NADH oxidized (pair of electron), complex I (NADH -Q - oxido reductase) and complex III (ubiquinone /Q-
cytochrome C- oxidoreductase) pump (translocate) 4 protons (H+) each, whereas complex IV (Cytochrome C-
oxidase) translocates only 2 protons (H+). In ETC. oxidative phosphorylation (ATP formation) occurs at these sites
(ie complex I, III & IV)
• In ETC, ATP is generated by F0-F1 ATPase (=ATP synthase)Q. Fo subunit forms proton channel and (FI sububit
L generate ATP (3ATPs per rotation)Q.
436 ■ A Complete Review of Short Subjects

A D P + PI ATP

V Site 1 y
FM N -Fe S „ Fe S 2,
NADH-
FeS , Fe Scso
C om plex I A DP + PI ATP ADP + PI ATP

V^site2^^ V^SUe3^^
c y t b s * - cyt baa-
CoQ F e S - cyt C,
►Cyt C C^ U -►Mol.O,
C om plex III Com plex IV

No ATP formed * Mobile


FAD-Fe SA, Fe S B, • Acts as a shuttle between
FeSc*cyt complex III and IV

Com plex II

F o u r c o m p le x e s o f E le c t r o n T r a n s p o r t C h a in

P a u l B o y e r 's B in d i n g C h a n g e M o d e l o f R o t a t io n a l
C a t a ly s is

72. C i.e . a a n d P s u b u n i t s r o ta te 73. B i.e . E x e r g o n ic & C a t a b o l ic ; E i.e . C y t o c h r o m e - C r e d u c t a s e in v o lv e d


[Ref: Chatterjea 6je P-120-130; Lehninger 3/e P- 661- 72]


Internal respiration is exergonic and catabolicQ and it uses cytochromes b, cj, c, a, a ft.
a
In b io lo g ic a l o x id a tio n s th e te rm e x o th e r m ic & e n d o th e r m ic a r e r e p la c e d b y e x e r g o n ic a n d e n d e r g o n ic . T h u s in
e x e r g o n ic a n d e n d e r g o n ic r e a c tio n s , free energy is released or absorbed respectively.
• I n t e r n a l r e s p i r a t io n is u t iliz a tio n o f O 2 a n d p r o d u c tio n o f C O 2 b y c e lls . It in v o lv e s o x id a tiv e p h o s p h o r y la tio n &
g e n e r a tio n o f A T P th r o u g h r e s p ir a to r y c h a in . T h e r e s p ir a to r y c h a in c o lle c ts , tr a n s p o r ts a n d o x id iz e s r e d u c in g
e q u iv a le n ts (-H o r e le c tr o n s ). T h e e n e r g y o f e le c tr o n tr a n s fe r is s u ffic ie n tly c o n s e r v e d in p r o to n g r a d ie n t a n d th e s e
r e a c tio n s a re h ig h ly exergonicQ. M o s t o f th e e - a r is e fr o m a c tio n o f d e h y d r o g e n a s e s th a t c o lle c t e le c tr o n fr o m catabolic
pathway 0 a n d fu n n e l th e m in to u n iv e r s a l e le c to n a c c e p to r - N A D , o r N A D P + o r F A D o r F M N .

74. A i.e. NADH, C i.e. NADPH; D i.e. ATP/Ref: H a rp e r ’s 2 8 /e P -1 0 9 -1 1 0 ; L ip p in cott 5 /e p. 7 9 -8 0 ; L eh n in g er 3 /e P -6 6 0 ; V asu devan 6 /e p. 23 4 ]
_A_
Inner mitochondrial transporter contains specific transporter that can y cytosolic NADH /NADPHQ, ADP & ATPQ.

75. B i.e. Exergonic & Catabolic; E i.e. Cytochrome C reductase involved 76. A i.e. Inhibits Cytochrome oxidase
77. A i.e. CO; B i.e. H 2S; C i.e. Cyanide 78. D i.e. Non-competitive irreversible inhibition
79. C i.e . C y a n id e 80. A i .e . C y a n id e 81. C i .e . B r o w n a d ip o s e t is s u e
82. C i.e. Uncoupling of oxidation & phosphorylation 83. A i.e. Atractyloside
84. B i.e. Inhibitor of oxidative phosphorylation 85. A i.e. Thermogonin
[R ef: H a rp e r 2 8 /e p. 1 0 8 -1 0 ; C h atterjea 7 /1 3 6 -3 7 ; V asu devan 6 /e p. 2 3 4 ; L eh n in g er 5 /e p 714; L ip p in cott 5 /e p. 77-7 9 ]

• Electron transport in complex IV - Cytochrome c oxidase (Cytochrome a, as) is inhibited by cyanide (CN-), carbon
m onoxide (CO), hydrogen sulfide (H2 S) and azideQ.
Biochemistry: Carbohydrate Metabolism ■ 437

|i s u b u n its ta k e s A D P a n d Pi

Brown adipose tissue is the specialized mammalian tissue in which fuel lo fo rm A T P . w h ic h is e x p e lle d
as r o ta tin g • s u b u n it s q u e e z e s
oxidation serves not to produce ATP but to generate heat Q. (i.e. oxidation e a c h | i s u b u m l. S o 3 A IH a re
g e n e r a te d p e i r e v o lu tio n
is not coupled with phosphorylation). This uncoupling of oxidative
. F 1 s u o c o m p le x o f
phosphorylation is due to presence of thermogenin (or I' 3 n a n d 3 |i s u b u n its
ATP
(fix e d lo m e m b r a n e
UCPl/uncoupling protein 1) a natural physiological uncoupler. So a n d d o n o t ro ta te )

brown fat is used for heat generation (not ATP production) in new born and
during hibernation in animals.
2-4-Dinitophenol, dicoumarol (vitamin K analogue), calcium, CCCP
, S u b u n it fo rm s
(Chloro carbonyl cyanide phenyl hydrazone), FCCP (trifluoro carbonyl
cyanide phenyl hydrazone),- valinomycin, aspirin are synthetic
uncouplers.
Atractyloside (glycoside) inhibits oxidative phosphorylation (i.e. ATP
synthesis)Q by blocking transporters (translocases) responsible fo r
movement o f ADP (into) & A TP (out) across the inner m itochondrial
membrane Q. Whereas oligomycin binds to ATP synthase (F0 domain so
name starts with O) closing the proton channel and blocking the flo w o f P ro to n g a s
- F o s u b c o m p le x
th ro u g h a n d ro ta te
protons 0. Fo and s u b u n its
(d is k o f C p ro te m s u b m it)

Uncouplers

86. A i.e. Oxidase [Ref: Robbins 7,h/60; Chatterjea 6/e P-18] 87. A i.e. Apoptosis [Ref: Robbins 7/e p-29-301

The electron transport chain system responsible fo r the


Cytochrome C has two very opposite roles. W ithin
respiratory burst is ‘NADPH oxidase' (on oxidase enzyme)
the mitochondrial membrane it serves as electron
The biochemical mechanism of microbial killing and degradation carrier in oxidative phosphorylation. But when it leaks
following phagocytosis is called ‘respiratory burst'. out into cytosol it combines w ith A paf I and
This is accomplished largely by oxygen dependent mechanisms. activates caspase-9 to m ediate apoptosis Q. This
'The generation o f reactive oxygen intermediates is due to rapid dual functioning is known as moonlighting.
activation o f an oxidase (NADPH oxidase)'.

88. A i.e. 1 [Ref: Harper 30/e p. 148[

Fuel 02 CO 2 RQ (=B/A)
Respiratory Quotient (R/Q) is the ratio of volume of
Consumed produced
CO 2 produced and O 2 consumed. It increases from
(L/g)=A (L/g)=B
pure fat (0.7) to pure protein (0.8) to pure
Carbohydrate 0.83 0.83 IQ
v carbohydrate (1= in fasting state)___________________
Protein 0.97 0.78 0.8
Fat 2.02 1.4 0.7
Alcohol 1.43 0.97 0.66
438 ■ A Complete Review of Short Subjects

Glycogen Metabolism & Storage Diseases

89. A i.e. Glycogenesis

Glycogenesis and Glycogenolysis

G ly c o g e n L <0 0
CGlucose
added New i -f)-Don3

film i o lio sp ln - R u le liinitiitt' e n ^ v n te


- Il I r a n * f o r a p a rt o f - P h o .sp h o r y lilie a c t iv ity
c h a in (a t le a s t o d u - C le a v e s I > A b o n d u n it a p p r o v
w h e n c h a in lu is f !
4 g l u e o s c r e s i d u e r e m a in on
< :lu e o sc r e s u l u c Glycogen Branching
- fl e s ta b lish e s b r a n d ilhc'i s i d e o f I —> 6 b o n d . synthase enzyme

( I - + 6 lin k a g e )

1. >4 G Lucosvl u n its


0-0 1 >4- Glucosidic bond

f ' o Unlabeled glucose residue

T r a n s f e r a D is a c e h a r h ie 0+0 1-«6- Glucosidic bond


u n it f r o m o n e b r a n c h to ,4C-labeled glucose residue
a n o th er branch
T he biosynthesis of glycogen. The m echanism of branching
C a u s e s fo rm atio n o f
C ih e o g c n h o n e b w C I o f g lu c o s e as revealed by adding 14C-labeled glucose to the diet in the
S v n lh a se o f L D l ' G l c i : C A o f le m m u living anim al and exam ining the liver glycogen at further
elu co .se re sid u e o f u K c o tic n
intervals

U D P glucose

U D P ( ili
P y ro p h o sp h o r y la se

- G ly c o g e n e s is

| G lu c o s e ~ |

• 0 —0 1 G lu c o s e r e s id u e s jo in e d
I0 -0 j by 1 4 - g lu c o s id ic b o n d s
I G lu c o s e re s id u e s jo in e d
' b y 1 - > B - g lu c o s id ic b o n d s

S te p s in g ly c o g e n o ly s is

90. B i.e . G ly c o g e n s y n th a s e 91. B i.e . C o m m o n ly s e e n in f a s t in g s t a t e t h a n i n fe d s ta te


92. C i.e . P h o s p h o r y la s e 93. B i.e . C o n v e r s io n to G l y c o g e n 94. D i.e . S t o r a g e a s g ly c o g e n
95. D i .e . A T P 96. C a n d E i.e . O x y t o c in d e c r e a s e s it a n d 5 s u b u n i t is c a t a ly t ic 97. D i.e . I n s u l i n
98. 6 g lu c o s id a s e
C i.e . A m y lo 1 , 99. A i.e . P h o s p h o r y la s e , G l u c a n t r a n s f e r a s e , D e b r a n c h i n g , P h o s p h o r y la s e
[Ref: Harper's 28/e p.158-62; Lippincott's 4/e p. 132-33; Chattergea & Shinde 7/e p. 327-30; Lehninger 5,h/ed pg 603-04]

- M ain enzyme for glycogen synthesis is glycogen synthaseQ.



- Gylcogenolysis ( g ly c o g e n m e ta b o lis m ) occurs in fasting & starvation®. T h e m a in e n z y m e o f g ly c o g e n o ly s is is
glycogen phosphorylase (or phosphorylase), w h ic h is allosterically inhibited (inactivated) by ATP, glucose 6PO4 and
free glucose (1st tw o in m u s c le & liv e r , 3 r d o n ly in liv e r). Factors activating or maintaining active phosphorylase
in c lu d e C-AMP dependent protein kinase ( b o th in liv e r & m u s c le ), 5 ' AMP & Ca++ (in m u s c le o n ly ). Hormones like
epinephrine, norepinephrine, glucagon and thyroid hormones a c t indirectly by increasing C-AMPQ a n d s o a c tiv a tin g
C - A M P d e p e n d e n t p r o te in k in a s e .
- I n w e ll fe d s ta te (a ls o c a lle d absorptive stage) insulin suppresses glucose -6 - phosphatase activity and causes glucose 6 -
phosphate to enter glycogenesis® (to fo r m g ly c o g e n ).
- Active glycogen phosphorylase a in both tissues (i.e. muscle and liver) is allosterically inhibited and inactivated by
ATP and glucose 6 - phosphate®. W h e r e a s in liver, but not in muscle, free glucose is also an inhibitor®. A c t u a lly w h e n
p h o s p h o r y la s e b is a c tiv a te d it is c a lle d p h o s p h o r y la s e a.
- 2 fo rm s - a n a c tiv e p h o s p h o r y la te d 'a ' fo r m a n d in a c tiv e d e p h o s p h o r y la te d V fo rm .
P h o s p h o r y la s e k in a s e e x is ts in
Active c-AMP dependent protein kinase p h o s p h o r y la te s in a c tiv e b fo r m to a c tiv e a fo r m . T h e p h o s p h o r y la te d e n z y m e
c a n b e in a c tiv a te d b y h y d r o ly tic r e m o v a l o f p h o s p h a te b y protein phosphatase 1. T h is e n z y m e is a c tiv a te d b y kinase
m ediated signal cascade initiated by insulin®.
Biochemistry: Carbohydrate Metabolism ■ 439

100. B i.e. M u s c l e 101. D i.e . G l u c o s e -6 - p h o s p h a t a s e IRef: Harper s 28/e P-146-148, 159; Lippincott 5/e p. 130; Vasudevan 6/e p. 1081
• The role of glycogen in liver is to provide free glucose for export
to maintain the blood glucose level; whereas in muscles the
Liver & Phosphorylase®
role of glycogen is to provide a source of glucose-6-P04 for Muscle Glucan transferase
glycolysis (to supply ATP for muscle contraction). Debranching enzyme
• This difference of fate of glycogen in liver & muscle is d/t i’
absen ce o f glu cose -6- p h o s p h a ta se enzym e in muscle®. Glucose 1 POj
Because of absence of this enzyme, which catalyzes the final
step of glycogenolysis, glycogen in muscles can only be Liver & Phosphoglucomutase
converted upto glucose- 6 - phosphate. Hence m uscle Muscle
glycogen olysis d oes n ot contribute to b lo o d glu cose r
directly®. Instead, in muscle, this glucose -6-P 04 can not Glucose 6 PO 4
cross plasma membrane and enters glycolysis producing
energy (3 A T P ) needed for muscle contraction. The gain of an
I
extra (1) ATP is d/t bypass of l sl step of glycolysis which
requires 1 ATP (If glycolysis starts from glucose only 2 ATPs Liver & Glucose 6 phosphataeO In m u scle
Kidney (endoplasmic reticulum) g lu c o s e 6
are produced). Lactic acid produced d /1 glycolysis crosses
p h o s p h a ta s e is
the plasma membrane & reaches liver to form glucose.
so glucose 6
a b s e n t,
• In liver glucose -6 . phosphate is transported by glucose- 6 - PO.| enters glycolytic
G lu c o s e
phosphate t r a n s lo c a s e into en d op lasm ic reticulum, where it cycle produce 3 A T P
is converted to glucose by g lu c o s e - 6- p h o s p h a t a s e (this for muscle contraction
enzyme is also used in last step of gluconeogenesis). The & form Lactic acid.
glucose moves out of ER to cytosol and then released by
hepatocytes into blood to maintin blood glucose levels until
gluconeogenesis starts.

104. A i.e . V o n g i e r k e 's d is e a s e 105. B i.e . V o n g i e r k e 's d is e a s e


106. C i.e . F a t m o b i l i z a t i o n is l e s s 107. A i.e M o r e f o r m a t io n o f p e n t o s e [R ef: Chatterjea 7/e P-331; 6/e p. 293]

V o n G i e r k e 's (ty p e l a g ly c o g e n s t o r a g e ) d is e a s e d / t d e fic ie n c y o f g l u c o s e 6 p h o s p h a t a s e e n z y m e Q is a u to s o m a l r e c e s s iv e


in in h e r ita n c e . It is c h a r a c t e r iz e d b y - C e lls (o f liv e r , r e n a l t u b u la r e p it h e liu m & in t e s t in a l m u c o s a ) c o n t a in in g n o r m a l
s t r u c t u r e d m e t a b o l i c a l ly u n a v a i la b l e g ly c o g e n , h e p a to -r e n o m e g a ly Q , d o ll lik e fa c e , h y p o g ly c e m ia , h y p e r - l i p o e m i a ,
h y p e r c h o l e s t e r o le m i a , h y p e r t r i g ly c e r id e m ia (d/t in c r e a s e d f a t m o b i l i z a t i o n ) ^ , f a t t y i n f i l t r a t i o n o f l iv e r , k e to s is Q ,
a c id e m ia , T l a c t ic a c id ( la c t ic a c id o s is )® , T u r ic a c id a n d s t u n t e d g r o w th .

108. B i.e . F a b r y 's d is e a s e ; D i.e . F r a g ile s y n d r o m e ; E i.e . K r a b b e 's d is e a s e


109. B i.e . D e b r a n d h in g e n z y m e 110. A i.e . G l u c a g o n e x c e s s [Ref: Harper's 28fe P-160, 298; Lippincott 5/e p. 130;
Lehninger 5/e p. 599; Chatterjea 7/e p. 330-31; Vasudevan 6/e p. 123, 111; Harrison 17/e p. 24591
• G lu c a g o n 1/t h y p e r g ly c e m ia as it inhibits glycolysis and stimulates gluconeogenesis.
• G lu c o s e 6 p h o s p h a t a s e d e f i c i e n c y ( T y p e Ia/ V o n G i e r k e 's d is e a s e ) , g ly c o g e n s y n th a s e d e f i c i e n c y (Type 0 glycogen
storage diseae), liver & muscle debranching enzyme deficiency (limit dextrinosis), liver & muscle phosphorylase
deficiency and liver phosphorylase kinase deficiency - cause f a s t in g h y p o g ly c e m ia .
• Hypoglycemia in uremia suggest multiorgan involvement.
440 ■ A Complete Review of Short Subjects

Glycogen Storage Diseases (Glycogenosis)

Type Name D eficient Enzyme Organ Clinical Features


involved
primarily
O - Glycogen synthase Liver Hypoglycemia (low blood glucose), hyper ketonemia
(high ketone bodies), early death.
la Von G ierke's Glucose 6 Liver® - Glycogen (normal structured but metabolically
disease phosphatase® unavailable) accumulation in liver & renal tubule cells
1/t hepatorenomegaly and failure (of both)
- Hypoglycemia, hyper lipemia, hypercholesterolemia,
hyper triglyceridemia (d/t increased fa t
mobilization)®, hyper ketonemia (ketosis), lactic
acidemia (lactic acidosis) and hyper uricemia
lb Endoplasmic reticulum Liver - High susceptibility to recurrent bacterial infections
glucose -6-Phosphate d/t neutropenia and impaired neutrophil function
transporter - Rest as Type la
(microsomal -6- P
translocase)
Ic - Microsomal Pi Liver As type la
transporter
II Pompe's Lysosomal glucosidase Skeletal & - Fatal accumulation of glycogen in lysosome
Cardiac
(a l —>4 & a l -» 6 ) or - Infantile-juvenile form: muscle hypotonia & death
Acid maltase muscle from heart failure by age 2.
- Juvenile-adult form : myopathy & muscular dystrophy
Ilia Forbe's or Cori's, Liver & muscle Liver, - Hepatomegaly in infants, myopathy (muscle weakness),
Limit dextrinosis debranching enzymeQ Skeletal & fasting hypoglycemia
Cardiac - Accumulation of characteristic branched
muscle polysaccharide (limit dextrin)
mb Limit dextrinosis Liver debranching Liver As type Ilia but no muscle involvement (weakness)
enzyme (muscle
enzyme normal)
IV Andersen's, Branching enzyme Liver, - Hepato-spleenomegaly, myoglobin in urine, death d/t
Amylopectinosis Skeletal cardiac or liver failure before age 5
muscle - Accumulation of polysachride with few branch points.
V Me Ardle's Muscle (Myo) Skeletal - Exercise induced cramps & pain d/t poor exercise
phosphorylase muscle tolerance; blood lactate is very low after exercise
- Muscle glycogen content is abnormally high (2.5-4%);
myoglobin in urine.
VI Heris Liver phosphorylase Liver Hepatomegaly (d/t accumulation of glycogen), mild
hypoglycemia & generally good prognosis.
V II Tarui's M uscle & erythrocyte Muscle, As type V with hemolytic anemia
phosphofructo kinase RBC
(PFK)-l
V III Liver phosphorylase Liver As type VI
kinase
IX Liver & muscle Liver, As type VI with accumulation of glycogen in muscle.
phosphorylase-kinase muscle
VIb Leukocyte & Liver Leukocytes As type VI
phosphorylase kinase & Liver
X C-AMP dependent Liver Hepatomegaly; accumulation of glycogen in liver
protein kinase
XI Fanconi-Bickel Glucose transporter Liver Failure to thrive, hypatomegaly, renal failure and rickets.
(GLUT-2)
Biochemistry: Carbohydrate Metabolism ■ 441

★ Fabry's disease and krabbe's disease are Disease Deficiency/Absent Enzyme


sphingolipidoses. Fragile X- syndrome is caused by Fabry's deficiency of a - G alactosidase®
abnormal expansion of trinucleotide repeats (CGG/ CCG disease
expansion lead to fragile site on chrom osom es) Farber's deficiency of ceram idase
disease
Forbe's A bsence o f debranching enzyme®.
disease

ill. A i.e . G ly c e r o l - 3 - p h o s p h a t e 112. C i.e . H e x o se m o n o p h o sp h a te sh u n t 113. D i.e . C O 2


114. A i.e . G lu c o s e6 p h o sp h a te d e h y d rg e n a se 115. D i.e . N A D P H 116. C i .e . N A D P H
117. C i.e . G lu c o s e 6 p h o s p h a t e d e h y d r o g e n a s e 118. C i.e . T r a n s k e t o l a s e 119. D i.e . 1 C O 2
120 . C i.e . G - 6- P - d e h y d r o g e n a s e 121. A i.e . L iv e r , C i.e . L a c ta t in g m a m m a r y g la n d ; D i.e . G o n a d s
122 . D i.e . H M P - S h u n t IRef: Lippincott's 5/e p. 145-148; Harper's 28jc p. 174-78; Chattcrgea & Shinde 7/e p. 352-38;Lehninger 5,h/e p. 8301

No ATP is directly produced or consumed in HMP shunt. Its m ajor function is to provide NADPH & pentose sugar®

ATP's produced in
1 1 1 1 1 1
Anaerobic Aerobic glycolysis Krebb's (TCA) Complete HMP p oxidation
glycolysis I cycle Aerobic shunt | |
i 8 ATP® per glucose 1 oxidation of J, Palmitic acid Stearic acid
2ATP® per molecule (old) - 12 ATP® per glucose OATP® i i
glucose 1
- 5/7 ATP (new) acetyl CoA (old) -129 ATP® (old) - 146 ATP® (old)
molecule -1 0 ATP (new) 30-32 ATP (new) -106 ATP® -120 ATP®
(new) (new)

123. A i.e. Glucose-6- PO 4 dehydrogenase I Ref: Lippincott's 4/e p. 1031

Glucose 6 phosphate dehydrogenase deficiency is most common and pyruvate kinase deficiency is 2nd most common
cause of enzyme deficiency related hemolytic anemiaQ.

124. D i .e . T h e o x id a t iv e p h a s e g e n e r a t e s N A D P H a n d n o n o x id a t iv e p h a s e g e n e r a t e s p y r u v a te IRef: Lippincott's 4/c p. 1471


125. D i .e . N A D P H [Ref: Lippincott 5/e p. 152-54; Lehninger 5/e p. 559]

The oxidative (irreversible) phase of HMP shunt produce NADPH and ribulose 5 phosphate^, whereas reversible non-

oxidative phase catalyze the interconversion of 3 ,4 ,5 ,6 , and 7 carbon sugars, and permit ribulose 5 phosphate to be
converted either to ribose 5 phosphate (needed for nucleotide synthesis) or to interm ediates of glycolysis - fructose 6 -
P 0 4 & glyceraldehyde 3- phosphateQ.

Calvin Cycle, Cori Cycle & Cahill Cycle

126. B , D , C i.e . S e d o h e p t u l o s e - 1 -7 - b i s p h o s p h a t a s e ; P h o s p h o r i b u l o s e k i n a s e ; G l y c e r a l d e h y d e 3 PO 4 d eh y d ro g en ase


[Ref: Lehninger 5/e p. 774-86]
f ----------------------------------------------------------------------------------------------------------------------------------------------------------------------
4 Calvin (photosynthetic) cycle enzyme - glyceraldehydes 3- phosphate dehydrogenase, ribulose 5- phosphate
kinase, fructose 1 ,6 - biphosphatase and sedoheptulose, 1 ,7 - biphosphatase a r e a c t iv a te d in d ir e c tly b y lig h t th r o u g h
th io r e d o x in - f e r r e d o x in t h io r e d o x in r e d u c t a s e s y s te m , w h ic h r e d u c e d is u lfid e b o n d s b / w 2 c y s te in e r e s id u e s c r it ic a l to
t h e ir c a ta ly tic a c tiv itie s .

- D isulfide bonded (oxidized) Cys residue, in dark (night) inactivates these enzym es. In day light, electrons flow from
photosystem I to ferredoxin to sm all, soluble, (oxidized) disulfide containing thiredoxin, reduces it in a reaction
catalyzed by ferredoxin- thioredoxin reductase. Reduced thioredoxin donates electrons for reduction (activation) of these
light activated enzymes.
- G l u c o s e 6 p h o s p h a t e d e h y d r o g e n a s e , the T 1enzym e of oxidative PP pathw ay is also regulated by light but in opposite
sense (i.e. light induced reduction inactivates it).
442 ■ A Complete Review of Short Subjects

127. A i.e . I n c r e a s e d r e le a s e o f a l a n i n e fr o m m u s c le 128. A i.e . I n c r e a s e d a l a n i n e l e v e l in m u s c le


129. C i.e . T r a n s a m in a s e e n z y m e & p y r id o x ia l P 0 4 b i n d i n g i s c o v a le n t ; D i.e . G lu t a m a t e is fo r m e d
130. A , B , C , E i.e . A c c e p te d b y a - k e t o g l u t a r a t e ; F u n n e le d to L -G lu t a m a t e ; P r e v e n ts o x id a t iv e b r e a k d o w n ; S G P T / S G O T ...
131. C i.e . I n h i b i t e d b y A D P & a c tiv a te d b y G T P IRef: Harper 29/e p 192-93; Lehninger 5/e p 681, 915, 548, 598; Lippincott 5je p 331-321

During first few days of fasting, there is rapid breakdown of muscle protein, providing aminoacids (alanine &
glutamine mainly) that are used by liver for gluconeognesisQ.
In the fasting state, the output of alanine from skeleton muscle is in far excess of its concentration in the muscle
proteins that are being catabolized. Because it is also formed by transamination of pyruvate produced by glycolysis
of muscle glycogen. Alanine is exported to the liver, where it is transaminated back to pyruvate, which serves as a
substrate for gluconeogenesisQ.

132. C i.e . T r a n s a m in a t i o n o f p y r u v a te to a l a n i n e

Cahill's Glucose - Alanine Cycle produces alanine from pyruvate (not lactate) by transamination. Because muscles
are incapabale of synthesizing urea, the a amino groups of aminoacids (i.e. ammonia) is transported from muscles
to liver in form of alanine. Alanine is also used in gluconeogenesis.
Cori's Lactic Acid Cycle: As gluconeogenesis does not occur in muscleQ, the lactate produced by anaerobic glycolysis
is transported to liver, where it is oxidized to puruvate. Pyruvate is converted to glucose which is transported to
muscles.

I Muscle I
Protein (aa)
Pyruvate Glutamate N H 4V break
S /' ' down
I Alanine aminotransferase
* A (A L T /S G P T )
Alanine * U-Ketoglutarate

Blood
.- '"Blood glucose alanine

Alanine I Liver
a-Ketoglutarate

G lucose < — Pyruvate ^""^Glutamate


Glucose
neogenesis

Alanine - Glucose (Cahill) Cycle


Glucose - Alanine cycle: Skeletal muscle transport N H 3 to
liver in form of alanine, resulting in reciprocal transfer of
Cori (Lactic Acid) Cycle
glucose to muscle, This cycle is called Glucose - Alanine cycle.

133. A i.e . F a tty a c id s [Ref: Harper 29/e p. 16]

Organ Fed Fasting Starvation


RBC Glucose Glucose -

Brain Glucose Glucose Ketone bodies


Heart Fatty acids Fatty acids Ketone bodies
Liver Glucose Fatty acids Amino acids
M uscles & Adipose tissue Glucose Fatty acids Fatty acid (M ajor source)^ & Ketone
Bodies (on prolonged starvation)

Gluconeogenesis

134. C i.e. Acetoacetate /Fatty acids/Palmitate /GH 135. C i.e. Methionine


136. D i.e. Lysine 137. A i.e. Oleate
Biochemistry: Carbohydrate Metabolism ■ 443

Gluconeogenesis Ketogenic A.A


It is the formation of glucose or glycogen from noncarbohydrate sources. Substrate for Those A.A that are
gluconeogenesis (as per priorities) are converted to
1. Glucogenic aminoacids grouped by site of entry in TCA cycle acetoacetate; or one of
its precursors (acetyl
I
Amino acid that Form Form Sucinyl CoA Form a- CoA or acetoacetyl
form pyruvate Oxaloacetate Mn - " 3 W ill m eetl Ketoglutarate CoA) - LeucineQ
Mn - "S ir Try All Lucy" Mn "H is Glues
39 Glass Cyst" Mn - "A s Per" Are Prolific A.A that are both
Sir S e rin e - A sp a ra g in e 3 - T h r e o n in e H is H istid in e G lucogenic &
T rv all T ry p to p h a n - A sp a rta te W ill - V a lin e G lu e s G lu ta m a te Ketogenic
3 T h r e o n in e • Fo rm F u m a r a te M eet - M e th io n in e G lu ta m in e
T - Tyrosine
9 A la n in e ® 1-L u cy - is o -L e u c in e A re A rg in in e
P P h e n y l a la n in e Tryptophan
G la ss G ly c in e P ro lific P ro lin e
C yst C y ste in e
T T y ro s in e I - Isoleucine
P - Phenylalanine
2. Lactate® 3. Pyruvate® 4. G lycerol® (obtained from lipolysis of fat) [M n " G < P < L " |
5. P ropion ic acid® (im p o r ta n t in ru m in a n ts )
138. ALL i.e. (Precursor for synthesis of cholesterol.., Form ketone .., Starting material for synthesis .., Arise from glycolysis)
139. B i.e. Malic Enzyme 140. A i.e. Glucose IRef: Harper 29th/e p. 123,164,187-189; Chatterjae 8/e p 338, 441;
Vasudevan 6/e p. 216, 450, 407; Thomas Miller (2002); Kaplan lecture notes (2007) p-201; Lippincott 4"‘/ed p g - 266,111-12, 220, 96,
119,192, 195-96 Lehninger 5th/ed pg- 616-17, 695, 557] ________________________________________________________
- Acetyl coenzyme A arises from pyruvate (the end product of aerobic glycolysis) by irreversible oxidative
decarboxylation catalyzed by pyruvate dehydrogenase (PDH) complex^
- Acetyl CoA is not a substrate for gluconeogenesis and cannot b e converted back to glucoseQ. This is because acetyl
CoA cannot be converted back to pyruvateQ since its carbon backbone is lost in citric acid cycle as C 02. Hence fatty
acids cannot be converted into glucose
- Substrates for gluconeogenesis are : proprionic acid (in ruminants), glucogenic amino acids, glycerol, pyruvate and
lactate (Mn- "Proper GPL")
- Acetyl CoA is used in oxidative phosphorylation (in TCA cycle) for energy production; in acetylation reaction
(coenzyme pantothenic acid) for detoxification & acetyl choline formation. It is also used in cholesterol synthesis,
steroidogenesis (steroid hormone & vitamin D 3 synthesis), lipogenesis (fatty acid synthesis and elongation, synthesis of
ether lipids like plasmalogens & PAF) and ketogenesis (ketone body formation).
- Acetyl CoA is a precursor or starting material (& substrate for enzymes) for synthesis of cholesterol & ketone bodies
(= substrate for HMG CoA synthetase) and fatty acid synthesis (= substrate for fatty acid synthase, acetyl CoA
carboxylase or malonyl CoA synthetase and malonyl transaceylase)
- M alic enzyme forms malic acid from pyruvic acidQ in presence of CO 2 & NADPH.

Carbohydrate (pyruvate), oxidation of fatty acids (beta- oxidation) and ethanol, ketogenic amino acids, ketone
bodies (ketolysis), HMG CoA, thioesterification of acetate and Clevage of citrate form Acetyl CoA, which is used in
T
TCA (Citric acid cycle /Oxidative Acetylation Cholesterol - Fatty acid
phosphorylation) reaction synthesis synthesis Ketogenesis
- Lipogenesis
i i i i 1
Energy (10 -A TP) + C 0 2 - Detoxification of
-Steroidogenes - Synthesis & Ketone body
- Combines with oxaloacetate to xenobiotics & drugs is elongation of fatty formation
form citric acid, which initiates like isoniazid, -Steroid acids i
TCA cycle to generate ATP via sulfonamides, & hormone, (lipogenesis) Energy
oxidative phosphorylation PAS vitamin Da - Synthesis of ether
- Each molecule of acetyl CoA - Acetyl choline formation lipids eg
produce 10 ATP (new concept) synthesis ★ Acetyl CoA & plasmalogense &
- M elatonin synthesis Malonyl CoA platelet activating
★ All pathways of fat, protein &
★ Pantothenic acid is are used in factor (PAF)
carbohydrate catabolism 1/t
coenzyme for lipogenesis
production of acetyl CoA which is
oxidized in TCA to yield ATP acetylation reaction.
444 ■ A Complete Review of Short Subjects

141. A i.e. Prevent hypoglycemia during prolonged fasting; D i.e. Excess of acetyl co A cause stimulation; E i.e. Carbon
skeleton of amino acid is involved in gluconeogenesis [Ref: Harper 29/e p. 187-94; Lippincott 6/e p. 331-32, 326, 329]
142. C i.e. Alanine & lactate both can serve as substrate

The carbon skeletons for gluconeogenesis are derived primarily from glucogenic aminoacids & lactate from muscle
and glycerol from adipose tissue^. Although the lactate produced in muscle (by anaerobic glycolysis during intense
exercise) is used by liver for gluconeogenesis (through Cori cycle); the muscles do not participate in
gluconeogenesis^. Liver and kidney are the major gluconeogenic tissues^.
Gluconeogenesis prevent hypoglycemia during short and long term (= prolonged) fastingQ i.e. begins 4-6 hours after
the last meal becomes fully active as stores of liver glycogen are depleted and peaks at 5-7 days of fasting .
Gluconeogenesis is stimulated by excess of acetyl CoA and decrease in fructose 2,6 biphosphate concentration^.

143. C i.e. Phosphoglucomutase [Ref: Harper's 29]e p. 179,187-90; Chatterjea 8/e p. 364-69]
144. A i.e. Phosphofructokinase 145. D i.e. Phosphotriose isomerase

- Neoglucognesis (gluconeogenesis) involves all 7 Irreversible Enzyme in Reversed in


enzymes of glycolysis involved in bidirectional Thermodynamic Glycolysis, Glycogen Gluconeogenesis by
reactions such as (1) phosphohexo isomerase ®, (2) Barrier Reactions metabolism (4th) key Enzymes
aldolase (3) Phosphotriose isomerase (4) Phosphoenol Pyruvate Kinase® Pyruvate
glyceraldehyde 3 phosphate dehydrogenase, (5 & pyruvate (PEP) -► carboxylase & PEP
6) phosphoglycerate kinase!mutase®, and enolase. Pyruvate carboxy kinase®
Whereas for thermodynamically irreversible Fructose 6 Phospho Fructose 1, 6
reactions of glycolysis, it uses pyruvate carboxylase phosphate —> fructokinase (PFK)l biphosphatase
& (7) phosphoenol pyruvate (PEP) carboxykinaseQ Fructose 1, 6
in place of pyruvate kinase; fructose 1,6 bi biphosphate
phosphatase® in place of phosphofrctokinase and Glucose —> Gluco/Hexo-Kinase® Glucose 6
glucose 6 phosphatase® in place of Glucose 6 phosphatase®
hexo /glucokinase phosphate
- Phosphoglucomutase is involved in Glycogen Glycogen —* Glycogen UDP Glc
metabolism® not gluconeogenesis. Glucose 1 phosphorylase, phosphorylase,
phosphate Glucan transferase, Glycogen synthase,
Phosphogluco Mutase® Debranching enzyme Branching enzyme
Glucose- 6-PC >4 Glucose - 1 - PO 4

146. A i.e. Acetyldehyde [Ref: Harper's 27/e p. 225-26; Lippioncott's 4/e p. 317-18]

Ethanol is oxidized to acetaldehyde by


alcohol dehydrogenase and
subsequently to acetate by aldehyde
dehydrogenase. Both reactions
produce NADH by transferring
electrons to NAD+. The abundance of
NADH (increased NADH /NAD+
ratio) favors the reduction of pyruvate
to lactate and of oxaloacetate to
malate. Pyruvate and oxaloacetate are
both intermediates in synthesis of
glucose by gluconeogenesis. So
alcohol consumption decreases
gluconeogenesis & precipitate hypoglycemia particularly if liver glycogen stores are depleted.
147. C i.e. ADP level 148. B i.e. Acetyl CoA stimulation of pyruvate carboxylase
149. B i.e . P y r u v a te c a r b o x y la t io n a c t iv a tio n 150. A i.e . L e v e l o f F r u c to s e 1 , 6- b i p h o s p h a t e i s h i g h e r t h a n n o r m a l
151. B i.e . L iv e r 152. D i.e . F r u c to s e 1 ,6 b i p h o s p h a t e
153. C i.e. Increased levels of Fructose 2, 6 - Bisphosphate [R ef:H a rp er 2 9 /e p. 187 -9 3 ; L ip p in cott 5 /e p. 1 1 7-122; L eh n in g er 5 /e p. 5 8 2 -8 7 ]
- Decreased concentration of glucose in blood & liver causes increased glucagon secretion (& f glucagon / insulin ratio)
leading to glycogenolysis followed by gluconeogenesis in liver. Increased levels of glucagon activates c-AMP
dependent protein kinase which intum activates F 2 ,6 BPase (fructose 2 , 6 biphosphatase or FBpase 2) and inactivates
phosphofructokinase (PFK)-2 leading to conversion of F2, 6 BP (Fructose 2, 6 biphosphatse) to fructose 6 phosphate. So
the concentration ofF2, 6 BP is decreased® and fructose 6 phosphate is increased which is converted to glucose.
Biochemistry: Carbohydrate Metabolism ■ 445

- Gluconeogenesis in fasting state is indicated by acetyl CoA induced instantaneous allosteric activation of hepatic
pyruvate carboxylase.Q
- Phosphofructokinase I takes part in glycolysis and FBPase 1 in gluconeogenesis, by catalyzing the formation &
breakdown of fructose 1 ,6 biphosphate respectively. Whereas bifunctional enzyme phosphor fructo kinase 2 (PK-2)
also has fructose 2 ,6 biphosphatase (FBP ase 2 or F 2 ,6 pase) activity and plays a unique role in regulation of both
glycolysis & gluconeogenesis by determining the levels o f fructose 2 ,6 bihosphateQ.
- Fructose 2 ,6 bihosphate (F2,6BP) is the most potent allosteric activator of PFK-1 (thereby increase glycolysis) and
inhibitor of fructose 1 ,6 biphosphatase /FBP ase 1/ F I, 6 pase (thereby decreasing gluconegenesis) in liverQ. The
reciprocal action of fructose 2 ,6 biPC>4, on glycolysis and gluconeogenesis ensure that both pathways, is not fully active
at same time. Therefore, decreased sensitivity of FBP ase 1 to regulation (inhibition) by fructose 2 ,6 biphosphate
increases gluconeogenesis; thereby decreasing levels of fructose 1 ,6 biphosphate, pyruvate and ATPQ.
- Fructose 1 ,6 biphosphate is under negative feed back control of fructose 2 ,6 biphosphateQ. If fructose 1 , 6
biphasphatase becomes less sensitive to the negative feed back of fructose 2 , 6 biPCX its level of activity will increase.
Thus more fructose 1, 6 biPC>4 will be cleaved into fructose 6 -PO 4 and levels of fructose 1, 6 biP 0 4 will decrease and be
lower than normal. The cycle will be shifted more toward gluconeogenesis. Less ATP would b e generated d/t decreased
glycolysisQ.
- Glycogenic capability of cell is determined by presence (activation) of fructose 1 ,6 biphosphataseQ. The conversion
of fructose 1 , 6-bisphosphate to fructose6-phosphate, to achieve a reversal of glycolysis, is catalyzed by fructose-1,6-
bisphosphatase. Its presence determines whether or not a tissue is capable of synthesizing glycogen not only from
pyruvate but also from triosephosphates. It is present in liver, kidney, and skeletal muscle but is probably absent from
heart and smooth muscle.
- In well fed state gluconeogenesis in liver is inhibited by ADPQ.
154. C i.e. Decreased gluconeogenesis [Ref: Lippincott 4thfed pg-322-23,329; Teitz 6thled pg-376,388; Chatterjea 7lh/ed pg-363-64]

Glycolysis increases and gluconeogenesis decreases after feedingQ.

155. B i.e. Citrate______________________________________________________________________________________________________


____________________ Regulatory & Adaptive Enzymes a/w Carbohydrate Metabolism____________________
Enzyme Enzyme Activity in Transcription Regulator Allosteric Regulation
High Energy state Low Energy Inducer Repressor Activator Inhibitor
(Carbohydrate state
Feeding) (Fasting/
Diabetes)
G lycogenolysis, Increase 1-6 Decrease 1-6
Glycolysis, & except 2 except 2
Pyruvate oxidation
1. Glycogen synthase T 1 Insulin, glucose Glucagon
6-phosphate
2. Hexokinase - - Glucose 6
phosphate
3. Glucokinase T I Insulin Glucagon
4. Phosphofructo­ T 1 Insulin Glucagon AMP, fructose 6- CitrateQ, ATP,
kinase (PFK)l P, fructose 2 , 6 glucagon
biphosphate, Pi
5. Pyruvate kinase T I Insulin, Glucagon Fructos 1, 6 ATP, alanine,
fructose biphosphate, glucagon, nor
insulin epinephrine
6. Pyruvate T I CoA, NAD+, Acetyl CoA®,
dehydrogenase insulin, ADP, NADH, ATP (fatty
pyruvate acid ketone -bodies)
Gluconeogenesis Glucacortico- Insulin Acetyl CoAQ ADPQ
7. Pyruvate steroids,
carboxylase glucagon, Glucagon? -
8. PEP carboxykinase epinephrine
9. Glucose 6-
phosphatase
446 ■ A Complete Review of Short Subjects

Fructose Metabolism

156. C i .e . H y p e r g ly c e m ia [Ref: Harper's 27/e P-183; Harrison 16/e P-2321]

Hereditary fructose intolerance due to deficiency o f aldolase B®, causes fructose induced hypoglycemia® despite the
presence of high glycogen reserve. The accumulation of fructose 1- PO 4 and fructose 1 , 6 - P 0 4 allosterically inhibit liver
phosphorylase

157. A i.e . F r u c to s e - 1 - P O 4 [Ref: Harper's 27/e P-183-84]

Fructokinase convert fructose to fructose -1 - P O fi. It is deficient in essential fructosuria

158. A i.e. Glyceraldehyde- 3- phosphate dehydrogenase [Ref: Harper's 27/e P-183-184; 26/e P-169]

G lyceraldehyde -3 phosphate dehydrogenase is an enzyme in the pathway fo r conversion o f fructose to pyruvate®.

Fructose Phospho- glucomutase: It diverts glycolysis substrate to uronic


Fructokinase acid pathway.

Glucose- 6- P 04 ■Glucose - 1 - P O 4
Fructose -1 - P Phosphoglucomutase

Aldolase B Lactate dehydrogenase


Glyceraldehyde Dihydroxyacetone - P Pyruvate Lactate
Lactate dehydrogenase
Triokinase Trio P Isomerase

Glyceraldehyde 3 - P Glucokinase: It has high affinity for glucose. So in presence of


both glucose & fructose, it preferentially binds glucose.
Glyceraldehyde - 3P
dehydrogenase ® Glucokinase
Glucose Glucose - 6 - PO4

1, 3 biphosphoglycerate

(via the normal glycolytic


pathway)

Pyruvate

Galactose Metabolism & Intolerance

159. D i.e. All of the above 160. D i.e. Galactose


161. A i.e. G alactose-l-Phosphate uridyl transferase 162. A i.e. G alactose-l-phosphate uridyl transferase
163. A i.e Mental retardation occurs; C i.e Defect in epimerase; D i.e Defect in galactose - 1 P O 4 uridyltransferase
164. C i.e. Galactosemia 165. B i.e. Galactosemia 166. D i.e. Galactitol

Glactosemia

An inherited A u tosom al recessive® disorder, in which there is inability to convert galactose to glucose in normal manner.
This may occur due to deficiency of following enzymes:
Biochemistry: Carbohydrate Metabolism ■ 447

Galactose I-PO 4 UDP galactose - 4 Epimerase Galactokinase


Uridyl Transferase (Rare) (Minor)
(classical & most Feature of uridyl trans ferase deficiency + Cataract®
com m on) - N erve deafness Affected infant is other wise asym ptom atic.
- M anifests with - H ypotonia
in a few days - Tin cellular UDP Galactose Galactose M etabolism
after birth (after Infants are subject to b a cteria l sep sis
ingestion o f (especially w ith E.coli)® this m ay be G a la to s e

milk)® leading cause of death. ATP


Failure to thrive B io ch em ical & urinary fin d in g G alactokinase I
H epto - Increase blood galactose level ADP

splenom egaly® - D ecreased blood glucose® (normal G alatose-l-PO ^ UD P- G lucose


Jaundice, Ascites oxidase-peroxidase test)Q B lo c k in in herite Galactose-1-PO^
(Classical type)
- H ypoglycem ia® g a la c to s e m ia ' uridyl transferase
- G alactosuria (reducing sugar
M ental G lucose -I-P O 4 UDP - Galactose
g alactose in urine gives positive A ___________
retardation® Benedict's test)® UD P-Galactose
- Cataract®, (oil Epim erase
- Album inuria
drop cataract)® - A m iaciduria: Glycine, Alanine, Serine ->■ Glycogen
UDP - G lucose -
Treatm ent: G ala ctose free diet®.

Cataract
G lucose
I--------------------- --------
In Galactosemia is due to In Diabetes is due to
accumulation of galactitolQ. accumulation of sorbito
•The increased galactose conc. in in lens.
blood and eye is reduced to TConcentration of glucosi
Galactitol by aldolase reductase, in blood & eye is reduced t
•Galactitol can not escape from lens sorbitol by aldolase
cells and accum ulates to form reductase.
cataract.

167. D i.e. Lactase 168. D i.e. Sucrase [Ref: Harper's 27/e P-184-185; Chatterjea 6/e P- 312-313, 264-65, Nelson 17/e P-1268]

This patient develops gas and diarrhoea follow ing consum ption o f dairy products. These are symptoms of 'Lactose
intolerance' due to deficiency of LactaseQ. Sucrase deficiency presents with sim ilar clinical features as in lactase
deficiencyQ. As lactase is not given as an option, sucrase is the 2nd b est choice.

Lactase deficiency Sucrase Isomaltase Enzyme Associated disorder


deficiency deficiency
• Due to deficien cy o f lactase®. • Due to deficiency of sucrose P- - Galactosidosis
Lactase® & isom altase galactosidase - GM1 gangliosidosis
L a c to s e ----------------------- >■ ualactose + • Clinical features in response deficiency - M ucopolysacharidosis IV
Glucose to ingestion of sugar (cane A: M orquio A
• Clinical features in response to ingestion of sugar and table sugar) a-glucosidase Type II glycogen storage
lactose (sugar in milk) - Bloating deficiency disease (Acid a l , 4
- B loatin g (gas)® - W atery diarrhea glucosidase deficiency :
- W atery diarrhoea® - Failure to thrive Pom pes disease)
- Failure to thrive
- A bdom in al cramp®

Alcohol Metabolism

169. B & C i.e. NAD+ & NADPH IR ef: L ip p in cott 4 l,'/ed p g - 3 1 7 -1 8 ; H a rp er 28"'/ed p g - 6 3 0 -2 9 , 2 1 8 -2 0 ; L eh n in g er 5"'led p g -5 4 7 ,664,5181
/--------------------------------------------------------------------------------------------------------------------------------------------------------------------------
In chronic alcoholism the rate lim iting com ponent for alcohol m etabolism excluding enzym e are NAPH (in MEOS) and
NAD+ (in m ajor pathw ay).
448 ■ A Complete Review of Short Subjects

170. C i.e. D-xylose absorption test

Glycosylated hemoglobin (HbAic, an index of average glucose level in preceding 12 weeks), Fasting blood sugar (FBS >
126 mg/dl), oral glucose tolerance test (OGTT) i.e. 2 hour post prandial glucose levels (> 200 mg/dl) and random blood
sugar (RBS > 200 mg(dl) are used to diagnose diabetes mellitus whereas D-xylose absorption test is used to determine
absorption of D-xylose in proximal small intestine (duodenum & jejunum) to detect intestinal malabsorption syndrome*?.

Carbohydrate Classification

171. C i.e. D -M an nose & L -M annose; D i.e. d-M annose & 1-M annose; E i.e. D -glucose and L glucose
172. C, E i.e. D -glucose & D -m annose, D - galactose & D -glucose
173. A i.e. M annose [Ref: Harper 28/e p. 114-15; Lippincott 5/e p. 83-85; Lehninger 5/e p. 236-40] *

' H
- Carbohydrate isomers that differ in configuration around only one carbon atom are called epimers. D-glucose and D-
mannose are C 2 epim ers w hereas D- glucose and D- galactose differ only at C 4. However, D-mannose & D-galactose
are not epimersQ as they differ at 2 carbons (2 & 4).
- Enantiomers (optical isomers or stereoisomers) are pairs of structures that are mirror images of each other but not
identical, sim ilar to left and right hand w hich are the sam e bu t opposite. These are non super im posable and are
designated as D and L-sugar. Each enantiom er often shows different chemical reactions with other enantiom ers. Because of
presence o f m any enantiom ers in living beings, there is usually a m arked difference in effects o f tw o stereoisom ers. For
exam ple only one of drug stereoisom er produces desired effect w hile the other does not.
v___________________________________________________________________________________________________

174. B i.e. Beta configu ration o f anom eric C2 [Ref: Harper 28/e p. 118-19; Lippincott 5/e p. 86-87;Lehninger 5/e p. 246-50; 'Food and
Nutrition supplements: Their role in health and disease' (Springer) 1st (2001)/130; 'Colonic Microbiota, Nutrition and Health' by
Gibson & Roberfroid P 1(1999)/105[

Inulin like fructan - prebiotics are non digestible in upper GI tract because of their beta (fi) configuration of anomeric
C2a.
C h a p t e r -2 FAT METABOLISM

QUESTIONS

Lipids of Physiological Significance 9. The follow ing fatty acid does not belong to W6 series
A . Linoleic acid (PGI 10; AIIMS 08) □
1. A ll a re tru e e x c e p t (PGI 02) B. Arachidonc acid □
A. Linoleic acid is found in soyabean oil □ C. Gamma linolenic acid □
B. Linolenic & linoleic acids are cis derivatives D . Alpha linolenic acid □
containing double bonds. □ 10. Gangliosides consists of: (PGI 08)
C. Arachidonic acid contain five double bonds. □ A . Long chain fatty acid (LCFA) □
D. Monoenoic acids contain one double bond at 9th B. Alcohol- sphingosine □
position. □ C. Phosphate □
2. W h i c h a m o n g t h e f o l l o w i n g i s a c a r d i o p r o t e c t iv e f a t t y D . Sailic acid □
a c id ? (NBE P 13; AI 11) E. Glycerol □
A. Palmitic acid □ 11. W hich o f the follow ing is correct in regards o f function
B . Stearic acid □ o f phospholipid: (PGI 05)
C. Oleic acid □ A . Cell-cell interaction □
D. Omega-3 fatty acids □ B. Signal transduction □
3. T r u e s t a t e m e n t a b o u t f a t t y a c id : (PGI 11) C. Surfactant □
A. Polyunsaturated FA is essential for membrane D . Acute inflammatory mediator □
structure □ E. Activates membrane enzyme □
B. Biologically arachidonic acid is essential to life □ 12. Glycosphingolipidis made up of: (PGI 10)
C. Hydrogenated vegetable oils contains Trans fatty A. Glucose □
acid □ B. Glycerol □
D. Most of the naturally occurring unsaturated FA C. Sphingosine □
exists as trans-isomer □ D . Fatty acids □
4. F a l s e a b o u t t r a n s f a t t y a c id (PGI 10, 09) E. Thromboxane A2 □
A. Increase risk of cardiovascular disease □ 13. Alcoholic group is found in: (PGI 08)
B. Fried foods have high content of TFA □ A . Ganglioside □
C. Hydrogenation increases TFA □ B. Sphingomyelin □
D. Hydrogenation decreases TFA □ C. Cerebroside □
E. It T LDL & i HDL □ D. Terpen □
5. T h e m o s t e s s e n t i a l f a t t y a c id i s (NBE P 13; AI 05) 14. Not a predisposing factor for atherosclerotic plaque
A. Linoleic acid □ formation? (AI 11)
B. Linolenic acid □ A. Apo E deficiency □
C. Arachidonic acid □ B. Alpha 2 - macroglobulin □
D. Palmitic acid □ C. Oxidised LDL □
6. E s s e n t i a l f a t t y a c id is/ are: (PGI 13) D. Increased homocyctiene □
A. Palmitic acid □ 15. W hich o f the follow ing phospholipid has antigenic
B. Linoleic acid □ activity- (NBE P 14,15)
C. Linolenic acid □ A. Plasmalogen □
D. Free fatty acid □ B. Cardiolipin □
E. Oleic acid □ C. Phosphatidylcholine □
7. M a x im u m s o u r c e o f l i n o l e i c a c id i s (PGI 13; AIIMS 04) D. Sphingoyelin □
A. Coconut oil □
B. Sunflower oil □ Acylglycerol & Sphingolipid Metabolism
C. Palm oil □
D. Vanaspati □ 16. Fatty acids are stored as- (NBE P 14,15)
8. W h ic h o f th e s e fa tty a c id s is fo u n d e x c lu s iv e ly in A. Cholesterol □
b r e a s t m il k : (NBE P 15; AI 01) B. Triglycerides □
A. Linolaete □ C. Sphingomyelin □
B. Linolenic □ D. None □
C. Palmitic □ 17. M olecular change in Lysosomal storage disorder is:
D. Docosahexanoic acid □ (AIIMS 07)
A. Defective fusion of lysosomes and phagosomes □
450 ■ A Complete Review of Short Subjects

B. Increased synthesis of some substrates □ Lipid Transport


C. Mutation of genes encoding lysosomal hydrolases □
D. Membrane defect of lysosomes □ 27. Which of the following lipoproteins does not move
18. Which of the following is/are not sphingolipidosis: towards charged end in electrophoresis or Which has
A. Tay sach's disease (PGI 13) □ least density (AIIMS 15,13; NBE P 15,14)
B. Sandhoff's disease □ A. VLDL □
C. Krabbe's disease □ B. LDL □
D . Wolman disease □ C. HDL □
E. Fabry's disease □ D. Chylomicrons □
19. True about gaucher disease: (PGI 09) 28. The human plasma lipoprotein containing the highest
A. Due to deficiency of enzyme sphingomyelinase □ percentage of triacylglycerol by weight is OR
B. Due to deficiency of enzyme Beta-Gluco- Triglycerides are maximum in OR Maximum content
cerebrosidase □ of triglycerides are found in:
C. Deposition of glucosylceramide □ A. VLDL (NBE P 15,13; AIIMS 14,13; PGI 14,12) □
D . Foam cell depositon □ B. Chylomicrons □
E. PAS staining positive □ C. HDL □
20. Sphingomyelinase Deficiency is seen in OR D. LDL □
Accumulation of sphingomyelin in phagocytic cells is 29. Dietary triglycerides are transported by
feature of: (AIIMS 14,12; AI 10) A . Chylomicrons (AIIMS 15,10) □
A. Niemann-Pick disease (NBE P 14,13; PGI 13,15) □ B. VLDL □
B. Farber's disease □ C. LDL □
C. Tay Sach's disease □ D. HDL □
D . Krabbe's disease □ 30. Apolipoprotein E is rich in- (NBE P 15)
21. Gaucher's disease is due to deficiency of enzyme : A. Lysine □
B. Arginine □
A. Sphingomyelinase (AI 11; PGI 14; NBE P 15,14,13) □
C. Histidine □
B. (3-Glucosidase (Glucocerebrosidase) □
D. Methionine □
C. Hexosaminidase-A □
31. All are true about LDL except- (NBE P 15)
D . P-Galactosidase □
A. More dense than chylomicron □
22. Splenomegaly seen in A/E: (PGI 06)
B. Smaller than VLDL □
A. Neimann pick's disease □
C. Transports maximum amount of lipid □
B. Krabbe's disease □
D. Contains maximum cholesterol □
C. GM2 gangliosidosis □ 32. Highest amount of cholesterol is present in OR The
D . Gaucher's disease □ most important carrier of cholesterol in plasma is:
E. Sialodosis □ A . HDL (PGI 14,09,02; NBE P 15,14,13) D
23. Krabbes disease is due to deficiency of- (NBE P 15) B. LDL □
A. SPhingomyelinase □ C. VLDL □
B. Beta galactocerebrosidase □ D. IDL □
C. Hexosaminidase □ E. Chylomicrons □
D. Arylsulfatase □ 33. M ain transporter of (dietary) cholesterol to peripheral
24. Nieman pick disease is due to deficiency of- (extrahepatic) tissue is: (PGI 15,13,11,08; NBE P 14)
A. Sphingomyelinase (NBE P 15) □ A . HDL □
B. Ceramidase □ B. LDL □
C. Galactosidase □ C. IDL □
D. Glucosidase □ D. VLDL □
25. Enzyme deficiency seen in genetic disease like: E. Chylomicron □
A. Tay sach's disease (PGI 03) □ 34. Lipoprotein associated with carrying cholesterol from
B. Gaucher's disease □ periphery tissues to liver is- (NBE P 15)
C. Cystic Fibrosis □ A. HDL □
D. Wilson's disease □ B. LDL □
E. Familial hypercholesterolemia □ C. VLDL □
26. Enzyme replacement therapy is available for- D. IDL □
A. Gauchers disease (NBE P 15,13) □ 35. In cholesterol transport which is not needed-
B. Galactosemia □ A. Liver (N B E P 1 5 )U
C. Fructosuria □ B. Kidney □
D. None □ C. Intestine □
D. Fat □
Biochemistry: Fat Metabolism ■ 451

36. Lipid with highest mobility is- (NBE P 15) 46. Which of the follow ing is an activator of LCAT-
A. HDL □ A . Apo B 100 (DNB 14; JIPMER 02) 0
B. LDL □ B. Apo B 48 □
C. VLDL □ C. Apo E □
D. Chylomicrons □ D. Apo AI □
37. Apoprotein A is found in : (AI 09; NBE P 15) 47. All of the follow ing statements about apoproteins are
A. Chylomicrons □ true Except: (AI 08)
B. VLDL □ A. Apoprotein A-I activates LCAT □
C. HDL □ B. Apoprotein C-I activates lipoprotein lipase □
D. LDL □ C. Apoprotein C-II inhibits lipoprotein lipase □
38. Which of the follow ing is maximum in HDL as D. Apoprotein C- II activates lipoprotein lipase □
compared to other lipoproteins? (AIIMS 16) 48. Best marker of dyslipidemia (AIIMS 09)
A. Cholesterol □ A. TG & cholesterol □
B. LDL / HDL ratio □
B. Triglycerides □
C. Cholesterol □
C. Apoproteins □
D. APO AI □
D. Fatty acids □
49. M ost important in causing coronary artery disease
39. Which of the follow ing has highest electrophoretic
(CAD) is (AIIMS 09)
m obility and least lipid content: (NBE P 14; PGI 16,14)
A. HDL □
A. Chylomicrons □
B. LDL □
B. HDL □
C. VLDL □
C. LDL □
D. Triglycerides □
D. VLDL □
50. All of the follow ing statements about LDL are true
E. IDL □
except: (AIIMS 04)
40. Good cholesterol found in: (PGI 12, 07)
A . It delivers cholesterol to cells □
A. HDL □
B. It contains only one Apoprotein □
B . LDL □
C. It is a marker for cardiovascular disease □
C. VLDL □ D. It contains Apo-B 4 □
D. IDL □ 51. All the statements regarding LDL receptors are true
41. Apo B 4s and apo Bioo are expressed as two different except: (AIIMS 11)
apo-proteins because of difference in : (AI 02) A . Present only inthe extra-hepatic tissues □
A . RNA editing □ B. Clathrin-coated pits contain the receptor on cell
B. RNA splicing □ membrane □
C. Chromosomal loci □ C. It is taken into the cells by endocytosis □
D. Apo-Bgene □ D. Increased cellular cholesterol down-regulates the
42. Which helps in the transport o f chylomicrons from synthesis of LDL receptors □
intestine to liver: (NBE P 13; AI 10) 52. Cholesterol present in LDL : (AIIMS 03)
A. Apoprotein B □ A . Represents primarily cholesterol that is being
B. Apoprotein A □ removed from peripheral cells. □
C. Apoprotein C □ B. Binds to a receptor and diffuses across the cell
D. Apoprotein E □ membrance. □
43. M ajor apolipoprotein o f chylomicrons- C. On accumulation in the cell inhibits replenishment of
A. B-100 (NBE P 15) □ LDL receptors. □
B. D □ D. When enters a cell, suppresses activity of acyl-CoA:
C. B-48 □ cholesterol acytranferase ACAT. □
D. None □ 53. All of the follow ing statements are true regarding
44. Endogenous triglycerides in plasma are maximally lipoproteins except:
carried in: (NBE P 13) A . VLDL transports endogenous lipids (AI 01) □
A. VLDL □ B. LDL transports lipids to the tissues. □
B. Chylomicrons □ C. Increased blood cholesterol is associated with
C. LDL □ increased LDL receptors □
D. HDL □ D. Increased HDL is associated with decreased risk of
45. VLDL is formed in- (NBE P 15) coronary disease □
A. Intestine □ 54. Which receptors are present in liver for uptake o f LDL:
A . Apo E (AIIMS 09) □
B. Liver □
B. Apo A and Apo E □
C. From chylomicrons □
C. Apo E and Apo B100 □
D. Blood □
D. Apo B100 □
452 ■ A Complete Review of Short Subjects

55. All of the following statements about Lipoprotein 64. Endogenous triglycerides in plasma are maximally
Lipase are true, Except (AI 09) carried in- (NBE P 15)
A . Found in adipose tissue □ A. VLDL □
B. Found in myocytes □ B. Chylomicrons □
C. Deficiency leads to hypertriacylglycerolemia □ C. LDL □
D . Does not require CII as cofactor □ D. HDL □
56. Lipoprotein with scavenging action is OR 65. Which is an abnormal lipoprotein- (NBE P 15)
Concentration of which is inversely related to the risk A. VLDL □
of coronary heart disease: (NBE P 15,13; PGI 15,13, 07) B. Chylomicron □
A. HDL □ C. LP (a) □
B. LDL □ D. LDL □
C. VLDL □ 66. Increased level of lipoprotein(a) predisposes to:
D. None □ A. Liver cirrhosis (AIIMS 06) □
57. A young male patient presents with LDL 600mg/dl, B. Atherosclerosis □
triglycerides 140 mg/dl. What would be the most likely C. Nephrotic syndrome □
finding on physical examination (AIIMS 11)
D. Pancreatitis □
A. Tendon xanthoma □
B. Planar xanthoma □
Cholesterol Synthesis & Excretion
C. Lipemia retinalis □
D . Tuberoeruptive xanthoma □
58. A patient was diagnosed with isolated increase in LDL. 67. In cholesterol synthesis, which is rate lim iting enzyme:
His father and brother had the same disease with A. HMG CoA reductasefNBEP 15,14; AIIMS 15,13,11) □
increased cholesterol. The likely diagnosis is: B. HMG CoA synthetase (PGI 14,12,06) □
A . Familial type III hyperlipoproteinemia (AIIMS 09)D C. 7 alpha hydroxylase □
B. Abetaliporoteinemia □ D. Phosphofructokinase □
C. Familial lipoprotein lipase deficiency (type 1) □ 68. Which of the follow ing enzymes is common to the
D . LDL receptor mutation □ synthesis of cholesterol and ketone bodies: (AI 12)
59. Which of the follow ing is increased in lipoprotein A . HMG -Co-A Reductase □
lipase deficiency: (AIIMS 02) B. HMG-Co-A Lyase □
A. VLDL □ C. HMG-Co-A Synthase □
B. LDL □ D. Thiokinase □
C. HDL □ 69. Cholesterol structurally belongs to molecule:
D. Chylomicrons □ A . Steroid (PGI 03) □
60. In Familial hypercholesterolemia there is deficiency B. Carbohydrate □
of: (PGI 08; AIIMS 05) C. Urea □
A . LDL receptor □ D. Peptide □
B. Apo protein A □ 70. Fatty acids of cholesterol are mainly: (PGI 2K)
C. Apo protein C □ A. Oleic acid □
D. Lipoprotein lipase. □ B. Linoleic acid □
61. Which of the following types of hypertriglyceridemia C. Linolenic acid □
is associated with an increase in chylomicron and D. Palmitic acid □
VLDL remnants? (AI 08) E. Stearic acid □
A . Type I □ 71. For reduction of cholesterol, the best management
B. Type Ha □ includes: (PGI 03)
C. Type III □ A . Cholesterol restricted diet □
D . Type IV □ B. High fiber diet □
62. In Abetaltipoproteinemia result in absence of: C. Diet with high unsaturated FA and low
A . Chylomicron (PGI 02) □
saturated FA □
B. LDL □
D. Diet with low unsaturated FA and high
C. VLDL □
saturated FA □
D . HDL □
E. Cholesterol restricted diet, high fiber and diet with
E. TG □
low saturated FA to unsaturated FA ratio □
63. A person on a fat free carbohydrate rich diet continues
72. Hypolipidemic agents act on: (PGI 02)
to grow obese. Which of the follow ing lipoproteins is
A . HMG CoA synthetase □
likely to be elevated in his blood?
B. HMG CoA oxygenase □
A . Chylomicrons (AI 04) □
C. HMG CoA reductase □
B. VLDL □
D. HMG CoA hydratase □
C. LDL □
D . HDL □ E. HMG CoA mutase □
Biochemistry: Fat Metabolism ■ 453

73. Bile acids are derived from: (AIIMIS 07) 83. Acetyl CoA is necessary for: (PGI 07)
A. Fatty acids □ A. Aminoacid synthesis □
B. Cholesterol □ B. Fatty acid synthesis □
C. Bilirubin □ C. Glucose storage □
D. Proteins □ D. All □
74. Bile acids synthesized in liver (Primary bile acids): 84. In fatty acid synthesis CO 2 loss occurs in which step
A. Lithocolic acid (PGI 2K) □ A. Hydration (PGI 02) □
B. Cholic acid □ B. Dehydration □
C. Chenodeoxycholic acid □ C. Condensation reaction □
D. Deoxycholic acid □ D. Reduction □
E. Taurocholic acid □ 85. Saturated aceyl enzyme formation causes:
75. Common intermediate in synthesis of steroid A. Freeing of PAN-SH site (PGI 08) □
hormones is: (AIIMS 1 6 ) B. Freeing of Cyst - SH site □
A. 7 Dihydrocholesterol □ C. Both □
B. Cortisone □ D. None □
C. Pregnenolone □ 86. Carbon atoms added in FA synthesis (AIIMS 01)
D. 7 Hydroxycholesterol □ A. 2 in 1st cycle & 4 in Ilnd cycle □
B. 4 in 1st cycle & 2 in Ilnd cycle □
Fatty Acid Synthesis And Metabolism
C. 2 in 1st cycle & 2 in Ilnd cycle □
76. Which biochem ical reaction does not occur in the D. 4 in 1st cycle & 4 in Ilnd cycle □
mitochondria: (NBE P 15,14,13; AIIMS 14) 87. A ll participate in Fatty acid synthesis except
A. Kreb's cycle/Urea cycle / TCA cycle □ A. Hydratase (AIIMS 02) □
B. Fatty acid (0) oxidation □ B. Reductase □
C. Gluconeogenesis/Ketogenesis □ C. Transacylase □
D . Fatty acid synthesis/EMP □ D. Dehydrogenase □
77. M itochondria is involved in all o f the follow ing, 88. PAN-SH site of fatty acid synthase complex accepts:
except: (AIIMS 03) A . Acetyl CoA (PGI 02; AIIMS 04) □
A . ATP production □ B. Malonyl CoA □
B. Apoptosis □ C. Propionyl Co A □
C. Tricarboxylic acid cycle □ D. All □
D. Fatty acid biosynthesis □ 89. True about mitochondrial chain elongation of fatty
78. NADPH is required for : (AI 08) acid is/are (PGI 09)
A . Gluconeogenesis □ A. Operates under anaerobic conditions □
B. Glycolysis □ B. Operates aerobically □
C. Fatty acids synthesis □ C. Common pathway □
D . Glycogenolysis □ D. Not a common pathway □
79. NADPH is used in: (AI 05) E. Pyridoxal-Phosphate & NADPH is required □
A . Fatty acid synthesis □ 90. In which organelle (s) of hepatocyte, the elongation of
B. Ketone synthesis □ long chain fatty acid takes place: (PGI 08)
C. Gluconeogenesis □ A. Endoplasmic reticulum (ER) □
D . Glycolysis □ B. Golgi body □
80. In synthesis of fatty acids energy is supplied by: C. Mitochondria □
A . NAD (AIIMS 02) □ D. Lysosomes □
B. FAD □ E. Ribosome □
C. GTP □
D . NADPH □
Triacylglycerol and Phosphoglycerol Synthesis
81. The most important source of reducing equivalents for
fatty acid synthesis in the liver is: (AIIMS 04)
91. Regarding synthesis o f triacyl glycerol in adipose
A. Glycolysis □
tissue, all of the follow ing are true except: (AI 07)
B. TCA cycle □
A. Synthesis from Dihydroxyacetone phosphate □
C. Uronic acid pathway □
B. Enzyme Glycerol kinase plays an important role □
D. HMP pathway □
C. Enzyme Glycerol 3 phosphate dehydrogenase plays
82. The first step in fatty acid synthesis involves:
an important role □
A . Acetyl CoA carboxylase (AI 06) □
D. Phosphatidate is hydrolyzed □
B. P-Hydroxyl CoA dehydrogenase □
C. Acetyl dehydrogenase □
D. Pyruvate kinase □
454 ■ A Complete Review of Short Subjects

P - Oxidation Of Fatty Acids 1 0 1 .T h e im m e d ia t e p r e c u r s o r i n th e f o r m a t io n o f


a c e t o a c e t a t e f r o m a c e t y l C o A i n th e l iv e r is:
92. The substance essential for transfer of fatty acids
A. Mevalonate (PGI 09) □
across mitochondrial membrane:
B. HMG CoA □
A. Cera tine □
C. Acetoacetyl CoA □
B. Ceratinine □
D. 3-hydroxy-butyryl □
C. Carnitine □
102.W h ic h o r g a n d o e s n o t u t i l is e k e t o n e b o d ie s :
D. Coenyzme A □
A. Liver (Hepatocyte) (NBE P 15,14; AIIMS 08,13) □
93. All are true about p-oxidation except: (AIIMS 13,11)
B. Brain □
A. Activation of FA is a must □
C. Skeletal muscles □
B. Carnitine palmitoyl transferase I is present in outer
D. Cardiac muscles □
mitochondrial membrane □
103.K e t o n e b o d i e s c a n n o t b e u t iliz e d b y : (AIIMS 13,15)
C. Carnitine palmitoyl trasferase II is present in
A. Heart □
mitochondrial matrix □
B. Brain □
D. FA activation requires 2ATP. □
C. Skeletal muscles □
94. p-oxidation of odd-chain fatty acids produces:
D. R BC s □
A . Succinyl CoA. (AI 03) □
104.I n a w e l l f e d s t a t e , a c e t y l C o A o b t a in e d f r o m d ie t is
B. Propionyl CoA □
l e a s t u s e d i n t h e s y n t h e s i s o f: (AI 02)
C. Acetyl CoA □
A. Palmity CoA □
D . Malonyl CoA □
B. Citrate □
95. How many ATP's are formed in case of stearic acid by
C. Acetoacetate □
P-oxidation- (AIIMS 14; NBE P 13)
D. Oxalosuccinate □
A. 7 □
105.1n a w e l l f e d s t a t e , t h e a c t iv it y o f C P T - I i n o u t e r
B. 18 □
m it o c h o n d r ia l m e m b r a n e i s i n h ib i t e d b y :
C. 56 □
A. Glucose (AIIMS 1 1 ) 0
D. 147 □
B. Pyruvate □
96. P-oxidation of palmitic acid yields (PGI 05)
C. Acetyl CoA □
A . 3 acetyl CoA □
D. Malonyl CoA □
B. 129 ATP net □
106.A11 o f th e f o l l o w in g s t a t e m e n t s a r e c o r r e c t a b o u t
C. 131 ATP net □ m e t a b o l i s m i n b r a in e x c e p t : (AI 03)
D. 16 Acetyl CoA □ A.Fatty acids are utilised in starvation □
E. 96 ATP from citric acid cycle □ B. 60% of total energy is utilized during resting stage □
97. Beta-Oxidation is peroxisome is differentiated from
C. Ketone bodies are used instarvation □
that occurring in mitochondria by: (PGI 03)
D. Has no stored energy □
A . Acetyl CoA □ I0 7 .1 f s t a r v a t io n e x c e e d s 7 d a y s , t h e m a jo r n u t r i t i o n a l
B. H 2O 2 formed □ s u p p ly o f t h e b r a in c o m e s f r o m (AI 04)
C. Different enzymes are found in different sites □ A. Fatty acids □
D . NADH is required □ B. Ketone bodies □
98. Refsum 's disease is due to deficiency of which of the
C. Protein breakdown □
following enzyme? (AIIMS 08)
D. Carbohydrate breakdown □
A . Malonate dehydrogease □ 1 0 8 .E n e rg y s o u r c e u s e d b y b r a in in l a t e r d a y s o f S t a r v a tio n
B. Thiophorase □ is : (AI04)
C. Succinate thiokinase □ A. Glucose □
D. Phytanic alpha oxidase □ B. Ketone bodies □
Eicosanolds C. Glycogen □
D. Fatty acids □
99. Which of the following is a suicide enzyme?
109.1n a p a t i e n t w i t h s t a r v a t io n f o r 7 2 h r s w h i c h o f th e
A. Lipoxygenase (AIIMS 13) □
f o l l o w in g w o u ld b e s e e n : (AIIMS 02)
B. Cyclooxygenase □
A. Increased glycogenolysis □
C. 5' nucleotidase □
B. Increased ketosis due to break down of fats □
D. Thromboxane synthase □ C. Degradation of proteins □
Ketogenesis (Ketone Body Formation) D. Increased gluconeogenesis □
100. All are true about ketone bodies except- (NBE P 15) 110.A l l o f th e f o l l o w in g a r e i n c r e a s e d i n s t a r v a t io n e x c e p t:
A . Lipolysis (PGI 12) □
A. Acetoacetate is primary ketone body □
B. Synthesized in mitochondria □ B. Ketogenesis □
C. Synthesized in liver C. Gluconeogenesis □

D . Glycogenesis □
D. HMG CoA reductase is the rate- limiting
E. Glycogenolysis □
enzyme
Biochemistry: Fat Metabolism ■ 455

111.All of the follow ing metabolic changes occur during 117.1n a s e r io u s ly i l l p a t i e n t , a d d it io n o f a m in o a c id s i n th e


prolonged fasting when compared with fasting for 12- d ie t r e s u l t s i n a p o s i t i v e n i t r o g e n b a l a n c e . T h e
24 hours, except: (AI 12) m e c h a n is m fo r th is is : (AIIMS 08)
A. Brain use of ketone bodies increases □ A. Increased absorption of amino acids from the
B. Liver gluoneogenesis increases □ intestine □
C. Muscle protein degradation decreases □ B. Enhanced rates of gluconeogenesis □
D. Brain use of glucose decreases □ C. Increased secretion of insulin □
112.W h i c h o f t h e f o l l o w in g m e t a b o l i c e v e n ts w i l l n o t o c c u r D. Increased Growth hormone secretion □
f o l l o w in g 1 2 -2 4 h o u r o f f a s t in g (AI 12) 118.S t a r v a t i o n a n d d ia b e t e s m e ll i t u s c a n le a d o n to
A. Increase in free fatty acids □ k e t o a c i d o s i s w h i c h o f t h e f o l l o w i n g f e a t u r e s i s in
B. Increase in ketone bodies □ f a v o u r o f k e t o a c i d o s i s d u e t o d ia b e t e s m e ll i t u s .
C. Decrease in Glycogen □ A. Increase in glucagon/insulin ratio, increased CAMP
D. Decrease in Serum Proteins □ and increased blood glucose □
113.I n s t a r v a t io n , b r a n u t i l iz e s g lu c o s e p e f e r e n t i a l l y b u t B. Decreased insulin, increased free fatty acid which is
n o t i n l iv e r b e c a u s e : (PGI 2K) equivalent to blood glucose (AIIMS 2K) □
A. Low km value of hexokinase □ C. Decreased insulin, increased free fatty acid which is
B. Low km glucokinase □ not equivalent to blood glucose □
C. High km glucokinase □ D. Elevated insulin and free fatty acid, equivalent to
D. High blood flow □ blood glucose □
1 1 4 .K e to n e b o d y f o r m a t io n w i t h o u t g l y c o s u r i a s e e n in : 1 1 9 .R o t h e r a 's te s t f o r d e t e c t io n o f : (AI 10)
A. DM (PGI 07) a A. Proteins □
B. DI □ B. Glucose □
C. Prolonged starvation □ C. Fatty acid □
D. Obesity □ D. Ketones □
115.W h i c h s u b s tr a te is/ are u s e d to p r o v id e e n e r g y f o r
120.A d e s t it u t e w o m a n i s a d m it te d to th e h o s p i t a l w i t h
a lte r e d s e n s o r i u m a n d d e h y d r a t io n ; u r in e a n a l y s i s
body:
s h o w s m il d p r o t e in u r ia a n d n o s u g a r ; w h a t o t h e r te s t
A. Ketone bodies (PGI 13) □
w o u ld b e d e s i r a b l e : (AI 01)
B. Glucose □
A. Fouchet □
C. Free fatty acids □
B. Rothera □
D. Creatine phosphate □
C. Hays □
E. Collagen □
D. Benedicts □
116.1n s t a r v a t io n , n i t r o g e n i s c a r r ie d f r o m m u s c le to liv e r
121.H a y 's s u l p h e r t e s t i s u s e d to d e te c t w h ic h o f th e
an d k id n e y b y : (PGI 07)
f o l l o w in g - (NBE P 15)
A. Alanine □
A. Bile salts in urine □
B. Aspartic acid and Serine □
B. Reducing sugar in urine □
C. Glycine □
C. Ketone bodies in urine □
D. Asparagines □
D. Urobilinogenin urine □
456 ■ A Complete Review of Short Subjects

ANSWERS & EXPLANATIONS:

Lipids of Physiological Significance

1. C i.e. Arachidonic acid contain five double bonds [Ref: Harpers 28/e P-121-23; Lippincott's 5/e p. 181-82]

- Monoenoic acids (FA with 1 unsaturated double bond) like palmitoleic, oleic and elaidic have double bond at 9th
positionQ (cis, d s & trans respectively).
- Linoleic has 2, linolenic has 3 and arachidonic acid has 4 double bonds (all are cis).

D i.e. Omega-3 fatty acids

Cardioprotective fatty acid is omega (cd) -3 fatty acids (ci>-3 PUFA)Q > monounsaturated fatty acids (like olive oil)Q

3. C, B, > A i.e. (Hydrogenated vegetable oils contains Trans fatty acid; Biologically arachidonic acid is essential to life >
Polyunsaturated FA is essential for membrane structure)
4. D i.e. Hydrogenation decreases TFA 5. A i.e. Linoleic acid 6. B i.e. Linoleic acid; C i.e. Linolenic acid
7. B i.e. Sunflower Oil 8. D i.e. Docosahexanoic Acid
[Ref: Lippincott's 6/e p. 182; Park 16/e p.407; Lehninger 5lhje p. 345, 815-16; Harper 28/e p. 122; Vasudevan 6je p.437]

M ost of the naturally occurring unsaturated fatty acids are cis-isomers. Trans-FA do not occur naturally in plants and
occurs only in small amounts in animals as ruminant fat (eg butter fat) arising from the action of micro organisms in
rumen. Cis isomers are less stable than trans FA.
Trans fatty acids formed during partial hydrogenation or hardening (saturation /deep frying)^ of vegetable oils - a
procedure to improve stability or shelf life. Trans - FA increase incidence of coronary heart disease by increasing
LDL and decreasing HDLQ.
Dietary essential fatty acids in humans are linoleic acid and a - linolenic acidQ, because we lack enzymes needed to
synthesize them. Plants provide these EFA. Linoleic acid is the precursor of a>-6-arachidonic acid and so
prostaglandins. So the arachidonic acid becomes EFA if linoleic acid is deficient in diet, a-linolenic acid is precursor
of co-3-fatty acids required for growth and development.
Linoelic acid is the most important essential fatty acid because it serves as the basis for production of other EFAs like
arachidonic acid & docosahexanoic acid. Arachidonic acid, a w-6 PUFA is essential as it is a precursor of
prostaglandins & leukotrienes.
Linoleic acid is the most important essential fatty acid, maximally found in Safflow er oilQ>Com oilQ> Sunflow er oilQ
Docosahexanoic Acid (DHA; 0)3,22:6) which is synthesized from a-linolenic acid or obtained directly from fish oils, is
present in high concentrations in retina, cerebral cortex, testis & sperm. DHA is particularly needed for development of
brain & retina and is supplied via the placenta and m ilk. Milk is the constant and continuous source of DHA as
placenta is not present at all time. Patients with Retinitis pigmentosa are reported to have low levels of DHA.
Plasma membrane contains both saturated & mono or poly unsaturated FA.

9. D i.e. Alpha linolenic acid [Ref: Harper's 27Ie P-123; 26/e P - 113,191]
Alpha linolenic acid belongs to W3 seriesQ. Rest others belong to W6 series.
Carbon atoms are numbered from the carboxyl carbon (carbon no. 1). The carbon atoms adjacent to carboxyl carbon are
also called, a, (3, Y respectively. The terminal methyl carbon is K/as W or n- carbon.

W3 Family W6 Family W7 Family W9


- Alpha linolenic Q - Gamma linolenicQ Palmitoleic - OleicQ
- Timnodonic - ArchidonicQ - Elaidic
- Cervonic - LinolicQ

The first double bond introduced into a saturated fatty acid (to make it unsaturated) is nearly always in A9 position.
The reaction is catalyzed by A9 desaturase in presence of O 2, NADH and cytochrome b5.
Since animals have A9 desaturase, they are able to synthesize W9 (oleic) family.
Linoleic (W6) or a - linolenic (W3) acids required for the synthesis of other members of W6 or W3 family must be
supplied in diet.
Biochemistry: Fat Metabolism ■ 457

A, B, D i.e. Long chain fatty acid (LCFA), Alcohol- sphingosine, Sailic acid
A i.e. Cell-cell interaction, B i.e. Signal transduction; C i.e. Surfactant; D i.e. Acute inflammatory mediator, E i.e.
Activates membrane enzymes [Ref: Lehninger 5/e p. 256, 350-55; 829; Chatterjea 7/e p. 45, 58- 61;
A, C, D i.e. Glucose, Sphingosine, Fatty acids Vasudevan 6/e p. 81-82; Lippincott 5/e p. 201-2, 209-10, 217]

Glycosphingolipid=Ceramide (Sphingosine/amino alcohol+Fatty acid)+Mono/oligo saccharide like glucose, galactose


Ganglioside (acidic glycosphingolipid) = Ceramide + Oligosaccharide+NANA (N-acetylneuraminic acid or sialic acid)

-----------------------------------------------------------------------------------------------------------------------------------------------
Glyco-sphingolipid is made up of ceramide (which is a long chain fatty add attached to amino group of sphingosine
through an amide linkage i.e. = sphingosine /amino alcohol + Fatty acid) attached directly to mono /oligo
saccharide (polar head) by an O-glycosidic bond. Glycosphingolipids differ from sphingomyelin (phospho-
sphingolipid) in that they do not contain phosphate (polar group); and differ from glycerophospholipids
(phosphoglycerides) that they do not contain glycerol.
G anglioside (addic, negatively charged glyco-sphingolipid) contain sphingosine alcohol (4- sphingenine) 1
molecule, long chan fatty acid (1 mol), Oligosaccharide polar head and 1 or more residues of N- acetylneurminic
acid (Neu 5 Ac), a sialic acid (often simply called sialic acid) giving it a negative charge, at termini. Ganglioside do
n ot contain glycerol and phosphate °.
Ceramide containing sphingosine amino alcohol is present in all sphingolipids (ie. phospho & glyco­
sphingolipids)0 .

A i.e. Ganglioside, B i.e. Sphingomyelin, C i.e. CerabrosidesfRe/: Lippincott's 5/e P-211; Harper's 28/e P-208-9]

Ceramide (i.e. sphingosine amino alcohol + fatty add) is synthesized in endoplasm ic reticulum (E R )from amino acid
serineQ. It is an important second messenger (signaling molecule) in apoptosis (programmed cell death), cell cycle, cell
differentiation and senescence. Ceramide is present in sphingomyelin, gluco/galacto-cerebrosides, globoside and ganglio
side.

B i.e. Alpha 2 - macroglobulin [Ref. Harper 28/e p. 216,232; 575; Vasudevan 6/e p. 292-300, 335]

Atherosclerosis is a/w increased LDL (or oxidized LDL), Lp (a) & homocysteine and decreased or defective apo B- ‘
100 & E°.
Alpha-2 (a-2) macroglobulin, a large plasma glycoprotein (720 kDa) containing Zn (zinc) and a unique internal cyclic
thiol ester bond (formed between cysteine & glutamine), is a panproteinase inhibitor (i.e. neutralize many proteases)
and target certain cytokines (like platelet-derived growth factor, transforming growth factor P etc) towards particular
tissues or cells. It is not a predisposing factor for atherosclerosis0 .

B i.e. Cardiolipin

Cardiolipin is only human glycero-phospho lipid w hich is antigenic0 . It is antigenic and is recognized by antibodies
raised against Treponema pallidum (syhillis bacteria)0 .

Acylglycerol & Sphingolipid Metabolism

B i.e. Triglycerides

Triglycerides = Triacy 1-glycerol = 3 Fatty Acids + Glycerol


Triglycerides (or TAG, acyl = fatty acid) is main storage form of energy. They are stored in fat cells (adipocytes) of adipose
tissue.

C i.e. Mutation of genes encoding lysosomal hydrolases [Ref: Harrison 17/e p- 2452-53; Harper's 27/e P-541]

Lysosomal storage diseases (LSD) are caused by mutations in genes encoding lysosom al hydrolases ° responsible for
breaking down of macromolecules, which results in accumulation of corresponsing substrate (macromolecule). LSDs are
classified on the basis of accumulated substrate into sphingolipidosis, mucolipidosis, mucopolysaccharidoses,
glycoproteinoses etc.
458 ■ A Complete Review of Short Subjects

18. D i.e . W o lm a n d is e a s e [Ref: Harper 29/e p. 235, 233; Lippincott's 6/e p. 212, 210; Vasudevan 7/e p. 193, 91-93; Chatterjea 8/e p. 11, 50, 63, 4241

Sphingolipidoses include (1) Neutral glycosphingo lipidoses (eg Fabry's, Gaucher's and Niemann Pick disease A
and B); (2) Generalized GM1 gangliosidoses; (3) GM 2 gangliosidoses (eg Tay Sachs & San d h o ffs diseases); (4)
Leukodystrophies (eg Krabbe globoid cell leukodystrophy, metachromatic leukodystrophy and multiple sulfatase
deficiency) (5) Lactosyl ceramidoses and; (6) Farber's disease.
W olman's disease and cholesteryl ester storage diseases are not sphingolipidoses as these are a/w accumulation of
triglycerides & cholesterol esters (not sphingolipid/ ceramide).

19. B, C, E i.e. Due to deficiency of enzyme Beta-Gluco-cerebrosidase, Deposition of glucosylceramide, PAS staining
positive [Ref: Harper 28/e p 210; Harrisonl7/e p- 2452-54; Robbins 7/e p 163-164; Vasudevan 6/e p.168-69; Chatterjea 7/e p 58-59;
Lippincott 5/e p 212; Lehninger 5/ 356]

Gaucher's disease is due to deficiency of Beta glucocerebrosidase /acid P- glucosidaseQ. This AR (autosomal
recessive) disorder results in accumulation of glucosyl ceramide (gluco cerebroside) usually kerasin within
phagocytic macrophages known as gaucher cells. These reticulo endothelial cells are found in liver, spleen, bone-
marrow, lymph node, tonsils, thymus, payer patches, and brain.
Foam or vacuolated cells occur in Niemann Pick disease^, Fucosidosis and (1-mannosidosis etc. But in contrast to lipid
storage diseases Gaucher cells are very rarely vacuolated and instead have a crumpled (or wrinkled) tissue paper
appearance of cytoplasm. The Gaucher cells are intense PAS (Periodic acid Schiff) staining positive and show
fibrillary crumpled cytoplasm d/t deposition of (glucocerebroside) lipid as bilayered stacks in enlarged & elongated
lysosomes. Shell like inclusion bodies are seen Tay sachs diselase.

All sphingolipidoses are AR except Fabry disease which is X-linked; and all can be fatal in early life. Prenatal diagnosis
can be made using cultured amniocytes or chorionic villi.

20. A i.e. Niemann-Pick disease 21. B i.e. P-glucosidase 22. B i.e. Krabbe's disease
[Ref: Harrison 17/e P 2452-57; Lippincott's 5/e P-208-217; Harper’s 28/e P-210, Chatterjea 7/e p. 60, Harrison 16/e p. 2316-17]

- Niemann-Pick (A+B) disease d/t deficiency of sphingomyelinase^ results in defective degradation of



phosphosphingolipids (i.e. spingomyelin) and accumulation of sphingomyelin in phagocytic cellsQ.
- Sphingolipidoses occurring d/t defective degradation of glyco-sphingolipid include - Gaucher disease (deficient (1
glucosidaseQ/ glucocerebrosidase); Fabry's disease (a-galactosidaseQ); Tay-Sach's disease (f)- Hexominidase AQ);
San d h o ffs disease (^-Hexominidase A&B)Q; Krabbe's disease (f) galactosidase/ galactosyl ceramidase def)Q; Farber's
disease (acid ceramidase)Q; Metachromatic leukodystrophy (aryl sulfatase AQ) and G M i, gangliosidase (f)
galactosidase defQ).
- Krabbe's disease has no hepatosplenomegalyQ.

23. B i.e. Beta galactocerebrosidase 24. A i.e. Sphingomyelinase


25. A i.e. Tay- sach's disease; B i.e. Gaucher's disease [Ref: Harper's 27/e P-215; Lippincott's 3/e P-210]

L y so so m a l D e f ic ie n t E n zy m e G ly c o g e n S to ra g e D e f ic ie n t E n z y m e O th e r D is e a s e s D e fe c t
S to ra g e D is e a s e D is o rd e r
Mutation in CF
C y stic f ib r o s is
G a u c h e r 's d is e a se P G lu co sid aseQ T y p e I (V o n G ie r k e 's G - 6- P h o s p h a ta se ^
gene located on
N ie m a n n - p ic k d s S p h in g o m y e lin a se e Q ds)
chromosome 7
F a b r y 's d s a - G a la cto sid a seQ T y p e I I (P o m p e 's ds) A cid m altaseQ
Wilson's ds Two mutant ATP
K r a b b e 's d s P - G la la cto sid a se Q T y p e I I I (F o r b e 's ds) D e b r a n c h in g en zym eQ 7B gene
T a y -s a c h ds H e x a sa m in id a s e AQ T y p e I V (A n d e rso n ds) B r a n c h in g enzym eQ Familial hyper Defective LDL
Meta chromatic A ry ls u lfa ta s e AQ T y p e V (M e A rd le ds) M u s c le cholesterolemia receptor
leucodys trophy p h o sp h o ry la s e ^
Type VI (Her's ds) L iv e r p h o sp h o ry laseQ .
Biochemistry: Fat Metabolism ■ 459

______________ Lysosomal Storage Disease______________

Lysosomal storage disorder is accumulation of specific macromolecule within cells that are normally metabolized by cells.
This is because of deficiency of one or other lysosomal enzyme.

Mucopolysaccharidoses (MPS)
Type Syndrome Enzymatic Defect Accumulated /Urinary metabolite
I Hurler or Scheie a- L- IduronidaseQ - Dermatan sulfateQ
- Hepran sulfate®
II Hunter Iduronate sulfatase
III A Sanfilippo A Heparan sulfate N- sulfatase (sulfamidase)
III B Sanfilippo B a-N- Acetyl glucosaminidase
m e Sanfilippo C Acetyl CoA Heparan sulfateQ
- a-glucosaminidase
- N-acetyl transferase
HID Sanfilippo D N- Acetylglucosamine 6 sulfatase
IV A M oriquio- A N-acetyl Galactosamine -6 - sulfatase - Keratan sulfate
- Chondroitin -6 sulfate
Morquio -B P- Galactosidase - Keratan sulfate
i v b

VI Maroteauxlamy N- acetyl galactosamine 4-sulfatase (aryl Dermatan sulfate


sulfatase P
VII Sly P* Glucronidase - Dermatan sulfate
- Heparan sulfate
- Chondroitin 4/6- sulfate
Clinical features of MPS: Coarse facies. Valvular heart disease, mental retardation, hepatosplenomegaly, skeletal
deformity, joint stiffness, corneal clouding, & vacuolated / granulated lymphocytes.

Mucolipidoses (MLS)

Disorder Enzyme Accumulated Clinical features


defect
I Sialidosis Sialidase (neuraminidase) Glycoprotein fragments MPS II phenotype - coarse
Sialyloligosaccharides facies, pebbly skin lesions,
joint stiffness
II I- Cell disease UDP- N- acetyl glucosamine: Glycoprotein fragments Coarse facies, gingival
glucoprotein- N- acetylgluco Glycolipids hypoplasia, absence of
samininylphospho transferase mucopolysacchariduria
(Acid hydrolases thus lack
phosphomannosyl residues)
III Phosphotransferase As with ML-II but deficiency is Glycoprotein fragments Coarse facies, stiffness of
incomplete Glycolipids hands & shoulder

Glycoproteinoses

Fucosidosis a -Fucosidase Glycopeptides, oligosaccharides


M annosidosis a , P Mannosidase a/ P Oligosaccharides
Aspartylglucosaminuria Aspartylglucosaminidase Aspartyl glucosamine, glycopeptides
Schindler's disease a-N - galatosaminidase

★ Clinical features - Coarse facies, enlarged tongue, angiokeratomas


460 ■ A Complete Review of Short Subjects

Neutral glycosphingolipidoses

Disorder Fabry disease Gaucher disease Nieman-Pick disease A&B


Deficient a- Galactosidase A® Acid-3- Sphingomyelinase®
enzyme glucosidase/glucocerebrosid
ase®
Stored Globotriacyl ceramide Glucosyl ceramide Sphingomyelin®
material
Clinical - Cutaneous angiokeratoma (i.e. - No CNS and - Pulmonary infiltrates & lung
features reddish purple skin rash)®, comeal opthalmological failure
dystrophy involvement in type 1 - Foam cells in bone marrow®
- Painful acroparesthesis i.e. burning (adult) form - Pleomorphic inclusion of lipid in
pain in lower extremity)® - PAS staining positive lysosomes concentric parallel
- Hypohydrosis, kidney & heart failure. Gaucher cells® in bone lamella (on electron microscopy =
- X- linked disorder, onset in childhood, marrow, cytopenia EM
- No hepatospleenomegaly, skeletal - Lipid as bilayered stacks in - Mental retardation, seizures,
disease and hematological elongated distended hepato spleenomegaly, macular
involvement lysosomes on EM degeneration
- Osteoporosis in type B only

Leukodystrophies

Disorder Enzyme defect Accumulated lipid Clinical Features


Krabbe's disease (globoid Galactosyl Galactosylceramide - Globoid bodies (glycolipid- laden
cell leukodystrophy) ceramidase/1) Galactosyl sphingosine macrophages) in white matter
galactosidase® - Mental & motor retardation
- Blindness & deafness
Metachromatic Arylsulfatase A® Cerebroside sulfate - Mental Retardation
leukodystrophy - Progressive paralysis
- Dementia in adult form
- Nerve stain yellowish brown with
cresyl violet (metachromasia)
M ultiple sulfatase Aryl sulfatases A, B & Sulfogalactosyl ceramide - Absent activity of all cellular
deficiency C and steroid (sulfatide), steroid sulfates, sulfatases
(Active site cysteine to C a sulfatase proteoglycans mucopoly - MR, HS omegaly, retinal
formyl glycine converting saccharides degeneration, & skeleletal
enzyme) involvement

GM 2 gangliosidoses

Tay Sach's (J- Hexosaminidase A® (HAD- GM 2 gangliosides Mental retardation, seizures, macrocephaly,
disease A) hyperacusis and cherry red spot in both®.
Sandhoff's P- Hexosaminidase A and B GM 2 gangliosides Hepatosplenomegaly & skeletal involvement
disease [HAD- A,B] only in sandhoff ds.

Disorder of neutral lipids

Wolman disease Acid lysosomal lipase Cholesterol esters, triglycerides Adrenal calcification,
hepatospleenomegaly, mild, mental
retardation
Cholestery ester Acid lysosomal lipase Cholesteryl ester Hepatomegaly, cirrhosis
storage disease
Farber disease Acid ceramidase Ceramide Macular degeneration, arthropathy,
subcutaneous nodules
Biochemistry: Fat Metabolism ■ 461

A i.e. Gauchers disease

Enzyme replacement therapy is available for some lysosomal storage disorders like (1) Gaucher's disease® (1st), (2)
Fabry's ds (3) Glycogen storage ds type II (4) Mucopolysaccharidosis I, II and VI (i.e. Hurler, Hunter and VI).

Lipid Transport

D i.e. Chylomicrons [Ref: Lippincott 5/e p-227-28; Chatterjea 7/e p 419-18; Harper 28/e p 213]

Chylomicron does not migrate (or move) towards charged end or anode on electrophoresis and remains at the origin ®. The
increasing order of mobility on electrophoresis is: Chylomicron < LDL (fi-regoin) < VLDL (pre fi- region) < HDL (oe-region)

B i.e. Chylomicrons [Ref: Lippincott's 5/e P-227-28] 29. A i.e. Chylomicrons

Lipoproteins may be differentiated on the basis o f their lipid & protein compositions. The lipoproteins containing the
largest percentage o f triacylglycerol are Chylomicrons (90%). Maximum content o f triglycerides (& exogenous TG) is
present in chylomicrons which carries exogenous dietary TG's to peripheral tissues ®.

Triglycerides (TG) are manly found in - chylom icron s & Chylmicrons: are found in chyle formed only by the lymphatic
VLDL®• system draining the intestine. They are responsible for transport
of all dietary lipids into circulation.

VLDL: transport TG from the liver to extrahepatic tissue.

★ The on ly ap olip op rotein , th a t rem ains a s so cia ted w ith


chylom icron s alon g its entire jou rn ey (from intestine to
liver) is ap o -B 4 8 )fi
B i.e. Arginine

Arginine rich apo-E (apoprotein-E) is found in VLDL®.

C i.e. Transports maximum amount of lipid 32. B i.e. LDL 33. B i.e. LDL 34. A i.e. HDL
B i.e. Kidney 36. A i.e. HDL [Ref: Harper's 27je P-218; 26/e P-206, Shinde & Chatterjea: 6/e P- 3861

OR GIN Chylomicron Chylomicron (float)

LDL (p) VLDL

Mobility VLDL (pre p ) LDL

HDL (a) HDL


Anode (+) (Settle in bottom)

Electrophoretic separation Ultracentrifugation (Density based)


462 A Complete Review of Short Subjects

Features of Lipoproteins

The density of lipoprotein decreases as the proportion of lipid to protein increases.

Composition
Diameter Density Protein Lipid Main Lipid
Lipoprotein Source (nm) (g/mL) (%) (%) Composition Apolipoprotiens
Chylomicrons Intestine® 90-1000 <0.95 1-2 98-99 Triacylglycerol A-I, A-II, A-IV1, B-
48, C-I, C-II, C-III, E
Chylomicron Chylomicrons 45-150 <1.006 6-8 92-94 Triacylglycerol, B-48, E
remnants phospholipids,
cholesterol
VLDL Liver® 30-90 0.95-1.006 7-10 90-93 Triacylglycerol B-100, C-I, C-II, C-
III
IDL VLDL 25-35 1.006-1.019 11 89 Triacylglycerol, B-100, E
cholesterol
LDL VLDL® 20-25 10.19-1.063 21 79 Cholesterol B-100
HDL Liver,
HDL, intestine, 20-25 1.019-1.063 32 68 A-I, A-II, A-IV, C-I,
h d l2 VLDL, 10-20 1.063-1.125 33 67 Phospholipids®, C-II, C-III, D3, E
HDL3 chylomicrons 5-10 1.125-1.210 57 43 cholesterol
PreP-HDL3 <5 >1.210 A -l
Albumin /free Adipose tissue > 1.281 99 1 Free Fatty acids
fatty acids
Abbreviations : HDL, high-density lipoproteins; • Diameter and Lipid Content: decreases from upward to
IDL, intermediate-density lipoproteins; LDL, low- downward i.e
density lipoproteins; VLDL, very low density - Chylomicron has highest diameter and highest lipid
lipoproteins. content
1 Secreted with chylomicrons but transfers to HDL - HDL has lowest diameter & has least lipid content.
2 Associated with HDL2 and HDL3 subfractions • Density and protein content: Increases on going downward
3 Part of a minor fraction known as very high i.e.
density lipoproteins (VHDL). - HDL has highest density & has max. protein content.
Chylomicron has least density®.

Chylomicrons VLDL LDL HLD


■Maximum content HDL • Lipoprotein >Are produced • Contain high • Synthesized in
of TG,s is seen in I particles with in the liver®. concentration liver & are
—» chylomicron® ■HDL are synthesized in - Lowest density® >Are composed of cholestrol released into
Maximum content the liver®. predominantly & cholesterol blood by
- Largest size®
of exogenous TG's ■This is highest density® of TG (also esters (bad exocytosis.
- Maximum
cholesterol)®
- » chylomicrons lipoprotein (because amount of lipid cholesterol • Remove free
•Maximum content ester & • Provide cholesterol from
least lipid content) • Are assembled
cholesterol) cholesterol to extrahepatic
of endogenous ■It has highest in intestinal the peripheral tissue & esterify
TG's is in —> electrophoretic mucosal cells®. •Carry
tissue®. it.
VLDL® mobility® endogenous
•Carry exogenous • LDL is
TG from liver • Esterified
■Maximum ■Apoprotein associated dietary TG's to causative for
to peripheral cholesterol is
cholesterol is in —» with HDL are APO A, peripheral coronary
tissue®. carried to liver
LDL®. APOCII, & APOE®. tissues®. disease®. where HDL is
degraded &
cholesterol is
removed.
• Protective in
coronary disease
(scavenging
action)®
Biochemistry: Fat Metabolism « 463

37. C i.e . H D L 38. C i.e . A p o p r o t e in s 39. B i.e . H D L 40. A i.e . H D L


I Ref: Lippincott's 4/e p 242- 43; Harper 30/e p. 254; Lehninger 5/e p. 836-37] .

' -----------------------------------------------------------------------------------
HDL contains g o o d c h o le s t e r o l^ . So there is an inverse relation between H D L ( H D L 2) concentrations and coronoary
heart disease. This is consistent with the f u n c t i o n o f H D L i n r e v e r s e c h o l e s t e r o l t r a n s p o r t ( i.e . f r o m p e r ip h e r a l t is s u e
to liv e r)Q .

Diseases (eg. diabetes mellitus, lipid nephrosis, hypothyroidism and other considtions of hyperlipedimea) in which
prolonged elevations of VLDL, IDL, LCD or chylomicron remnants in blood are often accompanied by premature or
severe atherosclerosis. This makes L D L : H D L c h o le s t e r o l r a t io a good predective parameter for atherosclerosis &
coronary heart disease.

HDL has h i g h e s t d e n s it y . It has h i g h e s t p r o p o r t io n o f a p o p r o t e in s ( m a i n l y A p o A s ) a n d p h o s p h o l i p i d s ^ . And HDL


m ig r a t e s l e a s t d u r in g e l e c t r o p h o r e s i s .

41. A i.e . R N A e d it i n g [Ref: Harper's 27 jc P-218] 42. A i .e . A p o p r o t e in B 43. C i.e . B -4 8

44. A i.e . V L D L 45. B i .e . L iv e r

V L D L i s f o r m e d i n l i v e r fr o m th e t r i g ly c e r id e s m a in ly . E n d o g e n o u s t r i g ly c e r id e s in p la s m a a r e m a x im m a ly c a r r ie d in
VLD LQ .

46. D i.e . A p o A I 47. C i .e . A p o p r o t e in C I I i n h i b i t s lip o p r o t e in lip a s e


iRef: Harper's 28/e p. 212-15; Chattergea 7/e p. 419-21; Devlin 5/e p. 712; Lippincott’s 100,177,189, 227 - 34]

L ip o p r o t e i n l ip a s e is a c t iv a te d b y p h o s p h o l i p i d , a p o p r o t e i n C I & C II a n d i n h i b i t e d b y a p o p r o t e in A l l a n d a p o p r o t e i r
CIIIQ.

48. B i.e . L D L / H D L r a t io 49. B i.e . L D L 50. D i .e . It c o n t a in s A p o - B 4


Ref: Harper 28th/ed pg-216; Lehninger 5lh/ed pg- 843; journal of American college of nutrition, Vol 27, No-1, 1-5 (2008) Chattarjea
7lh/ed pg. - 374-75; Lippincott 4,h/ed pg-234-31

LDL has h i g h e s t c h o l e s t e r o l a n d c h o l e s t e r y l e s t e r c o n t e n t ^ and possesses o n l y a p o B -1 0 0 a s t h e i r m a jo r a p o p r o t e in ^ . Its


So it is
p r im a r y f u n c t i o n i s to p r o v id e c h o l e s t e r o l ( w h e t h e r d ie t a r y o r e n d o g e n o u s ) to p e r ip h e r a l ( e x t r h e p a tic ) tis s u e ^ .
m o s t p o t e n t a t h e r o g e n ic lip o p r o t e in , i n c r e a s i n g t h e r i s k o f c a r d io v a s c u la r d is e a s e ^ and showing a positive correlation
between the incidence and plasma concentration of LDL cholesterol.

51. A i.e . P r e s e n t o n ly i n th e e x t r a - h e p a tic t is s u e s : IRef: Harper29/e p 242, 255-57; Lehninger 5/e p 837-41; Lippincott 5/e p 233-31]
52. C i.e . O n a c c u m u la t io n i n t h e c e l l i n h i b i t s r e p le n i s h m e n t o f L D L r e c e p to r s .
53. C i.e . In c r e a s e d b lo o d c h o l e s t e r o l is a s s o c ia t e d w ith i n c r e a s e d L D L r e c e p to r s 54. C i.e . A p o E a n d A p o B 1 0 0
s 7
- L iv e r a n d m a n y e x tr a h e p a t i c t is s u e s e x p r e s s L D L (a p o - B - 1 0 0 a n d E ) r e c e p to r s ^ . LDL re c e p to r s a r e p r e s e n t o n c e l l '"‘v
m e m b r a n e p i t s c o a te d w i t h c l a t h r i n ( p r o te in ) o n t h e c y t o s o l ic sid e Q ; a n d t a k e n in t o th e c e l l b y e n d o c y t o s is ^ (a fte r
b in d in g LDL). A n d c h o l e s t e r o l p r e s e n t i n L D L , o n a c c u m u la tio n in th e c e ll i n h i b i t s r e p le n i s h m e n t ( r e c r u itm e n t i.e .
t r a n s c r ip t io n ) o f L D L r e c e p to r s v i a S R E B P p a th w a y ^ .
- I n c r e a s e d c e l l u l a r c h o l e s t e r o l c o o r d in a t e ly s u p p r e s s c h o l e s t e r o l s y n t h e s i s (b y in h ib itin g H M G C o A s y n th a s e , H M G
C o A r e d u c t a s e e tc e n z y m e s ) a s w e l l a s u p t a k e ( b y d o w n r e g u la t i n g s y n t h e s i s o f L D L re c e p to rs )Q ; w h e r e a s c h o le s te r o l
e s t e r if ic a tio n is p r o m o te d (b y s t im u la tin g A C A T a c tiv ity ). T h u s th e LDL r e c e p to r a c t iv it y o n c e ll s u r f a c e is r e g u la te d b y
c h o le s te r o l r e q u ir e m e n t f o r m e m b r a n e s , s te r o id h o r m o n e s o r b ile a c id s y n th e s is .
464 ■ A Complete Review of Short Subjects

55. D i .e . D o e s n o t r e q u i r e C I I a s c o f a c t o r [R ef: L ip p in cott 4 ,h/e p 2 2 8 -2 9 ; L eh n in g er 5"'/ep 8 3 7 -3 9 ; 346, 649; H a rp er 2 8 u'/e p i 3 4 -3 5 ,2 1 5 -1 6 , 23 2 ]

In the capillaries of adipose, heart, skeletal muscle and lactating mammary tissues^, lipoprotein lipase is activated
by apo C I I aproprotein of chylomicron and VLDLQ and the enzyme degrades the triacyl glycerol to yield fatty acid
and glycerol.
Apo CII is essential for activation whereas apo AII and C III inhibits the lipoprotein lipase enzymeQ. So the patients
with a deficient lipoprotein lipase or apo CII (i.e. Type I hyperlipoproteinemia or Familial lipoprotein lipase
deficiency) show a very high (> 1000 mg/dl) accumulation of chylomicrons containing triacyl glycerol in plasma i.e.
hypertriacylglycerolemia even in fasting stateQ.

56. A i .e . H D L [Harper's 27je P-221-223; 26/e P-209, 210; Chatterjea 6/e P- 386]

HDL scavanges body cholesterol and blood vessel wall cholesterol by 'reverse cholesterol transport'-[Shinde P- 386]
The transport of cholesterol from the tissue to liver is known as 'reverse cholesterol transport'.

★ Class B scavenger receptor B I (SR -B I ) and ATP binding cassette transporter A I (ABCA 1) are involved in reverse
cholesterol transport^.
★ Reverse cholesterol transport involves efflux of cholesterol from peripheral cells to HDL, esterification of cholesterol by
PCAT (phosphatidyl choline: cholesterol acyl transferase), binding of cholesterol ester rich HDL (HDL-2) to liver and
steroidogenic cells, the selective transfer of the cholesteryl esters into these cells and the release of lipid depleted HDL
(HDL-3).
★ The efflux of cholesterol from peripheral cells is mediated by ABCA-1 and the uptake of cholesteryl ester by liver is
mediated by SRB1. Hepatic lipase degrade both triacyl glycerol and phospholipid, & participate in conversion of H D L 2
to H D L 3 , (HDL cycle). HDL2 concentrations are inversely related to incidence of coronary atherosclerosis. HDLc (HDLi)
is found in diet induced hyper cholesterolemic animals. It is rich in cholesterol & has only apo E apoprotein.
57. A i.e. Tendon Xanthoma 58. D i.e. LDL receptor mutation 59. D i.e. Chylomicrons
60. A i.e. LDL receptor 61. C i.e. Ill

63. B i.e. VLDL [Ref: Harper's 27/e P-219-220; 26/e P-211-12] 64. A i.e. VLDL
As VLDL is the main transporter o f endogenous triglycerides; VLDL derives Triglycerides from Two sources
all the conditions causing increased endogenous triglyceride I I
synthesis will also increase VLDL secretion. And these are- From hepatic synthesis of F.A From free fatty acids
, .. (Liver synthesize TG's from in circulation
- Well fed state (not starved) , __
acetyl COA, derived mainly |
- High level of circulating free fatty acids
- Carbohydrate rich diet (especially sucrose & fructose)® from carbohydrate) In Fat rich diet,
I Starvation, Diabetes
- Presence of high level of insulin & low level of glycogen
Ethanol (alcohol) ingestion weH fed state, mellitus
Carbohydrate rich diet
Biochemistry: Fat Metabolism 465

65. C i .e . L P (a) 66. B i.e. Atherosclerosis [Ref: Harper 27je p. 239; Lippincott's 4/e p. 237; Robbins 7/e p. 521]

Lipoprotein (a) /Lipoprtein little (a) /Lp(a) is associated with coronary heart disease due to atherosclerosis plus
thrombosis d/t inhibition of fibrinolysisQ.

Lipoprotein (a)
1---------------------------
Structure Circulating levels Excess cause
It is identical to LDL. Its distinguishing feature is the - Determined by genetics - Coronary heart disease
presence of an additional apolipoprotein molecule, - Diet rich in trans fatty acid due to atherosclerosis^
apo (a), that is covalently linked to apo B -1 0 0 . So increase Lp (a) - Thrombosis d/t inhibition
- Estrogen decrease both of fibrinolysis
Lp (a) = LDL + Apo (a)
LDL & Lp (a)
Apo (a) is structurally homologous to plasminogen -
the precursor of blood protease whose target is fibrin
the main component of blood clot.

Cholesterol Synthesis & Excretion

67. A i.e. HMG COA reductase 68. C i.e. HMG-Co-A Synthase


69. A i.e. Steroid 70. D i.e. Palmitic acid; E i.e. Stearic acid
[R ef: L eh n in g er 5 /e p 3 5 5 -6 0 , 8 3 1 -3 5 ; V asu d ev an 6/e p 147 -5 2 ; C h aterjea 8 /e p 4 3 1 -3 5 ; L ip p in cott 5 /e p 1 9 6 -9 8 ; H a rp e r 2 9 /e p 2 1 0 -1 3 , 251-551

Both cholesterol and ketone bodies are synthesize from (3) beta ☆ 2 Acetyl CoA |
hydroxyl (3) beta m ethyl glutary CoA (ie HM G CoA). So enzymes,
( Thiolase)
thiolase & HMG CoA synthase involved in conversion o f acetyl
CoA to HM G CoA are common to synthesis o f cholesterol and | Acetoacetyl CoA 1
ketone bodies bothQ. However, cytoplasmic HM G CoA synthase
is involved in cholesterol synthesis, whereas mitochondrial HMG Acetyl CoA ( HMG CoA synthasel
CoA synthase is involved in Ketone body synthesis.
Cholesterol synthesis is controlled by regulation of HMG COA HMG CoA
reductase, which is a rate lim iting enzymeQ.
Cholesterol, the major sterol in animal tissue has steroid nucleus^. Cholesterol Ketone bodies
(by cytoplasmic (In mitichondrla)
Of the cholesterol absorbed, 80-90% is esterified with long chain & ER enzyme)
fatty acids (mostly saturated) such as palm itic and steric acidsQ.
Whereas, substitution of saturated FAs by poly unsaturated FAs in
diet has beneficial effects and lowers cholesterol.

71. E i.e. Cholesterol restricted diet, high fiber diet with low saturated FA ... 72. C i.e. HMG COA reductase
73. B i.e. Cholesterol 74. B i.e Cholic acid; C i.e. Chemodeoxycholic acid. E i.e. Taurocholic acid
[Ref: Harper's 27/e P-230,236;26je P-218; Chatterjea 6/e P-374; KDT 5/e P-577]
Formation of gluco-corticoids (GC)
iiT ^ r
Choleserol is the precursor o f Bile and mineralo-cortcoids
Steroid synthesis Formation of androgens
acidsQ, Corticosteroids, sex
Formation of estrogens
hormones and vitamin D. Adrenal cortex
Formation of progesterone
Gonads

Small 80 to 90%
CO.* quantity Cholesterol h ■Cholic acid and chenodeoxycholic
Major pathway (Liver)
acid (Bile acids)

CoA-SH ATP
Cysteine
10% Neutral sterols T
’ 7-dehydro- <
cholesterol Intestine Glycine
Gholyl-CoA
Taurine J
Vitamin D3 > Coprosterol ■ Glycocholic • Tauro-cholic
(coprostanol) acid (3 parts) acid (one part)

Fate of Cholesterol in the Body


466 ■ A Complete Review of Short Subjects

75. C i.e . P r e g n e n o lo n e I Ref: Harper 30/e p. 5031

A ll m a m m a lia n s t e r o id h o r m o n e s a r e s y n t h e s i z e d f r o m c h o l e s t e r o l v ia a c o m m o n in t e r m e d ia t e p r e g n e n o lo n e ^ th ro u g h
a s e r ie s o f r e a c tio n s e ith e r in m ito c h o n d r ia o r E R .

Fatty Acid Synthesis

76. D i.e . F a tty a c id s y n t h e s i s / E M P 77. D i.e . F a tty a c id b i o s y n t h e s i s


78. C i.e . F a tty a c id s y n t h e s i s 79. A i.e . F a tty a c id s y n t h e s i s 80. D i.e . N A D P H 81. D i.e . H M P p a th w a y
[Ref: Harper’s 29/e P-217-24; Lippincott 5/e p. 184-186,194; Chatterjea 8/e p. 414-419]
'-------------------------------------------------------------------------------- j
- De n o v o s y n t h e s i s o f f a tt y a c id s ( L ip o g e n e s is ) o c c u r s ^ 3-Ketoacyl Reductase
i n t h e c y to so lQ . 3 Ketoacyl enzy me — ►.3-Hydrooxyacyl enzyme
- I n f a t t y a c id s y n t h e s i s e n e r g y i s s u p p lie d b y N A D P H .
A n d t h e m a in s o u r c e o f N A D P H f o r l i p o g e n e s i s is NAlDPH + H+ NADP+
p e n to s e p h o s p h a te p a th w a y (o r h e x o s e
m o n o p h o s p h a te / H M P p a th w a y ). N A D P H is u s e d as Enoyl reductase
d o n o r o f r e d u c in g e q u iv a le n ts in b o t h th e r e d u c tio n o f 2, 3 unsaturated acyl ^ Acyl enzyme
3 - k e to -a c y l a n d 2 ,3 - u n s a tu r a te d a c y l d e r iv a tiv e s in enzyme
fa ty a c id s y n th e s is . NADP H + H+ NA DP+

S i t e s o f B io c h e m i c a l R e a c t i o n N A D P H : U se & So u rces

□ C y t o p la s m □ Mitochondria U ses So u rces

- G ly c o ly s is Q - TCA or K reb's or Citric NADPH is used as an electron - H M P p a th w a y Q :

- H M P s h u n tQ acid cycleQ donor in many reductive syntheses Main source of


- Oxidative in the body. Examples are: NADPH
- F a tty a c id
decarboxylation (ATP 1. Extra-mitochondrial de-Novo - M a lic e n z y m e /
s y n th e s is Q
f a tt y a c id s y n th e s is Q M a la t e
- P r o t e in s y n th e s is Q production^
2. C h o le s t e r o l s y n th e s is Q d eh y d ro g en aseQ
- P a rt o f u r e a cy cleQ - Fatty acid (fl) oxidationQ
3 . S t e r o id s y n th e s s is Q : Malate is
- P art o f - Part of urea cycleQ 4 . S p h i n i g o l i p i d s y n th e s is Q oxidatively
g lu c o n e o g e n s is Q - KetogenesisQ 5. C o n v e r s io n o f o x id iz e d decarboxylated
- Part of gluconeogenesisQ g lu t a t h io n e to r e d u c e d to pyruvate and
□ G o l g i a p p a r a tu s □ Nucleus g lu ta th io n e Q NADPH is
DNA & RNA synthesis 6 . Conversion of P h e n y l a l a n i n e to produced.
Synthesis &
ty ro s in e Q - I s o c itr a te
packaging of □ Lysosomes 7. Conversion of m e t- H b to H bQ d eh y d ro g en aseQ
complexmolecules
Degradation of complex 8 . U r o n ic a c id p a th w a y Q
including
molecule
glycolipids,
glycoproteins & □ Nucleolus
lipoproteins rRNA synthesisQ

F a tty A c id S y n t h e s i s R e q u i r e s

Site B u ild in g M u lti E n z y m e s C o m p le x C o e n z y m e s & C o fa cto rs


B lo ck M a in e n z y m e R a te lim itin g e n zy m e
Cytsol® A ce ty l F a tty acid A ce ty l C oA M n - " B e H um an "
CoA® s y n th e ta s e co m p le x carb oxylase® - BiotinQ Be
(7 enzym e)® - H C O v (S o u rce o f C O 2) Hu
- Mn2*U M
- A TP A
- NADPHU N
Biochemistry: Fat Metabolism ■ 467

82. A i.e. Acetyle CoA Carboxylase; 83. B i.e. Fatty acid synthesis 84. C i.e. Condensation reaction
85. A i.e. Freeing of PAN-SH site 86. B i.e. 4 in 1st cycle & 2 in Ilnd cycle 87. D i.e. Dehydrogenase
88. B i.e. Malonyl CoA

- The starting material for the synthesis of fatty acid is CO2 ATP ADP+Pi
acetyl CoAQ. Formation of Malonyl-CoA from Acetyl
(C2) Acetyl-CoA Biotin
Co A is the initial and controlling step in fatty acid carboxylase Q Mn*
synthesis and catalyzed by acetyl -C oA Carboxylase.
(Rate lim iting enzyme)Q. This reaction is also called
CO 2- fixation reaction.
- In condensation reaction catabolized by 3 ketoacyl synthase, there is loss of 1 molecule of CO 2.
- Saturated acyl enzyme formation causes freeing of PAN-SH site of monomer II which w ill accept another malonyl
group (malonyl CoA) in each cycle.
- 4 carbon atoms are added in first and 2 carbon atoms in successive cycles of FA synthesis.
- 7 enzymes used in FA synthesis include acetyl CoA carboxylase, acetyl/malonyl transacylase, 3 ketoacyl synthase
(condensation), 3-KA reductase, hydratase (dehydration), enoyl reductase and thioesterase (decyclase) in this order.

89. A i.e. Operates under anaerobic conditions; D i.e. Not a common pathway; E i.e. Pyridoxal-Phosphate & NADPH ...
90. A & C i.e. ER and Mitochondria [Ref: Lippincott 4lh/ed p g - 187; Harper 28lh/ed pg- 196,199; Lehninger- 812-15] .

'— : -------------------------------------------------------------------------------------
Lipogenesis (i.e. the main pathway for denovo fatty acid synthesis) occurs in cytosolQ. Palmitate, a 16- carbon, fully
saturated long chain fatty acid (16: 0) is the principal endproduct of fatty acid synthase system in animal cells. It is the
precursor of other long chin fatty acid and may be elongated by addition of 2 carbon unit acetyl groups through the
action of fatty acid elongation system present in smooth endo plasmic reticulum (SER) and mitochondria^.
M itochondrial FA chain elongation is not a common pathway^. It operates under anaerobic conditions^ and is
favoured by high NADH /NAD+ ratio and pyridoxal phosphate (coenzyme)Q.

Triacylglycerol and Phosphoglycerol Synthesis

91. B i.e. Enzyme glycerol Kinase Plays an important role [Ref: Lippincott/s 4je p. 181-88,174- 76 , Harper 27/e p. 226, 209-11] .

>•
Triacylglycerol (TAG) is synthesized from acyl- CoA and glycerol 3 phosphate. Glycerol Kinase Catalyzes the activation
of glycerol to sn- glycerol 3 phosphate. Because the enzyme glycerol kinase is not expressed in adipose tissue or muscle,
glycerol cannot be utilized for provision of glycerol 3 - PO4, which must be supplied by glucose via glycolysis. So in
muscle or adipose tissue, most of glycerol-3 PO4 is formed from dihydroxy acetone phosphate by glycerol - 3 -
phosphate dehydrogenase^

P - Oxidation of Fatty Acids

92. C i.e. Carnitine [Ref: Harper's 28/e P-184-86; Lippincott 4/e p. 191]
93. C i.e. Carnitine palmitoyl trasferase II is present in mitochondrial matrix

Long chain activated fatty acid penetrate the inner mitochondrial membrane only in combination with camitineQ.
Carnitine, a carrier molecule, acts like a ferry boat and transport long chain acyl- CoA across mitochondiral membrane
otherwise inner mitochondrial membrane is impermeable to acyl -CoA._____________________________________________

94. B i.e. Propionyl CoA [Ref: Harper's 27/e P-188-89; 26/e P - 182,181; Shinde & Chatterjea P-348, 347]

P- oxidation of odd chain fatty acid produces acetyl CoA plus a molecule of propionyl -CoAQ.
But propionyl CoA is formed only in odd chain fatty acid oxidation (not in even fatty acids)

95. D i.e. 147 [Ref: Lehninger 5lh/e p. 65-57; Harper 28/e p. 191; 27/e p. 189]
96. B i.e. 129 ATP net; E i.e. 96 ATP from citric acid cycle 97. B i.e. H 2O 2 formed

A m odified form of p- oxidation is found in peroxisomes and lead to the formation of acetyl CoA and H2O 2Q
468 ■ A Complete Review of Short Subjects

98. D i.e . P h y t a n ic a lp h a o x id a s e [Ref: Harper 28th/e p.191, 493; Lippincott 4lh/e p.195; Lehninger 5lh/e p. 665; Nelson 17/e p.2077]
* " ^ ^ ^ —
Refsum 's disease is a r a r e a u to s o m a l r e c e s s iv e d is o r d e r c a u s e d b y deficiency of phytanic a oxides (N e ls o n ) / a -
hydroxylase (Lippincot) / Phytanoyl CoA hydroxylase (Lehninger) results in accumulation of phytanic acid due to its
decreased a - oxidation (i.e. hydroxylation at a carbon by fatty acid a hydroxylase)

Elcosanoids
99. B i.e. Cyclooxygenase [Ref: Harper 29/e p. 224-226]

Cyclooxygenase (COX) or prostaglandin H synthase enzyme is a suicide enzyme responsible for its self catalyzed
destructions.
COX has 2 activities, a cyclooxygenase (converting arachidonic acid & 2 O 2 to prostaglandin G2 or PGG 2) and peroxidase
(converting PGG 2 to PGH 2 an endoperoxide). After this PGH2 can be converted to prostaglandin D & E, prostacyclin
(PGI2) and thromboxane (TXA 2) by isomerase, PC synthase & TX synthase respectively. Switching off of prostaglandin
activity is partly achieved by self catalyzed suicidal destruction property of COX. However, 15 hydroxy prostaglandin
dehydrogenase inactivate prostaglandins rapidly.

Ketogenesis (Ketone Body Formation)


100. D i .e . HMG C o A r e d u c t a s e i s th e r a t e - li m it i n g e n z y m e [Ref: Harper 29/e p 210-14; Lippincott 5/e p 195-97]
101. B i.e. HMG CoA 102. A i.e. Liver 103. D i.e. RBCs 104. C i.e. Acetoacetate

• HMG CoA is an intermediate in the pathway of ketogenesis, and this is an immediate precursor o f acetoacetate^ (1st
ketone body to be formed). Aceto acetyl CoA is the starting material for ketogenesis.lt combines with acetyl CoA to form
HMG (=3 Hydroxy 3 methylglutaryl) CoA. This rate lim iting step is catalyzed by HMG CoA synthaseQ.
• Liver is the only organ which produces ketone bodies^ and add to the blood, but it lacks the enzyme responsible for
their degradation and utilization. This is the reason why liver does not utilize ketone bodies'?. Liver ( » ) and RBCs (d/t
lack o f mitochondria) are the only tissues not able to utilize ketone bodies for energy®. Liver lacks Succinyl CoA-
Acetoacetate CoA transferase enzyme which is required for 1st step i.e. activation of ketone bodies in ketone bodies
utilization.
• Ketosis is a metabolic adaption to starvation. Thus a ketone body (Acetoacetate) is formed in starvation not in well fed
state

Pathways Rate lim iting Enzymes


Glycolysis Phospho fructokinase (PFK)-l Q
Glycogenesis Glycogen synthaseQ
G Iy cogenoly sis Glycogen phosphorylaseQ
Gluconeogenesis Phosphoenol pyruvate (PEP) carboxykinaseQ; F I, 6
biphosphalase
Citric acid (TCA) cycle Alpha- ketoglutarate dehydrogenase
HMP shunt (PPP pathway) Glucose 6 phosphate dehydrogenaseQ
Fatty acid synthesis Acetyl CoA carboxylaseQ
Cholesterol synthesis HMG-CoA reductaseQ
Ketone body formation HMG CoA synthesaseQ
Bile acid synthesis 7-alpha hydroxylaseQ (regulated by fam esoid x-
receptor or FXR)
Lipolysis of adipose tissue Hormone sensitive triacylglycerol lipaseQ
Purine synthesis PP ribose - P-glutamyl amidotransferase
Uric acid synthesis Xanthine oxidaseQ
Urea cycle CPS I (mitochondria)
Pyrimidine synthesis CPS (carbamoyl phosphate synthetase II)
Purine salvage pathway HGPRT ase; APRT ase (adenine phosphoribosyl
transferase)
Heme synthesis ALA synthase
Catecholamine DOPA decarboxylase
GTP formation IMP dehydrogenase
Tetrahydro biopterin GTP cyclohydrolase I
formation
Biochemistry: Fat Metabolism ■ 469

105. D i .e . M a lo n y l C o A [Ref: Harper 29/e p 212-14, 247-48; Lippincott 5/e p 330-31,193-98]

In w ell fed state, the activity of CPTI (carnitine palm itoyl transferase I) in outer mitochondrial m embrane is inhibited
by melonyl CoAQ.

106. A i .e . U s i l iz e f a t t y a c id s i n S t a r v a t i o n ; 107. B i.e . K e t o n e b o d i e s ; 108. B i .e . K e t o n e b o d i e s


109. [Ref: Ganong 22/e P- 299-301,342-43, 349- 50; Lehininger 2/e P- 842- 45; Chatterjea 6/e P-
D i.e . I n c r e a s e d G l u c o n e o g e n e s i s
362-65,465; NMS 3/e P-517; Lippincott's 3/e P-294]

Energy is derived mainly from glycogenolysis (initial few hours) —» gluconeogenesis (upto 72 hours) - * Degradation <
o f fast (3 days - 2weeks) —> Break down of tissue protein (last 1 weeks)Q

110. D i.e . G l y c o g e n e s i s 111. B i.e . L iv e r g l u o n e o g e n e s i s in c r e a s e s 112. D i.e . D e c r e a s e in s e r u m P r o t e in s


[Ref: Lippincott 5/e p 321-33; Mark's (2012) p 35-36; Vasudevan 6/e p 107-108; Chattarjea 8/e p 535-35; Ganong 22/e p 350]

- Glycogenolysis and liver gluconeogenesis (initially by degradation o f glucogenic aminoacids of m uscle proteins ☆
followed by glycerol & lactate) are sources of energy in brief (18>stage) of starvation (ie 12 to 72 hours o f fasting).
Gluconeogenesis the main process, increases during brief fasting but decreases on prolonged fasting. In starvation,
glycogenolysis increases and glycogenesis decreasesQ. So in fasting (starvation) there is increase in glycogenolysis,
gluconeogenesis, lipolysis, ketogenesis and proteolysis^.
- During starvation serum proteins are not utilized as a source of energyQ. Brief fasting is associated with increased
proteolysis (break down) o f muscle proteins (not serum proteins) thereby decreasing total muscle proteins, whereas
the serum protein levels are unaltered (ie not decreased)Q. Proteolysis of muscles provide glucogenic aminoacids
(alanine & glutamine increasing liver gluconeogenesis
- On prolonged fasting (starvation), adipose tissue breakdown (beta-oxidation) becomes the main source o f energy;
thereby increasing production of ketone bodies. However it is important to note that this over produced ketone bodies
are utilized mainly by brain not by muscles as the utilization of glucose by brain and utilization of ketone bodies by by-
muscle decreases during prolonged starvation. In the same way, decreased gluconeogenesis in prolonged starvation is
responsible for decreased muscle protein degradation and decreased production of urea._____________________________

113. A i.e . L o w K m v a lu e o f h e x o k i n a s e C i .e . 114. C i .e . P r o lo n g e d S t a r v a t i o n [Ref: Lippincott's 3/e P-89, 293;


H ig h K m v a lu e o f g l u c o k i n a s e Lahninger 2/e P- 840 - 49]

H e x o k in a s e has lo w K m v a lu e i .e . h ig h Diabetes Differences in Ketosis of Starvation


a ffin ity fo r g lu c o s e s and it is present in a ll Mellitus
t is s u e i n c l u d i n g b r a in S ; where as - More Insulin & TGlucagon - Less
g l u c o k i n a s e has h i g h K m v a lu e i .e . lo w - More severe Ketosis - Less severe
a f f i n i t y f o r g lu c o se Q and it is present in liv e r - Ketonemia (ketone bodies in
a n d B - c e l l s S . That's why brain can utilize blood
glucose in starvation but the liver can not. - Ketonuria
The glucokinase functions only when Ketoacidosis
intracellular concentration of glucose in - More severe (decrease in blood pH or acid - Less severe
hepatocytes is elevated as in post absorptive buffering capacity of blood
state. buffers)
- Present^ Glycosuria - Absents
- Hyperglycemia Blood glucose level - Normal or less

115. A i .e . K e t o n e b o d i e s ; B i.e . G l u c o s e ; C i.e . F r e e f a t t y a c id s ; D i.e . C r e a t i n e p h o s p h a t e

free fatty acid ( in (3 o x id a t io n ) , ketone bodies (b y b r a in in s ta r v a t io n ), phosphocreatine


- G l u c o s e (in T C A c y c le ) ,
(creatine phosphate; in m u s c le s ) a n d glucogenic aminoacids a r e u s e d a s source of energy by body.
- Starvation leads to glycogenolysis(lst s ta g e ) , gluconeogenesis (2 nd), lipolysis (3 rd) a n d acidosis (ketoacidosis; 4 th stage)

116. A i .e . A l a n i n e [Ref: Lippincott 3/e P 294; Lahninger 2/e P 843]

I n s t a r v a t io n , a l a n i n e a n d g l u t a m i n e a r e q u a n t i t a t i v e l y t h e m o s t im p o r t a n t g l u c o n e o g e n i c a m in o a c i d s S . S o a la n in e
c a r r ie s n it r o g e n fr o m m u s c le to liv e r & k id n e y f o r f u r t h e r m e ta b o lis m & e n e r g y p r o d u c t io n d u r in g s ta r v a tio n .
470 ■ A Complete Review of Short Subjects

117. C i.e. Increased secretion of insulin [Ref: Ganong 22/e p. 345-46]

- Glucose, mannose, aminoacids (such as leucine, arginine, others) and f) ketoacids such as acetoacetate (derived from
fat catabolism) stimulate insulin secretin and have protein sparing effect^.

- Insulin has anabolic effect on metabolism. It decreases protein catabolism and fat catabolism for energy production and
increases aminoacid uptake & protein synthesis (positive nitrogen balance)^.

118. A i.e. Increase in glucagons / insulin ratio, increased C- AMP & increased blood sugar.
[Ref: Lehnenger 2/e P- 847- 49; Lippincott 2/e P 297; Ganong 22/e P- 350, 336- 49]

D iabetes M ellitus

- In diabetes little glucose is oxidized as fuel, except by the brain. The rest of tissues burn a large amount of fat,
particularly the liver where the amount of acetyl COA formed from fatty acids exceeds the capacity of the
tricarboxylic acid cycle to oxidize it. The excess acetyl COA is converted to ketone bodies 1/1 ketonemia, ketonuria &
ketoacidosis
- Diabetics not only have a defect in the tissue utilization of glucose but also appears to be metabolically poised to
produce maximum amount of glucose from amino acids (gluconeogenesis) and to prevent glucose from being
utilized to to form fat. (fat synthesis)
- Insulin is decreased & glucagons is increased 1/t Tglucagon /insulin ratioQ. Glucagon acts via elevation in C- AMPQ

119. D i.e. Ketones [Ref. Vasudevan 6/e p. 372, 319, 305, 64, 251-50, 288, 279; Lehninger 5/e p. 241; Satyanarayana 3rd/295; 'Practical
Biochemistry for Students' by Malhotra 4th/e 43-47, 'Laboratory Manual of Biochemistry' by Chary & Sharma (2004) / 21]

Rothera's - nitroprusside - pink/ purple permanganate ring test and Gerhardt's - ferric chloride - red wine color tests
are used for detection of ketone bodies in urine.

120. B i.e. Rotheras [Ref: Chatterjea 6/e P-652-60] 121. A i.e. Bile salts in urine

Owing to gradual decline in carbohydrate store & lack of Tests Used for
carbohydrates, energy is obtained from burning fat. This Rothera's test Ketone bodies in urineQ:
leads to an increase in free fatty acids (FFA). As FFA increase, (nitroprusside test) acetone and acetoacetate
ketogenesis is stimulated and excess of ketone bodies are Gerhard’s test (ferric Ketone bodies in urine:
formed which pass into blood, resulting in ketosis and chloride test) acetoacetate
ketonuria. Benedict's test
Reducing sugars in urine
The women is presenting with altered sensorium and Fehling's test
dehydration, which may be the presenting features of Hay's sulphur test B ile salts in urineQ
ketoacidosis. She has already been tested for blood sugar Fouchet's (borium
(rules out diabetic ketoacidosis) & protein. sulphate test)
The important investigation missed out in this patient is ketone Gmelin's (nitric acid Bile pigment: bilirubin
bodies in urine, which is important to reach to the diagnosis of test)
starvation ketosis. Ketone bodies are tested by Rotheras test. Vanden Bergh test
There appears no clinical relevance to test for bile salts & bile Bilinogens (stercobilinogen,
Ehrlich's test
pigments. urobilinogen)
Schlesinger's test Bilins (stercobilin, urobilin)
Test Done for
Ehrlich's aldehyde Porphobilinogen and
Fouchet's test Bile pigments urobilinogen in urine
test
Hay's sulphur test Bile salts
Benedicts test Blood sugar
Rotheras test K eton e b o d iesQ
A „ STRUCTURE, FUNCTION AND REPLICATION OF
Chapter -3
F INFORMATIONAL MACROMOLECULES

QUESTIONS

Purines and Pyrimidines C. Development of arthritis correlates with level of


serum uric acid □
1. (NBE P 15)
R a te l im i t i n g s t e p i n p y r i m id i n e s y n t h e s i s -
D. Uric acid crystals are best seen by polarising light
A. DIhydro-orotase □
microscope □
B. Omithine transcarbomolyase □
10. Specific test for gout is: (AIIMS 06)
C. Aspartate transcarbomoylase □
A. Raised serum uric acid level □
D. Carbamoyl phosphate synthase-1 □
B. Raised uric acid in synovial fluid of joint □
2 . P y r im i d i n e m e t a b o l i s m f e a t u r e s a/e (PGI 15,10)
C. Raised urea level □
A. Require CAD polypeptide chain □
D. Raised urease enzyme level □
B. 6-azauridine 1/t oritidinuria □
11. A ten year old child with aggressive behavior and poor
C. DHOA-dehydrogenase is mitochondrial □
concentration is brought with presenting complaints of
D. Reye syndrome decrease cytosolic carbamoyl
join t pain and reduced urinary output. M other gives
phosphate □
history of self m utilitative behavior stating that he
E. Urea cycle is linked □ tends to mutilate his fingers. W hich of the follow ing
3. W h a t i s in v o lv e d i n f o r m a t io n o f d - T M P f r o m d -U M P ?
enzymes is likely to be deficient in this child (AI 09)
A. N5, N10-methylene tetra hydrofolate (PGI 07) □ A. HGPRTase □
B. Form imino folate □
B. Adenosine Deaminase □
C. N5 formyl folate □
C. APRTase □
D. Dihydro folate □
D. Acid Maltase □
4. E n d p r o d u c t o f p u r in e m e t a b o l i s m i n n o n - p r im a te
12. Lesch Nyhan syndrome is due to deficiency of:
m a m m a ls i s : (NBE P 15,13; AIIMS 14,13,07)
A. Hypoxanthine phosphoribosyl transferase
A. Uric acid □
(HGPRTase) (PGI 0 8 ,1 2 ) 0
B. Ammonia □
B. Xanthine oxidase □
C. Urea □
C. Purine phosphorylase □
D. Allantoin □
D. Adenosine deaminase □
5. I n h u m a n s , e n d p r o d u c t o f p u r in e m e t a b o l i s m -
13. A 12 year old child presents with mental retardation
A. Allantoin (NBE P 15) □
and history of self mutilation. Which of the follow ing
B. Uric acid □
tests w ill help in determining the diagnosis (AI 12)
C. c o 2 □
A. Serum Lead Levels □
D. None □
B. Serum Alkaline Phosphatase Levels □
6. I n o s i t i c a c id i s b i o l o g i c a l p r e c u r s o r o f -
C. Serum Lactate Dehydrogenase Levels □
A. Uracil and thymine (Nimhans 07, jipmer 04) □
D. Serum Uric Acid Levels □
B. Purines and thymine (UP 05, AMU 06) □
14. W hich is common metabolic process in human as well
C. Adenylic acid and guanylic acid □
as bacteria: (PGI 01)
D. Orotic acid and uridylic acid □
A. Purine synthesis □
7. F a l s e r e g a r d in g g o u t i s : (AI 01)
B. Nitrogen fixation □
A. Due to increased metabolism of pyrimidines □
C. Mucolipid formation □
B. Due to increased metabolism of purines □
D. Nonoxidative photophosphorylation □
C. Uric acid levels may not be elevated □
15. Apart from occurring in nucleic acids, pyrimidines are
D. Has a predilection for the great toe □
also found in: (AIIMS 05)
8. G o u t i s a d is o r d e r o f : (AI 06)
A. Theophylline □
A. Purine metabolism □
B. Theobromine □
B. Pyrimidine metabolism □
C. Flavin mononucleotide □
C. Oxalate metabolism □
D. Thiamine □
D. Protein metabolism □
16. Salvage pathway of purine nucleotide synthesis are
9. W h a t i s n o t tru e a b o u t g o u t: (AIIMS 05)
used all except- (NBE P 15)
A. Abrupt increase in serum urate levels is more
A. Brain □
common a cause for acute gout than an abrupt fall in
B. Liver □
urate levels. □
C. RBC □
B. Patient may be asymptomatic with high serum uric
D. Leukocytes □
acid for years □
472 ■ A Complete Review of Short Subjects

DNA Structure B. 25% □


C. 46% □
17. True statements about DNA structure: (PGI 06) D. 27% □
A. All nucleotides are involved in linkage □ 27. Triple bonds are found between which base pairs:
B. Antiparallel □ A. A-T (AI 01) □
C. Parallel □ B. C-G □
D. Bases are perpendicular to DNA □ C. A-G □
E. Attached by hydroxy bond. □ D. C-T □
18. mRNA is a complimentary copy of - (SG PG I03) 28. Thermo-stability in DNA is contributed mostly by:
A. TRNA (Jipmer 02) □ A. A=T (AI 07; NBE P 13) □
B. RRNA □ B. G e C □
C. Ribosomal DNA □ C. Molecular base □
D. Both strands of DNA □ D. Parallel arrangement □
E. A single strand of DNA □ 29. In Hybrid RNA-DNA, DNA is 5CTTAAG3. The
19. Which is not found in DNA: (NBE P 14,13; AI 04) sequence of RNA w ill be: (PGI 11,15)
A. Adenine □ A. 5CTTAAG3 □
B. Thymine □ B. 5CUUAAG3 □
C. Guanine □ C. 5GAATTC3 □
D. Uracil □ D. 5GAAUUC3 □
20. The two strands of DNA are held together by: E. 5CTTUUG3 □
A. Van-der -Waal bond (AIIMS 07) □ 30. Which one of the following is the complementary
B. Hydrogen bond □ sequence of 5' TTAAGCTAC 3'? (AI 06)
C. Covalent bond □ A. 5 ' GTACGCTTAA 3' □
D. Ionic interaction □ B. 5' AATTCGCATG 3' □
21. True about DNA structure: (PGI 10) C. 5' CATGCGAATT 3' □
A. Purines are adenne and guanine & pyrimidines are D. 5' TTAAGCGTAC 3' □
uracil and cytosine □ 31. True about coding strand of DNA: (PGI 03)
B. Watson and Crick discovered structure in 1973 □ A. Template strand □
C. Deoxyribose-phosphate backbone with bases B. Minus strand □
stacked inside □ C. Runs at 5- - 3' direction □
D. Mainly consists of left handed helix □ D. Runs at 3' - 5' direction □
E. 5'-3' phosodiester bonding is present □ E. Plus strand □
22. Which form of DNA is predominantly seen: (NBE P 13) 32. True a bout coding strand of DNA: (PGI 03)
A. A (AI 06) □ A. DNA acts as template for mRNA □
B. C □ B. Minus strand □
C. B □ C. Plus strand □
D. Z □ D. Matches with RNA transcript hat encodes
23. Triplex DNA is due to (NBE P 14; AIIMS 11) protein □
A. Hoogsteen pairing □ 33. 5' TTACGTAC-3' after transcription what w ill be the
B. Palindromic sequences □ RNA: (PGI 08)
C. Large no. of guanosine repeats □ A. 5-TTACGTAC-3' □
D. Polypyramidine tracts □ B. 3'- TTACGTAC-5' □
24. Chargaff rule state that- (NBE P 13; UP 08) C. 5' CATGCATT-3 □
A. A+G = T+C □ D. 3'- CATGCATT-5 □
B. A/T = G/C □ E. 5'- GUACGUAA-3' □
C. A = U = T = G = C □ 34. Bromodeoxyuridine is related to DNA in: (AI 06)
D. A + T = G + C □ A. Uracil □
25. A nucleic acid was analyzed and found to contain 32% B. Adenosine □
adenine, 18% guanine, 17% cytosine and 33% thymine. C. Cytosine □
The nucleic acid must be: (AIIMS 06) D. Thymidine □
A. Single-stranded RNA □ 35. At the physiological pH the DNA molecules are:
B. Single-stranded DNA □ A. Positively charged. (AIIMS 13, NBE P 14) □
C. Double-stranded RNA □ B. Negatively charged. □
D. Double stranded DNA □ C. Neutral. □
26. In a sample of DNA, if adenine is 23% what w ill be the D. Amphipathic □
amount of guanine present: (PGI 13,16) 36. True about mitochondrial DNA: (PGI 14,11)
A. 23% □ A. Maternal inheritance □
Biochemistry: Structure, Function and Replication of Informational Macromolecules ■ 473

B. Same evolutionary origin □ B. Increased telomerase activity favours cancer


C. Not highly conserved & has high mutation rate □ cells □
D. Mitochondrial disease is associated mostly with C. DNA dependent RNA polymerase □
Mutation & some have deletion □ D. DNA polymerase □
E. Mostly encodes membrane protein □ 47. Action of 'Telom erase' is (NBE P 14; UP 04)
37. True about mitochondrial DNA (NBE P 14; AI 11) A. DNA repair □
A. UGA codes for tryptophan □ B. Longetivity of cell - Ageing □
B. Codes for 13 protein □ C. Breakdown of telomer □
C. High content of untranslated sequences □ D. None □
D. Circular double stranded DNA □ 48. Ends of chromosomes replicated by: (PGI 06)
E. Mitrochondrial disease occur due to Point Mutations A. Telomerase □
and Large-Scale Rearrangements □ B. Centromere □
38. Intron is not found in which DNA? (AIIMS 08)
C. Restriction endonuclease □
A. Nuclear DNA □
D. Exonuclease □
B. Mitochondrial DNA □
49. Which enzymatic mutation is responsible for
C. B DNA □
immortality of cancer cells : (AIIMS 01)
D. ZDNA □
A. DNA reverse tiansciptase □
39. NARP syndrome & myopathy is seen in
B. RNA polymerase □
A. Lysosomal disorder (AIIMS 15,14,09) □
C. Telomerase □
B. Glycogen storage disease □
D. DNA polymerase □
C. Mitochondrial disease □
50. All of the follow ing cell types contain the enzyme
D. Lipid storage disease □
telemerase which protects the length of telomeres at
40. M itochondrial DNA (gene) true is: (PGI 14,08,03)
the end of chromosomes, except: (AI 06)
A. Paternally inherited □
B. Maternally inherited □
A. Germinal a
C. Horizontal inheritance □ B. Somatic □
D. Vertical Mendelian inheritance □ C. Hemopoietic □
41. M itochondrial DNA is: (NBE P 15,14; AI 06) D. Tumor □
A. Closed circular □ 51. True about Histone protein: (PGI 12, 09)
B. Nicked circular □ A. Ribonucleoprotein □
C. Linear □ B. Present inside the nucleus □
D. Open circular □ C. Acidic □
42. Circular DNA is found in: (PGI 11) D. Basic □
A. Virus □ E. Glycoprotein □
B. Chloroplast □ 52. Histone acetylation causes? (AIIMS 11)
C. Bacteria □ A. Increased Heterochromatin formation □
D. Mitochondria □ B. Increased Euchromatin formation □
E. Fungus □ C. Methylation of cystine □
43. Total num ber of genes in a human being is- D. DNA replication □
A. 800,000 (DNB 12,08) □ 53. Components of chromosome are: (PGI 03)
B. 50,000 □ A. DNA □
C. 100,000 □ B. tRNA □
D. 30,000 □ C. m RN A □
44. Cellular bearings of herediatary disease seen in: D. rRN A □
A. DNA (DNB 12, 08; PGI 03) □ E. Histones □
B. Ribosome □ 54. The protein rich in basic amino acids, which functions
C. RNA □ in the packaging of DNA in chromosomes, is:
D. Membrane □
A. Histone (AI03) □
E. Proteins □
B. Collagen □
45. Highly repetitive DNA is seen in: (PGI 03)
A. Cloning of DNA □
C. Hyaluronic acid binding protein □
B. Microsatelite DNA □
D. Fibrinogen □
55. Histone has post - translational m odification by
C. Telomere □
all/except. (AIIMS 07)
D. Centromere □
46. True about telomerase or telomere is/are: (PGI 03)
A. Acylation □
A. They are present at the ends of eukaryotic
B. Methylation □
chromosome □
C. Phosphorylation □
D. Glycosylation □
474 ■ A Complete Review of Short Subjects

56. Euchromatin is the region of DNA that is relatively: B. DNA Polymerase II □


A. Uncondensed (NBE P 13; AI 06) □ C. DNA Polymerase III □
B. Condensed □ D. RNA Polymerase I □
C. Overcondensed □ 65. True-regarding DNA-replication: (PGI 07)
D. Partially condensed □ A. Occurs in the M-phase of the cell cycle. □
57. Nucleosome consists of: (PGI 15,10) B. Sister Chromatids are formed. □
A. Histone □ C. Follow base pair rule. □
B. DNA □ D. Semi conservative. □
C. RNA □ E. Single strand breaknd. □
D. DNA & RNA both □ 66. Correct sequence of enzymes required for DNA
E. Carbohydrate □ formation is: (PGI 01)
58. True about nucleosome: (PGI 09) A. DNA polymerase —) Protein unwinding enzyme —>
A. Use only one type of histone protein □ DNA ligase -> DNA Isomerase -> Polymerase -1 □
B. Each complex is separated from each other by non B. Protein unwinding enzyme —> Polymerase 1 —> DNA
histone proteins □ ligase-) DNA Isomerase-) DNA polymerase □
C. Regular repeating structure of DNA & histone C. RNA polymerase —» DNA polymerase III —>DNA
proteins □ polymerase I -> DNA ligase □
D. Reflect small nucleus □ D. RNA polymerase —> DNA polymerase III —> dNA
E. Core of DNA with protein wrapped □ ligase —> Exonuclease —>DNA polymerase I □
59. CG islands in our DNA are important fon (AIIMS 16) 67. False statements are: (PGI 11)
A. Acetylation □ A. In leading strands DNA is synthesized
B. Methylation □ continuously □
C. t-RNA synthesis □ B. Multiple origins of replication are possible for
D. DNA replication □ bacteria □
C. DNA replication proceeds in one direction □
DNA Synthesis and Replication D. Lagging strand stick by RNA primase □
E. DNA polymerase III- processive leading strand
60. True about prokaryotic DNA replication: (PGI 13) synthesis □
A. Conservative □ 68. The gaps between segments of DNA on the lagging
B. Semi conservative □ strand produced by restriction enzymes are rejoined
C. Unidirectional □ sealed by: (NBE P 15,13; AI 09)
D. Bidirectional □ A. DNA Ligases □
E. Semi discontinuous □ B. DNA Helicase □
61. True about DNA supercoiling is all except C. DNA topoisomerase □
A. Underwinding results in negative superhelical D. DNA phosphorylase □
density & supercoiling (PGI 09) □ 69. DNA synthesis requires: (PGI 16,10)
B. Superhelical density depends on length of DNA □ A. DNA polymerase □
C. Underwinding strain is normally accommodated by B. DNA ligase □
strand separation □ C. DNA topoisomerase □
D. Topoisomerase II change linking number in D. Primase □
increments of 2 □ E. RNA polymerase □
E. Most cellular DNA is underwound □ 70. DNA supercoiling is done b y :
62. Okazaki segments are required for OR formed during: A. DNA polymerase I (AIIMS 04,15) □
(AIIMS 14; NBE P 15,14,13; AI 10; PGI 14) B. DNA polymerase II □
A. DNA synthesis/ replication □ C. DNA topoisomerase □
B. RNA synthesis □ D. DNA gyrase. □
C. Protein synthesis □ 71. Which statements are true about E. coli chromosomal
D. None of the above □ DNA in relation with eukaryotic chromosomal DNA:
63. Okazaki fragments are formed during the synthesis of A. Circular (PGI 16,08) □
A. ds DNA (AIIMS 15; NBE P 15; AI 08) □ B. Packed into nucleolus □
B. ss DNA □ C. Positively Supercoiled □
C. m RNA □ D. Negatively sepercoiled □
D. t RNA □ E. Nucleoid present □
64. During replication of DNA, which one of the 72. Which DNA polymerase is /are involved in repair of
follow ing enzymes polymerizes the Okazaki mammalian DNA: (PGI 09)
fragments? (NBE P 15,14; PGI 14; AI 06) A. a □
A. DNA Polymerase I □ B. |3 □
Biochem istry: Structure, Function and R eplication of Inform ational M acrom olecules ■ 475

C. y □ C. Restriction endonuclease cut DNA chains at specific


D. e □ location □
E. 8 □ D. Endonuclease cut DNA at 5' terminus □
73. True about DNA polymerase in eukaryotes: E. Right handed helix of DNA is more common □
A. Components are a , p, y, A, e (PGI 08) □ 82. Photolyase helps in - (AIIMS 03; DNB 14)
B. P associated with repair □ A. DNA repair □
C. y associated with repair □ B. DNA replication □
D. A associated with synthesis of mitochondria DNA □ C. Protein synthesis □
E. Alpha is abundant amount □ D. None □
74. DNA polymerase have: (PGI 15,10; AIIMS 09) 83. Excessive ultraviolet (uv) radiation is harmful to life.
A. 3' —>5' polymerase activity □ The damage caused to the biological systems by
B. 5' -> 3’ polymerase activity □ ultraviolet radiation is by OR The primary defect in
C. 3' —» 5' exonuclease activity □ Xeroderma pigmentosa is: (AIIMS 10; AI 08)
A. Formation of thymidine dimmers □
D. 5' —>3' exonuclease activity □
B. Poly ADP ribose polymerase isdefective □
C. Exonuclease is defective □
DNA Damage, Repair and Protooncogene
D. Formation of adenine dimers □
84. Xeroderma pigmentation is caused due to a group of
75. Nucleoside is made up of: (PGI 13,10)
closely related abnormalities in:
A. Pyrimidine □
A. Mismatch repair (NBE P 14; AIIMS 15, 05) □
B. Histone □
B. Base excision repair □
C. Sugar □
C. Nucleotide excision repair □
D. Purine □
D. SO S repair □
E. Phosphate □ 85. Xeroderma pigmentosum is produced as a result of a
76. All of the follow ing abbreviations are true except:
defect in: (AI 04)
A. AMP-Adenosine monophosphate (PGI 09) □
A. DNA polymerase III □
B. CMP - Cytidine monophosphate □
B. DNA polymerase I □
C. GMP - Guanosine monophosphate □
C. DNA exonuclease □
D. TMP - Thymine monophosphate □
D. DNA ligase □
E. UMP - Uracil monophosphate □
86. A child develops blisters on exposure to sunlight.
77. On complete hydrolysis of DNA, we w ill get all of the
Irregular dark spots on the skin are also found. He is
follow ing except: (AI 04)
very likely to have a defect in which o f the follow ing
A. Adenosine □ mechanisms?
B. Purine base □ A. Nucleotide excision repair (AIIMS 12) □
C. Phosphoric acid □ B. Mismatch repair □
D. Dexose pentose sugar □ C. Recombination repair □
78. What w ill happen to DNA if salt is added to it:
D. Thymine dimmers □
A. Increase Melting point (Tm) (PGI 08) □
87. 5-methyl cytosine mutation of DNA (DNA
B. Decrease Tm □ methylation) leads to: (PGI 11)
C. Not affect Tm □ A. Deamination to uracil □
D. Melting lead to denaturation of DNA □ B. Deamination to thymine □
E. Both increase & decrease in Tm depending on C. Mispair with adenine □
concentration of salt □ D. DNA repair □
79. What is meant by melting of double stranded DNA: E. Methylation protects the host DNA from cleavage by
A. Splitting of double strands into single strand. □ its own restriction enzyme □
B. Splitting of DNA into fragments. (AIIMS 08) □ 88. DNA repair defect is seen in: (PGI 11, 08)
C. Formation of triple helix □ A. Xeroderma pigmentatosa □
D. Separation of bases. □ B. Bloom's syndrome □
80. Concentration of DNA is measured by: C. Ataxia telangiectasia □
A. Absorption at 260 nm (PGI 09) □ D. Li-Fraumani syndrome □
B. D-oxyribose estimation □ E. Retinoblastoma □
C. Infrared examination □ 89. The normal cellular counterparts of oncogenes are
D. Ultrasound examination □ important for the follow ing functions, except: (AI 06)
81. Incorrect statement are: (PGI 10) A. Promotion of cell cycle progression □
A. T4 DNA polymerase has 3'>5' exonuclese activity □ B. Inhibition of apoptosis □
B. Klenow fragment of DNA polymerase I function is C. Promotion of DNA repair □
almost similar to T4 DNA polymerase □ D. Promotion of nuclear transcription □
476 ■ A Complete Review of Short Subjects

90. True about Proto-oncogenes are: (PGI 07) A. 16 □


A. Normally involved in cell cycle proliferation □ B. 21 □
B. Produces tumor □ C. 256 □
C. Normally involved in suppression of tumour D. 64 □
production □ E. 265 □
D. Proto- oncogens on mutation causes cancer. □ 100.A c o d o n c o n s is t s o f - (AIIMS 03, WB 02)
91. Anti -carcinogens are: (PGI 07) A. One molecule of amono acyl-t RNA □
A. Carotenoids □ B. Two complementary base pairs □
B. Flavenoids □ C. 3 consecutive nucleotide units □
C. Curcumoids □ D. 4 individual nucleotides □
D. Benzene □ 101.A ll a r e tru e o f t h e g e n e t ic c o d e e x c e p t -
E. Lignase □ (AIIMS 16,14; DNB 15; NBE P 15,14,13; PGI 15,12)
92. Following required for Normal growth: (PGI 06) A. Degenerate □
A. Protooncogenes □ B. Universal □
B. Tumour suppressor genes □ C. Punctuation □
C. Oncogenes □ D. Non overlapping & Non ambiguous □
D. DNA repair genes □ 102.Chain initiation in protein synthesis is by-
93. Which is not a tumor suppressor gene? (AIIMS 08) A. AUG (MAHE 04, AIIMS 02, PGI 05) □
A. WT-1 □ B. GLA □
B. Rb □ C. UGA □
C. p53 □ D. UAG □
D. RAS □ 1 0 3 .N o n s e n s e c o d o n s b r in g a b o u t - (AIIMS 13,DNB 15)
94. Maximum damage to DNA is caused by: A. Elongation of polypeptide chain □
A. a-rays (AIIMS 03) □ B. Pre-translational modificastion of protein □
B. |3-rays □ C. Initiation of protein synthesis □
C. y-rays □ D. Termination of protein synthesis □
D. 5-rays and UV rays □ 104.A c o d o n c o n s i s t s o f - (AIIMS 12)
A. 2 complementary base pairs □
Genetic Code B. 3 complementary base pairs □
C. 4 complementary base pairs on RNA □
95. Amino acids not coded by triplet codon: (PGI 11) D. 4 individual nucleotides □
A. Lysine □ 105.A c o n d o n c o n s i s t s o f - (DNB 13; Raj 04, UP 05)
B. Hydroxyproline □ A. Two complementary base pairs □
C. Selenocysteine □ B. Three consecutive nucleotide units □
D. Pyrrolysine □ C. An individual ribosome □
E. Ornithine □ D. Four individual nucleotides □
96. True about genetic code except: (PGI 14,12, 09) 1 0 6 .S a m e a m in o a c id i s c o d e d b y m u l t i p l e c o d o n s d/t
A. Degenerate □ f o l l o w in g : (AIIMS 12,11,06)
B. Overlapping □ A. Degeneracy □
C. Ambigous □ B. Frame-shift mutation □
D. Universal □ C. Transcription □
E. Commaless □ D. Mutation □
97. S t o p c o d o n s a r e A/E: 1 0 7 .W o b b le h y p o t h e s i s - r e g a r d i n g t h e v a r ia t io n tru e is :
(NBE P 15,14,13; PGI 16,14,10; AIIMS 15,12) A. 3 - end of anticodon (PGI 15,14, 06) □
A. UAA □ B. 5 - end of anticodon □
B. UAG □ C. m-RNA □
C. UGA □
D. t-RNA □
D. UAC □
98. True about genetic code: (PGI 09)
Regulation of Gene Expression
A. AUA codes for methionine in mitochondria □
B. UGA codes for selenocysteine □
C. AUG codes for initiator codon in mammalian cell □ 108.Steroids act via nuclear receptors which interact with
D. AGA & AGG act as chain terminator in mammalian
DNA through (AIIMS 06)
A. Helix turn helix □
mitochondria □
E. UAA, UAG & UGA are stop codon □ B. Zinc finger motif □
99. If there are 4 nucleotides instead of 3 in codon, how
C. Histidine □
many amino acids may be formed? (AIIMS 11, PGI 07) D. Leucine zipper □
B iochem istry: Structure, Function and R eplication of Inform ational M acrom olecules ■ 477

1 0 9 .Z in c f in g e r is : (PGI 14, 06) B. Required only when inducer is present □


A. Nuclear receptor □ C. Mutant □
B. Membrane receptor □ D. Not regulated □
C. Receptor associated kinase □ 120.True among all is (BHU 04, UP 04)
110.S t e r o id r e c e p to r r e g u la t o r y p r o t e in : (PGI 15, 08) A. Repressor is dimer and a positive regulator □
A. Zinc finger □ B. CRP is gratuitous inducer □
B. Heliz-turn -helix □ C. Lactose is positive regulator □
C. Leucine zippor □ D. De-repression is d/t presence of glucose □
D. RNA □ E. Catabolite repression is mediated by CRP □
111.P r o t e in s w h i c h is h a v in g D N A b i n d i n g d o m a in in its 121.False statement is (PGI 04, Jipmer 06, WB 03, KA 04)
s tr u c tu r e : (PGI 13, 07) A. Repressor binds operator gene □
A. Zinc finger □ B. Regulator genes produce repressor subunits □
B. (3- sheet □ C. IP TG is inducer but not substrate □
C. P-band □ D. Regulator gene is inducible □
D. helix-turn helix □ 122.Lac operon transcription is induced by
E. leucine zipper □ A. Glucose (Delhi 03, UP 02, TN 0 1 ) 0
112.I n le u c i n e z ip p e r m o d e l, L e u c in e r e s id u e s e e n a f te r B. Glucose with inducer □
ev ery (AIIMS 09) C. Inducer without glucose □
A. 3 amino acids □ D. All □
B. 6 amino acids □ 123.AH of the follow ing statements about Lambda phage
C. 9 amno acids □ are true, Except: (AI 09)
D. 12 amino acids □ A. In Lysogenic phase it fuses with host chromosome
E. 7 amino acid □ and remains dormant □
1 13.B in d i n g o f p r o t e in s to D N A is r e g u la t e d b y : (AI 07) B. In Lytic phase it fuses with host chromosome and
A. Copper □ replicates □
B. Zinc □ C. Both Lytic and lysogenic phases occui• together □
C. Selenium □ D. In Lytic phase it causes cell lysis and releases virus
D. Nickle □ particles □
1 1 4 .L e u c in e - Z ip p e r c o m p le x i s (AIIMS 06) 124.1n conversion of DNA to RNA, enzyme required:
A. DNA binding protein □ A. DNA- Polymerase 1 (PGI 08) □
B. Membrane attack complex □ B. DNA Ligase □
C. B cell epitomes □ C. DNA-Polymerase III □
D. Receptor ligand protein □ D. RNA polymerase □
115.W h ic h i s t h e s m a l le s t f u n d a m e n t a l u n i t c o d in g f o r E. Primase □
D N A s y n th e s is : (PGI 01) Transcription
A. Cistron □
125.The sigma (a) subunit of prokaryotic RNA polymerase:
B. Operon □
A. Binds the antibiotic rifampicin (AI 0 6 )0
C. Replican □
B. Is inhibited by a-amanitin O
D. Anticodon □
C. Specifically recognizes the promoter site □
1 1 6 .T ru e a b o u t g e n e s (AIIMS 08)
D. Is part of the core enzyme 0
A. Smallest functional unit of genome □
.The atachment site of DNA dependent RNA
B. Not capable of independent expression □
polymerase to a gene is the - (PGI 05; UP 03)
C. Promoter & enhancer geens are typical example □
A. Operator □
D. Cistron is single functional unit □
B. Promoter □
1 1 7 .C A P i n L a c o p e r o n is a n e x a m p le o f :
C. Structural gene □
A. Positive regulator (AI 09, 07, AIIMS 11) □
D. Regulatory gene □
B. Negative regulator □
E. Repressor □
C. Constitutive expression □
.RNA polymerase does not require : (AI 04)
D. Attenuation □
A. Template (ds DNA) O
118.W h ic h o f th e f o l l o w in g a re s it u a t e d a w a y f r o m t h e
B. Activated precursors (ATP, GTP, UTP, CTP) □
c o d in g r e g io n : (PGI 06)
C. Divalent metal ions (Mn2+, Mg2+) O
A. Promoter □
D. Primer O
B. Enhancer □
.R-RNA is mainly produced in - (AIIMS 13, 06)
C. Operator □
A. Nucleus O
D. Structural gene □
B. Nucleolus O
11 9 .H o u s e k e e p i n g g e n e s a r e (Jipmer 02, WB 03)
C. Ribosome O
A. Inducible □
D. Endoplasmic reticulum O
478 ■ A Complete Review of Short Subjects

DNA c o n t r i b u t i n g to p o ly p e t id e
1 2 9 .P a rt o f e u k a r y o t ic C. Different a subunits are responsible for recognition
s y n th e s is : (PGI 15,11) specificity □
A. Exon □ D. Trans acting factors bind to CAAT/GC box-
B. Enhancer □ promoter proximal element. □
C. Leader sequence □ 1 3 7 .T A T A b o x is s e e n i n - (NBE P 15,13)
D. tRNA □ A. Promoter region □
E. ncRNA □ B. Palindromic region □
1 3 0 .N o t tr u e a b o u t e u k a r y o t ic g e n e : (PGI 14,11) C. Enhancer region □
A. Polycistronic mRNA □ D. Silencer region □
B. Noncoding intron □ 138.A y o u n g b o y w h o h a s d i f f i c u l t y i n g e t t i n g u p fr o m
C. Contain nuclear gene & pseudogene □ s i t t i n g p o s i t i o n w a s d ia g n o s e d to h a v e D u c h e n e 's
D. Modification of mRNA before transportation from m u s c u la r d y s tr o p h y . W h ic h o n e i s tr u e r e g a r d in g th e
nucleus □ m u t a tio n i n t h e d y s tr o p h in g e n e i n p r o m o t e r r e g io n :
E. Noncoding exons □ A. Initiation of transcription of the dystrophin gene
131.T h e f o l l o w in g is a g e n e r a liz e d d ia g r a m o f a ty p ic a l would be affected (AIIMS 1 0 ) 0
e u k a r y o t ic g e n e : (AIIMS 06) B. Capping of mRNA of dystrophin gene would be
affected □
Prom oter Region Polypeptide Coding Region C. Tailing of mRNA of dystrophin gene would be
affected □
D. Premature termination □
1 3 9 .N o n c o d in g RNAs a r e : (PGI 15,12,10)
w-
D ir e c tio n o f tr a n s c r ip tio n A. siRNA □
B. miRNA □
What is the most likely effect of a 2bp insertion in the C. tRNA □
middle of the intron? D. mRNA □
A. Normal transcription, altered translation □ E. rRNA □
B. Defective termination of transcription, normal 1 4 0 .N o t p r o d u c t o f t r a n s c r ip t io n : (PGI 14,11)
translation □ A. tRNA □
C. Normal transcription, defective mRNA splicing □ B. mRNA □
D. Normal transcription, normal translation. □ C. rRNA □
1 3 2 .P a rt o f M - R N A r e m o v e d d u r in g p r o t e in s y n t h e s i s -
D. cDNA □
A. Interon (AI 01, MP 05, WB 06) □ E. New strand of DNA □
B. Codon □ 141.G e n e t r a n s m it te d b u t n o t t r a n s la te d ; (AIIMS 04,14)
C. Exon □ A. Glycosyl transferase □
D. Suistron □ B. t-RNA □
1 3 3 .F u n c tio n o f e n d o n u c le a s e s - (Kerala 08, TN 07) C. Keratin □
A. Cut DNA at specific DNA sequences □ D. Histone □
B. To point out the coding regions □ 142.W h ic h o f th e f o l l o w in g s t a t e m e n t s r e g a r d in g m a tu r e
C. Enhancers □ c y t o p ls m ic m e s s e n g e r R N A i s tr u e (NBE P 14, AI 12)
D. To find out antibiotic resistance □ A. Transcribed from Nuclear DNA □
134.R e p li c a t i o n a n d t r a n s c r ip t io n a r e s i m i l a r p r o c e s s e s in
B. Has Thiamine in place of Uracil □
m e c h a n is t ic te r m s b e c a u s e b o t h : (AIIMS 03) C. Sugar is DeoxyRibose □
A. Use RNA primers for initiation. □ D. Its molecular weight is more than hn-RNA □
B. Use deoxyribonucleotides as precursors. □ 1 4 3 .N o b e l p r iz e i n 2006 ( r e c e n tly ) w a s g iv e n f o r th e
C. Are semi conserved events □ d is c o v e r y o f (AIIMS 09)
D. Involve phosphodiester bond formation with A. Lipoxins □
elongation occurring in the 5' - 3' direction. □ B. RNAI □
135.1n a DNA t h e c o d in g r e g io n r e a d s 5 '- C G T - 3 '. T h i s C. mt DNA □
w o u ld c o d e i n t h e RNA a s: (AIIMS 14, 02) D. |3-transcription factor □
A. 5-CGU-3' □ 144.M ic r o R N A t r a n s c r ib e d b y : (PGI 15, 08)
B. 5'GCA-3' □ A. RNA polymerase I □
C. 5-ACG-3' □ B. RNA polymerase II □
D. 5-UGC-3' □ C. RNA polymerase III □
1 3 6 .T ru e a b o u t e u k a r y o t ic i n i t ia t i o n is A/E (AIIMS 10) D. DNA polymerase □
A. Initiator sequence spans TSS (+1) □ E. Drosha □
B. Distal regulatory element (DRE) is exclusively 1 4 5 .N o rm a l r o le o f M ic r o R N A is : (AI 09)
found in eukaryotes □ A. Gene Regulation □
B iochem istry: Structure, Function and R eplication of Inform ational M a cro m o lecu les ■ 479

B. RNA splicing □ 1 5 5 .L e a s t p o s t t r a n s l a t i o n a l m o d if i c a t i o n o c c u r s i n
C. Initiation of Translation □ A. t-RNA (CMC 08, Jipmer 09, WB-03) □
D. DNA conformational change □ B. Prokaryotic r- RNA □
1 4 6 .P o ly (A ) ta il t r a n s l a t e s i n t o ( i.e . o n t r a n s l a t i o n g iv e r is e C. Eukaryotic r - RNA □
to): (AIIMS 11, 08) D. Prokaryotic mRNA □
A. Polyproline □ 156.W h i c h o f t h e f o l l o w in g i s n o t a p o s t t r a n s c r i p t i o n a l
B. Polylysine □ m o d if i c a t i o n o f RNA?
C. Polyalanine □ A. Splicing (AI 03) □
D. Polyglycine □ B. 5'capping □
1 4 7 .S o m e mRNA h a s s e l f s p l i c i n g a c t iv it y . T h e r e m o v a l o f C. 3' polyadenylation □
in t r o n s is/ are d o n e b y : (PGI 11) D. Glycosylation □
A. Occur in intron that form Ribozyme □ 157.1n RNA, g e n e d u r in g p r o c e s s in g u n d e r g o e s a l l, e x c e p t

B. Spliceosome □ A. Chemical hydrolysis (AIIMS 04) □


C. Self-splicing introns □ B. Terminal addition □
D. sn RNA □ C. Nucleoside modification □
E. Lariats intermediates are formed D. Nucleoside cleavage □
1 5 8 .A P O B48 & APO B100 is synthesized from the same
1 4 8 .In tr o n s a r e e x is e d b y (PGI 05)
mRNA; the difference between them is due to:
A. RNA splicing □
A. RNA splicing (AIIMS 11) □
B. RNA editing □
B. Allelic exclusion □
C. Restriction endonuclease □
C. Deamination of cytidine to uridine □
D. DNAase □
D. Upstream repression □
E. Helicase □
1 5 9 .P o s t t r a n s c r i p t i o n a l m o d if i c a t i o n in c l u d e s : (PGI 13)
149.A s e g m e n t o f a e u k a r y o t ic g e n e t h a t i s n o t r e p r e s e n te d
A. All RNA undergo post transcriptional modification□
i n th e m a tu r e m e s s e n g e r RNA i s k n o w n a s : (AI 04)
B. Capping of the pre-m RNA Involves the addition of
A. Intron □
7-methylguanosine to the 5'end □
B. Exon □
C. Poly A tailing occur at 3'end □
C. Plasmid □
D. Intron excision by spliceosome □
D. TATA box □
E. Primarily occur in cytoplasm □
1 5 0 .S p lic in g A c t iv it y is a f u n c t i o n o f :
1 6 0 .T ru e a b o u t r i b o z y m e s is : (AIIMS 15,12)
A. mRNA (AI 10, AIIMS 10) □
A. Cut DNA at specific site □
B. sn RNA □ B. GTPase activity □
C. r RNA □ C. Peptidyl transferase activity □
D. t RNA □ D. Participate in DNA synthesis □
151.W h i c h i s in v o lv e i n p r o c e s s in g o f p o s t - t r a n s c r i p t i o n a l 161.True about ribozymes are A/E (PGI 12, DNB 11)
m odification o f mRNA: (PGI 11) A. Catalytic activity □
A. Lariats formation □ B. Involved in transesterification □
B. Splicing □ C. Hammerhead metallo enzyme □
C. Capping at 3'end □ D. Peptidyl transferase □
D. Poly tail(A) formation at5'end □ E. Deamination □
E. Methylation □ 1 6 2 .A c ito m y c in D i n t e r f e r e s w i t h e n z y m e i n d u c t i o n b y
1 5 2 .S o m e m R N A h a s s e l f s p l i c i n g a c t iv it y . T h e r e m o v a l o f c o m b in in g w ith - (UPSC 03, AIIMS 04, AI 08)
i n t r o n s is/ are d o n e b y : (PGI 15,11) A. TRNA □
A. Occur in intron that form Ribozyme □ B. DNA □
B. Spliceosome □ C. R-RNA □
C. Self-splicing introns □ D. Repressor protein □
D. sn RNA □ E. Negative feedback inhibition system □
E. Lariats intermediates are formed □ 163.W h i c h o f t h e f o l l o w in g b l o c k s DNA r e p li c a t i o n w ith
1 53.C y t o p l a s m i c p r o c e s s d u r in g p r o c e s s in g is g e t t i n g in c o r p o r a t e d i n DNA s t r a n d : (PGI 07)
A. 5' capping (SG PG I05, CMC 03) □ A. Cytarabine □
B. Poly (A) tailing □ B. Nalidixic acid □
C. Methylation of tRNA □ C. Ciprofloxacin □
D. Attachment of CCA in tRNA. □ D. Paclitaxel □
1 54.A l l a re p r o c e s s in g r e a c t i o n s i n t-RNA e x c e p t (TN-02) 1 6 4 .T r a n s c r ip t o n is i n h i b i t e d b y : (PGI 06)
A. CCA tailing (WB-03, Delhi- 04, UP-05) □ A. Actinomycin D □
B. Methylation of bases □ B. Amanitin □
C. Poly (A) tailing □ C. Chloramphenicol □
D. Trimming of 5' end □ D. Streptomycin □
E. Puromycin □
480 ■ A Complete Review of Short Subjects

165.Inhibitors of DNA synthesis - (DNB 03, Bihar 08) Translation (Protein Synthesis)
A. Penicillin □
B. Polymyxin □ 1 7 5 .S y n th e s is o f r R N A t a k e s p la c e in - (NBE P 15,13)
C. Chloramphenicol □ A. Cytosol (AIIMS 1 4 ) 0
D. Actinomycin □ B. Nucleus □
166.hn RNA seen in : (AI 07) C. Nucleolus □
A. Spinal muscular dystrophy □ D. Mitochondria □
B. Sickle cell disease □ 1 7 6 .T h e c e l l u l a r c o m p o n e n t f o r p r o t e in s y n t h e s i s is OR
C. Hutchinson chorea □ T r a n s l a t i o n o c c u r s in : (AI 03, 04)
D. aThallasemia □ A. Ribosomes □
1 6 7 .A n e n z y m e t h a t m a k e s a d o u b le s t r a n d e d D N A c o p y B. Mitochondria □
f r o m a s i n g l e s tr a n d e d R N A t e m p la t e m o l e c u l e is C. Nucleus □
k n o w n a s: D. Cytoplasm □
A. DNA polymerase (AI 04) □ 1 7 7 .A m in o a c y l t-RNA is n o t r e q u i r e f o r - (NBE P 15)
B. RNA polymerase □ A. Proline □
C. Reverse transcriptase □ B. Lysine □
D. Phosphokinase □ C. Hydroxylysine □
1 6 8 .T ru e a b o u t r e v e r s e t r a n s c r ip t a s e : (PGI 03) D. Methionine □
A. Forms DNA from RNA □ 1 7 8 .T ru e r e g a r d in g a m in o a c y l t -R N A s y n t h e t a s e i s A/E
B. Forms RNA from DNA □ A. Isoaccepting t RNA (SGPGI 06) □
C. Important for replication of HIV □ B. Implement genetic code □
D. Amplification of DNA □ C. Attachment of amino group to 5' end □
E. DNA dependent RNA polymerase □ D. Editing function □
169.W h ic h o f t h e f o l l o w in g r e q u ir e t e m p la t e f o r it's 179.Which enzyme involved in translation is often
f o r m a t io n : referred to as ’Fidelity enzyme'; OR Fidelity of protein
A. Carbohydrate (PGI 01) □ translation depends on; OR In translation process,
B. Protein □ proof reading of tRNA is done by: (NBE P 15,14,13)
C. Lipid □ A. DNA polymerase (AIIMS 15,12, PGI 13) □
D. Nucleic acid □ B. RNA polymerase □
E. Phospholipid □ C. Amino acyl-t RNA synthetase □
170.The anticodon region is an important part of the D. Amino acyl-reductase □
A. r-RNA (Kerala 2K, KA 02, AP 03) □ 1 8 0 .P o ly p e tid e c h a in t e r m in a t io n i s e n h a n c e d b y :
B. m-RNa □ A. Stop codon (PGI 01) □
C. t-RNa □ B. Promoter □
D. hn-RNa □ C. Ribosomal unit □
17l.In transcription anticodon is seen in - (DNB 15,13) D. Pepitdyl transferase □
A. t-RNA □ E. UAA □
B. m-RNA □ 181. The hydrolytic step leading to release of polypeptide
C. r-RNA □ chain from ribosomes in catalysed by: (PGI 02)
D. DNA □ A. Stop codons □
1 7 2 .A b o u t t-RNA, TRUE is : (AIIMS 03) B. Pepitidyl transferase □
A. No intrabasal pairing □ C. Release factors □
B. Clover leaf pattern □ D. AUG codon □
C. Inverted T appearance □ E. Dissociation of ribosomes □
D. Each specific for one m-RNA □ 1 8 2 .A b o u t p e p t i d y l t r a n s f e r a s e tru e is (Jipmer 02)
1 7 3 .W h ic h ty p e o f RNA h a s t h e h i g h e s t p e r c e n t a g e o f A. Used in elongation & cause attachment of peptide
m o d if i e d b a s e ( m o d if ie d n u c le o t id e s ) ? OR W h ic h o f chain to A-site of t RNA (PGI 08) □
th e f o l l o w in g RNA h a s a b n o r m a l p u r in e b a s e s B. Used in elongation & cause attachment of peptide
A. mRNA (NBE P 15,13; AI 10; AIIMS 1 4 )0 chain to P site □
B. tRNA □ C. Used in initiation and cause 43S complex
C. rRNA □ formation □
D. snRNA □ D. Used in initiation & cause 48S complex formation □
1 7 4 .T h y m id y la te d R N A p r e s e n t in : (PGI 11) 1 8 3 .T e r m in a t io n i s c a u s e d b y a ll e x c e p t (AIIMS 06)
A. mRNA □ A. RF-1 □
B. rRNA □ B. UAA □
C. tRNA □ C. Peptidyl transferase □
D. 16-s-RNA □ D. 48 S complex □
Biochem istry: Structure, Function an d R eplication of Inform ational M acrom olecules ■ 481

1 8 4 .R ib o s o m e h a s f o l l o w in g e n z y m a t ic a c t iv it y ? B. N formyl methionine tRNA will be the first t - RNA


A. Peptidyl transferase (AIIMS 08) □ to come into action □
B. Peptidase □ C. EF2 shifts between GDP & GTP □
C. Aminoacyl tRNA synthetase □ D. Releasing factor releases the polypeptide chain from
D. GTPase □ the P site. □
1 8 5 .F o r 1 p e p t id e b o n d f o r m a t io n h o w m a n y h i g h e n e r g y 194.True about translation of protein is (PGI 08)
p h o sp h a te b o n d s a re r e q u ire d : (PGI 07) A. It has 3 steps initiation, elongation & termination □
A. 0 □ B. 1F-2 prevent reassociation of ribosomal subunit □
B. 1 □ C. IF-3 and 1A cause binding of initiating codon □
C. 2 □ D. IF 2 has a and |3 units □
D. 3 □ 195.43S preinitiation complex include all except:
E. 4 □ A. IF3 (AIIMS 03) □
186.W h ic h g r o u p o f a m in o a c id i s r e s p o n s i b l e f o r p e p t id e B. IF1A □
bond: (PGI 13) C. IF2P □
A. Amino group □ D. IF-4F □
B. Carboxyl group □ 196.IF4F include all except (PGI 05, JIPMER 09)
C. Side chain □ A. 4A □
D. Aldehyde group □ B. 4G □
E. Amide group □ C. 4E □
1 8 7 .T e r m in a t io n p r o c e s s o f p r o t e in s y n t h e s i s i s p e r fo r m e d
D. 4S □
by: (PGI 10) 197.Which of the follow ing does not undergo
A. Releasing factor □ phosphorylation by protein kinases in Eukaryotes:
B. Stop codon □ A. Threonine (AI 1 2 ) 0
C. Peptidyl transferase □ B. Tyrosine □
D. UAA codon □
C. Serine □
E. AUG codon □ D. Asparagine □
188.W h i c h o f th e f o l l o w in g is n o t r e q u ir e d f o r p r o t e in
198.Component of 50s subunit is/are: (PGI 11)
s y n t h e s i s ( in tr a n s la tio n ) o f e u k a r y o t e s : (AIIMS 11)
A. 23s □
A. RNA polymerase □
B. 28s □
B. Ribosomes □
C. 5s □
C. Peptidyl transferase □
D. 5.8s □
D. Amino acyl tRNA synthetase □
E. 16s □
1 8 9 .A m b e r c o d o n r e f e r s to (AIIMS 01)
199.Components of 60 S subunit of ribosome are:
A. Mutant codon □
A. 5.8 S (PGI 0 9 ) 0
B. Stop codon □
B. 23 S □
C. Initiating codon □
C. 1 6 S □
D. Codon for m ore than one am in o a cid s □
D. 18 S □
1 9 0 .S h i n e - D a l g a m o s e q u e n c e i n b a c t e r ia l mRNA is n e a r :
E. 28 S □
A. AUG codon (A I0 4 )D
200.True about ribosomes: (PGI 09)
B. UAA codon □
A. Conserved in nature □
C. UAG codon □
B. Role is to bring t-RNA and m-RNA together □
D. UGA codon □
C. DNA forms RNA and rotein are formedfrom RNA □
1 9 1 .tR N A f m e t w o u ld r e c o g n iz e : (PGI 09)
D. They are always free □
A. AUG □
E. Have two subunits □
B. UGC □
201.Regarding ribosome all is true except
C. GUG □
A. 16 S ribosome identifies shine- dalgamo sequence □
D. GCU □
B. Cytosolic ribosomes synthesize proteins of
E. UUG □
peroxisome (]ipmer-04, CMC-05) □
1 9 2 .F a ls e a b o u t e u k a r y o t ic p r o t e in s y n t h e s i s is :
C. Large subunit catalyzes peptide bond formation □
A. N formyl Met is the first-RNA to come into action □
D. Binding of aminoacyl tRNA uses 2ATP □
B. mRNA read from 5' to 3' (AIIMS 09) □
202.Vitamin required for post translational m odification of
C. Ef2 shift between GDP to GTP □
coagulants is : (AI 07)
D. Capping helps in attachment of mRNA to 40 S
A. Vitamin A □
ribosome □
B. Vitamin C □
1 9 3 .R a g a r d in g C y t o s o l ic E u k a r y o t ic g e n e e x p r e s s io n f a ls e
C. Vitamin B6 □
is : (AI07)
D. Vitamin K □
A. Capping helps in attachment of mRNA to 40 S
Ribosome □
402 ■ A Complete Review of Short Subjects

Immunoglobin 212.Sequence which is responsible for retaining proteins


in m e m b ra n e (UP 05, AMU 06, Delhi 03)
20 3 . Correct pair of Ig subclass with their heavy chain: A. Translocon □
A. IgM -ex (alpha) (PGI 10) □ B. Sec 61 complex □
B. IgG -y (gamma) □ C. Docking protein □
C. IgM - 5 (delta) □ D. Halt signal □
D. IgE- y- (gamma) □ 21 3 .N o t tr u e a m o n g t h e f o l l o w in g i s (PGI 05, TN 06)
E. IgA -y (gamma) □ A. Sec 61 translocon complex form passage way □
2 0 4 .C o rre c t o r d e r o f s w i t c h i n g o f Ig : (PGI 10) B. SRP-R is docking protein □
A. IgM to IgG □ C. SRP blocks elongation □
B. IgA to IgG □ D. SRP-R releases elongation block □
C. IgM to IgA □ E. SRP-R is ATP bound □
D. IgM to IgD □ 214.W h i c h o f th e f o l l o w in g g r o u p s o f p r o t e in s a s s is t i n t h e
E. IgE to IgG □ f o l d i n g o f o t h e r p r o t e in s ? (NBE P 14; AI 08, 04)
2 0 5 .T ru e a b o u t IgM (PGI 10) A. Pro teases □
A. Fix complement □ B. Proteosomes □
B. Increased in primary response □ C. Templates □
C. Fab region is composed of variable region □ D. Chaperones □
D. Fc is antibody -binding portion □ 2 1 5 .T h e p r im a r y r o le o f c h a p e r o n e s i s to h e lp in :
2 0 6 .T ru e a b o u t Ig g e n e r e a r r a n g e m e n t & Ig d iv e r s ity : A. Protein synthesis (NBE P 13; AI 03) □
A. Somatic mutations theory (PGI 1 0 ) 0 B. Protein degradation □
B. One loop and two loop joining theory □ C. Protein denaturation □
C. DNA rearrangement □ D. Protein folding □
D. Appropriate class switching □ 216.F o ld in g p r o t e in a r e : (PGI 09)
2 0 7 .T ru e a b o u t l o c a t i o n o f I g c h r o m o s o m e : (PGI 10) A. GLUT-1 □
A. k- 2 □ B. Calnexin □
B. k - 6 □ C. Cytochrome 450 □
C. k - 1 4 □ D. Insulin receptor □
D. 1 - 2 2 □ E. Protein disulfide isomerase □
E. X- 10 □ 217.A11 a re t r u e a b o u t c h a p e r o n e s e x c e p t- (NBE P 15)
208. What factor is responsible for deciding whether an A. Cause folding of proteins □
antibody w ill remain membrane bound or get B. Are lipid in nature □
secreted? OR Synthesis of an immunoglobulin in C. May have ATPase activity □
membrane bound or secretory form is determined by: D. Include heat shock proteins □
A. RNA splicing (AIIMS 14, 08; AI 11) O 2 1 8 .T ru e s t a t e m e n t a b o u t c h a p e r o n e s : (PGI 15,12)
B. Class switching □ A. Belong to heat shock proteins □
C. Differential RNA regulation (processing) □ B. Wide range of expression □
D. Allelic exclusion □ C. Present from bacteria to human □
___________Intracellular traffic & Sorting___________ D. Ubiquitin is the most important chaperones □
E. Also k/a stress proteins □
2 0 9 .S e c r e to r y p r o t e in s a r e s y n t h e s iz e d in : 2 1 9 .P r o te in s ta r g e te d f o r d e s t r u c t io n i n e u k a r y o t e s a re
A. Cytoplasm (AIIMS 05) □ c o v a le n t ly l in k e d to : (AI 04)
B. Endoplasmic reticulum □ A. Clathrin □
C. First in cytoplasm and then in Endoplasmic B. Pepsin □
Reticulum □ C. Laminin □
D. First in Endoplasmic Reticulum and then in D. Ubiquitin □
cytoplasm □ 220.P r o t e in s a r e s o r te d b y : (AI 08)
.Signal peptides are synthesized in (Jipmer 06, TN 05)
A. Golgi Bodies □
A. ER □ B. Mitochondria □
B. Free ribosomes □
C. Ribosomes □
C. Membrane bound ribosomes □
D. Nuclear Membrane □
D. Golgi body □
221.N o t a d is o r d e r o f p r o t e in m is f o ld i g ? (AI 11)
.Elongation arrest occurs due to (AP 05, KA 05, WB-06)
A. Alzheimer's disease □
A. SR P-R □
B. Tuberculosis □
B. SRP □
C. Cystic fibrosis □
C. Signal peptide □
D. CJD (Creutzfeld-Jakob disease) □
D. Docking protein □
B iochem istry: Structure, Function and R eplication of Inform ational M acrom olecules ■ 483

222.The secondary structure of prion particles is: B. 6 □


A. Alfa helix (AIIMS 08) □ C. 11 3
B. Beta bends □ D. 12 □
C. Beta sheets □ 231.O n e o f t h e f o l l o w i n g m u t a tio n is p o t e n t i a l l y l e t h a l - a)
D. Beta turns □ A. Substitution of adenine for cytosine (Delhi 10) G
2 2 3 .D e fe c t i n f o l d i n g o f t h e p r o t e in r e s u l t i n w h i c h o f th e B. Substitution of methylcytosine forcytosine □
f o l l o w in g c l i n i c a l d is e a s e : (AIIMS 02) C. Substitution of guanine for cytocine □
A. Kuru □ D. Insertion of one base □
B. Migraine □ 2 3 2 .S ic k le c e l l a n e m ia is t h e c l i n i c a l m a n i f e s t a t i o n o f
C. Hypothyroidism □ homozygous genes for an abnormal haemoglobin
D. Myopia □ molecule. The event responsible for the mutation in
224.I f c e l l u l a r p r o t e in s d o n o t f o l d in t o a s p e c if i c the B chain is - (DNB 10)
conformation, their function are affected. Certain A. Insertion □
disorders arise, if specific proteins are misfolded. B. Deletion □
Which of the follow ing disorders arises due to C. Nondisjunction □
conformational isomerization: (AIIMS 05) D. Point mutation □
A. Familial fatal insomnia □ 2 3 3 .N u ll m u t a t i o n is : (AI 04)
B. Hepatitis delta □ A. Mutation occuring in Non Coding region. □
C. Pernicious anemia □ B. Mutation that does not change theamino acid or end
D. Lesch-Nyhan syndrome □ product □
C. Mutation that codes for a change in progeny without
Gene Duplication any chromosomal change □
D. Mutation that leads to no functionalgene product □
225.Gene duplication plays an important role in the 234.A m u t a t io n i n th e c o d o n w h i c h c a u s e s a c h a n g e in th e
evolution of c o d e d a m in o a c id , i s k n o w n a s:(AIIMS 02)
A. mRna (AIIM S 11) □ A. Mitogenesis □
B. rRna □ B. Somatic mutation □
C. tRna □ C. Missense mutation □
D. hnRna □ D. Recombination □
2 3 5 .S o m a tic m u t a t io n s in v o lv e : (PGI 02)
Mutation A. Deletion □
B. Plasmid mediated □
(AIIMS
2 2 6 .S m a ll s c a l e m u t a tio n s in c l u d e a ll e x c e p t: 12) C. Frame-shift alteration in coding nucleotides □
A. Substitution □ D. Trinucleotide repeat mutations □
B. Deletion □ E. Non-sense alteration of DNA □
2 3 6 .F ra m e s h i f t m u t a t io n d o e s n o t e f f e c t c o m p l e t e a m in o
C. Paracentric inversion □
D. Insertion □ a c id s e q u e n c e i f it o c c u r s i n m u l t i p l e o f - (NBE P 15)
2 2 7 .B a s e s u b s t i t u t io n o f GAC (A s p ) to GAG (Glu) i s a n
A. 1 □
e x a m p le o f : (PGI 11) B. 2 □
A. Point mutation □ C. 3 □
B. Silent mutation □ D. None □
2 3 7 .B a s e s u b s t i t u t io n m u t a t i o n s c a n h a v e b e f o l l o w in g
C. Non-sense mutation □
D. Conserved mutation □ (AI 06)
m o l e c u l a r c o n s e q u e n c e e x c e p t:

E. Non-conserved mutation □ A. Changes one codon for an amino acid into another
228.W h i c h o f t h e s e i s a c o n s e r v a t iv e m u ta to r :
codon for that same amino acid □
A. Glutamic acid- glutamine (AIIMS 08) □ B. Codon for one amino acid is changed into a codon of
B. Histidine-glycine □ another amino acid □
C. Alanine- leucine □ C. Reading frame changes downstream to the mutant
D. Arginine-aspartic acid □ site □
229.I n s i c k l e c e ll a n a e m ia , th e d e f e c t c a n b e e x p l a i n e d a s
D. Codon for one amino acid is changed into a
h a v in g a r i s e n f r o m - (PGI 04, UP 02) translation termination codon □
2 3 8 .T r a n s itio n m u t a t io n o f GATCCT is : (PGI 06)
A. A base insertion in DNA □
B. A base deletion in DNA □ A. GGTCCT □
C. A base substitution in DNA □ B. GTTCCT □
D. None of the above □ C. GABCCT □
2 3 0 .H b s h a s d e f e c t i n a m in o a c id p o s i t i o n - (DNB 14,12) D. GrUGGT □
A. 4 □ E. GTTUCT □
484 ■ A Complete Review of Short Subjects

239.Trinucleotie repeats are found in: (PGI 09,15) 245.In sickle cell anemia replacement is: (P G I 08)
A. Huntington's disease □ A. Valbglu in b 6 □
B. Spinocerebellar ataxia □ B. Glu by val in b 6 □
C. Amyotropic latera sclerosis □ C. Hist val b 8 □
240.W hich of the following is an example of trinucleotide D. 6A □
repeat m utation- (NBE P 15) 2 4 6 .S ic k lin g i n 'H b S' d is e a s e i s p r im a r ily c a u s e d b y
A. Huntington's chorea □ A. Decreased Solubility (AI 09) □
B. Fragile-x-syndroem □ B. Decreased Stability □
C. Friedreich ataxia □ C. Altered function □
D. All fo the above □ D. Altered O 2 binding capacity □
241.In a mutation if valine is replaced by which of the 247. All of the follow ing are true about Sickle cell disease,
following would not result in any change in the Except (AI 09, AIIMS 11,10)
function of protein: A. Single nucleotide change results in change of
A. Proline (AIIMS 02) □ Glutamine to Valine □
B. Leucine □ B. RFLP results from a single base change □
C. Glycine □ C. 'Sticky patch' is generated as a result of replacement
D. Aspartic acid □ of a non polar residue with a polar residue □
242.W hich of the following can be a hom ologous D. HbS confers resistance against malaria in
substitution for valine in hem oglobin? (AI 04) heterozygotes □
A. Isoleucine □ 248. Technique (s) used to detect Gene Mutation is/are
B. Glutamic acid □ A. RT-PCR (PGI 1 2 ) 0
C. Phenylalanine □ B. Denaturing gradient gel electrophoresis □
D. Lysine □ C. DNA sequencing □
243.True about silent m utation in gene: D. Restriction fragment polymorphism (RFLP) □
A. No change in mRNA (PGI 09) □ E. Single-strand conformational polymorphism □
B. No change in Amino acid sequence in protein □ 249. All the following can be used to detect mutation,
C. No expression of protein □ except: (AI 02)
D. No change in expression of protein □ A. Single strand conformational polymorphism □
E. Termination of protein synthesis □ B. Ligase chain reaction □
244.True about Sickle cell disease are all, Except: C. Polymerase chain reaction □
A. Single nucleotide change results in change of D. DNA Sequencing □
Glutamine to Valine (AIIMS 1 0 ) 0
B. Deoxygenated Hb 1/1 exposure of sticky end d/t
replacement of nonpolar residue by polar residue □
C. HbS confers resistance against malaria in
heterozygotes □
D. RFLP results from a single base change □
Biochemistry: Structure, Function and Replication of Informational Macromolecules * 485

ANSWERS & EXPLANATIONS:

Purines and Pyrimidines


1. C i.e. Aspartate transcarbom oylase
H 20 zauk + ri +
Glutamate Carbamoyl
Glutamine Aspartic
phosphate
+ 2A TP + C 0 2 acid (AA)
(l) C a rb an o yl p h o sp h ate (CAP)

r
S v n th etase II
GAS = GAC (cytosol)
1
Activated by Inhibited by
Rate ©
I I limiting
A s p arta te
ATP UTP transcarbam o ylase
step

[
PR PP inhibited by
CTP&UTP
inhibition reversed
by ATP
NADH
+ H NAD + H 20 H+
Orotic Di Hydro
Carbamoyl Aspartic Acid
Acid Orotic Acid . ^ ^ (CAA)
(OA) D ih yd ro o ro ta se (DHOA) G D ih yd ro -o ro tase
d eh ydrogena se
(inner mitochondiral membrane;
a ll o th er enzym es are cytoso lic) Enzyme 1, 2, and 3 are three different catalytic
domains or a single CAD polypeptide chain
Enzym es 5 and 6 are catalytic domains of single
- bio lo g ic a lly irreversible polypeptide chain called UMP- synthase. Its low
O rotate reaction activity results in orotic acid urea. Deficiency
p h o s p h o rib o s y l / of both 5 & 6 ca uses type I and deficiency of only
transferase 6 l/t type II aortic aciduria
can use a llo p u rin o l &
" 5 -F U as substrate

Inhibited by 6 -Azauridine
(6-azauridylate) & allopurinol product
of enzyme 5 l/l oritidinuria & Orotic aciduria)

ATP ADP
© O M P ■ d ec arb o xylase Uridine 5 '
Orotidine 5 * monophosphate , UDP
Monophosphate
(OMP) = Orotidylic acid (UMP)* N ucleoside
M onophospho
C02
kinase
’ 1st pyrimidine nucleotide
formed

2. D i.e. Reye syndrom e decrease cytosolic carbam oyl phosphate


3. A i.e. N5 - N10 m ethylene tetrahydrofolate
[R ef: H arper's 28/e P- 298; C hattergea 7/e p. 2 1 4 -1 7 ; L ippin cott 5/e p. 3 0 2 -5 ; L ehn in ger 5 /e p. 8 9 2 -9 4 /

d-UMP (deoxy-uridine monophosphate) is


converted to d-TMP (deoxy-thymidine N A D PH NADP
monophosphate) w hich uses N5, N10 -
methylene tetra hydrofolate as a source of UDP deoxy UDP
methyl group (1 carbon & 2 hydrogen)Q.). R ibonucleotide
reductase
This is an unusual reaction in that THF ATP

(tetrahydrofolate) contributes not only 1


ADP J d-U M P
carbon unit but also 2 hydrogen atoms from SFU
pteridine ring, which causes oxidation of
^ N 5, N 10, m ethylene T H F
U TP
THF to DHF. Disorders of folate & vitamin 5 F dU M P
T h ym id ylate B 1 2 (methyl)
B 12 metabolism cause deficiency of TMP. Glulam ine ATP o s ynthase
Thymine analog, antitumor drug 5- Dihydro (H2) Folate
C TP synthase
flurouracil (5Fu) is converted to 5-F dUMP, Glutamate ^ V^ADP M tx D ihydro
T h y m id in e m ono folate
which becomes permanently bound to the phosph ate (TM P) G reductase
inactive thymidylate synthaseQ. So 5FU is a C ytid in e
THF
suicide inhibitor. Methotrexate (Mtx) and trip h o s p h a te (C TP )
antibacterial trimethoprim inhibits
dihydrofolate reductase, an enzyme which
reduces DHF to THFQ.
486 ■ A Complete Review of Short Subjects

4. D i.e. Allantoin 5. B i.e. Uric acid

End product of catabolism of purine (adenosine & guanosine) nucleotides

In hum ans In m am mals other than higher primates


I I
Uric acid*? (lack uricase enzyme) Allantoin*? (uricase converts
uric acid to allantoin)

6. C i.e., Adenylic & guanylic acid 7. A i.e. Due to increased m etabolism of pyrim idines 8. A i.e. Purine metabolism
9. A i.e. Abrupt increase in serum urate levels is more com m on a cause for acute gout than an abrupt fall in urate levels
10. B i.e. Raised uric acid in synovial fluid of joint [R ef: H arper's 28/e p .292-99; Lippincott's 5/e P -294-302 ; H arrison 16/e P 2046]j

( - Gout is a metabolic disorder of purine catabolism^, and characterized by high level of uric acid in the blood . Uric acid
^
is end product of purine catabolism^
- Pyrimidine nucleotide unlike purine rings can be opened & degraded to highly water soluble structure like, P- alanine,
P- amino isobutyrate, NH 3 & CO 2. Excess meabolism of pyrimidine therefore does not give rise to clinical
manifestation*?
- Human catabolizes purine (Adenine & Guamine) to uric acid, which is relatively insoluble acid.

11. A i.e. HG PRTase 12. A ie. Hypoxanthine phosphoribosyl transferase (HGPRTase) 13. D ie. Serum Uric Acid Levels
[R ef: L ehn in ger 5/e p. 893, 882; L ippin cott 4 lh/e p 296-97; H arrison 1 7 /2 4 4 7 -4 8 ; N elson 18/e p-629-30; H arp er 2 8"’/ed pg- 298-99]

Lesch - Nyhan syndrome is an X- linked, recessive disorder associated with a complete deficiency of hypoxanthine H
guanine phosphoribosyl transferase (HGPRT)Q and therefore has the inability to salvage hypoxanthine or guanine,
from which excessive uric acid are produced. They are poorly coordinated, mentally retarded, extremely hostile male
children who show compulsive self aggressive and self destructive tendencies; they mutilate themselves by biting
off their fingers, toes and lips'?. The features of hyper ureamia & gout are also presented.
Uric acid (urate) levels in urine and plasma (serum) is the key initial investigation in purine metabolism disorder
(like Lesch Nyhan syndrome) and should be measured to avoid missing the diagnosis'?.
V-

14. A i.e.Purine synthesis [Ref: Harper's 27/e P-301]

All forms of life (except protozoa) synthesize purine and pyrimidine nucleotides

15. D i.e. Thiam ine [Ref: Chatterjea 6 /e P-159; Harper's 27/e P-496-97]
--------------------------------------------------------------------------------------------
• Usually purine & pyrimidine occur in nucleic acid i.e. DNA and RNA
• Besides nucleic acid pyrimidines also occur in thiamine*?
• Purine is also a structural component of number of essential coenzyme:

- Coenzyme A - NAD+
- NADP+ - FAD+

Chem istry of thiamine (Vit Bi): Free thiamine is basic substance and contains: a) Pyrimidine*?; b) Thiazole ring;c) Sulphur

16. B i.e. Liver

- Liver is major site for denovo purine nucleotide biosynthesis'?. It does not use salvage pathway*?.
- Brain, RBC and polymorphonuclear leukocytes can not synthesize purine by denovo pathway. So they use salvage
pathway of purine synthesis'? (i.e. use exogenous purines formed by degradation of purines synthesized in liver).
Biochemistry: Structure, Function and Replication of Informational Macromolecules ■ 487

DNA Structure

17. A i.e. All nucleotides are involved in linkage; B i.e. Antiparallel; D i.e. Bases perpendicular to DNA
18. E i.e. A single strand of DNA 19. D i.e. Uracil 20. B i.e. H ydrogen bond
21. C i.e. Deoxyribose-phosphate backbone with bases stacked inside 22. C i.e. B
[R ef: H arp er 2 8/ed 289-90, 302-304; C hattarjae 7/e 225-27; L ippin cott 5/e 396-98; L ehn in ger 5/e 2 7 7 -8 2 1

i
The most common form of DNA is B- DNA*?. Within the cells most of DNA is B -D N A , although regions rich in
guanine and cytosine base pairs m ay assum e Z- conformation.

23. A i.e. Hoogsteen pairing [R ef: H arrison 17lh j!7 6 4 ; H arrison's 18lh/2312] _A_

Hoogsteen (non W atson-Crick) hydrogen pairing allows formation of DNA triplex'?; which is also facilitated by long
only pyramidine or only purine sequences in a given strand, 2Py + lP u or 2Pu + 1 Py strands, low pH, increased pKa
and protonated cytosine.
DNA helix bends w henever ^4 adenosine (A) residues appear sequentially in row in one strand (6 A produce 18°
bend). Hairpins or cruciform (cross shaped) structures are formed when self complementary palindromic sequence
(of DNA or RNA) form intrastrand base pairing in single or both strands respectively. Four stranded DNA (tetraplex
or quadruplex) occurs only for DNA sequences with very high proportion of guanosine residue (or very stable
guanosine /G tetraplex).

24. A i.e. A+G = T+C 25. D i.e. Double stranded DNA


26. D i.e. 27% [R ef: L ehn in ger 5/e p. 278; L ippincott 6/e p. 291, 397; S atyan arayan 3 / e p. 73]

Chargaff's Rules for Double stranded (ds) DNA

In all cellular DNAs, regardless of species- The base composition of DNA generally varies
from one species to another
number of adenosine residues is equal to num ber
DNA specimens isolated from different tissues
of thyrm idine residues (i.e. A=T)
of same species have the same base composition
number of guanosine residues is equal to the
The base composition of DNA in a given species
num ber of cytidine residues (G = C)
does not change with an organism's age,
From these relationships it follows that sum of purine nutritional state or changing environment.
residues equals the sum of pyrim idine residues;
Because in DNA, A=T and G=C; and A is 23'>. f
A+G=T+C thymine (T) will also be = 23%. T h i s makes 46'.-;
(23+23). The remaining 54% of DNA will be
Ratio of purine to pyrimidine is always around 1 guanine (G) and cytosine (C). But because the
A+G amount of G is equal to C each G & C will be 54/
T +C -2 7 % .

27. B i.e. C- G 28. B i.e. G = C IR ef: H a rp er’s 27/e P-311 ]

y
Thermostability in DNA is constributed mostly by C = G<? because there are 3 Hydrogen bonds between C & G<?.
G = C bonds are much more resistant to denaturation or melting than A= T rich region (2 Hydrogen bonds)

29. B i.e. 5CUUAAG3 30. A i.e. 5'G TA CG C TT A A '3' [R ef: H arper 2 9 /e p-3 4 3 -4 6 ; L ippin cott 5 /e p 396-98]

Complementary DNA sequence of 5' TTAAGCTAC3' is 3'AATTCGCATG5' or 5'GTACGCTTAA3'.


Complementry RNA sequence of 5CTTAAG3 template strand of DNA is 3GAAUUC5 or 5CUUAAG3
488 ■ A Complete Review of Short Subjects

Base Pairing Rule And Com plem entary Sequence

- Cytosine (C) is always Com plem entary sequence of 5'-3' end always 5'CTTAAG3'
paired with guanine (G) in proceeds from 3'- 5. Thus
y Complements
both DNA & RNA.
- Adenine (A) is always 5' T T A A GCG T A C 3' 3'GAATTC5' (in DNA) or
paired with Thiam ine (T) 3' GAAUUC 5' (in RNA)
in DNA and Uracil (U) in Complements
RNA (ie DNA-RNA hybrid) ^ Can be rewritten as

3'AATTCGCATG5' 5'CTTAAG (in DNA)


C=G
^ This can also be rewritten in 5'CUUAAG (in RNA
A = T (in DNA) /U (in
\ 5'- 3' as the following: hybrid)
RNA-DNA hybrid)
5' G T ACGC TT AA 3'

31. C i.e. Runs at 5 ' - 3 ' direction [R ef: H arper's 27/e P -348-350]
32. D i.e. M atches with RNA transcript that encodes protein.

In double stranded DN A the 2 strands are

Template strand Coding strand


- The strand that is copied into RNA molecule - The other strand (other than Template) is coding
- The information of this strand is read in 3' —> 5' strand because it m atches the RNA transcript that
direction^. encodes proteinQ
- The template strand for each gene will not necessarily be - The inform ation is read in 5' -¥ 3' direction^
the same strand of DNA double helix.

Minus stran d0: - Is synthesized from RNA


- Acts as a template for synthesis of plu s stran d
- Most retroviral recombination occur during minus strand synthesis.
Thus plus & Minus strands are met with replication of retrovirus.

33. E i.e. 5'- G U ACG UA A-3' [R ef: L ippincott 4»'/ed p. 419, 430]

• The template strand (5'-TTACGTAC-3') is read in 3'-5' direction and RNA product after transcription has sequence
that is complementary to the sequence of template strand but Thym ine (T) is replaced by Uracil (U) in RNA.
» The coding strand of DNA is complementry to its template strand. So it matches the RNA transcript except that T is
replaced by U. The information is read in 5 '—>3' direction. So
Biochemistry: Structure, Function and Replication of Informational Macromolecules ■ 489

35. B i.e. Negatively Charged [R ef: R on ald H ofm an h em atology 2/e P -2202]

DNA molecules are negatively charged®, this is evident as it is stained by a cationic dye (Methylene blue).

36. A i.e. M aternal inheritance; C i.e. Not highly conserved & has high m utation rate; D i.e. M itochondrial disease is ..
37. A, B, D, E i.e. UGA codes for tryptophan, Codes for 13 protein, Circular double stranded DNA, M itrochondrial disease
occur due to Point M utations and Large-Scale Rearrangem ents
38. B i.e. M itochondrial DNA 39. C i.e. M itochondrial disease 40. B i.e. M aternally inherited
41. A i.e. Closed Circular [R ef: L ehn in ger 5 ,h/ed p g-738-42; H arp er 28"'/ed p g -3 1 9 -2 0 ; R obbin 's 7/e p. 185; Teitz 6,hjcd pg-269;
E berhard: C olou r atlas o f gen etics 3/ed p g-134; A cta N eu ropathol (2006) 111: pg -6 1 0 -1 6 ]

- Mitochondrial DNA (mt DNA) is a closed circular double stranded® molecule that is transmitted by maternal
inheritance®and contains no or very few interons (untranslated sequences)®. It is not highly conserved has high
mutation rate (d/t point mutation, large scale rearrangements & som e deletions) and leads to m itochondrial
m yopathies, MELAS, Leber's hereditary optic atrophy, Leigh syndrome and NARP syndrome®.
- Mt DNA has 37 genes encoding. 2mt rRNAs, 22 mt tRNAs and 13 proteins; with a slightly different genetic code i.e.
UGA is tryptophan (not stop codon), AUA is methionine (not isoleucine) and AGA/AGG are stop codons (not
arginine).

42. A i.e. Virus; B i.e. Chloroplast; C i.e. Bacteria; D i.e. M itochondria [R ef: G reew ood 16/e p 10-11; A n an than arayan a 8/e p 10-13;
ja w etz 25/e p 2-4- 97- 100]

Circular DNA is found in viruses (eg polyoma virus has double stranded & porcine virus has single stranded circular
genome), prokaryotes (most bacterial DNA is circular), blue green algae as well as in eukaryotic cells like
mitochondrial DNA or plasmid DNA and in chloroplast.
A distinguishing characteristic of prokaryotes is their capacity to exchange small packets of genetic information carried
on plasmids. Plasmid are sm all, specialized genetic elem ent that are capable of replication within at least one cell line
or exhibit broad hoast range and may carry sets of specialized genetic information (eg drug resistance) through a
population. Plasmids were identified as small genetic elements carrying genes and capable o f in depen den t replication in
p rokary otes (bacteria & yeast). Unlike viruses, plasmids are naked DNA and donot carry genes necessary to encase
genetic material for transfer to new host. Sometimes plasmids may be found in eukaryotes like saccharomyces
cerevisiae). In genetic engineering, plasmid is a small genetic (DNA) element that is separate fro m an d can replicate
in depen den tly o f the chrom osom l D N A . So introduction o f D N A restriction fra g m en t in to a plasm id allow s the fra g m en t to be
am p lified m any tim es.

43. D i.e. 30,000 44. A i.e. DNA [R ef: H arper's 28/e P -316-18; L ehn in ger 5/e p. 950]

Hereditary information is preserved in DNA. Thus a ll genetic diseases are due to change in DNA®.

45. C i.e. Telom ere; D i.e. Centrom ere; B ie. M icrosatelite DNA [R ef: H arp er 2 8 /e 3 15-19; Lipincott 5/e 475; L ehn in ger 952-53]

Highly repetitive sequences also called simple sequence DNA or simple sequence repeats (SSR) or satellite DNA is ☆
mostly clustered in centromeres and telomeres®.

46. A i.e. They are present at the ends of eukaryotic chrom osom e; B i.e. Increased telom erase activity favours cancer cells.
47. B i.e. Longetivity of cell- aging
48. A i.e. Telom erase 49. C i.e. Telom erase 50. B i.e. Somatic
[R ef: L ehn in ger 5/e 1053-55, 953; H arp er 28/e 315-19; R obbin s 7/e p 308-9; L ippin cott 5/e 408-7; G an on g 22/e p 20]
r ---------------------------------------------------------------------------------------------------------------------------------- — -------
Telomerase is a multisubunit RNA containing complex related to viral RNA- dependent DNA polymerase (reverse
transcriptase), the enzyme which is responsible for telomere synthesis & thus for maintaining the length of telomereQ.
Telomerase, a RNA dependent DNA polymerase (reverse transcriptase), is the enzyme responsible for telomere
synthesis and thus for maintaining the length of telomereQ. Germinal (germ line), cancer and hemopoetic stem cells®
with telomerase activity do not under go cellular aging and apoptosis (death). Whereas, somatic cells without telomerase
activity undergo cellular aging and death after 40-60 replication cycles d/t critical shortening of telomere._______________
490 ■ A Complete Review of Short Subjects

si B i.e. P rese n t in s id e the nucleus; D i.e. Basic 52. B i.e. Increased Euchromatin formation
53. A i.e. D N A ; E i.e. Histones 54. A i.e. Histone 55. D i.e. Glycosylation
5b. A i.e. U n c o n d e n s e d 57. A, B i.e. Histone, DNA 58. C i.e. Regular repeating structure of DNA & histone..
I Ref: Cim tterjea 8 / p 83, 240; H arper 28/ed 312-13; L ehn in ger 5/e 963-66; L ippincott 5/e 409-11, 422; Satyanarayan 3/e p 79;
\'a su d n w i 6/e 471-72]

- Histones are small basic proteins rich in basic am inoacid arginine & histidine w ith alkaline isoelectric pH. It is a
sim p le protein® ie its complete hydrolysis only yields amino acids. They are soluble in water, salt solution & dilute acids
but not in ammonia and do not easily coagulate on heating. Histones are synthesized in cytoplasm and migrate to
n u cleu s to form conjugated nucleoprotein (chromatin) with nucleic acid (DNA)®. Histones act as repressor of template
activity of DNA in RNA synthesis. Histones also conjugate with porphyrins. Globin, the protein part of hemoglobin is
an atypical histone and it has predom inance o f histidine & lysine instead o f arginine.
- Nucleosome, th e fundam ental organizational units o f chrom atin (chromosome), consists of regular repeating "beads on a
string" arrangement in which histone octamer i.e. (H3 - m b - (H2A - H2B)2 forms the core (bead) around which 146 base
pair double stranded DNA is wrapped and 30 bp/54 bp linker DNA and Hi histone protein forms string.
- Histone acetylation (by histone acetyl transferase = HAT) causes increased (trancriptionally active) relatively
uncondensed euchromatin formation®. Acetylation or phosphorylation of lysine residue at aminoterminal decreases
positive (+) charge of basic histone protein, there by decreasing their association with negatively (-) charged DNA, thus
releasing nucleosomes and allowing access to DNA. Histone deacetylation (by histone deacetylase = HD AC) removes
acetyl group, restores positive charge and causes increased formation of transcriptionally inactive hetero chromatin
(relatively condensed).
- Histone is post translationally modified by ribosylation, ubiquitin binding, phosphorylation, acetylation and
methylation (RUPAM).

59. B i.e. M e th y la tio n [R ef: H arper 30/e p. 735; L ippincott 6/e p. 409,460; M T x]

M ic r o s a te llite Repeat Sequences (MiRS) o f 2-6 b a se p a irs e g AC, CG, CA & AT p r o d u c e heritable polymorphism an d
are used in constructing genetic linkage map. (See Tx o f H ig h ly re p e titiv e se q u e n ce s).

Epigenetic M odifications

Are non mutational changes to histone proteins & DNA that influence gene expression. These include
1 Post- translational modifications of histone such as acetylation, methylation, phosporylation & ubiquitylation.
2 . Methylation/demethylation of cytosine residue (nest to guanine) in CG islands of DNAQ.

- One X chromosome in females is condensed and inactivated to prevent overexpression of X linked genes. This X
inactivation occurs by hypermethylation of cytosine in CG islands and deacetylation of histone proteins;
- Methylation/demethylation of cytosine (in CG island) and DNA
acetylation of histone H3 & H4 have been found in cancer cells.
Methylation of C (by methyl transferase) turns off geneQ. C M e + M e th y l-T ra n s fe ra s e M eC —
M any epigenetic changes are reversible. 5' Azadeoxycytidine is an
inhibitor of methyl transferase and SAHA (Suberoyl anilide G - 5' A z a d e o x y C y tid in e G -
hydroxamic acid/Vorinostat) deacytylate histones. So both agents
Me
arc used in leukemias & lymphomas.

DNA Synthesis and Replication

bi). B i.e. Semi conservative; D i.e. Bidirectional; E i.e. Semi discontinuous


61. B, C i.e. Superhelical density depends on length of DNA, Underwinding strain is normally accommodated by strand...
62. A i.e. D N A synthesis /replication 63. A i.e. ds DNA 64. C i.e. DNA Polymerase III
65. B i.e. Sister chromatids are formed C i.e. Follows base pair rule D i.e. Semi conservative E i.e. Single strand breakend.
66. C i.e. RNA polymerase —> DNA polymerase III —> DNA Polymerase I —» DNA ligase.
Biochemistry: Structure, Function and Replication of Informational Macromolecules ■ 491

67. B i.e. M ultiple origins of replication are possible..; D i.e. Lagging strand stick by RNA..; C i.e. DNA replication ...
68. A i.e. DNA Ligases 69. A,B,C,D,E i.e. DNA polym erase, DNA ligase, DNA topoisom erase, Prim ase, RNA polym erase
70. D i.e. DNA gyrase [R ef: L ippin cott 5/e p 3 9 9-405; L ehn in ger 5/e p 954-58, 977-92; H arp er 29/e p 365-72; John Baynes 3/e p 4 18]

DNA replication is semiconservative (since half of DNA Replication (Elongation)


the original DNA is conserved in each of the daughter
I
DNA), semi-discontinuous (i.e. continuous in leading
Leading strand Lagging strand
& discontinuous in lagging strands) and bidirectional
RNA prim er (made by RNA prim er (made by Primase)
(d/t replication bubble/ fork) in both pro and
primase) I
eukaryotes. Although prokaryotes have single and
I DNA m ade by DNA
eukaryotes have multiple origins.
DNA chain made by poly m erase III
Okazaki fragments are short (small) discontinuous
segments of DNA synthesized on the lagging strand DNA polym erase III (in short Okazaki fragment,
in 5' —» 3' direction, during the process of double (addition of in an antiparallel fashion)
stranded (ds) DNA replication (synthesis)Q. On deoxyribonucleotides I
lagging strand, first an RNA primer is synthesized by or polymerization) Excision of RNA primer by DNA
primase and then DNA polymerase III binds to this polymerase I
RNA primer and synthesizes Okazaki fragments by DNA gap filled by DNA
adding deoxyribonucleotides (=polymerization) polymerase I
along the single stranded template. After this DNA Remaining nuclei sealed by DNA
polymerase I removes the RNA primer and fills the ligase.
created gap by DNA (nick translation). DNA ligase
seals the nick created by removal of DNAPI in
lagging strand (Nick sealing or sticking)

DNA replication occurs in S phase of cell cycle, is semi conservative process that follows base pair rule and yields
two pairs of sister chromatids from each chromosome. F or rep lication DNA is breaken ed (unwound) in to single
strands®.
Replication loops always initiate at a unique point in DNA termed origin (indicated by denaturation mapping o f Ross
inmari). Both (or one) ends of replication loop are dynamic points, termed replication forks, where parent DNA is
unwound and separated strands quickly replicated - 1/t bidirectional (or unidirectional) DNA replication.
DNA synthesis always proceeds in a 5' -► 3' direction and is semidiscontinuous. The nascent DNA is always
syn thesized in the 5'->3' direction becau se DNA p oly m era se can ad d a n u cleotid e on ly to the 3' end o f a DNA strand®.
This also means that the template is read from its 3' end towards its 5' end.
Leading or Forward (5' -»3') strand is synthesized continuously and in the same direction as replication fork moves.
Lagging (retrogade) (3'—>5') strand is synthesized discontinuously in short pieces (Okazaki fragments) in a direction
opposite to that in which replication fork moves. Genetic information found in DNA is copied & transmitted to
daughter cells through DNA replication.
Nick translation (i.e. filling the gap produced by removal of DNA or RNA strand such as RNA primer paired to DNA
templet) is done by 5'- 3' exonu clease activ ity o f DNA p oly m erase I® whereas nick sealing (i.e. filling the gap produced
by removal of DNA polymerase I on lagging strand) is done by DNA ligase®.
DNA gyrase, a type II topoisomerase found in bacteria & plants introduce n egative supercoils® in relaxed circular
DNA using ATP (energy). This facilitates the future replication of DNA because the negative supercoils neutralize the
positive supercoils introduced during opening of double helix.
Process of eukaryotic DNA replication closely follows that of prokaryotic DNA synthesis except that it has m ultiple
origins of replication (Ori) and RNA primers are removal by RNase H and FEN1 (rather than by DNA polymerase) in
eukaryotic cells. Prokaryotes have single Ori.
492 b A Complete Review of Short Subjects

Steps in DNA Replication

Id e n tify o rig in o f re p lica tio n S p e cific site w h e re re p lica tio n start.


(ori) A + T rich region a d ja c e n t to 'o ri'

5' 3' F o rm a tio n o f R e p lic a tio n fork L ead s to


B y lo ca ls s e p e ra tio n o f stra n d s I
P o sitiv e s u p e rc o ils in th e re g io n o f D N A
ah ead o f re p lic a tio n fo rk w h ich is re m o v ed
r U n w in d s t h e d o u b l e helix ®
H elicase
u R eq u ire s e n e rg y b y A T P .
I
Sin g le strand K eep th e tw o stra n d s o f D NA DNA topoisomerase
D N A binding s e p a r a t e Q. l& m
T 5' ATP AD P r 5’
Protein
+ Pi - T h e y h a v e b o th nuclease (stran d c u ttin g ) &
lig a se a ctiv ity .
R N A P rim e r --------------------------------------------------------- - F irs t a tra n sie n t n ic k is crea ted in on e
- R eq u ire d fo r in itia tio n o f D N A sy n th e sis b y D N A stra n d , w h ich re lie v e s su p e r c o ils, th en it is
p o ly m e ra se re sealed .
- It is sh o rt strech o f R N A th a t a re c o m p le m e n ta ry &
a n tip a ra lle l to D N A te m p la te Replication fork

- S y n th e siz e d b y P r im a s e a s p e c i f i c R N A p o l y m e r a s e 0

In ita tio n o f D N A s y n th e s is an d e lo n g a tio n b y


/ f%
DNA polymerase III

R e c o g n iz e s p rim e r & s y n t h e s i z e s D N A in 5 '- * 3 '


d i r e c t i o n <2 (read te m p la te in 3 ' - » 5 ' d ire c tio n ) b y
5 '- » 3 ' p o ly m e ra se a c tiv ity Bidirectional - Replication Bubble
Proof reading o f n e w ly sy n th e siz e d D N A b y its 3 -
5 ' e x o n u c le a s e a c t i v i t y Q.___________________________ L e a d in g (c o n tin u o u s ) strand
/
B o th stra n d o f d o u b le h elix s erv e as te m p la te fo r
c o n stru ctin g to w c o m p le m e n ta ry stra n d s

L a g g in g (d is c o n tin u o u s ) s tra n d
N ew D N A s tra n d s c o n sistin g of with O k a z a k i F ra g m e n ts (O F )

O n e le a d in g stran d O n e la g g in g stran d

I
S y n th e siz e d in 5 '- » 3 ' d irectio n towards the replication • S y n th e siz e d in 5 '—>3' d ire c tio n away from the replication
fork. fork in discontinuous manner.
R eq u ire s only one RNA primer • S h o rt stre tch e s o f d isc o n tin u o u s D N A calle d o k a z a k i
fr a g m e n t s .
• R e q u ire s m a n y R N A p rim e r
S’ —>3’ e x o n u c le a s e activ ity
DNA polymerase IQ
(d e g ra d e / e x c ise p rim er)

• 5 '—>3' p o ly m e ra se a ctiv ity fill the


g a p p ro d u ced b y p rim e r ex cisio n E x cisio n o f R N A p rim e r a n d th e ir re p la c e m e n t
• 3 '- » 5 ' e x o n u c le a se a c tiv ity is for b y D N A (N ic k tra n s la tio n )
p ro o f read in g. DNA ligaseQ \

J o in in g o f tw o s tre tc h e s o f D N A & fill th e g a p p ro d u c e d b y re m o v a l o f D N A p o ly m e ra se I.


S o it se a ls th e n ick on la g g in g stra n d (N ick sealin g )
Biochemistry: Structure, Function and Replication of Informational Macromolecules ■ 493

DNA Polymerase Complex

• D N A p o ly m e r a s e s I I I (p o l I I I ) c a ta ly z e s th e A t le a st o f 5 ty p e & are d e sig n a te d b y G reek A ll D N A P c o m p le x s h are 3 featu res


h ig h e s t r a te o f c h a in e lo n g a tio n (p o ly m e r iz tio n ) letter ra th e r th a n ro m a n n u m b e rs (i.e. a , (3,y, 1. C h a in E lo n g a tio n : ac co u n ts for
a n d is the m o s t p r o c e s s iv e (p o ly m e riz in g 0 .5 M b 5, e) n u cle o tid e p e r se co n d ra te ( n t /s ) at
D N A in o n e cy c le on le a d in g stran d ). a is a m u lti s u b u n it e n z y m e . w h ich p o ly m e r iz a t io n o ccu rs
O n e s u b u n it h as p r im a s e a c tiv ity , w h ich
• P o l III is > lM D a (la rg e ), m u lti (1 3 ) su b u n it 2 . P r o c e s s iv ity : is n u m b e r of
in itiates s tra n d sy n th esis on lead in g stran d
p ro te in c o m p le x . Its a sso cia tio n w ith 2 id en tical n u c le o tid e s a d d e d to n a s c e n t ch ain
& a t b eg in n in g o f e a c h O k azak i fra g m e n t on
b e t a s u b u n it s o f D N A s l i d i n g c la m p m a rk e d ly b efo re th e D N A p o ly m e ra s e
la g g in g stra n d .
in cre a se s th e s t a b ilit y , ra te o f c h a in e lo n g a tio n P rim a s e su b u n it sy n th esizes a s h o rt R N A d ise n g a g e s from th e tem p late.
(2 0 -5 0 n t / s ) a n d p r o c e s s iv ity (1 0 0 to > 5 0 p rim e r th at is e x te n d e d b y a 5 ’ —* 3 ' 3. P r o o f r e a d in g : is id en tificatio n of
th o u s a n d n u cle o tid e s) of D N A p o ly m e ra s e III p o ly m e r a s e a c tiv ity , w h ic h a d d s a sh o rt c o p y in g e rro rs an d c o rre c tin g th em .
enzym e. p iece of D N A
E .c o li E u k a r y o tic F u n c tio n
• D N A p o ly m e r a s e I & I I (p o l I & I I) a re m o stly P o l 5 is th en re cru ite d to c o m p le te D N A
syn th esis of le a d in g s tra n d & e lo n g a te each I G ap fillin g
in v o lv e d in p r o o f r e a d in g a n d D N A re p a ir.
O k azak i fra g m e n t, u sin g 3 ' —>5' e x o n u cle a se (s y n th e s is ) o f
L P d h m era se
a c tiv ity to p ro o fre a d the n e w ly sy n th e siz e d la g g in g stra n d
i II m DNA follo w in g
5 '—» 3 ' P o ly m e r a s e + + + D N A p o ly m e ra s e 8 a sso cia te s w ith DNA
a c tiv ity ^ p ro life ra tin g cell n u c le a r a n tig e n p ro tein , II
rep licatio n ,
3 '—» 5 ' E x o n u c le a s e + + + w h ich se rv e s as a s lid in g D N A c la m p re p a ir &
a c tiv ity (P ro o f (sim ilar to (3 su b u n it o f D N A p o ly m e ra s e III reco m b in a tio n
re a d in g )Q D N A p ro o f
DNA F u n c tio n P ro o f
5 '—> 3' E x o n u c le a s e +Q re a d in g &
p o ly m e r a s e R e a d in g
a c tiv ity ^ (E x c is io n - r e p a ir
re p a ir/ n ic k tr a n s la t io n ) Pol a C o n tain s p r im a s e
P D N A re p a ir^
R e p a ir + + - In itia te s D N A
y M ito c h o n d r ia l
s y n th e s is
P o ly m e r iz a tio n ra te 1 6 -2 0 40 2 5 0 -1 0 0 0 DNA
(n u c le o tid e / s) P o ip R e p a ir - s y n th e s is ^
P r o c e s s iv ity 3 -2 0 0 1500 > 5 lacs Pol e R e p a ir + III E P ro cssiv e ,
(n u cle o tid e s a d d e d le a d in g stran d
Pol y R e p lic a te s +
b efo re p o ly m e ra s e
m ito ch o n d rial syn th esis
d isso ciates)
1
DNA DNA G a P rim a s e
Su bu nits 7 >10
Pol 8 E lo n g a te s lead in g + 8 P ro c e s siv e ,
S tru ctu ra l g en es pol A pol B pol C
s tra n d s & ok azak i la g g in g s tra n d
(d n a E )
frag m en t syn th esis.
E n d o n u c le a s e s a re e n z y m e s th a t c le a v e s in t e r n a l
b o n d s i n D N A & RN A Q .

71. A , D, & E i.e. Circular, N egatively sepercoiled & N ucleoid present Ref: Lehninger 5lh/ed pg- 949-50; Greenwood 16/ed pg-10-12
/■ 1
E. coli (bacteria) has single, closed, circular, double stranded (ds) negatively supercoiled DNA chromosome o f about
1.7 nun (46,39,675 bp) long in their nudeioidu. Bacteria have no nucleus and nuclear membrane

72. B, D i.e. 0; e [Ref: Lippincott 4thfed pg- 407; Harper 29/367; Lehninger 5th/ed pg. 989]
73. A, B, & C i.e. Components are a , 0, y, A, e, 0 associated w ith repair, & y associated w ith repair
74. B i.e. 5'—» 3' Polymerase activity; C i.e. 3 '—t 5' exonuclease activity; D i.e. 5'-> 3 ' exonuclease activity

DNA Damage, Repair and Protooncogene

75. A, C, D i.e. Pyrim idine, Sugar, Purine [Ref: Lehninger 5/e 271-75; Chatterjae 7/e 206-8; Vasudevan 6/e 457-59-296;
76. D, E i.e. TM P - Thym ine monophosphate, UM P - Uracil monophosphate Lippincott 5/e 291-93; Harper 28/e 285-87]

• Nudeosides are N-glycosides formed by addition of pentose sugar (D-ribose or 2 deoxy D-ribose) to a purine
(adenine, guanine) or pyrimidine (cytosine, thymine & uracil) bases. Nucleosides with purine bases (A, G) have -
'sine' suffix viz - adenosine, guanosine^, whereas pyrimidine nucleosides have 'dine' suppix viz - cytidine, thymidine
and uridineQ.
• Nucleotides are phosphorylated nucleosides. Suffix - sine /-nylate /-nylic acid for purine & - dine /-dylate /-dylic add

for pyrimidine are used. So UMP is uridylate or uridine now phosphate and TMP is thymidylate or thymidine mono
phosphate
494 ■ A Complete Review of Short Subjects

77. A i.e. Adenosine

78. A & D i.e. Decrease Tm & M elting lead to denaturation of D N A


79. A i.e. Splitting of double strands into single strand. [Ref: Lippincott's 4/397-98,Harper's 28/303-4;Clmtterjea 7/228-229;Lodish 6/e pi 17]

— ---------------------------------------------------------------------------------
Seperation of double stranded (ds) DNA into single stranded DNAQ is known as denaturation or melting of DNA. It is
promoted by increasing temperature or decreasing salt concentration. So increased salt (monovalent cation)
concentration increases TmQ.

80. A i.e. Absorption at 260 nm [Ref: Harper 28/e 288; Lehninger 5/e 276]

Concentration (and quality) of nucleotides and ncleic acid (DNA /RNA) is measured in terms of "absorbance at 260
nmQ" with a Ultraviolet (UV) spectrophotometer.

81. D i.e. Endonuclease cut DNA at 5' terminus [Ref: Lehninger 5/e 979-82; Harper 28/e 311-404; Vasudevan 6/e 475]

• B-DNA with right handed helix is most commonQ. N ucleic acid (i.e. DNA & RNA both) is degraded by nuclease and
specifically RNA by ribonucleases (RNases) and DNA by deoxy ribonucleases (DNases). Exonucleases degrade nucleic
acid from one end of molecule in only 5'-»3' or 3 '—>5' direction, removing nucleotides only from 5' or 3' end
(terminal)Q respectively of one strand of a double stranded or a single stranded DNA.
• Endonucleases are capable of cleaving internal phosphodiester bonds to produce either 3' - OH and 5' - P 0 4 terminals
or 5' - hydroxyl and 3' - phosphoryl terminals. Hence endonucleases cut (degrade) in the internal site of nucleic acid to
produce nicks, reducing it to sm aller & sm aller fragments. Restriction endonucleases cleave DNA only at specific
nucleotide sequences^.
• DNA polymerase I (Korenberg's enzyme) has 5' —» 3' polymerase activity (responsible for polym erization), 3' —>5'
exonuclease activity (responsible for proof reading) and specific (or exclusive) 5' —>3' exonuclease activity (responsible
for nick translation and excision repair). Due to 5 '—>3' exonuclease activity, DNA polym erase I can replace a segm ent of
DNA or RNA paired to template strand, in a process k/a nick translation. M ost other DNA polym erases lack 5 '—>3'
exonuclease activity and when this is removed the rem aining large fragment of DNA polym erase I is k/a Klenow
fragment, which retains the polym erization & proof reading activities.
• DNA polymerase of T4 bacteriophage of E.Coli and Klenow fragment (of DNA polymerase I) have almost similar
functions^ but the 3 '—>5' exonuclease activity of T4 DNA polymerase is 200 times higher than that of klenow fragm ent
making it preferable for blunting DNAs with 3' over hangs. W hile klenow fragm ents are preferred for oligonucleotide
mutagenesis reactions as it displaces oligonucleotides dow nstream as it polymerizes.

82. A i.e. DNA repair [Ref: Chattergea 7/e p. 239-40]

Photoreactivation (light induced repair), recombinational repair, mismatch repair (or sister strand exchange), and
excisional repair^ are various DNA repair mechanisms.

83. A i.e. Formation of thymidine dimmers 84. C i.e. Nucleotide excision repair
85. B i.e. D N A Polymerase I > D i.e. D N A ligase 86. A i.e. Nucleotide excision repair
[Ref: Harper's 27/e P-345-46,343; Lehninger 5/e p. 1003]
Biochemistry: Structure, Function and Replication of Informational Macromolecules ■ 495

The damage produced by UV rays is caused due to formation of pyrimidine (mainly thymine) dimersQ.
The thymine dimmer prevent DNA polymerase from replicating^ the DNA strand beyond the dimmer, thus
inhibiting the repair of defective DNA.
In Xeroderma Pigmentosa mechanism for repair ('nucleotide excision repair')Q of damaged DNA is defective d/t
formation of Thymidine dimersQ.

Xeroderma Pigmentosa Repair o f thymine (pyrimidine) dimmer by Excision repair


I I
An au tosom al recessive condition in UV rays
which cells can not repair the I
damged DNA D am age o f DNA by fo rm a tio n o f thym ine dim erQ
The clinical syndrome include
| U V sp ecific en don u clease.
m arked sen sitivity to sun lig h tQ
resulting in Damaged sequence is recognized and cleaved
- Skin cancersQ
Synthesize new DNA strand in
- Prem ature d ea th s
5'- 3' direction using sister
Arises because of defect in
DNA strand as template
'nucleotide excision' p a th w a y of
p oly m erase I 5'- 3 ' exon u clease activity
repair due to defect in either of
removes the damaged sequence
three:
- UV specific en donuclease (m ost
Removal of damaged DNA & form ation of new DNA strand
com m on defect)Q
- DNA p oly m erase 1Q DNA ligase
- DNA ligase0 Joining of new DNA strand w ith the m ain chain

87. B i.e. Deam ination to thiam ine; D i.e. DNA repair > E i.e. M ethylation protects the host DNA from cleavage by its
own restriction enzyme I Ref: Vnsudeven 6/e p477; Harper 29/e p370-74; Lippincot 5/e p 398, 460-62, 422; Lehninger 5/e p 289-92, 302, 993-951

Deamination of cytosine forms uracil and deamination of 5 methyl cytosine forms thymine. TD G & M BD4 DNA
uracil glycosylase remove either U or T residues paired with G, generated by deamination.
DNA methylation of adenine at 5' GATC repair mismatched base pairs. And methylated parental (template) strand is
protected from cleavage / digestion by its own restriction enzyme.

88. A, B & C i.e. Xeroderma pigmentatosa, Bloom 's syndrome & Ataxia telangiectasia
[R ef: H arp er-2 8 tl,/ed pg- 330-33; C hatterjea 7th/ed pg-239-41; L ehn in ger 5 thjed pg- 1003-992; H arrison 17/ed p g-396]

Ataxia telangiectasia, Bloom 's syndrome, Fanconi's anemia, Xeroderma pigmentosa, hereditary non polyposis colon
cancerQ and few breast cancers are due to DNA repair defects.

89. B i.e. Inhibition of apoptosis [R ef: R obbin s 7th1293,295; w w .lu n g can cerbooksan d n ew sletter.com /w h at_ is_ can cer.h tin j

Oncogenes may function by inactivation of DNA repair genes, but not by promotion of DNA repair. Promotion of DNA
repair is protective from oncogenesis and is hence the answer of exclusion.

Oncogenesis and DNA repair: (Inactivation of DNA repair genes leads to oncogenesis)
Cancer generally involves m ultiple incidents of DNA damage.
Inactivation of DNA repair genes allow build up of genetic m istakes w ith each succeeding round of cell division.
This 'genetic instability' may lead to oncogenesis.
Apoptosis: (Inhibition of apoptosis leads to oncogenesis)
Apoptosis or program m ed cell death is an inherent method of protection against developm ent of cancer
Inhibition of apoptosis by oncogenes prom otes cancers.
Cell cycle progression: (Enhanced cell cycle progression leads to oncogenesis)
Cell cycle is a highly ordered sequence of events that lead to cell growth and division
In normal cells 'C ell Cycle Control genes' control the progression of cells through cell division cycles.
Deregulation of signals controlling the progression of cell through cell division cycles leads to oncogenesis.
Nuclear transcription (Factors causing enhanced nuclear transcription cause oncogenesis.
Transcriptional factors enter the nucleus bind the DNA and cause proliferation and oncogenesis.
496 ■ A Complete Review of Short Subjects

90. A i.e. Normally involved in cell cycle proliferation; D i.e. Proto- onchogenes on m utation cause cancer
91. A i.e. Carotenoids [Ref: Casciato: Onchology 5/e p-22, 25; Harrison 16/e p- 1072, Robbins 7/e p. 100, 288, 292- 95]

• Protooncogenes are normal cell genes that participate in cellular functions related to growth and proliferation Q.
Proteins encoded by protooncogenes may function as growth factor, ligands and receptors, signal transducers,
transcription factors and cell cycle regulators 0 .
• Protooncogenes on activation become oncogenes that induce cancer ®. The process of activation include gene
amplification, point m utation, insertion mutation, retroviral transduction (integration), chromosomal translocation
and protein- protein interaction.
• Onchoproteins encoded by oncogenes have similar functions as their normal counterparts but they are constitutively
expressed and endow the cell with self sufficiency in growth. Carotinoids are anticarcinogens®.

92. A ie. Proto-oncogenes; B ie. Tumor suppressorgene; D ie. DNA repair genes 93. D i.e. RAS [Ref: Robbins 7/e p 289-309]

• Genes that promote autonomous cell growth in cancer cells are called oncogenes and their normal cellular
counterparts® are called proto-oncogenes®. RAS is an oncogene®.
Growth factor Growth factor receptors Proteins involved in Nuclear regulatory
signal transduction proteins
Protoonco Encode Protooncoge Encode Protooncogen Related to Protooncoge Related to
gene ne e ne
HST-1, Fibroblast ERB-B2 & EGF-receptor RAS-K/H/N GTP binding MYC-C/N/L Transcriptiona
1NT-2 growth factor ERB-B1 family I activator®
(ECRF)
TGF a Transforming FMS Colony ABL Nonreceptor
growth factor- stimulating factor tyrosine Cell cycle regulators
a (CSF)-l kinase
HGF Hapatocyte RET Receptor for BRAF RAS signal Cyclin D/E Cyclins
growth factor neuro-trophic transduction
factors
SIS Platelett KIT Receptor for stem p-catenin WNT-signal CDK-4 Cyclin
growth factor cell (steel)factor transduction dependent
P-chain kinase
• MYC - protooncogene is expressed in almost all eukaryotic cell. MYC - protein is translocated to nucleus (sometimes as
hetrodimer with MAX protein) and is a potent transcription activator. MYC activation causes cell proliferation, induces
histone acetylation, protein synthesis and cell motality & decreases cell adhesion & proteinase activity. Cells in culture
undergo apoptosis if MYC activation occurs in absence o f growth factors®, but it is not clear whether MYC induced
apoptosis occurs in vivo.
• Retinoblastoma susceptibility protein (RB), in hypophosphorylated state, prevents cell from replicating by forming complex
with transcription factor E2F (E2F/D Pl/RB - complex). Phosphorylation of RB dissociates the complex, release E2F,
eliminates main barrier to cell cycle progression & promotes cell replication. Cyclin D, which is the first cyclin to increase
in cell cycle, (appears in Gj phase but becomes detectable in S phase) activates CDK4 and form cyclin D- CDK4 complex
that phosphorylates RB. During M phase RB is again hypophosphorylated. /

Normal growth requires - protooncogenes not oncogenes, tumor suppressor genes, genes involved in apoptosis and
DNA reapir<2.

94. A i.e. a-rays [Ref: Grainger 4/e P-139; Park 17/e P-521]

Maximum damage to DNA is caused by a- RaysQ.

Radiation Content Penetrating Power Ionizing Power Damaging Power


a-particle Helium nuclei (i.e. 2 protons & 2 PoorestQ (1) MaximumQ M ost damaging
neutrons)
P-particle Either high energy electron or Greater than a-particle Lesser than a-particle < a-particle
antimatter counter part positron ( 100 )
X-Ray Low energy photonsQ More than P-particles Lesser than P-particle < P-particle
7 -Radiation Very high energy photons^ More than X Rays (10000) M inim um IonizingQ Least damaging
i.e. M ost Penetrating^
Biochemistry: Structure, Function and Replication of Informational Macromolecules ■ 497

95. B i.e. Hydroxyproline; E i.e. Ornithine 96. B, C i.e. Overlapping, Ambigous 97. D i.e. UAC
98. A, C, D, E i.e. AUA codes for methionine in mitochondria, AUG codes for initiator codon in mammalian cell, AGA &
AGG act as chain terminator in mammalian mitochondria, UAA, UAG & UGA are stop codon
99. C i.e. 256 100. C i.e. 3 consecutive nucleotide units 101. C i.e. Punctuation
102. A i.e. AUG 103. D i.e. Termination of protein synthesis 104. B i.e. 3 complementary base pairs
105. B i.e. 3 consecutive nucleotide unit 106. A i.e. Degeneracy 107. B i.e., 5-end of anticodon
[Ref: Lippincott 5/e p 432; Harper's 29/e P- 270; Lehninger 5/e p 1085

Genetic Code and Codons

Genetic code is a dictionary that identifies a corrospondence b/w a sequence of nucleotide bases and a sequence of amino
acids. And codon is each individual genetic word of this dictionary that is com posed o f 3 nucleotide bases®.

■Condons are present in m- RNA in language of A Characteristics of Genetic Code


(adenosine), G (guanine), C (cytsine), and U (uracil)
nucleotides. Specificity (Unambiguous): which means a particular codon
■There nucleotide sequences are always written from 5' to always codes for the same amino acid.
3' end. Each codon consist of a sequence o f 3 nucleotides® Universal: which means the specificity of genetic code is
i.e. it is a triplet code. conserved from very early stages of evolution to mammals
•Since there are only 4 different nucleotides (A, G, U,C) (between species) and among different tissues in a species.
in m- RNA and codon consists of sequence of 3 The exceptions are
nucleotides; each codon must consist of >1 purine or Mitochondrial translational machinery reads four codons
pyrimidine nucleotide and there are 64 (43) differen t differently from t-RNA molecules in cytoplasm of even same
com b in ation s o f codon®. cell. So mitochondria requires only 22 t- RNA molecules to
■61 of 64 genetic codons code for 20 common amino acids read their genetic code, where as cytoplamic translation
- phenylalanine (Phe), leucine (Leu), isoleucine (lie), system has a full complement of 3' tRNA.
m ethionine (Met), Valine (Val), serine (Ser), proline
(Pro), threonine (Thr), alanine (Ala), tyrosine (Tyr), Codon Cytoplasm Mitochondria
histidine (His), glutamine (Gin), asparagine (Asn), AUA lie Met (Methionine)
lysine (Lys), aspartic acid (Asp), glutamic acid (Glu), UGA Termination Trp (Tryptophan)
Cysteine (Cys), tryptophan (Trp), arginine (Arg), and codon
glycine (Gly). AlfG designating m ethionin e is initiating AGA Arginine Stop or chain
codon®. AGG termination codon
■3 codones - UAA, UAG, UGA do not code for any
amino add and are known as stop (termination or Degeneracy (Redudancy): which means som e am inoacids
nonsence) codon. When one of these appear in m-RNA are coded by multiple codons®. For example six diffrent
sequence, it indicates that syntihesis of protein coded codons specify serine as well as arginine
by that mRNA is completed^. Non overlapping, non punctuated and commaless: which
means the code is read from a fixed starting point as a
If codon consisted of Number of codons continuous sequence of bases taken 3 at a time without any
2 nucleotide sequence eg. AU (4)2 = 16® overlapping or comma (punctuation) between the codons. For
3 nucleotide sequence eg AUG (4)3= 64® example ABCDEFGHI is ABC/DEF/GH1.
4 nucleotide sequence eg AUG (4)“= 256 <3
498 ■ A Complete Review of Short Subjects

Middle Bas Unnatural Expansion of Genetic Code


5' or U C A G 3' or Third - Codon UAG is the best target (of scientific research) for
First Base encoding a new aminoacid as it is least used of the three
Base termination codons and strains with tRNAs selected to
U Phe Ser Tyr Cys U recognize UAG do not exhibit growth defects.
Phe Ser Tyr Cys c - Over 30 unnatural aminoacids with a unique reactive chemical
Leu Ser Stop Stop A group like ketone, azide, photocrosslinker (a functional group
Leu Ser Stop Trp G designed to form a covalent bond with a nearby group when
c Leu Pro His Arg U activated by light), a highly florescent amino acid, aminoacid
Leu Pro His Arg c with a heavy atom (Br) for use in crystallography and
Leu Pro Gin Arg A aminoacid with long chain cysteine analog that can form
Leu Pro Gin Arg G extended disulfide bonds have been added to genetic code.
A lie Thr Asn Ser U
Induced Variation in Genetic Code: Nonsense Suppression
lie Thr Asn Ser c
lie Thr Lys Arg - Nonsense mutation produce a termination codon in interior of
A
Met Thr Lys Arg gene, prematurely halting translation, producing inactive
G
G Val Ala Asp Gly incomplete polypeptide. The gene can be restored to normal
U
Val Ala Asp Glyc function if a 2 nd restorative mutation (k/a nonsense
Val Ala Glu GlyA suppressors) either suppresses the effects of termination codon
Val Ala Glu GlyG or converts the misplaced termination codon to a codon
specifying an aminoacid.
Natural Expansion (Evolution) of Genetic Code - Mostly nonsense suppressors involve mutations in t-RNA genes
to produce altered (supprssor) t RNA (mostly with a single base
- There are actually 22 rather than 20 amino acids
substitution in their anti codon) that can recognize the termination
specified by known genetic code. The 21s1 amino
codon and insert an aminoacid at that position.
acid selenocysteine (Sec) is coded by UGA and 22nd
- A cell usually has several copies of each tRNA. Nonsense
amino acid pyrrolysine (Pyl) is encoded by UAG. So
suppression does not completely disrupt normal information
in effect, UGA and UAG doubles as a codon for both
transfer in cell because these suppressor mutations usually
termination and (very occasionally) selenocysteine (in
involve minor (weakly expressed) t RNA, leaving the normal
bacteria w ith formate dehydrogenase or in mammals
tRNA to read its codon normally. However nonsense
with glutathione peroxidase protein) and pyrrolysine
suppression can produce enough full length active protein from
(in methanogens or methane producing anaerobic
a gene with nonsense mutation to allow the cell to survive.
archaea).
- Nonsense suppression does not always lead to abnormally long
- Unlike hydroxyl lysine or hydroxyproline,
proteins or creation of suppression t-RNA with mutation in
selen ocystein e arises co tra n slatio n a lly during its
anticodon. Eg suppression of UGA nonsense codon generally
incorporation into peptides. Unlike selenocysteine -
involves tRNA trp that normally recognize UGG and inserts Trp
pyrrolysine was attached directly to a dedicated
at that position. Alteration of G to A at 24 position in arm of
tRNA by a cognate pyrrolysyl tRNA synthetase.
tRNA allows it to read both UGA and UGG (& insert Trp).

Regulation of Gene Expression

108. B i.e. Zinc finger m otif 109. A i.e. Nuclear receptor 110. A i.e. Zinc finger
111. A i.e., Zinc finger D i.e., Helix turn helix E i.e., Leucine zipper [Ref: Harper's 28/e p. 381-84; Lippincott's 5/e p. 423,450,18]
112. E i.e. 7 Amino acid 113. B i.e. Zinc 114. A i.e. DNA binding protein

Binding o f transcription regulatory proteins to DNA is regulated by several motifsQ - the helix turn helix, the leucine
zipper and the zinc finger. However, the binding m otif for steroid receptor fam ily and thyroid (nuclear) receptor family
is zinc fingerQ.

115. A i.e. Cistron IR ef: C h atterja e 7/e 2 5 3 ; V asu devan 6 /e 4 7 3 -5 6 1 -6 0 ; H a rrison 17/e p -3 8 5 ; H a rp er 2 8 /e 3 7 0 -7 1 ; L eh n in g er 2 7 1 -4 7 7 -9 4 8 ]
116. A, C, D i.e. Smallest functional unit of genome, Promoter & enhancer geens are typical example, Cistron is single ,
117. A i.e. Positive regulator

CAP - cAMP regulator is a positive regulator fo r Lac operonQ because its presence is required for gene expression.

Operon is the segment of DNA strand, consisting of cluster of several genes involved in metabolic pathway.
Biochemistry: Structure, Function and Replication of Informational Macromolecules 499

Operon = structural genes coding for enzym es of metabolic pathw ay + regulatory genes that determ ine their
transcription
1
Structural genes for Coded protein Regulator genes/ proteins & inducers
Lac operon
Lac Z P galactosidase Positive regulator CRP = CAP + CAMP®
Lac y Perm ease Negative Lac I gene®
regulator Repressor protein
Lac A Acetyl ase Inducer Lactose®
IPTG (gratuitous inducer)

118. A i.e. Promoter; B i.e. Enhancer; C i.e. Operator [R ef: H arper's 2 8 /e p. 3 7 0 -3 7 3 ; L ippin cott's 5 /e p. 4 4 9 -5 3 ; C hattergea 7/e p. 2 5 3 -5 8 1
119. D i.e., Not regulated 1 2 0 . E i.e., Catabolite repression is mediated by CRP
121. D i.e., Regulator gene is inducible 1 2 2 . C i.e. Inducer without glucose

125. C i.e. Specifically recognizes the promoter site 1 2 6 . B i.e. Promoter


127. D i.e. Primer 1 2 8 . B i.e., Nucleolus
[Ref: Lippincott's 5/e p. 419, 422, 424; Harper's 28je p. 337, 345; Lehninger 5/e p. 1024-32; Chatterjea 7/e p. 242-43]

- The sigma (a) subunit (factor) enables RNA polym erase to recognize prom oter regions on DNA®.
- Like DNA polymerase, RNA polymerase uses nucleoside triphosphate as substrates & releases pyrophosphate each time a
nucleoside monophosphate is added to growing chain. And as with replcation, transcription is always in 5' —>
3'direction. However, incontrast to DNA polym erase, RNA polym erase (both in eukaryotes and prokaryotes) does not
require primer and has no known proofreading activity®.

129. A i.e. Exon > B i.e. Enhancer 130. A i.e. Polycistronic mRNA; E i.e. Non coding exon
131. D i.e. Normal transcription, normal translation 132. A i.e. Interon 1 3 3 . A i.e. Cut DNA at specific DNA seq.
[Ref: Harper 29/e p 360-641

- Interons are non coding region of gene which are processed out of primary transcript. Hence any change (insertion or
removal of base pairs) in intron w ill not effect transcription or translation®.
- Exons (coding regions) o f gene are expressed as m-RNA and contribute to polypeptide synthesis®. Enhancer and
repressor (silencer) are cis acting (ie located on same chromosone, whose transcription affect) distal regulatory
elements that regulate (increase & decrease respectively) the rate of initiation of transcription by pol II in eukaryotes.
- Leader sequence at 5' end of mRNA, 3 ' flanking sequence and non coding (nc) or non protein coding (npc) or non
messenger (nm) or small (s) or functional (f) RNA like rRNA, t-RNA, sn RNA, sno RNA, sea RNA, st RNA, mi RNA,
si RNA, etc do not translate into protein.
- Endonucleases cut DNA at specific sequence with in the molecule (as opposed to exonuclease which digest from the ends
of DNA) are key tools in recombination DNA research. These enzymes were called restriction enzymes because their
presence in a bacteria restricted the growth of certain bacterial viruses called bacteriophage.
500 ■ A Complete Review of Short Subjects

In eukaryotic cells, small mobile DNA elements k/a jumping DNA or jumping genes, E ukaryotic gene
that clearly are not viruses are capable of transpoising themselves out and in of the host
genome in way that affect the junctioning of neighbouring DNA sequences. These mobile
Non coding
jumping genes can carry flanking regions of DNA and therefore profoundly affect
regions
evolution. Alu family of moderately repeated DNA sequences has structural similarity to
(Interons)
termini of retrovirus, allowing the latter to move in and out of mammalian genome.
Double Interpose
Processed genes (eg for alpha globin molecules) provide direct evidence for transposition
helical DNA between
in human genome. Processed genes consist of DNA sequences identical or nearly
is packaged exons and
identical to those of m-RNA for appropriate gene product (ie the 5' non translated
into a more are always
region, the coding exon region without intron representation, and 3' poly A tail are all
compact removed
present contiguously). This processed DNA sequence arrangement must have resulted
structure from
from the reverse transcription o f an approp riately processed in-RN A from which introns had
d /1 basic precursor
been removed and poly (A) tail added. And the only known mechanism this reverse
protein RNA before
transcriptase could have used to integrate into human genome is a transposition event.
histone transport
That's why processed genes have short terminal repeats at each end like transposed
E nhancers into
sequences in lower organisms.
are the DNA cytoplasm
In absence of transcription & genetic selection for function, many processed genes are
sequences in occurs.. The
randomly altered through evolution so that they now contain nonsense codons that
gene that process by
preclude their ability to encode a functional intact protein. These are k/a pseudogenes
increase the which
Thus a eukaryotic gene contains neudear & mitochondrial genes, pseudo genes and
rate of interons are
processed genes; coding exons and non coding introns.
transcription removed
M onocistronic mRNA encodes only single polypeptide (protein) chain (like in most
& silen cers and exons
eukaryotes), whereas polycistronic mRNA encodes >1 or many proteins eg prokaryotes
repress are ligated
(bacteria), lac operon.
transcription together is
Exons are spliced together to form mRNA and introns are spliced out of precursor RNA.
called RNA
However, eukaryotic- heterogenous nuclear (hn) RNA or pre-m RNA or primary
splicing
transcript must undergo several post transcriptional modifications like capping ,
splicing & addition of poly (A) tail before it is exported from nucleus to cytoplasm for
protein synthesis.

134. D i.e. Involve phosphodiester bond formation with elongation occurring in the 5'- 3' direction.
IR ef: H arper's 2 7 je P -348; C hatterjea 6/e P -232J

In both DNA and RNA synthesis, the general steps of initiation, elongation and termination occur in 5'- 3' direction
w ith the formation of phosphodiester bondsQ.

135. A i.e. 5' - CGU - 3' [R ef: H arper's 2 8 je p. 336-46; L ippincott 5/e p. 418-22]

- In transcription, the template strand of DNA serves as a template for RNA synthesis and is always read in 3'-»5'
direction. So the primary transcript (m-RNA) is complementary to template standQ.
- The opposite non template DNA strand (which is complementary to template strand) is also called coding strand
because it is identical in base (nucleotide) sequence to mRNA transcribed from that DNA (gene), with U in mRNA in
place of T in coding strand o f DNA. So 5'GCT3' coding strand means 5'CGU3' mRNA transcript.

136. C i.e. Different a subunits are responsible for recognition specificity 137. A i.e. Prom oter region
138. A i.e. Initiation of transcription of the dystrophin gene would be affected
[R ef: L ippincott 5/e 430-423; H arper 28/e 553-54; H arrison 17/e 2682-83; C hattarjac 7/e 243; R obbin s 7/e p-1337]

Let me rephrase this question to make it straight forward, and then it would be - what is the consequence of alteration
(i.e. mutation) in prom oter region?

• Promoter region of transcription unit is the portion of DNA having specific consensus sequences (like TATA box
/CAAT box /GC box etc) which is identified by the RNA polymerase enzyme for binding and formation of preinitiation
complex (PIC) in prokaryotes or RNA polymerase II/pol II-transcription complex in eukaryotes - a process ultimately
resulting in initiation o f transcription.
Biochemistry: Structure, Function and Replication of Informational Macromolecules ■ 501

• So this becom es very obvious, that any alteration /mutation / damage in promoter would typically prevent formation
of PIC or pol II transcription complex, resulting in a decrease in the initiation of m-RNA synthesis (transcription) 1/t
decreased protein synthesis. And sim ilarly promoter induction would increase the transcription rate Q.
• Capping, tailing, termination and splicing are not associated with promoter and so are not a consequence of prom oter
m utations. How ever, capping & tailing defects can result in m RNA with decreased stability and splicing defects in
mRNA in which to few or too many introns have been removed.

139. A, B, C, E i.e. siRNA, m iRNA, tRNA, rRN A I Ref: H arp er 28/e 3 07-10; L ehn in ger 5/e 1 1 4 6 -1 0 6 0 -3 3 -1 0 7 6 /

• Noncoding (nc) RNAs are all R N A s that do not en cod e protein. They are also called non protein codin g (npc) RN A , non-
m essen ger (nm ) R N A , sm all (s) R N A , an d fu n ctio n a l (f) RN A . The gene (DN A sequence) from which a nc-RNA is
transcribed as end product is called non coding RNA gene or RNA gene. A_

• Non coding (nc) RNAs include ribosomal (r) RNA, transfer (t) RNA, small nuclear (sn) RNA, small nucleolar (sno)
RNA, sea RNA, small temporal (st) RNA, micro (mi) RNA, small interfering (si) RNAQ, pi RNA, rasi RNA, line RNA,
antisense RNA, and long nc RNAs like Xist & HOTAIR.

• M essenger (m) RNA is a coding RNA.

140. D i.e. C-DNA; E i.e. New strand of DNA 141. B i.e. t RNA

Genetic inform ation stored in genes (chromosome) All RNA molecules (eg r RNA, tRNA, mRNA etc) are
transm itted to daughter cells through DNA replication, produced from transcription o f DNAQ. Whereas, as a new
is expressed through transcription to RNA and in cases strand of DNA is formed by DNA polymerase III enzyme in
of mRNA, subsequent translation into proteins. tRNA process of DNA synthesis /replication. And complementary
and rRNA are not translated Q. (c) DNA is prepared by using mRNA as a tem plate by enzyme
reverse transcriptase (= RNA dependent DNA polymrase)® in
process of reverse transcription.

142. A i.e. Transcribed from Nuclear DNA

- m-RNA is transcribed from one strand of ds nuclear DNAQ. In Com ponent DNA RNA
eukaryotes, the primary transcript (immediate product of
Sugar 2-deoxy D- D -ribose
transcription) in nucleus is heterogenous nuclear (hn) RNA not
ribose
mature mRNA. This hn RNA is very heterogenous, can be >10-50
times longer (with higher molecular weight) containing both N ucleotides Adenine (A), Adenine (A),
exons & introns, and is very unstable. By post transcriptional Cytosine (C), Cytosine (C),
modification (processing) eg splicing hn RNA is converted to Guanine (G), and Guanine (G) and
smaller molecular weight final transcript (ie mature mRNA). Thym ine (T) Uracil (U)
- DNA contains deoxy ribose sugar and thymine (T) nucleotide, whereas RNA (including mRNA) contains ribose
sugar and uracil (U) nucleotide.

143. B i.e. RNAI 144. B & E i.e. RNA polymerase II & Drosha 145. A i.e. Gene Regulation
R ef: H arper 2 8 ,hle p. 348-50, 3 0 6-10,366; L ehn in ger 5"'/ed pg- 1 0 3 0 -3 3 ,1 1 4 5 ,1 0 4 5 -4 6 ,1 1 4 6 ; L ippin cott 4 thfed pg- 422-24; Lodish-
M olecu la r B iology 6/ed pg-348

' -----------------------------------------------------------------------------------------------------------------------------------------------------------------------------H
• Micro-RNA (mi-RNA) is transcribed by RNA polymerase IIQ as primary mi-RNA (pri-mi-RNA). It is processed by 2
endoribonucleases of RNase III family:
1. Drosha (with E)GCR8, in nucleus to precursor mi-RNA or pre-mi-RNA) and
2. Dicer (with TRBP in cytoplasm to nearly mature mi-RNA or small interfering RNAs = si -R N A s or small temporal
RNAs = st RNAs paired with a short RNA complement)
• RNA helicase removes the complement and mature mi- RNA is incorporated into protein complexes to form RNA
induce silencing complex (RISC).
• By this process of gene silencing by RNA interference (Nobel prize 2006 given to Andrew Fire & Craig mello) mi-
RNA, st- RNA and si - RNA play a important role in gene regulation by inhibition of gene expression (& so
translational process)Q
5 02 h A Complete Review of Short Subjects

146. B i.e. Polylysine [Ref: Harper's 27/e p. 366; Lippincott's 4/e p. 432, 424-27; Genes & development 21 : 519- 524, 2007]

Codon AAA GGG ccc TTT / UUU


Translation product Lysin® Glycine Proline Phenylalanine
Poly A tail translates into polylysine®

147. A i.e. O ccur in intron that form Ribozyme; C i.e. Self-splicing introns; E i.e. Lariats interm ediates are formed
148. A i.e.RNA splicing 149. A i.e. Intron 150. B i.e. snRNA
[Ref: Harper 28,h/ed pg- 307, 346-48; Lippincott 5,h/ed pg- 426-27; Lehninger 5lh/ed pg- 1146,1043-44]

- Self splicing introns function as ribozym e and do not require external protein enzym es or high energy cofactors (eg
ATP). Intron forms lariat in group II but not in group I self splicing introns.
- Splicing is process of rem oving introns (i.e. the segm ent of gene that is not represented in m ature m -RNA) from
prim ary transcript and joining (or ligating together) exons (RNA sequences that code protein)®. It is mediated by sn
RNP or snrups formed from sn RNAQ.
- Small nuclear RNAs (sn RNAs) are involved in RNA splicing®; small nucleolar RNAs (sno RNAs) are involved in r
RNA modifications®; m icro RNAs (mi RNAs) & small tem poral RNAs (st RNAs) are involved in inhibition
(regulation) of gene expression (gene silencing)®; small interfering RNAs (si RNAs) are involved in RNA
interference (RNAi)®; and B2 RNA binds to pol II and block transcription of many genes during heat shock
- Mammalian genomes seem to encode more non coding RNAs (nc RNAs) than coding m RNAs. nC RNAs are RNAS
that do not encode proteins including r RNAS, t RNAs, mi RNAs, si RNAs, st RNAs and sn RNAs.

151. A i.e. Lariats form ation; B i.e. Splicing; E i.e. M ethylation


152. A i.e. Occurs in introns that form ribozym e; C i.e. Self splicing introns;E i.e. Lariat interm ediates are formed
153. D i.e. Attachm ent of CCA in tRNA. 154. C i.e. Poly (A) tailing 155. D i.e. Prokaryotic mRNA
156. D i.e. G lycosylation 157. A i.e. Chem ical hydrolysis 158. C i.e. Deam ination of cytidine to uridine
[Ref: Vasudevan 6/e p 485-86; Harper 29/e p 389-94; Lippincott 5/e p 427, 457-58; Lehninger 5/e p 1036-49]

- Least post translational m odification occurs in prokaryotic mRNA, which is generally identical to its primary
transcript.
- Post translational modification of t-RNA includes removal of introns from anticodon loop, trimming of 5' & 3' ends,
methylation / reduction / deamination / alkylation / rearranging glycolsidic bond to produced modified bases like
methylated bases, dihydrouracil (D) & pseudo uracil (40 bases in nucleus, whereas cleavage and attachment of CCA
tailing occur in cytoplasm®.
In RNA, gene during processing undergoes nucleoside m odifications, nucleoside cleavage and terminal addition but
not chem ical hydrolysis®.
- Post translational m odification of m RN A involves 5' ca p p in g , 3' polyadenylation (addition of poly 'A ' tail at 3'
end), splicing (removal of non coding intervening or intron sequences and ligation / joining of coding exons) by Sn
RNA / Sn RNPs / Snurps or self splicing d/t ribozym e activity of self splicing introns with formation of lariat
interm ediates, RNA editing and secondary methylation®.
- Apo B-48 and Apo B-100 are synthesized from same Apo B gene and same ApoB- m-RNA. Apo B 100 is a 100 kDa
protein synthesized in liver by full length translation of corresponding mRNA of Apo B gene. Apo - B-100 forms
part of LDL, IDL and VLDL. Apo B-48 is a 48 KDa protein (48% shorter form of Apo B-100) synthesized in intestine
by partial translation of sam e m RN A of Apo B gene. Apo B 48 forms part of chylom icron & chylom icron rem nant.
This difference between the sizes of Apo B100 and Apo B48 occurs because post transcriptional processing (editing)
of Apo B mRNA , deaminates the cytidine (C) to uracil (U) in intestine at 2153 position. After cytidine deamination
the CAA codon (which codes glutam ine in liver) becomes UAA (nonsense or stop codon) in intestine. This results in
shorter apo B-48 protein being made in intestine (and incorporated into chylomicron) than is made in the liver full
length Apo B-100, incorporated in to VLDL.

159. A i.e. All RNA undergo post transcriptional m odification; B i.e. C apping of the pre-m RNA Involves the addition of 7-
m ethylguanosine to the 5'end; C i.e. Poly A tailing occur at 3'end; D i.e. Intron excision by spliceosom e

N early all eukaryotic & prokaryotic RNA (except prokaryotic m-RNA) are extensively processed prim ariy with in
the nucleus (for eukaryotes) before they becom e functional®.
Post transcriptional modification of mRNA includes capping (i.e. 7 m ethyl guanosine addition to 5' end), poly A tail
addition at 3' end, rem oval of introns & joining of exons by spliceosome®.
Biochemistry: Structure, Function and Replication of Informational Macromolecules ■ 503

160. C i.e. Peptidyl transferase activity 161. E i.e. Deam ination [Ref: Harper 29/e p. 68, 348]
'
Ribozyme is RNA with catalytic activity^. Examples include peptidyl transferase, selfsplicing group I introns, RNase PT
. . . .
☆ i

& hammerhead metallo enzymeQ of virusoids (requiring Mg2+). These are involved in transesterification,
phosphodiester bond hydrolysis (cleavage), RNA metabolism (splicing & endoribonuclease), peptide bond formation
(peptidyl transferases) and site specific RNA cleavage^.

162. B i.e. DNA 163. A i.e. Cytarabine 164. A i.e. Actinom ycin D - B i.e. Am anitin 165. D i.e. Actinom ycin
[Ref: Chatterjea 7/e p. 245,252; Lippincott's 4/e p. 424,397; Harrison 16/e p. 479, 794, 635, Harper 27je p. 351,378]
166. A i.e.Spinal m uscular dystrophy IR ef: L ip p in cott 5 je 4 2 5 -2 6 ; H a rp er 2 8 /e 3 1 7 -3 4 8 -4 9 -3 0 8 ; L eh n in g er 5 /e 1 0 3 5 -4 0 ; V asu d ev an 6/e p. 484-851

P-thalassemia and spinal muscular atrophy occur due to defective splicing of hetrogenous nuclear (hn) RNA.

167. C i.e. Reverse transcriptase [Ref: Harper's 27/e P -316,410; Lippincott's 3/e P- 405]
168. A i.e. Form s DN A from RNA; C i.e. Im portant for replication of HIV

Reverse transcriptase (RT) is an RNA dependent DNA polymerase that produces double stranded DNA copies of
RNA template^. In retrovirus (eg HIV), reverse transcriptase uses RNA as its template (Genome of retrovirus is single
stranded RNA) for synthesis o f viral DNAQ.
RT moves along the template in 3'—>5' direction, synthesizing DNA in 5' —>3' direction.

DNA polymerase Synthesize DNA from DNA&


RNA polymerase Synthesize RNA from DNA®
Phosphokinase Adds a phosphate m oiety Q
★ Am plification o f DNA is done by Polym erase Chain reaction (PCR)Q.

169. B i.e. Protein; D i.e. Nucleic acid [Ref: Harper’s 27jeP-314, 316]

Template is required for protein and nucleic acid synthesis^.


- Nucleic acid synthesis require DNA as a template
- Protein synthesis require mRNA as a template

170. C i.e. t - RNA 171. A i.e. t - RNA 172. B i.e. C lover leaf pattern 173. B i.e. t-RNA

The t-RNA's contain many modifications o f the standard bases A,U,G, and C, including methylation, reduction,
deamination and rearranged Glycosidic bonds.

174. C i.e. t RNA

The T\|/C arm of tRNA has sequence of thymidine^, Pseudouridine, and cytosine.
The T V)/C arm of tRNA is involved in binding of amino acyl - tRNA to the ribosomal surface at the site of protein
synthesis.
hn RNA is heterogenous nuclear RNA, they are processed to generate the mRNA, which then enter the cytoplasm to serve
as template for protein synthesis.

Translation (Protein Synthesis)

175. C i.e. Nucleolus 176. A i.e. Ribosom e 177. C i.e. H ydroxylysine


178. C i.e. Attachm ent of am ino group to 5' end 179. C i.e. Am ino acyl t- RNA synthetase
180. A i.e. Stop Codon; E i.e. UAA; D i.e. Peptidyl 181. B i.e. Peptidyl transferase; C i.e. Release factors
182. A i.e. Used in elongation & cause attachm ent of peptide chain to A-site of t RNA
183. D i.e. 48 S com plex 184. A i.e. Peptidyl transferase 185. E i.e. 4 186. A i.e. Am ino group; B i.e. C a rb o x y l..
187. A, B, D i.e. Releasing factor, Stop codon, UAA codon 188. D i.e. Am ino acyl tRNA synthetase
[Ref: Harper 29/e p 400-408; Lippincott 5/e p 438-45; Lehninger 5/e p 1075-95]
504 ■ A Complete Review of Short Subjects

Translation is the process by which 'Ribosom es' convert the Functions of


information carried by mRNA to the synthesis of proteinQ. Thus
Ribosome is a cellular component on which all interaction take Nucleolus
place to synthesize proteinQ.
Initial step of protein synthesis is activation of aminoacid by Site of r-RNA synthesisQ
t-RNA to form aminoacyl t-RNA. This reaction is ctalyzed by
amino aceyl t RNA synthase. Because hydroxyproline & Ribosomes
hydroxylysine are not represented in genetic code (as they are
synthesised post translationally) so they do not have t-RNA - Exists free in cytoplasm
capable of acccepting themQ. - Contains 65% r- RNA & 35% protein
Aminoacyl tRNA synthetase is responsible for proof reading, - Synthesize protein (i.e. tran slate m-RNA) th at
editing & high fidelity of translation (protein synthesis)^ as it rem ain w ithin cellsQ.
attaches aminoacids to their corresponding (isoaccepting) tRNAs-
an interaction that is also called second genetic code. It attaches Rough Endoplasmic Reticulum
aminoacyl groupto 3' position of terminal residue.
Peptidy transferase (a ribozyme) catalyze peptide bond - Coated with Ribosomes
formation^ and causes attachment of growing peptide chain to - Responsible for protein synthesis (only
tRNA of A site. The formation of peptide bonds (for protein chain integral membrane protein & protein that are
elongation) is catalyzed by peptidyltransferase, an activity to be exported)
intrinsic to 23 S r RNA found in 50 S ribosomal subunitQ - Initial folding of polypeptide chains with
(Lippincott) /28S rRNA of 60S ribosomal subunitQ (Harper). formation of disulfide bond.
Because the peptidyl transferase activity resides in rRNA [Thus this is Ribosome which is responsible for
component of ribosome, it is a ribozyme and indicates - direct protein synthesis in RER]
role of r-RNA in protein synthesis. Energy required for
formation of 1 peptide bond is 4 high energy phosphate bonds Smooth Endoplasmic reticulum
(2ATP + 2GTP)Q.
Peptide bond is formed between amino group (of amino acyl - Steroid sy n th esis0
tRNA at A site) and carboxyl group carbon (of peptidyl t RNA in P - Detoxification process
site). - Plays important role in cardiac & skeletal
Termination of protein synthesis occurs when a stop codon muscle contraction.
(UAA, UAG, UGA) appears in 'A ' site. These codons are
recognized by release factors (RF-1, RF-3). RF with GTP & ★ Central Dogma: The flo w o fin fo rm a tio n
peptidyl transferase promotes hydrolysis of bond between the fro m DNA to RNA to p rotein Q.
peptide & t RNA occupying the 'P ' siteQ.
RNA polymerase is required for transcription (mRNA synthesis)
not for translation (protein synthesis)Q

189. B i.e Stop Codon 190. A i.e. AUG Codon 191. A i.e. AUG 192. A i.e. N formyl Met is the first - RNA ...
193. B i.e. N formyl methionine t RNA will be first t RNA to come into action.
194. A i.e. It has 3 steps initiation, elongation & termination; D i.e. IF-2 has a &P units 195. D ie. IF-4F 196. D i.e. 4S

------------------------------------------------------------------------------------------------------------------------------------------------------------------------------ N
Three codons are called as Stop codons or nonsense codons. These are UAA (Ochre), UAG (Amber), UGA (Opal)Q
Shine Dalgamo sequence is a sequence of nucleotide bases (5' - UAGGAGG-3') located 6- 10 base upstream of
AUG codon on mRNA molecule^. It helps ribosomes in recognizing the nucleotide sequence that is essential for
initiation of translation.
Although amino acid methionine (Met) has only one codon AUG, all organism have 2 types of t RNA for methionine
tRNA fMet incorporating N-formyl methionine (f-met) in response to initiation codon AUG and tRNAMel
incorporating methionine (Met) in response to AUG codon in internal position.
M ethionine having anticodon UAC is the first amino acid required in binding to the initiation codon AUG on mRNA.
In bacteria (prokaryotes) & mitochondria the initiator t-RNA carries an N - formylated methionine aminoacidQ,
whereas in eukaryotes, the initiator tRNA carries a methionine that is not formylatedQ.
- Translation has 3 steps initiation elongation & termination. IF2 has a - P subunits. IF4F has 4A, 4G, & 4E (ie AGE)
L components. 40/43 s preinitiation complex has 1A, 2P and 3 initiation factors (IF).___________________________________^
Biochemistry: Structure, Function and Replication of Informational Macromolecules ■ 505

197. D i.e. Asparagine: IR ef: L ehin in ger 5 /e p 1096-97, 224-26; Sm ith 8/e p 9 6 -2 2 1 1 .

H
Serine, threonine and tyrosine undergo phosphorylation by protein kinase (in eukryotes) during post translational
modifications
Extra carboxyl group is added to glutamate residue of blood clotting protein prothrombin (carboxylation). Lysine
undergo methylation in some muscle proteins (mono & dimethyllysine), and calmodulin (trimethyl lysine); carboxy
group of glutamate undergo methylation removing their negative charge.

198. A i.e. 23s; C i.e. 5s 199. A, E i.e. 5.8 S, 28 S


200. A, B, C, E i.e. Conserved in nature, Role is to bring t-RNA and m -RNA together, DNA forms RNA and rotein are
formed from RNA, Have two subunits IR ef: L eh n in g er 5 /e .3, 1076-79: H a rp er 28/e 3 0 8 -1 0 ; L ip p in cott 5 /e 436; V asm leven 6/e 493-4871
201. D i.e. Binding of am inoacyl tRNA uses 2ATP .

^>1
Component of 50s subunit are 23s & 5s; and components of 60s subunit are 28s, 5.8s and 5s.
Ribosome is a highly conserved natural machine for protein synthesis that brings m-RNA & t-RNA together^. It has
2 subunits the large subunit catalzes peptide bond formation whereas small subunit is responsibl for accuracy. 16s
rRNA component of 30s subunit identifies & binds shine Dalgam o sequence. Binding of aminoacyl tRNA to A site
uses 1 GTP. Free cytosolic ribosomes synthesize proteins required in cytosol, nucleus, mitochondria & peroxisomes.
Whereas RER bound ribosome form proteins for various plasma membranes.

202. D i.e. Vitam in K


Generation of biologically active clotting factors involves the post translational modification of Glutamate residues of the
precursor proteins to ycarboxyglutam ate residue by a specific carboxylase enzyme that is dependent on 'vitamin K'.
Vitam in C is also involved in post translational m odification, but not of coagulants.^

Immunoglobin

203. B i.e. IgG -7 (gamma) 204. A, C i.e. IgM to IgG, IgM to IgA
205. A, B, C i.e. Fix com plem ent, Increased in prim ary response, Fab region is com posed of variable region
206. A, C, D i.e. Somatic m utations theory, DNA rearrangem ent, Appropriate class switching
207. A, D i.e. K- 2 , 1 - 2 2 [R ef: L ehn in ger 5 /e 1 0 1 4 -1 6 ,1 0 4 0 -4 1 ,1 7 0 -7 3 ; H arper 28/e 349, 361, 322, 576-80; L ippin cott 5/e 461-62-426;
V asu devan 6/e 561-62, 5 54-58; A n an tn arayan 7/e p 96; H arrison 17/e p 203 6 ; C hattarjae 7/e 2561

• In humans, genes for light chains-Kappa ( k ) is in chromosome 2; for light chain lambda (X) is in chromosome 22 and
genes for heavy chains are in ch rom osom e 140.
• Each of the 5 classes of immunoglobulin has a characterstic type of heavy chain - alpha (a) in IgA, gamma (y) in IgG,
delta (5) in IgD, mu (p) in IgM and epsilon (e) in IgE.
• Im munoglobin has 2 identical light (L) & 2 identical heavy (H) chains, each containing variable (V) & constant (C)
regions. Variable regions of light- & heavy chains (i.e. VL & VH) bind to specifc antigen and form a part of Fab
(antigen binding fragment).
. IgM is natural antibody produced in primary response to antigen. It does not cross placenta, therefore, the fetus even
though it carries an incompatible antigen, is protected from natural antibodies of mother. It fix es complem ent® (like IgG)
and agglutinates bacteria (like IgA & G). It has antigen receptors on B cells, surface (IgD is found on surface of B cells
where it acts as a receptor for antigen).
• Newly mature B cells produce both IgM and IgD, which can switch to other class of immunoglobulin (i.e. IgG, IgA or
IgE) in a process k/a isotype/class switching.

208. C i.e. Differential RNA regulation R ef: L ehn in ger 5 ,h/ed p g - 1014-16; T asukn: Im m u n oglobu lin g en es 2/e pg -2 3 7

Synthesis and diversity im munoglobin in membrane bound or secretory form is determ ined by d ifferen tial /altern ativ e ‘
RNA processin g of heavy, Kappa and lambda light chains and immunoglobin gene rearrangements.
506 ■ A Complete Review of Short Subjects

Intracellular traffic, sorting and folding, Chaperons

209. B i.e. Endoplasmic reticulum [Ref: Vasudevan 6/e p. 493; Harper 28/e p. 487-911

The secretory proteins and transport proteins are synthesized in ribosomes attached to the endoplasmic reticulumQ.
Where as proteins which have to be delivered in cytosol are synthesized in free polyribosomes.

210. C i.e. M em brane bound ribosom es 211. B i.e. SRP 212. D i.e. Halt signal
213. E i.e. SRP-R is ATP bound 214. D i.e. Chaperones 215. D i.e Protein folding
216. B, E i.e. Calnexin, Protein disulfide isom erase 217. B i.e. Are lipid in nature
218. A, B, C, E i.e. Belong to heat shock proteins; W ide range of expression; Present from bacteria to hum an; Also k/a stress

- Clathrin is a peripheral membrane protein, that helps in endocytosis. Pepsin is proteolytic enzyme secreted by chief
cells of stomach and is used for digestion. Laminin is found in extracellular matrix a most abundant glycoprotein
present in basement membrane

220. A i.e. Golgi bodies

Trans - golgi is involved Role of golgi apparatus in membrane synthesis


in sorting of proteins^. I-------------------------------------------------------------- 1

- Processing of oliosaccharide chains of Sorting & seggregation of various


membrane and N- linked glycoprotein and proteins prior to their delivery to their
- Contains enzymes involved in O- appropriate intracellular destination
glycosylation I
I Trans- port of golgi apparatus is
By all parts of golgi apparatus^ particularly involved^.

221. B i.e. Tuberculosis [Ref: Harper 28/e p 497-98, 493; Vassudevan 6/e 590, 495]

Alizheimers, cystic fibrosis and prions diseases (eg Creutzfeld Jakob's disease, Kuru, fatal familia insomnia etc)Q occi
due to protien misfolding.

222. C i.e. Betasheets [Ref: Harper's 28/ep. 39; Lippincott's 4/e p. 22; Harrison 17/e p. 2646]
r -------
There is no difference in amino acid & gene sequence, primary structure & post translational modifications between
normal cellular isoform of non infectious (host) prion protein (PrPc) and infectious (pathological) prion proteins (PrPsc).
The key to becoming infectious lies in 3 dimensional conformation i.e. a num ber of a -helices present in non
infectious PrPc are replaced by /? sheets in infectious (PrPsc) formQ.
Biochemistry: Structure, Function and Replication of Informational Macromolecules ■ 507

223. A i.e. Kuru [Ref: Harper's 28/e P-39, 496-97; Lippincott's 3/e P- 21-22] 224. A i.e Fam ilial fatal insom nia

Kuru, fam ilial fatal insom nia & Creutzfeldt Jacob's disease are d/t protein m isfolding.

Gene Duplication

225. A i.e. mRna [Ref: Lehninger 5/e p. 1060-61, 325, 34, 30]
/
Gene duplication and m utation is a path to generate new enzym atic activities by evolution of superfluous (extra) gene
producing new m RN A & new enzyme with new substrate specificity.

Mutation

226. C i.e. Paracentric inversion

C hrom osom al (or large scale) m utations involve large part of chromosom e eg entire gene, m any nucleotides etc.
Examples include inversions (paracentric & pericentric), translocations (reciprocal, Robertsonian/ centric fusion,
intrachromosomal, inter chromosom al), duplications and deletions (term inal deletion & interstitial deletions).
Small scale (base pair or gene) m utations occur in individual base pair (nucleotide) or a small group of base pairs
(gene). Examples include base pair substitution or point m utation (transition, transversion, silent/ m issense/
nonsense mutation) and fram e shift m utation ( d/t deletion or insertion).

227. A i.e. Point m utation; D i.e. Conserved mutation 228. C i.e. Alanine - leucine
229. C i.e. A base substitution in DN A 230. B i.e. 6 231. D i.e. Insertion of one base
232. D i.e. point m utation 233. D i.e. M utation that leads to no functional gene product 234. C i.e. M issense m utation
235. C i.e. Frame shift alteration in coding nucleotide; D i.e. Trinucleotide repeat mutation; E i.e. Nonsense alteration of DNA
236. C i.e. 3 237. C i.e. Reading frame changes dow n stream to the m utant site 238. A i.e. G G T C C T
239. A i.e. H utingtoris disease B i.e. Spinocerebellar ataxia 240. D i.e. All fo the above 241. D i.e. Aspartic acid
242. B i.e. Glutam ic acid 243. B, D i.e. No change in am ino acid sequence, No change in expression of protein
[Ref: Harper 28/e 355-58; Lippincott 5/e 433-34, 460,427; Chattarjae 7/e 258-60; Lehninger 5/e 29-289,947,993; Vasudevan 6/e 501-
503-262; Harrison 16/e p 370]

C onserved m utation is a type of acceptable m issense m utation that is caused by single base substriction (or point
m utation) and results in translation of am inoacid that has sam e properties as of original am inoacid eg glutam ic acid
(Glu) to aspartic acid (Asp)Q.
Genetic code GAA & GAG encode G lutam ic acid (Glu); and G A C and G A U encode Aspartic acid.

M utation

Mutation is defined as any alteration in primary nucleotide sequence of DNA regardless of functional consequences. It can involve
(i) Germ cells to get transmitted to progeny or, (ii) somatic cells, which is non hereditary and may result in cancers etc.
Types include. .
I------------------------------------------
I. Base Substitution (or Point) M utation II. Mutations altering the am ount or structure of
protein produced by translation.
- It involves substitution (replacem ent or change) of one
1. Fram e Shift M utation
nucleotide base with another baseQ and can be of 2 types
Transition Trans version
Purine base is replaced by It changes purine to either of Insertion type Deletion type
another purine base or two pyrim idines or
pyrim idine is replaced by pyrim idines to either of 2 •Insertion or deletion of one or two or non m ultiples of
another pytim idine baseQ. punnes. 3 nucleotides into the coding gene results in an m-
Purine (A) <-> purine (G) Purine Pyrimidine RNA in which the reading fram e is distorted upon
<->
Pyrimidine(T)e->Pyrimidine (C) A/G T/C translation. This reading fram e change
508 ■ A Com plete Review of Short Subjects

It may result in disappearance of already present restriction /alteration/distortion in an mRNA, all the way down
site or creation of a new restriction site by a particular to the mutant site is a feature of frame shift mutation
restriction endonuclease (RE) 1/t generation of larger or smaller & is not seen in point mutations.
fragmens resectively on digestion with that RE. In otherwords, - The machinery translating the mRNA can not
point mutation may 1/t RFLPQ (restriction fragment length recognize that a base is missing (deleted) or added
polymorphism).
(inserted), since there is no punctuation in reading of
u :A|a
(T e rm in a tio n c o d o n ) codons - is the reason of altered reading frame.
t
- This can result in production of garbled protein (i.e.
Nonsense protein with completely/radically different amino acid
mutation
sequence) or truncated/short /prematurely terminated
- proteins (d/t premature creation of termination
Silent mutation
ffl§K\ - codon) or perhaps reading through the normal
(Codon Tor serine)
termination codon until another nonsense codon is
Missense
mutation
encountered, (d/t deletion/insertion just prior to or
u cM within non sense codon)
(Codon Tor serine)
- Following an insertion in a gene, a deletion (or vice
©CA versa) can restore the proper reading frame. And the
(Codon fo r prollne)
corresponding mRNA on translation would contain a
Possible effect of changing a single nucleotide garbeled amino acid sequence b/w insertion &
base in the coding region of an mRNA chain deletion and a correct sequence after that.
- The resulting m-RNA w ill show a change only at a point ■If 3 or multiples of 3 nucleotides are deleted/inserted
complementary to a base change at the corresponding locus. in a coding region, the corresponding number of
Point mutation or changing a single nucleotide base on aminoacids is missing/added in the proteinQ. Loss of
mRNA chain may 1/t any of these 3 effects. Effects of point 3 bases maintains the reading frame but can result in
mutation may be serious pathology like cystic fibrosis (CF).
1 - In CF, deletion of one codon (i.e. 3 bases) result in loss
Silent Mutation Missense Nonsense of 1 aminoacid phenyl-alanine at 508th position
Mutation Mutation (AF508) in CFTR (cystic fibrosis trans membrane
• If mutation affects non-
essential DNA or if it has •Codon •The codon conductance regulator) protein. CFTR protein
a negligible effect on containing containing the normally is a chloride channel in epithelia cells & its
function of a gene, it is k/a the changed changed base loss 1/t production of thick, sticky secretion in lungs &
silent mutation. base may may becom e a pancrease causing damage to both.
stop/term in ation UCA ucc UAU GGCU
•The codon containing the code for a
co d on Q. This S er S er V Gly
different
changed base may code
aminoacid. creation of stop
Addition of U
for same amino acid. This
The codon at
occurs d/t redundancy
substitution inappropriate site ecu
/degeneracy of genetic is called a non
UCA AUG GCU
of a Ser Pro M et A la
code. sense mutation.
mistaken/
• No apparently detectable missense/ • It would result in Deletion of C
effect usually occurs d/t or incorrect premature
change in 3rd nucleotide amino acid termination of UCA CUA UGG
of genetic code. And is called a peptide chain and S er Leu Rrp
because of w obble missense only a fragment F r a m e S h i f t m u ta tio n d/t i n s e r t io n & d e le tio n o f 1 b a s e 1/t
phenomenon translation mutation. of intended p r o d u c tio n o f g a r b le d p r o te in

of a gene is least sensitive protein molecule


• The 2. Trinucleotide Repeat Expansion/Dynamic
to a change at 3rd position.
incorporatio will be produced. Mutation
• For example CUU, CUA, n of •There is high A sequence of 3 nucleotide bases that is repeated in
CUG, and CUC all code mistaken probability that tandem becomes amplified in number, so that too
leucine and both UCU & different prematurely many copies of triplet occur.
UCA code serine aminoacid, terminated If this occurs in coding region, the protein will contain
depending partially produced many extra copies of one aminoacid Eg CAG
• Example: Flomologous
upon its peptide or protein amplification in Hutington disease 1/t insertion of
substitution of valine at
location in fragment will not
67th position of P-chain- many extra glutamine aa in Hutington protein making
Biochemistry: Structure, Function and Replication of Informational Macromolecules 509

HbA with another specific function in its it unstable that accumulates causing
aminoacid has no protein - assigned role. Eg. neurodegenerative disorder.
apparent effect and result might be /3-thalassemia - If amplification occurs in non coding regions, the
in functionally normal acceptable • If a stop codon is result may be decrease in amount of protein produced,
hemoglobin types. (mostly), changed into a eg, fragile X syndrome & myotonic dystrophy. In
partially coding codon, this fragile X syndrome the amplification results in gene
Hemoglob Homologous acceptable or 1/t elongation of silencing through DNA hyper methylation (as we
in type substituted unacceptable
protein to know transcriptionally active DNA is
aminoacid at
produce run on hypomethylated whereas hypermethylation silences
67th position on
polypeptide. Eg gene expression).
P-chain of Hb-
A Hb constant
spring (a- Disease Repeat Gene product
Normal Valine
thalassemia). Hutington disease CAG Hutington
H bA
Spinocrebellar CAG Ataxin 1, Ataxin 2, SC3
Hb- G lutam ic
ataxia 1, 2, 3 (MJD1), Alpha 1A voltage
M ilwauke acid® /glutamate
(Machado Joseph dependent Ca++ channel,
e
disease), 6 , 7, 12 Ataxin 7, Protein phosphatas
Hb-Bristol A spartic
2A
acirfO/aspartate
Dentorubral CAG Atrophin
Hb- A lanineQ
pallidoluysiane
Sydney
atrophy (DRPLA)
X-chromosome CAG Androgen receptor
spinobulbar
Acceptable Partially Unacceptable muscular atrophy
Acceptable mutations (DRPLA)
Mutations
Friedreich ataxia GAA Frataxin
-Change in aa has no -It will result - It results in
Fragile -X - CGG FMR Vi protein
functional in a protein totally non
syndrome /GCC
consequence i.e. it molecule functional
does not alter the with partial protein, that Dystrophia CTG / Myotonin protein kinase
may be myotonica CUG
normal function of but
protein abnormal incompatibl
F r a g ile X S y n d r o m e
function e with
- Conserved . 3' A A A
mutation means -H b S hinders normal life.
( C G G ) 7-50
altered aa has the normal - For example
same properties of function HbM M y o to n ic D y s t r o p h y

original aa eg since it 1/t mutation 3' A A A

glutamic acid to sickle cell generate


( C U G ) 5-35
aspartic acid. Most anemia molecules
3' A A A
point mutation when that oxidize
would result in mutant gene Fe2+ of heme ( C A G ) 11-34
replacement of one is present in to Fe3+,
I
aa by another with homozygou producing A g g r e g a t e d P ro te in w ith a b n o rm a l

rather similar s state and is methemoglo glu tam in e repeats

functional groups -a considered bin, which


natural mechanism partially cannot 3. Splice Site Mutation
to avoid drastic acceptable transport Improper splicing can alter the way in which introns
changes. The because HbS oxygen are removed from pre mRNA 1/t production of
resulting protein does not aberrant proteins.
may not be bind & Eg incorrect splicing of pl-globin mRNA 1/1 p-
distinguishable release O 2 thalassemia and in myotonic dystrophy gene silencing
from the normal abnormally. occurs d/t splicing alterations.
one.
Protein Hb-Hikari (p-chain) Hb-S/Sickle Hb-M Suppressor Mutations
Cell (p- (Boston), a-
•Suppressor mutations can suppress/counteract some
chain) Chain
effects of missense, nonsense & frame shift mutations
510 ■ A Complete Review of Short Subjects

Aminoaci In HbA-P-chain-61 In HbA-(p- In HbA-a- by producing abnormally functioning, mutated,


d change position, lysine is chain)-6lh chain- 58lh suppressor t-RNA molecules with alterations in
replaced by position position, anticodon region. These are capable of binding to &
asparagine glutamate is histidine is decoding altered mutated codones, thereby
replaced by replace by suppressing the effects of mutations in distinct
61 Lysine
valine tyrosine mutated mRNA encoding structure genes.
—>Asparagine
6,Glutamic 58Histidine - Suppressor t RNA usually result in decreased viability as
acid—yvaline ->Tyrosine these are incapable of distinguishing b/w a normal & a
mutated codon. Eg nonsense S-tRNA can suppress
Codon Transversion Trans version CAU orCAC
GAA or GAG normal termination signals to allow a read through
change AAA or AAG
1 1 when it is not desirable and a frameshift S-tRNA may
| I> < 4 1 1 UAU UAC read a normal codon plus a component of a juxtaposed
AAU or AAC GUA GUG codon to provide a frame shift when it is not desirable.
* Null mutation 1/1 no functional gain - Since reading through of translation has on occasion
been observed, S-tRNA may exist in mammalian cells.

24 4 . B i.e. Deoxygenated Hb 1/t exposure of sticky end d/t replacement of nonpolar residue by polar residue
24 5 . B i.e. Glu by val in b6 246. A i.e. Decreased Solubility
247. C i.e. 'Sticky patch' is generated asa result of replacement of anon polar residue with a polar residue
[R ef: H a rp er 2 8 ,hled p g -4 9 , 35 7 , 3 9 7 -9 8 ; L ip p in cott 5 /e p. 4 7 5 -7 7 ; R ob bin s 7,,'/ed p g - 402, 628; E ich ler m ed ical B ioch em istry (20 0 6 ) p -4 4 ; G ribben -
M o lec u la r h em atolog y (2 0 0 9 ) p g -9 ; Jo h n B ay n es b M a rek D o m in icz a k m ed ical B ioch em istry 3 /rd p g -5 3 -5 5 ,4 3 8 ,4 7 5 ; L eh n in g er 5 ,hjed p g 1 6 8 -6 9 ]

In sickle cell anemia- Glutamic acid is replaced by valine at sixth position in f) - globin chainQ.
Sticky patch is generated as a result of replacement of polar residue (negatively charged glutamate amino acid) with
a nonpolar hydrophobic residue (valine)Q. This change makes HbS (sickle cell hemoglobin) less soluble, when
deoxygenated and cause sicklingQ.
HbS confers resistance against malaria in heterozygotesQ.

24 8 . ALL i.e. (RT-PCR, Denaturing gradient gel electrophoresis, DNA sequencing, Restriction fragment polymorphism
(RFLP), Single-strand conformational polymorphism
24 9 . B i.e. Ligase chain reaction
[Ref: Harrison's 18lh/508-7; Harrison 17lh/e/d p406; Eming's Elemens of Medical Genetics by Muller b Young P-63],

Mutations can be detected by DNA sequencing, PCR, RT PCR, single strand conformational polymorphism (SSCP),
RFLP, and denaturing gradient gel electrophoresis^.
CYTOGENETICS AND MOLECULAR GENETICS,
Chapter -4
h RECOMBINANT DNA & GENOMIC TECHNOLOGY

QUESTIONS

Cytogenetics B. Non- disjunction □


C. Inversion □
1. The long and short arms of chromosomes are D. Isochromose □
designated respectively as: (NBE P 13; AI 06) E. Deletion □
A. p and q arms □ 9. Aneuploidy is due to: (AI 04)
B. m and q arms □ A. Non-disjunction at meiosis C
C. q and p arms □ B. Mosaicism C
D. 1 and s arms □ C. Deletion C
2. Karyotyping under light microscopy is D. Translocation □
A. L Banding (AIIMS 14, 09) □ 10. Differential expression of same gene depending on
B. G banding □ parent of origin is referred to as: (AI 08)
C. C Banding □ A. Genomic imprinting u
D. FUdR banding □ B. Mosaicism □
3. Rapid method of chromosome identification in C. Anticipation □
intersex is: D. Nonpenetrance □
A. FISH (AIIMS 07) D 11. Normal parent with 2 siblings having osteogenesis
B. PCR □ imperfecta. Pattern of inheritance is
C . SSCP □ A. Mutation (AIIMS 10) D
D. Karyotyping □ B. Anticipation □
4. Which of the following technique can be used for exact C. Genomic imprinting u
localization of a genetic locus? (AIIMS 13) D. Germline mosacism □
A. Chromosome painting □ 12. Prenatal diagnosis at 16 weeks of pregnancy can be
B. Western blot □ performed using all of the following, except:
C . Comparative genomic hybridization □ A. Amniotic fluid (AI 06) □
D. FISH □ B. Maternal blood G
5. Subtelomeric rearrangement of genes is frequently C. Chrionic villi □
associated wih mental retardation. All of the following D. Fetal blood □
techniques can be used to diagnose them except: 13. Rich source of IgG is: (AIIMS 04)
A . FISH (AIIMS 1 5 ,11)D A. Stomach □
B. MAPH □ B. Visceral peritoneum G
C . cGH array □ C. Small intestine □
D. MALDI □ D. Large intestine C
6. Which of the following is used to detect the 14. Rh factor is (AI 08)
chromosomal differences between neoplastic cells and A. IgM antibody □
their normal counterparts? (NBE P 13, AIIMS 12) B. Mucppolysaccharide □
A . PCR □ C. IgG antibody □
B. Comparative genomic hybridization □ D. Fatty acid □
C . Spectral karyotyping □ E. Glycoprotein □
D. Western blotting □ 15. Barr body is found in the following phase of the cell
7. Nanotechnology has found tremendous application in cycle:
the diagnosis of cancers because of all of following A. Interphase (AI 05) □
advantages, except: (AIIMS 12) B. Metaphase □
A . Nanocrystals exibit bright, photostable C. GI phase □
fluorescence □ D. Telophase □
B. Nanocrystals have a narrow spectrum wavelength □ 16. Movement of protein from nucleus to cytoplasm can
C . Peak spectrum wavelength is tunable □ be seen by (AIIMS 11)
D. Nanocrystals exhibit a narrow difference between A. FISH □
their excitation and emission peak spectra □ B. FRAP □
8. Transfer of genetic material in 1 meiotic division C. Confocal microscopy □
between 2 non-homologous chromosomes: D. Electron microscopy □
A. Translocation (PGI 12,10) □
512 ■ A Complete Review of Short Subjects

Luminescence 26. Correct statements regarding restriction endonuclease


is/are: (PGI 03)
17. Fluorescence means (AIIMS 11) A. Restriction endouclease recognizes specific sites of
A. Spontaneous illumination in dark □ DNA sequence □
B. Release of longer wavelength light on absorbing B. Restriction endonuclease recognizes short sequence
light of shorter wavelength D of DNA □
C. Release of shorter wavelength light on absorbing C. It acts at 5' - 3' direction □
light of longer wavelength □ D. It acts at 3' - 5' direction □
D. Release of equal wavelength light at constant rate □ 27. DNA fragments formed by the action of Restriction
18. Which statement is incorrect? (AIIMS 09) Endonucleases, are separated by: (NBE P 14, AI 08)
A. Chemiluminescence: excited electron in higher orbit A. Gel electrophoresis □
comes to lower orbit by emitting energy in form of B. Agarose gel eletrophoresis □
photon □ C. Paper Chromatography □
B. Bioluminescence: is a form of chemiluminescence □ D. High pressure liquid chromatography □
C. Photoluminiscence: absorbed radiation emitted as 28. DNA restriction is done by the following method:
light □
A. Paper chromatography (AI 06) □
D. Electrophotochemiluminescence: Light-emitting B. Electrophoresis agargel method □
reaction which take place by the use of electrical C. Spectrophotometer □
current □ D. Spectrometry □
Firefilies produce light due to: (NBE P 13; AIIMS 08) 29. After digestion by restriction endonucleases DNA
A. NADH □ strands can be joined again by (AIIMS 11,10)
B. GTP □ A. DNA polymerase □
C. ATP □ B. DNA ligase □
D. Phosphocreatinine □ C. DNA topoisomerase □
Nephelometry is based on the principle of D. DNA gyrase □
A. Light attenuated in intensity by scattering □ 30. True about Restriction enzymes: (PGI 15, 09)
B. Refraction of light (AIIMS 07) □ A. Palindromic □
C. Reduced transmission of light □ B. Produce DNA sticky end □
D. Filtration of solutes by kidney □ C. Restrict replication of DNA □
DNA estimation can be done by (AIIMS 12) D. Restriction sites are not specific □
A. Spirometer □
E. Breaks at sugar-phosphate bond □
B. Spectrophotometer □ 31. True statement about Restriction endonuclease:
C. pH meter □ A. Palindromic sequences observed (PGI 12) □
D. Sphygmometer □ B. Protects bacteria from infection by virus □
Immunoflorescent probes are used in: (PGI 15, 13) C. Present only in eukaryotes □
A. FRET □ D. Restrict replication of DNA □
B. Microarray □ 32. W hich of the following represents the most
C. RIA □ charachteristic function of Type II Restriction Enzyme
D. Recombinant DNA □ A. Prevent folding of proteins (NBE P 14, AI 12) □
E. ELISA □ B. Remove formed DNA □
C. Prevent supercoiling □
Restriction Endonuclease D. Cut DNA at palindromic sites □
33. True about DNA methylation: (PGI 10)
23. RFLP, True are: (PGI 08) A. Alters gene expression □
A. Endonuclease cuts DNA at nucleotide level: □ B. Genetic code remains intact □
B. It acts at specific site □ □
C. Role in carcinogenesis
C. Only cohesive ends are produced □ D. Protective mechanism against cleavage by restriction
D. Only blunt ends are produced. □ endonuclease □
24. Restrinction Endonucleas function —> true about: 34. Bacteria aquires restriction endonuclease by horizontal
A. Cut both the strands of ds DNA (PGI 06) □ transfer. The result would be
B. The cut ends produced are sticky □ A. Bacteria can undergo mutation (AIIMS 10) □
C. The cut ends produced are blunt. □ B. Bacteria causes death of host DNA □
D. Cuts single strand of DNA □ C. Bacteria cannot survive because it lacks DNA
25. Restriction endonuclease is : (NBE P 15,13, AI 07) methylase that protects the host DNA □
A. Break single stranded DNA □ D. It helps in DNA proof reading in bacteria □
B. Break double stranded DNA □
C. Break peptide chain □
D. Break RNA □
Biochemistry: Cytogenetics and Molecular Genetics, Recombinant DNA & Genomic Technology ■ 513

Polymorphism and RFLP PCR

35. True about polymorphism is: (PGI 14, 08) 44. True about polymerase chain reaction (PCR):
A. Single locus —> multiple normal alleles A. Carried out by thermostable DNA polymerase □
B. Single locus —> multiple abnormal alleles □ B. Exponential (PGI 15, 09) □
C. Single phenotype:Single locus —> multiple normal C. Additive □
alleles □ D. Specific □
D. Single phenotype: Single locus - 4 multiple abnormal E. Single-stranded DNA is required □
alleles n 45. In PCR: (PGI 14, 08)
36. Correct statement about Restriction fragment gene A. Thermostable enzyme is needed □
A. Detected by Southern blot (PGI 15,11) □ B. 211 copies formed after 'n' numbers of multiple □
B. Detected by Northern blot □ C. Non specific □
C. Used for identification of gene for genomic D. Thermolabile enzyme □
mapping rj E. Primer is needed. □
D. RFLP is a DNA variation sequence 0 46. True about polymerase chain reaction is:
37. Restriction fragment length polymorphism is used for: A. Enzymatic DNA amplification (NBE P 13, AI 09) □
A. Analysis of chromosome structures (AI 03) □ B. Recombinent DNA amplification □
B. DNA estimation n C. Seperation of protein fragments is serum :J
C. Synthesis of nucleic acid D. None n
D. Detecting proteins in a cell □ 47. W hich of the follow ing is not true about PCR:
38. For measurement of long DNA molecules (50-100 KB) A. Thermostable enzyme is used (AI 04, PGI 05) □
follow ing is used (NBE P 13, AIIMS 08) B. Annealing is done after DNA denaturation □
A. Chromosome walking □ C. Specific primers re required □
B. Nitch □ D. Required atleast 1 week time for synthesis □
C. RFLP □ E. DNA polymerase has to be added on each cycle □
D. SSLP □ 48. The enzyme used in polymerase chain reaction
39. RFLP, true is /are: (PGI 06) (PCR) is: (AIIMS 09,13)
A. Detects mutation □ A. Thermostable enzyme □
B. Recognizes triuncleotide repeates □ B. Enzyme stabiliser □
C. Detects deletion □ C. Inorganic ion □
D. Blunt ends are produced □ D. Inorganic metal □
E. Always short ends are produced □ 49. True statements about PCR: (PGI 08)
40. M icrosatellite sequence is: (NBE P 15, AI 06) A. Protein amplification □
A. Small satellite □ B. DNA amplification □
B. Extra chromosomal DNA □ C. Same as western blot test □
C. Short sequence (2-5) repeat DNA □ D. Detection of infecting organisms □
D. Looped-DNA □ 50. True about PCR: (PGI 13,10)
A. Primers required □
DNA Fingerprinting B. DNA dependent RNA polymerase □
C. DNA polymerase □
D. RNA dependent DNA polymerase □
41. DNA fingerprinting is based on possessing in DNA of:
51. Enzyme(s) used in polymerase chain reaction is/are:
A. Constant Tandem Repeat (PGI 14,08) □
A. Restriction endonuclease (PGI 11) □
B. Variable Number Tandem Repeats (VNTR) □
B. DNA polymerase □
C. Non- repeatative sequence □
C. Alkaline phosphate □
D. Exon □
D. RNA polymerase □
E. Intron in eukaryotes □
E. Reverse transcriptase □
42. DNA fingerprinting was founded by: (PGI 06)
52. Which o f the follow ing is/are features of polymerase
A. Watson □
used in recombinant synthesis: (PGI 2K)
B. Galton □
A. Thermostable □
C. Jeffrey □
B. Doesn't require primer □
43. DNA finger printing is done by: (PGI 07)
C. Unusually active □
A. Splitting DNA □
D. Replicates double stranded DNA □
B. D N A ofW BC □
53. In PCR Acquaticus thermophilus is preferred over
C. DNA from nucleated cells □
E.coli. because: (PGI 07)
D. All of the above □
A. Thermostable at temperature at which DNA
liquefies. □
514 ■ A Complete Review of Short Subjects

B. Proof reading done. □ C. To identify desired chromosomal DNA insert in


C. Done in more precisely. □ plasmid vectors □
D. Does not require primer. □ D. To detect host DNA in situ □
E. Better DNA replication. □ 63. True about Monoclonal antibody: (PGI 11, 08)
54. True about DNA polymerase used in PCR: (PGI 13) A. Produced by hybridoma technology □
A. Obtained from virus □ B. Non-specific □
B. Obtained from bacteria □ C. Requires in small quantity □
C. Used for joining the two strands □ D. Used for blood grouping □
D. It is heat stable □ 64. True about Hybridoma: (PGI 08)
E. Add nucleotide □ A. Immortalise myeloma cell □
55. Which of the following is used in PCR? (AIIMS 07) B. Hybridoma cell produced by fusion of T-cell &
A. Ca++ (NBE P 15) □ myeloma □
B. Mg++ □ C. Cell is of human origin □
C. Li+ □ D. Prior immunization is done □
D. Na+ □ E. Produce monospecific & monoclonal Ab □
56. For PCR which of the following is not required: 65. Methods of producing fusion of cells are A/E
A. Taq polymerase (AIIMS 14) □ A. Attaching inactive viral particles on cell membrane □
B. d-NTP (PGI 15; NBE P 14,13)0 B. Adding polyethylene glycol (AIIMS 10) □
C. Primer □ C. Applying a small electric current (AI 11) □
D. Radiolabelled DNA probe □ D. Reducing surface tension □
57. For PCR which of the following is not required: 66. In gene cloning, largest fragment can be incorporated
A. Taq polymerase (NBE P 13, AIIMS 11) □ in:
B. d-NTP □ A. Plasmid (AIIMS 05) □
C. Dideoxynucleotides □ B. Bacteriophage □
D. Magneswium □ C. Cosmid □
58. SYBR Green Dye is used for: (AIIMS 07) D. Retrovirus □
A. HPLC □ 67. In DNA transfer the vectors used from smallest to
B. Immunofluorescence □ largest is:
C. PCR □ A. Cosmids, Plasmids, Bacteriophage (PGI 07) □
D. ELISA □ B. Plasmids, Bacteriophage, Cosmids □
59. Real time PCR is used for: (AIIMS 13) C. Bacteriophage, Cosmides, Plasmids □
A. Multiplication of RNA □ D. Cosmids, Bacteriophage, Plasmids □
B. Multiplication of proteins □ E. Plasmids, Cosmids, Bacteriophage □
C. Multiplication of specific segments of DNA □ 68. Which is true about phage DNA (PGI 07)
D. To know how much amplification of DNA has A. Antibiotic susceptibility □
occurred □ B. Restriction enzyme sites □
60. Prenatal diagnosis of Hemophilia is best done by: C. Hexagonal DNA □
A. PCR (A I1 0 )D D. Carries short segment of DNA □
B. Linkage analysis □
C. Cytometry □ Site Directed Mutagenesis
D. Microarray □
69. Which test uses oligomer with single base pair
Ligase Chain Reaction substitution as primer (AIIMS 09)
A. PCR □
61. DNA Amplification is done in: (PGI 06) B. RFLP □
A. PCR □ C. Error coded mutation analysis with PCR □
B. NASBA (Nucleic acid sequential based D. Site directed mutagenesis □
amplification) □
C. Ligase chain reactions □ Site Specific Recombination
D. DNA sequencing □
70. Site specific recombination is (AIIMS 09)
Cloning A. Palindromic □
B. Ser-form Holliday intermediate □
62. In molecular cloning, Blue-white screening is used for: C. RE + ligase □
A. To screen for recombinant vectors (AIIMS 08) □ D. Inversion in same orientation/ Non precise □
B. To detect gene mutations □
Biochemistry: Cytogenetics and Molecular Genetics, Recombinant DNA & Genomic Technology ■ 515

Linkage Analysis C. DNA sequencing □


D. Northern blot analysis □
71. Which of the follow ing statement is true about 80. W hich of follow ing techniques is used for detection of
Linkage analysis? (AIIMS 07) variations in DNA sequence and Gene expression:
A. Detection of Characteristic DNA polymorphism in a A. Northern Blot (AI 11) □
family is associated with disorders □ B. Southern Blot □
B. Useful to make pedigree chart to show affected and C. Western Blot □
non-affected family members □ D. Microarray □
C. Used to make a pedigree chart to show 81. W hich of the follow ing tests is not used for detection
non-paternity □ of specific aneuploidy: (AI 10)
D. Non-Gene maping method of genetic study □ A. FISH □
B. RT-PCR □
C. QF-PCR □
Analyzing Technique
D. Microarray □
82. Gene expression can be detected by all except:
72. True about western blot: (PGI 11, 09)
A. RT-PCR (PGI 14,12,11) D
A. Separation of proteins are based on mass □
B. cDNA microarray □
B. SDS-PAGE electrophesis is used for separation □
C. Southern blot C
C. Enzyme linked antibodies are used □
D. Northern blot □
D. Confirmatory in AIDS □
E. Immunohistochemistry □
73. Antigen antibody reaction is detected by:
A. ELISA (PGI 09) □ Gene Library
B. Southern blot □
C. Northern blot □ 83. True about gene library: (PGI 14,12,11)
D. Western blot □ A. Also known as chromosome □
E. Eastern blot □ B. Library that contains books on gene □
74. Southern blot is used to visualize- (NBE P 15,14) C. Computer base with all gene knowledge □
A. RNA □ D. DNA Nucleotide or fragment □
B. DNA □ E. Complete collection of gene copies of one organism □
C. Protein □
Recom binant DNA Technology or Genetic Engineering
D. Antibody □
75. Western blot technique is done for (AIIMS 14, 09) 84. Biotechnology is/are used for (PGI 09)
A. Mitochondrial RNA □ A. Viral vaccine production □
B. Double strand DNA □ B. To cure genetic disorder □
C. r- RNA □ C. Genetic crop □
D. Protein □ D. Gene production □
76. N othem blot is used for the separation of: E. Production of genetic products □
A. mRNA (AI 07, NBE P 13) □ 85. Starting material for production of insulin from
B. DNA □ bacteria is: (AIIMS 11)
C. Protein □ A. Genomic DNA from beta pancreatic cells of humanC]
D. Protein DNA interaction □ B. Genomic DNA from lymphocytes of human D
77. Northern blot test is used for: (NBE P 14, AIIMS 15, 08) C. mRNA from beta pancreatic cells of human □
A. DNA analysis □ D. mRNA from lymphocytes of human □
B. RNA analysis □ 86. Reverse transcriptase is: (PGI 11)
C. Analysis of proteins □ A. DNA dependent RNA polymerase □
D. Enzyme analysis □ B. RNA dependent DNA polymerase □
78. Probe used in western blot: (PGI 14, 08) C. DNA dependent DNA polymerase C
A. Antibody □ D. RNA dependent RNA polymerase n
B. M-RNA □ E. RNA polymerase □
C. DNA □ 87. True about DNA reconstruction technology:
D. t-RNA □ A. Restriction endonucleases involved □
E. PCR □ B. DNA ligase is used (PGI 08) □
79. The follow ing methods can be used to detect the point C. Acid phosphatase is used □
mutation in the beta (jl)-globin gene that causes sickle D. Reverse transcriptase needed □
cell anemia, except: (AI 06) 88. Enzymes used in DNA research programme are except:
A. Polymerase chain reaction with allele-specific A. Polymerase (PGI 07) □
oligonucleotide hybridization □ B. Exonuclease □
B. Southern blot analysis □ C. Nuclease □
516 A Complete Review of Short Subjects

D. Alkaline phosphatase □ 97. Two transgenic plants are grown. One plant has a gene
E. None □ encoding a fluorescent pigment incorporated in its
89. Which of the following is a strategy used to increase genome. The other plant has a fire fly luciferase gene
the yield of protein produced in recombinant protein incorporated in its genome. Which of the two plants
synthesis? will glow in the dark?
A. Promoter induction (AI 11) A. Both plants will glow ( A I ll) O
B. Genes for protease inhibitors Li B. Neither will glow □
C. Translation initiation □ C. First one will glow □
D. Translation and transcription termination r1 D. Second one will glow □
90. Study of structure and products of gene is:
A. Genomics (PCI 05) Gene Therapy
B. Protenomics □
C. Bioinformatics □ 98. Purpose of Gene therapy: (PGI 03)
D. Cytogenetics □ A. Replacement of abnormal gene by normal gene □
E. Pharmacogenomics □ B. Replacement of normal gene by abnormal gene □
91. Study of multiplication of proteins in disease process C. Knock out of abnormal gene □
is called: (AI 07) D. Introduction of viral gene □
A. Proteomics □ 99. Gene therapy methods are: (PGI 03)
B. Genomics A. Electroporation □
C. Gly comics [J B. Intranuclear injection □
D. Nucleomics LI C. Site directed mutagenesis □
92. Techniques used for protein expression proteomics D. Retrovirus □
study include(s): (PGI 12) lOO.The first gene therapy (somatic enzyme) was
A. Gene Array Chips □ successfully done in: (PGI 07)
B. Mass Spectroscopy □ A. SCID □
C. PolyAcrylamide Gel Electrophoresis (PAGE) □ B. Phenylketonuria □
D. RT-PCR □ C. Thalassemia □
E. Two-Dimensional Electrophoresis □ D. Cystic fibrosis □
E. Alkaptonuria □
Transgenic Organism

93. Mutation that completely disrupt the function of the


Genetic Disorder
gene used in which of the following technique:
A. Knock-out (PGI 09) □ 101.True abut 'X ' Chrom osom e inactivation:
B. Non-sense mutation □ A. X gene (PGI 0 6 ) 0
C. Restrictin fragment length polymorphism □ B. RNA interference □
D. Targeted gene disruption □ C. Seen in Male □
E. Knock -in □ D. Seen in Female □
94. True statement about transgenic mice: (PGI 10) 102.W hich of the following are X- linked recessive
A. Developed from DNA insertion into fertilized egg □ disorders:
B. Have same genome as parents except one or more A. Marfan Syndrome (PGI 08) □
genes □ B. Phenylketonuria □
C. Identical genome to parent mice □ C. Neurofibromatosis □
D. Produced by breeding over several generations □ D. Duchenne muscular dystrophy □
E. Homozygous are selected □ E. G-6 -P dehydrogenase deficiency □
95. Which of the following methods is most suited to 103.Father to son inheritance is never seen in case of:
assess function of a gene: (AI 12) A. Autosomal dominant inheritance (AIIMS 03) □
A. Southern blot □ B. Autosomal recessive inheritance □
B. Northern blot □ C. X- linked recessive inheritance □
C. Gene knockout animals □ D. Multifactorial inheritance
D. Transgenic animals □ 104.An affected male infant b om to norm al parents could
96. Nude mouse is used in genetics because of the be an example of all of the follow ing, except: (AI 06)
following properties: (PGI 11) A. An Autosomal dominant disorder □
A. Absence of thymus □ B. An Autosomal recessive disorder □
B. Xenograft can be sustained for weeks □ C. A polygenic disorder □
C. Severe combined immunodeficiency mice is same D. A vertically transmitted disorder □
as nude mice □ 105.Autosom al recessive disease are: (PGI 08)
A. Tuberous sclerosis □
Biochemistry: Cytogenetics and Molecular Genetics, Recombinant DNA & Genomic Technology ■ 517

B. Duchenne muscular dystrophy □ 109.True about MHC: (PGI 03)


C. Cystic fibrosis □ A. Transplantation reaction □
D. Ataxia telangiectasia □ B. Autoimmune disease □
E. Factor VIII deficiency □ C. Immunosupression □
106.A child with a small head, m inor anom alies of the face D. Involved in T-cell function □
including a thin upper lip, grow th delay, and E. Situated at long arm of chromosome □
developmental disability can have all of the following, 110.MHC class II antigens are located on w hich of the
except: (AI 06) follow ing cells: (PGI 2K)
A. A chromosomal syndrome □ A. Platelets □
B. A teratogenic syndrome □ B. Dendritic cell □
C. A mendelian syndrome □ C. RBC □
D. A polygenic syndrome □ D. T-cell □
107.1n a family, the father has widely spaced eyes, increased E. G-cell □
facial hair and deafness. One of the three children has F. B-cell □
deafness with similar facial features. The mother is 111 .MHC II is/are presented by: (PGI 12)
normal. Which one of the following is most likely pattern A. Macrophage □
of inheritance in this case? (AI 06) B. Dendritic cells □
A. Autosomal dominant □ C. Lymphocyte □
B. Autosomal recessive □ D. Epithelial cells □
C. X-linked dominant □ E. Platelets □
D. X-linked recessive □ 112.W hich of the follw oing m olecular phenom enon in Ig
108.G ene for m ajor histocom patibility com plex and HLA genes is responsible for affinity m aturation of
gene is located on: (AIIMS 08, 09) antibody response
A. Long arm of chromosome 8 . □ A. Chain shuffling (AI 08) □
B. Short arm of chromosome 8 □ B. Junctional diverity □
C. Long arm of chromosome 6 □ C. Somatic hypermutation □
D. Short arm of chromosome 6 □ D. Altered RAA splicing □
518 ■ A Complete Review of Short Subjects

ANSWERS & EXPLANATIONS:

Cytogenetics

C i.e. q and p arms [R ef: R obbin s 711'/170-73; h ttp://w w w .rin g-chrom osom e-20.org/w hat_are_chrom osom cs.n spxj

The short arm of chromosome is desingated as 'p ' (for petit) and the long arm is refered to as 'q ' (the next letter of
alphabet).

2. B i.e. G banding [R ef: N agesh R ao: H um an gen etics (1988) 78. 21-26]
3. A i.e. FISH [R ef: Robbin's 7/e p. 171- 73; h ttp ://m em bers.aol.com /chrom in fo/fishin fo.htm ]

FISH, chromosome painting and spectral karyotyping (SKY) are rapid methods of chromosome identification^.

4. D i.e. FISH [R ef: H arper 29/e p. 442]

Gene maping localizes specific gene to distinct chromosomes and thus define a map of human genome. This exact
localization of genetic locus can be done by FISH (very sensitive technique which utilizes fluorescence rather than
radioactively labeled porbes), somatic cell hybridization and in situ hybridization (simpler, more direct procedure
adding radioactive probe to metaphase spread chromosome).
Chromosomal painting hybridizes multiple (coloured) fluorescent labeled DNA probes to chromosomal DNA. This
produces a multicolored or painted effect with a unique color at each site of hybridization in metaphase or inter phase
cells. So CP identifies both numerical & structural choromosomal aberrations with high sensitivity and specificity.

5. D i.e. M ALDI [R ef: T eitz 6/e p 1225-28; H arrison 18/e p 510-12 665; H arp er 29/e p-442]

- f t
Subtelomeric rearrangements of gene eg chromosomal duplications or deletions too small to be detected by conventional
cytogentic analysis are detected by c-GH (comparative genomic hybridization), multiprobe FISH (fluorescence in situ
hybridization), MAPH (multiplex amplifable probe hybridization) and MLPA (multiplex ligation-dependent probe
amplification). Subtelomeric rearrangements are significant cause of idiopathic & familial mental retardation.

6. B i.e. Comparative genomic hybridization

Comparative genomic hybridization (FISH-CGH) can be used to detect chromosomal differences betw een neoplastic
cells and their counterparts^.

7. B i.e. Nanocrystals have a narrow spectrum wavelength

Nanocrystals (or Quantum dots) are inorganic, semiconductor crystals which exhibits very bright (strong) photostable
fluorescence em issions Q. It can be used as fluorescent probes in biological staining and diagnosis. Compared with
conventional fluophores; the nanocrystals are photochemically & optically stable (i.e. very durable) and their electronic
& optical properties (i.e. absorption band and emission spectra w ave length) can be adjusted (or tuned) by changing their
size Q or by growing it smaller or larger. So nanocrystal size controls a widely tunable absorption band resulting in
widely tunable emission spectra. So nanocrystals have tunable broad & continuous excitation spectrum (i.e. broad
spectrum w ave length )G because of greater chemical (size) flexibility. But they have a narrow, tunable (adjustable),
symmetric em ission (peak) spectra (in comparison to conventional fluorophores).

8. A i.e. Translocation [R ef: R obbin s 7/e p-173 - 7 5 ; H arrison 16/e p. 384- 85] 9. A i.e. Non disjunction at meiosis

- Aneuploidy is a condition in which a cell contains other than haploid number of chromosomes or an exact multiple on
it. It produced by non- disjunction o f chromosomes a t m eiosis0 and is common in cancer cells.
- Term 'Ploidy' is used to refer number of chromosomes in cells. Euploid: Normal resting diploid cell. Tetraploid: Cells
just before division.
Biochemistry: Cytogenetics and Molecular Genetics, Recombinant DNA & Genomic Technology ■ 519

Cytogenetic Disorders (Chromosomal Mutations)

I. Due to abnormal number o f chiqffiosoliies


• Euploid means any exact multiple of haploid number (23) and aneuploidy means chromosomes not in exact multiple of 23.
• The usual cause of aneuploidy are

I------------------“
Non disjunction Deletion
O G D — O C+
' -.-J Ss

I I
Occurs when a h om olog ou s p a ir o f One h om olog ou s ch rom osom e in
chrom osom e fa ils to disjoin a t I s 1 m eiosis or on e ch rom atid in
m eiotic divisionQ, or two m itosis lags behin dQ & is left out of
chromatids fail to seperate either at cell nucleus. The result is one
2 nd meiotic division or in somatic normal cell and one with
cell divisions resulting in 2 monosomy
aneuploid cells. If this occurs in All autosomal monosomies are
gametogenesis, gametes formed fatal d /1 loss of too much genetic
have either an extra (n+1) or one less information
(n-1) chromosome. Fertilization of Monosomies & trisomies of sex
such gametes by normal gametes chromosomes & trisomy of 21
result in - trisomic (2 n+l) or autosomal chromosome are
monosomic (2 n-l) zygotes compatabile with life.

• Deletion means loss of portion of chromosome & may be of 2 types


- Terminal deletion at ends
- Interstitial deletion refers to 2 breaks within chromosome arms f/b loss of material b /w the breaks & fusion of
broken ends (most common type)
• Ring chromosome is special form of deletion, produced when break occurs at both ends of chromosomes with
fusion of damaged ends.
• Inversion refers to rearrangement that involves two breaks within single chromosome with inverted reincorporation
o f segment.
- Paracentric inversion involves only one arm of chromosome
- Pericentric inversion has breaks on opposite side of centromere
• Isochromosome results when one arm of chromosome is lost and the other is duplicated resulting in a
chromosome containing 2 short arms or 2 long arms only. Most common isochromosome (in live birth) involves long
arm of X [i (x) (q 10)]. & is associated with monosomy for genes on short arm of X and trisomy for genes on long
arm of X
• Translocation 1/t transfer o f a segment o f chrom osom e to another nonhom ologous chromosomeQ
- Reciprocal translocation is mutual swapping o f ends between any tw o non hom ologous chrom osom es <3. It may be
balanced or unbalanced (eg during meiosis).
- Robertsonian translocation (or centric fusion) is tanslocation b/w two acrocentric chromosomes (13,14,15, 21, & 22).
The break occurs close to centromere of each chromosome and transfer of segments 1/t one very long & one extremely
small chromosome. 14-21 fusion may cause Down syndrome.
A B B A
u u
r Paracentric
lo T il
A
I
( 180 °)
t « - B ----- I
A i.e. Genomic Imprinting [Ref: Robbins 7/e p 186; Harrison 16/e p. 375] 11. D i.e. Germline mosacism

D ifferential expression of same gene depending on parent of origin is referred to as genomic imprinting.
Germline (Gonadal) mosaicism results from a mutation only affecting cells destined to form gonads -
postzygotically during early embryonic development. So the individual (parent) is phenotypically normal but can
transmit the disease to offsprings through mutant gametesQ. And because the progenitor cells of gametes carry
mutation, there is a high possibility that more than 1 child of such parent would be affected.
520 ■ A Complete Review of Short Subjects

Genomic Im printing

It is selective inactivation o f a gene or set o f genes on either the maternal or paternal chrom osom e Q. It occurs in ovum or sperm
before fertilization & then is stabily transmitted to all somatic cells through mitosis and so can be of two types

Maternal genomic imprinting Paternal genomic impringint

-
. . . I
It is inactivation of maternal allele (genes or -
I
It is inactivation of paternal allele (gene or
chromosom e) Q chromosom e)
- Paternal allele are functional - Functional allele are provided by only maternal
- Exam ple is Anglem an syndromeQ chromosom es
- Exam ple is Prader W illi Syndrome^

12. D i.e. Fetal blood [R ef: D utta 6"'/108; w zvw .am ericanprcgnancy.org/prenatal testin g]

'Cordocentesis is perform ed under local anaesthetic usually after 18 weeks of gestation'-Dutta 6,h/1 0 8
Cordocentesis is a procedure that takes a blood sample from one of the blood vessels in the baby's um bilical cord, to
directly test the baby's blood. Cordocentesis can only be done later in the pregnancy, from about 18-24 weeks, w hen the
umbilical cord has adequately developed.
Prenatal diagnosis

Chorionic villus sampling Amniocentesis M SAFP Cordocentesis


(choronic villi) (Amniotic fluid) (Maternal blood) (Fetal blood)
10-12 weeks 14-16 weeks 15-18 w eeks 18-20 weeks.
lOweeks to term
(transabdominal)
P ren atal diagn osis at 16zveeks can therefore be perform ed using all o f the above except feta l blood.

13. C i.e. Small intestine (Most probably) M ost abundant im m unoglobin in blood —> IgG.
14. A i.e. IgM antibody » E i.e. Glycoprotein IRef: Harrison 17/e p. 22 5 5 ; Chakraborty Microbiology 1/e p. 141; Davidson 20/e p. 1074]

Rheumatoid (Rh) factor in rheumatoid arthritis is an autoantibody o f IgM class directed against Fc fragm ent o f IgG®.

Antibodies (immunoglobins) are glycoprotein s. And Rh factors is an autoantibody of any im m unoglobin class directed
against Fc fragm ent of IgG im m unoglobin (Davidson).

15. A i.e. Interphase ]Ref: Tortora Principles o f A natom y & Physiology 10/e P-1098; Robbins 7/e P -178; Chandrasom a Taylor 3/e P- 227]

The inactive X can be seen in the interphase nucleus as a darkly staining sm all mass in contact w ith the nuclear
membrane known as the Barr body or X chromatin. Barr body is the inactivated X chromosome. In non dividing
interphase cells it remains tightly coiled and can be seen as a dark staining body w ithin the nucleus.

16. B i.e. FRAP [R ef: L ehn in ger 5/e p 383, 434-36, 760]

( :-----------------------------------------------------------------------------------
- FRAP (fluorescence recovery after photobleaching) primarily measures rate of lateral diffusion of lipids and it can b e
used to investigate diffusion of biological macromolecules (such as proteins etc) including lateral diffusion in the plane
of membrane and movements into & out of nucleus.
- FRET is used to visualize biochemistry of living cells such as to measure cAMP and activity of PKA.
- AFM (atomic force microscopy) is used to yield 3D contour map of membrane (surfaces) and visualize membrane
proteins (eg microdomains or rafts etc)_________________________________________________________________________________

Luminescence

17. B i.e. Release of longer wavelength light on absorbing light of shorter wavelength
18. None [R ef: L ehn in ger 5"'/e p. 435, 509; T eitz 6th/ed pg- 63, 79-80, 167-68, 179]

Fluorescence means release of longer wavelength (lower energy) light on absorbing light of shorter wave length
(higher energy)Q
Biochemistry: Cytogenetics and Molecular Genetics, Recombinant DNA & Genomic Technology ■ 521

19. C i.e. ATP IR ef: L ehn in ger 5 lh/ed pg- 509, 332; T eitz 6lh/ed p g -79]

In firefly, ATP is used in a set of reactions that convert chemical energy into light energyQ. So fire fly flashes are glowing
reports of ATP (i.e. flashes in firefly indicate presence of ATP & SOLIFE)Q

20. A i.e. Light attenuated in intensity by scattering [R ef: Tietz 6/e p. 8 0-179]

Nephelometry determines turbidity by measuring intensity of scattered lightQ.


W hen light passes through a medium with dispersed particles of different refractive index, it is attenuated in intensity by
scattering. The intensity of scattered light depends on its w ave length & particle size. How ever scattering is directly
proportion al to the con cen tration o f p articu late m atter. This principal is used to m easure turbidity by;
I------------------------------------------------------- 1
Turbidimetry Nephelometry
I I
Intensity o f unscattered tran sm itted light Intensity o f scattered lig h t Q usually but not necessarily
is m easu red Q at right angle to the incident beam is measured.

21. B i.e. Spectrophotometer IR ef: L ehn in ger 5/e p. 78; T eitz 5/e p 152]

Quantitative estimation of DNA (or RNA) can be done by UV spectrophotometer at about 260 (-280nm) and
densitometry (after gel electrophoresis).

22. A i.e. FRET; B i.e. Microarray; C i.e. RIA; E i.e. ELISA [R ef: C hatterjea 8/e p. 834; L ippin cott 6/e p. 472]

Im munoflorescent probes are used in radioimmunoassay (RIA), immunoradiometric assay (IRMA), ELISA, FIA
(fluorescence IA), chemiluminescence (C)IA, FRET (fluorescence resonance energy transfer) and microarray.

Restriction Endonuclease

23. B i.e. It acts at specific site 28. B i.e. Electrophoresis agar gel method
24. A i.e. Cut both strands of ds DNA; B i.e. Cut ends 29. B i.e. DNA ligase
produced are sticky; C i.e. Cut end produced .. 30. A, B, C, E i.e. Palindromic, Produce DNA sticky end,
25. B i.e. Break double stranded DNA Restrict replication of DNA, Breaks at sugar-phosphate..
26. A i.e. Restriction endonuclease recognizes specific 31. A, B, D i.e. Palindromic sequences observed; Protects
sites of DNA; B i.e. Restriction endonuclease bacteria from infection by virus; Restrict replication ...
reconises short sequence of DNA 32. D i.e. Cut DNA at palindromic sites IRef: Harper 29/e p 434-36;
27. B i.e. Agarose gel electrophoresis Chatterjea 7/e 261-63; Lippincott 5/e 465-66; Lehninger 5/304-7,1052-55]

- Restriction endonuclease (restriction enzyme) makes 2 incisions through sugar phosphate backbone (phospho
diester bond) of each strand o f ds DNAQ at specific sequence k/a recognition sequences or restriction site. So RE
restricts viral (bacteriophage - DNA) replication^ and protects bacteria (prokaryote) from infection by virusQ.
- RE cuts both strands of double stranded (ds) DNA at specific restriction sites with palindromic (inverse repeat)
arrangement; thus producing smaller, manageable fragments with short sequenes and sticky /blunt endsQ. These
restriction fragments can be isolated by agarose gel /polyacrylamide - electrophoresis^ or HPLC.
- A fter digestion by restriction endonucleases the DNA ends can be ligated (joined /annealed) b y ' DNA-ligaseQ

F e a tu re s T ype I RE T y p e III RE T y p e II R E
Structure, S in g le la rg e , m u ltis u b u n it, S in g le la rg e , m u ltis u b u n it, Simple h o m o d im e rs (2 identical
components & m u ltif u n c t io n a l enzyme containing both m u ltif u n c t io n a l enzyme proteins linked together) with
activities e n d o n u c le a s e (n u c le a s e ) & m e th y la s e containing both e n d o n u c le a s e s e p a ra te n u c le a s e & m e th y la s e
a c tiv itie s in o n e e n z y m e (n u c le a s e ) & m e th y la s e a c tiv itie s activities (ie both not in one enzyme).
in o n e e n z y m e
N u c le a s e s u b u n it - H e te ro - tr im e r - H e te ro -d im e r - H o m o d im e r
s tru c tu re & - R e q u ir e A T P for cleavage - A T P n o t r e q u ir e d for cleavage - A TP not required for cleavage
R e q u ir e m e n ts fo r - Cofactors A T P , M g ++, S A M - Cofactors M g +*, S A M - Cofactors M g ++
c le a v a g e - 2 recognizition sites in a n y o r ie n ta tio n - 2 recognizition sites in h e a d to - S in g le recognizition site
h e a d o r ie n ta tio n
522 ■ A Com plete Review of Short Subjects

S ile o f D N A - Cleaves at ra n d o m s ite s th a t c a n b e > - Cleaves D N A , 25 bp (24-26) - Cleaves D N A within recognized


c le a v a g e and 1 0 0 0 b a s e p a irs (bp) away from from recognition site (towards 3' sequence itself (ie at or near RS)
m e th y la tio n recognition sequence (RS) end) - Methylates at RS
- Methylates at RS - Methylates at RS
DNA - Present - Absent - Absent
tr a n s lo c a tio n - R e q u ir e A T P fo r m o v in g a lo n g D N A - Require A TP for moving along - R e q u ir e n o A T P
DNA
R e s tr ic tio n By single multifunctional enzyme By separate enzymes sharing a Separate restriction (nuclease) &
m o d ific a tio n common subunit modification (methylase)
E n z y m e tu rn o v e r Absent Absent Present
Type II RE is first isolated by Hamilton Smith (Beta H ind 1) and Daniel Nathans first used them to cut D N A for m apping & analyzing
genes & genome.

33. A i.e. Alters gene expression; B i.e. Genetic code remains intact; C i.e. Role in carcinogenesis; D i.e. Protective
mechanism against cleavage by restriction endonuclease [Ref: Lehninger 5/e 994-96, 292, 223, 233; Wikipediaj

• DNA m ethylation alters the gene expression pattern in cells-for example it may cause X -chrom osom e in activation ,
tum or su ppressor gene silencing (in cancer cells)®, suppression of expression of repetitive elements / deleterious
elements / viral genes that have been incorporated into genome of host over time
• E. coli has 2 prominent DNA m ethylation system , both using S-adenosyl methionine (SAM) as a methyl group donor.
Adenine or cytosine methylation in restriction - m odification system , serves as a p art o f defence m echanism again st
cleavage by restriction endonuclease®. It helps the cell to distinguish its DNA from foreign DNA by marking its own
DNA with methyl group and destroying (foreign) DNA without the methyl group.
Mismatch repair mediated by Dam methylase improves the overall fidelity of DNA replication by fetor of 102 - 103.
Dam (DNA adenine Methylation) methylase, methylates adenosine residue to N6- methyl adenosine within the
palindromic (51) GATC (3) sequence. This can serve to distinguish parent/template strand (which is methylated) from
unmethylated (transient state) newly synthesized strand. So replication mismatch in vicinity of hemi-methylated
GATC sequence are then repaired a/t the information the methyated parent strand.
• In eukaryotic cells ~ 5% of cytidine in DNA are methylated to 5 methyl cytidine. Methylation is most common it CpG
islands (a C G rich region, upstream of promoter region and P signifies phosphodiester bond between them) producing
methyl -CpG symmetrically on both strands of DNA.

34. C i.e. Bacteria cannot survive because it lacks DN A m ethylase that protects the host DNA

Any bacteria is protected against its own restriction endonuclease (RE) by a companion enzyme site specific DNA
m ethylase w hich m ethylates the restriction sites recognized by th at p articu lar RE®. In other wors, every bacteria has
its own pair of RE & site specific DNA methylase
So if any bacteria aquires new /different RE by horizontal transfer (i.e. from other bacteria) this restriction enzyme
will cleav e (lyse) the b acterial DNA and k ill bacteria® because the restriction sites for this RE are not methylated &
protected.

Polymorphism and RFLP_________________

35. C i.e., Single phenotype, single locus, m ultiple normal allelle


[Ref: Harper's 27/e p. 413-16: Lippincott's 4/e p. 473-79; Robbins 7/e p. 189-91,146,150]

1 '-------------------------------- ;— — ' 1---------:— ;--------


Polymorphism is presence pfsfhgle normal phenotype with m ultiple normal allele (DNA sequence varioations) in
single locus (site)Q in >1% pophfeticm.

36. A i.e. Detected by southern blot; C i.e. Used for identification of gene ..; D i.e. RFLP is a DNA variation sequence
37. A i.e. Analysis of Chromosal structure 38. C i.e. RFLP
39. A i.e. Detects M utation B i.e., Recognizes trinucleotide repeats C i.e., Detects deletion D i.e., Blunt end produced
[Ref: Lehninger 5/e p 319; Lippincot 5/e p 474-76]
- RFLP is a DNA sequence variation observed by cleaving the DNA into fragments by restriction endonuclease (RE)
e n z y m e These restriction fragments are separated by agarose gel electrophoresis on the basis of size and detected by
southern blotQ.
- RFLP results from single nucleotide polymorphism (SNPs), single base changes (mutation), deletions, insertions
(copy num ber variations = CNVs), length polymorphism /variable num ber of tandem repeats (VNTR), like mini
satellite repeats and microsatellite or short tandem repeats (STR). All of these result in gain /loss of a restriction site;
thereby producing an extra /fewer fragments.
Biochemistry: Cytogenetics and Molecular Genetics, Recombinant DNA & Genomic Technology ■ 523

- RE cuts both strands of double stranded (ds) DNA at specific restriction sites with palindromic (inverse repeat)
arrangement; thus producing smaller, manageable fragments with short sequenes and sticky (cohesive, overlapping,
staggered)/ blunt ends <2. These restriction fragments can be isolated by agarose gel /polyacrylam ide - electrophoresis Q or
HPLC.
- RFLP is used for identification o f gene fo r genomic mapingQ, establishing linkage (thus analyzing chromosomal
structure and measurement / identification of DNA fragments or molecules of different size), which by process of
chromosomal w alking will eventually define disease locus and isolate any gene in unknown location of large DNA.

40. C i.e. Short sequence (2-5) repeat DNA [R ef: R obbin s 7ll'/307; H arrison s 16"'/449; h ttp ://on ew eb .u tc.ed u /~ M arg aret-
K ovach/m icrosat.htm l 1

'M icrosatellites are tandem repeats of one to six nucleotides scattered throughout the genome'

DNA Fingerprinting

41. B i.e. Variable N um ber Tandem Repeats (VNTR)

Variable number of tandem (one after another) repeats (VNTR), is unique for any individual & therefore serves as
molecular DNA finger print. VNTR may be short tandem (microsatellite) or large tandem (minisatellite) repeats.

42. C i.e. Jeffrey 43. D i.e. All of the above [R ef: L ehn in ger 5/e p. 3 19-21; Lippincott's 5/e p. 83, 475: H arp er 2 8 /e p. 404]

PCR
44. A, B, D i.e. Carried out by therm ostable DNA polym erase; Exponential; Specific
45. A & E i.e. Therm ostable enzym e is needed & Prim er is needed. 46. A i.e. Enzym atic DNA am plification
47. D i.e. Required atleast 1 week time for synthesis; E i.e. DNA polym erase has to .... 48. A i.e. T herm ostable enzyme
49. B i.e. DNA am plification; D i.e. Detection of infecting organism 50. A i.e. Primers required, C i.e. DN A polym erase
51. B i.e. DNA Polym erase 52. A i.e. Therm ostable, D i.e. Replicates double stranded DNA
53. A i.e. Therm ostable at tem perature.. 54. B i.e. Obtained from bacteria; D i.e. It is heat stable; E i.e. Add nucleotide
55. B i.e. M g++ 56. D i.e. R adiolabelled DNA probe 57. C i.e. Dideoxynucleotides 58. C i.e. PCR
[R ef: V nssudevnn 6"'/e p. 6 1 6 -1 8 ; L eh n in g er 5"'/e p. 3 1 7 -1 8 ; C hnttarjea 7,h/e p. 2 6 7 -7 2 ; H a rp er 2 8 lll/e p. 395, 3 1 9 ; L ip p in cott 5 lh/e p. 4 7 9 -8 2 ;

59.

60 .
524 A Complete Review of Short Subjects

Ligase Chain Reaction

61. A i.e. PCR > B i.e. NASBA; C i.e. Ligase chain reaction IRef: Lippincottjs 4/e p. 479-83: Chatterjea 7/e p. 267- 71; Robbins 7/e p- 3611

DNA amplification techniques include - polymerase chain reaction (PCR), real time PCR, nucleic acid sequence based
amplification (NASBA), ligase chain reaction (LCR) and Gap LCRQ. These are also k/a nucleic acid amplification tests
(NAAT's).

Cloning

62. C i.e. To identify desired chrom osom al DNA insert in plasmid vectors.
63. A i.e. Produced by hybridom a technology; C i.e Requires in small quantity; D i.e. Used for blood grouping
64. A, D, & E i.e. Im m ortalise m yeloma cell, Prior im m unization is done & m yeloma & Produce m onospecific &
monoclonal Ab [Ref: Harper 29/e p 647-48; Vasudevan 6/e p 617]

- ft
- Hybridoma is an immortal (permanent) cellQ produced by cell fusion o f B cells (previously immunized or injected
with antigen) and myeloma cells o f mouseQ (or suitable animal). It produces m onospecific monoclonal antibodies^
which can be humanized (used in humans) by attaching the complementarity - determined regions [CDRs], (the site
that binds antigens) onto appropriate sites in human immunoglobulin molecule. Attempts to produce human
monoclonal antibodies are underway.
- M onoclonal antibodies are produced by hybridoma technology (in vitro) or by one clone secreting specific type of
antibody in m ultiple myeloma (in vivo). It is required in minute (only microliter) quantities and can b e used for
blood groupings.

65. D i.e. Reducing surface tension [Ref: Harper 28/e 580J


Cell fusion (eg. as used in hybridoma technique to produce monoclonal antibodies) can be brought about by

Exposure to polyethylene glycol gels


Electric field induced (i.e. applying small electric current) cell fusions
Laser induced cell fusion
Attaching inactive viral particlesS like inactivated rota virus and Sendai virus.

66. C i.e. Cosmid [Ref: Harper's 27/e P-406-08]

Larger fragment of DNA can be cloned in cosmideS as it combine the best features of plasmide & phage.

67. B i.e. Plasm id, bacteriophage, cosmid [Ref: Chattergca 7/e p. 262/

* Plasmid is sm allest & most Vector /V ehide- DNA insert size (i.e. can accept DNA
com m only used vectors DNA fragment of)
★ 1 Kb = 1000 nucleotide long base Plasmid (PBR 322) 0.01 - 10 kb (smallest)Q
Bacteriophage 1 0 -2 0 kb
(Lambda charon 4A)
Cosmids 35 - 50 kb (largest)P

68. B i.e. Restriction enzyme sites [Ref: Harper's 27/e P- 407; Anantha Narayan 6/e p. 426- 28]

Bacteriophage (or phages) are viruses that infect bacteria and play an important role in gene transfer between bacteria by
mechanism of transduction that confer an tib iotic resistanceP. They usually have lin ear DNA m olecu leQ into which foreign
DNA can be inserted a t several restriction enzym e sitesQ. It can carry 10- 20 kb long DNA segm entQ.
Biochemistry: Cytogenetics and Molecular Genetics, Recombinant DNA & Genomic Technology ■ 525

Site Directed Mutagenesis, Site Specific Recombination, Linkage Analysis & Analyzing
__________________________(Hybridization) Techniques__________________________

69. D i.e. Site directed mutagenesis [Ref: Lehninger 5je p. 312-13]

70. C i.e. RE+ligase 71. A i.e. Detection of characteristic DNA polymorphism in a family is associated with disorder
72. A i.e. Separation of proteins..; B i.e. SDS-PAGE electrophesis..; C i.e. Enzyme linked antibodies..; D i.e. Confirmatory..
73. A, D i.e. ELISA, Western blot 74. B i.e. DNA 75. D i.e. Protein
76. A i.e. m -RN A 77. B i.e. RNA analysis IRef: Harper 29/e p 439-40; Vasudevan 6/e p 612-14;
Lippincott's 5/e p. 485; Lehninger 5/e p. 174,1010-1012; Robbins 7/e p-189-91; Human genetics 2000,8:738-43]

Technique Sample Gel used Probe Purpose


analyzed
Southern blot DNA® Yes Radioactive DNA probes Detects DNA changes & mutation
A llele specific DNA No Allele specific oligo Detects DNA mutation
oligonucleotide nucleotide probe
(ASO)
Microarray M-RNA or No DNA probe - G en otypin g (i.e., gene variation or m utation) by using DNA sample.
C-DNA - G ene expression a n a ly sis (i.e., patterns of m-RNA production) by using m-
RNA & converting it to c-DNA by reverse transcriptase.
- Analyze thousands o f gene & mRNA at same time
- Tum or fingerprint or transcriptional profile used to determine the
differing (global) pattern of gene expression in two different cells types eg
normal & cancer cells (in order to develop most appropriate form of
chemotherapy)._________________________________________________________
RNA® Yes Measures mRNA® amounts & sizes
P rotein ® Yes Measure protein amounts

Protein, Yes Examines protein - DNA (protein-nucleic acid) interactions


DNA (?)
Protein or No A ntibody® (specific for Detects proteins (antigens) or antibodies
antibodies protein to be measured)

Protein® Yes Measures abundance, distribution, post translational modifications, function


& interactions of cellular proteins
T h e b ase sequ en ce o f D N A that have b een cloned & pu rified can be determ ined in the laboratory b y - S a n g e r d id eoxy en zym atic
(m anual) m eth od and M axam & G ilb e rt ch em ical (m an u al) m eth od.

78. A i.e. Antibody IRef: Lippincott's 4/e p. 470- 74,483-84; Harper's 27je p. 409-12,313-14]
526 ■ A Complete Review of Short Subjects

79. D i.e. Northern blot analysis [Ref: Harrisons 16lh/365, 378, 595]

Definitive diagnosis of a point m utation in the beta globin gene requires DNA analysis. Northern blot test is a
technique used to study the RNA, rather than DNA and hence is the answ er of exclusion.
Polymerase chain reaction, DN A Sequencing and southern blot testing are rapid methods for DNA analysis, and can
all be used to detect the point m utation in the globin gens.
T h e advent of polymerase chain reaction (PCR), allele specific oligonucleotide hybridization and automated DNA
sequencing has made it possible to identify globin gene mutations in a few days'. T h e southern blot technique is a fast
w ay of analyzing a small num ber of DNA fragm ents, for instance the sickle cell mutation. Before PCR and cheap fast
sequencing became available, the southern blot was the universal workhorse.
The northern blot is a technique to analyze RNA rather than DNA. This is thus the answ er of exclusion.

80. D i.e. M icroarray [Ref: Lippincott 5/e 483-85; Vassudeven 6/e 614; Lehninger 5/e p. 325-28]

DNA Microarrays (DNA-Chips) is used for detection or screening of variations in DNA sequences and gene expressionQ
analysizing thousands of gene at a same time.

81. D i.e. Microarray ]Ref: Harrison 17/e p 406, 390; Robbin's 7/e p 171-73]

- FISH (Fluorescent in situ hybridisation) and PCR (polymerase chain reaction) like RT-PCR and QF PCR tests can detect
specific anemploidyQ.
- Whereas, microarray based CGH (comparative genomic hybridization) test can be used as high resolution whole
genome scan for the detection or screening of unknown (non specific) abnormalities, mutation or genomic imbalance
including deletions, duplication and aneuploidies. Microarray based CGH cannot detect balanced translocations or
inversions.

- So FISH and PCR can detect specific (known) chromosomal abnormalities whereas microarray based CGH is better
suited for screening unknown (non specific) chromosomal abnormalities.

82. E i.e. Immuno histochemistry » C i.e. Southern Blot [Ref: Harrison 18/e p 508; Lippincott 5/e p 465-85]

Immunohisto chemistry detects antigens (eg proteins) in abnormal (eg cancerous) or normal cells of a tissue by antibodies
binding specifically to that antigen (so immune means antigen-antibody binding and histo means in tissue). It is used to
find distribution & localization of biological marker in various tissues.

Gene Library

83. D, E i.e. DNA Nucleotide or fragment, Complete collection of gene copies of one organism [Ref: Lehninger 5/e 315;
Lippincott 5/e 469-70; Chatterjea 7/e p-265; Vassudeven 6/e p 516; Salyanarayan 3/e p 596-97]

Genomic DNA (gene) library contain a copy of every DNA nucleotide sequence in genomeQ, whereas. c-DNA (gene)
libraries contain those DNA sequences that transcribe as m RNA.

Recombination DNA Technology or Genetic Engineering

84. A, B, C, D, E i.e. Viral vaccine production; To cure genetic disorder; Genetic crop; Gene production, Production of
genetic products [Ref: Lehninger 5/e 337; Vasudevan 6/e 512-14; Chattarjea 7/e-275, 279]
85. C i.e. mRNA from beta pancreatic cells of human: [Ref: Harper 28/e p388; Lehninger 5/e p 334-37, 316]

Starting material for production of insulin from bacterial cells (E.coli) using recombination DNA technology is mKNA
from beta pancreatic cells of human®.

86. B i.e. RNA dependent DNA polymerase [Ref: Harper's 29/436; Cjhatterjea 7lh/e p. 261-63; Lehninger 5/e p. 305]
87. A, B & D i.e. Restriction endonucleases involved, DNA ligase is used, Reverse transcriptase.. 88. E i.e. None
Biochemistry: Cytogenetics and Molecular Genetics, Recombinant DNA & Genomic Technology ■ 527

Enzymes used in recombination DNA technology are nuclease (BAL-31, S I), exonuclease (III, X), endonuclease
(restriction), DNase I, DNA polymerase I, DNA ligase, reverse transcriptase (RNA dependent DNA polymerase) Q,
polynucleotide kinase, terminal transferase and alkaline phosphatase (not acid phosphatase)Q

89. B i.e. Genes for protease inhibitors I Ref: H erper 2 8 /e 3921

- Expression vector is a vector which synthesize/produce/express the protein coded by a gene introduced via DNA
recom bination technology. So it is used to produce protein by genetic engineering and detect specific c-DN A molecules
in library.
- Expression vectors are specially constructed so that they contain very active inducible promoters, proper in phase
translation initiation codons, b o t h t r a n s c r ip t io n & t r a n s la t i o n t e r m in a t io n sig n a lsQ (to increase transcription &
translation) and appropriate protein processing signals, if needed (to produce processed function protein).

- To enhance the final yield of protein product, some expression vectors even contain genes that code for protease
inhibitors^.

90. A i.e. Genomics 91. A i.e. Protenonom ics


92. A, E > B, C i.e. (Gene Array Chips, Tw o-D im ensional Electrophoresis >M ass Spectroscopy; PolyA crylam ide Gel
Electrophoresis (PAGE) [R ef: L ippin cott's 4/e p. 4 8 4 -8 5 ; H arp er 2 9 /e p 28-33; S ted m a n ’s diction ary 27 je p-739]

r- Study of structure & products of gene is genomics. Genomics enables protein to be identified from small amount of ^7 ‘
sequence data.
- Study of multiplication of proteins in disease process is called proteomics. The goal of proteomics is to identify the
entire complement of proteins elaborated by a cell under diverse conditions^.
- Two dimensional electrophoresis, gene arrays (DNA chips), multidim ensional protein identification technology
(Mud PIT) employing successive rounds of chromatography are used to survey protein expression in proteomics.
After 2D electrophoresis, individual polypeptides are extracted & analyzed by Edman sequencing or mass
spectroscopy.
- Protein purity is most widely assessed by polyacrylamide gel electrophoresis (PAGE) - SD S (sodium dodecyl sulfate).
Isoelectric focusing (IEF) is used in conjunction with SD S-PA GE for 2D (two dimensional) electrophoresis, which
seperates polypeptides based on pi on one dimension and molecular mass (Mr) in second. 2D electrophoresis is
particularly used for separating the components of complex mixtures of proteins.

Transgenic Organism

93. A, D i.e. Knock-out, 94. A i.e. Developed from DNA insertion into fertilized egg; B i.e. Have same
Targeted gene disruption genome as parents except one or more genes; E i.e. Homozygous are selected:
528 ■ A Complete Review of Short Subjects

97. A i.e. Both plants will glow [R ef: L ehn in ger 5/e 316, 434, 332]

- Both the plants expressing genes for firefly luciferase and (green /yellow /blue /cyan) fluorescent protein of jelly fish
(Aequorea victoria) vriRglow in dark®.

- Fluorescent probes are derived from green fluorescent protein (GFP) of jelly fish. It is an 11-stranded (3-barrel and light
absorbing /emitting center (chromatophore) of protein comprises a modified (oxidized) form of tripeptide - Ser65 - T y r 66
- Gly 68 located within the barrel. Oxidation of tripeptide is catalyzed by GFP itself, with no other protein or cofactor
required other than oxygen®. So it can be cloned into virtually any cell, where it can serve as a fluorescent marker for
itself or for any protein to which it is fused. When exited by absorption of photon of light GFP emits fluorescent, photon
in green region. A related protein (m RFP1) fluoresces red light.
- Actually luciferase containing transgenic plants emit quite weak light and a prolonged exposure (~ 24 hour) pictures
show them glowing in dark.

Gene Therapy

98. A i.e. Replacem ent of abnormal gene by normal gene.


99. A i.e. Electroporation, B i.e. Intranuclear infection, C i.e. Site directed m utagenesis, D i.e. Retrovirus
100. A i.e. SCID IRef: Harper's 28/e P-380, 400, 504, 625: Lippincott’s 5/e P- 485-56; Lehninger 5/e p. 331-35, 312, 1004; Chatierjea 7ih/e p. 278-87]

- The goal of gene therapy is replacement of abnormal mutant gene by normal gene®. ----------------------------------f r
- Technique to transfer genes (foreign material) into cells include - transfection, microinjection, electroporation, site
directed recombination, transduction using adenovirus or retrovirus and plasmid liposome complex® etc.
- SCID (Severe combined imuunodeficiency disease) or bu bble baby syndrome is the first disease treated
successfully by gene therapy.

Genetic Disorder

101. A i.e. X-gene; D i.e. Seen in fem ales [R ef: K um ar & C lark 6/e p. 172-73; H arrison 16/e p. 375]
Although females have 2X chromosomes (XX), but due to inactivation (lyonization) genes on one X chromosome becomes
transcriptionally inactive. Thus X inactivation prevents expression of m ost genes on one of the X- chrom osom e in every
cell of female®.

102. D & E i.e. Duchenne m uscular dystrophy & G- 6-P dehydrogenase deficiency
103. C i.e. X - linked recessive inheritance [R ef.: C handrasom a T aylor 3 nl/e P -234, 235, 236; R obbin s 6"'/e P -146]

Single X-chromosome of son is achieved from his mother, not father; so x-linked diseases never transmit from father
to son®.

104. A i.e. Autosom al dominant disorder [R ef: H arrison 16/e P- 374]

Autosomal dominant disease cannot occur in offspring o f an unaffected individuals®. Because the allele is dominant it
will always express phenotypically (clinically) and hence both parents can't be normal in autosomal dominant disorders
even if they have a single abnormal gene.

105. C & D i.e. Cystic fibrosis & Ataxia telangiectasia [R ef: H arrison 17/ed pg- 398-401, 494; R obbin s 7lh/ed p g - 151-52]

Ataxia telangiectasia & cystic fibrosis are autosomal recessive disorders®. Tuberous sclerosis is AD® and Duchene /
Becker muscular dystrophy and hemophilia A (factor - 8 deficiency)® are X-linked recessive disorders.

106. D i.e. A polygenic syndrom e [R ef: w w w .som .tu lan c.edu /departm en ts/peds_respcare/gen etic.htm ]

A syndrome with facial anomalies including a thin upper lip, growth delay and developmental disabilities is likely to
result from either a teratogenic congenital malformation, a single gene defect (Mendelian) or a chromosomal disorder. A
trait is called polygenic if multiple genes are thought to contribute to the phenotype and includes conditions like diabetes
and hypertension. A polygenic disorder is the least likely diagnosis amongst the options provided and the single best
answer of exclusion.
• M endelian Syndrom e presenting with above features is the Cornelia de Lange syndrome. (Am sterdam Dwarf
Syndrom e). This is a mendelian syndrome with either autosomal recessive or autosomal dominant inheritance, and is
characteristically associated with a thin upper lip, small head size (microcephaly) and other developmental disabilities.
Biochemistry: Cytogenetics and Molecular Genetics, Recombinant DNA & Genomic Technology ■ 529

• Teratogenic syndrome presenting with features in question is the fetal alcohol syndrome. Fetal alcohol syndrome
results from the teratogenic effects of alcohol on the fetus during pregnancy and characteristic features include a thin
upper lip, small head size (microcephaly) and other developmental abnormalities.
• An example of a chromosomal disorders presenting with features in question is the M iller D icker Syndrome. (M D S).
Miller Dicker Syndrome is a chromosomal syndrome with a mendelian pattern of inheritance. Chromosomal anomalies
include deletion of short arm of chromosome 17 (17 p 13.3) and characteristic features include a thin lip, microcephaly
and other developmental disorders.

107. A i.e. Autosom al dom inant (most likely) [Ref: http:ljwww.babyhearing.org/HearmgAamplificationl Causes/BasicGenetics.pdf;
http:jjwww.emedicine.com/pedltopic2422.htm]

Waardenburg's Syndrome (WS) is a rare autosomal dominant syndrome characterized by pigmentary disturbances (skin,
hair, iris), sensorineural hearing loss, and other developmental anomalies such as dystopia canthorum (widely spaced
eyes) and blepharophimosis.
The syndrome that closest matches the features provided in the question is Waardeburg syndrome type-I. This is
inherited as an autosomal dominant fashion and hence is the answer o f exclusion.
As the question does not provide us with details as to the sex of the three children, a sex linked inheritance pattern cannot
be worked out. Also the combined presentation of an affected father, unaffected mother and one affected child among
three children is possible with both an autosomal dominant and an autosomal recessive inheritance pattern. In absence of
any more details, the answer to this question cannot be deduced through the exploration of various permutations and
combinations. How both autosomal dominant and autosomal recessive inheritance can be responsible for the above
combination:

(A-affected allele and a normal allele) (‘A ’-affected allele and ‘a ’ normal allele)
If the disease is transmitted in an Autosomal Dominant If the disease is transmitted in an Autosomal recessive
then: fo rm :
Father (affected) will be either 'AA' or 'Aa' Father (affected) will be 'AA'
Mother (unaffected) will be 'aa' Mother (unaffected) may be 'aa' or 'Aa'
If father is AA and mother aa, the offsprings w ill be If father is AA and mother aa, the offsprings will be
1. a A - 100% affected 1. aA -1 0 0 % not affected, (but carriers)
If father is Aa and mother aa, the offsprings will If father is AA and mother Aa, the offsprings will be
1. Aa - 50% affected 1. Aa - 50% not affected, carrier
2. aa - 50% not affected 2. AA - 50% affected
The scenario in the above question therefore is possible The scenario in the above question therefore is possible
autosom al dominant inheritance if fa th er is Aa and with autosom al recessive inheritance if father is aa and
m other is aa mother is Aa

108. D i.e. Short arm of chromosome 6 [Ref: Harrison 18/e p.497, 2680, 2685-94; Ganong 22/e P- 526; Robbins 7je P 203]
109. A i.e. Transplantation reaction; B i.e. Autommune disease; D i.e. Involved in T -C e ll function
110. B i.e. Dendritic cells; F i.e. B-cell 111. A, B i.e. Macrophage; Dendritic cells; C i.e. Lymphocytes

- Genes encoding major hi^i|piiipatibility complex (MHC) or


because MHC encoded ai|i|gfts-were initially detected on
short aim of chromasoitte ISil&Stis involved in
- MHC II are expressed ( p i p ^ j^ ) b y antigen presentingcells
and activated T cellsQ.

112. C i.e. Somatic Hypermutation [Ref: Principles of Cellular & Molecular Immunology by Austin and Wood 1993/408, 449, 450]

A ffinity Maturation refers to the increased affinity of antibodies for their specific antigen as the antibody response
continues. This phenomenon is not seen in IgM responses.
Those B-cells expressing receptors with a higher affinity for the antigen are selctively expanded, even as the levels of
antigen fall. Thus B-cells with higher affinity are generated during the 1° & 2° immune respones in large part due to
Somatic hypermutations that occur in V h genesQ.
(VH=Variable region of Heavy chain).
Chapter -5 STRUCTURE S FUNCTIONS OF PROTEINS AND ENZYMES

Q U E S T IO N S

Structure of Protein B. Molecular aggregation □


C. Precipitation □
1. The primary structure of a protein refers to: (AIIMS 04) D. Quaternary structure □
A. Linear structure and order of the amino acids 9. An Immunoglobulin molecule represents the
present □ follow ing level of organized protein structure: (AI 12)
B. Regular confirmational forms of a protein □ A. Primary structure □
C. Complete three-dimensional structure of the B. Secondary structure □
polypeptide units of a given protein □ C. Tertiary structure □
D. Subunit structure of the protein □ D. Quartenary structure □
2. Alpha helix and Beta pleated sheet are examples of: 10. True about Isopeptide bond: (PGl 11)
A. Primary structure (NBE P 13; A I 04) □ A. It makes protein resistant □
B. Secondary structure □ B. Bond is formed b/w carboxyl terminus of one
C. Tertiary structure □ protein and the amino group of a lysine residue on
D. Quaternary structure □ another □
3. Which one of the follow ing is not true for an a-helix: C. Involves in post-transcriptional modification of
A. It is one of the most important secondary structure □ protein □
B. It has a net dipole moment (AIIMS 04) □ D. Enzyme acts as catalyst for the bond formation □
C. All hydrogen bonds are aligned in the same 11. Stability of the toxoid in snake's venom is due to:
direction □ A. Disulfide bonds (NBE P 14; AIIMS 09) □
D. Long stretches of left handed a-helices occur in B. Hydrogen bonds □
proteins □ C. Ionic bonds □
4. An amino acid which does not participate by ahelix D. Van Der Wall's bond. □
formation is: (AI 06) 12. Keratin in skin is softer than keratin in nail because
keratin in skin has: (AI 12)
A. Leucine □
B. Glycine □ A. Less number of disulphide bonds □
B. Less number of salt bridges □
C. Proline □
C. High sodium content □
D. Lysine □
5. An alfa helix of a protein is most likely to be disrupted D. Different affinity for water □
if a missense mutation introdues the following amino 13. Keatin contains- (NBE P 15)
acid with in the alpha helical structure: (AIIMS 02) A. Arginine □
A. Alanine □ B. Histidine □
B. Aspartic acid □ C. Lysine □
C. Tyrosine □ D. All □
D. Glycine □ 14. Biomineralization is seen in: (AIIMS 13)
6. Folding of protein chain is due to : (AIIMS 97) A. Pinna □
A. Amide bond □ B. Molluscs shell □
B. Hydrogen bonds □ C. Hooves of cattle □
C. Phosphodiesterase bonds □ D. Nails □
15. D ifferent sequence of amino acids having similar
D. Disulphide bond □
structure of proteins is an example of
7. The 3 dimensional shape of a protein is maintained
A. Divergence (AIMS 07) □
mainly by: (AIIMS 03)
B. Convergence □
A. Strong covalent interactions □
C. Opportunistic □
B. Interactions with other proteins □
D. Incidental □
C. Multiple weak interactions □
16. There are more than 300 variants of human
D. Interactions with prosthetic groups □
hemoglobin gene. Among these only a few are fatal.
8. Proteins are linear polymers of amino acids. They fold
Hence, the most important factor to be conserved in a
into compact structures. Sometimes, these folded
protein for its function is the:
structures associate to form homo-or hetero-dimers.
A. Amino acid sequence (AI 06) □
Which one of the following refers to this associated
form? (AI 06) B. Ligand binding residues □
A. Denatured state □ C. Structure □
D. Environment □
Biochemistry: Structures & Functions of Proteins and Enzymes ■ 531

17. W hich of the following is not true in relation to 26. Characteristics of glycoprotein: (PGI 07)
glycoproteins: (P G I05) A. Protein linked with glycosidic bond □
A. Are proteins to which oligosaccharide are covalently B. Core protein □
attached □ C. Sugar residues are long in carbohydrate portion of
B. The carbohydrate units of glycoprotein have several glycoprotein. □
repeats □ D. Participate in cell surface recognition. □
C. Carbohydrate content is same but proteins are 27. All are true about denatured proteins except:
different □ A. Amino acid sequence remains intact □
D. Carbohydrate chain is often linear □ B. The biological function remains intact □
18. W hich of the follow ing is a com ponent of C. The isoelectric pH changes (AIIMS 08) □
polysaccharide- (PGI 04) D. Recovery after denaturation is not possible □
A. Synovium □ 28. True about denaturation of proteins is all except:
B. Glucosamine □ A. Unfolding occurs (AIIMS 08) □
C. Glucoronic acid □ B. Disruption of secondary structure occurs □
D. Ascorbic acid □ C. Sequence of amino acids remain the same □
19. Regarding proteoglycans, false is: (AI 07) D. Biological activity is retained □
A. Chondroitin sulfate is a Proteoglycan □ 29. Random arrangement of molecule results in
B. They hold less amount of water □ A. Crystallization (AIIMS 08) □
C. They are made up of sugar and aminoacids □ B. Precipitation □
D. They carry charge □ C. Denaturation □
20. Com pounds with large am ount of carbohydrate & D. High Solubility □
small am ount of protein (Jipmer 06, SG P G I07) 30. Protein that precipitates on heating to 45° C and
A. Glycoprotein □ redissolves on boiling is: (AI 12, 07)
B. Glycosaminoglycan □ A. Bence Jones Protein □
C. Proteoglycan □ B. Gamma globulin □
D. Glycocalyx □ C. Albumin □
21. W hich of the follow ing am inoacids in a protein D. Myosin □
com m only acts as a potential O -G lycosylation site for 31. Optical transmission through a solution depends on:
attachm ent of an oligosaccharide unit: (AI 12) A. Time (PGI 03) □
A. Glutamine □ B. Concentration □
B. Cysteine □ C. Scale □
C. Serine □ D. Path length □
D. Asparginine □ E. Spectrophotometer
22. All steps of N- glycosylation occur in ER except 32. Cytochrome C of the bacteria has 50% identity of
A. Dolichol synthesis (Jipmer 04, TN 05) □ amino acid sequence with that of human. W hich of the
B. Glycosyl transferase □ follow ing is the most conserved parameter in these
C. Protein- oligosacharide transferase □ two proteins?
D. Final trimming □ A. Quarternary structure (AI 04) □
23. G lycosyl transferase of O -glycosylation is found in B. Tertiary structure □
A. RER (WB 03, Assam 04) □ C. Amino acid sequence □
B. SER □ D. Loop and turn segments □
C. Golgi □ 33. Which of the follow ing is a denaturing substance?
D. Lysosome □ A. Guanosine (AIIMS 05; AI 06) □
24. Difference betw een High m annose & com plex B. Guanidine □
glycoprotein (AMU 03, UP 02) C. Glutamate □
A. Core pentasacharide □ D. Glycine □
B. Protein chain □ 34. The structural proteins are involved in m aintaining the
C. Asparagine □ shape of a cell or in the formation of matrices in the
D. Presence of G1C - NAc - Gal - NANA in complex body. The shape of these proteins is: (AI 06)
glycoprotein □ A. Globular □
25. Follow ing organelles are involved in the form ation of B. Fibrous □
N- glycosylated product: (PGI 07) C. Stretch of beads □
A. Golgi apparatus □ D. Planar □
B. Nucleolus □ 35. Structural proteins OR Keratin is OR In a centrifugal
C. SER □ distribution, protein that is precipitated first:
D. RER □ A. Fibrous (AIIMS 08) (NBE P 14) □
E. Peroxisomes □ B. Globular □
532 ■ A Complete Review of Short Subjects

C. Branched □ 45. Which of the following estimates blood creatinine


D. Compound □ level most accurately (AIIMS 06)
36. True about Collagen: (PG111) A. Jaffe method □
A. Triple helix □ B. Kinetic jaffe method □
B. Beta- pleated structure □ C. Technicon method □
C. Vit C is necessary for post-translational D. Enzyme assay □
modification □ 46. False positive test of protein in dipstick test seen in:
D. Glycine residue at every third position □ A. Chlorpropamide (PGI 07) □
37. Which of the following are intracellular events B. IV contrast agent □
occurring in fibroblast during synthesis of collagen: C. Viral infection □
A. Hydrolysis of procollagen to collagen (P G I08) □ D. IV administration of drugs □
B. Glycosylation of proline □
C. Formation of triple helix □
D. Formation of covalent cross link b/w fibrils □
E. Self assembly of tropocollagen molecules into
47. Protein separation based on mass /molecular weight
fibrils □
(size) is /are done in all except: (PGI 12)
38. The 40 nm gap in between the tropocollagen molecule
A. Ultrafiltration □
in collagen which serve as the site of bone formation is
B. Native gel electrophoresis □
occupied by which of the following- (AIIMS 11, 06)
A. Carbohydrate □ C. 2D gel electrophoresis □
B. Ligand moiety □ D. Gel Filtration chromatography □
C. Ca++ □ E. Ultracentrifugation □
D. Fe+++ □ 48. Protein fragments separation is /are done by:
39. Albumin and globulin are classified as: (PGI 07) A. Western blot (PGI 10) □
A. Conjugated proteins □ B. Chromatography □
B. Simple globular proteins □ C. Centrifugation □
C. Unconjugated proteins □ D. Ultrafiltration □
D. Derived proteins □ E. Electrophoresis □
40. Which is /are not transport protein (s): (PGI 12) 49. In SDS-PAGE (Sodium Dodecyl Sulfate-
A. Transferrin □ Polyacrylamide G el Electrophoresis) proteins are
B. Collagen □ separated on basis of: (PGI 09)
C. Ceruloplasmin □ A. Mass □
D. Hemoglobin □ B. Charge □
E. Albumin □ C. Density □
41. Which of the following is/are storage protein : D. Molecular weight □
A. Myoglobin (PGI 1 1 ) 0 E. Solubility □
B. Ovalbumin □ 50. Proteins are separated on the basis of size by:
C. Ricin/Hepcidin □ A. SDS-PAGE (PGI 03) 0
D. Ferritin □ B. HPLC □
E. Glutelin □ C. Affinity Chromatography □
42. A Protein estimation test is confused with: (AI 06) D. Ion-exchange Chromatography □
A. Phosphates □ 51. Separation of proteins based on size is done by:
B. Nitrates □ A. Affinity chromatography depending on charge □
C. Sulphates □ B. SDS-Polyacrylamide gel electrophoresis (PGI 01) □
D. Bile salts □ C. Ion exchange chromatography □
43. Intact peptide bond is necessary for which test: D. High performance liquid chromatography □
A. UV diffraction (PGI 08) □ E. Electrophoresis □
B. Ninhydrin □ 52. Protein purification and separation can be done by all
C. Diazo reaction □ except:
D. All □ A. Chromatography (AI 07) □
44. All of the following assays are used for estim ation of B. Centrifugation □
total serum protein, except (AI 12) C. Electrophoresis □
A. Biuret □ D. Densitometry □
B. Bradford □
C. Lowry's □
D. Bromocresol green □
Biochemistry: Structures & Functions of Proteins and Enzymes ■ 533

Ammonium Sulfate Fractionation Electrophoresis

53. Protein is purified using ammonium sulfate by: 61. True regarding electrophoresis are all except (PGI 09)
A. Salting out (AIIMS 14, 09) □ A. Isoelectric focusing use ampholytes & pi □
B. Ion exchange chromatography □ B. Adversely affect structure & function □
C. Mass chromatography □ C. Depends on size not on shape □
D. Molecular size exclusion □ D. Commonest method of purifying protein □
E. Use electric field & quickly estimate no. of proteins □
Chromatography 62. Supercoiled DN A is separated by relaxed D N A by:
A. ELISA (AIIMS 1 4 ,12)D
54. The following separation technique depends on the B. Gel electrophoresis □
molecular size of the protein: (AI 03) C. DNA footprinting □
A. Chromatography on a carboxymethyl (CM) cellulose D. DNA fingerprinting □
column □ E. Microarray assay □
B. Iso-electric focussing □ 63. Which process separates the fragments of DNA :
C. Gel filtration chromatography □ A. Gel centrifugation (AIIMS 07) □
D. Chromatography on a diethylaminoethyl (DEAE) B. Paper chromatography □
cellulose column □ C. High speed centrifugation □
55. M olecular separation of two proteins with same charge D. Thin layer chromatography □
can be done by: (PGI 07) 64. The two fragments of DNA are separated by:
A. Ion exchange chromatography □ A. High speed centrifugation (AIIMS 06) □
B. Dialysis □ B. Thin layer chromatography □
C. Gel diffusion chromatography □ C. Paper chromatography □
D. Electrophoresis □ D. Gel chromatography □
56. The best method to differentiate proteins is by: 65. The m olecular weight of a protein can be determined
A. Gel chromatography (PGI 09) □ by - (AIIMS 04)
B. Affinity chromatography □ A. Native Poly Acrylamide Gel Electrophoresis
C. Ion exchange electrophoresis □ (PAGE) □
D. None of the above □ B. Sodium Dodecyl Sulphate PAGE □
57. In chromatography mass movements of the substances
C. Isoelectric focussing □
is due to: (AIIMS 01)
D. Ion Exchange Chromatography □
A. Diffusion □
66. A protein with molecular weight of 100 kD is
B. Electrophoresis □
subjected to SD S PAGE electrophoresis. The SD S
C. Paper chromatography □
PAGE electrophoresis pattern show two widely
D. Osmosis □ separated bands of 20kD and 30kD after addition of
58. In which type of chromatography, the proteins are Mercaptoethanol. The true statement regarding this
bound to another substance: (AIIMS 01) will be. (AI09)
A. Hydrophobic chromatography □ A. The protein has undergone complete lysis □
B. Affinity chromatography □ B. The protein is a monomer of 20 kD and 30kD
C. Paper chromatography □
protein □
D. Gel chromatography □
C. The protein is a dimmer of two 20kD and 30kD
59. Method of chromatography in which molecules that
proteins □
are negatively charged are selectively released from
D. The protein is a tetramer of 20kd and 30kD proteins □
stationary phase into the positively charged molecules
67. Hemoglobin electrophoresis is based on (AIIMS 07)
in m obile phase is termed as: (AI 10)
A. Molecular weight □
A. Affinity chromatography □
B. Charge ^ □
B. Ion-Exchange chromatography □
C. Solubility □
C. Adsorbtion chromatography □
D. Calorimetric properties □
D. Size-Exclusion chromatography □
60. Fastest moving amino acid in m e th a n o l- chloroform
electrophoresis is OR Which amino acid migrates Detection of Protein Structure & Amino acids
fastest on chromatography on carboxy methylcellulose
medium? (AIIMS 14,09,08) 68. Method (s) to determine protein structure is /are:
A. Glycine □ A. X-RAY Crystallography (PGI 09,10) □
B. Aspartic Acid □ B. NMR Spectroscopy □
C. Valine □ C. Electrophoresis □
D. Leucine □ D. Infra red spectroscopy □
534 ■ A Complete Review of Short Subjects

69. All of the following can determine the protein C. Reversible inhibitor □
structure, Except: (AIIMS 08) D. None of the above □
A. High performance liquid chromatography □ 79. In competitive inhibition- (PGI 12, 05)
B. Mass spectrometry □ A. Vmax unchanged □
C. X- ray crystallography □ B. Apparent Km unchanged □
D. NMR spectrometry □ C. Apparent Km decreased □
70. Sequence in a long chain of protein is identified by-
D. Vmax decreased □
A. Restriction fragment length polymorphism (RFLP) □
E. Rate independent of substrate concentration □
B. Chromosome walking (AIIMS 08) □
80. Disopropyl phosphofluoridate (DFP) reacts with serine
C. Leucine Zipper □
proteases irreversibly and therefore is: (PGI 06, 04)
D. SSOP □
A. Allosteric inhibitor □
71. The tertiary structure of protein is detected by:
B. Competitive inhibitor □
A. X- ray diffraction /crystallographs. (AIIMS 09) □
B. Spectrophotometry □ C. Non competitive inhibitor □
C. Electrophoresis. □ D. A repressor □
D. Chromatography. □ 81. The type of enzyme inhibition (in which succinate
72. Trypsin cleaves : (PGI 10) dehydrogenase reaction is inhibited by malonate) is an
A. Arginine □ example of: (AIIMS 06)
B. Glutamate □ A. Non competitive □
C. Lysine □ B. Uncompetitive □
D. Proline □ C. Competitive □
73. All is true about fragment cleavage in 1° structure D. Allosteric □
determination Except (PGI 09) 82. True about reversible non-competetive inhibitors:
A. Trypsin cleaves lysine residue (C) □ A. Lower Vmax (PGI 13,10,09) □
B. Trypsin cleaves arginine next to proline □ B. Lower Km □
C. Trypsin & lysyl endoproteinase cleave C terminal C. Not affect Km □
of lysine □ D. Not affect Vmax □
D. Cysteine reduced by 2 mercapto ethanol □
E. Affect both V max & Km □
E. Performic acid break disulfide bond
83. True about competitive inhibition of enzyme:
74. Chymotrypsin cleaves carbonyl terminal of: (PGI 11)
A. T Km (PGI 1 3 ,1 0 )0
A. Phenylalanine □
B. Arginine □ B. I Km □
C. Lysine □ C. T Vmax □
D. Tryptophan □ D. No change in Km and Vmax □
E. Tyrosine □ E. V max remain same □
75. M olecular size is assesed by : (PGI 04) 84. In noncompetitive antagonism the true statement is:
A. Sedimentation □ (NBE P 14,13; PGI 16,14,12)
B. Absorption mass spectroscopy □ A. Km value decrease; V max normal □
C. Liophilization □ B. Km value decreased; V max decreased □
D. Salting out □ C. Km value normal; V max decreased □
76. The substance present in the gall bladder stones or the D. Km value decreased; V max increased □
kidney stones can be best identified by the following 85. Km increased but Vmax same enzyme is: (PGI 14,09,08)
techniques:
A. Competitive (NBE P 15,14) □
A. Fluorescence spectroscopy (AI 03) □
B. Non competitive □
B. Electron microscopy □
C. Irreversible □
C. Nuclear magnetic resonance □
D. Uncompetitive □
D. X-ray diffraction □
77. Ninhydrin test is used for- (NBE P 15) 86. Km value is defined as: (PGI 04)
A. Bile salts □ A. Substrate concentration at Vmax/2 □
B. Amino acids □ B. Substrate concentration of twice Vmax □
C. Nucleic acid □ C. Substrate concentration of thrice Vmax □
D. Lipids □ D. Substrate concentration of one third Vmax □
87. Enzyme specificity is given by- (NBE P 14)
Enzyme: Kinetics A. K m □
78. Substance which binds to substrate other than catalytic B. Vmax □
enzyme is- (NBE P 15) C. Both □
A. Competitive inhibitor □ D. None □
B. Non-competitive inhibitor □
Biochemistry: Structures & Functions of Proteins and Enzymes ■ 535

88. True about Km (PGI 09) 96. One of the following enzyme is not a protein:
A. Half the substrate concentration at which velocity is A. DNAase (AIIMS 02) □
maximum □ B. Abzyme □
B. Substrate concentration atwhich reaction rate is half C. Eco RI □
the maximum □ D. Ribozyme □
C. Michaelis constant □ 97. Ribozym es is/are : (PGI 02)
D. Dissociation constant of enzyme-substrate complex □ A. Splicing of heterogenous RNA (hnRNA) to form
E. Dissication constant of enzyme-product complex □ mRNA □
89. Hormone substrate concentration affects the velocity B. Splicing of polypeptide chain and mRNA □
of enzymatic action, this is known as - (UP 01) C. Transcription of mRNA □
A. Zimmerman reaction (DNB 2K, AIIMS 0 4 )0 D. Recognition of codons in introns □
B. Salkowski reaction □ E. RNA polymerase □
C. Michales Menton reaction □ 98. Enzyme activity is expressed as: (PGI 06)
D. Lieberman-Burchard reaction □ A. Millimoles /lit? □
90. M ichaelis-M enten hypothesis states that -
B. Milli gm/lit? □
A. Rate of enzymatic reaction is independent of
C. Mg/dl □
substrate concentration (DNB 01, MP 02, Jhar 04) □
D. Micromoles/min □
B. Rate of non enzymatic reaction is proportinal to
99. True about enzyme specificity: (PGI 01)
substrate concentrate □
A. Amount of enzyme required per second, per mole or
C. Km is the enzyme-substrate complex association
product formation □
constant □
B. Number of sites per substrate □
D. Enzyme-substrate complex formation is essential in
C. Amount of enzyme binding with various
enzymatic reaction □
substrates □
91. Which of the following enzymes are activated in
dephosphorylated state: (PGI 09) D. Number of enzyme units per milligram of enzyme
A. HMG Co A reductase □ protein □
B. Glycogen phosphorylase □ E. Amount of enzyme causing transformation of 1 pmol
C. Glycogen phosphorylase kinase □ of substrate per minute under standard conditions □
D. Citrate lyase □ 100.Enzymes Ea,Eb and Ec are under allosteric control of
E. Glycogen synthase □ S2 and P I and P2 respectively. If Ec is defective what
92. Enzyme regulated by phosphorylation: (PGI 08) would occur -
A. Glucokinase □
B. Glycogen synthetase □ A. Accumulation of SI (AIIMS 01) □
C. Pyruvate dehydrogenase □
B. Accumulation of PI □
D. Isocitrate □
C. Accumulation of S2 □
E. Citrate lyase □
93. All of the following enzymes are regulated by calcium
D. Decrease of S2 □
101.Non- functional enzymes are all except:
or calmodulin, except: (AI 06)
A. Alkaline phosphatase (AIIMS 08) □
A. Adenylate cyclase □
B. Acid phosphatase □
B. Glycogen synthase □
C. Lipoprotein lipase □
C. Guanylyl cyclase □
D. Gamma glutamyl transpeptidase □
D. Hexokinase □
102.True about isoenzym es is: (AIIMS 11)
94. "All enzymes are not proteins." This statement is
justified by: (AI 02) A. Same quaternary structure □
A. All enzymes do not follow the Michaelis Menton B. Same distribution in different organs □
hypothesis □ C. Same enzyme classification with same umbers □
B. RNAs act as ribozymes □ D. Catalyze the same reaction □
C. Antibodies take part in the catalysis of many 103.True about isoenzym es is/are: (PGI 01)
reactions □ A. Different km value □
D. Metals are involved in attachment to enzymes and B. Act on different substrate □
catalysts. □ C. Consist of multimeric complex □
95. D efinition of Ribozyme: (PGI 06) D. Same electrophoretic mobility □
A. RNA molecular that acts catalytically to change it self E. Have different physical properties □
or another RNA molecule □ 104.The predom inant isozym e of LDH in cardiac m uscle is:
B. t-RNA □ A. LD-1. (A I0 5 )O
C. m-RNA □ B. LD-2. □
D. Ribosome □ C. LD-3. □
E. Ribonucleoprotein □ D. LD-5. □
536 ■ A Complete Review of Short Subjects

105.1n which of the following conditions the level of 115.W hich of the following enzym es does not participate
creatinine kinase-1 increases? (AI 04) in oxidation-reduction reactions? (AIIMS 13)
A. Myocardial Ischemia □ A. Oxygenases □
B. Brain Ischemia □ B. Peroxidases □
C. Kidney damage □ C. Hydroperoxidases □
D. Electrical cardioversion □ D. Dehydrogenases □
106.Force not acting in an enzyme substrate com plex : 116.A11 of the follow ing enzymes are involved in oxidation
A. Electrostatic (AI 01) □ - reduction reactions, Except (AI 09)
B. Covalent □ A. Dehydrogenases □
C. Van der waals □ B. Hydrolases □
D. Hydrogen □ C. Oxygenases □
107.The serine proteases do not include (AIIMS 03) D. Peroxidases □
A. Pepsin □ 117.Detoxification of drugs is controlled by: (AP 10)
B. Trypsin □ A. Cytochrome □
C. Elastase □ B. Cytochrome P450 □
D. Chymotrypsin □ C. Cytochrome C □
108.Active sites of serine protease contain: (PGI 03) D. Cytochrome A □
118.All of the following are true regarding oxygenases
A. Histidine □
e x c e p t:
B. Lysine or threonine □
A. Incorporate 2 atoms of oxygen(AIIMS 11, AI 07)0
C. Arginine □
B. Incorporate 1 atom of oxygen □
D. Serine □
C. Required for hydroxylation of steroids □
109.1f chym otrypsin m olecule undergoes a ser-195-ala
D. Required for carboxylation of drugs. □
mutation then : (AI 08)
119.W hich of the following is not a rate lim iting enzyme:
A.Chymotrypsin will not bind the substrate □
A. HMG CoA Reductase (AIIMS 13,11,08) 0
B.Chymotrypsin will bind the substrate as well as
B. Phosphofructokinase □
cause cleavage □
C. Acetyl CoA carboxylase □
C. Chymotrypsin will bind the substrate but will not
D. Malanoate dehydrogenase □
cause cleavage □
120.1n which of the following reactions is Magnesium
D. No affect will be observed □
required:
HO.Tyrosinase is - (AIIMS 06, UP 12)
A. ATPase (A102)O
A. Oxidase □
B. Dismutase □
B. Transferase □
C. Phosphatase □
C. Lyase □ D. Aldolase □
D. Isomerase □ 121.Coenzymes a r e organic compounds :- (PGI 04)
111.Hexokinase is a: (AIIMS 13, NBE P 15) A. Lipoprotein □
A. Transferase □ B. Proteinaceous □
B. Reductase □ C. Non-protein □
C. Oxidoreductase □ D. Any of the above □
D. Oxidase □ 122.The combination of main supporting enzyme is called:
112.Fum arase / Decarboxylase is: (PGI 14, NBE P 15,13) A. Apoenzyme (PGI 03, AIIMS 01) □
A. Oxidoreductase □ B. Coenzyme □
B. Transferase □ C. Holoenzyme □
C. Oxidase □ D. Constitutive enzyme □
D. Lyases □ 123.W hich of the following acts as co-enzym es and not as
113.Enzym es m ediating transfer of one m olecule to co-factors: (PGI 06)
another are OR Enzym es that m ove a m olecular group A. Pyridoxine □
from one m olecule to another are know n as: (AI 08,05) B. Biotin □
A. Transferases □ C. Thiamine □
B. Oxidases □ D. Folic acid □
C. Lysases □ E. Co-balamin □
D. Peptidases □ 124.Coenzyme, in an enzym atic reaction usually functions
114.Addition of w ater in C-C bond is by the enzyme: to :
A. Hydroxylase (PGI 04) □ A. Activate the substrate (AIIMS 06) □
B. Dehydrogenase □ B. Increase the active sites of apoenzyme □
C. Hydrolase □ C. Enhance the specificity of aponzyme □
D. Hydratase □ D. Accept one of the cleavage products □
Biochemistry: Structures & Functions of Proteins and Enzymes ■ 537

125.Thiamine deficiency decreases cellular m etabolism 132.Selenium deficiency causes: (PGI 13,12)
because: A. Dermatitis □
A. Thiamine is a coenzyme for pyruvate dehydrogenase B. Cardiomyopathy □
and alfa-ketoglutarate dehydrogenases (AIIMS 08) □ C. Diarrhoea □
B. Activity of transketolase is inhibited □ D. Alopecia □
C. It is required for the process of transamination □ E. Gonadal atrophy □
D. It is a cofactor in oxidative reduction □ 133.Selenium is co-factor for : (NBE P 15,14,13; AI 08)
126.V itam in K is required for: (AIIMS 14, 07) A. Glutathione peroxidase (PGI 14) □
A. Carboxylation □ B. Glutathione reductase □
B. Hydroxylation □ C. Glutathione synthetase □
C. Oxidation □ D. Glutathione dehydrogenase □
D. Reduaction □ 134.Abzyme is a/an (AIIMS 08)
127.Which coenzyme is responsible for carboxylation A. Isoenzyme □
reaction? O R Treatment of M ultiple Carboxylase B. Allosteric enzyme □
Deficiency is: (NBE P 14,13; AIIMS 08, AI 10) C. Abnormal enzyme □
A. Biotin □ D. Antibody with a catalytic activity □
B. FAD □ 135.Ceruloplasmin contains- (NBE P 15)
C. NADH □ A. Zn □
D. Thiamine pyrophosphate □ B. Cu □
128.W hich of the follow ing acts as cofactor after it's C. Se □
m odification: (PGI 01) D. Fe □
A. Vit-C □ 136.Copper containing enzymes is/are All except:
B. Pantothenic acid □ (NBE P 15,14; AIIMS 14; PGI 15,14,12)
C. Biotin □ A. Superoxide dismutase □
D. Zinc □ B. Cytochrome oxidase □
E. Copper □ C. Myeloperoxidase □
129.Zinc is cofactor for: (AIIMS 09) D. Tyrosinase &c Amine oxidase □
A. Alcohol dehydrogenase □ 137.Biological role o f m etallothioneins is to sequester
B. Pyruvate Carboxylase □ harmful metal ions. These bind (AIIMS 04)
C. Pyrvate Carboxylase □ A. Cd++, Cu++ & Zn++ □
D. Alpha- KG dehydrogenase □ B. Al+++, Hg++ & NH„+ □
130.Zinc acts as co-factor in A/E: (PGI 14,12,08) C. Pt++, As+++ & PO 4- □
A. Lactate dehydrogenase □ D. Fe+++, Na+ & K+ □
B. Carbonic anhydrase □ 138.Increased copper excretion in urine is seen in all except
C. Glutathione peroxidase □ A. Primary sclerosing cholangitis (AIIMS 07) □
D. Alkaline phosphatase □ B. Wilson's disease □
E. Superoxide dismutase □ C. Primary biliary cirrhosis □
131.Carboxypeptidase contains w hich m ineral- D. Hepatocellular carcinoma □
A. Copper (NBE P 15) □
B. Zinc □
C. Iron □
D. None □
538 ■ A Complete Review of Short Subjects

ANSWERS & EXPLANATIONS:


Structure & Functions of Proteins

1. A i.e. Linear structure and order of amino acids present 2. B i.e. Secondary structure
3. D i.e. Long stretches of left handed a - helices occur in proteins 4. C i.e. Proline
5. B i.e. Aspartic acid 6. B i.e. Hydrogen bond 7. C i.e. M ultiple weak interaction
8. D i.e. Quaternary structure 9. D i.e. Quartenary structure
[Ref: Lehninger 5/e p 140-117, 92; Lippincott 5/e p 16-17; Vasudevan 6/e p 32-33; Harper 29/e p 36-40; Chattarje 8/e p 89-911

- Discription of all covalent bonds (mainly peptide and disulfide bonds) linking aminoacid residues in a linear
structure refers to primary structure of protein.
- Secondary structure of protein refers to description of steric relationship (spatial arrangement) between aminoacids
located relatively near each other in a selected segment of main polypeptide chain, without regard to the
conformation of its side chains or its relationship to other segments. Examples of secondary structures include a-helix,
P pleated sheet and f) turns etc.
- Naturally occurring L-aminoacids can (theoretically) form either right or left handed a-helices, but extended left handed
a-helices are theoretically less stable and have not been observed in proteins®.
- Proline and glycine have the least tendency (proclivity) to form a-helices ®.
- Large number of charged am inoacids like aspartate (aspartic acid)®, glutamate (glutamic acid), arginine, lysine or
histidine can also disrupt a-helix by forming ionic bonds or electrostatically repelling each other.
- Secondary structure is stabilized by hydrogen bonds & disulfide bonds; Cooperative hydrogen bonding in repeating secondary
structures have an important role in guiding the protein folding process®. H ow ever, location of bends (including f)
turns) in polypeptide chain and the direction and angle of these turns are determined by the numer & location of
specific bend producing am inoacid residues, such as Pro, Thr, Ser, and Gly. Interacting segments o f polypeptide chains
are held in their characteristic tertiary (3-dimensional) positions by several kinds o f w eak interactions® (mainly
hydrophobic bonds, vanderwall forces, ionic interactions) and sometimes by covalent bonds (such as disulfide cross
links) between the segments.
- Some proteins contain >2 polypeptide chains or subunits which may be identical (homo) or different (hetero).
Quartemary structure of protein refers to three dimensional arrangement of these protein subunits. Therefore, hom o-or
hetero-dim ers form ed between 2 polypeptide chains is an example o f quaternary structure®. Similarly an immunoglobin
molecule represents hetero-tetrameric quartemary structure between 4 polypeptide (2 heavy + 2 light) chains.

★ Secondary and tertiary structure of protein maintain the 3- dimensional shape of protein and the main bonds involved
are weak non covalent bonds. (Thus 3-D shape is maintained by multiple weak interaction)^.
Biochemistry: Structures & Functions ot Proteins and Enzymes ■ 539

10. All i.e. (It makes protein resistant, Bond is formed b/w carboxyl terminus of one protein and the amino group of a
lysine residue on another, Involves in post-transcriptional m odification of protein, Enzym e acts as catalyst for the bond
form ation)

Isopeptide bond is an amide bond formed betw een the side (not m ain) chain carboxy terminal o f one protein (with
either glutamate or aspartate) and am ino group of lysine residue on side chain of another target protein. Bond formation

is either spontaneous or enzym e (transglutm inase) catalyzed, involves post translational m odification and makes
protein resistant.

11. A i.e. Disulfide bond [Ref: Lippincott's 3/e P-19; Harper's 27/e P-33-35]

A disulfide linkage contribute to stability of three dimensional shape of protein molecule.


Many disulfide bonds are found in proteins that are secreted by cell, to stabilize the structure and prevent them from becoming
denaturated in the extracellular environment.
Since sn ake's venom is a protein secretion it is m o st p ro b a b ly sta b iliz ed by d isu lfid e bond®.

12. A i.e. Less num ber of disulphide bonds 13. D i.e. All
[Ref: Vasudevan 6 /ep 584; Harper 29je p 626-27, 43-44; Lehninger 5/e p 71,123]

Keratin in skin is softer than keratin in nail, horn & hooves because skin keratin has less number o f disulfide bonds®
between Cys residues of adjacent polypeptide chains. Keratin contains H istidine, Arginine & Lysine (M n: HALQ in
1:12:4).

14. B i.e. M olluscus shell [Ref: Internet]

Biom ineralization (ie mineralization caused by cell mediated phenomenon in living organisms to harden or stiffen
existing tissues) is seen in m olluscus shell, sea shells (i.e. calcium carbonate in invertebrates), bone & teeth in
m am m als and birds (i.e. CaCOj & CaPC >4in vertebrates), silicates in algae & diatom s, and copper, iron, gold in
bacteria.
H oof and nails are thickened keratin.

15. B i.e. Convergence [Ref: Convergent evolution among immunoglobins F : M Frick;]

Convergence is an evolutionary process by which distinctly unrelated organisms (protein) aquire morphologically similar
traits, (function), where as divergence produces different traits (functions) in related same ancestor organisms (proteins).
Convergence may be
r T
Sequence convergence Structural convergence M echanical convergence Functional convergence
I I I
>1 aminoacids or Proteins of different Proteins with different Proteins with different amino
aminoacid sequence of am inoacid sequences am inoacid sequences & acid sequences, structure and
two different proteins assum e sim ilar m otif strucure come to act by mechanisms come to serve
come to resemble structures^ sim ilar mechanism^ same function.

16. B i.e. Ligand binding residues [Ref: Lehninger 4"’/160; Harper's 27/e P-369]

Function in biochem istry revolves around a reversible protein ligand interaction.


A molecule bound reversibly by a protein is called a 'Ligand'. A ligand binds at a site on the protein called the 'binding
site', which is complementary to ligand in size, charge and hydrophobic or hydrophillic character. As long as the ligand
binding residues are preserved to allow interaction between the ligand and the binding sites the function will essentially
be preserved.
• Amino acid sequence alterations are compatible will functionally normal haemoglobin: (Acceptable missense mutations)
eg. Haemoglobin Hikari has been found in at least two families of Japanese people. This haemoglobin has asparginine
substituted for lysine at position 61 in IL (i chain, (change in AA sequence). This replacement of specific lysine with
arginine apparently does not alter the normal function of the (3 chain in these individuals.
• As long as the domains/Ligand receptors are preserved, a variation in structure is still compatible with normal function.
• Environmental changes do not explain why only a few out of 300 variants of human globin gene are incompatible will
function.
540 ■ A Complete Review of Short Subjects

17. C i.e. Carbohydrate content is same but proteins are different D i.e. Carbohydrate chain is often linear
18. B i.e. Glucosam inoglycan 19. B i.e. They hold less am ount of w ater 20. C i.e. Proteoglycan
21. C i.e. Serine 22. D i.e. Final trimming 23. C i.e. Golgi
24. D i.e. Presence of G1C - NAc - Gal - NANA in com plex glycoprotein 25. D i.e. RER > A i.e. Golgi apparatus
[Ref: Lippincott 5/e p 444; 166-70; Vasudevan 6/e p 124]
/ --------------------------------------------------------------------------------------------------------------------------------------------------------------------------
O-glycosylation involves addition of a specific oligosaccharide unit (sugar chain) to the terminal (OH) group of se, *
or threonine aminoacids (commonly) and tyrosine, hydroxylysine or hydroxyproline (rarely). Glycosyl transferase
responsible for O-glycosylation is found in golgli complex®.
Dolichol synthesis, glycosyl transferase and protein oligosaccharide transferase steps of N-glycosylation occurs in
endoplasmic reticulum (ER). W hereas final trimming & completion occurs in golgi complex
N linked complex oligosaccharide (glycoproteins) contain G lc NAC, NANA & Fuc, whereas high mannose contain
pramiraly Man (mannose).

26. A i.e. Protein linked with glycosidic bond; D i.e. Participate in cell surface recognition
27. B i.e. Biological function rem ains.. 28. D i.e. Biological activity is retained 29. C, B ie. D enaturation, Precipitation
30. A i.e. Bence Jones proteins [Ref: Harper's 27/e P-45-46; Leninger 2/e P- 141,160, 63, 66- 61; Lippincott's 4/e p. 20,171;
Ananthanarayan 7/e p. 90; Harrison 16/e p. 2511

- Bence Jones protein in urine can be identified by its characteristic property of coagulation (precipitation or clumping V - i
due to denaturation) when heated to 50 - 60° C. But the clumps begin to desappear (d/t renaturation) when the
sample is further heated to boiling point (70 - 100°C)Q Again on cooling clumps reappear (Heat test).
Cryoglobulins (immunoglobulin proteins) precipitate when cooled and dissolve on heating to higher temperatures
Pyroglobulins (associated with multiple myeloma, macroglobulinemia & other neoplastic disorders) resembles Bence
Jones proteins in that both precipitate on heating to about 50°C but do not dissolve when cooled.

31. B i.e. Concentration; D i.e. Path Length [Ref: Lehninger 3/e P - 121; 2/e 314, 207]

Measurement of light absorption is an important tool for Where,


analysis of nucleotides and nucleic acids. The fraction of Io = Intensity of incident light
the incident light absorbed by a solution at a given wave I = Intensity of transmitted light
length is related to the thickness o f absorbing layer (i.e. e = Molar absorption coefficient (in liters /mole- centimeter)
path length) and the concentration o f absorbing spaces ®. C = Concentration of absorbing species in moles /liters
The two relations hips are combined into the Lambert /= thickness of light absorbing sample in centimeters
Beer law given in integrated form as- log Io/I = Absorbance, and low assumes that the incident light
is parallel and monochromatic (of single wave length) and
Log — = £Cl solvent and solutes are randomly oriented.
I
32. B i.e. Tertiary structure [Ref: Homology of pseudomonas cytochrome C-551 with eukaryotic Ctyochrome C by northwest university
school of medicine]
• Tertiary structure of cytochrome C in bacteria and humans are highly conserved. Pseudomonas (bacterial)cytochrome C
differs from eukaryotic (human) cytochrome in
- total no. of amino acids
- amino acid sequence.
• Comparison of primary protein structure with the tertiary structure revealed that functionally crucial a.a residues fold
closely together in tertiary structure while they were distant to each other in primary structure.
33. B i.e. Guanidine [Ref: Lippincott's 3/e P-21, 23, Lehninger 3/e P 192]

Guanidine is a denaturating substance®, results in loss of secondary & tertiary structure of proteins

Denaturation of Protein
Results in the unfolding and disorganization of the protein's secondary and tertiary structures (3-D structure), sufficient
to cause loss of function. The primary structure is intact®. Some denaturant are
Heat® Urea® - Guanidine hydrochloride®
Extremes of pH® Detergents® - Organic solvents eg. alcohol & acetone
V
Loss of secondary and tertiary structure® leading to Loss of function
Biochemistry: Structures & Functions of Proteins and Enzymes ■ 541

34. B i.e. Fibrous proteins 35. A i.e. Fibrous IRef: Harper's 27 /e P-38; 26/e P-30; Chatterjea 6/e P-78; Lehninger 5/e p. 1251

- Proteins responsible for m aintaining the f t Fibrous protein Globular protein


structural strength of cells (i.e. cytoskeleton) When the axial ratio of length: When the axial ratio of
and tissues include collagens, keratin and width of a protein molecule is length: width of a protein is
myosin etc. These proteins are classified as more than 10 , it is called fibrous less than 10 , it is called
fibrous proteins. 'Structural proteins' are protein. Polypeptide chains are globular protein.
usually fibrous proteinQ, and include arranged in long strands or Polypeptide chain is folded
proteins of skin, cartilage, nail (such as sheets. into spherical or globular
collagen and proteoglycan). Fibrous protein Usually consists largely of single shape
centrifuge more rapidlyQ because of their type of secondary structure Often contain several types
rod like shape. Structures that provide support, of secondary structures
- Collagen is the most abundant of the fibrous shape and external protection to Most enzymes & regulatory
proteins. Other prominent fibrous proteins vertebrates are fibrous protein. proteins are globular protein
include keratin and myosin. These proteins Thus structural proteins are Examples: Myoglobin,
represent a primary source of structural usually fibrous proteinsQ. Hemoglobin (i.e. storage
strength for cells (the cytoskeleton) and Examples: Collagen, a- keratin proteins or transport
tissues. of hair etc, (i.e. structural proteins)
protein)^

36. A i.e. Triple helix 37. C i.e. Formation of triple helix [Ref: Harper 28,hjed pg-528-30; Lippincott 4,h/ed pg- 43-49]

- Collagen is a triple helical structure^ in which 3 a chains are wound into right handed superhelix with occurance of
glycine residue at every 3rd position^ of triple helix of a-chain. V itam in C (ascorbic acid) is necessary (as cofactor) for
hydroxylation of proline & lysine w hile procollagen is converted into collagen during post translational
modification^.
- Removal of leader or signal sequence (to form pro a - chains), hydroxylation (not hydrolysis) of proline & lysine
residues, glycosylation of some hydroxylysine not proline residues with glucose or glucosyl galctose, interchain and
intrachain disulfide bond formation at carboxy terminal propeptide extension (while amino terminal extension
propeptide forms only intTachain disulfide bonds) and triple helix formation^ are intracellular events of collagen
formation.
- Cleavage of amino & carboxy terminal propeptide extensions (by N & C procollagen peptidases) releasing triple
helical tropocollagen; spontaneous self assem bly of tropocollagen molecules to form collagen fibrils; assembly of
collagen fibrils in orderd, quarter overlapping, parallel, staggered alignment; oxidative deamination of E
aminogroups of lysyl and hydroxyl lysyl residues to aldehydes; and formation of intra and inter chain covalent cross
links via Schiff bases and aldehyde condensation products 1/t mature collagen formation are extracellular events of
collagen formation.

38. C i.e. Calcium

f t
40 nm gap between the typrocollagen molecues serve as nucleation site for deposition of hydroxyapatite
[Ca5(/>0 4),(0/f)] with some phosphate] crystals in boneQ.

39. B i.e. Simple globular protein [Ref: Chatterjea 7/e p. 81-82; Lehninger 5/e p. 85]

Plasma proteins like album in and globulin are sim ple globular proteinQ.

40. B i.e. Collagen: IRef: Harper's 29/e p 48, 632; Chattarjae 8/e p 105-06]

Albumin, ceruloplasmin, transferrinn and hem oglobin are transport proteins whereas collagen is a structural protein.

41. B, D, E > A i.e. Ovalbumin, Ferritin, G lutelin > M yoglobin [Ref: Chatterjaea 8/e p 617, 83-85; Satyanarayan 3/el63; Harper 29/e 635]
542 ■ A Complete Review of Short Subjects

Ricin is a phytotoxin. W hereas, ovalbumin, ferritin and glutelin are storage proteins.
Hepcidin is the ch ief regulator protein known to p lay a central role in system ic iron hem ostasis <2. It bind to cellular
iron exporter ferroportin and triggers its internalization and degradation. When plasma iron levels are high, hepatic
synthesis of hepcidin increases, thus reducing iron absorption in intestine (mucosal block); preventing recycling of iron
from macrophages and reducing placental iron transfer. This leads to reduction in circulating iron levels
(hypoferremia). The opposite occurs when plasma iron levels are low.

42. A i.e. Phosphates IRef: Chatterjea 6/e P- 657]

A protein estim ation test is confused with phosphates 0- as presence of phosphate give false positive test

43. B i.e. Ninhydrin [Ref: Lippincott's 3le P - 14-15]

Amino acids can be detected and quantified by reaction with ninhydrin after seperation of amino acid from mixtures. The test
requires intact peptide bonds Q.

44. D i.e. Brom ocresol green [Ref: Teitz 6/e p 687, 80-179]
-
Bromocresol purple (BCP) and bromcresol green (BCG) dye binding assay (ie A lb a c p & AH jb c g ) is a m e th o d for
estimation of total serum albumin (not total protein).

45. D i.e. Enzyme assay ]Ref: Reliability of plasma creatinine method in infants & children marie fose clermont - Clinical pediatrics,
vol. 25, No 11, 569- 572 (1986); Recommendations for improving serum creatinine methods- Gary L. Myers, Clinical chemistry 52 : 5
- 1 8 - 2006; Measurement of serum creatinine - Micheal whiting].

Gas chromatography - isotope dilution mass spectrometry (GC- IDMS) is considered the method of choice for
establishing the true concentration of creatinine in serum because of its excellent specificity > HPLC > Enzyme assay >
A lkaline picrate m ethods Q.

46. B i.e. IV Contrast agent; A i.e. Chlorpropamide [Ref: Schnier: Manual ofnephrolgy 6/e p - 116-17]

Protein - dipstick test


I I
False positive False negative
Concentrated urine - Highly alkaline urine (> 7ph) •Non albumin proteins
Gross hematuria - Iodinated IV contrast agentQ ■Albumin in small amount
Presence of mucus, semen - Contaminated with chlorhexidine,
or leukocytes benzoalkonium, penicilline,
________________________________ aspirins, oral hypoglycemic agents

Methods of Protein Separation (Purification)

47. B, C i.e. Native gel electrophoresis; 2D gel electrophoresis


48. B, C, D, E i.e. Chrom atography, Centrifugation, Ultrafiltration, Electrophoresis 49. A, D i.e. M ass; M olecular wt
50. A i.e. SDS- PAGE 51. B i.e. SDS- Polyacrylam ide gel electrophoresis 52. D i.e. Densitometry
[Ref: Chatterjea 6/e P-749- 50, NMS 4/e P 19- 23; Harper's 27/e p. 21-24]

SD S polyacrylamide gel electrophoresis is method of protein seperation & purification on the basis o f molecular sizeQ.
Whereas electrophoretic mobility of protein in native (non-denaturating) gel electrophoresis and 2-D gel electrophoresis
depends on both charge (PI) and its mass (hydrodynamic size). Densitometry is a method to quantitate protein in
tissue Q.
Biochemistry: Structures & Functions of Proteins and Enzymes 543

Method of Separation & Purification of Protein


(based on)

Technique Property used • Ammonium - Reversed phase


Denaturing gel/ Molecular sulfate chromatography
SDS-PAGE weight (mass) precipitation - Hydrophobic
(polyarcylamide High performance (bulk method) interaction - Affinity
Charge, size,
liquid i.e. chromatography (absorption)
gel) affinity
electrophoresis^ chromatography - Salting in chromatogra
(HPLC) - Salting out phy
Gel filtration Molecular
• Partition - Precipitation
(size exclusion) weight (size) Ion exchange Charge
chromatography by
chromato- chromatography
graphyQ antibodies
Chromato focusing Pi
Dialysis/Ultra Size chromatograpgy
★ Ammonium sulphate, precipitation,
filtration
centrifugation, and filtration are bulk methods
Ultracentrifugat Mass/density
ion Isoelectric focusing P 1
★ pi is pH at which the net (IEF)
charge of molecule is zero. electrophoresis
And electrophoresis in Native gel Mass/Charge
general is for separation of electrophoresis
charged molecules.
2D gels Molecular
electrophoresis weight & pi

Ammonium Sulfate Fractionation

A i.e. Salting out [Ref: Lehninger 5,h/ed pg-85-92; John Baynes Mechanical Biochemistry 3rd/ed p g - 14-15; Harper 28lh/ed pg-21 ]

Ammonium Sulfate /(NPLih SO 4 = (Salt) Fractionation

Early purification (fractionation) utilize differences in protein solubility which depends on pH, temperature, salt
concentration etc.

Salts of divalent ion such as ammonium sulfate / (N H ^ SO4 and magnesium chloride (MgCb) are far more effective than
salts of monovalent ions eg NaCl, KC 1, and NH4CI. Ammonium sulfate is particularly effective and is often used to sa lt
out protein^.

In low - The solubility of proteins is lowered at high salt concentration, an effect called salting out
concentrations, Increasing the ionic strength (or salt concentration) gradually decrease the solubility of
salt increase the proteins. So addition of certain salts eg ammonium sulfate can selectively precipitate some
solubility of proteins, while others remain in solution
many proteins, - At 33-40% saturated (NhLi)2 SO4, human immunoglobins are precipitated (salted out) while
a phenomenon albumin remains soluble. Most proteins will precipitate from an 80% saturated (NFLib SO4.
known as Saturated ammonium sulfate is about 4.1 mmol/L
salting in
544 ■ A Complete Review of Short Subjects

Chromatography

54. C i.e. Gel filtration Chromatography 55. C i.e. Gel diffusion Chomatography 56. A i.e. Gel Chromotography
57. A i.e. D iffusion 58. B i.e. Affinity chromatography
[R ef: C hatterjea: 7/e p. 769-72; H arper's 28/e p. 22-23; V asudevan 6/e p. 601-41

- G el filtration chromatography separates protein according to their molecular sized (molecular weight).Q ☆
- Proteins whose charges are same, can be separated by Gel diffusion chromatography.^
- A ffinity Chromatography separates proteins by their binding specificities. The proteins retained on the column are
those that bind specifically to a ligandQ (substance) that has an inherent ability to bind with the target molecule to be
purified.

59. B i.e. Ion-Exchange chromatography 60. B i.e. Aspartic acid


(R ef:L eh n in g er 5 " je p. 8 5 -8 8 ,1 7 3 , 3 6 4 -6 5 ; Joh n B ay n es M ec h a n ica l B ioch em istry 3"‘/e p. 16; H a rp er 2 8 " je p .2 2 ; V asu devan 6 /e p. 601-41

In cation exchange chromatography solid stationary matrix has negatively charged groups (i.e. -v e charged carboxy
methyl cellulose, O - CH2 - C O O is used as cation exchanger). So in the mobile phase, proteins with a net positive
charge (i.e. cations) move slowly than those with net negative charge (anion)Q. Hence aspartate (aspartic acid) being a
negatively charged amino acid will move fastest in carboxymethyl cellulose or methanol chloroform (cation exchanger)
ion exchange chromatography.

Electrophoresis

61. C & D i.e. Depends on size not on shape & Com m onest method of purifying protein
62. B i.e. Gel electrophoresis 63. A i.e. Gel centrifugation 64. D i.e. Gel chrom atography
I Ref: L ehn in ger 5 :h/ed pg- 958-56; T eitz 6 lh/ed pg- 278; M icklos: D N A scien ce 2/ed p g-438-39; P ierce: G enetics 2/e p-515; John Baynes
M echan ical B iochem istry 3 ni/ed pg- 467-66]

The fragments of DNA produced by endonuclease can be isolated by electrophoresis on agrose or polyacrlamide gelsQ.
Relaxed DNA is separated from supercoiled & linear DNA by gel (agarose /polyacrylamide) electrophoresis^

65. B i.e. Sodium Dodecyl Sulphate PAGE 66. C i.e. The protein is a dimmer of two 20kD and 30kD proteins
[R ef: L ehn in ger 5 ,h/ed pg- 88-91; H arper 2 8 l,l/ed pg-23-24; John B ayn es M echan ical B iochem istry 3 rd/ed pg-16-17]
-----------------------------------------------------------------------------------------------------------------------------------------
Most widely used method for determination of purity and molecular weight of proteins is SD S- PAGE i.e.
polyacrylamide gel electrophoresis in the presence of anionic detergent sodium dodecyl sulfateQ

SD S PAGE gel electrophoresis separates proteins on the basis of their relative m olecu lar w eight alon e. Since the results of
SDS PAGE electrophoresis show two bands only, the protein that un derw en t SD S -P A G E is com posed o f p olypeptides w ith two
differen t m olecu lar w eights (O ne o f 20 K D an d the other o f 30kD ). If the protein in question was a m onom er its m olecular
weight should have been 50kD. Since the protein has a m olecular weight of lOOkD, the protein is likely to be a dim mer of
two 20 kD proteins (Which form one band) and two 30 kD proteins (W hich form the other band).

67. B i.e. Charge [R ef Cahill, M athew . H an d book o f D iagn ostic Tests. Sprin ghou se, 1995. ;Jacobs, D avid S., et al. L aboratory Test
H an dbook. 4th ed.; P agana, K athleen D eska. M osby's M an u al o f D iagn ostic an d L aboratory Tests. St. L o u isl9 9 8 ]

Hemoglobin electrophoresis is based on charge0

Hemoglobin electrophoresis is a test that measures the different types of hem oglobin in the blood. Hem oglobin (Hgb) is
com prised of many different types, the most com mon being Ai, A 2, F, S, and C. Hgb Ai is found in normal red blood cell,
Hgb A? is a minor com ponent of normal hem oglobin, Hgb F in the fetus, Hgb S in sickle cell disease, Hgb C in hemolytic
anemia. Each o f the m ajor hem oglobin types h as an electrical charge o f a d ifferen t degree, so the m o st useful m eth od fo r
separatin g and m easuring norm al and abn orm al hem oglobin s is electrophoresis.
This process involves su bjectin g hem oglobin com pon en ts fro m dissolved red blood cells to an electric field . The com ponents then
m ove aw ay fro m each other at differen t rates, and when separated form a series o f distin ctly p igm en ted bands. The bands are then
com pared w ith those o f a norm al sam ple. Each band can be further assessed as a p ercen tag e o f the total hem oglobin , thus in dicatin g
the severity o f an y abnorm ality.
Biochemistry: Structures & Functions of Proteins and Enzymes ■ 545

Detection of Protein Structure

A, B, E i.e. X-RAY Crystallography, NMR Spectroscopy, Electrophoresis [Ref: Vasudeven 6/e 34; Harper 28/e 23-27-37-39;
A i.e. High performance liquid chromatography Lehninger 5/e 75-90; Lippincott 5/e 21; John Baynes 3/e p 15-17, 55
B i.e. Chromosome walking 71. A i.e. X-ray diffraction /Crystallography

High performance (pressure) liquid chromatography (HPLC) is a modem refinement of column chromatography and

is used for protein separation, purificationQ and identification. It can not determine protein structures.
Electrophoresis separates (& purifies) proteins based on migration of charged proteins in an electric field. It can not
determine protein structures.
Higher level (2°, 3° & 4s) o f protein structure is studied by X-ray crystallography, NMR sepectroscopy, infrared and
ultraviolet spectroscopy^.

A i.e. Arginine; C i.e. 73. B i.e. Trypsin cleaves arginine next to 74. A,D,E i.e. Phenylalanine; Tryptophan;
Lysine praline Tyrosine
[Ref: Harrison 17"' J1764; Harrison's 18"l/2312Lehninger 5"‘led pg-95-96; Vasudevan 6/e p. 33; Harper 28/e p. 462; John Baynes
Mechanical Biochemistry 3rd/ed p g - 18-19]

Reagent & Cleavage Point

Trypsin cleaves peptide bonds on C- terminal side of Reagent (Biological Cleavage point
arginine and lysine residues, provided the next source)
residue is not prolineS. Chymotrypsin cleaves
Trypsin (bovine - Lysine, Arginine (C) provided
peptide bond on carbonyl (C) terminal of next residue is not prolineS
pancrease)
phenylalanine, tryptophan and tyrosineS
- Elastase esters of small neutral
Lysyl endopeptidase cleaves c- terminal side of aliphatic aminoacids,
lysineQ and cyanogens bromide cleaves C- terminal chymotrypsin esters of aromatic
side of methionine. aminoacids.
Disulfide bonds interfere with sequencing procedure. Lysyl endoproteinase C Lysine (C) carboxy terminal
So before cleavage proteins with cysteine and cystine (bacterium Lysobacter
residue are either oxidized by performic acids to enzymogenes)
cysteic acid or reduced by dithiothreitol (or f) Submaxillarus protease Arginine (C) terminal
mercaptoethanol)Q and idoacetate (to (mouse submaxillary
carboxymethylated cysteine) to irreversibly break gland)
disulfide bonds. This avoids spontaneous formation Cyanogen bromide Methionine (C)
of inter or intra molecular disulfides during analysis. chemical
For determination of structure of protein: Purified V8 protease Aspartate, Glutamate (C)
protein (by process of salting out —^dialysis —» ion (staphylococcus aureus)
exchang & gel filtration chromatography —>SDS Asp - N- protease Aspartate, Glutamate (N
PAGE) is cleaved by protelytic degradation. The (Pseudomonas fragi) terminal)
cleaved peptides fragments are again purified by Chymotrypsin (bovine Phenylalanine, Tryptophan,
reverse phase HPLC and then sequenced on pancreas) Tyrosine (C) or Leucine
automated protein sequencer, using Edman's Pepsin (Porcine stomach) Leucine, phenyalanine (Phe),
degradation technique, Now M ass spectrometry, Tryptophan (Trp), Tyrosine (Tyr)
MALDI - TOF, ESI -M S and indirect hybrid - N amino side
techniques are also used to determine 1 ° protein
structure.

B i.e. Absorption mass spectroscopy 76. D i.e. X-ray diffraction [R ef: H arper's 2 8 /e P -2 6 -2 7 ; C am p bells U rology 8 /e P- 32741

Polarizing microscopy, X-ray diffraction and infrared spectroscopy are techniques for analysing renal and gall bladder
stones.
546 ■ A Complete Review of Short Subjects

77. B i .e . A m in o a c id s [Ref: Chatterjee 5/e p. 88]

Test/Reaction Used to Identify Aminoacids (group


or specific)
U s e d to d e te r m in e A m in o a c id s e q u e n c e i n
p o ly p e p t i d e c h a in by tagging (N) amino
terminal with S a n g e r 's r e a g e n t (1 f lu r o -2 4 -
Aldehyde test & Hopkins- Tryptophan (Indole ring)
D i n it r o b e n z e n e ) or E d m a n 's r e a g e n t
Cole-Reaction
( P h e n y - is o t h io - c y a n a te ) . Tagged aminoacid
Sulphur/Nitroprusside Test Cysteine (Sulphydryl group)
is hydrolysed and identified.
Millan (Nasse) Test Tyrosine (Phenol group)
Pauli's Diazo Test Histidine (Imidozole gp) or Tyrosine
Sakagauch's Test Arginine (Guanidinium gp)

Enzymes

78. B i .e . N o n - c o m p e t itiv e i n h i b i t o r

Any substance that can decrease the velocity o f an enzym e catalyzed reaction is k / a i n h i b i t o r ( I). The inhibition can be
r e v e r s i b l e or irreversible. R e v e r s i b le i n h ib i t o r s bind to enzymes through non covalent bonds and can be dissociated
(removed) from its binding site (by dilution) without loosing its enzyme activity. Whereas, i r r e v e r s i b l e i n h ib i t o r s bind to
enzyme through covalent bonds and can be removed from its site of binding only at the loss of enzymatic activity (i.e.
i r r e v e r s i b l e i n h ib i t o r s p o is o n e n z y m e s ) . Two most common types of reversible inhibiton are competitive & non
competitive. However the non- competitive can also be irreversible.

C o m p e t it iv e I n h i b i t i o n N o n - C o m p e t itiv e - I n h i b i t i o n
D ir e c tio n ReversibleQ Reversible or irreversibleQ
S tru ctu re o f Resemble substrate (structure) and Bear n o (or little) structure r e s e m b l a n c e to substrate
i n h i b i t o r (I) so are termed s u b s t r a t e a n a lo g s .
B i n d i n g s it e Inhibitor binds to s u b s t r a t e b i n d i n g Inhibitors bind enzymes at sites d is t i n c t f r o m s u b s t r a t e b i n d i n g
p o r t io n o f a c t iv e s i t e (i.e. both s i t e (i.e. substrate (S ) and inhibitor (=1) bind enzyme (= E ) at
inhibitor & substrate bind to s a m e d if f e r e n t s it e s ) and so there is no competition between inhibitor &
s i t e and so inhibitor competes with substrate for enzyme.
the substrate (S) for that site) thereby
blocking access by the substrate
E I S c o m p le x - Inhibitor can not bind to enzyme - - Inhibitor can bind to ES complex
substrate (ES) complex - Both El and E I S c o m p le x e s can form; however EIS generates
Either El or ES complex is formed product at negligible rate.
and EIS is not possible____________
R a is in g O v e r c o m e s i n h i b i t o n (i.e. at C a n n o t b e o v erco m e (i.e. inhibitor d e c r e a s e s V m a x )
s u b s tr a te sufficiently high substrate
c o n c e n t r a t io n concentration, V m a x w i l l b e s a m e as
(i.e . d ilu t io n ) observed in absence of inhibitor)
S u b s tr a te D e c r e a s e d (i.e. once inhibitor D oes n ot change(i.e. non competitive inhibitors donot interfere
a f f i n i t y to binds to enzyme, the enzyme with binding of substrate to enzyme)
enzym e cannot bind to substrate) Same amount of substrate is needed to achieve Vi Vmax (i.e. K m is
So more substrate is needed to u n a ffe c te d )
achieve 16 Vmax (i.e. K m t)
Biochemistry: Structures & Functions of Proteins and Enzymes ■ 547

M ichaelis - Y - V- 'S I v Vm„[S]


Menten Km' +,I)+S Km ‘ ^ jl K m l + IS]
Ki
equation
Lineweaver -
-L = J ^ - [ 1+^ ] x — + —!— M t Non-competitive
Burk plot V Vm Ki [S] v m„
V m ax >, f Com petitive
d ecreased /
/ No inhibitor

F o r Non-Competitive - Inhibition V m ax
sam e

1 Km r (i)i 1 1 r (i)i ------------------- ©


— = ------- 1 + — x — + -------+ 1+ — 0 r
V V^L KiJ [S] VmM L Ki J Km Km
H] sam e increase

Line-Weaver Burk (Do uble reciprocal) plot


Eadic - No change in Y intercept (Vmax) but No change in slope (-Km) but Y intercept is lowered and x intercept
Hofstee plot possesses steeper slope and smaller declines in value
x intercept.

Inhibitor (I) V J

Enzyme (E)

79. A i.e. Vmax unchanged C i.e. Non competitive inhibitor 81. C i.e. Competitive
82. A, C i.e. Lower Vmax, Not affect Km A, E i.e. T Km, V max remain same
84. C i.e. Km value normal; V max decreased A i.e. Competitive 86. A i.e. Substrate concentration at Vmax72
87. A i.e. Km [Ref: Chattarjea 7/e p 121-24; Harper 28/e 70-74-77-79; Lippincott 5/e 62-64; KDT 6 /e p 40]

In non competitive inhibition, there is no competition betw een substrate & inhibitor because inhibitor is structurally
different and it binds to enzyme at a site other than substrate binding (catalytic) siteQ.

Enzym atic Inhibition

Competitive Non competitive ★ Km: The Michaelis constant Km is the


I I substrate concentration at which the
- Km is increased0 - Km is unchanged® reaction velocity is equal to Vt Vma®. So
- Vmax is unchanged® - Vmox is decreased® it measures specificity o f substrate for
enzymeQ.

88. B , C i.e . S u b s t r a t e c o n c e n t r a t io n a t w h i c h r e a c t i o n r a t e i s h a l f t h e m a x im u m ; M i c h a e l i s c o n s t a n t
89. C i .e . M i c h a l e s - M e n t o n r e a c t i o n 90. D i .e . E n z y m e s s u b s t r a t e c o m p le x f o r m a t io n i s e s s e n t i a l i n e n z y m a t ic r e a c tio n
91. A , E i.e . H M G C o A ; G l y c o g e n s y n t h a s e 92. B ,C ,E ie . G l y c o g e n s y n t h e t a s e , P y r u v a t e d e h y d r o g e n a s e & C it r a t e ly a s e
Ref: Lippincott 4lh/e p.63,110; Lehninger 5lh/e p. 220-27, 571-77; Harper 28lh/e p.81; Chattarjea & Shinde 7/e p. 118-19]

Phosphorylation is the most important type of regulatory - Some enzymes are regulated by reversible
modification covalent modification
- It involves addition or removal of phosphate group from - Common modifying group include:
serine, threonine, tyrosine or histadine residue of enzyme phosphoryl, acetyl, adenylyl, uridyl, methyl,
- ATP is used as PO 4 donor amide, carboxyl, myristoyl, palmitoyl, prenyl,
- P r o t e i n k i n a s e e n z y m e c a t a l y z e s p h o s p h o r y l a t i o n (i .e a d d i t i o n
hydroxyl, sulfate and adenosine di phosphate
o f P O 4) a n d p h o s p h o p r o t e in p h o s p h a t a s e c a t a l y z e s
ribosyl groups
d e p h o s p h o r y l a t i o n (i.e . r e m o v a l o f P O 4 g r o u p ) .
548 ■ A Complete Review of Short Subjects

Enzyme Activity state - Even entire proteins eg ubiquitin & sumo are
P = Phosphorylated Low High used as specialized modifying groups
D = Dephosphorylated (Inactive) (Active) Ubiquitin is added to protein as a tag that pre
Acetyl - CoA carboxylase P D destines them for proteolytic degradation Q.
Glycogen synthase P D Sumo is attached to eukaryotic nuclear
Pyruvate dehydrogenase P D proteins with role in regulation of
HMG - CoA - reductase P D transcription, chromatin structure & DNA
Glycogen phosphorylase D P repair.
Citrate lyase D P - ADP ribosylation (derived from NAD) occurs
Phosphorylase b kinase D P in bacterial enzyme dinitrogenase reductase (1/1
HMG-CoA reductase D P regulation of biological N 2 fixation),
kinase diphtheria toxin & cholera toxin

93. D i.e . H e x o k in a s e [Ref: Harper's 28/e p. 450; Vasudevan 6 /e p. 412, 587; Lehninger 5/e p. 436-37]

Adenyl cyclase, guanylate cyclase and glycogen synthase enzymes (but not hexokinase)Q are activated by calcium and
calmodulin.

94. B i.e . R N A s a c t a s r ib o z y m e s [Ref: Harper's 27/e P-315, 318, 364]

Ribozymes are RNA molecules with catalytic activities^, which acts upon phosphodiester bonds in RNA & cause its
hydrolysis [intron splicing event]. RNAs are not protein, but meet all the classic criteria of enzymes therefore all
enzymes are not protein.

95. A i.e., RNA molecular that acts catalytically to change itself or another RNA molecule 96. D i.e. Ribozyme
97. A i.e. Splicing of heterogenous RNA to form mRNA; D i.e. Recognition of codons in intron
[Ref: Harper's 27/e P-315, 318, 364; 26/e P-308, 356; Lippincott’s 3/e P-436]
Ribozymes are RNA molecules with intrinsic catalytic activity. These are generally involved in transesterification
reactions, & most are concerned with RNA metabolism (splicing and endoribonuclease). Example: Sn RNAs, rRNA
Ribozymes
I------------------------------------------ 1---------------------------------------------- 1
_______________________SnRN A_____________________ _____________________ rRNA________________
- Small nuclear RNAs (Sn RNA) in assosiation with small - rRNA is involved in elongation of polypeptide
nuclear ribonucleoprotein particles (Sn RNPs) facilitate chain during eukaryotic protein synthesis
splicingQ and thus acts as enzyme. - The formation of peptide bonds is catalyzed by
- Sn RNA act upon phosphodiester bond of RNA & causes peptidyl transferase, an activity intrinsic to 23S
its hydrolysis necessary for conversion o f pre mRNA (hn rRNA found in 508 ribosomal unit.
RNA) to mature RNAQ. [Intron splicing event]__________

98. D i.e. Micromoles /M inute [Ref: Lippincott's 3/e P- 54, NMS. Biochemistry 4/e P-59-60; Vasudevan 4/e p- 44]
99. E i.e . A m o u n t o f e n z y m e c a u s in g t r a n s f o r m a tio n o f 1 p m o l o f s u b s tr a te p e r m in u t e u n d e r s ta n d a r d c o n d it i o n

The specific activity of enzyme is usually expressed as p. mol of substrate transformed to product per minute per
milligram of enzymes under optimal condition of measurement^.
1 in t e r n a t i o n a l u n it (IU ) = 1 micromole / minute Q
Modern unit of enzyme activity, 1 katals (kat) = 1 m ole /second (i.e. conservation of 1 moles of substrate per second)
Enzymes are highly specific, interacting with one or few specific substrates & catalyzing only one type of chemical reaction
Rate of metabolic processes may thus be regulated by changes in the catalytic efficiency of specific enzymes.

100. C i.e . A c c u m u la t io n o f S 2 [Ref: Harper's 27/e P-75-77; 26/e P- 74]

Ea © Eb © \ Ec © * -------- \
S i---------------------------------►
S2 -------------------------------- ►Pi---------------------------------- ►P2
- Defective Ec would block the conversion of Pi to P2 & thus Pi start accumulating.
- This increased Pi exerts a negative feedback effects on Eb and decreases the conversion of S2 to Pi & thus S 2 accumulates.
- Increased S2 exerts a positive feedback on Ea & increases the conversion of Si to S2. The more S 2 accumulates the more
enzyme Ea is activated to convert Si to S 2 & thus S 2 goes on accumulating.
Biochemistry: Structures & Functions of Proteins and Enzymes ■ 549

101. C i.e. Lipoprotein lipase [Ref: Harper 28u'/e p. 59]

Lipoprtein lipase is a functional enzym e (i.e. has physiological role) that hydrolyze lipids in lipoproteins
(chylom icrons & VLDL) into free fatty acids and glycerol molecule^.

102. D i.e. Catalyze the same reaction


103. A i.e. Different Km value; C i.e. Consist of multimeric complex; E i.e. Have different physical properties
104. A i.e. LD-1 105. B i.e. Brain Ischemia
[Ref: Chatterjea 8/e p 123, 644-647, 5 2 2 ; Lippincott 5/e p 65-66; Das 8/e p 94; H arper's 29/e P-63-65; Harsh M ohan 4/e 296; D ebajyoti D as 8/e P 941

- Isozym es (iso enzym es) catalyze the sam e reaction but at different ratesQ. T hey differ from each other structurally ☆
(d/t difference in quarternary structure), electrophoretically (d/t different n um ber of charged acidic or basic amino
acids), physically ie in physical properties such as heat resistance (d/t genetically determ ined differences in
am inoacid sequence). They differ in pH optim a, Km and Vmax values as well as in enzym atic classification num bers.
Isoenzymes have different tissue distribution and m ay contain various com binations of different subunits of
m ultim eric complex^!.
- Isoenzym e LDH -1 with four 'H ' subunit predom inates in cardiac tissued, since heart expresses the H subunit almost
exclusively. CPK-1 is found in brain ischem iad

106. C i.e. Vander Waals [Ref: Textbook of Biochemistry Thomas M.Devlin 4/e P 156]

'Vander W alls' forces are too weak, to actively participate in formation of enzym e- substrate complex.

Various models have been proposed to explain the substrate specificity of enzymes e.g. 'Lock & Key' method
• Substrate fits in the binding site of enzyme just as key fits into proper lock (strong interaction). Forces contribute in
binding are:- Hydrogen bondingd
- Ionic (electrostatic) bondingd
- Hydrophobic interaction
• Enzymes function to decrease the activation energy so that reactions can occur at normal body temperature. The
substrate form a co v alen t bon dd with the enzymes active site, & accelerates the reaction.

107. A i.e. Pepsin 108. D i.e. Serine [Ref: Harper's 27/e P-53-55!
109. C i.e. Chym otrypsin w ill bind the substrate but w ill not cause cleavage

Serin e-195 is a highly reactive residue of chymotrypsin molecule. It endows chymotrypsin with its 'Reactivity' and not its
ability to bind to substrate. In absence of S e r-195 residue, chym otrypsin is inactivated i.e., it w ill not cleave, however, it
still has the ability to bind (by virtue of other remaining residues).

110. A i.e. Oxidase 111. A i.e. Transferase 112. D i.e. Lyases


113. A i.e. Transferase 114. C i.e. Hydrolase 115. None » > B i.e. Peroxidase
116. B i.e. Hydrolases 117. B i.e. Cytochrome P 450 118. D i.e. Required for carboxylation of drugs
[Ref: Chattergea & Shinde 7/e p. 469,116; Harper's 28/e p. 52; Lippincott's 5/e p. 53-54; Lehninger 5,h/ed pg- 816, 518-20]

t- Transferase - enzym es catalyze transfer of a particular group (containing O , N- or P-) from one substrate to anotherQ. ☆
W hereas, hydrolase bring about hydrolysis by addition of waterd.

- Fum arase is a lyased, glutamine synthetase is ligased, tyrosinase is oxidased and hexokinase is a transferased.
- Enzym es involved in oxidation (rem oval of e~) and reduction (gain o f electrons) are called oxido - reductase and are
classified into 4 groups: oxidase, oxygenase, dehydrogenase, hydroperoxidase (peroxidase and catalase)d.
- H ydrolase is another class of enzym e that catalyzes hydrolysis i.e. cleavage o f bonds by addition o f water.
- Carboxylase enzym e, w hich belongs to ligase class is required for carboxylationd
- O xigenase (w hich belongs to oxido-reductase class) cause incorporation of 2 atom s o f O 2 (dioxygenase) or 1 atom of
O 2 (m onoxygenase o r hydroxylase)
550 ■ A Complete Review of Short Subjects

6 - Classes of enzymes (International Union of Biochemistry = IUB) Mn - “TO HILL”


Class Feature Example
Transferase Catalyze transfer of C - N-, or P- - Aspartate & alanine transminase (AST/ALT)
containing group from one substrate to - HexokinaseQ, Hexose 1- phosphate uridyl
another<2, such as transferase
Serine <-------- - Glycine - Phosphoglucomutase
SHT - Ornithine carbamoyl transferase
- Serine hydroxy methyl transferase (SHT)
Oxidoreductase Catalyze oxidation reduction reactions, - Alcohol dehydrogenase, lactate dehydrogenase,
such as glucose - 6 - phosphate dehydrogenase
Lactate dehydrogenase - Glutathione reductase
Lactate <----------------* Pyruvate - Xanthine oxidase, TyrosinaseQ

Hydrolases Catalyze hydrolysis i.e., cleavage of bonds - Pepsin, trypsin, esterases


by addition of waterQ, such as - Glucose 6 phosphatase, glycoside hydrolases
Urea + H 20 — ------- >C 0 7 + 2 NH-, - UreaseQ
Urease A J

Isomerases Involved in isomerization of substrate i.e., - Triose phosphate isomerase, retinal isomerase
catalyze racemization of optical or - UDP- glucose epimerase, racemase
geometric isomers, such as - Methyl malonyl CoA mutase (MMCM)

Methyl Malonyl <----------------^Succinyl


CoA MMCoAM CoA
Lysases Catalyze cleavage of C - C , C - S, & certain - FumaraseQ, Argino- succinase
C - N bonds and facilitate removal of small - Histidine decarboxylase^
molecules from a large substrate - Pyruvate decarboxylase^
Pyruvate — p>rum,e >Acetaldehyde
decarboxylase

Ligases Catalyze formation of bonds between - DNA ligase, Pyruvate carboxylase (PC)
carbon and O, S, N coupled to hydrolysis - Alanyl t-RNA synthetase
of high energy phosphates and involved in - Glutam ine synthetase^
joining together two substrates.
PC
Pyruvate + C 0 2 -^ r-^ y O x alo acetate

ATP ADP
119. D i .e . M a l o n a t e d e h y d r o g e n a s e 120. C i.e. Phosphatase [Ref: Harper's 27fe P-78-79; Chatterjea 6/e P-559]

Reactions 1
M agnesium is required as co-factor Magnesium is required
Glycolysis Phosphofructokinase^ in: in covalent modification
Glycogen G lycogen synthetase® [Mnemonic: "C T PARK"] of enzymes by
synthesis C - Carboxylase A- Adenyl cyclase phosphorylation,
Glycogenolysis Glycogen T - Transketolase R-Ribonuclease dephosphorylation of
phosphorylase® P - Phosphatase^ K- Kinase seryl residues.
T C A C y le Isod trate - Peptidase
dehydrogenase®
Fatty acid Acetyl C O A ATP ADP

synthesis Carboxylase®
Cholesterol H M G C O A reductase<2
synthesis -O -PC V -
Ketone body HM C O A synthetaseQ
synthesis
Bile a d d synthesis 7- a - hydroxylaseQ
h 2o
Catecholamine Tyrosine hydroxylaseQ
synthesis
Urea synthesis Carbamoyl
transferaseQ
Biochemistry: Structures & Functions of Proteins and Enzymes ■ 551

121. C i.e. Non protein 122. C i.e. H oloenzym e


123. A i.e. Pyridoxine; B i.e. Biotin; C i.e. T hiam ine; D i.e. Folic acid 124. D i.e. A ccept one of the cleavage products
125. A i.e. Thiam ine is a coenzym e for pyruvate dehydrogenase & a ketoglutarate ... 126. A i.e. C arboxylation
[Ref: Lippincott's 4je p. 54- 55,101,110, Harper's 27/e p. 50-51,490; Chattergea b Shinde 7/e p. 114-16 ; Vasudevan 4/e p. 37- 39]

Except for ribozym e, w hich is RNA0, all the enzymes are protein in nature with large molecular weight. Some enzymes
require, molecules other than proteins for enzymic activity. Holoenzym e is active enzyme with its nonprotein component,
whereas the enzyme without its nonprotein moiety is termed an apoenzyme and is inactive. So the protein part o f enzyme is
called apoenzym e, non-protein part - coenzym e, cofactor or prosthetic group, and the complete structure holoenzym e.

127. A i.e. Biotin [Ref: Nelson 17je p. 412]

M ultiple carboxylase deficiency is a group of disorder resulting from defective utilization of biotin. Since b iotin is a
coenzym e fo r carboxylase enzym es (carboxylation), it results in m alfunction of all carboxylases and organic acedimia. It
is treated b y biotinQ.

128. B i.e. Pantothenic acid [Ref: Harper's 27/eP-503]

The active form of pantothenic a d d is CO- enzyme A (COA) & the A cyl carrier protein (ACP). Pantothenic acid acts as a
cofactor after its m odification^ ...

Pentothenate 4- phosphopantothenate 4- phosphopantothylcystine

ATP ADP ATP+ ADP+P


Cysteine CCh

Coenyme A Dephospho coenzyme 4- Phosphopantetheine

ADP ATP Ppi ATP

129. A i.e. Alcohol dehydrogenase 130. C i.e. G lutathione peroxidase 131. B i.e. Zinc

Functions of Zinc

Zinc forms an integral part of Others: • Role in Vit A


several enzymes (metallo enzymes) - A lco h ol dehydrogenase, la cta te d eh y d rog en ase0, metabolism
in the body: glutamate dehydrogense, malate dehydrogenase, - Stimulate release of
• Superoxide d ism u taseQ in cytosol: Carbon monoxide dehydrogenase, D-2 hydroxy acid Vit A from liver
Cu-Zn protein complex with two dehydrogenase, galactitol 1 P 0 4 5 dehydrogenase - Participate in the
Zn++ per molecule of the enzyme. - A lka lin e p h o s p h a ta se0, 8 ALA dehydratase regeneration of
• C arbon ic an hydraseQpresent in - Retinine reductase, DNA & RNA polymerase, Rhodopsin in eye
RBC, parietal cells and renal histone deacetylase during dark
tubular epithelial cells, one Zn++ - P lactamase, lactoylglutathione lyase, insulin adaptation (as present
per molecule degrading enzyme. in Retinine reductase)
• Leucine- Am ino- peptidase of - Glutamae carbboxypeptidase II, glutamyl • Role in insulin
intestinal juice aminopep tidase secretion
• Carboxyl peptidase A - of - ADAM (a disintegrin & metalloprotease domain) of • Role in growth &
p an creatic juice proteinases (eg TACE i.e. TNF a converting Reproduction
enzyme) • Role in wound healing

132. B i.e. C ardiom yopathy 133. A i.e. G lutathione Peroxidase


[Ref: Harper 29/e p 118, 201, 270, 541; Harrison 18/e p 604; Chattrjea 8/e p 211, 630-31, 560, 801; Indian journal of Medial Science
1190; 44(7): p. 173-7]
552 ■ A Complete Review of Short Subjects

■--------------------------------------------------------------------------------------------------------------------------------------------------------------------------
- Selenium in form of selenocysteine is a component (as a prosthetic groop) of glutathione peroxidase, thioredoxin, and
%V
deiodinase that converts thyroxine to triodothyronine.
- Selenium deficiency causes Keshan disease (endem ic cardiom yopathy) and Kaschinbeck disease (endemic
degenerative osteoarthritis or osteopathy particularly affecting children). Low selenium levels also increase the risk of
throm botic episode, cancer (specially G I, m ore so oesophegeal & orolaryngeal cancers). Concomitant deficiency of Se
& iodine m ay worsen clinical manifestations of cretinism.
\ _

134. D i.e. Antibody with catalytic activity [Ref: NMS Biochemistry 4/e P - 109]

Abenzym es are catalytic antibodies (antibody + enzyme)Q

135. B i.e. Cu 136. C i.e. M yeloperoxidase 137. A i.e. Cd++-C u++ & Z n++ [Ref: Harper 29/e p 640]

--------------------------------------------------------------------------------------- !------------------------------------------------------------------------------------H'
- M etallothioneins regulate tissue levels of copper (Cu2+), zinc (Zn2+), cadm ium (Cd2+), and m ercury (Hg2+)
- C opper containing enzym es include ferroxidase (ceruloplasm in), lysyl oxidase, ascorbic acid oxidaseQ, amine oxidae,
cytochrom e oxidase, superoxide dism utase and tyrosinase^

____________________ Metallothioneins____________________ Enzymes that contain Copper


- Are group of small proteins ( 6 . 5 KDa), found in cytosol of cells,
particularly of liver, kidney & intestine A m ine oxidaseQ
- They have high content of cysteine. The SH group of cysteine binds Superoxide dismutaseQ
the metal C ytochrom e oxidaseQ
- Tissue levels of Cu2+, Zn2+, Cd2+ and Hg2+ i.e. (Copper, Zinc, TyrosinaseQ___________
Cadmium & mercury)^ are regulated by m etallothioneins
- It may function to store the above metals in nontoxic form & are
involved in their over all m etabolism in the body.
- The concentration of metallothionines in tissue is increased by
acute intake of metals or adm inistration of certain horm ones or
cytokines.

138. D i.e. H epatocellular carcinom a [Ref: Robbins 7/e p. 910; Harper's 27/e p. 595-97; Chattergea 7/e p. 584-86]

- Copper is excreted in the bile into GI tract from which it is not reabsorbed. So normally urine contains only traces of
copper. Since copper h om eosta sis is m aintained a lm o s t exclusively by b iliary excretionQ, any disease which hampers
its biliary excretion will lead to accumulation of Cu in liver —> increased blood levels & increased urinary excrition. So
ch o lestatic hepatitis, prim ary b iliary cirrhosis and prim ary sclerosing ch olan g itis sh ow in creased copper excretion in
urineQ.
- W ilson disease has defect in copper binding P type ATPase (ATP 7B protein) which cause defective excretion of copper
into bile & reduction in incorporation of copper into apoceruloplasmin 1/1 accumulation of Cu in liver, brain, kidney &
RBC.
Chapter -6 METABOLISM OF PROTEIN & AMINO ACIDS

QUESTIONS

Fate of Amino Group 10. Glutamine synthetase is a - (AIIMS 03, Delhi 05)
A. Isomerase □
1. Transfer of an amino group from an am ino acid to an B. Ligase □
alpha keto acid is done by? (AI 11) C. Lyase □
A. Transaminases □ D. Transferase □
B. Aminases □ 11. Source of ammonia in urine (jipmer 02)
C. Transketolase □ A. Glutaminase □
D. Decarboxylase □ B. Urease □
2. Transm ination of pyruvate with glutam ate produces C. Glutamate dehydrogenase □
A. Oxaloacetate & aspartate (AI 03) □ D. Arginase □
B. Alanine & asparate □ 12. The amino acid producing (major source of) ammonia
C. Oxaloacetate & a-ketoglutarate □ in kidney is: (AIIMS 07,13)
D. Alanine & a-ketoglutarate □ A. Glutamine □
3. Trans-am ination of pyruvate and glutam ic acid leads B. Alanine, Aspartate □
to the form ation of (AI 06) C. Methionine □
A. Oxaloacetate □ D. Glycine □
B. a-ketoglutarate □ E. Glutamate
C. Aspartate □ 13. Oxidative deamination is catalyzed by-
D. Malate □ A. Glutaminase (NBE P 15) □
4. All are involved in non toxic transport of amm onia B. Glutamine synthase □
except C. Glutamate dehydrogenase □
A. Glutamine synthetase (WB 10, UP 09) □ D. None of the above □
B. Glutaminase □
C. Alanine cycle □ Urea Cycle
D. SGPT □
5. The am ino acid which serves as a carrier of am m onia 14. In urea cycle, hydrolysis of arginine forms- (NBE P 15)
from skeletal m uscle to liver is: (NBE P 15,14; AI 06) A. Citrulline □
A. Alanine □ B. Ornithine □
B. Methionine □ C. Carbomoyl phosphate □
C. Arginine □ D. Arginosuccinase □
D. Glutamine □ 15. W hich of the follow ing enzyme (s) is/are not involved
6. G lutam ine in blood acts as: (PGI 08) in urea cycle: (PGI 12)
A. NH 3 transporter □ A. Glutamate dehydrogenase □
B. Toxic element □ B. Argininosucinate synthetase □
C. Stored energy □ C. a-KG dehydrogenase □
D. Abnormal metabolite □ D. Isocitrate dehydrogenase □
7. W hich end product of citric acid cycle is used in E. Fumarase □
detoxification of am m onia in brain (AIIMS 01) 16. Urea cycle occurs in : (NBE P 14; AI 11)
A. Oxaloacetate □ A. Liver □
B. Alphaketoglutarate □ B. G.l.T. □
C. Succinate □ C. Spleen □
D. Citrate □ D. Kidney □
8. In M etabolic alkalosis, w hich is true about excretion in 17. Urea cycle enzymes are: (PGI 14, 09)
urine: (PGI 03) A. Glutaminase □
A. More of N H 3 □ B. Asparginase □
B. Less of aceto acetic acid □ C. Arginosuccinate synthetase □
C. Betahydroxy butyric acid □ D. Ornithine transcaroxylase □
D. Less ammonia □ E. Glutamate dehydrogenase □
9. Am m onia is detoxified (removed as) in brain to: 18. Urea is produced by the enzyme- (NBE P 13; PGI 12)
A. Urea (NBE P 15,13; AIIMS 16,15; AI 09) n A. Urease □
B. Glutamine □ B. Uricase □
C. GABA □ C. Arginase □
D. Uric acid □ D. Glutaminase □
554 ■ A Complete Review of Short Subjects

19. Urea is mainly formed in liver and to small extent in- D. Presence of Ser-His-Asp catalytic triad at the active
A. Spleen (DNB 15; AMU 03) □ site □
B. Heart □ 29. Guanido acitic acid is formed in....from....-
C. Muscle □ A. Kidney; Arginine+Glycine (AI 11, DNB 15) □
D. Brain □ B. Liver; Methionine + Glycine □
20. Which of the following is true in relation of urea cycle: C. Liver; Cysteine + Arginine □
A. First 2 steps in cytoplasm (PGI 05) □ D. Muscle; Citrulline + Aspartate □
B. First 2 steps in mitochondria □ 30. Creatinine is synthesised from - (DNB 10; NBE P 14,13)
C. Defect of enzyme of any step can cause deficiency A. Glycine, arginine and methionine □
disease □ B. Glycine and methionine □
D. Urea is formed by N H 3 , glutamic acid and CO 2 □ C. Ornithine and glycine □
E. Citruline is formed by combination of carbomoyl D. Thymine and ornithine □
phosphate and L. Ornithine □ 31. Creatine is made up of all, except- (NBE P 15)
21. Sources of the nitrogen in urea cycle are: (AI 01) A. Glycine □
A. Aspartate and ammonia □ B. Alanine □
B. Glutamate and ammonia □ C. Methionine □
D. Arginine □
C. Arginine and ammonia □
32. All of the follow ing amino acids are converted to
D. Uric acid □
succinyl-CoA, except: (AI 06)
22. Urea cycle, true is all except (AMU 10, UP 09)
A. Methionine □
A. Aspartate converted to fumarate □
B. Isoleucine □
B. Ornithine-citrulline transporter aminoacids □
C. Valine □
C. N-acetyl glutamate is allosteric inhibitor □
D. Histidine □
D. Aspartate-arginino succinate shunt □
33. Which one of the following can be a homologous
23. A child presents with m etabolic lactic acidosis along
substitution for isoleucine in a protein sequence?
with hyperam m onem ia and convulsion is due to
A. Methionine (AI 06) □
A. Reye's syndrome (AIIIMS 2K)□
B. Aspartic acid □
B. Mitochondrial enzyme disorders (MELAS) □
C. Valine □
C. Urea cycle defect □
D. Arginine □
D. Phenylketonuria □
34. At physiological pH, the carboxy-terminal of a peptide
24. Hyperammonaemia inhibit TCA cycle by depleting:
is: (AIIMS 05)
A. Oxaloacetate (PGI 09) □
A. Positively charged □
B. a-ketoglutarate □
B. Negatively charged □
C. Citrate □
C. Neural □
D. Succinyl Co-A □
D. Infinitely charged □
E. Fumarate □
35. Iso electric pH is: (AIIMS 05)
Amino A. Point at which all amino acids are in zwitter ion
form □
25. Cystine s formed by- (NBE P 15)
B. Point at which pH of ions are same. □
A. Hydroxylation of cysteine molecule □
C. Point at which pH of ions does not alter. □
B. Carboxylation of cysteine molecule □
D. Point at which positive and negative charges on
C. Peptide bond between two cysteine molecule □
the amino acid are equal. □
D. Disulfide bond between cysteine molecule □
36. At isoelectric pH protein: (PGI 03)
26. Initial amino acid in prokaryotic protein synthesis-
A. Have Net charge 'O’ □
A. Arginine (NBE P 15) D B. Are positively charged □
B. Methionine □
C. Are negatively charged □
C. Formyl-methionine □ D. Don't migrate □
D. Alanine □ 37. The amino acid residue having an im inoside chain is:
2 7. Tryptophan is glucogenic & ketogenic by producing- A. Lysine (AI 05) □
A. Acetyl coA & alanine (NBE P 1 5 ) 0 B. Histidine □
B. Acetoacetate & fumarate □ C. Tyrosine □
C. Acetoacetate & arginine □ D. Proline □
D. Arginine & alanine □ 38. Sulphur containing amino acid is/are: (PGI 15,12,06)
28. A common feature of all serine proteases is: (AI 06) A. Aspartic acid □
A. Autocatalytic activation of zymogen precursor □ B. Glutamine □
B. Tight binding of pancreatic trypsin inhibitor □ C. Methionine & Cysteine □
C. Cleavage of protein on the carboxyl site of serine D. Glycine □
residues □ E. Serine □
Biochemistry: Metabolism of Proteins and Amino Acids ■ 555

39. Replacing alanine by which amino acid will increase 49. Neutral amino acid is- (NBE P 15)
UV absorbance of protein at 280 nm wavelength: A. Aspartate □
A. Leucine (AIIMS 08) □ B. Arginine □
B. Proline □ C. Glycine □
C. Arginine □ D. Histidine □
D. Tryptophan □ 50. Acidic amino acids are- (PGI 10,03)
4 0 . Property of photochromosity is seen amongst the A. Asparagine □
following amino acids OR The property of proteins to B. Arginine □
absorb ultraviolet rays of light is due to: (AI 11, 09) C. Glutamine □
A. Unsaturated aminoacid □ D. Lysine □
B. Aromatic aminoacid □ 51. Basic amino acids are: (AI 01; AMU 04)
C. Monocarboxylic acid □ A. Aspartate and glutamate □
D. Dicarboxylic acid □ B. Serine and glycine □
4 1 . Aromatic amino acids are A/E OR Aromatic ring is C. Lysine and arginine □
present in all except- (NBE P 15,14,13; AIIMS 15,13) D. None of the above □
A . Tryptophan □ 52. M ost basic amino acid out of the follow ing is:
B. Tyrosine □ A. Alanine (NBE P 13) □
C. Lysine □ B. Arginine □
D. Phenylalanine □ C. Histidine □
4 2 . Hydrophobic aminoacids are: (PGI 10) D. Lysine □
A . Methione □ 53 . Basic amino acid (s) is /are A/E O R At physiological
B. Isoleucine □ pH, which of these amino acids has a positive charge?:
C. Tyrosine □ A. Arginine (PGI 16,15,09; AIIMS 16,15) O
D. Alanine □ B. Proline □
E. Asparagine □ C. Lysine □
4 3 . Polar amino acid (s ) is/are: (PGI 15,12) D. Histidine □
A . Serine □ 54. Which of these amino acids will migrate slowest to the
B. Tryptophan □ anode end at the physiological pH- (AIIMS 02)
C. Tyrosine □ A. Aspartic acid □
D. Valine □ B. Glycine □
E. Lysine □ C. Lysine □
4 4 . Non Polar Amino Acids are: (PGI 10) D. Valine □
A. Alanine □ 55. Non-Essential amino acid is - (AI 08, DNB 09)
B . Tryptophan □
A. Tyrosine (AIIMS 15,13)D
C. Isoleucine □
B. Phenylalanine □
D. Lysine □
C. Lysine □
E. Tyrosine □
D. Threonine □
4 5 . Which of the follow ing side chains is least polar 56. An essential amino acid in man is -
A . Methyl (AI 09) □ A. L-methionine (AIIMS 14,12) □
B. Carboxyl □ B. L-tyrosine □
C. Amino □ C. L-serine □
D. Phosphate □ D. L-proline □
4 6 . Leucine is a aminoacid with a: (AI 08) 57. Aminoacid that enters the TCA cycle for
A . Nonpolar side chain
gluconeogenesis & Ketogenin in nature- (PGI 06)

B. Polar side chain A. Pheylalanine □

B. Alanine □
C. Negatively charged side chain □
C. Glycine □
D. Positively charged side chain □
4 7 . Polar amino acid are all except- (NBE P 15) D. Serine □
58. Essential amino acid is- (NBE P 15,14,13)
A . Glutamic acid □
A. Alanine □
B. Histidine □
B. Serine □
C. Glutamine □
C. Arginine □
D. Methionine □
D. Proline □
4 8 . Most non polar Amino Acid is : (A im
59. All ar essential amino acid except-
A . Leucine □
A. Phenylalanine (AIIMS 14; AI 08; NBE P 15,14,13) □
B. Glycine □
B. Lysine □
C. Arginine □
C. Leucine □
D. Lysine □
D. Glycine □
556 ■ A Complete Review of Short Subjects

60. Essential amino acid amongst the following- 70. Which of these amino acids does not have anomeric
A. Arginine (NBE P 15,14,13) □ carbon atom OR Optically inactive Amino Acid is OR
B. Lysine □ Flexibility of protein depends on:
C. Threonine □ A. Valine (NBE P 13; AIIMS 14,02; PGI 10)D
D. All □ B. Alanine □
61. Semiessential amino acids are- (PGI 15,13) C. Tyrosine □
A. Arginine □ D. Glycine □
B. Histidine □ 71. M ajor from of folic acid to transfer one carbon is-
C. Glycine □ A. Methylene THF (N B E P 1 5 )D
D. Phenylalanine □ B. Formyl THF □
62. Negative nitrogen balance is produced in humans by C. Methyl THF □
elimination from the diet of all of the following amino D. ALL □
acids except - (DNB 13; Delhi 10) 72. Vitam in involved in 1 carbon m etabolism OR Which
A. Leucine □ vitamin is required for transfer of 1 - carbon unit:
B. Methionine □ A. Folic acid (NBE P 15,13; PGI 15,14; AIIMS 14) □
C. Lysine & Threonine □ B. Thiamine □
D. Serine □ C. Pyridoxine □
63. All of the following are essential aminoacids except: D. Vitamin A □
A. Methionine (AIIMS 15, 07) □ 73. Which vitamin is related to a co-factor in glycine
B. Lysine □ Metabolism is- (AIIMS 03)
C. Alanine □ A. V, E □
D. Leucine □ B. Folic acid □
64. Substitution of which one of the following amino C. Thiamine □
acids in place of alanine would increase the D. Pantothenic acid □
absorbance of proteins at 280 mm: (AIIMS 05) 74. Conversion of glycine to serine requires: (PGI 02)
A. Leucine □ A. Folic acid □
B. Arginine □ B. Thiamine □
C. Tryptophan □ B. Vit. C. □
D. Protein □ C. Fe2+ □
65. Which one of the following amino acids is most likely D. Pyridoxal phosphate □
to be found in the transmembrane region of a protein 75. Amino acid with dissociation constant closest to
A. Lysine (AIIMS 04) □ physiological pH is OR Most stable amino acid at the
physiological pH is OR Maximum influence of local
B. Arginine □
milieu on ionization is on OR Which of the following
C. Leucine □
amino acid is active at neutral pH OR Which amino
D. Aspartate □
acid can protonate & deprotonate at neutral pH OR At
66. All biologically active proteins are (AIIMS 03)
physiological pH buffering action in blood is caused
A. L-forms □
by: (NBE P 15,14,13; AIIMS 13,10; PGI 14, 09)
B. D-forms □
A. Serine □
C. Mostly D form □
B. Histidine □
D. D and L form □
C. Threonine □
67. The conversion of an optically pure isomer
D. Proline □
(enantiomer) into a mixture of equal amounts of both
76. Which of the following is true about glutathione
dextro and levo forms is called as: (AI 03) A. Contain sulfhydral group (PGI 14,13) □
A. Polymerization □ B. Forms met Hb from Fib □
B. Stereoisomerization □ C. It does not detoxity superoxide radicals □
C. Racemization □ D. Transport aminoacid across cell membrane □
D. Fractionation □ E. Part of enzymes □
68. Amino acid with double chiral is- (NBE P 15) 77. In glutathione which amino acid is reducing agent-
A. Phenyalanine □ A. Glutamic acid (AIIMS 07) □
B. Threonine □ B. Glycine □
C. Tryptophane □ C. Cysteine □
D. Tyrosine □ D. Alanine □
69. Amino acid which lacks chirality is: (NBE P 14; AI 10) 78. Regarding glutathione which of the following is/are
A. Lysine □ true:
B. Leucine □ A. It helps in absorption of certain amino acids □
C. Histidine □ B. It inactivates some enzymes (PGI 02) □
D. Glycine □ C. It helps in membrane transport □
D. It helps in conjugation reactions □
Biochemistry: Metabolism of Proteins and Amino Acids ■ 557

79. Amino acids secreted in cystinuria are all except: C. Glutamine □


A. Ornithine (AIIMS 01) □ D. Methionine □
B. Histidine □ 90. Ammonia is detoxified in brain by-
C. Arginine □ A. Uric acid (JIPMER 01, AMU 08, J&K 03) □
D. Lysine □ B. Glutamine □
80. Cystinuria is associated with excretion of which A.A in C. Creatinine □
urine: (PGI 09) D. Urea □
A. Cystine, lysine, Tyrosine and alanine □ 91. The urea in the brain is derived from- (AI 01, MP 06)
B. Cystine, lysine, ornithine and alanine □ A. Glutamine □
C. Cystine, ornithine, lysine and arginine □ B. Glutamine acid □
D. Cystine, ornithine, leucine and arginine □ C. Lactate □
81. Aminoacid containing indole ring is - D. Uric acid □
A. Arginine (DNB 12, PGI 10) □ 92. Phenylalanine is the precursor of all the following,
B. Tryptophan □ except:
C. Tyrosine □ A. Tyrosine (AI 02) □
D. Histidine □ B. Epinephrine □
82. The following is false about Tryptophan (NBE P 14,13) C. Thyroxine □
A. Non essential amino acid (DNB 08; AI 06) □ D. Melatonin □
B. Involved in serotonin synthsis □ 93. Increased risk of Myocardial infarction is associated
with which amino acid (AIIMS 04)
C. Involved in niacin synthesis □
A. Methionine □
D. Involved in melatonin synthesis □
B. Homocystine □
83. The amino acid that can be converted into a vitamin
C. Ornithine □
A. Glycine (N B E P 1 3 )D
D. Valine □
B. Tryptophan □
94. All will predispose to atherosclerosis except:
C. Phenylalanine □
A. Homocystinemia (PGI 05) □
D. Lysine □
B. Fibrinogen □
84. Which of the following amino acids is involved in the
C. Calcium □
synthesis of thyroxine - (NBE P 13,11)
D. Lipoprotein A □
A . Glycine □
95. In phenylketonuria the main aim for first line therapy
B. Methionine □
is: (AI11)
C. Threonine □
A. Limiting the substratefor deficient enzyme □
D. Tyrosine □
B. Giving the missing amino acids by diet □
85. An amino acid not involved in urea synthesis is -
C. Replacement of enzyme □
A. Ornithine (DNB 07,13) □
D. To replace deficient product □
B. Citrulline □
96. PKU is a congenital amino acid metabolic disorder. In
C. Arginine □ one of the follow ing rare variants of PKU Dihydro
D. Aspartic acid □ Biopterin synthesis is affected. The enzyme deficient
86. M elatonin is synthesised from- is: (PGI 08)
A. Tryprophan (PGI 14,12; AIIMS 15; NBE P 15,13)D A. Histidine decarboxylase □
B. Serotonin □ B. Phenylalanine hydroxylase □
C. Phenylalanine □ C. Dihydropterin reductase □
D. Histidine □ D. Tyrosine deficiency □
87. Which of the following is precursor of adrenalin and 97. What results due to incomplete oxidation of
thyroxine synthesis - (AIIMS 14,12; UP 08) phenylpyruvic acid: (AIIMS 03)
A. Phenyl alanine □ A. Albinism □
B. Tyrosine □ B. Alkaptonuria □
C. Tryptophan □ C. Phenylketonuria □
D. None of the above □ D. Tyrosinemia □
88. Which of the following is not synthesised from 98. Ferric chloride test in PKU gives: (PGI 06, 08)
tyrosine - (AIIMS 14,12, 08) A. Red □
A. Norepinephrine □ B. Green □
B. Melatonin □ C. Blue □
C. Thyroxine □ D. Yellow □
D. Dopamine □ 99. Mousy odour urine is seen in. (PGI 11,07)
89. The substance from which ammonia is produce by the A. Maple syrup urine □
kidney is- (AIIMS 06, CMC 08, Jipmer 09) B. Phenylketonuria □
A . Glycine □ C. Isovalericaciduria □
B. Alanine □ D. Cystinuria □
558 ■ A Complete Review of Short Subjects

100.Tyrosinemics are more susceptible to develop: C. Thiamin linked decarboxylase □


A. Adenocarcinoma colon (AIIMS 07) □ D. Leucine transaminse □
B. Melanoma □ 110.Correct com bination of urine odour in various
C. Retinoblastoma □ metabolic disorders: (PGI 13)
D. Hepatic carcinoma □ A. Phenylketonuria-Mousy odour □
101.A 40 yr old wom an presents with progressive B. Tyrosinemia-Rotten cabbage □
palm oplantar pigm entation. X-ray spine shows C. Hawkinsuria-Potato smell □
calcification of intervertebral discs. On adding D. Maple syrup disease-Rotten tomato □
Benedict's reagent to urine, it gives greenish brown E. Alkaptonuria-Rotten egg □
precipitate and blue-black supernatant fluid. W hat is 111.Phosphorylation of amino acid by: (PGI 08)
the diagnosis? (AIIMS 10) A. Serine □
A. Phenylketonuria □ B. Tyrosine □
B. Alkaptonuria □ C. Leucine □
C. Tyrosinemia type-2 □ D. Tryptophan □
D. Argininosuccinic aciduria □ 112.Succinyl Co-A is formed by: (PGI 08)
102. 1f urine sam ple darkens on standing: the m ost likely A. Histidine □
condition is : (AI 01) B. Leucine □
A. Phenylketonuria □ C. Valine □
B. Alkaptonuria □ D. Lysine □
C. Maple syrup disease □ 113.A baby presents with refusal to feed, skin lesions,
D. Tyrosinemia □ seizures, ketosis organic acids in urine with normal
103.M etabolites of tryptophan can give rise to: (PGI 02) am m onia; likely diagnosis is: (AI 01)
A. Diarrhoea □ A. Proprionic aciduria □
B. Vasoconstriction □ B. Multiple carboxylase deficiency □
C. Flushing □ C. Maple syrup urine disease □
D. Can predispose to albinism □ D. Urea cycle enzyme deficiency □
E. Phenylketonuria □ 114.The com parison of the amino acid sequence of
104.1n maple syrup urine disease the amino acids, excreted Cytochrom e C from different species shows many
in the urine are-fPGJ 15,13,10; NBE P 14,13; AIIMS 14,09) variations. M ost of these variations are found:
A. Leucine □ A. Randomly (AI 06) □
B. Isoleucine □ B. Only in helical regions □
C. Valine □ C. Only in strand regions □
D. a, b & c □ D. Mainly in loop regions □
105.W hich of the following am inoacid is excreted in urine
in m aple syrup urine disease : (AI 09)
Hemoglobin, Bile Pigment & Porphyrin
A. Tryptophan □ 115.Hemoproteins are: (PGI 14,09)
B. Phenylalanine □ A. Cytochrome C (Oxidase) □
C. Leucine □ B. Cytochrome 450 □
D. Arginine □ C. Myoglobin □
106.Branched chain ketoacid decarboxylation is defective D. Hemoglobin □
in (NBE P 13; AI 11) E. Catalase □
A. Maple Syrup urine disease □ 116.Haemoglobin is a buffer because of OR In
B. Hartnup disease □ hem oglobin, iron is bound to - (NBE P 15,13)
C. Alkaptonuria □ A. Histidine residue (AIIMS 14,12) □
D. GMI Gangliosidosis □ B. Glycoprotein nature □
107.Disease of branched chain amino acid includes: C. Weak acidic nature □
A. Phenylketonuria (PGI 15,13) □ D. Iron molecule □
B. Maple syrup disease □ 117.False statem ent about hem oglobin structure?
C. Taysach's disease □ A. Hb has 2 polypeptide (AIIMS 12,10) □
D. Isovaleric acidemia □ B. Iron is present in ferrous state □
E. Niemann-pick disease □ C. Hb structurally similarto myoglobin □
108.Orotic aciduria is due to deficiency of- (NBE P 15) D. Ferrous ions are in porphyrin rings □
A. Decarboxylase □ 118.1n hem oglobin the innate affinity of hem e for carbon
B. Tyrosinase □ monoxide is diminished by the presence of:
C. Isomerase □ A. His F 8 (AIIMS 15,10,02) □
D. Homogentisate oxidase □ B. His E7 □
109.Sweaty feet odor in body fluids occur d/t deficiency of C. Gly B6 □
A. Biotin (KA 09, AI 06) □ D. Thr C4 □
B. FAD linked dehydrogenase □ 119.True statem ent about hem oglobin is- (NBE P 15)
Biochemistry: Metabolism of Proteins and Amino Acids ■ 559

A. Each hemoglobin molecule is made of 4 polypeptides B. Converts a hydrophobic compound to a hydrophilic


of each subunit □ one □
B. Two alpha and two beta subunits having a O 2 C. Enables the bilirubin molecule to cross the cell
attached to each subunit □ membrane □
C. Each hemoglobin molecule binds to only one 0 2 = D. Is increased during neonatal jaundice □
molecule □ 129.Bilirubin in conjugation with glucoronic acid
D. Each hemoglobin has one heme molecule □ conjugation has properties: (PGI 07)
120.Heme is which porphyrin- (NBE P 15) A. Able to cross cell membrane □
A. Type 1 □ B. Hydrophillic to hydrophobic □
B. Type II □ C. Hydrophobic to hydrophilic □
C. Type III □ D. Lipid soluble □
D. Type IV □ 130.Detergent action of bile salts is due to: (AIIMS 07)
121.Haem oglobin unlike m yoglobin shows: (PGI 12, 08) A. Hydropathic □
A. Parabolic curve of oxygen association □ B. Acts as a zwitter ion □
B. Co-operative effect of combine O 2 □ C. Amphipathic □
C. Cooperative index of 81 □ D. All □
D. Hill's coefficient of 1 □ 131.All are true except- (NBE P 15)
122.Which of the following statement characterize both A. Porphyrinogens are coloured □
Hemoglobin and Myoblogin: (PGI 03)
B. Porphyrins emit flurorescence by UV light □
A. Non-helical □
C. Pyrrole rings of porphyrins are joind by methenyl
B. Subunits which are held together by hydrogen
bridges □
bonds. □
D. Pyrrole rings of porphyrinogens are joind by
C. Binds with 2 Heme □
methylene bridge □
D. Heme at hydrophobic pockets □
132.W hich of the follow ing is lum inous (coloured): (AI 11)
E. Bind with 1 molecule of O 2 □
A. Porphyrin (NBE P 13) D
123.W hich is an allosteric protein: (AIIMS 05)
B. Zymogen □
A. Transferrin □
C. Chromatin □
B. Ceruloplasmin □
D. Albumin □
C. Phosphofructokinase □
133.Prophobilinogen in urine produces pink colour with:
D. Haemoglobin □
A. Fouchet's reagent (AI 03) □
124.T rue regarding conversion of deoxyhem oglobin to
oxyhem oglobin is : (AIIMS 02) B. Benedict's reagent □
A. Binding of O 2 causes release of H+ □ C. Sodium nitropruside □
B. One mole of deoxyhemoglobin binds two moles of D. Ehrlich's aldehyde reagent □
2,3 DPG □ 134.All of the follow ing types of porphyrias are autosom al
C. pH of blood has no affect on the binding of O 2 □ dom inant except: (AI 12)
D. Binding of O 2 causes increased binding of 2,3,DPG □ A. Acute intermittent porphyria □
125.Decreased glycolytic activity im pairs oxygen transport B. Porphyria cutanea tarda □
by hem oglobin due to: (AI 03) C. CEP (Congenital Erythropoietic Porphyria) □
A. Reduced energy production □ D. Hereditary coproporphyria □
B. Decreased production of 2,3-bisphosphoglycerate □ 135.The enzym e deficient in erythropoietic porphyria is:
C. Reduced synthesis of hemoglobin □ A. PBG deaminase (PGI 04) □
D. Low levels of oxygen □ B. Uroporphyrin II cosynthetase □
126.Bilirubin is absent in urine because it is: C. Coprophyrin □
A. Distributed in the body fat (AIIMS 09) □ D. Ferrochelatase □
B. Conjugated with glucoronide □ 136.H epatom egaly is an essential feature of A/E
C. Not filterable □ A. Von Gierke's disease (AIIMS 10) □
D. Lipophilic. □ B. Nieman Pick disease □
127.Pigment providing colour to stool is: (PGI 07) C. Hurler syndrome □
A. Stercobilinogen □ D. Hepatic porphyria □
B. Urobilinogen □ 137.Heme synthesis requires (PGI 02)
C. Mevobilirubin □ A. Ferrous iron □
D. Bilirubin □ B. Glycine □
128.The conjugation of bilirubin to glucuronic acid in the C. Alanine □
liver; (AIIMS 03) D. Succhyl CoA □
A. Converts a hydrophilic compound to a hydrophobic E. Lead □
560 ■ A Complete Review of Short Subjects

ANSWERS & EXPLANATIONS:

Fate of Amino Group


1. A i.e. Transaminases [Ref: Lehninger 5/e p 677-83; Lippincott 5/e p 250-53; Harper 29Ie p 274-78; Chatterjea 7/e p 443-50;
Vasudevan 6/ep 174-76]

Transaminase (Amino-transferases) transfer an a-am ino (NHa+) group from L amino acid to an alpha keto acidQ.
$
2. D i.e. Alanine & a- ketoglutarate 3. B i.e. a- ketoglutarate

Transamination of pyruvate and glutamic acid (or glutamate) 1/ 1 formation of alanine and a-keto (oxo) glutarateQ.

4. B i.e. Glutaminase 5. A i.e. Alanine

The amino acid used primarily by muscle, for transport of ammonia to liver is alanine. Most other tissues however use
glutamine

6. A i.e. NH 3 transporter 7. B i.e. Alphaketoglutarate 8. D i .e . Less of NH3 9. B i.e. Glutamine


[Ref: Lippincott's 3/e P-254, 255; Harper's 27je P-247-250]

Glutamine is the sole form of transport of ammonia in brainQ.


Glutamine is an amide of glutamic acid which is formed from alpha - ketoglutarate. (Product of TCA cycle)Q.
In metabolic alkalosis, there is decreased ammonia excretion^ in urine.

Transport of Ammonia to Liver

Ammonia is transported to Liver from peripheral tissue for its ultimate conversion to urea. Two mechanisms are available
in humans for this transport _____________________________ | _____________________________

M e c h a n i s m f o u n d i n M o s t tissu e M e c h a n is m in M u scle
I I
U ses G lu ta m in e 0 U ses A la n in e 0

Ammonia Transport

E n te rs U re a C y cle A s G lu ta m in e 0 M etabolism of Ammonia


M o s t im p o rta n t ro u t of P r o v i d e a n o n to x ic sto ra g e &
d isp o sa l tra n sp o rt fo rm o f N H i° T C A C y c le
F o r m a t i o n o f u r e a in l i v e r & M a jo r m e ch a n is m for re m o v a l of
th e n e x c re te d in u r i n e f r o m NH3 in b ra in 0 . a -K e t o g lu ta r a te
k id n e y F o rm a tio n o f g lu ta m in e fro m
T ra n sam in ase
A p o r tio n o f u r e a d if f u s e s g lu ta m a te & N H 3 o c c u rs p rim a rily
fro m b lo o d in to in te s tin e & in m u s c l e & L i v e r
is c l e a v e d to N H i & C O 2 b y G l u t a m i n e is a n a m i d e o f g l u t a m i c G lu ta m a te
ATP
b a c te ria l u re a s e a c i d ( g l u t a m a t e ) w h i c h is form ed
G lu ta m in e
fro m a - K e to g lu ta ra te 0 . I . I s y n th e t a s e
I G lu ta m in e | 1
G lu ta m in a s e
Ammonia production (NLLp ion) from renal amino acid (eg
glutamine) increase in metabolic acidosis and decrease in ---------
G lu ta m a t e
metabolic alkalosisQ. [To maintain acid- base balance]
U r in e < = [ U r e a
Biochemistry: Metabolism of Proteins and Amino Acids ■ 561

10. B i.e. Ligase 11. A i.e. Glutaminase 12. A i.e. Glutamine


[Ref: Harper's 28/e p. 242-46; Lippincott's 4/e p. 250-53; Ganong 22/e p. 721; Chatterjea 7/e p. 116]

- Glutamine synthase is a mitochondrial enzyme of


ligaseQ class. Ligase enzymes are involved in joining ATP A D P +Pi
G lutam in e
together two substrates. L - Glutamate synthetase
- In renal tubular cells (kidney), ammonia (NH3) is + IL - G lu t a m in e
produced by hydrolysis o f glutmine (by enzyme A m o n ia ( N H 3 )
G lutam in ase
glutaminase)Q. Secretion of NPLi+ in urine maintains
acid base balance and conserve cation.

13. C i.e. Glutamate dehydrogenase

- Oxidative deamination is catalyzed by D & L-aminoacid L - G l u t a m a ^ ll .- G n r i_^ a-ketoglutarate + NH3


oxidase.
- But oxidative deamination of L-Glutamate is done by L- NAD(P)+ NAD(P)H+ + H+
Glutamate dehydrogenase (GDH)Q.

Urea Cycle

14. B i.e. Ornithine [Ref: Harper's 29/e p 267, 274-78; Chattargea 8/e p 476-77]
15. A, C, D, E i.e. Glutamate dehydrogenase; alpha-KG dehydrogenase; Isocitrate dehydrogemase; Fumarase .

' ---------------------------------------------------------------------
- a-KG dehydrogenase, isocitrate dehydrogenase and fumarase are enzymes of citric acid (TCA) cycle.
- L- glutamate dehydrogenase, deaminates L-glutamic acid (glutamate) to a-ketoglutaric acid (a-keto glutarate) and NH3.
- Enzymes of urea cycle include carbamoyl phosphate synthase I (rate lim iting = pacemaker enzyme)Q, L-om ithine
transcarbamoylase, arginosuccinate synthase (defect L/t citrullinem ia type 1), arginosuccinate lyase (defect 1/t
arginosuccinic acidemia), argin ase, N-acetylglutamate synthase, mitochondrial membrane ornithine permease
(encoded by ORNT1 gene is mitochondrial omithine carrier and deficiency results in HHH syndrome ie
hyperomithinemia, hyperammonemia, and homocitrulinuria syndrome) and mitochondrial aspartate /glutamate carrier
enzyme CTLN2 (defect 1/1 citrullinemia type 2).

M IT O C H O N D R IA CYTOSOL
N il, + C O ,

A cetyl N-acctyl 2A TP C a rb am o yl P 0 4
C oA glutamate N-acetyl glutamate sy n th a se I
synthase (Allosteric activator)
- M ito c h o n d ria l
G lu ta m a te
2A D P + -^ - Hale limiting (pacem aker)
enzyme V
G lu ta m in a se
C a rb a m o y l P 0 4
O rn ith in e
(fo rm ed in cy to so l &
G lu ta m in e
e n te r m ito c h o n d ria ) Arginase Q
(Extensively in Liver)

O rn ith in e-carb am o y l (C y to so lic )


transferase/
L -o rn lth in c- tran scarb am o ylase Kidney & Intestine
Brain lacks
lack arginase
this enzyme

I,-A rg in in e
C itru llin
(fo rm ed in m ito c h o n d ria
& tra n sp o rte d to cy to so l) A rg in o su c cin a s e
(Cytosolic)

Arginosuccinic
(Cytosolic)
acid synthase
J ru m ara.se
A rg in o -s u c cin a le M ala te

Aspartate-Argininosucdnate Shunt
A sp a r ta te ^__ ()A A M a la te
d e h y d ro ge n ase

Sources of Carbon & Nitrogen in Urea (NH2CONH 2) NH: • from N H 44


Mn: "O CC-A SA " i.e. 0 = (c
NH, ■ Ironi A s p n r ta tc
Omithine + C arb am o y l —> Citrullin —>Argino-Succinate -»
Arginine Q
from H CCT,
562 A Complete Review of Short Subjects

16. A i.e. Liver 17. C, D i.e. Arginosuccinate syn th etase, Ornithine transcaroxylase

Urea cycle occurs in liverQ and involve carbamoyl PO 4 synthetase 1 (rate limiting /pace maker mitochondrial enzyme),
ornithine trans carbamoylase (brain lacks this), argininosuccinate synthetase, argininosuccinase and arginase enzymes
(deficient in kidney & intestine and produce urea)Q.

18. C i.e. Arginase 19. D i.e. Brain [Ref. Nelson 16/e P -1214,1246; Harper's 27/e P-100-101]
20 . B i.e. First 2 steps in mitochondria; C i.e Defect of enzyme of any step can cause deficiency disease;
E i.e. Citruline is formed by com bination of carbom oyl phosphate and L. Ornithine
21 . A i.e. Aspartate and amm onia 22. C i.e. N-acetyl glutam ate is allosteric inhibitor 23. A i.e Reye's syndrome
[Ref: Lehninger 5/e p. 683-85; Harper 28/e p. 243-46; Lippincott 5/e p. 253-57; Vasudevan 6/e p. 177-79; Chatterjea 7/e p. 448-50]

Urea cycle defect (only hyperammonemia seen, no lactic acidosis, no convulsions) & Phenylketonuria can be easily ruled
out. There is confusion between MELAS & Reye's syndrome.
___________________I_____________________
r I
MELAS (Mitochondrial Reye's Syndrome
Encephalopathy, Lactic (Syndrome of acute encephalopathy and
Acidosis & Stroke like fatty degeneration o f liver)
__________episodes)__________
Children with MELAS may be No known etilogical factor has been proved. Aspirin or salicylates used for
normal for the first several certain viral respiratory infections might precipitate Reye's syndrome.
years, but they gradually The m ajor site of injury is the mitochondria. The activities of hepatic
display delayed motor and intramitochondrial enzymes, involving Ornithine transcarbamylase(OTC),
cognitive development and Carbamoylphosphate synthatase(CPS) and pyruvate dehydrogenase are
short stature. The clinical reduced, often to less than half of their normal values.
syndrom e is characterized HyperammonemiaQ may result from decreased activities of OTC and CPS.
by- Lactic acidosisQ results from decreased activity of pyruvate dehydrogenase.
• Recurrent stroke like
Clinical fe a tu re s: Laboratory fe a tu re s:
episodes manifesting as
- Classical Reye syndrome exhibits a - CSF- is normal except for elevated
hemiparesis, hemianopia
stereotypic, biphasic course. It pressure
or cortical blindness.
usually occurs in a previously - Hyperammonemia
• Lactic acidosis, ragged red
healthy child. - Respiratory and metabolic acidosis
fibres or both
- A prodromal febrile illness, an - Features of hepatic failure
• At least two of the
upper respiratory tract infections or - Increase AST, Increase ALT,
following:
chicken pox is followed by an Increase LDH, Increase PT
focal or generalized
interval in which the child has - The striking and characteristic gross
seizures
seemingly recovered. pathological feature of Reye
dementia
- 5-7 days later there is abrupt onset syndrom e is a yellow to white liver,
recurrent migraine
of symptoms. reflective of high content of
headaches
- Vomitting, irrational behaviour, triglycerides. There is
vomitting
seizures, stupor & coma. m icrovescicular fatty steatosis*of
• Full expression of disease
- Jaundice is absent liver, kidney and brain.
leads to dementia, a
- Brain shows gross cerebral edema.
bedridden state, and death
often before age 20 .

24. B i.e. a-ketoglutarate [Ref: Chatterjea 7/e p. 447-48; Vasudevan 6/e p. 180]

Hyper ammonaemia inhibit TCA cycle by depleting a-keto glutarateQ

Amino Acids

25. D i.e. Disulfide bond between cysteine m olecule 26. C i.e. Form yl-m ethionine 27. A i.e. Acetyl coA & alanine
Biochemistry: Metabolism of Proteins and Amino Acids ■ 563

Amino Acids: Summary

Glycine Histidine Glutamine


- Simplest^ a.a - Most stable a.a at physiological - Storage and transport form of
- Optically inactive® pH® ammonia®.
- Lacks chirality® - Acts as buffer at pH 7® - Major pathway for removal of NHi in
- Synthesize Creatine® (with arginine - Protonate & deprotonate at brain
& methionine) neutral pH® - Precursor of purines and pyrimidines
- Used in heme synthesis®. - Precursor of histamine®.

Phenylalanine /Tyrosine Cysteine M ethionine


• Phenylalanine is Sulphur containing amino - Sulphur containing a.a
precursor of tyrosine acid - Methionine form S-adenosylm ethionine (SAM), major
• Tryrosine is a precursor Two cysteine join by methyl group donor in bodyQ
of: disulphide bond to form - Precursor of hom ocysteine^- metabolite associated with
- Catecholamines^ cystineQ atherosclerotic vascular disease^.
- ThyroxineQ Can be synthesize in body - In eukaryotes, initial aminoacid in protein synthesis (or
- melaninQ from methionine. aa carried by initiator tRNA) is methionine. Whereas, in
Responsible for reducing prokaryotes & in mitochondria it is N-formyl
action of glulathioneQ methionine (i.e. m ethionine modified by
transformylase or Formyl transferase.)

Tryptophan Arginine Alanine


- Precursor of niacin and serotonin® - Member of urea cycle M ost basic amino acid 0
- Serotonin form melatonin - Precursor o f nitric Transport form of ammonia from
- 60mg tryptophan give rise to lm g Niacin®. oxide ® muscle.
- Also kla a - amino /} -3 indole propionic acid®
- Precursor of Vitamin niacin and serotonin
- Tryptophan is both glucogenic (by producing
alanine) & ketogenic (by producing acetyl CoA)®

28. D i.e. Presence of Ser-His-Asp catalytic triad at the active site:


29. A i.e. Kidney; Arginine + Glycine 30. A i.e. Glycine, arginine and methionine
[Ref: Harper's 27/e P- 274 - 77; Lippincott's 4/e p. 287- 88]

- A common feature o f all serine proteases is presence o f Ser-His-Asp catalytic triad at the active site.
- Glycine, arginine and methionine all participate in creatine biosynthesisQ. M n- "G A M "

Serine Protease

These are protein digesting enzymes. They contain serine residue at the active centerQ
They form charge relay network with histidine and asparate. This charge relay network functions as proton shuttle. This
proton shuttle donate a proton to serine & activate it. The activated serine breaks down peptide. Serine proteases include
- Trypsin °, E lastase c, Thrombin, Chymotrypsin 0

B i.e. Alanine 32. D i.e. Histidine [Ref: Lippincott's 3/e P-261 [

Methionine, Valine, Isoleudne and Threonine are the four aminoacids that form succinyl- CoA. Histidine forms a
ketoglutarate and not Succinyl CoA.

33. C i.e. Valine [Ref: Harper's 27/e P-15]

Isoleucine is an aminoacid with an aliphatic side chain.


Amongst the options provided valine is the only other amino acid with an aliphatic side chain and is the answ er of
choice here. Substitution of isoleucine with valine in a protein sequence is an example of a homologous or
conservative substitution.

34. B i.e. Negatively Charged 35. D i.e. Point at which positive and negative charges on the amino acids are equal.
36. A i.e. Have net charge 'O ' [Ref: Lippincott's 3/e P - l]

At physiological pH (7.4), all amino acids have both a negatively charged carboxyl ion (COO)Q and a positively
charged ammonium ion (-NHs+). The amino acids are therefore dipolar ion (Zwitter ionsQ)
564 ■ A Complete Review of Short Subjects

Zwitter Ion Isoelectric Point


Molecules that contain an equal number of ionizable groups The pH at which an amino acid bears no net
of opposite charge and therefore bear no net charge. chargeQ i.e. where the sum of the positive charges
Carboxyl and aminogroup of amino acid can protonate & equals the sum of negative charges^.
deprotonate depending on the pH of surrounding medium.
Thus the net charge depends on surrounding pH__________

37. D i.e. Proline 38. C i.e. M ethionine & Cysteine 39. D i.e. Tryptophan
40. B i.e. Aromatic amino acid 41. C i.e. Lysine 42. A, B, C, D i.e. M ethione, Isoleucine, Tyrosine, Alanine
43. A, C, E i.e. Serine, Tyrosine, Lysine 44. A, B, C > E i.e. Alanine, Tryptophan, Isoleucine > Tyrosine
45. A i.e. Methyl 46. A i.e. Nonpolar side chain 47. D i.e. M ethionine 48. A i.e. Leucine
Ref: Lehninger 5lh/e p.73-76; Lippincott 5lh/e p. 2-9; Harper 30,h/e p.15-21; Vasudevan 6/e p. 20-23; Chatterjea 7/e p. 74-761

- Proline is a unique amino acid and has an immino groupQ (=NH) instead of an amino (NH2) group found in other
amino acid.
- M ethionine & Cysteine are sulfur containing aminoacidsQ. M ethionine is nonpolar (hydrophobic) aa<2.
- The property of photochromicity (i.e. absorbance of ultraviolet light at 250-290nm esp 280nm) is seen with aromatic
amino acid (tryptophan > tyrosine > phenylalanine).
- Hydrophobic (non polar) am inoacids h a v e n o c h a rg e o n th eir R g ro u p o r sid e ch a in . Aliphatic (eg methyl, methylene,
thioether & imino) side chains a n d a ro m a tic sid e ch a in s a re n o n p o la r. So methyl (CH3) side chain of alanine; p ro p y l
( C 3H 7) sid e ch ain o f valine; b u ty l ( C 4H 9) sid e ch a in o f leuicine & isoleucine; th io e th e r sid e ch a in o f m e th io n in e ; a n d
im ino group / pyrrolidine containing side chain of prolineQ a re n o n p o la r.
- Methyl (CH3) side chain of alanine is nonpolarQ. Serine, threonine, tyrosine containing hydroxyl group and cysteine
containing sulfhydryl group, are polar aminoacids with neutral/uncharged/nonionic side chain. Positively charged
basic antino (NH 3 ) group side chain of histidine, arginine and lysine ; and negatively acidic carboxyl (COO) side
chain of aspartic acid and glutamic acid is polar.

49. C i.e. Glycine 50. A i.e. Asparagine; C i.e. Glutamine

(L-a)- Amino acid Classification (Based on Side Chain)

Polarity and Solubility Side Chain Content


1 I 1
Hydrophobic (Nonpolar) Hydrophillic Aliphatic Hydroxyl (OH) group
Neutral aa ( L e h n i n g e r ) (Polar) aa - Glycine - Tyrosine
- Alanine - Threonine
T Tryptophan* • Uncharged (Neutral)
- Valine - Serine
Tyrosine** - SerineQ - Aspargine
- Leucine Mn: "OH! Try Three sari"
M Methionine - Threonine - Cysteine
- Isoleucine
L Leucine - TyrosineQ - Glutamine Sulfur atoms
- Proline
V Valine - Cysteine
• Acidic (Negatively charged - - Methionine
I Isoleucine - Methionine
Proton donors)
A Alanine Mn: "M eet Sulphuric Cyst"
Mn: "Acidic as G lue" Mn: 'GAVLI-PM '
P Phenylalanine - Aspartic acid Immino acid/ (Secondary
Proline - Glutamic acid Aromatic amino group)
G Glycine* - Asparagine* - Tyrosine ProineQ
- G lutam ine* - Tryptophan Basic group
M n - " Try To Meet Lucy - ArginineQ
- Phenylalanine
Valley Is All Phen's • Basic (Positively charged - - LysineQ
- Histidine
problem - G " or "TAM IL Proton acceptor) - HistidineQ
VAP-G" Mn: "H is basics are loose"
Mn: "Try To Pen His Acidic Group
However in aromatic side - Histidine (weakly basic or - Aspartic acid (aspartate) Q
Arrow"
chain aa tyrosine > neutral) * - Glutamic acid (glutamate) Q
tryptophan are more polar - Hydroxy lysine Amides of acidic group
than phenylalanine. ★ = - Arginine (most basic)Q - Asparagine
Relative - LysineQ - Glutamine

51. C i.e. Lysine and arginine 52. B i.e. Arginine


53. B i.e. Proline 54. C i.e. Lysine
Biochemistry: Metabolism of Proteins and Amino Acids ■ 565

- At physiological pH basic aminoacid (positive charged a.a) will move slowest towards anodeQ (positively charged
electrode). Basic amino acids are: ArginineB (Most basic), Lysine<2and Histidine 0 (Weekly basic).
- Acidic amino acids include aspartic acid (aspartate) & glutamic acid (glutamate)Q. Others are neutral aa.

55. A i.e. Tyrosine 56. A i.e. L- methionine 57. A i.e. Phenyalanine


58. C i.e. Arginine 59. D i.e. Glycine 60. D i.e. All
61. A i.e. Arginine B i.e. Histidine 62. D i.e. Serine 63. C i.e. Alanine
[Ref: John Baynes 3je p. 248; Lehinger 5/e p. 686]

Threonine. Lysine, Phenylalanine, Methionine, Isolcucine & Leucine, Tryptophan. Valine > Histidine » Arginine are
nutritionally essential amino acids. Among these last two (histidine & arginine) are semi- essential, and arginine is more
essential than histidine.

The Likely M Is Trying Very H- Ard (H Ard)


Mn
Threonine Lysine Phenyl Methionine Isoleucine & Tryptophan Valine Histidine Arginine
alanine Leucine

Classification of Amino Acids on the Basis of Metabolism

Essential Amino Add Ketogen Glucogenic Both Ketogen G lucogenic G lucogenic

Lu- Leucine Vali- Valine


icA . A A. A. Glucogen ic &

Ly- Lysine Iso- Isoleucine


ic& 13 K etogenic
■J3
T h i-
Ketogeni G Leucine Isoleucine M eth ion ine
Threonine Phenyl- Phenylalanine 01
c A.A. CA Lysine Phenyl T hreonine
Her- Histidine) S em i- Metho- M ethionine ifl
Leucine V - Valine Isoleucin W alanine V aline
A rg i- A rgininei Trypto- Tryptophan
Lysine C / see - Cysteine, e T ryptopha Histidine*
e ss en tia l^
Serine Tyrosine n
IMn: 'LuLi Thi His Argi V ali Iso Phenyl M etho t
All - Alanine Tryptoph - - Histidine
Trypto'] or "Please Visit To TIM H A LL" ■3 *
His - Histidine an E 5 Arginine
Three Phenylal to 1
- Threonine u.
Conditionally Essential
Asses - Aspartate anine H ydroxy T yrosine A rginine*
Required to some degree in young growing animals Are - Arginine -lysine Alanine
& during illness Grass - Glycine, A sparagine
- Arginine - Glutamine - Proline Glutam ate 13 A spartate
- Cysteine - Glycine - Tyrosine Prone - Proline / G C ysteine
V
H ydroxy CA
CA
G lutam ate
Non essential Amino Adds proline U G lutam ine

- Alanine - Aspartate - Serine


G G lycine
In Lehninger, H istidine is essential and 0
2 Proline,
- Asparagine - Glutamine A rginine is conditionally essential.
hydroxyproli
ne
Serine
64. C i.e. Tryptophan
Most amino acid do not absorb visible light & are thus colourless. The only amino acids that may absorb light are
Aromatic amino acids i.e. Tryptophan, Tyrosine, Phenylalanine & Histidine®. Mn: "Try To Pen His Arrow".
Aromatic aminoacids absorb ultraviolet lights (250- 290 nm). Tryptophan makes 'major contribution to ability of most
proteins to absorb light in the region of 280 nm.

65. C i.e. Leucine [Ref: Lippincotjfs3/c P-3- 41

Hydrophobhic or Non polar Cell membrane


aminoacid are present in
Cell membrane is a complex structure composed of lipid, proteins and carbohydrate.
transmembrane region®.
Membrane lipids form bilayer. The unique characteristic of this layer is that it has its
Hydrophillic or polar amino
hydrophobic ends in the centre of membrane while its hydrophilic ends lie on the
adds are present on outer
outer surface of membrane.
surface of membrane.
Membrane proteins are associated with lipid bilayer. So
Leucine is a hydrophobic a.a
The hydrophilic amino acids of the protein should protrude at the inside & outside
so it lies in transmembrane
faces of membrane.
region®___________________
While hydrophobhic a.a should lie in the centre (transmembrane region)®.
566 ■ A Complete Review of Short Subjects

66. A i.e. L-form s [Ref: Harper's 28/e P-14; Chatterjea 6 /e P-25] 67. C i.e. Racemization

Natural protein of animate anglplants generally contain L-amino acidsQ. D- amino add occur in bacteria.
Racemic or DL mixture: Wheh ajiud amount of D & L isomers are present^, the resulting mixture has no optical activity
and called Racemic mixture. '
Asymmetric Carbon: A carbon atom to which 4 different atoms or group of atoms are attached. Glycine lacks
asymmetrical carbon atom so does not exhibit isomerismQ.

Isomerism
Two types of isomerism are shown by a.a, because of the presence of asymmetrical carbon atom.

Compound which are identical in composition and Optical isomers or 'enantiomorphs' rotate the plane
differ only in spatial configuration are called 'Stereo polarized light and exist as either dextro rotatory (d)
isomers' and laevorotatory (L) forms.
All aminoacids except GlycineQ exist in D & L isomers All amino acids except glycine have optical activityQ

68. B i.e. Threonine 69. D i.e. Glycine 70. D i.e. Glycine

All aminoacids have one chiral (or asymmetric) a carbonQ which has 4 different groups attached to it except

Glycine Aminoacid which lacks chirality or does not have anomeric (asymmetric/chiral)
carbon and so is optically inactive.
Threonine & Isoleucine Aminoacid with 2 chiral centers (with double chirality)Q.

Glycine

- Glycine does not posses asymmetric carbon - Smallest^ and simplestQ amino acid
atom andistherefore optically Inactive i.e. - Amino acid responsible for flexibility of proteinsQ.
lacks chirality (handednessM*. - Is optically inactive^, does not exist in isomeric form.
- The a - Carbon of each a.a is attached to four - It lacks chiralityQ.
different chemical groups & |s therefore chiral or - Glycine with arginine & methionine (GAM) synthesize
optically active. Glycine is ah exception because creatineQ.
its a - carbon has two Hydrogen atoms & - Glycine (with succinyl COA) is used for Heme synthesisQ.
therefore is optically inactive: ________________

71. A i.e. Methylene THF 72. A i.e. Folic Acid 73. B i.e. Folic Acid

Tetrahydrofolic add and S- adenosyl methionine act as a carrier compound for single carbon unitsQ. Methylene
THF is major form of folic add to'transfer one carbonQ.
J
Source of 1 carbon unit Synthesis using 1 carbon units
One Carbon Unite
insF-1
- One carbon units can exist in form of P y r u v a te

methane, methanol, form aldehyde, form ic


acid and carbonic acid, all of which except
methane can be incorporated in various N 5N ‘° m e t h y l e n e
TH F
synthetic pathways. M e t h y l- T H F —> M e th io n in e
- Sources of one carbon unit are serine,
glycine, choline, histidine, and form ated c c >2 + n h T M P + D ih y d r o fo la te

- Tetrahydrofolate and S-adenosyl X nwA


methionine are carrier o f one carbon
units Q. The active form of folic acid, tetra
hydrofolic acid (THE) can carry carbon N- Form im inoglu tam ate . G lu ta m a te

unit bound to nitrogen N 5 or N10 or to (F I G lu )


I
both N 5 and N 10 in form of formyl, -k e to g lu ta r a te
F o r m y l - m e th io n in e

formimino, or methyl (attached to N 5 ) ,


iFormyl- THF] P u rin e
methylene, methenyl group (bridging N5
& N10) and formyl (attached to N10).
Biotin functions as coenzyme to transfer CO 2 in carboxylation reactions (enzyme carboxylase). But CO 2 is not
considered as member of 1 carbon pool.
Biochemistry: Metabolism of Proteins and Amino Acids ■ 567

74 . A i.e. Folic acid

Glycine is converted to serine by addition of methylene group from N5, N10 - methylenetetra hydrofolic add® and
requires pyridoxal phosphate as coenzyme for serine hydroxymethyl transferase®.

75. B i .e . H i s t i d i n e

Histidine

- Most stable amino add at Histidine is unique, since pKa of its imidazole group permits it at pH7
physiological pH® (physiological) to function either as a base or as an add catalyst i.e. it has
- Can serve as best buffer at capadty to protonate or deprotonate at neutral pH®.
pH 7® Minor change in pH, changes the ionization charge on histidine to buffur pH
- Can protonate & change®. Therefore it can serve as a buffer & is most stable amino add at
deprotonate at neutral PH® physiological pH®._____________________________________________________

76. A i.e. Contain sulfhydral group ; D i.e. Transport aminoacid across cell membrane; E i.e. Part of enzymes
77. C i.e. Cysteine [Ref: Harper's 27/e P-636; Lippincott's 3/e P-4, 5,146; Harper's 27/e P-17-18; Lippincott's 3/e P- 4, 5]
78. C i.e. It helps in membrane transport, D i.e. It helps in conjugation reaction

— ^
Glutathione acts as a reducing agent in body and its cysteine residue is responsible for its reducing property®.
Glutathione (GSH) is a tripeptide® of three amino acids: [Mn:GGC]: Glycine, Cysteine (Provides SH group), Glutamate

79. B i.e. Histidine 80. D i.e. Cystine, Ornithine, Lysine, Arginin 8 1 . B i.e. Tryptophan 8 2 . A i.e. Non essential A.A.
83. B i.e. Tryptophan 8 4 . D i.e. Tyrosine 85. C i.e. None 86. A i.e. Tryptophan
87. B i.e. Tyrosine 88. B i.e. M elatonin 89. C i.e. Glutamine 90. B i.e. Glutamine
91. A i.e. Glutamine 9 2 . D i.e. M elatonin [Ref: H arper's 27/e P-259-62; 26/e P- 2 5 7 ; Lippincott's 5/e p. 250; Chatterjea 7/e p. 477]

~~ Epinephrine®
/ Catecholamine ---------------------------- Norephinephrine
• Phenylalanine -4 Tyrosine® — ------- Thyroxine® Dopamine
\N___ Triodothyronine
’----- Melanin®

• Melatonin is formed from tryptophan®. Tryptophan —» serotonin® —> melatonin®


i
Niacin®

93. B i.e. Homocytine 94. C i.e. Calcium [Ref: Harrison 16/e P. 1430,1432/

- There is a strong Co- relation between hyperhomocytinemia and coronary events (MI)®.
- Fortification of food with folic add to reduce NTD has lowered homocystine level in population.

95. A i.e. Limiting the substrate for deficient enzyme 96. C i.e. Dihydropterin reductase
97. C i.e. Phenylketonuria 98. B i.e. Green 99. B i.e. Phenylketonuria
[Ref: Lippincott's 5/e P- 270-71;Lehninger 5/e p. 696-99; Chatterjea 7/e p. 469-70; Harper's 28/e P-254-55, 237; Harrison 17/e p.
2471-70; Nelson 18/e p. 530] .

Phenylketonuria is caused by defidency of phenylalanine hydroxylase & /or dihydrobiopterin (BH2) reductase, which
regenerates BH4 from BH2®. It presents with mousy (musty) odour in urine and emerald green colour in FeCU test®.
The aim of first line therapy is to limit the substrate (i.e. phenylalanine) for defident enzyme®

100. D i.e. Hepatic carcinoma [Ref: Lehninger 5/e p.696-97; Chatterjea: 7/e P- 471; Harper's 28/e P-254; Lippincott 5/e p. 269;
Vasudevan 6/e p. 209-10]

In Tyrosinemia type I, h e p a t o c e l l u la r c a rc in o m a ® occurs in children who survive beyond 2 yrs.

101. B i.e. Alkaptonuria 102. B i.e. Alkaptonuria


[Ref: Lehninger 5/e 694-698; Vasudeven 6/e 208-10; Chatterjae 7/e 470-71; Harper 28/e 254; Lippincott 5/e 269, 274; Harrison 17/e
2470-73; Nelson 18/e p 534; Weedon skin pathology 2/e p 440]
568 ■ A Complete Review of Short Subjects

Alkaptonuria a condition d/t defective homogentisate 1 ,2 - dioxygenaseQ (1/1 defective tyrosine degradation) usually * '''
presents after age of 40Q with characteristic triad of Homogentisic aciduria (which blacken/darkens on exposure to air
/standing and is strongly Benedict's positive), Ochronosis (i.e black pigmentation of cartilage & collagenous
connective tissue), and Arthritis of large jointsQ, (Mn - "Alkaptonuria - Black-HOA = p n")

103. A i.e. Diarrhoea; B i.e. Vasoconstriction; C i.e. Flushing [Ref: Chatterjea: 6/e P- 451; Harper's 27/e P-262]

• Trytophan is an essential amino acid and is precursor of Serotonin, Melatonin & Niacin.
• Clinical effects of Serotonin or 5Htase: Symptoms occur due to effect on smooth muscle.
- Cutaneous vasomotor episodes of flushingQ.
Potent vasoconstriction^.
Chronic diarrhoeaQ
Respiratory distress & bronchospasm.
• Albinism accompanies defective melanine, product of tyrosine biosynthesis

104. D i.e. a, b & c 105. C i.e. Leucine 106. A i.e. Maple Syrup urine disease
107. B i.e. Maple syrup disease; D i.e. Isovaleric acidemia [Ref: Harper 29/e p. 284, 293-95; Lippincott6/e p. 266; Vasudevan 7/e
p.230; Chatterjea 7/e p. 493; Satyanarayan 3/e p. 376; Lehninger 5/e p. 694-701 ]

Diseases of branched chain aminoacids include (1) maple syrup (burnt sugar) urine disease or branched chain
ketonuria (BCKU) and interm ittent BCKU (less severe) both due to defect of branched chain a ketoacid
dehydrogenase^ resulting in defective oxidative decarboxylation of leucine, isoleucine & valineQ; (2) Isovaleric
acidemia d/t defective isovaleryl CoA dehydrogenase^ and; (3) Hypervalinemia d/t defective valine transaminase.

108. A i.e. Decarboxylase 109. B i.e. FAD linked dehydrogenase


110. A i.e. Phenylketonuria-Mousy odour; B i.e. Tyrosinemia-Rotten cabbage

Sweaty feet odor in body fluids a/w neurological problems occur in deficiency o f FAD linked dehydrogenase specific
for isovaleryl COAQ.

M etabolic Disease Enzyme D eficient Urinary Feature


Maple syrup urine disease Branched chain ketoacid Sweat maple syrup (burnt sugar)
dehydrogenase^ odourQ
Methyl-melonic aciduria Methyl-malonyl CoA mutase (isomerase)
Isovaleric academia Isovaleryl-CoA dehydrogenase Sweaty feet (cheesy) odor in body
fluids & breath
Tyrosinemia (Tyrosinosis) I Fumarylacetoacetate hydroxylase Boiled or rotten cabbage/mushroom
like odour
Tyrosinemia II Tyrosine transaminase (Tyrosine
aminotransferase)
Neonatal tyrosinemia Hydroxyphenyl pyruvate hydroxylase
Albinism Tyrosinase
Alkaptonuria Homogentisate oxidaseQ Urine turns black on air exposure or
standing
Phenylketonuria Phenylalanine hydroxylaseQ Mousy (musty) urine odourQ
Orotic aciduria OMP decarboxylase
Homocys tinuria Cystathionine synthatase
Lesh-Nyhan Syndrome Complete deficiency of enzyme HGPRT
Cystinosis Cystine reductase
Hawkinsinuria Swimming pool odouiQ

BCAA Disease Defective Enzyme


MSUD (BC Ketonuria) BCKA Dehydrogenase
Intermittent branched chain ketonuria BCKAD (less severe)
Isovaleric acidemia Isovaleryl CoA dehydrogenase
Hypervalinemia Valine transaminase

111. A i.e. Serine, B i.e. Tyrosine [Ref: Chatterjea 6/e P-446; Harper's 27/e P-78-791
Selective phosphorylation, specific for serine & tyrosine Q residue or threonine and subsequent dephosphorylation is
catalyzed by protein kinase & phosphatase.
In human, activity of many enzymes are regulated by covalent modification such as ATP dependent phosphorylation &
subsequent dephosphorylation.
Biochemistry: Metabolism of Proteins and Amino Acids ■ 569

112. C i.e. Valine »

Succinyl COA is formed by Valine C, Isoleucine, Methionine. [Mn: "Succesful V I M " ]

113. B i.e. M ultiple carboxylase deficiency [Ref: Nelson 16/e P-356]

In the degradation of valine, leucine, isoleucine (essential, branched chain aminoacids), the intermediate metabolites are all
'organic acids', and deficiency of any of the degradative enzymes (except for transaminases) causes acidosis due to
accumulation of organic acids before the enzymatic block. The patient in question has ketosis as well as skin manifestation
and thefore the answer is Multiple carboxylase deficiency. Normal levels of ammonia rule out urea cycle enzyme
deficiency.

Common Features
- Refusal of feed - Vomiting - Acidosis
- Dehydration - Neutropinia - Hypoglycemia

Ketosis Non I^etosis


I
No skin manifestation Skin manifestation Normal NHa level TNH s
I
M ultiple carboxylase deficiency^ Urea cycle defect
I
Propionic academia^ Charateristic odour - Acyl-CoA dehydrogenase deficiency
I - 3 hydroxy 3 methyl glutaric aciduria
- Methylmalonic academia - Maple syrup urine disease
- Keto-thiolase deficiency - Isovaleric academia

114. A i.e. Randomly (Most probable answer of exclusion )

Most variations in aminoacid sequence of cytochrom e c affect prim ary structure in a way not to affect the secondary and
tertiary structures which are more important for function. The amino acids that cannot be replaced without a major
modification of the enzymic properties of the cytochrome c are not altered. Thus the amino acids at locations that
determine the secondary structure of protein (loops, helix or strands) are preserved while aminoacids that do not
contribute to the determination of secondary and tertiary structure are preferentially altered. These alterations are likely
to be distributed randomly as they result from point mutations on the responsible gene and this is the single best answer
of exclusion here.

Also note that it is the tertiary structure which is most important for function and hence it is the most conserved
parameter. Some changes in the secondary structure (helix, loop or strands) m ay be seen as a result of random changes in
the polypeptide sequence of amino acids. These changes may affect the 'helical regions' or the 'loops' or the 'strands'
randomly and should not be confined to only the helical regions, the loops or the strands.

Hemoprotein, Hemoglobin, Porphyrins & Bile Pigment


115. A , B, C, D, E i.e. Cytochrom e C; Cytochrom e 450; M yoglobin; Hem oglobin; Catalase
[Ref: Harper 28/e 271-72; Chattarjae 7/e 82; Lippincott 5/e 25,34; Lehninger 84-85]

H em oproteins are a group of chromo proteins that Hemoproteins Function


contain heme as a tightly bound prosthetic group. And Hemoglobin Transport O 2 in blood
heme is a complex of protoporphyrin IX and ferrous M yoglobin Storage of O 2 in muscle
(Fe2+) ion. The iron is held in center by bonds to 4 Cytochrome C (oxidase) Electron transport chain
nitrogens of porphyrin ring. The heme iron can form two Cytochrome P - 450 Hydroxylation of
additional bonds. In hemoglobin & myoglobin one of xenobiotics
these is with histidine residue of globin and other is Catalase Degradation of H 2O 2
available to bind O 2. Tryptophan pyrrolase Oxidation of tryptophan
Per-oxidase Oxidative enzyme

116. A i.e. Histidine residue 117. A i.e. Hb has 2 polypeptide chain


[Ref: Harper's 27/e P- 41-45; 26/e P- 40; Chatterjea 6/e P-133-34]

Heamoglobin has 4 polypeptide chainsQ, two identical a - chains and two identical P chains.
Iron (Fe2+) o f Hb is bound to histidine & four pyrrole ringsQ.
570 ■ A Complete Review of Short Subjects

118. B i.e. His E 7 [Ref: Harper's 27fe P-44; 26/e P- 40]

119. A i.e. Each hemoglobin molecule is made of 4 polypeptides of each subunit 120. C i.e. Type III
121. B i.e. Co-operative effect of combine O 2 122. D i.e. Heme at hydrophobic pockets
[Ref: Harper's 27/e P- 42-44; Chatterjea 6/e P-134, Lippincott's 3/e P-26]

Heamoglobin

r
Iron Porphyrins • The Hb tetramer is arranged a
• Iron is in ferrous state (Fe2+)Q in • Is a tetra pyrrole structure two dimmers (a |3)i & (a p)2
functional Hb • Four pyrrole (I to IV) -> • The two polypeptide chains
• 4 gm of Fe2+ per mole of Hb joined by methylene or within each dimmer are held
• Iron can form six co- ordinated bondsQ M ethylodene bridge^ together primarily by
- Linked to four pyrrole rings through • Vinyl is attached to I and II hydrophobic interaction^.
their nitrogen atomQ. ringsQ. • Each polypeptide chain
- 5th linkage to nitrogen of imidazole ring • Propionic acid at 6 th & 7th contains a 'hem e' in heme
of histidine of polypetideQ. position of heme of III & IV PocketQ. Thus 1 Hb contains 4
- On other side co-ordinated position of Pyrroles heme units
Fe2+ is available to bind to O2 • Hb contains mostly
hydrophobic amino acids
• Within the pH range of 7.0 to 7.8,
internally (forming 'hem e
most of the physiological buffering Pyrrole N
ring pockets')Q & hydrophilic a.a on
action of Hb is due to imidazole C ----------CH = C
their surfaces.
group of amino acid histidineQ.
• Imidazole H+ group can give up H+
(Proton) and accept H+ depending
on pH of medium.
• Other dissociable buffering group
responsible for buffering capacity of
Hb
a) Acidic-COOH group
b) Basic-NH2 group
c) Guanidino group
Histidine
1 Globin

123. D i.e. Hemoglobin [Ref: Lehninger 3/e P-214, 278]


Hemoglobin is allosteric proteinQ and Phosphofructokinase is allosteric enzymeQ.

Results from its quaternary structures. Is one in which binding of a ligand to one site - Is one in which binding
There are four subunits in Hb & each affects the binding properties of another site on of a modulator to the
subunit can bind an O 2 molecule. The the same protein enzyme induces a
binding of O 2 to one subunit increases Allosteric proteins are those having 'other conformational change
the affinity of other subunits for O 2 shapes' or conformation induced by binding of in the enzyme making it
Hb binds O 2 with a sigmoid binding ligands referred to as modulators. more active or less
curveQ because of its allosteric property. The conformational changes induced by active.
(Myoglobin binds O 2 with a hyperbolic modulator inter convert more active & less - The same concept of
binding curve) active forms of protein. Modulator may be allosteric protein is
either inhibitor or activator applied on the enzymes.
Biochemistry: Metabolism of Proteins and Amino Acids ■ 571

124. A i.e. Binding of 0 2 causes of H+ [Ref: Harper's 27/e P - 42-45]

Binding o f 0 2 to deoxyhemoglobin in lungs causes release of proton (H+)Q. This is responsible for Bohr effect.

125. B i.e. Decreased production of 2 ,3 - biphosphoglycerate

2 ,3 - DPG is produced by an alternative pathway in 2 ,3 - Biphosphoglycerate Pathway in Erythrocytes


glycolysis. It is formed form 3-
Glucose - 6 - P O 4
phosphoglyceraldehyde, which is a product of
I
glycolysis via Embden- Meyerh of pathway^. It is
3- Phosphoglyceraldehy
very plentiful in red cells. If the glycolytic activity is
I
decreased, the level of 2 ,3 - DPG in RBCs will
decrease. Decreased level of 2 ,3 DPG
concentration:
- Shifts the 0 2-Hb curve to leffQ
- Increases the 0 2 affinity of HbQ (impairs O 2
transport).

Pyruvate

126. D i.e. Lipophilic 127. A i.e. Stercobilinogen [Ref: Lippincott's 3fe p 287, 281; Chatterjea 6/e p. 476, 478; Harper's 27je 155]

Unconjugated bilirubin is absent in Hb Spleen, liver


• Ineffective erythropoiesis
urine because it is highly lipid • O th e r sources Hem e
soluble (lipophilic)Q but conjugated 1
Biliverdin
bilirubin can be present in it. I think 1
that the question is about Bilirubin

unconjugated bilirubin as 90% ^ln blood


bilirubin in serum is unconjugated.
Bilirubin + albu m in
Brown color of stool is due to (u n c o n ju g a te d bilirubin)
Intestine
stercobilinQ.
Bilirubin • glucoronide

G lu co ro n id e is
Liver re m o ve d by
gut bacteria

Bilirubin
I
U robilinogen

O xid is ed by
bacteria

Stercobilinogen
(brown pigment of feces)

In u r i n e

( g iv in g its y e llo w c o lo r )

128. B i.e Converts a hydrophobic com pound to a hydrophilic one [Ref: Lippincott's 3/e P-281; Chatterjea 6/e P-478]

Liver uptakes unconjugated bilirubin (hydrophobic) & conjugates it with glucoronic add generating bilirubin
glucoronide which is water soluble^.

Bilirubin is produced in reticuloendothelial cells of spleen and liver.

Unconjugated bilirubin : Lipophilic®


Not soluble in water
- Liver uptakes unconjugated bilirubin &
Bound to albumin for its transfer in plasma
Conjugation with glucuronic acid
Conjugated bilirubin: - W ater soluble®
(Bilirubin glucuronide)
572 ■ A Complete Review of Short Subjects

129. A i.e. Able to cross cell membrane; C i.e. Hydrophobic to hydrophilic [Ref: Lippincott 4/e p. 282; Harper's 27/e p. 289]

- Bilirubin is only sparingly (slightly) soluble in water, but its solubility in plasma is increased by non covalent binding to
albumin. Each albumin molecule has one high & one low affinity site for bilirubin. Only 25 mg of bilirubin can be tightly
boud to albumin at its high affinity site in 100 ml plasma. The excess amount bound loosely & can be easily detached.
- Bilirubin enters hepatocytes via facilitated transport system and binds to cytosolic proteins (ligandin, protein Y), which
to keep it solubilized & prevent its efflux back into blood stream before conjugation.
- Bilirubin is nonpolar & would persist in cells if not rendered water solubleQ. Solubility of bilirubin is increased in
hepatocytes by addition of 2 molecules of glucuronic acid to convent it to a polar form. This process k/a conjugation is
catalyzed by microsomal bilirubin glucuronyl transferase and use uridine diphosphate - (UDP) - glucuronic acid as
glucuronate donor. /

- So conjugation of bilirubin makes them polar, water soluble, hydrophilic and able to cross cell membrane^

130. C i.e. Am phipathic [Ref: Ganong 21/e P 477, 505] j,


----------------------------------------------------------------------------------------------------------------------------------------------------------------------------1 4
Fats are finely emulsified in the small intestine by the detergent action of bile salts, lecithin and monoglycerides.
Detergent action of bile salts are due to their amphipathicQ nature i.e., they have both hydrophilic and hydrophobic
domains.

- Bile salts tend to form cylindrical disks called micelles with hydrophilic surface facing out & hydrophobic surfaces in.
- These micelles take up lipids, solubilzes them & provide a mechanism for their transport to the enterocytes.

131. A i.e. Porphyrinogens are coloured 132. A i.e. Porphyrins [Ref: Harper's 27je P-282; 26/e P- 273]

Porphyrins are colored Guminous) and fluorescence^ wherase porphyrinogens are colorless (non-luminous)Q.

- When porphyrins dissolved in strong mineral acids or in organic solvents are illuminated by ultraviolet light, they emit
a strong red fluorescence®.
- The double bonds joining the pyrrole rings in porphyrin are responsible for characteristic absorption & fluorosence.
- Soret band: - The sharp absorption band near 400nm. shown by porphyrin solution (in 5% HC1)
- Is characteristic of all porphyrins regardless of the side chain present.

Feature Porphyrinogen Porphyrins


Color, Fluorescence and Colorless & Do not emit Colored compounds and emit red fluorescence on
Lum inosity fluorescence (nonluminous)Q UV illumination (i.e. are l u m i n o u s ) ^
Pyrolle ring join by M ethylene (-CH 2-) bridge Q M ethenyl (-HC = ) bridge)
At 400nm No absorbance Characteristic absorbance

Porphyrinogens are reduced porphyrins as they contain extra 6 hydrogens. They can be autooxidized (by light) to their
respective porphyrins.

133. D i.e. Ehrlich's aldehyde regent [Ref: Practical boochemistry, C Raj Gopal and B D Toora 1/e P 47]

Reagent Substance Ehrlich's aldehyde reagent test for porphobilinogen &


Ehrlich's aldehyde Porphobilinogen & urobilinogen
UrobilimogenQ Take 5ml of urine +5 ml of Ehrlich's reagent
Fouchet's reagent Bile Pigment (Mix and allow to stand for 10 minutes)
Benedict's reagent GlucoseQ I
Sodium nitroprusside KetonesQ Add 5ml of saturated sodium acetate & mix Next add 5ml
chloroform & shake for few seconds
(allow layer to separate)
J ________________
i------------------------------r 1
Chloroform Increased Porphobilinogen in Aqueous layer
layer turns urine turns pink
pink - Acute intermittent porphyria I
4- - Variegate porphyria Porphobilinogen
Urobilinogen - Hereditary coproprphyria
Biochemistry: Metabolism of Proteins and Amino Acids ■ 573

134. C i.e. CEP 135. D i.e. Ferrochelatase


[R ef: H arper's 27/e P -283-286 ; V asudevan 6/e p. 2 45-50; C hatterjea 6/e P -472- 73, Lippincott's 3/e P -2791 .

To i
- Erythropoietic protoporphyria is due to deficiency of enzyme ferrochelatase^.
- All porphyrias are autosomal dominant (AD) except congenital erythropoietic porphyria (autosomal recessive, AR),
ALA dehydratase (porphobilinogen) deficiency (AR) and X-linked protoporphyria (X-linked).

136. D i.e. Hepatic porphyria

--------------------------------------------------------------------------------------------------------------------------------------------------
Hepatomegaly is an essential feature of von G ierek's disease (all glycogen storage diseases except type 2 ,5 ,7 ), Hurler ^
(Scheic) syndrome (& all mucopolysaccharidosis) and Nieman-Pick disease. But the hepatic involvement of hepatic
porphyria is variableQ. Patients with chronic hepatic porphyria (porphyria cutanea tarda), however may show liver
enlargement d/t fatty infiltration but frank sings of hepatomegaly and iron overload are rare.

137. A i.e. Ferrous ion; B i.e. Glycine; D i.e. Succinyl CoA [R ef: H arp er 26/e Pg 271-275]

Heme Synthesis
• First step is - Formation of ALA
from succinyl-COA & Glycine.Q Succinyl CoAQ A LA s y n t h t a s e S- A m in olevu lin ate (A LA )
• ALA is synthetised in + (in term ediate p ro d u ct is
mitochondria then enters the GlycineQ______ P y rid ox al phosphateQ a -am in o /? K etoadipate)
cytosol to form porphobilinogen.
- Porphobilinogen is converted to Tivo m olecu les o f A LA D eln/dratasc P orphobilin ogen (PEG )
A LA ( P 1 p recu rsor o f pyrrole)

o
uroporphyrinogen in presence of
uroporphyrinogen synthatase. Inhibition
(Intermediate product is hydroxy
methyl bilane) (Pb) Lead®
- Uroporphyrinogen is converted to coproporphyrinogen by uroporphyrinogen decarboxylase.
- Coproporphyrinogen enters the mitochondria where it form protoporphrinogen and then converted to protoporphyrin.
- Protoporphyrin with ferrous ion form heme in the presence of Ferrochelatase (Heme synthtase).
Chapter-7 GENERAL BIOCHEMISTRY

QUESTIONS

Water, pH, Buffers & Bioenergetics 10. True statement regarding covalent bonds is : (AI 07)
A. Electrons have same spin □
1. M etabolic water is- (NBE P 15) B. Electrons have opposite spin □
A. Ingested water
C. They are weak bonds □
B. Water infused iv □
D. None of the above □
C. Water produced by metabolism □
11. Strongest bond amongst the follow ing is (AI 06)
D. None H
A. Hydrophobic □
2. Monoprotic acid are (PGI 09)
B. Electrostatic □
A . Formic acid
C. Hydrogen bond □
B. Carbonic acid 7
D. Wander wall’s □
C. Acetic acid 7
12. Weakest bond is : (AIIMS 07)
D. Citric acid 77
A. Covalent □
E. Nitric acid
B. Hydrogen □
3. In a solution the concentration of hydrogen ion [H+] is
1 x 10-6 moles/litre. The pH of the solution w ill be: C. Electrostatic □
D. Vander wall □
A . Three (AIIMS 06) 77
B. Six 13. Energy required for maintenance of bodily functions
C. Nine 77 of a person weighing 40 kgs is: (PGI 01)
D. Twelve C A. 1000 kilocalories □
4. The pH of a solution containing 5 millim ole per liter of B. 15000 kilocalories □
H+ ions is closest to what value? (AIIMS 16) C. 2000 kilocalories □
A. 2.3 7] D. 2500 kilocalories □
B. 1.7 E. 3500 kilocalories □
C. 3.5 _7 14. Difference between reversible and irreversible
D. 4.2 r reaction is (AIIMS 03)
5. The buffering capacity of a buffer is maximum at pH A. Entropy □
equal to (AI 03) B. Temperature □
A . 0.5 pKa J C. Work done □
B. pKa 7 D. Amount of heat production □
C. pKa+1 77 15. Regarding a crystal, the true statement is : (AI 02)
D. 2pKa ij A . Molecules are arranged in same orientation with
6. A buffer that is most effective at a pH of about 4.5 is; different confirmation □
A . Acetate buffer. (AIIMS 14, 03) J B. Molecules are arranged in different oreintation with
B. Bicarbonate buffer fj different confirmation □
C. Phosphate buffer 777 C. Molecules are arranged in same orientation and
D. Tris buffer □ same confirmation □
7. HCO 3/H2CO 3 is the best buffer because it is ?(AIIMS 16) D. Molecules are arranged in different oreintation but
A. pKa near physiological pH □ with same confirmation □
B. Its components can be increased or decreased in the
body as needed □ Xenobiotics
C. Good acceptor and donor of H+ ions □
D. Combination of a weak acid and weak base □ 16. All are true about glutathione except: (AIIMS 10,08)
8. The presence of 2 extra pairs of electrons on the oxygen A. It is a tripeptide □
in water molecule results in : (AIIMS 02) B. It converts hemoglobin to methemoglobin □
A. Makes water a non-polar solvent □ C. It conjugates xenobiotics and it is co-factor of various
B. Forms covalent bonds in the ice. □ enzymes □
C. Electro negative charge on water molecule (71 D. It scavenges free radicals and superoxide ions □
D. Electro positive charge on water molecule □ 17. True about Glutathione except- (NBE P 15)
9. Strongest bond out of the following is : (AI 08) A. TRIpeptide □
A. Electrostatic □ B. Formed form glutamic acid, glycine, cysteine □
B. Hydrogen n C. Act as antioxidant in reduced state □
C. Hydrophobic [7 D. All of the above □
D. Vanderwall's r
Biochemistry: General ■ 575

27. All of the follow ing act as antioxidants except:


18. W hich of the follow ing elem ents is know n to A. Vit. D (PGI 15,13,09; AIIMS 16,08; NBE P 15,14,13) O
influence the body's ability to handle oxidative stress? B. Vit. C & E □
A. Calcium (AI 04) □ C. Selenium □
B. Iron □ D. Glutathione peroxidase □
C. Potassium □ 28. The m ineral having action like vitam in E-
D. Selenium □ A. Calcium (NBE P 15,13) □
19. W hich of the follow ing enzym e are associated w ith
B. Iron □
break dow n of H 2O 2. (PGI 11)
C. Selenium □
A. Catalase □
D. Magnesium □
B. Perioxidase □
29. T he m ain function of vitam in C in the b ody is-
C. Oxidase □
D. Oxygenase □ A. Coenzyme for energy metabolism (NBE P 15,14) □
E. Hydrolase □ B. Regulation of lipid synthesis □
20. H 2O 2 is breaked o r form ed by w hich of the follow ing C. Involvement as antioxidant □
enzym e: (PGI 03) D. Inhibition of cell growth □
A. Oxidase □ 30. M ajor function of vitam in E in the body- (NBE P 15)
B. Oxygenase □ A. Regulation of energy metabolism □
C. Hydrolase □ B. Carboxylation reaction □
D. Peroxidase □ C. Blood clotting □
E. Catalase □ D. Protection of biological membrane from free
21. W hich of the follow ing is n ot a phase I Reaction: radical damage □
A. Oxidation (AI 12) □ 31. R eperfusion injury is caused by: (PGI 06)
B. Reduction □
A. Vitamin E □
C. Hydrolysis □
B. Superoxide ion □
D. Conjugation □
C. Calcium ion □
22. The m ain enzym e responsible for activation of
xenobiotics is D. Magnesium ion □
A. Cytochrome P-450 (A I03) □ 32. W hich of these have an tioxidant properties: (PGI 12)
B. Glutathione S-transferase □ A. Tocopherol □
C. NADPH cytochrome P-450-reductase □ B. Reduced Glutathione (GSH) □
D. Glucuronyl transferase □ C. Citrulline □
23. The conjugation of bilirubin to glucuronic acid in the D. Lycopene □
liver; (AIIMS 03) 33. True about G lutathione reductase: (PGI 01)
A.Converts a hydrophilic compound to a hydrophobic A. Sulphur containing enzyme □
one □ B. Important in methemoglobinemia □
B. Converts a hydrophobic compound to a hydrophilic C. Free radical scavenger □
one □ D. All □
C. Enables the bilirubin molecule to cross the cell
34. Enzym e responsible fo r respiratory b urst reaction
membrane □
A. Dehydrogenase (AIIMS 08) □
D. Is increased during neonatal jaundice □
B. Peroxidase □
24. W hat is m etabolised like xenobiotics (AIIMS 05)
C. Hydroxylase □
A. Myoglobin □
B. Bilirubin □ D. NADPH - oxidase □
C. Biliverdin □ 35. R eactive oxygen interm ediates are released by:
D. Haemoglobin □ A. Catalase (AI 08) □
25. Bile salts undergo xenobiotics: (PGI 07) B. NADPH oxidase □
A. After conjugation with taurine and glycine □ C. Glutathione peroxidase □
B. After conjugation with lysine □ D. Superoxide dismutase □
C. After conjugation with betaglucuronic acid □
D. After conjugation with derived proteins □ Minerals
Free Radicals
36. W hich is non essential mineral: (AIIMS 10)
26. W hich of the follow ing reduces oxidative stress except:
A. Sodium □
A. Superoxide dismutase (PGI 1 1 ) 0
B. Manganese □
B. Catalase □
C. Glutathione peroxidase □ C. Iron □
D. Xanthine oxidase □ D. Lead □
E. Ceruloplasmin □
576 ■ A Complete Review of Short Subjects

Vitamins 47. Thiamine is essential for (AI 05)


A. Isocitrate dehydrogenase □
37. Which of the following is a Water Soluble Vitamin? □
B. Succinate dehydrogenase
A. Folic Acid (AI 08) □ □
C. Pyruvate dehydrogenase
B. Vitamin A □ □
D. Acetyl CoA synthetase
C. Vitamin K □ 48. Thiam ine is not used in which of the following
D. Linolenic Acid □ reactions (AIIMS 01)
38. NAD acts as a cofactor for (PGI 09) A . Lactate to pyruvate □
A. Citrate Synthetase □ B. Alphaketoglutarateto succinyl CoA □
B. Isocitrate dehydrogenase □ C. Glucose to pentose □
C. a-ketoglutarate dehydrogenase □ D. Oxidative decarboxylation of a-keto amino acids □
D. Malate dehydrogenase □ 49. Which of the vitamin deficiency lead to lactic acidosis
E. Succinyl thiokinase □ A . Riboflavin (AIIMS 10) □
39. ALL are involved in energy m etabolism except- B. Thiamine □
A. Vitamin B, (NBE P 15) □ C. Niacin □
B. Vitamin B3 □ D. Pantothenic acid □
C. Vitamin B7 □ 50. Riboflavin deficiency is assessed by- (NBE P 15)
D. Vitamin B 12 □ A. Transketolase □
40. Gene for folic acid absorption is present on B. Glutathione reductase □
A. Chromosome X (AIIMS 08) □ C. PDH □
B. Chromosome XXI □ D. None □
C. Chromosome V □ 51. Which vitamin deficiency causes glossitis and
D. Chromosome Xi □ cheilosis- (NBE P 15)
41. The gene for folic acid transporter is located on which A. Thiamin □
chromosome: (AIIMS 08) B. Riboflavin □
A. 5 □ C. Folic acid □
B. 15 □ D. Vitamin A □
C. 21 □ 52. The activity of the following enzyme is affected by
D. X □ biotin deficiency: (AIIMS 03, AI 06)
42. Which of the following statement about thiamin is A . Transketolase, □
true? (AIIMS 08) B. Dehydrogenase. □
A. It is a co-enzyme of lactate deydrogenase □
C. Oxidase. □
B. Its deficiency is associated with scurvy □ D. Carboxylase. □
C. Its co enzyme function is done by thiamine 53. Active moiety of CoA is (WB 10, TN 09)
monophosphate □ A . Acetyl group □
D. Its co-enzyme for pyruvate dehydrogenase and a- B. Pantoic acid □
ketoglutarate dehydrogenase □ C. Thiol of P-alanine □
43. Thiam ine requirement increases in excessive intake of: D. Thiol of pantetheine □
A. Carbohydrate (AIIMS 09) □ 54. Which vitamin is necessary for coenzyme A synthesis
B. Amino acid □ A . Pantothenic acid (AI 05) □
C. Fat □ B. Ascorbic acid □
D. Lecithine □ C. Biotin □
44. Thiamine level is best monitored by: D. Pyridoxine □
(AI 12; NBE P 15,14,13; AIIMS 15,13) 55. Panthothenic acid is coenzyme of which of the
A. Transketolase level in blood (RBC) □ following reaction (s): (PGI 08)
B. Thiamine level in blood □ A. Dehydrogenation □
C. G 6 PD activity □ B. Oxidation □
D. Reticulocytosis □ C. Decarboxylation □
45. Enzyme activity measured in beri beri is: (PGI 15,13,05) D. Reduction □
A. RBC Transketolase □ E. Acetylation □
B. Transaminase □ 56. Patient presenting with pellagra, parkinsonism,
C. Decarboxylase □ convulsions, anemia and kidney stones has deficiency
D. Deaminases □ of (AI05)
46. Thiamine acts as a cofactor in: (AI 04) A. Coenzyme A □
A. Conversion of pyruvate to acetyl-CoA □ B. FADH □
B. Transamination reactions □ C. Niacin □
C. Oxidation in respiratory chain □ D. Pyridoxal phosphate □
D. Conversion of pyridoxal to pyridoxal phosphate □
Biochemistry: General ■ 577

57. Which is not pyridoxine dependant: (AIIMS 08) C. Requires in metabolism of methylmalonyl CoA □
A. Homocystinuria □ D. Requires for conversion of pyruvate to lactate □
B. Maple syrup urine disease □ 68. Vitamin which is excreted in urine is : (AIIMS 07)
C. Oxaluria □ A. Vitamin A □
D. Xanthinuria. □ B. Vitamin C □
58. Cause(s) of homocystinuria: (PGI 12) C. Vitamin D □
A. Deficiency of vit B 6 □ D. Vitamin K □
B. Deficiency of vit B 12 □ 69. All of the follow ing are required for hydroxylation of
C. Deficiency of folic acid □ proline in collagen synthesis excep t: (NBE P 13)
D. Deficiency of pantothenic acid □ A. O2 D
59. Co-enzyme used in transamination (AIIMS 03) B. Vitamin C □
A. NAD □
C. Dioxygenases □
B. Biotin □
D. Pyridoxal phosphate □
C. Pyridoxal phosphate □
70. Vitam in required for hydroxyproline to proline
D. Riboflavin □ conversion: (NBE P 14; PGI 07)
60. Intrinsic factor of Castle is secreted by
A. Vitamin C □
A. Chief cells (AIIMS 10) □
B. Vitamin E □
B. Parietal cells □
C. Pyrodoxal P O 4 □
C. Mucous cells □
D. Biotin □
D. |3cells □ 71. During the formation of hydroxyproline and
61. Vitamin B 12 deficiency has A/E (PGI 10, WB 10)
hydroxylsine, the essential factors required is/are:
A. Folate trap □
A. Pyridoxal phosphate (PGI 03)D
B. Decreased methyl malonyl CoA □
B. Ascorbic acid □
C. SACD □
D. Megaloblastic anemia □ C. Thiamine pyrophosphate □
62. Vitamin B 12 is absorbed in the (NBE P 14; AI 06) D. Methylcobalamine □
A. Stomach □ E. Biotin □
B. Duodenum □ 72. Aminoacyl t-RNA is required for all excep t: (AI 02)
C. Ileum □ A. Hydroxyproline □
D. Colon □ B. Methionine □
63. Cobalt forms a component of which vitamin C. Cystine □
A. Biotin (AI 05) □ D. Lysine □
B. Vitamin B12 □ 73. Gamma carboxylation of glutamic acid in clotting
C. Vitamin A □ factor II, V II and protein C is dependent on
D. Tocopherol □ A. Vitamin K (NBE P 14,13; AI 08)□
64. Not a dietary source of vit. B12- (AIIMS 08) B. Vitamin C □
A. Fish □ C. Vitamin A □
B. Meat □ D. Vitamin E □
C. Soyabean □ 74. The Sim ilarity between V it C & V it K is: (AI 09, 05)
D. Liver □ A. Both help in conversion of proline to hydroxy­
65. Vitamin B 12 and folic acid supplementation in proline □
megaloblastic anemia leads to the improvement of B. Both help in post-translational modification □
anemia due to: (AI 02)
C. Both have anti infective activity □
A. Increased DNA synthesis in bone marrow □
D. Both are involved in coagulation cascade □
B. Increased Hemoglobin production □
75. In vitamin C deficiency, post translational
C. Erythroid hyperplasia □
modification o f which amino acid is defective:
D. Increased iron absorption □
A. Lysine (AIIMS 07) □
66. Vitamin B 12 acts as co-enzyme to which one of the
B. Alanine □
follow ing enzymes? (AIIMS 14,12; PGI 13; A I10)
C. Glycine □
A. Isocitrate dehydrogenase □
D. Arginine □
B. Homocysteine methyl transferase □
76. V ita m in K is iv o lv e d in the posttranslational
C. Glycogen synthase □
m o dification ? (AI 11)
D. G- 6 -P dehydrogenase □
A. Glutamate □
67. All are true about vitamin B 12, except- (NBE P 15)
A. Active form is methylcobalamine □ B. Aspartate □
B. Requires for conversion of homocysteine to C. Leucine □
methionine □ D. Lysine □
578 ■ A Complete Review of Short Subjects

77. Carboxylation of clotting factors by vitamin K is 84. Which of the following is a component of the visual
required to be biologically active. Which of the pigment rhodopsin: (AIIMS 05)
following amino acid is carboxylated? (AIIMS 15,13,08) A. P-Carotene □
A. Histidine □ B. Retinal □
B. Histamine □ C. Retinol □
C. Glutamate □ D. Retinoic acid □
D. Aspartate □ 85. Vitam in A is stored m ainly as retinol esters in:
78. HDN is seen due to deficiency of vitam in: (PGI 09) A. Kidney (AIIMS 05) □
A. A □ B. Muscle □
B. C □ C. Liver □
C. K □ D. Retina □
D. E □ 86. Which of the follow ing combinations of biologically
79. Vitamin required for post translational modification of active molecules does vitamin A consists of
coagulants is : (AI 07) A . Retinol,retinal and retinoic acid (AIIMS 04) □
A. Vitamin A □ B. Retinol, retinal and tetrahydrofolate □
B. Vitamin C □ C. Retinal, conjugase and retinoic acid □
C. Vitamin B6 □ D. PABA, retinal and retinaldehyde □
D. Vitamin K □ 87. Vitamin A intoxication causes injury to
80. Vit K is needed for which of these post transational A . Lysosomes (AIIMS 07) □
m odification processes- (AIIMS 01) B. Mitochondria □
A.Methylation □ C. Endoplasmic reticulum □
B.Carboxylation □ D. Microtubules □
C.Hydroxylation □ 88. M etabolic bone disease is caused by excess intake of
D. Transketolation □ which vitamin: (PGI 02)
81. The vitamin used for post-translational modification A. A □
of glutamic acid to y-Carboxy-glutamate is (AI 04) B. B □
A. Vitamin D □ C. C □
B. Vitamin E □ D. D □
C. Vitamin A □ E. E □
D. Vitamin K □ 89. Which of the follow ing does not contain keratin
82. Vitam in K dependent coagulation factors include: A. Potato (AIIMS 04) □
A. II and III (AI 10) □ B. Tomato □
B. IX and X □ C. Carrot □
C. Ill and V □ D. Spinach □
D. VIII and XII □
83. Vitam in K dependent clotting factor is:
A. Factor VII (AIIMS 10) □
B. Factor I □
C. Factor XI □
D. Factor XII □
Biochemistry: General 579

ANSWERS & EXPLANATIONS:

Water, pH, Buffers & Bioenergenetics

1. C i.e. Water produced by metabolism [Ref: Naik 1/e p. 2 71 ]

M etabolic water is water produced during metabolism of food. It is ~ 150-350 ml/day.

2. A, C, E i.e. Formic acid, Acetic acid, Nitric acid

--------------------------------------------------------------------------------------------------------------------------
Monoprotic acids (donating 1 proton) are hydrochloric acid (HCl), nitric acid (HNOs), form ic acid (HCOOH), acetic acid
(CH3 COOH) and benzoic acid 6. Biprotic acids are sulfuric acid (H2SO 4) and carbonic acid (H2CO 3)0 and triprotic acids
are phosphoric acid (H3 PO4) and citric arid®

3. B i.e. Six [Ref: Harper's 30/e P-10-13]

i.e. pH is the negative log of


pH = -log [H+]
hydrogen ion concentration. Dissociation constant of H2O —> K = r — ?—
[h 2o ]
In this case pH = - log [106]
Ion product for water Kw= (K) [H2O] = [H+] [OH ]
= - (-6 ) = + 6.0
In case if H+ conc. is 3.2 x lO-6
Bracket represents molar concentration (strictly speaking
pH = - log [3.2 x lO-6]
molar activities). Since one mole (mol) of water weighs 18g,
= - log [3.2] - log [1 0 6]
one liter (L) (lOOOg) of water contains 1000/18 = 55.56 mol
= - [ 0 .5 ] - [ - 6 ]
Since the probability that a hydrogen ion in pure water will
= - 0.5 + 6 = 5.5
exist as a hydrogen ion is 1.8xl0 '9 the molar conc. of H+ (or
So you can see that as H+ conc. increases pH decreases OH ) in pure water is 1 .8 x 10-9 times of molar conc. of pure
I would like to tell this much about dissociation constant water (55.56) = ~ 10 2
1 gm of water contains 3.46 x 1022 molecules. To state that
the probability that a hydrogen exists as an ion is 0.01 i.e. Dissociation constant, K = — — — - = 1.8 x 10 16 mol/L
that a hydrogen atom has 1 chance in 100 of being an ion 55.56
and 99 chances out of 100 of being part of water molecule. Ion product Kw = (1.8xl0 -16 mol / L) (55.56 mol / L) = 10'14
The actual probability of hydrogen atom in pure water mol2 /L2
existing as a hydrogen ion is - 1 .8 x 1 0 9 and as molecule is Kw equals 10'14 (mol /L)2 for all aqueous solutions, even
1. Stated another way fo r every hydrogen ion and solutions of acids or bases. We shall use Kw to calculate pH
hydroxyl ion in pure w ater there are 1 .8 billion or 1 .8 x 109
of acidic & basic solutions.
w ater molecules.

4. A i.e. 2.3

pH = - log [H+] = -log [5 x 1 0 3] = -log (5) - log [10-3]


= -log(5) - (-3) = -log (10/2) + 3 = -1 + log 2 +3
= 3 - 1 + log 2 = 2+ log 2 = 2+0.3 = 2.3Q ★ [log 10 = 1 & log 2 = 0.3010]

B i.e. pKa [Ref: Harper's 30/e P -ll-13; Guyton 11/e p. 385] 6. A i.e. Acetate Buffer

"A t values close to pKa, the bu ffer solution resists changes in pH most effectively", i.e. a bu ffer system is most
effective when the pH of the system is near its pka valueQ.

Buffers

• Buffering is the tendency of a solution to resist a change in pH more Buffer Systems (BS)
effectively after addition of a strong acid or base than does an equal ICF Phosphate BS
volume of water. Both in Protein BS (i.e. Hb, plasma
• Solutions of weak acids & their conjugate bases (& vice versa) exhibit ICF & ECF protein & amino acid BS)
buffering. Strong acid/bases do not have buffering quality as great as ECF Carbonic acid-Bicarbonate
the weak acid/bases. pKa refers to the pH a t which the concentration
o f the acid equals that o f the baseQ.
580 ■ A Complete Review of Short Subjects

A t pH = pKa A t pH < pK A t pH > pK


the solution i.e. more acidic environment much of the acid buffering i.e. more alkaline
would contain capacity of the solution has already been neutralized so environment, much of the
acid and base in adding even minute quantities of acid would decrease the pH, alkaline buffering
equal amounts, i.e. it no more has buffering capacity to acids, but it would still capacity has been
and therefore have buffering capacity towards bases. neutralized after all that's
would resist a the reason why the pH
Buffer Pka
change most rose above pK in the first
Phosphate 6.8
effectively in place so adding even
Bicarbonate 6.1Q
either direction. Acetate minute quantities of alkali
4.76
Tris [hydroxymethyl aminomethane] would increase the pH.
8.3
Note: The effective range of a buffer is one pH unit higher or low er than pKa<2.

7. B i.e. Its com ponents can be increased or decreased in the body as needed

- Bicarbonate - carbonic acid buffer system has low elem ent concentration and has pK of 6.1 (physiological pH is 7.4)
which means the buffer operates where the slope is low or buffering power is poor (as the pH & pK difference is > 1).
- D espite these facts, the bicarbonate buffer system is the m ost powerful (best) extracellular buffer in the body. This
apparent paradox is mainly due to the fact that the tw o components of this buffer system i.e. H C O r & CO 2 (bicarbonate
& carbon dioxide) are regulated by kidney and lungs respectively in the body as needed^.

8. C ie Electronegative charge on w ater molecule 9. A i.e. Electrostatic 10. B i.e. Electrons have opposite Spin
11. B i.e. Electrostatic 12. D i.e. V ander wall [Ref: Harper's 27/e P - 5-7; Lehninger 2/e P - 39-45; Chatterjea 6/e P - 84]

In decreasing order of strength various bons are covalent bond (strongest)Q > disulphide bond> electrostatic (salt
bridges)^ > hydrogen bond > vander wall forces (weakest)Q

★ Disulphide bond is strong high energy covalent bond between two cysteine residues.

13. A i.e. 1000 Kcal [Ref: CMDT 1263; Park’s SPM 424]

Energy required for basal m etaboism = 1 K.cal/hr/kg of body weight^

So in 24 hours for a 40kg individual, it is 40x24 = 960 K cal.

Basal energy Expenditure is 10% less than resting energy Expenditure

14. C i.e. W ork done [Ref: Lehninger 2/e P 390-92; Bahl 24/e P-270-74; 200 ]
15. C i.e. Same conform ation & molecules are arranged in same orientation [Ref: Chemistry by Mishra, Bhushan, Sharma/105] .

!
A crystal lattice is a three dim ensional, highly ordered and regular arrangem ent of atom s, m olecules or ions. A crystal
%
is form ed from sm all units, (that have the sam e orientation and sam e conform ation^), such that w hen these units are
repeated in all possible directions, the structure of the crystal is still maintained.________________________________________^

Xenobiotics
16. B i.e. It converts hem oglobin to methem oglobin 17. D i.e. All of the above 18. D i.e. Selenium
[Ref: Lippincott's 4/e p. 148; Harper's 27/e p. 620, 636-37,180- 81, 96; Chattergea 7/e p. 478-79]

- Selenium in the form o f 'selenocysteine' is a com ponent of enzym e glutathione peroxidaseQ. 'Glutathione peroxidase
serve to protect proteins, cell membranes, lipid and nucleic acids from oxidant molecules'. Glutathione prevents
oxidation of hem oglobin to methemoglobinQ. It acts as antioxidant in reduced stateQ.
- Reduced glutathione is tripeptide thiol (y- gluatam yl cystein glycine) consisting of 3 amino acids - glutam ic acid,
cysteine and glycineQ. The glutamate is linked to cysteine through y - carboxyl linkage. And sulfhydryl (SH) o r thiol
group o f cysteine, w hich accounts for its strong electron donating (nucleophilic) character of glutathione^, is the
business part of molecule.
Biochemistry: General ■ 581

19. A i.e. Catalase; B i.e. Perioxidase 20. A i.e. Oxidase; D i.e. Peroxidase; E i.e. Catalase [R ef: H arper's 27/e P -96, 2041

Oxidase & superoxide dismutase generate H202, where as catalase and peroxidase are associated with breakdown of
h 2o 2q.

21. D i.e. Conjugation [R ef: H arp er 29/e p 6767-682; V asudevan 6/e p 447-501
22. A i.e. Cytochrome P-450 23. B i.e. Converts a hydrophobic compound to a hydrophilic one
24. B i.e. Bilirubin 25. C i.e. Conjugation with P glucoronic acid

f t
Phase I biotransformation reactions include Hydrolysis. Hydroxylation, Reduction, Oxidation, Oxidative
Deamination, Dealkaylation and Epioxidation. M n " High RODE". Whereas phase II reactions include
Glucronidation, Sulfation, Conjugation (with glucronic acid, glutathione, sulfate or aminoacids), Acetylation and
M ethylation. Mn "5G -SCA M ".
Cytochrome p450 monoxygenase is the main enzyme responsible fo r activation o f xenobiotics (phase I hydroxylation
reaction)®.
B ilirubin is metabolized like xenobiotics; it is conjugated with beta glucronic acid in hepatocytes to convert a nonpolar
hydrophobic bilirubin to a p olar hydrophilic one which is readily excreted in bile®.

Free R adicles

26. D i.e. Xanthine oxidase 27. B, C, D i.e. Vit.C & E, Selenium , Glutathione peroxidase 28. C i.e. Selenium
29. C i.e. Involvem ent as antioxidant 30. D i.e. Protection of biological mem brane from free radical damage
31. B i.e. Superoxide ion [R ef: H arp er 28/e 4 82-85; C h attarjae 7/e 198; L ippin cott 5/e 148; V assudevan 6/e 241-588-409-4291

f t
- Xanthine oxidase increases oxidative stress by generating superoxide free radicles^
- Ischemic reperfusion injury is caused by reactive oxygen species (ROS)/free radical or pro oxidant like superoxide
ion<3
- Antioxidants blocking free radical formation include vitamin A (Beta carotene), C and E; superoxide dismutase,
catalase and selenium containing glutathione peroxidase enzymes; ceruloplasmin, transferrin, ferritin, and lactoferrin
transport proteins etc.
- M ain function of vitamin E is antioxidationQ. It prevents oxidation of other antioxidants (eg vitamin A, C &
carotenes). It protects biological membrane from free radical damageQ formed by peroxidation of PUFA. So it prevents
rancidity of fats.
V__________________________________________________________________________________________________________________ ,

32. A i.e. Tocopherol; B i.e. Reduced Glutathione(GSH); D i.e. Lycopene


33. C i.e. Free radical scavanger [Ref: Harper 29/e p. 200-2011

- Tocopherol (vitamin E), lycopene (in red tomatoes) and


reduced glutathione (GSH) have antioxidant properties.
- In RBC, the pentose phosphate pathway is the only source of
NADPH for reduction of oxidized glutathione by enzyme
glutathione reductase. Glutathione reductase, a flavo protein
enzym e containing FAD ca ta ly zes oxid ised glu tath ion e to reduced glutathione® . Reduced glutathione (GSH) removes
H 20 2 in a reaction catabolized by glutathione peroxidase (a selenocysteine i.e. contains selenium analog of cysteine).
Since H2O 2 accum ulation in RBC causes oxidative damage to cell m em brane 1/t hemolysis & decreased life span. The
PPP, glutathione peroxidase and reduced glutathione (GSH) protect erythrocyte against oxidative damage (i.e. have
antioxidant properties).
★ Glutathione = Glycine + Cysteine + Glutamate
★ Methhemoglobin reductase is im portant for conversion of m ethhem oglobin back to hemoglobin.

34. D i.e. NADPH - oxidase 35. B i.e. NADPH oxidase


[R ef: H arp er 2 8 /e p . 4 82-85; R obbin s 7/e p. 16-17, 60-61; G uyton 11/e p.16; G an on g 22/e p. 518]
582 ■ A Complete Review of Short Subjects

Minerals

36. D i.e. Lead [Ref: Vasudevan 6/e p 411; Styanarayan 3/e p 403; Chatterjea 7/e p 707-570]

Aluminium, cadmium, mercury and lead minerals are toxic and should be avoided^.

Vitamin

37. A i.e. Folic Acid [Ref:; Lippincott's 4/e p- 373; Ganong 22/e p. 313-16; Devlin 5/e p - 1154, Harper's 28/e P- 468]

Vitamin B complex and C are water soluble, whereas vitamin A, D, E and K are fat soluble®.

38. B, C, D i.e. Isocitrate dehydrogenase, a-ketoglutarate dehydrogenase, M alate dehydrogenase


[Ref:Chatterjea 7/e p 171;Lehninger 5/e p 518]

Niacin (Vitamin B 3) in biologically active form of NAD+ (nicotinamide adenine dinucleotide) and NADP+ (nicotinamide ☆
adenine dinudeotide phosphate). NAD+ act as cofactor for various enzymes such as
malate/alcohol/lactate/glyceraldehyde 3-P/ a-glycero-P/a-ketoglutarate & pyruvate dehydrogenase. NADP+ is cofactor
for G6PD & glutathione reductase. NADP+ or NAD+ act as cofactor for glutamate & isocirate dehydrogenase®.

39. D i.e. Vitam in B 12

5 Vitam ins o f B group involved in energy m etabolism are (1) B1 (Thiamin)®, (2) B2 (Riboflavin)®, (3) B3 (Niacin)®,
(4) B5 (Pentothenic acid)® and (5) B7 (Biotin/Vitamin H)®.

40. B i.e. Chrom osom e XXI 41. C i.e. 21

Folic acid transporter gene (SLC 19 AI solute carrier family 19) is located on long arm o f chrom osom e 21 (21q) ® at band
22.3. Transport of folate occurs by potocytosis which functionally couples folate receptor, folate transporter and V- type h+
pump.

42. D i.e. Its co-enzym e for pyruvate dehydrogenase and a-ketoglutarate dehydrogenase 43. A i.e. Carbohydrate
44. A i.e. Transketolase level in blood 45. A i.e. Transketolase 46. A i.e. Conversion of Pyruvate to acetyl COA
47. C i.e. Pyruvate dehydrogenase 48. A i.e. Lactate toPyruvate 49. Bi.e. Thiam ine
[Ref: Harper 28/e p. 154, 273-74; Lippincott 5/e p 111, 266,145-46, 378-79; Vasudevan 6/e p 391-92; Lehninger 5/e p 549-51;
Chatterjea 7/ep. 166-67] j,

------------------------------------------------------------------------------------------------------------------------------------------------
Vitamin Bi (Thiamine) in form of thiamin diphosphate (TDP) or TPP act as a coenzyme for transketolase (in PPP) and
3 dehydrogenase multienzyme complexes i.e. Branched chain, Alpha ketoglutarate and Pyruvate - dehydrogenase
enzymes. (Mn "Truly Dehydrated- B A P")
Thiamin has a central role in energy yielding metabolism (especially of carbohydrates) as it is required for pyruvate
dehydrogenase (converting pyruvate to acetyl CoA in carbohydrate metabolism)® and a-ketoglutarate
dehydrogenase (in citric acid cycle). So its requirement increases on excess intake o f carbohydrate®. And for the same
reasons thiamine deficiency causes decreased energy production®.
Because of role of TDP in pyruvate dehydrogenase; thiamin deficiency in subjects on high carbohydrate diet results in
increased plasma pyruvate and lactate eventually 1/1 potentially life threatening lactic and pyruvic acidosis®.
Thiamin deficiency 1 /t - beriberi, W ernicke- korsakoff syndrome®, polyneuritis and lactic acidosis®. Thiamin
nutritional status is best assessed by erythrocyte transketolase activation®.

50. B i.e. G lutathione reductase 51. B i.e. Riboflavin


52. D i.e. Carboxylase [Ref: Lippincott 5/e p. 381, 266; Harper 28/e p. 479; Vasudevan 6/e p. 400; Chatterjea 7/e p. 179-80]

R iboflavin deficiency is assesses by glutathione reductase®.


Biotin is a coenzyme of carboxylase enzymes®, that catalyze CO 2 transfer reactions (CO 2 fixation reaction).
Biochemistry: General ■ 583

Vitamin V it B 2 (Riboflavin) Vit B 3 (Niacin) Folic acid


Coenzyme FMN & FAD NAD & NADP Tetrahydro folate (TH 4F)
Function • FMN and FAD are 2 coenzymes of • NAD and NADP are its active Its active form,
this vit, and are electron carriers^ in forms which operate as hydrogen Tetrahydrofolate is a carrier of
oxidoreduction reaction & electron transfer agent one carbon unit; synthesis of
• Consituent of: • Deficiency seen in: methionine, thymine and
- Cytochrome C- reductase - M aize eaterQ purine
- Warburg yellow enzyme - Carcinoid syndromeQ
- Fumarate dehydrogenase - Heartnup disease^
Deficiency - Seborrhic dermatitis Pellagra^ c/b "3D s" - Megaloblastic anemiaQ
- Angular stomatitis, Glossitis^ - Diarrhoea - Neural tube defect^.
- CheilosisQ - Dermatitis
- Dementia
Features - Heat & acid resistant - In body synthesized from • Folic acid reduces risk of
- Destroyed by light & alkali tryptophanQ Neural tube defect &
- Used as food additive because of its - 60mg tryptophan= lm g niacin - HyperhomocystinemiaQ
yellow color - High doses used to treat
- Deficiency detected by hyperlipidemiaQ
measurement o f RBC glutathione - In overdose causes cholestatic
reductaseQ jaundice

53. D i.e. Thiol of pantetheine 54. A i.e. Pantothenic acid


55. E i.e. Acetylation [Ref: Harper 28/e p. 479; Chatterjea 7lh/ed pg 177-76; Lippincott 5thled pg-381; Vasudevan 6/e p. 398-99]

Pantothenic acid is necessary for synthesis of coenzyme A (CoA)Q, which functions as a coenzyme for acetylation
reactionsQ

56. D i.e. Pyridoxal phosphate 57. B i.e. M aple syrup urine disease
58. A i.e. Deficiency of vit B&; B i.e. D eficiency of vit B 12; C i.e. Deficiency of folic acid 59. C i.e. Pyridoxal phosphate
[Ref: H arper 28/e p. 252-58, 475-76; Lippincott 5/e p. 378; Vasudevan 6/e p. 396-97; Chatterjea 7/e p. 3 7 2 - 7 3 ; Lehninger 5/e p. 864-65, 878-831 ,

Pyridoxal phosphate (active Be) dependent conditions (in which it is used in treatment) are H om ocvstinuria. Oxaluria,
Cvstathioninuria and Xanthurenic acid uriaQ, Mn - "H O C X or Hom o Ox Siton Zen"
Homocystinuria is vitamin B6, B12 and folate, dependent^
M aple syrup urine disease is d/t defective branched chain a-ketoacid dehydrogenase enzyme; and it may be a/w
thiam in (vitamin Bi) deficiency^
Methylmalonyl aciduria is seen in vitamin Bu deficiency^.
Figlu (N- formimimino-glutamate) uria following a dose of histidine occurs in folate (folic acid) deficiency (Histidine
load test). AICAR (amino imidazole carboxamide ribosyl 51-? ) uria also occurs in folate deficiency.

60. B i.e. Parietal cells 61. B i.e. D ecreased m ethylm alonyl CoA
62. C i.e. Ileum 63. B i.e. Vitam in B 12 64. C i.e. Soyabean 65. A i.e. Increased DN A synthesis in bone m arrow
[Ref: Harper's 28/e P-500-02, 476-77; Lippincott 5/e p. 374-76; Vasudevan 6/e p. 403-5; Chatterjea 5/e p. 185-89]

Cobalt containing V itam in B 12 is found only in food of animal origin and is absorbed from distal ileumQ. B 12 & folic
acid supplements treat pernicious (megaloblastic) anemia by increasing DNA synthesis in bone marrowQ.

66. B i.e. Hom oeystine M ethyl transferase [Ref: Lippincott 3ni/373; Chettcrjea Shinde 6,h/181]

Vitamin B 12 is required as a coenzyme to homocystine methyl transferase involved in the synthesis o f m ethionine from
homocystine.
584 ■ A Complete Review of Short Subjects

M etabolic role of Vitamin B 12

Biologically active forms of vitamin B 12 are called as 'cobamide coenzymes' and act as coenzymes with several enzymes for
various metabolic reactions. These include

Vitamin B12
(Deoxyadenosyl cobalamine)
1. Isomerization of methyl malonyl co-A to succinyl CoA®;Methyl malonyl CoA ►Succinyl CoA
Methyl malonyl CoA mutase

N5-Methyltetrahydrofolate Tetrahydrofolate

2. M ethylation of Homocystine to methionine®: Hom ocystine --------------- > Methionine


Vitamin B 12 (Methyl-cobalamine)
3. M ethylation of pyrimidine ring to form thymine.
V itam in
4. Conversion of ribonucleotides to deoxyribonucleotides in DNA synthesis: R ib o n u c le o tid e s ------------------ »•D eoxyribon u cleotid es
Reductase
Vitamin B 12
5. Required for metabolism of diols: Ethylene glycol ---------------------- ►Acetaldihyde

Vitamin B 12
6. In Bacteria for interconversion of glutamate and P methyl aspartate: Glutamic acid > [3 Methyl aspartate

67. D i.e. Requires for conversion of pyruvate to lactate

D eficiency of Vitam in B12

• 3 enzymes methyl malonyl CoA mutase (or isomerase), leucine amino mutase, and methionine synthase are Bn
dependent

Methyl malonic acid uria Folate Trap and Pernicious Anemia

- Methyl malonyl CoA is formed as an intermediate in


catabolism of valine, threonine, isoleucine & methenine
aminoacids and (by carboxylation of propionyl Co A arising
in catabolism of) cholesterol side chains, isoleucine & fatty
acids with odd numbers of carbon atoms or directly from
proprionate
(H 4) F o late
(T H F )
-FA (odd numbers)
-Cholesterol side chain L Methyl------ B^— ►Succiny! - Effect of B 12 def. is most pronounced in rapidly
Malonyl MM y
- A A (Valine, Threonine, dividing cells of bone marrow erythropoietic tissue &
CoA Isomerase
isoleucine, Methionine) intestinal mucosa as these need both the N5 Nm
(mutase)
- Propionate____________ methylene & Nm formyl forms of THF for syn thesis o f
n ucleotides required fo r DNA rep lication ®
- In Bn deficiency methyl malonyl accumulates and its urinary - In B 12 deficiency, d/t dysfunction of methionine
excretion provides a means of assessing B 12 nutritional status. synthase methylation of homocysteine by N5 methyl
- Accumulation of unusual fatty acids and their incorporation THF is hampered 1/1 accumulation of both.
in cell membrane of nervous system may account for - Because methylated form cannot be directly converted
neurological manifestations. to other forms of tetrahydro folate (THF), the folate is
- H yper hom ocystein em ia, hom o cystinuria l/t card iov ascu lar trapped in N 5 methyl form, which accumulates. The
d iseases, lik e atherosclerosis, throm bosis, hypertension and level of other form decreases 1/t functional folate
MIQ deficiency (needed for purine & TMP synthesis).
Decreased DNA synthesis disturbs erythropoiesis,
causing immature precursor of RBC to be released in
circulation 1/t megaloblastic /pernicious anemia.
Biochemistry: General ■ 585

• Subacute com bined degeneration (SACD) & Dem yelination


- D /1 methionine deficiency in CNS, there is irreversible degeneration of spinal cord, as a result of failure of methylation
of one arginine residue on myelin basic protein.
Therefore inadequate methylation of phosphotidyl
ethanolamine to phosphatidyl choline 1/t deficient Precursor- -> ll
*. Uracil-
TT—~1 Q
■♦Thiamine ■ DNA (Bone, marrow
formation of myelin sheath of nerves and mucosa of GI tract)
Folic
demyelination. RNA (Nervous
Acid
- N iew eg's hypothesis explain why only B 12 def. is system)
a/w neurological lesions whereas, both B12 and folic acid deficiency cause megaloblastic anemia. This occurs because
folate is only concerned with DNA synthesis, whereas B 12 is required for both DNA & RNA synthesis.
• Both B 12 & fo la te correct m egaloblastic anemia by increasing DNA synthesis ®. Only folate supplements may put some
people at risk as this would rectify (correct) megaloblastic anemia but may hasten development of irreversible nerve
damage.
• Schilling (radioactive lebelled Cobalt-60) test is done to assess B12 deficiency.

68. B i.e. Vitamin C [Ref: Harrison 16/e p- 406-07, 2469]

- Vitam in C is a water soluble vitamin, which is mainly excreted in urine®. Its intake above 100 mg/day results in saturation
of its metabolic capacity and excretion into urine. So large doses (1 gm TDS/81p) are used to acidify urine in preventing
and treating UTI as some drugs act better in acidic urine.
- Vitamin A, D, E, K are fat soluble and so excreted mainly in bile and very minimally in urine.

69. D i.e. Pyridoxal phosphate 70. A i.e. Vitamin C 71. B i.e. Ascorbic acid 72. A i.e. Hydroxyproline

- Collagen most abundant protein of Hydroxylation of Proline Require


mammalian tissue, contains- Glycine & Proline
- Hydroxyproline: is formed by the post - Proline (substrate)
translational hydroxylation of proline by prolyl - a - ketoglutarate
hydroxylase. - Enzyme: Prolyl
- There is no t-RNA capable o f accepting hydroxylase® (Mixed
hydroxyproline or hydroxylysine ® and function oxygenase)
inserting them into an elongating polypeptide - M olecular O 2®
chain. Thus preformed hydroxyproline & - Ascorbate (vitam in C)®
hydroxylysine of ingested food proteins are - Iron (Fe2+)
not incorporated into collagen

73. A i.e. Vitam in K 74. B i.e. Both help in post translational m odification 75. A i.e. Lysine
[Ref: Harper's 28/e P-472-473, 479-80; Lippincott's 5/e p. 377-78, 389-90; Vasudevan 6/e p. 407-10; Devlin 5/e p. 1146]
r
- Both vitam in C and vitam in K are required for post translational modification®.
- Vitam in C is required for p ost translational m odification of proline & lysine in collagen synthesis®.
- V it K is required for post translational m odification of glutam ate residue for generation of clotting factors®.

76. A i.e. Glutamate 77. C i.e. G lutam ate 78. C i.e. K 79. D i.e. Vitam in K
80. B i.e. Carboxylation 81. D i.e. Vitam in K

Vit K function as a C 02
coenzyme fo r carboxylase ® G lutam ate (glutamyl) y Carboxy glutam ate (glutamyl)
enzyme responsible for
post translational Pyruvate carboxylase ®
m odification of blood Vitamin K
clotting factors II, VII, IX, (hydroquinone )®
X and prothrombin. These Precursors of clotting Carboxylation Carboxylated activated
proteins are initially factors 2, 7,9,10 and reaction clotting factors 2, 7, 9 ,1 0 &
synthesized as inactive prothrom bin ®_______ v§) prothrombin______________
precursors containing
glutamyl residuces. Vit K o
activates these clotting WarfannQ
factors by carboxylation o f their glutam ic acid residues ®. Carboxylated glutamyl residues on activated clotting factors
confer them the capacity to bind Ca++ and phospholipid surface.
586 ■ A Complete Review of Short Subjects

82. B i.e. IX and X [Ref: Ganong 23/e 535] 83. A i.e. Factor VII

- Vitamin K dependent clotting factois (VKCF) are


clotting factors II (prothrombin), VII (Proconvertin),
IX (Christmas factor), X (Stuart Prover Factor),
protein C, protein S and protein Z®.
- Liver diseases, Warfarin over dose and mlabsorption
cause combined deficiency of VKCFs (i.e. 2, 7, 9,10,
C ,S & Z) and leads to bleeding.
- Two types of autosomal recessive (AR) disorder i.e.
VKCF deficiency 1 (VKCFD1) is a/w point mutation V itam in K epoxide

in gamma-glutamyl carboxylase gene (GGCX) and


VKCF deficiency 2 (VKCFD-2) is a/w point mutation in vitamin K epoxide reductase gene (VKOR).

84. B i.e. Retinal 85. C i.e. Liver 86. A i.e. Retinol, Retinal, and Retinoic acid

87. A i.e. Lysosome [Ref: O.P. Ghai 6/e p. 121; Harrison 16/e p. 408]

Vitamin D intoxication leads to rupture of lysosomal membraneQ.

88. A i.e. A; D i.e. D [Ref: Harper's 27/e P-492-94]


> ■■ ■ ■ ■■
Vitamin -A intoxication leads to demineralization of bone, hypercalcemia, hyperostosis^.
Vitamin D intoxication leads to hypercalcemia & hyperphosphatemia^

89. A i.e. Potato [Ref: Chatterjea 6/e P-145]


Diet
____________________ I
Plant sources store vitamin A in the form of Diet consisting of following will
carotene Rich sources of carotene are - Tomatoes®, produce deficiency of vitamin A -
Carrots®, Green yellow vegetables®, Spinach®, Maize, Wheat, Rice, Tubers (Potatoes)®
Papaya, Mango, Corn___________________________
BIOCHEMISTRY

Review Notes with High Yield Text


Chapter 1. CARBOHYDRATE METABOLISM: REVIEW NOTES

Energy Production per Molecule of Glucose

Common in both aerobic & anaerobic conditions Reaction occurring in aerobic conditions only
Steps at which energy is utilized (ATP consumption)
NDA+ NADH
\Glucokinase\
G lucose — — — ►Gl ucose- 6- P04 : - 1 ATP Glyceraldehyde — > 1,3 -
3 phosphate Glyceraldehyde biphospho
ATP ADP 3*-P” glycerate
dehydrogenase
D u c to -c C P 1PhosPho fru cto kin a se \ > F m r.tnse l fi : - 1 ATP
~ ~ '\ A biphosphate In absence of oxygen, reoxidation of NADH at
ATP ADP
glyceraldehyde -3P - dehydrogenase stage cannot take
Steps where energy (ATP) is produced
place in electron transport chain.
\Phosphoglycerate kinase ATP gained from 1NADH according to old concept is
1,3 - biphospho 3 Phosphogl- + 2 ATP
glycerate ycerate 3ATP but according to new concept it is 1.5 or 2.5
ADP ATP depending on shuttle. So for 2NADH, 5 ATP will be
I Pyruvate kinase I produced (or 6 a/t old concept)
2 Phosphoe ------- ►(Enol) pyruvate + 2 ATP Because 1 glucose molecule yields 2 molecules of
nolpyruvale
ADP ATP glyceraldehydes 3-phosphate, the overall production of
NADH per molecule of glucose is 2
Net ATP produced in anaerobic condition=4-2=2ATPQ
So for 2 NADH; 5ATP will be produced (or 6 a /1 old
O 2
G l u c o s e ( C 6>
> H exose T r ip le
P h o s p h a t e ----- ► P h o s p h a t e — ► P y r u v a t e ( C j ) Q —
concept)
G l y c o g e n (C o ) (C„)
(G) (C„) j Hence the net ATP gain in glycolysis in Aerobic
conditions is 5 (from NADH) + 4 - 2 = 7 ATP (a/t new
concept) / 6 (from NADH) + 4-2 = 8 ATP (a/t old concept)
Embden-Meyerhof Pathway of Glycolysis
Glycolysis is the cytosolic pathway of all mammalion
cells for the m etab olism o f g lu cose (or glycogen) to Aerobic Condition Anaerobic Condition
pyru vate and lactate®. Under aerobic - Lactate® is a final product
- In glycolysis glucose (C<,) is converted into 3C units condition final - Erythrocytes, which lacks mitochondria (O2
su bstan ces; pyru vate (C3) and la cta te (C3)®. product is supply) completely reliant on glucose as
- All the enzymes of glycolysis are present in cytosol®. pyruvate® their metabolic fuel.
- This is n ot a com plete breakdow n® of glucose as pyruvate further enters mitochondria to completely degraded into CO 2 and
H 20 .
- It is unique in the sense that it can function either aerobically or anaerobically.
• Fructose - 2, 6 Biphosphate is not a part of glycolytic pathways but is formed during glycolysis.
• When the body has abundant supply of glucose:
Fructose 2,6 biphosphate is formed from fructose - 6 PO 4 by the action of phosphofructokinase -2 (PFK-2).
• If Glucose supply is not adequate:
Fructose 2, 6 biphosphate is converted back to Fructose - 6 - PO 4 by PFK -2 which now acts as fructose - 2, 6 biphosphotase
• Phosphofructokinase -2 is a bifunctional enzyme.
• Fructose 2, 6 biphosphate has a unique ro le to p la y in the regulation o f G ly coly sis and Gluconeogenesis®.
- P ositiv e a llosteric effector o f p h osp h ofru ctok in ase -1® (rate limiting enzyme of glycolysis)
- In h ib itor o f Fructose 1, 6 B iphosphatase® (enzyme of Gluconeogenesis)
• At each of 3 points where glycolytic reactions are bypassed by alternative gluconegenic reactions, simultaneous operation of
both pathways would consume ATP without accomplishing any chemical or biological work, ie ATP is hydrolysed and large
amount of chemical energy is dissipated as heat without any useful metabolic work being done. Therefore these uneconomical
bypass processes are called futile cycle. However, substrate cycle is a better name because these provide advantages for
controlling pathways.
That means
Ted Fructose 2, 6 - Biphosphate —» T Glycolysis, -L Gluconeogenesis
4-ed Fructose 2, 6 - Biphosphate —> -I Glycolysis, TGluconeogenesis
590 ■ A Complete Review of Short Subjects

Phosphogfycerate mutw|

(2) 2-Phosphoglycerte

Opposing pathways of Glycolysis & Gluconeogenesis

Oxidase - Peroxidase Test Strips


It is new highly specific method for estimating glucose in blood & urine. Glucose oxidase enzyme produce hydrogen peroxide &
gluconic acid from glucose. And peroxidase catalyzes the reaction of H2O2 with colourlesspotassium iodide to brown iodide. This
produces a colour change, the intensity of which may indicate glucose concentration in some tests (such as Boehringer, Diastix).
Horseradish peroxidase (HRP) are used for histochemistry & ELISA because of their excellent stability, low molecular weight and its
ability to produce chromogenic products (such as peroxidase labelled antibodies).
Glucose + H 20 + 0 2 — — >H 20 2 + Gluconic acid ( 8 -gluconolactone)

Peroxidase Q 'K I

Brown Iodide

Inhibitors of Metabolic Pathway

Kreb's cycle Glycolysis


F l u o r o a c e t a t e inhibits A c o n ita s e Q 1. I o d o - a e t a t e ® inhibits G ly c e r a l d e h y d e 3 -P - d e h y d r o g e n a s e Q
A c o n ita s e
Glyceraldehyde
C itra te ■*-------------------► C is-aco n itate
G ly cerald eh y d e * ^ <^e^yc*roSen££e>i ^ 3 _ b ip h o sp h o g ly cerate
A r s e n it e inhibits a - k e t o g l u t a r a t e d e h y d r o g e n a s e Q
3 p h o sp h ate
a -K e to g lu ta r a te
d e h y d ro g e n a se
2. F lu o r id e inhibits E n o la s eQ
a -k e to g lu ta ra te -*----------------------------- ► Su ccin yl C O A
E n olase
M a l o n a t e inhibits s u c c in a t e d e h y d r o g e n a s e O . 2 p h o sp h o g ly cerate P h o sp h o en o lp y ru v ate
S u ccinate Mg2+/Mn2+
d e h y d ro g e n a se
S u ccin ate ■*-------------------------------- ► F u m a ra te
Biochemistry: Carbohydrate Metabolism ■ 591

Enzyme (Genes) and Pathways regulated by Insulin

Increased Expression
D ecreased Expression

In Glycolysis In Pentose phosphate In Fatty acid synthesis I


- Hexokinase II® pathway (NADPH) - Pyruvate PEP Gluconeogenesis
- Hexokinase IV (i.e. - Glucose-6-phosphate dehydrogenase carboxykinase
glucokinase)® deydrogenase ■ Acetyl CoA Glucose-6- Glucose release
- Phosphofructokinase - 6 - phosphogluconate carboxylase phosphatase to blood
(PFK-1) dehydrogenase ■ Malic enzyme
- Pyruvate kinase (NADPH)
In Fatty acid desaturation
- Stearoyl CoA deydrogenase - ATP citrate lyase
In regulation of glycolysis/
(provides acetyl CoA)
gluconeogenesis
In Triacyl glycerol synthesis ■ Fatty acid synthase
- PFK-2 / FBP ase-2
- Acyl-CoA- glycerol complex
transferase

Hexokinase and Glucokinase

• Hexokinase, which catalyzes the entery of glucose into glycolytic pathway Features Hexokinase Glucokinase
(by converting glucose to glucose 6 P O 4 ) , is a regulatory enzyme. Humans (I-III) (Hexokinase
have 4 isoenzymes (I to IV) IV)
• Predominant isoenzyme type of myocyte (muscle) is hexokinase II > I. It Found in All tissues Adult liver
has low Km and high affinity fo glucose. It normaly acts at or near its including fetal only®
maximal rates (because Km=0.1 is much lower than blood glucose conc. & adult liver
4-5 mM) and is allosterically inhibited by their product glucose 6 - Km Low (0.1 mm)® High (10 mM)®
phosphate Affinity (for High Low
• Predominant hexokinase isoenzyme of liver is hexokinase IV glucose)
(glucokinase) which differs from hexokinase I-III of muscle in 3 respects Stability More Less (labile)
I. Km or glucose concentration at which glucokinase is half saturated (10 phosphorylate Any hexose Glucose
mM) is higher than usual concentration of glucose in blood. Because an (non specific) (specific)
efficient glucose trasporter (GLUT-2) rapidly equilibrates glucose Allosteric Glucose - 6 - Fructose-6 -
concentration in cytoplasm (of liver cell) & blood, the high km of inhibition P 0 4® phasphate®
glucokinase allows its direct regulation by blood glucose level. Glucose No change Increased
feeding
High blood glucose level Low blood glucose level Fasting /DM Not much Decreased
(After carbohydrate rich meal) (after fasting) change
' I I GH, Inhibit Inhibit
Excess glucose transported into Glucose concentration in Glucocortico-
hepatocytes & converted to glucose - 6 hepatocyte is low due to its low steroids
P O 4by glucokinase. Because it is only affinity (high km) and the glucose Insulin Induce Induce
half saturated at 10 mM glucose, its generated by gluconeogenesis Function Make glucose Clear glucose
activity continues to rise as glucose leaves the cell before being available to from blood after
concentration rises to >10 mM trapped by phosphorylation. tissues for meals (leves >
So its main function is to clear glucose So it glucose avaiiM e to oxidation at 100 mg/dl)
from blood at levels > 1 0 0 mg/dl®. tlssue at iower biood gtucose lower blood
glucose level
levels®.
2. Glucokinase is not inhibited by glucose - 6 - PO4® and it can therefore continue to operate when the accumulation of
glucose - 6- PO4 completely inhibits hexokinase I- III.
3. Glucokinase is inhibited by reversible binding of regulatory protein specific to liver (and more tightly in presence of
allosteric effector fructose - 6- phosphate). Glucose competes with fructose 6 -PO4 for binding & cause dissociation of
regulatory protein from hexokinase (in nucleus), relieving the inhibition. Whereas during fast, (glucose <5 mM), fructose 6 -
PO4 triggers inhibition of glucokinase by regulatory protein, so liver does not complete with other organs for scarce glucose
* Glucose enter into cells by GLUT - 2 in the liver under control of insulin. GLUT -2 has higher km value for glucose hence
transport glucose only when blood sugar is high.
★ GLUT-4 Work in muscle cells and adipocytes.
592 ■ A Complete Review of Short Subjects

Shuttle System

Mitochondrial (inner) membrane is impermeable to NADH. The NADH generated in glycolysis are transported from
cytoplasm to mitochondria for oxidation by shuttles (mainly matale)

____________Malate shuttle ^ .________ Glycerophosphate shuttle______________________


- This is more common and universal - This is present in some tissue (e.g. brain, white skeletal muscle) and absent in
shuttle used mainly in liver, kidney & others (e.g. Heart muscle)
heart - FAD<3 dependent so only 2 ATP® (old)/1.5 ATP (new) are produced.
- Oxidation of reduced NADH + H* in - When body utilizes this system, net ATP production is 36 ATP® (old)/30
mitochondria yields 3 ATP® (old)/ 2.5 ATP (new.)
ATP (new)
- When body utilized malate shuttle, net Malate shuttle
Mitochondrial membrane
ATP production by glycolysis - TCA
cycle per glucose molecule oxidized
will be 38 ATP®/32 ATP (new).
- The complexity of this system is d/t
impermeability of mitochondrial
membrane to O.AA, which must react
with glutamate & transaminate to
aspartate and a - ketoglutarate before
transport through mitochondrial
membrane & reconstitution to O.A.A
in cytosol.___________________________

Glycerophosphate shuttle
M ito c h o n d ria l m e m b ra n e
C yto so l M ito c h o n d rio n

NAD a -g ly c e ro p h o s p h a te - -► a -g ly c e ro p h o s p h a te FAD

a -g ly c e ro -d e h y d r o g e n s e s a -g ly c e ro -P -d e h y d r o g e n s e s

NADH
D ih y d r o x y a c e to n e -P <- D ih y d ro x y a c e to n e -P F A D .H ,
+H
R e s p ira to ry c h a in

Fate of Pyruvate

• 7 reversible reactions of glycolysis are used in synthesis Phosphoenolpyruvate


of glucose from lactate to pyruvate (i.e.
Phosphoenolpyruvate | Pyruvate kinase ]
gluconeogenesis). But the 3 irreversible reactions must G D P< carboxykinase 6
be circumvented by 4 alternate reactions that Pyruvate
enegetically favour the synthesis of glucose. And these
reactions are unique to gluconeogenesis. The first road GTP— '
V V
IT
block in synthesis of glucose from pyruvate
Pyruvate
(gluconeogenesis) is irreversible conversion o f PEP to O xaloacetate
B io
i otin
t i n ]j | Pyruvate carboxylase Q \
pyru vate by pyru vate kin ase in glycolysis® . In N ADH -
A D P -^ t C02
gluconeogenesis, pyruvate is first carboxylated by Oxaloacetate
O x ;'
pyruvate carboxylase to OAA, which is then converted
NADH
to PEP by PEP - carboxykinase. NAD+ -

• Conversion of phosphoenol pyruvate(PEP) to w. NAD +


NA
M alate Malate Citrate
pyruvate is one of the three irreversible step (barrier)
in simple reversible glycolysis. So this conversion is
achieved by 2 enzymes.
V Su ccin yl C o A
J iM l T O C H O N D R iA l

- Pyruvate carboxylase: Pyruvate has to be first converted to oxaloacetate in mitochondria.


- Phosphoenol pyruvate carboxykinase: OAA is then converted to phosphoenolpyruvate in cytosol
Biochemistry: Carbohydrate Metabolism ■ 593

Fate of Pyruvate (End product of glycolysis)


1---------- 1
Oxidative decarboxylation by Carboxylation by biotin Anaerobic glycolysis occurs in Transamination by
pyruvate dehydrogenase complex, dependent pyruvate carboxylase tissue which are poorly alanine amino
in tissues with high oxidative converts pyruvate to vascularized/or lack transferase
capacity such as cardiac muscle, oxaloacetate (OAA). This mitochondria such as exercising (ALT/SGPT)
irreversibly converts pyruvate replenishes the citric acid muscle, cornea & lens o f eye, kidney converts pyruvate
to acetyl CoA (a major fuel of cycle intermediates and medulla, testes, leucocytes & RBC. to alanine®
TCA cycle and a building provides substrate for Lactate formed by action of lactate
block for fatty acid synthesis ) gluconeogenesis. dehydrogenase is the final product.

Arsenic (As) poisoning

Arsenite (trivalent As) inhibits Arsenate (Pentavalent As) competing with inorganic phosphate as a substrate
(dehydrogenases) enzymes that require (also occurs)
lipoic acid co-enzyme (primary - Arsenate competing as a substrate for glyceraldehydes -3- phosphate
mechanism) dehydrogenase, forms a complex that spontaneously hydrolyzes to
E2 of pyruvate dehydrogenase (PDH) form 3-phosphoglycerate. By bypassing the synthesis of 1 ,3-BPG and
complex® phosphate transfer from it, the cell is usually deprived of energy
a-ketoglutarate dehydrogenase® usually obtained from glycolysis. So it inhibits net ATP & NADH
Branched chain a- ketoacid production without inhibiting the pathway itself.
dehydrogenase® - As also replaces phosphate on FI domain of ATP synthase resulting in
formation of ADP- arsenate that is rapidly hydrolyzed.

Citric Acid Cycle/Krebb’s Cycle

Citric acid cycle is a series of reaction


in mitochondria® that oxidize acetyl Acetyl-CoA(C2)
Malate
CO-A and reduce coenzyme that Oxaloacetate (C«) Citrate synthase
Dehydrogenase
upon reoxidation are linked to the *■ CoA-SH
NADH+FT FLO
formation of ATP.
Malate
(C „) NAD+
Fumarase
Mn - 1"C , C is, I, 0 , q, S;, F, M , 0"| /
Fumarate (C4)
fluoro
acetate
❖ In bracket carbon content of each
>FADH,
com pound is written
❖ Release of C O 2 1 /t decrease in one Succinate Dehydrogenase
Aconitase
carbon atom •FAD
❖ Two irreversible steps of TCA are MalonateQ
- O xaloacetate citrate
- a-ketoglu tarate succinyl CoA
❖ 0 indicates inhibitors of TCA Isocitrate
Succinate Dehydrogenase
❖ New concept
thiokinase "
1N A D H = 2.5 ATP
1 FA D H 2 = 1.5 A TP Succlnyl-CoA(Q)l NADH+FP

a-Ketoglutarate NAD Arsenite®


Dehydrogenase Isocitrate
Dehydrogenase
q-Ketoglutafate (Cs) I*
594 ■ A Complete Review of Short Subjects

Inhibitors of TCA (Citric Acid /Kreb’s) Cycle Inhibitor of TCA Enzyme Inhibition Type
• Aconitase which isomerizes citrate to isocitrate, is cycle
non competitively inhibited by a rat p oison Fluoroacetate Aconitase® Non-competitive
flu oroacetate® . Fluoroacetate (with acetyl CoA) is
(Rat poison)
converted to fluoroacetyl CoA, which condenses
w ith o x a lo a ceta te to form® fluoro-citrate - a potent Arsenite (Trivalent Alpha- Non-competitive
non competitive inhibitor of aconitase - resulting in Arsenic) ketoglutarate
citrate accumulation. dehydrogenase
• Alpha ketoglutarate dehydrogenase enzyme (which Malonate Succinate Competitive
converts a-ketoglutarate to succinyl CoA by oxidative dehydrogenase®
decarboxylation) is non-competitively inhibited by
Ammonia B y depletin g A lpha-ketoglu tarate®
arsenite (trivalent arsenic) causing the substrate
Intoxication
alpha ketoglutarate to accumulate. Arsenite forms a
stable complex with thiol group oflipoic acid making it
unavailable to serve as a coenzyme. Acetyl CoA Oxaloacetate (OAA)
• Oxidation of succinate to fumarate by succinate
dehydrogenase is competitively inhibited by
malonate. Succinate dehydrogenase is the only
enzyme of TCA cycle that is embedded in inner
mitochondrial membrane and so also functions as
complex II of electron transport chain. Acotinase
„ .. Isocitrate
• Hyperammonemia (TN H 3) usually due to urea cycle
defect inhibits TCA cycle by depletin g a-
ketoglutarate® .

4-B Complex Vitamins are Essential in the TCA (Citric Acid) Cycle
....... , „,l , ....------------ — 1
RiboflavinQ (B2) Niacin T hiam in eO (Bi) Pentothenic add
In the form of FAD, a In the form of NAD, the electron As thiamine diphosphate As part of coenzyme
cofactor in acceptor coenzyme for three (TDP/TPP), coenzyme for A, the cofactor
- a- ketoglutarate dehydrogenase decarboxylation in a- attached to active
dehydrogenase complex - Isocitrate dehydrogenase ketoglutarate carboxylic acid
- succinate dehydrogenase. - a- ketoglutarate dehydrogenase dehydrogenase reaction. residue such as acetyl
- Malate dehydrogenase CoA & succinyl CoA.

Energy Production in TCA Cycle


I ll
I
NADH Production FADH2 Production ATP /GTP synthesis
• 3 m olecu les o f NADH® are produced in 2 m olecu le o f FADH 2® is produced in 1 GTP/ATP® is produced by substrate
one turn of cycle in the following the following reaction level phosphorylation [ It is production
reaction - S u ccin ate ■*------------------- ► F u m a ra te
of ATP 'at the substrate level' without
- Iso citrate ■*-------------- *-O x a lo su ccin a te
s u c c in a t e the p articip atio n o f electron transport
Is o c itr a te d eh y d rog en ase® chain.]®
d eh y d rog en ase®
S u ccinyl C oA •S u ccinate
Each FADH 2 gives 2ATP® (old concept) S u c c in a t e
- a - k eto g lu tarate Su ccinyl C o A
/ 1.5 ATP (new concept) t h io k in a s e ®
a - k e t o g lu ta r a te
d eh y d rog en ase® O ver a ll 12 ATP® (old) / 10 ATP (new) are formed per turn of the citric acid

■M alate ■OAA
ATP Production
M a la te
Substrate New value Old value
D ehydrogen ase®
NADH 2.5 3
Each NADH gives 3 ATP (old concept) / FADH 1.5 2
2.5 ATP (new concept). So NADH Glucose 32 38
Acetyl CoA 10 12
oxidation in one TCA cycle provides
Palmitate 106 129
9ATPO (old) / 7.5 ATP (new).
Biochemistry: Carbohydrate Metabolism ■ 595

Regulation of Citric Acid Cycle by

Availability of ADP® Supply of NAD+® Regulation of selective enzyme activity


- TADP -> Stimulates TCA cycle - TNAD+ —» Stimulates cycle Citrate synthase
- TATP/ iA D P —» cycle depressed - TNADH -^Inhibits cycle Iso citra te dehydrogenaseQ
a - ketoglutarate dehydrogenase complex

In addition to above
Succinate dehydrogenase is inhibited by OAA, and availabily of OAA depends
on malate dehydrogenase which is controlled by NADH / NAD+ ratio.

Involvement of the Citric Acid Cycle in Transamination and Gluconeogensis

Valine, Isoleucine and H y d ro x y p ro lin e 'l L a c ta te + ► A c e to c e ta te


Methionine® produce S e r in e I
succinyl COA which is an T h r e o n in e [ L e u c in e , P h e n y la la n in e ,
important component of G ly c in e J Transaminase!
T y ro s in e , T ry p r o p h a n , L y s in e

TCA cycle. T r y p to p h a n - ► A la n in e ■«- -W P y ru v a te A c e t y l- C o A I

[Mn: Successful VIM ] P h o s p h o e n o lp y r u v a te


c a r b o x y k in a s e L e u c in e ,
_ P h o s p h o e n o l- Is o le u c in e
G lu c o s e
p y ru v a e
Oxaloacetate T ry p to p h a n

A s p a r a g in e ► A s p a rta te
T r a n s a m in a s e '
T y ro s in e
p h e n y la la n in e T

A s p a rta te C itr u lin e

C itr a te
A r g in o s u c c in a te
★ The bold arrows indicate the
main pathway of Is o le u c in e 1
-► I S u c c in y l-C o A | A rg in in e
gluconeogenesis m e th io n in e r
v a lin e J

| a - k e t o g lu t a r a te
P r o p io n a te

Transaminase
H is tid in e "|
P r o lin e I -► G lu t a m a t e
G lu ta m in e ]
A rg in in e

Glucose - Alanine Cycle


- Alanine transportes toxic ammonia (amino group) from skeleton muscles to liver via a pathway called glucose - alanine
cycle.
- It is important to remember that resting muscles use fatty acids as its major fuel source. Whereas exercising muscles
initially use glycogen stores (by anaerobic glycolysis & convert it to lactate / pyruvate) and then use free fatty acids
provided by mobilization of TAG from adipose tissue (when glycogen reserves are depleted) so these aminoacids are not
directly utilized as fuel source by muscles.
- During first few days of fasting, there is ra p id breakd ow n o f m uscle p rotein , p rovidin g a m in o acid s (alanine & glu tam in e
m ainly) th a t are used by liver fo r gluconeognesis® . Because muscles do not have glucagon receptors, muscle proteolysis is initiated
by fall in insulin and is sustained by the rise in glucocorticoids. Alanine & glutamine are produced by catabolism of branched
chain aminoacids. By sev eral w eek s o f fastin g , ra te o f m uscle p ro teo ly sis d ecreases p arallelin g a decline in the need fo r
glu cose a s a fu el fo r the brain w hich h as started using keton e b od ies as a source o f energyQ.
■In tissues that degrade aminoacid for fuel (mainly skeletal muscle), amino group is collected in form of L-glutamate (d /1
transamination)
596 ■ A Complete Review of Short Subjects

- This glutamate (containing toxic a-amino group) can be transported to liver by converting it to glutamine or it can transfer
its a-amino group to pyruvate, a easily available product of muscle glycolysis by the action of alanine aminotransferase
(ALT/SGPT). The alanine so formed passes into blood to liver.
- In liver cell cytosol alanine aminotransferase (ALT/SGPT) transfers amino group from alanine to a-ketoglutarate forming
pyruvate (which enters gluconeogenesis) and glutamate (under go oxidative deamination)
- Vigrously contracting muscles operate anerobically producing pyruvate & lactate (from glycolysis) and NH 3 (from aa break
down). Alanine-glucose cycle in concert with Cori cycle send these products to liver where pyruvate & lactate are
incorporated into glucose (by gluconeogenesis and returned back to muscle) and NH 3 is converted to urea for excretion. So
the energetic burden of gluconeogenesis is imposed on liver and all available ATP in muscle is devoted to muscle
contraction.
- In the fasting state, the output of alanine from skeleton muscle is in far excess of its concentration in the muscle proteins
that are being catabolized. Because it is also formed by transamination of pyruvate produced by glycolysis of muscle
glycogen. Alanine is exported to the liver, where it is transaminated back to pyruvate, which serves as a substrate fo r
gluconeogenesisQ. The use of alanine to transport ammonia from skeleton muscles to liver is an example of intrinsic
economy of living organisms. Thus glucose - alanine cycle provides an indirect way of utilizing muscle glycogen to
maintain blood glucose in fasting state. The ATP required for hepatic synthesis of glucose from pyruvate is derived from
the oxidation of fatty acids.

Reversible Oxidative Deamination: (L) Glutamate Dehydrogenase (GDH)

Transamination (1st step in catabolism of aa) transfers amino group of many a-amino acids to a-keto glutarate, thus converting it
to L-glutamate. The reaction predominatly occurs in cytosol of hepatocytes (liver cells).
L glutamate is transported from cytosol to mitochondria of hepatocyte. Only (predominantly) mitochondrial matrix contain
glutam ate dehydrogenase which oxidatively deaminates glutam ate to a-ketoglutarate (a-keto acid) and free am onia Q (a-
amino group is liberated as NH3). A/t vasudevan this reaction occurs in only liver; a/t Lippincott 252 this occurs in liver &
kidney.
GDH, a Zn containing metalloenzyme of mitochondrial matrix, is a tetramer (i.e. has 4 polypeptide chain each containing 1
Zn molecule). It is the only enzyme that can use either NAD+ or NADP+ as coenzyme Q (or acceptor of reducing equivalent).
NAD+ is used in oxidative deamination and NADPH is used in reductive amination (i.e. reverse reaction).
Oxidative deamination is a freely reversible reaction Q. The direction depends on relative concentration of substrate,
product (glutamate, a-ketoglutarate, N H 3) and ratio of oxidized to reduced coenzyme. So in well fed state glutamate levels
are high resulting in oxidative deamination, whereas in starvation it synthesize amino acids by reductive amination.

a-Ketoglutarate + N H 3
- R e d u ctiv e a m in a tio n — — O x id a tiv e d e a m in a tio n

Liver GDH is allosterically activated by ADP and inhbited by ATP, GTP and NADH®. Thus when energy levels are low in
cell, amino acids are catabolized by GDH for energy production. Mutations that alter GTP binding site cause permanent
activation of GDH, & result in hyper insulinism-hyper-ammonia syndrome characterized by TNH3 and -Iglucose in blood.
Trans deamination is sequential action of transamination f/b oxidative deamination, and provide a pathway where by the
amino groups of most amino acids can be released as NH3. Transamination takes place in cytoplasm of all cells of body (mainly
liver, kidney, muscle, heart etc) and form glutamate. Toxic glutamate is transported to liver as non-toxic glutamine (most
tissue) and alanine (from muscles), and finally oxidatively deaminated in mitochondria of liver (mainly).
Minor pathways of deamination are L/D/Mono-amino oxidase; dehydrase, desulfhydrase and histidase (non oxidative) etc.

High Energy Compounds Low Energy Compounds

These compounds release at least 7 Kcal /mol (i.e. > 7 K cal/mol or > 25 These liberate < 7 Kcal/mol on hydrolysis at
KJ/mol) on hydrolysis; they have large A G° (free energy of hydrolysis) from 7- pH7 (i.e. lower than energy liberated by
15 Kcal/mol. The high energy bond is indicated by a squiggle bond (~). These hydrolysis of ATP to ADP + Pi)
include
Biochemistry: Carbohydrate Metabolism ■ 597

AG° Energy These include


High Energy Compounds (Kcal/ release
mol) (KJ/mol) Low Energy Compounds A G°
• Phosphate Compounds (Kcal/mol)
1. Pyrophosphates: Nucleotides like ATP, GTP, UTP, 1. A D P -A M P + Pi 6.5
UDP-glucose 2. Glucose 1 phosphate 5.0
- ATP -» ADP + Pi 7.3 30.5 3. Fructose 6 phosphate 4.0
- ATP -> AMP + PPi 10.7 45.6 4. Glucose 6 phosphate 3.5
2. Phosphagens: like Creatine phosphate (CP) or 5. Glycerol 3 phosphate 2.2
phosphocreatine 10.5Q 43.1 ★ All valuses of A G° and energy released
3. Acyl phosphates: like 1 , 3-Biphosphoglycerate (BPG) 10.1 49.4 are negative (-)
4. Enol phosphates like phosphoenol pyruvate (PEP) 14.8 61.9
5. Carbamoyl phosphate 12.3 51.4
6. Amino acyl adenylate (amino acyl AMP/cycline AMP) 12.0 50.0
7. Acetyl phosphate 10.3
8. Arginine phosphate 10.0
9. Inorganic pyrophosphate 8.0
» Sulfur Compounds
10. Sulfonium compounds like S- adenosyl methionine 8.0
(SAM)
11. Thioester CoA derivatives like Acetyl CoA, Succinyl 7.7
CoA, Fatty acyl CoA, HMG CoA
12. Adenosine phosphosulfate (active sulfate)

Redox Potential (Eo) & Electron Transport Chain (ETC) R e d o x P air R edox
p o te n tia l
Mitociondrial ETC consists of a series of sequentially acting electron carriers, accepting &
(E ° v o lts )
donating either 1 or 2 electrons. Free energy is released as electrons are transferred from
F erred o xin (F e *3 /F e * 2) - 0 .4 3 2
an electron donor (reducing agent /reductant) to an electron acceptor (oxidizing agent /
2 H * / H 2 (at p H 7) - 0 .4 1 4
oxidant). 3 types of electron transfer occur and term reducing equivalent is used to
a -K e to g lu ta r a te -0 .3 8
designate a single electron equivalent in any form.___________________________________
/Is o c itra te
- Direct transfer of electrons (e-) to cytochromes for reduction of Fe3+ to Fe2+
A c e to a c e ta te / p - 0 .3 5
- Transfer as a hydrogen atom, (H+ + e* = «H) to FMN, FAD and coenzyme Q
H y d ro x y b u ty rate
(Ubiquinone)
- Transfer as hydride ion (:H ) bearing 2 electrons to NAD*______________________ N A D P+ / N A D PH - 0 .3 2 4

• Oxidation (loss of e-) of one substance is always accompanied by reduction (gain of e-) of N A D +/ NADH - 0 .3 2 0

another substance. For example oxidation of NADH to NAD* is accompanied by (a/b) L ip o ate (o x / red) - 0 .2 9
reduction of FMN to FMNH 2. So oxidation - reduction reactions are sum of 2 separate G lu tath io n e (o x /r e d ) - 0 .2 3
half reactions one an oxidation & other a reduction reaction. Thus NAD+ /NADH 2 form a FA D / FA D H 2 - 0 .2 2
redox pair and FMN /FMNH 2 forms other.
P y ru v a te / L actate - 0 .1 9
• Standard recuction potential /Redox potential (E°) in volts, is a quantitative measurement
O x a lo a c e ta te / M a la te - 0 .1 7
of the tendency of a redox pair (couple) to donate or accept electron under standard potential. The
pairs are ordered from most negative to most positive E° i.e. is in increasing order of E°. 2 H * / Hz (at P H 0) 0

More negative the E°, the greater is the tendency of reductant member of that pair to lose F u m a ra te / S u ccin a te + 0 .0 3
electron. More positive the greater is the tendency of oxidant member of that pair to U b iq u in o n e / + 0 .0 4 5
accept electron. So electron flow spontaneously from lower to higher (i.e. more negative to more U b iq u in o l

positive) redox potential. b + 0 .0 7 7


• The change in free energy (A G°) is directly related to A E° (i.e. E° of electron accepting F e 3* / F e 2* in C, + 0.2 2
pair - E° of electron donating pair). The standard free energy required for C y to c h ro m e s C + 0 .2 5 4
phosphorylation of ADP to ATP is + 7.3 Kcal/mol.____________
a + 0 .2 9
NADH FM N
A G = n F A E° 33 + 0.35
Eo = - 0 .3 2 V 1 Eo = - 0 .2 2 V F e *3 / Fe* + 0.771
F = F a ra d a y co n sta n t
O 2 / H 2O 1 + 0 .8 2
(23.1 K C a l/V o lt-m o l) N AD * FM N H 2
n = N u m b e rs of e - transferred
(1 for cy to ch ro m e , 2 for N A D H , F A D H 2 and C o en zy m eQ )
598 ■ A Complete Review of Short Subjects

T ra n sp o rt of a p air of e- from N A D H (co m p le x I) to O 2 via E T C p ro d u ces 5 2 .5 8 K cal an d so p ro d u ce 3 A T P s from 3 A D P & 3 Pi (3 x 7.3 = 21.9 K
C a l/m o l) p e r O ato m re d u ce d . H en ce F : 0 ra tio is 3 :1 for N A D H (an d 2:1 for F A D H 2 b ecau se co m p lex I is b y p assed ). T he rem ain in g e n erg y is
used in an cillary reaction s o r released as heat. B u t n o w recen t stu d ies h av e sh ow n that 2.5 A T P is p ro d u ce d p e r 0.5 m ol (1 / 2 O 2) = p e r ato m of O
from N A D H ( P :0 ratio = 2.5 :1 ). A n d F A D H 2 gen e ra te 1.5 A T P ( P :0 ratio = 1.5:1)

____________________________ Electron Transport Chain (ETCyResplratory Chain_____________________________


• Energy rich molecules like glucose are ultimately metabolized to CO 2 and water. The metabolic intermediates of these
oxidation reactions donate electrons to NAD /FAD to form energy rich coenzymes NADH/FADH2.
• These energy rich reduced coenzymes, in turn, donate electrons to a specialized series of electron carriers/donors arranged aftredox
potential (E°) - collectively called ETC.
• ETC is present in inner mitochondrial membrane & is the final common pathway in aerobic cells by which electrons obtained
from different fuels of body flow to oxygen. This requirement of O 2 makes the ETC the respiratory chain, which accounts for
the greatest portion of body's use for oxygen.
• Electrons flow through the ETC through a redox span of 1.1 volts from NAD+ / NADH to O 2 /H2O
• The components of respiratory chain contain 4 large protein complexes (initial 2 are flavoproteins & last 2 are cytochromes)and
2 mobile electron carriers - coenzyme Q & cytochrome C.
• The chain reaction of e- transport starts with formation of NADH (by various substrate dehydrogenases removing hydrogen
atom as in TCA cycle - isocitrate / a-ketoglutarate / malate - dehydrogenase). Transfer of hydride ion (:H~, bearing 2
electrons) and a free proton (H+) to NAD+ 1/t formation of NADH (having 2 e-) plus a free proton (H+). ETC consists of

C o m p o n e n t of ETC P ro s th e tic F e a tu re s
G ro u p / E lectro n flo w
C o m p le x I: F M N , non h em e iron - It is a larg e L sh ap ed 42 sub unit p rotein w h ich accep ts 2e- & 1 H * from N A D H an d transfer it
NADH su lfu r p rotein s (F e S ) to C o Q
d eh y d ro g en ase/ - 12 K cal/ m ol of released e n e rg y is utilized to p u m p 4 p ro to n s (H +)G ou t of m ito ch o n d ria into
N A D H -) FM N
N A D H -U b iq u in o n e in ter m em b ran o u s sp ace.
i
(C o Q )
o x id o r e d u c ta se / Fes

N A D H -C o Q i
re d u c ta se C oQ

C o m p le x II: FA D , FeS - It is a sm all 4 su b u n it p ro tein w h ich accep ts e- from su ccin ate & tran sfer it to C oQ .
S u c c in a te - It is n o t a p ro to n p u m p as en e rg y liberated is not en o u g h to p u m p p roton s.
d e h y d ro g e n a s e - It also con tain h e m e b th at is n o t in d irect p ath of e transfer b u t serv e to re d u c e the freq uency
/ Su ccin a te - Q - S u c cin a te -) FA D w ith w h ich e~ le a k o u t o f sy s te m , m o v in g from su ccin ate to m o lecu lar O 2 to p ro d u c e reactive
R e d u c ta s e 1 o x y g en species
Fes
1
Z oQ
C oenzym e Q - T his o n ly n o n p r o te in m e m b er o f ET C is a lipid soluble benzoquinone with long hydrophobic isoprenoid tail (side chain).
(C o Q )/ U b iq u in o n e/ Q - It is called ubiqu in one as it is ubiquitous (present everywhere) quinone derivative. Menaquinone (of bacteria) & plastoquinine
(of chloroplast) he v e a n a lo g o u s roles
- It can a cce p t 1 0 2 electron s to b eco m e sem iq u in on e o r ubiquinol resp ectively. So it can a ct at the ju nction of 2 e- d o n or
& l e - a cce p to r ((2 cycle).
- B ecau se it is sm ; ill & h y d ro p h o b ic, Q is a m o b ile c a rrie r freely diffusible w ith in the lipid b ilayer of in ner
m itoch on d rial rrlem b ran e an d can shu ttle b / w o th er less m obile e- carriers,
- It has a cen tral r ole in cou p lin g e- flow to p ro to n (H + ) m o v m en t as it carries b o th e- & H+.
- F o r tr a n s fe rr in g to c o m p le x I I I it a c c e p ts h y d r o g e n a to m (e- & H + ) fro m ,
1 . F M N H 2 p ro d u c ed in c o m p le x I (fro m FM N )
2. F A D H 2 p ro d u ct ;d in c o m p le x II
3. F A D H 2 p ro d u ct ;d in (3-oxidation b y a c y l C o A d e h y d ro g e n a s e (th ro u g h E T F - u b iq u in o n e - o x id o r e d u c ta s e ) and by
g ly c e r o l 3 p h o s p h a te d e h y d ro g e n a s e . T hese b y p ass co m p lex II & I.
C o m p le x I I I: F e S , T e m e group - It tran sfers electro n from C o Q to c y to ch ro m e C . - 10 K c a l/m o l of released e n e rg y is utilized to
C y to c h r o m e b C i (a p o rphyrir ring p u m p 4 proto nsQ o u t of m ito ch o n d ria in to in ter m em b ran o u s space.
c o m p le x & iro n, whic h is & H + tran sp o rt in co m p lex III & in volves c y to c h ro m e b i, bH (b 5 6 2 , b 5 6 6 ),
- Q c y c le c o u p les e-
/ U b iq u in o n e re v e r sibly - S (an u n u su al iron sulfur p rotein in w h ich on e of the iron ato m is
c y to c h ro m e C i a n d R ie s k e F e
c y to c h ro m e C o x id o ­ convt ;rte d fr om linked to 2 histidine -S H rath er than 2 cystein e- SH g r o u p s ) , o xid atio n of Q H 2 (ubiquinol) to Q
re d u c ta s e / Fe3* t 0 F e2+ 1o rm ) (u biq uinon e) is co u p led to red u ctio n of 2 m olecu les of c y to ch ro m e via c y to ch ro m e C
C y to c h ro m e
re d u c ta se Q -> C y t b
i
Cyt C ,
1
C y tC
Biochemistry: Carbohydrate Metabolism ■ 599

Cytochrome c - Soluble carrier protein of intermembrane space


- After its single heme group accepts an electron from complex III, cytochrome c moves to complex IV to donate
electron to a binuclear copper centre___________________________________________________________________
Comple IV 2 Heme groups - It transfer electrons from cytochrome C to the final acceptor O2 converting (reducing) it to water
Cytochrome c (of cyt a, 83), (HzO).
oxidase /cytochrome 2 copper io n s ol - O2 remains tightly bound to complex IV (as) until it is fully reduced to H2O, this minimizes the
oxidase Cua, Cub release of ROS.
- It pumps 2 proton (H+) out into inter membrane space for every pair of electron passing down the
C y tC - CuA chain by complex IV
i
Cyta
i
Cyt a3
I
CuB
1
02

Oxidative Phosphorylation

• As electrons (e ) are transported down the ETC, they lose much of their free energy. Some part of this energy is captured &
stored as ATP (from ADP + Pi) in a process called oxidative phosphorylation. The remainder (not trapped as ATP) energy is
used for ancilliary reactions like Ca+2 transport into mitochondria & to generate heat.
• Electron transfer down the ETC is energetically favoured because NADH is a strong electron (e ) donor and O 2 an avid
acceptor.
• M itchelle- chemiosmotic hypothesis of proton pump & ATP synthase
- It states that ETC is responsible for ATP production b/o electro-pH gradient produced d /1 associated proton pumping (i.e.
proton pump is a link b/w ETC and oxidative phosphorylation).
- As we know, electron transport through complex I, III and IV also transports (pumps) 4 ,4 and 2 protons (H+) respectively
across inner mitochondrial membrane from matrix to inter membrane space.
- This associated proton transport creates an electrical gradient (outside is positive relative to inside) of 0.15-0.20V; and a pH
gradient (outside pH is lower than inside d/t more H+ ion concentration) of 0.75 -1.4. This generates a proton motive force
(PMF) of 0.224V corresponding to a free energy change of 5.2 Kcal/mol of protons.
- The energy generated by proton gradient is enough to synthesize ATP by ATP synthase (complex V). So the proton gradient is
a link that couples oxidation (ETC) to phosphorylation (ATP synthesis)
- Complex V: ATP synthase (also k/a F 1/F0 ATPase because it can also catalyze hydrolysis of ATP to ADP + Pi) has Fo and Fi
subunits. F0 unit spans inner mitochondrial membrane and serves as proton channel, whereas FI unit phosphorylates ADP to
ATP.
- Protons that have been pumped out to inter membrane space, creating electrical and pH gradient, they reenter the
mitochondrial matrix by passing through a proton channel in F0 domain of ATP synthase (complex V).
- This re-entery of protons rotates Fo unit thus producing conformational changes in extra membranous Fj domain, and at the same
time, dissipating the pH and electrical gradients. The conformational changes of Fi unit allow phosphorylation of ADP + Pi to
ATP, and release ATP.
• Paul Boyer's Binding Change mechanism of rotational catalysis
- It states that ATP synthesis by proton gradient is comparable to a 'water driven hammer minting coin'. Fo is wheel, flow of proton
is water fall, and structural changes in Fi 1/t ATP coin being minted for each turn of wheel.
- 3 Beta (catalytic sites) sub units of FI has 3 conformational states

1. L state binds to substrate & product with with loose affinity


2. T state binds substrate tightly & catalyzes ATP synthesis
3. O state is open and does not bind to substrate or product; i.e. release product (ATP)

- For complete rotation of Fi unit through 3 states 10 protons are translocated. As the ATP is released, the original conformation
of enzyme (Fi) is assumed
• So the energy is actually required for producing conformational change (not for ATP synthesis). And the energy surplus
produced by proton gradient is stored as chemical energy ATP.
600 ■ A Complete Review of Short Subjects

________________________________ Mitchell’s Chemi-Osmotic Theory______ 1 Z _____ I


M itchell's chemiosmotic theoryQ postulates that energy from oxidation of respiratory chain components is coupled to the
translocation of hydrogen ions (proton, H+) from the inside to outside of inner mitochondria membrane Respiratory chain
com plexes I, I II & IV® acts as a proton pump. Inner mitochondrial membrane is impermeable to ions in general but particularly
to protons which accumulates outside the membrane creating a electro- chemical potential difference (A(0.H+)
The A|J.H+ is used to drive a membrane located ATP synthase which functions as a rotary motor to form ATP.

Fo Subunit complex Fi Subunit complex


- A membrane protein complex which forms proton channel® - Composed of 9 polypeptide chains (3a, 3p, ly, 1 a & le)
- It is a disk of “C" protein subunit project into matrix and contain phosphorylation mechanism
- Attached to it is "bent axle" formed by Y-subunit & e- subunit® - 3 a & 3- f i subunits, w hich are fix ed to the m em brane and
- Protons passing through the disk of "C " units cause it & d o n ot rotate®
attached Y- unit to rotate - The Y- subunit fits inside Fi sub complex
- Uncoupling agent dinitrophenol allow leakage of H+ across the - ADP & Pi are taken up sequentially by the fi subunits to form
membrane, thus collapsing the electrochemical proton gradient. ATP, which is expelled as the rotating Y subunite squeeze
Oligomycin specifically blocks conduction of H* through Fo subunit each P subunit in turn. Thus 3 ATP m olecu les are
gen erated p er rotation®.
★ Ionophores are lipophilic molecules that complex specific cations & facilitate their transport eg valinomycin (K+), dinitrophenol.
★ Familial infantile mitochondrial myopathy & renal dysfunctions involves severe diminution or absence of most oxido
reductases of the respiratory chain. MELAS (mitochondiral encephalopathy lactic acidosis & stroke) is caused by mutation in
mitochondrial DNA & is d/t NADH: ubiquinone oxidoreductase (complex I) or cytochrome oxidase deficiency. It may be
involved in Alzheimer is disease & diabetes mellitus.

Transporter System in Inner Mitochondrial Membrane


Although the outer membrane is freely permeable to most ion & small molecules, the inner mitochondrial membrane is
impermeable to most small ions like H+, Na+, K+; and small molecules such as ADP, ATP, Pyruvate and other metabolites
important to mitochondrial function. The relative impermeability of inner mitochondrial membrane necessitates exchange
transporters
Inner mitochondrial membrane is freely Transporters Exchange /Transport
permeable to unchanged small molecules such as, 1. Pyruvate symport Transport pyruvate & hydrogen ions in same
- O2 - H iO direction (inwards)
- CO2 - NH3 2. Phosphate transporter Exchange phosphate for hydroxyl ions
- Monocarboxylic acids such as 3-OH-butyric acid, 3. Dicarboxylate Exchange dicarboxylate (malate 2-) ions for
acetoacetic & acetic transporter phosphate
Outside Insinde 4. Tricarboxylate Exchange tricarboxylate (citrate) for malate
/
transporter
1 5. a-ketoglutarate Exchange alpha ketoglutrate for malate
transporter
6. Adenine Nucleotide Exchange ADP for ATP
2 transporter
0 3
7. Glutamate-Aspartate Glutamate is converted to aspartate and
transporter aspartate is exchanged for H+ion
3
Transporters are required for
^ ------- N
- Long chain fatty acids require carnitine system
4
— 0 - Pyruvate
C ilra te + H
- Dicarboxylate & tricarboxylate anions and aminoacids
^ ------- N
5 - ADP enter & ATP exit®
a -ke to
g lu ta ra te 2’ ^ - NADH is transported by malate and glycerophosphate shuttle
- NADPH®
6
- Mitochondria accumulate cations such as K*,Na*, Ca2* and Mg2* & Pi
S *
- Creatine phosphate shuttle bring outside the ATP generated in
★ N- ethyl- M aleimide, hydroxycinnamate and mitochondria
atractyloside inhibit pyruvate symport, phosphate - Alpha-ketoglutarate transporter and glutamate/aspartate
transporter and adenine nucleotide transporter transporter are used in malate shuttle for transfer of reducing
* Transporters are named on the name of product equivalents from cytosol into mitochondria.
they take inside mitochondria - Transporter for glutamine, ornithine, neutral amino acids and
carnitite (carnitine acyl carnitine translocase).
Biochemistry: Carbohydrate Metabolism ■ 601

Poisons Inhibiting the Respiratory Chain

1. Inhibitors o f Electron Transport Chain (ETC)

• ETC inhibitors, prevent the passage (transport) of electrons by binding to a specific component of chain, thus blocking the
oxidation-reduction reaction. So all electron carriers located before the block are fully reduced, whereas those located after the
block are oxidized.
• Inhibition of ETC blocks ATP synthesis as both processes are tightly coupled. And reactive oxygen species (ROS) like superoxide
(02 *), hydroxyl radicals (OH*) and hydrogen peroxide (H 2O 2) are produced d/t incomplete reduction of O 2 to H 2O. ROS can
damage DNA, proteins & cause lipid peroxidation.

Site of action Inhibitor (Poison) M echanism


Complex I: NADH-CoQ - Amobarbital (amytal) & seco barbitals = Blocks e- transfer through complex I (i.e. from Fe
reductase (or NADH Barbiturates S to CoQ / ubi quinone)
dehydrogenase complex) - Rotenone (fish poison & insecticide)
- Myxothiozole
- Piericidin A (antibiotic) Blocks e- transfer by com petin g w ith CoQ
- Chlorpromazine (tranquilizer)
- Guanethidine (a hypotensive alkyl
guanide)
Complex II: - M a lo n a te0 Competitive inhibitor of complex II inhibiting e-
Succinate -C oQ Reductase transfer from succinate to FAD
(Succinate dehydrogenase) - Carboxin Inhibit e- transfer from Fe S to CoQ (through
-TTFA complex II)
Complex III: - Antimycin A Inhibits transfer of e- from cyt b to Q .
CoQ - Cyt C reductase - BAL (British anti lewisite or
(Cytochrome reductase) Dimercaprol), antidote of war gas
- Napthoquinone
- Phenformin (hypoglycemic drug)
Complex IV: Cytochrome - Carbon monoxide (CO)Q Capture O 2 binding site
oxidase (Cyt c oxidase) - Cyanide (CN-) Inhibit terminal e- transfer from a3 to molecular
- Hydrogen sulfide (H2S) O 2 (arrest respiration)^
- Azide (N 3-)
* DCMU (Dichlorophenyl dimethyl urea) competes with Q B for binding site in PS II

2. Inhibitors o f Oxidative Phosphorylation (ATP synthesis)

-O ligom ydn (antibiotic) -Atractyloside (glycoside) Ionophores


-Venturicidin Bongregate (pseudomonads toxin) - Ionophores are lipid soluble
- Binds to Fo domain of ATP synthase, closing the Blocks adenine nucleotide compounds that increase the
proton (H+) channel, blocking reentry of protons transporter (translocases) permeability if lipid bilayer to
through ATP synthase into mitochondrial matrix responsible for m ov em en t o f ADP certain ions.
-This l / t accumulation of H+ at higher concentration in inter in to & ATP ou t o f the m itoch o-n d ria - These are of 2 types
membrane space. Because the pH & electrical gradient Q a cross the inner m itoch on d rial 1.Valinomycin (mobile ion
cannot be dissipated, e- transport stops b/o difficulty membraneQ carrier)
of pumping any more protons against steep proton ■It In h ibits ATP-ADP exchange0. 2 .Gramicidin (channel formers)
gradient. ■Adequate supply of ADP is blocked -Valinomycin allow K+ to
-Inhibition of phosphorylation inhibits e- transport thus preventing phosphorylation permeate mitochondria and
-So Oligomycin completely blocks oxidation & and ATP formation. dissipate proton (H+) gradient.
phosphoryla-tion by blocking the flow of protons Oligomycin & venturicidin inhibits F 0 & CF0 ; DCCD (dicyclohexyl
through ATP synthase and prevents stimulation of O2 carbodimide) blocks proton flow through F 0 & CF0; and aurovertin
uptake by ADP & phosphorylation of ADP to ATP inhibits FI domain of ATP synthase (complex V)
602 ■ A Complete Review of Short Subjects

3. Uncouplers of Oxidative Phosphorylation

• Uncouplers uncouple oxidation (electron transport chain) from phosphorylation. This dissociation of oxidation from
phosphorylation will allow oxidation to proceed, but the energy instead of being trapped into ATP by phosphorylation is
dissipaled as heat.
• Uncouplers create a proton leak removing proton gradient i.e. allowing protons to re-enter mitochondrial matrix without
capturing any energy as ATP. The energy is released as heat and the process is called non-shivering thermogenesis.
• Uncouplers are (mostly) toxic in vivo, causing respiration to become uncontrolled, since the rate is no longer limited by
concentration of ADP or Pi. These allow mitochondria to use O 2 regardless of whether or not there is any ADP or Pi available.
• Uncouplers (eg DNP) are amphipathic & increase permeability of lipoid inner mitochondrial membrane to protons, thus reducing
electro- chemical potential & short circuiting ATP- synthase. In this way oxidation can proceed without phosphorylation (i.e.,
without ATP synthesis). So energy is lost as heat (chemiosmotic theory). So these are toxic in vivo. Examples are

_________ Natural/Physiological Uncouplers________ Synthetic Uncouplers

-Uncoupling proteins (UCP) occur in inner -2, 4 D in itroph en ol* (2,4-DNP)


mitochondrial membrane (of mammals including - 2 ,4 Dinitrocresol ★ (2,4-DNC)
humans) adipocytes. Thermogenin (UCP-1) is -C C C P * (Chloro carbonyl cyanide phenyl hydrazone; the most active
responsible for non shivering thermogenesis (heat uncoupler)
production) in brown fat to maintain body heat, in -F C C P * (tri fluora carbonyl cyanide phenyl hydrazone)
response to cold in neonates and during arousal in - Dicoumarol (vitamin K analogue, anti coagulant)
hibernating animals. UCP-1 is activated by fatty acids - Aspirin & other salicylates in high doses; this is the reason of fever
and brown fat (unlike white fat) uses almost 90% of d/t toxic overdose
its respiratory enrgy for thermogenesis. UCP-2 and 3 - V alinom ycin (produced by streptomyces; a K+ ionophore, transport
are also found in humans but their importance K+ from cytosol to matrix & H+ viceversa thus decreasing proton
remains unclear. gradient)
- Long chain fatty acids (FA) in brown fat - Calcium (Ca++ transport into mitochondria is coupled with uptakes
- Essential fatty acid (EFA) deficiency of Pi and oxidative phosphorylation. But the energy produced is
- Excess of thyroxine hormone spent to pump the Ca++ into mitochondria. So e" transport proceeds
- Unconjugated hyperbilirubinaemia but no energy is stored as ATP)

★ Are hydrophobic proton carriers

Mechanism of Apoptosis

Extrinsic (death- receptor- initiated) pathway Intrinsic (mitochondrial) pathway

Death receptors are members of TNF (tumor ■Growth factors & survival signals stimulate antiapoptotic. Bel - 2
necrosis factor esp TNF1) family that contain a protein (named after B cell lymphoma & is homologous to C. elegans
cytoplasmic death domain involved in protein- protein C ed 9)
protein interaction. The pathway is initiated by •Antiapoptotic Bel- 2 & Bel- x proteins normally reside in
engagement of cell surface death receptor. mitochondria & cytoplasm
When Fas binds to its ligand FasL (a death When cells are deprived of survival signals or subjected to stress Bel
receptor) their cytoplasmic death domain binds - 2 / Bcl-x are lost from mitochondrial membranes & replaced by
with FADD (Fas associated death domain), pro-apoptotic Bak, Bax, and Bim. So the permeability of
which in turn binds an inactive form of ca sp ase - mitochondrial membrane increases and cytochrome C & AIF leaks
8 (& in hum ans ca sp ase- 10). out.
Multiple procaspase 8 molecules cleave one Cytochrome C binds to Apaf-1 (apoptosis activating factor 1,
another to generate active caspase - 8 . This homologous to Ced 4 of c. elegans) and complex activates caspase - 9
pathway can be inhibited by FLIP. ■Apoptosis inducing factor (AIF) enter cytosol & bind to various
inhibitors of apoptosis.
Biochemistry: Carbohydrate Metabolism ■ 603

Glycogen Phosphorylase and Glycogenolysis

Mobilization of stored glycogen is caused by glycogen V asop ressin , o xy to cin ,


phosphorylase, which degrades glycogen to glucose 1 an g ioten sin II b y C a ” o r P IP 2

phosphate. It is often referred to simply as phosphorylase -


so honoured because it was the first phosphorylase to be V
C-AMP independent Ca++
discovered. It was the first enzyme shown to be CAMP dependent
- calmodulin sensitive
allosterically regulated & controlled by reversible phosphorylase kinase PKA
phosphorylation and for which detailed 3 dimensional
structures of active & inactive forms were revealed by x-ray
crystallography.
I +ve

Active phosphorylase kinase b


Phosphorylase exists in 2 inter convertible forms:
phosphorylase a which is catalytically active, and
phosphorylase b which is less active (or inactive). This Phosphorylase b Phosphorylase a
conversion from inactive to active phosphorylase is done by (less activ e) (active)

active phosphorylase kinase b or phosphorylase kinase a,


an enzyme responsible for transferring a phosphoryl group Phosphoprotein <___
to ser residues in each of the 2 identical subunits of phosphatase 1 (PP1) -v e Inhibitor 1
glycogen phosphorylase.
Epinephrine & nor epinephrine (in muscle during
G lu cag o n (liver), E p in ephrin e,
activity) or glucagon (in liver when blood glucose level is
T C a + + , T A M P (m u scle)
too low) increase concentration of C-AMP, which inturn
activates C- AMP dependent protein kinase (protein
Myocyte Hepatocyte
kinase A, or PKA). PKA then phosphorylates & activates X molecule
Epinephrine Glucagon
phosphorylase b kinase (to phosphorylase kinase a),
which catalyzes phosphorylation and activation of serine
ATP Cyclic AMP (20x)
residues of glycogen phosphorylae, and thus stimulating
glycogen breakdown. In muscle this provides fuel for
glycolysis to sustain muscle contraction whereas in liver it
counters the low blood glucose levels. Inactive PKA Active PKA (lOx)
Both phosphorylase and phosphorylase kinase (a) are
dephosphorylated and inactivated by phosphorylase a Inactive Active phosphorylase
phosphatase or phosphoprotein phosphatase 1 (PP1). In phosphorylase kinase b(lOOx)
resting muscles. Protein inhibitor 1, when activated after kinase b
phosphorylation by cAMP dependent protein kinase,inhibit
PP1. In liver insulin increases activity of phosphodiesterase Inactive glycogen Active glycogen
(which hydrolyze C-AMP) and indirectly stimulate PP1 phosphorylase b phosphorylase a
resulting in decreased glycogenolysis. (1000 x)
In liver, cAMP independent glycogenolysis occurs in
response to ai adrenergic stimulation by epinephrine or
norepinephrine. This involves stimulation of Ca++ / +ve +ve Glucose 1 POr
Glycogen
calmodulin sensitive phosphorylase kinase by Ca++. It is (10,000x molecules)
also activated by vasopressin, oxytocin, & angiotensin IIQ
acting either through Ca++ or PIP2 ▼
Glycolysis Glucose
Ca++ ion is responsible for intiation & synchronization of
both muscle contraction and glycogenolysis (or activation I
of phosphorylase)Q. Muscle phosphorylase kinase (c Muscle contraction T Blood glucose

activates phosphorylase) is a tetramer of a, (3, y, and 8 Allosteric control of phosphorylase occurs through
subunits.
- a, and P, subunits contain serine residue that are activated In muscle In Liver
by cAMP dependent protein kinase. I____ I ,
I-------
- 8 subunit is identical to Ca++ binding protein calmodulin Activation Inhibition Activation Inhibition
& binds 4 Ca++ I I I I
- y catalytic subunit is activated (by 8- calcium binding) C a ^ (th ro u g h 8 -A T P Q CAM P - A TPQ
su b u n it of - G lu cose 6 - G lu cose 6
even while the enzyme is in dephosphorylated state b; the
p h o sp h o ry lase kinase) p h o sp h ate p h o sp h ate
fully activated & phsphorylated a form occurs in high
C- AMP - F r e e g lu c o s e Q
concentrations of calcium. 5' AM P
604 ■ A Complete Review of Short Subjects

Factors maintaining active Allosteric Inhibitors of glycogen Factors maintaining


phosphorylase phosphorylase (inactivating it) inactive phosphorylase
- C- AMP dependent protein kinaseQ - ATP & Glucose 6 phosphate (in - Protein phosphatase IQ
- Adenyl cyclase, Cyclic AMP and hormonesQ muscles & liver both)Q (PP-1)
increasing C-AMP.such as epinephrine (in - Glucose (in liver but not in - Glucose 6 phosphate^
muscle), norepinephrine, glucagon (in liver) & muscles) Q - InsulinQ
thyroid hormone - Phosphodiesterase
- Calmodulin component of phosphorylase kinase (hydrolyze cAMP)
- CalciumQ
- 5AMP (allosteric activator in muscle only)
- Inhibitor-1 (inhibits PP-1)

— Epinephrine
— Norepinephrine
C -A M P : Increased b y —
Glucagon
— Thyroid horm one

Glycogen
Pathway of glycogenesis and of (1—>4 and 1-»6 glucosyl units),
glycogenolysis in the liver

In well fed state (also called absorptive


stage) insulin suppresses glucose -6-
phosphatase activity and causes glucose 6
- phosphate to enter glycogenesisQ (to
form glycogen).

Two high-energy phosphates are used in


the incorporation of 1 mol of glucose into
glycogen. (+), stimulation; (-), inhibition.
Insulin decreases the level of cAMP only
after it has been raised by glucagons or
epinephrine - i.e. it antagonizes their
action^. Glucagon is active in heart muscle
but not in skeletal muscle. At asterisk:
Glucan transferase and debranching UDP triphosphate (UTP) Glucose 1-phosphate

enzyme appear to be two separate Mg21 1 1 Pho9phofllucomutase~|


activities of the same enzyme.
6-phosphate » Toglycolysisandpentose
^ AD p phosphatepathway
Ghjcose-6-'
Mg2* I Glucokinase ]
Phosphatase ,
P:
Biochemistry: Carbohydrate Metabolism n 605

Von Gierke’s Disease (Type la Glycogen storage Disease)

Inheritance ARQ M etabolic Abnormalities & Clinical Presentation


Chromosome 17 • Fasting hypoglycemia d/t unavailability of glycogen
• Hyper lipemia (hyper lipidemia), acidaemia & ketosis occurs as a result of utilizing
Enzyme Glucose 6
fat as energy source (i.e. increased fat mobilization).
defect phosphatase®
• Excess acetyl CoA obtained from (3- oxidation is diverted for increased cholesterol,
Organs Liver (primary),
fatty acid and ketone body synthesis resulting in xanthomas, fatty infiltration of
affected kidney, intestinal
liver, hepatorenomegaly (protuberant abdomen) & doll like face with fatty cheeks
mucosa
• Persistent hypoglycemia has 2 effects
Glycogen Normal in structure
- Inhibits insulin secretion which inturn inhibits protein synthesis 1/t stunted growth
Stored but m etabolically not
(dwarfism) and thin extremities
available®
- Stimulates secretion of catecholamines which cause muscle glycogen to break down
Appearance Doll like face with fatty producing lactic acidosis
cheeks, thin
• Hyperuricemia an gout (d/t more pentose fo rm a tio n and d ecreased excretion)®. It
extremities and
has 2 reasons
protuberant abdomen
(d/t hepato-reno 1. Increased production:
megaly) - A ccum ulated G-6- P h osp h a te enters the HM P shunt and l/t in creased production
o f pentoses®
Glucose 6 phosphatase (present in - Pentose acts as substrate for PRPP synthetase (Phospho Ribosyl pyro phosphate
liver but not in muscle) converts synthetase) and result in increased production of purines
glucose 6-P 04 (obtained from - Purines are catebolized to uric acid.
glycogenolysis) to glucose. So deficiency 2. Decreased renal excretion of uric acid is caused by increased lactic acid level which
1/t hypoglycemia. competes with uric acid for excretion.

Glucose-6-PQ4 Dehydrogenase (G6PD) Deficiency

It is a X-linked recessive disorder <3


that causes episodic hemolytic
anemia because of decreased
ability ofR B C s to deal with
oxidative stresses, due to
im paired generation o f NADPH®.
G6 PD is the first and rate
limiting enzyme o f the HMP
shunt which is responsible fo r the R0 Ie of the p en to se p h o sp h ate p a th w a y in the g lu tath io n e p ero x id a se reaction of ery th ro cy tes.
production o f NADPH in the (G -S-S-G , o xid ized g lu tath io n e; G -SH , red u ce d g lu tath io n e; Se, selen iu m co facto r)
cells Q.
NADPH acts as a cofactor fo r glutathione reductase® that generates reduced form of glutathione.
Reduced glutathione is a potent antioxidant and removes H2O 2 by enzyme glutathione peroxidase
In the absence of reduced glutathione, H2O 2 accumulates and decrease the life span of RBC by causing oxidative damage to
cell membrane leading to hemolysis.
Favism by eating Fava beans (eg as an ingredient of falafel- a Mediterranean food) l/t hemolytic anemia, jaundice, even kidney
failure & death may occur in peoples with G6PD deficiency.
606 ■ A Complete Review of Short Subjects

G lu co s-6 -P O ,
6- Phosphogluconate Pathway / Hexose
Mono Phosphate (HMP) Shunt / Pentose NAD P*
Phosphate Pathway (PPP)/Warburg-Dickens- G lu c o s-6 -P O p
[R a te lim itin g e n z y m ef*
_____________ Lipman-Pathway d eh y d ro g en ase
NAD PH
It is an alternate pathway for complete oxidation of 6 - P h o s p h o g lu c o n o la c to n e
glucose, which is not meant fo r production o f energy
(ATP)0 . Like glycolysis, enzymes of PPP are Gluconolactone
Hydrolase
cytosolic<3. But unlike glycolysis
6 - P h o s p h o g lu c o n a te
the irreversible oxidative portion of PPP occurs in
specialize tissue like liver, lactating mammary NADP+
glands & adipose tissueQ which are active in fatty 6 -phosphoglunonate
dehydrogenase
acid synthesis, in adrenal cortex0 which is active in NADP++ H +
NADPH dependent synthesis of steroids and in 3 - K eto 6 - p h o s p h o g lu c o n a te
erythrocytesQ which require NADPH to require
glutathione reduced. It also occurs in lens, cornea,
testes and ovary but not in non lactating
mammary gland and unimportant in skeletal
muscle®. Whereas glycolysis occurs in all tissues.
It is multicyclic process, 3 molecules of glucose 6
phosphate gives rise to 3 molecules of C 02 and 3
molecules of 5 carbon sugar, which rearranges to X y lu lo s e - 5 P 0 4
give 2 molecules of glucose 6- phosphate and 1 A
molecule of glyceraldehyde 3- phosphate. CO2 is
produced in HMP shunt which is never produced in
E.M. pathw ay<2.______________________
6 C arbon 5 C arb o n S u g a r (3 m o lecu les) + 3C O ;
G ly c e r a ld e h y d e - 3 P 0 4 S e d o h e p a tu la s e - 7 P 0 4
G lu co se - 6 P
(3 m olecu les) 1 R e a rra n g e
6 carbo n G lu c o s e - 6 - P (2 m o lecu les) +
3 carbon G ly c e ra ld e h y d e -3 -P (1
m olecu le)

Oxidation is achieved by dehydrogenation using F r u c to s e - 6 P 0 4 E r y th o s e - 4 P 0 4


NADP+ not NAD* as hydrogen acceptor<2.
<HMP shunt can be divided into 2 phases: (1)
oxidative non reversible phase that 1/t formation
of ribulose 5 -PO4, CO2 and 2 molecules of NADPH
for each molecule of glucose - 6 - PO4 oxidized. (2)
F r u c to s e - 6 P O 4 G ly c e r a ld e h y d e 3 - P 0 4 (3C)
non-oxidative reversible phase that occurs in all
cells synthesizing nucleotides & nucleic acids; and Phospho Phosphotriose

mainly involve two transketolase (transfer 2 carbon


hexose Isomerase i isomerase
1 / 2 F r u c to s e 1,6 Bi P O 4 (6 C)
units) and one transaldolase (which transfer 3
G lu c o s e - 6 -POq Fructose 1,6 bi
carbon units) reactions. phosphatase
1 Glucose 6 PO 4 dehydrogenase is the rate limiting
I
1 / 2 F r u c to s e 6 P O 4 (6 C)
enzyme and uses NADP+ as a coenzyme®.
Transketolase requires thiamine pyrophosphate
(TPP)®. Other enzymes requiring TPP are pyruvate
t
G lu c o s e 6 P 0 4 ( 6 C)
de carboxylase®of pyruvate dehydrogenase
complex, a —ketoglutarate dehydrogenase®of * It is m ulticyclic
TCA cycle, & branched chain (X -keto acid process, 3
deydrogenase® of branched chain aminoacid m olecules o f
metabolism. g lu cose-6 P 0 4 enter
1 No ATP is directly produced or consumed in HMP the cycle,
shunto. Its m ajor function is to provide NADPH®
producing 3 m ols
(required for fatty acid & steroid synthesis) and
o f C 0 2 and 3 m ols
pentose sugar® (for nucleic acid & nucleotide
synthesis). Summary of PPP is______________
3 g lu co se - 6- F O i 2 g lu co se - 6- P 0 4 + 1
g ly c e ra ld e h y d e -3 - P 0 4 +
N A D P+ 3 C O 2 + 6N A D PH + 6 H*

* In s u lin & th y ro id h o rm o n e e n h a n c e the a c tiv ity o f


H M P sh u n t
Biochemistry: Carbohydrate Metabolism ■ 607

Carbon (CO2) Assimilation /Fixation = Calvin Photosynthetic Carbon Reduction Cycle


Plants synthesize carbohydrate from CO2 and
water by process of photo synthesis in plastids
(chloroplast). It occurs in 3 stages - fixation of CO2
into 3 phosphoglycerate, conversion of 3
phosphoglycerate to glyceraldehydes 3 phosphate
and regeneration of ribulose 1, 5 biphosphate from
triose (glyceraldehydes 3) phosphate.
Overall 3 molecules of ribulose 1,5 biphosphate
fixes 3 molecules of CO2 to form 6 molecules of
glyceraldehyde 3 phosphate (18 carbons) in
equilibrium with dihydroxy acetone phosphate.
The formation of glyceraldehyde from 3
phosphoglycerate is reversal of corresponding
glycolysis steps, with one exception that cofactor is
NADPH rather than NADH.
In 3rd stage 5 of 6 molecules of triose
(glyceraldehyde 3) PO4 (=15C) are used to
regenerate 3 molecules of starting material ribulose
1,5 biphosphate (15 carbons) involving many of
the same reactions used in oxidative pentose
phosphate pathway (but in reverse direction). This
reductive pentose phosphate pathway also uses
few extra enzymes like sedoheptulose, 1, 7
biphosphatase etc that make the reductive cycle
irreversible.
Fructose 6 phosphate is a key intermediate in stage
3 and can lead either to regeneration of ribulose 1,
5 biphosphate or to synthesis of starch.
6 NADPH and 9 ATPs are required for synthesis
of each triose phosphate (glyceraldehyde 3 PO4)
from CO2 & H2O. so summary of calvin cycle is p ro d u c tio n b y g ly c o ly s is o r
c o n v e r te d to f ru c to s e 6 p h o s p h a le
u ltim a te ly f o r m in g s ta rc h (in
c h lo r o p la s t) o r s u c ro s e (in
c y to p la s m ) to bo s to r e d for la te r u s e

3 CO2 + 5 H2O + 6NADPH + 9 ATP -» Glyceraldehyde 3 phosphate (G3P) + 6 NADP+ + 9 ADP + 2H* + 8 Pi

Fate of

A cetyl CoA Pyruvate


• In Citric Acid Cycle
C itr a te S y n th a se
A c e ty l C o A + O .A .A ------------------------------- ►C itric A cid
• In Cholesterol Synthesis
T h io ta s e H M G C oA
A c e tv l C o A - -> A c e to A c e ty l C o A - » H M G C oA
S y n th a se
in F a tty A c id S y n th e s is

A c ety l C oA C a r b o x y la s e
A cety l C o A :-------------------------:----------* M a lo n y l C o A

F a tty A cid
S y n th a s e E n z y m e (Gluconeogenesis) acids (Gly. Ser. Cyst/
cystme. Thr)

Acetyl Transacylase M alonyl Transacylase

* A cety l C oA carb oxylase is also kn o w n as M alo n y l CoA


A c e ty l-M a lo n y l E n z y m e
S y n th e tase
• In P o x id a tio n

Thiotase
3 K e to a c y l C o A - ■A cy l C o A + A cety l C o A
608 ■ A Complete Review of Short Subjects

Gluconeogenesis

* The carbon skeletons for gluconeogenesis are derived


primarily from glucogenic am inoacids & lactate from G lu c o s e
Glucokinase
Glucose-6-phosphalase
muscle and glycerol from adipose tissue ®. Although the Hexokinase
G lu c o s e 6 - P
lactate produced in muscle (by anaerobic glycolysis
during intense exercise) is used by liver fo r
t
F ru c to se 6 -P
Fructose 1,6 |Phosphofructokinase \
gluconeogenesis (through Cori cycle); the m uscles do not -biphosphatase F r u c t o s e 1 ,6 , b ip h o s p h a t e
p articipate in gluconeogenesis®. Liver and kidney are the
m ajor gluconeogenic tissues®. I----------- D i h y d r o x y a c e l o n e p h o s p h a ic
G ly c c rn ld c h y d c -3 -P

> Gluconeogenesis prevent hypoglycem ia during short and Glycerol 3-phosphate


long term (= prolonged) fasting® i.e. begins 4-6 hours I J - d i p h o s p l m g l y c r i c a c id Dehydrogenase

after the last meal becomes fully active as stores of liver G lv c c m l-3 -P
3 - p h o s p h o g l v c e r i c a c id
glycogen are depleted and peaks at 5-7 days of fasting .
>Gluconeogenesis is stim u lated by excess o f acetyl CoA i
2 - p h o s p h o g l y c c r i c a c id
[ Glycerol Kinase ]

[G ly c e r o l]
and decrease in fru ctose 2, 6 biphosphate concentration®. t
1. Although acetyl CoA cannot be used as a substrate for ► P h o s p h o e n o lp y r u v a lc

Phosphoenol
gluconeogenesis, it is an allosteric activator of pyruvate carboxy
\Pyruvate kinase

pyruvate carboxylase and an allosteric inhibitor of kinase P y r u v a te { L a c ta ic )

pyruvate dehydrogenase. So acetyl CoA (produced by


(3 oxidation) pushes pyruvate to gluconeogenesis
2. Gluconeogenesis is favoured by activation of fructose
O x a lo a c c ia le
1,6 biphosphatase ( F 16 BPase d/t decreased
availability of its inhibitor F ,l, 6 biphosphate) and by O x a lo a c e ta te

induction of PEPCK by glucagon. / N


■ M a la ie C l"
>It is synthesis of glucose or glycogen from non
carbohydrate precursors (substrates) such as glucogenic a - k c io g lu ia r a t e «

aminoacids (forming pyruvate, oxaloacetate, alpha KG, F u m a ra te J / R u m in a n t ( m a jo r s o u rc e )


S u c c in y l C o A < [P r o p io n a te ]
succinyl CoA, fumarate), lactate, glycerol and In n o n r u m i n a n t (eg h u m a n s)
propionate. Liver and kidneys are major gluconeogenic ii a ris e s fro m |i o x id a tio n o f
o d d c h a in F A ( r u m in a n t fu th
tissues; the kidney contributing upto 40% of total glucose o x id a tio n o r i s o l c u c i n c &
c h o le s t e r o l s id e c h a in .
synthesis in fasting state and more in starvation. M in o r s o u r c e o f g lu c o s e

Gluconeogenic key enzymes are expressed in small


intestine but their role in fasting state is unclear. M a jo r p a th w a y s o f g lu c o n e o g e n e s is an d g ly c o ly s is in liv e r
•A supply of glucose is necessary for nervous system
(brain) & erythrocytes (RBC). Glucose is important in
maintaining the level of intermediates of citric acid cycle even
when fatty acids are the main source of acetyl CoA. A c y l C o A S y n th e ta s e I P ro p lo n y l C o A C a rb o x y la s e |
P r o p io n a t e ------------- ; = ; ------------- ►-P r o p io n y l------------- ► D -M e th y l
Gluconeogenesis also clears lactate produced by muscles &
[ M g **] CoA | B lo tln ~] m a lo n y l C o A
erythrocytes, and glycerol produced by adipose tissue. Failure
of gluconeogenesis is fatal b/o hypoglycemic brain
M e th y l m a lo n y l
dysfunction and excessive gluconeogenesis in critically ill C oA Racem ase

(injured / infected) patients also has poor outcome. Enter M a th y lm a lo n y l C o A M u ta s e |


' It involves glycolysis, the citric acid cycle plus some special citric acid 4 S u c c in y l C o A ►L -m e th y l
cycle V lt B12 c o e n z y m e | m a lo n y l C o A
reactions.
Deficiency
' Main Pathway of gluconeogenesis is a reversal o f
glycolytic p ath w ay but thermodynamic 'barriers' M e th y l m a lo n y l a c id u ria

prevent a simple reversal of glycolysis. There are four such


energy barriers which are irreversible. Three non equilibrium
reactions in glycolysis and one in glycogen metabolism
prevent simple reversal of glycolysis for glucose
/glycogen synthesis (gluconeogenesis) and they are
1Propionate is a major source of glucose in ruminants. It
non ruminant (eg humans) propionate arises from beta-
oxidation o f odd chain fa tty acids (eg ruminant lipids),
oxidation o f isoleucine & side chain o f cholesterol®, and
is a minor substrate for gluconeogenesis.
Biochemistry: Carbohydrate Metabolism ■ 609

Regulation of Gluconeogenesis & Glycolysis

Since glycolysis and gluconeogenesis share the same pathwy but in opposite directions, they must be regulated
reciprocally. A v a ila b ility o f g lu c o g e n ic p re cu rso rs (su b stra te s) is the most important factor, along with c irc u la tin g le v e l
o f g lu c a g o n , in movement to movement re g u la tio n o f g lu c o n e o g e n e s is .
In carbohydrate metabolism enzyme activity is re g u la te d b y 3 m e ch a n ism s

S lo w A dap tive ch anges in enzym e C ovalent M o d ificatio n Allosteric M odification


activity (Ind uction & R ep ression ) from b y R ev ersib le
It is in s ta n ta n e o u s and examples of allosteric regulation
alteration in rate o f Enzym e Sy n th esis P hosp ho ry latio n
include
o r D egradation or both. - It is rapid
- Induction k repression of key ■In response to fastin g A. A llo s te ric a ctiv a tio n o f h e p a tic p y ru v a te
enzymes (catalyzing physiologically (decrease in blood ca rb o x y la s e b y a cety l C o A in c re a se s g lu c o n e o g e n e s is
irreversible non equilibrium reactions) is glucose), g lucagon & and re c ip ro ca l in h ib itio n o f p y ru v a te d e h y d ro g e n a se
slow & req u ires several hours. ep in ep h rin e, inhibit by acetyl CoA sparing pyruvate from oxidation (in TCA).
- Activity of enzymes catalyzing glycolysis and stimulate ■ G lu c o n e o g e n e s is in fa s tin g sta te is indicated by increased
reactions in opposite direction varies gluconeogenesis in liver activity (activation) o f pyruvate carboxylase by Acetyl
recip rocally so the effects are by in creasin g cA M P CoAQ. As a result of increased Iipolysis (in fasting state &
reinforced. concen tration. c-A M P diabetes), the liver is flooded with fatty acids. The rate of
- For example in su lin secreted in d ep end ent protein
formation of acetyl CoA by beta- oxidation of these fatty
response to excess of blood glucose k in ase activity leads to
acids exceeds the capacity of liver to oxidize it to CO2 &
enhance synthesis of key enzymes of phosphorylation &
H2O. As a result, acety l C o A a ccu m u la te s and ca u se
glycolysis and antagonize the effect of inactivation of pyruvate
a llo s te ric a c tiv a tio n o f h e p a tic p y ru v a te ca rb o x y la s e . At
epinephrine, glucocorticoids and k in ase
glucagon stimulated cAMP thereby the same time acetyl CoA in h ib its p y ru v a te
F astin g / D iab e tes d e h y d ro g e n a se (by activating PDH kinase). Thus acetyl
suppressing synthesis of key enzymes of
gluconeogenesis. i ------ If CoA can divert pyruvate towards gluconeogenesis and
D ecrease Increased away from TCA.
Excess o f g lu co se (C arbohydrate d in su lin glucocorticoid
Feedin g) Beta- oxid ation o f FA (fasting)
k s, glucagon
j : g lycoly sis (cA M P), _______ I_______
Increased D ecreased enzym es ep in ep h rin e A cetyl CoA
In su lin G lucocorticoid , and enzymes i
G lucagon, of g lu co n eo­ A llo steric activation o f R ecip rocal in h ib itio n of
1
E pin eph rine g e n e sis_______ pyruvate carboxylase (in liver) PDH
In creased activity
I I
o f enzym es 1
in vo lv ed in D ecreased M e t a b o lic F a te o f P y ru v a te

glucose activity I I
u tiliz a tio n (synthesis) of key Pyruvate is converted to Inhibits conversion of pyruvate
(glycolysis & enzym es of oxaloacetate (entering to acetyl CoA. Thus pyruvate is
lipogenesis) g lu co neo gen esis gluconeogenesis) diverted away from TCA

B. A llo s te ric re g u la tio n b y E n e rg y le v e ls w ith in c e lls

A small
- 5 'A M P acts as a n in d ic a to r o f en erg y sta tu s o f c e lls.
decrease in ATP (d/t its use in energy requiring proesses)
causes a several fold increase in AMP so that AMP acts as a
m e ta b o lic a m p lifie r (s e n s itiv e s ig n a l) o f ch a n g e in en e rg y
sta tu s o f c e ll.
- Energy rich state of cell (ie high ATP & low AMP levels)
stimulate gluconeogenesis (an energy requiring pathway).
Whereas energy poor state (elevated levels of AMP) increase
glycolysis (for energy /ATP formation)
P F K -1 a n d F ru cto se 1 , 6 B ip h o s p h a ta s e are re c ip r o c a lly
re g u la te d (a llo s te ric a lly ) b y A M P

- F ru cto se 1 , 6 b ip h o s p h a ta s e is a llo s te r ic a lly in h ib ite d b y


AMP (decreasing gluconeogenesis) and simultaneously AMP
1)
a ctiv a te s p h o s p h o fru c to k in a s e (p h o s p h o fru c to k in a s e
increasing glycolysis & p h o sp h o r y la s e increasing
glycogenolysis.
610 ■ A Complete Review of Short Subjects

- PFK1 occupies a key position in regulating glycolysis in response to energy status of cell. It is activated by 5' AMP and
is inhibited by citrate & normal ATP concentrations. As a consequence of inhibition of PFK-1, glucose 6 phosphate
accumulates, which in turn inhibits uptake of glucose in extrahepatic tissue by inhibition of hexokinase.

C. PFK-1 & FBP aselare reciprocally regulated (allosterically) by Fructose 2, 6 Biphosphate

■Fructose 2, 6 Biphosphate is most potent positive allosteric activator of phosphor fructokinase 1 (increasing glycolysis)
and inhibitor of fructose 1, 6 biphosphatase /FBPase 1 /or F 1, 6 pase (decreasing gluconeogenesis). Thus it (F2, 6 BP)
plays a unique role in regulation o f glycolysis & gluconeogenesis in liver®.
■PFK1 is virtually inactive in the absence of F2, 6 biphosphate (d/t inhibition caused by ATP, citrate glucagon). F2 , 6
biphoshate increases PFK l's affinity for its substrate fructose 6 phosphate and reduces its affinity for allosteric inhibitors
ATP & citrate; thereby increasing glycolysis. F2, 6 BP has opposite effect on FBPasel : it inhibits FBpase 1 (or fructose 1, 6
biphosphatase) by reducing its affinity or increasing km for its substrate fructose 1, 6 biphosphate

Fructose 2 ,6 Bipjiosphate (F 2,6 BP)


In well fed state with abundant
Allostric activation of Allosteric inhibition of supply of glucose (& so fructose 6
PFK-1 by increasing its FBPase 1 (Fructose 1, 6 ___________ phosphate)___________
affinity for its substrate biphosphatase or FI, 6 I I
fructose 6 phosphate & pase) by reducing its Allosteric Stimulation of PFK-2 Glucagon enhances
reducing affinity for its affinity or increasing Km and inhibition of FBPase 2 by FBPase 2 and
allosteric inhibitors ATP for its substate fructose 1, 6 fructose 6 phosphate inhibits PFK-2
& citrate biphosphate activity

I I
Increase glycolysis Decrease gluconeogenesis
■Therefore decreased sensitivity of FBpase 1 to regulation (inhibition) by fructose 2, 6 biphosphate increases
gluconeogenesis; thereby decreasing levels o f fructose 1, 6 biphosphate, pyruvate and ATP®.
Cellular levels of allosteric regulator fructose 2, 6 biphosphate (F2, 6BP) depends on the relative rates of its formation
(by phosphofructokinase 2 or PFK2) and breakdown (by fructose 2, 6 biphosphatase or FBPase 2). PFK-2 and FBPase
2 are 2 separate enzymatic activities of a single bifunctional protein (enzyme). This bifunctional enzyme is under
allosteric control of fructose 6 phosphate which stimulates kinase (PFK-2) and inhibits phosphatase (FBpase 2). The
balance of these activities are regulated by glucagon & insulin; as glucagon enhances FBPase 2 activity & inhibits
PFK-2 activity (via cAMP dependent protein kinase) whereas insulin has opposite effect.

Feed - Fast Cycle

After Feeding After Fasting (starvation)


• Liver is a glucose producing rather than a glucose using - T glycogen degradation (glycogenolysis)
tissue. However after a meal containing carbohydrate, - T gluconeogenesis
the liver becomes a net consumer of glucose, retaining ~ - T fatty acid oxidation
60 of every 100 gm of glucose presented by portal system. - Tsynthesis of ketone bodies (ketogenesis)
• Hepatic metabolism of glucose is increased by - T degradation of TAG__________________
[ 4 0 -i
- Tphosphorylation of glucose
- T glycogen synthesis
- T activity of HMP
- T glycolysis
- i gluconeogenesis

• Fat metabolism: T Fatty acid & TAG synthesis


• Aminoacid metabolism: Tprotein synthesis and T amino
acid degradation
But the liver has limited capacity to degrade branched
chain aminoacids leucine, isoleucine and valine; they
pass through the liver essentially unchanged and are Sources of blood glucose after ingestion of lOOg of
preferentially metabolized in muscle. glucose
Biochemistry: Carbohydrate Metabolism ■ 611

Fructose Metabolism

Hexokinase *
D-Fructose
->■ Fructose 6 -P O 4
(d ie t)
In extrah ep atic tissue, hexokinase
Does not act on glucose & unlike
Fu rctokinase* catalyses ph osph orylation of m ost
glucokinase its activity is not
affected by fasting or insulin, hexose sug ars inclu d ing fructose
* B lo ck in esse n tia l
w hich explains w hy fructose is
F ru cto seu ria Q ■
cleared from blood in diabetic In liv er
* Fou nd in liv er
patients at a norm al rate Fructose undergoes m ore rapid
(K id n ey & in testin e)
glycolysis in liver than does glu cose
Fructose - 1 - PO 4 b ecause it bypasses the regulatory
step catalyze by Phosph ofructokin ase.
Aldolase B *
D -G lycerald eh yde en ter glycolysis via
I Block in Hereditary p h osp h orylation to G lyceraldehyd e
Fructose intoleranceQ 3 -PO 4
D H A P & G ly cera ld ch y d e-3 -P 0 4 m ay
en ter glycolysis or m ay be su b strate for
A ld olase to e n te r g lu c o n e o g e n e s is
Di-Hydroxy-Acetone Phosphate D - Glyceraldehyde ( m a jo r f a t e o f liv e r fr u c t o s e )
(DHAP)
ATP T riokin ase

Glyceraldehyde
3 phosphate

\
Enter to Gluconeogenesis Enter Glycolysis

< In L iv e r ---------------- Extrahepatic

Hereditary Fructose Intolerance

Hereditary fructose intolerance is due to deficiency of fructose 1, 6- biphosphate aldolase B in the liver, kidney and
intestine®.
M echanism C lin ic a l Presentation
• Aldolase B catalyzes fructose I -P O 4 • This is severe disease of infants that appears with the ingestion of fructose
to DHAP and D- glyceraldehyde. containing food
• Its deficiency causes excessive rise • Administration of fructose or sucrose leads to Hypoglycemia® accompanied
of fructose -1- PO4 and fructose in by nausea, vomiting, convulsion & profuse sweating.
blood® • Continued intake leads to
i - Hepatomegaly®
Hypoglycemia® - Jaundice®
- Proximal renal tubular dysfunction®
- Intellectual impairment®
Treatment: Complete elimination of all sources of sucrose, fructose and sorbitol from the diet.
612 ■ A Complete Review of Short Subjects

Ethanol (Alcohol) Metabolism


I
I
M ajor p ath in liver in volves 2 oxidation reaction s by a lco h o l & Minor path takes place via cytochrome P450
aldehyde dehydrogenase & NAD+® dependent microsomal ethanol oxidizing
system (MEOS) involving NADPH® & O 2
Ethanol This system increases in activity in chronic
Alcohol NAD* alcoholism & may account for increased
dehydrogenase metabolic clearance__________
NADH Ethanol
/ N AD PH + O 2
A cetaldehyde tN A D H
M EOS
NAD* N A D P * + H 2O
A ldehyde M assive
dehydrogenase D isulfiram increase in A cetaldehyde
cytosolic ★ Acetaldehyde is highly reactive & can form
NADH N ADH adducts with protein nucleic acids & other
I molecules
A ceta te
* Ethanol interpolates into biological membranes,
I
expanding them and increasing their fluidity. It
A c e ty l C o A • •TFA synth.
alters membrane active transport, AP, &
neurotransmitter release esp affecting brain.

Favours reduction of - Inhibits fatty acid oxidation Inhibits glycerophosphate dehydrogenase 1/t
pyruvate to lactate and - Testerification of FA to form triacyl elevated glycerophosphate
oxaloacetat| to malate glycerol resulting in fatty liver

Tlactate /pyruvate ratio ^gluconeogenesis (because both pyruvate & oxaloacetate are intermediates)
- Hyperlacticacidemia I
- 1 excretion of uric acid Precipitate hypoglycemia in individuals who have depleted their stores of liver
- Aggravate gout glycogen or are using insulin.

Classification of Carbohydrate Structure

Aldose and Ketose Isomers


• Monosacchrides are classified a/t the number of carbon atoms they contain Isomers are compounds that have same chemical
eg triose for 3C, tetrose for 4C etc. Most oxidized functional group of formula but have different structures. For
aldoses is aldehyde and ketoses is ketone. In other words, if the carbonyl example glucose, fructose, galactose, mannose,
group is at an end (Ci) of carbon chain (in an aldehyde group) the idose, talose, gulose, allose, and altrose all are
monosacchride is aldose and if the carbonyl group is at any other position isomers with same chemical formula C 6H 12O6.
(eg C2 in a ketone group) the monosaccharide is a ketose. Most aldoses have
-ose (except few like glyceraldehyde) and ketoses have - ulose (except few Carbon G eneric A ldoses Ketoses
3 Trioses G lycerose D ihydroxy-
like fructose) suffix.
(C3H6O3) (g ly ce ra ld e h y d e ) acetone
Epimers 4 Tetroses Erythrose, Erythrulose
( G H s O d Threose
• Carbohydrate isomers that differ in configuration around only one carbon
5 Pentoses R ibose, Xylose Ribulose
atom (with the exception of carbonyl carbon) are called epimers. For
(C3H ,0O 5) A ra b in o se, Xylulose
example L y xo se
- Erythrose and threose are C2 epimers 6 H exoses Glucose, Fructose,
- Ribose is epimer of arabinose (at C 2 ) and xylose (at C 3 ) but arabinose and (C6H12CM G ulose, P sico se,
xylose are not epimers as they differ in C 2 & C 3 positions. G alactose, S o rb o se ,
- Lyxose is epimer of Xylose (at C 2 ) and arabinose (at C 3 ) but arabinose and M annose, T a g a to se
Xylose are not epimers as they differ in C 2 & C 3 positions. A llo se , A ltro se,
Id o se , T a lo se
- Glucose is epimer of mannose (at C 2) and galactose (at C 4 ) but mannose &
7 H eptoses - Sedoheptulose
galactose are not epimers.
(C7HMO7)
- Allose is epimer of altrose (C2), glucose (C3) and gulose (C4) but altrose, 9 N onoses N euram inic Acid
glucose & gulose are not epimers (CoHiaCM
- Talose is epimer of galactose ( C 2 ) , idose ( C 3 ) and mannose ( C 4 ) but
galactose, idose & mannose are not epimers.
Biochemistry: Carbohydrate Metabolism ■ 613

'C H O CH O Cl 10 CH O CIIO CHO CHO CHO


1 1 1 1 i I I Pyran/Furan; Haworth Projection; Chair
H - 2C - OH-C- H -C - HO - C - H H - C - H O -C- H - C - OH HO - C - Conformation
OH H OH OH H H
• The ring structure of monosaccharides are
I
H - C - H -C - HO- C - H HO-C-H H -C - H - C - HO- C - H HO- C - H
either similar to pyran (a 6 membered ring
OH OH OH OH consisting of 5 carbon & 1 oxygen) or furan (a 5
I I membered ring consisting of 4 carbon & 1
H - C - H -C - H -C - H-C-OH HO- C - H H O -C - I HO - C - H H O -C -
oxygen) and are called pyranose or
OH OH OH H
1 1 1 1 I I furanose.
H - C - H -C - H -C - H-C-OH H -C - H -C -O H H -C - • Cyclic structures are drawn in Haworth
OH OH OH OH OH OH projection in which bonds nearest to the viewer
I I I
C H 2OH C H 2OH C H 2OH C H 2OH CH2OH CHzOH CH2OH CH2OH
(viewing from side & above the plane of
ring) are bold and thickened and OH groups
(T )-A llose ^ fe -A ltro s e fo -G lu c o s e , D- D - G u lo s e ) D -I d o se D -G a la c to se D -T a lo se are above or below the plane of ring.
However, the 6 membered pyranose ring is
Enantiomers (Optical/Stereo-Isomers) actually in form of (two) chair conformation and
is not planar as Haworth perspective
• Enantiomers (optical isomers or stereoisomers) are pairs of structures that
suggested. Two conformations (like chair)
are mirror images of each other but not identical, similar to left and right
are interconvertible w ithout the breakage o f
hand which are the same but opposite. These are non super imposable and
covalent bond whereas 2 configurations (like
are designated as D and L-sugar. Each enantiomer often shows different
Alpha & Beta) can be inter converted only
chemical reactions with other enantiomers. Because of presence of many
by breaking a covalent bond.
enantiomers in living beings, there is usually a marked difference in effects
• If the hydroxyl group of anomeric carbon of
of two stereoisomers. For example only one of drug stereoisomer produces
ring sugar is not linked to other compound
desired effect while the other does not.
by a glycosidic bond, the ring can open and
• Isomers that are mirror images of each other are called enantiomers (optical
the sugar is called reducing sugar. Because it
isomers) and the 2 membes are designated as 'D' or 'L' sugars. In L form the
acts as a reducing agent with chromogenic
OH group on the asymmetric (chiral) carbon (i.e. a carbon linked to 4 different
Benedict's or Fehling's solutions. So the
atoms or groups) farthest from carbonyl carbon is on left, whereas in D
reagent gets reduced and sugar becomes
isoform it is on the right. Racemase enzymes inter convert D & L
oxidized. Only state of oxygen in aldehyde
enantiomers. In general, a molecule with n chiral centers have 2n
group decide if sugar is reducing or non
stereoisomers (enantiomers). Glyceraldehyde (triose) has 1 chiral carbon so
reducing.
2 1 = 2 stereoisomers, whereas hexoses have 4 chiral carbons so 24 = 16
stereoisomers.
• The presence of asymmetric carbon atoms provides optical activity to compound. A
beam of polarized light, when passed through a solution of optical isomer, is
rotated either to right, dextro rotatory (+) or to left, (levorotatory (-).
Stereochemistry (D/L forms) is independent of the direction of rotation of polarized
light, so D (+), D (-), L (+) or L (-) forms exist. Glucose being dextrorotatory is
also called dextrose and naturally occurring fructose is D (-).
Fischer Projection Formula; Diastereomers (Anomers)
• In Fischer projection formula to represent 3 dimensional sugar structure on
paper, horizontal bonds project out of plane of paper towards the reader,
a-D-G luco f)-D- Gluco
whereas vertical bonds project behind the plane of paper, away from reader.
pyrnose
• > 99% of sugars with 5 or more carbons exist in cyclic (ring) form (i.e. < 1 %
is in open chain or acyclic form) in which the aldehyde group (of aldose) or
Keto group (of ketose) has reacted with the alcohol (OH) group on the same
sugar to form hemiacetals or hemiketals. This cyclization makes the 2 C hair conform ations
carbonyl carbon (C 1 of aldose & C2 of ketose) asymmetric (now called
anomeric carbon) generating alpha and beta anomers. Because these are
not mirror images of each other they are also called diastereomers, and can
be spontaneously inter converted by mutarotation process. The alpha
anomer has same side and beta anomer has opposite side orientation of
hydroxyl (OH) group at anomeric & farthest chiral centers, in Fischer
projection. Biological enzymes use alpha or Beta forms preferentially like
glycogen is synthesized from alpha where as cellulose from Beta - D
glucopyranonose.
614 ■ A Complete Review of Short Subjects

_________________________________ Inulin like Fructan Prebiotics (ILF)_________________________________

Prebiotics are nondigestible food ingredients that should pass harmlessly through upper gastrointestinal tract to be a substrate for
advantageous probiotic agents (colonic bacteria) in the large bowel. These beneficially affect the host by selectively stimulating the
growth or activity of health promoting colonic bacteria like lactobacilli & bifidobacteria.
Inulin is a polysaccharide of fructose (& so a fructosan) found in roots & tubers of dandelions, dahlias and artichokes. It is
easily soluble in water and is used to determine GFR. But it is not hydrolyzed by intestinal enzymes and acid. So non
digestible oligosaccharide (NDO)- inulin like fructans are most widely used & studies prebiotics.
Inulin like fructans are non digestible in upper digestive tract b/o their ^-configuration of anomeric C 2Q. Their Beta
glycosidic bonds (linkages) resist the hydrolysis by enzymes & acid. As human digestive enzymes a-am ylase (in saliva)
and disaccharidases (like isomaltase, maltase, sucrase, lactase synthesized by small intestinal mainly jejunal mucosal cells)
can break a (1 -> 4) and a (1 -> 6) glycosidic bonds respectively; trehalase breaks a (1 -> 1) bond.
Humans lack (1- (1 —>4) endoglucosidase and therefore are unable to digest cellulose containing f l ( l —>4) glycosidic bonds
between glucose residue and chitin (exoskeleton of arthropods) containing f i d —>4) glycosidic linkage b/w N-acetyl glucosamine
residue.
ILF pass through small bowel without degradation and with out inhibiting (even actually promoting) the absorption of
nutrients & minerals like Fe+2, Ca+2 and Mg +2 (i.e. harmless passage through upper digestive tract).
ILF reach colon in intact form and become substrates for beneficial health promoting probiotic bacteria like bifidobacteria and
lactobacilli.
Chapter 2. FAT METABOLISM: REVIEW NOTES

Fatty Acids
F.A. are aliphatic carboxylic acids. The chain may be saturated (containing no double bonds & end in -'A noic') or unsaturated
(containing one or more double bond & end in - 'Enoic'). Carbon atoms are numbered from carboxyl carbon (carbon No 1). The
carbon atoms adjacent to carboxyl carbon (Nos. 2, 3 & 4) are also k/a a, fd& /respectively, and the terminal methyl carbon is K/a
a) or n- carbon. A indicates number & position of double bonds eg A9 = a double bond between carbons 9 & 10; co9 = a double
bond on the 9th carbon counting from to carbon. So

1 2 3 4 9 10 16
c- C - C- C- c- c- c- c- c == c- c- c- c- c- c- c
a P Y

(0<Z)
{

this is 16 carbon atom monoenoic acid (unsaturated fatty acid with one double bond) that is A9 and o f

Saturated Fatty Acids (SFA) Unsaturated Fatty Acids (UFA)


Saturated fatty acids (having no double bond) - Contain one (monounsaturated) or more double bond
are major component of meat product (beef (polyunsaturated). Are named monoenoic, dienoic trienoic, tetra enoic,
fat) making it white, greasy & solid at room pentaenoic & hexaenoic according to numbers of double bond 1,2 3, 4, 5,
temperature. Other main sources are dairy & 6 respectively
product and some vegetable oils such as - Monounsaturated fatty acids (MUFA) like olive oil rich in oleic acid
coconut &l palm oils (used in latin American & (when substituted for saturated FA in diet) lower both total plasma
Asian food). cholesterol & LDL cholesterol, but maintain or increase HDL
Consumption of saturated fat is a/w cholesterol; thus decreasing incidence of coronary heart disease.
increased incidence of coronary heart - Effect of polyunsaturated fatty acid (PUFA) on cardiovascular disease is
disease (by increasing total plasma influenced by location of double bond ( 0)3 = at 3rd bond position / <06 =
cholesterol & LDL cholesterol), and at 6th bond position starting from methyl end).
increased risk of prostate & colon cancer. So - Consumption of 0)6 or n-6 PUFA (mainly linoleic acid, 18:2 (9,12))
limiting intake of saturated fats is strongly when substituted for SFA, lower plasma cholesterol & LDL
advised. cholesterol, but HDL which protects against coronary heart disease is
SFA with 14 (Myristic) & 16 (palmitic) carbon also lowered. So the powerful benefits of lowering LDL are only
chain lengths are most potent in increasing partially offset because of decreased HDL. Nuts, avocados, olives,
serum cholesterol. Whereas, stearic acid (18 soyabean and various vegetable oils including sesame, cottonseed, corn oil,
carbon) found in chocolate etc has little effect sunflower oil, canola oil, safflower oil etc are common sources of 0)6 PUFA.
on blood cholesterol. - Dietary 0)3 PUFA suppress cardiac arrhythmias, reduce serum triacyl
Name Number of glycerols, decrease tendency of thrombosis, lower BP and substantially reduce
(Atoms) risk of cardiovascular mortality (i.e. are cardioprotective) but they have
Acetic 2 little effect on LDL or HDL cholesterol level. The to? PUFA mainly Alpha-
Propionic 3 linolenic acid, 18:3 (9 ,12,15) are found in plants (its acceptable range is
Butyric 4 0.6 - 1.2% of total calories). Flax seed oil, canola oil, soyabean oil are
Valeric 5 rich in co-3 PUFA. Fish oil (salmon fat) containing long chain (0-3- DHA
Caproic 6 (docosahexaenoic acid) and EPA (eicosapentaenoic acid) are (0-3
Capric 10 PUFAs. Two fatty fish meals per week are recommended.
Laurie 12 - M editerraneen diet rich in MUFA (olive oil) and co-3 fatty acids (from
Myristic 14 fish oils & some nuts) but low in saturated fats is a/w i LDL & total
Palmitic 16 cholesterol but little change in HDL: hence decreased risk of coronary
Stearic 18 heart disease. Plasma triacyl glycerols are unchanged.
Lignoceric 24 - Arachidonic acid a precursor of prostaglandins & leukotrienes is (0-6
PUFA.
616 ■ A Complete Review of Short Subjects

No. of Carbon No. of double Position of Family Common Systemic Name Occurrence
atoms bonds double bonds Name
16 1 A9 CO7 Palmitoleic Cis- 9- Hexadece- noic In nearly all fats
18 1 A9 CO9 Oleic Cis- 9- octadecenoic Possibly the most common
fatty acid in natural fats (olive
18 1 A9 Cl)9
Elaidic Trans-9- actadecenoic oil)
Hydrogenated and ruminant
fats
18 2 9,12 CO6 Linoleic all -cis- 9,12- octadeca Corn, peanut, cottonseed,
dienoic soyabean, and many plant oils
18 3 9,12, 6 CO6 y - Linolenic all - cis- 6 , 9 , 12 - Some plants, eg. oil of evening
octadeca trienoic primrose, borage oil; minot
fatty acid in animals
18 3 9 ,1 2 ,1 5 CO3 a - Linolenic all -cis- 9,12,15 Frequently found with linoleic
octa-deca trienoic acid but particularly in
linseed oil
20 4 5, 8 ,1 1 ,1 4 CO6 Arachidonic all -cis-5 ,8 ,11,14- Found in animals fats and in
Eicosatetraenoic peanut oil; important
component of phospholipids
in animals
20 5 5, 8 ,1 1 ,1 4 ,1 7 CO3 Timnodonic all -cis- 5, 8 ,1 1 ,1 4 ,1 7 - Important component of fish
Ecosapentaenoic oils eg. cod liver, mackerel,
menhaden, salmon oils.
22 6 4, 7 ,1 0 ,1 3 ,1 6 , CO3 Cervonic all -cis- 4, 7 ,1 0 ,1 3 ,1 6 , Fish oils, phospholipids in
19 19- Docosahexaenoic brain.
24 1 15 Neurvonic
acid

Fatty acid type Metabolic Effect Clinical Effect


LDL HDL
Trans fatty acid T I Increased incidence of coronary heart disease
Saturated fatty acid T Little effect Increased incidence of coronary heart disease
Increased risk of prostate & colon cancer
Monounsaturated fatty acid i A Decreased incidence of coronary heart disease
(MUFA)
CO-6 polyunsaturated fatty i i Decreased incidence of coronary heart disease
acid (PUFA)
CO-3 Polyunsaturated fatty Little effect Little effect Decreased incidence of coronary heart disease
acid (PUFA) Decreased risk of sudden cardiac death
Decreased tendency for thrombosis, reduce BP & serum
triacylglycerols and suppress cardiac arrhythmias

Trans Fatty Acids (TFA)

Procedure & Purpose Biological Significance


Lipid rich food, when exposed too long to oxygen (air), may - Trans fatty acids do not occur naturally in
spoil and become rancid d/t oxidative cleavage of double bond in plants, but occur in small amount in animals
unsaturated fatty acids, which produces aldehyde and (ruminant fat arising from action of
carboxylic acids of shorter chain length & therefore higher microorganisms in the rumen)
volatility. - Many fast foods are deep fried in partially
To improve/increase the shelf life and volatility (stability at hydrogenated vegetable oils & therefore
high temperature in deep frying), commercial vegetable oils contain high levels of trans fatty acids. So TFA
are subjected to partial hydrogenation /hardening. is a major component of many commercial
Hardening converts many cis double bonds in FA to single bonds baked goods, like cookies, cakes and most deep
Biochemistry: Fat Metabolism ■ 617

and increase the melting temperature of oils, so that they are fried foods. A clue to presence of TFA is
more nearly solid at room temperature. Margarine is produced "partially hydrogenated" word used in the list
from liquid vegetable oils by partial hydrogenation. of ingredients.
But the undesirable effect of partial hydrogenation Dietary intake of trans fats raise the level of
procedure is production of trans fatty acids i.e some cis triacyl glycerol & LDL (bad) cholesterol,
double bonds are converted to trans double bonds®. So trans increase the body's inflammatory response
fatty acids are chemically unsaturated fatty acids but behave (another risk factor for heart disease) and
like saturated fatty acid (i.e. they are atherogenic). decrease the level of HDL (good) cholesterol
Soft margarine, low in trans fatty acid (or containing not at and so a/w higher incidence of cardiovascular
all) is produced by refined technology disease, and avoiding these fats in diet
substantially reduces the risk of coronary heart
disease.
Consumption of trans fatty acid is a/w
increased risk o f cardiovascular disease,
diabetes mellitus ® (d/t increased insulin
resistance) and adversely affect endothelial
Trans Cis
function.
Fatty Acid

Essential Fatty Acid


Dietary essential fa tty acids in humans are linoleic acid and a- P a lm ita te , (16: 0) D ^ P a lm ito le a te
linolenic acid®, because we lack enzymes needed to synthesize them. 1 6 :1 (A9)

Plants provide these EFA. Linoleic acid is the precursor of (0-6 - I


iD
arachidonic acid and so prostaglandins. So the arachidonic acid S te a r a te , (18: 0) -► L o n g e r sa tu ra te d
FA
becomes EFA if linoleic acid is deficient in diet, a-linolenic acid is
precursor of co-3-fatty acids required for growth and development.
Those fatty acids that cannot be synthesized by human bodies® and O le a te
must be provided in diet. The most important essential fatty acid is 18: 1 (A9)
-» Linoleic acid® (because it serves as a basis for the production of I
other EFA) D (d e sa tu ra tio n ) in
EFA Dietary Source D ieta ry sources o f linoleic acid p la n ts o n ly

L in o le ic a c id c S a fflo w e r oil
S o y a b e a n oilQ
S a fflo w e r
oilQ
73%Q
(m ax)Q
I
L in o le n ic acidQ
C o m oilQ 57% L in o le a te
A r a c h id o n ic acid M e a t , eg g , m ilk ° .
S u n flo w e r 56% 18: 2 (A912)
E ic o s a p e n ta e n o ic acid F is h oilQ oilQ D
D o c o s a h e x a n o i c A cid Q B r e a s t m ilk S oy abean 51% In p lan ts
oilQ o n ly , D
Humans require but do not have the enzymatic capacity to r
a - L in o le n a te y -L in o le n a te
synthesize omega-3 PUFA a linolenic acid (ALA; 18:3 (A9' 12-15), and
must therefore obtain it in diet. However from ALA; humans can 18: 3 (A912- 15) 18:3 (A6-912)

synthesize 2 other omega PUFAs: eicosapentaenoic acid (EPA; 20 : 5


(A5' 8’ n-14-,7) and docosahexaenoic acid (DHA; 22:6 (A 4-7-10- 13<16' 19). 1 I'
An imbalance between dietary ratio of omega 6 & omega 3 (optimal O th e r P U F A s E c o s a trie n o a te

is between 1:1 and 4 :1 ) PUFAs is associated with increased risk of 2 0 : 3 (A811- 14)

cardiovascular disease.
Mammalian hepatocytes can introduce double bonds a t A9 position 1 °
o f FA but not between C-10 and m ethyl terminal end. So mammas A ra c h id o n a te

can not synthesize linoleate, 18:2 (A9’12) or ex- linolenate 18:3 (A9'n- 20: 4 (A5-8- 11’ 14)
1 S )Q
8E = E lo n g a tio n , D = D e sa tu ra tio n
However, plants can synthesize both; the desaturases that introduce
double bonds at A12 and A15 positions are located in ER and
chloroplast. So these are essential fatty acids for mammals; they
must be obtained from dietary plant material. Once ingested
linoleate may be converted to other PUFAs, particularly y
linolenate®, eicosatrienoate and eicosatetraenoate (arachidonate), all
of which can be made only from linoleate
618 ■ A Complete Review of Short Subjects

(Polar) Membrane Lipids


Lipids are heterogenous group of compounds that are relatively insoluble in water and soluble in nonpolar solvents like ether and
chloroform.
All major lipids have either glycerol or sphingosine back bone, to which 1 or more long chain alkyl groups and a polar head group
are attached. In glycerophospholipid, galacto (sulfo) lipid and triacyl glycerol, the alkyl groups are fatty acids in ester linkage
whereas, sphingo (both phospho & glyco) lipids contain single fatty acid in amide linkage to sphingosine backbone.
I----------------------------------------------------------------------------------------- 1
Phospholipids (PL) Glycolipids (GL)
- Phospholipids contain phosphoric acid residue in addition to fatty Glycolipids contain both carbohydrate and lipid.
acid and alcohol The carbohydrate portion is antigen
- So PLs are phosphatidic acid derivatives in which the phosphate is determinant and so the glycolipids are
esterified with the OH group of suitable alcohol (glycerol or sphingosine). antigenic. Two types include
- Both types of PLs are found in membranes & play a role in
generation of lipid signaling molecules.

1
Glycero-phospholipids Sphingo-phospholipids Glyco-sphingolipids Galactolipids
(Phospho-glycerides) (Phospho-sphingolipids) (sulfolipids)

F a tty acid FA

Ceramide Ceramide o
F a tty acid o V FA
— FA bO FA u
o C >,
4- tc
PO — A lc o h o l c X
M o n o or
3 M o n o or
x Oh
a — P0 4
cn o lig a sa c ch a rid e D isa cc h a rid e SO4
cn C h o lin e

- Phosphoglyceride contain glycerol backbone linked to


T
All sphingo-lipids (ie sphingosine In
2 fatty acid, a phosphate group and an alcohol (like containing lipids) are formed from ceramide. galactolipids,
serine/ ethanolamine/ choline/ inositol/ glycerol) Ceramide is synthesized from serine aa in ER 1 or 2
- Phosphatidic acid is the simplest (parent) and works as 2nd messenger in cell galactose
phosphoglyceride (PG) and is the precursor of other cycle/differentiation/ senescence and residues are
members of this group. All PGs contain (or are apoptosis. 3 types of sphingolipids are: connected by
derivative of) phosphatidic acid (diacyl glycerol with phosphosphingolipids (eg sphingomyelin), glycosidic
a PO 4 group on 3rd carbon). glycosphingolipids (eg cerebrosides, linkage to C-3
- PGs are formed from phosphatidic acid (PA) and globosides and gangliosides) and sulfatides of 1, 2 diacyl
an alcohol (or sulfo-glyco sphingolipids). glycerol
C o m p o n e n ts Sphingolipids are composed of 1 mol of long Galactolipids
G ly c e ro p h o s p h o ­ H ea d g ro u p Com m on N e t c h a rg e
chain amino alcohol sphingosine (4- found
lipid (a t p H 7) sphingenine) backbone linked to 1 mol of predominantl
P h o s p h a tid ic acid (P a ren t -1 long chain fatty acid through an amide y in plants are
(P A ) com p ound) linkage (at C 2) (i.e. = ceramide) and a polar localized in
head (P O 4 in phosphospingolipids & internal
P h o sp h o tid y l C h o lin e 0
c h o lin e (L e c ith in ) mono/oligo-saccharide in glycosphingolipid) (thylakoid)
P h o sp h a tid y l E th a n o la m in e PA 0 group joined by phosphodiester bond in membrane of
e th a n o la m in e phosphosphingolipids and by O-glycosidic chloroplasts.
(c e p h a lin ) bond in glycosphingolipids. They make
P h o sp h a tid y l serin e S e rin e -1 C- 2 bears fatty acid (saturated or 70-80% of
P h o sp h a tid y l G ly cero l -1 monounsaturated with 16,18, 22 or 24 carbon) total
g ly cero l in amide linkage. membrane
P h o sp h a tid y l M y o In o sito l -4 Ceramide is structurally similar to diacyl lipids of
in o sita l 4 - 5, 4, 5- glycerol & is parent compound.______________ vascular
b ip h o sp h a te b ip h o sp h a te I plants and are
C a r d io lip in D ip h o sp h a tid -2 Sphingomyelin Glycosphingolipids most abundant
yl g ly cero l (sphingophosp - It consists of ceramide membrane lipid
ho-lipid) (sphingosine + fatty acid) and in biosphere.
Biochemistry: Fat Metabolism ■ 619

Saturated (Ci6 or Cis) fatty acid at Ci and unsaturated It consists of antigenic carbohydrate - This
fatty acid (Cis or C20) is attached in ester linkage to C 2 ceramide (i.e. (mono/oligosaccharide) phosphate
(2 nd carbon) of glycerol. sphingosine - Like phospholipids, these are free lipid is
A highly charged or polar head group is attached amino essentially present in all evolved to
through a phosphodiester linkage to third carbon. In alcohol membranes but are particularly conserve P 04,
backbone abundant in nerve tissue. They a limiting
this head group PO 4' bears a negative charge at neutral
attached at C 2 are located in outer leaflet of plant nutrient
pH. The polar alcohol may be negatively charged
to long chain plasma membrane, where they in soil, for
(phosphotidylinositol 4,5, biphosphate), neutral
FA through interact with extracellular more critical
(phosphotidyl serine) or positively charged (phosphotidyl
an amide environment and play role in roles
choline or phosphotidylethanolamine).
linkage) and a regulation of cellular inter - Sulfolipids in
Phosphoacylglycerol containing choline (i.e.
polar head action, growth and plant
phosphotidyl choline /Lecithins) are the most
group. development. So when the cell membrane
abundant phospholipids o f cell membrane® &
Phosphoryl division/growth is bear negative
represent a large proportion of body's store of choline.
choline or dysregulated, there is dramatic charged
Choline is important in nervous transmission as
phosphoetha change in glycosphingolipid sulfonate
acetylcholine and as store of labile methyl groups.
nolamine (as composition of head group
Dipalmitoyl lecithin is very active surface active
polar head plasmamembrane - In this
agent and a major constituent of the surfactant®.
groups) are - Glycosphingolipids have been sulfonated
Phosphatidyl ethanolamine (cephalin) &
attached to identified as a source of blood lipid, glucose
phosphatidylserine (found in most tissue) differ from
carbon 1 of group antigens, embryonic residue is
phosphatidyl choline only in that ethanolamine or
sphingosine antigens, tumor antigens. joined to
serine, respectively, replaces choline.
through - They serve as cell surface diacylglycerol
Phosphatidylinosilot is a precursor o f second
phosphodiester receptors for cholera & tetanus in glycosidic
messengers (i.e. help in signal transduction)®.
bond. toxins and for certain microbes linkage.
Lysophospholipids are intermediates in the
It is found in & viruses,
metabolism of phosphoglycerols.
as an - Genetic disorder a/w inability
In cardiolipin 2 phosphatidic acids (PA) share a
constituent of to degrade glycosphingolipids
single glycerol (i.e. 2PA esterified through their PO 4
myelin in 1/1 lysosomal accumulation of
groups to glycerol) hence the name is diphosphatidyl
nervous these compounds,
glycerol. It is found exclusively in inner
tissue. - These can be of two types
mitochondrial membrane, where it maintain ETC. It
,______________ I______________ ,
is antigenic and is recognized by antibodies raised
against Treponema pallidum (syhillis bacteria)®. Neutral Acidic
Some PGs have ether linked fatty acids (Ether lipids) (Uncharged) glycosphingolipids = (Negatively charged)
like Neutral glycolipids glycosphingolipids
1 . Plasmalogens which constitute 10% of PL of - It consists of ceramide linked to It consists of ceramide 1/1
brain & heart muscles. These resemble 1 or more sugar residues at N-acetyl neuraminic acid
phosphotidyl ethanolamine (abundant in nerve tissue) carbon 1 = NANA/sialic acid (in
&/ or phosphotidylcholine (abundant in heart 1. Cerebroside (i.e. ceramide mono gangliosides) or a sulfate
saccharide)/are the simplest group on a galactose (in
muscle) but possess an ether link on Sn -1 instead
of ester link (i.e. ester linked fatty acid at carbon 1 neutral glycosphingolipids and sulfatides)
is replaced by ether linked unsaturated alkyl they contain either a galactose Negative charge at
(galactocerebroside) or glucose physiological pH is
group).
2 . Platelet activating factor (PAF), an unusual ether (glucocerebroside). provided by NANA/sulfate
Galactocerebroside are found in group____________________
phosphoglyceride has saturated alkyl group in
ether link to carbon 1 and an acetyl residue plasma membranes of neural Gangliosides are
(rather than FA) at carbon 2 of glycerol backbone. tissue and gluco cerebroside, derivatives of ceramide
PAF is synthesized by variety of cells. It binds to which occur in plasmamembrane oligosaccharide (glucosyl
surface receptors triggering potent acute of non neural tissue serves ceramide) that contain
inflam m atory & throm botic events®. PAF is one primarily as an intermediate in the oligosaccharide as their
of the most potent bioactive molecule it activates synthesis & degradation of more polar head group & >1N
inflam m atory cells & m ediates hypersensitivity, complex glycosphingolipids. acetyl neuraminic acid
anaphylactic & acute inflam m atory reactions®. It ■ As the name suggest, (Neu 5 Ac = NANA) a
causes p latelet aggregation & degranulation® and cerebrosides are found mainly in sialic acid at termini. These
activate neutrophils & alveolar macrophages to brain & peripheral nervous tissue. are found in ganglionic
Different galacto /gluco- cells of CNS, particularly at
generate superoxide radicals® (killing bacteria).
cerebrosides may differ in type the nerve endings. They
620 ■ A Complete Review of Short Subjects

of FA attached to sphingosine. have receptor & other


2. Globoside (ceramide functions. Notation G (for
oligosaccharide) are produced ganglioside).
by attaching additional mono Subscriptio M, D, T, or Q
saccharide (including N-acetyl indicate whether there is 1
galactosamine) to glucocerebrosides. (mono) 2 (di), 3 (tri) or 4
Globosides contain 2 or more (quatro) molecules of
sugars, usually D- glucose, D- NANA in ganglioside.
galactose or N-acetyl D Additional number
galactosamine designate monomeric
sequence of carbohydrate
Cerebroside (gluco/galacto) = attached to ceramide. GM3
Ceramide+Glucose/Galactose is the simplest ganglioside
(Cer - Glc / Gal) G represent ganglioside; M
Globoside (Lactosylceramide) is a monosialo containing;
P la te le tt a c tiv a tin g fa c to r
C e r - G lc -G a l & 3 is a number assigned
Globoside (Forsman antigen) on the basis of
★ All cells except mature RBCs can synthesize = Cer - Glc - 2 Gal - 2 Gal Nac chromatographic
phospholipids, whereas triacyl glycerol synthesis migration. Gmi,is receptor
occurs only in liver, lactating mammary glands, Neutral Storage Lipid (triacyl in intestine for cholera
adipose tissue and intestinal mucosal gland. glycerol) toxin.
★ Lung maturity of fetus is gauged by determining 2. Sulfatides are
the dipalmitoyl lecithin (DPPC) to sphingomyelin sulfoglycosphingolipids i.e.
(L/S ratio) in amniotic fluid. A > 2 ratio indicate cerebrosides that contain
maturity and major shift from sphingomyelin to sulfated galactosyl
DPPC synthesis that occur in pneumocyte at 32 residue. These
weeks gestation. sulfogalactosyl ceramide
★ A ceramide (containing 30 carbon FA) is major are found predominantly
component of skin & regulates it's water in nerve tissue (myelin) &
permeability. kidney.____________________

Ceramide

P h o s p a tid y lc h o lin e O n co m b in a tio n w ith O lig o s a c c h a r id e


4- I U D P s u g a r re sid u e )
Sphingomyelin M o n o s a c c h a rid e S u g a r re s id u e 4
I------------- 1 Globoside
U D P -g a la c to s e U D P -g lu c o s e
NANA
4 4
Galactocerebroside Glucocerebroside Ganglioside

Risk of Atherosclerosis & Coronary Heart Disease (CHD)

LDL cholesterol (especially oxidized LDL) uptake by macrophage is responsible for formation of foam cells - the hallmark
of atherosclerotic plaque. A positive correlation exists b/w plasma LDL cholesterol concentration & atherosclerosis. LDL is
degraded in liver (70%) and extrahepatic tissues (30%), both expressing LDL (apo-B-100, E) receptor.
LDL (apo-B-100, E) receptor is defective in familial hypercholesterolemia, increasing blood LD1 cholesterol levels and
causing premature atherosclerosis.
Apoprotein A-I is a measure of HDL (good) cholesterol & apo-B measures LDL (bad) cholesterol. Hence ratio of Apo B/AI
is the most reliable index of atherosclerosis (cardiovascular disease). Ratio of 0.4 is very good and ratio of 1.4 has highest risk
of heart accidents.
Lipoprotein (a), which inhibits fibrinolysis, consists of 1 mol of LDL attached to 1 mol of apo (a). It increases risk of coronary
heart disease (& atherosclerosis) if levels are > 30 mg/dl.
Biochemistry: Fat Metabolism ■ 621

High plasma homocysteine levels (> 15 |imol/L), increased hs-CRP (i.e. high sensitivity C-reactive protein), high levels of
serum triglyceride & long chain saturated fatty acids (e.g. stearic acid), hypertension, diabetes mellitus and smoking are
risk factors for atherosclerosis and coronary heart disease.
Atherosclerosis & coronary heart disease occurs in familial lipoprotein (a)/Lp(a) excess, familial hypercholesterolemia type
II a (defective LDL receptor apo B-100), familial type III hyperlipoproteinemia/ broad beta / remnant removal disease/
familial dysbeta Iipoproteinemia (abnormal apo E). Coronary heart disease also occurs in familial hyper triacylglycerolemia
type IV and hepatic lipase deficiency.

Sphingolipidoses

Sphingolipidoses (a part of lysosomal lipid storage diseases) are a group of autosomal recessive (except for Fabry
disease which is X-linked) inherited diseases that are caused by a genetic defect in the degradation (catabolism) of
sphingo (or glycosphingo) lipids containing sphingosine.
Common features of all sphingolipidoses include
1. Rate of synthesis of stored sphingolipid is normal but a specific lysosomal hydrolytic enzyme for degradation is
deficient resulting in accumulation of usually only a single sphingolipid containing ceramide (the substrate for
enzyme). It can also result from defects in lysosomal activator proteins (eg saposins) that facilitate access of
lysosomal hydrolases to short carbohydrate chains as degradation proceeds.
2. Enzymatic activity is affected in all tissues similarly. Accumulation of sphingolipids in neurons cause
neurodegeneration and shortening of lifespan.
3. The incidence is low in most populations except for Gaucher, Tay Sachs and Niemann- Pick disease (which show
high incidence in few ethinic groups). All are A R except Fabry's disease which has x-linked inheritance.
4. Can be diagnosed by measuring enzyme activity (in cultured fibroblasts or peripheral leukocytes) or by DNA
analysis. Prenatal diagnosis uses cultured amniocytes or chorionic villi. Histological examination can be also useful eg
shell like inclusion bodies in Tay Sachs and wrinkled (crumpled) tissue paper appearance of cytoplasm in Gaucher
disease.
5. Treatment is not effective for many diseases. Although recombinant human enzyme replacement therapy and bone
marrow transplantation can successfully treat Fabry's and Gaucher's disease. Substrate deprivation therapy (to stop
synthesis), chemical chaperone therapy and gene therapy (under investigation) are other alternatives.
Sphingolipidoses include (1) Neutral glycosphingo lipidoses (eg Fabry's, Gaucher's and Niemann Pick disease A and B);
(2) Generalized GM1 gangliosidoses; (3) GM 2 gangliosidoses (eg Tay Sachs & Sandhoff's diseases); (4)
Leukodystrophies (eg Krabbe globoid cell leukodystrophy, metachromatic leukodystrophy and multiple sulfatase
deficiency) (5) Lactosyl ceramidoses and; (6 ) FarbePs disease.
W olman's disease and cholesteryl ester storage diseases are not sphingolipidoses as these are a/w accumulation of
triglycerides & cholesterol esters (not sphingolipid/ ceramide).
M ultiple sclerosis is a demyelinating disorder (not sphingolipidoses) in which the lipid composition of white matter
resembles that of gray matter because of loss of both phospholipids (esp ethanolamine plasmalogen) and of
sphingolipids from white matter. The CSF shows raised phospholipid levels.

Lysosomal Storage Diseases without

AR inheritance Neurological involvement Hepatospleenomegaly


I I _________________________I___________________
MPS II - Hunter - MPS IV A, Morquio - Tay- sach's disease Q
Fabry's disease - MPS VI B, Morquio - Fabry's disease Q
Both are X-linked - MPS VI, Maroteaux- lamy - Mucolipidos is III; pseudo- Hurler polydystrophy
- Cholesterylester storage - K rabbe’s diseaseQ
disease - Metachromatic leukodystrophy
- Gaucher's disease type I * Cholestery ester storage disease (only hepatomegaly)
622 ■ A Complete Review of Short Subjects

I
Skeletal involvement Opthalmologic involvement Hematological involvement
I I
Tay- sach's disease MPS III (Sanfilippo) A, B, C, D - GM 2 gangliosidoses (i.e. Tay Sach's &
Fabry disease Gaucher's disease type 1 Sandhoff's disease)
Niemann- Pick disease, neuropathic Fucosidosis - Fabry's disease
type A P- Mannosidosis - Krabbe's disease
Krabbe's disease aspartyl glucosaminuria - Metachromatic leukodystrophy
Metachromatic leukodystrophy Krabbe's disease - Disorders of neutral lipids (i.e. Wolman,
Wolman's disease Wolman disease Faber & cholesteryl ester storage disease)
Cholesteryl ester storage disease Cholesteryl ester storage
Farber disease disease

★ Neurological involvement means mental retardation for all lysosomal storage diseases (LSD) except Fabry disease where it
is in form of painful acroparesthesias (i.e. burning pain in lower extremities )Q. And in GM2 gangliosidoses (Tay sach's &
Sandhoff’s disease), Niemann- Pick disease, /? -Mannosidosis and sialidosis (myoclonus), MR is associated with seizures.

★ In all MPS and ML opthalmological involvement, is in form of com eal clouding except in MPS II, Hunter disease, which causes
retinal degeneration. And ML III, pseudo- Hurler polydystrophy also show mild retinopathy and hyperopic astigmatism
along with corneal clouding. Retinal degeneration is also present in multiple sulfatase deficiency; and macular degeneration in
Farber disease & Niemann - Pick disease A & B. Optic atrophy is seen in metachromatic leukodystrophy; comeal dystrophy
& vascular lesions in Fabry disease. Cherry red spot is found in GM2 gangliosidoses (Tay sach's & Sandhoff's disease),
Sialidoses and, GM1 gangliosidosesQ.

Classification and Separation of Lipoproteins (Based on)

Electrophoretic mobility Density (SP unit) based Apolipoproteins


(Frederickson's Method) ultracentrifugation (Alaupovic Method)
- Lipoproteins are separated on their (Gofman-Method)
Lipoprotein Apolipo Density
charge/ size ratio. Mostly on paper Increase in lipid /protein ratio
Family protein class
/agarose electrophoretic media. decreases the density of lipoprotein and
- Increasing order of electrophoretic increase the floatation property of A A -I& HDL
mobility towards charged (anode) lipoprotein. All
end is Svedberg units of floatation (SF unit) is B Apo B LDL &
the rate at which lipoprotein floats (B48 & VLDL
L ip o p r o t e i n M i g r a t e in t o through NaCl and is = 1CH3 B 100)
C h y lo m ic r o n D o n o t m ig r a te &
cm/S/dyne/g at 26°c C Apo Cl, LDL,
r em a in a t o r ig in Q In ultracentrifugation separation c ii , cm VLDL &
L D L (|J- P- g lo b u lin r e g io n technique, only the order of LDL & VLDL HDL
lip o p r o te in ) is reversed (as compared to D Apo-D h d l3
V L D L (p r e -P - P re P -g lo b u lin r e g io n
E Apo E LDL,
lip o p r o te in ) Chylomicron Floats d/t lowest
(arginine VLDL &
H D L (a - oil g lo b u lin r e g io n density
rich) HDL
l ip o p r o te in ) VLDL
LDL
HDL Settles below d/t
highest density
Biochemistry: Fat Metabolism ■ 623

Chylomicron VLDL LDL HDL Key:


Protein
{ H ydrophilic
Triacylglycerol 90% 60% 8% 5% Phospholipid

Protein 2% 5% 20% 40%


A Triacylglycerols y
Phospholipids 3% 15% 22 % 30% I Hydrophilic
Cholesteral & Cesters 5% 20% 50% 25% Cholesterol and r layer
cholesteryl esters J
Liporoteins is composed of
1

Neutral Lipid are Surrounding shell


(Hydrophobic) (Hydrophillic)
- Triacylglycerol - Proteins (Apolipoproteins)
- Cholesterol and chlolesterol esters - Phospholipids
- (Unesterified cholesterol)

Chylomicrons & VLDL

In testin e Fat Lvier Fatty acid


(D ieta ry tr ig ly c e rid e s ) Q (E n d o g en ou s tr ig ly c e rid e s ) Q

Carried by Carried by
C h ylom icron (nascent) V L D L (nascent)
<B-48)<3 (B-100)O
(A p o

C hylom icron (B ^ , C, E) V L D L (B ,m, C, E)

Lipoprotein lipase

- located on wall of blood cappilaries


- Anchored to endothelium by heparan sulphate
HDL - found in adipose tissue, lactating mammary gland, heart, aorta,
lung, diaphragm, kidney, spleen.
FA
Extra - A c tiv a to r s : P h o s p h o lip id & apo C -II ^ F A ^ M o s t of it
hepatic
- In h ib ito r s : apo A I 1 & apo C -I1I® Supplied to
- It is not active in adult liver and not present in blood. Extrahepatic A p p r o x im a te s iz e a n d d e n sity of
tissue
- Released on heparin administration. s e ru m lip o p ro te in s. E a ch fa m ily of
- Insulin enhances its synthesis in adipocytes
lip o p ro te in s e x h ib its a ra n g e o f
- Function : It hydroli/es triglycerides (to FA & glycerols) of
siz e s an d d e n sitie s; th is fig u re
chylomicron and VLDL resulting in loss of 9 0% TG & loss of apo-C.
s h o w s ty p ical v a lu es
C h ylom icron rem nant V LD L rem nant (ID L) [N o te: T h e w id th o f th e rin g s
<B«, E) (Bioo/ E) a p p r o x im a te s the a m o u n t o f each
c o m p o n e n t]
“ 90% o fT G is lost - precursor of IDL
“ Cholesterol & cholesterol esters retained - It can be :
" They are taken up by liver by apo E specific
receptor m ediated endocytosis. R eceptor can be :
LD L (apo B-100, E) i I
Converted to T ak en up by liv er via
LRP (LDL receptor related protein) LDL LDL (apo B-100, E)
(M ainly) receptors

i
Liver Liver

O n ly on e m olecu le o f B 100 is presen t in each of


Cholesterol ester & rem aining Triglycerides
these lip o p rotein p articles & this is conserved
are hydrolyzed and m etabolized by hepatic
d u rin g transform ation. So each LD L p article is
lipase
derived from a sin gle p recu rso r V LD L particle.
624 ■ A Complete Review of Short Subjects

Dietary TG

Final destruction
in liver, extrahepalic
tissu e s (e.g.
lym pho-pcytes.
fibroblasts via
endocylosis)

Apolipoprotein
Lipoproteins are composed of neutral (non-polar) lipid core (containing triacyl glycerol and cholesteryl ester) surrounded by a
single surface layer shell of amphipathic (polar) apolipoproteins, phospholipids and nonesterified (free) cholesterol. These
amphipathic compounds are oriented so that their polar groups are exposed on the surface, thus making the particle soluble in
aqueous solution. The triacyl glycerol & cholesterol of lipo proteins are either exogenous (obtained from diet) or endogenous
(de novo synthesized). The protein part of lipoprotein is k/a apoprotein or apolipoprotein which constitutes nearly 70% of
some HDL and as little as 1% of chylomicron. Some apolipoproteins are integral & cannot be removed, whereas others are
free to transfer to other lipoproteins. Actual apolipoprotein & lipid content of each lipoprotein class is variable because of
constant interchange of these between them.
- A po A I & a p o A llQ are major apoproteins of HDL ( a -lipoprotein). The main apoprotein of LDL ( ft - lipoprotein) and
VLDL (pre f3 -lipoprotein) is B-100Q. Chylomicron mainly contains B- 48Q. B-100 is one of the longest single polypeptide
chain & is synthesized in liver, whereas, B-48 is 48% of B-100 d /1 introduction of stop signal by RNA editing enzyme. It is
formed by same mRNA but in intestine.
Apo B-48 and Apo B-100 are synthesized from same Apo B gene and same ApoB- m-RNA. Apo B 100 is a 100 KDa protein
synthesized in liver by full length translation of corresponding mRNA of Apo B gene. Apo - B-100 forms part of LDL, IDL
and VLDL. Apo B-48 is a 48 KDa protein (48% shorter form of Apo B-100) synthesized in intestine by partial translation of
same mRNA of Apo B gene. Apo B 48 forms part of chylomicron & chylomicron remnant. This difference between the
sizes of Apo B100 and Apo B48 occurs because post transcriptional processing (editing) of Apo B mRNA , deaminates the
cytidine (C) to uracil (U) in intestine at 2153 position. After cytidine deamination the CAA codon (which codes glutamine
in liver) becomes UAA (nonsense or stop codon) in intestine. This results in shorter apo B-48 protein being made in
intestine (and incorporated into chylomicron) than is made in the liver full length Apo B-100, incorporated in to VLDL.
Apoprotein C - I , C- II and C- III are smaller polypeptides freely transferable between chylomicrons, VLDL and HDL.
Apoprotein E is found in chylomicron, chylomicron remnant, HDL and IDL.
Functions of apoproteins are

1
Enzyme cofactors /inhibitors As ligand on lipoprotein particle for Form part of structure
I interaction with liprotein receptors in I
Enzyme Apoprotein function tissue eg. - Apoprotein B is structural
Lipoprotein lipase - A po-C II (main) & protein for chylomicron,
C l are co fa c to r s Q LDL receptor B-100 & EQ VLDL, IDL & LDL
- A po A ll & a p o C III LDL receptor related APo-E - Apoprotein AI is structural
are in h ibitorsQ protein (LRP) or protein for HDL.
Lecithin: Apoprotein AI is remnant receptor
cholesterol acyl cofactor HDL receptor Apo AI ★ LCAT is also a part of HDL
transferase (LCAT)
Cholesteryl ester Apo - Cl is inhibitor
transfer protein
Biochemistry: Fat Metabolism ■ 625

Low Density Lipoprotein (LDL)


i-------------------------------------------------------------- —
>It is derived from VLDL (after going through IDL stage) by loss of It is measured indirectly by using Friedewald
triacyl glycerol. equation i.e.
>Each LDL paricle contain a single apo B-100 m olecule Q and is 1. LDL cholesterol (in mg/dl) = Total cholesterol -
derived from single VLDL precursor HDL cholesterol - (0.20 x Triglycerides)
' It has highest cholesterol and cholesteryl ester content V It 2. LDL cholesterol (in mmol/1) = Total cholesterol -
transports 70% of total cholesterol HDL cholesterol - (0.45 x Triglycerides)
' The primary function of LDL particle is to provide cholesterol to American Heart Association guide lines (2003) for
peripheral tissues (or return it to liver). fasting LDL cholesterol levels & risk of CAD
’ Cholesterol in LD1 is called bad cholesterol because it is LDL Fasting Level
transported to arteries, where it is oxidized by free radicles and CAD Risk
Mg/dL Mmol/L
retained in arterial proteoglycans starting the formation of
<100 < 2.6 Optimal LDL,
atherosclerotic plaques. So it increases risk o f coronary artery
corresponding to reduced
disease Q, stroke and peripheral vascular disease.
but not zero risk
While there is growing consensus that levels o f apolipoprotein
(apo) B and the ratio o f apo B /apo- AI are more accurate 100-129 2.6-3.3 Near optimal LDL
predictors of CVD risk but it will take time before apoprotein 130-159 3.3-4.1 Borderline High LDL
terminology is recognized by general public & recommended by 160-189 4.1-4.9 High LDL level
National cholesterol Education program. Till then it may be more >190 >4.9 Very high LDL,
efficacious to continue adhering to already familiar & proven corresponding to highest
measurements of LD L-cholesterol /HDL- cholesterol ratio. risk of heart disease

LDL (apo B 100, and apo E) Receptors


Liver and many extrahepatic tissues express the
negatively charged glycoprotein LDL (apo B-100, E) C holesterol LDL. receptor
receptors 8 on the cell membrane in depressed coated pits LDL re c e p to r_
synthesis
C la th rin
in coated pits
that are coated on cytosolic side with a protein called
m m ^ C h oleste ro l
-In h ib it •
clathrin (which stabilizes the shape of pit) Unesterified
synthesis
E Lysosom e Cholesterol pool
It is so designated because it is specific fo r apo B-100 M ainly in m em branes — Stim ulale— ► ACAT

but not B-48®, which lacks the carboxy terminal


domain of B-100 containing the LDL receptor ligand, Cholesterol
e sierification
and it also takes up lipoproteins rich in apo E®.
So these receptors can take up LDL, VLDL remnant (i.t
IDL), and chylomicron remnant. The LDL is
metabolized via LDL receptor. Approximately 30% of
LDL is degraded in extrahepatic tissue and 70% in
liver.
The glycoprotein LDL receptor spans the membrane,
the B100 binding region being at the exposed amino
P re |VHDL H D L3
terminal end. After binding , LDL is take up intact
with receptor by endocytosis Q. The vesicle containing LDL loses its clathrin coat & fuses wit similar vesicles, forming larger
vesicles called endosomes. The pH of endosomes fall d/t proton pumping activity of endosomal ATPase which allows
separation of LDL from its receptor. The receptor migrates to one side of endosome, whereas LDL stays free within the lumen
of vesicle. This structure is called CURL - Compartment for uncoupling of Receptor & Ligand. This leads to

Hydrolysis of apoprotein and cholesteryl esters (present In LDL) 111 lysosome and LDL receptors are recycled
translocation o f cholesterol into the cellsQ_______________________________________ back to the cell surface

This influx of cholesterol inhibits cholesterol synthesis and uptake by Because cholesterol is esterified
- Inhibiting transcription of genes encoding HMG CoA synthase, HMG CoA reductase & by ACAT (acetyl CoA-
other enzymes involved in cholesterol synthesis cholesterol acyl transferase),
- Inhibiting synthesis (replenishment/recruitment or transcription) o f new LDL cholesterol influx stimulates
receptors Q via SREBP (sterol regulatory element binding protein) pathway, thus ACATactivity promoting
decreasing their number in plasma membrane cholesterol esterificationQ
■ LDL receptor is defective Q in familial hyper cholesterolemia, a genetic condition which increases blood LDL cholesterol levels
and cause premature atherosclerosis and coronary artery disease (CAD)C.
626 ■ A Complete Review of Short Subjects

Lipoprotein Lipase (LL)

It is an extracellular enzyme that is located on capillary walls, anchored to endothelium by negatively charged proteioglycan chains
of heparan sulfate.
It is not normally found in blood; however following heparin
injection, LL is released from its heparan sulfate binding sites
into circulation.
Lipoprotein lipase is found in capillaries of most tissues but
predom inantly in adipose tissue, cardiac & skeletal muscles,
lactating mammary glands Q, and also in spleen, lung, renal
medulla, aorta and diaphragm. Adult liver does not have this
enzyme 0 (Lippincot) / It is not active in liver - Harper
Hepatic lipase, bound to sinusoidal surface of liver cells, does
not significantly attack chylomicrons or VLDL triacylglycerol,
but rather is involved in chylomicron remnant and HDL
metabolism. It is also released by heparin.
Triacyl glycerols (TAG) o f chylomicron and VLDL are
hydrolyzed by lipoprotein lipase Q, progressively through
diacyl glycerol to mono acyl glycerol and finally to free fatty
acids and glycerol. Both phospholipids & apo CII are required
as cofactors fo r LL activity, w hile apo A II and apo CHI act as
inhibitors 0 .
The formed fatty acids are stored (by adipocyte) & used for
energy (by cardiac & skeletal muscles, and renal cortex). And if
not immediately taken up by cells, some of it are transported
back to circulation attached to albumin until their uptake does
occur.
Lipoprotein lipase synthesis and transfer to luminal surface of
capillary is stimulated by insulin (fed state). The highest
concentration o fL L is in cardiac muscles Q, reflecting that fatty
acids provide much of the energy for cardiac functions.
Heart lipoprotein lipase has a low Km and adipose enzyme has a large Km0 (about 10 times more than heart LL) for triacyl
glycerol. This allows heart continuing access to circulating fuel, even when plasma lipoprotein concentrations are low
whereas, adipocytes can remove fatty acids from circulating lipoproteins and store them as TAG only when lipoprotein
concentrations are elevated. This also enables the delivery o f fatty acids to be redirected from adipose tissue to heart in
starved state Q (when plasma TAG is low). A similar redirection occurs in lactating mammary glands for milk synthesis.

Frederickson Classification of Hyperlipoproteinemias

Typ N am e B a s ic d e fe c t E le v a te d E le ctro p h o re tic T r ig ly VLDL LD L HD C h o le s R e fr ig e r a ti C lin ic a l featu re


e lip o p ro te in c la s s ific a tio n c e rid e s (p re p) (P) L (a ) te ro l o n te st

I F a m ilia l - L P L d e fic ie n c y Q C h y lo m ic r o n s H y p er T T 1 N o rm a C rea m y - E rru p tiv e


lip o p ro te in - A b n o rm a l L P L Q c h y lo m ic ro n a e m 1 la y er, x a n th o m a s
lip a s e - A p o C -II ia in fra n a te - P a n c re a titis
d e fic ie n c y d e fic ie n c y Q c le a r or - A u to so m a l
/ ch y lo m icro n e ca u sin g in a ctiv e slig h tly re ce ssiv e
m ia sy n d ro m e LP L tu rbid - No
(L a ctesc en t a t h e r o s c le r o s is Q
)
I la F a m ilia l h y p e r - D e fe c t iv e L D L T ld l & H y p e r (3- N o rm a t (m ay T t C le a r (y ello - T e n d o n tu b ero u s
c h o le s te r o le m i r ecep to r 0 h y p e r c h o l e s t e lip o p ro te in e m ia 1 (-) or b e) o ra n g e tint) x a n th o m a s
a/ d efectiv e - M u ta tio n is a p o - r o le m ia Q (h eav y p b an d ) T - C o ro n a ry &
apo B B 100, P C S K -9 , p e rip h e ra l
/ sito stero lem ia A B C G 8, a th e ro scle ro sis
- A BC G S& A R H - A u to som al
d o m in a n t <2
- N o p a n c re a titis
Biochemistry: Fat Metabolism ■ 627

lib Familial U nknow n LD L & H ea v y p) an d p re N T t N or C lea r to - C o ro n a ry &


combined V LDLQ P b an d T slig h tly p erip h e ra l
hyperlipidemi turbid ath e ro scle ro sis
a - N o p a n c re a titis
& n o x a n th o m a

III Familial - A b n o r m a l i t y in C h y lo m ic r o n Broad P T t 1 to T u rb id to - P a lm a r


dysbetalipopr a p o £Q & VLD L lip o p ro te in a e m ia o p aq u e tu b e ro e rru p tiv e
oteinemia/Bro r em n a n ts o f (Broad floating w ith thin X a n th o m a s
ad P disease / d e n sity P band) cle a r layer - C o ro n a ry &
Remnant <1.019 ( p erip h e ra l
removal (P - V L D L f a th e ro scle ro sis
disease - A u to so m al
d o m in a n t
- N o p a n crea titis

IV Familial hyper - O ver V LD LQ w ith H y p e r p re p T T I i T T u rb id to - A u to so m a l


triacylglycerol p ro d u c tio n of ch o le ste ro l lip o p ro te in a e m ia opaque d o m in an t
emia V L D L o fte n rise an d H D L - C o m m o n ly
w ith g lu co se & LD L ten d a sso c ia te d w ith
in to le ra n c e & to b e c o ro n a ry h eart
h y p e rin su lin e m i su b n o rm a l d ise a se , ty p e II
a d ia b e te s
- Un known m e llitu s, o b esity ,
/Apo A-V a lc o h o lism &
ad m in istra tio n of
p ro g e sta tio n a l
h o rm o n e s
- N o p a n c re a titis
& no x a n th o m a
V Familial hyper - A po A - V / C h y lo m ic r o n H e a v y (h y p e r) t t I I t C rea m y - A u to so m a l
triacylglycerol unknow n & VIDLQ c h y lo m ic ro n & lay er d o m in a n t
emia p re P b a n d s in fra n a te - E ru p tiv e
tu rb id to x a n th o m a s
o p aq u e - p a n c re a titis
(L a cte sc e n t
)
★ N orm al TG's in 2 & Ted s. ch o lestero l in 2,3®

Abetallpoproteinemia Dysbetalipoprotelnemia
Pathogenesis Caused by mutation in the gene encoding microsomal Is due to defective form o f apoprotein E®.
triglyceride transfer protein (MTP), this prevent [Apo- E plays a crucial role in the catabolism
loading of apo - B with lipid & there is defective of chylomicron and VLDL.]
synthesis or secretion o f A po- B®
Lipoprteins level Absent / JChylomicron® tChylomicron remnant®
Absent / 4- VLDL® tVLDL remnant®
Absent / J-LDL® LDL normal
Plasma lipid Low cholesterol® (d/t decreased LDL) Increased TG 's (mainly)Q
abnormalities Low TG,s® (d/t decreased chylomicron & VLDL) Increased VLDL cholesterol®
Clinical features - M alabsorption o f fat® - Tuberous xanthoma®
- Early childhood diarrhea & failure to thrive. - Deposition of cholesterol in palmar creases
- Neuropathy® (striae palmaris)®
- Spinocerebellar degeneration (Ataxia)® - Risk fo r atherosclerosis® & its complication
- Retinitis pigmentosa® is increased
- Acanthocytosis®
Treatment - Vitamin E • Drugs:- HMG COA reductase inhibitor
- Low fat, High caloric diet - Febrates, Niacin
- Estrogen replacement therapy (In postmeno
- pausal women)
• Low - cholesterol, low fat diet
628 ■ A Complete Review of Short Subjects

Cholesterol: Structure, Esterification & Transport

Sterols are structural lipids present in the membranes of most eukaryotic cells. It has steroid nucleus, consisting of 4
fused rings, 3 with 6 carbons and 1 with 5 carbons. The steroid nucleus is almost planar and is relatively rigid; the fused
ring do not allow rotation about C-C bonds.
Cholesterol, the major sterol of animal tissues, is amphipathic, with a polar head (OH group at C3) and a non polar
hydrocarbon body (steroid nucleus & hydrocarbon side chain at C-17) about as long as a 16 carbon fatty acid in its
extended form.
Plants have stigmasterol & fungi have ergosterol. Bacteria cannot synthesize sterols. The sterols of all eukaryotes are
synthesized from 5 carbon isoprene subunits, as are fat soluble vitamins, quinones and dolichols
Cholesterol is the most important steroid (sterol) and precursor of all other steroids in body eg: - Corticosteroids, Sex
hormones, Bile acid Q, Vitamin D®.
Cholesterol occurs in body as free form (that exchanges readily between tissues & liproproteins) and ester form. Of the
cholesterol absorbed, 80-90% in the lymph is esterified with long chain fatty acids (to form cholesterol ester). Esterified
form is transported as lipoproteins & is not readily exchangeable and used for storage also. Highest proportion of
circulating cholesterol is found in LDL (beta lipoproteins) which takes cholesterol to tissue & also in HDL which carry
it from tissues to liver fo r degradation (scavanger)Q.
Esterfication of cholesterol: Cholesterol may be combined with long chain FA such as Linoleic acid (50%), Oleic acid
(18%), Palmitic acid (11%), Arachidonic acid (5%), and other fatty acid (16%).
_L
1
In tissue In Plasma Fate o f Isop en ten y l P yrophosphate & C h olesterol

A3 Isop en ten y l P yrophosphate


By transfer of By transfer of acyl group (mostly
acyl group from
acyl COA to
unsaturated) from lecithin to
cholesterol by enzyme. LCAT
\
Vitamin A, E, K
cholesterol by (Lecithin: cholestero acyl
enzyme ACAT transferase) Rubber, Plant Carotenoids
(acyl: LC A T hormones
L e cith in + ______ ^ L y so le c ith in +
cholesterol acyl abscisic acid &
C h o le ste ro l c h o le s te ro l e ste r Dolichols; Quinone electron
transferase) gibberellic acid
carriers eg Ubiquinone &
Plastoquinone

Chlorophyll (Phytochain)

C h olestero l

7 I \
B ile a cid s V ita m in D S te r o id H o rm o n e s
(ch o lic, (g lu co & m in e ra lo -
Polar ch enodeoxy- c o rtic o ste ro id ,
Head c h o lic , g ly c o c h o lic a n d ro g e n , e stro g e n ,
& ta u ro ch o lic) p ro g e ste ro n e )
Cholesterol with Steroid Nucleus

Cholesterol Synthesis

Sir John Comforth & Vladim ir Prelog described synthetic pathway and received Nobel Prize (1975).
Cholesterol is synthesized in liver, adrenal cortex, testes, ovaries & intestine. Although all nucleated cells can synthesize
cholesterol, including arterial walls. Enzymes for cholesterol synthesis are partly located in endoplasmic reticulum (ER) and
partly in cytoplasm.
Acetyl CoA is the starting m aterial & source o f all carbon atom s in cholesterol Q. The synthesis can be divided into 5 steps.
Biochemistry: Fat Metabolism ■ 629

1. Synthesis of Mevalonate

[ Thiolase Aceto Cholesterol synthesis is controlled by regulation o f HMG


2Acetyl CoA k --------► ACe‘°, COA reductase, which is a rate limiting enzyme®.
' acetyl
CoA
HMG COA Reductase activity

Increased by Decreased by
• Insulin • Mevalonate (immediate product)
• Thyroid • Cholesterol (ultimate product)
• Glucagon
• Glucocorticoid
• Drugs:- H y p olip id em ic drugs®
- Atorvastatin
- Simvastatin
- Lovastatin
Statins - Other statins

2. Formation of Isoprenoid units by loss of CO 2

Mevalonate is phosphorylated sequentially using ATP by 3 kinases (mevalonate, 5 phospho mevalonate & 5
diphosphomevalonate kinase) and then decarboxylated by 3, 5 diphospho mevalonate decarboxylase to form 5 carbon
atom active isoprenoid unit (ie isopentenyl diphosphate).

3. Condensation of 6 Isprenoid units form Squaline 4. Formation o f parental steroid, Lanosterol by


Cylization of Squalene
Isopentanyl d iphosphate Dimethyl allyl
Endoplasmic reticulum enzyme, squalene epoxidase
(5 C arbon) diphosphate (5C)
( Isomerase ] converts squalene to squalene 2,3 epoxide by
transferring methyl group on C14 to C13 and on C 8 to
C14. Ring closure (cyclization) is catabolize by
oxidosqualene-lanosterol cyclase to form lanosterol.

5. Formation Cholesterol (in ER membrane)

Methyl groups on C 14 & C 4 are removed to form 14


desmethyl lanosterol & then zymosterol; The double bond,
Dolichol is
form ed by between C 8-C9 is remove to C 5-C 6 in 2 steps forming A7-24
Cis PT; & Cholestadienol and then desmosterol (24-
side ch ain of
dehydrocholesterol). The double bond of side chain is
ubiquinone &
heme a are reduced to finally produce cholesterol by A24 reductase
form ed by (enzyme inhibited by triparanol).
trans-prenyl
tra n sfe ra se
630 ■ A Complete Review of Short Subjects

Measures to Lower Blood Cholesterol (CH)

Dietary alteration Hypolipidemic drugs


Diet lo w er in ch o lestero l & Drugs Mechanism of action Effect on lipid
satu rated fa tty acid®
• HMG- COA Reductase >LCH synthesis by inhibition of • tL D L
Increase dietery fiber® intke inhibitor HMG -C O A reductaseQ receptors^
Substitution of PUFA and
Lovastatin • Tl d l
monosaturated FA for
Simvastatin • -iTG's
saturated FA. i.e. satu rated /
Atorvastatin • Th d l
unsaturated FA ra tio is low®.
Lipid lowering effect of PUFA • Bile acid sequestrant • I Bile acid absorption, T T ldl
(Poly Unsaturated FA): Cholestyramine hepatic conversion of CH to T hdl
Tcholesterol excretion from Cholestipol bile acidQ
intestine • tLD L receptor on hepatocytes
Tcholesterol oxidation • Fibric acid derivatives • TActivity of lipoprotein T tg
(catabolism to bile acid) Gemfibrozil lipaseQ T ldl
LDL receptor upregulation. • I release of FA from adipose T hdl
tissue
• Nicotinic acid • J- L ip oly sis in adipocytes® i LDL
• I Production of VLDL i TG's
T hdl

Bile Acids Formation of Bile Acids


C h o le s te r o l
N A D PH + H +
Primary bile acids Secondary bile acids 7- a-hydroxylase®
o2
Synthesized in liver from Produced in intestine - R a t e lim it in g step® C -v ita m in M n: "N O C C "
cholesterol from primary bile acid C y to c h ro m e P -4 5 0
- C holic and by bacterial action 7 -o -O H c h o le s te r o l
chen odeoxy cholic - Deoxycholic acid I N A D P H + H*
acid® - Lithocholic acid & 7-a-hydroxylase®
r Oi M n: "N O C C "
- G lycine & Taurine their glycine and ■R a t e lim it in g s t e p <3 ^ C o A -S H
conjugate® of cholic & taurine conjugate. ------------------------P ro p io n y l C o A
chemodeoxycholic acid [N O C C ]

[in Liverj

Primary bile acids & Cholyl CoA Chenodeoxy cholyl CoA


their Glycine and
Glycine Taurine I
Taurine Conjugate
I---------------- '----------------- 1 Glyco and Taurochenodeoxy
Glycocholic acid® Taurocholic acid® cholic acid®

In intestine In iiitestine

Decojugation Decojugation
(Bacteria) + (Bacteria) +
7-a-dehydroxylation 7-a-dehydroxylation

Secondary bile acids Deoxycholic acid® Lithocholic acidQ


Biochemistry: Fat Metabolism ■ 631

De Novo Fatty Acid Synthesis Fatty Acid Synthase Complex I I I

Fatty acid synthesis requires • It is a polypeptide containing 7 enzyme activities - Harper


I______
I--------- • It is a multienzyme complex, made up of an ellipsoid dimer of
Multienzyme Coenzymes & identical polypeptide monomeric units arranged in head to tail
complex Cofactors fashion. Each monomeric unit contains six enzymes & an ACP molecule
- 7 enzyme Mn- "B e Human" ( Acyl carrier protein) - Chaterjae.
multicomplex Fatty • It is functional only when the two monomeric units are in association
- BiotinQ Be
acid synthase <2 with each other. Order of enzyme from end to end in each monomeric
- Acetyl Co enzyme A - HCO 3- Hu
unit is.
carboxylase (rate (Source of CO 2)
lim iting enzyme) - Mn2+Q M M nem onic - "K A S T A A M - H E R K A R A C P " i.e.
- ATP A A cetal
Building Block

Acetyl CoA®
- NADPW N
< M alonyl H ydratase, Enol,
R eductase, K eto A cyl R edu ctase, A C P - Thioesterase
Site
• Active sites of enzyme
C ytosol0

• This takes place in cytosol. A cetyl Co enzyme AQ is At Ketoacyl Synthase end At ACP end
primer (starting material) for formation of palmitic I I
acid (palmitate Cu fatty acid). Propionyl CoA acts as Cysteine - SH (Cys - SH) Pantothenyl - SH (Pan- SH)
primer for synthesis of long chain fatty acid having This site of monomer I accepts This site of monomer II accepts
an odd number of carbon atoms. Acetyl - CoA malonyl -CoA
• 1 st and rate limiting step is form ation o f m alonyl
from acetyl CoA® in presence of acetyl CoA Mnemonic- "1st AC of Ilnd PM " i.e. Monomer I accepts Acetyl CoA &
carboxylase0 by fixation of CO 2 (so called CO 2 has Cys - SH active site; Monomer II has Pan-SH active site & accepts
fixation reaction.) Malonyl CoA.

CO;

Mn**Q
A c e ty l C o A M a lo n y l C o A

ATP |Acetyl-CoA carboxylase®} aDP


| C o n ta in B io lin , B io tin c a r b o x y la s e , B o lic c a r b o x y la s e
c a r r ie r p r o te in , T r a n s c a r b o x y la s e , R e g u la lo r v a llo s te r ic s ite

T h is is a r a t e lim it in g ir r e v e r s ib le r e a c t i o n U
C itr a te is an a c tiv a to r a n d p a lm ity l C o A is in h ib ito r o f
en zym e

Fatty Acid Synthesis

Once malonyl CoA is synthesized rest of fatty acid


synthesis reaction takesplace with FA synthase
complex. Now Acetyl CoA binds to cys -SH end of
KAS (unit I) and malonyl CoA binds to Pan-SH end of
ACP (unit II) The coenzyme A is removed from both of
these and Acetyl (acyl) - malonyl enzyme is formed
This whole reaction is catalyzed by enzyme
transacylase.
I
Now, the acetate attacks malonate ( that means
acetyl group from Cys-SH site is shifted to malonyl
group of Pan-SH site) to form 'aceto - acyl- ACP'.
As Aceto acetate is attached to pan - SH of
monomer II, Cys - SH of monomer I becomes free.
This is a condensation reaction catalyzed by
632 ■ A Complete Review of Short Subjects

•CO ,
condensing enzyme - ketoacyl synthase. There is loss A celyl-CoA C 2- -► *M a lo n yl-C o A C3
1
o f one molecule o f CO2 I
j Acetyl-CoA
( g ) 1 carboxylase
HS - Pan - ( T ) -C y s - SH, ►CoA
i Acetyl
Malonyl C„ transfer from
Iransacylase >
Iransacylase
Now aceto- acetate (or 3- ketoacyl group) HS - Cys - ( ^ ) - Pan - SH - Cob+r (T) 10(T)
C.
undergoes reduction, dehydration and reduction Falty acid synthase
m ultienzyme complex — Gy n - Sf A cetyl CoA
again, catalyzed by ketocyl reductase, Hydratase an
enoyl recuctase respectively, to ultimately form E)
—( ^ 2 ^ ) - Pan - S - Malonyl CoA
saturated acyl - S- enzyme. All the above three Acyl (acelyl)-m alonyl enzyme
©
reactions occur on Pan-SH of monomer 11. . Consdensalion
3-Ketoacyl ; reaction
^ y n lh a s e : (acetyl attacks
i malonyl)
- ( T ) - Gyn - S - Free
Once saturated acyl - S - enzyme is formed; it is now
transferred to the 'cys -S H ’ of monomer I, freeing the —{ ^ 2 ^ } - Pan - S -A c e lo -A cetyi;

Pan - SH site ®. (which will accept another malonyl 3-Ketoacyl enzyme


(acetoacetyl enzym e)
group in each cycle, adding two carbon atoms per
3-Ketoacyl
cycle) . reductase By new malonyl
C oA combining to
PAN-SH of
I - ( T ) - Cys - S - m onomer II
O 0 Saturated acyl
This cycle starts with 2 carbon atom- acetyl CoA & n I residue is
- Pan - S -C -C H 2-C H -C H j
displaced onto
3 carbon atom - malonyl CoA 1/t formation of 5 D (-)-3-Hydroxyacyl enzyme free cysteine SH
group of
carbon atom compound acyl - malonyl enzyme. Hydratase ( 4) m onom er I
With a release of CO 2 (i.e. 1 Carbon atom) H ,C H Dehydration
ultimately a 4 carbon atom compound, saturated
- ( T ) - C ys - £
acyl enzyme is formed. With repetition of each
cycle 3 carbon atoms are added (in form of malonyl - ( j ^ - Pan - S -|6 '-C H = C H -C H ,
CoA) and 1 carbon atom is released (in form of 2,3-Unsaturated acyl enzyme
NADPH + H '-
CO 2); thus resulting in addition o f 2 carbon atom in
each cycle®. So after 7 cycles (1 first + 6 repetitions N AD P«-
[Enoyl reductase
©
H20
= 4+ 6x2 = 16C)® - ( T ) - Cys - SH
jThiogsterasel
Palmitic acid (16 - C compound) is formed.
After cycling through
s tep s( 2 ) - ( 5 ) seven times
' - CA2 J - "
Pan - S -C -C H Z- CHZ- CHZ
i (S )A cyl enzym e (saturated)
Palmilate
Palmitic acid is liberated from the enzyme complex by K e y : 0 ( 7 ) , individual monomers of fatty acid synthase

the seventh enzyme- thioesterase (deacylase)


B io sy n th e sis o f lo n g -c h ain fatty ac id s. D eta ils o f h o w a d d itio n o f a m a lo n y l re s id u e c a u se s the
a c y l ch a in to g ro w by tw o c a rb o n ato m s. (C y s, c y s te in e re sid u e ; P an , 4 ’-p h o sp h o p a n te th e in e ).
T h e b lo c k s in d a rk b lu e c o n ta in in itia lly a C 2 u n it d eriv e d fro m a c e ty l-C o A (as illu stra te d ) and
s u b se q u e n tly th e C o u n it fo rm e d in re a c tio n 5.

Fatty Acid Chain Elongation


Mitochondrial System Microsomal System
1. N ot a common pathway® 1. Usual common pathw ay fo r chian elongation®
2. Operates in mitochondria® 2. Operates in "microsomal" system. Chain elongation of FA
takes place in endoplasm ic reticulum (ER) ®
3. Palmityl-CoA is usually the starting material and 3. Acyl group that may acts as "Primer" molecule; May be
converted to stearyl-CoA. Other long-chain FA may be saturated FA series from CIO to C16 and some unsaturated C18
elongated. FA. End-product is next higher homologue of the 'Primer' acyl-
CoA molecule.
4. Operates under "anaerobic" conditions. It is favoured by 4. Requires presence o f O2®
a high NADH/NAD* ratio in cells Q. Also in presence of
excessive ethanol oxidation in liver.
5. Acetyl-CoA (two carbon unit) is directly incorporated 5. Acetyl moiety (two carbon) is added through Malonyl CoA
into the palmityl CoA molecule and not directly by incorporating C-2 units.
6 . NADPH is required which is provided by HMP shunt. 6 . NADPH is required as a reductant provided by HMP shunt.
7. Pyridoxal-P is required as a coenzyme® of the 7. Pyridoxal-P is not required®
"condensing enzyme" in the first reaction to incorporate
C-2 unit.
Biochemistry: Fat Metabolism ■ 633

Animal Cell P la n t cell

E n d o p la s m ic r e tic u lu m M ito c h o n d ria


- P hosp holipid s ynthesis - No |J oxidation
M ito c h o n d r ia ■
- S le ro l syn th es is (la te s ta g e )
- F atty acid (i-oxidation
- F a tty a c id e lo n g a t io n
- F a tly acid e lo n g atio n
- F a tty a c id d e s a tu r a tio n
- A c etyl C o A produclio n
(i.e d o u b le bond introduction
- K e to n e b ody synth es is
n e arly a lw a y s a l A9
position initially)
- o x id a tio n o f fa tt y a c id s

C y to s o l - P eroxisom e
- N A D P H production - Fatty acid oxidation (
(P P p athw ay; m alic HA)
enzym e) - Catalase peroxidase:
- N A D P H /N A D P + high H ,0 ,^ H ,0
- F a tly acid synthesis
- Isop renoid a n d sterol
s yn th es is (e a rly s ta g e )
C h lo ro p la st
- NA D PH , ATP production
- N A D P H /N A D P + high
- Fally acid synthesis

Lysophosphatidate
(l-a c y lg ly c e ro l-3-P 0 4 )

Acyl CoA
A cyl
(unsaturaled usually)
tra n sferase

2-Monoacyl glycerol
(M A G ) Phosphatidatc
C oA

( 1 ,2 d ia c y lg ] y c e r o l- P 0 4 )
CTP

v
PPi

C D P-D G -synlluise
r
CDP-diacylglycerol
Cardiolopin

M AG s ' In o s ito l
PI
acyl tran sferase f A cyl C oA
Phosphatidate
f H2OU
(in testine
Syn thase
mucosa)
phosphohydrolase IM P
S eC oA
S sP i
Phosphotidyl
inositol (PI)
1,2, Diacylglycerol (DG)
Y
P hospho
CDR ^ CDP-eholinc A cyl C oA K in a se I
ATP

c th a n o la m i/u j
r e th a n o la m in e
Phosphochoiine DG A cyhransferase (rate limiting)d N ADP
tran sferase transferase Q

V MP » CMP
‘ C oA PI-4-phosphalc
/• ATP
Phosphotidyl Phosphotidyl Triacylglycerol K in a se
ethanalamine choline ADP

PI-4,5-biphosphate

• So 2 molcules of acyl CoA, combines with glycerol 3 PO 4 to form phosphatidate in two steps catalyzed by glycerol 3 PO4
acyltransferase and 1 acylglycerol-3 -P 0 4 acyl transferase. P h o sp h a tid a te is h y d roly zed by p h osp h ory la se in to 1 ,2 d ia cy l
g ly cero l0. Diacyl glycerol acyl transferase, rate lim iting enzym e, catalyzes the on ly step sp ecific fo r triacyl g ly cerol synthesis.
• Choline & ethanolamine are first activated by ATP phosphorylation followed by CTP linkage to form CDP - choline or CDP -
ethanolamine which reacts with 1 ,2 - diacylglycerol to form either phosphotidyl choline or phosphotidylethanol-amine.
Phosphotidyl serine is formed from phosphotidyl ethanolamine by reaction with serine, and decarboxylation reverses the
reaction. In liver phosphotidyl ethanolamine gives rise to phosphotidyl choline by methylation.
634 ■ A Complete Review of Short Subjects

•These are formed by acylation of triose phosphates; and phosphatidate is the common precursor of triacylglycerols, many
phosphor-glycerols and cardiolipin. The biosynthesis is regulated by the availability of free fatty acids. Those that escape
oxidation are preferentially converted to phospholipids, and when this requirement is satisfied they are used for
triacylglycerol synthesis. Glycerol 3 PO4 & DHAP are intermediates in glycolysis making connection between carbohydrate &
lipid metabolism.

Carnitine

Chemistry & Synthesis Functions


Carnitine is (3- OH Y Trimethy ammonium butyrate Carnitine is a carrier molecule. It facilitates transpost
IP- OHGTB]
It is found in meat diet or is endogenously I 1
synthesized from lysine® and methionine in liver In mitochondria From mitochondria
(mainly) and kidney but not in skeletal or heat Transport long chain Transport acetyl CoA &
muscles. acyl - CoA in acetoacetyl Co from
Skeletal muscle contains about 97% of all carnitine in mitochondria for 13- mitochondria to cytosol
body. oxidation

Fatty acids must be first activated before they can be oxidized®. 'Active' FA (acyl - CoA) are formed in cytosol and for (3-
oxidation to occur it has to be transferred to mitochondrial matrix. Acyl CoA needs a transport system as it is impermeable to
inner mitochondrial membrane.
ATP A M P + PPi

FA activation FA transport across


mitochondria
Fatty acid is Long chain acyl CoA is
converted to an impermeable to inner
'active fatty acid' mitochondrial
or acyl CoA in membrane but bcome
presence of ATP permeable by
and co enzyme A converting into 'acyl-
by the enzyme carnitine' by the enzyme
acyl - CoA carnitine palmitoyl
synthetase transferase 1 (CPMT1)
(thiokinase). which is present in the
Acy- CoA outer mtochondrial
synthetase is membrane.
present in Now Acyl carnitine is
endoplasmic transported inside the
reticulum, mitochondria by the
peroxisomes & enzyme carnitine -
mitochondria acylcarnitine translocase
(inside & on which also export one Acylcarniline A c y l-C o a ► p-O xidation

outer membrane) molecule of carnitine


out. Role of carnitine in thetransport of long-chain
This is the only
faty acids through the inner mitochondrial
step in FA Acyl - carnitine is
membrane. Long-chain acyl-CoA cannot pass
degradation converted to acyl CoA
through the inner mitochondrial membrane, but
where ATP is in mitochondrial matrix
its metabolic product, acylcarnitine, can.
used & form by Carnitine palmtoyl
AMP®. transfase II (CPMT-2)
located on the inner
membrane
Biochemistry: Fat Metabolism ■ 635

P - Oxidation of Fatty Acid

It involves successive cleavage of Comparison of long chain, even


FA chain between the a (2 ) and /3(3) - Fatty acid
numbered, saturated FA synthesis & B
carbon atom (hence the name 13- C o a — SH-s A TP
oxidation (degradation)
oxidation) with release of Acetyl A cyl-C oA M g:

CoA.
synthetase Feature FA synthesis Degradatio
K A M P + PPi
n
Oxidation of acyl CoA to acetyl CoA Acyl-CoA
C side Occurs in Fed state Fasting &
is catalyzed by 'Fatty acid oxidase (O utside)
(after starvation
complex' in mitochondrial matrix ; Inner m itochondrial m e m b rane © Carnitine transporter

M side
carbohydrate
The successive steps are: (inside) rich meal)
Dehydrogenation —> Hydration —> Favouring High Low insulin
Acyl-CoA
Dehydrogenation. In l sl hormone insulin/ / glucagon
FA D
Dehydrogenation reaction, 2H glucagon ratio
A cyl-C oA
atoms are removed with the
formation of FADH 2(2ATP) in
dehydrog enase

'X k F A D H j
rp
^ -u 2o
ratio
Primary site Liver Muscle,
Respiratory
presence of acyl -COA A -tra n s -E n o y l-C o A chain Cellular site Cytosol liver
dehydrogenase, and Enoyl CoA (A2 H 20 Mitochondr
A -E n o y l-C o A
trans enoyl) is formed. hydratase ia
The hydration (addition of 2 H2O) Acyl (acetyl) Citrate Carnitine
occurs in presence of enoyl CoA L(+)-3-hydroxy- group carrier From From
acyl-C oA
hydratase with the formation of 3- mitochondri cytosol to
NAD*
OH (hydro) acyl CoA L(+)-3-hydroxyacyl- a to cytosol mitochondri
i
C oA dehydrog enase 2 .5 -P a
Further dehydrogenation (removal
N A D H + H^r - 4 - > H20
Respiratory Phosphopant Acyl carrier Coenzyme
of H2) catalyzed by 3 -O H - acyl CoA
3-K elo ac yl-C o A chain -etheine protein, A
dehydrogenase to form k e to acyl C o A -S H
containing coenzyme A
CoA & NADH (3ATP)Q. Thiotase active
carriers
_____________ i ____________
-A c y l-C o A A cetyl-C oA
Coenzymers NADPH NAD+,
Dehydrogenation f/b Hydration (reduction) FAD
f/b Dehydrogenation = (oxidation)
Reactions 2 Carbon Malonyl Acetyl CoA
Enoyl CoA f/b Hydroacyl CoA d o n o r/ CoA (donor (product of
f/b ketoacyl CoA = Products product of 1 acetyl beta
2CO , group) oxidation)
Activator Citrate
Finally 3 - keto acyl CoA is split by
Inhibitor Long chain M alonyl
thiolase and result in formation of:
fatty acyl CoA
CoA (inhibits (inhibits
acetyl CoA CPTI)Q
carboxylase)
Product of Palmitate Acetyl CoA
pathways
Repetitive 4 Condensatio Dehydro­
steps n, reduction genation,
/ hydration,
In this way a long chain fatty acid is
dehydration, dehydroge­
degraded completely to acetyl CoA
reduction nation,
(C-2unit). Thus palmitoyl - CoA (Cu)
(CRDR) thiolysis
form 8 acetyl CoA from 7 cycles.
(DHDT)
636 ■ A Complete Review of Short Subjects

P - Oxidation of fatty acid

_____________ Even Chain FA__________ __________________________Odd Chain FA______________________


2 carbons at a time are cleaved from acyl - With odd chain FA, P- oxidation produces acetyl CoA, until a three-
CoA in every cycle to form Acetyl - CoA carbon (propionyl -CoA) residue remains®.
(Zcarbon unit substance)® Propionyl CoA is the only part o f fatty acid that is glucogenic® [a s it
is converted to succinyl CoA, a consituent of TCA cycle]

Fatty Acid Number of C atoms ATP Production in p-oxidation of Fatty Acids


Acetic 2
Propionic 3 Steps Product ATP New
Butyric 4 formed cone
Valeric 5 Old ept
Caproic 6 concept
Laurie 12 • Each P oxidation cycle
Myristic 14 - Acyl CoA dehydrogenase 1 FADHz 2 1.5
Palmitic® 16 - P-Hydroxyacyl-CoA 1 NADH 3 2.5
Stearic® 18 dehydrogenase
★ In any even numbered FA, the no, of P oxidation • Total ATP produced 5 4

cycles would be " _i and no. of TCA cycles would • Each acetyl CoA, on
2 ’ oxidation in TCA (citric
be n /2 acid cycle)
* So ATP produced by palmitic (n o -16) acid is - Malate / Isocitrate/ Alpha 3 NADH 9 7.5
(a) ketoglutarate
- — - 1 = 7 x 4 = 28 A T P
2 dehydrogenase
- Succinate dehydrogenase 1 FADH2 2 1.5
= 8 x lO = 8 0 A T P i .e .l 0 8 A T P - 2 A T P = 106 A 7 7 5
2 - Succinyl CoA synthetase 1 GTP 1 1
• Total ATP produced 12 10
• Energy utilized in -2 ATP In all
activation of FA cycles

• Acording to older concept 1NADH (Mitochondrial) yields 3 ATP,


1 NADH (cytosolic) yields 3 or 2 ATP depending upon shuttle system, and 1 FADH 2 yields 2 ATP. So net gain of
ATPs in palmitic acid (16 carbon) used to be 129 ATP and in stearic acid (18C) 146 ATP.
For calculating this a formula derived by my friend is

Net ATP gain = llx n o o f carbon atoms In ?


2 2
(in even number fatty acid)

• But according to new concept 1 NADH (mitochondrial) yields 2.5 ATP, 1 NADH (cytosolic) yields 1.5 or 2.5 ATP
depending on the shuttle syste and 1 FADH 2 yields 1.5 ATP. Therefore, net gain in palmitic acid would be 106 ATP®
and in stearic acid 120 ATP®. So now suggested formula is

Net ATP gain ★ The formulas are also valid for even
(in even number fatty acid) = 7n- 6 numbered unsaturated fatty acids.

• 1 mol. Of palmitate involves net production of 106 mol of ATP via P oxidation, whereas only 26 mol of ATP are
produced when acetoacetate is the end product and only 21 mol of ATP when 3 hydroxybutyrate is the end product.
Biochemistry.' Fat Metabolism ■ 637

ATP Production (old)

___________________ Stearic Add__________________ ______________________ Palmitic Acid____________________


Stearic acid (Cis) is C 17 H 31 COOH. In 0- oxidation Palmitic acid is 16 carbon compoundsQ and it will
it will undergo 8 cycle [2-C removed in each cycle undergo 7 cycles & p rodu ce 8 acety l CoAQ (for citric acid
+2 (C2) units remaining in last cycle], producing 9 cycle) i.e.
acetyl CoA
- ATP from NADH & FADH2 in cycle = 7 x 5 = 35 ATPQ
- Each cycle produces = 5 ATP.
.'. 8 cycle produces = 8 x 5 = 40 ATP - ATP from acetyl CoA in Citric acid cycle = 12 x 8 = 96
ATPQ
- Each acetyl Co A produces = 12 ATP - ATP spend in activation = -2ATP
.■. 9 acetyl Co A produces = 12x 9 = 108 ATP
- Total ATP produced =148 ATP - 2 ATP
Net gain_________________= 146 ATP____________ Net gain = 129 ATPQ

p- Oxidation

In peroxisomes In Mitochondria
• Facilitates oxidation of very long chain FA (C20, C 22), do • Oxidation of both long & short chain FA.
not attack shorter chain FA.
• p - oxidation sequence ends at octanoyl - CoA & acetyl • End product is acetyl CoA.
CoA. Both further oxidized in mitochondria.
• In 1st oxidation step: (i.e. 1st dehydrogenation step; FAD
linked) Flavoprotein dehydrogenase passes electrons to • In l sl oxidation step:
O 2 producing H 2O 2Q, which is broken down by catalase Electron removed from flavoprotein
into H 2O & O 2. Energy released dissipates as heat dehydrogenase passes through respiratory chain to
• This system is not linked directly to phosphorylation & 0 2 to produce H 2 0 & this process is accompanied
ATP generation by ATP synthesis.
• Other functions:
To shorten side chain of cholesterol in bile acid formation. • This system is directly linked to ATP generation.
Synthesis of ether glycerolipids, cholesterol and dolichol.

Impaired oxidation of Fatty Acids: l/t disease often associated with hypoglycemia

Carnitine deficiencies Medium Chain fatty Zellweger's (Cerebrohepatorenal)


Secondary deficiency occurs in Acyl CoA syndrome
Decreased synthesis of carnitine in patients of liver Dehydrogenase • Very long chain fatty acids
disease, malnutrition or those on strictly vegetarian diets (MCAD) deficiency (VLCFA) i.e. >20 carbons,
Increased requirements as result of pregnancy, trauma, • Autosomal recessive, undergo a preliminary (3
severe burn, infection most common inborn oxidation in peroxidomes. The
Increased losses d/t hemodialysis or renal leakage (eg error of fatty acid shortend FA is then transferred
in preterm infants). oxidation to mitochondria for further
Genetic deficiency may be of 2 types • Infants are oxidation
CPT-1 deficiency affects on ly liverQ, where an particularly affected • So absence of peroxisomes in all
inability to use long chain fatty acid for fuel greatly because they rely on tissues (i.e. Zellweger
impairs the tissues ability to synthesize glucose during milk (which contains syndrome) or the inability to
a fast, resulting in reduced fatty acid oxidation and primarily medium transport VLCFA across
ketogenesis with hypoglycemia, coma and death. chain FA) for their peroxisomal membrane (i.e. X-
CPT II deficiency affects prim arily sk e le ta l and nourishment linked adrenoleukodystrophy)
card iac musclesQ and, when severe, the liver resulting • Dicarboxylic auduria 1/t accumulation of VLCFA (Cn,-
in cardiomyopathy, muscle weakness with myoglobinemia is characterized by C,w)
following prolonged exercise excretion of C6 - CIO • In ZS, accumulation of polyenoic
638 ■ A Complete Review of Short Subjects

Sulfonylurea drugs (glyburlde /glybenclamide, and co- dicarboxylic acids & acids in brain causes severe
tolbutamide) used in tt of type 2 diabetes reduce fatty by non ketotic neurological symptoms & death
acid oxidation & therefore hyper glycemia by inhibiting hypoglycemia, in 1st year of life
CPT-1 caused by MCAD Refsum's Disease
M alonyl CoA inhibits CPT-1 thus preventing long deficiency. AR disorder caused by deficiency
chain acyl group entery into m itochondrial matrix. The Jamaican Vomiting o f phytanic -a o x id a se / a
phosphorylation & inhibition o f acetyl CoA Sickness hydroxylase /phytanoyl CoA
carboxylase decrease malonyl CoA production, Eating of unripe fruit hydroxylase® resulting in
removing the break on fatty acid oxidation®. ofakee tree, containing accumulation of phytanic acid
hypoglycin toxin d/t its decreased aoxidation in
Presence of methyl group on p-carbon (eg in branched causes JVS peroxisome.
chain phytanic acid) makes P oxidation impossible and Toxin inactivates Phytanic acid impairs membrane
these fatty acids are catabolize in peroxisomes by a medium & short chain function, protein prenylation and
oxidation. acyl CoA gene expression causing
to oxidation ( of w cabon i.e. most distant from carboxyl dehydrogenase, primarily neurological
group) occurs in endoplasmic reticulum of liver & inhibiting (3- symptoms.
kidney, and preffered substrates are fatty acids of 10-12 oxidation and Phytanic acid is found in dairy
carbon atoms. causing hypoglycemia. products, & ruminant fat & meat. It
involves dietary restrictions.

Ketogenesis: Synthesis of Ketone Bodies in Liver

•Acetoacyetyl CoA is the starting material & this can arise in two ways:
- directly from terminal 4- carbon of fatty acid during (3- oxidation
- as a result of condensation of two acetyl CoA molecule (i.e. 2 C units) formed in |3 oxidation by reversal o f th iolase
reaction.
• Acetoacetate is the 1st ketone body to be formed. It can be formed from Acetoacyetal in two steps:
- Initially condensation of Acetoacyetyl with another acetyl CoA to form |3-OH - (3- methyl glutaryl CoA (HMG CoA) by
enzyme HMG CoA synthesis.
- Then HMG CoA is split into Acetoacetate and Acetyl CoA by the enzyme HMG CoA lyase. This is a commited step.
• Both enzymes (HMG Co synthase & lyase) of ketogenesis must be present in mitochondria for ketogenesis to take place.
This occurs only in liver and rumen epithelium. D(-) - 3 - Hydroxybutyrate is quantitatively the predominant ketone body
present in blood stream (ketonemia or hyper ketonemia) and urine (ketonuria) in ketosis.___________________________________

________________________ Ketolysis: Use of ketone Bodies in Periphral Tissues as Fuel_______________________


- Although liver constantly produces low levels of ketone bodies, their production becomes much more significant during fasting,
starvation, type II diabetes mellitus, twin lamb disease & in lactating cattle. Non pathological form of ketosis occurs under
conditions of high fat feeding & after severe exercise in post absorptive state.
- Under certain metabolic conditions (depletion of available carbohydrate coupled with increased mobilization/lipolysis of
FFF) associated with high rate of fatty acid oxidation, liver produces ketone bodies, which is used as respiratory substrate
(fuel) by extrahepatic tissue. Ketone bodies is collective name given to 3 compounds®: 1. Acetone 2. Acetoacetate 3. Beta
hydroxybutyrate. In a well fed state, concentraton of ketone bodies in blood does not exceed lm g 1100ml®.
- Extrahepatic tissues including brain but excluding cells lacking mitochondria (RBC) efficiently oxidize acetoacetate &
3(OH) butyrate. In contrast, liver actively produces ketone bodies but lacks
thiophorase (Succinyl CoA : acetoacetate CoA transferase) and so is Organ Fed Fasting Starvation
unable to use ketone bodies as fuel®. Brain Glucose Glucose Ketone
- Substrate utilized for energy production ----------------------------------------------
bodies®
Heart utilizes F.A even in fed state. Resting muscles use FFA as fuel source while Heart Fatty Fatty acid Ketone bodies
exercising muscle initially use glycogen reserve (glucose) and then FFF. acid®
- While acetoacetate & 3(OH) butyrate are oxidized by extrahepatic tissue Liver Glucose Fatty acid Amino acid
as fuel, acetone is difficult to oxidize (in vivo) and is mainly volatilized in Muscles Glucose Fatty acid Fatty acid
lungs. Both acetoaectate & 3(OH) butyrate are m oderately strong acids and are buffered depleting alkaline reserve, thus
causing fatal ketoacidosis (in very severe cases).
Biochemistry: Fat Metabolism ■ 639

Pathway of ketogenesis

F a lly a c y l C o A 2 A c c iv l C o A

A c c to a c e ty l C o A (slurring m uterial)

A c e ly l C o A -

I HMG CoA synthase I


C oA

H M G C oA
( 3 -O H -3 -m c lh y l g lu ta ry l C o A )

I HMG CoA lyase


A c c ty l C o A

A c e to a c e la te - 1st k c io n e lo b e fo rm e d
- o lh e r tw o k e to n e b o d ie s c a n b e d e r iv e d

3-OH butyrate / ^r o m l ^ * s ; ‘ A c e t o n e b y s p o n t a n e o u s d e c a r b o x y l a t i o n
• 3 -O H -b u iy r a le b y e n z y m e 3 -O H -b u ly r a lc
dehydrogenase ^
L 3 ^ J NADH+ d eh y d rog en ase

A kNAD+

3 -h y d r o x y b u ty r a le
I predom inant k eton e body
in b lo o d & urine in kelosis}

Regulation of Ketogenesis

Ketogenesis is regulated at 3 crucial steps

1. During m obilization of FFA (free fatty acids) from adipose tissue (lipolysis): Liver extracts ~ 30% of FFA passing through
it both in fed & fasting condition. FFA are the precursors of ketone bodies in liver. Therefore factors increasing /decreasing
FFA mobilization from adipose tissue increase / decrease ketogenesis. So factors affecting adipose tissue lipolysis, FFA
release and ketogenesis include

Decreasing Factors Increasing Factors


- Insulin, Nicotinic acid, Prostaglandin Ei, Adenosine & - Epinephrine, norepinephrine, glucagon, ACTH, alpha and
high concentration of FFA inhibit lipolysis by inhibiting beta M SH (melanocyte stimulating hormone), TSH , GH
synthesis of c-AMP at adenylyl cyclase site acting through Gi and Vasopressin increase lipolysis of triacyl glycerol by
protein. activating hormone sensitive lipase (through adenyl
- Lipolysis is more sensitive to changes in concentration of insulin cyclase)
than are glucose utilization & esterification. Insulin also - For optimal effect most of these require glucocorticoids and
stimulates phosphodiesterase (removing c-AMP) and thyroid hormones as facilitatory or permissive lipolytic
lipase phosphatase that inactivates hormone sensitive endocrinal factors.
lipase (HSL). FFAs also inhibit HSL. - On stimulation with lipolytic hormones, perilipin promote
- Perilipin protein is involved in formation of lipid droplets TAG degradation; thus regulating balance between TAG
in adipocytes and inhibit lipolysis in basal condition by storage & lipolysis in adipocytes
preventing access of lipase enzyme to stored triacyl
glycerol.

2. During entery of FFAs for B oxidation pathway by CPTI (into mitochondria): After uptake by liver, FFAs are either beta-
oxidized (in fasting/ starving) or esterified to TAG & phospholipid (in fed state). The entery o f FFAs into Beta oxidation
pathw ay is regulated by CPTI (carnitine palm itoyl transferase I)@ and the remainder of FA taken up by liver is esterified.
640 ■ A Complete Review of Short Subjects

In Fed State In Fasting / Starvation

I I
Because of low C PTI activity, fatty acid beta oxidation is More acyl CoA (FFA) enter beta oxidation by CPTI
depressed and nearly all FFA entering liver is esterified gateway into mitochondria and balance of FFA uptake not
to acyl glycerol and is transported out in VLDL oxidized is esterified

Acetyl CoA formed in beta oxidation is oxidized in citric


acid cycle or it enters the pathway of ketogensis.
- Liver is unique as it synthesize ketone bodies, primarily
3- hydroxybutyrate (beta-hydroxybutyrate) for use as fuel
by peripheral tissues, but not by liver itself. Ketogenesis is
favoured when concentration of acetyl CoA produced
from beta- oxidation, exeeds the oxidative capacity of
TCA. So as the level of serum FFA is raised,
G lu c a g o n
proportionately more FFA is converted to ketone bodies
& less is oxidized in TCA to CO 2. The fate of acetyl CoA
between ketogenesis & TCA is regulated so that the total
A c y l g ly c e r o ls
ATP produced from the oxidation of FFA remains constant as CPT I Gateway
Insulin -
M a lo n y l C o A
concentration of FFA in serum changes. Into mitochondria
& p oxidation
- 1 mol of palmitate produces 106 mol of ATP via beta
oxidation & CO 2 production in TCA, whereas only 26 Palmitate
ATP are produced when acetoacetate is end product and
only 21 ATP when 3 hydroxybutyrate is end product. So
ketogenesis allows liver to oxidize increasing quantities
of FAs. At the same time, ketogenesis releases CoA, A c e ty l C o A

ensuring its availability for continued fatty acid Ketogensslsl T C A C y c le


oxidation.
K e to n e B o d ie s CO2

R e g u la tio n o f K e to g e n e s is

Metabolism In Fed & Fasting


I----------------------------------------- I
Brain M etabolism Other Organs
I I
Brain tissue uses glucose as an exclusive flu e l, both Liver is the only source o f ketone body production, but it does
during the fed state as w ell as in fasting stateQ, except not utilize ketone bodies Q
during starvation when it can adapt to use ketonebodies Liver utilize glucose in fed, fatty acid in fasting, and aminoacid in
as an energy source 0. starvation
Brian does not utilize fa tty acids as a fu el 0. And brain Heart utilize fatty acid as main substrate in fed & fasting state;
contains no fuel reserves Q and so must be continuously and ketone bodies in starvation.
supplied with fuel from the blood. In brain 60% o f
Muscles do not utilize ketone bodies Q. They utilize glucose in
total energy is utilized during the resting stage Q
fed state and fatty acid in fasting and starvation.
Biochemistry: Fat Metabolism ■ 641

_______________Effects of Starvation On Metabolism


I--------------------------------
Ist Stage IInd Stage (last for Illrd - Stage
(first 2 to 3 days i.e. 48-72 hour) longest period, usually over 2 weaks) I
I I When the fat stores
• Glycogenolysis: liver glycogen is first 90% of energy requirement will be are almost
metabolized but d/t its limited storage it derived from fat & remainder 10 % exhausted energy is
cannot last long from protein derived from
• Gluconeogenesis: initially from glucogenic breakdown of tissue
Since the adipose tissue represents
protein
amino acids (alanine glutam ate ®) and then largest amount of stored fat 2 nd stage
by glycerol & lactate will last for longest period Digestive enzymes.
Followed by muscle
Glucagon secretion increases during Adipose tissue is broken into FFA
protein are first to
starvation. It reaches a p eak on 3rd day o f which is oxidized for energy
be utilized, as these
fa s t at the time o f m axim al production and used for ketone body
are least required
gluconeogenesis ®. There after glucagons formation.
level declines as the ketones & fatty acids This stage last for
The burning of fat is reflected by a
become the major source of energy. less than one week &
lower RQ (~ 0.73) i
G anong-350 leads to death.

________ Metabolic Alterations During Fasting / Starvation________


Absorptive (fed) state is 2-4 hours period after ingestion of a normal meal. During this period, transient increase in plasma
glucose, amino acids and triacyl glycerol (TAG; d/t increased chylomicron synthesis in intestinal mucosa) occur. The elevated
insulin to glucagon ratio (in response to increased glucose) make it anabolic phase characterized by increased synthesis of
TAG, and glycogen to replenish fuel stores and enhanced synthesis of protein. During absorptive phase, virtually all tissues
use glucose as fuel.

Fasting begins if no food is ingested after absorptive phase. Metabolic changes include

Features During Brief Fasting During Prolonged Fasting


Liver - Glycogenolysis (glycogen degradation) maintains blood - Liver glycogen is nearly exhausted after 10-18
glucose level initially (a transient early response) hours of fasting, therefore glycogenolysis can no
- Gluconeogenesis begins 4-6 hours after last meal and longer maintain glucose level.
becomes fully active after depletion of liver glycogen stores. - Rate of gluconeogenesis is decreased in prolonged
Glucogenic amino acids & lactate from muscles and glycerol from fasting to spare functional proteins and therefore
adipose tissue is utilized. less protein is degraded to provide glucogenic
- Beta-oxidation of fatty acids is increased for energy aminoacids
production and ketone body formation. FAs obtainjm,ed - Beta oxidation of FA and production of ketone
from adipose tissue lipolysis becomes the main fuel. bodies continue to be increased. Ketone bodies can
However RBC and brain continue to use glucose mainly. be used for fuel by most tissues (except liver)
Blood levels of ketone bodies (esp 3-hydroxy butyrate) including brain, once their level in blood is
increase gradually. sufficiently high.
Adipose - Decreased FA & TAG synthesis d/t decreased glucose transport by insulin sensitive GLUT-4 into adipocytes
Tissue - Increased degradation of TAG (by hormone sensitive lipase) resulting in increased release of FFAs and glycerol.
- Decreased uptake of FA (in adipocytes from circulating TAG of lipoproteins) d/t low lipoprotein lipase activity.
Skeletal - Rapid breakdown of muscle protein providing aminoacids - Rate of muscle proteolysis decreases paralleling a
Muscle for gluconeogenesis decline in the need fo r glucose as a fuel fo r brain
- Resting muscle use FFA and ketone bodies for fuel which has begun using ketone bodies as a source o f
energy®.
- FA is mainly used as source of fuel. The increased
use o f ketone bodies by the brain as a result o f their
increased concentration in blood is correlated with
decreased use o f ketone bodies by muscle®.
Brain, - Urea production d/t proteolysis increases - Urea production by liver decreases
RBCs - Brain and RBC use glucose as fuel - Brain use of glucose decrease but RBCs still need
glucose as fuel
642 ■ A Complete Review of Short Subjects

Biochemical Tests for Detection of


I---------------- 1
Ketone Bodies Reducing Sugars Bile salts
- Ketone bodies (such as Benedict's test and Fehling's test are used Hay's test detect bile salts (sodium salts of
acetone, acetoacetic acid/ to detect reucing sugars like glucose in taurocholic acid & glycocholic acid) in urine.
diacetic acid, and urine. Positive test indicates obstruction in biliary
betahydroxy butyric acid) So the patient with diabetic ketoacidosis passage.
are produced exclusively in will give positive Benedict's test and
liver ® as products o f fa t Rothera's test both®, whereas urine of Bilirubin
catabolism® usually in patient with starvation will give a negative - Fouchet's (barium sulphate) test, G m elin's
diabetes mellitus Benedict's test but a positive Rothera's (nitric acid) test and Vanden Bergh
(ketoacidosis) & starvation. test®. (diazo/diazotized sulphanilic acid reagent)
- Ketone bodies serve as an Benedict's reagent contain copper sulphate, tests detect bilirubin.
alternate source of energy sodium carbonate & sodium citrate. - Conjugated bilirubin being water soluble
especially in brain (& other gives direct Vanden Bergh test (immediate
parts of CNS) providing 50- R educing sugar —> Enediol '1v / Cu** <— CuSO-i color); whereas free bilirubin, being water
70% of brain energy needs insoluble give indirect reaction (i.e. color
Oxidized A Reduced
during starvation. This is an after extraction with alcohol)
adaptation for survival Sugar acid C u ++- » C u O H - » C u 20
during starvation. Yellow Red Bile pigments
- Ketone bodies in urine are Benedict's is a semiquantitative test i.e. color - Uro & steco-bilinogens (UBG & SBG) give
detected by Rothera's and roughly indicating concentration of sugar positive Ehrlich's (para-amino
Gerhardt's test. blue (absence), green (0.5%), yellow (1%), benzaldehyde reagent) test to form red
orange (1.5%), and red (> 2% i.e. > 2 color.
Rothera's test Gerhardt's gm/ 100 ml reducing sugar) - Schlesingeris test is positive for Uro &
test Any reducing sugar (like glucose, fructose, Sterco-bilins (UB & SB) to give brilliant
Also called Also called lactose, galactose and pentose/xylulose) will green fluorescence with zinc ions.
Nitro ferric give positive Benedict's test. So Benedict's - Rosenbach's test
prusside test chloride test test is positive in________________________ Bile
Detects Detects
- Glucosuria (diabetes mellitus) pigments Detection - Tests
acetone & acetoacetic
- Fructosuria (fructose Fouchet's; Ehrlich's Schlesingers
acetoacetic acid
in tolerance/aldolase B deficiency, Gmelin;
acid
fructokinase deficiency) Vanden
Urine (5ml) is FeCl.i is
- Lactosuria Bergh
saturated with added drop
- Galactosuria (galactose - I-PO 4- uridyl Bilirubin + - -
solid by drop to
transferase deficiency) Bilinogens +
ammonium 3ml urine. - Pentosuria (xylulosuria) (UBG,
sulphate. And - Presence of non carbohydrate reducing SBG)
Adding few appearance of substance in urine like ascorbic acid Bilins - - +
drops of red wine (vitamin C), homogentisic acid, (UB + SB)
sodium color glucuronides and salicylate.
nitroprusside indicates
* Presence of proteins is detected by heat
f/b liquor presence of
coagulation test, acetic acid test and
ammonia 1/t ketone bodies
Haller's nitric acid test®. Whereas
appearance of
benzidine test detects presence o f blood
purple/pink
permanganate
(hemoglobin > myoglobin)®.
ring
Chapter 3. STRUCTURE, FUNCTION AND REPLICATION OF INFORMATIONAL
MACROMOLECULES: REVIEW NOTES

Pyrimidine Synthesis & Catabolism SH


Unlike purine ring, which is constructed on a preexisting ribosome 5 PO 4, (derived from PRPP) the pyrimidine ring is
synthesized from aspartic acid (contributing N j , G, Cs, & G atoms), amine N 2 of glutamine (source of N3) and CO 2 (source
of G atom of ring). Although both purine & pyrimidine synthesis require glutamine, aspartic acid & PRPP as essential
precursors.
The steps are Mn- "G A S CAP AA CAA DHOA OA OM P UMP U D P" see the rhythm AA, OA, MP, DP
It uses 2 multifunctional-multicatalytic polypeptides, which ensures efficient channeling of carbamoyl phosphate to
pyrimidine synthesis.
First 3 enzymes (i.e. CPS II, Aspartate trans carbamoylase and Dihydroorotase) are three different catalytic domain of
single CAD polypeptide chain (name derived from l sl letter of each domain).
Next (4th) non multifunctional enzyme DHOA dehydrogenase is the only m itochondrial enzyme®
O rotate phosphoribosyl transferase and O rotidylate (OMP) decarboxylase ® are also catalytic domains of a single
polypeptide chain of bifunctional enzyme UM P synthase. Orotic aciduria is caused by defect of single (type II) or both
(type I) domains and result in poor growth, megaloblastic anemia & excretion of large amounts of orotate in urine. Uridine
administration improves condition.
Allopurinol & 6-azauridine drugs may precipitate oritic aciduria & oritidinuria by inhibiting OMP decarboxylase (i.e type
II orotic aciduria)®. This enzyme is actually blocked by product of allopurinol & enzyme OPRT and 6 -azauridylate.
Orotic aciduria occurs in deficiency of liver mitochondrial ornithine transcarbamoylase enzyme of urea cycle and Reye
syndrome (in which severly damaged mitochondria is unable to utitize carbamoyl phosphate). Excess carbam oyl
phosphate exits to cytosol® and becomes available for cytosolic over production of orotic acid (by stimulating pyrimidine
biosynthesis).
Carbamoyl Phosphate U n l i k e p u r i n e r in g , w h i c h is n o t c l e a v e d in h u m a n c e lls , th e
Synthetase p y r i m i d i n e r i n g is o p e n e d & d e g r a d e d b y dihydropyrim idine
I II
dehydrogenase ( D H P D ) & B e ta -u r e id o p r o p io n a s e to h ig h ly w a te r
Occur in Mitochondri Cytosol
s o lu b le p r o d u c ts B eta-alanine (from uracil) an d B eta-am inoisobu tyrate
Used in Urea cycle Pyrimidine (from thym ine), w ith p r o d u c tio n o f N H 3 & C O 2 . D H P D d e fic ie n c y
synthesis r e s u lt s in B eta-hydroxybu tyric aciduria; a disorder o f pyrim idin e
Activator N-acetyl PRPP
catabolism (and also o f Beta am in o acid m etabolism s in c e f o r m a t i o n o f
glutamate
B e t a - a la n in e & B e t a - a m i n e is o b u t y r a t e is a l s o i m p a ir e d ) is a ls o k / a
Nitrogen Ammonia 7-amide group of
com bined uraciluria-thym inuria. H ig h w a te r s o lu b ility o f e n d
source glutamine
p r o d u c t s r e s u lt s in f e w c l in ic a l f e a t u r e s .

Purine Synthesis
GDP G T P A s p a rtic ac id NAD NADH X a n th in e
M o n o p h o s p h o ta e
Adenylosuccinate IM P dehydrogenase
(X M P )
Synthetase
G lu ta m in e
ATP
Adenylosuccinase Transam idinase/
GMPsynthetase
'A D P
F u m a ra te n5n10 G lu ta m a te
M e th y l THF
A denosine
G u a n o s in e M o n o
M o n o p h o s p h a te
p h o s p h a te G M P
(A M P )

S o u r c e s o f a to m s in p u r in e r in g
M ycophenolic acid is reversible, uncompetitive inhibitor o f IMP dehydrogenase. It deprives rapidly proliferating T & B
cells of nucleic acids and used to prevent graft rejection.
644 ■ A Complete Review of Short Subjects

Uric Acid Synthesis (Purine Catabolism)


U ric A c id Synthesis (P urine C atabolism )

AM P Autosomal recessive deficiency l/t severe GMP


H20 ^ 1 com bined Im m uno deficiency (SCID)involving
T-cell, B-cell, & NK cell depletion
(lymphocylopenia). If untreated l/l death
R* - ' 4 before 2 yrs.
Adenosine Guanosine
H20 -J |A de n o s in ed e am in a se (ADA)] Purin e nucleoside
phosphorylase

Guanine
^ [Purine nucleoside phosphorylase] AR deficiency, is less severe,
Hypoxanthine Purine nucleoside phosphorylase affects only T-cells and cause
recurrent infection &
neuro development delay
X anthine Oxidase
’" t
A lto p u rin o l - Guanase
Inhib its xanthine oxidase
[u ric a cid synthesis in h ib ito r0]

|Xa^rtne |

A llo p u rin o l ~ X anthine oxidase

Uric acid (blood)

Increased uric acid


excretion (u ric o s u ric p ro b e n c ic f +
dru9s) Sulfinpyrazone

Uric acid (urine in humans) A llan toin


bsen t im hum ans (in mammals other
than higer primates)

★ Hyperuricemia result in deposition of sodium urate crystals in kidney and joints


★ Hyperuricemia does not always lead to gout, but gout is usually preceded by hyperuricemia.

Gout
This is characterized by hyperuricemia with recurrent attack o f acute arthritic joint inflammation, caused by deposition o f uric acid
crystal. .-----------------------------------------------.--------------------L----------------------------------------------------------------- -
Types Clinical Features Lab Diagnosis Treatment
Primary gout ■The vast majority o f hyperuricemic Serum uric acid level can In Acute Gout
(hypeuricemia patient are asymptomatic. Any factor be normal or low a t the • N SAIDS : eg. Indomethacin® , Naproxen
): that cause either an abrupt increase time o f acute attack®. , Piroxicam
Due to inborn or decrease in the serum urate levels Serum uric acid is - strong anti inflammatory action
error of may provoke an acute attack, the alm ost alw ays elevated - effective in terminating the attack®
metabolism best correlations being factors that at som e time ® & can be • Colchicine
and is caused cause an abrupt fall®. used to follow cource of - It has no effect on blood uric acid level®
by ■Mostly affecting middle- aged to elderly hypouricemic therapy. - It does not inhibit the synthesis or
most often d/t men and postmenopausal women. Radiologically: promote the excretion of uric acid.
urate crystal ■Usually only one joint is affected, 1st Tophi ®: eccentric soft - It inhibits release o f glycoprotein® which
less often d/t m etatarsophalangeal join t is M.C tissue selling which may aggravates inflammation by increasing
T urate involved® calcify lactic acid production [Lactic acid —> I
synthesis »Polyarticular acute gout is seen in - D efinitive diagnosis pH —> urate crystals precipitated]
Secondary male hypertensive patient with 1 1 • Corticosteroids:
A spiration A spiration
hypeuricemia: ethanol abuse & postmeno pausal - In refractory cases &
of synovial of
Caused by women - Those not tolerating NSAIDs /
fluid of Tophaceous
other disease ■The 1st episode frequently begins at affected deposits in Colchicine
eg: night with dramatic joint pain and joint tissue In Chronic Gout
Cancer swelling 1 • Uricosuric drugs : Probencid® and
Chronic renal ■Joint becomes warm, red & tender Strongly birefringerent sulfinpyrazone® promotes uric acid
insufficiency ■Tophi ®: aggregate of monosodium needle shaped MSU excretion & reduce blood level.
Myeloprofifera urate monohydrate (MSU) crystals crystals with negative • Allopurinol® inhibits uric acid synthesis
tive disorder form in extraarticular & articular elongation under by inhibiting xamthine oxidase®, [an
Drugs structures, cause deformity and polarized light®. enzyme responsible for uric acid
destruction of hard and soft tissue. synthesis]
Biochemistry: Structure, Function and Replication of Informational Macromolecules ■ 645

Purine Nucleotide Biosynthesis


The total- body urate level is the net result between urate production and excretion. Urate production influenced by, dietary
intake of purine, pyrimidine, de novo synthesis of purine, nucleic acid turnover and salvage by phosphoribosyl transferase
activity.

De novo biosynthesis Salvage Pathway


• It begins with their metabolic It recycles the free bases & nucleotides released from nucleic acid breakdown.
precursors amino acids, ribose 5 Purines that result from normal turnover of cellular nucleic acids or obtained
phosphate, C O 2 & N H 3 . from diet and not degraded, can be reconverted into nucleoside triphosphates
• Purine ring is contributed by number and used by the body.
of compounds PRPP
Inherited deficiency of HGPRT
- A.A [Glycine, Aspartic acid,
H y p o x a n th in e
glutamine]
- C 02 Phosphoribosyl m m sferase Result in increased level of PRPP and le d level
(HGPRT)
- N 10 - Formyltatrahydrofolate of IMP and GMP causingTed de novo purine
- Ribose -5 P O 4 [Product of HMP synthesis
_L
shunt] _£
• Ribose -5 - P O 4 is activated by ATP to G u a n in e (IM P Complete deficiency Partial deficiency of
IHOPRTI
form phosphoribosyl pyrophosphate of HGPRT HGPRT
(PRPP) by PRPP synthetase. PRPP PPi

- PRPP concentration is major L esch - N y h an K e lle y -S e e g m ille r


A d e n in e - w : ►A M P
determinant o f purine synthesis syn drom e® syn drom e® ______
A d e n in e
- PRPP acts as a substrate in salvage P h o sp h o r ih o s y llr a n s fe r a s e

pathway »X linked recessive disorder of purine metabolism which Hyperuricemia


- PRPP synthetase is inhibited by AMP. becomes manifest by the age of 2 years and seen almost Gout
& GMP (end product inhibition) exclusively in male children. No CNS
C 02 * Homozygous males are affected & carrier females are manifestation
i G lycine
Aspartate .c asymptomatic
\ 4^ \
N C ' »It demonstrates the importance of salvage pathways.
NV
Hypoxanthine & guanine arise constantly d/t breakdown of
/ *" nucleic acids. In absence of HGPRT-ase, PRPP level rises &
Formal
purines are produced by denovopathway resulting in high
\ /
Am ide N o f levels of uric acid production and gout like damage to tissue.
glutam ine
»The brain is especially dependent on salvage pathway and
O rigin of ring atom s of purine this is the reason for central nervous system damage in
Nucleotides Lesch-Nyhan syndrome
(AMP, G M P ) 4 -

Hyper Uricemic Neurological (CNS)


Symptoms Symptoms
Nucleosides Salvage
[Adenosine, G uanosine] Pathway Hyperuricemia, -M ental retardation® (intellectual disability)
Hyperuricosuria - Poor coordination (spasticity, choreoathetosis &
Diet ] y ' G outy arthritis l/t jo in t dystonia)
Bases [- pain® - Com pulsive aggressiveness (extremely hostile)®
Uric acid crystaluria - Show compulsive self-destructive tendencies &
Nephrolithiasis (urate s e lf m utilating behaviou r is characteristic
Urate Tophi
nephropathy, urinary feature® and occurs despite intact sensory
tract stones) (including pain) modalities. Mostly fingers, mouth
U rine Intestine
Obstructive uropathy (1/t & oral mucosa are mutilated. They mutilate
* Adenosine deaminase deficiency
reduced urinary ou t put)® themselves by biting off their fingers, toes & lips.
1/t immunodeficiency d/t
dysfunctional T cells and B cells I
1
causing fatal infectios in infants T/t
Purine nucleoside phosphorylase Early diagnosis + Allopurinol
deficiency is a/w severe deficiency Prevent or eliminate all the problem d /1 hyperuricemia
of T cells but apparently normal B No effect on neurological abnormalities
cell function.
646 ■ A Complete Review of Short Subjects

Structure of DNA
•Erwin-Chargaff in late 1940s postulated Charagaff rules Secondary Structure
that in a DNA, - Number of adenosine (A) residues is equal - It consists of 2 helical, antiparallel and complementary
to number of thymidine (T) residues (A=T) and number of chains that wound around the same axis to form a double
cytidine (C) residues is equal to number of guanosine (G) helix. Anti parallel orientation means that the 3 '—>5'
residues (C=G) phosphodiester bonds of both chains run in opposite
- Hence sum of purine residues (A + G) = Sum of direction.
pyrimidines residue (T + C) -The chains are paired in antiparallel manner i.e. 5' end of
•Rosalind Franklin & Maurice W ilkins analyzed DNA by one strand pairs with 3' end of other strand and thus the
X-ray diffraction and deduced that DNA molecules are phosphodiester bonds in two interwoven strands run in
helical, with 2 periodicities along their long axis, a primary opposite direction. In other word one strand run in 5' ti 3'
one of 3.4A° and a secondary one of 34A°. 'direction and other in 3' to 5' direction, (analogous to a
•James Watson & Francis Crick in 1953 postulated 3 highway carrying traffic in opposite directions).
dimensional model of DNA with following features - 2 Antiparallel chains are not identical in either base
-2 h e lica l, antiparallel and complementary DNA chains sequence or composition. Instead they are complementary
wound around the same axis to form a right handed double helix. i.e. Adenine (A) is alw ays paired w ith thymine (T) by 2
- Hydrophillic backbone formed of alternating deoxy ribose hydrogen bonds and cytosine (C) is alw ays paired with
sugar and phosphate groups (linked by 3'-5' guanine (G) by 3 hydrogen bonds®. The base pairs are
phosphodiester bond) lie on the outside of double helix perpendicular to the axis o f helix®. A = T; C = G
facing, the surrounding water. Whereas the purine & -This specific base pairing 1/1 Chargaff's rule i.e. in any
pyrimidine bases o f both strands are stacked inside® the sample of ds DNA (double stranded DNA) the amount of
double helix, with their hydrophobic & nearly planar ring adenine equals the amount of thymine and the amount of
structures lying very close together & perpendicular to long cytosine equals the amount of guanine. So the total
axis. amount o f purine equals the total amount o f pyrimidine®
-Each nucleotide base of one strand is paired with the base ( a + g / t + C = l)
of other strand in same plane. The offset pairing of 2 stands - Each of the two chains (strands) of double helix are coiled
1/1 formation of major & minor grooves. Original model around a common central axis called the axis of symmetry.
had 10 base pairs or 34A° (3.4 nm) per turn of helix but - Hydrophillic deoxyribose- phosphate backbone of each strand is
subsequent measurements revealed 10.5 base pairs or 36 A° on the outside whereas hydrophobic bases are stacked inside.
(3.6 nm) per turn. The structure resembles twisted ladder.
- 3 hydrogen bonds link C and G (C=G) and 2 link A and T. -Tw o strands are held together by both
So separation of strands is difficult in DNAs with higher 1.Hydrogen bonds between purine & pyrimidine bases®
C=G concentration.)_____________________________________ 2 .Vander walls and hydrophobic interaction between
stacked adjacent base pairs.
Primary structure _____
-In ds DNA, genetic information resides in the sequence of
- Each DNA is a polymer of about 1010 deoxyribo nucleotide nucleotides on 3' —» 5' strand, the template strand which is
monomers. These deoxynucleotides are of 4 types - deoxy copied during m-RNA synthesis. It is also k/ a non coding
adenylate (containing adenine), deoxy guanylate (containing strand. The opposite strand with 5' to 3' polarity is
guanine), deoxy cytidylate (containing cytosine) and deoxy considered coding strand, because it matches the sequence
thymidylate (containing thiymine). o f RNA transcript (but containing uracil in p lace o f
- These monomeric nucleotides are held in polymeric form in thymine)®.
each strand by covalent 3', 5' - phosphodiester linkage®. -One polypeptide strand of ds DNA is alw ays
-Phosphodiester bond joins 3'- hydroxyl group (OH) of complementary® of the other. So the m-RNA transcript with
deoxypentose (2'- deoxy D- ribose) sugar of one nucleotide to a 5' to 3' polarity is complementary to the tem plate strand
the 5'-hydroxyl group of deoxy pentose sugar of an adjacent with its 3 ' to 5' polarity, and matches the coding strand®.
nucleotide through a phosphate group. (Harper)
-The end of strand which bears 5- hydroxyl or phosphate -The glycosidic bonds b/w sugar & bases are not directly
terminal is called 5 end. And the opposite end with free 3' opposite each other and two grooves of unequal width are
OH or 3' phosphate group is k/a 3' end. So the polymer formed. The edge of helix that measures >180° from
show polarity®. The nucleotides are alw ays written and read glycoside bond to glycoside bond is called major groove,
in 5' —> 3' direction®. and if < 180°, it is called minor groove.
- So the primary structure consists of linear back bone o f - The hydrogen atoms (proton) in the bases can shift from one
deoxyribose sugar and phosphate. While the purine or ring nitrogen or oxygen atom to another k/ a
pyrimidine bases project laterally from the backbone at 90° tautomerisation - thus interconverting the position that can
(perpendicular®). severe as hydrogen bond donor & acceptor
Biochemistry: Structure, Function and Replication of Informational Macromolecules ■ 647

P - (5’ End)

5 '/ 3'
Bases D e o x y r ib o s e P h o s p h a te
(A d e n in e , su gar in d ie s te r / —
P
th y m in e , c y to sin e lin k a g e
& g u a n in e)
5' / 3/
T
I
D e o x y ri b o n u c le o s id e
5 '/ 3'
1—
D e o x y r ib o n u c le o s id e T
OH-(3' End)

★ RNA is far less stable than DNA because the 21-hydroxyl group of RNA (absent from DNA) functions as nucleophile during
hydrolysis of 3 '—>5' phosphodiester bonds. However in absence of phosphodiestreases, hydrolysis of phosphodiester bonds
of DNA occur only over long periods of time, despite an extremely favourable AG. and so DNA even persists in fossils.

D N A S tra n d s
C o d in g -5 ' - T G G A A T T G T G A G C G G A T A A C A A T T T C A C A C A G G A A A C A G C T A T G A C C A T G - 3'
T e m p l a t e - 3 '- A C C T T A A C A C T C G C C T A T T G T T A A A G T G T G T C C T T T G T C G A T A C T G G T A C -5 '
R N A tr a n s c r ip t 5 '- p A U U G U G A G C G G A U A A C A A U U U C A C A C A G G A A A C A G C U A U G A C C A U G -3 '

Unusual Structures Variations In DNA (Chromosome)

Bending of DNA Helix Palindromes & Hairpin or Cruciform (Cross shaped) Structures
DNA helix bends whenever >4 adenosine (A) residues Palindrome word / phrase / sentence /sequence is identically
appear sequentially in row in one strand (6 A produce read either forward or backward; such as ROTATOR and NURSES
18° bend). RUN.
Palindromic sequences of DNA have twofold symmetry over two
Triplex DNA & Hoogsteen Pairing strands of DNA and in order to superimpose one repeat on the
- Nucleotides participating in Watson-Crick base pairing other, it must be rotated 180° about the horizontal axis then 180°
can form additional hydrogen bonds, particularly with
functional groups arrayed in the major groove. For
example protonated cytidine residue (C+) can pair with
about the vertical axis.
A
guanosine residue of a G = C nucleotide pair; and a
thymidine (T) can pair with adenosine of A = T pair.
Palindromic sequences are self complementary within each
C = G • C+ T = A •T strand and therefor can form hairpin or cruciform (cross shaped)
structures with intrastrand base pairing. When only one strand of
Hoogsteen (non Watson-Crick) pairing at N7, N 6 and
DNA (or RNA) is involved the structure is called hairpin and
O positions of purines (Hoogsteen positions) allow
when both strands of a duplex DNA are involved it is called
the formation of triplex DNAs. The triplexes are
cruciform.
formed most easily within long sequences containing
only pyrimidines or only purines in a given strand; or
in DNAs containing 2 pyrimidine & 1 purine or 2
purine & 1 pyrimidine strands. The triplexes are most
stable at low pH because the C = G • C+ triplet requires
protonated cytosine residue (= is WC hydrogen bond
& • is Hoogsten hydrogen bond). Normally pKa of
cytosine is 4.2, which is altered to > 7.5 in triplex. It is
important to understand that Hoogsteen pairing is
essential for triplex DNA formation, whereas other
factors like long polypyramidine / polypurine only
sequences in a strand, 2 + 1 purine / pyramidine Hairpin
g \ y a

strands, low pH, increased pKa & protonated cytosine


Cruciform
are facilitatory.
648 ■ A Complete Review of Short Subjects

Tetraplex/ Quadruplex - DNA & G/Guanosine Mirror Repeats


Tetraplex Mirror repeat has symmetrical inverted repeat sequence within
■Four stranded DNA (tetraplex or quadruplex) occurs each strand. So super imposing one repeat on the other requires
only for DNA sequences with very high proportion of only a single 180° rotation about the vertical axis.
guanosine residue (or very stable guanosine /G T T A G C A C C A C G ATT
t — i— —
r i— r
tetraplex). _i i i i i i i i i_ _i i i i L_ C - >
A A T C G T G G T G C T A A
■Guanosine (G) tetraplex is very stable over wide range
of conditions Mirror repeats donot have complementary sequences with in the
same strand and so cannot form hairpin or cruciform structures.

Human Mitochondrial DNA (mt DNA)


•Mitochondria contain their own genome which •Genetic code of mt DNA differs slightly from standard code: mt RNA
is a closed circular piece of double stranded reads 4 codons differently
DNAQ that is composed of heavy (H) and light Codon Normal M tDNA
(L) strands or chains.
AG A 1 Terminator or
•Mt DNA is double stranded for most of its length -Arginine
AGG stop codon
except at the replication and transcription control
VGA Terminator Trypthophan
region (the D-loop), It is not packaged in
AU A Isoleucine Methionine
nucleosomes
• Proteins & structures encoded by mt DNA
•Human mt DNA contains 16,569 base pairs
- Encodes 2 mitochondrial ribosomal RNAs large (16s) and small (12S) units
(16.599 kb) in length and 37 genes that encode 13
- Encodes all 22 mt tRNA molecules
protein complexes of respiratory chain, 2r
- Encodes 13 protein subunits of respiratory chain (of a total of about 67-
RNAs and all 22 mt tRNAs requited for
mitochondrial protein synthesis^ Harper/ 79- Lehninger)
•However the great majority of mitochondrial Respiratory chain protein Total no. of No. of sub units
proteins - about 900 different types (of which complex subunits encoded by mt DNA
about 54 out of 67/ 66 but of 79 proteins of I NADH dehydrogenase 43 7
respiratory chain)- are encoded by nuclear genes, II Succinate dehydrogenase 4 0
synthesized on cytoplasmic ribosomes, then III Ubiquinone: cytochrome C 11 1
imported into & assembled in the mitochondria. oxidoreductase (cytochrome b)
•1% of cellular DNA is in mitochondria as each IV Cytochrome oxidase 13 3
mitochondria contains 2-10 copies of mt DNA. V ATP synthase 8 2
Each human cell contains variable number of Total 79 13
mitochondria depending on the energy • Disorders due to ( p o i n t m u t a t i o n , l a r g e s c a l e r e a r r a n g e m e n t s & s a m e
requirement of a particular cell. So each cell has d e l e t i o n s ) defect in mt DNA: (Mn- "Pearson No Progressive Leigh Deaf
large number of mt DNA owing to the fact that Labour Kissed Me Adequtely")
mt DNA replication is not limited to one per cell
cycle as it is in nuclear genome. This greater Disease & Phenotype Mt DNA Mutation &
abundance, compared with that of nuclear DNA, Inheritance (mostly)
makes mt DNA attractive for testing applications Pearson (PEAR) syndrome i.e. pancreatic Large deletion, Heteroplasmic,
in which sample DNA is limited (eg crime scene & bone marrow failure Sporadic somatic mutations
investigation, pathogen detection, & NARP syndrome i.e. neurogenic muscle Point mutation in ATPase
paleontology) weakess (with sensory or sensory motor subunit 6 gene,
•Mt DNA is transmitted between generations by neuropathy), ataxia, retinitis pigmentosa Heteroplasmic; Maternal
non mandalian maternal inheritance, with the due to mutation in MT-ATP 6 gene.
mitochondria coming from oocytes and not Leigh syndrome i.e. progressive
(usually) from sperm neurological disease with motor &
•Mitochondrial and nuclear DNA are located in intellectual developmental delay, signs &
different places in the cell. During fertilization, symptoms of basal ganglia / or brain
the sperm and egg cell nuclei fuse to form an stem disease and raised lactate
embryo. The egg cell is very large compared to concentration in blood and CSF.
the sperm, so although the cells' nuclei fuse, the Progressive external opthalmoplegia Single deletion / duplication;
rest of the cell mass in the embryo comes from (CEOP) Heteroplasmic; Sporadic
the egg only. Nuclear DNA is therefore co­ somatic mutations
inherited but the mitochondrial DNA, which is Deafness: Progressive sensorineural A 1555 G mutation in 12 s
Biochemistry: Structure, Function and Replication of Informational Macromolecules ■ 649

located outside of the nucleus, is always deafness induced by amino glycoside RNA; Homoplasmic; Maternal
maternally inherited because all mitochondria in antibiotics.
a foetus and later adult are derived from the LHON syndrome i.e. Leber's hereditary G17778 A, G 3460A, T 14484 C;
mitochondria in the mother's egg. All children optic neuropathy (atrophy) Homoplasmic (±); Maternal
from affected mother will inherit the disease but Kearns- Sayre Syndrome (KSS) i.e. 5Kb common deletion;
it will not be transmitted from an affected father opthalmoplegia, pigment degeneration of Heteroplasmic; sporadic
to his children. retina and cardiomyopathy somatic mutation
•Mt DNA is not highly conserved & has high MELAS syndrome i.e. mitochondrial Point mutation in tRNA leu;
mutation rate (5-10 times that of nuclear DNA) myopathy with encephalopathy, lactic Heteroplasmic; Maternal
which accumulate through out the life of acidosis and stroke
organism. It is because M ERRF syndrome i.e. myoclonic Point mutation in t RNA lys;
- Mt DNA suffers greatest exposure to reactive epilepsy and ragged red fibres disease Heteroplasmic; Maternal
oxygen species produced in respiratory chain d/t mutation in mitochondrial gene that
- It has less effective proof reading system at encodes t RNA specific for lysine (t RNA
correcting mistakes made during replication and Lys)
at repairing DNA damage M M C syndrome i.e. Maternally
- It contains no (or very few) introns (or inherited myopathy and cardiomyopathy
untranslated sequences)^
ADM IM Y syndrome i.e. autosomal
•Rapid accumulation of mutation provides
dominant mitochondrial inherited
sufficient data, to allow the comparison of
myopathy with mitochondrial deletion.
closely relatede species or population with in the
Somatic mutations in cytochrome
same species providing evidence about
complex III deficiency, muscle pain,
evolutionary origins of primates & other species.
ragged red fibers
•It can be used to study evolutionary
* Deafness & LHON are homoplasmic and others are heteroplasmic usually.
relationship (phytogeny) of organisms & build
All have maternal inheritance except Pearson syndrome, KKS and chronic
evolutionary tree as well as for genealogical
progressive external opthalmoplegia (CPEO) which are sporadic somtic
research into person's maternal line (eg prince
mutations mostly.
Williams has Indian origin maternal line).

Types of DNA
Axis o f sym m etry
F e a tu re A -D N A B- DNA Z -D N A
P re v a le n c e N o t fo u n d u n d e r M ost O c c u rs in reg io n th at h a v e
p h y sio lo g ic a l c o n d itio n s. c o m m o n ® a se q u e n c e o f a lte rn a tin g
W h en B D N A c ry sta ls p u rin e s & p y r im id in e s eg
a r e d ried , th e lo n g th in p o ly G C rich in g u a n in e &
B - D N A b e c o m e s sh o rt c y to sin e b a se pairs.
stu b b y A - D N A
H e lix h a n d e d n e ss R ig h t R ig h t® L eft®
S u g a r p u c k e r co n fo rm a tio n C -3 1 e n d o C -2 1 e n d o C -2 ' e n d o f o r p y r im id in e s
a n d C -3 ' e n d o f o r p u r in e s
G ly c o s y l b o n d c o n firm a tio n A nti A n ti A n ti fo r p y rim id in e s; S y n
fo r p u rin e s
D e o x y r ib o se - p h o sp h a te L in ea r L in e a r Z ig z a g ®
backbone
M a jo r g ro o v e P rese n t P re se n t N o n e x is tin g o r c o n v e x
M in o r g ro o v e P rese n t P re se n t D e e p c le ft
R e p e a tin g u n it b a se p a irs 1 1 2
B a s e p a irs /turn 11 10.5 12
In c lin a tio n o f b a se p a ir to axis ~ 20 ° (19°) - 1 ° ( 1 .2 °) - 1 0 ° (9°)
o f h elix
B a s e tilt n o rm a l to h e lica l a x is 20 ° 6° 7°
R o ta tio n / b a se p air -3 3 ° -3 5 ° 30 °
P itc h (n m ) 2 .4 6 3 .4 0 4 .5 6
D ia m e te r (n m ) 2.55 2 .3 7 1.84
H e lic a l R is e / b a se p a ir a lo n g 0.2 6 0 .3 4 0 .3 7
h elix a x is (n m )
650 ■ A Complete Review of Short Subjects

In Human genome

Haploid genome of each human cell consists of 3 x 109 base pairs (3000 mega bases) of DNA and about 1.7 x 107
nucleosomes subdivided into 23 chromosomes (chromatids). Each of these 23 chromatids would contain on the average 1.3
x 108 nucleotides in one ds DNA molecule.
Entire haploid genome contains sufficient DNA to have nearly 1.5 million average sized genes but studies suggest that
humans have < 1 lakh proteins encoded by -1% of human genome composed of exonic DNA containing < 25,000 protein
coding genes (previously 30-35 thousand). This means most of the DNA is non-protein coding i.e. its information is never
translated into an aminoacid sequence of a protein molecule.
Pathologic effects of genetic diseases are expressed at levels:
i) DNA ii) RNA iii) Protein iv) Cell & organ consequences.

Eukaryotic DNA: Human Genome

> 50% of eukaryotic DNA is unique or non repetitive and at least (i.e. >) 30% consists of repetitive sequences, About 30%
of genome is genes (for protein) but only 1.5% is coding sequences (or exons) carrying information for RNA or protein
products.
Non coding (non-translated) DNA segments in genes are called intervening sequences or introns (28.5%), and the coding
segments are called exons. Genes for histones have no introns and genes for muscle protein titin have 178 introns.
1% of cellular DNA is in mitochondria. Repetitive sequence DNA can be classified as

Also called simple sequence DNA or simple sequence repeats (SSR) - Defined as being present in numoers of <106
and constitute 3% of human genome copies per haploid genome, are not clustered but are
SSR is also called satellite DNA because its unsual base composition interspersed with unique sequences.
often causes it to migrate as satellite bands (separated from the rest o f - About half (45%) of human genome is made up of
DNA) when fragmented cellular DNA samples are centrifuged (in MRS which is derived from transposable
cesium chloride density gradient). elements (transposons) - a kind of molecular
Are usually <10 basepair long and sometimes repeated millions of parasite, efficiently making a home (or trapped)
times per cell. But may be 5-500 bp long, repeated many times in with in the host genome during evolution.
tandem - These DNA segments range from few hundred to
VNTR (variable number of tandem repeats) are short sequences of several thousand bp long, that can move from one
DNA at scattered locations in genome, repeated in tandem (i.e. one location to another in the genome (transposition)
after another). The number of these repeat units varies from person to through an RNA intermediate by action of
person, but is unique for any given individual and, therefore, serves as reverse transcriptase (i.e. retroposons).
a molecular finger print. Microsatellites are a type of VNTR. - Many transposons have genes transcribed by
M icrosatellite repeat sequences (M iRS) consist of 2-6 bp repeated RNA polymerase II encoding m-RNA (containing
upto 50 times as both dispersed and grouped tandem arrays. It most indistinguishable caps) and proteins that catalyze
commonly is found as dinucleotide repeats of AC on ones strand and transposition process.
TG on opposite strands-occuring upto 1 lack times in genome. Several - Although these do not encode RNA or proteins
other repeats may be CG, CA and AT. It produces polymorphism (i.e. used in human cells, they have played a major
clinically harmless DNA variation that does not affect phenotype) & is a role in human evolution -as movement of
heritable trait. Because of their unique number (for an indvidual), usual transposons can 1/t redistribution of other
association with most genes and ease of detecting them using PCR, genomic sequences.
MiRS Eire useful in constructing genetic linkage map. - Most trEtnsposons are inactive, evolutionarily
M icrosatellite instability (i.e. increased tri nucleotide repeats) can lead to altered by mutations, however, some may be
diseases like fragile X syndrome d/t unstable p(CGG)n, myotonic active moving (transcribing) at a low frequency.
dystrophy (CTG) and Hutington's chorea, spinobulbar muscular - Depending on their length MRS can be
atrophy and Kennedy disease (all d /1 unstable CAG). 1. Long interspersed repeat sequences (LINEs):
HRS-DNA is transcriptionally inactive (i.e. does not encode proteins or
- 6 - 8 Kbp (1 kbp = lOOObp) long and have 20-50
RNA) but (unlike transposons) have Em identifiable functional importance
thousand copies (Harper)/8.5 lack LINEs
in cellular metabolism, because much of it is associated with -
(Lehninger) in human genome.
centromeres & telomeres. It may play a structural role also.
- It is species specific families of repeat element.
Biochemistry: Structure, Function and Replication of Informational Macromolecules ■ 651

I
Centromere Telom ere - Are shorter (70-300 bp long) and have >1 lack
- It is adenine- thymine-(A-T) Telomeres are short T-G rich copies per genome (Harper)
rich region containing repeats at the ends of eukaryotic - Of 1.5 million in human genome >1 million are
repeated DNA sequences from chromosomes that help stabilize the Alu family, so called because they generally have
102 - 106 bp size (i.e. thousands chromosome. one copy of recognition sequence for Alul,
of tandem copies of short Human telomeres have variable restriction endonuclease (Lehninger).
segments in same orientation) number of '5' - TTA G G G -3" - Alu elements (family) a/f 5-6% human genome
which connects sister repeats which may extend upto (i.e. 5 lack copies per haploid genome). Harper it
chromatids. several Kb. is highly conserved within as well as between
- Metazoan centromere is bound mammalian species.
by nucleosome containing - SINEs are m obile elements, capable of jumping
CEN P-A (a H3 histone variant from one region to another region of genome and
protein & other centromere can 1/1 disastrous events as insertion of Alu
binding proteins, forming sequences may cause neurofibromatosis.
Kinetochore which provides the
attachment fo r mitotic spindle.
- 1.5 - 11 kbp long, 4.5 lack copies of these are
- This anchoring is essential for
present in human genome.
equal & orderly chromosomal
- Although evolutionarily related to retrovirus
segregation (distribution)
they are trapped & cannot move from one cell to
during mitosis._______________
another.

S is t e r c h r o m a tid I
4. Transposon remnants I
length

Sister chrom atid 2

Telomeres
• Telomeres are complexes of non-coding DNA & protein located at the ends of linear • It is a multisubunit RNA containing-
eukaryotic chromosome. (Greek telos = end). protein complex with a RNA
• It consist of many tandem (one after another) repeats (> 20 to > 1500) of short DNA dependent DNA polymerase (i.e.
sequence/oligo nucleotide (dimeric to octameric). Telomere usually has TG rich reverse transcriptase) activity.
longer strand and AC rich shorter complementary strand. The general formula is • It is responsible for telomere
w here x & v = Betw een 1 & 4 synthesis and maintenance of
r
5 '- (T, Gy)n and num ber of telom ere telomere length.
3' - (Ax C y)n repeat n range from 20 -100 in 3 ' fa 5' • Prevention of telomere shortening
single cell eu karyotes to T e lo m e re E u k a ry o tic D N A T e)o m ere
with every cell division, d /1
>1500 in m am m als
activity of telomerase enzyme is
the cause of cellular immortality
Telomeres very in length from few Species Telom ere Repeat
in cancer cells, germline cells and
dozen bp in protozoa to several Sequence
stem cells as it protects from
thousands bp in mammals (humans). In Human (TTAGGG)n
cellular aging & apoptosis.
humans, non coding hexameric Thermophilic (TTG GG G )n
• Somatic cells lack telomerase and
sequence AG 3T 2 base paired to ciliated protozoa
undergo cellular aging, senescence
complementary CA sequence forms Yeast I(TG)i-3 (TG)M]n (i.e. no longer able to divide) and
telomere. Plant (TTTAGGG)n death (apoptosis )Q after 40-60
replications when telomeres
T e lo m e re become critically short or gone. So
The TG strand is longer than its AC d u p le x
cells.
complement, leving a region of single DNA-
C A b in d in g With telomerase Without
stranded (ss) DNA o f few hundred s tra n d p ro te in s
activity telomerase
nucleotides in length at the 3' end.
activity
Single stranded region (end) is folded
- Germinal (Germline) Somatic
back on itself, and paired with its cellsQ cells O
complement in ds portion of telomere, - Cancer (tumor)/
forming a T-loop structure. The looped neoplastic cells <2
DNA is bound (stabilized) by several - Stem cells like
telomere binding proteins, including hemopoetic cellsQ
652 ■ A Complete Review of Short Subjects

TRF-1 & TRF-2 (telomere repeat binding factor) with the latter protein involved in • Telomerase contains a protein that
formation of T loop. It maintains the structural integrity and confer stability to acts as reverse transcriptase and a
chromosomal ends. T-loops protect 3' end, making them inaccessible to nucleases and short piece of (-150 nucleotide
enzymes that repair double strand breaks, and allow repair system to distinguish a long) AC rich RNA that acts as a
true end from a break in ds DNA. So telomeres protect chromosomal termini from fusion template for synthesis of already
dr degradation. longer TG (3') strand.
•Because DNA replication (by Telomeres Repeats
• CA-rich RNA template (containing
DNA polymerase) requires a
~ 1.5 copies of Cy Ax telomere
template & primer, and beyond DN A 5' - T T A G G C - T T A G G G T T A G G - 3'

II II II repeat) base pairs with GT rich


the end (extreme 5' end) of D N A 3 ' - A A T C C C - 5'
single stranded 3' end oftelomeric
lagging strand there is no DNA A C r i c h T e l o m e r a s e e x le n d th e T G r i c h a lr e a d y
DNA. The extra overhanging length
template available for pairing o f an l o n g e r 3 ’ e n d o f D N A b y u s in g its R N A a s te m p la te

of this RNA acts as a template for


RNA primer. So telomeres are
not readily replicated by DNA
D N A 5 ' - T ;A G < T 2A G 3 - TTA G G G - 3' synthesis (lengthening) of already
II I I II II I
polymerase and consequently A 2U C 3 - A A U C C C - A 2U C 3 - longer Tx Gy strand of telomere.
3' Reverse transcriptase uses this
telomeres shorten with each cell
division in most normal somatic R e v e r s e tr a n s c rip ta s e RNA template to synthesize DNA
R N A te m p la t e
s y n th e s iz e DNA in 5 ’ 3'
human cells. direction, u sin g th e RNA
in 5 '—> 3 ' direction, extending the
•There is a linear, inverse
te m p la te , th u s
le n g th e n in g th e 3' e n d
already longer 3' (TG) end.
relationship between telomere However, unlike retroviral reverse
length and cell senescence D N A 5 ' - T 2A C t - / T 2A G 3 - T 2A G 3 - T T A G G G - TTAGGG - 3 '
transcriptase, telomerase copies
(cessation of cell division) or age. /III III I I II II I II II I only a small segment of RNA that it
D N A 3' - / ^ A 2U Q - A 2U C 3 - A A U C C C - A A U C C C ^
So telomerase is a mitotic clock carries with itself.
/y ________________________ _
in that their length in most cells • Having synthesized one copy of the
R N A p r im e r w hich is
is inversely related to the c o m p le m e n ta ry to 3 ’ e n d repeat, the telomerase repositions
number of times cells have o f D NA is s y n th e s iz e d by
p r im a s e . T h e 3 ' e n d o f (translocates) to the newly synthesized
divided. th is p rim e r s e r v e s a s a n
a c c e p to r for DNA end and the process is repeated further
•Once the telomeres are p o ly m e r a s e .
lengthening already longer 3' end
shortened beyond a certain D N A S ' - T 2A G 3 - T -> A G i T -> A G t T 2A G t T2AG3 (rich in TG).
point, the loss of telomere I II -h-hh - h - h h I I I
D N A 3 ' - A 2T C 3 < - A 2TG-A2TG • A : U G "P rim ase • Once the GT rich 3' end (strand) is
function leads to activation of
T appropriately lengthened, primase
p53 dependent cell cycle check D N A polymerase
D N A p o ly m e r a s e s y n t h e s i z e s c o m p l e m a n t a r y 5 ' A C can use it as a template to
points, causing proliferation
r i c h e n d i n 5 ' - > 3 ' d i r e c t i o n a n d R N A p r i m e r is synthesize a complementary RNA
arrest & apoptosis. Thus telomere rem ov ed .

shortening functions as a clock that fragment (primer) opposite to TG


counts cell divisions. In germ cells, strand. The RNA primer is
telomere shortening is prevented extended by DNA polymerase &
by the sustained function of primer is removed.
enzyme telomerase, which • This means 3' TG rich end is
maintains their ability to self synthesized by telomerase and
replicate extensively. Cancer cells complementary 5' AC rich end is
show reactivation o f telomerase synthesized by DNA polymerase.
activity and maintenance of
telomere length.

Structure of Chromosome &.Chromatin Nucleosomes


Chromatin is the chromosomal material in the nucleus of eukaryotic cells. It is of 2 Nucleosomes are functional or originational
types - euchromatin & heterochromatin. units o f chromatin and appear as a "beads
on a string" arrangement on electron
Less condensed (more open) & w eakly staining, transcriptionally active 0
microscopy.
regions of chromatin are called euchromatin. So the euchromatin is the
dispersed (uncondensed) portion of chromatin that is transcriptionally Flat protein cylinder consisting of histone
active during interphase (when RNA transcription occurs). In other words, octamer containing 2 molecules each of
euchromatin contains genes capable o f transcribing genetic information into H3, H4, H2A & H2B form tine structural
proteins that are necessary for young cells maturation, metabolic function core of individual nucleosome beads
or DNA repair. (H3-H4)2 tetramer itself can confer
nuclesome like properties on DNA and so
Biochemistry: Structure, Function and Replication of Informational Macromolecules ■ 653

- Wheres, highly condensed & strongly staining heterochromatin is has a central role in nucleosome
defined as a (chromotin) structure that does not alter inits condensation formation. Addition of 2 (H2A-H2B)
throughout the cell cycle (in contrast to euchromatin that decondense dimmers stabilize the primary particle &
during interphase) firmly binds two additional half turns of
DNA previously bound only loosely to
• Chromatin consists of fibers containing (histone) protein and DNA in nearly equal
(H ,-H 4)2.
proportions (by mass). Chromatin contains
Around the surface of disc shaped
- Double stranded (ds) DNA 1 _ . histone octamer (i.e. nucleosome protein
v ' - r o r m n u c le o s o m e s
- Small basic proteins k/a histonesd core), the 1.75 superhelical turns of DNA
- Mostly acidic and larger (than histones) non histone proteins, that are wrapped (wound), forming left
include enzymes involved in DNA replication & repair, and proteins handed solenoidal supercoil (or
involved in RNA synthesis, processing and transport to cytoplasm in underwound or negatively supertwisted
smaller amounts helix), protecting 146 bp of DNA from
- RNA (small quantity)__________________________________________ digestion by a nuclease and forming the
• DNA (in transcriptionally or potentially transcriptionally) active chromatin nucleosome core particle.
has larger regions (~ 1 lack bases long) that are relatively more sensitive to Neighboring nucleosome core particles
digestion by nuclease (like DN ase I, a low sequence specific nuclease that makes (beads) are linked by approximately 30 bp
single strand cuts in nearly any segment of DNA that is not protected, or bound (Harper)/50 bp (Lippincot)/54 bp
by protein into its component deoxynucleotides). The sensitivity to DNase I (Lehninger) long DNA segment termed as
digestion reflects only a potential for transcription rather than transcription itself linker. Histone HI binds to the linker
and can be associated with a relative lack of 5-methyl deoxycytidine (meC) in DNA. The bead plus the connecting DNA
DNA and particular (phosphorylation, acetylation etc) covalent modification of that leads to the next bead form the
histones. nucleosome. So nucleosomes are
• Shorter (100-300 nucleotides long), Hypersensitive (10 times more sensitive) repeating units containing histones & 200
sites to DN ase I exist in larger active chromatin. It is formedby binding of non­ bp DNA (146 bp bound to core & 54 bp
histone regulatory transcriptional factor proteins and proteins involved in linker)Q.
maintaining access to template strand and result in structural conformation
Tight wrapping of DNA around histone
that favours access of nuclease to DNA. Hypersensitive regions are often
core requires removal o f l helical turn in
located immediately upstream from the active gene that is capable of being
DNA, but because this process does not
transcribed.
break the DNA (or change the linking
• Inactive chromatin is high in meC content in DNA and lower in covalent
number), the formation of a negative
modifications of histones. It can be of 2 types
solenoidal supercoil must be
- Constitutive hetero chromatin, found in centromere and telomeres (i.e.
accompanied by a compensatory positive
chromosomal ends), is always condensed and thus essentially inactive.
super coil in unbound region of DNA.
- Facultative heterochromatin, is at times condensed but at other times is Eukaryotic topoisomerase relax the
uncondensed (appears as euchromatin) and is actively transcribed. Like X unbound positive supercoil (leaves the
chromosome of females that is active only during gametogenesis. negative supercoil fixed to histone core)
• Approximately 1 m long DNA of 46 chromosomes is packed in small nucleus and results in an overall decrease in
of each cell. This becomes possible only b/o multi levels of DNA compaction, linking number. This is how eukaryotic
condensing it to > 10000 (104 i folds. DNA is underwound even through
Chromatin Packing Dia­ Features eukaryotic cells lack enzymes that
form (Condensation) meter underwind DNA.
Ratio Majority of core histone proteins interact
Nacked 1 2 nm with the DNA on the inside of supercoil
double without protruding except their amino
helical DNA terminal tails which extend outside and
10 nm 7-10 10 nm Beads on string form of chromatin are available for covalent modification.
chromatin
Nucleosome (histone) core exhibit non
fibrils of
random preferential binding (phasing)
nucleosomes
to specific DNA sequence that has a local
30 nm 40-60 (Harper) 30 run 10 nm fibril is further supercoiled with
abundance of A=T base pairs in DNA
chromatin /100 6-7 nucleosomes per turn to form 30 nm
where it is in contact with histones
fiber of (Lehninger) chromatin fiber: Hi histones stabilize it
facilitating the compression of minor
superhelical and faces of nucleosome of successive
grooves that is needed for tight wrapping.
nucleosomes turns would be nearly parallel to each
Nucleosomes bind well to sequences
other.
654 ■ A Complete Review of Short Subjects

Loops 8000 300 nm - 20-100 kbp (X 1000 bp) anchored in a where AA or AT or T T dinucleotides are
(Domains) scaffolding (supporting) matrix with staggered at 10 bp intervals. Phasing is
in the nucleus (i.e. nuclear matrix) d/t physical flexibility of certain
containing Hi histone and topoisomerase sequences that are able to accommodate
II, organize to form loops. the regions of kinking within the
- Looped domains contain a set of supercoil.
related genes corresponding to one or Nucleosomes are deposited on DNA in a
more separate genetic functions, stepwise manner (during replication or
containing both coding & non coding other process that require a transient
regions of cognate gene or genes. displacement) mediated by RCAF
Rosette 6 loops condense to form 1 rosette (replication coupling assembly factor) which
Coil 30 rosettes condense to form 1 coil includes acetylated H3 & H4, chromatin
Chromatids 10 coils condense to form each chromatid (of 1400 nm assembly factor-1 /CAF1 and anti
diameter) silencing factor /ASF-1). (H i-H ^
tetramer binds first f/b (H2A - H2B)
Chromatin Proteins
dimmer.
1. Histone proteins
- Histones are small, basic proteins^, which package the DNA into structural units
called nucleosomes.
N u c le o s o m e co re
-Carboxy terminal (2/3) of histone molecules are hydrophobic, while amino
terminal (1/3) is rich in basic' amino acids.
-Histone proteins are most abundant chromatin protein. These are positively charged
at physiological pH because of their high content of basic amino acids arginine
and lysine and hence form ionic bonds with negatively charge DNA. Histones,
along with positively charged ions like Mg++ neutralize the negatively charged
DNA phosphates.
-There are five classes of histones: HI, H2A, H2B, H3, H4. H I histones are least
tightly bound to chromatin and are, therefore, easily removed with salt solution,
after which chromatin becomes soluble. The organizational unit of this soluble
chromatin is the nucleosome. N ucleosom e contains four type of histones- H2A,
H2B, H3, H4; collectively called core histones.
-The histones interact with each other in a very specific ways. H3 and H4 form a
tetramer containing two molecules of each (H3-H4)z, while H2A and H2B form
dimmers (H2A-H2B). Under physiological conditions, these histone oligomers
associate to form the histone octamer (H3-H4)2 - (H2A-H2B)2.
- H3 and H4 histones are most conserved among eukaryotes f/b other core
histones (H2A, H2B). This indicates that function of histones is identical in all
eukaryotes and that the entire molecule is involved in carrying out this function. Organization of human DNA illustrating
- HI histone is most tissue specific & species specific of histones. It facilitates the the sequence of nucleosomes
packing of nucleosoe into more compact structure.
-The four core histones are subjected to at least six types of covalent modification.
Mn: "RUPAM " i.e. Ribosylation, Ubiquitin binding, Phosphorylation,
Acetylation, Methylation.___________________________________________________
Covalent Modification DNA Repair
ADP-Ribosylation - D N A r e p a ir
Covalent linkage to Ubiquitin
- Monoubiquitylation - G e n e a c tiv a t io n , r e p r e s s io n , a n d
h e te r o c h r o m a t ic g e n e s ile n c in g
- Sumoylation (SUMO; s m a ll - T r a n s c r ip tio n r e p r e s s io n
u b iq u it in - r e la te d m o d ifie r )
- P h o s p h o r y la tio n o f H i - C o n d e n s a t io n o f c h r o m o s o m e s d u r in g \ m
r e p lic a t io n c y c le
A c e ty la tio n
- o f c o r e h is to n e s - C h r o m o s o m a l a s s e m b ly d u r in g D N A -
- o f H 3 & H 4 h is to n e s r e p lic a t io n
- A c tiv a t io n & in a c t iv a tio n o f g e n e tr a n s c r ip tio n
M e t h y la t io n - A c tiv a t io n & r e p r e s s io n o f g e n e tr a n s c r ip tio n
Biochemistry: Structure, Function and Replication of Informational Macrom olecules ■ 655

- Histone acetylation (by histone acetyl transferase = HAT) causes increased


(trancriptionally active) relatively uncondensed euchromatin formation®.
Acetylation or phosphorylation of lysine residue at aminoterminal decreases
positive (+) charge of basic histone protein, there by decreasing their
association with negatively (-) charged DNA, thus releasing nucleosomes and
allowing access to DNA. Histone deacetylation (by histone deacetylase =
HDAC) removes acetyl group, restores positive charge and causes increased
formation of transcriptionally inactive hetero chromatin (relatively
condensed).
2. Topoisomerase
It relax the compensatory positive plectonemic supercoil in unbound region of DNA
produced d/t negative solenoidal supercoiling of DNA around histone core.
3. SMC (Structural maintenance of chromosomes) proteins
- SMC proteins maintain structures o f condensed chromosomes and consist of 5
domains globular amino (N) & carboxy (C) terminals with partial ATP
hydrolytic site (on each and becomes complete on joining). These 2 N & C
terminals are joined by 2 a-helical coiled coil motifs joined by a hinge domain.
- Cohesins & Condensins are 2 SMC proteins. Condensins bind to DNA and
creates positive super coils (i.e. DNA becomes over wound in contrast to
underwinding introduced by binding of nucleosomes). So condensins are
essential to chromosomal condensation as cell enters mitosis.
- Cohesins link sister chromatids together immediately after replication & keep
them together as chromosomes condense to metaphase - a process essential
for proper segregation of chromosomes at cell division. Cohesin & Kleisin
form a ring around replicated chromosomes that tie them together (expand &
contract in response to ATP hydrolysis) until separation is required at cell
division.

_________ Supercoiling & Linking Number_________

Supercoiling means the coiling of a coil. DNA is coiled in form of a double helix, with both strands of DNA coiling around
an axis. The further coiling of that axis upon itself produce DNA super
coiling, which is a manifestation o f structural strain. When there is no net
bending of DNA axis upon itself, the DNA is said to be in relaxed state.
Supercoiling is an intrinsic property of DNA tertiary structure which is
requied for DNA compaction. Replication & transcription of DNA also affect
& are affected by supercoiling. It is studied in topology, a branch of
mathemetics.
Closed circular (B form) DNA with one turn of double helix per 10.5 base
pair is relaxed. Supercoiling is a manifestation of structural strain. And in
almost every instance, the strain is a result of underwinding of DNA double
helix in the closed circle. In other words, DNA has few er helical turns than
would be expected fo r the B form structure. An 84 bp segment of a circular DNA
in relaxed state would contain 8 double helical turns, or 1 of every 10.5bp. If
one of these turns were removed, there would be (84 bp) / 7= 12 bp / turn rather than 10.5 bp/ turn in B- DNA. This
deviation from most stable form would result in a thermodynamically strained DNA
In principle, the strain could also be accommodated by separating the two DNA strand over a distance of about 10 bp. But in
isolated circular DNA the strain introduced by under winding is generally accom m odated by supercoiling rather than strand
seperation®, because coiling the axis of DNA usually requires less energy than breaking the hydrogen bonds that stabilize
paired bases. However underwinding represents a form o f stored energy and makes separation o f DNA strands easier
M ost cells maintain DNA in under wound state to fa cilita te its com paction by coiling®.
Linking number, is a topological property of ds DNA, because it does not vary when DNA is bent or deformed, as long as
both strands remain intact. Linking number can be defined as the number of times the 2nd strand pierces the surface of 1st
strand which is visualized as circle. It is defined as positive (+ve) if strands are interwound in right handed helix and negative
(-ve) in left Imnded helix (not encountered).
656 ■ A Complete Review of Short Subjects

Number o f base pairs


Linking number = --------------------------------------------------------------------------------------
Number of base pairs per turn (which is 10.5 in relaxed form)

So linking number is number of turns.

• Superhelical density or specific linking difference (a) is a measure of number of turns removed relative to the number
present in relaxed DNA. It is independent o f the length o f DNA molecule®. The negative sign indicates that the change in
linking number is d/t underwinding o f DNA®. So the supercoiling induced by underwinding is called negative supercoiling.
Over wound results in positive superwinding (which is a mirror image of negative supercoiling).

_ ALk (number of turns removed)


Lko (number of turns present in relaxed DNA)

o = - 0.01 means that 1% of helical turns present in B form of DNA has been removed. The degree of underwnding in
cellular DNAs generally falls in range of 5-7%, that is o = - 0.05 to - 0.07.
• Linking number (Lk) can be changed by + 1 by breaking one DNA strand, rotating one of the ends 360° about the unbroken
strand, & rejoining the broken ends. This change has no effect on no. of base pairs. Lk can be broken down to twist (Tw) and
writhe (Wr), which are geometric rather than topological properties, because they may be changed by deformation.

Lk = Tw + Wr
Writhe is a measure of coiling of helix axis and twist (Tw) determines the local twisting or spatial relationship of
neighbouring base pairs.
Topoisomerase enzyme s increase or decrease the extent o f DNA underwinding and so the linking number
- Topoisomerase type I, act by transiently breaking one of the two strands, passing the unbroken strand through the break
& rejoining the broken end; they change L k in increm ents o fl® .
Topoisomerase type II break both DNA strands & change linking num ber in increm ents o f 2®.

DNA Replication (Initiation, Synthesis, Elongation and Termination) ^

Initiation of DNA Synthesis


• It fo llo w s b ase p a ir rule an a is sem iconservative® - Two strands of double stranded DNA molecules are antiparallel. The
because, although the parental double strands are DNA replication in eukaryotes & prokaryotes occur on b oth strands
separated into two halves & therefore is not sim ultan eously by a single asym m etric DNA p oly m erase I II dimer®.
conserved as entily, each of the individual parental Because, an enzyme capable of polymerizing DNA in 3' to 5'
strands remains intact in one of the two daughter direction does not occur in nature, so both o f newly replicated DNA
chromatids. Each strand of double helix (strand) strands cannot grow in the same direction simultaneously.
serves as template for constructing a complementry
- Replication exhibits polarity because DNA polymerases responsible
daughter strand (semiconservative strand).
for copying the DNA templates are only able to read parental
• The site at which the DNA replication is initiated is nucleotide sequences in 3' —> 5' direction and they synthesize the
termed the origin of replication (Ori). In new DNA strands only in 5' —» 3' (antiparallel) direction
prokaryotes, Dna A (a DNA binding protein) binds
- Because of unidirectional (5' —» 3') synthetic activity of polymerase
to repeated nucleotide consensus (highly
and the antiparallel nature of two strands, the synthesis of DNA
conserved) sequences located within the origin.
along the two strands is different and in opposite direction. DNA
Binding of 20-30 Dna A molecules to ori induces
polymerase synthesize new DNA strand only in 5' —> 3'direction,
unwinding, which separates strands in AT rich
which is towards (in the direction o f advancing) replication fork in leading
region adjacent to Dna A- binding sites, that is easy
strand and away from replication fork in lagging strand. DNA
to unwind and is called DNA unwinding element
polymerase require primer (a short stretch of RNA), synthesized by
(DUE). The E coli replication origin is called ori C.
DNA primase. The leading strand only needs one, whereas lagging
Functionally similar autonomously replicating
strand needs many primers. The leading strand is rep licated in a
sequences (ARS) or replicators occur in yeast cells.
continuous fashion® . However, for the lagging strand, RNA prim ers
The ARS contains degenerate 11 bp sequence,
are p eriod ica lly ad d ed by DNA prim ase® along the strand. DNA
origin replication element (ORE) that binds to set
polymerase III elongates these RNA primers to form Okazaki
of protein analogous to dna A of E.coli and form
fragments.
origin recognition complex (ORC).
Biochemistry: Structure, Function and Replication of Informational Macrom olecules ■ 657

• Now the hexameric protein Dna B - binds to So the leading strand is produced as a single, long continuous
separated DNA strands. D na B has helicase strand (in 5 '—»3' direction) from a single primer with the same
activity that catelyzes ATP mediated unwinding of overall forward direction. But the lagging strand is synthesized in
DNA helix. DNA gyrase also participates in small fragments (1000-5000 bp length) termed O kazaki fragm ents in
separation of strands. As this complex continues a discontinuous manner, in 5'—>3' direction, but at the same time it
unwinding DNA strands in both direction from Ori, faces towards the back end of the preceding RNA primer rather than
SSB P (single stranded DNA- binding protein) coat toward the unreplicated portion. So overall DNA synthesis is semi
the separated strands to in h ib it th eir rea sso cia tio n discontinuous (continuous in leading & discontinuous in lagging
(i.e. p reven t prem atu re reanneiling)®. DNA strand).
replication begins a t origin o f rep licatio n (ori) in S Replication proceeds from single origin (ori) in prokaryotic bacterial
p h ase. T he stran ds are sep era ted lo c a lly fo rm in g 2 chromosome. In eukaryotic metazoan (mammals etc), the replication
rep licatio n forks® . Double stranded DNA's is bidirectional (ie occurs in both directions) and replication
replication is bidirectional. Helicase unwinds, proceeds from m ultiple origins in each chromosome (~ 100 in
seperates the double helix, which produces positive humans). Thus replication occurs in both directions and both
supercoil in the region o f DNA ahead o f replication fork. strands are replicated sim ultaneously generating replication
DNA topoisomerase I & II remove suprecoils. bubbles.
Entire protein complex at replication fork k/a DNA replicase system
:o Initiate Replication or replisom e come into action. Parental DNA is unwound by DNA
B helicase and resulting topological stress is relieved by
Protein No of Function topoisomerases (I, II, gyrase). Each separated strand is stabilized by
sub SSB (or RPA) protein. Dna B helicase and Dna G primase form
units functional unit within replication complex k/a primosome. Dna B
helicase bound in front, unwinds the DNA at replication fork as it
Dna A 1 - Recognizes replication origin
moves along the lagging strand template in 5' —» 3' direction and the
protein (ori) sequence and opens duplex
primer is synthesized by Dna G primese in 5 ' —» 3' direction but the
at specific sites in origin.
direction opposite to that in w hich D na B helicase is moving.
- This member of AAA + A T pase However the first primer laid down serves to prime leading strand
protein family has ATP bound DNA synthesis in opposite direction.
active & ADP bound inactive Once the strands are sufficiently unwound (seperated) by helicase,
forms. It has higher affinity for R the DnaG primase (of primosome) synthesize RNA oligo nucleotides
sites (5 repeat of 9 base pair complementary to each parental DNA strand which serves as primer
consensus sequence = TT (A/T) for DNA synthesis. Once each RNA primer has been laid down, two
TNCACC) than I sites in ori. R- DNA polymerase III core subunit complexes are assembled, one at
sites can bind to both active & each of the primed site.
inactive Dna A protein whereas, Both leading and lagging strands are produced by a single
I sites bind only the active ATP asymmetric DNA polymerase III dimer tethered to the lagging
bound Dna A allowing strand. This is accomplished by using one set o f core subunits (core
discrimination. polymerase) to synthesize the leading strand, while the other set of
core subunits cycles from one Okazaki fragment to the next on the
- 8 molecules of ATP bound
looped lagging strand (ie core subunits are attached to both strands).
(active) Dna A proteins form a
Looping of lagging strand brings together the two points of
helical complex and bind at R
polymerization.
and I sites in origin (ori) Tight
right handed helical wrapping of Prokaryotic DNA chain elongation is catabolized by DNA
DNA around this complex polymerase III using 5' deoxyribonucleoside triphosphate as substrate. It
produces a positive supercoil. p r o o f read s & h as 5'ex on u clease activ ity . RNA prim ers are
The resultant strain in near by rem ov ed by DNA p o ly m era se I using its 5' —» 3' ex on u clease
DNA 1/ 1 denaturation in A=T activity® . It fills the gap with DNA and proofreads (during synthesis).
rich DUE (DNA unwinding So DNA polymerase I rem ov es the RNA prim er, rep laces it w ith
element) region of ori (ie DNA an d ev en tu ally jo in O k a z a k i fragm ents® . Finally, DNA lig ase
tandem array of three 13 bp se a ls n ick an d jo in th e lagging stran d DNA fra g m en ts to fo r m a
consensus sequence contin uous strand® by catalyzing final phosphodiester linkage.
GATCTNTTNTTTT). Eukaryotic DNA has 5 types. Pol a is a multisubunit enzyme with
primase & 5' —> 3'polym erase activity. It adds a short piece of DNA
HU, FIS*, 2 - Histone like bacterial protein
to RNA, primer. Pol 5 completes DNA synthesis on leading strand
IHF (HU) or factor U, factor for
and elongates each lagging strand fragment using 3' —> 5 'exonuclease
658 ■ A Complete Review of Short Subjects

inversion stimulation (FIS) and activity to proofread. Pol P & pol e are involved with DNA repair. Pol
Integration host factor (IHF) are y replicates mitochondrial DNA. Eukaryotes have many more ori and
*= DNA binding proteins which contain a licencing factor to insure that excess amount of unfinished
Identical stimulate initiation by facilitating replicating DNA do not accumulate.
sub units formation o f helical Dna A complex.
- When the 3' end of elongating Okazaki fragment, reaches the 5' end
- FIS & IHF have specific binding of previously synthesized, DNA polymerase III release the template
site in ori whereas HUS does not & find another primer further back along lagging strand synthesizing
have a specific site. 2 subunits of another okazaki fragment.
FIS are identical (not of IHF and - Proteins of replisome include Dna B helicase, DnaG primase, DNA
HU) gyrase (topoisomerase II), SSB, DNA polymerase III & I and DNA
ligase.
Dna C 6* - An AAA - ATPase protein
protein required for Dna B (helicase) DNA - Polymerize deoxyribonucleotide (DNA) and share 3
binding or londing onto Polymerase comon properties
separated DNA strands in the
1. Chain elongation: It is d/t 5' -3' polymerase
denatured region at origin,
activity accounting for the rate (in nucleotides per
(keystep in initiation)
second = ntd/s) at which polymerization occurs
- Hexamer of Dna C (with ATP 2. Processivity : It is an expression of number of
bound to each subunit) forms nucleotides added to nascent (new) chain before
tight complex with hexameric, the polymerase disengages from the templet.
ring shaped Dna B helicase. This 3. Proof reading: It is d/t 3' - 5' exonuclease activity
interaction opens the ring of that identifies copying errors & corrects them.
Dna B and results in loading of DNA - Gap filling in lagging strand, proof reading & repair
2 ring shaped Dna B hexamers polymerase I - Mostly involved in host clean up functions during
in DUE, one onto each strand.
replication, recombination & repair (d/t 5' —» 3'
The ATP bound to Dna C is
exonuclease activity)
hydrolyzed, releasing the Dna C
and leaving Dna B (helicase) - It locates space (nick) between 3' end o f DNA newly
bound to DNA. synthesized (by polymerase III) & 5' end o f adjacent RNA
primer.
- Next, it removes RNA nucleotides (primer) ahead of
itself moving in 5 '—>3' direction (by its (5' —> 3'
DnaB 6* - Helicase (Dna B) binds to single
exonuclease activity) and synthesizes DNA/chain
protein strand DNA near replication
elongation (by its 5' —» 3' polymerase activity).
(Helicase) fork (V) and then move along
During synthesis it also proofreads the new chain
&M CM the SS DNA (that becomes
using 3' —» 5' exonuclese activity.
helicase lagging strand) in 5' -> 3'
- This removal - synthesis - proofreading continues
direction into the neighboring
one nucleotide at a time, until the RNA is totally
double stranded region, forcing
degraded & replaced with DNA
them apart -in effect processive
unwinding the ds DNA to - The 5 ' -> 3' exonuclease activity is specific to DNA
provide ss DNA as it travels. It polym erase I and is responsible fo r Nick translation®
requires energy provided by - 5 '—>3' exonuclease can remove 1 or upto 10, properly
ATP. Dna B is principal helicase base paired or altered, nucleotides (either
of E.coli. ribonucleotides or deoxyribonucleotides) of RNA or
DNA at a time. This ability is important in repair of
- Thus both Dna B halicases damaged DNA.
loaded on to the two strands of
When 5' —>3' exouclease domain is removed the
DNA travel in opposite
remaining large fragment is k/a Klenow fragment
directions, creating 2
replication V or Y forks. DNA - DNA proof reading & repair
polymerase
- All other proteins at replication II
fork are linked directly or DNA - Processive leading strand and oka z a k i fragm ent
indirectly to Dna B. pol III polymerse synthesis®. It is a assembly o f 13 protein subunits (of a
holoenzyme is linked through x III different types)
Biochemistry: Structure, Function and Replication of Informational Macrom olecules ■ 659

subunit. Subunit No. per Gene Function


holo-
- In eukaryotes, a
enzyme
heterohexameric, ring shaped
complex of minichromosome Core a 2 pol C Polymerization activity
maintenance (MCM) helicase polymerase e 2 (dna E) 3' 5' Proof reading
(MCM 2 to MCM 7) functions 2 mut D exonuclease
6
like prokaryotic Dna B helicase. (dna Q) Stabilization of e subunit
MCM is loaded on to DNA by a hoi E
hexameric protein complex ORC
Clamps P 4 dna N DNA donut shaped
(origin recognition complex) =
sliding clamps required
Key step
for optimal processivity
Stabilizing 4* - Seperation of DNA double helix (one dimer clamp per core
proteins: is promoted by single stranded subassembly).
Single DNA binding protein (SSB) in Clamp x 2 dna X Stable template binding;
stranded prokaryotes and replication loading (y) core enzyme dimerization
DNA protein A (RP-A) in eukaryotes. complex
Y 1 dna X* Clamp *Clamp loading
Binding
- These bind co-operatively & loader (y) complex
protein £ 1 hoi B
stoichiometrically to SS DNA Clamp loads (3
(SSB) & 8 1 hoi A
with out interfering with the loader subunits on
Replication
abilities of neucleotides to serve lagging strand
Protein A Clamp
as template. So these stabilizes at each okazaki
(RPA) opener
the single stranded structure as fragment. It has
replication fork progresses. 5 subunits of 4
- These proteins (not enzymes) different types
keep the two strands o f DNA x2 y SlS
separated by preventing X (Chi) 1 hoi C Interacts with SSB
premature reanneiling o f ds
V (Psi) 1 hoi D Interacts with y and %
DNAQ
DNA ligase - Nick sealing enzymes that seals (ligates) nick
- Protect DNA from nuclease that created by removal of DNA polymerase 1 in lagging
cleaves ss DNA strand (backbone of DNA in form of broken
- Bind to ss DNA created by phosphodiester bond) by using ATP
helicase cooperatively - i.e., the - Forms final phosphodiester linkage b/w 5' P 0 4
binding of one molecule of SSB group on nascent DNA chain synthesized by DNA
protein makes it easier for polymerase III & 3' OH group on chain mede by
additional molecules of SSB DNA polymerase I
protein to bind tightly. ★ i|/subunit is generated by translational frame shifting (of portion of
DNA 4 - A DNA topopscmerase II gene for x subunit) that 1/t premature translational termination
Grayse relieves torsional strain /
topological stress (positive
super coil or suprtwist) induced 2 replication forks of circular prokaryotic DNA meet at terminus
ahead of replication fork by the region containing m ultiple copies of 20 bp sequences called Ter.
helicase induced unwinding Ter sequences function as binding sites for protein terminal
process. utilization substance (Tus) and create a trap that a replication fork
- A type 11 topoisomerase that has can enter but cannot leave.
unusual property of being able - Tus - Ter complex can arrest a replication fork from only one
to introduce negative supercoils direction. Only one Tus-Ter complex functions per replication cycle -
into relaxed circular DNA by the complex first encountered by either replication fork.
using energy (ATP)Q. This
- Because opposing replication forks usually halt when they collide,
facilitates the future replication of
Ter sequences would not seem essential, but they may prevent over
DNA because negative
replication by one fork in the event that the other is delayed or hated
supercoiling neutralize positive
by an encounter with DNA damage or other problem. So, w hen eith er
supercoils introduced during
rep lication fo r k fir s t encounters a Tus-Ter com plex, it h alts; the oth er
opening of ds DNA. Also aids
rep lication fo r k h a lts w hen it m eets the fir s t arrested fo r k .
transient strand separation.
660 ■ A Complete Review of Short Subjects

DnaG 1 - DNA primase is a specific RNA - Final few hundred bps are replicated by unknown mechanism
(Primase) polymerase that synthesize completing two topologically interlinked (catenated) circular
short (~ 10 nucleotide long) chromosomes (k/a catenanes). The circles are not covalently linked,
stretches of RNA that are but because they are inter wound and each is covalently closed, they
complementary and antiparallel cannot be separated. Separation of catenanes requires topoisomerase
to DNA template and are k/e IV (a type II topoisomerase) which transiently breaks both DNA
RNA primer strands of one chromosome and allowing the other chromosome to
pass through the break.
As with DNA synthesis the - In eukaryotes, telomeres are synthesized.
direction of primer synthesis is
Topoisomerase Relieves torsion strain (supercoiling or
5' -» 3'. One primer (& primase) supertwisting) that result from helicase induced
is required for leading strand unwinding Q in the region of DNA ahead of
and many for lagging strand. replication fork. The swivel function is provided by
introducing nicks in strand, thereby allowing
Helicase associates with primase
to provide it proper access to the unwinding & then resealing the nicks. These are of
2 types
template. This mobile complex
of helicase & primase is called Type I Have both nuclease (strand cutting) and ligase
primosome. topoisomerase (strand / nick sealing) activity. They do not require
ATP, but rather appear to store the energy for
Primer synthesis is important phosphodiester bond they cleave, reusing the
since, DNA polymerase cannot energy to reseal the strand.
initiate DNA synthesis denovo It reversibly creates a transient nick (cut) in one strand
(without primer). of ds DNA. The intact DNA strand is passed
through the break before it is resealed, thus
Hda Homologous to Dna A (or Hda)
relieving (relaxing) accumulated supercoils.
is a AAA + ATPase that binds to
beta subunit o f DNA polymerase HI Type I topoisomerase relax negative supercoils in E.
and interacts with Dna A to coli, and both negative & positive supercoils in
stimulate hydrolysis of its bound eukaryotes. (Positive supercoil is produced by
ATP. This ATP hydrolysis twisting in the direction o f tightening of strands so the
disassembles the Dna A DNA would contain more turns of helix than
complex at origin. That is why, relaxed DNA. Whereas, the negative supercoil is
loading o f DNA pol III beta produced by twisting in the direction o f loosening the
subunits on to the DNA signals strands; so the negatively super twisted DNA
completion o f initiation phase would contain fewer turns of helix than relaxed
DNA).
Dam 1 - Methylates N 6 position of Type II Make transient break in both strands and relieve both
Methylase adenine within the palindromic topoisomerase positive & negative supercoils by utilizing ATP
sequence (51) G ATC at origin (in
Required in both prokaryotes & eukaryotes for
Ecoli). Dam stands for DNA
separation of interlocked molecules of DNA
adenine methylation and it
following chromosomal replication.
affects timing of replication
initiation. C ontinuous leading strand synthesis using
single prim er (D N A polym erase III) in
direction tow ards replication fork
Immediately after replication,
DNA is hemimethylated at ori ie
■5’
parent strands methylated but
<^ R e p lic a tio n fork m o v e m e n t"
newly synthesized strand do
3’-
not. The DNA must be fully -►3'
methylated before it again binds *-8 8 —8 8 ---------- — 5'
D N A gyrase
Dna A and initiates new (Topoisom erase II) r
D N A B helicase D naG SSB D iscontinuous legging
replication eg cycle. on parent strand
travel in 5 '-> 3' prim ase stabilize strand synthesis
relives torsional
d irection and form p rim er single (by D N A polym erase III
strain o f unw ounding
unw ound DNA in 5 '-» 3 ' strand using m ultiple prim ers
d irection in a looped strand
Biochemistry: Structure, Function and Replication of Informational Macromolecules ■ 661

Components of Nucleic Acid

Nitrogen -Bases Pentose Sugar: Nucleoside


- Purines & pyrimidines are nitrogen & carbon - Addition of pentose sugar to nitrogenous base produces a
containing heterocyclic aromatic compounds with a nucleoside
planar character (i.e. two or more bases are - Sugar is D-ribose in ribonucleoside (of RNA) and 2-deoxy-
positioned with the planes of their rings parallel D-riobse in deoxy ribionucleotide (of DNA).
like a stack of coins)- a feature that facilitates their - Carbons of pentose sugars are prime numbered l 1 to 5'
close association or stalking that stabilizes double distinguishing them from those of heterocycles. Hence, 5'
stranded DNA. carbon of nucleoside (or nucleotide) means a carbon atom in
- Larger purine molecules and smaller pyrimidine pentose ring rather than atom in base.
molecules are numbered in opposite direction for - All bases are attached to pentose sugar by a P-N- glycosidic
their 6 atom main ring i.e. clockwise fo r pyrimidine & bond between 1 st carbon of pentose sugar and almost
anteclockwise for purine. always to the N -l of pyrimidine or N-9 of purine.
- Nucleosides are N-glycosides formed by addition of pentose
- Free purines and pyrimidines with - NH 2 group are
sugar (D-ribose or 2 deoxy D-ribose) to a purine (adenine,
weak bases. Physiological pH strongly favour oxo
guanine) or pyrimidine (cytosine, thymine & uracil) bases.
and amino groups of purine and pyrimidine
Nucleosides with purine bases (A, G) have - 'sine' suffix viz
exhibiting Keto-enol and amine- imine tautomerism.
- adenosine, guanosine 0 , whereas pyrimidine nucleosides
have 'dine' suppix viz - cytidine, thymidine and uridine 2 .
Purine Bases Pyrmidine Present in
Bases Phosphate/Phosphoric Acid: Nucleotide
Adenine = A Cytosine = C Both DNA
- Nucleotides are phosphorylated (i.e. phosphate esters of)
G uanine = G & RNA
nucleosides.
Thym ine = T DNA
Uracil = U RNA Nuceotide = Nucleoside + Phosphate or Phosphoric acid
Hypoxanthine (6 M inor = Purine / Pyrimidine Base + Pentose (D-ribose / 2 deoxy
oxopurine) bases D-ribose) sugar + Phosphate / Phosphoric acid
Xanthine (2, 6- formed in
dioxopurine) catabolism - Nucleotides may contain 1, 2 or 3 phosphate groups and are
Uric acid (2, 6, 8- of purines called mono, di or tri phosphate nucleotides (i.e. MP, DP, or
trioxopurine) TP)
N6, N6 N4 acetyl Unusual - l sl phosphate (phosphoryl) group is attached (esterified) to
dimethyl cytosine Bases hydroxyl (OH) group of pentose sugar by ester linkage on 5 1
adenine Dihydro uracil (mostly) or 3' (rarely) position. Since most are 5'-nucleotides
* Methylation acetylation, glycosylation or reduction or nucleoside 5' phoshate (i.e. have phosphate at 5' position)
in some viral DNA & tRNA bases forms unusual the prefix "5 '" is usually omitted when naming them.
buses which may aid in its recognition by specific - Suffix sin ^ - nylate/- nylic acid" for purine nucleotides &
enzymes or protect it from being degraded by d in ^ - dylate/-dylic acid" for pyrimidine nucleotides are
nucleases. used.
★ T and U differ in that only T has a methyl group.

Pentose Sugar Nitrogenous Base Nucleoside Nucleotide (Mono /Di /Tri- phosphate), NDP NTP
M ono phosphate (MP)
D-ribose sugar Adenine (A) Adenosine Adenosine /Adenvlate /Adenvlic acid- ADP ATP
forming simple mono phosphate (AMP1
nucleosides & Guanine (G) Guanosine Guanosine /Guanvlate /Guanvlic acid - GDP GTP
nucleotides in monophosphate (GMP)
RNA and 2-deoxy Cytosine (C) Cvtidine Cvtidine /Cvtidvlate /Cvtidvlic acid CDP CTP
ribose sugar mono phosphate (CMP)
forming deoxy Thym ine (T) Thvmidine Thvmidine /Thvmidvlate /Thvmidvlic TDP TTP
(or d-) acid mono phosphate (TMP)
nucleosides & Uracil (U) Uridine Uridine /Uridvlate /Uridvlic acid mono UDP UTP
nucleotides in phosphate (UMP)
★ Xanthine base form Xanthosine nucleoside and hypoxanthine base forms inosine nucleoside of RNA.
662 ■ A Complete Review of Short Subjects

Denaturation / Melting of DNA and T m


«The ds-DNA can be separated into two strands Tm & Factors affecting Tm
(melted) in solution by increasing the temperature »It is the temperature at which DNA is half denatured. In other words,
or decreasing the salt concentration. Denaturation is the strand of a given DNA molecule separate over a temperature
also called a helix to coil transition. range. The midpoint is called the melting temperature or Tm. It is
>Not only do the two stacks of bases pull apart but calculated by nearest neighbour method.
the bases themselves unstack while still >At Tm, the absorbance of DNA at 260 nm is increased by 18.5% i.e.
connected in the polymer by the phosphodiester half of 37% increased in absorbane when DNA is completely
backbone. denatured. It is called hyperchromicity phenomenon or
>Concomitant with the denaturation of DNA hyperchromic effect.
molecule is an increase in the optical absorbance o f >It is in influenced by
purines & pyrimidine bases - a phenomenon Both length o f DNA molecule & specific nucleotide sequence. DNA rich
referred to as hyperchromocity of denaturation. in C = G base pairs with 3 H bonds melts at higher temperature than
DNA loses its viscosity on denaturation . that rich in A = T base pairs with 2 hydrogen bonds.
>The denaturation (melting) is used to analyze the Salt concentration of solution influence the Tm because negatively
structure of DNA. Denaturation & renaturation charged P 0 4 in two strands are shielded by positively charged ion
of DNA are basis of nucleic acid hybridization. (cation). Decreased ionic concentration decrease the shielding effect,
•Separate strands of DNA will renature or thus increase the repulsive forces b/w strands and reducing Tm.
reassociate when appropriate physiological So a ten fo ld increase in m ovalent cation concentration increase the
temperature & salt conditions are achieved; this Tm by 16.6°CQ
reannealing process is often referred to as Formamide, which is commonly used in recombinant DNA
hybridization. If melted sample of DNA is experiments and urea both of them destabilizes hydrogen bonding
slowly cooled the absorbance of solution between bases, thereby lowering Tm. This allows the strands of
decreases, which is an indication of DNA or DNA hybrids to be separated at much lower temperatures
complementary strands being pulled again. and minimizes the phosphodiester bond breakage that can occur at
Annealing can occur at a temperature below Tm higher temperatures.
of DNA (i.e. 70°c). It is fastest at 20°c below Tm
(i.e. 50°c) Applications of Tm
>Hybridization when combined with gel Estimate amount of G & C content
electrophoresis (that separate nucleic acids by In PCR
size) & radioactive or fluorescent labeling To detect sequence differences b/w strands & different DNAs
resulting analytic technique is called southern As a proxy for equalizing the hybridization strength of molecule eg
(DNA-DNA) & Northern (RNA - DNA) oligonucleotide probe of DNA microarray.
blotting.

Ultraviolet Spectrophotometry of Nucleic Acids


Electron delocalization among atoms in aromatic rings of purine & pyrimidine bases give most of the bonds partial double bond
character. All nucleotide bases (d/t conjugated double bond of purine / pyrimidine) absorb ultraviolet lightQ and nucleic
acids (DNA /RNA) are characterized by a strong absorption at near 260 nm wavelengths.
Mutagenic effect of UV light is d/t its absorption by nucleotides in DNA 1/t chemical modification.
Although spectra is pH dependent, all nucleotides of DNA /RNA absorb light at wave length close to 260 nm, at pH 7. So
concentration of nucleotides & nucleic acids are often expressed in terms of "absorbance at 260 nm".
DNA purity (quality) measurement is based on the fact that the clean/pure DNA has twice (1.8 -2 times more) optical density
(OD) at 260 nm then at 280nm. So if OD-260/OD-280 is lesser, it indicates contamination, which decreases OD ratio. Hence
purify the DNA again.

Resultant DNA Damages

I. Single base alteration II. 2 - Base alteration


- Depurination (rarely depyrimidination) - UV induced thymine- thymine (pyrimidine) dimer
- Deamination of cytosine (C) to uracil (U) - Bifunctional alkylating cross linkage
and adenine (A) to hypoxanthine III. Chain breaks d/t ionizing radiation, oxidative free radicle formation or
- Alkylation of base radioactive disintegration of backbone element
- Base-analog incorporation IV. Cross linkage between
- Insertion or deletion of nucleotide Bases in same & opposite strands
DNA & protein molecules eg histone
Biochemistry: Structure, Function and Replication of Informational Macromolecules ■ 663

DNA Repair Mechanism

DNA DNA lesion DNA Repair Fidelity of Procedure


Damaging produced Pathway Repair
Agents
Replication Base (single or 2-5 Mismatch +++ Corrects a single mismatch base pair (eg C to A rather than T
(copying) errors base unpaired loops) Repair (MMR) to A) or short region of unpaired DNA. The defective region
mismatch; Insertions; in newly synthesized unmethylated strand is recognized by
Deletions (mut H) endo nuclease that makes a single strand cut at an
adjacent methylated GATC palindrome sequence (present in
opposite orientation on two strands). The defective DNA
strand is removed through mutation (by exonuclease),
replicated or replaced (by DNA helicase II, SSB, DNA
polymerase III) and relegated (by DNA ligase). Dam
methylase methylates N6 adenines within (51) GATC sequence
of parent (template) strand. Replication mismatches in vicinity
of hemimethylated GATC sequence are then repaired
according to information in methylated template strand. If
both strands are methylated (in vitro) at GATC, few
mismatches are repaired and if neither strand is methylated
repair occurs but does not favor either strand.
Oxygen radicles Abnormal bases like Base Excision +++ Base removal by N-glycosylase, a basic sugar removal,
; Hydrolysis; uracil, hypoxanthine, Repair (BER) replacement
Alkylating xanthine, alkylated
agents bases, small
pyrimidine dimmers,
Abasic sites; single
strand breaks; 8
oxoguanine lesions
UV light, Large structural Nucleotide +++ Removal of an approximately 30-nucleotide oligomer and
chemicals DNA lesions like Excision Repair replacement
pyrimidine ( NER)
dimmers, Bulky
adducts
Ionizing Double / Single Double strand Synapsis, unwinding, alignment, ligation
radiation, X- strand breaks; intra / break repair.
rays, Antitumor inter strand cross - Homologous ++
drugs links recombination
(HR)

+
-Non
Homologous
End Joining
(NHEJ)
Any Direct Repair

Pyrimidine dimmers DNA


O 6 methyl guanine photolyases O
methyl-guanine
-DNA methyl
transferase
Aik B protein 1 methyl
guanine 3
methyl cytosine
664 ■ A Complete Review of Short Subjects

Mismatch Repair (MMR) Base Excision Repair (BER)


- Mut L protein forms complex with Mut S protein - DNA glycosylases remove purine /pyrimidine created by spontaneous
and the complex binds to all mismatched bases deamination and create AP (abasic or apurinic) site, eg uracil DNA
(except C-C). Mut H binds to Mut L and glycosylase removes the uracil created by spontaneous deamination of
unmethylated GATC (encountered by Mut L- Mut S cytosine in DNA (it does not remove T from DNA or U from RNA).
complex).
- Humans have 4 types of DNA uracil glycosylase: most abundant UNG
- DNA on both sides o f mismatch is threaded (looped) is a/w replisome & removes occasional U inserted in place of T during
through Mut L- Mut S complex and both legs of loop replication; h SMUG1 removes any U residues that occur in ss DNA
move through complex simultaneously (equivalent during replication or transcription; TD G & M BD 4 remove either U or
to complex moving in both directions along DNA). T residues paired w ith G, generated by deamination o f cytosine or 5
Mut H attached to complex has site specific methyl cytosine respectively 0.
endonuclease activity which is activated on
encountering hemimethylated GATC sequence and - AP endonuclease cut DNA strand containing the AP site and a
cleaves unmethylated strand on 5' side of G in segment of DNA containing AP site is removed.
GATC. - Repair polymerase (DNA polymerase I) replaces DNA and fills the
- When mismatch is on 5' side (right) of cleavage site defect and DNA ligase seals the remaining nick (ligates the space
the unmethylated strand is unwound (by helicase II) created by DNA polymerase I rem oval)
and cleaved in 3' —>5' direction from cleavage site
by exonuclease I or exonuclease X (both of which
3 a t c g g c t H a t c c g a t 5
degrade DNA strand in 3' —»5' direction). I I I I I I I I I I I I I I I
,. T A G C C G A G T A G G C T A

Whereas mismatch or 3' side (left) use either H ea t e n erg y

exonuclease V II (which degrade ssDNA in 5' —>3' or A T C G G C T l A T C C G A T

3'-> 5' direction) or Rec J nuclease (which degrade ss I I I I I I I I I I I I I I I


T A G C C G A G T A G G C T A
DNA in 5 '—>3' direction).
U [ URACIL DNA G LYCO SY LASE |

a t c g g c t H a t c c g a t

I I I I I I I I I I I I I I I
T A G C C G A G T A G G C T A

I N UCLE ASE S |

A T C G G C T C C G A T
I I I I I I I I I I I I
T A G C C G A G T A G G C T A

| DN A POLYMERASE * DNA LIQASE ~~|

CHj CH3
A^ T C G G C T 1 A T C C G A T
I I I I I I I I I I I I I I I
-------------------------------------------* ---------------------- 3
T A G C C G A G T A G G C T A
StNGLE-SrTE STRAND CUT
BY GATC ENOONUCLEASE

CHj CHj
Nucleotid Excision Repair (NER)
------------------------------------------- H---------------------- 3

DEFECT REMOVED
BY EXONUCLEASE - Larger DNA defects that cause large distortion in DNA helix are
CH, CHj
repaired by NER.
------------------- 3' - A multisubunit excinuclease enzyme hydrolyze 2 phosphodiester
1 DEFECT BEPAWED
BY POLYMERASE bonds one on either side of distortion. This corresponds in E.coli &
CH j CHj bacteria to 5th phosphodiester bond (PDB) on 3' side & 8 lh PDB on 5'
side creating 12 to 13 nucleotide fragment; and in humans to 6 th PDB on
REUGATEO 3' & 22nd on 5' side producing a fragment of 27-29 nucleotides.
BY UGASE

CH j CH3 - Excinuclease is a endonuclease with a unique capacity to catalyze 2


specific endonucleolytic cleavages. Following the dual incision the
excised oligonucleotide are replaced from duplex and resulting gap is
filled by DNA- polymerase I in E.coli and DNA polymerase e/8 in
__________ Double Strand Break Repair ________ humans. DNA ligase seals nick.

The protein Ku and DNA-dependent protein kinase - In E.coli ABC excinuclease has 3 subunits Uvr A, Uvr B (A2B complex
combine to approximate the two strands and unwind of both units scans DNA and binds to site of lesion; Uvr A 2 dimer then
them. The aligned fragments form base pairs; the extra dissociates leaving a tight Uvr B- DNA complex) and Uvr C (binds to B
Biochemistry: Structure, Function and Replication of Informational Macromolecules ■ 665

ends are removed, probably by a DNA-PK-associated subunit and Uvr B makes an incision at 5th PDB on 3' side whereas Uvr
endo-or exonuclease, and the gaps are filled in; and C makes an incision at 8 th PDB on 5' side of lesion).
continuity is restored by ligation.
77........ )//
Ku and DNA-PK bind

Approximation

?/
/A

Unwinding

(| im 7 /

Alignment and base pairing

/A t \ t i n
\
Liga tio n
/A \ I I I I I I I I I I I I 1 )7 /

Disease Defect in DNA repair Feature


Xeroderma Nucleotide excision repair pathway - Autosomal recessive (AR)
pigmentosa specilly thymine dimmers cannot be
- Marked sensitivity to sunlight (UV rays) 1/t multiple
incised d /1 deficiency o f enzyme nicking
skin cancers and premature death
endonuclease (m.c), DNA polymerase I or
DNA ligase, helicase (Subunits of
transcription factor TFIIH)
Ataxia telangiectasia - Autosoml recessive (AR)Q
- Cerebellar ataxia, lymphoreticular neoplasms,
increased sensitivity to damage by x-ray, oculo
cutaneous telangiectasia & IgE deficiency.
Fanconi's anemia Chromosomal instability & defective - Autosomal recessive (AR)
repair o f cross linking damage - Anemia & increased frequency of cancer
- Defective gene located on chromosome 20q & 9q
Bloom 's syndrome Chromosomal breaks & rearrangements - Facial erythema, photosensitivity and telangiectasia
are seen d/t defective DNA ligase
Heriditary non Faulty mismatch repair due mutation - Most common inherited cancer
polyposis colon cancer in hM SH 2 gene (located - Cells which harbour hMSH2 (50-60% cases) or hMSH-
(HNPCC) inchromosome 2 ) that encodes human 1 mismatch repair enzyme, are unable to remove
analog of E.coli M uts p rotein that is small loops of unpaired DNA, & the microsatellite
involved in mismatch repair. DNA thus increase in size. It affects either expression
or function of a protein critical in surveillance of cell
cycle in colon cells.
666 ■ A Complete Review of Short Subjects

Wobble Hypothesis
- It explains the degeneracy o f genetic code, a
mechanism by which t RNAs can recognize more
than one codon for a specific aminoacid, in which
the base at 5' end of anticodon ( = 1st base of
anticodon) is not as spatially defined as the two
other bases.
- Movement of this 1st base o f anticodon (5' end)
allows non traditional (i.e., not according to
watson- Crick rule) base pairing with the last base
o f codon (at 3 ' end). This is called wobble and
allows single t RNA to recognize more than one
codon.
- So there need not be 61 tRNA types to read 61
codons. For example two codons for arginine,
AGA & AGG, bind to same anticodon having
uracil at 5' end (UCU), or 3 codons for glycine -
UCG mRNA
GGU, GGC, and GGA can bind to one anticodon
CCI. (I = inosine nucleotide)
T ra d itio n a l base p a irin g observed N on traditional base pairing b/w 3 rd (3') p o sitio n
* Codon of mRNA is recognized by anticodon of in I s1 & 2 nd p o sitio n o f codon: o f co d o n an d 1 st (5 ') p o sitin o f a n tico d o n : W o b b le
tRNA and it follows the rule of complimentary & t-R N A m RN A
tRN A m -R N A
antiparallel binding, i.e. the mRNA codon is read A U
A U
5' —>3' by an anticodon pairing in the flipped G C ,U
G C
3 '—>5' orientation. When writing the sequences of H C ,U ,A
U A
both codons & anticodon, the nucleotide sequence U A,G
C G
must always be listed in the 5'->3' order.
★ H = h y p o x a n th in e (b a se o f in o sin e)

Characteristics of Motif Containing Proteins


Control o f transcription requires high affinity and specificity in binding o f regulatory proteins to the correct region o f DNA,
which is provided by motifs®. Binding must be high affinity to specific sites and low affinity to others. Binding must be high
affinity to specific sites and low affinity to others. The DNA - protein interactions are maintained by hydrogen bonds & vander waals
forces. The presence of motifs in unknown protein suggest that the protein may bind to DNA.
Small regions o f m otif make direct contact with DNA. The rest of protein may provide ligand binding sites, trans activation
domains, or contact surface for formation of hetero dimers, & interaction with coactivatior or corepressor. It may be involved
in dimerization of monomers of binding protein. Proteins w ith helix turn helix & leucine zipper motifs form symmetrical
palidrom es and their DNA binding sites are symmetrical palindromes®. Whereas, in proteins with zinc finger motif, the
binding site is repeated 2- 9 times®.

Helix turn helix motif Zinc- finger motif Leucine- zipper motif
Each monomer consist of three antiparallel fi- sheets & They are series of repeated Carboxyl terminal of DNA
three a- helix. domains (2-9) in which each binding protein C/EBP, forms
- (33 sheets for dimmer. is centered on a tetrahedral an a- helix in which there is
- oc3 helices form DNA recognition surface. coordination w ith Zn®. periodic repeat o f leucine
Examples of transcription regulatory proteins containing In TF III A (transcription residue a t every 7th position®.
helix turn helix motif regulatory protein) 9 zinc This structure allow two
ions present in repeating identical monomer or hetero
Regulatory protein Organism CoO ordination complex dimer (eg, fos- Jun or Jun- Jun)
lac repressor, CAP E. Coli formed by pair o f cystine (C) to 'Zip together' in a coiled coil
Cro, A cl, tryptophan & 434 represor Phage separated by 12-13 amino acid & form tight dimeric complex.
Homeo box proteins, pit 1, Octl, Mammals from a pair o f histidine residue. This enhance association of
Oct2 Transcription regulatory separate DNA binding domain
proteins containing Zn- with their target
finger motif
Biochemistry: Structure, Function and Replication of Informational Macromolecules ■ 667

Regulatory Orgainsm • Transcription regulatory


protien containing leucine zipper motif
Gene 32 protein E. coli Regulatory protein Organism
Gal 4 Yeast GCN-4 Yeast
S e r e n d ip ity , Drosophila C- myc, n- myc, 1- Mammals
H u nch back myc
CRE binding protein
TF III A Xenopus
C/EBP
S teroid receptor Mammals
C y s -H ls z in c finger Fra 1, fos
fa m ily Q, Spl
Jun
Zn finger motif Leucine Zipper motif
* G e n e e x p re ssio n is c o n tro lle d by re g u la to ry s e q u e n c e s o f D N A th a t lie in non c o d in g region s o f g e n o m e fla n k in g a co d in g g en e. T h e se are C is -
a c tin g , if th e y in flu en ce e x p re ssio n o f g e n e o n ly on sam e c h ro m o so m e a n d tr a n s a c tin g if in flu e n c e s th e e x p re s s io n o f g e n e on oth er
c h r o m o s o m e (th a t is d iffe re n t fro m its site o f o rig in ).

Gene Clstron
A segment o f DNA molecule that contains the information required for synthesis of a -As we know; some enzymes &
functional biological product, whether protein or RNA, is referred to as a gene proteins are composed of 2 or more
Edward T a tu m & George B e a d l e proposed molecular definition of gene in 1940 a /1 non identical subunits. Thus 1 gene- 2
which a gene is a segment o f genetic material that determines or codes for one enzyme" the enzyme concept is not always valid.
on e gen e-on e enzym e h y p o t h e s is . Later this concept was broadened to 1 g e n e - 1 - C i s t r o n is the sm a llest unit o f
p o ly p e p tid e , because many genes code for a protein that is not an enzyme or for one gen etic expression Q. It is the basic
polypeptide of a multi subunit protein. genetic unit coding f o r one
A /1 modem biochemical definition, a gene is all the DNA that encodes the primary p o ly p ep tid e ch a in /stru ctu re o f
sequence o f some final gene product, which can either be polypeptide or an RNA with a subunit o f a protein moleculeQ.
structural or catalytic function. -So one gene- one enzyme concept
G e n e is a f u n c t i o n a l u n i t that encodes a RNA product which is mostly (but not always) might more accurately be regarded
translated into a protein, which exerts activity within or outside the cell. (Harrison-385). as o n e c i s t r o n - o n e s u b u n i t concept.
Gene is regulated by transcription & corresponds to a unit o f inheritance -A single mRNA that encodes >.1
I n d u c ib l e g e n e s , with relatively low basal rates o f transcription are upregulated (i.e. sepeatily translated proteins is
expression increased) in response to an in d u c e r or a c t iv a to r (a p o s i t i v e r e g u la t o r ). By referred to as p o ly c i s t r o n i c m
contrast repressible genes, with high basal rate of transcription are often down R N A . Eg polycistronic lac operon
regulated by r e p r e s s o r (negative regulator). m-RNA.
H o u s e k e e p i n g g e n e s are c o n s t i t u t iv e l y expressed meaning that they are expressed -But this concept is also changing as
at a reasonably constant rate & not known to be subject to regulation. A mutation many cistrons could be spliced &
resulting in constitutive expression of what was formerly a regulated gene is k/a rearranged. In other words, m a n y
c o n s t i t u t iv e m u t a tio n . g e n e s to g e th e r c a n p ro d u c e o n e
(or s t a b i l i t y ) g e n e s encodes proteins that function in repair o f major genetic
C a re ta k e r p e p t id e c h a in b y means of
defect that result from aberrant DNA replication, environmental carcinogens or t r a n s p o s it i o nof genes or g e n e
ionizing radiation. So mutation in these genes l/t mutation (unrepaired damage) in r e a r r a n g e m e n t or s o m a t ic
other genes including protoonchogenes & tumor suppressor genes and thus to re c o m b in a tio n o f D N A o r g e n e
cancer. Examples include ATM gene, XP gene (1/t Xeroderma pigmentosa), BRCA1 s h u f f l i n g (described by Susumu
gene (1/t breast cancer), and p53 gene (1/t Li Fraumeni cancer syndrome.) Tonegawa for antibodies).

Jacob & Monod’s Operon Concept for Prokaryotic Gene


In prokaryotes, genes involved in a metabolic pathway are often present in a linear array called an o p e r o n . The c i s t r o n is the
smallest unit of genetic expression coding for subunit of a protein molecule (One cistron - one subunit concept). P o ly c is t r o n ic
m R N A , predominantly found in prokeryotes is a m-RNA that encodes more than one seperately translated proteins. Operon
(transcription unit of prokaryotes) consists of
l------------------------------------------------------------------------------------------ 1
Structural (coding) genes Regulatory region consisting
- Carries the codon that code for Promoter (P) gene
proteins involved in a particular - It is situated on the upstream (5'- side of transcription start site b/w operator &
metabolic pathway regulator gene.
- In lac operon of E coli (responsible - It is the site, where DNA dependent - RNA p oly m erase (RNP) binds to fo rm
for lactose metabolism), 3 prein itiation com plex (PIC)Q, and commence transcription.
668 ■ A Complete Review of Short Subjects

structural genes lac Z, lac Y, & lac - C a t a b o l it e g e n e a c t iv a to r p r o t e in (C A P ) in c o n ju n c t io n w i t h c - A M P (als k/a c-AMP


A code respectively for ff- regulatory protein = C R P ) is required for efficient formation of PIC. So it is a p o s itiv e
galactosidase, (hydrolyze lactose r e g u la t o r . Adenylate cyclase which forms c-AMP from ATP, is activated by absence of
or fi- galactoside to galactose & glucose. So the lac operon transcription on ly occurs in absen ce o f glucose® ( c a ta b o lite
glucose), permease (responsible for r e p r e s s io n ) . And presence of glucose prevents it by depleting c-AMP.
permeation of galactose into cell), Operator (O) genes
and thiogalactoside transacetylase. - It is a segment of ds DNA (i.e., gene) that regulates the activity o f the structural genes.
- In lac operon, it is 27 bp long segment situated between promoter site (at which RNAP
binds) and lac Z (transcription initiation site) and show a t w o fo ld r o ta t io n a l s y m m e tr y
Inducible Constitutive
and a n in v e r te d p a lin d r o m e in 21 bp long region as shown below
regulated (house keeping)
genes genes
5' - AATTGT d ------------ 3'
-Their -They are
expression expressed at a 3 ' ........... ..........................C
T G T T A A 5'
: <-------------
increases in reasonably When attached to operator (o) gene, the r e p r e s s o r (LacI) acts as a n e g a t iv e r e g u la t o r and
response to an constant rate and prevents transcription of operator and structural (lac /Y/A) genes.
inducer or are not subjected
activator (a to regulation® Regulator (I) genes
positive -The product of -It is a constitutive gene, that forms subunits of lac repressor (MW 38,000), A lac repressor
regulatory these genes are molecule (L a c I r e p r e s s o r p r o t e in ) is formed by union of 4 subunits and has high affin ity
signal) required all the f o r o p erator locus®.
- In general, time® in a cell. - At any time only 2 subunits of repressor binds to operator, mostly in its major groove
inducible - A mutation without interrupting the base paired double helical nature o f operator DNA.
genes have resulting in -Normally 20-40 repressor tetramers, occupy > 95% of lac operator, thus ensuring a low
relatively low constitutive (but not zero) lac operon transcription in absence of inducer.
rates o f expression of what - P r e s e n c e o f la c t o s e o r g r a t u it o u s in d u c e r (i.e., lactose analog which induces lac operon
transcription was formerly a but is not a substrate) such as i s o p r o p y l t h i o g a la c t o s id e ( I P T G ) on bacteria growing on
regulated gene is poorly utilized carbon source (eg succinate) results induction of lac operon enzymes by
called constitutive d e -r e p r e s s io n . This occurs because lactose / IPTG bind to free & attached repressor &
mutation decrease its affinity 103 times for operator by inducing coformational change.

★ The first base at transcription start site (of DNA) is assigned a position of +1. There is n o base designated " O " . A base is
assigned a n e g a t iv e n u m b e r , if it occurs upstream ( i.e., prior to or on left side or towards the 5'end of) the transcription start
site. And in the same way m o r e p o s i t i v i t y indicates more downstream (i.e., after to or on the right side or towards 3 -end)
distance from +1 position.
★ E n h a n c e r and r e p r e s s o r ( s ile n c e r ) , can either increase or decrease the rate of transcription initiation of eukaryotic gene
respectively. They can be lo ca ted upstream o r d ow n stream o f transcription sta rt site (+1) an d even w ithin transcribed
p ortion o f som e genes®. I n c o n t r a s t to proximal & upstream p r o m o t e r s , these can exert their effect when located even hundreds
or thousand bases away from transcription unit on same chromosome and function in orientation independent fashion. Enhancers
are promiscuous, they can stimulate any promoter (homologous or hetrologus). Enhancers work by binding one or more
proteins and by facilitating binding of basal transcription complex to promoter.

No inducer or Inducer is present with glucose With inducer & no glucose

Constitutive regulator
gene (I) O perator (I)
iP ro m o te r la c Z la c Y la c A
la c Z lac Y lac A m
- T w o s u b u n its bind a t a lim e and
p re v e n t tra n s c rip tio n o f o p e ra to r RNAP •Acetylase
RNA * \ Permease - Production
a n d la c Z /Y /A g e n e s
p o ly m e ra s e Repressor |)-galactosidase _
R e p re s s o r (R N A P ) subunits ^ o of Can’t bind
i operator (De-repression)
s u b u n its
• Inducers
\ - Lactose
R e p re s s o r (te tra m e r)
- IPTG (gratuitous inducer)
is a n e g a tiv e re g u la to r)
Biochemistry: Structure, Function and Replication of Informational Macromolecules ■ 669

Bacteriophage Lambda (A.): Gentetic Switch & Lysogenlc and Lytic Pathways

Bacteriophage lambda (A.) is a temperate virus that infects E. coli bacteria and injects its 45, 000 -bp- ds linear - DNA genome
into the cell.
Depending upon the nutritional state of h o st, the (A.) virus will enter eith er lysogen ic or ly tic pathw ays® depending on
which of 2 sets of viral genes is turned on
I
,________________________________________________________ _______________________________________________________ .

Lysogenic Pathway Lytic Pathway


Poor growth conditions favor lysogeny - Good growth conditions promote lytic
In lysogenic pathway, the viral DNA becomes integrated into bacterial pathway
chromosome where it replicates passively as the bacterial DNA and cell - In lytic pathway, viral DNA replicates
divides. itself and direct synthesis of viral
This d o r m a n t g e n o m i c a l l y in t e g r a t e d virus is called p r o p h a g e and the proteins
cell that horbours it is called a l y s o g e n . - About 100 new complete protein packed
A prophage can be induced by DNA damaging agents such as viral particles are formed. And the
ultraviolet rays to enter lytic phase by turning on different set of genes proliferating viruses induce the lysis of
(genetic switch) cell

Right operator (O r )

Right
L e ft OOOCtfOCCXX^
Cro
Gene for repressor protein gg Gene fo r regulatory protein
(Cro)
(cl)

C is a c tiv e , e v e n ly
s p a c e d 3 ta n d e m ly
a rra n g e d s im ila r b u t n o t
id e n tic a l

' O R3 OR2 ORI I

R e p re s s o r
R N A & 2 3 6 aa , 17 b P
Cro RNA&
27 K D a c l | | 66aa.,9KDa
repressor cro protein
protein R epressor Prom oter C ro prom otor

O r 3 Q r 2 O r 1
Or 3 Or2 Or 1
RNA i 1i 1i 1
polymerase TV
I i

Pepressor promoter cro promoter


i ^ _i____________ 0=3
" 'R '- '
Od2 Ool
W R4
I 1 r-----
Pepressor promoter * Cro promoter

V\Ar^
Pepressor promoter Cro promoter

Prophage Early lytic phase


670 ■ A Complete Review of Short Subjects

The lytic - lysogenic genetic switching in (X) is centered


around 80 bp ds DNA right operator (O r) genome which is
flanked on its left side by gene for repressor protein (cl) and
on its right side by genes encoding regulatory protein cro
(Mn "R is not on right side")
O r can be subdivided into 3 discrete, evenly spaced, tandemly
arranged, similar but not identical, 17 bp cis active DNA
elements termed operators O rI, O r2, Or3. These can be
bound by either cl or cro proteins, however by different binding
affinities, a feature that regulates molecular switch.
DNA between cro & repressor genes also contain 2 promoter
sequences that over lap the recognition (Or) sites and direct
Lysogenic Lytic
binding of RNA polymerase in either right ward direction (& pa^iw ay pathway
so transcribing cro protein) or leftward direction (& so
transcribing cl protein)
Repressor protein (cl) is a 236 aminoacid, 27 kDa product of
repressor gene. It exists as two domain molecule in which the
amino terminal domain binds to operator DNA (preferably
O rI) and carboxy terminal domain promotes the association
of one repressor protein with another to form a dimer. A
dimer of repressor molecules binds to operator DNA much
more tightly than does the monomeric form.
Cro protein is a 66 aminoacids, 9 kDa product of cro gene. It
has a single domain that operates both operator binding
(preferably to Or3) and dimerization. It also binds to operator
DNA more tightly as dimer.
When the (k) is in its lysogenic- prophage - dorm ant state- the cl repressor gene is the only gene that is expressed.
When bacteriophage is in lytic sta te - cro gene is expressed. So when the repressor gene is on, cro gene is off, and when the cro
gene is on, the cl repressor gene is off - a paradigmatic example of a m olecular transcription switch.

1
In lysogenic (dormant- prophage) lambda, the When a DNA damaging signal (eg UV rays) strike lysogenic host bacteria,
repressor (cl) dimer protein binds fragments of single stranded DNA are generated that activate a specific
preferentially to O rI and and enhances coprotease rec- A coded by bacterial gene
binding (by a factor of 10 ) of another cl dimer - Rec A protease hydrolyze portion of repressor protein that connects the
to Or2. The affinity of repressor for Or3 is least amino terminal to carboxy terminal

- Such cleavage of repressor domains causes the repressor dimers to


•O rI occupancy blocks the binding o f RNA dissociate, which in turn cause dissociation of repressor (cl) from O r2 &
polymerase to right ward promoter & in that way eventually from O rI
prevent cro formation (negative regulation).
- RNA polymerase immediately has acess to the rightward promoter &
Binding of repressor to Or2 enhances the binding commences transcribing the cro gene
o f RNA polymerase to leftward promoter that
- Newly synthesized cro protein binds to operator region, but its order of
overlaps Or3 and there by enhances
preference is opposite to that of repressor i.e. Cro binds most lightly to
transcription & expression of repressor (cl)
O r3 but there is no co operative effect of Cro at Or3 on binding of Cro to
gene (positive regulator)
Or2. At increasing concentrations of Cro, the protein will bind to O r2 and
•This dual regulation is responsible for stable eventually O rI
state of dormant lambda bacteriophage. When
- Occupancy of Or3 by Cro immediately turns off transcription from the left
intracellular cl protein concentration becomes
ward cl promoter & so preventing any further expression of repressor
very high, this excessive repressor will bind to
gene. Molecular switch is thus completely thrown in lytic direction. The
O r3 and by doing so diminish transcription o f
cro is expressed and repressor gene is fully turned off irreversibly
repressor gene from leftward promoter until the
cl concentration drops & repressor dissociates - In very high concentrations Cro occupy O rI and reduce expression of its
itself from Or3 own gene in the final stages of lytic cycle.
Biochemistry: Structure, Function and Replication of Informational Macromolecules ■ 671

DNA Dependent - RNA - Polymerase (RNAP)

It is the key enzyme in transcriptionc. In both, prokaryotes & eukaryotes, RNA polymerase does not require primer & has no proof reading
activityQ.

Prokaryotic RNA polymerase (Bacterial DNA dependent)

3' -» 5' e x o n u c le a s e a c t iv e s it e (as of DNA polymerase) & so e r r o r r a te f o r tr a n s c r ip t io n


R N A P la c k s a s e p a r a t e p r o o f r e a d in g
Because many copies of RNA are produced from single gene and eventually all are
i s h ig h e r t h a n t h a t f o r D N A r e p li c a t i o n .
degraded, so a mistake in RNA is of less significance to cell as compared to DNA which is permanently stored.

Structure & Function Requirements


>It recognizes prom oter nucleotide sequence® at the begining of length of RNA polymerase holoenzyme reuires:
DNA that is to be transcribed.
1. Template of double stranded DNA (or
It m akes a com plem entary RNA copy o f DNA tem plete (antisense)
occasionally single stranded DNA)
strand, w hich is identical to coding (sense) strand®; with U replacing the
2. Four ribonucleotide triphosphates GTP, UTP,
T. RNA is synthesized from its 5' end to its 3' end, antiparallel to its DNA
ATP, CTP®.
template strand.
3. Mg** or Mn**®
And then it recognizes the termination region (i.e., the end of DNA sequence
to be transcribed). Type
It is a multisubunit- enzyme, with -400 kDa core complex consisting of Only one type of RNAP synthetizes a ll o f RNAs (i.e.
two identical a subunits, similar but not identical 0 an d f j subunits and a Qfw m-RNA, t-RNA, & r-RNA) except f o r the short RNA
subunit (a(3p'o>). P subunit binds Mg++ and composes catalytic subunit. prim ers needed f o r DNA replication ®. RNA primers
It is multi subunit metalloenzyme containing 2 zinc molecules and are synthesized by p r im a s e enzyme.

Core- complex (E) Sigma ( a ) subunit


-Sigma factor binds tr a n s ie n t ly to core
It is ~ 400 kDa c o m p le x fo rm e d b y 5 u n its (g^ P P ' cd)
& directs the enzyme to specific
1
binding sites on DNA. It en ables
Coreenzyme a/ft subunit
RNAP to recognize p rom oter regions
- Consists of two identical a subunits, sim ilar but n ot Function of this 5th
on DNA® and to form the preinitiation
iden tical 0 a n d 0 subunits (ai00® , an d p ossess 5'—>3' subunit is unclear
complex (PIC).
polym erase activity®
-It decreases RNAP's affinity for non
-It lacks specificity and cannot recognize promoter region
promoter regions, while simultaneously
★ Termination of transcription requires termination factors, such p f a c t o r of E. coli. increasing holoenzyme affinity fo r
promoter DNA (dual role)
-It is responsible for p r o m o te r
r e c o g n i z a t io n s p e c if i c i t y of RNAP as
bacteria has multiple o factors
recognizing different genes, a 70 is the
most common type
-Sigma (a) unit plus the core enzyme
makes up the h o l o e n z y m e (cfcPP'cifl-a).

Eukaryotic (Mammation) RNA Polymerase (Nuclear DNA dependent)

- Eukaryotes have 3 ty p e s of RNA polymerases (RNAPs) with size ranging from 5 lack to 6 lack MW and all having 2 la r g e r a n d
m u ltip le (upto 14 in pol-III) s m a lle r subunits. However, these subunits do e x h ib it e x t e n s iv e a m in o a c id ( & 3 d im e n s io n a l
s tru c tu re ) h o m o lo g ie s with prokaryotic RNA polymerase.
- a- Amantin obtained from A m a n ita p h a llo id e s (d e a th c a p ) is a specific d if f e r e n t ia l i n h ib i t o r (i.e. differentiates different types)
of eukaryotic RNAP. It b locks tran slocation o f RNAP during p h osp h od iester bond form ation® . Different forms/ types of
eukaryotic RNAPs are
672 ■ A Complete Review of Short Subjects

RNAP-type a-Amantin Product


sensitivity Main Detailed
1/ P o ll Insensitive r-RNA Synthesize only one type of pre-r RNA (pre ribosomal RNA) in nucleolus
which forms 28S, 18S and 5.8S rRNAs
II (Pol II) High m-RNA, mi-RNA, Synthesize nuclear precursor of m-RNA and certain small non coding (nc)
sn RNA RNAs such as mi-RNA, sn RNA and sno RNA
III (Pol III) Intermediate t-RNA, 5S r RNA Forms small RNAs including t-RNA, 5S rRNA & some sn RNA & sno RNA
- Pol II, responsible for transcription and eukaryotic gene expression consists of 12 subunits. The two largest subunits RBP1 (190
KDa) and RBP 2 (150 kDa) are structurally similar (homologous) to prokaryotic p' and P subunits respectively. RBP 3 and 11
are homologous to prokaryotic a- subunits.
- Largest subunit has characteristic lo n g c a r b o x y ta il consisting of many r e p e a ts o f c o n s e n s u s 7 ( h e p te d ), a m in o a c id s e q u e n c e -
Y S P T S P S - (i.e . T y r o - S e r - P r o - T h r - S e r - P ro - S e r ). T h i s c a r b o x y te r m in a l r e p e a t d o m a in (C T D ) has 27 repeated units
(18 exactly matching the consensus) in yeast and 52 (21 e x a c t) in mammalian (mouse & human) pol II. The CTD is separated
from main enzyme by unstructured linker sequence.
- P h o s p h o r y la tio n o f S e r & T h r aminoacids of CTD a c tiv a te s and d e p h o s p h o r y la tio n of these residues d e a c tiv a te s pol II (i.e.
increasing & decreasing activity respectively). The phosphorylation & dephosphorylation is essential for promoter clearance,
elongation, termination and even appropriate RNA processing. So cells with pol II lacking CTD tail are incapable o f activating
transcription and hence are inviable.
- CTD is both a substrate for various kinases (including kinase ofT F IIH ) and binding site fo r many RNA processing enymes as well as
nuclear transport proteins - thus serves to couple transcription initiation with m-RNA splicing, 3' end formation and transport to
cytoplasm.
- Pol II can a/w M e d ia to r or M e d proteins to form p o l I I h o lo e n z y m e . Meds are essential for activating & repressing (i.e. regulating)
transcription. Since mediators (like T F I I D ) are required for actions of activators, they are also called c o a c tiv a to r s / c o r e g u la to r s .
Over 30 meds (Med 1- Med 31) are described in human cells.
★ M it o c h o n d r ia l RNA p o ly m e r a s e more closely resembles bacterial (than eukaryotic) enzyme and is single (1 in number).
* RNA p o ly m e r a s e le a v e s it s f o o t p r in ts o n a p ro m o te r. Foot printing analysis tells the precise location of protein, binding site
(like that for RNAP) in a DNA by showing missing bands (gaps i.e. foot prints) corresponding to that location. In this 2 identical
radioactively labeled DNA (one with and other without RNAP) are cut with DNase and separated by p o ly a c r y la m id e g e l
e le c t r o p h o r e s is (side by side) - the missing bands / gaps in DNA bound to RNA indicate the prescise location of protein
binding site.
RNA Iranscripl

Comparison Between DNA-Replication and Transcription


Similarities: 1) The general steps of initiation, elongation and termination occurs in 5'- 3' direction®
2) Both involve large, multi component initiation complex.
3) Adherence to Watson - Crick base pairing rule (i.e. Hydrogen bonding between A= T & between C = G)
DNA Replication Differences Transcription
In this process DNA molecule serves as a template fo r Definition In this process mRNA is synthesized from a
the formation o f daughter DNA molecule DNA template
Deoxyribonucleotides are used in DNA synthesis Nucleotides Ribonucleotides are used in RNA synthesis
— Bases 'U ' replaces 'T as the complementary base
pair for 'A'
Entire genome is copied. Copied Genome Only a small portion o f genome is transcribed.
Proof reading occurs. Proof Reading No proof reading
Present in DNA polymerase Endonuclease & Absent® in RNA polymerase
Exonuclease activity
Initiation of DNA synthesis requires Primer® [a Primer® N o t needed®
short length of RNA, about 10-200 nucleotides]
Biochemistry: Structure, Function and Replication of Informational Macromolecules ■ 673

S e m ic o n s e r v a t iv e R e p li c a t i o n : When the two strands of DNA double helix are separated, each can serve as template for
replication of new complementary strand producing two daughter molecule. This process is called semiconservative replication
because although the p aren tal duplex is sep a rated in tw o h alv es (& therefore n ot con served a s an entity), each o f in dividu al
p aren tal strands rem ains in tact in on e o f the tw o new duplexes®.

Transcription

DNA constitutes the genetic master plan of an organism, which is however, expressed through RNA - "the working copie of
DNA" - This copying process (i.e. synthesis of mRNA from DNA) in which a DNA strand serves as a template is called
t r a n s c r ip t io n . The 2 complementary DNA strands (of ds DNA) have different roles.

Template Strand Non-template or Coding Strand


- Template strand of DNA is t r a n s c r i b e d o r c o p ie d into - The opposite DNA strand (which is complementary to
an m-RNA molecule. In other words it i s u s e d a s a template strand) is called n o n t e m p l a t e s tr a n d .
t e m p la t e to m a k e t h e c o p y of that part of DNA (i.e. - Non template DNA strand is also called c o d i n g s tr a n d
gene). because w i t h t h e e x c e p t io n o f T f o r U c h a n g e s , it
- The in form ation o f tem p late stran d is read ou t in 3' to c o r r e s p o n d s e x a c t ly t o t h e s e q u e n e o f m R N A primary
5' direction®. transcript, which encodes the protein product of that gene.
- So t e m p la t e s t r a n d i s c o m p le m a n t a r y t o m R N A - So C o d i n g ( n o n - te m p la t e ) s t r a n d o f D N A is id e n t i c a l to
t r a n s c r ip t in nucleotide /base sequence. m R N A t r a n s c r ip t (e x c e p t T f o r U )

1----------------------------------------------
- This means that the information for protein to be produced is encoded in (or is identical to) coding /non template strand
of DNA (oriented 5' to 3'). But the opposite DNA strand acts as template for transcription and is read in 3' to 5' direction.
- In ds DNA containing many genes the template strand for each gene will not necessarily be the same strand. So any
strand can serve as a template strand fo r some genes and coding strand for other genes. So genes can be transcribed off both
strands of DNA.

Gene A Gene B Gene C Gene D DNA coding strand


(5') C G C T A T A G C G T T T (31) strand
DNA template
(3’) G C G A T A T C G C A A A (5') strand
- Template strands-

(51) C G C U A U A G C G U U U mRNA transcript


Arrow indicates direction (polarity) of transcription (3‘)

- R e g u la t o r y s e q u e n c e s that control transcription are designated by the sequences in the c o d i n g s tr a n d .

• T r a n s c r ip tio n u n it (TU) is the region of DNA that includes signals for all 3 phases of transcription i.e. i n i t ia t i o n , e lo n g a t i o n
& t e r m in a t io n . TU extends from promoter to termination region and the initial RNA product of RNA polymerase (RNAP) which
is synthesized in 5' to 3'direction is called p r im a r y tr a n s c r ip t.
• The s t r a n t i n g p o in t o f t r a n s c r ip t io n i n D N A ( u s u a lly ) c o r r e s p o n d s to the 5' nucleotide of m-RNA. The b a s e a t th e
t r a n s c r i p t i o n s t a r t s i t e is assigned + 1 ( f i r s t p o s it iv e ) p o s i t i o n . There is n o base designated " O " . By convention, DNA base
pairs that corresponds to the beginning of an RNA molecule are given positive numbers & those preceding the RNA start site
are given negative numbers. In other words, a base is assigned a n e g a t iv e n u m b e r if it occurs p r io r to ( to w a r d s t h e 5' e n d or
u p s t r e a m of or to the l e f t of) transcription start site (+1) and these bases are called 5' f l a n k i n g s e q u e n c e s . Whereas the bases
situated a f t e r / d o w n s t r e a m / t o w a r d s 3' e n d & r i g h t o f T S S (+ 1 ) are assigned p o s i t i v e numbers in increasing order (i.e. + 10
is more distal to +3). The sequenes down stream of transcription termination site are termed 3'- F l a n k i n g s e q u e n c e s .
Although all regulatory sequences function only in ds DNA, by convention, these elements (eg TATA box etc) are d e s c r ib e d
i n 5' to 3' d ir e c t io n a s b e in g o n c o d i n g s tr a n d .

1. Initiation

• Transcription begins with the a s s e m b ly / b in d in g o f R N A p o ly m e r a s e ( R N A P ) h o l o e n z y m e to (the template strand of)


p r o m o t e r r e g io n of DNA - forming a p r e - i n i t i a t i o n c o m p le x ( P IC ). The RNAP can bind with low affinity to many regions
of DNA, but instead it scans DNA at a rate of 1 0 3 bp /s to find a specific DNA region to which it binds with h i g h e r a f f i n i t y .
These higher affinity binding, non-transcribing zones are termed p r o m o t e r s and it is the binding of RNAP with promoters
that e n s u r e s a c c u r a te i n i t i a t i o n ( f i d e li t y ) of transcription.
674 ■ A Complete Review of Short Subjects

• Promoter contains characteristic consensus sequences (i.e. ideal sequences in which the base shown at each position is the
base most frequently but not necessarily always encountered at that position). Because these sequences are on the same
molecule of DNA as the gene being transcribed, they are called Cis-acling elements. No one consensus sequence (CS) is
found in all core promoters i.e. different bacteria /organism have slightly different CS, but all promoter have 2 common
features.____________________________________________________________________________________________________________
- These tend to be in the same position relative to transcription start site (+2).
- Sequences b/w 2 promoters have no similarity but still provide critical spacing functions that facilitate recognition.
• Promoter regions can be located upstream & downstream both, (eg it extends b/w - 70 and +30 in E.coli). Prokaryotic
promoters (-35 sequene, -10/pribnow box) are recognized by a (sigma) subunit of RNAP holoenzyme. Eukaryotic
promoters (TATA box, Inr, DPE etc) are recognized & assembled to RNAP by general transcription factors (GTF) like
transcription factor II (i.e. TF required by RNAP II) A to H (except C & G) Because these TFs are encdoded by different
genes, synthesized in cytosol, and must transit to their site of action - they are called trans acting factors.

2. Elongation
• Beginning of elongation phase is defined when,_______________________________________________________________________
- Primary transcript (characteristically starting with purine is > 10 nucleotides in length)
- Sigma (a) subunit is released and protein N usA binds to elongating RNAP competitively
- Core enzyme forming elongation complex is able to leave /clear the promoter sequence and move along the template
strand in a processive manner (i.e. promoter clearance)
• Prokaryotic RNAP has an intrinsic unwindase activity 1/ 1 formation of transcription bubble (DNA unwinding) of about 20
bp/ RN AP molecule which remains constant through out transcription (i.e. initiation & elongation). Its size is determined
by RN AP & is independent of D N A sequene.
• Elongation proceeds at a rate of 50-90 nucleotides per second. Because DNA is a helix, movement of transcription bubble
requires considerable strand rotations. So movement of RNAP along DNA tends to generate a positive superoil (i.e
overwound DNA) in front of transcription bubble and negative supercoil (underwound DNA) behind it. This topological
problem in cells is relieved by action of topoisomerases which rapidly eliminates positive supercoils & regulate the level of
negative supercoiling.
• So topoisomerases both preceeds & follow s progressing RN AP to prevent formation of superhelical tensions that would
increase the energy demand to unwind the DNA in front of RNAP.
• RNAP requires, D N A template, all 4 nucleotide 5' triphosphates (ATP, GTP, CTP and UTP), M g++ and Zn++for synthesis
(elongation) of RNA strand - by adding ribonucleotide units to 3' hydroxyl end, building RNA in 5'-»3' direction.
• Just like DNA replication, transcription (RNA elongation) is in 5' to 3' direction antiparallel to its template strand (which
is read & copied in 3' to 5' direction). But unlike i t , RNAP does not require primer for initiation & has no proof reading
activity
• Bases are added a/ 1 Watson-Crick base pairing rule and pyrophosphate (PPi) is released after each addition (except of 1st
initiating purine base, which remains intact throughout transcription). PPi is rapidly degraded to 2 mol of inorganic
phosphate (Pi) by ubiquitous pyrophosphatases, making over all reaction irreversible.
• Growing end of newly synthesized RNA strand base pairs temporarily with DNA template strand to form short 8 bp long
RN A -D N A double helix in the transcription bubble.
• RNA hybrid duplex peels off shortly after its formation & DNA duplex reforms.
• In eukaryotes, TF IIF remains associated with pol II throughout elongation. Elongation factors like Ell; pTEFb, SII (TF IIS)
and elongin (S III)- greatly enhance the activity of pol II. pTEFb phosphorylates pol II within CTD and suppress pausing.

3. Termination
• RNAP is highly processive (i.e. adds) great number of bases before dissociation)- necessarily so because if it gets detached
prematurely, it would have to start from beginning again. So elongation ofSS-RNA continues until termination signal is reached.
• 2 types of termination are
1. Spontaneous /Intrinsic Rho (p) independent termination.
• In mot prokaryotic genes a sequence in DNA template strand is spontaneously terminating which produces
- Nascent (newly formed) RNA transcript with self complementary sequences, permitting the RNA to fold back on itself
forming a hair pin structure centered 15 to 20 nucleotides before the projected end of RNA strand. This hair pin has G C
rich stem stabilized by hydrogen bonds and loop o f non complementary bases.
- Highy conserved string of 3' 'A' in template DNA is transcribed into a string of TJ's at the 3' end just beyond hair pin.
• RNAP pauses on reaching termination site with this structure. The bonding of string of Us to complementary As of DNA
template is weak. Dissociation of RNA transcription from DNA template in this area is facilitated by formation of hair pin
structure and zipping up of DNA double helix behind RNAP.
Biochemistry: Structure, Function and Replication of Informational Macromolecules ■ 675

2. Rho (p) dependent termination


- Termination site (in template strand of DNA) lacks repeated A string but usually includes CA-rich sequence k/a rut (rho
utilization) element
- This requies addition p protein that is a hexameric ATPase (adenosine triphosphatase) with helicase activity.
- Rho binds to C rich-p-recognition site near the 3' end of nascent RNA and migrates in 5' —>3' direction, along RNA using
its ATPase activity until it reaches the transcription complex (RNAP) that is paused at termination site.
- A TP dependent R N A -D N A helicase activity of p- protein separates the RNA-DNA hybrid helix-causing the release of
RNA. (this enz also promotes translocation along RNA-Lehninger.)
• On completion of transcription NusA dissociates from RNAP, which itself dissociates from DNA. The o- factor can now
bind to RNAP to initiate new transcription - in a cycle called sigma (a) cycle.
• Summary: Transcription initiates with binding of RN AP to promoter region of DNA (on template strand) to form PIC - a
process which is guided by a- subunit in prokaryotes & TFs in eukaryotes. This 1/t formation of closed (wound DNA) f/b
open complex (with unwound DNA) or transcription bubble. Nucleotides are added (i.e. RNA is synthesized) in 5'—»3'
direction antiparalled to temlate strand a/1 base pairing rule. In both prokaryotes & eukaryotes 1st nucleotide is usually
purine and its 5' triphosphate end is maintained (not degraded) in mRNA.

P ro k a ry o tic TU

R egu lated E xp ressio n B asal/C onstitutive - E x p ressio n -------


P rom oter Proxim al
D istal R eg u lato ry E lem en ts ^ Elem ent (50-200bp
« P ro m o ter
proxim al to prom oter)
( 10 3 - 1 ()5 bp fro m prom oter)

-2 5 / -3 2 -3 to +5 +25
£ )
TATA/ Inr D PE
H ogness B ox

'I I D - fll DA
r II D
kTBP/ XTAF>
cTAF/

Direction of transcription

-cmmr ^
Coding strand 5 '-
Temptale strand 3 -
u U U U U U U -3 '

L o c a l u n w in d in g o f D N A c a u se d by R N A p o ly m e ra se
676 ■ A Complete Review of Short Subjects

Prokaryotes Eukaryotes
Promoters 1. "-35 sequence": It is a consensus 1. Hogness/TATA-box (-25 or -32 sequence):
(Cis-acting sequence of 8 nucleotie pairs (5'- It is the major assembly point usually located 25 bp upstream from
element): TGTTG-ACA-3') centered TSS (+1). This region has a sequence of TATAAAG and the
Named & approximately 35 bp upstream (left) consensus sequene is 5' - TATA-AA-3', which is recognized by
positioned a/1 of transcription start site (+1). RNA T A T A - binding protein (TBP) subunit of TF IID It is functionally
location of binds & forms closed complex. similar to prokaryotic TATA box
5'to3' base 2. "-10 sequence" or Pribnow/TATA- 2. Initiator sequence (Inr): It spans the transcription start site (from-3
sequence in box: It is a 6 nucleotide pairs of A+T to +5) and consist of CS
coding/non­ rich consensus sequence (5'-TATA- "Y V A N T Y Y", where N is any nucleotide and
template AT-3') centered more proximally A Y is pyrimmidine (Lehninger)
strand about 10 bp upstream (left) to T5S T C A*i G T T , where A+i is first transcribed
(+1). Closed f/b open complex (k/a T C nucleotide. (Harper).
transcription bubble) is formed d/t
TSS is very near or usually within Inr hence forming open complex
melting /unwinding of short stretch
with unwound DNA. TBP associated factor (TAFs) subunit of
(-14 bases) of DNA-as TATA box
TFIID recognize & help binding it to pol II.
lack GC base pairs.
3. UP (Upstream promoter):
3. Downstream promoter element (DPE):
It is A T rich recognition element
It is located 25 bp downstream (+25) of TSS and has
occurring b/w -40 & -60 in highly CS - (5'-A G A CGTG - 3') which is recognized by TAF subunit of TF IID
expressed genes. It is bound by a- G T
subunit of RNAP.
Promoter - This cis acting upstream element 1. C A A T box:
proximal located proximal to promoter is not
It is located between 70 & 80 bp upstream (-70 to -80) of TSS (+1) and
elements found in prokaryotes
has CS (CCAATC) which is recognized & bound by trans acting
(located b/w - It long with (a/w) promoters is
transcription factors like CTF (als called C/E PB, NF-1, NFY)
50-200 bp required for basal (Constitutive)
2. G C box with CS (GGGCGG) recognized by Spl
proximal to expression of gene.
promoter) - It determines frequency of even and Element Consensus Sequence Activator/Factor that bind to
function in both direction. (CS) them
- PPEs bind to trans elements GC G,CG2 Spl
(proteins/activators) through D N A CAACTGAC MyoD
binding domains (DBD) and others IGGANsGCCAA NF-1
(eg TF, pol II, coregulators) through C C
activation domain (AD) API TGAGTCA Jun, Fos, ATF
C A
Ig Oc tamer A T G C A A A T Oct 1, 2, 4, 6
Heat Shock (NGAAN), HSF
Serum GATGCCCATA SRF
response
Distal - These are cis acting (i.e. located on same chromosome) DNA sequences that regulate (i.e. increase or
Regulatory decrease) the rate of initiation of transcription by Pot Ilin eukaryotes. These are not found in these
Elements - prokarvotes. These are usually located distallv (i.e. 103 to 105 from promoter site. However they can
Enhancer & 1. be located upstream (to 5' end) or down stream (to 3' end) of transcription start site and even within the
Repressor transcribed protein coding portions.
(Silencer)
2 . function in orientation independent manner (in both direction).
O nly found in
eukaryotes 3. occur on either strand of DNA.
4. exert effects when located close to or even tens o f thousands o f bp away from promoter/TU.
5. be of 2 types - enhancer (response elements) and repressor (silencer) that bind TFs to increase & decrese
transcription initiation respectively. So these are required for regulated expression.
- These enhancer / silencer - binding TFs can interact with other TFs bound to promoter with RNA pol II by
bending & looping the DNA.
Factors for - Sigma (a) subunit of prokaryotic RNAP is responsible for - Various general transcription factors
promoter promoter recognition specificity of enzyme and its binding to (GTFs) like transcription factor for pol II i.e.
specific promoter 1/t formation of PIC TF II from A to H (except C & G) are
binding required for PIC formation as RNAP II alone
Biochemistry: Structure, Function and Replication of Informational Macromolecules ■ 677

- Functions of different sigma subunits are cannot discriminate b/w promoter & other
<j70 House keeping regions of DNA. GTFs in order of binding
o54 Modulations of cellular nitrogen levels assembly (i.e. basal components of
ct38 Stationary phase gene transcription and their functions) are
ii
o32 Heat shock gene
o28 Chemotaxis & flagella genes
o24 Extracytoplasmic functions like heat shock functions
018 Extracytoplasmic functions like ferric citrate transport F

Component Function
s
TFII D -It consists of 1 TATA binding protein (TBP) & 14 TBP associated factors (TAFs)
- It is the only factor that is independently capable of recognizing & specific, high affinity
binding to (TATA box) promoter DNA through its TBP subunit.
-TBP binds to TATA box in minor groove of DNA (whereas most TFs bind in major groove)
causing around 100° kink/bend in DNA helix, which facilitates the interaction of TAFs with
other components of transcription initiation complex (TIC).
-Because of various TAFs, TF-IID is also an important component of TIC ofpol I and II, even iftliey
do not contain TATA box.
-All transcription factors (including TBP of IID) are basal components i.e. they support basal
transcription but not augmented /enhanced transcription. However, TAFs (of IID) are essential
for activator (like Sp 1 etc) enhanced /augmented transcription - and so are coactivatior
/coregulator: (Coregulators are required for action of activators and examples include - TAFs,
Meds, chromatin modifiers and remodelers)._________________________________________________
TFIIB It binds to TBP (of II D) and to DNA on either side of TBP (of II D) and recruits (+ binds) RNA
(subunits 1) polymerase II - TFIIF complex.________________________________________________________
TFIIA (3 - It stabilizes the TF II and TBP (II D) on promoter DNA, resulting in a ternary complex that is
sub units) more tightly bound & more precisely located on TIC. This TBP- promoter IIA-IIB complex then
attracts & tethers pol II- IIF complex.
- Its binding is not always essential but is very important at non consensus promoters where TBP
binding is relatively weak. 1IA binds prior to IIB
TF IIF (2 It binds tightly to pol II and targets/brings it to promoter both by binding with TF-IIB and by
sub units) reducing the binding of polymerase II to non specific sites on DNA.______________________________
TF IIE (2 Recruits TF II H and has ATPase & helicase activities
sub units)
TF II H (12 - After binding of IIH, closed complex of 60 bp (-30 to + 30) size is formed - as binding of each TF
subunits) has increased the size.
- Helicase activity of IIH melts /unwinds DNA (a process requiring ATP) near TSS there by
creating an open complex.
- Kinase activity of IIH phosphorylates pol II (within the CTD) allowing it to clear the
promoter.
- It recuruits nucleotide excision repair protein. So the repair of damaged DNA is more efficient
in template strand (than non template strand) of actively transcribing (than inactive genes.)
Promoter - Transcription machinery cannot access promoter of eukaryotic genes b/o their wrapping up in nucleosomes.
Acessibility Binding of TFs to enhancers causes recruitment of chromatin remodeling & modifying coregulatory factors
(like Swi/Snf, SRC-1, p300 /CBP or P/CAF) - ultimately 1/t nucleosome eviction (removal) and opening of
promoter to become accessible.
- Interconversion of hetero chromatin (i.e. highly condensed, most inactive segments of DNA) and
euchromatin (i.e. relatively relaved, highly transcribing genes) is k/a chromatin remodeling. Acetylation,
mediated by HATs (histone acetyl transferases) removes positive charge on lysine residues at amino terminal
of histone and thus decreases the interaction b/w histones and negatively charged DNA - ultimately
converting hetero chromatin to euchromatin and removing nucleosomes. HDACs (histone deacetylase)
restores positive charge building stronger interactions b/w histones & DNA thus forming nucleosomes.
- So this epigenetic code of histone - protein modifications can increase or decrease gene transcription.
Mutations in code writers (proteins that catalyze) or code readers (proteins that differentially bind) modified
histones 1/1 diseases.______________________________________________________________________________
678 ■ A Complete Review of Short Subjects

Micro- RNA (mi-RNA), si RNAs, st RNAs, sRNAs & RNA interference


• Micro-RNAs are non coding (nc RNA i.e. they do not
encode protein) RNAs about 22 (21-25) nucleotide long,
found in multicelluar eukaryotes (eg worms, flies, plants &
mammals).
- mi- RNA gene w ith p rom o ter, c o d in g & term in atio n sig n als
• Upto 1 % o f human genome may encode mi RNAs & mi
RNAs may target upto l/3 rd o f human m-RNAs
RNA polymerase II (Pol II)
• mi - RNAs are synthesized from much larger precursors
in several steps. The transcription units of pri-mi RNAs
are located either in introns (intronic DNA) o f other genes or I
(and coexpressed with) host genome. Therefore mi RNA
encoding genes must possess a distinct promotor, coding
region, and polyadenylation / termination signals.
• Most mi RNAs are transcribed by RNA polymerase II
into primary transcripts (pri-mi RNAs) which is 5' - 5 ’ C ap
3 (A )A n
capped & 3' - polyadenylated. pri-mi RNA is 500-1000
P re cu rso r mi RNA (pre-
P rim ary tra n scrip t o r pri-m i RNA
nucleotide long single stranded RNA with extensive 5 ’ - c a p p e d & 3 '- p o l y a d e n y l a t e d , 5 0 0 -
mi RN A )
I n t r a m o le c u la r 2 ° (hairpin)
intramolecular 2 ° (double stranded hair pin) structure, 1 0 0 0 n u c l e o t id e s ( l e n g t h ) , s in g l e
structure is preserved. B u t
s tr a n d e d w ith e x t e n s i v e i n t r a m o le c u la r
which is maintained following processing by Drosha - s e c o n d a r y ( h a ir p in ) s tr u c tu r e
D ro sh a D G C R 8 n u cle a se
tr im s b o t h 5 ’ & 3 ’ en d
D G C R 8 nuclease. A nuclear RN ase for double stranded g e n e r a t i n g 70-80

(d/1 2° structure) RNA called Drosha acts with a nuclear nucleaotide pre- miRNA

double stranded RNA binding protein called D G C R 8 in A s y m m e t r ic a l

humans (Pasha in drosophila) and cleaves the hair pin u n w in d i n g & o n e o f


th e tw o r e s u lt i n g 2 1 -
region out of long precursor RNA generating a 70-80 2 2 n u c l e a t id e lo n g D i c i n g b y m u lti s u b u n it
s tr a n d is s e l e c t e d
nucletide long pre- mi RNA
• pre - mi RNA is exported through nuclear pore by J
exportin 5 into cytoplasm, where dicer nuclease (ds RNA M i-R N A duplex
( n e a r ly m a tu r e m i R N A
specific RNA ase called Dicer act with cytoplasmic ds p a ir e d w ith a s h o r t

RNA binding protein called TRBP) in humans to form mi- S e le c te d strand (m i R N A ) is R N A c o m p le m e n t )

in corp orated in to protein


RNA duplex. Ultimately one of the 2 strands (21-22 co m p le x e s su ch as R N A indu ced
nucleotide long) is selected for loading into RISC or RNA sile n cin g co m p le x (R I S C )

induced silencing complex to form a mature functional mi


RNA. H y b rid ize via
fo rm a tio n o f
• mi RNA is complementary in sequence to particular regions imperfect RNA
of m RNA. So mi RNAs typically hybridize, via the RNA duplexes
w ithin 3 ’
formation of imperfect RN A-RN A duplexes within the 3' un translated
untranslated regions (3' UTRs) of specific target m-R NAs reg io n s ( 3 ’ U T R )
o f target m R N A
leading to either translational arrest (near perfect match)
or mRNA degradation (partal match). In either case, the Near perfect
mRNA and thus the gene that produces it is silenced. complementarity Partial Complementarity
(b/w mi R N A & targ et
• Micro (mi) RNA are non coding regulatory RNAs that mRNA) |
play im portan t role in gene expression 0. These inhibit Translational repression
m- RNA is cleaved
gene expression by RISC mediated gene silencing. It is (b lo c k s tran slation o f targ et m
R N A )___________________________
used as a mechanism to protect against invading RNA
viruses (esp important in plants which lack an immune system) & to control the activity of transposons & developmental
timing.
• Many mi RNAs are present only transiently during development, & are referred to as small temporal RN AS (st RNAs).
• If an investigator introduces into an organism a duplex RNA molecule corresponding in sequence to virtually any mRNA,
Dicer nuclease cleaves the duplex into short segments, called small interfering RN As (siRNAs). These short (21-25
nucleotide long) si RNAs usually form perfect RN A-RNA hybrid with their targets potentially anywhere within the length
of m RNA where the complementary sequence exists. Formation of such RNA-RNA duplexes between si RNAs and mRNA
results in reduced specific protein production because the si R N A - m RNA complexes are degraded, mostly in specific
cytoplasmic organelles termed P- bodies. The process is k/ a RN A interference R N A i for which Nobel prize was given to
Andrew Fire & Craig Mello in 2006.
Biochemistry: Structure, Function and Replication of Informational Macromolecules ■ 679

RN Ai is activated when RNA molecule occurs in double stranded pairs in cell. Ds RNA activates biochemical machinery
which degrades those mRNA that carry a genetic code identical to that of ds RNA. When such m RNA molecules disappear,
the corresponding gene is silenced and no protein of encoded type is made.
So RN A i is extremely useful research tool as it allows genetic researchers to knock out specific genes, observe the
consequent disruptions, and so determine exactly what the gene does. It has already revealed genes responsible for diabetes
and muscle problems. In lab it has been used to block HIV and polio virus infections.
Nearly all mi RN As and si RN As cause inhibition of gene expression. Bacteria also contain small hetrogenous regulatory
RNAs called sRNAs. Bacterial sRNAs range in size from 50-500 nucleotides, and like eukaryotic mi RNA / si RNA also
control a large array of genes. Similary sRNAS often repress, but some times activate protein synthesis by binding to specific
m RNA
Mi RNAs are considered to be endogenously expressed counterparts o f Si RNAs

Mechanism &Types of Splicing


There are 4 classes of introns
1 & 2 Self Splicing Introns (SSI)
5' S plice (D onor) sll< 3' S plice (A ccep to r) s ite (3' e n d o f Intron)
- It function as ribozym e, does not require protin enzymes and high energy (S' end o f In tro n ),

cofactors (such as ATP) for splicing


- Splicing mechanism involves two transesterification reactions in which a G pU r

new phosphodiester bond (5' - 3' PDB in group I and 5'-2' PDB in group II) is
pG - 3 ’(OH) o f G
form ed at the expense o f old bond maintaining the balance of energy. These (gua n o sin e . GMP, GDP.
GTP) n e u cleo p h ile a llcks
reactions are very similar to DNA breaking and rejoining reactions p h o sp h a te a t 5 ’ sp lice she
loim m g norm al 3'-5' PDB
promoted by topoisomerase and site specific recombinases.
Self splicing introns may be of 2 types
Group I SSI Group II SSI d U-3'(QH) G pU c

Found in some nuclear, mitochondrial Found in primary transcript of


and chloroplast genes that code for r mitochondrial or chloroplast m Newly Ireed 3' (OH ) o f S' exon
a tta c k s 5' p h o s p h a te at splice
RNAs, mRNAs & t-RNAs; and some RNAs in fungi, algae & plants; and a c c e p to r site form ing 3 -5'
PDB, resulting in excision of 5’ p G p A ^ . |n tr o n
G -(O H )3'
introns in bacteria some introns in bacteria introns & ligation of ex o n s

-Nucleophile in first transesterification - 2' hydroxyl group of adenosine U pU c

reaction is 3'-hydroxyl group of (A) residue (known as branch


guanine nucleoside or nucleotide site) in intron is nucleophile.
G ro u p I S S I
(guanosine, GMP, G D T, or GTP) - 2 '—»5' phosphodiester bond and
-3' —> 5' phosphodiester bond is formed branched lariat (loop) like
but lariat (loop) like intermediate is intermediate structure is formed.
Intron
not formed The excised intron is released as a
lariat, in which adenosine has 3
phosphodiester bonds. Lariat is
degraded.______________________
2 Transeterifications reactions include 2 Transesterification reactions are
2' (OH) of A in branch point
-3' (OH) of G (guanosine, GM P, GDP, -21 (OH) group of A (adenosine) intron attacks phosphate at
GTP cofactors) is used for nucleophile in the branch point sequence of 5' splice site forming
u n u sual 2 ’—►S' PDB &
attack (not energy). It attacks the intron act as nucleophile, la riat s tru c tu re o f in tro n
phosphate at 5' splice site (junction of attacking the phosphate at 5'
5' exon & intron = 5' end of intron = splice site forming unusual 2' 3 U-3'(OH)

splice donor site) form ing a normal 5' p h osp h od iester bon d an d
3'- 5' phosphodiester bond with 5 1 creating a la ria t (loop) like Newly freed 3' (OH) o f 5' exon
attacks 5' phosphate at splice
end of intron. structure<2. acceptor site forming 3'-5'
-3 ' (OH) of 5' exon that is displaced in -Newly freed 3' (OH) of 5' exon PDB, resulting in excision of
introns & ligation of exons
this step then acts as nuceophile and attacks 5' phosphate at splice
triggers 2 nd transesterification reaction acceptor site (3' end of intron or 5'c 2 UpU c

by attacking 5' phosphate at splice 3' splice site) forming 3' —» 5'
G rou p I I S S I
acceptor site (3' end of intron or 3' phospodiester bond, resulting in
splice site) forming 3'—>5' excision of introns & ligation of
phosphodiester bond. This results in exons.
precise excision of introns & ligation of
exons.
680 ■ A Complete Review of Short Subjects

3. Splicesom al Introns i
5' — |~5‘ Exon \ Py N Py Py Pu APy A G | J E xo n 1— 3'
Branch site consensus
• Most introns are not self splicing. This largest class of introns includes those sequence 20-40 bp
upstream of 3' en d in
found in nuclear mRNA primary transcripts Intron

•These are called splicesomal introns, because their removal occurs within and is 5' Splice (Donor) Site * Py = Pyrimidine • 3* Splice (Acceptor) Site
• S 'E a d o flo tr a n Pu = Purine - 3' End of Intron
catalyzed by large protein complex called splicesom e. Within the splicesome, the • J im c tlo a o f 5 ' ( L e fto r N = Any nucleotide - Junction of 3* (Right or
D o a o r) E io o -a o d l o lio n Acceptor) Exon and Intron
introns undergo splicing by same la ria t (loop) fo rm in g m echanism as group
I I intronsQ.
• Splicesomal introns have dinucleotide sequence GU at 5' end and A G at 3' | 5' Exon | G -
end and these mark the site where splicing occurs. The GU and AG N ucleophile Attack o f A
at S' end o f Intron result In
sequences at the beginning and end respectively, of introns are invariant. Lariat (loop) form ation

• U1 sn RN A contains a sequence complementary to sequences near the 5'


G
splice site of nuclear m RNA introns; and U1 binds to this region of primary 15 E«on ha(OH) — ag|3-Enon |
transcript. U2 is paired to intron at a position encompassing the A residue that
Newly freed 3' (OH) of 5' exon
become nucleophile. This adenyl residue in branch point sequence is typically attacks (S') p h o sp h ale a t splice
ac ceptor site forming 3'-S' PDB
located 20-40 nucleotides upstream from 3' end of intron being removed. r/i excision of lanal intron &
ligation of exons.
The branch site identifies the 3' splice site. Base pairing of U2 causes a bulge
that displaces and activate adenylate (of branch site in intron), whose 2' 5‘ C ap — | 5' Exon | 3' E xon~|— 3' Tail ^ • Lariat (loop)

(OH) w ill form the lariat structure through a 2'- 5' phosphodiester bond.
• After Ui and U2, the remaining sn RNPs (U4 - U 6 complex and U5) bind to form an inactive spliceosome. Internal
rearrangements activate complex in which Ui & U4 have been expelled and U 6 is paired with both U2 and 5' splice site. This
is followed by catalytic steps similar to and group II SSI. A TP is required for the assembly of spliceosome, but not for RNA
cleavage - ligation
• Some less common spliceosomes contain U 11 and U 12 snRNPs in place of U 1 and U2. Whereas U I and U2 containing
spliceosome remove introns with (5') GU and A G (3') terminal sequences, the U 11 and U12 containing spliceosome remove
rare class of introns that have (5') AU and A C (3') terminal sequence to mark the intrionic splice site.
• Splicing apparatus is tethered to CTD o f RNA polymerase III coordinating splicing & transcription and providing a
mechanism for bringing 2 splice sites together. After splicing introns (lariat) remains in nucleus and is degraded.

4. Class 4 Introns are found in t RN As


• Distinguished from group I and group II introns in that the splicing reaction requires A TP and endonuclease
• Splicing endonuclease (ribonuclease) cleaves the phosphodiester bond at both ends of introns and two exons are joined
by a mechanism similar to DNA ligase reaction.
Splicing and sn RNAs sno RN As and r RN A modification
Uracil rich Small nuclear RNAs (sn RNAs) are non - Small nucleolar RNAs (sno RNAs) are nc RNAs that guide
coding RN As that play pivotal role in RNA nucleoside modification and cleavage reactions in pre- rRNAs.
processin g an d splicingQby forming small nuclear - In yeast about 70 and in humans > 200 sno RNAs are found
ribonucleo protein particles (SnRNPs or - sno RNAs are 60-300 nucleotides long and many are encoded with in
"snurps") the introns o f other genes.
SnRNAs vary in size from 90-300 Nucleotides. Some
- Each sno RNA includes a 10-21 nucleotide sequence that is perfectly
complementary to some site in rRNA. The conserved sequence elements
Name Length Localization Molecules
in the remainder of sno RNA fold into structures that are bound by
(nucleotides) per cell
sno RNP proteins.
UI 16 5 Nucleoplasm lx lO 6 - The most common nucleoside modification in eukaryotic r RNAs are,
U2 188 Nucleoplasm 5 x l0 5 again, conversion of uridine to pseudouridine and ado met
U3 216 Nucleolus dependent nucleoside methylation (often at 2' -O H groups). These
3 x l0 5
reactions rely on sno-RNA protein complexes or sno RNPs, each
U4 139 Nucleoplasm 1x10 s consisting of a sno RNA & 4-5 proteins
U5 118 Nucleoplasm 2 x l0 5 - There are 2 classes of sno RNPs, both defined by Key conserved
U6 106 Perichromatin sequence elements referred to as lettered boxes.
3 x l0 5
granules 1. Box C/D sno RNPs function in 2 ' - O - methylations
4.55 95 Nucleus & 3 x l0 5 2. Box H/ACA sno RNPs are involved in pseudouridylylation
cytoplasm
7 SK 280 Nucleus & 5x10 s
cytoplasm
Biochemistry: Structure, Function and Replication of Informational Macromolecules ■ 681

- Besides its role in splicing, the sn R N A U1 directly Splicing of Primary Transcript


binds to transcription factor TF IIH and may
• Primary transcript contains 2 types of sequences
regulate TF IIH or affect the coupling behvcen
transcription & splicing.
• U], U2, U4, U5 & U6 - SnRNAs are involved in Introns (Intervening) sequence Exon (Expressed) sequence that
intron removal and processing of mRNA that do not code the genetic contain genetic information &
precursors into mRNA. U7 - snRNA is involved in information code for various amino acids
p ro d u ctio n o f c o rre ct 3 ' e n d s o f h is to n e m R N A ,
which lacks a poly (A) tail. • It undergoes extensive processing (RNA splicing) to convert into
functional RNA which involves cleaving out of introns and splicing
- A 331 nucleotide nc RNA called 7 S K , abundant in
together of exons, facilitated by Sn RNP or snurp of spliceosome.
mammals, binds to pol II transcription elongation
This process occurs in nucleus and mature m RNA is then
factor P-TEFb (that modulates mRNA gene
transported to cytoplasm where it is translated into protein.
transcription elongation by RNA polymerase II)
• The process of removing introns (RNA sequences which do not code for
and repress transcriptase elongation.
proteins) and joining exons (RNA sequences that code for proteins) is called
- In S L E , autoantibodies develop against their own ‘Splicing'.
nuclear proteins like SnRNPs. • The process o f splicing is m ediated by Small nuclear
ribonucleoprotein particles (sn RNP) or 'Snurps' which are form ed
S p lic e S ite M u ta tio n & A lte r n a tiv e S p lic in g from sn RNA
• The molecular machinery that accomplishes splicing is called a
- Splice site mutation can 1/1 improper splicing &
spliceosom e which is formed when the primary transcript combines with
production of aberrant proteins. 15% of all generic
the Sn RNP. Spliceosome consists of primary transcript, fiv e (Sn­
disease are caused b/o fa c u lty s p lic in g and
RNA) sm all nuclear RNAs (Ul, U2, U4, U5, U6) and > 60 proteins
example includes beta thalassemia (with defective
collectively these form a small nuclear ribo nucleoprotein (Sn RNP)
beta -globin protein)
complex or snurp.
result
- A lte ra n tiv e p a tte rn s o f m R N A s p lic in g • The binding ofsn RNP brings the sequences of neighboring exons into
from tissue specific developmental & adaptive control correct alignment for splicing.________________________________
mechanism (such as chromatin epigenetic markers). u, 1st binds to 5' exon-interon boundary
This produces diverse set of proteins from a limited set
u2 Binds to branch site and this expose the nucleophilic A
of genes and example includes multiple isoforms of
residue
tropomyosin protein.
u y iv u * Mediate an ATP dependent protein mediated
- Tissue specific regulation of gene expression can unwinding that results in disruption of the base paired
be provided by alternative splicing by control U4-U6 complex with the release of U4. U6 is then able to
elements in promoter or by use of alternative interact first with U2, then with Ul. This approximate
promoters. Example, in s u lin & c A M P c o n tro l the 5' splice end, the branch point with its reactive A,
g lu c o k in a s e (G K ) g e n e tra n s c rip tio n in liv e r and 3' splice site. This alignment is enhanced by U5.
w h ile g lu c o se c o n tro ls G K e x p r e s s io n in This process also results in formation of loop or lariat
p a n cre a tic b e ta -c e lls . structure.
U2-U6 Two ends are cleaved, by U2-U6 with in the SnRNP
complex.
U6 is essential since yeast deficient in this are not viable.

Primary Transcript and Post Transcriptional Modification of hn-RNA


The RNA formed and released from DNA template is known as primary transcript. The primary transcript of m-RNA is hn-
RNA (and of mi-RNA is pri-miRNA).
A primary transcript is a linear, RNA copy of a trancriptional unit - i.e., segment of DNA between initiation & termination
sequence. The primary transcripts of both prokaryotic & eukaryotic t-RNA and r-RNA as well as eukaryotic m-RNA, 55
RNA, Sn RNAs, RNA processing mechinary are inactive and undergoes modifications for activation. In contrast, prokaryotic
mRNA is generally identical to its primary transcriptP.
Nearly all eukaryotic & prokaryotic RNA (except prokaryotic m-RNA) are extensively processed primariy with in the
nucleus (for eukaryotes) before they become functionalQ. The processes of transcription, RNA processing & RNA transport
from nucleus are highly co-ordinated by - SAGA (in yeast) & P/CAF (in humans) and TREX (transcription export).
Transcriptional co-activator (SAGA or P/CAF) link transcription activation to RNA processing by recruiting TREX that links
transcription elongation complex, RNA splicing machinery and nuclear export.
682 ■ A Complete Review of Short Subjects

Modification of Eukaryotic mRNA Modification of Transfer


(t) RNA
- All primary transcript synthesized in nucleus by RNA polymerase II is k/ a heterogenous Both eukaryotic &
nuclear (hn) RNA. This pre-mRNA (primary transcript) the immediate product of gene prokaryotic t-RNA precursor
transcription, is very heterogenous and can be > 10-50 times longer than mature m-RNA. molecules are modified by
This hn RNA is very unstable, exists as hn RNP (ribonucleoprotein) and must undego
- Removal o f introns from
post transcriptional modification (processing) to appear in cytoplasm as m-RNA.
anticodon loop
- Addition of 7-methylguanosine cap at 5' terminal, poly (A) tail at 3' terminal; secondary
methylation of 2' hydroxy and N7 adenyl residues and splicing (introns removed & - Trimming of sequences
exons joined) converts hn-RNA to m-RNA. both at 5' and 3' end. 16
nucleotide sequence at 5'
• Capping end is cleaved by RNase P
- It is the 1st processing reaction for hn-RNA occuring in (a ribozyme).
the nucleus 0 .
- R eplacem en t o f uracil
- Cap structure (7 m ethyl guanosine) is attach ed
residues a t 3' end by CCA
b ackw ard s to 5' end o fh n RNA® forming unusual 5'
7 M e th y l
sequence®, catalyzed by
—>5' triphosphate linkage.
G u anosyl C A P nucleotidyl transferase. 3' OH
- Nuclear guanylyl transferease enzyme is required for
of A ribose is point of
creation of GTP (guanosine triphosphate) part of cap.
attachment
- Secondary methylation of 2' hydroxy (of 1st & 2nd
nucleotide) & N 7 of adenylyl residues of terminal - Many bases are converted
guanine occurs in cytosol®, and is catalyzed by to modified unusual bases
5 '- 5 ' T r ip h o s p h a te
guanine-7 methyl transferase. S- adenosylmethionine is lin k a g e including D ihydrou racil
the source of methyl group. (D), pseu do uracil ( <F) and
- 5 ' capping is required f o r recogn ization o f mRNA by m eth y lated b ases (m)® by
tran slation m achinery, efficien t tran slation in itiation - 1st n u cleotid e methylation, reduction,
o f mRNA
an d protection o f 5' end o f m-RNA fro m a tta c k by 5' O -C H deamination, rearranging
—>3'exonuclease®. Cap fa c ilita te s binding o f m-RNA glycosidic bonds &
to 43 S in itiation complex® (Harper) /4 0 S ribosom e nucleotide alkylation
unit® (Lippincot). - M eth y lation occurs in
- The guanine of cap is methylated at N7 position, and nucleus w h ereas cleav ag e &
1st & 2nd nucleotides adjacent to cap (of m RNA) are attach m en t of CCA are
methylated at 2 1 -O H position. So the 7-methyl cytoplasmic functions.
guanosine triphosphate cap is linked to 5' end (at 5'-
Modification of
hydroxyl position of 2' - O - methyl ribonucleoside)
Ribosomal (r) RNA
through three phosphate (i.e. triphosphate linkage).
Both prokaryotic (5S, 16S &
• Addition of poly (A) tail at 3' end 23S) & eukaryotic (5.8S, 18S,
& 28S) rRNA are
- The m-RNA is cleaved ab o u t 20 nucleotides dow n stream o f a consensus recognition synthesized from long
sequence, ca lled the p o ly aden ylation sign al sequence (AAUAAA)® found near 3' end. precursor molecule
- Poly (A) tail is added to new 3' end by nuclear enzym e p oly ad en y la te polym erase® , •All ribosomal (r) RNA
using ATP as the substrate. This tail is not transcribed from DNA and contain 40-200 molecules except 5S r-RNA,
adenine nucleotide. are processed from a single
- Tail helps stabilize the m-RNA, facilitate their exit from nucleus and protects 3' end from 45 S precursor RNA
attack of 3' —» 5'exonucleaseQ. molecule in nucleolus
- The presence of poly (A) tail does not necessarily means that the precurssor mRNA will • E u karyotic 5S rRNA is
appear in cytoplasm, becau se a ll p o ly (A) taild hnRNA d o n ot contribute to hn-RNA®. sy n th esized by RNA
Similarly n ot a ll cyto p la sm ic m-RNA h av e p o ly A tails (although m o st have)®; m-RNA p oly m erase II & m od ified
coding fo r histones & some interferons do not have poly (A) tails seperately®.
- Poly (A) tail is gradually shortened as m-RNA enters cytoplasm. The r-RNA genes are
lo ca ted in n u cleoli (in
• Removal of introns & joining of exons (splicing) eukaryotes)®. Because of
lack o f amplification, large
- Maturation of m-RNA involves rem oval o f non- coding (introns o r intervening number of genes and high
sequences®) from primary transcript and join in g o f rem aining coding sequenes (or transcription rate is
exons)® required to synthesize
Biochemistry: Structure, Function and Replication of Informational Macromolecules ■ 683

- The process of removing introns & joining exons is called splicing and the molecular sufficient copies of rRNA to
machine accomplishing the task is k/a spliceosom e. The splicing occurs in nucleus form the 107 ribosomes
&serves to bring the actual aminoacid coding sequences (exons) together. required for each cell
- Primary tanscript from histone genes contain no interons replication.
- A hn RNA can be spliced in >2 ways to produce multiple variations of m-RNA & - The precursor (pre
therefore proteins. ribosome RNA) are cleaved
in nucleolus by
• RNA Editing r i b o n u c l e a s e s to yield
- It changes the coding information at m-RNA level, so that coding sequence of mRNA diffes intermediate size pieces of
from that of cognate DNA (exception to central dogma) RNA, which are further
- APo B gene transcribes 100 kDa apo B-100 protein in liver and 48 kDa apo B-48 protein in trimmed to produce
intestine; because cytidine deam inase converts a CAA codon in m-RNA to UAA at single required rRNA
specific site. So rather than coding glutamine this becomes termination codon.
- In parasitic protozoa (trypanosom es) large regions of m-RNA are synthesized without
uridylate and U residues are inserted later by RNA editing.
5' c ap p in g
5' 5' Poly (A) tail at 3‘ end
7 m e th y l G T P c a p w ith
AUG AUG 5 '- * 5' trip h o s p h a te AAUAA M AAAAAAAAA AA OH-j
lin ka g e

(Gin) i = = > | UAA | (Stop codon)


1 CAAI
RNA Editing
Cytidine
UUA Deamination UUA

Translation

V V .
I Apo B-100 I Apo B-48

Liver (VLDL) Intestine (Chylomicron)

Ribozymes

Ribozymes are R N A m o l e c u l e s w i t h e n z y m a t ic / c a ta ly tic a c t iv it y . Examples include p e p t id y l tra n s fe r a s e Q , s e l f s p l i c i n g


g ro u p I in tro n s , R N a s e P , and h a m m e r h e a d r i b o z y m e . Most of the activities of these ribozymes are based on 2 fundamental
reactions: t r a n s e s t e r i f ic a t i o n and p h o s p h o d i e s t e r b o n d h y d r o ly s is ( c le a v a g e ). The substrate for ribozyme is often an RNA
molecule and it may even be part of ribozyme itself; so most ribozyme are concerned with RNA metabolism ( s p l i c in g &
e n d o r ib o n u c le a s e ) . Recently a r-RNA component has been implicated in hydrolyzing an aminoacylester & p la y s a cen tral ro le
in p ep tid e bon d fo rm a tio n (peptidyl transferases)Q.
Ribozymes vary greatly in size (40 nucleotide hammer head to 400 nucleotide SS group I intron). 3-dimensional structure is
important for function. Ribozymes are inactivated by melting, or by addition of denaturing agents or complimentary
oligonucleotides which disrupt normal base pairing patterns or by changing essential nucleotides.
Certain virusoids (virus like elements) have small RNA genomes and usually require other virus to assist in their replication
and /or packaging. H a m m e r h e a d r i b o z y m e s are small segments of virusoidal RNAs, that promote site specific RNA cleavage
reactions associated with replication. Hammerhead ribozyme is a metalloenzyme requiring Mg++ ions for activity and their
secondary structures is shaped like a head of hammer.
S i d n e y A ltm a n & T h o m a s C z e c h were awarded Nobel Prize (in 1989, Chemestry) for discovering catalytic property of
RNAs. And this discovery changed the central dogma (DNA—»RNA—>protein) that RNAs are not just the transmitters of
genetic information but can also act as enzyme.
684 ■ A Complete Review of Short Subjects

Inhibitors of
Transcription Translation (protein synthesis)

Refamycin, Binds with 0-subunit o f polymerase to Streptomycin - Interferes with binding of f-met t
Streptovaricin block initiation o f transcription (highly basic RNA to ribosomes & thus inhibiting
Actinomycin D Forms a complex with double stranded trisaccharide) initiation process
DNA & prevents movement of core - Leads to misreding of m- RNA
enzyme & thus in hibiting p rocess o f Puromycin Release nascent polypeptide chains
chain elongation® before their synthesis is complete. It
Streptoglydigin Binds with (3-subunit of prokaryotic binds to A site on ribosome and
polymerase & thus inhibiting elongation inhibits the entry of aminocyl t-RNA
Heparin Binds to (3-subunit & inhibits (both in bactarial & mammalian cells).
transcription in vitro Tetracycline Binds to 30 S subunit & inhibits
a-Am anitin - B lo cks m-RNA fo rm a tio n by binding of aminoacyl t-RNA & so
(Death cap, in hibiting RNA p oly m erase II® inhibiting initiation process.
destroying - Blocks translocation of RNA Chloramphenicol Inhibits the peptidyl transferase
angel) polymerase during transcription. activity of 50s subunit, thus inhibiting
D N A replication process of elongation.
Cyclohexim ide - Inhibits peptidyl transferase activity
Cytarabine - Cell cycle S phase specific
of 60S ribosomal subunit in
(cytosine antimetabolite that becomes
eukaryotes
arabinoside) phosphorylated intracellularly to an
- Also inhibits elongation.
active triphosphate form which
Erythromycin Binds to 50s subunit & inhibits
in hibits DNA p oly m erase an d b locks
translocation
DNA syn thesis by in corporating into
Diptheria toxin Binds EF-2 in eukaryates & blocks its
DNA®
capacity to carryout translocation
- 1- f} - D-arabinofuranyl cytosine-
Ricin & abrin - Prevent aminoacyl t-RNA from
triphosphate is active metabolite
(lectins) binding to A site
Anthracyclins DNA inter calaters, inhibiting
- Inactivates eukaryotic 28 s ribosomal
(eg topoisomerase H, leading to DNA breaks
RNA
daxaorubicin)
Sparcomycin - Inhibits peptidyl transferase &
Etopaside - Arrest cell growth in G 2 phase
release factor dependent termination
- Causes DNA breaks by affecting
Cc - Sarcin (toxic - Prevents aminoacyl t- RNA binding
DNA topoisomerase II function
RNA ase) by cleaving a single phosphodiester
Nalidixic acid Inhibits bacterial DNA gyrase
bond in 28 S r- RNA
Ciprofloxacin Inhibits bacterial DNA gyrase, which
★ Paclitaxel enhances polymerization o f tubulin; a mechanism
nicks the double stranded DNA,
opposite to that of vinca alkaloids
induces negative supercoil and reseals
★ Actinom ycin D (dactinomycin) intercalates into the narrow
the nicked ends
groove of DNA double helix, thus interfering w ith DNA and
RNA synthesis®.
Site of am ino a d d
attac h m e n t

RNA structure
I
consists of

c o n sistin g o f - A d en in e
Ribose sugar Phosphate in diester _____________ » - G u a n in e
Bases
- C y to sin e
_______linkage_______
- U racil

Producing
I Anticodon loop

Ribonucleotide
i Anticodon

Polymers of which form C h a ra c te ristic tR N A stru c tu re


Biochemistry: Structure, Function and Replication of Informational Macromolecules ■ 685

I— I___________
___________ rRNA____________ _________________ tRNA_________________ _________ mRNA_________
-80% o f RNA in cell® -15% of total RNA -5% of total RNA
- Associates with protein - Smallest of the three species of RNA - Most heterogenous type
- Four size species in eukaryotes -C on tain s unusual b ases (pseudouridine, - 5' 'Cap' of 7- methyl
(285,185,5.85 and 5S) thymine)® guanosine
- Sructural component o f ribosomes - E xtensive in trachain b ase pairing® -3' Poly- A 'tail'
-Serve as site f o r protein - C lov er le a f pattern® -Used as tem plate f o r
synthesis®. - At least on e sp ecific type o f m olecu le f o r protein synthesis®. It
['S' - Svedberg unit, related to each o f 20 am ino acid s fo u n d in portein®. carries genetic information
molecular wt & shape of - Serve as adaptor molecule that carries a from DNA to cytoplasm.
compound]___________________ specific a.a. to ribosome / mRNA complex

F o ld e r tR N A s tru c tu re fo u n d in cells

Translation (Protein Synthesis) ] 2 :


The tran slation o f mRNA com m ences n ear its 5' term inal. The m essage is read fr o m 5 ' to 3', concluding w ith the fo r m a tio n o f
ca rb ox y l term in al o f the protein®. Occurs in three phases: Initiation, Elongation & Termination

Initiation Sequence Initiation Codon


Although amino acid methionine (Met) has only one codon AUG, all organism
Shine Dalgamo Kozak
have 2 types of t RNA for methionine tR N A fMet incorporating N-formyl
Sequence Consensus
methionine (f-met) in response to initiation codon AUG and tRNAMef
Sequence
incorporating methionine (Met) in response to AUG codon in internal position.
• Present in prokaryotes. • Present in
Codon - AUG
Shine Dalgamo eukaryotes
I------------ 1
sequence is a • This In prokaryotes In Eukaryotes
sequence o f sequence
1 1 1 1
nucleotide b ases (5' - surrounds Cytosolic Ribosomes
At-Initiation At Internal site Mitochondria
UAGGAGG-3') AUG and & Chloroplast Polypeptide synthesis begin
site
lo ca ted 6 -1 0 base determines
N-formyl Methionine N-formyl with Methionine (rather than f-
upstream o f AUG the initiating
methionine (Met) is methionine Met) but again cells use
codon on mRNA codon o f
(f-Met) is incorporated (f-Met) is specialized initiating t-RNA that
molecule®. It helps mRNA.
incorporated by by t- incorporated is distinct from tRNAMet used at
ribosomes in • In this t RNAfM*' RNA M et by t AUG codons at interior
recognizing the sequence -3 RNA™* positions of m-RNA.
nucleotide sequence and +4
that is essential for positions • Initiating amino acid N-formyl - methionine (f-Met) arrives with tRNA fmet at
initiation o f relative to ribosome as N-formyl methionyl tRNAf-Mel (f-Met - t RNA fM**), which is formed
translation. AUG are in 2 steps.
• Sequence of occupied by - Met -tRNA synthetase aminoacylates (attaches) aminoacid methionine to tRNA
nucleotide base (5'- purine rMel (and also tRNA Met) with the help of ATP
UAGGAGG-3') (A&G)
686 ■ A Complete Review of Short Subjects

located 6 - lObp
upstream of AUG GCCA-3 /
codon on mRNA at GCCAUGG +4
its 5' end. More selective & specific (for only Met residue attached to tR N A ^ 1) -
• The 16 S ribosomal transformylase transfer a formyl group from N10-formyl tetrahydrofolate to
RNA component of amino group of met residue.
30 S ribosomal
subunit has a Met-tRNA™'* Transformylase fM et-t
nucleotide sequence + N“ - f-THF RNAfmel +
at its 3' end, that is THF
complementary to all
or part of shine • Methionine having anticodon UAC is the first amino acid required in binding to
Dalgamo sequence the initiation codon AUG on mRNA. In bacteria (prokaryotes) & mitochondria the
• Thus mRNA 5' end i n i t i a t o r t - R N A c a r r i e s a n N - f o r m y l a t e d m e t h i o n i n e a m i n o a c i d ® , whereas in

& 3' end, of 16 S eukaryotes, the i n i t i a t o r t R N A c a r r i e s a m e t h i o n i n e t h a t i s n o t f o r m y l a t e d ® .


rRNA can form • Formyl group is added to methionine by enzyme transformylase which uses N 10
complementary base formyl THF as carbon donor.
pair & facilitating
• The initiator t-RNA is the only t - RNA to be recognized by eIF2 (in eukaryotes) /
binding o f mRNA on
IF2 (in prokaryotes) and the only t RNA to go directly to P site.
30 S ribosomal subunit.

I. Steps of Initiation of Protein Synthesis


It can be divided into 4 steps (for ease of writing
eukaryotic initiation factor i.e. elF is written as IF)
D is so c ia te s into
1. Ribosomal dissociation into 40s & 60s subunits.
IF (i.e. e IF) 1A and 3 binds to 40 s subunit which
delays its reassociation.
2. Formation of 40s/43 S preinitiation complex by IF 3 & 1 A
binding of ternary complex & 40s subunit b in d s & prevents
G T P , LF-2 & m e l t R N A i T e r n a r y co m p le x
b ind to 4 0s su b u n it lo form p reinitiation
its reassociation
com p lex
- GTP binds to IF2 to form binary complex & then
to met t RNAi (a t RNA that binds initiation I F 2 has a , p <£ y s u b u n it & is o n e o f the
codon AUG & code methionine) to form ternary co n tro l point o f in itiation
— a : In a ctiv a te s G T P - G D P re c y c lin g
complex
p rotein I F 2 -P & prevents fo rm a tio n o f
40/ 43 S p r e in it ia tio n N
- Ternary complex binds to 40 s subunit to for 43 s S ta b iliz e c o m p le x J p re in iliatio n c o m p le x (P IC )
*— (5 : P ro m o te s fo rm atio n o f PIC
preinitiation complex, which is stabilized by IF 1A 1 -T I
&3 IF - 3 , 1A , G T P , I F -2 &
m et t R N A i I 4 0 S
- IF (elf)2 is one of the two control points for m eth y l G T P C ap
protein synthesis initiation in eukaryotes and IF 4 F , m R N A

consists o f e%f}& y subunits®. IF2a is m RNA


A TP | 5'
phosphorylated (on serine 51) by at least 4
different protein kinase (HCR, PKR, PERK & N ■ M e t h y l G T P C a p a t 5 ‘ e n d o f m R N f a c i l i t a t e s »'fs
b in d i n g w i t h 4 3 S P J C °
GCN2) that are activated when a cell is under - C ap b in d in g p r o te in IF 4 F has 3 (A, G , E) co m p o n en ts
4 £ re c o g n iz e & b in d m 7 G c a p & c o n t r o l ra te o f
stress and energy expenditure required for 43/ 48 S in itia
itiatio
tio n ^
\ in it ia t io n 0
protein synthesis would be deleterious. c o m p le
lex
x J 4 G , a s c a ffo ld in g c o m p o n en t b in d s 1F3 (& 4 0 5 ), 4E ,
4A and 4 B (A T P ase- h elicase com p lex)
Phosphorylated IF-2a binds tightly to and - 4 A 4c 4B sim p lifie s cap 2° stru ctu re by A T P a se & A TP
inactivates the GTP-GDP recycling protein e IF- T ra n slo ca te s 5 '-> 3 ' & sc a n s m R N A d ep en d en t h elicase activity
in itiation co d o n (S '-m o s t A U G )
2 B. Thus preventing formation of 43 s preinitiation
d eterm in ed b y K o z a k - C o n s e n s u s s e q u e n c e
complex & blocking protein synthesis.
3. Formation of 43/48 S initiation complex by IF -5
h y d ro ly zes G T P
binding of 40/43s preinitiation complex to o f IF 2
mRNA. r \_____— I F 3 , 1 A , 2 and 4 F are released and recy cle d
( 8 0 S in it ia t io n c o m p l c i J < m c t-t R N A i o n P site ready for elo n g atio n
- MGTP (methyguanosyl triphosphate) cap at 5'
terminal o f most eukaryotic m RNA facilitate its
binding to 43S preinitiation complex®.
Biochemistry: Structure, Function and Replication of Informational Macromolecules ■ 687

- Cap binding protein IF 4F (which consist of Difference Prokaryotes Eukaryotes


4A, 4G, 4E units) binds to the cap through 4E m- RNA Polycistronic i.e. have several Monocistronic with
protein and to 40 S ribosome subunit through coding regions; each with its one coding region
4G component. IF (eIF)3 is a key protein initiation & termination
because it binds to 4G component of 4F with codon producing a seperate
high affinity and it links this complex to 40 S polypeptide
ribosomal subunit (in 43 S preinitiation
Am ino acid 18 a a/sec 6 aa/sec
complex)
incorporation rate
- IF 4B (4B) binds & reduces (melts) the complex
2° structure of 5' end of mRNA through Initial amino acid F orm y lated m ethionin e (f- M ethionine® (Met)
ATPase & ATP dependent helicase activity. met)
Formation of 48 s initiation comlex require Initial t-RNA t-RNA fMe* tR N A Met
ATP hydrolysis Transcription & Translation starts before Uncoupled
- 4E is responsible for recognition & binding of translation transcription is ended
m7 G cap structure at 5' end of m RNA, a rate
(coupled)
lim iting step in tran slation. So 4E con trol rate
o f initiation®. 4G is a scaffoid in g rotein® Initiation factors (IF) 3 i.e. IF1, IF2, IF3 >10 designated elF
which binds to 4E, 4A and 4B (i.e. ATP ase- Determination of - Shine D alg am o sequence® K o z a k consensus
helicase complex that unwinds RNA) and to initiating codon - Facilitated by IF-2 bound to sequence®
eIF3, which links the complex to 40 s GTP Facilitated by eIF-2
ribosomal subunit. bound to GTP (plus
additional elF)
Elongation: Delivery - Facilitated by EF-Tu & EF- Facilitated by EF1 a
of aminoacyl t-RNA Ts and EF 1 Py
to A site
- After binding of 43 S preinitiation complex
with m RNA cap, and melting of 2° structure Nucleotide exchange - EF - Ts (as it exchanges its EF lPy (as it exchanges
near 5' end of mRNA through the action of 4B factors GTP for GDP on EFTu) its GTP for GDP on EF
helicase and ATP, the 43/48 S initiation la
complex translocates 5' - 3' & scans mRNA for Peptidyl transferase
23 S rRNA o f 50 S ribosomal 28 S rRNA o f 60 S
suitable initiation codon. Generally this is 5'- activity catalyzing
subunit (ribozyme) ribosomal subunit
most AUG, precisely determined by Kozak- peptide bond
(ribozyme)
consensus sequence that surround AUG (i.e. formation
Requires EF-G & GTP Requires EF-2 & GTP
purine at - 3 and + 4 positions relative to AUG). Translocation
-3 -1 +4 Termination -Release factor (RF) 1 -eRF recognizes all 3
I I I recognizes UAA & UAG, UAA, UAG and UGA
GCCA / GCC AUG G RF2 recognizes UAA & and release nascent
4. Formation of 80 S initiation complex by UGA polypeptide chain
combination of 43 S initiation complex with 60 - RF-3 (bound to GTP) causes - eRF-3 (bound to GTP)
S ribosomal subunit release of RF-1 or RF-2 as function like
- Involves hydrolysis of GTP bound to IF2 by GTP is hydrolysed prokaryotic RF-3 &
IF5, resulting in release and recycling of release eRF
initiation factor bound to 48 s initiation
complex and rapid association of 40S & 60S
subunits to form 80 S ribosome.
- met - t RNAi is on P site ready for elongation.

II. Elongation III. Termination


It involves the addition of aminoacids to carboxy end of Termination of protein synthesis occurs when a stop
growing chain and movement of ribosome from 5' end 3' end codon o f m-RNA (UAA, UAG, UGA) appears in A ' site.
of mRNA that is being translated (translocation) These codons are recognized by release factors (RF-1, RF-
3). R F w ith GTP & p ep tid y l tran sferase p rom otes
h y d roly sis o f bon d betw een the p ep tid e & t RNA
occupying the 'P' site®.
688 ■ A Complete Review of Short Subjects

Peplidyl/
A cceplor/Am inoacyl 1RNA
Polypeptide' acceptor site
site

N e w t -R N A w ith a m in o a c id bin ds to A -s i/ e v
+ EH- 1A | with expenditure of I GTP|
=\ (R F3 + G TP)
- Relaxing fa c to r 1
(R F 1) recognizes
stop codon ®
- R F 1 with peptidyl
transferase hydrolyze
Peptidyltruns ft rase: causes a tta chm en t o f g r o w in g peptide bond between peptide
c h a in to t R N A o f A site ,y
& t-RNA at p site Q &
- Peptide bond is formed b/w a-amino gp of amino acyl
prom otes release o f
t R N A of A site and carboxy g p o f t RN A of P site
- Peptidyininsfera.se is component of 2H S RN A of 60 S newly synthesized
subunit of ribosome & 23S RN A of 50S subunits of ribosome protein from P site.
Because the r R N A or ribosomes catalyze reaction, these are L Cause dissociation
ribozym es
o f ribosom e from
m RNA

Sites: Function

A site P site E site


EF-2, GTP Donor tRNA carrying Contains Deacylated tRNA
EF2 binds to & displaces the peptidyl tRN A from
A site to P site. appropriate amino acid tRNA with moves once the peptide
So deacylated tRNA moves on E (Exit site) from
which it leaves ribosome
on its acceptor stem, is growing bond is formed & it
E F 2 shifts b/w G T P to G D P Le . E F 2 - G T P c o m p le x is h y d ro ly ze d positioned before that peptide will soon be exiting the
to F .F 2 -G D P * * in moving m RNA forward by one codon & lead to
A site fre e f o r o cc u p a n c y b y n e w t R N A - E F l A - G T P com p lex. aa. is incorporated in chain ribosome.
protein.______________

Total energy expenditure: tRNA+A.A= 2ATP


- 2 GTP in elongation } 4ATP

Peptide Bond Formation & Translocation Evidence that rRNA Is Peptidyl Transferase

Peptidyl transferase activity (of 23S r R N A o f 50 s o r 2 8 Ribosomes can form peptide bonds even when proteins are
S r R N A o f 60 S r i b o s o m e )C is responsible for peptide removed or inactivated
bond formation. Few r RNA sequences are highly conserved in all species
This ribozyme catalyzes the reaction between the These conserved regions are on the surface o f RNA molecule
amino group of aminoacyl t RNA at A site and carboxy RNA can be catalyst
carbon of peptidyl tRNA in the P site, forming a Mutations that result in antibiotic resistance at the level of
peptide bond from an ester bond. protein synthesis are more often found in r RNA than in
Because the amino acid on aminoacyl t RNA is already protein component of ribosome
activated no further energy is required fo r this reaction X-ray crystal structure of large subunit bound to tRNA
The reaction result in attachment o f the growing peptide suggest this.
chain to the t RNA in the A site, the which makes t RNA
of P site to become deacylated.
Codon
The recently deacylated tRNA is attached to P site by
its anticodon and to an exit (E) site by the open CCA n -1 n n +1
tail. 5’Cap
Elongation factor (EF)-2 binds and s h i f t s b e t w e e n G T P
t o G D P ( i . e . E F 2 G T P i s h y d r o l y z e d t o E F 2 G D P ) Q to

move the ribosome down one codon on m RNA, so


that
1. A site is vacant & ready to receive next charged t E n
site *+1 site
n-1
RNA
n+l p
2. P site contain peptidyl tRNA that has been
site
translocated (displaced) from A site
3. Deacylated t RNA is on E site, from which it leaves
the ribosome.
Biochemistry: Structure, Function and Replication of Informational Macrom olecules 689

Essential Com ponents Required for 5 Steps of Protein Synthesis (in Prokaryotes)

Activation of Initiation Elongation Term ination & Folding & Post translational
amino acids -m R N A (Shine-Dalgarno -Functional 70S Ribosom e recycling modification
- 20 aminoacids sequence) ribosome (initiation -Term ination /Stop Enzymes & cofactors required
- 20 aminoacyl- -Initiation codon (AUG) in complex) codons (U AA,U AG , for
tRNA synthetase m RNA -Am inoacyl tRNAs U G A in m RNA - Removal of initiating &
(of 2 classes) -N-form ylm ethionyl tR N A (as specified by -Release /Termination signal sequences
-> 32 tR N A s (i.e. f-Met + tR N A fMet) codon) factors (RF-1, RF-2, -A ddition of acetyl, methyl
-A T P -30S subunit of ribosome - Elongation factors RF-3, RRF) carboxyl, phosphoryl group
-M g++ -50 S subunit of ribosome (EF-Tu, EF-Ts, EF-G) E F - G etc or oligosaccharide or
■Initiation factors (IF-1, IF- -GTP I F-3 prosthetic groups to
2, IF-3) -M g++ aminoacid residues
G TP -Proteolytic cleavage &
■Mg++ modification of terminal
residues.

Aminoacyl t- RNA Synthetase (AtRS) 2 E


- This family of enzyme is required fo r attachment o f amino Based on substantial differences in primary & tertiary structures
acids to their corresponding t RNAs. Each amino acyl and in reaction mechanism AtRS has been divided into 2 classes.
tRNA synthetase recognizes one specific amino acid
and one or more corresponding t RNAs (isoaccepting t Class I Class II
RNAs)®. F o r a m in o a c i d s w i t h 2 o r m o r e Arg, Cys, Gin, Glu, lie, Leu, Met, Ala, Asn, Asp, Gly, His,
the same enzyme usually
c o r r e s p o n d in g tR N A s , Tip, Tyr, Val - (add tRNA Lys, Phe, Pro, Ser, Thr
aminoacylates all of them. The extreme specificity o f synthetase to each) (tRNa synthetase)
synthetase in recognizing both A.A & t-RNA is In 2nd step, the aminoacyl group is In 2nd step, aminoacyl
responsible fo r high fidelity o f tranlation o f genetic transferred in itially to 21(OH) group is directly
message®. So, it implement genetic code by acting as group o f 3' terminal A residue then transferred to 31
molecular dictionaries® that can read both 3 letter code to 3' (OH) group by hydroxyl group of
of nucleic acid and 20 letter code of aminoacid. transesterification reaction terminal A adenylate
- It catalyzes 2 s t e p r e a c t i o n that result in covalent attachment o f carboxyl group o f a specific aminoacid to the 3' end o f its
corresponding/ cognate/isoaccepting tRNA®. The overall reaction r e q u i r e s A T P which is cleaved to AMP and inorganic pyro
phosphate (PPi). Thus 2 high energy phosphate bonds are expended fo r activation o f each aminoacid molecule®, making the
over all reaction irreversible. In 1st step the enzyme bound intermediate - aminoacyl adenylate (aminoacyl - AMP) is formed
and in 2 nd step the aminoacyl group is transferred from e n z y m e b o u n d a m in o a c y l A M P i n t e r m e d i a t e to 3 ' (O H ) o f t e r m in a l
A ( a d e n y la t e ) of corresponding tRNA.
- Amino acyl tRNA synthetase, aminoacylates tRNA that
accomplishes 2 ends: ( 1 ) i t a c t i v a t e s a n a m in o a c i d f o r p e p t i d e M g *+
A m in o a c id + t- R N A + A T P ------------------- ► A m in o a c y l tR N A + A M P + P P i
b o n d f o r m a t io n and ( 2 ) it e n s u r e s a p p r o p r i a t e p l a c e m e n t o f
a m in o a c i d s i n g r o w i n g p o ly p e p t i d e c h a in . As the identity of A m inoacyl tRNA
Synthetase
amino acids attached to tRNA is not checked on ribosome, the
attachment of correct amino acid to tRNA is essential fo r
fidelity o f protein synthesis®.
- In addition to extreme specificity & high fidelity it has a proofreading or editing activity® that can remove mischarged
aminoacid from the enzyme or tRNA molecule. The effect o f forcing the substrate specific binding through 2 sucessive steps (filters) is
multiplicative in proofreading.
- Incorrect aminoacids bind to s e p a r a t e h y d r o ly t i c ( p r o o f r e a d in g ) site in few AtRS. The active site of some AtRS that activate
aminoacids with no close structural relatives ( e g C y s - t R N A s y n t h e t a s e ) can sufficiently discriminate between the proper
substrate and incorrect aminoacids, and demonstrate l i t t l e o r n o p r o o f r e a d i n g a c t iv it y . Overall error rate of
protein synthesis (1 mistake in 1 0 4 aa incorporated) is higher than that of DNA relication.
- I n t e r a c t i o n b e t w e e n a m in o a c y l t R N A s y n t h e t a s e a n d t R N A i s c a l l e d t h e s e c o n d g e n e t i c c o d e b/o its critical
role in maintaining accuracy of protein synthesis. Conserved sequences of tRNA cannot be used for
discrimination. Discriminatory sequences are mostly concentrated in aminoacid & anticodon arm of tRNA. For
example primary determinant of tRNA recognition by Ala-tRNA synthetase is s i n g l e G = U b a s e p a i r i n t h e
a m in o a c i d a r m of t RNA Ala. So even a short synthetic RNA minihelix with c r i t i c a l G=U b a s e p a i r i n g but lacking
most of remaining tRNA structure is amino acetylated specifically with alanine almost as efficiently as the complete tR N A ^ .
690 ■ A Complete Review of Short Subjects

Energy Requirements In Protein Synthesis

Energy requirements Formation of peptide bond


Total 4 ATP and 5GTP are used in protein synthesis in following steps - It is catalyzed by peptidyl transferase,
a component of 28 S RNA of 60S
- Activation of 2 high Formation of 43 S pre initiation 1 GTP (Harper) /50 S (Lippincott) ribosomal
aminoacid & energy P O 4 complex involves binding of GTP by subunit. It is an example of ribozyme
formation of bonds elF- 2. This binary complex binds to activity (i.e. r-RNA catalyzes the
aminoacyl t- RN A (ATP - met t RNAi, (methionine having reaction)Q and a direct role o f RBA in
complex —> AMP) anticodon UAC) specifically involved protein synthesis.
- Formation of 43 S 1 ATP (or in binding to initiation codon.
- a - amino (NH2) group o f new aminoacyl
initiation complex high Formation of 80 S initiation 1 GTP t- RNA in 'A ' site combines with
when e IF4A and energy P O 4 complex (i.e. binding of 60 S subunit
carboxyl (COOH) group of peptidyl t-
elF 4B bind and bond) and 48 S initiation complex) involves
RNA occupying the 'P' (peptidyl or
reduce the complex hydrolysis of GTP bound to elF- 2 by
polypeptide) side. At initiation, P site is
secondary structure eIF-5 occupied by aminoacyl - t
5' end of m RNA Binding of aminoacyl t RN A to 'A' 1 GTP
RNA met*. Because the aminoacid on
through ATPase site requires elongation factor EF 1A
aminoacyl t RNA is already activated,
and ATP dependent and GTP
no further energy source is required for
heticase activity For translocation movment of m- 1G PT this reaction Q.
- Formation of 48 S 1 ATP (P0 4 RNA forward by 1 codon and
- In eukaryotic cell, a single ribosome is
initiation cmplex bond) leaving the A site open for
capable of synthesizing 400 peptide
by association of m- occupancy of another tRNA- EF1A-
bonds each minute.
RNA with 43 S pre GTP- aminoacid complex requires
- Energy required for formation of one
initiation complex. EF2 and GTP
peptide bond include the equivalent of
Releasing factor 3 (RF3) with bound 1 GTP
hydrolysis of 2 ATP to ADP and 2GTP
GTP is required for termination
to GDP, or the hydrolysis of 4 high
process
energy phosphate bonds

Ribosomes Energy Required in Protein Synthesis


■Ribosome is a cytoplasmic nucleoprotein structure that 4 high energy bonds are required for addition of one aminoacid
acts as a machinery fo r protein synthesis from m to growing polypeptide chain
RNA templates. Its role is to bring m-RNA and t-
RNA togetherC to translate the information encoded
in m-RNA (and transcribed from genes) into
polypeptides.
•1 ribosome can synthesize 5-6 peptide bonds
/second. So many ribosomes, situated 80-100
Additional ATP & GTP are required for initiation in eukaryotes.
nucleotides apart, work on same m-RNA
Whereas additional GTP is required fo r termination in both
simultaneously and these aggregates are called
eukaryotes & prokaryotes Q.
polyribosomes or polysomes. Ribosomes may be
attached on walls of endoplasmic reticulum (ER) to P r o k a r y o lic r ib o s o m e E u k a r y o tic r ib o s o m e

form rough ER or may exist free in cytoplasm.


■RER associated ribosomes synthesize proteins that
are exported from cell, are integrated into plasma,
ER or golgi membranes or lysosomes. Free cytosolic
ribosomes synthesize protein required in cytosol,
nucleus, mitochondria & peroxisomesQ. However
mitochondria contain its own ribosome
•Ribosomes can be free (cytoplasmic) or membrane
(ER) bound, contain about 65% of r-RNA and 35%
of protein, are roughly spherical with a diameter of
about 18nm - seen only on electron microscope and
accounts for almost 25% of dry weight of cell.
•Ribosomes are large complexes of r-RNA and
Biochemistry: Structure, Function and Replication of Informational Macromolecules ■ 691

proteins with one large and one small subunits. The Ribosomal Composition
small subunit binds m-RNA, thus guiding interaction
b/w m-RNA codon and anticodon o f t-RNA to read the 7 0 S (P ro k a ry o tic) 8 0 (E u k a ry o tic )
genetic information with exiquisite fidelity. Hence 1 1 1 1
small subunit is responsible for accuracy, whereas 50s 30 S 60S 40 S

the large subunit catalyzes the formation of 1 1 1 1


- 23 S -1 6 S - 28 S -1 8 S
peptide bond.
-5 S - 21 P rotein s - 5.8 S - 3 3 -3 5 P rotein s
- Prokaryotic ribosome with sedimentation coefficient
- 3 3 -3 6 - 5 S
70S (S=Svedberg units, a parameter sensitive to P rotein s - 5 0 P rotein s
molecular size & shape) is composed of 2 unequal
subunits with sedimentation coefficient of 30 S and
50S. Whereas eukaryotic 80S ribosome has 60S & 40S.
- Each ribosome has 3 binding sites fo r t-RNA - A (for incoming aminoacyl t RNA), P (for peptidyl - t RNA) and E (for empty t RNA
about to exit), each of which extend over both subunits. Together they cover 3 neighbouring codes.
- Both ribosomal subunits can be broken down into their RNA and protein components, then reconstituted in vitro nearly identical in
structure and activity to native subunit. Ribosomal subunits are huge r-RNA molecules that form the structural core. The proteins
are secondary elements in complex decorating the surface (forming Worm like structures). There is no protein within 18A° of active
site for peptide bond formation. The large r-RNA component performs peptidyl transferase activity and thus ribosome is a
ribozyme.
- Ribosomal RNAs (28S + 16S) constitute nearly 70% of total eukaryotic cellular RNA. All r-RNA molecules except 5S r RNA,
which is independently transcribed are processed from a single 45S precursor RNA molecule in nucleolus. The highly
methylated r-RNA are packaged in nucleolus with the specific ribosomal proteins. In cytoplasm, ribosomes remain quite
stable and capable of many translational cycles.
- 16s r-RNA o f 30 subunit o f p ro k a ry o tic rib o som e id en tifies an d binds shin e - D a lg am o sequence®.
- The r-RNA residues of large subunit of ribosomes that catalyze peptide bond formation (and have peptidyl tranferase activity)
contain highly conserved nucleotide sequence^ - the ribonucleotide octet (5') AUAACAGG (3') in all cells.
- Many negatively charged phosphoryl groups in RNA backbone limit the stability of very large RNA molecules. So incorporation of
divalent cations or positively charged groups and certain peptides promote stability. Ribozyme catalyzed synthesis of peptides could
thus initially have evolved for structural maintenance of large RNA molecules but the greater catalytic potential of proteins
marked the beginning of new era of protein dominated metabolic system.

Protein Kinase & Protein Phosphorylation


• Attachment of phosphoryl groups to specific aminoacid residue of a Eukaryotic Protein Phosphorylate
protein is catalyzed by protein kinases; removal of phosphoryl groups Kinase (PK)
is catalyzed by phospho protein phosphatases. Serine-Threonine PK Serine (95%); also
• Phosphorylation is the commonest type o f regulatory post translational recognizes threonine (3%)
modification. The addition of phosphoryl group to a Ser, Thr, or Tyr Tyrosine PK Tyrosin (< 1%) only
residue introduces a bulky charged group into a region that was only Dual Function PK Serine (Sr), threonine (Thr),
moderately polar. The oxygen atoms of a phosphoryl group can tyrosine (Tyr)
ATP ADP
hydrogen bond with one or several groups in a protein, commonly the
amide groups of peptide backbone at the start of an a-helix or the
charged guanidinium group o f an Arg residiue. The 2 negative charges on a
phosphorylated side chain can also repel neighbouring negatively ©
NH2 i Protein l-C O O H NH2 i Photpho-Proteln |. o - P o f
charged (Asp or Glu) residue. The interactions with arginine, aspartic
acid or glutamic acid, histidine & lysine side chains is usually
secondary to and after primary attachment with serine (Sr), threonine
(Thr) or Tyrosine (Tyr)
• When modifications caused by phosphorylation occur in a region of an enzyme critical to its three dimensional structure,
phosphorylation can have dramatic effects on enzyme conformation and thus on substrate binding and catalysis, (either
stimulating or inhibiting). For example phosphorylation stimulates substrate binding & catalysis in glycogen phosphorylase
of muscle & liver, whereas it inhibits isocitrate dehydrogenase.
• Protein kinases catalyze the transfer of negatively charge phosphoryl (phosphate) group of ATP to hydroxyl moiety of only
three aminoacids ie serine (Sr, 95%), threonine (Thr, 3-4%) and tyrosine (Tyr in < 1%) to form chemically stable phosphate
ester in eukaryotes. Whereas in prokaryotic bacteria, fungi & plants phosphorylate histidine and aspartate (or glutamate) by
histidine and aspartate protein kinases.
692 ■ A Complete Review of Short Subjects

Plasma Immunoglobulins (Antibodies)


• Antibodies are produed by B (or plasma) cells and provide humoral immunity. It destroyes target cells by following
mechanisms.
- Activating classical complement pathway i.e. complement fixation (by IgG & IgM)
- Antibody dependent cell mediated cytotoxicity (ADCC) i.e. antibody coating will sensitize the target cells for killing by killer
(k) cells (IgG)
- Agglutination (by IgM > A > G)
- Opsonization (IgG coating helps & makes phagocytosis easy as its Fc portion can fix with macrophage).
- All immunoglobins contain a minimum of 2 identical light (L) chains (23 -kDa) and 2 identical heavy (H) chains (53 - 75 kDa)
joined together by disulfide bonds forming a Y shaped tetramer (L2 H 2). Each chain has 2 ends - amino (NH 3+) and carboxy
(COO') and 2 regions.

I------------------------------------------ ---------------
Constant (C) region Variable (V) region
- It has constant amino acid sequence, and located towards carboxy (C) terminal. - It has variable amio acid sequence &
- Half (1/2) of light (L ) chain towards carboxy end (k/a C l ) and approximately three located towards amino terminal.
quarters (3/4) in IgG, A, D or 4/5 in Ig M,E of heavy chain towards carboxy terminal - First 108 amino acids (out of 214)
(k/a C h ) - constitute the constant region. in light chain i.e. amino terminal
- Constant domain have a characteristic, well conserved structural m otif - the half of light chain ( V l ) and first
immunoglobulin fold in all P-class of proteins. There are 3 to 4 of these constant 1 1 8 aa (out of 440) in heavy chain
domains in each heavy and 1 in each light chain. i.e. approximately one quarter
Ch Cl (1/4) of heavy chain towards
- Constant region of heavy chain ( C h ) is divided into 3 (in IgG, IgA, Ig - Distinguish amino terminal ( V h) constitute
D) or 4 (in IgM, IgE) parts - CHi, Ch2, CH3, CH4- ed by the variable region.
- Distinguished by differences in their CH regions, humans have 5 differences - Variable region are quite
different types of heavy chains - determining 5 classes of in their Cl heterogenous and no two
(constant variable regions have identical
Immuno-globulin Heavy Chain CH domains (number) region of amino acid sequence.
Gamma (y) light chain), - Variable regions are comprised of
IgG 3]
IgA Alpha (a) humans relatively invariable regions and
3 I' C h1 ,C h2,
Delta (5) have 2 hypervariable regions.
IgD 3 Ch3
different - L chains have 3 and H chains
IgM Mu (|i) 4^
[ types of have 4 com plem entarity
IgE Epsilon (e) 4 - Also C h4
light chain - determ ining regions (CDRs) or
- Hinge region between CHi and CH 2 domains is the area in which kappa (k) hyper variable regions - which
papain (proteolytic enzyme obtained from papaya) and pepsin and lambda comprise the antigen binding site
cleave the immunoglobulin molecule. Papain cleaves distal disulfide (X). located on amino tips of Y and
bond to produce 2 antigne binding fragments (Fab) and 1 Any dictates amazing specificity of
crystallizable fragment (Fc) whereas, pepsin digestion cleaves immunoglo antibodies.
proximal S-S bond to generate 2 Fab fragments joined together by bulin - Polypeptide zone b/w 2 hyper
distal S-S bond and lFc. molecule variable regions is k/a frame
- Hinge region allows both Fab arms to move independently thus always work regions.
confers flexibility to bind to antigen sites which may be at variable contains - About 5 to 10 aminoacis in each
distance (on bacterial surface). either 2X or hypervariable CD region
- Constant regions of immunoglobulin particu larly b a sa l 2 k but contribute to antigen binding site,
cry sta liz ab le frag m en t (Fc) i.e. CH2, CH3 (and CH4 in IgM & IgE) never a which is formed by CDRs of both
are responsible for class specific effector functions (like mixture of V h & V l
transplacental passage, complement fixation, opsonization, both. - Epitope or antigenic determ inant
agglutination, binding to macrophage & mast cells, mediation of In human k is the site on antigen to which an
allergic response, presence in secretions, antigen receptors on B cells chains are antibody binds. Large antigen
and polymeric forms containing J chains of immunoglobulins. more binds with all of CDRs whereas
- 1 molecule of Ig has 2 Fab so it is termed divalent. frequent small antigen interact with only
(60%). few (or one) CDR i.e . a
m utual complem entarity b/w
epitope & CDR surface. Binding
involves non covalent forces and
Biochemistry: Structure, Function and Replication of Informational Macromolecules ■ 693

A m in o T e r m in a l
bonds (electrostatic, vander
P a p a in C le a v a g e Walls, hydrogen and hydrophobic
bonds)
- Various combinations of H & L
Vl — L ig h t ch ain (L )
chain CDRs can give rise to
CL
tremendous diversity of
antibodies (specificity) k/a
- P e p s in c le a v a g e site
combinatorial diversity.
Ch2
- Antigen binding fragment (Fab)
Fc < has light chain (Vl & Cl) and VH
-H e a v y ch a in (H )
P e p s in C le a v a g e P a p a in C le a v a g e
Ch3 & C hi of heavy chain.

• Both light chain and heavy chain products IrG IrA IrM IgD IgE
of multiple genes like - variable region (V) Heavy (H) Chain Y A (a) R 6 e
gene, joining region (J) gene, constant Heavy chain 4 4 5 4 5
region (C) gene and diversity region (D) domains
Usual Structrue Monomer (1) Monomer Monomer Monomer Monomer
genes. V, J, & C genes occur for both L & H
/Units (1) Dimer (1). (1) (1)
chains - Vl/Vh, J, Cl/h genes whereas
(2) Pentamer (5)
diversity region (D) g e n e s o c c u r o n l y f o r Hexamer (6)
h e a v y (H) c h a in s . / gene bears no relationship Dimmer (2)
to / chain o f IgA or IgM. Antibody valence 2 2,4 10,12 2 2
• All normal classes & subclasses of Additional Units in - S&J J piece - -
immunoglobins are called i s o t o p e s if they polymers
contin s a m e l ig h t c h a in and identical Subclasses (Isotypes) Gl, G2, G3, G4 Al, A2 - - -

variable domains o f both their light chains % of total Ig in 75 15 9. 0.2 0.004


( V l) and heavy chains (Vh) indicating s a m e
serum
Serum concentration 1000 200 120 3 0.05
This means different
a n t i g e n s p e c if i c i t y .
(mg/dl)
classes (isotopes) of immunoglobulins are
Serum half life 23 6 5/10 3 2.5
differentiated by their d if f e r e n t C h r e g i o n s (days)
which are combined with the same antigen Rate of synthesis 35 65 5 0.4 0.016
specific Vh region. (mK/kg/day)
• Human genome contains only ~ 29,000 Sedimentation 7 7 o r ll* 19 7 8
coding genes but can produce 109- 1010 coefficient (S)
different immunoglobin types b/o g e n e Molecular weight 150 170 or 900 180 190
r e a r r a n g e m e n ts , providing the immease
(X1000) 400*
Complement + (Gi, Gz, G 3 ) + - -
diversity needed for recognition of
Fixation-Classical +(G.) + +
enormous number of antigens. I m m e n s e Alternative
a n t i b o d y d iv e r s ity depends on a number complement
of factors including - t h e e x is t e n c e o f activation
m u l t i p l e g e n e s e g m e n t s (30 XV, 300 kapa Natural antibodies, +
(k) V; 1000 heavy chain V, 4J, 15D, 1C Primary response
segments), their recombinations (somatic antibody Antigen
recetors on B cells
rearrangement o f DNA or transposition o f
Transplacental +
genes)®, the c o m b in a t io n s o f d if f e r e n t passage,
l i g h t (L ) & h e a v y (H ) c h a in s , ju n c t i o n a l Opsonization
d iv e r s it y (i.e. addition or deletion of (produce opsonin
random number of nucleotides in joining Cjb), Antibody
certain gene segments 1/1 inaccuracies in dependent cell
V-J, V-D and D-J recombination), N - r e g io n mediated cytolysis
(ADCC)
i n s e r t io n (nucleotides inserted b/w D and
Fixation to mast +
J by terminal transferase), combinatorial cells
association, multispecificity, and a high Mediation of
frequency o f somatic mutationQ. allergic responses
• C- t e r m in a l t r a n s m e m b r a n e d o m a in is Found in secretions - + - - -

present in membrane bound antibodies Agglutination + ++ +++


694 ■ A Complete Review of Short Subjects

but absent in secreted (circulating)


antibodies. The shift from membrane * Form is found in secretions (saliva, milk, tear etc) & fluids of respiratory, genital &
bound form to secreted form of intestinal tracts,
immunoglobulins in volves p o ly A -site
ch oice (i.e. d ifferen tial RNA processing)Q.

v, - v2 - v3 H V300 D1 — c>2 ----- °20 — Jl — h — h C o n sta n t

s z
v3 D2 J2
---- C o n sta n t

• A gene can give rise to multiple products by differential RNA processing i.e. - either by poly A site choice or alternative
splicing or both. If there are >2 sites for cleavage and polyadenylation, use of one closest to 5' end will remove more (& use
of one closest to 3' end will remove less) of primary transcript sequence-thus generating diversity in variable domain of Ig
heavy chain. The mRNA site where cleavage occurs is marked by 2 sequences.
- Highly conserved (5') AAU AAA (3') sequences about 10 to 30 bases upstream on the 5' side of cleavage site
- Less well defined GU rich sequence, 20 to 40 nucleotides downstream of the cleavage site.
Cleavage generates free 3'-OH end of mRNA. Poly (80 •250) A residues are immediately added by polyadenylate
polymerase enzyme that does not require template but uses cleaved mRNA as primer. Poly A site choice generates
diversity in variable domains and is responsible for deciding w h eth er an an tib od y w ill rem ain m em brane bound o r g et
secretedQ. Alternative spling produce 3 different form of myosin heavy chains whereas, both mechanism come into play to
produce 2 different hormones calcium regulating hormone clacitonin in thyroid and calcitonin gene regulated peptide
(CGRP) in brain from single transcript.

Protein Sorting (Localization /Targetting)

A major sorting decision is made early when proteins are synthesized either on membrane bound polyribosomes (ofRER) or on free
polyribosomes o f cytoplasm. Protein synthesized on
,________________________ I___________________________ .
Cytosolic-Free Polyribosomes Membrane bound (RER)- Polyribosomes
- Protein synthesized on free - Proteins synthesized in RER contain a signal peptide that mediate their
ribosome lack signal peptide and attachment to membrane of endoplasmic reticulum.
are delivered into cytosol. - Proteins synthesized on RER may be destined to various membranes (eg ER
- They may be further directed to membrane, golgi apparatus membrane and plasma membrane), to lysosome
mitochondria, nuclei, and and for secretion.
peroxisomes by specific signals or - Golgi apparatus (all parts) is involved in O-glycosylation and processing o f
remain in cytosol if they lack a oligosaccharide chains of membrane & other N-linked glycoproteins and sorting
signal. o f various proteins prior to their delivery to appropriate intracellular destination,
★ Proteins that contain targeting (itrans golgi network performs this role)
sequence that is subsequently
removed are called pre proteins.
Whereas in pre-pro-proteins a
second peptide also has to be
removed.

Intracellular Traffic & Sorting of Proteins

Signal peptides & protein localization Chaperone (Folding) Proteins


>Signal peptide is an extension that is found in proteins syn thesized , ■Chaperone (literally meaning older women incharge o f
on m em brane bound p o ly rib oso m esQ. But proteins whose entire young unmarried woman on certain social occasions) are the
synthesis occurs on free polyribosome lack this signal peptide proteins that have a role in assembly and proper folding o f
(Blobel - Sabatini hypothesis) newly synthesized proteins so that they have biological
Biochemistry: Structure, Function and Replication of Informational Macromolecules ■ 695

•Signal peptide is also k/a leader or signal sequence, presequence or activity. The exit of properly folded protein after
transient insertion singal. And it synthesis from ER may be a rate limiting step.
- Contain 12-35 amino acids with a central cluster of hydrophobic amino •P re s e n t in a wide range o f sp e c ie s fro m b a c te ria to
acids & at least one positively charged aminoacid near amino terminal. h u m a n s in v a rio u s c e llu la r co m p a rtm e n ts su ch as
Methionine is usually the amino terminal aminoacid cytosol, mitochondria, & lumen of endoplasmic
- Is usually (not always) located at the amino terminal; and cleaved reticulum a n d m a n y a re so c a lle d heat shock protein
off at the carboxyl terminal end of an Ala residue by signal (hsp)®
peptidase
•Bind mainly to hydrophobic regions of unfolded &
•Signal peptide is involved in process of protein insertion into ER aggregated proteins and act as a quality control or
membrane & lumen in following manner editing mechanism for detecting misfolded or o th e rw is e
- The m-RNA encodes an amino terminal signal peptide. It is d e fe c tiv e p ro te in s.
recognized by signal recognition particle (SRP) that blocks
•Some are induced by conditions that cause unfolding of
translation after about 70 amino acids® have been polymerized
newly synthesized proteins eg. any stress to cell like
(40 buried in large ribosomal subunit & 30 exposed).
radiation, heavy metals, free radicals, toxins, elevated
- SRP causing elongation arrest has 6 proteins & 7 S RNA (related
temperature & chemicals®; and hence called stre s s
to Alu family of highly repeated DNA sequences)
p ro te in s . H e a t s h o c k p ro te in s were first identified in
- SRP-R (a receptor for SRP) is an integral docking protein on ER
response to heat shock.
membrane, which has a and fd subunits. /? subunit spans the
•Most exhibit A T P a se a c tiv ity & bind ADP & ATP. The
membrane & a is bound to GDP/GTP®
chaperon- ADP complex has high affinity for unfolded
- SRP imposed elongation block is released as the SRP-signal
proteins, when bound ADP is replaced by ATP.
peptide-polyribosome complex binds to SRP-R®; thus SRP guides
Chaperon - ATP complex releases segments of proteins
the signal peptide to SRP-R & prevents premature folding and expulsion
that has folded properly.
of proteins.
•Some c h a p e ro n e fo ld in g p ro te in & e n z y m e lo ca te d in
- Translocation consists of 3 membrane bound proteins (Sec 61
ro u g h e n d o p la s m ic re tic u lu m are
complex) that form a protein conducting channel® in ER. The
insertion of signal peptide into conducting channel, while the other end of
- Im m u n o g lo b in h e a v y c h a in b in d in g p ro te in (B iP ,
parent protein is still attached to ribosome is termed cotranslational
p re w e n t in lu m e n )
insertion
- G lu c o s e r e g u la te d p ro te in (G R P ) 94
- Protein should be in unfolded state prior to entering conducting
- C a ln e x in (calcium binding protein located in ER
channel- otherwise it may not be able to gain access to the channel.
membrane that also binds to MHC antigen) & variety
When the process is complete the ribosome is dissociated into 2
of other serum proteins, retaining them in ER until
subunits and signal peptide is hydrolysed by signal peptidase,
glycoprotein has folded properly.
located on luminal side of ER.
- C a lr e tic u lin (calcium binding protein)
- Amino acid sequence k/a halt or stop transfer signal, retains
- P ro te in d is u lfid e is o m e ra s e (P D I), an enzyme that
proteins in membrane as it prevents passage through membrane
promotes rapid reshuffling of disulfide bonds® until
of ER, of proteins that are integral ER membrane proteins eg.
correct set is achieved.
cytochrome p 450
- P e p tid y l p ro ly l is o m e ra s e (P P I) enzyme accelerates
- N- glycine chains are added (cotranslational glycosylation)
folding o f proline containing proteins® by catalyzing
occurs as proteins transverse the inner part of ER membrane.
the cis-trans isomerization aminoacid residue (x) - Pro
• Some sequences that direct proteins to specific organelle
bond
T a rg e tin g s e q u e n c e (or co m p o u n d ) T a rg e te d o r g a n e lle
• Misfolded proteins in ER are transported back to
Signal peptide sequence Membrane of ER p ro te a so m e s present in cytosol by re tro g ra d e tra n s p o rt
Amino terminal KDEL sequence (Lys- Asp Luminal surface of through translocon for ERAD. It may be powered by
- Glu- Leu) in ER resident proteins in COPI ER ATP ase present in proteasomes. Before entering
vesicles proteasomes, most proteins are ubiquitinated.
Di-acidic sequences (Asp-X-Glu) in Golgi membranes
membrane proteins in COP-11 vesicles * About other options: P ro te a se s are enzymes that
Amino terminal sequence (20 - 80 residues) Mitochondrial hydrolyze proteins®. P ro te o s o m e s (eg 26 S component)
matrix help in degrading the proteins marked with ubiquitin®.
N u cle a r lo c a liz a tio n s ig n a l (N L S ) eg Pro2 - Nucleus T e m p la te is involved in the synthesis of nucleic acid.
Lys2 - Arg - Lys- Val
PTS (peroxisomal matrix targeting sequence) Peroxisome
eg Ser - Lys - Leu
M an n o se 6 p h o sp h a te Lysosome®
696 ■ A Complete Review of Short Subjects

________ Protein Degradation and Traffic________


• Accumulation of misfolded proteins in endoplasmic reticulum (E R s t r e s s ) can induce u n f o l d e d p r o t e in r e s p o n s e (U P R ).
This results in increased folding capacity of ER (by increasing chaperones) and prevents a buildup of unproductive &
potentially toxic protein products by inhibition of translation of misfolded proteins & induction of protein synthesis
involved in degradation of misfolded proteins.
• R e t r o t r a n s lo c a t io n o r d is lo c a t io n is the process by which misfolded proteins are transported back across ER to enter
p ro tea som es in cytoplasm. C h a p e r o n e s (of ER & cytosol) target misfolded proteins and p olyubiquitin -bindin g protein s
escort them to proteasomes. Energy for translocation is partly supplied by p 9 7 an A A A -A T P a s e (i.e. ATP Associated with
various cellular Activities).
• 3 enzymes - a c t i v a t i n g , c o n ju g a t in g and lig a s e (latter confers substrate specificity) are involved in attaching ubiquitin to
target protein. Minimum of 4 ubiquitin molecules must be attached to commit a target molecules for degradation.
Ubiquitin can be r e u s e d after cleaving by d e u b i q u i t i n a t i n g enzymes.
• Proteasome is a large cylindrical structure composed o f 50 subunits. It has 1-2 regulatory caps containing ATPase for unfolding
target proteins and a hollow core for degrading it to small peptides. It has a important role in presenting small peptides
(produced by degradation of viruses etc) to major histocompatibility (MHC) class I molecules, a key step in antigen
presentation to T-lymphocytes.
• Vesicles involved in e n d o c y t o s is and /or carrying cargo to lysosomes are c o a te d w i t h c la t h r in . Clathrin is involved in
transport in post golgi apparatus locations including endosomes, plasma membrane (PM) and trans golgi network (TGN).
• Proteins synthesized on membrane bound polyribosomes (RER) is destined for golgi apparatus (GA) or PM. C O P I I
( C a lt h r in f r e e ) t r a n s p o r t v e s i d e s are involved in antegrade (ER to GA) transport and CO PI (clathrin fre e) tran sport v esicles
are involved in retrograde (GA to ER) transport. Secretory & transport v e s i c l e s carrying cargo from GA to PM are also clathrin
free.
• Key steps and m o le c u la r s w it c h e s in the life cycle of non clathrin coated vesicles are b u d d i n g [Sar 1 (GTPase); Sec 12
(guanine nucleotide exchange factor = 'GERF), coat proteins (complet bud formation & contribute to curvature)]; p i n c h i n g
o f f bud, U n c o a t in g / c o a t d is a s s e m b l y (Sari has key role in both assembly & uncoating of coat proteins), vesicle ta r g e t in g
(Rab proteins / Ras related proteins / monomeric GTPases first observed in rabbit brain), t e t e r in g (or attachment) of vesicles
to membrane (Rab effector proteins), d o c k in g (Vesicle or V- SNAREs pair with cognate target or t-SNAREs, forming 4 helix
bundle; SNAREs = SNAP Receptors), f u s i o n (interaction of SNAREs on same membrane forms eis - SNARE complex and
on two different membranes forms trans SNARE complex or S N A R E p in ) and d is s o c i a t i o n (require NSF or NEM sensitive
ATPase and a-SNAP or soluble NSF attachment protein). ARF (a GTPase or ADP ribosylated factor) is involved in
formation of both COP I & clathrin coated vesicles.
There are two major enzyme system responsible for degradation.
r
Ubiquitin- Proteasome pathway or Endoplasmic reticulum Lysosomes
associated degeneration (ERAD)
• Energy dependent, requires ATP • Non energy dependent
• Mainly degrade endogenous protein (synthesized within cell) • Degrade extracellular protein:
• a- carboxyl glycine of ubiquitine becomes co- valently - Plasma protein
attached to lysine residue of protein I - Cell surface membrane protein
Protein tagged with ubiquitine is recognized by proteolytic • Extracellular proteins are taken into cell by
proteasome endocytosis & then degraded by lysosomal
i enzyme
Protein degraded to Amino acid
Biochemistry: Structure, Function and Replication of Informational Macromolecules ■ 697

Diseases d/t

Conformational abnormalities in intracellular transport o f proteins and enzymes (d/t Peroxisomal abnormalities
mutation) resulting in ER stress and protein misfolding. 1. Hyper oxaluria type 1
D is e a s e A ff e c te d p ro te in 2. Hyper pipecolic academia
A lz h e im e r 's d is e a s e P - a m y lo id Q 3. Zellweger and pseudo zell
P rio n d is e a s e s (lik e k u ru , C r e u tz fe ld t J a k o b 's P r P sc wager syndromes
d is e a s e , fa ta l fa m ilia in s o m n ia )
4. Adrenoleuko dystrophy
C y s tic f ib r o s is CFTR
(ALD), neonatal ALD and
a -A n titry p s in d e fic ie n c y w ith liv e r d ise a se a l - A n t i try p sin
psoudo neonatal ALD
C h e d ia k H ig a sh i s y n d ro m e L y so so m a l tr a ffic k in g re g u la to r
C o m b in e d d e fic ie n c y o f fa c to r 5 & 8 M a n n o se b in d in g le c tin (E R G IC 53) 5. Acatalasemia
D ia b e tes m e llitu s (so m e) In su lin re ce p to r (a -s u b u n it) 6 . Rizomelic
A D fa m ilia l h y p e rc h o le ste ro le m ia L D L re ce p to r chondrodysplasia
G a u c h e r d ise a se P -g lu co sid a se punctuata
H e m o p h ilia A F a c to rs 8, 9 7. Infantile Refsum disease
H e re d ita ry h e m o ch ro m a to s is H FE
8 . Glutaryl CoA oxidase
H e rm a n sk y -P u d la k sy n d ro m e p -3A su b u n it o f A P -3 a d a p to r co m p le x
deficiency
1-c e ll d ise a se N - a c e ty l g lu co sa m ie p h o sp h o tra n s fe ra s e
L o w e o c u lo c e re b ro re n a l s y n d ro m e P IP 2 - 5 - p h o sp h a ta se
Mn-" Hyper ZAR-G
T a y S a c h 's d ise a se P -h e x o sa m in id a se
deficiency"
V o n W ille b ra n d d ise a se V o n W ille b ra n d fa c to r

Prion Proteins & Prion Disease


It is a protein that is highly resistant to proteolytic degradation. It exists in two isoforms
i-------------------------------------------------------------------------------------- 1
Noninfectious normal Pathological prion protein (PrPsc)
cellular prion protein (PrP0) It rich in /3-sheetsQ with many hydrophobic aminoacyl side chains exposed to solvents.
- Human prion related protein PrPK molecules therefore associates strongly with one another forming insoluble protease
is a glycoprotein, encoded on resistant aggregates of fibrils (similar to amyloid)
short arm o f chromosome One pathological prion or prion related protein can serve as template for conformational
2 OQ transformation of many PrPc molecules. So the prion disease can be transmitted by
- It is monomeric, rich in protein alone without involvement o f DNA or RNA°.
a — helix0 and does not It causes transmissible spongiform encephalopathies (TSE) or prion disease. These fatal
cause disease neurodegenerative diseases are characterized by spongiform changes, astrocytic gliomas
- It is precursor of PrPsc. & neuronal loss resulting from insoluble aggregates in neural cells. These include mad
cow disease (bovine spongiform encephalopathy) in cattles, scrapie in sheep, and
Creutzfeldt- Jacob disease (most common), Gerstmann- Straussler - Sheinker disease,
Kuru, & fa ta l fam ilial insomnia in humans

Protein Misfolding
Any alteration in protein folding causes wide Protein folding is a complex, trial and error process that can sometimes result
variety o f neurodegenerative disease in in improperly folded molecules. Deposits of these misfolded proteins are
mammals, perhaps the most common o f which associated with a number of neurological diseases.
is prions disease. Prions diseases include:
- F a t a l f a m i l i a l in s o m n ia Q ~| Prions diseases Alzheimer's disease
- C r e u tz fe ld t - Ja k o b 's d ise a se -» H um ans - Occur because of defect in - Occurs due to accumulation o f misfolded
- KuruQ J the secondary - tertiary protein called fi- amyloid°.
- S c ra p ie -> Sheep
protein structure due to - The A p amyloid that is deposited in brain is
- B o v in e spongiform en c e p h a lo p a th y - » C ow
defect o f folding o f the derived by proteolytic cleavages from larger
peptide chain0. amyloid precursor protein
- Prions stands for - A (J peptide when aggregated in (3-pleated
proteinaceous infections sheet, is neurotoxic.
particle devoid o f nucleic - Apolipoprotein E has been implicated as a
acid°. potential mediator of this conformational
transformation
698 ■ A Complete Review of Short Subjects

Gene Duplication Homologs, Paralogs & Orthologs


• Gene duplication is any duplication or amplification - When 2 genes share readily detectable sequence similarities
of segment of DNA (chromosome) that contains a (nucleotide sequence in DNA or aminoacid sequence in proteins
gene. they encode), their sequences are called hom ologous and proteins
This second copy of a whole gene is introduced into they encode are homologs.
chromosome as a result of defective replication o f the - 2 homologous genes occurring in the same species are called
DNA (chromosome) such as inhomologus recombination, paralogous and their protein products are paralogs. Paralogous
retrotransposition or duplication of entire chromosome. genes are derived from gene duplication followed by gradual
■The second gene copy is superfluous (extra) and free from changes in the sequences of both copies. Paralog proteins are
selective pressure (the process o f natural selection) and similar in sequence of aminoacid & three dimensional structure
mutation in this gene will not be deleterious. So it both, although they commonly acquire different functions during
becomes a means by which the cell may evolve, by their evolution
producing a new gene with a new function - 2 homologous genes occurring in different species are called
(proteip/enzyme) while retaining the original gene & orthologous and their protein products are orthologs. Orthologs
gene function because (organism with) extra gene are commonly have same functions in both organisms.
accumulate mutations faster than (organism with) - Synteny is conserved order of genes on chrom osom e over a large
single function gene. segment of genome of closely related species (eg human & mouse).
Freedom from pressure of natural selection and It provides additional evidence for an orthologous relationship
consequences of mutations in extra copy of gene allow between genes at identical location within the related segment.
them to acquire new capability (such as metabolism of - Genes in segments of hum an chrom osom e 9 & mouse
new substrate) that may increase the fitness of chrom osom e 2 exhibit very high degree of homology (orthology)
organism for survival in new ecological niche as well as synteny (same gene order).
(environment). 3 0 0 Gene X I Gene X DOC
D Q Q Gene X p Q C Extra Copy

M u tltlo n

1
- No deleterious effect because original gene is there
to produce normal mRNA— • protein— ►function
P rev en t g e n etic - May acquire new capability such aa m etabolism
O rig in a l fu n c tio n
of new substrate by evolution of new m RNA &
v a ria tio n of gene product
protein
& e v o lu tio n la lost

Sickle Cell Hemoglobin (HbS)


A point mutation involving T- to P artially acc ec pte d (A ) to (U ) s ubstitution
m lssense (P o in t) m utation in corresponding
- A DNA substitution (which Invo lving (T ) to (A ) posilionof m R N A and Codes Amino acid In protein
intum results in an 'A - to- U' substitu tio n In p globulin so codon
ge ne (D N A ) at 6th position
change in the m-RNA) GAA/QAG j_ Potar Itasativ'hr C h a r** _ ^ HbABchaln |
corresponding to the 6 th codon of
1 Glutamate at B<________ I------- - ---------- 1
P globin gene result in sickle cell GUA/GUG
Non polar hydrophobic
HbS - Bchain
[ Valina at B6
disease
The corresponding single nucleotide change within the codon 1° Hydrophobic sticky 2° Complementary Sticky site
would be G A A or G A G of glutamic acid to G U A or GU G of patch on oxy & deoxy on deoxy HbA and deoxy
valine. This is a partially accepted missense mutation; partially HbS HbS
I I
accepted because HbS does bind & release O 2, albeit abnormally; Produced d/t Produced d/1 deoxygenation
missense because it hinders normal function & result in sickle cell replacement of polar glu I
anemia. -» non polar valine Decreased solubility
Restriction fragment length polymorphism (RFLP) is an inherited
difference in the pattern of restriction (endonuclease) enzyme i
digestion d/t presence or absence of an extra restriction site. This Polymerization & Sickling
single base change in sickle cell disease result in RFLP®.
The mutation results in single amino acid substitution (GLu 6(^) - <
Val): i.e. a surface localized charged (polar, hydrophilic) aminoacid p a Sticking site
glutamate is replaced by hydrophobic (non polar) amino acid
a p
valine (without any charge)® at position 6 in two ff-chains. The R >
group of valine has no electric charge, whereas glutamate has a Oxy H bA Deoxy HbA
negative charge at pH 7.4. So HbS has 2 fewer negative charges (terminate polymerization)
than HbA (1 for each P-chain)
Biochemistry: Structure, Function and Replication of Informational Macromolecules ■ 699

This replacement creates a 'S t i c k y h y d r o p h o b i c p a t c h ' (contact 4 <


point) on the outer surface at position 6 of (3-chain of both oxy HbS Sticky patch
and deoxy HbS (but not in Hb A)
C o m p le m e n t a r y s t ic k y p a t c h is found in both H b A a n d H b S , but >
is exposed only in their d e o x y g e n a te d (T ) s t a t e . O xy H b S D eoxy H b S
• HbA remains a true solute at rather higher concentrations because of polar exterior surface that is compatable and non reactive
with nearby Hb molecules. In contrast HbS w hen deoxygen ated is less solu ble°.
Thus at low PO 2, deoxy HbS can polymerize to form long, insoluble fibers. These twisted helical fibers d isto rt the erythrocyte
in to characteristic s ic k le sh a p e0.
• Binding of d e o x y H b A terminates fiber polymerization, since HbA lacks the second sticky patch necessary to bind another Hb
molecule.
• In dilute solutions, HbS has interactions with O 2 (P50, Hill coefficient) that are similar to those for HbA. However Bohr effect on
concentrated HbS is more pronounced 1/1 greater release of O 2 in the capillaries & increased propensity for sickling. Sickled RBCs
are less deformable less freely mobile through microvasculature & often block blood flow especially in spleen & joints. These
cells lose water, become fragile, & have a considerable short life span 1/t hemolysis & anemia.
• In h e te r o z y g o u s (H b A/ H b S , s i c k l e c e l l t r a it) individuals, sickling decreases by at least a factor of 1000, thereby accounting
for the asymptomatic natue except during extreme physical exertion. For reasons that remain to be elucidated, h etrozy g osity is
a s so c ia ted w ith an in creased resistan ce to m a la r ia 0, specially for lethal forms like plasmodium falciparum. HbS trait causes
parasites to grow poorly or die at low O 2 concentration b/o low K* levels caused by potassium efflux from RBC on heboglobin
sickling.
•This is the selective advantage that HbA/HbS hetrozygotes exhibit over either normal HbA/HbA or homozygous HbS/ HbS
and offer an explanation for persistence of HbS in gene pool d/t natural selection. Emerging t/t include inducing HbF
expression to inhibit polymerization.
• 90% of human genome variation occurs d/t s i n g l e b a s e c h a n g e i n D N A and is k/a s i n g l e n u c l e o t i d e p o ly m o r p h is m (S N P s
or pronounced " s n i p s " ) . This results in d is a p p e a r a n c e (of already present) or c r e a t i o n of a new r e s t r ic t i o n s i t e by a particular
restriction, endonuclease enzyme.
In either case, cleavage with that restriction endonuclease results in fragments of l e n g t h s d i f f e r i n g f r o m n o r m a l (i.e.
RFLP=restriction fragment length polymorphism), which can be detected by DNA - hybridization (southern bloting). In Hbs,
point mutation abolishes the recognition site of r e s t r ic t io n e n d o n u c le a s e M s t I I that recognizes the nucleotide sequence C C
T N A G G (N=any nucleotide). A to T mutation in (3s globin gene codon eliminates a cleavage site, resulting in generation of
la r g e r 1 .3 5 k g f r a g m e n t s (in place of normal 1.15 kb fragments) on DNA digestion with Mst II restriction endonuclease.

Methods used for Detection of Mutation


Common Techniques Principle Other Techniques Principle
Cytogenetic analysis Unique visual appearance of Single-strand PCR of DNA segment: Mutations
various conformational result in conformational change and
Fluorescent in situ Hybridization to chromosomes polymorphism (SSCP) altered mobility
hybridization (FISH) with fluorescently labeled Denaturing gradient PCR of DNA segment: Mutations
probes gel electrophoresis result in conformational change and
Southern blot Hybridization with genomic (DGGE) altered mobility
probe or cDNA probe after RNAse cleavage Cleavage of mismatch between
digestion of high molecular mutated and wild type sequence
DNA Oligonucleotide Hybridization of PCR products to
Polymerase chain Amplification of DNA segment specific hybridization wild -type or mutated
reaction (PCR) (OSH) oligonucleotides immobilized on chips
Reverse transcriptase Reverse transcription, or slides
PCR (RT-PCR) amplification of DNA segment Microarrays Hybridization of PCR products to
- absence or reduction of wild-type or mutated oligonucleotides
mRNA transcription Protein truncation test Transcription/translation of cDNA
DNA sequencing Direct sequencing of PCR (PIT) isolated from tissue sample
products Pyrosequencing Clonal amplification of single DNA
Sequencing of DNA segments fragments on microparticles followed
cloned into plasmid vectors by massive parallel sequencing
Restriction fragment Detection of altered restriction Multiplex ligation- Quantification of PCR-generated
polymorphism (RFLP) pattern of genomic DNA dependent probe amplicons reflecting the number of
(southern blot) or PCR amplification (MLPA) copies of a specific DNA sequence
products
700 ■ A Complete Review of Short Subjects

Chapter 4. CYTOGENETICS AND MOLECULAR GENETICS, RECOMBINANT


DNA & GENOMIC TECHNOLOGY: REVIEW NOTES

Normal Karyotype & Chromosomal Nomenclature Conventional Cytogenetic Analysis


•The study of chromosomes - Karyotyping - - Theoritically, chromosomal preparations for
Arm R e g io n Band S u b b an d
is the basic tool of cytogenetics. analysis can be obtained from any actively
Chromosome spread is produced by use of dividing tissue by causing the cells to arrest
mitotic spindle inhibitors (eg. colcemid) to in metaphase (the stage of cell cycle when
arrest mitosis in dividing cells in metaphase, chromosomes are maximally condensed).
and then to stain the chromosome. In However, in practice only amniocytes or
metaphase spread every chromosome takes chorionic villi (for prenatal testing) and
the form of two chromatids connected at the blood, bone marrow or skin fibroblasts (for
centromere. A Karyotype is a photograph post natal studies) are used.
(image) of stained chromosome pairs - Analysis of blood, bone marrow & chorionic
arranged in order of decreasing length (in villi involve short term culture technique
metaphase spread stage). Idiogram is details that yield result in 1-3 days; whereas other
of banding of chromosome. tissues require long term culture (1-3 weeks
•Chromosomes have characteristic banding of processing).
patterns created by trypsin and Giemsa stain; - Cells are isolated at metaphase or
hence called G banding. The chromosomes prometaphase and treated chemically or
need to be stained in order to see them with enzymatically to reveal chromosome bands.
a microscope. When stained the Analysis of number of chromosomes and
chromosomes look like strings with light distribution of bands on individual
and dark 'bands'. Each chromosome arm is chromosome allow the identification of
defined further by numbering the bands, the numerical or major structural abnormalities.
higher the number, the further that area is - Each human chromosome has a centromere
from the centromere. /primary constriction (that divides
•Every cell of the human body has 46 X-Chromosome chromosome into short (p) and long (q)
chromosomes occurring in 23 pairs. Chromosomes are laid out in pairs, from largest arms° and is responsible for segregation
(#1) to smallest (#22). The last pair are called the sex chromosomes labeled X or Y. during cell division) and telomeres or
Female have two X chromosomes (XX), and males have an X and a Y chromosomal ends capping the p & q arms.
chromosome (XY). Each chromosome consists of single
• Each chromosomes has a p and q arm; P (petit) is the short arm and q (next chromatid copy of double helix DNA prior
latter in the alphabet) is the long arm°. When a karyotype is made the q arm to DNA replication and 2 identical sister
is always put on the bottom and the p on the top. The arms are separated by a chromatids after DNA replication (until cell
region known as the centromere. So Xp 21.2 means segment located on short division including metaphase)
arm of X-chromosome, in region 2, band 1 and subband 2. a

p
Q
Short arm
Long arm
M K it
+ additional chromosome fl t
i t' <» <
- missing chromosome >J X rr J<
6 7 8 9 10 12
T translocation i»
■■
del deletion M II )(
13 14 15 16 17 18
dup duplication 4*
n\ ! «. T• # »
inv inversion
19 20 21 22
der derivative
r ring chromosome Karyotype
I isochromosome
Biochemistry: Cytogenetics and Molecular Genetics, Recombinant DNA & Genomic Technology ■ 701

Different Types of Banding

G- Banding R - Banding
It is a technique used in cyto genetics to produce visible - It is reverse G banding i.e. the dark regions are
karyotype by staining condensed chromosomes in cells euchromatic, guanine - cytosine rich regions and the
blocked in mitosis. bright bands are hetrochromatic adenine - thymine
The metaphase chromosomes are treated with trypsin (to rich regions.
partially digest the protein) & stained with giemsa. The
acid- saline- giemsa protocol utilizes acetic acid fixation, T- Banding
air drying, denaturing chromosomes mildly with
Visualize telomeres
proteolytic enzymes, salts, heats, detergents or u rea,
and finally giemsa stain. C- Banding
It yields a series of lightly & darkly stained bands. The
Giemsa specifically stain the centromere and other
dark regions tend to be hetrochromatin, late replicating,
regions containing constitutive hetrochromatin i.e. 2 °
A T rich and the light bands are euchromatic, early replicating
constrictions of human chromosomes 1, 9,16, & distal
GC rich.
segment of Y chromosome long arm.
FUdR (Fluro- deoxy- uridine Banding)
FUdR is used to induce fragile sites in chromosomes. Q-Banding
Fragile sites on chromosome undergo breakage at an - Metaphase chromosomes ae stained with a
increased frequency under certain culture conditions fluorescent quinacrine mustard stain to produce
Common or constitutive fragile sites (c- fra) can be temporary Q bands on chromosome. The pattern of
induced by low folate levels. Chemicals such as FUdR, bands is similar to that seen in G banding
aphidicolin and caffeine also enhance c- fra. - It is useful in identifying Y chromosome, Y bodies in
Rare or heritable fragile sites (h-fra) are located on interphase nuclei and various DNA polymorphisms
autosomes (16), and the 17th is on x- chromosome. involving satellites and centromeres.

Molecular Cytogenetics: Fluorescence in Situ Hybridization (FISH)

C on ven tion al cytogen etic kary oty p in g is applicable only to cells that are dividing or can be induced to divide in vitro. This limitation
can be overcome with DNA probes labelled with fluorescent dyes that recognizes chromosome specific sequences - a technique k/ a
FISH. The prim ary adv an tag e o f in terphase FISH is th at it can be p erform ed very rap id ly (usually <24 hours) becau se cell
g row th is n ot required®.

Method Types of FISH & Array Technology & their Utilty


■In principle, FISH is • FISH can be utilized to identity specific chromosomes, characterize de novo duplications or
similar to other DNA- deletions and clarify subtle chromosomal rearrangements. However, its greatest utilization
D N A hybridization in constitutional analysis is in detection o f microdeletions and in cancer cytogenetics for analysis
method in which of structural rearrangements. Though conventional cytogenetic studies can detect some
fluorescent labeled DNA microdeletions, initial detection &/or confirmation with FISH, is essential.
probe is hybridized with • It can demonstrate subtle microdeletions, complex translocations, telomere alterations, &
target DNA to specific rearrangements in metaphase cell that are not readily detectable by routine
counterstain it & allow its karyotyping
detection under
fluorescene microscope • To physically map (localize) a newly isolated gene & to identify extra material of unknown
origin.
■Sample DNA (interphase
nuclei, or metaphase • It diagnose microdeletion syndromes such as Cri-du-chat / Miller - Dieker / Smith-
spread chromosome) is Magenis/Kallman / Williams / Wolf-Hirschhom/Prader-Willi-Angelman/Di-George/Velo
denatured into cardial- facial / CATCH-22 / Shprintzen - syndromes.
complimentary strands - In addition to metaphase FISH, other types include
and fluorescent labelled 1. In te rp h a s e FISH
DNA probe is added. Probe
- It can be used to make a rapid diagnosis in instances when metaphase chromosome
hybridizes with sample DNA
preparations are not yet available; also increases number o f cells available fo r examination , allows
at the target site as the
for investigation of nuclear organization, and provides results when cell do not progress to
sample DNA reanneals
metaphase.
(reforms) back into double
702 ■ A Complete Review of Short Subjects

helix. - When there is strong clinical indication of any trisomy, the aneuploid screen test (uncultured
■Then the probe signals are amniotic fluid interphase FISH study) is performed on amniotic fluid cells, using DNA probes
visualized under fluorescent fo r chromosomes 13, 18, 21 X and Y. The results are obtained usually within 24 -48 hours and will
microscope & sample DNA ascertain -60% abnormalities detected prenatally.
is scored fo r presence or - Interphase FISH is used to enumerate one or more chromosomes in interphase nuclei, as well as to
absence o f the signals. detect specific chromosomal rearrangements characteristic of certain cancers However, its
Types of FISH application is not only limited to interphase nuclei. Many site specific translocations a/w
Probes specific types of malignancies (cancer cytogenetics) can be identified. For example, probes for
both Abelson (Abl) onchogene and break point cluster region (bcr) involved in CM L are
1 .Chromosome libraries labeled in red and green respectively; so the fusion of both genes in CML combines the
(chromosome specific fluorescence and appears as a yellow hybridization signal.
paints) - Oncogene activation by D N A sequence amplification (& overexpression of gene product)
are probes that hybridize to leads to increase in DNA copy number causing cytogenetically recognizable chromosome
sequence that span the alteration k/ a homogenous staining regions (HSRs) if integrated within chromosome or dou ble
entirety o f chromosome from m inutes (d mins) if extrachromosomal. It can be identified through comparative geneomic
which they are derived hybridization, FISH which allow visualization of chromosomal aberrations. Microarray based,
and as a result they can non cytogenetic approaches identify changes in copy number and short tag based sequencing
used to paint individual approaches are used to evaluate amplification. With both microarrays & sequencing
chromosomes technologies, the entire genome can be surveyed for gains and losses of DNA sequences, thus
2.Repetitive probes pinpointing chromosomal region likely to contain gene important in development or
recognize am plified D N A progression of cancer
sequences present in • In addition to standard metaphase & interphase FISH, several enhanced techniques include.
chromosomes. Eg alpha-
2. Spectral Karyotyping (SKY) and multicolor FISH (mFISH)
satellite D N A probes are
complementary to DNA - In, chromosomal painting whole chromosomes can be labeled with a series of fluorescent
sequences found at the DNA probes that binds to a particular chromosome at multiple sites. Chromosomal painting
centromeric regions of all has limited ability to visualize all 46 human chromosomes simultaneously. This hurdle has been
human chromosomes. overcome by spectral karyotyping (or spectacular Karyotyping = SKY) which uses a
combination of 5 fluorochromes & computer generated signals to visualize entire human
3.Single copy probes,
genome.
available d/t human
genomic project, can be as - SKY and m-FISH use combinatorially labeled probes that create a unique color for individual
small as lk b (though chromosomes. It is useful in identification o f unknown chromosome material (eg accessory marker
normally are much larger) chromosomes containing additional material) and most commonly in complex rearrangements
and are packaged into seen in cancer specimen
fosm ids (40 kb) or 3. Fiber FISH
bacterial artificial - Chromosomes are mechanically stretched which provides a higher resolution of analysis than
chromosomes (BACs) conventional FISH
(100 -200 kb).
4. FISH - Comparative Genomic Hybridization (CGH)
- C om parative gen om ic hy brid ization (FISH-CGH) can be used to d etect ch rom osom al
★ The mobility ofd s DNA in differen ces betw een n eop lastic cells an d th eir c o u n t e r p a r t s It can quickly and efficiently analyse
gel electrophoresis copy number variations (CNVs) in relation to ploidy level in a test sample DNA compared to
depends on strand size & another reference sample (which is mostly related). It detects amplification or deletions of
length but is relatively smaller DNA region or whole chromosome by comparing DNA of a patient (target) with a
independent o f nucleotide normal person.
sequence. However, - It can be used only when DNA in available from a specimen of interest (eg cancer). The entire
mobility o f single stranded DNA specimen from the sample of interest is labeled in one color (eg green) and the normal
DNA is highly affected by a control DNA specimen is labeled by another (eg red) color. These are mixed in equal amounts
small change in nucleotide and hybridized to normal metaphase chromosomes. Computerized analysis of (red to green)
sequence because color ratio determines where the DNA of interest may have gains or losses of material.
relatively unstable SS
DNA forms intrastrand 5. Array - CGH (a-CGH, Chromosomal Microarray Analysis / CMA, Virtual Karyotyping)
base pairing, loop & folds - Similar to FISH - CGH, except that the test DNA is hybridized to DNAs that are spread on
that give the single strand arrays, rather than hybridized to normal chromosomes.
a unique 3rd structure, - C om paritive gen om ic h y brid ization (CGH) directly compares the DNA, content o f differentially
regardless of its length. labelled normal & tumor cells by their co- hybridization to normal metaphase chromosome
Single stranded spread or to series of genomic DNA clones aligned on glass slides (array CGH). In this way
Biochemistry: Cytogenetics and Molecular Genetics, Recombinant DNA & Genomic Technology ■ 703

c o n f o r m a t io n tumor specific alterations in gene copy number can be ascertained.


is - It provides a higher resolution (upto 50kb). C G H a r r a y s utilize either BACs or o li g o n u c l e o t i d e s ,
p o ly m o r p h is m ( S S C P )
electrophoretic seperation whereas g e n o t y p in g a r r a y s utilize single nucleotide polymorphisms ( S N P a rr a y s ). It is used in
of single stranded nucleic prenatal diagnosis & study of cryptic ch rom osom al im balan ces in p atien ts w ith m en tal
acids based on their retardation an d m u ltiple con gen ital an om alies. It is also used to d etect m icrod eletion s an d
differences in nucleotide m icro d u plication s in cancer an d p rev iou sly u nidentified gen om ic disorder®. This technology
sequence (often a single is still in development, but it will ultimately be the i n i t i a l m e th o d o f s t u d y f o r c o n s t i t u t io n a l
base pair) that results in a a b n o r m a litie s .
different secondary
★ Both M A P H ( m u lt ip le x a m p li f i c a t i o n & p r o b e h y b r i d i z a t i o n ) and M L A P ( m u lt i p l e x lig a t io n
structure & a measurable
d e p e n d e n t p r o b e a m p li f i c a t i o n ) techniques for detection of small scale copy number changes
difference in its mobility
& gene deletions rely on sequence specific probe hybridization to genomic DNA followed by
through gel. So the SSCP
PCR amplification of hybridized probe and semiquantitative analysis of resultant PCR
can detect point mutations.
product. The relative peak heights / band intensities indicate initial concentration of each
target.

Barr Body

• When two X chromosomes are present in a cell (as in a normal female) one of them becomes inactivated and condensed
on the nuclear membrane and is called the 'B a r r b o d y '. This process is termed as X chromosome lyonization (inactivation)
• Presence or absence of Barr bodies helps in ascertaining the sex o f an individuals
- Absence o f Barr body indicates that the patient has only one X chrom osom e (eg. Normal m ale XY or Turner’s syndrome(XO))
- Nuclei of cells in females (XX) contain a darkly staining Barr body that is not present in the nuclei of cells in males.
• Barr bodies are most easily seen in a smear of squamous epithelial cells obtained by s c r a p i n g th e b u c c a l m u c o s a ®
• Barr bodies react differently to histological stains and are best seen as dark staining bodies with in the nucleus of non
dividing interphase cells.

Fluorescence Recovery After Photobleaching (FRAP) Fluorescence Resonance Energy


Tranfer (FRET)
Lateral diffusion can be shown experimentally by attaching f lu o r e s c e n t p r o b e s to
- F l u o r e s c e n t p r o b e s derived from
the head groups of lipids/proteins and using the f lu o r e s c e n c e m ic r o s c o p y to
naturally occurring fluorescent protein,
follow the probes over time. Two techniques involve
the g r e e n f lu o r e s c e n t p r o t e in ( G F P ) of
jelly fish Aequorea victoria or other
F lu o r e s c e n c e T a g g in g o f M e m b r a n e S i n g l e P a r t ic le T r a c k i n g
variants like B F P ( b lu e ) , Y F P ( y e llo w ) ,
S u rfa ce
C F P (c y a n ) along with f lu o r e s c e n c e
• Lipids (protein etc) on the outer leaflet •The motion (movement) o f a single m ic r o s c o p y are used to visualize
(layer) of membrane are l a b e l e d b y fluorescently labeled lipid (protein/ biochemistry of living cells by means of
reaction with a membrane -impermanent etc) molecule in the cell membrane F R E T (flu o re s c e n c e r e s o n a n c e e n e rg y
f lu o r e s c e n t p r o b e , s o t h e s u r f a c e is is recorded on video by fluorescence t r a n s f e r ) . FRET can be used to m e a s u r e
u n i f o r m ly l a b e l e d w h e n v ie w e d w i t h microscopy on a much shorter time c A M P a n d a c t iv it y o f P K A (p y r u v a te
f l u o r e s c e n c e m ic r o s c o p e . A small area scale (eg 50 ms = 2250 frames with k i n a s e A ).
(5|im2) of a cell surface with fluorescent time resolution of 25gs=40000 - When two molecules are v e r y c l o s e
tagged lipids is p h o t o b le a c h e d b y i n t e n s e frames/s). ( w i t h i n 1 to 5 0 A ° ), the energy of excited
l a s e r r a d i a t i o n so that the irradiated patch donor molecule passes directly to near by
•The pattern of movement
n o l o n g e r f lu o r e s c e s (emit fluorescent acceptor molecule without emission of a
confirms r a p id l a te r a l d i f f u s i o n
light) when viewed with less intense (non photon, exciting the acceptor (the
w i t h in a c o n f in e d ( s m a ll d is c r e t e )
bleaching) light in the florescence efficiency of FRET is in v e r s e ly
r e g i o n w i t h o c c a s io n a l h o p s into
microscope. p r o p o r t io n a l to 6 th p o w e r o f d is ta n c e
an adjoining region. It suggests
b e t w e e n d o n o r & a c c e p to r ) . The acceptor
• However, the photo bleached membrane that lipids are corralled by
molecular fences that they can now decays to its ground state by
region r e c o v e r s i t s f lu o r e s c e n c e w i t h in
f lu o r e s c e n c e (which can be located to
m il l i s e c o n d s as unbleached lipid occasionally jump by h o p
specific regions of single living cell); the
molecules diffused into the bleached patch d if f u s i o n . Membrane proteins (like
em itted p h oton h as lo w er energy (longer
and bleached lipid molecules diffuse away glycophorin & C l - H C O 3 - exchanger
in RBC) immobilized by their w av e length) than both exciting light and
from it by rapid lateral diffusion.
association with spectrin (a the flu orescen ce em ission ofdonori®.
• The r a te o f f l u o r e s c e n c e r e c o v e r y a f t e r
filamentous cytoskeleton protein)
704 ■ A Complete Review of Short Subjects

p h o to b le a c h in g o r F R A P is a m e a su re o f form the fences that define the Atomic Force Microscopy (AFM)
r a te o f la te r a l d if f u s i o n of the lipids regions of relatively unrestricted In AFM, the s h a r p t ip o f a m ic r o s c o p e
lipid motion. p r o b e attached to a (flexible) cantilever is
• FRAP can be used to assess structure o f
membranes (biological & artificial both), m o v ed a cro ss a n u n e v e n su rfa ce eg
define fluid mosaic model and investigate m e m b r a n e . E le c t r o s t a t ic & v a n d e r W a l ls
diffu sion (m otion) o f b iolog ical between the tip and sample
in te ra c tio n s
m acrom olecu les including lateral produce a force that moves the probe or
diffusion m ovem ents w ith in the p la n e o f sample up and down (in z dimension) as
m em branes and m ovem ents into & ou t o f H o p D i f f u s i o n o f S i n g l e L ip id
the probe encounters hills (up) and valleys
nucleusQ. M o le c u le (downs) in the sample. A laser beam
reflected from the cantilever detects motion
Photobleaching of small area of 1A°. A series of scans in plane of
by intense laser beam membrane (x & y dimensions) yields a
t h r e e d im e n s io n a l c o n t o u r m a p o f t h e
with resolution near the atomic
su rfa ce
Labeled by scale - 0.1 nm in vertical and 0.5 to 1 nm in
lateral dimensions. When several images of
Fluorescent J — Q individual units are superimposed with
Normal cell Pr°be compute, a h i g h r e s o l u t i o n im a g e o f
membrane p r o t e in is yielded. So AFM is u s e d to
v i s u a li z e m e m b r a n e p r o t e in s such as
m e m b r a n e (m ic r o d o m a in s ) r a f t s (enriched
Uniformly labeled Non fluorescent Recovery of in sphingolipids, cholesterol & GPI linked
view on fluorescence irradiated fluorescence proteins) or s i n g l e m e m b r a n e p r o t e in such
microscope (FM) photo bleached d/t lateral as bacteriorhodopsin, aquaporin & Fo
patch (on FM) diffusion (proton driven rotor of chloroplast ATP
synthase) etc.

Movements of Lipid In Membrane Bllayer

Uncatalyzed Lateral Diffusion Catalyzed Transbilayer Translocations

Lipid and protein molecules d if f u s e la t e r a lly to facilitate


• U s e t r a n s lo c a t e r s ( f l i p p a s e s , f lo p p a s e s & s c r a m b la s e s )
(o r u n d e r g o B r o w n ia n m o v e m e n t w i t h in t h e transbilayer movement of lipid by a path that is much f a s t e r a n d m o r e
b i l a y e r ) by changing places with e n e r g e t i c a l l y f a v o u r a b le .
neighbouring molecules at very fast rate • The combination of asymmetric biosynthesis of membrane lipids (ie
(lpm/s). This rapid uncatalyzed lateral diffusion different lipids on different sides), slow flip flop diffusion and presence
in the plane o f membrane bilayer tends to of selective energy dependent translocaters is responsible for
randomize the position of individual transbilayer asymmetry in lipid composition, creating a larger surface on
molecules in a few seconds. one side o f membrane and generating curvature essential in budding o f vesicles.
• 3 - ty p e s o f t r a n s lo c a t e r 's in c l u d e

I---------------------------------- 1------------------------------- 1
i ,Lm -Li./-!* 1r Flippase Floppase Scramblase
.w M m u m
Very fast
-Structurally & functionally •Are members of - Are proteins that move
(1pm/s)
related to P t y p e A T P a s e A B C t r a n s p o r te r any m e m b r a n e
(active transporters); move f a m i l y ; all of p h o s p h o l i p i d across the
Uncatalyzed Transbilayer (Flip-Flop) PE which actively membrane d o w n it s
Diffusion (phosphatidylethanolamine transport c o n c e n t r a t i o n g r a d ie n t
) & P S (phosphatidyl hydrophobic (from leaflet where it is i
Transbilayer (flip-flop) diffusion of a lipid serine) f r o m o u te r substrates higher concentration to
molecule from one leaflet of membrane to ( e x t r a c e llu la r ) to c y t o s o lic outward across the leaflet where it has a
other occurs v e r y s lo w ly (ti/2 in days) ( lu m e n a l) l e a f l e t (s id e ) o f plasma lower concentration), ie
because it requires polar (charged) head p la s m a m e m b r a n e . They membrane. in e i t h e r d ir e c tio n
group to leave its aqueous environment and consume about 1 ATP per ■Flippase to w a r d s e q u i li b r i u m .
move into hydrophobic interior of membrane, molecule of phospholipid transport plasma -They d o n o t r e q u i r e A T !
a process with large, positive free energy translocated membrane b u t a r e a c t iv a te d b y
Biochemistry: Cytogenetics and Molecular Genetics, Recombinant DNA & Genomic Technology ■ 705

change. However, in the endoplasmic reticulum - Keeping PS out of phospholipids increase in cytosolic
this movement is essential for lipids to get extracellular surface protects from cytosolic calcium (Ca++), which
from their site of synthesis to their eventual cells from apoptosis (inner) to may result from cell
point of deposition as membrane (programmed cell death) by extracellular activation, cell injury or
glycerophospholipids are synthesized on cytosolic protecting against (outer) leaflet apoptosis.
surface whereas sphingolipids are synthesized/ macrophages that carry PS and use ATP
modified on luminal surface ofE R membrane. receptors.

Luminescence

I t is th e e m is s io n o f lig h t o r r a d ia n t e n e r g y w h e n a n e le c t r o n retu rn s fr o m a n e x c ite d o r h ig h e r en erg y le v e l to a

lo w e r en erg y l e v e l ® . It c a n b e caused by chem ical or bioch em ical reactions, electrochem ical reaction, electro
m agnetic radiation (photons), s u b a t o m i c m o v e m e n t s o r s tr e s s o n c r y s t a l . It is a cold body radiation a s t h e
e m i s s i o n o f l i g h t b y a s u b s t a n c e is n o t r e s u l t i n g f r o m h e a t i n g , w h i c h d i f f e r e n t ia t e s it f r o m incandescence (ie l i g h t

e m i t t e d b y a s u b s t a n c e a f te r h e a t i n g ) . It c a n b e o f v a r i o u s t y p e s .
,__________________________ I__________________________,
Chemi/Bio/Electrochemiluminescence & Luminometry Photoluminescence
•These are types of luminescence in which the excitation event is It is a kind of luminescence where energy is supplied
caused by a chemical, biochemical or electrochemical reaction and by photons (electromagnetic radiation). It can be of 2
not by photo illumination (or photoluminescence)® as seen in types
fluorescence.
• Physical event for light emission in chemi / bio / electro- I. Phosphorescence & Phosphometry
chemiluminescence is similar to fluorescence in that it occurs from an - It is a type of luminescence produced by certain
excited singlet state, & the light is emitted when the election returns
substances after absorbing radiant energy or other
to the ground state
type o f energy®
It is distinguished from fluorescence in that
•This type of light emission is measured by luminometry
1. It continues even after radiation causing it has
I ceased
Chemiluminescence Bioluminescence Electiochemilumine- 2. The decay time of emission of
- It is emission of light - It is special form scence (ECL) phosphorescence light is longer (1(H to 10 2 S)
when an electron ®f - It differs from 3. Phosphorescence shows a larger shift in
returns from an chemiluminescenc chemiluminescence in emitted light wavelength than fluorescence.
excited or an higher e found in that the reactive species Phosphometry measures phosphorescence
energy level to a lower biological that produce electro
energy level systems® chemiluminescent Fluorescence & Flurometry
- Excitation event is - In it an enzyme or reaction are electro Fluorescence is emission of electromagnetic
caused by chemical photoprotein or chemically generated radiation by a substance after the absorption of
reaction & involves biological from stable precursors energy in some form (eg the emission of light of one
oxidation of an catalyst (eg at the surface o f an color / wave length when a substance is excited by
organic compound luciferase & electrode® irradiation with light of a different wavelength. In
such as luminol, acquorin) - Ruthenium (Ru+2), tris other words fluorescence occurs when a molecule
isoluminol, increase the (bipyridyl) chelate is absorbs light at one wave length and reemits light
acridinum ester, or efficiency of the M.C. used ECL a t a longer wave length (or lower energy)®
luciferin by an luminescence label & ECL is It is differentiated from phosphorescence in that its
oxidant such as reaction generated at an life time is < 10 mili seconds after excitation ceases.
Eg. fluorescent paint stops glowing as soon as UV
706 ■ A Complete Review of Short Subjects

H 2O 2, hypochlorite, - The quantum electrode via oxidation lamp is turned off but phosphorescent paint keeps
or O 2 yield is ~ 0.1 - reduction type of glowing for a while.
- Light is emited from 1 0 % for reaction with An excited fluorescent molecules such as g r e e n
the excited product chemilumi­ tripropylamine. f lu o r e s c e n t p r o t e in ( G F P ) o r Y F P o r C F P etc can
formed in oxidation. nescence and 10 - This chelate is dispose of the energy from absorbed photon by
These reactions 30% for relatively small, very either__________________________________________
occur in presence bioluminescence stable & has been used 1 .F lu o r e s c e n c e ie em itting a p h oton o f slightly
catalyst such as to label haptens or longer w a v e length (low er energy)® or
enzymes (eg alkaline large molecules (eg 2 .N o n r a d ia tiv e fluorescence resonance energy
phosphatase, protein or transfer ( F R E T ) , where energy of excited donor
horseradish oligonucleotides) passes directly to a nearby (within 1 to 50 A0)
peroxidase, & ECL is used both in acceptor molecule without emission of a photon,
microperoxidase), immunoassays and exciting the acceptor. The acceptor ow decays to
metal ion and metal nucleic acid assays. It ground state by fluorescene; emitted photon has
complexes (Cu++ & has the advantage of lo n g e r w a v e le n g t h ( lo w e r e n e r g y ) than both
Fe+3 phthalocyanine improved reagent original exciting light & donor emission.________
complex) & hemin stability, simple An atom / molecule that fluoresces is termed
reagent preparation f lu o r o p h o r e
and enhanced is measurement of emitted
F lu o r o m e t r y
sensitivity. fluorescence light.

Bioluminescence Cycle

• B io lu m i n e s c e n c e is a special form o f chemiluminescence found in biological system®.


• Bioluminescence requires considerable amount of energy. In firefly, ATP is used in a set of reactions that convert chemical
energy into light energy®
• The light is produced through a chemical reaction involving PPi
l u c i f e r i n , (a heat resistant complex carboxylic acid
substrate), l u c i f e r a s e (enzyme), A T P and O 2
• The generation of light flashes requires activation of
luciferin by pyrophosphate cleavage of ATP to form
l u c i f e r y l a d e n y la te . In the presence of molecular oxygen
(0 2 ) and luciferase, the luciferin (adenylate) undergoes a
multistep oxidative decarboxylation to oxyluciferin This
process is accompanied by emission of light. A
• Colour of flashes differ in different firefly species and seems > O xyluciferin
to be determined by difference in the structure o f luciferase.
Luciferin is regenerated from oxyluciferin in a subsequent
series of reactions.
• It has the highest quantum efficiency (Qc= 88 %). The
quantum yield (total photons emitted per total molecule
reacting) is ~ 0.1- 10% for chemiluminescence and 10-30% for bioluminescence
• Since the light producing reaction is d e p e n d e n t o n A T P . It allows researchers to determine the presence of ATP and hence
the p r e s e n c e o f l if e .
• In lab, pure firefly luciferin and luciferase are used to measure minute quantities of ATP by the intensity of light flash
produced. As little as a few picomoles (1 0 12 mol) of ATP can be measured in this way
• This method is used by NASA to f i n d l i f e o n o t h e r p la n e t s . It is also used for a n t i b i o t i c s u s c e p t i b i l i t y t e s t in g . If light is
produce on mixing above mixture to cultured sample of TB bacteria and antibiotic, it indicates the failure of drug and
presence of growing and thriving bacteria.
• In g e n e a c t iv a tio n t h e r a p y , if light is visible on ultraviolet cameras, this means that genes treated with luciferase are active.
Biochemistry: Cytogenetics and Molecular Genetics, Recombinant DNA & Genomic Technology ■ 707

Spectrophotometer : Lambert Beer Law & Absorption of Light by Molecule


• Biomolecules absorb light at characteristic wave lengths, just as tryptophan, tyrosine and to a much lesser extent
phenylalanine absorb u ltr a v io let light n ear a w av elen g th o f 280 nmQ.

Select and transmits 'o transmitted light r '


light of particular -''J V W W V Y O
wave length Intensity of 1
Lamp emits incident light Dectector
Monochromator Sample
broad spectrum
light cuvette (I = path length)
with c moles/litre of
absorbing molecules

The fraction of incident light absorbed by solution at pven wavelength depends on thickness of absorbing layer
(pathlength) and concentration of the absorbing species. So Lambart Beer Law is Absorbance = log _P = eCl
__________________ I
where e is molar extinction coefficient and it has been assumed that incident light is parallel, mono chromatic (of single wave
length) and solvent & solute are randomaly oriented. So each successive millimeter path o f absorbing solution absorbs not a constant
amount but a constant fraction o f light. However, w ith fix e d p ath length absorbing layer, absorbance is directly proportional to the
concentration o f absorbing solute®. So it can be used in qu atitative estim ation o f DNA, RNA or am inoacids0.

Restriction Enzyme / Restriction Endonucleases (RE)

- These are originally called restriction enzym es because their presence in a given bacteria restricted the g ro w th o f certain
b a c teria l viruses (i.e. bacteriop h ag es)Q. This defensive enzym e protects the host bacterial DNA from the DNA genome of
foreign organisms (particularly infective bacteriophages) by specifically in activ a tin g th e in v ad in g p h a g e DNA by d ig estion 0.
- Werner Arber discovered that their biological function is to recognize an cleave foreign DNA (eg the DNA of infecting virus) and
such DNA is s a id to b e restricted. R E restrict DNA rep licatio n (v ira l-rep lica tio n ).&
- However, REs are found only in cells that also contain site specific D N A m ethylase enzym e-w h ich m eth y la tes th e h o s t DNA,
rendering it an u n su itable su bstra te f o r d igestion by restriction en zym es (and thus p ro tectin g it fr o m RE)Q. So restriction
enzymes and site specific DNA methylase are companion enzymes an always exists in pairs in a bacterium - and sometimes
called as restriction m odification system .
- Immense size of DNA was the main obstacle in its molecular analysis, which was over come by a group of bacterial enzyme
called restriction endonucleases (restriction enzymes). RE cleav es d ou b le stran d ed (d s) DNA in to sm aller, m ore m an a g eab le &
p recisely d efin ed DNA segm ents 6 called restriction fragm ents. RE used as m olecular scissors or chem ical knife is the key
enzyme in recombinant DNA technology or genetic engineering. There are 3 types of Res
- Restriction endonucleases recogn ize an d c lea v e (cu t o r sp lice) d ou b le stran d ed DNA in a p red icta b le m anner a t sp ecific
sequ en ces0 k/ a (recognition sequences or restriction sites) that are typically 4-6 (Lehinger) /4-7 (Harper) bp long, to generate
a set of smaller fragments. Restriction enzymes are endonucleases ie they break nucleic acid chains somewhere within the
DNA rather than at the ends of molecule.
- To cut DNA, a RE makes 2 in cision s through each su g ar - p h o s p h a te b a c k b o n e (i.e. p h o s p h o -d iester bon d o f ea ch stran d) o f
d ou b le stran ded DNA0. The average size of D N A fragm ents produced by cleaving genomic DNA with RE depends on the
frequency with which a specific restriction site occurs in DNA; which intum depends on their size. In a DNA in which all 4
nucleotides are equally abundant [i.e. (C=G) = (A = T) =50%] and randomly distributed, a RE tht recognizes a 6 bp sequence
cuts once every 4096 (46), whereas another RE that recognizes a 4 bp sequence cuts once every 256 bp (44) and so produces
smaller DNA fragments. The fragment size produced by RE cleavage can be increased by simply terminating the reaction
before completion - partial d igest - or by using special class of endonuclease called hom ing endonuclease (by group I
introns) which cleave 14-20 bp DNA sequence. However, in natural DNA the recognition sequence tend to occur less frequently
because neucleotides are neither random nor equally abundant.
- If a piece of DNA from a species is made to react with a specific RE, a characteristic array of cut DNA fragments will be
produced, this is k/a restriction m ap. The ends of all fragments have same DNA sequence. These restriction fragments of
known size are isolated (enriched) by electrop h oresis on a g a ro se o r p o ly a cr y la m id e gel® or H P L C . Restriction map serves as
DNA fingerprint because the fragments of DNA from one organism will have a characteristic pattern.
708 ■ A Complete Review of Short Subjects

S p e c ific ity S tic k y & B lu n t e n d s R e s tr ic tio n s ite &


RE cleaves DNA so as to produce a 3' - OH group on n o m e n c la t u r e
- Restriction endonucleases have specific
one end & 5 1 - P O 4 group on other end. DNA cut results A DNA sequence
recognition sites with p a lin d r o m e
recognized by RE
a r r a n g e m e n t where they cut ds DNA
is called restriction
(at specific DNA sequence )Q with in
S tic k y / C o h e s iv e / O v e r l a p p i n g B lu n t e n d s site. RE that
the molecule producing short
S ta g g e re d e n d s Cleaves both recognizes 4 base
fragments (other enzymes cut DNA
- Produced due to s t a g g e r e d c u t strands of DNA at pair sequences
randomly)
(by Taql, Bam HI), resulting in the opposing produces more cuts
- RE recognizes short stretches of DNA
DNA fragments having single p h osp h od iester and shorter
a b o u t 4 to 7 b ase p airs long (usually
stranded (SS) sequences that are bonds®, leaving no segments than the
having 4 or 6 base pairs)
complementary to each other. unpaired bases on RE requiring
- These sequences differ for each RE and
- This leaves 2-4 nucleotides of the ends. sequence of 6 base
are p a lin d r o m e s , i.e., they show two
one strand unpaired at each Produced d/t pairs.
fold rotational symmetry. P a lin d r o m e
resulting end. These unpaired straight cut, (by The 1st letter of
(meaning to r u n b a c k w a r d s ) are also
strands are called s t ic k y - e n d s Hpal, Hae III) that name of RE is
known as in v e r te d r e p e a t s e q u e n c e s .
because these can base pair are double stranded from the genus o f
It is similar to a word that reads
with each other or with & therefore donot bacterium from
backwards or forwards similarly eg
complementary sticky ends of form hydrogen which it is isolated
madam. Which means with in a short
other DNA fragments. bonds with each (E). Next two
region of double helix, the nucleotide
- D N A l ig a s e joins sticky ends other. letters are from
sequence on one strand, read 51 —> 3/
produced by same RE B a c te r io p h a g e T 4
name o f species (Co).
is identical to that of the 2nd strand, also
- T e r m in a l t r a n s f e r a s e enzyme The additional
read in 5 1 —> 3' direction. So, if rolated l ig a s e can
add new ends & change it into covalently joint subscript letter
180 degrees around the axis of
blunt end. blunt ended indicates the type
symmetry - the structure remains
fragments._________ or strain (R) and a
same.
final roman number
G G c c indicate the order
c c G G
in which the enzyme
was discovered in
Straight cut by Hae III that particular
I organism (I) eg.
G G C C ECoRI.
C C G G

Sticky ends Blunt ends

- After isolating target DNA (restriction) fragment, it is joined to similarly digested cloning vector (i.e. vector digested by some
RE) by DNA ligase® (which catalyzes formation o f new phosphodiester bonds by using ATP). Fragments produced by 2 RE (eg EcoRI
& Bam HI) will not link. New DNA sequences can be created by inserting synthetic DNA fragments ( li n k e r s ) b/w the ends
that are being ligated. P o ly l i n k e r s are inserted DNA fragments with multiple recognition sequences for RE (often useful later
as points for inserting DNA by cleavage & ligation). So it has an important role in recombinant DNA technology and preparing
chimeric DNA molecules.
- Ligating sticky ends is technically easy but is a/w the problems like sticky ends may reconnect with themselves, without net
gain of DNA, forming heterogeneous tandom inserts or sticky end sites may not be available in a required (convenient)
position. To escape these problems blunt ends can be generated. Blunt ends can be ligated directly (however not directional)
by b a c t e r io p h a g e T 4 l ig a s e - a technique though less efficient than sticky end ligation, has advantage of ligation of any pairs
of ends. 2 alternatives to circumvent problems listed above include
1. H o m o p o ly m e r t a il i n g by t e r m in a l t r a n s f e r a s e i.e. if poly d (G) is added to vector and poly d (C) is added to 3' end of foreign
DNA, the two molecules can ony link to each other.
2 .Adding synthetic sticky ends ( li n k e r s )
- Some pro /eu-karyotic r e c o m b i n a s e s (like CRE recombinase identifying bacterial lox P sites or Flp recombinase identifying yeast
FRT sites) are used as an adjunct to RE to catalyze DNA in corporation.
Biochemistry: Cytogenetics and Molecular Genetics, Recombinant DNA & Genomic Technology ■ 709

Polymorphism

Any two nonrelated individuals share 99.9% of their DNA sequences. So the remarkable diversity of humans is encoded in only
0.1% of DNA (~ 3 million base pairs). Genomic variations are differences in sequence of DNA among individuals. And genomes
of non-related people differ at about 1 o f 1200 to 1,500 DNA bases (or 0.1% o f genome). These variations are due to.
I-------------------------------------------------------------------------------------------1
Polymorphism Mutations
- It is n o r m a l, c l i n i c a l ly h a r m le s s - It is i n f r e q u e n t (< 1 % ) b u t p o t e n t i a l l y h a r m f u l g e n o m e v a r i a t i o n th a t is
v a r ia t io n s o f D N A s e q u e n c e , t h a t d o e s a s s o c ia t e d w i t h s p e c if i c h u m a n d is e a s e
n o t a f f e c t t h e p h e n o ty p e Q - Single mutation may 1/t many end effects ( p l e io t r o p i s m ) or mutations at
- Traditionally defined as s e q u e n c e several genetic loci may produce the several genetic loci may produce the
v a r ia t io n a t a g iv e n lo c u s ( a ll e l e ) in same trait ( g e n e t ic h e t e r o g e n e i t y ) .
> 1 % p o p u la t io n . - Structural alterations of P-globin
- It occurs ~ once in every 500
nucleotides or about 107 times per Alteration Disease Function affected
genome due to deletion, insertion or Point mutation Sickel cell disease Protein folding
single base substitution occuring in |3-Thalassemia Transcription control, Frame shift &
non-coding region like introns & non sense mutation RNA processing.
intergenic region or at sites that cause Deletion P°- Thalassemia MRNA production
no change in function of encoded Hemoglobin lepore
protein.
Rearragnement P-Thalassemia type III MRNA production
- So polymorphism is single phenotype
single locus and multiple normal
alleleQ.

Restriction Fragment Length Polymorphism (RFLP)


Human genome contains s e q u e n c e v a r ia t io n s a t a g iv e n l o c u s ( a ll e l e ) th a t d o n o t a f f e c t p h e n o ty p e i n > 1 % p o p u la t io n .
This property of the molecule to exist in > 1 form is k/a p o ly m o r p h is m . So polymorphism is p r e s e n c e o f s i n g l e n o r m a l
p h e n o t y p e , s i n g l e l o c u s (s ite ) a n d m u l t i p l e a l l e l e ( s e q u e n c e v a r ia t io n s ) in >1% of population. Polymorphism is a normal
variation and is clinically harmless. Whereas mutation is infrequent, (seen in <1%), abnormal, and potentially deleterious genetic
variation that may lead to pleiotropism (one mutation - many effects) or genetic heterogeneity (many mutation - one effect).
R e s t r i c t i o n f r a g m e n t le n g t h p o ly m o r p h is m (R F L P ) is a g e n e t i c v a r i a t i o n t h a t c a n b e o b s e r v e d b y c l e a v i n g t h e D N A in to
f r a g m e n t s ( r e s t r ic t io n f r a g m e n t s ) w ith a r e s t r ic t i o n e n d o n u c le a s e . The existence of two or more types of restriction
fragment patterns is called RFLP. The length of restriction fragment is altered if the genetic variation (polymorphism,
mutation etc) alters the DNA so as to create or abolish a site of restriction endonuclease cleavage (a restriction site).
Differences in DNA sequence ( p o ly m o r p h is m o r m u t a tio n ) can create or abolish a site o f restriction endonuclease cleavage (a
r e s t r ic t i o n s ite ) and thus result in variation in length o f restriction fragments. If DNA of an individual has g a in e d a r e s t r ic t io n
s it e , then restriction enzyme cleavage yields at least one additional fragement. And if mutation cause a l o s s o f r e s t r ic t i o n s it e
few er fragments are produced. Two types of DNA variation that result in RFLP are.

Single nucleotide polymorphism (SNPs, pronounced Length polymorphism/


Snips) or site polymorphism Tandem repeats
It is DNA variation that just involves single base. SNPs Human DNA contains short repetitive sequence of non
are responsible fo r 90%> o f human genome variation. coding DNA. This v a r ia b le num ber o f tandem (one after
Typically the SNPs are biallelic (i.e., only two choices an other) rep eats ( V N T R ) , is u n i q u e f o r a n y i n d iv id u a l &
exist at a given site) and can occur anywhere in t h e r e f o r e s e r v e a s a m o l e c u l a r f i n g e r p r in t
genome - within intron, exon or intergenic regions. It may be of two types
However, <1% o f SNPs occur in coding region. I
Serve both as physical landmark within the genome & Microsatellite DNA polymorphism Minisatellite repeats
as a genetic marker whose transmission can be - S h ort (2-6 b a se pairs) in herited Larger (1-3 kb), &
followed from - parent to child. It is genetic marker o f tandem rep eats o f u sually < lkb® repeat motifs is
choice fo r study of complex genetic traits. - Occurs ~ 50,000-1 lac times in usually 15-70 base
human pairs.
710 ■ A Complete Review of Short Subjects

5 '— A - [ c ] -------------G ------3' = Individ u al I | ca ca cacaca ca |


A lle le A
5’— A - [ a ]------------ G ------3' = Individual 2
SN P jc A C A C A C A j
A lle le B
I
7.0 K b ----- J ca c a |
A lle le C
1 (M utan t)
A = d ise a s c , B & C == n o rm al

7.5 Kb
r— “1 ;
2 (N orm al)

7.5 Kb N orm al allele


P e rso n A P e rso n B P e rso n C
7 .0 Kb M utant allele
S o u t h e r n b lo t a n a ly s is
C a r r ie r A f f e c te d N orm al

S o u th e r n b lo t a n a ly sis

Procedure
DNA sample is treated with restriction endonuclease Gene Unknown [ 3' Intact DNA

enzymes which cleaves DNA in restriction fragments of


O verlapping fragm ents
defined length. These fragments are separated by same DNA with gene
agarose gel electrophoresis on the basis of size. . } ! be known in fragment 5
Detection of RFLP relies on southern blotting
Chromosomal W alking
hybridization procedure. DNA is denatured by soaking
agarose get in alkali, and then blotted on to nylon
(nitrocellulose) paper
to reproduce the distribution RFs in gel. This nitrocellulose membrane is immersed in radioactively labeled DNA probe (southern-
blotting). Autoradiography (x-ray exposure to film) reveals the fragments to which the probe hybridizes. The method is very
accurate: However, it requires large samples o f undergraded DNA (>25ng) that is often not available at crime scene.
• Clinical Applications of RFLP
- Tracing chromosome from parent to offspring: An individual who is heterozygous for a polymorphism has a sequence
variation in the DNA of one chromosome and not in the DNA of companion (homolgous) chromosome. In such cases each
chromosome can be traced from parent to offspring by determining the absence or presence of polymorphism. This is useful
in disputed parenthood.
- Prenatal diagnosis of genetic disease: Disease will produce alteration in size & distribution of RE fragments of DNA
obtained from fetus (blood, amniotic fluid or chorionic villi). Examples include___________________________________________
1. Sickel cell anemia can be directly diagnosed by RFLP, as the point ( A —>T ) mutation abolishes recognition site o f restriction
endonuclease mst II that recognizes nucleotide sequence CCTNAGG (N = any nucleotide). Normal DNA digested with Mst
II yields a 1.15 kb fragment whereas a 1.35 kb fragment is generated from (3sgene.
2. Phenyl ketonuria can be indirectly diagnosed using RFLP. Mutation in phenylalanine hydroxylase (PAH) gene do not
directly affect any RE site. To establish diagnosis one has to analyze DNA o f affected family member with a goal to identify
genetic markers or variants (RFLP) that are tightly linked to the disease. Once these polymorphism marker that are consistently /
closely linked to a disease producing mutation gene - (ie they closely show close linkage and are coinherited) are
identified. The RFLP analysis can be carried out for prenatal diagnosis.
- RFLP results from single base changes /mutation (eg sickle cell disease) or from deletions or insertions (copy number
variations = CNVs) of DNA into a restriction fragment (eg thalssemias) and have proved to be useful diagnostic tools. Thy
have been found at known gene loci and in sequence that have no known function; thus RFLPs may disrupt the function of
gene or may have no apperent biological consequence.
- RFLPs/ SNPs are inherited in m endilian fashion, so these can be used in definition o f inherited disease in which the functional
deficit is unknown. RFLPs / SNPs can be used to establish linkage groups, which in turn by the process of chromosomal
walking, will eventually define the disease locus. Chromosomal walking involves repeated cloning o f overlapping DNA
segments (of 100-200 kb) to isolate/determine any gene in unknown location o f large piece o f DNA. In chromosomal walking, a fragment
representing one end o f long piece o f DNA is used to isolate the next overlapping segment. The direction of extension is
determined by restriction mapping and the procedure is repeated sequentially until the desired sequence is obtained. X
chromosome linked disorders are particularly amenable to chromosomal walking since only a single allele is expressed and a
complete linkage map (& genomic sequence) of this chromosome have been determined. The gene for X-linked disorder
Duchenne muscular dystrophy was found using RFLPS (& chromosomal walking). Similarly, defect was localized (linked) to
terminal region of short arm of chromosome 4 in Huntington disease and to alpha-globin locus on chromosome 16 in
polycystic disease.
Biochemistry: Cytogenetics and Molecular Genetics, Recombinant DNA & Genomic Technology ■ 711

- Comparison of genetic makeup of humans & related species (genetic maping)


- For DNA finger printing (DNA typing or DNA profiling): Alee Jaffreys described this technique in 1985. It is based on
sequence polymorphisms (slight sequence differences) between individuals. It is a potent weapon of crime detection
(forensic medicine).
1. Some sequence changes affects restriction site ami result in RFLPs. It is very accurate method but requires larg sample of
undergraded DNA (>25ng). That amount of DNA is often not available at a crime scene.
2. More sensitive DNA typing method use PCR and focus on short tandem repeats (STRs). An STR locus is a short DNA
sequence, repeated many times in tandem at a particular location in a chromosome; mostly & 4bp long. The DNA sequences
flanking STR are unique to each type of STR & identical (except for very rare mutations) in all humans. PCR primers are
added to this flanking DNA & designed to amplify the DNA across the STR. Since each human inherits one chromosome
from each parent, the STR lengths on two chromosomes are often different generating 2 signals from 1 individuals. If multiple
STR loci are analyzed, a profile can be generated that is essentially unique to a particular individual. PCR amplification allows DNA
fingerprints to be obtained from < lng of partially degraded DNA, an amount that can be obtained from a single hair follicle,
a drop of blood, a small semen sample that might be months or years old. Combined DNA Index System (CODIS) uses 13
STR loci for DNA typing in USA. The amelogenin gene present on human sex chromosome, has slightly different flanking
DNA on X and Y chromosomes. It is used as a marker for sex determination of DNA donor. Multiplex STR kits include 13
CODIS loci, amelogenin and 2 additional loci (total 16). The chances of accidental match between two individuals in human
population is < 1 in 108 (quintillion).

Microsatellites

- Microsatellites" are defined as loci (or regions within DNA sequences) where short sequences of DNA (nucleotides;
adenine- A, thiamine- T, guanine- G, cytosine- C) are repeated in tandem arrays. This means that the sequences are repeated
one right after the other. The lengths of sequences used most often are di-, tri-, or tetra-nucleotides.
- They can be used for DNA fingerprinting, in forensic and paternity/parentage work. At the same location within the
genomic DNA the number of times the sequence (eg.AC) is repeated often varies between individuals, within population,
and/or between species. So one population may commonly have 13 AC's (dinucleotide repeat sequence) repeated in a row
while another population has 18 AC's repeated at the same location within the genomic DNA.
- Predicting the progression of disease: Microsatellites are acutely prone to replication errors that result in expansions and
contractions of repeat unit (repeat length variability) because of misalignment of the template and daughter strands,
Variability of microsatellite repeat lengths leads to genomic instability and, plays a role in the progression of disease,
presumably by gene inactivation. DNA mismatch repair (MMR) enzymes maintain genomic stability by removing
replication errors from DNA.
- Defective DNA mismatch repair (MMR) enzymes in association with microsatellite instability (MSI) are thus implicated in
the progression of several diseases and cancers. Examples includes: Fragile syndrome, Huntington's chorea, Myotonic
dystroply, Spinobulbar muscular atrophy, Kennedy's disease, Hereditary non polyposis colon cancer
- Provides clue to genetic bases of disease: The association of a specific polymorphism in affected family members and the
lack of this association in unaffected members may be the first clue about genetic bases of a disease.

DNA Fingerprinting (DNA Typing or Profiling)

• Alec J. Jeffrey® of Leicester University (U.K), in 1984 serendipitously discovered fundamentals of genetic finger printing.
• Because every person's DNA sequence is different, the fragment in DNA specimen from one individual to another are
different in number & length. DNA finger printing uses restriction fragment length polymorphism (RFLP) or repeat
sequence DNA to establish a unique pattern of DNA fragments for an individual. The DNA markers analyzed for such
finger printing are most commonly short tandem repeat polymorphisms. VNTR (variable number of tandem repeats) are
short sequences of DNA at scattered locations in genome, repeated in tandem (one after another) that give rise to
polymorphism in chromosome. Short tandem repeats are similar to VNTR but smaller in size.
• DNA fingerprinting (by PCR) has revolutinized the analysis of evidence from crime scenes in determining whether the
sam ple comes from specific individual and varification o f paternity®. DNA fingerprinting is done by nucleated cells® eg
blood® (WBC & RBC), semen, vaginal epithelial cells, tooth pulp, bone marrow, hair roots, muscle, skin, mucous
membrane.
712 ■ A Complete Review of Short Subjects

Polymerase Chain Reaction (PCR)


PCR, invented by Karry M ullis, is an in vitro (test tube) enzymatic method of am plifying a selected (target) sequence of
DNA. It is like Xerox machine fo r gene copying. It is sensitive, selective (specific) and extremely rapid means of am plifying
any desired D N A sequence which can be as short as 50-100 bp and as long as lOkb.
It is so sensitive that it allows the DNA in a single cell, sperm or hair follicle; from any source - virus, bacteria, plant, animal
or, human even when the targetedsequence makes up less than one part in a million of the total initial sample to be
amplified. PCR has allowed successful enzymatic cloning of DNA from > 40,000 years old sample.
Specificity is based on the use of 2 oligonucleotide primers that hybridize to complementary sequences on opposite strands
o f separated ds (double standed) DNA0. (So ds DNA is required not SS DNA).
Each newly synthesized strand can act as a template for the successive cycles thus each cycle of amplification doubles the
amount of D N A in sample resulting in an exponential increase in target DNA, hence the name polymerase chain reaction.
Typically 20-30 cycles run during PCR amplify the DNA by a million to billion folds in a few hours. 20 cycles provide
amplification of 2 20 (i.e. 106) and 30 cycles 230 (i.e. 109).
Major advantages of PCR over traditional cloning methods (using recombinant DNA techniques), as a mechanism for
amplifying specific DNA sequence are sensitivity, speed and less technical difficulty. It is an in vitro (test tube) method not
requiring living cell. The cycles are generally repeated by automated Tempcycler.
The double stranded (ds) D N A is first heated to separate (melt) the 2 strands of template DNA containing the target
sequence (denaturation); Vast excess of 2 primers (one for each flanking sequence) is added and allowed to bind DNA
(annealing or priming); each DNA strand is copied by thermostable D N A (Taq) polymerase enzyme obtained from
Thermus aquaticus, starting from the primer sites (chain extension) in presence of all 4 dXTPs / dNTPs (deoxyribo
nucleoside triphosphate). So it is enzymatic (non-recombinant) D N A amplification. The cycles of denaturation, annealing
& chain extension are repeated.
Clinical applications of PCR utilize its ability to am plify (increase) the minimal (or almost negligable) amount o f DNA, which is
then detected by southern blot technique. So it is used in detecting low abundance nucleic acid sequence (DNA). For
example.

Detection of infectious disease Detection of variations & mutations in genes PCR in Law
- To detect bacterial, viral etc infectious To detect allelic polymorphism, specific aneuploidyQ, - DNA
disease agents in early and latent phase to compare normal cloned gene with an uncloned fingerprinting by
(when the agent load is very less). So mutant gene and to study evolution using DNA from means of PCR has
PCR offers rapid & sensitive method of archeological samples. revolutionized
detecting viruses that have long latency PCR (f/b electrophoresis & southern blot) is used for forensic analysis of
such as ITIV. detection of direct mutational analysis to prenatally or crime scene and
postnatally diagnose the presence or absence of paternity verification.
- Can detect even one virus or bacteria
inherited disorders like Cystic fibrosis, hemophilia, f - The DNA markers
present in sample
thalassemia & sickle cell anemia etc Q. It also detects analyzed for such
- To detect viral load by reality-PCR
carriers. fingerprinting are
★ PCR uses cells obtained from Detects DNA abnormalities characteristic to cancer and most commonly
amniocentesis or chorionic villus establish precise tissue types for transplantation. short tandem repeat
sampling for prenatal diagnosis; For quantitative RNA analysis after RNA copying & m- polymorphism.
somatic cells for postnatal dignosis and RNA quantitation by RT-PCR (using retroviral reverse - Sex determination
nucleated cells for finger printing. transcriptase).
To score in vivo protein - DNA occupancy using
chromatin immunoprecipitation assay._______________

Steps of a PCR Types of PCR


• Each cycle doubles the amount o f DNA in the 1. Conventional DNA based PCR
sample, leading to an exponential (logarithmic) 2. Reverse transcriptase PCR (RT-PCR)
increase in DNA with repeated cycles. Steps in - In this mRNA or rRNA is starting material which is converted to
each cycle are. complementary DNA (c-DNA) by reverse transcriptase. So it is a normal
PCR preceded by reverse transcription
1. Primer construction:- Flanking sequence is
- Tth polymerase (instead of Taq polymerase) from Thermus
nucleotide sequence of short segment on each
thermophilus is used which has both reverse transcriptase & DNA
side of target DNA. It brackets the DNA
polymerase activity.
sequence o f interest. In PCR it is not necessary
- In normal PCR, DNA is detected which could be from living or non
to know the nucleotide sequence o f target
living organism but in RT-PCR, mRNA is detected indicating its
DNA but it is necessary to know the flanking
origin from living organism (only living cells chang DNA to mRNA).
sequenceQ.
Biochemistry: Cytogenetics and Molecular Genetics, Recombinant DNA & Genomic Technology ■ 713

- Two synthetic single stranded oligonucleotides So it is used in expression mapping i.e. determining when & where
(usually 20-35 nucleotides long) which are certain genes are expressed.
complementary to respective flanking 3. A s y m m e t r ic a l P C R is used to generate single strands by adjusting
sequence® are constructed. These serve as primer concentrations to favor one strand. After 10-15 cycles the
primers. The 3'- OH end o f each primer points second primer is used up and only one strand complementary to first
towards the target sequence®. Primer is added strand continues to be copied up.
in vast excess. 4. I n v e r s e P C R amplifies the sequences on either side of known stretch
by circularizing a piece of DNA & hybridizing primers in an opposite
2. Denaturation of D N A duplex (94-98°C). The
orientation.
DNA to be amplified is heated to seperated
5. N e s te d P C R , improves sensitivity & specificity by using 2 sets o f primers
double stranded DNA (ds DNA) into single
in 2 successive PCR runs. This means a fragment is amplified by using
strands. (SSDNA)
a set of primers (eg genus specific) and second amplification of some
3. Annealing of primers to SSD N A (37-60°C). internal sequence is done by using another set of primers in first run
Separated strands are cooled & allowed to product.
bind (anneal) to the 2 primers (one fo r each - This reduces contamination in products d/t amplification of
strand) at flanking sequence®. Primer is added unexpected primer binding sites but requires more detailed
in vast excess. knowledge of sequence and proves hybridization may not be
4. Chain extension of primer (~72°C) necessary.
- DNA polymerase & deoxyribo nucleoside 6 . A n c h o r e d (o n e s id e d ) P C R o r R A C E (rapid amplification of (cDNA
triphosphate (d-NTP)® are added in excess to ends)
initiate synthesis of two new chains - It is used to amplify the ends of m-RNA transcripts when one end of
complementary to original DNA chains, sequence is known using a specialized reverse transcription - (RT) PCR
starting at primer site. DNA polymease adds - To unknown end a segment of guanine is covalently attached which
nucleotides to 31- OH end o f primer® and serves as template with poly cytosine primer. It is used to obtain
strand grows across the target DNA making (amplify) unknown 5' end (5' RACE-PCR) or 3' end (3' RACE-PCR).
complementary copies of target. 7. P C R u s i n g o t h e r D N A p o ly m e r a s e s
- DNA lig ase in lig ation am p lificatio n reactions®
- Magnesium concentration is critical® as it
- T 7 R N A p o ly m e r a s e in transcription based systems
affects DNA polymerase activity and fidelity,
- Q P r e p li c a s e from bacteriophage
denaturation temperature, primer annealing
8 . I n s it u (paraffin embedded or cytospin coated glass slides) P C R
& dimer formation and PCR specificity. 0.5-
a m p li f i c a t i o n (by protease + DNA polymerase) a n d p r o d u c t
5mM is usually used Mg++ concentration.
d e te c t io n (by hybridization or direct incorporation of biotin /
Excess results in accumulation o f nonspecific
digoxigenin labeled nucleotides)
amplification products seen as multiple bands on
9. R e a l T i m e P C R ( R T - P C R ) has many advantages over conventional
agarose gel, whereas, insufficient M g++ result in
RT-PCR
reduced yield of desired PCR products.
- R T - P C R o r q u a n t i t a t i v e ( q ) P C R , c o m b in e s P C R (D N A )
• Cycles of heat denaturation (seperation), by eliminating gel
a m p li f i c a t i o n a n d d e t e c t io n in t o a s i n g l e s te p
annealing (binding) of primers to flanking electrophoresis for detection. It is a software driven automated easy to
sequence and extension of annealed primer perform procedure that can be easily standardized. It has higher sensitivity,
with DNA polymerase are repeated. Initially, specificity, reproducibility and sample output (~ 200 sam ples/day)
E.coli DNA polymerase was used in PCR, that - Allows for quantitative measurement and rapidity d /1 reduced cycle
was destroyed & so has to be added by each times & removal of post PCR detection procedure. The amplicon
heat denaturation cycle. The advantage of could be visualized as amplification progressed. It is u s e d to k n o w
using thermostable DNA polymerase (such as h ow much am p lificatio n o f DNA h as occurred®.
Taq polymerase from a thermophilic bacteria - A ll rea l tim e PCR rely upon flu o r escen t reporter (probes) such as
Thermus aquaticus®) is that the enzyme itself SYBER - green, T aq man, and m olecu lar Beacons® for detection &
is not denatured and therefore does not have quantitation. The signal of which increases in d irect p rop ortion to the
to be added at each successive cycle®. am ou nt o f PCR p rod u ct in reaction [ Q u a n t it a t iv e R e a l t im e P C R
• The amplified DNA sequences can be ( R Q - P C R ) ].
analyzed by gel electrophoresis &ethidium - It can detect viral load (no. o f viruses present in sample) so help planning
bromide staining, southern hybridization treatment.
(blotting), direct sequence determination, RFLP / 10. Q u a n t it a t iv e (Q -) P C R (preferably real time) is used to quantify PCR
sequencing and incorporation o f biotin / product i.e. determining whether a sequence is present or not and if
digoxigenin labeled dUTP during amplification it is present the number of copies in sample.
and detection by colorimetric/ luminescent 11. M u l t i p l e x P C R uses multiple - unique primer sets within a single
techniques. PCR targeting multiple genes in a single test run that would
otherwise reuire several repeated PCR runs.
714 ■ A Complete Review of Short Subjects

5- 3' Flanking Target Flanking


3- Sequence Sequence Sequence
5'
Heat - Denaturation (seperation) 5' -3 '
Direction of 3' 5'
chain growth Primer
Detection of pre or post natal disease with Primer
known mutation eg cystic fibrosis, sickle cell 5' 3/—> Direction of
anemia, {i-thallasemia & hemophila Growth
3 ' ------------------------------ ^ ^ - ^ — 5'
Annealing & chain extension
Normal Gene
4
PCR Primers
3'
./
3- 5'
Mutant Gene 5- 3'

PCR followd by CSGE 5'


(conformation sensitive Gel Formation of double copy
Electrophoresis) or dHPLC
(de na turing High Direct Mutation Indirect linkage
Permormance Liquid Analysis (eg by PCR) Analysis
chromatography of products)
Involves identification o f specific Involves segregation o f genetic
base pair change or mutation markers known to be linked to a gene
responsible fo r disease responsible for disease. And it is
63bp
determined that wheres an
60bp individual inherited a chromose
present in family that carries the
abnormal gene
Homozygo Hetroygo Homozygo As it is a direct method, not DNA analysis of multiple family
us Normal us Mutant us Mutant based on segregation of members is required to identify the
(carrier) (Diseased)
genetic markers in family markers that segregate with
I members, it does not require abnormal chromosome in each
D/t
DNA analysis of multiple affected pedigree
missing
family members.
base pairs
Involves physical isolation Dose not involve physical isolation
PCR
and sequencing of gene & sequencing of gene
product is
Determines unequivocally Being less specific it's use is
shorter
whether suspect has inherited restricted to genetic diseases if
the disease or not. So has mutation sequence is not known.
become gold standard

Ligase Chain Reaction (LCR) Nucleic Acid Sequence Based Amplification


LCR, is amplification of target DNA after multiple - NASBA is a continuous, isothermal, enzyme based method of
rounds of denaturation, annealing & ligation of nucleic acid amplification. It amplifies RNA (especially or DNA from
two pairs of synthetic nucleotide (which hybridize a pool of total nucleic acid.
to complementary strands of a target DNA) with a - It uses a mixture of reverse transcriptase, RNA polymerase,
thermostable ligase ribonuclease- H, and 2 transcript specific DNA primers
Gap- LCR reduces the background generated by - 5 1 extension of forward primer contain promoter sequence for
target independent blunt end ligation. In this DNA bacteriophage T7 DNA dependent RNA polymerase and 5 1
polymerase fills in a gap b/ w annealed probes extension of reverse primer has complementary binding sequence
which are sub sequently joined by DNA ligase. for an electro chemiluminescent (ECL) tag.
★ D N A sequencing is the process of determining the exact order of 3 billion bases (A,T, C & G) that make up the DNA of 24
different human chromosomes.
Biochemistry: Cytogenetics and Molecular Genetics, Recombinant DNA & Genomic Technology ■ 715

Gene / DNA / Molecular Cloning


It is a technique by which large number of identical DNA molecules can be produced. In this technique a DNA fragment to
be cloned (amplified) is inserted into the DNA strands of cloning vector eg. plasmid, phages or cosmids to make a new
recombinant or chimeric DNA. When the DNA strand of vector multiplies, it also multiplies the inserted DNA fragment.
Clones produced is later removed by using restriction enzymes.
Cloning of DNA fragment involves 4 steps
- Fragmentation by restriction endonuclease involves isolation of DNA of intrest (that is to be cloned) and linearization of
circular vector DNA that contain same sticky ends
- Ligation is gluing together of DNA of interest with vector DNA to make a new recombinant or chimeric DNA molecules,
by DNA ligase enzyme.
- Transfection is inserting newly formed pieces into cells. And finally the cells are cultured. In this way the chimeric DNA
(so the DNA of interest) is amplified.
- Screening or selection fo r selecting or identifying the cells that have been successfully transfected with the vector DNA
containing the desired insertional DNA sequence in the required orientationQ. This can be done by.

Antibiotic resistance Colour selection markers which provide blue white screening PCR, restriction
markers ( a -factor complementation) on X- gal (a colourless galactose fragment analysis & or
I sugar) medium D N A sequencing
Allow only cells in which I I
the vector has been It is based on lac operon. The hydrolysis of x-gal by (3 glactosidase Congirmatory method
transfected, to grow cause blue colour in colonies & indicate that colonies contain
unligated vector. Whereas, white colonies indicate insertion of
foreign DNA & loss of cells ability to hydrolyse marker.

Hybridoma Cells and Monoclonal Antibodies


•Generally when an antigen is injected into an animal it results in Myeloma cell B Cell
formation of polyclonal (not monospecific) antibodies by mixture of B cells.
- Do not produce - Produce specific
These antibodies are directed against a number of different sites
antibody antibody
(epitomes or determinants on antigen).
- Immortal - Not immortal
• By Kohler- M ilstein method, limitless amounts of monoclonal -
- HGPRT-ve rendering - HGPRT+ve
m onospecific (specific for one epitope)antibodies can be obtained, by
salvage path of
producing hybridoma cell. The method involves cell fusion & resulting
purine
permanent cell line is called a hybridoma.
• B cells previously treated with an antigen C or mixture of antigens &
obtained from mouse spleen are mixed with mouse myeloma cells in
presence of PEG (polyethylene glycol). PEG causes cell fusion and
formation of hybridoma cell, which when cloned produce monoclonal
monospecific antibodies.
• Hybridoma cells can b e frozen & stored and subsequently thawed when
Fused in presence of
more of antibody is required; this ensures its long term supply. It can
polyethylene glycol (PEG)
also be grown in the abdomen of mice, providing relatively large
supplies of antibodies. Attempts to produce human monoclonal
antibodies are under-way. Hybridoma - cells
• M onoclonal antibodies are produced by hybridoma technology (in
vitro) or by one clone secreting specific type of antibody in multiple - Immortalized (like parental
myeloma (in vivo). In monoclonal antibody preparation all molecules myeloma cell)
are specific against a particular antigenQ and have increased avidity - Produce antibody & are
for attaching with antigen. So in a reaction where polyclonal antibodies HGPRT +ve (gained from
require 1 ml the monoclonals require very little (only 1 microliter) parental B- cells___________
quantity. So the initial cost of production may be high but it decreases
gradually in long run. I
• Monoclonal antibodies are now useful in detection of 1) serum Grown in presence of H AT (Hypoxanthine -
proteins, 2) enzymes, 3) hormones, 4) drugs 5) bacterial & viral aminopterinthymidine) containing medium
antigens, 6 ) cell surface receptors, 7) HLA antigen, and 8) Cancer
cells etc
716 ■ A Complete Review of Short Subjects

Monoclonal monospecific antibodies are used in


I----------------- B Cell will die Hybridoma cell
M yelom a cell
- To measure the amounts of many - Monoclonal antibodies
in medium as will also die in will survive &
individual proteins made in mice can be
they are not presence of grow because
- To determine nature of infectious humanized (used in
immortal HAT, since the they are
agent humans) by attaching the
aminopterine hypoxanthine -
- To subclassify both normal (eg complementarity determining
in HAT guanine-
lymphocytes) & tumor cells (eg regions [CDRs) (i.e. the sites
suppress phosphoribosyl-
leukemia) that binds antigen) onto
purine transferase
- To direct therapeutic agents to tumor appropriate sites in a human
synthesis by (HGPRT) +
cells immunoglobin molecule.
denovo 1
- To accelerate removal of drug (eg This produces an antibody
pathway by If cloned
digoxin) from circulation in toxicities that is very similar to
inhibiting produce
- N ephelom etric a ssay s o f b loo d human antibody, thus
reutilization of monoclonal
com ponents and b lo o d grouping® markedly decreasing
tetrahydrofola monospecific
- As immunosuppressants in clinical immunogenecity & chances
te antibodies
conditions like autoimmune disease of anaphylactic reactions
(Rheumatoid disease) - Quantitative preparation of
- To do ELISA tests specific cells and pure
antigens

Cloning Vectors

________Plasmid______ ______________ Phages____________ _________________ Cosmids______________


- Accept DNA pieces - Accept DNA fragment 10-20 kb long - A ccept larger frag m en t o f DNA, 35-50 kb
about 6 - 1 0 kb long - Linear DNA molecule into which long®.
- Small, circular, duplex foreign DNA can be inserted at - Are plasmids that contain DNA sequence,
DNA molecules several restriction enzyme sites. called cos sites, required for packaging
- Confer antibiotic lambda DNA into phage particle.
resistance to host cell
- Its complete DNA Cloning Capacities
sequence is known
hence precise location Vector D N A insert size
of restriction cleavage Plasmid, PBR322 0.01- lOkb (kilobases)
site for inserting the Phage 10 - 20 kb
foreign DNA is
Cosmid 3 5 - 50kb®
available
BAC (bacterial artificial chromosome), 5 0 -250kb
PAC (E. Coli bacteriophage PI based)
vectors
YAC (Yeast artificial chromosome) 500- 3000kb

Site Directed Mutagenesis: Alternation in cloned genes produce modified proteins

Cloning can be used not only to overproduce proteins but to produce protein products with subtle alteration in native forms.
Specific aminoacids may be replaced by site directed mutagenesis in two ways
Biochemistry: Cytogenetics and Molecular Genetics, Recombinant DNA & Genomic Technology ■ 717

If the When known restriction sites are not


appropriate present, oligonucleotide - directed
(known) mutagenesis can create a specific DNA
restriction sequence change
sites flank A short synthetic DNA strand with a
the sequence specific base change is annealed to single
to be altered, stranded copy o cloned gene. The
researchers mismatch of a single base pair in 15-20 bp
can simply does not prevent annealing at
remove a appropriate temperatures.
DNA This annealed strand acts as a primer for
segment & synthesis of a duplex DNA (with one
replace it mismatch), which is then used to
with a transform cells.
synthetic Cellular DNA repair systems will remove
one that is and replace the altered base and restore
identical to the gene to its original sequence in 50%
Plasmid contains Transformation
gene wilh desired of E.coli cells;
original cases whereas, the other h alf will remove base-pair change repair of DNA

except for (a)


and replace the normal b ase , retaining
the desired the desired mutationQ
change Changes can also be introduced that
involve more than 1 basepair. Or even
the parts of 2 genes can be ligated to In E.coli cells, about half the
plasmids will have gene wilh
create fused gene and fusion protein. desired base-pair change
(b)

Two approaches to site-directed mutagenesis

Site Specific Recombination (Exchange) / SSR

SSR occurs only at a particular short (20-200 bp), unique DNA sequence - the recombination site (RS), where the recombinase
enzyme acts. Recombination sites recognized by site specific recombinase are non palindromic (partially asymmetrical) 0 and 2
sites align in same orientation during reaction . The outcome depends on location & orientation of recombination sites. If 2 sites
are on

Same DNA molecule with Two different DNA molecules

! 1. If one or both DNAs are circular


Same orientation Opposite orientation .. . .
I rr | the result is insertion

Deletion of intervening Inversion


DNA

Recombinase is a site specific endonuclease and ligase in one package Q. Auxiliary proteins may regulate timing or outcome
of reaction.
2 classes of SSR system, with rely on either Tyr or Ser residues in the active site. A separate recombinase recognize and
bind to each RS on same or two different DNA molecules. One DNA strand in each site is cleaved at a specific point &
recombinase becomes covalently linked to DNA at cleavage site through phospho tyrosine bond. This transient protein
DNA linkage preserves phosphodiester bond that is lost in cleaving the DNA so ATP is unnecessary in subsequent steps.
The cleaved DNA strands are rejoined to new parteners to form H olliday intermediates, with new phosphodiester bonds
created at the expense of protein DNA linkage. The reaction is completed by repeating the process at a 2nd point within
each of 2 RS.
SRS system that employ an Ser residue active site , both strands o f each RS are cut concurrently & rejoined to new
partners without the Holliday intemediatesQ. In both systems, exchange is precise and reciprocalQ, regenerating RS when
the reaction is complete.
718 ■ A Complete Review of Short Subjects

Linkage Analysis : Indirect DNA Diagnosis


- Direct gene diagnosis is only possible if nucleotide sequence of mutant gene is known, which is not the case in most genetic
disorders. Therefore m utant gene is indirectly tracked on the basis o f its lin kage to d etectab le gen etic m arkers such as
n aturally occuring v ariation s or polym orphism ® ; SNPs, RFLP, STRs (short tandem repals or micro satellites)
- In essence, one has to determine whether a given fetus or family member has inherited the same relevent chromosomal
region as a previously affected family member.
- Linkage analysis is a statistical technique used to identify location on a chromosome of a given gene involved in a trait or
disease relative to a known location of chromosomal marker, in terms of recombination frequency rather than as a specific
physical distance. The greater the frequency of recombintion (segregation) between two genetic markers, the farther apart
they are assumed to be.
- With linkage analysis all members of family are genotyped for polymorphic markers and then compared to determine which
markers are present in family members affected by the disorder. So the primary aim is to determine whether there exist
pieces of genome that are passed down through families (with multiple patients) in a pattern that is consistent with a
particular inheritance model and that is unlikely to occur by chance alone. Since linkage analysis does not detect mutations in
gene directly, further investigation is required once genome of interest is identified.
- Pedigree is a diagram of family relationship that uses symbols to represent people (square = male, circle = female) and lines
to represent genetic relationship (horizontal line = mating, vertical line extending downward from a couple represent their
children). It is often analyzed to determine the mode o f inheritance (dominant or recessive). All the individuals that possess the
trait / disease are shaded.

Blotting & Hybridization Techniques


- Visualization (or detection) of a specific D N A segment Southern Northern Western
DNA Blot RNA Blot Protein Blot
(Southern) or RN A fragment (Northern) or particular
antigen/protein (Western) in a complex mixture o f thousands o f
contaminating molecules requires convergence of a number of
techniques (like restriction enzymes, Agarose / SDS-PAGE
electrophoresis, denaturing, blotting renaturing, fixing, probe
O n a pp lyin g Gel (Agarose /
hybridization which are collectively termed blot transfer. d ire ct
SDS-PAG)
e le c trica l
- Blot technique was developed by EM Southern in 1975, to c u rren t
Electrophoresis
s e p a r a te s s a m p le
ne gativ e
detect a specific segment of DNA in the whole genome. c h arg e d D N A b ased on m ass

Southern blotting is named after its inventor, Edward Southern m ove


tow a rd s
(s iz e ) : sm a ll
segments m ove
and others (northern, western) began as laboratory jargon. a n ode more rapidly

Procedure Colony / Plaque


Phage Hybridization
(Bacterial o r phage
- Electrophoresis: For blot transfer, the sample (DNA or RNA D N A )_________________

or protein) is isolated from cell line or tissue and this complex C overing Ihe g e l with
nylon m e m b ra n e r/i
mixture is pipetted into a well in an agarose or sodium d od ecy l e x a c t replica of
DNA/RNA/ protein
su lfate (SDS) - p oly acry lam id e g el f o r electrophoresis. pattern within th e gel.
Dna is d e n a tu re d to
E lectrophoresis seperates sam ple (D N A /R N A /P rotein ) m ak e s s by e x p o su re to
mild alkali b efo re this;
according to m olecu lar size (mass)®; the smaller fragments a n d b o u n d / fix e d to
nylon membrane by
more most rapidly. In southern blotting, sample DNA is e x p o su re to h e a l o r UV

digested with one or more restriction enzymes before pipetting and


exposed to direct electrical current during electrophoresis.
A d d in g L a b ele d
DNA being n egatively changed m ove tow ards anode®. P robe; w h ich is c -
ONA - cPNA D N A fo r D N A &
RNA • CDNA
- Blotting (Adsorption/Transfer) to Nitrocellulose (Nylon) R N A blot;
a n tib o d y fo r
membrane: DNA / RNA / Protein within the gel is pro te in blot; and
n u c leic acid fo r
transferred (blotted or adsorbed) over to nitrocellulose or s o u th w e s te rn blot

nylon paper (membrane) by covering the gel with paper,


resulting in an exact replica of the pattern on the gel. In
Southern blotting, DNA is denatured by exposure to mild alkali W a s h in g excess
p ro be & A u to rad io
(like NaOH) to make it single stranded before adsorption. During g ra p h y sh ow ing
sp e cific probe
the transfer the SDS is largely removed from some proteins, binding
and there is enough refolding & restoration of conformation so
that the antibody probe can react with protein on membrane.
Biochemistry: Cytogenetics and Molecular Genetics, Recombinant DNA & Genomic Technology ■ 719

- A dding Probe & Hybridization: Before adding probe, DNA is bound (fixed) on the nitrocellulose membrane by exposure to
heat (temp at 80°c) and or UV. Then paper is exposed to labeled probe. The probe is complementary D N A strand (c-DNA)
labeled with radioisotope or nonradioactive fluorescent label in Southern (DNA) and Northern (RNA) blot tests. The nucleotide
sequence ofcDNA is complementary to DNA or RNA. There will be many DNA/RNA fragments on membrane but only few
pieces that contain the target DNA / RNA, hybridize with cDNA probe to make a D N A -cD N A hybrid in Southern blot or
RNA-cDNA hybrid in Northern blot. Similarly a labeled antibody probe forms complex with protein (antigen) in Western
(immuno) blot test and labeled nucleic acid probe forms complex with protein in south-Western blot test.
- W ashing & Autoradiography: After thorough washing to remove excess probes, the x-ray plate or imaging screen is placed
over the nitrocellulose paper in dark for few days. The radiation from fixed probe will produce its marks in form o f several
specific bands corresponding to the DNA/RNA/protein fragment that recognized the probe.

Variants

- In Southwestern hybridization blot technique, proteins are separated by electrophoresis, blotted to a membrane, renatured,
and analyzed for an protein-DNA (nucleic acid) interaction by incubation with a specific labeled nucleic acid probe.
- Colony or Plaque or Phage H ybridization is a method by which specific clones of bacteria or phage grown as colonies on
agar plate are identified and purified. Nitrocellulose paper is overlaid over agar plate sticking clones from each colony which
are fixed permanently by heat (UV and denatured or lysed with NaOH so that its DNA is available to hybridize with cDNA
probe. It results in a clonal isolate (bacterial colony) or individual phage plaque containing a unique DNA insert.

Use

- Blotting procedure are useful in determining amount of gene (DNA)/RNA/ protein in a given tissue / sample. If a specific
base is changed and a restriction site is altered these procedures can detect point mutations and gene alterations (deletion,
insertion or rearrangements) because the required electrophoresis step sepeates the molecules on the basis of size. Therefore
Southern, Northern & Western blot transfer techniques are used to size, identify and quantitate specific gene (DNA), RNA
and protein - respectively. Mutant genes like HbS, DMD, PKU & cystic fibrosis as well as presence of viral DNA (HBV, HBC)
can be identified by Southern blot method. Similarly W estern b lo t m eth od confirm s d iag n osis o f AIDS® (in ELISA positive or
HIV positive patients) by demonstrating presence of antibodies in patients serum to a specific HIV protein.
All hybridization procedures depend on specific base pairing of complementary nucleic acid strands. Perfect matches
hybridize readily & withstand stringent conditions (high temperatures and low salt concentrations). In the hybridization &
washing reactions, less than perfect matches do not tolerate such stringent conditions; thus hybridization either never occur or
is disrupted during washing steps. Gene hom ology detection and cross species comparison of a gene can be done by varying
stringency.

Probes

I I
D N A probe Synthetic Biotinylated probes A n t ibodie
ib o d ie s
I oligonucleotide probe I I
Probe is a short single stranded piece of I Disposal of radioactive When no information
DNA, labelled with a radioisotope (eg P-12) - If sequence of all or probes is expansive, so now is available to guide
or a non- radioactive probe (eg. biotin), and part of target DNA is vitamin biotin coupled the synthesis of a probe
can pick a specific gene or DNA sequence known, single nucleotide are used to for direct detection of
out of the mixture of thousands or stranded synthesize probes DNA of interest.
millions of irrelevant DNA fragments oligonucleotide probes Biotin binds to egg white In this case an tibod y
cleaved by restriction endonucleases. The of 20-30 nucleotides protein avidin. Avidin can be la b eled p ro b e d irected
nucleotide sequence of a probe is can be synthesized that attached to fluorescent dye ag ain st the p rotein o f
complementary to the DNA of interest, are complementary to detectable optically with interestQ is used to
called the target DN A. gene of interest great sensitivity. Thus a identify which
Utility of probes depends on phenomenon - A llele specific DNA fragment that bacterial colony
o f hybridization (or annealing) in which a oligonucleotide (ASO) hybridizes with biotinylated produces the protein &
single stranded sequence of a target DNA probe is used to detect probe can be made visible by therefore contains
binds to a probe containing a the presence of sickel immersing the gel in a CDNA of interest.
complementary nucleotide sequence. cell mutation in fi-globin solution of dye coupled
gene avidin.
720 ■ A Complete Review of Short Subjects

DNA-Microarray / DNA - Chips


N o rm a l C a n c e r c e ll m -R N A is i s o l a t e d fro m 2
• DNA microarrays contain thousands (500- ty p es o f c e lls o r c e lls a t 2
5000) o f immobilized DNA probes /sequences s ta g e s o f d e v e lo p m e n t; e a c h
(few dozen to hundreds of nucleotide m R N A r e p r e s e n t s a ll g e n e s

long) from known genes organized in an e x p r e s s e d in t h a t c e ll o r s t a g e .

area no longer than a microscope slide.


DNA segments (from DNA libraries) are
amplified by PCR and placed on small wells C o n v e r t m R N A to c D N A b y
re v e r s e tr a n s c r ip ta s e , u s in g
in a solid polystyrene plates, using robotic
f lu o r e s c e n tly la b e le d d e o xy
devices. Upto million such spots are
r ib o n u c le o tid e tr ip h o s p h a te s .
deposited in a predesigned array on a
surface area of just few cm2. An alternate
way is to synthesize DNA directly on the
A d d c D N A s to m i c r o a r r a y ;
solid surface using photolithography. f l u o r e s c e n t c D N A s h y b r i d i z e to
• C-DNA (obtained directly or from mRNA c o m p le m e n ta ry s e q u e n c e s o n
of patients particular cell type or stage) is m i c r o a r r a y _________________________
added to each well, hybridize and
fluorescence is assessed to assess genes
being expressed in those cells/stage.
• This is based on principles of nucleic acid
hybridization like Southern or Northern
blot tests but allows simultaneous study
of multiple genes or entire genome rather
o#o E a c h flu o re s c e n t s p o t
r e p r e s e n t s a g e n e e x p r e s s e d in
t h a t c e ll o r s t a g e . E g
- G re e n (c ro s s e d ) s p o t fo r

o o##o
n o r m a l c e l l s p r o d u c e th i s .
than single gene. Hybridization means
- R ed (p la in ) s p o t fo r c a n c e r
binding of complementary strands of c e lls p r o d u c e t h i s
nucleic acid a /1 Watson-Crick rules (i.e. A
= T and G=C binding). Southern blot for OOOiO - Y e llo w o r m ix e d ( d o tte d
s p o t f o r b o th c e lls p r o d u c e
DNA & Northern blot fo r RNA allows the
study /detection o f single gene whereas
microarray technique (for DNA & RNA)
allows detection o f multiple genes or entire
o#ooDNA Microarray Analysis of gene expression
th i s

genome. So it can be considered as multiple


Southern or Northern blot analysis
running in parallel.
• Microarrays are used to analyze a sample for presence of m utation / gene variation (genotyping) or to determine the
patterns of mRNA production (gene expression analysis).
- For genotyping and mutational analysis, the cellular sample is genomic DNA. M icroarray based C G H (com parative
genom ic hybridization) test can be used as high resolution whole genome scan for the detection or screening of unknown
(non specific) abnormalities, mutation or genomic imbalance including deletions, duplication and aneuploidies. Microarray
based CGH cannot detect balanced translocations or inversions
- Microarray is the m ost powerful tool for screening gene expression profile of biological samples. It allows researcher to
explore, expression patterns o f thousand o f genes (or entire genome) in a single analysis thus permitting com prehensive analysis
of gene expression profile. For expression analysis the mRNA from a particular cell type (eg normal and cancer cells) or
from same cell type in different stags of development is converted to cDNA & labeled with fluorescent tag. This mixture is
then exposed to microarray chip. The amount of fluorescence bound to each spot is a measure of the amount of that
particular mRNA in the sample. This tells which genes are expressed in which stage of development or in which cells. The
spots that fluoresce provide a snapshot of all genes being expressed in the cells at the moment they were harvested - gene
expression on genome wide scale.
- For a gene of unknown function, the time & circumstances of its expression can provide important clues about its role in the
cells.
- Tum or finger print or transcriptional profile of cancers to tailor treatment regimens.
- Genetic fingerprint of different diseases / malignancies - classifying them, understanding genetic pathophysiology and
identifying diagnostic or prognostic markers.
Biochemistry: Cytogenetics and Molecular Genetics, Recombinant DNA & Genomic Technology ■ 721

Methods for Analysis of Gene (DNA, Mutation) & Products Gene Expression (RNA and Protein)

Methods used for Detection of Mutation A nalysis of Gene Expression Product


(& Analysis of Gene or DNA)
Determination of mRNA levels
1. Cytogenic analysis 8 . Single stran ded 1.Northem blot®
2. Fluorescent in situ con form ation al In this mRNA from a particular cell is converted to
2. M icroarrays:

hybridization polym orphism (SSCP)®. cDN A and amplified by RT-PCR or rererse transcriptase PCR
(FISH) 9. D en atu ratin ggradien t Analysis of Proteins
3. Southern blot® gel electrophoresis - Amount o f protein in cells donot always directly correspond to amounts of
4. P oly m erase chain (DGGE)Q mRNA as some mRNAs are translated more efficiently & some
reaction (PCR)® 10.RNAse cleavage proteins undergo post translational modifications.
5. R everse ll.Oligonucleotid specific - For detecting one or lim ited num ber of proteins, antigen (protein)
tran scriptase PCR hybridization labeled antibody binding is used eg in
(RTPCR)® 12.Microarrays (c D N A 1.ELISA (Enzyme linked immuo sorbent assay)
6 . DNA sequencing microarrays or D N A 2.Western (Immuno) blots
an d DNA cloning® chips) - When analyzing abundance & interactions of large num ber of
7. R estriction 13.Protein truncation test proteins (called proteom ics), automated methods are employed such
frag m en t (PIT). as.
polym orphism 14.Pyrosequencing l .T w o d im e n s io n a l (2 D ) g e l e le c t r o p h o r e s is ®
(RFLP)Q 15.M ultiplex ligation l.M a s s s p e c tr o sc o p y ®
dependent probe 3 .M u l t i d im e n s io n a l l iq u i d c h r o m a t o g r a p h y ®
amplification (MLPA)
4.Bioinformatics

DNA (Gene) Library


DNA or gene library is collection o f cloned restriction fragments o f DNA o f an organism. It can be of 2 types

Genomic - DNA library Complementary (c) DNA library


• It is the largest type of DNA library, created when the It is a specialized library designed to study gene fimction - so it
complete genome o f a particular organism is cleaved into includes only those genes that are expressed i.e. are transcribed
thousands offragments by restriction endonuclease and all the into mRNA. So cDNA libraries lack introns & control
fragments are cloned by insertion into a cloning vector. regions.
• This (amplified DNA fragments) represent the entire It is prepared by using m-RNA a s a tem p late to m a k e a
genome of an organism including introns and control com plem en tary DNA (cDNA) ca ta ly z ed by enzym e reverse
regions. It is prepared by R E & cloning in following way tran scrip tase (RNA dependent DNA polym erase) 0. (The
- 1st step is partial digestion of D N A (of an organism) by mRNA is isolated from tRNA and r-RNA by the presence
restriction endonucleases (RE) in which either the amount of its poly tail)
or the time of action of enzyme is limited. The resulting double stranded c-DNA can be amplified by
- R E that cut very frequently (recognize 4bp) are generally PCR or cloning (i.e. inserting into a suitable vector and
used. Complete digestion is not done as it would creating a population of clones) resulting in c-DNA
fragment the gene of interest (i.e. it is not contained in libraries.
any one clone) as most genes contain > 1 restriction sites. To aid mapping of large genomes, cDNAs can be partially
Whereas partial digestion provides gene of interet in sequenced to produce a useful type of STS called expressed
DNA fragments of range of sizes (~ 20 kb) sequence tag (EST).
- Too small or too large fragments (inadequate for cloning) are cDNA libraries are made more specialized by cloning
removed by electrophoresis or centrifugation. Clonic cDNAs into a vector that fuses cDNA sequence with the
vector (BAC or YAC) plasmid is cleaved with the same sequence for marker or reporter gene-thus forming
RE and ligated to partially digested genomic DNA reporter construct. 2 useful markers include
fragments. Green fluorescent protein (GFP) gene which generates
- Now each transformed yeast or bacterium cell grows into fusion protein which is highly fluorescent protein that
a colony or clone of identical cells, each bearing the same literally lights up.
plasmid containing one of many genes represented in Epitope tag protein that bounds mono clonal antibody &
overall library. precipitates.
• Hybridization methods can order individual clones in Because cDNA has no intervening (intron & control regions
sequence by identifying clones with overlapping etc) sequences, it can be cloned into an expression vector
722 ■ A Complete Review of Short Subjects

sequences. Sequence tagged site (STS) can provide for synthesis of eukaryotic proteins (like human insulin) by
landmarks for genomic sequencing projects. A set of over bacteria. These (plasmid of) expression vector contain
lapping clones represents a catalog for a long contiguous bacterial promoter (for transcription) and Shine Dalgarno
segment of a genome, often referred to as a contig. sequence /SDS (to initiate translation).
SDS

C loned
D N A (cD N A )

D E
:

So the sequence of partially digested &


cloned fragments is 1 —> 2 —>3 P ro te in

Biotechnology / Recombinant DNA technology / Genetic Engineering


It is a method by which a living organism or its parts (i.e. DNA/genes) are used to change/ incorporate a particular character to
another living organism.
It involves use of restriction endonuclease (to cut a particular gene) and PCR or cloning (to amplify it). It can be used for
I
> Quantitative production of human biomolecules, S o m a tic G e n e G ene Production of specific
genetic products by bacteria eg_____________ T h erap y p r o d u c t io n D N A probes for
- Human insulin I I diagnosis/
- Erythropoietins, anticoagulants It is intracellular New genes can identification of
- Plasminogen activators delivery of desired be produced to - Antenatal genetic
- Cell growth factos, GF, Colony stimulating genes to cure a be incorporated disease
factor genetic disorder® in - Mutation /
- Interferons, interleukins0 by correcting an - Genetic onchogenesis
- Recombinant vaccines eg rabies vaccine, hepatitis existing cropsQ - Sex, parent or
B vaccine abnormality. - Transgenic criminal etc.
- Monoclonal antibodies It is tried in SCID, animals
- Superoxide dismutase_________________________ cystic fibrosis,
This method produces biological substances without Duchene MD,
any risk of contamination (like hepatitis B virus) familial hyper
• Creation of gene library cholesterolemia etc.

Production of Human Insulin by Genetic Engineering


- Because obtaining a human DNA with specific insulin gene is a laborious process, it is easier Insulin m RNA of human
to begin the process by obtaining insulin mRNA from beta cells o f pancreas®, where insulin pancrease beta cells
is produced in abundance (and obtaining specific mRNA is easier). ■4'
- Complementary (c) D N A containing the insulin gene is prepared by using human insulin c-DNA by reverse
mRNA (from pancreatic beta cells) as a template and reverse transcriptase enzyme. transcriptase
- This c-DNA containing insulin gene to be cloned is inserted into plasmid vector. Bacterial
Cloned into plasmid
plasmids are small, circular, duplex DNA molecules whose natural function is to confer
vector to produce many
antibiotic resistance to host cell. They exist as single or multiple copies within bacterium and
copies of same gene
replicate independtly from the bacterial DNA while using primarily the host replication
machinery. The complete DNA sequence of many plasmids are known; so the precise Bacteria (E.coli) grown in
location of RE cleavage sites for inserting foreign DNA (for cloning) is available. Plasmids optimal production
are easily separated from host chromosome because of smaller size and the desired plasmid conditions
- inserted DNA can be easily removed by cutting plasmid with enzyme specific for d'
restriction site into which the original piece of cloned DNA was inserted. Plasmids produce Produced insulin is
m illions of clones containing same insulin gene. collected isolated &
- Bacteria (E.coli) containing insulin gene are grown in optimal conditions; insulin produced is purified
isolated, collected & purified.
Biochemistry: Cytogenetics and Molecular Genetics, Recombinant DNA & Genomic Technology ■ 723

Recombination DNA Technology (Research) & Enzymes Used in it


• It is a genetic engineering which effects artificial modification of genetic constitution of a living cell by introduction of foreign
DNA experimental techniques. It involves.
- Splicing the DNA by restriction endonuclease,
Preparation o f chimeric DNA molecule followed by
Cloning for production of large number of target DNA molecules
• The biological tools required are - enzymes, vector (vehicle) D N A (i.e. bacterial plasmids, cosmids & bacteriophage), and
passenger (foreign) D N A (i.e. complementary DNA, synthetic or random DNA to be introduced into the vector). Enzymes
used are
Enzymes used in Recombinant D N A Technology / Genetic Engineering
Enzyme Function Primary Use
Restriction Endonuclease - C le a v e D N A s at a sp e cific b a s e s e q u e n ce s k/ a Key enzyme u se d as chemical knife, in
(Type II) recognition sequences or restriction sites. re c o m b in a n t D N A te ch n o lo g y
- Cut (splice or cleave) D N A o f an y so u rc e in to sh o rt O rig in a lly ca lle d restriction enzymes b eca u se
p ie c e s in a sequence specific manner in c o n tra s t to th eir p re se n ce in a g iv en b a c te ria re stric te d the
o th e r m eth o d s, e g e n z y m a tic , p h y sic a l o r ch e m ic a l g ro w th o f c e rta in v iru se s (b a c te rio p h a g es)
th a t b re a k D N A in ra n d o m fash ion .
Example Cleaved Sequence Bacterial Source
E co RI G |A A T T C E. C o li R y 13
C TTA A G|
H Pa I GTT AAC H a e m o p h ilu s p a ra in flu e n z a e
CAA TTG
Bam H I G I GATC C B a c illu s a m y lliq u ifa c ie n s H
C CTAGl g
Eco R II 1C C T G G E. C o li R 245
GGACC1
P s tI CTG CA |G P ro v id e n c ia stu a rtii 164
G 1A C G T C
DNA ligase C a ta ly z e s th e fo rm a tio n o f n ew p h o to d ie ste r b o n d s in Jo in in g D N A m o le c u le s e g a ft e r d ig e s tio n b y R E.
a re a ctio n th a t u se s A T P (o r s im ila r c o fa c to r) b e tw ee n D N A s t r a n d s c a n b e j o i n e d a g a i n b y D N A lig a seQ .
D N A m o le c u le s

Bacteriophage
X Exonuclease R e m o v e s n u c l e o t i d e s f r o m 5' e n d s o f d s D N A t o e x p o s e D N A se q u e n cin g
s in g le s t r a n d e d 3' e n d Q
Exonuclease III R e m o v e s n u c l e o t i d e s f r o m 3 ' e n d s o fD N A Q D N A se q u e n cin g ; m a p p in g p ro te in -D N A
in te ra ctio n s
BAL 31 Nuclease D e g ra d e s b o th th e 3 ' a n d 5 ' e n d s o f D N A P ro g re ssiv e sh o rte n in g o f D N A m o le c u le s
SI Nuclease D e g r a d e s s i n g l e - s t r a n d e d D N A 'i R e m o v a l o f " h a ir p in " in sy n th e sis o f c D N A ; R N A
m a p p in g stu d ie s (b o th 5 ' an d 3' en d s)
DNase I U n d er a p p ro p ria te c o n d itio n s , p ro d u c e s sin g le ­ N ick tra n sla tio n ; m a p p in g o f h y p e rse n sitiv e site s;
stra n d e d n ick s in D N A m a p p in g p ro te in -D N A in tera ctio n s.
Alkaline phosphatase - D e p h o sp h o ry la te s 5 ' e n d s o f R N A a n d D N A R e m o v a l o f 5 '- P 0 4 g ro u p s p rio r to k in a se la b e lin g
- R e m o v e s te rm in a l p h o sp h a te fro m e ith e r 5 ' o r 3' to p re v e n t se lf-lig a tio n .
en d s (o r b o th ) - L e h n in g er

Thermostable DNA - S y n t h e s iz e s d o u b l e - s t r a n d e d D N A f r o m s in g le ­ S y n th e sis o f d o u b le -stra n d e d c D N A ; n ick


polymerase I (E. coli) s tr a n d e d DNAQ tra n s la tio n ; g e n e ra tio n o f b lu n t e n d s fro m stic k y
- F il ls g a p in d u p le x b y s t e p w i s e a d d i t i o n o f en d s.
n u c le o t id e s t o 3' e n d
Reverse transcriptase S y n t h e s iz e s D N A f r o m R N A t e m p l a t e Q (i.e. m ak es S y n th e sis o f c D N A fro m m R N A ; R N A (5' en d )
(R N A d e p e n d e n t D N A D N A c o p y o f an R N A m o le c u le ) m a p p in g stu d ies.
p o ly m e r a s e ) Q
Polynucleotide kinase - A d d s a p h o sp h a te to 5 '-O H en d o f p o ly n u c le o tid e “ P la b e lin g o f D N A o r R N A
to la b e l it o r p e r m it lig atio n .
- T ra n sfe rs te rm in a l p h o sp h a te (y p o sitio n ) fro m A T P
to 5 '-O H g ro u p s o f D N A o r R N A
Terminal transferase A d d s n u c le o tid e s (homopolymer tails) to 3 '-O H e n d s Homopolymer tailing
o f D N A (lin ea r d u p lex ).
724 ■ A Complete Review of Short Subjects

_____________Genomics_____________ ______________________ Proteomics______________________


- Study o f organism's entire genome (i.e. set o f - Quantitative & qualitative study o f entire proteome (all expressed
genes)® protein)®. Preteom e is the set of all proteins expressed by a cell at
- Genome is set of all genes expressed by an particularly time. It is more complicated as it involves the study o f all
organism protein expressed by a genome, including their relative abundance,
distribution, post translational modifications, functions, &
- Genome o f an organism is constant, so it's less interactions with other macro molecules®.
complicated process and involves study of - It aims to identify the entire complement of proteins expressed by a cell
structure of genome including maping & under diverse condition. So it provides more complete, dynamic & advanced
sequencing and enables primary sequence o f picture of growth, aging & disease. Example includes i d e n t i f i c a t i o n o f
protein to be identified easily® from small p r o t e in s w h o s e l e v e l s o f e x p r e s s io n c o r r e la t e w ith m e d ic a lly
amounts of sequence data. All that remains is like whose appearance or disappearance is
s i g n i f i c a n t e v e n ts ,
to aquire open reading frame (ORF) that associated with a specific physiological condition or disease.
encodes protein. - Basic steps include s a m p le p r e p a r a t io n , p r o t e in s e p a r a t io n , p r o t e in
- Picture provided is both static & incomplete i m a g i n g a n d i d e n t i f i c a t i o n . It can be done by______________________
and represents the begining of the task of in which the expression of m RNA that
1 .G e n e A r r a y (D N A C h ip s )
undarstanding the dynamics of growth, aging encodes protein is detected. It is more sensitive the 2 D gels,
& diseases process. particularly with respect to low abundance proteins. However,
changes in expression of mRNA encoding a protein do not
____________ Glycomics____________
necessarily reflect comparable changes in level of corresponding
Deals with the structure & function o f chains o f protein.
sugar (oligosaccharides)®. Glycome refers to
2 . M u l t i d im e n s io n a l p r o t e in i d e n t i f i c a t i o n t e c h n o lo g y (M u d P IT )
the identity of the entirely of carbohydrates in
employs successive rounds o f chromatography to resolve the peptides
an organism
produced from digestion of complex biological sample into many
___________Bioinformatics___________ simpler fractions that can be analyzed by m a s s s p e c tr o s c o p y .
A scientific discipline that combines the tools 3 .2 - D i m e n s i o n a l e l e c t r o p h o r e s is : I s o e l e c t r i c f o c u s i n g (IE F ) is u s e d
and techniques of mathematics, computer i n c o n ju n c t io n w ith S D S - P A G E for 2D (two dimensional)
science and biology with the aim of electrophoresis, which seperates polypeptides based on PI on one
understanding the biologic significance of dimension and molecular mass (Mr) in second. 2D electrophoresis is
variety of data. particularly used for separating the components of complex
mixtures of proteins. Individual polypeptides are then extracted and
___________ Cytogenetics___________
analyzed by E d m a n s e q u e n c i n g or m a s s s p e c tr o s c o p y .
Branch of genetics concern with structure and
★ Proteins & peptides must be p u r i f i e d prior to analysis by various
function of cell, especially the chromosome.
c h r o m a t o g r a p h y t e c h n iq u e s . Then protein p u r ity is a s s e s s e d b y S D S -
________ Pharmacogenomlcs________ P A G E (sodium dodecyl sulfate polyacrylamide gel electrophoresis).
E d m a n r e a c tio n enables peptide & proteins to be sequenced. M a s s
The study of genetically determined variations
s p e c tr o s c o p y is more sensitive method for determining sequences of
in responses to drugs in human or in laboratory
peptides and proteins. MS determines mass of molecules and can be
organisms.
used to analyze metabolites, carbohydrates and post translational
modification such as phosphorylation or hydroxylation that add readily
identified increaments o f mass to a protein.

Human Genome Project

Principle Methods Used Major Findings

Procedure Comments More than 90% of the


Detection of specific For instance, a small deletion of band Xp21.2 was important genome has been sequenced;
cytogenic abnormalities in cloning the gene involved in Duchenne muscular gaps, large and small, remain
dystrophy to be filled in.
Extensive linkage studies Large families with defined pedigrees are desirable. Estimated number of protein
Dominant genes are easier to recognize than recessives -coding genes ranges from
Biochemistry: Cytogenetics and Molecular Genetics, Recombinant DNA & Genomic Technology ■ 725

Use of probes to define Probes identify STSs, RFLPs, SNPs etc; thousands, covering 30,000 to 40,000.
marker loci all the chromosomes, are now available. It is desirable to • Only 1.1-1.5% of the genome
flank the gene on both sides, clearly delineating it. codes for proteins.
Radiation hybrid Now the most rapid method of localizing a gene or DNA • There are wide variations in
mapping fragment to a subregion o f a human chromosome and feature of individual
constructing a physical map. chromosomes (eg in gene
Use of rodent of human Permits assignment of a gene to one specific chromosome number per Mb, SNP
somatic cell hybrids but not to a subregion. density, GC content,
Fluorescence in situ Permits localization of a gene to one chromosomal band. numbers of transposable
hybridization elements and CpG islands,
Use of pulsed-field gel Permits isolation of a large DNA fragments obtained by use recombination rate).
electrophoresis (PFGE) of restriction endonucleases (rare cutters) that result in very • Human genes do more work
to separate large DNA limited cutting of DNA. than those of the roundworm
fragments or fruit fly (eg alternative
Chromosome walking Involves repeated cloning of overlapping DNA segments; the splicing is used more
procedure is laborious and can usually cover only 100-200 frequently).
kb. • The human proteome is
Chromosome jumping By cutting DNA into relatively large fragments and more complex than that
circularizing it, one can move more quickly and cover greater found in invertebrates.
lengths of DNA than with chromosomal walking. • Repeat sequences probably
Cloning via YACs, BACs, Permits isolation of fragments of varying lengths constitute more than 50% of
cosmids, phages, the genome.
plasmids • Approximately 100 coding
Detection of expression The mRNA should be expressed in affected tissues regions have been copied
of mRNAs in tissues by and moved by RNA-based
Northern blotting using transposons.
one or more fragments of • Approximately 200 genes
the gene as a probe may be derived from
PCR Can be used to amplify fragments o f the gene; bacteria by lateral transfer.
• More than 3 million SNPs
DNA sequencing Establishes the highest resolution physical map. Identifies
have been identified
open reading frame. Facilities with many high throughput
instruments could sequence millions of base pairs per day.
Databases Comparison of DNA and protein sequences obtained from
unknown gene with known sequence in databases can
facilitte gene identification

_______ Nude (Nu) Mouse / Nude Defect_______


Nude defect is characterized by absence of both hair and thymus. A profound T cell defect can result from faulty thymus
development (eg Di George syndrome d/t detection in long arm of chromosome 22 and CHARGE - coloboma of eye, heart
anamoly, choanal atresia, retardation, genital and ear anomalies syndrome d/t CHD 7 deficiency) leading to virtually no T
cells. However, expansion of oligo clonal T cells can occur; these cases require thymic graft.
Nude mouse is a laboratory mouse mutant with disruption of FOX N1 gene (an autosomal recessive mutation on chromosome
11 nu locus), causing lack of body hair and absent /deteriorated thymus (=thymus aplasia)®. Because of lack of thymus,
nude mice can not produce mature T cell lymphocytes resulting in deficiency/greatly reduced number of T cells. Nude
mouse do not generate cytotoxic effector cells, CD 8/4 T cells and are insusceptible to graft versus host disease. Therefore
these can receive various types of Xenograft and allograft transplants, in form of tissue and tumor grafts and sustain them
for weeks® (used in research to test new methods of imaging & treating tumors).
Nude mouse are unable to mount most immune responses such as
1. Lack graft rejection (d/1 lack of CD4+ and CD8 + T cells)
2. Lack cell mediated immune response (which requires CD4 + and/or CD8 + Tcells)
3. Lack delayed type hypersensitivity response (requires CD4 + T cells) and some antibody formation that require CD4
+ helper T cells.
4. Lack killing of malignant / virus infected cells (which requires CD8 + cytotoxic cells)
Most nude mice are leaky (more so on aging) and have few T cells. Therefore, now these are less popular in research and
knock out (eg RAG1 & RAG2 knock out) mice with more complete defects in immune system are preferred.
726 ■ A Complete Review of Short Subjects

Genetically Modified Animals


G e n e t i c a l ly m o d if i e d a n i m a l s are produced by transgenic pronuclear injection, forward genetics (i.e. mutations created
randomly by ENU=N-ethyl-N-nitrourea), targeted knock in and knock out, congenic strains (mating of inbred donor, diseased
phenotype with inbred recipient strain to identify genome responsible for disorder) and cloning.
Transgenic Expression (Animals) Targeted (Directed or Conditional) Approaches;
>Transgenic animals are produced by pronuclear injection Knock out & Knock In
of transgene (genomic DNA or cDNA construct) into the
Creation of gene knock in and knock out (targeted) take
nuclei of fertilized ovum. There is no way to control where
advantage of the fact that a segment o f DNA (gene) can be substituted
that gene eventually resides (ie random integration o f
by another homologous (identical) or nearly identical DNA segment by
transgene). Transgene has variable copy number and
recombination. This allow integration of deletions that disrupt
variable expression in each individual founder.
gene function ( ta r g e te d k n o c k o u t) or integration of selected
Transgenic (animals) expression can be_________________
mutations into t a r g e te d g e y e (i.e. targeted knock in)._________
1. L o s s o f f u n c t i o n m o d e ls using a n t i s e n s e and
d o m in a n t n e g a t iv e t r a n s g e n e s
T a rg e te d K n o c k In T a rg e te d K n o c k O u t (G e n e
2. G a i n o f f u n c t i o n m o d e ls d/t o v e r e x p r e s s io n using
A n im a ls D i s r u p t i o n ) A n im a ls
tissue specific promoter
3. Inducible expression (tetracycline, ecdysone)________ • Introduction o f a • It is substitution of functional gene with
»It can be produced by microinjecting one or few cloned subtle mutation (s) inactive gene by homologous
genes into the nuclei o f fertilized egg (ovum) o f into a gene by recombination in embryonic stem cell,
experimental animal (eg mouse)®. The successful substitution of a • It is selective removal o f a gene from
introduction of recombinant DNA into an animal can be normal endogenous genome. Gene knockout animals are made
illustrated by an experiment that permanently altered an gene with a mutant by creating a mutation that totally disrupts
easily observable inheritable physical trait. Microinjection gene (carrying a the function o f a gene. In other words,
of DNA into the nuclei of fertilized mouse eggs can specific mutation) rather than introducing a functional
produce efficient transformation (chromosomal by homologous gene into animal, a non functional
integration). When the injected eggs are introduced into a recombination in version is inserted,
female mouse & allowed to develop, the new gene is often embryonic stem cell • It is a way of establishing the function o f
expressed in some of new bom mice. is called (ta r g e te d ) inactivated gene or to study
k n o c k in . It occurs corresponding human disease,
* Unlike, somatic cell gene replacement therapy (or gene
therapy) which would not be passed on to offspring, this d /1 substitution of • Both Mi RNAs and DNA recombinases
process o f generating transgenic animal would alter germ cell endogenous gene inactivate gene expression,
lines and be passed on to offsprings. with gene carrying a • Techniques disrupting targeted genes
»If gene becomes successfully integrated into a subtle mutation (i.e. take advantage of particular promoter
chromosome, it will be present in both somatic cells and adding few genes). enhancer combinations driving
germline of resulting animal & is passed along from •These are expression of DNA recombinases or mi
generation to generation. predominantly used RNAs, both o f which inactivate gene
*So transgenic animals have same genome as parents in mice and can expression (targeted knock out).
except few (1 or more) genes® that have been integrated or accurately model • It is predominantly used in mice. Tissue
inactivated. Those in which the germline has been altered human disease specific knock out is possible (Cre /
can be identified by testing their offspring. By careful •Targeted expression Iox). Absence o f phenotype may occur
breeding of these mice, t r a n s g e n ic l in e in which all the of onchogene is d/t gene redundancy® or subtle
mice are homozygous for new gene or genes can be used to study phenotype. For eg d e le t i o n o f H P R T
established. mechanism of g e n e (H p r t) d o e s n o t c a u s e
neoplasia & to c h a r a c t e r is t ic f e a t u r e s o f L e s c h N y h a n
* Hundrends of transgenic animals are established and
generate s y n d r o m e i n m ic e b/ o their reliance on
these are useful for analysis o f tissue specific effect on gene
expression and effects o f over production o f gene products (eg immortalized cell adenine phosphoribosyl transferase
those from the growth hormone gene or onchogenes) and in lines. Eg expression (APRT) in purine salvage. D e l e t i o n o f
discovering genes involved in development. of simian virus (SV) r e t i n o b l a s t o m a g e n e ( R b ) e n c o d in g p
40 large T antigen 1 0 5 d o e s n o t 1/t r e t i n o b l a s t o m a or other
»A giant super mouse is produced by injecting the gene for
under direction of human syndromes in mice. However,
rat growth hormone into fertilized mouse egg or
insulin promoter mice with combinatorial deletion of
transgenic cows or goats can be designed to produce
induces formation several Rb related proteins exhibit
human proteins like clotting factor in their milk.
of islet cell tumors. features similar to human disorder.
* Similarly genetic hypogonadism in mice can be treated (in
offsprings) by injecting DNA with coding sequence for 1 , 1
This replacement of one gene in embryonic stem cell creates a
GnRH (gonadotropin releasing hormone) precursor.
heterozygous transgenic animals. The mating in 2 such animals will
Biochemistry: Cytogenetics and M olecular Genetics, Recom binant D N A & G enom ic Technology ■ 727

• Transgenic stratigies can be divided into 2 main result in 25% o f homozygous offsprings (a/t mendelian law). Careful
approches breeding will establish t r a n s g e n i c ( a n i m a l ) l i n e in w h ich a ll
1) Expression of a gene by random insertion into genome an im a ls are h om ozy g ou s f o r n ew gen e/g en es® .
(pronuclear injection of trransgene)
F o rw a rd G e n e tic s
2) Deletion or targeted mutagenesis of a gene by
homologous recombination with the native endogenous - Mutations created randomly by ENU (N-ethyl-N-nitrourea)
gene (knock out, knock in). - Selection of phenotype followed by genetic characterization
• Transgenic expression of gene can be useful for studying - Useful for identifying novel genes
disorders that are sensitive to gene dosage. Eg over : -i Congenic strains
expression of PMP 22 gene 1/ 1 high level of peripheral - Mating of an inbred donor strain with a disease phenotype with
myelin protein 22 (type I Charcot-Marie tooth disease); an inbred recipient strain in order to define the genomic region
SRY gene in females induce formation of testis. T ransgenic responsible for the disorder
in trodu ction o fC A G trin u cleotide rep ea t cau ses - Useful for mapping disease-causing genes
H utingtons d isease, w h erea s targ eted d elitio n (kn ock out) Cloning
ofH u tin g ton 's d ise a se gen e d oes n ot induce n eu rolog ical - Introduction of nucleus into enucleated eggs (nuclar transfer)
disorder®, (ie gain of function for CAG encoded - Successful in several mammalian species including sheep (Dolly),
polyglutamine containing protein) mice, cows, monkeys
• Transgenic strategies can be t\sed as precursors to gene - Cloning of genetically identical individuals
therapy. Eg expression of dystrophin corrects Duchenne's - May effect lifespan
muscular dystrophy in mouse - Ethical concerns

Gene Therapy
- It implies to correct the basic genetic abnormality. The principle is to insert the normal, cloned DNA fo r defective gene into somatic
cells.
- G ene therapy is a procedure involving inserting (or sometimes deleting portions o f gene in diseased patients for the purpose of
cure. In somatic gene therapy, the recipient's genome is changed but the change is not passed along the next generation.
Whereas, in germ line gene therapy, change is passed to next generation.
- Antisense gene therapy or gene silencing selectively inactivate (or turn off) a mutated gene by RNA interference using short
synthetic double stranded siRNA (small interfacing RNA)
I
• Procedure involves Techniques to transfer genes (foreign material) into cells
- Replacem ent® : Mutant gene removed & replaced with - S ites d irected recom bnation®
normal gene. - Soil bacterium Agrobacterium tumefaciens (containing
- Correction : Only mutated area of affected gene would be Ti plasmid) and Agrobacterium E. coli are used for
corrected & the remainder will be unchanged. plants
- A ugm entation: Introduction of foreign genetic material into a - In tran u clear m icroinjection®
cell to" compensate for the defective product of mutant gene. - Transduction using viral vectors like retrovirus,
- Gene of interest is usually administered via a viral (retro, adenovirus, adeno associated virus, herpes simplex virus and
adeno, adenoassociated, lenti viruses etc) vector or non viral lenti viruses. Adenovirus lacks a mechanism for
plasm id liposom e (cationic lipid) com plex, used as gene integrating DNA into chromosome. Therefore adenoviral
transfer vehicle. vector is expressed for only a short time and then
•G ene t h e r a p y i s u s e d i n ___________________________________________ destroyed. This is useful if the objective is transient
- S C I D (severe com b in ed im m unodeficiency disease) or expression o f gene. Adenovirus are used for
bubble bay syndrom e, which occurs d/t mutation in either introduction of cystic fibrosis gene in respiratory tract
adenosine deaminase gene or a X-linked gene coding for an cells. Other possible vectors are- Epstein Barr viruses,
interleukin receptor subunit, is the first disease that was herpes, simian virus 40, papilloma, non human
successfully treated by gene therapy. lentiviruses, and hepatitis viruses.
- C ystic fibrosis® - Transfection using non-viral methods like plasmid
- Familial hypercholesterolemia liposome complex using calcium phosphate precipitated
- Duchene muscular dystrophy, Hemophilia, Leber's DNA. Liposomes (cationic lipids) are small vesicles
hereditary optic neuropathy. consisting of lipid bilayer that encloses an aquous
- Lesh - Nyhan syndrome, phenylketonuria (PKU) are most likely compartment.
candidates for future gene therapy. Single gene disorders are - Electroporation : It employs pulsed electric field using
best candidates for gene therapy. However, unfortunately small electric current, which transiently increase the
most prevalent disorders such as heart disease, high BP, permeability o f plasma membrane thus allowing intake of
Alzheimer's disease, diabetes and arthritis are multigene extracellular macromolecules including DNA.
disorders. M n-"SITE "
728 ■ A Complete Review of Short Subjects

Inheritance of X-linked Recessive Disease

X linked recessive inheritance is a sex M a le F e m a le An affected male does not transmit


linked disorders i.e. they are the disorder to his sons, but all
&
transmitted by sex chromosomes. daughters are carriers
Remember that all sex linked diseases Sons o f heterozygous woman have
[xVl (xjxh-|xfcl (xx> -H U
are linked to X chromosome so it is 50% chance o f receiving the
quiet obvious that none of the sex abnormal gene
linked disease (whether X-linked It usually affects males because o f
I X Y | (X X ) (X X ) |X Y | |?| y | I X Y l ^ x ) $| x)
recessive or X-linked dominant) will be the presence o f Y chromosome which
transmitted from father to son, as is not homologous to X chromosome
[xy] Normal female and male
father transmits only 'Y' chromosome and therefore the mutant genes on
to the son. Other characteristic of this Female carrier the X is not paired with alleles on
disorder is that it is mostly seen in the Y
males. Where as the diseases Affected male
It is rare in females only possible
transmitted through non sex when mother is carrier and father
Affected female
chromosomes (autosomes) have equal has overt disease.
male and female incidence.
,_____________________ I_____________

Most of the X linked disorders X-linked Recessive Disorders


are X linked recessive. There are (1) Duchenne & B ecker m uscular dystrophy® (8) Fragile X syndrome
only few X-linked dominant (2) H em op h ilia A an d B® (9) Lesch Nyhan syndrome
conditions. These are (3) Chronic granulomatous disease (70% XLR, 30%> AR) (10) Fabry disease
- Pseudohypoparathyoidism (4) G lu cose 6 -PO 4 dehydrogenase deficiency® (11) Hunter's disease
- Hypophosphetemic rickets. (5) Agammaglobulinemia (12) Colour blindness
- Vit D resistant rickets (6 ) Wiskott Aldrich syndrome (13) Ocular albinism
- Incontinentia pigmenti (7) Diabetes insipidus (14) Sideroblastic anemia

Autosomal Recessive (AR) Disorders

I
Metabolic Hematopoetic Nervous
All Albinism0 See Sickle cell anemia0 Friedrich ataxia®
Thalasemia - a & P° Neurogenic muscular atrophy
Anti a l Antitrypsin deficiency0 And Leucocyte adhesion
Spinal muscular atrophy
deficiency
Ha Hartnup disease0 Check Chediak - Higashi Skeletal
He Hemochromatosis0 syndrome0
AH Alkaptonuria
Ho Homocystinuria0 My Myeloperoxidase
Pets Osteopetrosis
Gla Galactosemia0 deficiency
Fail Klippel Feil syndrome
Gly Glycogen storage disease0 Fan Fanconi syndrome Don's Danlos Ehler syndrome
La Laurence Moon Biedl Exam
Syndrome0 Mn - "See And Check My Fan"
Urinary
Ly Lysosomal storage disease0
Endocrine - Barter syndrome®
Will Wilson's disease0
- Congenital adrenal hyperplasia - Gitelman syndrome®
Use Usher's syndrome0
- Pendred syndrome - Nephronophthisis
Cystic Cystic Fibrosis0
- Nephrogenic diabetes insipidus (also XL)
Urea Phenylketonuria0
- AR polycystic kidney diseasse®
Mn - "All Anti Ha He Ho Gla Gly, La
Ly will Use Cystic Urea" Gastrointestinal
- Familial Mediterranean fever
Other
Bloom syndrome
Biochemistry: Cytogenetics and Molecular Genetics, Recombinant D NA & Genom ic Technology ■ 729

Genetically Transmitted Disorders


Normal parents means parents who are 'clinically normal' with no manifestation of any 'disease' during any part of their life
time. But these may still be 'abnormal' (1) genetically i.e. carrying single allele of recessive disease that needs 2 recessive alleles
to be manifested or (2 ) non genetically carrying silent or infective agents.

I I I 1
Autosomal dominant (AD) Autosomal recessive (AR) Polygenic Vertically
I I disorder transmitted disorder
For a child with an autosomal For a child with an autosomal
I I
dominant disorder to be affected recessive disorder to be affected
he needs to have atleast two A trait is called Passage of disease
at least one parent needs to have
recessive alleles, one from each 'polygenic' if causing agent
the affected allele
parent. multiple genes on (pathogen) 'vertically'
Because the allele is dominant it will
thought to from mother to baby
always express phenotipiclly and B ecau se the a llele is recessive, it
contribute to the during the perinatal
hence both parents can not be normal m ight n o t express in either
phenotype. period is called
Genotype of Affected Child with paren ts an d both p aren ts can be
Polygenic disorders 'vertical transmission'.
autosomal dominant disorder p h en oty p ica lly norm al.
include A v ertically
Genotype of Affected Child with
Hypertension tran sm itted agent
Tt Tt autosomal recessive disorder night lie dorm an t in
Diabetes, Asthma
(phenotypically (phenotypically the m oth er an d n ot
abnormal parent normal parent 2) Tt Tt A m a le in fan t w ith
m an ifest in her
D_ (phenotypically (phenotypically asth m a o r d ia b etes
clin ically (norm al
llrSrTS] normal parent 1) normal parent 2) m ay w ill be b o m to
n orm al parents
m other) but m ay
m an ifest in the child.
Tt
(affected offspring TT
in AD disorder) (affected offspring
in AR disorder)

Types of Defects

Genetic Congenital Malformations


I (eg Teratogenic)
Single gene defect Polygenic (Multifactorial) Chromosomal Abnormal development
(Mendelian disorder) Diabetes Pyloric stenosis - Abnormal unrelated to genes or
■ Autosomal dominant Hypertension Hirschspning number of chromosomes.
disorders Coronary / ischemic disease chromosomes (eg Teratogenic event)
■ Autosomal recessive heart disease Cleft lip or cleft (Trisomy,
disorders Congenital heart palate Monosomy etc)
X linked dominant disease Gout Polyploidy
disorders Neural tube defect Schizophrenia - Abnormal
X linked recessive shape /Structural
disorders abnormalities

Wadenburg Syndrome Type -1

Waardenburg's Syndrome (WS) is a rare a u to so m a l d om in an t syndrom e characterized by pigmentary disturbances (skin, hair, iris),
sen sorineural hearing loss, and other developmental anomalies such as d y stop ia canthorum (w id ely sp a ced eyes) and
blepharophimosis.
730 ■ A Complete Review of Short Subjects

1-----------------------------
Type I (Waardenburg) Facial features Aural features
D istinguishing Dystopia Broad high nasal root - Sensineural hearing loss (SNHL) (deafness)
Features canthorum Synophrys or medial flaring, Eye features
or both, of the eyebrows - Complete or segmental heterochromia
(Widely spaced
Hypoplastic alae nasi - Brilliant sapphire blue eyes
eyes)
Dystopia canthorum (lateral Skin features
Hearing loss (25%)
displacement of inner canthi - Hypopigmentation, possibly on the face,
Inheritance Autosomal or w id ely spaced eyes or trunk, or limbs with or without an associated
Dominant (AD) hypertelorism) white forelock
Gene Defect/ P Hair features - Patches ofhyperpigmentation in some
Function AX3/transactivates White forelock (present at families with WS
promotor for MITF birth or developing later; also Less commonly associated findings
may disappear later) White - Neural tube defects
body hair, eyebrows, - Sprengel shoulder (ie, congenital upward
eyelashes scapular displacement)
Dark tufts of hair or black - Cleft lip or palate
forelock - Hirschsprung disease (primarily in cases of
Premature graying (i.e. <30 y) WS4 but reported in families with WS I and
WS2) Contractures and limb muscle
hypoplasia (in patients with WS3)

Major Histocompatibility Complex (MHC)/ Human Leukocyte Antigen (HLA) Complex


• In humans the gene encoding MHC or HLA complex are clustered on a sm all segment (4 megabase region) o f chromosome 6
short arm® (6p 21.3)®. HLA class I and class II genes whose products are critical for immunological specificity and
transplantation histocompatibility® and they play a major role in susceptibility to a number of autoimmune diseases.
• Function: Binds peptide fragments of foreign protein for presentation to appropriate antigen specific T- Cells. Thus involved in:
1) Transplantation reaction® 2) Immune response & tolerance 3) Disease susceptibility (autoimmune ds®, infectious &
Inflammatory disease) 4) Complement component.
• They are classified into 3 categories

Feature Class I Antigen (MHCI) Class II Antigen (MHC II) Class III (MHC III)
Expressed on A ll n ucleated cells & platelets® Antigen presenting cells including D endritic Class III is present
(presented by) cells® in lymphnode & spleen, Langerhans in between I & II
cells®, Macrophages, B cells® & a c tiv a ted T
cells®
Encoded by 3 closely linked loci designated HLA-D (HLA DP, D®, DR) C4B, BF, C2 TNF a
(component loci) HLA-A, HLA-B and HLA-C &P
Encoding Cell surface glycoprotein involved C ell surface glycoprotein involved in Components of
molecule in antigen presentation antigen presentation complement
system
Recognized by CD 8 + Tcells®, NK cell recognizition CD4+ helper Tcells®
Function - Immune response against - Graft versus host response
intracellular infection, tumors, - Mixed leucocyte reaction (MLR)
allografts - Influence functional capacity of mature
- Graft rejection peripheral T cells.
- Cell mediated cytolysis
- Component of hormone receptor
Chapter 5. STRUCTURE & FUNCTIONS OF PROTEINS AND ENZYMES:
REVIEW NOTES

Structural Organization of Protein

Primary Strue hire Secondary Structure


- Primary structure is linear sequence of - Secondary structure of protein refers to description of steric
aminoacids held together by covalent bonds relationship (spatial arrangement) between aminoacids located
(mainly peptide and disulfide bonds). It is relatively near each other in a selected segment of main polypeptide
insufficient to perform chemical /physical task. chain, without regard to the conformation of its side chains or its
- The aminoacids are numbered from N terminal relationship to other segments.
end (free NH2/amino group of terminal amino - A regular secondary structure occurs when each dihedral angle ($ &
acid) as no 1 towards the C (free carboxy) terminal. \y) remains the same or nearly the same throughout segment. Where a
Peptide bonds form the backbone and side chains regular pattern is not found, the secondary structure is called
of aminoacid residues project out side the peptide undefined or random coil (misnomer as polypeptide back bone in any
back bone protein is not random).

•a-carbons of adjacent aminoacid residues are - The secondary structure of a polypeptide segment can be completely
separated by three covalent bonds C„ - C - N - Ca. defined if ()>& \j/angles are known for all aminoacids.
The peptide (C-N) bond has a partial double - Circular dichroism (CD) spectroscopy, based on principle that
bond character that keeps the entire 6 atom structural asymmetry in a molecule gives rise to differences in
peptide group in a rigid planar configuration. absorption of left handed versus right handed plane polarized light, is
Peptide C-N bond cannot rotate freely; rotation is a method for assessing secondary structure and moniter folding in
permitted about N-C0 and C „ - C bonds. So proteins. It can determine whether proteins are properly folded,
backbone of polypeptide chain can be pictured as a estimate the fraction of protein that is properly folded in either of
series of rigid planes sharing a common point of common secondary structures, an moniter transition b/w folded &
rotation at Ca. unfolded states. For proteins spectra are obtained in far UV region
Peptide conformation is defined by 3 dihedral (190 to 250nm).
angles (or torsion angles) called 0 (phi) with - Three dimensional secondary structure is produced by folding or
rotation occurring about N-Ca bond; \|f(Psi) with coiling of short (3-30) contiguous segments of polypeptides (peptide
rotation occurring about Ca-C bond ; and to chain primary structure) and production of cross linkages or bonds
(omega) with central peptide (C-N) bond, where mainly hydrogen bonds. It is insufficient to perform a particular physical
rotation is constrained and mostly (99.6% times) or chemical task. Organization may result in formation of geometrically
peptide bond is in trans configuration (omega = + ordered units such as a helix, (3 sheets, loops and bends-
1 8 0 ° ) or very rare in cis configuration (co=0°). I--------------------- -------------------- 1
Theoritically values of cj) & q; can be between - 1 8 0 ° a- Helix P- Pleated Sheets
and + 1 8 0 ° but many values (eg both 0 °) are • Are regular- helical coils of peptide chain P- pleated structure
prohibited d/t steric interferences. Allowed values • Formed by intra chain hydrogen bonds®b/w is formed by inter
are obtained by Ramchandran plot. carbonyl O of 1st amino acid and amide N of chain hydrogen
4th amino acid residue. bonds®.
Tertiary Structure • a - helix may be left handed or right handed They are produced
- Left handed ahelix is less stable® than right when hydrogen

Tertiary structure refers to over all three
handed a - helix because of steric interference bonds are formed
dimensional arrangement of all atoms in a
b/w the C =0 and the side chains between carbonyl
protein. It indicates the 3-D confirmation of a
- Only right handed a - helix has been found in oxygen and amide
single polypeptide chain ie in 3D space how
protein structure hydrogen of two
secondary structure features helices, sheets, loops
• In one complete turn of a - helix there are 3.6 adjacent chains.
and bends assemble to form domains and how
amino acid residue. The adjacent chains
these domains relate spatially to one another.
• Proline is never found in a- helix®.
may be either
Tertiary structure refers to spatial arrangement o f parallel or
• Small or uncharged amino acids as alanine,
long range aspects o f am inoacids® (in comparison antiparallel.
phenylalanine & leucine are often found;
to secondary structure that includes adjacent Most common a.a
whereas more polar a.a. such as arginine,
aminoacids). Proteins with tertiary structure are to form P- pleated
glutamate & serine may repel & destabilize a-
sufficient to perform a particular task. sheet - Glycine,
helix.
732 ■ A Complete Review of Short Subjects

- On the basis of tertiary structure, proteins can be Alanine, Serine


A lpha (a) H elix (Pauley & Corey)
divided into structural fibrous proteins (having P roline occurs
•This spiral structure is the most common and
simple repeating elements of secondary structure) although it tends
stable secondary conformation (structure) for a
and globular proteins (often containing several polypeptide chain. The polypeptide bond to disrupts sh eet
types of secondary structure in same polypeptide by producing
forms the back bone, side chain of aminoacids
chain). kinks®
extend outward and the structure is stabilized
Silk fib ro in s a
- Complex structures of globular proteins can be by hydrogen bond b/w N— H & C= O groups
protein o f silk
analyzed by examining folding patterns called of main chain
w orm is rich in p-
m otifs (super secondary structures or folds). A • In a-helix, the polypeptide backbone is tightly
wound (twisted) around an imaginary axis p lea ted sheets®.
motif is a recognizable folding pattern involving 2
(drawn longitudinally through the middle of
Triple Helix
or more elements of secondary structure and the
Collagen rich in
connection (s) between them. It can be very simple helix) and side chains (R-groups) protrude
prolin e and
as ji -a - P loop or very elaborate such as P barrel. outwards from helical back bone. The
hydroxy proline,
Motif is not a hierarchial structural element falling polypeptide backbone of a-helix is equally
can n ot fo rm a -
between secondary & tertiary structure; it is a twisted about each a-carbon atom with a phi (<j>)
helix or p p lea ted
folding pattern that can describe a small part of angle = -57° and psi (vp) angle = -47°. Each
sheet. I t form s
protein or an entire polypeptide chain. A motif complete turn of helix contains ~ 3.6
triple helix®.
segment may or may not be independently stable. aminoacyl residues with translation (i.e.
Stabilized by
A domain is a p art o f p oly p ep tid e chain th a t can distance b/w each aminoacid) = 0.15 nm (1.5
interchain
fo ld stab ly and independently® or could undergo A°) and pitch (i.e. distance per turn) = 0.54 nm
hydrogen bonds
movements as a single entity with respect to the (5.4 A°).
b/w different
entire protein. Multiple domains of large proteins • Structure of a-helix is stabilized primarily by
polypeptide
may have different functions and may retain their hydrogen bonds formed b/w
chains.
native 3D structure even when separated. - Oxygen (O) of peptide bond carbonyl and
Thousands of known protein structures are hydrogen (H) atom of peptide bond nitrogen
Reverse Turns or
assembled from a repertoire of few hundred of 4lh residue down the peptide chain (Harper)
|J- Bends
motifs. - Hydrogen (H) atom attached to
P-bends reverse the
electronegative nitrogen atom of peptide
direction of
Quaternary Structure linkage and the electronegative carbonyl
polypeptide chain,
oxygen (O) atom of 4th aminoacid on
• Q u artem ary structure results fro m in teractions helping it form a
aminoterminal side of peptide bond
betw een the subunits o f m ulti subunit (m ultim eric) compact globular
(Lehninger).
proteins®. So quaternary structure is 3D chain.
• Every peptide bond (except those close to end
arrangement of multiple polypeptide chain Usually occur on
of helix) participate in hydrogen bonding. So
proteins the surface of
each turn is held to adjacent turn by 3-4
■Some multimeric proteins have a repeated globular protein
hydrogen bonds (supplemented by Van der
structural unit, consisting of a single subunit or a A.A frequently
W alls interactions), conferring significant
group of subunits known as promoter. Promoters found in P- bends
stability.
are usually related by rotational or helical Proline
• Experimentally, a-helix can form in
symmetry (ie individual subunit can be super Glycine
polypeptides consisting of either L o r D
imposed on other). Examples of rotational
symmetry include cyclic symmetry, dihedral
stereoisomeric aminoacids, with all residues of Super Secondary
symmetry and icosahedral symmetry (in order of one type because mixing o f D & L aminoacids Structures
will disrupt the structure. Theoretically, Greek key
complexity).
naturally occurring L aminoacids can form p-meander:
•A multimeric protein with few subunits is called
oligomer. Dimer contains 2 polypeptide chains. either right or left handed a-helices but consists o ( 5
extanded left handed a-helices are unstable pleated sheets
Homodimers Heterodimers and not observed. connected by reverse
Contain two copies Contain different • Alanine show the g rea test tendency to form turns. It contains
of same polypeptide polypeptide subunits a-helix®. Keratins protein of hair, nail and nearly as many
chain skin and hemoglobin /myoglobin are rich in a- hydrogen bonds as
Represented as a P
Represented as X 2 helical structure, whereas chymotrypsin an a-helix.
virtually lack it. Not all aminoacids form a- P- a- P unit
Examples: Creatinine kinase (dimer), lactate
dehydrogenase (tetramer), hemoglobulin helix, prolin e & glycine h av e lea st p rocliv ity
(tetramer of 2 alpha + 2 beta chains) and to form a-helixes®.
immunoglobulin (tetramer of 2 heavy + 2 light
chain)
Biochemistry: Structures & Functions of Proteins and Enzymes ■ 73 3

3 -M E A N D E R

Aminoacids Affecting Stability or Disrupting a-Helices

- Proline G lycine Charged aminoacids When present in Am inoacids at the end of


- Hydroxy-proline I Disrupt a-helix by forming ionic large number or helix
I B/o small bonds or electro statically repelling close together - 4 aminoacids at each end do
Since peptide size, and each other not fully participate in
bond nitrogen of Bulky side hydrogen bonding
proline lacks a conformation + - chains • Electrical dipole of each
hydrogen atom to al flexibility, it - Arginine - A sp artate - Tryptopan peptide bond is aligned
contribute to a induces - Lysine (asp artic acid)® through hydrogen bonds of
hydrogen bond, it bends. - Histidine - Glutamate A A that branch helix, resulting in a net dipole
can only be stably (glutamic acid) at f) carbon (i.e. along the helical axis that
accommodated ★ For stability +ve charged aa are 1st C in R group increases with helix length
within the first turn found 3 residues away from -ve next to a-C) ■ For this reason negatively
of a-helix. When charge aa. - Valine charged aminoacids are
present else - Isoleucine found near amino terminal
where, proline and positively charged amino
Bulk & Shape of
disrupts a-helix. acids are found near carboxy
AA
Nitrogen atom of terminal of a-helix; where
- Asaparagine
proline is a part of they have stabilizing
- Serine
rigid imino ring & interaction
- Threonin
rotation about N- ____________ I___________
Ca bond is not Positive (+) charged
possible. Hence aminoacid at amino (+)
proline terminal) and negatively (-)
introduces a charged aminoacids at
destabilizing carboxy (-) terminal of a-
kink in a-helix. helix is destabilizing._______

Various Types of Bonds

Hydrophobic (weak) interactions are predominantly important for protein's stabilizing conformation (protein stability)
Hydrophobic residues are largely buried in the protein interior, away from water and the number of hydrogen bonds and
ionic interactions with in protein is maximized, thus reducing the number of hydrogen bonding & ionic group that are not
paired with suitable partner. Because of favourable free energy change and cooperative bonding (i.e. one bond favours the
formation of next bone), the hydrogen bonds often h av e an im p ortan t role in guiding the protein fo ld in g process®.
(Langerhan's 115)
After folding into their native conformations, some protein form interchain or intrachain disulfide bridges between Cys residues. In
eukaryotes, disulfide bonds are common in proteins to be exported from cells. The cross links formed in this way help to
protect the native conformation of protein from denaturation in extracellular environment, which can differ greatly from intracellular
conditions & is generally oxidizing, - (Lehninger)
734 ■ A Complete Review of Short Subjects

Higher order of protein structure (tertiary & quaternary) are stabilized primarily & often exclusively - by non covalent
interactions, principle among these are h y drop h obic in teractionn s® that drive most hydrophobic aminoacid side chains into
the interior of protein shielding them from water. Other significant contributors include hydrogen bonds and salt bridges
between carboxylates of aspartic and glutamic acid and the oppositely charged side chains of protonated lysyl, argininyl, and
histidyl residue. Collectively these confer a high degree of stability to the biologically functional conformation of protein.
Some (not all) peptides form covalent disulfide (S - S) bonds linking sulfhydryl group of cysteinyl residues by oxidation
(requiring O 2). Intrapolypeptide disulfide bonds enhance the stability of folded conformation of a peptide, while
in terpolypeptide disu lfide bon d stabilize quarternary structure of certain oligomeric proteins.
D isulphide bonds: are the S- S bonds between - SH group of two distant cysteine residues. A given cysteinyl residue can
form many disulfide bond pairs, only one of which is appropriate for biologically folding. The enzyme disulphide isomerase
facilitates shuffling of - S-S bonds by accelerating the rate at which disulphide undergo mutual exchange, thereby accelerating
process of protein folding and stabilizing proteins native conformation.

Bond M ainly Responsible For


I
Primary Secondary structure & Higher order Protection of native conformation of Help in folding
structure Folding___________ (tertiary & protein from denaturation I
I quaternary) I Auxiliary protein
P eptide H ydrogen bond® protein structure D isulfide (Covalent) bond Chaperones (Hsp-
bonds® High degree of I__ 70)
I I
stability to Chaperonins (Hsp-
★ Chaperones (i.e. Hsp 70 or 70 kDa Intrapolypeptide Interpolypeptide
biologically 60)
heat shock protein) bind hydrophobic disulfide bond disulfide bond
functional Disulfide
aa., shield them from solvent, prevent I I
conformation of isomerase
aggregation, thus providin g Enhance sta b ility Stabilize
protein (Protein Disulfide bonds
opportun ity fo r fo rm a tio n o f o f fo ld ed Quarternary
stability)
ap p rop riate 2 ° structure and folding®. conform ation® structure of
Chaperonins (Hsp 60) act later in oligomeric
folding process by providing sheltered proteins
N on cov alen t
environment (d/t its donut shape) in H yd rop h obic
which a polypeptide can fold until all bonds® (main)
hydrophobic regions are buried in its Hydrogen bond
interior, eliminating aggregation. Salt bridges
★ Auxiliary proteins assist folding

Isopeptide Bond

Isopeptide bond is an amide bond formed between the side (not main) chain carboxy terminal of one protein (with either
glutamate or aspartate) and amino group of lysine residue on side chain of another target protein.
Bond formation is either spontaneous (in gram positive bacterial pilli & HK 97 bacteriophage capsid formation) or enzyme
(transglutaminase) catalyzed (between lysine & glutamine). Spontaneous proximity induced isopeptide bond formation
requires presence of glutamic acid, and is used in development of isopeptag (a new peptide tag) which can spontaneously &
irreversibly react with its target through a covalent isopeptide bond. This molecular tool is used in development of new
protein architectures - in vivo protein targeting and fluorescent microscopy imaging.
Isopeptide bond formation is involved in post translational m odification of proteins. Interchain isopeptide bonds make
proteins resistant and produce stably linked protein dimmers, multimers & complexes like blood clots, as only few
enzymes can hydrolyze this bond.
A small peptide Glutathione, has isopeptide bond b/w side chain of glutamate & amino group of cysteine. Similary
ubiquitin protein's C terminal glycine residue gets attached to lysine side chain of other protein by isopeptide bond.
Biochemistry: Structures & Functions of Proteins and Enzymes ■ 735

Keratin: Fibrous - Intermediate Filament (IF) Protein

- Keratin is a fibrous protein and like all other fibrous proteins (such as IF P ro tein s
collagen & silk fibrion) is adapted for structural function. Keratins, like
all other fibrous proteins (eg collagen & silk fibrion) are insoluble in Alpha -keatins IF Proteins Distribution
water, a property conferred by a high concentration of hydrophobic are part of a Keratins Epithelial
amino acids both in interior of protein and onits surface broader family -Type I (Acidic) cells, hair,
Right handed Alpha Helix and Left Handed Super Helix with Plenty of of proteins k/a -Type II (Basic) nails, horns,
Hydrophobic Aminoacids. intermediate hooves,
- a-keratin (helix) secondary structure is right handed a helix. Two strands filament (IF) feathers etc
of a-keratin (right handed helix), oriented in parallel (with their amino terminals proteins. IF is Lamins A, B Nuclear
at the same end), are wrapped about each other to form a super twisted an intracellular and C lamina
coiled coil with its helical axis twisted in a left handed super helix fibrous system Neurofilaments Neurons
(forming tertiary structure). The super twisting amplifies the strength of of filaments - L (low),
with an axial - M (medium),
the overall structure, just as strands are twisted to make a strong rope.
periodicity of 21 - H (high)
This super twisting to form a coiled coil is also responsible for discrepancy
nm and a Vimentin like
between the 5.4 Ac per turn predicted for a helix by Pauling & Corey and
diameter of 8-10 - Peripherin - Neurons
5.15 to 5.2 A° repeating structures observed in x-ray diffraction (of hair).
nm that is - Glial - Glial cells
- The areas (surfaces) where 2 -a helixes touch each other are made up of
intermediate fibrillary acid
hydrophobic aminoacid residues, their R groups meshed together in a
between that of protein
regular interlocking pattern - this allows a close packing of polypeptide
microfilaments - Desmin - Muscle
chains with in left handed supertwist. That's why a keratins are rich in - Vimentin - Mesenchymal
(6nm) and
hydrophobic am inoacids Ala, Val, Leu, lie, Met and Phe®. cells
microtubules
Disulfide Bonds & Strength of Quaternary Structure
(23nm).
- Intertwinging of 2-a helical polypeptides (coiled coils) forms quaternary
structure of keratin. It is stabilized by disulfide cross links between pairs
They are all elongated, fibrous molecules with a
o f cysteine (Cyst) residues o f adjacent polypeptide chains® (enhancing the
central rod domain, an amino terminal head and a
strength of fibrous proteins).
carboxy terminal tail. Mature filaments are
- Protein a keratin, formed by all vertebrates, is the chief structural
composed of tetramers packed together in a
component of hair, nails, horns, hooves, wool, feathers, scales, claws,
helical manner forming a structure like rope. They
quills and much of the outer layer of skin. So a- keratins form tough,
are structural component of cells, and most are
insoluble protective structures of varying hardness and flexibility evolved relatively stable components of cytoskeleton, not
for strength. In a-K eratin s, the quaternary structure (formed by undergoing rapid assembly and disassembly and
intertwinging o f 2-a-helical polypeptide chains) is stabilized by disulfide not disappearing during mitosis as do actin and
bond cross links between pairs o f cysteine (cyst) residues o f adjacent microtubular filaments. Lamins, which form a
polypeptide chains®. mesh work in apposition to inner nuclear membrane, is
- The extent of hardness, strength and flexibility of a-keratins depends on an important exception - lamins subsequent to
the number of disulfide bonds (sulfur content / cysteine residues). The phosphorylation disassemble at mitosis and
more the number o f disulfide bonds between Cys residues o f adjacent reappear when it ends.
polypeptide chains, the more the hardness, toughness & strength and
lesser the flexibility o f the keratin isQ. So decreasing order of disulfide
bonds (or hardness) is ----------------------------------------------------------------
- Hair can be waved/curled by moist heat and stretching out a-helix to
fully extended P -conformation. Reducing agent (thiol /or sulfhydryl
group) is applied with moist heat - the reducing agent cleaves disulfide
cross link and moist heat breaks hydrogen bonds, thus causing a-helix to Least (Because of (Upto 18% of
uncoil. After some time reducing agent is washed and oxidizing agent is number of more disulfide residues are
applied to establish new disulfide bond pairs between Cys residues of disulfide bonds these are cysteines
adjacent polypeptide chains but not the same bond pairs as before the bonds; less flexible & involved in
more much harder disulfide
treatment. Similarly straightening of hair can also be done.
pliable than soft bonds)
(flexible) keratins)
than hard
keratins
736 ■ A Complete Review of Short Subjects

Glycoprotein

• Are protein s th a t contain olig osacch arid e (glycan ) chains cov alen tly attach ed to their p oly p ep tid e b ack b on esQ
• Carbohydrate chain is usually sh ort ( 2 - 10 units), branched & h av e sev eral rep eatsQ. Their carbohydrate content ranges
from I- 85% by weight.
• Once a glycoprotein has been purified, the use of mass spectrometry and high resolution NMR spectroscopy can often identify
the structures of its glycan chains. Analysis of glycoproteins can be complicated by the fact that they often exist as
glycoform s; i.e. protein s w ith id en tical am in oacid sequence but som e w h a t d ifferen t olig osacch arid e com p osition s 0 .
• Oligosacchride chains encodes biological information and this depends upon their constituent sugars, their sequences and their
linkages.
• Oligosacchride chain of glycoprotein is responsible for - physiochemical properties (eg solubility, viscosity, charge,
conformation, denaturation & binding sites), biological activity, protection against proteolysis, insertion into membranes,
intracellular migration, sorting and secretion. It affects embryonic development & differentiation and site of metastasis.
• Eight sugars predominate in glycoproteins and these are mannose, fucose (are GDP linked); glucose, galactose, xylose, N-
Acetyl galactosamine, N- Acetyl glucosamine (are UDP linked) and N- Acetyl neuraminic acid (linked to CMP to form
nucleotide sugar)
• N- Acetyl neuraminic acid (Neu Ac) is usually found at the termini of oligosaccharide chains attached to subterminal galactose
(Gal) or N- acetyl galactosamine (Gal Nac). Other sugars are in internal positions. Neuraminidase is an exoglycosidase (i.e. act
externally); endoglycosidase F & H act internally; where as galactosidase is both exo & end glycosidase
• Based on their linkage between their polypeptide chains & oligosaccharide chains; glycoprotein can be of 3 types:
i) O- linked i.e. containing O- glycosidic linkage, involving
hydroxyl chain of serine/ threonine and sugar N- acetyl Three inhibitors of enzymes involved in the
galactosamine (G al N ac - Ser /Thr) glycosylation of glycoproteins and their sites of action
ii) N- linked i.e. containing N- glycosidic linkage, involving Site o f Action
Inhibitor
amide nitrogen of asparagine and N- Acetyl Glucosamine
Tunicamycin Inhibits GlcNAc-P transferase, the
(G lc N ac - Asn)
enzyme catalyzing addition of
iii)GPI - \\nkedl G ly co p h osp h a tid y l in o s ito l-lin k e d i.e.
GlcNAc to dolichol-P, the first step
glycan (oligosacchride) is linked to carboxyterminal of a.a.
in the biosynthesis of
via phosphoryl ethanolamine.
oligosaccharide-P-Pdolichol
• G lycophorin a R B C g ly cop rotein contain b oth O an d N lin ked
Deoxynojirimycin Inhibitor of glucosidases I and II
oligosacchridesQ . Almost all plasma proteins except albumin
Swainsonine Inhibitor of mannosidase 11
are glycoproteins. Selectins (a glycoprotein) play a key role in
inflammation & lymphocyte homing. Zona pellucida contain
glycoproteins ZP 1-3, that functions as a receptor for sperm (esp ZP - 3) & induce acrosomal reaction.
• Asialoglycoprotein receptor is involved in clearance of glycoprotein from plasma by hepatocytes.
• Tunicamycin, deoxynojirimycin & swainsonine inhibit N but not O- glycosylation. The extension of O- linked chains can be
prevented by Gal NAC - benzyl.

Glycosaminoglycans (GAG) Proteoglycan


•GAGs are large com plexes o f n egatively charged, long, - GAGs are gen erally a sso cia ted w ith a sm all am ou nt o f protein
unbranched, hetrop oly sacch arid e chain s gen erally form in g p roteog ly can s w hich contain > 95% ca rb oh y d rateQ.
com p osed o f a repeating d isacch arid e unit [acid ic sugar - Whereas, glycoproteins contain primarily protein & a small
aminosugar]„Q amount of carbohydrate
•They h old large am ount o f waterQ thereby producing the - All the GAGs, except hyaluronic acid, form proteoglycans. It is also
gel like matrix that forms the basis of body's ground the only GAG which is not sulfated and also found in bacterias.
substance, & lubricating other structures - Proteoglycan monomer resembles a bottle brush as it consists of a
•Because of their strongly negative charges, these core protein to which linear GAG
hetropolysaccharide chains are extended in solution chains are covalently attached.
occupying space & repelling each other. Each chain may be composed
■They are surrounded by a shell of water molecules & when of > 200 monosaccharide chains,
brought together, they slip past each other (like same remain separated from each
poles of magnet) This 1/t slippery consistency of other b/o charge repulsion.
synovial fluid & mucous secretions. - Linkage b/w core protein & carbohydrate (GAG) chains is most
■When compressed, the water is squeezed out and GAGs commonly through a serine residue & a trihexoside (galactose -
occupy smaller volume, on release of compression the galactose - xylose) respectively. An O glycosidic bond is formed
GAGs spring back to their original hydrated volume thus b/w hydroxyl group of serine & Xylose.
Biochemistry: Structures & Functions of Proteins and Enzymes ■ 737

cushioning. ' " Xylose- A cid ic s u g a r ~j

Tend to attract Unh ^ w L Serine - Galactose- A m ino s u g a r j^


? r
polycations & OH Galactose
positive ions such as Hyaluronic '"Y I Serine Trihexoside Repeating
Na+/K+which aci =j|— proieooivcan side chain disaccharide unit
result in their ability
to attract water by osmotic pressure.
Core protein
- Proteoglycan monomers associate with hyaluronic acid molecule to form proteoglycans aggregates primarily through ionic
interactions. Association is stabilized by small link proteins.
- PGs can act as sieves restricting the passage of large molecules into ECM but allowing free diffusion of small molecules.
- Syndecan (membrane), versican & aggregan (extracellular matrix) and neurocan & cerebrocan (nervous system) are
proteoglycans.
* So proteoglycan=GAGs+Protein 025%) =[Amino & Acidic sugar],, + proteins

Glycosylation: Glycosidic Linkages of Glycoproteins


Addition of carbohydrate (sugar or oligosaccharide) side chains to specific aminoacid residues of proteins forming glycoprotein
is called glycosylation. There are two types of glycosylation (& glycosides).

N-Glycosylation and N - Linked Glycosides O- Glycosylation & O-Linked Glycosides


In N-Glycosylation sugar chain is attached to am id e (NH2) - In O-Glycosylation sugar chain is attached to hydroxy (OH)
group o f asparagineQ side chain (Glc Nac - Asn) group o f eith er serine or threon in eQ R group (Gal Nac -
N-linked oligosaccharide may be of complex or high- Ser/Thr)
mannose variety both containing same core - O - glycosylation begins with transfer of an N-acetyl
pentasaccharide. But additional sugars for complex galactosamine (from UDP-Nac) onto a seryl or threonyl R group.
oligosacchride are NANA (N-acetyl neuraminic acid), Fuc The glycosyl transferases responsible for stepwise synthesis
(L-fucose) & Glc NAc (N-acetyl glucosamine), whereas of oligosaccharide are bound to m em brane o f g olg i
high mannose oligosaccharides contain primarily com p lex 6 .
mannose (Man) only - Tyrosine, hydroxylysine and hydroxy proline aminoacids
Endo plasmic reticulum (synthesis) & golgi complex may also be involved in O-glycosylation.
(final processing) are involved ill N- glycosylationQ.

ER Golgicom plex
■Protein & dolichol (80-100 - Final trimming &
carbon ER membrane lipid) completion of oligo
linked oligosaccharide are first saccharide chains by
synthesized. addition of variety of
- Sugars added to dolichol by sugars (Nac
membrane bound glycosyl glucosamine, Nac
transferase are first N- acetyl galactosamine,
glucosamine f/b mannose & mannose, fucose,
glucose NANA) to produce
■Oligosaccharide is transferred complex glycoprotein
from dolichol to asparagine side or
of protein by protein - - They are not
oligosaccharide transferase processed, leaving
present in ER branched, mannose
Trimm ing of carbohydrate containing chains in a
chain begins in ER by removal high- mannose C om plex-glycoprotein
of mannosyl & glucosyl residues. glycoprotien.
738 ■ A Complete Review of Short Subjects

Glycoproteins Feature Glycosaminoglycans


Are proteins to which hetro oligosaccharides Composed Protein including core & link protein + hetro polysacc
are covalently attached® of haride chain®
- Short hetrooligo saccharide chain® Chain - Long hetropolysaccharide chain
- Branched, may or may not be negatively - Unbranched, N egatively charged®.
charged. - Repeating disaccharide units® [acidic sugar-amino sugar]n
- No repeating disaccharide units - Acidic sugar is D- glucronic acid or L- iduronic acid and
aminosugar is D-glucosamine or D- galacto-samine
- Cell surface recognition® Function - Chondroitin 4 & 6 sulfates, keratan, dermatan & heparan
- Cell surface antigenicity sulfate, heparin & hyaluronic acid are 6 major types
- Components of extracellular matrix & mucin - Bind large amounts of water, producing gel like ground
- Globular proteins in plasma substance supporting cellular & fibrous component of
tissues
- D/t viscosity also k/a mucopolysaccharides
- Mediate cell-cell signaling & adhesion
- N-glycosylation occurs in ER lumen & O- Synthesis - Glycosaminoglycans are synthesized in golgicomplex®
glycosylation in golgi complex® - Sulfonation of amino sugar is last step. Source of sulfate is
- In golgi complex proteins are sorted to their 3' - phophoadenosyl -5 - phosphosulfate.
proper destination such as inclusion in cell - All except hyaluronic acid form proteoglycan monomer,
membrane, lysosome or secretion from cell which consist of a core protein to which linear chains of
glycosamino glycans are attached.

Denaturation

• Disruption of native structure is termed denaturation. Physically denaturation may be viewed as randomizing the
conformation of a polypeptide chain® with out affecting its primary structure.
• Since no covalent bonds in the back bone of the polypeptide chain are broken during this, the primary structure remains
intact
• The comparatively weak forces responsible for maintaining secondary, tertiary and quaternary structure of protein are
readily disrupted with resulting loss of biological activity.
• The biological activity of most proteins is destroyed by strong acids or bases, heat, ionic detergents (amphipaths),
chaotropic agents (urea, guanidine)®, heavy metals (Ag, Pb, Hg) or organic solvents.
• Denatured proteins are generally less soluble and they precipitate® (not always).
• Denaturation results in unfolding & disorganization of secondary & tertary structure of protein without hydrolysis of
peptide bonds. Under ideal conditions it may be reversible, in which case the protein refolds into original native
structure when denaturing agent is removed. However most proteins, once denatured remain permanently
disordered®.

Collagen: Structure & Formation

- All collagens have a triple helical structure. 3 of these alpha chains are then wound into right handed superhelix. A
characteristic feature of collagen is occurrence of glycine residue at every 3rd position of triple helix of a- chin. This is
necessary because glycine is the only amino acid small enough to be accommodated in the limited space available down the
central core of triple helix.
- Repeating (Gly- X-Y)n structure is an absolute requirement for formation of triple helix while X & Y can be any other amino
acids, about 100 of X are proline & 100 of Y are hydroxyproline both of which confer rigidity
- Biosynthesis of collagen involves.
Biochemistry: Structures & Functions of Proteins and Enzymes ■ 739

I Intracellular events in fibroblast, osteoblast or chondroblast Extracellular events


1. Cleavage of signal or leader peptide to form pro a chains (from 6. Cleavage of N and C terminal extension
pre-pro a chains) peptides of procollagen molecules.
Like most secreted proteins produced for export, collagen is Extracellular enzymes called procollagen
synthesized on ribosomes in a precursor form preprocollagen, which amino proteinase & procollagen carboxy
contain a leader sequence that facilitates binding of ribosomes to proteinase remove the extension peptides at
RER & directs the polypeptide chain into the lumen of endoplasmic amino and carboxy terminal ends, respectively.
reticulum. The signal sequence is cleaved in ER to yield a precursor Cleavage may occur within crypts or folds in the
of collagen called pro a chain. cell membrane, releasing triple helical
2. Hydroxylation of prolyl & some lysyl residues: proline & lysine tropocollagen molecules which spontaneously
residues found at Y position may be post translationally modified (i.e. assemble into collagen fibers.
hydroxylated) to hydroxy proline & hydroxy lysine residues, through the 7. Self assem bly of tropocollagen molecules into
action of prolyl hydroxylase & lysyl hydroxylase enzymes. It quarterly staggered collagen fibrils
requires molecular oxygen (Oz), reducing agent vitam in C (ascorbic Tropocollagen molecules spontaneously
acid) and aketoglutarate as cofactors. associate to form collagen fibrils. They form an
So in scurvy collagen fibers cannot be cross linked greatly decreasing ordered, parallel, overlapping array, with
the tensile strength adjacent collagen molecules arranged in a
3. Glycosylation of some hydroxyl- lysyl residues by addition of quarter staggered pattern (such that each is
galactose or galactosyl- glucose through an O-glycosidic linkage (that displaced longitudinally from its neighbour by
is unique to collagen): Like hydroxylation, glycosylation also takes slightly less than a quarter of its length)
place in ER 8. O xidtive demination of e- amino groups of
4 Formation of intrachain & interchain disulfide bonds in extension lysyl & hydroxylysyl residues to aldehydes
peptides: Procollagen molecule contains polypeptide extension and formation of intra and inter chain cross
(extension peptides) of 20-35 kDa at both its amino & carboxy links
terminal. Both extension peptides have cysteine residues. While Lysyl oxidase, a copper containing enzyme that
amino terminal propeptides form only intrachain disulfide (S - S) oxidatively deaminates the e- amino group of
bonds, the carboxy terminal extension peptide forms both intra & lysine & hydrolysine residues, yielding reactive
interchain S-S bonds. aldehydes
5. Formation o triple helix: The reactive aldehydes (allysine & hydroxy
Formation of these disulfide bonds assists in formation of triple allysine) can condense with e aminogroup of
helix, winding starts from carboxy terminal end. After formation of unoxidized lysyl or hydroxylysyl residues in
triple helix, no further hydroxylation or glycosylation can take place. neighbouring collagen molecules to form
Procollagen molecules are translocated to golgi apparatus, where they collagen crosss links, & thus, mature collagen
are packed in secretory vesicles. The vesicles fuse the cell membrane & fibers
release procollagen molecules into extracellular space. Histidine may also be involved in certain cross
links.

Tropocollagen or Collagen Molecule


It is a subunit of larger collagen aggregates such as fibrils & is approximately 300 nm long and 1.5 nm in diameter, made up of
three left-han ded alp h a helix p oly p ep tid es stran ds (its name is not to be confused with the commonly occurring alpha helix, a
right-handed structure). These three left-handed helices are twisted together into a right-han ded triple helix or "super helix", a
quaternary structure stabilized by numerous hydrogen bonds. With type I collagen and possibly all fibrillar collagens if not all
collagens, each triple-helix associates into a right-handed super-super-coil that is referred to as the collagen microfibril. Collagen
fibrils are collagen molecules packed into an organized overlapping bundle. Collagen fibers are bundles of fibrils.
A distinctive feature of collagen is the regular arrangement of amino acids in each of the three chains of these collagen subunits - Gly-Pro-Y
or Gly-X-Hyp, where X and Y may be any of various other amino acid residues. So P roline o r hydroxyproline constitute about
1/6 &, G lycine accounting for the 1/3 of the sequence, this means that approximately half of the collagen sequence is not glycine
or proline.
In the fibrillar collagens, the tropocollagen molecules are staggered from each other by about 67nm (a 'D ' unit and changes
depending upon the hydration state of the aggregate). Each D-period contains 4 and a fraction collagen molecules. This is because
300 nm divided by 67 nm does not give an integer (the length of the collagen molecule divided by the stagger distance D).
Collagen fibrils / aggregates are arranged in different combinations and concentrations in various tissues to provide varying
tissue properties. In bone, entire collagen triple helices lie in a p arallel, staggered array. 40 nm g ap s betw een the ends o f the
tropocollagen subunits serve as n ucleation sites fo r the d ep osition o f long, hard, fin e cry stals o f the m ineral com ponent, w hich
is (approxim ately) h ydroxyapatite, CawfPO^e (O H hw ith som e p h osp h ate. It is in this way that certain kinds of cartilage turn
into bone. Type I collagen gives bone its tensile strength.
740 ■ A Complete Review of Short Subjects

Simple & Conjugated Protein


Proteins can be divided on the basis o f their solubility and their physical properties
Sim ple Conjugated protein
proteins Are simple proteins combined with a non- protein group called prosthelic group. Protein part is called
On complete apoprotein and entire molecule is called holoprotein.
hydrolysis yield C la ss N u cleo - M uco- L ip o G ly c o P h o sp h o C h ro m o p ro te in M e ta llo p ro tein
only amino p ro tein p ro tein p ro tein p ro tein p ro tein H em o - Flav o V isu a l
acids (m u co id s) p ro tein p ro tein p u rp le
Examples: P ro s th e tic D N A H exo- L ip id s C a rb o P h o s p h a te H e m e (F e - R ib o f l a v in C a r o t e n - Fe Z n Ca Cu Mo
- A lbum inQ g ro u p RNA s a m in e h y d ra te p o r p h y r i n ) ( f la v in ) o id
- G lobu lin 0 n u c le o tid e

- Protamines E x a m p le C h r o m a tin M u c in Bi l i p o ­ I g C C a s e in Hb S u c c in a te

d e h y d ro g e n a s e ,
- Histones p ro te in O v o v ite llin M b d e h y d ro ­

c e r u lo p la s m in
P la s t o c y a n i n ,

D in itro g e n a s e
C y tc , P450

C a lm o d u lin
g en ase

c a r b o n ic
A lc o h o l
F e rritin
Transport or Binding Plasma Proteins
Plasma Proteins Function (Bind & Transport) Plasma Proteins Function (Bind & Transport)
Transthyretin Thyroxine (T4) and retinol (forms a Haptoglobin Extracorpuscular (free) hem oglobin
(Prealbumin) complex with retinol binding protein) (phenotype 1 - 1 ) ; peroxidase activity
Album in Osmotic protein binds & transports (phenotype 2 - 1 )
various ligands like ions (Ca++), metals Sex hormone binding Testosterone, estradiol
(Cu2+, Zn2+), b iliru b in , free fatty globulin
acids, metheme, steroids, other Hemopexin (HPX) = Heme
hormones & drugs Beta-B-Globulin
Retinol binding Binds & transports retinol Transferrin Iron (Fe)
protein (Siderophillin)
Ceruloplasmin Contains & transports Cu++ (albumin is Trans cobalamine I, II, Vitam in B12
probably more important in III (cfep mobility)
physiological transport of Cu++); has <Xi (HDL) Transport lipid, fat soluble vitamins,
ferroxidase activity. hormones and remove cholesterol
Corticosteroid Cortisol Lipoproteins from tissue to liver (scavanger
binding globulin action)
(Transcortin) P (LDL) Transport same but carries
Thyroid binding Thyroxine (T4, T3) cholesterol to tissues.
globulin (TBG)
★ Hemoglobin is a tetrameric protein of RBC that transport O 2 to tissues and return C O 2 & protons to lungs.
_______________ Storage Proteins_______________ Heat Test for Protein______
These are biological reserves of metal ions and aminoacids Take 2/3 of test tube full of urine sample. Hold the test tube
such as found in plant seeds, egg whites and milk. Examples from the bottom and heat the top portion of tube. Add a drop of
includes 1% acetic acid. Do not mix,_________________
1. Plant albumins: legumelin in legumes and leucosin in
Normally If Urinary pH is
cereals.
(when Urine Alkaline
2. Animal albumins: ovalbumin in egg and lactalbumin &
PH is Acidic)
casein in milk
White - Proteins may not be precipitated owing to
3. Globulins: ovoglobulin in egg and lactoglobulin in milk
turbidity or formation of alkaline meta proteins, which
4. Gliadins (prolamines) very rich in proline but poor in
precipitates in are not precipitated on heating
lysine: gliadin of wheat and hordein of barley
the heated - P h osp h ates m ay h ow ev er be p recip itate and
5. Glutelins (rich in glutamic acid): glutelin in wheat and
portion g iv e a fa ls e p o s itiv e testQ.
oryzenin of rice
indicating the -O n acidification with 1 % acetic acid, drop by
6 . Ferritin and hemosiderin stores iron
presence of drop, phosphate will disappear and
7. M yoglobin , a monomeric protein of red muscles stores
albumin. precipitate of albumin will appear on
oxygen as a reserve against oxygen deprivation (Harper
acidification.
29/48)* ____________________
Biochemistry: Structures & Functions of Proteins and Enzymes ■ 741

Test Methods for Estimation of Substances

Total Seram Protein Estimation Blood Glucose Concentration Tests for Lipids
• Aztoor-king, Folin Wu & Nelson- • Brown paper (grease spot) test
•Calorimetric Assay (common methods)
Somogyi chemical methods. •Sudan red test
1. Biuret® protein assay •Glucokinase, Hexokinase & Glucose
2. Low ry's (F olin -C iocalteu )® protein dehydrogenase enzymatic methods. Uric Acid
assay • Phosphotungstic acid (PTA)
3. Sm ith (B icinchoinic acidd)® protein Tests of Carbohydrate chemical method
assay • M olisch's test • Uricase enzymatic method
4. B rad ford ’s (C oom assie G-250 dye) • Barfoed's, Benedict's and Fehling's
method® tests for detection of reducing sugar Blood Urea
* Nephelometric, Turbidimetric, Direct (ie with aldehyde & ketone group). • Diacetyl monoxime (DAM)
photometric, UV visible spectroscopy, Seliw anoff's test differentiate chemical method
Refractometric methods between aldose & ketone sugars. • Urease & glutamate
>Kjeldahl method, Amido black protein • Bial's Resorcinol test detect presence dehydrogenase enzymatic
assay, O-phthaldehyde & Nano orange of pentose (5C sugars) methods
methods.

Clinical Laboratory Based Analytic Systems for Measuring Seram Creatinine to Assess GFR

I I I
Alkaline picrate methods Enzymatic criatinine method High performance GC- isotope
I liquid dilution mass
• Jaffe method (1886) overestimates serum • It uses these enzymes and this reaction chromatography spectroscopy
creatinine by as much as 15-25% d/t „ . . . Crealininuxe _
C r e a tin in e ----------- — --------- > C
Cre a tin e
reatin
(HPLC) I
presence of interfering substance, .I C
Crca
reatinase
I Method of
particularly proteins (albumin), acorbate, Sarcosins oxidase Fairly sensitive & choice
* J .
pyruvate, glucose, bilirubin, fetal hemoglobin F o rm a ld e h y d e * Sa’rc o sin e specific method and
& drugs + H2O2 + g ly cin e include cation -
• Kinetic Jaffe assay have improved • The H2O2 generated can be measured by exchange / normal-
specificity & minimized suceptibility to spectrophotometrically (Roche creatinine phase / reversed-
interfering substances. plus assay), using leuco dye (vitros dry phase and reversed
• Technicon continuous flow autoanalyzer chemistry system), or with blood gas phase ion-pair
(Skeggs) provided the first opportunity to analyzer electrodes (Radiometer dual chromatography.
efficiently process increasing workloads & electrode system) Sample
incorporated on line dialysis to remove • Bilirubin and creatinine interference is deprotinization
protein & protein bound interfering reduced by using more efficient H 2O 2 improves
substance such as bilirubin, but smaller acceptor (triiodo- hydroxy benzoic acid), specificity.
molecules such as glucose, pyruvate, potassium ferocyanide& detergents.
acetoacetate & cephalosporins were still Ascorbate & dopamine are other
able to cross the dialysis membrane & interfering substances.
cause falsely increased results.

Chromatography
This is a method of separation of molecules that involves passing (diffusing) a solution (the m obile phase) through a
medium (immobile or stationary phase) that shows selective absorption for different solute components based on their
size, charge, hydrophobicity & affinity etc. So chromatography separates or purifies proteins on the basis of charge, size,
hydrophobicity or affinity.
The stationary /immobile phase or matrix may be a sheet of filter paper (paper chromatography) or a thin layer of cellulose,
silica or alumina (thin layer chromatography or TLC) or a column (cylinder container of glass, plastic or metal) containing
small spherical beads of modified cellulose, acrylamide or silica whose surface has been coated with functional groups)-
Column chromatography
Either ascending or descending chromatography can be done based on the mobile phase being applied from the bottom or at
the top respectively. Depending on methods of separation types of chromatography include-
742 ■ A Complete Review of Short Subjects

Size: Gel Filtration (Size Exclusion) Chromatography Biological Activity: Affinity Chromatography
- Separates protein on the basis of m olecu lar size® (stokes - Affinity chromatography separates proteins by their
radius). binding sp ecificities. The p rotein s retain ed on the
- A number of gels (sephadex) when hydrated acts as molecular colum n are th ose th at bind sp ecifica lly to a ligand®
sieves. The pores on the surface of these gel allow smaller (substance) that has an inherent ability to bind with the
molecules to penetrate deeper whereas larger molecules are target molecule to be purified.
left outside, hence also called exclusion chrom atography. - It exploits the high selectivity of most proteins for their
- The procedure is extrem ely gentle, permitting seperation of ligands. So enzymes can be purified by using coenymes
labile substances e.g. protein, nucleic acid like DN A or inhibitiors as stationary phase. And so antigen-
fragments®. antibody, hormone-receptor, enzyme-substrate, inhibitor-
- The procedure is highly reproducible with minimal losses. cofactor can be sparated.
- Used to determine molecular weight of purified protein. - After proteins that do not bind to ligand are washed
Hydrophobicity: H ydrophobic Interaction Chromatography through Bound proteins are eluted (washed out of
- Seperates proteins based on their tendency to associate with a column) either by competition with soluble free ligand
stationary phase matrix coated with hydrophobic groups (eg or by disrupting protein ligand interactions using urea,
phenyl Sepharose, octyl Sepharose) guanidine hydrochloride, mildly acidic pH or high salt
- Proteins with exposed hydrophobic surfaces adhere to the matrix concentration.
via hydrophobic interactions, that are enhanced by a mobile - Stationary phase matrices available comercialy contain
phase of high ionic strength ligands such as NADP+ or ATP analogs and Ni*2 (that
- Nonadherent proteins are first washed away. The polarity of bind protein with an attached polyhistidine tag & a
mobile phase is then decreased to weaken hydrophobic interactions glutathione matrix that binds a recombinant protein
by gradually lowering salt concentration f/b adding glycerol or linked to glutathione - S- transferase
ethanol (if interactions are very strong). Charge: Ion- Exchange Chromatography
Solubility: Partition Chromatography - Separates proteins depending on the net electric charge
- In this the components of mixture to be separated are o f protein®.
partitioned b/w the two phases (eg solid - liquid, liquid- - Chromatography on carboxym ethyl cellu lose or diethyl
liquid, gas-liquid etc) depending on the solubility (partition am ino m ethyl cellu lose column® separates protein based
co-efficient) of the particular substances. Stationary phase on electrostatic attraction b/w charged molecules &
may be either solid or liquid over which a liquid or gaseous oppositely charged groups on ion exchange resins.
mobile phase moves Size, affinity & charge; High Performance /Pressure
- Purifies proteins based on their relative affinity of different Liquid Chromatograph (HPLC)
proteins for a given stationary or a mobile phase, which in - The stationary phase used in classic column
turn depends on composition of both stationary & mobile chromatography are spongy material whose
phase. compressibility limits flow of the mobile phase
- Proteins that interact more strongly with stationary phase are - HPLC makes use of high pressure pumps that speed
retained longer and provide a basis of partition (purification) the movement of mobile phase (protein molecules)
- Gas liquid chromatography (GLC), TLC or ascending / down the column, as well as higher quality
descending paper chromatography is performed and some chromatographic incompressible (sillica or alumina
location reagents (eg ninhydrin for aminoacids & peptides; microbeads as) stationary phase that can withstand the
sulphuric acid for phospholipids; diphenylamine for sugars) is crushing force of pressurized flow
sprayed on paper to identify RF (relative flow) value. RF - Reduced transit time limits the diffusional spreading
(ratio of fronts) value is the ratio of distance traveled by the of protein bands & thus greatly improves resolution.
substance (solute) to the distance traveled by the solvent in paper - Used for high resolution separation of proteins peptides
chromatography. The RF is constant for a particular solvent at and aminoacids based on their charge, size &
a given temperature and the substance or aminoacid can be hydrophobicity. The eluates are monitored by ultraviolet
identified by their RF value. absorption, refractive index or fluorescence. So it can give
Adsorption coefficients of molecules d /t van der waals high resolution separation of mixtures of lipids or
interaction with stationary phase: Absorption Chromatography peptides whose property differ only slightly.
- Protein mixture is applied to a column under conditions - Reversed phase HPCL uses a hydrophobic stationary
where the protein of interest associates with the stationary phase o f aliphatic polymers 3-18 carbon atoms in length.
phase so tightly that its partition coefficient is essentially And peptide mixtures are eluted (separated) using a
unity gradient of water miscible organic solvent such as
- Non adhering first washed & then stabilizing forces are acetonitrile or methanol.
gradually disrupted by gradient of increasing salt
concentration.
Biochemistry: Structures & Functions of Proteins and Enzymes ■ 743

Ion Exchange Chromatogaphy (IEC)

It separates (purifies) protein on the basis of differences in the sign & magnitude of net electric charge at a given pH.
In IEC, the stationary phase (column matrix) is a synthetic polymer (resin) containing bound (attached) charged groups -
negatively charged (carboxylic or sulfonic group) in
cation (+ ion) exchanger; and positively charged (tertiary (_ — N e g a tiv e c h a rg e d
or quaternary amines) in anion (- ion) exchanger. The \ / 'ons 9ets attached
+) ' to s ta tio n a ry p h a s e
protein which has to be separated (purified) is percolated an d m o v e s lo w
through the stationary phase in liquid form and it forms
mobile phase of IEC. The proteins of mobilephase interact
with resin of stationary phase by charge - charge
interaction (electro static attraction). Since the net charge
on a protein is determined by the pH, this separation is also
based on ionic character of proteins & aminoacids (isoelectric
point).
9
P o s itiv e c h a rg e d io n s are
re p e le d , s o m o v e s fa te r

Proteins, which are polyvalent compete against monovalent


ions for binding to the stationary phase - thus the term ion
exchange
A n io n - E x c h a n g e C h r o m a to g r a p h y
The affinity of each protein for charged group on column
(stationary phase) is affected by the pH (which determines the ionization state & net charge of molecule) and concentration
of competing free salt ions. Separation can be optimized by gradually changing the pH and or salt concentration of the
mobile phase so as to create a pH or salt gradient.

On the basis of charge of stationary phase, It is of 2 types

1
Cation (Positive ion) Exchanger Anion (Negative ion) Exchanger
When a more positively charged ion (or molecule) is When a more negatively charged ion (or molecule) is
exchanged with another positively charged component exchanged with another negatively charged component
bound to negatively charged stationary phase, the bound to positively charged stationary phase the process is
process is called cation exchanger. Cation exchange called anion exchanger. Anion exchange resins have
resins have negatively charged sulphonic or carboxylic positively charged quaternary nitrogen (N+)

Stationary Charge Negative Stationary Charge Positive


phase - Carboxymethyl phase Example - D iethyl aminoethyl (DEAE)
Example
(solid (solid cellulose (- O- C 2H 4 - NH+ [C2H5]2) =
cellulose (0-C H 2- C 0 0 )
matrix) matrix) Strong anion
- Carboxylate (carboxylic)
Resin Resin - Tertiary or quaternary amines
group.
- Sulfate (sulphonic) (nitrogen)
group - Dowex - 3 (weak anion)
Slowest m oving Positively charged aa. - Amberlite IRC - 50 (weak cation) ??
amino acids (in because they interact with (Vasudevan)
mobile phase) stationary phase Slowest m oving Negative charged
amino acids
Fastest m oving amino N egatively charged
acids (in mobile Fastest m oving Positively charged
amino acids
phase)

★ At pH 7.5, albumin is negatively charged (pi = 4.8) and immunoglobin is positively charged (pi = ~ 8 ). So albumin (anion)
w ill move slow ly in an anion exchanger and immunoglobin (cation) w ill move slow ly in a cation exchanger.
744 ■ A Complete Review of Short Subjects

Electrophoresis

I I
Electrophoresis is separation of proteins Isoelectric Focusing Two dimensional
based on m igration o f charged p rotein s in an Electrophoresis
electric field® It determines isoelectric point (pi) of It is combining isoelectric
It is n ot g en eally used to purify p rotein s in a protein and separates protein focusing and SDS
large am ounts becau se it often adversely according to their pi. electrophoresis sequently
affects the structure & function o f proteins® It is based on the fact that each It is more sensitive analytical
and simpler alternatives are available protein has a different pH at which it method than either
It is an analytical method that quickly estimates is electrically neutral (pi) electrophoretic method alone
number of different proteins in a mixture or the A pH gradient is generated within It permits fractionation of
degree of purity of a particular prtein polyacrylamide matrix by allowing complex mixtures of proteins
preparation. Its advantage is that protein can ampholytes or ionic buffers (i.e. low It separates proteins of
be visualized as well as separated. molecular weight organic acids & identical molecular weight
Polyacrylamide gel acts as a molecular sieve bases) to distribute themselves in an that differ in pi, or protein
slowing the migration of proteine ~ in electric field generated across the gel. with similar pi values but
proportion to charge - to - mass ratio. Applied protein migrate until it different molecular weight
Migration is also affected by size & shapeQ. reaches the region where pH
The force moving the macromolecule is matches its isoelectric point (pi), the
electrical potential (E) pH at which the molecules net
charge is zero.

E le ctro p h o re tic m o b ility (g) = = ______ N e t c h a r g e ( z )


E F r ic tio n a l C o ff ie ic n t ( f )

Discrimination / Seperation of Nucleic Acid (DNA & RNA)


Seperation by size through a gel matrix in presence of an electric field (i.e. gel electrophoresis) is the m o st com m on technique
to discrim in ate different nucleic acids®
Both DNA & RNA are negatively charged and will migrate towards a positively charged electrode.
Seperation is primarily based on molecular weight with smaller molecules traveling faster than larger ones. Physical
conformation (or shape) also affect & form the basis of separation. Because single stranded RNAgenerally hs a high degree
of secondary structure. Electrophoresis of RNA is usually performed under denaturating conditions to abolish these
structures. Electrophoresis of DNA is performed under non denaturing or denaturing conditions (to abolish hetroduplex,
nicked or superhelical circular structures) depending on amplification. Under most gel conditions.

F a s t e s t m o v in g fo r m ® I Migrates at rate (form a band)


S lo w e s t m ig r a tin g b a n d ® intermediate b/w uncut supercoiled &
nicked relaxed forms

When very large molecules (> 50kb) have to be separated, pulsed electrical fields are employed to help move these
molecules through the polymer matrix (Pulse- field gel electrophoresis, PFGE). Conventional gel electrophoresis use
current applied in single direction. Very large DNA fragments are thought to orient themselves with the voltage field. Once
oriented, molecules of different lengths migrate at slow but similar rates. The trick to separating these large fragments is to
make them change direction at regular intervals; longer DNAs take longer to orient correctly and so lag behnd shorter
fragments. So in PFGE, an alternating current is applied to the gel, changing the direction of electrophoresis at intervals.
That's why a normal agarase gel electrophoresis may take 1 - 4 hr, whereas PFEG often require 24 hrs to complete.
Polyacrylamide gel is suited for high resolution separation (upto 0.1% size differences) of short molecules (upto 2kb), whereas
agarose gel can separate nucleic acid fragments from 20 bp to 10 mB (10,000 kb) but with limited resolution (usually to a size
difference 2-5%)
Biochemistry: Structures & Functions of Proteins and Enzymes ■ 745

SDS - PAGE

A denaturing anionic detergent, sodium dodecyl sulfate (SDS) [dodecyl denoting 12 carbon chain = CH 3 (CH2)n - SO 4 - Na+] is
used in poly acrylamide gel electrophoresis (PAGE) system to separate, resolve & purify protein subunits according to
molecular weight.
Acrylamide is polymerized & cross linked to form porous matrix (stationary phase) which acts as a molecular sieve.
SDS denatures protein, & disrupts the hydrophobic interactions of protein so that the protein is partially unfolded and assumes a
similar (rod like) shape.
SDS binds to protein at a ratio of 1 m olecule of SD S per 2 peptide bonds (or amino acid residues) that is roughly
proportional to molecular weight of protein (or length of peptide chain).
The large number of bound anionic SDS contribute a large net negative charge, rendering the intrinsic charge of protein
insignificant and conferring on each protein a similar charge - to mass ratio.
Because SDS treated proteins have similar shape and charge to mass ratio; SDS - PAGE sep arates protein s a lm o st
exclu sively on the b asis o f m ass (m olecular weight)®, with smaller polypeptides migrating more rapidly.
A purified protein preparation can be readily
(NH4)2
analyzed for homogeneity (purity) on SDS-PAGE by Marker Crude Axon Cation Purified
extract so4 excha excha protein
staining with sensitive & specific dyes, such as
preciptat nger nger
Coomassie blue, or with a silver staining technique, 111 ---- 1
which binds to protein but not to gel itself.
Thus the researcher can moniter the progress of 73 HD —
protein purification as the number of bands decreases
after each new fractionation step. When compared with
48
positions of protein with known molecular weight, the
position of unidentified protein can provide a good 3
o
3 □ i— 1 |= > Moleci lar
Wt~45
approximation of its molecular weight. 0 34 = 1
SDS- PAGE separates the component polypeptides
o f multimeric proteins when used with 2-
29 = 1 =
merceptoethanol or dithiothreitol to reduce and
break disulfide bonds. A separate band appears for
15 ----- 1 =
each component.
So in above case 100 kD protein has 2 bands of 20 kD S D S - PAGE
& 30 kD.
Hence Protein has 2 components of 20 kD & 30 kD (i.e. 50 kD). Since to be of 100 kD there must be 2 units of 50 kd (50x 2 =
100). So the 100 kD protein is a dimer of 20 kD & 30 kD component.

Detection of Protein Structure

Primary Structure detection H igher three dimensional


I (Secondary, tertiary and
Sanger's Reagent Method quaternary) structure detection
Fragmenting polypeptide chain by endoproteases & chemical reagents.
Automated protein sequencer (sequenator) using Edm an's degradation - X ray cry stallog rap h y (or
technique® and D N A sequencing. diffraction)®
Mass spectrometry to d etect m olecu lar m ass, sequence o f p oly p ep tid es & - Polychromatic X-ray
co v alen t m o d ification s sim ultaneously® (more commonly used method) crystallography (Laue
Protein sequencing, identification and molecular weight determination of intact diffraction)
protein can also be done by matrix-assisted laser desorption ionization - time - NMR (nuclear m agnetic
of flight (MALDI - TOF) mass spectrometry and Electrospray ionization mass resonance) spectroscopy®
spectrometry (ESI-MS) - In fra-red spectroscopy®
Once partial aminoacid sequence is obtained, one can determine nucleotide - Ultraviolet (UV) light
sequence of DNA that encodes this polypeptide segment. Chemically spectroscopy
synthesized oligonucleotide primers can be used to identify amplify (by PCR) - Optical rotatory dispersion
and isolate gene containing appropriate sequence. (Indirect hybrid technique) - Circular dichromism
746 ■ A Complete Review of Short Subjects

Mass Spectrometry
- It determines the m ass o f m olecu les® (< 1000 Da by
conventional spectrometers and larger mass by use of flight
spectrometers)
- Initial difficulty of volatilizing large organic molecules has
overcome by MALDI (m atrix assisted laser desorption and
ion ization ) and electrospray dispersion (eg. Nanospray) which
permit the masses of even large polypeptides (>100000 Da) to
be determined with extraordinary accuracy (± lD a)
- Peptides inside the mass spectrometer are broken down in
smaller units by collisions with neutral helium atoms (collision spectrometer. A mixture of molecules is vaporized in an
ionized state in the sample chamber S. These mole­
induced dissociation) and the masses of the individual fragments cules are then accelerated down the flight tube by an
are determined. electrical potential applied to accelerator grid A. An ad­
justable electromagnet, E, applies a magnetic field that
- Since peptide bonds are much more labile than the carbon- deflects the flight of the individual ions until they strike
carbon bonds, the most abundant fragments will differ from the detector, D. The greater the mass of the ion, the
higher the magnetic field required to focus it onto the
one another by units equivalent to one or two amino acids. detector.
- Since with the exception of leucine and isoleucine- the molecular
Mass increase resulting from common posttranslational
mass of each aminoacid is unique, the sequence of the peptide
modifications
can be reconstructed from the masses of its fragments.
- It detects covalent modification i.e. is ideal for detecting the Modification Mass Increase
phosphate, hydroxyl and other groups on post translationally (Da)
modified aminoacids. As each adds a specific and readily Phosphorylation 80
identified increment of mass to the modified amino acid Hydroxylation 16
- Tandem mass spectrometry can determine mass by analyzing Methylation 14
complex peptide mixture with out prior purification. It employs the Acetylation 42
equivalent of 2 mass spectrometers linked in series. It can detect Myristylation 210
metabolic abnormalities such as PKU, ethyl malonic Palmitoylation 238
encephalopathy, & glutaric academia type I. Glycosylation 162
* Protein purity is assessed by p oly acry lam id e g el electrop h oresis (PAGE)®. The most widely used method is SDS- PAGE i.e.
polyacrylamide gel electrophoresis in the presence of the anionic detergent sodium dodecyl sulfate (SDS).
* Sanger's reagent (1 fluoro,2,4- dinitrobenzene) was used to determine the polypeptide sequence of insulin. It als react
with e- amino group of lysine.
* Edman's reagent (phenyl- iso thiocynate) selectively label the amino terminal residue of a peptide. It enables peptides and
proteins to be sequenced.

Enzyme Inhibition

Enzyme/Drug Competitive Inhibitor Substrate


Lactate Dehydrogenase Oxamate Lactate
Aconitase Transaconitate Cis-aconitate
Succinate Dehydrogenase Malonate Succinate
HMG-CoA Reductase HMG HMG-CoA
Dihydrofolate Reductase Amethopterin 7,8 Dihydrofolate
Allopurinol Xanthine oxidase, which produce uric acid from hypoxanthine Hypoxanthine
Sulfonamide Synthesis of folic acid (impermeable to bacterial wall) from PABA PABA
Methotrexate Folate reductase & prevent formation of FH 4 Folic acid
MAO Inhibitor eg Ephedrine & MAO (mono amine oxidase) Cate -cholamines
amphetamine
Physos-tigmine Acetyl cholines terase (destroy ACH into choline &acetate) Ach
Dicoumarol Act as anticoagulant by inhibiting Vit K Vit K
Succinyl Choline Competitively fixes on post synapti c receptor. As it is not Ach
hydrolysed easily by the enzyme acetyl cholinesterase, it
produces continued depolarization & muscle relaxation
Biochemistry: Structures & Functions of Proteins and Enzymes ■ 74 7

Regulation of Enzyme Activity

1. Allosteric Inhibition

• Allosteric enzymes are regulated by molecules k/a effector or m odifiers that bind noncovalently at a site other than the
active site (k/a allosteric site) on a different region of enzyme molecule. Effector can alter the affinity &/or the maximal catalytic
activity of enzyme.
• Positive effectors increase and negative effectors inhibit enzyme activity. When the substrate itself serves as an effector it is
called homotropic effector but when both are different it is called heterotropic effector.
•Does not follow Michaelis Menten hyperbolic kinetics. Instead it gives a sig m oid curve & sh o w co op era tiv e binding.®
•Allosteric in hibitors shift the substrate saturation curve to the right & a ctiv ators shift it to left.
• Allosteric enzymes may be of K & M series according to their kinetics.
- In K- enzymes eg aspartate carbamoylase & phospho fructokinase, allosteric Enzyme A llosteric A llosteric
inhibitor lowers substrate affinity to raise km, but Vmax is unchanged ac tiv a to r in h ibitor
- In M -enzymes eg acetyl CoA carboxylase, the allosteric inhibitor reduces G lu tam ate ADP ATP, NADH
the maximum velocity, but no change in km o r su bstrate affin ity. D ehydrogenase
Allosteric activators produce a fall in K enzymes and a rise in Vmax
H exo kin a se ICD ADP G - 6 -P, ATP
•When the final product allosterically inhibits the enzyme it is called feed
Protein K inase c-AM P -
back allosteric inhibition
- A sparte tran scarbam oylase (AT case) which catalyzes first reaction of Pyruvate A cetyl CoA ADP
ca rb ox y lase
pyrimidine synthesis is allosterically inhibited by cytidine triposphate (CTP)
- Threonine deaminase first enz. of isoleucine synthesis is inhibited Phospho fructo C itrate
allosterically by isoleucine kinase

•A metabolite may cause feed forward allosteric activation eg FI. 6 Bi PO 4activates pyruvate kinase
•Allosteric effector oppositely influences two allosteric enzymes catalyzing reverse reations eg. AMP activates phospho fructo
kinase & inhibits F.D. pase

2. By covalent modification eg phosphorylation & dephosphorylation o f enzyme.

Michaelis - Menten model & equation Catalytic Constant (Kcat) & Efficiency
(Kcat/Km) ______________

According to M-M- model, enzym e (E) reversibly com bines w ith its Comparision of relative catalytic activity of
su bstrate (S) to fo rm enzym e - su bstrate (ES) com plex th a t subsequently different preparations of same enzyme or
y ield s product (P), regenerating the free enzyme® different enzymes can be done in
Impure enzyme preparations by
E + S- -> ES
Specific activity=
Protein concentration
M- M- equation describes how reaction v elo city varies w ith su bstrate
concentration®.
- Homogenous enzyme by
M ax im u m v elo c ity
In itia l V elocity (Vq) =
M ic h a e lis c o n s t a n t ( O + M V max
Turnover number=
Moles o f enzyme
Where, [S] is substrate concentration & K m = (K - 1+ K 2 )/ K , ■And it is
assumed that concentration of substrate is much greater than enzyme,
If number of active sites is known
concentration of [ES] does not change with time and [p] is very small so
rate of back reaction can be ignored. Conclusions extracted are
C a t a ly t ic c o n s ta n t ( K c a l) =
Number o f active sites (S ,
748 ■ A Complete Review of Short Subjects

It is best measure and unit is 1/time


Michaelis constant (Km) - Rate of reaction velocity is directly
• Catalytic efficiency is best expressed in terms
- It is characteristic of enzyme & p ro p ortio n a l to enzym e concentration
of ratio of Kcat/km
its particular substrate and a t a ll su bstrate concentrations®. Eg.
reflects the affin ity o f enzym e when [E] is halved, Vo & Vmax are
Inhibitory Potency
to th at substrate® reduced to half of original
- It is equal to substrate - Order of reaction
It is the concentration of inhibitor that produces
concentration at which
50% inhibition ( I C 5 0 )
reaction velocity is equ al to When [S]« Km W h en [S ]» Km
1/2 Vmax®. It does n ot change - Velocity of reaction - Velocity is
w ith enzym e concentration®. is approximately constant and equal
I I proportional to to Vmax (i.e., V =
Sm all (low) Km Large (High) substrate Vmax)
Reflects high Km concentration i.e., - Zero order
affinity of Reflects low V a [5 ] reaction,
enzyme for affinity of E - I s1 order reaction independent of
substrate, as for S, because with respect to substrate
low a high substrate. concentration
concentration of concentration ★ Line weaver - Burk (double -
S is needed to of substrate is reciprocal) plot is a straight line
half saturate E, needed to half obtained by plotting 1/Vo versus 1 / isj-
i.e., to reach 1/2 saturate It can be used to calculate Km & Vmax,
Vmax. enzyme as well as to determine the mechanism
of action of enzyme inhibitors.

Calcium - Calmodulin

Calmodulin is calcium dependent (& binding) regulatory protein that is homologus to muscle protein troponin-C in structure &
function. It can bind to 4 calcium ions. Binding of Ca++ with calmodulin activates it (in a similar way of cAMP activates PKA).
Enzymes & process activated by Ca++ & mediated by calmodulin are

- A denylyl Ca++ dependent protein kinases (CaM Phosphorylase kinase - RNA helicase
cyclase® (brain) kinase I-IV) phosphoprotein - Myosin light
- C-AMP Ca++ - Mg++ ATPase phosphatase 2B chain kinase
phosphodiesterase Ca++ pump (plasma membrane calcium Phosphoinositide 3 - NAD+ kinase
- C-AMP gated olfactory channel ATP ase) kinase - Nitric oxide
- Cyclic neucleotide Ca++ phospholipid dependent protein PhospholipaseC synthase
kinase Phospholipase A2 - NMDA type
phosphodiesterase
Ca++ release channels of sarcoplasmic Pyruvate carboxylase glutamate
G uanylate cyclase®
reticulum receptor
Pyruvate
C-GMP gated Na+ & Ca++
Ca++ dependent Na+ channel dehydrogenase
channels of rods & cones
(paramecium) Pyruvate kinase
G lycogen synthase® Calcineurin (phosphoprotein phosphatase
Glycerol-3 phosphate 2B)
dehydrogenase L-type calcium channels
Glutamate decarboxylase Cytoskeletal protein
Biochemistry: Structures & Functions of Proteins and Enzymes ■ 749

Plasma Enzymes can be of Two Types


Features Functional Enzymes Non Functional Enzymes
Function Has physiological role No known physiological role
Normal concentration in High Low
plasma in comparision to
tissue
Substrate in blood Always present Absent
Site of synthesis Mostly in liver In different tissues eg liver, heart, muscle
Cause of origin Physiological need Pathological process eg
- Cell damage & necrosis in viral hepatitis or MI
- Obstruction of normal path (bile duct = alkaline P O 4 )
- Meastasis
Clinical importance No Diagnosis & prognosis of disease
Example L ipoprotein lipase® etc A cid & a lk a lin e p h o s p h a ta s e , y glu tam y l tm speptidase® etc.

Isozymes / Isoenzymes

Isozymes are the physically distinct forms of the same enzyme


that catalyze the same reaction, and d iffer fro m each oth er • Lactate dehydrogenase is a tetrameric enzyme ct consists or
structurally, electrop h oretically an d im m unologic ally®. four subunits
Isozymes arise through gene duplication in higher organisms. . These subunits can occur in two isoform i.e.
Isoenzymes may differ in their______________________________ - H isoform (for heart)
1. Quartem ary structure as many isoenzyme multimeric - M isoform (for muscle)
complexes contain differen t subunits in differen t All these isozymes though different physically they catalyze
(various) com binations® such as CPK1 has BB, CPK 2 the same reaction of oxidation of LA to PA
has MB and CPK 3 has MM compositon. In normal serum, LDH2 (H3M) is the most prominent isoenzyme.
2. Physical properties such as heat resistance because of
genetically determined differen ce in am in oacid LDH Isoenzymes
sequence® Type Polypeptide Electrophorectic Tissue rich in
3. Electrophoretic m obility (allowing them to be separated chains m obility isoenzyme
type
from each other by electrophoresis) because of different
electrical charges on the isoenzymes due to difference LDH-1 (H4) H H H Fastest moving found in Heat
H d/t highest myocardium® resistance
in contents of charged acidic & basic aminoacids)
negative charge
4. Properties such as sensitivity to particular regulatory
b/o higher Asp
factors or substrate affinity (eg glucokinase) that adapt & glutamate
them to specific tissues or circumstances. content
5. Tissue distribution: Different organs frequently LDH-2 (H3M)
contain characteristic proportions of different HHHM
isoenzymes. So pattern of isozyme in plasma may serve
LDH-3 (H2M2)
as means of identifying the site of tissue damage.
HHMM
6 . Although isoenzymes catalyze same reactions they differ
LDH-4 (HM3) Heat labile
in rate of catalysis, pH optima, Kmax & Vmax and even
HMMM
enzyme classification.
LDH-5 (M4) slowest moving found in liver
★ Each enzyme has a enzyme code number (EC No) of 4 digits MMMM (Hepatic) ®
separated by points; first being the main class, second
indicating type of group involved in reaction (sub class),
Isoenzyme of CPK > .- *
third denotes substrate on which group acts (sub sub class)
and fourth digit is serial number of a particular enzyme.
• Creatinine phosphokinase exists as three isoenzymes in
human tissue:
Serum Non functional Diagnostic Use
1. C P K -1 or CPK - BB : Found in Brain
enzyme
2. CPK- 2 or CPK - MB : Found in Myocardium and skeletal
Amylase,Lipase Acute Pancreatitis
muscle.
Lactate dehydrogenase Liver diseases 3. CPK- 3 or CPK - MM: Found in skeletal muscle
isoenzym e 5
750 ■ A Complete Review of Short Subjects

L actate d e h y d r o g e n a se M y o c a r d ia l in farction • 1 is essen tially fou n d in


T h u s creatin ine p h o sp h o k in a se
(isozym e) brain & w ou ld be in creased in brain ischaem ia®
A m in o tran sferase
- A sp a r ta e am in otran sferase - M y o c a r d ia l in farction C r e a tin e p h o s p h o k in a s e (C P K ) -2 -C P K M B

(A S T , or S G O T )
-A la n in e am in otran sferase - V itra l H ep atitis
(A L T or S G P T ) Normally Myocardial infarction
C e r u lo p la sm in H ep ato len ticu la r • CPK -2 (MB isozyme) occurs • Elevation of CK-MB
d e ge n era tio n (W ilso n 's in v e r y sm all q u a n tity is o e n z y m e occu rs w ith in 4

disease) a c co u n tin g for as less as 2 % hours, m a x im u m in 24 hrs,

L iv e r d isea se
of total CK activity o f plasma. then falls rap id ly .
7 -G lu ta m y l tra n sp ep tid a se

C r e a tin in e k in a s e (M B ) M u s c le disorder, M I
• C K M B also exists in 2 form: • A ratio o f C K M B j: CKMB2
- C K M B i : E xtracardiac above 1.5 is highly sensitive
A c id p h o sp h a ta se M etastatic C a o f p ro state
form for the diagnosis of acute MI
A lk a lin e p h o sp h a ta se Bon e disorders, o b stru ctiv e
C K M B 2 : C a r d ia c form after 4-6 hours of onset of
live r d isea se
myocardial ischaemia.

Oxido- Reductase Enzymes

O x id a tio n in v o lv e s removal of electron /hydrogen or hydride ion from a (reduced) substrate or incorporation of O2in to a (reduced)
substrate. A fte r w h ic h the r ed u ce d su bstrate b eco m es o x id ize d . Oxygen, flavin nucleotides (FMN, FAD) and nicotinamide coenzymes
(NAD* or NADP*) are u s e d as h y d r o g e n acceptor. H 2 O 2 ca n b e u s e d as e- accep to r & d o n o r bo th . T h u s oxidation is always
accompanied by reduction o f an electron (h y d ro gen ) acceptor.

1---------------------------- 1
Oxidase Dehydrogenase Hydroperoxidase
C a t a ly z e rem o va l o f D e h y d r o g e n a e s e ca n n o t u s e o x y g e n - T h e se u s e H 2 O 2 or o rga n ic p e ro x id e as su b stra te &
h y d r o g e n from as h y d r o g e n acceptor. So these p ro tect b o d y from h a rm fu l p ero xid es.
su b strate u s in g O 2 as transfer h y d r o g e n from o n e substrate
- A r e o f 2 types: p e r o x id a s e s & ca ta la se
hydrogen acceptor and to an o th er in a c o u p le d o x id a tio n -
- C a ta la se is a h e m o p ro tein co n ta in in g 4 h e m e g ro u p . In
fo rm in g H 2O or H 2 O 2 red u ctio n reaction
ad d itio n to p o s se ss in g p e ro x id a se a c tiv ity it uses H 2O 2
as p ro d u c t
C y to c h ro m e s are iron co n ta in in g
as electron d o n o r a n d accepto r
C ytochrom e oxidase h e m o p ro tein s in w h ic h iron ato m s
2 H 2O 2 ^ 2 H 2O + O 2
(aai) contain copper® o scillates b e tw e e n Fe+2 & Fe+3 C a ta la s e

d u r in g o x id a tio n & redu ction . A ll


F lav o p ro te in - P ero xid a se s are fo u n d in m ilk & in le u k o cyte s, platelets,
cy to c h ro m es (except aa3) i.e.
c o n ta in in g o xid a ses & tissues in v o lv e d in eico san o id m etabo lism . It redu ces
are L amino oxidase cytochrome b, Ci, c of respiratory chain
p ero x id es u s in g electron accepto rs su ch as asco rb a te,
(FMN linked), and cytochrome p-450 & bs of
q u in in e s , & c y to c h ro m e C
Xanthine oxidase (Mo endoplasmic reticulum are regard ed
Peroxidase
d e h y d r o g e n a se
containing) and H 2O 2 + a h 2 2 h 2o + a

aldehyde T h e y u se c o m m o n c o e n z y m e e g Oxygenase
dehydrogenase (F A D N A D + as h y d r o g e n carrier. B ecause
C a t a ly z e direct transfer & in corp o ration o f O 2 into a
lin k ed m o ly b d e n u m these reactions are reversible, these
substrate
& n on h e m e iron r e d u c in g eq u iv a le n ts (N A D + ) are
con tain in g) freely transferred in w ith in the cell.
*' Biochemistry. Structures & Functions <Jf Proteins and Enzymes ■ 751

Many dehydrogenase employ (niacin) Other depend on (riboflavin) flavoproteins (FMN or


nicotinamide coenzymes (NAD+ or NADP+) FAD)
I I
- Glucose -6 - phosphate dehydrogenase (NADP+) - Glycolate oxidase (FMN)
- Glutamate dehydrogenase (NADP+ or NAD+) - NADH dehydrogenase / complex I (FMN)
- Acyl CoA dehydrogenase

J
- Isocitrate dehydrogenase ^
- a-ketoglutarate dehydrogenase - Dihydrolipoyl dehydrogenase
- Malate dehydrogenase - Succinate dehydrogenase
- Lactate dehydrogenase - Glycerol-3- phosphate dehydrogenase
- Alcohol dehydrogenase - Thioredoxin reductase
- Glyceraldehyde-3-P04 v. - Electron transferring Flavoprotein (ETF)
dehydrogenase

Oxygenase
It oxidizes a substrate by catalyzing incorporation of oxygen into a substrate. It belongs to oxido- reductase family and is called
direct oxidase. It is divided into 2 sub groups
Dioxygenase Monooxygenase (Mised function oxidase, Hydroxylase)
- Cause in corporation o f Cause in corporation o f on ly one atom o f m olecu lar oxygen in to substrate®. The 2nd O 2 atom
b oth atom s o f O2into is reduced to water.
substate® i.e. An electron donor co substrate is necessary. Since main substrate is hydroxylated, it is also
A + 0-) — —--------------- > AO->
Dioxygenase c/a hydroxylase. Monooxygenase
- Examples include liver A - H (Main substrate) + O2 + ZH 2 (Cosubstrate)- Hydroxylase ^
enzymes, iron A - OH (hydroxylated) + H2O + Z ^
containing 3- hydroxy They are also called mixed function oxidase/oxigenase to indicate that they oxidize two
anthranilate dioxygenase different substrate simultaneously. Examples include cytochrome P-450 and phenylalanine
(oxidase), & hydroxylase (responsible for PKU).
homogentisate C ytochrom es P 450are (hem e containing) m onoxygenases® which take part in h y d rox y lase
dioxygenase (oxidase), cycle® (NADH, NADPH & Fe2 S 2 required), d etox ify drugs in liver m icrosom es® (with
and L- tryptophan 2, 3 cytochrome bs, Fe2 S2 not required).
dioxygenase (pyrolase) M itochondrial cytochrome P450 is found in steroidogenic - adrenal cortex, testis, ovary and
that utilize heme. placenta and is concerned with hydroxylation at C 22& C 20in side chain cleavage & at 11B & 18
positions during synthesis of steroid chromones from cholesterol. It catalyzes 1 a & 25
hydroxylation in vit. D metabolism (in kidney) and 7 ot hydroxylase & 27 hydroxylase in
bile acid synthesis (in liver).

Holoenzyme
It is active enzyme (protein part) with its non protein component

Non protein part

Transient association Stable & tight


incorporation
It is inactive Coenzyme Cofactor I
enzyme - If non-protein part is a small thermostable low - If non protein Prosthetic group
(protein part) molecular weight organic molecule, it is termed moiety is metal ion - These are tightly &
without its non coenzyme such as Fe++ or stability incorporated
protein - Coenzyme that only transiently associate with the Zn++, it is termed into protein (enzyme) by
component. enzyme are called cosubstrate (second substrate or cofactor covalent or non covalent
coenzyme - cosubstrate). Co substrate dissociate from - Bind reversibly in forces.
enzyme in an altered state, eg NAD+ & coenzym e A® transient dissociable - Metals are most common
- Coenzyme serve as recy clab le su bstrate shu ttles - o r manner either to prosthetic groups.
group transfer agents® that transport many substrates enzyme or substrate Metalloenzymes contain
from their point of generation to their point of - Enzymes requiring a tightly bound metal
utilization. It also stabilizes unstable substrates such as metal ion as ion, that is not
hydrogen atoms or hydride ions. cofactor are k/a dissociated even after
■Components of coenzymes are- vitam in B®, adenine, metal activated several steps of
ribose & phosphoryl moieties of AMP or ADP enzymes. purification.
752 ■ A Complete Review of Short Subjects

Vitamin B component Part of / coenzyme Function


Pantothenic acid Coenzym e AO Co ASH is acyl group carrier
Cobalamine (B]2) Cobamide Coenzym e in transfer o f 1 carbon fragm ent & m etab olism o f fo lic acid 0
Folic acid Tetrahydro folate Coenzym e in transfer o f 1 carbon fragm en t0
(TH4F)
Niacin (nicotinamide / N A D & N AD P Coenzyme in redox (oxidation and reduction) reactions
nicotinic acid)
Riboflavin (B 2) FMN & FAD
Pyridoxine, pyridoxal, Pyridoxal phosphate Coenzyme in transmination & decarboxylation of amino acids & glycogen
pyridoxamine (Be) phosphorylase
Thiamine (Bi) - Thiamine pyro • Coenzyme for oxidative decarboxylation by 3 multi enzyme complexes
phosphate (TPP) - P yruvate dehydrogen ase® (catalyze pyruvate to acetyl Co A)
- Thiamine di - a -ketog lu tarate dehydrogenase® (in citric acid cycle)
phosphate (TDP) - Branched chain ketoacid dehydrogenase for leucine, Isoleucine & valine
• Coenzyme for Transketolase ( (X ketoacid decarboxylation) in pemtose
phosphate (PPP)
Biotin - C oenzym e in ca rb ox y la tion ® reactions in gluco neogenesis & fatty acid
synthesis
Vitamin C (ascorbic acid) Coenzym e in hydroxylation o f prolin e & lysine in collagen sy n th esis0
Vitamin K - Coenzyme in carboxylation of glutamate residue in proteins of blood clotting (factor 2, 7, 9,10,
(phylloquinoses, protein C, S and Z) and bone matrix (osteocalcin) to form y -carboxy g lu tam ate residue0.
menaquinones)
★Coenzyme prosthetic group are small organic molecules that associate permanently with an enzyme & return to their
original form by enzyme. Eg are FMN, FAD, TPP, pyridoxal phosphate, biotin.

________________Selenium & Selenocysteine________________ Enzymes Cofactors


KG dehydrogenase TPPQ
- Selenium in form of selenocysteine is a component (as a prosthetic
groop) of glutathione peroxidase, thioredoxin, and deiodinase that Carbonic anhydrase Zn®
converts thyroxine to triodothyronine. Lactate /Alcohol Zn®
dehydrogenase
- Selenocysteine, the 21st aminoacid is present at the active site of
Xanthine oxidase Mo
enzymes and participate in catalytic mechanism of these enzymes.
Tyrosinase ~l Cu
So the replacement of selenocysteine by cysteine can significantly impair
Cytochrome oxidase
catalytic activity.
Carboxylation B io tin®
- Unlike hydroxy proline & hydroxy lysine, selenocysteine arises by Transamination Pyridoxine®
cotranslational insertion from a previously modified t-RNA (t- Mg®
Kinase & peptidase
R N Asec). Serine provides the carbon skeleton of selenocysteine.
Hb & Cytochrome Fe
Biosynthesis of selenocyteine requires selenate (SeO^-), cysteine,
Hydrolase, decarboxylase & Manganese
ATP, a specific unusual t RNASC'Cand enzymes. The U G A anticodon
transferase
of t RN Asec signals STOP and this selenocysteine specific UGA stop
codon is identified by selenocysteine insertion element (ie a stem
loop structure in untranslated m RNA).

I Ligase | [ Selenophosphate Synthetase I


tRN Asec + Serine ------------- ► tR N A ser ' —► * Selenophosphate
Selenium

Cysteyl-t RNAsec » Aminoacrylyl tRNAsec * Selenocystelyl tRNAsec

- In presence of specific elongation factors that recognize selenocysteyl - 1 RNAsec, selenocysteine is incorporated into
protein.
- In erythrocytes pentose phosphate pathway (PPP) is the only source of NADPH for the reduction of oxidized glutathione
catalyzed by glutathione reductase (containing FAD). Reduced glutathione removes H 2O 2 in a reaction catalyzed by
glutathione peroxidase. Glutathione peroxidase contains selenium an alog o f cystein e (selenocysteine) as a p rosth etic
group®. In erythrocytes and other tissues, it catalyzes the destruction of H 2O2and lipid peroxidases, protecting membrane lipid
Biochemistry: Structures & Functions of Proteins and Enzymes ■ 753

and hemoglobin against oxidation. This reaction is important since accumulation of H 2O 2 decreases life span of RBCs by
causing oxidative damage to cell membrane proteins, lipids & nucleic acids 1/1 hemolysis. In other tissues, NADPH can
also be generated by reaction catalyzed by malic enzyme. So PPP & glutathione peroxidase protect RBCs against
hemolysis.

NA DP+ 2 G - S H (Reduced Glutathione) H 2O 2


Glutathione
FAD
Reductase
NADPH + H + GSSG (Oxidized Glutathione) 2 H 2O

Selenium has sparing effect on vitamin E reducing its requirements and similarly vitamin E reduces selenium
requirement. Selenium & sulphur protect against toxic effects of heavy metals such as cadmium, Hg & Ag (b/o high
affinity).
Seafood, muscle meat & cereals are rich dietary sources of selenium, although the Se content of cereals is determined by
soil concentration. Scandinavia, China & New Zealand have low soil Se content. Daily requirement's are 0.1 to 0.2 mg/kg
(~ 50-100 Hg in 70 kg adult)

Selenium D eficiency Diseases Selenium Toxicity


1. Keshan disease: Endemic cardiomyopathy 1. Selenosis (chronic exposure) presents with garlicky
(enlargement, arrhythmia & ECG changes) mainly breath (d/t dimethyl selenide), chronic dermatitis, skin
affecting children & young women with intake < rash, hair & nail brittleness and loss, myopathy,
20 pg/d. irritability & other abnormalities of nervous system (in
2. Kaschinbeck disease:Endemic degenerative humans). Animals develop impaired vision, movement
osteoarthritis (osteopathy) mainly in children disorder (blind staggers), alkali disease (anorexia,
between 5 & 13 years with enlarged dysfunctional alopecia, loss of vitality, myocardial atrophy & liver
joints & retarded growth. necrosis), paralysis & death.
3. Liver cell necrosis, exudative diasthesis, pancreatic 2. Acute toxicity in animals present with diarrhea, raised
degeneration, muscular dystrophy, myopathy, PR & temperature, tetanic spasm, laboured breathing,
infertility, growth failure, and dilatation of heart 1/1 respiratory failure & hemorrhagic necrosis of various
CCF. (in animals) tissues.
Chapter 6. METABOLISM OF PROTEIN & AMINO ACIDS: REVIEW NOTES

Biosynthesis of Urea
Biosynthesis of urea has 4 stages (1) transamination, (2) oxidative deamination of glutamate (3) ammonia transport, and (4)
urea cycle.

1. Transamination (Braunstein & Kritzmann Reaction) 2. Deamination


Despite being most abundant in atmosphere, N 2 is preciously handled in biological system, It is rem ov al o f N o f
because only few micro organism can convert N 2 to biologically useful form (as N H 3 / amino am in oacid as NH3
group). Aminoacids derived from dietary protein are the source of amino groups in humans. Oxidative deamination is
The presence of this a-am in o group keeps amino acids safely locked a w a y fro m ox id ativ e catalyzed by D & L-
breakdow s® . So removal of a amino group is essential obligatory step in energy production
aminoacid oxidase. Despite
or catabolism of all aminoacids. absence of D aminoacids in
tissues, the D-aa-oxidase
Transamination is a reversible reaction in which a - NH 2 group of one aminoacid is
enzyme is much greater than
transferred to a a - ketoacid resulting in formation of new amino acid a new keto acid. Once
L-aa-oxidase
they have reached the liver, transamination is the first step in catabolism of most amino
L aminoacid oxidae is auto
acids and involves only intermolecular transfer of amino (NH2) group without formation
oxidizable flavoprotein (FMN),
(splitting out) of ammonia ( N H 3 ) . No net deamination & occurs because a-amino acid is
which is reoxidized, when
deaminated & a-keto acid (mostly a-keto glutarae) become aminated.
reduced, at substrate level
directly by O 2 forming H 2O 2
Aminotransferase (Transaminase) Enzyme without participation of any
electron carrier (like
- This transfer of a-amino group from L Salient Features cytochrome or ETS). Toxic
amino acid to a-ketoacid is enzymed by - Transamination interconverts p airs o f ex- H 20 2 is converted to 0 2 & H20
amino transferase or transaminase enzyme am ino acid s and a -k e to acids, transferring a- by catalase.
- Enzyme is found in mitochondria and cytosol am ino nitrogen (NH 2 group) w ith ou t splittin g
of cells through out the body especially those of ou t o f am m on ia (NH]) to a-k eto g lu ta rate Feature L-AA D-AA
liver, kidney, intestine, muscle, heart and brain. form in g glutamate®. oxidase oxidas
- Each aminotransferase is specific for one - Alanine pyruvate (or alanine) e
pair of substrate but non specific for other aminotransferase and glutamate - a- Flavoprote FMN FAD
pair. Each is specific for 1 or at most few ketoglutarate (or glutamate) in
am ino group don or on w hich their nam es aminotransferase catalyze the transfer of Acts on L- D-
are based. B ecause the accep tor o f amino amino
amino (NH 2) group to pyruvate (forming
am inogroup is alm ost alw a y s a -k e to (oxo) acids acids
alanine) or to a-ketoglutarate (forming
glutarate®. O ther rare recipien t k eto acid s
glutamate). Since alanine is also a substrate Extracted Not; Freely
are o x a lo a ceta te or pyruvate. Example are
for glutamate aminotransferase, the a-amino with water because
a. Alanine (Alanine-Pyruvate) free form bound
nitrogen from all aminoacids that undergo
Aminotransferase (ALT)= Serum
tran sam ination can be concentrated in to
Glutamate Pyruvate Transaminase and
glu tam ate. Therefore tran sam ination reaction tissue
Glutamate (Glutamate a-ketoglutarate) co llects the am inogroup fro m m any differen t
Aminotransferase am in o acid s in fo rm o f L-glutamate®. And Non oxidative deamination
Transfer aminogroup of alanine to a-keto then L-glutamate functions as aminogroup has a very minor role in N H 3
(oxo) glutarate, resulting in formation of donor for biosynthetic pathways or for formation and can occur in
pyruvate & glutamate (glutamic acid) excretion pathways that lead to the 1. Sulfur containing amino
elimination of nitrogenous waste products. acids like cysteine b
A la n in e ^ L T (S G P T ) Pyruvate
- a-ketoglutarate p la y s a cru cial ro le by homocysteine by
a -k eto g lu ta ra te
G lu tam ate accepting am in o (NH2) groups fro m m o st desulfhhydrase (removing
G lu tam ate (glutam ic
am in o acid s an d thus becom in g glutam ate® . H 2S).
AT acid)
So transamination collects amino groups from 2. Hydroxy amino acids like
many different aminoacids in form of L- serine, homoserine &
b. Aspartate Aminotransferase (AST)=
glutamate (i.e. called fu nn elin g o f threonine by aminoacid
Serum Glutamate Oxaloacetate,
am inogroups to L -g lu tam ate). This is dehydrogenases requiring
Transaminase (SGOT)
important because L-glutamate is the only amino pyridoxal (B6)- P as
Exception to the rule that
acid that undergoes oxidative deamination at an coenzyme.
aminotransferase enzyme funnel amino
appreciable rate in mammalian tissue. So 3. Histidine by histidase
groups to form glutamate. Because during
formation of ammonia from a-amino group forming urocanic acid &
aa catabolism, it transfer aminogroup of
thus occur mainly via a-amino nitrogen of L NH3.
glutamate to oxaloacetate forming
glutamateQ (by oxidative deamination)
Biochemistry: Metabolism of Proteins and Amino Acids ■ 755

aspartate (which is used as source of - Transamination occur in almost all amino Deamination of L-
nitrogen in urea cycle) & a-ketogluterate. acids except threonine, lysine (loose amino Glutamate =
group by deamination), proline, T ransdeamination
G lu tam ate a -k eto g lu ta ra te hydroxyproline, and cyclic iminoacids.
O xalo acetate SG O T A sparate
- Transamination is not restricted to a-amino L-Glutamate formed as a main
groups. The B-amino group of ornithine (but product of transamination is
★ However in biosynthesis (when reaction is not e amino group of lysine) readily not deaminated by L-amino
reversed) the aspartate becomes donor & undergoes transamination. acid oxidase but by hepatic L-
a-ketoglutarate acceptor, - Glutamate produced by transamination can be glutamate dehydrogenase
c. Formation of alanine by transamination oxidatively deaminated or used as (GDH).
of pyruvate. Amino donor may be aminogroup donor in synthesis of non GDH is a tetramer
glutamate or aspartate thus forming a- essential amino acids. metalloenzyme containing 1
ketoglutarate or oxaloacetate - Equilibrium con stan t for transamination is ~1 Zn++ in each of 4 polypeptide
making it readily reversible and allowing amino chain. It can use NAD+ or
Pyruvate

x
Alanine
acid biosynthesis (by addition of amino group NADP+ as coenzymes and
G lu tam ate a -k e to g lu ta ra te o r to keto acid when supply of amino acid in diet release nitrogen as ammonia.
or A spartate o x a lo a c e ta te is not adequate) and amino acid degradation It is allosterically inhibited by
(by removal of amino group when diet is rich ATP, G TP and N A D H and is
in protein) both. activated by ADP.
Reaction is freely reversible
Coenzyme Pyridoxal Phosphate (Vitamin Biderivative) and equilibrium constant
favours glutamate (amino
P yridoxal p h osp h ate coenzym e (or p ro sth etic group) is co v alen tly bound to enzym e's activ e acid) biosynthesis but quick
site by an S chiff b ase (aldim ine) linkage® to £ amino group of Lys residue and serves as a removal of NH 3 to urea cycle
carrier of amino group (and thus generating pyridoxamine phosphate which transfers this & a-ketoglutarate to TCA
amino group to a-keto acid). cycle favours ammonia
Transamination occurs via ping pong reaction which means the l sl substrate reacts & formation.
product must leave the active site before 2 nd substrate can bind.

L-Glutama^e L-GDH ^a-ketoglutarate +


■*NH3
NAD(P)+ NAD(P)H+ + H+

Conversion of alpha-amino
nitrogen to ammonia by
coupled action of glutamate
aminotransferase and G D H is
called transdeamination.

PIN G -P O N G M E C H A N ISM O F T R A N SA M IN A T IO N

Non toxic Transport of Ammonia (& Amino group)

I n Skeletal Muscle
In most tissues (especially brain) nucleotide degradation - In tissues that degrade aminoacid for fuel
produce free ammonia, which being toxic is immediately (mainly skeletal muscle), amino group is
trapped by glutamate (glutamic acid) to form nontoxic collected in form of L-glutamate (d/t
glutamine. This reaction is catabolized by glutamine transamination)
synthetase and require ATP. - This glutamate (containing toxic a-amino
group) can be transported to liver by converting
it to glutamine or it can transfer its a-amino
L -G lu ta m a te + N H j ' ’ L - G lu ta m in e
G lu ta m in e S y n th e ta s e group to pyruvate, a easily available product of
T o x ic N o n lo x ic muscle glycolysis by the action of alanine
aminotransferase (ALT/SGPT). The alanine so
Aspartic acid may undergo similar reaction to form asparagine,
formed passes into blood to liver.
756 ■ A Complete Review of Short Subjects

w h ic h tr a p s N H 3 in b r a in (m in o r) - In liver cell cytosol alanine aminotransferase


Glutamine is non toxic transport form o f NH 3and serves as a source (ALT/SGPT) transfers amino group from
of amino group in biosynthetic reactions. It is normally present in alanine to a-ketoglutarate forming pyruvate
blood in much higher concentrations than other amino acids. (which enters gluconeogenesis) and glutamate
Excess glutamine (of amount that is required for biosysthesis) is (under go oxidative deamination)
transported to liver, intestine and kidney, where enzyme - Vigrously contracting muscles operate
glutaminase reverse the reaction and convert glutamine to anerobically producing pyruvate & lactate (from
glutamate (glutamic acid) and NH 3 (INf-Lp). The toxic NTLp glycolysis) and N H 3 (from aa break down).
from intestine, kidney & other tissues is rapidly removed from Alanine-glucose cycle in concert with Cori cycle
blood by liver and converted to urea. send these products to liver where pyruvate &
In kidney (renal tubular cells) am m on ia (NH4 +) production lactate are incorporated into glucose (by
fro m intra cellu lar am ino acids m ainly glu tam in e increases in gluconeogenesis and returned back to muscle)
m etab o lic acid osis and decreases in m etab o lic alkalosis® . So in and N H 3 is converted to urea for excretion. So
metabolic acidosis not all NTLp produced in kidney is released the energetic burden of gluconeogenesis is
into blood to form urea in liver; some is excreted directly into the imposed on liver and all available ATP in
urine thus facilitating cation (positive ion) conservation and muscle is devoted to muscle contraction.
regulating acid base balance.
Glutaminase and asparaginase deam in ate glu tam in e &
asparagines thus releasing toxic NH 3 ®

Hyper-Ammonemia: NH3 Intoxication


N H 3 is produced by protein catabolism and is mainly converted to urea (by urea cycle) in liver. Some N H 3 is also used for
transamination of a-ketoacid to a-amino acid and in synthesis of glutamine.
Disorders of urea cycle (i.e. any enzyme of urea cycle) would result in hyper ammonemia, encephalopathy & respiratory
alkalosis. Brain is very sensitive to ammonia. Clinical features include mental retardation, lethargy, irritability & vomiting.
Block in earlier steps of urea cycle results in more symptoms since ammonia itself accumulates, whereas deficiency of later
enzymes result in accumulation of less toxic other intermediates and hence symptoms are less. Hyper ammonemia type I
(d/t CPS I- deficiency), hyperammonemia (d/t O TC def.), hyperornithinemia (defect in ornithine transporter protein),
citrullinemia (def. of argininosuccinate synthetase), argininosuccinicacid uria (def. of argininosuccinate lyase) and
hyperargininemia (arginase def) are disorder of urea cycle. Increase NH 3 concentration causes
___________________I_________________
I 1
Increased tranamination of a- Increased glutamine synthesis from glutamate (glutamic acid) in
ketoglutrate (a TCA cycle intermediate) to brain leading to
form glutamate (glutamic acid)
Decreased brain cell Increased brain glutamine level and its
pool of glutamic acid outflow
I
1 Increased tryptophan level in brain cells
D ecreased m itochon drial p o ll o f a - Decreased because glutamine is carried out by same
k etog lu tarate inhibiting TCA cycle & synthesis of G A B A transporter which allows entery of
cellu lar respiration®. (inhibitory tryptophan in brain cells
neurotransmitter)
i
Increased synthesis of serotonin

Urea Cycle / Krebs - Henseleit Cycle / Ornithine Cycle

Urea cycle leadin g to fo rm a tio n o f urea ta k es p la ce in liver®. Urea is the most important disposal form of NH 3 or amino
group derived from amino acid catabolism. One mol of urea synthesis requires 1 mol each ofNH4+ (ammonium ion) & of a-
amino nitrogen of aspartate and 3 mol of ATP (but 4 high energy phosphate bonds as 1 ATP of these 3 is converted to
AMP thus using 2 PO 4 bonds.)
•One nitrogen of urea molecule is supplied by free ammonia and other by aspartate. However, glutamate is the immediate
precursor of both ammonia (through oxidative deamination by GDH) and aspartate nitrogen (through trans amination of
oxaloacetate by AST). In effect, both nitrogen (N) atoms of urea arise from L-glutamate, which in turn gathers them from
aminoacids. The carbon (C) & oxygen (O) atoms of urea are derived from CO 2 (or HCO 3 )
Biochemistry: Metabolism of Proteins and Amino Acids ■ 757

It is a cyclic process with 6 participating aminoacids and 5 enzymes. There is not net loss or gain of ornithine, citrulline,
argininosuccinate, or arginine. However, ammonium ion (NH4+), C O 2,3 A T P (or 4 high energy phosphate bonds) are
consumed and there is net conversion of (oxaloacetate via) aspartate to fumarate.

Aspartate + NH 3 + CO 2 + H2O + 3ATP —> Fumarate + Urea + 2ADP + 2AMP + 2Pi + PPi

However fumarate formed in urea cycle may be converted to malate, which when oxidized to oxaloacetate produce 1
NAD PH equivalent to 2.5 ATP, so net energy expenditure reduces to only 1.5 high energy phosphates.
First two reactions i.e. formation of carbamoyl phosphate (catabolized by ca rb am o y l p h o s p h a te sy n th etase 1 enzym e) and
formation of citrulline (catabolized by ornithine transcarbamoylase) takes place in mitochondria. Citrulline is transported
from mitochondria to cytosol and remaining steps i.e. formation of argininosuccinate (enz. argininosuccinate synthetase), its
cleavage to arginine & fumarate (enz. argininosuccinate lyase), and cleavage of arginine to ornithine and urea (enz
arginase) takes place in cytosol. Ornithine returns to mitochondria to react with another molecule of carbamoyl P O 4 .
Ornithine is regenerated with each turn of urea cycle, much in the same way the oxaloacetate is regenerated in citric acid
cycle. Ornithine & citrulline are basic aminoacids that are not incorporated into cellular proteins because there are no
codons for these amino acids.
2ATP molecules are required to make carbamoyl P O 4 and 1 ATP (2 high energy phosphate bonds) to make
argininosuccinate. First step of urea cycle i.e. condensation of CO 2, ammonia and 2 ATP 1/t formation of carbomoyl
phosphate by carbamoyl P O 4 synthetase I is the rate lim iting step (& enzyme). This enzyme is only active in presence of
N-acetyl glutamate (NAG) an a llosteric a c tiv a to r th a t enhances the affin ity o f enzym e fo r ATP®. NAG is synthesize by
N-acety glutamate synthetase from acetyl CoA and glutamate.
Because fumarate produced in urea cycle is also an intermediate of citric acid cycle, both cycles are inter connected (k/a
Krebs bicycle) by aspartate - argininosuccinate shunt. However each cycle can operate independently. And fumarase &
malate dehydrogenase used in urea cycle are cytosolic (not mitochondrial like TCA cycle).
Kidney & Intestine contribute to most of body arginine because they possess all the urea cycle enzyme except arginase.
Brain can synthesize urea from citrulline, but lacks enzyme for formation of citrulline from ornithine.
Thus neither brain nor Kidney can form urea in significant amount.

Serine Proteases

The activ e site o f serine p ro tea ses contain • Pancreatic trypsin inhibitor is serine protease inhibitor only for trypsin
three critical am in o acids Serine, and not for other serine protease enzymes such as chymotrypsin or
H istidine and A spartate. These residues elastase. Serine proteases enzymes tightly bind the respective protease
are often referred to as the ca ta ly tic triad. inhibitors instead of normal substrates.
Chymotrypsin, Elastase and Trypsin are the
three serine proteases that function essentially Serrne protease
as enzymes.Each of these enzymes contain the Trypsin Pancreatic trypsin inhibitor
'catalytic triad' within their active sites but Chymotrypsin Alpha-1 antichymotrypsin
differ with respect to their target cleavage
Elastase Alpha-1 antitrypsin
sites.- www.rpc.msue.edu
They are activated by proteolytic • Serine proteases attack the carbonyl group of the peptide bond that is to
activation and not autocatalytic be cleaved (and not essentially the one adjacent to serine residue)
activation. They are secreted by the Serine protease -
pancreas as inactive zymogens into small Chym otrypsin Peptide bond next to arom a tic residues
intestine- and became active through (Phenylalanine, Tyrosine Tryptophan)®
proteolytic cleavage events, either in a Trypsin Peptide bond next to b asic residues (Lysine or
subcellular compartment or in an Arginine)®
Elastase
extracellular space where they are
Peptide bonds next to H yd rop h obic residues
required for digestion.
(Alanine)®
758 ■ A Complete Review of Short Subjects

Creatinine & Creatinine Biosynthesis


L- Arginine Ornithine
Glycine -► Glycbcyamine (Guanidoacetate)
Amidino- transferase or
Liver
Transamidinase Kidney)®
S- Adenosyl methionine \ / A T P
v Gau a n id o a c e ta te

P i+ T hO
S A d en o sy l
H o m o cy ste in e
AM M e th y l tra n sfera se

ADP
N o n en zy m atic
Creatinine ■<- Creatine phosphate

S-Adenosyl methionine (Liver)

Gluanido - acetate
S-Adenosyl homocysteine / Methyl transferase

Creatine
ADP \ A >»ATP
P'-
Creatine Kinase J
ATP t / \ ;P
aD
N o n en zy m atic (sp on tan eo u s)
Creatinine ■
<---------- Creatine Phosphate
(excreted in u rin e & is
Pi + H 2O
in d ica tiv e o f to tal c rea tin e P O 4
co n ten t an d m u scle m ass)

Catabolism of Carbon Skeletons of Aminoacids

A.A that form Oxaloacetate A.A that form Fumarate A.A. that form Sucdnyl CoA
Asparginine (Aspartate) - Phenylalanine - M ethionine - Isoleucine
- Tyrosine - Valine - Threonine

A.A That form a-ketoglutarate A.A.that form Pyruvate A.A that form Acetyl CoA/Acetoacetyl CoA
- Glutamine - Alanine - Cystine - Leucine - Tryptophans
- Proline - Serine - Threonine - Isoleucine - Phenyalanine
- Arginine - Glycine - Lysine - Tyrosine
- H istidine

Classification of Ammino Acids (aa) on the basis of R-Group or Side chains

Nonpolar (Hydrophobic or Water insoluble) side chains Polar (Hydrophillic or Water soluble)
side chains
Nonpolar side chains does not gain or lose protons or participate in hydrogen The most hydrophilic (polar) R groups are
or ionic bonds. Oily or lipid like side chains promote hydrophobic interaction. those that are either positively or
In proteins found in polar environment or aqueous solutions - the side chains negatively charged
of nonpolar aminoacids tend to cluster together in interior of protein I
(hydrophobic effect) and help give it its three dimensional shape. However in Polar, Uncharged (Neutral or Non
hydrophobic environment such as membranes the nonpolar side chains are ionic) side chain
found on the outside surface of protein interacting with lipid environment - Have zero (0) net charge at neutral pH,
although cysteine and tyrosine can lose
Aliphatic (Nonpolar / Aromatic (Non polar/ relatively proton at alkali pH
Hydrophobic) Hydrophobic) Side chain - They contain functional group that form
Side chain)
hydrogen bond with water making them
Glycine Sim plest structure® All can participate in hydrophobic
Very small (H) side interactions. Serine Hydroxyl group
chain makes no real Tyrosine & tryptophan are Threonine Hydroxyl group
contribution to significantly more polar than Cysteine Sulfhydryl group
hydrophobic phenyl alanine, because of tyrosine (weak acid)
interactions
Biochemistry: Metabolism of Proteins and Amino Acids ■ 759

Alanine M ethyl sid e chain® hydroxyl group& nitrogen of Asparagine Amide group
Valine C3H7 side chain tryptophan indole ring Glutamine Amide group
Leucine & C4H9 side chain Tyrosine hydroxyl (OH) group can Tyrosine Hydroxyl group
Isoleucine form hydrogen bonds - Asparagine & glutamine are the amides of
Alnine, leucine, Isoleucine & Tryptophan & tyrosine, an d to 2 other aa, aspartate & glutamate
Valine (mn "ALL Is W ell") tend to much lesser extent p h en ylalan in e respectively
cluster together within proteins ab so rb u ltrav io let light a t a w av e - Cysteine is oxidized to form a covalently
stabilizing protein structure by length o f 280nm.® linked dimeric amino acid called cystine,
means of hydrophobic interactions (photochromocity) in which 2 cysteine molecules are joined
Phenylalanine has benzene, tyrosine by disulfide bond making it strongly
has phenol and tryptophan has indole hydrophobic (non polar)
groups respectively._______________ ★ Cysteine & tyrosine may show
★Hydropathy index is negative hydrophobic character when present in
Methionine Sulfur containing (AG< O) for polar or charged interior of protein. Glycine may be
amino acid with aminoacids; and positive (AG> O) included in this group also.
thioether side chain for nonpolar or more hydrophobic
Proline Side chain form a aminoacids. All of aliphatic & Polar ionic, Positively charged
distinct cyclic (ring aromatic aa have (+) value except (Basic) side chain /aa
shaped), rigid 5 glycine (- 0.4), tryptophan (-0.9),
membered tyrosine (-1.3), and histidine (-3.2). Lysine Second primary amino at e
secondary (rather Hence histidine is not included & position on its aliphatic chain
than primary amino tyrosine is relatively included in Arginine +ve charged guanidinium
group k/a imino non polar group group
group. It reduces ★Histidine is a positively charged Histidine Aromatic imidazole group
structural flexibility,
(basic) aa with NH 3 side chain. It
and contributes in Polar, Ionic, Negatively charged
formation of fibrous also has aromatic im idazole side
(Acidic) side chains
structure of collagen chain. As the only aa. having an
imidazole side chain with pKa near 2 aa. with a net negative charge at pH 7
and often interrupts are aspartate & glutamate each of which
a-helices found in neutrality, histadine m ay be
p o sitiv ely charged (p roton ated has 2nd carboxyl (COO-) group.
globular proteins. Aspartic acid & glutamic acid contain P
form ) or uncharged a t pH 7)®.
It facilitate many reactions by carboxyl and y carboxyl groups
serving as proton donors / respectively.
acceptors.
*a- carbon atom of aminoacid is optically or chirally active as it is attached to 4 different groups. G lycine is the
exception and is optically inactive b/o 2 hydrogen atoms attached.
Aminoacids with asymmetrical a-carbon center can exist in two 'D ' and 'L' mirror image forms k / a optical isomers,
stereoisomers or enantiomers. Only 'L' (levorotatory) alpha aminoacids occur in proteins.
Naturally occurring 'D ' - aminoacids are free 'D ' serine & 'D'-aspartate in brain; 'D ' alanine & 'D ' glutamate in cell
walls of gram (+) bacteria; D aminoacids in some antibiotics & peptides produced by bacteria, fungi etc.

Polar -Ionic - Charged - Amino acids

Basic (Positively charged) Acidic (Negatively charged)


Monoamino-dicarboxylic acids Diamino-monocarboxylic acids
At physiological pH the side chains are fully ionized At physiological pH the side chains are fully ionized
(polar) and positively charged containing an extra (polar) and negatively charged containing an extra
positively charged amino group negatively charged carboxylic group.
These contain one negative charged carboxylic (- Acidic aminoacids contain 1 amino and 2 carboxylic
COOH / - COOT group and two positive charged groups. Hence called mono amino dicarboxylic acids
amino (-NH 2) groups. Hence called di amino-mono Acidic aminoacids are proton donors
carboxylic acid Examples include aspartic acid (or a s p a r ta te ) and
Basic aminoacids accept protons glutamic acid (or glutamate). These 2 are written in all
Examples include : Arginine (most basic), lysine, books but chatterjae also includes glu tam in e &
hydroxyl-lysine, and histidin e (w eakly b asic /neutral)® asp arag in es in this group. But these two are included in
polar uncharged (neutral) in Lehninger (Go with this)
760 ■ A Complete Review of Short Subjects

Functions of Glutathione

Antioxidant:
G -S-S-G
- Reduced glutathione (GSH) can d etoxify H20 2 by g lu tath ion e peroxidase® N A D P H + H +. 2 H iO
'^ ^ (oxid ized ) ^
and itself converted into oxidized form (G-S-S-G).
G lu ta th io n e G lu ta th io n e
- Oxidized glutathione contains two glutathione linked by disulphide bond R e d u c ta s e P e ro x id a se
between its cysteine residue, hence its reducing p roperty is becau se o f 2 G-SH
cystein e sulfhydryl group®. NADPT V h 20 2
(redu ced)
2. It is important intracellular reductant, helping to maintain essential SH gp of
enzymes in reduced state.
3. Membrane transport: It acts as a carrier in tran sport o f certain am ino acid across m em brane in kidney®.
Amino acid + GSH —>y - Glutamyl amino acid + Cystein glycine
H elps in conjugation reaction ®: Toxic xenobiotics are conjugated to nucleophilic GSH (glutathione) to convert to more polar,
hydroxylated derivatives.

Cysttnuria f f f t j g o u n Cysteine
Is a disorder of proximal tubule s reabsorption of filtered cystine and Cysteine is a sulphur containing am ino acid®.
dibasic amino acid (Ornithine, Lysine, Arginine). So there is excretion Cystine is formed by two molecules of
of - Cystine, Ornithine, Lysine & Arginine^ cysteine, joined together by S- S bond.
It is most common genetic error of amino acid transport affecting Cysteine can be synthesized in body from
aminoacid carrier in small intestine & proximal tubule of kidney (as methionine.
both share same transport carrier) for COAL amino acids. Prominent a.a in protein of nails, hairs and
Complications: Cystine is relatively insoluble a.a and therefore keratin of skin.
precipitates in renal tubules, ureter and bladder to form cystine
calculi®.
Diagnosis: By C yanide- N itroprusside test®._________________

Risk Factors For Atherosclerosis (Hence Ml)


_L
Major
Emerging risk factors
Non modifiable Potentially modifiable - L ipoprotein (a)®
I I - H om ocysteine®
Increasing age - Hyperlipidimea - H yperfibrinogenem ia®
Male gender - Hypertension
- Proinflammatory
Family h/o premature - Cigarette smoking
factors
CHD - Lifestyle risk factors
ob esity (BM I >30 kg/m 2)®.
- Physical inactivity.

Phenyl Ketonuria (PKU) 3 2 :

It is m o s t c o m m o n c l i n i c a l l y e n c o u n t e r e d in b o r n e r r o r o f a m i n o a c i d m e t a b o l i s m . I t is b i o c h e m i c a l l y c h a r a c t e r i z e d b y a c c u m u la t i o n o f
p h e n y l a l a n i n e a n d a d e fic ie n c y o f ty ro sin eQ .
In th is a u to som a l recessiv e d iso rd er, d u e to d e fic ie n c y o f p h e n y l a l a n i n e h y d r o x y la s e (P A - 4 m o n o o x y g e n s e , a m ix ed fu n c tio n o x id a se ),

P h e n y la la n in e h y d ro x y la se re q u ires a c o fa c to r te tra P h e n y l a la n in e 5 , 6 ,7 ,8 - T e t r a h y d r o b i o p t e r i n
h y d r o b io p te r in , w h ich c a rrie s e- fro m N A D P H to O 2 &
N A D (P )'
(B H « ) /K At
b e c o m e s o x id iz e d to d ih y d ro b io p te rin . It is su b s e q u e n tly o *\ f \ /

re d u ced b y e n z y m e d ih y d r o b io p te r in re d u c ta s e in a
Phenylalanine I
re d u c ta se in a re ctio n th a t re q u ire s N A D P H . S o h y p e r hydroxylase .A
reductase
p h e n y la ta n in e m ia m a y a lso b e c a u se d b y d e fic ie n c y o f
e n z y m e s sy n th e siz in g B H i o r re g e n e ra tin g BH« fro m B H 2
H 2 ° / \ A
(i.e. d ih y d ro b io p te rid in e re d u cta se)
T y ro s in e ^
N A D (P ) H + H*
7 , 8 - D i h y d r o b i o p t e r i n ( B H 2)
(Q u in o n o id f o r m )
■' Biochemistry. •Metabolism <Jf Proteins and Amino Acids ■

Hyperphenylalaninemia, d/t dihydrobiopterine


reductase deficiency (Type II and III), is
accompanied with BH 4deficiency. BH4 is also
required fo r tyrosine hydroxylase and tryptophan
hydroxylase® leading to synthesis of neurotransmittcrs
L-DOPA, catecholamines (from tyrosine) and 5-hydroxy
tyrptophan & serotonin (from tryptophan). So in type II
& III PKU simply restricting phenyl alanine and
providing tyrosine does not reverse CNS effects d/t
deficiencies in neurotransmitters. Replacement therapy
with BH4 or L- DOPA and 5-OH- tryptophan is also
needed. Supplementing the diet with tetrahydro
biopterin itself is ineffective as it is unstable and does
not cross blood brain barrier.
Elevated phenylalanine undergoes trans amination to form phenyl pyruvate, phenyl lactate and phenyl acetate.

Pathophysiology ■ Clinical features:


- Hypopigmentation®d / t impaired
In P h e n y lk e to n u r ia In n o rm a l in d iv id u a l melanin synthesis 1/t fair hair,
(A ltern a tiv e p athw ay) light skin, blue eyes
- P henylalanin e h ydroxylase (I)
C a te c h o la m in e s 0 - CNS symptoms: Mental
P h en y l - D ih y d ro b io p terin redu ctase (II, III)
' T h y r o id h o rm o n e ® retardation, seizures,
p y ru v ic a c id t T ransyn[inase
w a g in g M K ►
[p h en y la la n in e T y r o s in e * M ela n in e® hyperactivity, tremors, failure to
(P PA )
a k e to g lu ta r a le
T y ra m in e walk, talk & grow and IQ < 50
-B H 4 P h en o l - Excretion of phenylalanine & its
- BH 2 (b io sy n th esis d efect, IV , V )
catabolite in urine: PPA, PAA
P h en y l lactic P henyl a cetic - Mousy (musty) odour urine®: PAA
- A b se n t in PK U
A c id (P LA ) acid (P A A ) conjugates with glutamine and
- So in P K U a cce sso ry
excreted as phenyl acetyl
m etab o lic p ath w ay
+ L G lu ta m in e glutamine.
o f p h e n y la la n in e is
activ ated ■ Diagnosis
P h en yl a cety l g lu ta m in e
[u rin e m o u s y ad ou r® ]
• Antenatal prenatal diagnosis is
facilitated by DNA probes to
detect enzyme defect & tandem
Types of PKU & other PA metabolism defect
mass spectrometry (most reliable
to estimate blood PA levels).
1 Defect in phenylalanine hydroxylase (classic)Q
• Neonatal screening and diagnosis
II Defect in dihydrobiopterin (BHi) reductase^
I II 6 Pyruvoyl-tetra hydropterin synthase defect of PKU by elevated blood levels
IV D efect in d ih y d ro b io p terin (B H 2) b io sy n th esis
of PA
V - It is important because the disease
M a te rn a l P K U W h en w o m an w ith P K U w h o is n o t on low p h en y l alan in e d ie t is treatable by dietary means and
b eco m e p re g n a n t neonates lack symptoms.
It ca u ses m icro ce p h a ly , m en tal re tard atio n & co n g en ital h eart - The infant with PKU has normal
a n o m a lies in fetu s) blood levels of phenylalanine (PA) at
T r a n s ie n t b e n ig n h y p e r P te rin -4 c a rb in o la m in e d e h y d ra ta se d e fic ie n c y birth because the mother clears
p h e n y la la n in e m ia increased blood PA through
G T P C y c lo h y d r o la s e I D efect o f PA m etab o lism
placenta.
d e fic ie n c y
- Normal PA levels may persist
Treatment until the new born is exposed to 2-
4 days of protein feeding. Thus
T h e m a in a im fo r l sl lin e th e ra p y is to lim it t h e i n t a k e o f s u b s t r a t e (i.e. p h e n y l a la n in e ) f o r
d e f i c i e n t e n zy m e® .
screening tests are done after this
period to avoid false negatives.
In P K U , b lo o d p h e n y l a la n in e is m a in ta in e d c lo s e to th e n o rm a l ra n g e b y fe e d in g s y n th e tic
a m in o a c id p re p a ra tio n s lo w in P A , su p p lem en ted w ith so m e n a tu ra l food (su ch as fru its,
- False positive in premature
v e g eta b les & so m e cere a ls) selected for th eir low p h en y la la n in e con ten t.
infants may reflect delayed
maturation of enzymes of PA
B e ca u se P A is an esse n tia l a m in o a cid , ov erz ea lo u s treatm en t that resu lt in blood PA levels below
catabolism.
n orm al sh ou ld be a v oid ed b eca u se this can 1/t p o o r g ro w th & n eu ro lo g ica l sy m p to m s
- The diagnosis is confirmed
In P K U , ty ro sin e c a n n o t b e sy n th esiz e d fro m P A , and th erefo re it b e c o m e s a n e s se n tia l
through quantitative
a m in o a c id th at m u st b e su p p lie d in diet.
determination of PA levels.
T re a tm e n t m u st b eg in d u rin g th e first 7-1 0 d a y s o f life to prev en t m en tal retardation (as e a rly as
- Plasma phenylalanine level: Ted to
b etter).
15- 65mg% (N- 1- 2mg%)_________
762 ■ A Complete Review of Short Subjects

D isco n tin u a tio n o f P A re stricte d d iet b e fo re 8 y e a rs o f ag e is a sso c ia te d w ith p o o r • Guthrie rapid screening test uses
p erfo rm a n ce o n IQ test. Bacillus subtilis strains which
L ife lo n g re s tric tio n o f d ie ta ry P A is re c o m m e n d e d . In d iv id u a ls w ith P K U a re ad v ised to cannot grow in a medium devoid
a v o id a sp a rta m e , an a rtificia l sw e e tn e r th a t c o n ta in s p e n y la la n in e. of phenylalanine and the growth is
P la sm a p h e n y la la n in e c o n c e n tra tio n s sh o u ld b e m a in ta in ed b e tw e e n 2 an d 6 mg/dL (1 2 0 an d proportional to its content.
3 6 0 gm ol/L) w ith th erap y. A n d d ie ta ry p h e n y la la n in e restrictio n is u su a lly in stitu ted if b lo o d • FeCh test®: (older less reliable
p h e n y la la n in e lev els a re > 4 mg/dl (250 pm ol/L) test)
S o m e v e ry m ild c a ses o f P K U sh o w in crea sed to lera n ce to d ie tary p ro tein s & im p ro v e d - Screening test for presence of
m e ta b o lic c o n tro l w h en treated w ith tetra h y d ro b io p te r in (5-20 m g / k g / d a y ) w h ich is an phenylpyruvate in urine
esse n tia l c o fa c to r o f p h e n y l h y d ro x y la se
- FeCL gives em erald green w ith
P re g n a n cy risk s (M a te rn a l P K U ) can b e m in im iz ed b y a ssu rin g strict p h e n y la la n in e
um ie in PKU®.
re stric tio n s 2 m o n t h p r i o r t o c o n c e p t io n & th r o u g h o u t g e s t a t i o n an d c o n tin u in g life lo n g PA
re stric te d d iets. M a te rn a l P K U 1/t in crea sed risk fo r co n g en ita l d e fects, m icro ce p h a ly ,
• Phenylalanine load test (iv) detects
c o n g e n ita l h e a rt a b n o rm a litie s, an d s e v e re m en ta l & g ro w th re ta rd a tio n a fte r b irth . PKU carriers.
P h e n y la la n in e is a terato g en .

Hyper Tyrosinemias
T e tra h y d ro b io p te rin (BH-t) N A D P+
* - a t e & - ic a c i d a r e s a m e e g p y r u v i c
P h e n y la la n in e h y d ro x ylase S 5 2 f ^
(4-m on o xyg enase), a m ixed function K a c id = p y r u v a t e
oxid ase in liv er 7 V . N A D PH
★ T y r o s i n e is c o n v e r t e d to D O P A
D ih y d ro b io p terin (BH 2) ''
(dihydroxy phenylalanine) b y
- F o rm m e la n in , c a te c h o la m in e s & th y ro id h o r m o n e tyrosinase ( o r tyrosine 3 -
monoxygenase) in m e l a n o b l a s t a n d
a - k e to g lu ta ra te
R ich n e r-H an h art tyrosine hydroxylase i n a d r e n a l
ocu lo cu tan eo u s T y ro sin e ( -a ketoglutarae) tra n sa m in a se m e d u l l a . B o t h a r e d i f f e r e n t , s o in
T y p e I I ty ro sin em ia (am in o tran sfe rase ) requ ire PP (B t) & v ita m in C
ty r o s in a s e d e fic ie n t a lb in is m ,

G lu ta m ic ac id
e p i n e p h r i n e s y n t h e s i s is n o r m a l .
T y r o s in e h y d r o x y la s e r e q u ir e s
Para H ydroxy p h en y l p yru vic acid (pH PPA ) N A D P H & B H 4 ( s i m i l a r to
p h e n y la la n in e h y d r o x y la s e ).
T y p e II I N eo n atal
C u con tain in g
ty ro sin e m ia ;
p H P P A - o xid ase / d io xyg en ase
H ered itary ;
/ hy d ro xy lase, requ ire vitam in C & Bn
H aw k in su ria

H om ogen tisic Acid


(2, 5-dih ydroxy p h en y l actic acid)

F e con tain in g
A lk ap to n u ria H o m o g en tisate oxid ase
(o r 1-2 d io xyg en ase), V itam in C

(4) M a le y l - A ceto; cetate (M A A )

MAA
lso m e ra s e , req u ire G lutathione

Fum aryl A cetoacetate

T y p e I H ep ato ren al F u m ary l -a ce to a c e ta te


ty ro sin e m ia h y d ro xylase
(T y ro sin o sis) >

Fum arate A ceto -acetate


(G lu co g en ic)

A c e ta te A cetyl C oA (K etog en ic)


Biochemistry: Metabolism of Proteins and Amino Acids ■ 763

Type Type I (Tyrosinosis) Type II (Richner Neonatal (Type Hereditary Hawkins-inuria


Hepato renal Hanhart III)
syndrome)
Oculocutaneous
Features
Enzyme defect Fum aryl aceto aceta te Tyrosine am ino pHPPA pHPPA- oxidase PHPPA oxidase
hydrolase® transferase hydroxylase
Malelyl acetoacetate (transaminase)® pH PAA = para Hydroxy phenyl pyruvic acid = p-OH-phenyl
isomerase (+) pyruvate = 4-OH-phenyl pyruvate
Inheritance AR AR AR AR AD
Clinical Feature - Manifest in first 6 - Ocular: painful -Transient Biochemically similar to -Transient failure
months of life (acute corneal erosions relative type III but differs in that it to thrive
form) & cause death with photophobia deficiency of is -Metabolic
rapidly d/t liver failure - Cutaneous: enzyme d/t - More aggressive & severe acidosis in
- Cabbage like odor, palmo-plantar delayed - Not suppressed by infancy
hypoglycemia, keratosis maturation or vitamin C - Swimming p o o l
vomiting, diarrhea, - Mental vitamin C - pHP lactic acid excretion is odour®
failure to thrive, rickets. retardation deficiency more
- Major organs affected (mild to -Transient - Death occurs before 6
are liver, kidney & moderate) hypertyrosine months but those who
peripheral nerves causing mia with survive later on develop
cirrhosis, liver failure, normal liver 1. Renal tubular absorption
h epato cellular function defects like aminoacid
carcinoma® (in chronic -Occasional /protein uria, hyper
type when children mental delay phosphaturia
survve beyond 2-10 2 . Hypophosphatemic
years); renal tubular rickets, hypoglycemia
acidosis and peripheral 3. Nodular cirrhosis, hepato
neuropathy cellular carcinoma
Urinarly - Fumaryl acetoacetate & its Tyrosine & its - pHPPA and its - Same with more pHPLA Hawkinsin a
excretion metabolite succinyl metabolites metabolites derivative of
acetone N-acetyl tyrosine & pHPLA and pHPPA
- pHPPA and pHP lactic tyramine pHP acetic
acid (PHPLA) acid
- Tyrosine - Tyrosine, N-
acetal tyrosine,
tyramine
Plasma tyrosine 6 -12 mg/dL 4 -5 mg/dL - - -
Treatment - Dietary restriction of protein especially phenylalanine & tyrosine
- Substrate restriction using nitisinone in tyrosinemia type I
- Vitamin C (ascorbic acid) in type III neonatal tyrosinemia

Alkaptonuria / Alcaptonuria

Etio-pathology Clinical presentation Management


It is autosomal recessive (AR) inborn error (hereditary defect) in • Mild condition - Diets low in protein
catabolism (degradation ) of phenyl alanine & tyrosine in which compatible with fairly especially in
Garrod's classic ideas concerning heretable metabolic disorder was normal life. Patients are phenylalamine (< 500
proposed. Other disorders of Garrod's tetrad are albinism, usually asymptomatic mg/day) & tyrosine
cystinuria & pentosuria. until about 40 years o f reduce homogentisic
It occurs d/t deficiency of Fe containing liver enzyme age®. However dark acid levels & pigment
homogentisic (homogentisate) oxidase also k/a homogentistate 1, staining of diapers may deposition (Lippincott)
2-dioxygenase responsible for cleavage of aromatic benzene ring of sometimes indicate the - Ascorbic acid (vit.C)
homogentisic acid resulting in formation of 4-maleyl - disease in infants but may prevent oxidation
acetoacetate. usually no symptoms of homogentisic acid.
With lack of this enzyme, levels of homogentisic acid increase in blood, are present until later - Nitisinone [2-(2-Nitro 4-
and spill over into urine and tissue. It is oxidized by polyphenol in life. trifluoromethyl benzyl)
oxidase to benzoquinone acetate, which polymerizes and binds to • Most striking clinical 1, 3- Cyclo hexanedione],
connective tissue 1/t pigmentation (ochronosis) & arthritis. feature is blacken in g o f a drug used in
Vitamin C deficiency (in premature infants) and experimental urine on standing® tyrosenemia type I
764 ■ A Complete Review of Short Subjects

application of a-a'-dipyridyl-chelating agent of Fe may produce which is also the only reduces excrection of
alkaptonuria. sign of disorder in homogentisic acid in
children. As other urine and might prevent
Homogentisic Acid Fe containing liver features develop after long term complications of
enzyme Homogentisic or 30-40 years age. alkaptonuria, in
Polyphenol Oxidation_
4- M aleyl • Classical triad in conjunction with a low
oxidase
Homogentisate oxidase Acetoacetate includes-"HOA" protein diet.
Benzoquinone acetate (or 1-2 dioxygenase)

I------------------
Homogentisic aciduria Ochronosis Arthritis
•Patient's urinale contain high Deposition & accumulation of oxidized Ochronosis also involves joint
levels of homogentisic acid which homogentisic acid (benzoquinone acetate) cartilage 1/t arthritis which may
is oxidized to dark (black) pigment in cartilage and collagenous be severely crippling. It
pigment on standing or connective tissue 1/t ochronosis. This results - Starts with pain, stiffness &
exposure to air (sun light and in black (gray-blue- brown), pigmentation limitation of movements of
oxygen). and (generalized) darkening of_____________ large joints.
• Urine turns b la ck on standing® Sclera b/w cornea & canthii® - It involves large joints of
or air exposure when it also Ear cartilage (concha, anthelix & fnally spine (intervertebral disc)
becomes alkaline. (Alkaptonuria helix) followed by hip, knee &
meaning to suck up oxygen Nose cartilage shoulder.
greedily in alkali) Tympanic membrane, larynx, skin, and - It resembles rheumatoid
■When urine is kept in testube heart valves. arthritis b/o acute
darkening starts from surface and Skin (pigmentation) involving excerbations, but unlike RA
involves entire sample with in p alm op lan ta r area®, dorsum of hand & small joints of hands & feet
few hours. feet, face & neck are spared.
B enedict's test is strongly Intervertebral disc & large joints® - Black pigment deposition at
positive® and so is Ferric Pigmented renal and prostatic calculi (stones) intervertebral disc f/b
chloride (FeCla) test develop occasionally calcification. Radiological
■Urine organic acid analysis or D /1 pigmentation, valvulitis & calcification findings resemble
specific calorimetric test detect (of mitral & aortic) heart valves, the osteoarthritis with
homogentisic acid in urine. incidence of degenerative heart disease & characteristic narrow ing o f
MI increases. jo in t sp aces & ca lcifica tio n o f
in tervertebral disc®.

Maple Syrup Urine Disease (MSUD) or Branched Chain Ketonuria

Biochemical Defect Clinical Features, Types & Management


Reaction Aminoacids Enzyme • Feeding problems, vomiting, dehydration, severe
involved metabolic acidosis and characteristic sweet maple syrup or
Oxidative Leucine Branched chain burnt sugar od o r o f urine®, which gives disease its name. If
decarboxylati (especially), a-ketoacid untreated may 1/t mental retardation, physical disability &
on isoleucine, valine dehydrogenase death
- Autosomal recessive (AR) disorder in which there is deficiency • Variants of MSUD are
(partial or complete) in branched chain a-keto acid - Classic type is most common & most svere, show
dehydrogenase (decarboxylase) enzyme complex. This symptoms with in first few days of life and lethal in first
enzyme is responsible for o x id ativ e d ecarbox y lation o f weeks of life. Leukocytes or cultured skin fibroblasts of patients
branched chain a - k eto acids - leucine, isoleu cin e and valine®. show little or no enzyme activity.
- In MSUD, defect or deficient decarboxylation of leucine, - Intermediate & intermittent types have higher (3-15% of
isoleucine & valine d/t deficiency of branched chain a- normal) enzyme activity. So symptoms are milder & of late
ketoacid dehydrogenase enzyme leads to accumulation of onset (from infancy to adulthood)
these aminoacids and their corresponding a-keto acids (a-keto - Thiamin dependent M SUD achieve increased activity of
isocaproic acid, a-keto- fi-methyl valeric acid, a-ketoisovaleric acid) enzyme if given larger doses of thiamin.
and a-hydroxy acids (reduced a- ketoacids) in blood (plasma or • Early prenatal diagnosis (by enzyme analysis in cell),
serum), CSF and urine. Toxic effects interfere with brain neonatal screening and life long dietary restriction is
functions. essential if the child with MSDU is to develop normal.
- Unlike other ketoacidosis, Rothera's test is positive even in Treatment entails strict control of diet, restricting the intake
boiled & cooled urine. of valine, isoleucine and leucine to the minimum required to
permit normal growth.
Biochemistry: Metabolism of Proteins and Amino Acids ■ 765

★Hartnup disease is d/t defect in membrane transport mechanism of monoamino mono carboxylic (neutral) amino acids such
as tryptophan by small intestinal mucosa & proximal renal tubule. Indole derivatives of unabsorbed tryptophan formed by
intestinal bacteria are excreted. Large losses d/t intestinal malabsorption and failure of renal reabsorption limits tryptophan
availability for niacin biosynthesis and accounts for pellagia like features.
★ GM 1 gangliosidosis is d/t defect in activity of lysosomal enzyme galactosidase resulting in pathological accumulation of
GM1 gangliosides in lysosomes.

Catabolism of Branched Chain Aminoacids (BCAA)


- BCAA, leucine, isoleucine & valine are essential Leucine Valine Isoleucine
aminoacids. In contrast to most other aminoacis
these are metabolized (oxidized) as fuels primarily Transamination: B6 (PP) d e p e n d e n t (B ra n c h e d c h a in a m in o a c id
a m i n o t r a n s f e r a s e p r e s e n t i n m u s c l e , f a t , k i d n e y & b r a i n b u t a b s e n t in li v e r ) .
by peripheral tissues mainy muscles, adipose,
kidney and brain rather than by liver. Because
a -k e to is o a -k e to is o a -k e to - P -m eth y l Valeric
these tissues contain aminotransferase, absent in Caproic acid Valeric acid acid
liver, that acts on BCAA & produce corresponding I I I
a-ketoacids. Oxidative decarboxylation: 5 c o f a c t o r s a r e TPP, NAD, FAD, CoA, lipoic
- Branched chain a-ketoacid dehydrogenase acid
enzyme catalyzes oxidative decarboxylation of all (Branched chain
. a-ketoacid- dehydrogenase) ^
three a-ketoacids produce their acyl CoA
Isovaleryl CoA
T i
derivatives & release C 0 2. This reaction is analogus Is o b u ty ry l C o A a -M e th y l b u ty ry l C o A

to oxidative decarboxylation caused by pyruvate I I


dehydrogenase complex & a-ketoglutarate FAD linked Dehydrogenation
dehydrogenase as all 3 enzymes are similar in
structure, share same mechanism and cofactors - 3 - M e th y l C r o to n y l
C oA
TPP, NAD, FAD, Lipoate & CoA. Defect in this
step 1/t maple syrup urine disease.
,,Biotin
- FAD linked dehydrogenation (analogous to acyl [Acetyl CoA|
CoA dehydrogenase of P-oxidation yields a-P- 3 -M e th y l g lu ta c o n y l Propionyl CoA K e to g e n ic
C oA
unsaturated acy CoA derivatives. Defective
Biotin
isovaleryl CoA dehydrogenase is a/w seaty feet
odor in body fluids) H M G C oA Methylmalonyl CoA
- M ethyl malonyl CoA mutase (isomerase) enzyme
and coenzyme vitamin B22 (adenosyl cobalamine)
rearrange MMCoA to succinyl CoA. Deficiency of Acetyl CoA MMCoA Mutase
MMCoA mutase or Bi2 result in methyl malonic + (Ado B,2)
academia & aciduria, in which both propionate & Acetoacetate
' I
methyl malonate are excreted in urine. Patient Ketogenic Succinyl CoA
presents with metabolic acidosis, developmental Glucogenic
retardation, symptoms resembling ethylene glycol BCAA Disease Defective Enzyme
poisoning and death. MSUD (BC Ketonuria) BCKA Dehydrogenase
- Leucine is ketogenic (yields acetyl CoA & Intermittent branched chain ketonuria BCKAD (less severe)
acetoacetate); isoleucine is both ketogenic & Isovaleric acidemia Isovaleryl CoA dehydrogenase
glucogenic (yields acetyl CoA & succinyl CoA) and
Hypervalinemia Valine transaminase
valine is glucogenic (yield succinyl CoA)._________

Amphibolic Intermediates formed from the Carbon Skeletons of Amio Acid


766 ■ A Complete Review of Short Subjects

Comparison between Hb & Myoglobin

Similarities Differences
I I
Single polypeptide Hemoglobin M yoglobin
chain of myoglobin is -Tetram eric structure<2, consists of 4 polypeptide chains - Monomeric structure
structurally similar to -Can bind 4 O2 m olecu le 6 with one polypeptide chain
individual -Hb-C>2dissociation curve is sigm oid sh a p ed <2. - Combines with one m ole
polypeptide chains of -P ositiv e Co- op erativ ity (or C o-op erativ e binding o f 0 2. Q
Hb. kin eticsQ: binding of heme with O 2 facilitates binding - Myoglobin -O 2
Polypeptide chain of of other O2 molecule, a property that permits it to bind dissociation curve is —»
Hb & Mb has stretches a maximal quantity of O 2 at the respiratory organ & to R ectangular hy p erbola. 0
of a - h elica l structure deliver a maximal quantity of O 2 at peripheral tissue.
& a hem e binding Carbon monoxide (CO) binds to isolated heme about 25000 times more strongly than does
pocketsQ. oxygen but to myoglobine only 200 times. CO binds to
Polypetides contain
mostly nonpolar amino
acids internally, these Hemoglobin M yoglobin
non- polar a.a forming CO binds to isolated heme in its preferred Presence of d ista l E-7 h istid in eQ. Hinders
a structure stabilized by orientation, i.e, with all three atoms (Fe, C, binding or CO at 180° angle (Preferred angle)
hydrophobic interaction O) perpendicular to the plane of heme & forces it to bind in a less favoured
The heme group sits in configuration and reduce the strength of
a cervice, lined with 0 heme - CO bond.
X) III
non- polar a.a with
0 C
exception of two 1 1
histidine residue Fe Fe i o c
I
(Proximal & distal).
Thus hem e lies in the Preferred angle for bonding of O 2 & CO
hy drophobic p o c k et in Angles for bonding of O 2 & CO to heme iron
both Hb & M b0. of m yoglobin

Hemoglobin performs two major transport 2 ,3 - Di/BI-Phosphoglycerate (2 ,3 - PPG)


functions
1. Transport of Ozfrom lungs to peripheral tissues. I------------
Binding with Hb Feature
2. Transport 0/CO 2 and protons (H+) from the
peripheral to the lungs. One molecule of BPG is - One m ole o f deoxyhem oglobin binds one
bound per Hb tetramer m ole o f 2, 3 DPGQ.
I------------ in the central cavity - 2,3- DPG binds to deoxygenated (T) form
In Lungs In Tissues formed by all four of Hb & binding of O 2 to Hb decreases
Binding of O 2 to Binding of protons subunits. This cavity is the binding of 2,3- DPG
deoxyhemoglobin cause to Hb causes release sufficient for BPG, only - 2,3- DPG decreases the affinity of Hb
release of protons (H+). & of O 2. when the Hb is in the T molecule for O 2 thus increase in DPG
this is responsible for the I form (deoxy genated conc. shifts O2- Hb dissociation curve to
Bohr effect form) right.
In tissue increase in
I BPG forms salt bridges - Factors that affects conc. of 2,3 DPG in
H* cone (i.e. JpH ),
Released H+ combine with the terminal RBC
decreases the
with HCCb', forming affin ity o f aminogroups of both (J-
carbonic acid. Which is hem oglobin fo r O 2 Q. chains via Val N A 1 and Ted 2,3- DPG l e d 2,3- D PG
broken down into CO 2 & Lys EF6 and His H21. - Hypoxia - F etal HbQ
H2O and CO 2 is exhaled BPG stabilizes the - GH - In bank blood
out. deoxygenated form (T) - Exercise that is stored—»
of Hb that must be - T3, T4 less O 2 is
broken prior to - Anemia released to the
conversion to R state. - Altitude high tissues.
Biochemistry: Metabolism of Proteins and Amino Acids ■ 767

Porphyria

Porophyrias: Are group of disorders due to abnormalities in the pathway of biosynthesis of heme, resulting in accumulation &
increased excretion of porphyrins or its precursors. Depending on whether enzyme deficiency occurs in Liver or bone marrow.

1. Hepatic Porphyria

Acute Intermittent Porphyria Varigate Porphyria Hereditary Coproporphyria

■D/t deficiency of uroporphyrinogen I - An acute disease caused by - An acute disease caused by


syn thase (H ydroxym ethyl bilan e synthase)® deficiency in p ro to deficiency in coproporphyrinogen
- Porphobilinogen & ALA accumulates in porphyrinogen ox id ase ® oxidase®
urine. - Protoporphyrinogen III & other - Coproporphyrinogen III accumulate
- Urine darkens on exposure to light and air intermediates prior to the block in the urine
accumulate in the urine - Patients are photosensitive
Patient are n ot p h oto sen sitiv e ®
- Patients are photosensitive
Abdominal pain & neuropsychiatric
symptoms

2. Erythropoietic porphyria

Erythropoietic Protoporphyria Congenital Erythropoietic Porphyria


D/t deficiency in ferrochelatase® D/t deficiency of uroporphyrinogen III synthase®
Protoporphyrins accumulates in erythrocytes, bone Uroporphyrinogen I & coproporphyrinogen I
marrow and plasma accumulates in urine
Patients are photosensitive Patients are photosensitive

3. Both

Porphyrea Cutanea Tarda Treatment


■Affects both liver and erythropoietic cells. - Medical support for pain & vomiting during acute attack
M ost com m on type®. - I. V hemin: decreases synthesis of ALA synthase
■Chronic disease caused by deficiency in - Avoidance of sunlight
uroporphyrinogen decarboxylase. ® Ingestion of (J- carotene (free radical scavanger)
Uroporphyrin accumulates in urine ★ All porphyries are AD disorder except congenital erythropoietic
Patients are photosensitive porphyria which is AR
★ Photosensitivity: patients skin itches & burns (pruritis) when
exposed to visible light (result of porphyrin mediated formation of
superoxide radicles from 0 2)
768 ■ A Complete Review of Short Subjects

Heme Synthesis

V V H em e
M itoch on d ria

Su ccinyl C oA + G lycin e E ry th ro p o ietic


Ferro ch elatase (F ei++)
++) mm
p rotop o rp h yria
A L A syn th ase P yrid oxal Fe2+^
ra te lim itin g p h osp h ate P ro to p o rp h y rin III
en zy m e in L iv er
Frotoporphyrinogen
a -A m in o P K etoad ip ate oxidase or light in vitro V arigate p o rp h yria
(rap id ly d ecarb oxylated ) 6H
P ro to p o rp h y rin o g en III
-phyrir
A L A syn th a se
C o p ro p o rp h y rin o g en H ered itary

5 -A m in o le v u lin ic A cid
(A LA )
oxidase
+ cop rop orp h y ria

C o p ro p o rp h y rin o g e n III

JY _ r \.
C yto so l 2 m o lecu les o f A LA
*
C o p ro p o rp h y rin o g en I I I
L ig ht C op ro
► p orp hy rin

A L A D ehydratase
6H HI
L e a d P o is o n in g
(Z n containing) I P o rp h yria cutan ea
tarda
U ro p o rp h y rin o g en
P o rp h o b ilin o g en (PBG )
d ecarb o xy lase (U P D )
( 1 st p re cu rso r o f p yrro le)

4 m olecu les of PB G in
h ead to tail m an n er
r 4C 02
U ro p o rp h y rin o g e n 1
s y n th ase or A c u te in t e r m it t e n t
PB G d eam in ase o r p o r p h y r ia
H M B syn th ase U ro p o rp h y rin o g en
III sy n th ase
6H
H y d ro x y m e th y lb ilan e (H M B)
sp o n tan eo u s
I
C o n g en ital eryth rop oietic
U ro p o rp h y rin o g en III - J -
Lig ht
U ro
p orp hy rin
III
6H cyclizatio n
p o rp h yria

U ro p o rp h yrin U ro p o rp h y ro n o g en I
L ig ht
4C 02A U PD
Light
C o p ro p o rp h y rin I C op ro p o rp h y rin o g en I

Site of heme biosynthesis are L iver & B one m arrow (erythropoietic cells)Q
Chapter 7. GENERAL BIOCHEMISTRY: REVIEW NOTES

Acids

Monoprotic Acids (HA) Polyprotic Acids


These donate 1 proton (H+) per
I-----------------------------
molecule
Diprotic Acids (H2A) Triprotic Acids (HjA)
Example include
• Can donate upto 2 protons per molecule • Can donate upto 3 protons per
- Mineral acids - HCl
(hydrochloric acid) & H N O 3 during process of ionization molecule during process of ionization
(Nitric acid)Q • Can undero 2 dissociations (ionizations) • Can undergo 3 ionization & have 3
- Monocarboxylic organic & have 2 dissociation constants, first dissociation constants (K1>K2>K3)
acids like fo rm ic acid typically greater than second (K1 > K2) • Examples include
(HCOOH), acetic acid • Examples include - Phosphoric acid (H3PO 4)
(CH 3 COOH) and benzoic - Sulfuric acid (H 2SO 4) - Citric acid 0
acidQ. - C arbonic acid Q (H 2CO 3)
Diprotic Acids Triprotic Acids
Monoprotic Acids
Carbonic pK, 6.37 Phosphoric pKi 2.15
Formic pK 3.75 10.25 pK 2 6.82
pK 2
Lactic PK 3.86 Succinic pK, 4.21 pKj 12.38
Acetic PK 4.76 5.64 Citric pKi 3.08
pK 2
Ammonium pK 9.25 Glutaric pK, 4.34 pK 2 4.74
ion
pK 2 5.41 pK 3 5.40

Different Types of Bonds

Covalent Bond Non covalent Interactions


- Strongest force While of lesser magnitude make significant contribution to the structure, stability and functional
that holds competence of macro molecules of living cells.
molecules I I 1
together Electrostatic Bonds Hydrogen Hydrophobic Vandar
- E lectron have /Salt Bridges Bond Interactions Wall Forces
op p o site spinQ - Occurs between -It is the electrostatic interaction •A tendency of - Arise from
- Each individual oppositely between the hydrogen nucleus of nonpolar compounds attraction
DNA strand is charged groups one water molecule and unshared to self associate in an between transient
held together by with in or electron pair of another aqueous environment. dipoles generated
covalent bonds; between -Compared to covalent bonds, these This self association is by rapid
but the two molecules are quite weak & requires ~ 4.5 k neither attraction nor movements of
strands of helix - Comparable in cal /mole energy to break it (about bond (so hydrophobic electron on all
are held together strength to 4% of energy) required to rupture bond is misnomer). neutral atoms
by noncovalent hydrogen bond the O-H bond (110 k cal/mol) It arises from the need -Significantly
interactions (eg. but act over larger to minimize weaker than
- Influences the physical properties of
Hydrogen bonds distances energetically hydrogen bonds
water and accounts for its
between exceptionally high viscosity, surface unfavourable but potentially
- Facilitate the
nucleotide bases tension and boiling point. interactions between extremely
binding o f
(Watson-Crick numerous.
charged -In solid state each water molecule is nonpolar groups and
base pairing) & water - Act over a very
molecules & ions associated with 4 other water
vandarwall short distances (2 -
to proteins and molecules and ~ 3.5 in liquid state Contribute stability to
forces.
nucleic acids. -Responsible for ice formation the interiors of 4A)
- Show phenomenon of co-operativity proteins.
770 ■ A Complete Review of Short Subjects

Glutathione

Reduced glutathione is tripeptide thiol (y- gluatamyl cystein glycine) consisting of 3 amino acids - glutamic acid, cysteine
and glycineQ. The glutamate is linked to cysteine through y - carboxyl linkage. And sulfhydryl (SH) or thiol group of
cysteine, which accounts for its strong electron donating (nucleophilic) character of glutathioneQ, is the business part of
molecule.
G -S-S-G
Reduced glutathione is important reducing agent in the tissue, and NA D PH +H + 2 H ,0
can chemically detoxify hydrogen peroxide. The reaction is
catabolized by selenium (Se) requiring glutathione peroxidaseQ HM P
V
(O xidized)

G lutathione
V
Glutathione
enzyme and form oxidized glutathione (glutathione di sulfide Shunt reductase peroxidase
GSSG) which no longer has protective properties. The cell
regenerates reduced glutathione (GSH) in a reaction catabolized 2G SH H2O2
2H + N AD P + (reduced)
by glutathione reductase, which contains FAD as prosthetic group
and uses NAD PH as a source of reducing electron^.
Liver is the largest GSH reservoir and because of its reducing property it has potent antioxidant action, protects from
oxidative stress conjugates Xenobiotics, acts as carrier and coenzymes & cofactorQ.

Xenobiotic metabolic Antioxidant: protect from oxidative A s a carrier y-glutamyl A s intracellular reductant
conjugation stress cycle
- Prevents oxidation of SH group
I I I
containing enzymes eg
- Nucleophilic GSH - Accumulation of H 2O 2 may It transport certain
glyceraldehyde-3-P-
conjugate with toxic decrease life span of RBC by amino acids across
dehydrogenase
electrophilic causing oxidative damage of cell membranes in kidney
- GSH & glutathione trans
xenobiotics such as membrane 1/t hemolysis.
Aminoacid + GSH hydrogenase cause reductive
carcinogens - R educed glu tathion e rem oves H 2O 2
—> y- Glutamyl cleavage of S-S linkage of
- Glutathione S in a reaction ca ta ly zed by
amino acid+ thyroglobulin glycoprotein
transferase present in g lu tathion e peroxidase® an enzyme
liver cytosol catalyze cyteinyl- glycine
that contains the selenium analogue
Acts as coenzyme / co factor
the reaction. o f cystein e (selenocysteine)® The above reaction I
- These conjugates are - In RBC, HMP shunt is the only catalyzed by y- glutamyl GSH acts as coenzyme with
ultimately excreted in pathway that produces NADPH for tranferase (GGT) glutathione transhydrogenase,
urine as mercpturic reduction of oxidized glutathion present in plasma formaldehyde dehydrogenase,
acid which is formed (catalyzed by glutathione membrane of renal glyoxylase, maleyl acetoacetate
by removing glutamyl reductase) tubular & bile ductular isomerase, cyclo- oxygenase (PG
& glycinyl group and - So G6PD deficien cy im pairs cells and in ER of
generation o f NADPH, and l/t synthatase system) and for liver
addind acetyl group. hepatocytes.
h em olytic an em ia ® when subjected enzyme for activation of
to antimalarial prim aquin, aspirin, methionine to form SAM.
su lfon am ides o r f a v a beans (vicia
fa v a - fa v is m ) Q
M easurem ent of erythrocyte transketolase and its activation by thiamine diphosphate is used to assess thiam ine nutritional
statusQ.

Oxido-Redutase
In biological systems, oxidation (loss of electron) is always accompanied by reduction of electron acceptor. Enzymes involved in
oxidation & reduction are called Oxidoredutase and are classified into four groups.
1
Oxidase Dehydrogenase Hydroperoxidase Oxygenase
Catalyze the removal of • Cannot use oxygen as a Hydroperoxidase p ro tec t the bod y Catalyze the direct
hydrogen from a substrate hydrogen acceptor. They ag ain st harm fu l peroxidases® (which transfer &
using oxygen as a hydrogen transfer H+ from one can generate free radicals). These are incorporation of 0 2
acceptor, forming H 20 2 or substrate to another, thus of 2 types into a substrate
HzO causing one substrate to be molecule
1. Catalase
Some oxidase contain Cu oxidized at the expense of
- use H 2O 2a s electron d on or & - Dioxygenase
and others are flavoproteins another
incorporate both
Biochemistry: General ■ 771

Cytochrome oxidase/aa3 is a Useful in enabling oxidative electron acceptor® & its function is atoms of molecular
hemoprotein containing 2 process to occur in the assumed to be the destruction of 0 2 into the substrate
atoms of Fe atom (that absence of 0 2 such as during H20 2Q formed by the action of ■Monooxygenases
oscillates between Fe2* and anaerobic phase of oxidase (mixed function
Fe3*) and 2 atoms of Cu. glycolysis and as - It is found in blood, bone marrow, oxidase hydroxylase)
components of respiratory mucous membrane, kidney & liver. incorporates only
Xanthine oxidase contain
chain one atom of
m olybdenu m Q; aldehyde 2. Peroxidase
dehydrogenase contain These depend on molecular O 2 into
reduces FfeC^ using various substrate
FAD, molebdenum & non nicotinamide coenzyme
electron acceptors such as
heme iron; L-amino acid (NAD+ or NADP+) or ■Cytochrome p450 are
ascorbate, quinones & cytochrome
oxidase contain FMN riboflavin (FMN) or FAD monooxygenase
C
Poisoned by CO cyanides Except for cytochrome important for
found in milk, leucocytes, platelets detoxification of
and H2SQ. oxidase, other cytochromes
and other tissues involved in many drugs & for
are classified as
eicosanoid metabolism hydroxylation of
dehydrogenase.
In erythrocytes g lu tath ion e steroids.
p ero x id ase con tain in g selenium Q
destroy H2O 2.
Superoxide dism u tase p rotects aerob ic organism s ag ain st oxygen tox icity (superoxide anion fre e ra d ica ls O2-)0

OJ OJ 2H+ “► H2O 2 + O 2
S u p e r o x id e
d is m u ta s e

Oxidative Stress
It is accumulation of oxygen derived free radicals due to inappropriate prooxidant - antioxidant balance.
I---------------------------------------- '------------------------------------
Pro-oxidant Antioxidants (AO)
• These are chemical compounds or reactions capable of generating potentially toxic reactive These are compound
oxygen species (ROS) or free radicals, which contain single unpaired electron in an outer orbit. and reactions that
These are produced by /in either block free radical
- Absorption of radiant energy (UV ray, X-ray, ionizing radiation) formation or inactivate
- Ischem ic - reperfusion injury0 induced by restoration of blood flow in ischemic tissue. (scavenge) their
action. These include
- Microbial killing by phagocytes and cellular aging.
antioxidants
- Enzymatic metabolism of exogenous chemicals or drugs (eg. C C I 4 generate C C I 3 )
- Reduction - oxidation reactions of normal metabolism.
- Transition metals such as iron (Fe) and copper (Cu) donate or accept free electrons during
intracellular reaction and catalyze free radical formation as in Fenton reaction (ferrous form is
used)
- Nitric oxide (NO), can act as free radical and can be converted to highly reactive peroxynitrite
anion (ONOO-) as well as N 0 2 and N O 3 - .

Naturally occurring AO According to action Used commercially


Vitamins : Beta carotene (a Preventive AO: They inhibit chain initiation and initial (in vitro) as food
type of vitamin A), C production of free radicals preservatives to
(ascorbic acid), E (alpha prevent lipid
- Catalases
toco p h ero l)Q. peroxidation (esp
- Glutathione peroxidase PUFA) & increase
Zinc (trace element),
- Chelators of metal ions like shelf life
selenium Q
1.Transferrin, ferritin, lectoferrin and ceruloplasm in are storage
Caffeine, epigallocatectin and transport proteins that minimize levels of reactive Fe and
gallate = ECGg (in green tea - Propyl gallate
Cu by binding them (and so OH- formation)
leaves); clorogenic & melanic (PG)
2.EDTA (ethylene diamine tetra acetate)
acids (coffee beans); - Butylated
curcuminoids (turmeric); 3.DTPA (diethylene triamine penta acetate) hydroxy anisole
772 ■ A Complete Review of Short Subjects

proanthocyanidines (grape Chain Breaking AO: They interfere with chain propogation (BHA)
seeds); polyphenols (apple); - Superoxide dismutase both cytosolic & mitochondrial - Butylated
flavonoid querectin (apple & hydroxy toluene
- Vitamin E (alpha tocopherol)® is most effective
onions); lycopen e (red (BHT)
tomato)®-, a-lipoic acid (yeast - B eta carotene® (a type of vitamin A)
- Vitamin E
& liver); and lutein & - Uric acid (urate)
Used as therapeutic
zeaxanthin (egg yolk) - Selenium containing g lu tath ion e peroxidase®
agents
- Phenols or aromatic amines
Dimethyl thio urea
Synzym e (M-40403) Scavanger of free radicals
Dimethyl sulfoxide
A synthetic enzyme that works - Superoxide dismutase (SOD), scavange superoxide anion ( o ’ )
Allopurinol
like body's own scavenger anti and produce H 2O2.
Vitamin C
oxidants (ex SOD) to mop up - Catalse scavange H ?0 2
Vitamin E
free radicals. This manganese -Glutathione peroxidase and reduced g lu tath ion e (GSH)®
based experimental product (for
(Se dependent scavenge H 2O 2)
treating stroke) is more powerful
- NADPH dependent Glutathione reductase
than traditional antioxidants
like vitamin E & C. - Vitamin C, uric acid and polyphenols obtained form plant
food act as water soluble free radical trapping AO.
- Ubiquinone & Beta-carotenes are lipid soluble radical
trapping antioxidants in membranes & plasma lipoproteins.

Free Radicals or Reactive Oxygen Species (ROS)

Free radicals or reactive oxygen species (ROS) are atoms or groups of atoms that have an unpaired electron and are powerful
oxidants. These include superoxide anion (O 2' ), hydrogen peroxide (H2O 2), peroxyl radicals (ROO-), hypochlorite (HOC1) and
very reactive hydroxyl radicals (OH-) that react with proteins, nucleic acid, lipid etc.

I
Reaction & enzyme generating free Reactions & enaymes scavenging free radicals
radicals (and causing damage) (protective)
_________________ I____________ I
- NADPH oxidase (produce superoxide
Superoxide dismutase (SOD), scavange superoxide anion ( 0 -, ) and
anion)®
- M yeloperoxidase (produce h y pohalou s produce H20 2.
acids) ® Catalse scavange HjOi
- Fenton and H aber - W eiss reaction Glutathione peroxidase (Se dependent scavenge H2O 2)
(produce hydroxyl radicals) ® NADPH dependent Glutathione reductase
- Xanthine oxidase

• Fenton reaction:

1Ff-’ + + H 7 O 7— > O H ° + O H - + Fe3+ I

• Heber - weiss reaction (iron catalysed):

O 2 + H 2O 2" OH°+OH + 0 2
Biochemistry: General ■ 773

H2 O2 - Myeloperoxidase (MPO) - Halide System


H20 2 - Myeloperoxidase (MPO) - Halide System is the m ost efficien ct b actericid al system in neutrophils®. It has 3
components.
NADPH - oxidase Halide & Myeloperoxidase (MPO)
- Its activation (when phagocytic cell, engulf bacteria) is - H20 2 generated by NADPH oxidase system (after
associated with a rapid increase in 0 2 consumption and metabolism spontaneous dismutation) is not able to efficiently kill
(the respiratoryu burst) and generation of superoxide (O f ) anion microbes by itself. This is compensated by enzyme
by following reaction.____________________________ myeloperoxidase present in azurophilic granules of
meutriophils.
NADPH + 2 0 2 -» NADP + H+ + 0 2 - MPO is responsible for green colour of pus and act on
- There is only a lag of 15-60 seconds in phenomenon of H 20 2 in presence of halide to produce hypophaloius
respiratory burst. NADPH is mainly generated by pentose acids.
phosphate pathway, whose activity increases markedly during - Cl- is the halide usually employed as it is present in
phagocytosis. Electron transport chain system - NADPH high concentration in plasma & body fluids. And
oxidase is composed of a hetrodimer polypeptide (of 91 KDa + hypochlorite (HOC1 or bleech) which is a powerful
22KDa) cytochrome b558, located in plasma membrane along oxidant and is highly microbicidal is usually produced.
with two cytoplasmic polypeptides of 47 and 67 KDa H 2O 2 + H+ + X- Myeloperoxidase^ HOX + H20
- Two molecules of superoxide anion spontaneously produce
(X- = C1-, Br-, I- or SCN-; HOX = HOC1 etc.)
hydrogen peroxide (H 20 2) by a process of spontaneous
- In normal tissue, HOC1 combines with amine and form
dismutation.
chloramines, which are less powerful oxidants and act as
O ’ + 0 ’2 + 2 H+ -+ H20 2 + 0 2 microbicidal agents without causing tissue damage.

- Superoxide dismutases tremenduously speed up the above reaction. This group includes both copper-zinc superoxide
dismutase found in cytosol and manganese superoxide dismutase localized in mitochindria.
* Superoxide dismutase=SOD (present in cytosol & mitochondria), catalse (present in peroxisome) and glutathione
peroxidase (present in cytosol & mitochondria) protects cells from oxidative stress and damage in following way.
2 Superoxide anion ( 0 2 ~) + 2H+ SOD a H 20 2 + 0 2

Group 1: Group 2: Major Group 3: Trace or (Minor Group 4: Additional Group 5: Non-
Components of (Principal or Macro) or Micro) Elements / Trace Elements essential or Toxic
organic macro Elements /Minerals Minerals Are possibly Element
molecules Are nutrilionally These are essential and essential (necessary) No known role and
(carbohydrate, important and required in <100 mg/day but exact function is enter body through
lipid, protein) requirement > 100 amount. Dificiency can unknown pollution or
which form 90% mg/day The deficiency 1/t serious disorders - Barium (Ba) contaminants.
of solid matter can prove fatal. The 7 - Iron (Fe)® - Bromine (B) Non-essential
- Carbon (C) macrominerals - Iodine (I) - Cadmium (Cd) elements
- Hydrogen (H) constitute 60-80% of - Copper (Cu) - Nickel (Ni) - Silver (Ag)
- Oxygen (O) body's inorganic matter - Cobalt (Co) - Lithium (Li) - Gold (Au)
- Nitrogen (N) - Calcium (Ca) - Chromium (Cr) - Silicon (Si) - Bismuth (Bi)
- Magnesium (Mg) - M anganese (Mn)® - Sn - Rubidium (Ru)
- Phosphorus (P) - Molybdenum(Mo) - Vanadium (Vn) Toxic Elements
- Sodium (Na)® - Fluoride (F) - Chromium (Cr) - L ead (Pb)®
- Potassium (K) - Selenium (Se) - Mercury (Hg)
- Chloride (Cl) - Zinc (Zn) - Aluminium (Al)
- Sulfur (S) ★ Cr & Cd are included - Cyanide (CN )
in different groups in - Arsenic (As)
different books. - Boron (Bo)
- Cadmium (Cd)
774 ■ A Complete Review of Short Subjects

Vitamin

Water- Soluble Vitamins Fat-Soluble Vitamins

Differs from fat soluble vitamins in following ways • Vitamin A (retinol, (3


- Readily excreted once their concentration surpasses the threshold, so toxicities are rare carotenes)
- Deficiency is quick on an inadequate diet as their metabolic stores are labile & • Vitamin D (cholecalciferol)
depletion occurs in few weeks or months • Vitamin K i.e.
- Are coenzymes so status can be assayed by measuring enzyme activities - Ki = Phylloquinone (in green
vegetables)
Vitamin B complexQ Vitam in C - K 2 = Menaquinone
(ascorbic acidQ) (synthesized by intestinal
I------------------ 1
bacteria)
Energy releasing Haemopoietic Other
- K 3 = Menodione (synthetic
Vitamin Bi (thiamine) - F olic acid® - Vitamin B 6
compounds that can be
Vitamin B 2 (riboflavin) - Vitamin B 12 (pyridoxine)
metabolized to
Vitamin Ba (niacin) (Cobalamine) - Pyridoxal
phylloquinones)
Biotin - Pyridoxamine
» Vitamin E (tocopherols)_____
Pantothenic acid

Vitamin B3 (Niacin)
NADP+ - Cofactor NAD+ - Cofactor
- Glucose- 6 -phosphate dehydrogenase (G-6 -PD (glucose-
M alate dehydrogenase (malate —>oxaloacetate)
6-PO4 to 6 -phosphogluconate)
Alcohol dehydrogenase (ethanol —> acetaldehyde
- G lutathione reductase
Lactate dehydrogenase (pyruvic acid —» lactic acid)
Either NADP+ or NAD+ - Cofactor Glvceraldehvde-3-phosphate dehydrogenase
- G lutamate dehydrogenase (glutamate —» a-ketoglutarate (glyceraldehyde 3P —»1, 3 diphosphoglycerate)
+ NH3) q-Glvcero-P-dehvdrogenase
- Isocitrate dehydrogenase (isocitrate —> oxalosuccinate) a-Ketoglutarate dehydrogenase (a-ketoglutarate —> succinyl
Mn- "Glu3 Is M AL Gly Kit/Pit-PDC" CoA)
All are dehydrogenase except glutathione reductase Pyruvate dehydrogenase complex (PDC) (pyruvate —>acetyl
sounds like "Glue in small gly pit/Kit" CoA)

Vitamin Bj (Thiamin) or Aneurine (i.e. relieves neuritis)


Chemistry and Mechanism Deficiency
■Thiamin is a basic substance that has a thiazole ring, pyrimidine & •Thiamin deficiency can 1/t 3 distinct
sulphur (S). syndromes
•Thiamin pyrophosphate (TPP) also k/a thiamin diphosphate (TDP) 1. Chronic peripheral neuritis
is biologically active & storage form of vitamin Bl and is formed by 2. Beriberi (Singhalese word meaning
transfer of pyrophosphate group from ATP________ weakness) occurs in areas where polished rice
ATP AM P (lacking aleurone layer) is major diet. It is
Thiamin - -»TPP (TDP) characterized by dry skin, irritability,
TPPTransferase
disordered thinking, progressive paralysis
Thiazolium ring, the functional part of TPP, has a relatively acidic d/t peripheral neuritis in adults
proton at C-2. Loss of this acidic (C2) proton produces a carbanion, - Acute pernicious (fulminating) beri beri (or
which is the active species in TPP dependent reactions. In other shoshin beri beri) is c/b high out put cardiac
words, C2 in thiazolium ring of TPP ionizes rapidly, thus producing active failure, edema and metabolic abnormalities
species carbanion. Carbanion readily adds to carbonyl groups (eg without peripheral neuritis. (Wet beri beri)
pyruvate) and thiazolium ring of TPP stabilizes carbanion - Dry beri beri : Peripheral neuritis & CNS
intermediate by providing an electron deficient (electrophilic) manifestations predominate
structure into which the carbanion electrons can be delocalized by - Infantile beriberi occurs in infants born to
resonance. This "electron-sink" action of thiazolium ring facilitate thiamin deficient mothers and show
carbon-carbon bond cleavage adjacent to carbonyl group in reactions like tachycardia, vomiting, convulsions, death.
decarboxylation of a-ketoacids (eliminating CO 2) and in chemical 3. Wernicke encephalopathy with Korsakoff
rearrangements in which an activated acetaldehyde group is transferred psychosis (W-K syndrome) is a/w thiamin
from one carbon atom to another.
Biochemistry: General ■ 775

Physiological Role deficiency in chronic alcoholism ® & narcotic


abuse. It is c/b ap ath y , atax ia, am m esia,
- Thiamin has a central role in energy yielding metabolism and nystagm us & opthalm oplegia®
especially of carbohydrates. So thiam in requirem ents in crease in •Lactic and pyruvic acidosis
excess in ta ke o f carb oh y d rates and its deficien cy lead s to decreased High carbohydrate diet in thiamin deficient
energy production® subjects result in increased level of pyruvic
- TDP (TPP) is coenzyme for oxidative decarboxylation and
acid or pyruvate (d/t PDH dysfunction)
transketolation reactions such as
which is then converted to lactic acid thus
I. Oxidative Decarboxylation of a-Ketoacids causing life threatening lactic acidosis.
3 Multienzyme complexes that catalyze oxidative decarboxylation Abnormal blood LA/PA ratio is a specific
are pyruvate dehydrogenase/PDH (in carbohydrate metabolism indicator
synthesizing Acetyl CoA from pyruvate); a-ketoglutarate *Decreased energy production because acetyl
dehydrogenase (forming succinyl CoA from a-KG, in citric acid CoA is not formed (d/t PDH dysfunction)
cycle); and bran ched chain k e to a c id dehydrogenase /BCKD (in and citric acid cycle is hampered (d/1 a-K G D
metabolism of leucine, isoleucine & valine). dysfunction)
>Accumulation of pentose (5C) sugar in RBC
d/t retardation of transketolase
Pyruvate Pyruvate D ehydrogenase (PD H) ^ A ce ty lC o A + C O 2
>B ranched chain ketonu ria (M aple syrup urine
TPP, FA D , Lipoate, CoA, NAD* ^ disease)® variant (d/t retardation of BCKD
NADH enzyme)
a-K G D ehydrogenase (Ca** activator) SucdnvlCoA >Bracken disease, chastek paralysis,
a-Ketoglutarate-------------------------------------------y> + CO2
x opisthotonus, catatorulin effect (i.e. decreased
TPP, FAD, Lipoate, CoA, N AD* \
O2 uptake by B1 deficient brain) and
NADH
saturation effect (i.e. low urinary excretion
after test dose in thiamin deficient) in animals.
a-K etoisocaproic Branched chain a-ketoacid
►I s o v a l e r y l C o A
acid (Leucine) dehydrogenase e n z y m e
a-K etoisovaleric
C o e n z y m e s : TPP, FAD, ► Iso b u ty r y l + CO2+ NADH
acid (v a lin e )
N A D , Lipoate (Lipoic C oA
a-K eto P-methyl acid), CoA
a -M e th y l __
valeric acid
b u ty ry l C o A
(Isoleucine) Diagnosis

II. Formation and degradation of oc-ketols by Thiamin (Bl) deficiency is best diagnosed by
transketolase (transferring 2 carbons in PPP/HMP shunt) observing an increase in RBC tran sketo lase
T ransketolase TPP G lyceraldehyde- activity® on ad d ition o f thiam in
Ribose-5-PC>4(5C)
>3-PO 4 (3C) p y ro p h osp h a te (TPP)®

Xylulose-5PO i(5C) Sedoheptulose


7-PO , (7C)
III. Reduction of pyruvate to ethanol (in yeast but not in
humans): Fermentation /Non oxidative decarboxylation
Pyruvate decarboxylase ^ E th an0, (alcohol) + C q 2
Pyruvate
TPP

Biotin (Vitamin B7)

Biotin, carboxybiotin & biocytin (e- Coenzyme in Carboxylation reaction (by Carboxylase enzyme)
aminobiotinyllysine which require proteolysis • Coenzyme biotin serves as a carrier of activated carbon dioxide in
to release biotin) is widely distributed in biotin dependent carboxylation reactions. Biotin covalently binds to e-
many foods and synthesized by intestinal flora in amino groups of lysine residues of biotin dependent carboxylase
excess of requirement. So dietary deficiency is (apo) enzymes to form biocytin residue of holoenzyme. This
unknown except among people conversion is catalyzed by holocarboxylase synthetase. 1-N-
Maintained on total parenteral nutrition for carboxybiocytin formed from C 0 2 in ATP dependent reaction is
many months the reactive enzyme-biotin-CC>2intermediate, which transfers
Eat abnormally large amount of (>20) raw carboxy group to substrate.
eggs per day. Uncooked egg white contains
776 ■ A Complete Review of Short Subjects

glycoprotein avidin that tightly binds biotin » Biotin dependent 4 carboxylase enzymes are___________
and prevents its absorption from intestine. - A cetyl CoA carbox y lase® : catalyzes Acetyl CoA -»
- Multiple carboxylase deficiency, results from Malonyl CoA
defect in ability to link or remove biotin from - P yruvate carboxylase® : Pyruvic acid —» Oxaloaceticacid.
carboxylases - P ropion yl CoA Carboxylase® : Propionyl CoA —> Methyl
- Congenital holocarboxylase synthetase deficiency malonyl CoA
• Deficiency of biotin 1/1 anorexia, nausea, - 3 Methyl-crotonyl CoA carboxylase: 3 methyl crotonyl
glossitis, dermatitis, spectacle eyed CoA —>3 Methyl glutaconyl CoA_________________________
appearance (d/1 circumocular alopecia), and • Carbamoyl phosphate synthetase (of urea & pyrimidine synthesis),
Leiner's disease (i.e. exfoliative dermatitis or malic enzyme (converting pyruvate to malate) and addition of CO 2
erythroderma desquamativum in young to form C 6 in purine ring are some biotin independent
infants) carboxylation reactions.

Pantothenic Acid (Vitamin B5)

Biochemistry - It is a p art o f coenzym e A (such as acety l CoA, succinyl


CoA, fa tt y acy l CoA, HMG CoA) and acy l carrier protein
- Pantothenic acid consist of peptide linked P-alanine &
(ACP) o f fa tt y acid synthase®
pantoic acid (dihydroxy-dimethyl butyric acid).
- It has a central role as a carrier of acyl group in
- Peptide linkage o f pantothenic acid to fi-m ercep toeth an ol metabolism. Active thiol (sulfhydryl-SH) group of
am ine (which is obtained from aminoacid cysteine) forms (1 - betamerceptoethanol amine in pantetheine functional moiety
phospho)-pantetheine. This process requires 2 ATP (or 1 A of CoA or ACP carries & transfer acyl groups as activated
TP & 1CTP) thiol esters.
- Terminal thiol or sulfhydryl (-SH) group of - Its deficiency is extremely rare as vitamin is widely
betamercaptoethanolamine in pantetheine functional moiety is the distributed (Pantos means every where). Burning foot
active site where acy l groups are carried®. syndrome, sleep disturbance & spectacle alopecia may (+)
- Coenzyme (Co) A is formed when pantoic acid moiety (of occur
pantetheine) is linked to adenosine 3’ phosphate by - Its metabolic roles are
pyrophosphate bridge (a process requiring 2ATP) T----------------------------- 1
- So CoA is a nucleotide and requires expenditure of 4 high Formation of acetyl Formation of • Lipid
energy bonds (ATP) in synthesis. It can be denoted as CoA- CoA which is utilized Succinyl CoA metabolism
SH to denote the active thiol site. in which is utilized - [3 oxidation of
- A cetylation in FA
reactions® - Heme - Synthesis of
- Acetyl choline (porphyrin) FA
formation synthesis • Activation of
- C holesterol - Ketolysis branched
(—asteroids), keton e (ketone body chain amino
bod y & fa tty acid degradation) by acid such as
synthesis® extrahepatic valine &
- Elongation of FA tissue leucine involve
- Synthesis of ether - TCA cycle, CoA
lipids eg detoxification • Formation of
plasmalogen & PAF - Gluconeogenesi adreno cortical
- TCA cycle, s, activation of hormones.
Detoxification acetoacetate
Biochemistry: General ■ 777

Vitamin B6 / Pyridoxine

• Six derivatives of pyridine have vitamin Physiological role


activity: pyridoxine (occurs > Bf, is important in am in o acid m etab olism fo r transam ination, deam in ation ,
primarily in plants); pyridoxal, decarb ox y lation and con d en sation Q.
pyridoxamine (in animal foods H is tid in e — P y rid o x a l > H is ta m in e
mainly) and their 5' phosphates.
T y r o s in e — P h o s p h a te (B 6) > D o p a m in e , N E , E p in e p h r in e
• B io log ically activ e coenzym e is
G lu ta m a te — D ependent > GABA
p y ridoxal 5'-phosphateQ which can
T r y p to p h a n D e c a r b o x y la tio n > S e r o to n in
be obtained for from all 3 precursors
O r n ith in e & M e th io in e — > S p e r m id in e , S p e r m in e (in v o lv e d
i.e. pyridoxine (alcohol), pyridoxal in D N A p a ck a g in g )
(aldehyde) & pyridoxamine (amine).
• 80% of body's total vitamin Be is - So defective allergic response & gastric acid secretion (d/t histamine);
pyridoxal (5') phosphate (PP) in deterioration of parkinson's disease (d/t dopamine); epileptic seizures (d/t
muscles mostly a/w glycogen GABA, an inhibitory neurotransmitter) can result from B6 deficiency.
phosphorylase. This is not available in
deficiency, but is released to become
available in starvation, when
glycogen reserves become depleted.
Then it is mainly available in liver &
kidney, to meet increased
requirements for gluconeogenesis
from aminoacids
• PP terminates the action of steroid
- Bg deficiency results in (sideroblastic / sidero achrestic i.e. microcytic
hormone by removing the hormone
hypochromic) anemia (d/t defective ALA synthase in heme /porphyrin
receptor complex from DNA binding.
synthesis), xanthurenicaciduria and pellagra (d/t defective kynureninase). One
So there is increased sen sitivity to
action s o f low concentrations o f vitamin (Bs) is necessary for synthesis of other vitamin (niacin). So niacin is not
cortisol, androgen, estrogen and strictly a vitamin as it can be synthesized from tryptophan (essential aa) in body.
vitam in D, w hich m ay l/t M e th io n in e ►[H o m o c y ste in e ] PLP d ependent ►C y s ta th io n in e &
developm en t o f horm on e dependent ~ T T r a n s s u lfu r a tio n b y th en c y ste in e
cancers o f breast, uterus, & p ro sta te N 5 m e th y l T H F ------ S e r in e C y stath ion in e fl-sy n th a s e &
and vitam in Be statu s m ay a ffect the y c y s ta th io n a se
prognosisQ M e th y l- B 12 d e p e n d e n t
•Isoniazid, OCP, cycloserine and alcohol re m eth y la tio n by
may induce B6 deficiency. m eth io n in e sy n th a se

• Vit B 6 status is assessed by assaying I


M e th io n in e
-Activation coefficient i.e. activation
of RBC transaminase by pyridoxal - Bg deficiency cases hyperhomomo cystinemia & homocystinuria 1/t
phosphate added in vitro. cardiovascular diseases and oxaluria 1/t kidney stones.
-Xanthurenic acid index - B6 is required in glycogen metabolism as a cofactor (& constituent of) glycogen
(tryptophan loading test) i.e. phosphorylase (4 molecule of PLP per enzyme molecule)
excretion of Xanthurenate (or • B6 is required in CoA synthesis, sphingomyelin synthesis (so def.causes peripheral
Xandhurenic acid) in urine after a neuritis d/t defective myelination), arachidonic acid synthesis (from linoleic acid),
feeding test dose of tryptophan is intramitochondrial FA synthesis and K+ transport (from out to inside).
diagnostic of B6 deficiency
N 5, N>°, M e th y l T H F TH F
* B6 is the only water soluble vitamin
with significant toxicity. Sensory
G ly c in e ^ ^ S e r in e
neuropathy occur at > 200 mg / day P L P d ep endent
doses (used to treat premenstrual I se r in e h y d ro xy l
syndrome). Substantial improvement G ly o x y la te m e th y l
I tra n s fe ra s e
with some residual damage (i.e. recovery
O x a la te (P rim a ry
is not complete) occurs on
o x a lu r ia ty p e 1)
discontinuation of vitamin.
778 ■ A Complete Review of Short Subjects

Vitamin B12 (Cobalamine, Extrinsic Factor of Castle, Animal protein/Anti Pernicious Anemia-Factor)

Structure Source & Absorption


• Vitamin B 12 or cobalamins are those corrinoids (i.e. •It is synthesized exclusively by microorganisms & (for all
cobalt containing compounds possessing corrin practical purposes) fou n d on ly in fo o d s o f an im al origin®. Vit
ring) which show vitamin activity, (some corrinoid B 12 has no p la n t sources® which means that strict vegetarians
growth factors have no vitamin activity & even are (vegans) are at risk of developing B 12 deficiency.
antimetabolite) 1Small amounts of B 12 formed by bacteria on surface of fruits
• Central portion of B12, consists of 4 pyrrole rings may be adequate to meet the requirements
(reduced & extensively substituted) surrounding a 12 factors required for absorption of B 12 are
single cobalt (Co) atom and is k/a Corrin ring Cobalophilin secreted in saliva bind to B 12 when gastric acid &
system. This differs from porphyrins only in that 2 pepsin release the vitamin from protein in food.
of pyrrole rings (i.e. ring 1 & 4) are linked directly Intrinsic fa c t o r (IF) o f castle®, a small g ly cop rotein secreted by
rather than through a methene bond. p a r ieta l cells o f g astric mucosa® (in cardiac end & fundus but
• 1 cobalt (Co) atom (with 6 valency) is joined to 4 not in pylorus) binds only to active vitamin B 12 vitamers (and
pyrrole rings, 1 DBI (i.e. 5 , 6 Dimethyl not other corrinoids) in duodenum, where cardiophilin is
Benzimidazole riboside) ring. The 6 th valency is detached (hydrolyzed) from B 12 by trypsin of pancreatic juice.
satisfied by any of the following group cyanide That's why pancreatic insufficiency result in excretion of
(added to get stable crystals in oral preparations cobalophilin bound vitamin B 12 causing its deficiency. 1
CN has no physiological function, k/a cyano- molecule of IF can combine with 2 molecules of B 12
cobalamine); hydroxyl (k/a hydroxyl cobalamine or 1 Vit B j 2 is ab sorb ed from d ista l third o f ileium® by receptors
B^a is superior more active form that is retained that internalize whole IF-B 12 complex but not free IF or Bn. Bn
longer in body; can be injectible), SO 4, Cb or N O 2 seperates from IF by releasing factor (RF) secreted by
(i.e. nitro-cobalamine or B12C) duodenum, and Ca++
• Coenzyme forms of cobalamine are methyl and 1 Vitamin Bn is transported in blood as methyl Bn with trans
adenosyl cobalamine. These are formed in body cobalamin glycoprotein; and stored in liver as ado-Bn with
when 6 th valency (like CN-) is replaced by methyl or transcorrin in amounts sufficient for 2-3 years (an exception as
5 1 - deoxyadenosine group respectively. Methyl- B complex is not stored in body)
cobalamine is major form seen in cytoplasm of cells
& blood circulation, whereas Ado (adenosine) B12
is the major storage form seen in liver

.6 ' • - ;
x Uunaat oa *<k u n /

BK»>5 <571 «— r j< & ;r


LunwolGUt

Absorption of Vitamin Bn

Functions of Vit K Functions of Vit C (Acorbic acid)

• Main function of ascorbate is as reducing agent in


Synthesis & activation of Oxidative Synthesis of
different reactions because it can reversibly chang
clotting factors phosphorylation calcium binding
b/w ascorbic acid & dehydro ascorbic acid
I I bone proteins
I (reversible oxidation & reduction)
- Vitamin K is required in - Being associated
- Vit K is required in • Co-enzyme for copper containing hydroxylases eg
hepatic synthesis and with
post translational mitochondiral synthesis of two - Dopamine P hydroxylase involved in synthesis of
m odification (by lipid, vitamin K bone proteins. catecholamines (epinephrine & norepinephrine)
y carboxylation of is a necessary O steocalcin and from tyrosine in CNS and adrenal medulla.
cofactor in bon e m atrix G la - Peptidylglycine hydroxylase, hydroxylating terminal
glutamate residues) of
oxiative protein® that glycine residue
Biochemistry: General ■ 779

calcium binding clotting phosphorylation contain y- carboxy • Co enzyme for a-ketoglutarate linked iron
factors (proteins) glutamate. containing hydroxylases, in which hydroxylation of
prothrombin, 2, 7, 9,10®, Dicumarol Osteocalcin also substrate is linked to decarboxylation of a-
protein C & protein S ®. (vitamin K contains ketoglutarate examples are
- Vitamin K dependent antagonist)® is hydroxyproline, - Proline & lysine hydroxylases are required for
clotting factors are 2, 7, 9, uncoupler of so its synthesis is hydroxylation of prolyl (proline) & lysyl (lysine)
10 and protein C & oxidative also dependent on residues o f collagen, in postsynthetic m odification o f
protein S® phosphorylation both vitamin K and procollagen to colagen®.
- Carboxylation is inhibited C; and also its - Proline hydroxylase is also required in synthesis of
by dicumarol (natural synthesis is osteocalcin & C lq component of complement.
analog o f vitamin K) and induced by vitamin - Trimethyllysine & y-butyrobetaine hydroxylases
warfarin® (synthetic D. The release of are required for carnitine synthesis.
analog of vitamin K) osteocalcin into - Aspartate P-hydroxylase is required for post
- Vitamin K deficiency l/t circulation provides translational modification of precursor of protein C
hemorrhagic disease o f an index of vitamin (a vitamin K dependent protease that hydrolyse
newborn (HDN)® D status. activated factor V in blood clotting cascade)
★ In presence of warfarin, Warfarin treatment - Vitamin C is antioxidant (just like vitamin E & P
Vitamin K epoxide can not in pregnancy 1/t carotene) and can decrease incidence of common cold;
be reduced to quinone fetal bone and some chronic diseases like cataract, coronary
(by warfarin sensitive abnormalities heart disease & cancer. Its deficiency 1/t scurvy,
reductase), but (fetal Warfarin Barlow's disease (infantile scurvy), hemorrhagic
accumulates & excreted. syndrome) d/t tendency & poor wound healing
A high dose o f vitamin K defective synthesis - Help in deposition of collagen in bone
is an antidote to of bone calcium - Maintenance of Capillary integrity
warfarin overdose®, binding proteins - Help in Absorption of Fe
which is reduced to caused by vit K - Prevent formation of Atheromatous plaque
active hydroquinone by deficiency. - Involved in Electron Transport chain: contribute to
warfarin insensitive
microsomal hydroxylation reaction. [M n : "C A T"]
enzyme.

Vitamin A

r T
Structure Transport & Storage Functions Deficiency Prevalence criteria for
• Active form - Dietary retinal is - Vision® - Night blindness determining the xeroplathalamia
- Retinol® transported as - Maintenance of - Xeropthelmia
- Retinal ® retinyl esters in reproduction - Impotence Criteria Prevalence in
- Retinoic acid® chylomicrons - Antinfective® - Growth population at
• Retinal is a ■Stored form is - Antioxidant® retardation risk (6 month
component o f retinyl esters® - Anticarcinogenic® - 6 yr)
visual pigment mainly in liver & Bitol's spot >0.5%
Visual cycle in Retina
rhodopsin®. adipose tissue Cormeal >0.05%
• p Carotenes are A ll trans retinol ulcer
found in certain I Serum >5%
All trans retinal
plants, cleaved to retinal
1
yield 2 mol. Of Mobilized form 11- C is retinalQ Corneal >0 .01 %
retinal i — Opsin xerosis
R h od op sin Nightblind­ > 1%
|Lighlj J, — ►Opsin ness
A ll trans retinal
780 ■ ■ A Complete Review of Short Subjects

Vitamin A Intoxication

I-------------------------------------------—
Acute Chronic
Seen after administration of >100 mg in children or > 150 mg in adult Seen after chronic ingestion (for
l sl noticed in arctic explores who ate polar bear liver. several months) of > 6mg/day in
P manifests as increased in tracran ial pressure, bulging fo n tan els (in children or >15 mg/day in adults.
children), seizure, vertigo, dip lop ia, ex fo lia tiv e derm atitis® and death. It manifests as

P seudotum or cerebri® - Alopecia - Bone - Liver Teratogenic effects


with increased TCP & - Dryskin demineralization fibrosis - Amenorrhea
papilledema - Cheliosis - Bone pain - Portal - Spontaneous abortion
Vomiting - Glossitis - Hypercalcemia hypertension - Craniofacial abnormality
- Hyperlipidemia - Hyper - Valvular heart disease
- Lymphadenopathy lipidemia

Vitamin D

D2D3 7 -O H -c h o le c a lc ife ro l • It behaves like a steroid hormone, binding to


(D iet) (S kin ) a nuclear receptor protein.
U V ligh t • Deficiency:
V it D3 - Rickets (In children)
2 5 (O H ) Liver<2 - Osteomalacia (In adults)
• Toxicity: Vitamin D is most toxic of all
2 5 (O H ) V it D 3
vitamins
■(O H ) K id n ey Q - Tb p
- T serum Ca2+ - - Vaso constriction
1.25 (O H )2 V it D i [A ctiv e form ]
L Calcinosis [soft tissue
calcification]
T a b so rp tio n o f C a 2" fro m g u t T m in era liz a tio n o f b o n e - Nausea
- Stupor
for detailed information on PGMEE books, visit our website www.jaypeebrothers.com, for detailed information on PGMEE books, visit our website www.jaypeel

-^ACROSS
A Complete Review of Short Subjects

Why this book?


• The first and the most sought after subjectwise book for PGMEE and the bestselling MCQs
book on short subjects.
• The book which has given a new approach and horizon to PGMEE preparation
• Thoroughly revised and updated edition w ith new layout
Book covers all questions from 2001 onwards
• All MCQs o f All India (NBE Pattern), AIIMS, PGI, DNB, and State-based MCQs up to 2016 have
been included
Detailed, yet concise, short notes are added for each chapter
Subjects divided into theory and MCQs and furthe r divided into chapters and subchapters,
according to their topics such tha t the study material is easily retainable and understandable
for students
Each new edition is not only an addition o f few hundred new questions; as each line o f the
book is com pletely revised and updated based on the latest editions o f various standard
textbooks everytime. Controversial questions are explained w ith com plete references
• A com pletely revised, researched, concise and condensed (PGMEE-oriented) presentation o f
im p o rta nt topics and basic concepts w ith numerous self made tables, figures, flo w charts and
m nemonics th ro u g h o u t the book, to save student's ample tim e for preparation
Provides a m ethodology to approach PGME exam ination; includes valuable pedagogical
features and many topics o f medicine, surgery and pharmacology, coinciding w ith short
subjects
Indispensable for course review and PGME examinations. Authors recommend to start
reading and retaining this book after 2nd professional, during clinical postings.

Available at all medical bookstores


or buy online at www.jaypeebrothers.com
ISBN I T a - I B - S B V O - m O - 7

4 JAYPEE BROTHERS
Ifr Medical Publishers (P) Ltd.
jAVPEE w w w .ja ypee bro thers.co m

You might also like